You are on page 1of 2088

1.

The following statements are true about SIDS (sudden infant death
syndrome) except:
a) Both prone and side sleeping increase risk of SIDS.
b) All babies should sleep in supine position including infants with
micrognathia and obstructive sleep apnea.
c) Most common cause of postneonatal death 1 mo – 1 yr of age in developed
countries.
d) Future SIDS victims may have repeated fatigue during feeding and profuse
sweating during sleeping.
e) Caffeine and theophylline decrease apnea and improve respiratory pattern.
However, no medication can prevent SIDS.

1. (b) All babies should sleep in supine position except in infants with
micrognathia and obstructive sleep apnea.
2. A 2-year-old boy was brought to the ER with a history of difficulty
breathing and coughing for the last 2 days. His mother has asthma. Three
other siblings do not have asthma. Mother denies any foreign body
aspiration.
Physical examination: afebrile, mildly tachypneic, wheezing is noted in the
right chest, and the left lung sounds are clear. Chest X-ray reveals
overdistended right lung, especially in expiratory film. Appropriate
management should be:
a) Bronchodilator
b) Pulmonary physiotherapy
c) Thoracotomy
d) CPR (cardio pulmonary resuscitation)

e) Removal of the bronchial foreign body


3. A child was extubated and developed postextubation subglottic edema.
The next step in management:
a) Furosemide
b) Reintubate immediately
c) Spironolactone
d) Nasal CPAP (continuous positive airway pressure)
e) Inhalation of aerosolized racemic epinephrine

4. All of the following are indications for tonsillectomy except:


a) Obstruction due to tonsils
b) Tumor of tonsills
c) Middle-ear deafness
d) Peritonsillar abscess with prior history of tonsillitis
e) Recurrent peritonsillar abscess
5. A female child appears in the ER with a severe asthma attack. She has
been using albuterol inhalation pump.
Physical examination reveals tachypnea, intercostal and subcostal
retractions, and expiratory wheezes. She is in a respiratory failure. The
most important finding suggests impending respiratory failure:
a) Hypoxia
b) Hypercarbia
c) Fatigue
d) Lethargy
e) Hypocarbia

6. A 15-year-old girl appears in the clinic with history of a dry cough with
harsh, ‘barking’ quality for the last 2 weeks. She has no fever. She had
history of mild whitish vaginal discharge for the last 3 weeks. She had no
problems in sleeping at night. Her mother said that she did not cough at
night while sleeping. Physical examination reveals mildly injected throat.
Most likely diagnosis:
a) Pharyngitis
b) Chlamydia
c) Croup
d) Laryngeal papilloma
e) Habit cough
7. A 14-year-old girl appears with sudden onset of left chest pain for the
last 2 hours. She has no fever. She denies history of trauma. Her
menstrual period is regular, usually lasts for 5 days, and presently, she is
on day 3 of her period. Most likely diagnosis:
a) Pneumomediastinum
b) Pneumothorax
c) Pulmonary embolism
d) Asthma
e) Angina
7. (b) Catamenial pneumothorax

8. A 7-year-old boy is diagnosed to have a obstructive lung disease. All of


the following are abnormal except:
a) Total lung capacity
b) Residual volume
c) Vital capacity
d) Functional residual capacity
e) Diffusing capacity of carbon monoxide
8. (e) Diffusing capacity of carbon monoxide
9. A 2-week-old girl appears in the clinic for a routine check up. She has
biphasic stridor that is more prominent during inspiration. She has
hoarseness and barking cough. Most likely diagnosis:
a) Subglottic hemangioma
b) Laryngeal nodule
c) Tracheomalacia
d) Croup
e) Laryngeal web

10. A preterm infant with respiratory distress syndrome was placed on a


mechanical ventilator. The infant developed right pneumothorax. The
most important factor causing pneumothorax in a patient on mechanical
ventilator:
a) High PEEP
b) High PIP
c) Low oxygen
d) High flow rate
e) High expiratory time

11. Most common complication of a tympanostomy tube:


a) Hearing loss
b) Otorrhea
c) Cholesteatoma
d) Otitis media
e) Otitis externa
12. Pulmonary function test results in a patient with cystic fibrosis:
a) Restrictive lung disease
b) Obstructive lung disease
c) Initially restrictive, then obstructive lung disease
d) Initially obstructive, then restrictive lung disease
e) Normal lung function

13. A 6-year-old boy had a modified Blalock-Tausig shunt for tetralogy of


Fallot. About 12 hours after surgery, the boy developed respiratory
distress. Chest X-ray revealed pleural effusion. Most likely diagnosis:
a) Hemothorax
b) Chylothorax
c) Pneuthorax
d) Hydrothorax
e) Diaphragmatic hernia

14. A child is diagnosed to have a pleural effusion. All of the following are
the characteristics of a transudative pleural fluid except:
a) Total protein is less than 3 g/dL.
b) Lactate dehydrogenase level is low.
c) Total WBC count is less than 2000/mm3
d) Predominance of monocytes
e) A ratio of pleural protein to serum protein is more than 0.5
15. A 10-day-old male newborn is diagnosed to have a cystic fibrosis.
Sweat chloride test results are normal. The most likely interpretation:
a) He does not have a cystic fibrosis.
b) Sweat chloride test results are always abnormal in a cystic fibrosis newborn.
c) A sweat chloride test should not be performed before 1 year of age.
d) Sweat chloride test results are not reliable in first week of life.
e) Sweat chloride test should be performed after 6 months of age.

16. A 4-year-old male asymptomatic child appears for a routine check up.
He had an ear infection 3 years ago. Physical examination reveals bilateral
middle-ear effusions. Most likely diagnosis:
a) Acute otitis media
b) Chronic otitis media
c) Otitis externa
d) Middle ear cholesteatoma
e) Eustachian tube dysfunction or obstruction

17. Most common cause of childhood nasal polyposis:


a) Allergic rhinitis
b) Allergic sinusitis
c) Vasomotor rhinitis
d) Cystic fibrosis
e) Deviated nasal septum
18. Most common clinical manifestation of cystic fibrosis:
a) Failure to thrive
b) Abnormal stools
c) Meconium ileus
d) Nasal polyps
e) Respiratory symptoms

19. The best initial diagnostic study for a child with respiratory distress:
a) Arterial blood gas
b) Chest X-ray
c) Chest CT scan
d) Chest MRI
e) Oxygen saturation by pulse oximetry

20. A 3-year-old girl appears with a sudden onset of cough, dyspnea, and
hoarseness for the last 3 hours. Mother noticed a small amount of fresh
blood with mucus inside her mouth. She was playing and eating prior to
this episode. Physical examination reveals a croupy cough and mild
cyanosis. The appropriate management:
a) Throat culture
b) Nasopharyngeal culture
c) Remove a foreign body from esophagus
d) Remove a foreign body from larynx
e) Remove a foreign body from nasopharynx
21. A 2-year-old boy appears with a sudden onset of cough, dyspnea, and
hoarseness for the last 6 hours. Mother denies fever. He was playing prior
to this episode. Physical examination reveals bilateral wheezing, audible
slap, and palpable thud. The appropriate step in management:
a) Throat culture
b) Nasopharyngeal culture
c) Remove a foreign body from esophagus.
d) Remove a foreign body from trachea.
e) Remove a foreign body from stomach

22. Preferred investigative procedure for a lung mass:


a) MRI
b) Fluroscopy
c) CT scan
d) Chest x-ray
e) Pulmonary function test

23. Least likely complication of bronchoscopy:


a) Hypoxia
b) Bronchospasm
c) Arrythmia
d) Pneumopericardium
e) Laryngospasm
24. Inhaled glucocorticoid therapy can cause dysphonia in patients with
asthma. The cause of dysphonia is:
a) Vocal cord myopathy
b) Fungal infection in vocal cord
c) Vocal card paralysis
d) Pharyngitis
e) Laryngitis
25. Most common side effects of inhaled glucocorticoids are oral thrush
and dysphonia. The cause of oral thrush formation is:
a) Local immunosuppression
b) Dry mouth
c) Excessive salivation
d) Herpes simplex virus
e) Infectious mononucleosis
26. The mean airway pressure (MAP) is increased by all of the following
parameters except:
a) Increased positive inspiratory pressure (PIP)
b) Increased positive end expiratory pressure (PEEP)
c) Increased inspiratory flow
d) Prolonged expiratory time
e) Increased ventilatory rate without changing the inspiratory time
27. (d) Prolonged expiratory time does not increase the MAP. Prolonged
inspiratory time without changing the rate causes reversal of inspiratory–
expiratory ratio (I:E) resulting in an increased of MAP.
28. Pulmonary interstitial emphysema (PIE) can be prevented by:
a) Avoiding high positive inspiratory pressure (PIP)
b) Avoiding high oxygen use
c) Increasing high PIP
d) Increasing mean ventilatory pressure
e) Avoiding low positive end-expiratory pressure (PEEP)
28. (a) Avoidance of high PIP and mean ventilatory pressure can prevent the
development of PIE.
29. The treatment of pulmonary interstitial emphysema (PIE) includes all
of the following except:
a) Selective intubation and ventilation of the involved lung and bronchus
b) High frequency ventilation
c) Oxygen
d) Bronchoscopy performed in patients with mucus plug
e) General respiratory care
30. The predominant source of bleeding in pulmonary hemorrhage is:
a) Tracheal
b) Bronchial
c) Interstitial
d) Laryngeal
e) Alveolar

31. The treatment of pulmonary hemorrhage includes all of the following


except:
a) Blood transfusion
b) Increased PEEP
c) Intratracheal administration of epinephrine
d) Suctioning to clear the airway
e) Increased PIP
31. (e) Increased PIP does not stop pulmonary hemorrhage. High frequency
ventilation (HFV) is useful to stop pulmonary hemorrhage in some cases.
32. The preferred therapy for patients with recurrent respiratory
papillomatosis is:
a) Intralaryngeal sclerosing agent
b) Intralaryngeal epinephrine
c) Intralaryngeal corticosteroids
d) Reassurance
e) Excision with CO2 laser
33. A newborn developed noisy breathing and mild respiratory distress.
He was diagnosed with recurrent respiratory papillomatosis.
He acquired the disease from his mother:
a) At the time of delivery
b) In utero
c) Immediately after birth
d) During nursing
e) Within 7 days after birth

34. The recurrent respiratory papillomatosis (RRP) is caused by the


following types of human papillomavirus (HPV):
a) Types 1 and 3
b) Types 2 and 4
c) Types 6 and 11
d) Types 12 and 14
e) Types 13 and 15
35. All of the following environmental irritants can cause cough, asthma,
or chronic lung disease except:
a) Ozone
b) Nitrogen dioxide
c) Marijuana smoke
d) Tobacco smoke
e) Carbon dioxide
35. (e) Carbon dioxide and oxygen are not irritants; particulate matters are also
irritants.
36. The restrictive lung disease includes all of the following conditions
except:
a) Asthma
b) Pneumonia
c) Scoliosis
d) Pulmonary edema
e) Respiratory distress syndrome
37. The obstructive lung disease includes all of the following conditions
except:
a) Pulmonary fibrosis
b) Emphysema
c) Bronchiolitis
d) Meconium aspiration
e) Bronchopulmonary dysplasia
37. (a) Pulmonary fibrosis is a restrictive lung disease. Restrictive lung disease
also includes pneumothorax, pulmonary collapse, diaphragmatic hernia,
persistent pulmonary hypertension due to collapse, pulmonary interstial
emphysema, and pulmonary tuberculosis.
38. All of the following conditions cause fixed extrathoracic obstruction
except:
a) Laryngomalacia
b) Tracheal stenosis
c) Epiglottitis
d) Enlarged tonsils
e) Polyp of the vocal cord
38. (a) Laryngomalacia causes variable extrathoracic obstruction.
39.. The preferred test for pulmonary function is:
a) Pulmonary scan
b) Pulmonary angiogram
c) Lung CT scan
d) Lung MRI
e) Arterial blood gas
39. The most common cause of acquired laryngotracheal stenosis is:
a) Infection
b) Reflux of gastric acid
c) Reflux of gastric pepsin
d) Bleeding
e) Endotracheal intubation
40. The most common organism in patients with Lemierre disease is:
a) Group A Streptococcus
b) S. aureus
c) Haemophilus influenzae
d) Klebsiella
e) Fusobacterium necrophorum
40. (e) Fusobacterium necrophorum (anaerobic oropharyngeal bacteria)
41. A 16-year-old healthy adolescent appears with sudden onset fever,
respiratory distress, and cough. He has been suffering from acute
tonsillopharyngitis for the last 3 days. The chest x-ray reveals multiple
bilateral cavitary nodules and mild pleural effusions. The preferred
therapy is:
a) Penicillin
b) Ciprofloxacin
c) Vancomycin
d) Gentamicin
e) Ceftazidime
41. (a) Lemierre disease is treated with intravenous penicillin or cefoxitin;
surgical drainage is indicated in patients with extrapulmonary metastatic
abscess.
42. The following medication should be avoided in patients with influenza:
a) Acetaminophen
b) Ibuprofen
c) Aspirin
d) Antihistamine

43. A 15-yar-old healthy boy appears with a stabbing pain in the chest.
The pain radiates to the neck. He denies history of trauma. The
physical examination reveals subcutaneous emphysema. The most likely
diagnosis is:
a) Pneumothorax
b) Fracture ribs
c) Critical aortic stenosis
d) Myocardial infarction
e) Pneumomediastinum
44. A child is suspected to have recurrent aspirations. The initial study
should be:
a) Milk scan
b) Barium-swallow
c) Upper GI series
d) Ultrasonography
e) Plain chest x-ray
45. A 3-month-old child with SIDS (sudden infant death syndrome) was
brought to the ER. The physical examination most likely reveals:
a) External bruises
b) Enlarged liver
c) Enlarged spleen
d) Proptosis
e) Petechial hemorrhage

46. The methacoline challenge testing is performed to diagnose a suspected


case of:
a) Food allergy
b) Bronchial asthma
c) Cystic fibrosis
d) Tuberculosis
e) Cardiac arrythmias
47. If a mother is PPD positive and has a negative chest x-ray, the newborn
infant should be:
a) Separated from the mother
b) Given INH prophylaxis
c) Evaluated with a chest radiography
d) Evaluated with a PPD testing
e) With the mother
48. A mother has an active pulmonary tuberculosis. All of the following
statements are true about management of the newborn except:
a) The newborn should receive INH therapy.
b) The newborn should be isolated from the mother regardless of her symptoms
during INH therapy.
c) The newborn should be isolated from the mother if she has drug-resistant
tuberculosis, she is noncompliance, and she is ill enough to require
hospitalization.
d) The newborn should receive INH therapy until the mother’s sputum cultures
are negative for at least 3 months.
e) The newborn should receive a Mantoux tuberculin skin test after 3 months
of age. If positive, INH should be continued for a total duration of 9-12
months.
48. (b) The INH treatment of the newborn is very effective. Therefore, a
separation of the mother and infant is no longer mandatory if the mother is
asymptomatic.
49. The preferred therapy for a pregnant woman with an active
pulmonary tuberculosis is a combination of:
a) INH, pyrazinamide, and rifampin
b) INH, ethionamide, and ethambutol
c) Rifampin, ethambutol, and ethionamide
d) INH, rifampin, and ethambutol
e) Streptomycin, INH, and rifampin
49. (d) INH, rifampin, and ethambutol are preferred therapies; aminoglycosides
and ethambutol are teratogenic; the use of pyrazinamide may not be safe during
pregnancy.
50. A 12-year-old boy appears with a persistent cough, headache, fever,
malaise, and hoarseness for the last 6 days. Coryza is absent.
He produces a frothy, white sputum. Initially the cough was
nonproductive. The physical examination reveals a fine crackles on the
right chest. The chest x-ray reveals right lower lobe interstitial infiltrates.
The preferred therapy for this patient is:
a) Symptomatic therapy
b) Ampicillin
c) Cefotaxime
d) Ceftriaxone
e) Azithromycin
50. (e) Azithromycin, erythromycin, or clarithromycin is used in Mycoplasma
pneumonia. Coryza is usually present in viral infections. Pneumonia in a
school–aged children, especially if cough is a persistent finding, is always
indicate Mycoplasma pneumonia.
51. Neurologic complications of patients with Mycoplasma pneumonia
includes all of the following except:
a) Guillain-Barre syndrome
b) Bell palsy
c) Transverse myelitis
d) Hyperacusis
e) Aseptic meningitis
51. (d) Deafness, cerebellar ataxia, brainstem syndrome and acute
demyelinating encephalitis are also neurologic complications of Mycoplasma
pneumonia
52. The most common site of pulmonary atelactasis in children is:
a) Right upper lobe
b) Right middle lobe
c) Right lower lobe
d) Left upper lobe
e) Left lower lobe
53.. Tonsillectomy is indicated in all of the following conditions except:
a) A patient who experienced six tonsillar infections that were treated with
antibiotics in the preceding year.
b) A patient who experienced five tonsillar infections that were treated with
antibiotics in each of the preceding two years.
c) A patient who experienced three tonsillar infections that were treated with
antibiotics in each of the preceding three years.
d) A patient who experienced four tonsillar infections that were treated with
antibiotics in each of the preceding three years.
e) A patient who experienced seven tonsillar infections that were treated with
antibiotics in the preceding year.
53. a) Tonsillectomy is indicated in a patient with seven or more tonsillar
infections that were treated with antibiotics in the preceding year,
five or more tonsillar infections that were treated in each of the preceding 2
years,or three or more tonsillar infections that were treated in each of the
preceding 3 years.
54. The most common organism in patients with a empyema (purulent
pleurisy) is:
a) Staphylococcus aureus
b) Group A Streptococcus
c) Tuberculosis
d) E. coli
e) Streptococcus pneumoniae
55. The most common organism in patients with a post-traumatic
empyema is:
a) S. pneumoniae
b) S. aureus
c) E. coli
d) Pseudomonas
e) S. epidermidis
56. The most common foreign body in patients with aspiration is:
a) Popcorn
b) Dried beans
c) Sunflower seeds
d) Watermelon seeds
e) Nuts
57. A child appears with hemoptysis. A screening method includes all of
the following tests except:
a) Complete blood counts
b) Chest x-ray
c) Prothrombin time
d) Chest CT scan
e) Partial thromboplastin time
57. d) CT-scan is not a screening tool but may be useful to make a diagnosis.
Bronchoscopy is both diagnostic and therapeutic. A rigid bronchoscopy is used
to remove foreign body. A flexible bronchoscopy is used in patients with
noncopious bleeding.
58. A child is intubated for respiratory failure. An objective of mechanical
ventilation is the following:
a) Not to normalize arterial blood gas tension
b) Not to maintain an adequate gas exchange
c) Oxygen saturation should be maintained between 95-100%.
d) Arterial PCO2 should be maintained between 35-45 mm Hg.
e) Arterial pH should be maintained between 7.35-7.45.
58. a) An objective of mechanical ventilation is not to normalize arterial blood
gas tension but to maintain adequate gas exchange (i.e., some degree of
hypoxia, O2 saturation 85-90%; moderate hypercarbia, PCO2 60-80 mm Hg
are acceptable if the patient’s condition is stable).
59. All of the following organisms cause acute pharyngitis in children
except:
a) Virus
b) Group A beta-hemolytic Streptococcus
c) Group C Streptococcus
d) Haemophilus influenzae
e) Arcanobacterium hemolyticum
59. d) Haemophilus influenzae and Streptococcus pneumoniae may be cultured
from throat but their role in causing pharyngitis is not clear. Viruses are the
most common cause of pharyngitis. However, group A-beta-hemolytic
Streptococcus (GABHS) is the most common bacterial cause of pharyngitis.
60. Posteroanterior x-ray of a neck reveals a “steeple sign”. The most
likely diagnosis is:
a) Fracture clavicle
b) Fracture cervical vertebra
c) Croup
d) Laryngeal polyps
e) Tracheoesophageal fistula
60. c) “Steeple sign” represents typical subglottic narrowing that is present in a
patient with croup. This sign may be absent in croup or may be present in a
normal child or a patient with subglottic stenosis.
61. Lateral x-ray of the upper airway in a 2-year-old boy reveals a “thumb
sign”. The most likely diagnosis is:
a) Laryngeal polyps
b) Epiglottitis
c) Subglottic hemangioma
d) Laryngeal edema
e) Subglottic stenosis
62. The first symptom noted in patients with a common cold is:
a) Nasal congestion
b) Cough
c) Fever
d) Sore throat
e) Runny nose
63. A 7-year-old boy appears with cough, mild fever, nasal obstruction,
and rhinorrhea for the last 5 days. The symptoms began with sore throat
about 7 days ago. The mother also noted a change in the color and
consistency of the nasal secretions. A physical examination reveals a stuffy
nose, swollen, erythematous nasal turbinates, and mildly injected throat.
The next step in management
is:
a) Amoxicillin
b) Erythromycin
c) Cefuroxime
d) Amoxicillin-clavulanate
e) Symptomatic therapy
63. e) Symptomatic therapy is indicated for patients with a common cold.
Please remember, a change in color and consistency of the nasal secretions
does not indicate a sinusitis or bacterial infection.
64. A nasal secretion of a child reveals predominant polymorphonuclear
leukocytes. The most likely diagnosis is:
a) Rhinovirus infection
b) Haemophilus influenzae infection
c) Allergic rhinitis
d) Moraxella catarrhalis infection
e) Group A beta-hemolytic Streptococcus infection
64. a) Rhinovirus; the presence of polymorphonuclear leukocytes does not
mean a bacterial superinfection.
65. A nasal secretion of a child reveals predominant eosinophils. The most
likely diagnosis is:
a) Rhinovirus infection
b) Streptococcus pneumoniae infection
c) Influenza virus infection
d) Respiratory syncitial virus
e) Allergic rhinitis
66. A child appears with a drooling, trismus, unilateral throat pain, and
ipsilateral referred otalgia. A physical examination reveals the right tonsil
displaced medially and downward by the ipsilateral swollen palate and
anterior tonsillar pillar. The preferred diagnostic study is:
a) Throat culture
b) ASLO titer
c) CT scan of the affected area
d) Lateral x-ray of the neck
e) X-ray of maxillary sinuses
66. c) CT scan of an affected area or needle aspiration and culture can make the
diagnosis of peritonsillar abscess.

67. The preferred therapy for patients with a retropharyngeal or lateral


pharyngeal abscess without respiratory distress is:
a) Surgical drainage and penicillin
b) Azithromycin and gentamicin
c) A third-generation cephalosporin and penicillin
d) Nafcillin and surgical drainage
e) A third-generation cephalosporin and ampicillin-sulbactum
67. e) A third-generation cephalosporin and ampicillin-sulbactum or
clindamycin; surgical drainage is indicated in a patient with respiratory distress
or failure to improve with antibiotic therapy.
68.. The preferred therapy for patients with a retropharyngeal or lateral
pharyngeal abscess with respiratory distress is:
a) Surgical drainage and penicillin
b) Surgical drainage and ampicillin
c) Surgical drainage and ampicillin-sulbactum
d) Surgical drainage and clindamycin
e) Surgical drainage and a third-generation cephalosporin and clindamycin

69. Clinical uses of pulmonary function testing (PFT) include all of the
following except:
a) PFT usually makes the specific diagnosis.
b) PFT can detect an obstructive lung disease.
c) PFT can detect a restrictive lung disease.
d) PFT can detect a degree of functional impairment.
e) PFT is useful in a preoperative evaluation.
69. a) PFT rarely makes a diagnosis. PFT can detect a functional impairment
in patients with a normal physical examination but minor
complaint. PFT is useful in determining responses of a bronchodilator in
patients with an obstructive lung disease.
70. All of the following conditions are true in patients with a cystic fibrosis
(CF) except:
a) Autosomal recessive
b) Chromosome 7
c) Chronic obstructive lung disease
d) Pancreatic insufficiency
e) The most prevalent mutation of the CF transmembrane regulator (CFTR) is
the deletion of a single tyrosine residue at amino acid 508.
70. e) A deletion of a single phenylalanine residue at amino acid 508.

80. The following statement is not true about restrictive lung disease:
a) The duration of inspiration is increased.
b) The duration of expiration is reduced.
c) The chest x-ray reveals a decreased lung volume.
d) The physical examination reveals grunting and crackles.
e) The amplitude of respiratory movements is shallow.
80. a) Duration of inspiration is reduced. Respiratory rate is increased.
Inspiratory muscles are accessory muscles. Retractions are present.
81. All of the following statements are true about extrathoracic obstructive
disease except:
a) Duration of inspiration is reduced.
b) Respiratory rate is decreased.
c) Duration of expiration is unchanged.
d) Physical examination reveals a inspiratory stridor.
e) Chest x-ray findings are normal.
81. a) Duration of inspiration is prolonged. Inspiratory muscles are accessory
muscles. Retractions are present. Amplitude of respiratory movements are
normal or decreased.
82. The following statement is not true about the intrathoracic obstructive
lung disease except:
a) Duration of inspiration is prolonged.
b) Respiratory rate is either normal or increased.
c) Duration of expiration is prolonged.
d) A physical examination reveals expiratory wheezes.
e) A chest x-ray reveals an increased lung volume.
82. a) Duration of inspiration is unchanged. Both inspiratory and expiratory
(abdominal) muscles are accessory muscles.
Retractions are present. Amplitude of respiratory movements are either normal
or reduced.
83. The most common presentation in patients with a tracheal foreign
body is:
a) Wheezing
b) Stridor
c) Positive chest x-ray
d) Choking and aspiration
e) Positive soft tissue x-ray of the neck
83. e) Positive posteroanterior and lateral soft tissue x-ray of the neck (92% of
cases); answer (d) is present in 90% of cases; answer (b) is present in 60% of
cases; answer (c) is present in 58% of cases; answer (a) is present in 50% of
cases
84. A 2-month-old infant appears with a brassy cough. The most likely
diagnosis is:
a) Subglottic hemangioma
b) Bronchiolitis
c) URI
d) Chlamydia infection
e) Vascular ring
84. e) Vascular ring causes brassy cough. Brassy cough is also noted in patients
with tracheitis or habit cough. Brassy cough and stridor are noted in patients
with a laryngeal obstruction or pertussis
85. A child appears with a staccato cough. The most likely diagnosis is:
a) Habit cough
b) Subglottic hemangioma
c) Laryngomalacia
d) Croup
e) Chlamydia pneumonitis

86. A 10-year-boy appears with a history of cough only during the daytime
for the last 2 months. However, he sleeps well at night without coughing.
The boy is coughing during the physical examination. A physical
examination reveals a normal throat and clear breath sounds in both
lungs. The most likely diagnosis is:
a) Chlamydia infection
b) Bronchial asthma
c) Sinusitis
d) Habit cough
e) Carrier of group A Streptococcus
87. A congenital subglottic hemangioma is associated with the following
anomaly:
a) Renal
b) CNS
c) Liver
d) Spleen
e) Cutaneous
87. e) Cutaneous lesions are present in 50% of cases.

88. A child who is a known asthmatic appears with a sudden onset of


cough, dyspnea, tachypnea, shallow respiration, and tachycardia.The child
has no fever. A physical examination reveals a mild cyanosis, decreased
breath sounds and crackles over the right upper thoracic cavity. The most
likely diagnosis is:
a) Right pneumothorax
b) Left pneumothorax
c) Right upper lobe pneumonia
d) Right upper lobe atelactasis
e) Right lower lobe bronchiectasis
88. d) Pulmonary atelactasis can occur in asthma, pneumonia, and trauma.
Massive collapse of one or both lungs most commonly occurs after a surgery.
The most common site of atelactasis is the right upper lobe.
89. A child appears with a moderate amount of bleeding through the
mouth. All of the following findings indicate a hemoptysisvexcept:
a) Bright red color
b) Frothy
c) Nausea
d) Accompanied by cough
e) Alkaline pH

90. A child appears with a massive bleeding through the mouth and nose.
The following finding does not indicate a hematemesis:
a) Dark red blood
b) Acidic pH
c) Nausea
d) Frothy
e) Presence of food particles
91. A child appears with a fever, cough, sputum production, tachypnea,
dyspnea, hemoptysis, and weight loss. A physical examination reveals
intercostals retractions, crackles, decreased breath sounds, and dullness to
percussion in the right upper chest area. The chest x-ray reveals
parenchymal inflammations with a cavity containing air-fluid level in the
right upper lobe. The most likely diagnosis is:
a) Right upper lobe TB
b) Right upper lobe pneumonia
c) Right upper lobe abscess
d) Right upper lobe cyst
e) Right upper lobe obstruction

92. The preferred diagnostic study in patients with a pulmonary abscess


is:
a) Chest x-ray
b) Lung scan
c) Lung CT scan
d) Bronchoscopy
e) Sputum culture
93. The following statement is not true about pulmonary aspirations and
abscesses:
a) A secondary lung abscess is more common in the right side, particularly in
immunocompromised children.
b) A primary lung abscess is more common in the right side.
c) If the child is upright, the posterior segments of upper lobes are affected.
d) If the child is in recumbent position, left and right upper lobes are affected.
e) In a recumbant position, apical segment of the right upper lobe is affected.
94. a) A secondary lung abscess is more common on the left side

95. A 2-year-old boy appears in the ER with a history of foreign body


inside the nose. A physical examination reveals a small perforation of the
nasal septum. The foreign body is removed. The most likely foreign body
is:
a) Bead
b) Bean
c) Crayon
d) Stones
e) Disk battery
95. e) Disk batteries cause pain and local tissue destructions within a matter of
hours

96. A child had a nasal operation that required nasal packing. A follow-up
appointment is given in ENT clinic. The mother missed the appointment.
Subsequently, the child became ill, developed fever and a bad smell from
the nasal packing area. The most serious complication is:
a) Dislodgement of the packing
b) Excessive bleeding
c) Apnea
d) Brain abscess
e) Toxic shock syndrome
96. e) Toxic shock syndrome most commonly occurs from nasal surgical
packing
97. A 5-year-old girl appears with a purulent unilateral nasal discharge
for the last 3 days. A physical examination reveals unilateral purulent
secretions but no foreign body is visualized. The next step in management
is:
a) Amoxicillin-clavulanate
b) Cefuroxime
c) Azithromycin
d) Reassurance
e) Suctioning the secretions
97. e) Suctioning the secretions is indicated; the foreign body is usually located
behind the secretions; often a topical decongestant and headlight are needed to
remove the foreign body.
98. A routine newborn physical examination at birth reveals a nasal septal
deviated to the right. The most common cause is:
a) Choanal stenosis
b) Choanal atresia
c) Congenital syphilis
d) Frontal encephalocele
e) Trauma from delivery
99. The preferred management for newborns with a deviated nasal septum
secondary to trauma during delivery is:
a) Reassurance
b) ENT outpatient appointment after 7 days
c) ENT outpatient appointment after 1 month
d) CT scan of the nose
e) Immediate realignment using blunt probes cotton applicators, and topical
anesthesia
100. The following statement is not true in patients with a pulmonary
sequestration:
a) The majority of sequestrations are intrapulmonary.
b) Extrapulmonary sequestrations are almost always involve the left lung
c) Extrapulmonary sequestrations are strongly associated with a diaphragmatic
hernia.
d) Surgical removal is indicated.
e) The arterial supply in sequestration comes from a pulmonary artery and
venous drainage returns to the left atrium through pulmonary veins.
100. e) The arterial supply in sequestration comes from systemic arteries,
especially from the aorta and venous drainage returns to the right atrium,
especially through inferior vena cava.
101. The most common presentations in patients with a primary ciliary
dyskinesia are:
a) Situs inversus, otitis media, and chronic sinusitis
b) Chronic sinusitis, otitis media, and wheezing
c) Bronchiectasis, chronic sinusitis, and asthma
d) Asthma, pneumonia, and bronchiectasis
e) Productive cough, sinusitis and otitis media
101. e) Productive cough, sinusitis, and otitis media are present in almost all
patients; about 50% of patients have Kartagener syndrome (i.e., situs inversus,
otitis media, chronic sinusitis and airway disease leading to bronchiectasis).
Primary ciliary dyskinesia is known as immotile cilia syndrome
102. The mode of inheritance in patients with a primary ciliary dyskinesia
is:
a) Autosomal recessive
b) Autosomal dominant
c) X-linked recessive
d) X-linked dominant
e) Unknown
103. The gold standard to make the diagnosis of primary ciliary dyskinesia
is:
a) CT scan of paranasal sinuses
b) Pulmonary function tests
c) CT scan of lungs
d) Ultrasonography of sinuses
e) Nasal biopsy
103. e) Nasal (or bronchial) biopsy or scraping reveals quantitative
documentation of missing dynein arms or random orientation of cilia under
electron microscopic examination.
5104. A pulmonary function test in older children with a primary ciliary
dyskinesia reveals:
a) Normal
b) Restrictive lung disease
c) Obstructive lung disease
d) Both restrictive and obstructive lung diseases with an equal frequency
e) More restrictive than an obstructive lung disease
104. c) Obstructive lung disease
105. The cardiac output in a normal healthy newborn is about:
a) 75 mL/kg/minute
b) 150 mL/kg/min
c) 200 mL/kg/minute
d) 300 mL/kg/min
e) 350 mL/kg/minute
106. An adolescent boy appears with a recurrent, severe nose bleeds for
the last 6 months. He experienced several bleeding episodes more
frequently for the last 1 month. He denies history of a major trauma.
However, he has a long nose and often experienced minor injuries. A
physical examination reveals a nasal mass. He experienced a major
laceration in the right leg but the bleeding sopped spontaneously. The
most likely diagnosis is:
a) Hemophilia A
b) Nasal septal hematoma
c) von Willebrand disease
d) Kiesselbach plexus bleeding
e) Juvenile nasopharyngeal angiofibroma
106. e) Juvenile nasopharyngeal angiofibromas peak in adolescent and
preadolescent boys and also noted less than 2 years of age. He has no bleeding
disorder, therefore, answers (a) and (c) are wrong. Answer (d) is wrong
because the incidence of Kiesselbach plexus bleeding decreases during
adolescence.
107. The preferred initial diagnostic study in patients with a juvenile
nasopharyngeal angiofibroma is:
a) AP x-ray of the nose
b) Lateral x-ray of the nose
c) Excisional biopsy
d) Examination under anesthesia
e) CT scan with contrast
107. e) CT scan with contrast or MRI is the initial procedure of choice.
Arteriography, embolization, and extensive surgery may be required.
108. The preferred therapy to eradicate streptococcal carriage is:
a) Penicillin
b) Amoxicillin
c) Erythromycin
d) Clindamycin
e) Azithromycin
109. A 2-year-old boy appears with fever, sore throat, neck pain, drooling,
and decreased oral intake for the last 48 hours. A physical examination
reveals neck stiffness, torticollis, muffled voice, and bulging of the
posterior pharyngeal wall. The most likely diagnosis is:
a) Laternal pharyngeal abscess
b) Peritonsillar abscess
c) Acute epiglotittis
d) Meningitis
e) Retropharyngeal abscess
109. e) Please remember, retropharyngeal nodes involutes after 5 years of age.
Retropharyngeal abscess is common in less than 3-4 years of age and presents
with bulging of posterior pharyngeal wall in less than 50% of cases. The
patient may also appear with stridor, respiratory distress, and refuse to move
the neck.
110. A 2½-year-old girl appears with a fever, dysphagia, and enlarged
cervical lymph nodes. A physical examination reveals a bulging of the left
lateral pharyngeal wall and medial displacement of the left tonsil. The
most likely diagnosis is:
a) Acute adenoiditis
b) Peritonsillar abscess
c) Acute pharyngitis
d) Acute left tonsillitis
e) Left lateral pharyngeal abscess
111. A 12-year-old boy appears with a fever, sore throat, dysphagia, and
trismus for the last 2 days. He was diagnosed with a viral acute
tonsillopharyngitis 5 days ago. A physical examination reveals a right
tonsillar bulging with displacement of the uvula to the left.
The most likely diagnosis is:
a) Right peritonsillar abscess
b) Left peritonsillar abscess
c) Retropharyngeal abscess
d) Acute right tonsillitis
e) Acute uvulitis
111. a) Right peritonsillar abscess; this is common in adolescents. There is
usually the recent history of acute tonsillopharyngitis. An assymmetric tonsillar
bulging is diagnostic.
112. A 3-year-old girl appears in the ER with a history of sudden
development of fever and sore throat for the last 6 hours. She developed
dysphagia, drooling, and dyspnea for the last 1 hour. The child is sitting
upright, learning forward with the chin up, and opening mouth while
bracing on the arms. She was completely asymptomatic prior to this
episode. Her immunization status in unknown. Recently, her parents
immigrated from a developing country. The most likely diagnosis is:
a) Acute tonsillar abscess
b) Acute epiglottitis
c) Acute retropharyngeal abscess
d) Acute adenoiditis
e) Acute peritonsillar abscess
112. b) Acute epiglottitis; less common in the USA due to H. influenzae type b
vaccinations but still common in unimmunized or underimmunized children
between 2-4 years of age.
113. A 2-year-old girl appears in the ER at night with a sudden onset of
barking, metallic cough, and respiratory distress for the last 2 hours. She
experienced a mild coryza and hoarseness for the last 24 hours. The child
is asymptomatic during the day except for the cough and hoarseness. She
experienced a similar episode about 2 weeks ago. Mother denies history of
fever. The child appears anxious and frightened. A physical examination
reveals bilateral noisy inspiratory sounds and mild intercostal retractions.
The most likely diagnosis is:
a) Spasmodic croup
b) Acute infections laryngitis
c) Bacterial tracheitis
d) Laryngomalacia
e) Acute infectious laryngotracheobronchitis
113. a) The patients with spasmodic croup presents without fever and viral
infections. The allergy and psychological factors are important. The children
are usually symptomatic during the evening and nights. Laryngoscopy usually
reveals pale, watery edematous larynx but normal epithelium. In acute
infectious laryngotracheobronchitis, the laryngeal epithelium appears
erythematous, edematous, and destroyed.
114. The most common organism in patients with a bacterial tracheitis is:
a) Nontypable H. influenzae
b) Moraxella catarrhalis
c) Streptococcus pneumoniae
d) Pseudomonas aeruginosa
e) Staphylococcus aureus
114. e) S. aureus; answers (a) and (b) also cause bacterial tracheitis.
115. The most common organism in patients with an infectious upper
airway obstruction is:
a) Mycoplasma pneumoniae
b) H. influenzae type b
c) Streptococcus pyogenes
d) Influenza viruses
e) Parainfluenza viruses
115. e) Parainfluenza viruses (types 1, 2, and 3) are responsible for 75% of
cases.
116. The most common organism in patients with an epiglottitis and
underimmunized or unimmunized is:
a) Streptococcus pyogenes
b) Streptococcus pneumoniae
c) Mycoplasma pneumoniae
d) Parainfluenza viruses
e) Haemophilus influenzae type b
117. The most common organism in patients with an epiglottis and
immunized is:
a) Haemophilus influenzae type b
b) Staphylococcus epidermidis
c) Streptococcus pyogenes
d) Pseudomonas aeruginosa
e) Parainfluenza viruses
117. c) Streptococcus pyogenes, S. pneumoniae, and S. aureus are the most
common organisms.
118. The most common organism in patients with a croup
(laryngotracheobronchitis) is:
a) Influenza type A
b) Adenovirus
c) Respiratory syncitial virus
d) Mycoplasma pneumoniae
e) Parainfluenzae viruses
118. e) Parainfluenzae viruses (types 1 and 2); type 3 causes bronchiolitis and
pneumonia.
119. A 3-year-old boy appears with a barking cough, hoarseness, low-
grade fever, and dyspnea for the last 6 hours. The child has been suffering
from rhinorrhea, mild cough, pharyngitis, and fever for the last 3 days.
His condition worsens with agitation and crying. A physical examination
reveals tachypnea, mild retractions, inflamed pharynx, coryza, and hoarse
voice. The most likely diagnosis is:
a) Acute epiglottitis
b) Spasmodic croup
c) Croup
d) Acute pharyngitis
e) Acute infectious laryngitis

120. The most common congenital laryngeal anomaly in infant is:


a) Laryngomalacia
b) Subglottic stenosis
c) Subglottic hemangioma
d) Laryngeal web
e) Vocal cord paralysis
121. A full term newborn appears with low pitched, inspiratory stridor at
birth. The stridor worsens with crying, agitation, and feeding. The
newborn is NPO and receiving intravenous fluids. He was born by an
elective cesarean section with Apgar scores are 8 and 9 at 1 and 5 minutes
respectively. The most likely diagnosis is:
a) Vocal cord paralysis
b) Laryngeal web
c) Laryngeal atresia
d) Laryngeal nodule
e) Laryngomalacia
122. e) The infants with laryngomalacia has a low pitched, inspiratory stridor,
that worsens with crying, agitation, and feeding. The infants with vocal cord
paralysis, laryngeal web, and laryngeal atresia usually appear with high pitched
cry.

123. The preferred diagnostic study in infants with laryngomalacia is:


a) AP x-ray of the neck
b) Lateral x-ray of the neck
c) Flexible laryngoscopy
d) Barium swallow of the esophagus
e) Flexible bronchoscopy
224. The pulmonary arteriovenous fistulas are most commonly present is:
a) Williams syndrome
b) Down syndrome
c) Turner syndrome
d) Prader-Willi syndrome
e) Osler-Weber-Rendu syndrome
125. e) Osler-Weber-Rendu syndrome (hereditary hemorrhagic telangiectasia
type 1) is associated with angiomas of the GI tract, liver, nasal and buccal
mucous membranes.

126. A 5-month-old boy appears with respiratory distress for the last 24
hours. He had two episodes of pneumonia and four episodes of otitis media
and six episodes of diarrhea. He is delayed for his growth and
development. A physical examination reveals bilateral rales and right
otitis media. All of the following findings are true for this patient except:
a) Total CBC lymphocyte counts are 1600/mm3.
b) Chest x-ray reveals bilateral infiltrates.
c) Blood culture is positive for Streptococcus pneumoniae.
d) Serum immunoglobulin levels are elevated.
127.. The most common cause of bronchiectasis in the developed countries
is:
a) Cystic fibrosis
b) Ciliary dyskinesia
c) TB
d) Measles
e) Right middle lobe syndrome
128. The preferred diagnostic study in patients with bronchiectasis is:
a) Chest x-ray (AP view)
b) Bronchoscopy
c) Chest x-ray (lateral view)
d) Bronchography
e) CT scan
128. e) Thin-section high-resolution CT scanning has replaced bronchography
as the gold standard. CT scan typically reveals cylindrical (“tram lines,”
“signet ring appearance”), varicose (bronchi with “beaded contour”), cystic
(cysts appear in “strings and clusters”), or mixed types.
129. The most common combination of pathogens causing pneumonia is:
a) RSV and Mycoplasma pneumoniae
b) RSV and Chlamydia trachomatis
c) Streptococcus pneumonia and S. aureus
d) Streptococcus pneumonia and C. trachomatis
e) Streptococcus pneumonia and RSV
129. e) Streptococcus pneumoniae with either RSV or Mycoplasma pneumonia
130. The most common causes of empyema are:
a) S. aureus and S. pneumoniae
b) S. aureus and Pseudomonas aeruginosa
c) S. pneumoniae and RSV
d) S. pneumoniae and Mycoplasma pneumoniae
e) S. pneumoniae and Chlamydia trachomatis
131. The top five most common clinical manifestations in patients with
cystic fibrosis are the following except:
a) Acute or persistent respiratory symptoms
b) Failure to thrive (or malnutrition)
c) Abnormal stools
d) Meconium ileus
e) Nasal polyps
131. e) Nasal polyps or sinus diseases occur less frequently than rectal
prolapse, electrolytes or acid-base abnormalities. Answers (a), (b), (c), and (d)
are in the order of decreasing frequencies (i.e., answer (a) is the most common
presentation). Hepatobiliary diseases occur less frequently than nasal polyps
132. The parents noticed salty taste when they kiss the child. The most
likely diagnosis is:
a) Facial eczema
b) Acrodermatitis enteropathica
c) SLE
d) Cystic fibrosis
e) Ectodermal dysplasia

132. The diagnostic criteria in patients with cystic fibrosis (CF) include all
of the following except:
a) Presence of typical clinical manifestations and two elevated sweat chloride
concentrations (60 mEq/L or more) performed on separate days
b) A history of cystic fibrosis in a sibling and identification of two CF
mutations
c) A positive newborn screening test and two elevated sweat chloride
concentrations performed on separate days
d) A history of CF in a sibling and an abnormal nasal potential difference
measurement
e) A history of CF in a sibling and positive newborn screening test
132. e) Diagnostic criteria for cystic fibrosis are the following: Presence of
typical clinical manifestations (respiratory, GI, or GU) or history of CF in a
sibling or positive newborn screening test
PLUS Laboratory evidence for CFTR (cystic fibrosis transmembrane regulator)
dysfunction: Two elevated sweat chloride concentrations performed on
separate days or identification of two CF mutations or an abnormal nasal
potential difference measurement

133. The following statement is not true in patients with cystic fibrosis:
a) Sexual development is often delayed.
b) Sexual function is usually impaired.
c) Majority of males are azoospermic.
d) Female fertility rate is reduced.
e) Females may have cervicitis and accumulation of tenacious mucus in the
cervical canal.
133. b) Sexual function is usually unimpaired. Sexual development is often
delayed by an average of 2 years. Females may experience secondary
amenorrhea. Pregnancy is well tolerated in CF women with good pulmonary
functions.
134.. All of the following complications are due to use of aminoglycosides
in patients with cystic fibrosis except:
a) Hypomagnesemia
b) Hyperuricemia
c) Hearing loss
d) Vestibular dysfunction
e) Renal tubular dysfunction
134. b) Hyperuricemia and colonic stricture can occur due to the use of very
large doses of pancreatic extracts.

135. A child has cystic fibrosis. The PFTs (pulmonary function tests) are
not performed until:
a) 1-2 years of age
b) 3-4 years of age
c) 5-6 years of age
d) 7-8 years of age
e) 9-10 years of age
135. c) 5-6 years of age; by this time most patients have obstructive pulmonary
disease. A decrease in mid-maximal flow rate is an early finding that suggests
small airway obstructions.
136. The following statement is not true about pulmonary function tests in
patients with cystic fibrosis:
a) Restrictive lung changes occur early in the disease process.
b) Obstructive lung changes typically present by 5-6 years of age.
c) Small airway obstruction is manifested early by decrease in mid-maximal
flow rate
d) Residual volume and functional residual capacity are increased early in the
course of the disease.
e) Presence of an obstructive lung disease and modest response to
bronchodilator are diagnostic of cystic fibrosis.
136. a) Restrictive lung changes occur late in the disease process (i.e.,
decreased total lung capacity and vital capacity) and are due to extensive lung
injuries and fibrosis
137. The presence of the following organisms on culture of lower airways
or sputum is diagnostic of cystic fibrosis in children:
a) Streptococcus pneumoniae
b) Mucoid forms of Pseudomonas
c) Mycoplasma pneumoniae
d) RSV
e) Chlamydia trachomatis
137. b) Mucoid forms of pseudomonas; the presence of Staphylococcus aureus,
Pseudomonas aeruginosa, or Burkholderia cepacia organism suggests CF
138. The most common mechanism of arterial hypoxemia in pulmonary
disease is:
a) Ventilation-perfusion mismatch
b) Ventilation failure
c) Perfusion abnormalities
d) Arterial obstruction
e) Venous obstruction
138. a) Ventilation-perfusion mismatch (or decreased) i.e., alveolar PO2 is
lower (e.g., 60) than normal (e.g., 100), therefore blood that passes through this
unit of alveoli achieve lower (e.g., 89%) than normal (e.g.,95-100%) oxygen
saturation. Supplemental oxygen therapy increases alveolar PO2 and arterial
PO2.

139. All of the following statements are true in infants who are sleeping
supine or on their side except:
a) They develop less fever at 1 month of age.
b) They develop less stuffy nose at 6 months of age.
c) They have less trouble sleeping at 6 months of age.
d) They have less outpatient visits for ear infection at 1 and 2 months for
sleeping supine.
e) They have less outpatient visits for ear infection at 3 months for sleeping on
their side.
139. d) Infants have less outpatient visits for ear infection at 3 and 6 months for
those who are sleeping supine.
140. The preferred therapy in patients with antral choanal polyps is:
a) Surgery
b) Intranasal steroid sprays
c) Systemic steroids
d) Local decongestants
e) Systemic decongestants
140. The preferred therapy in patients with nasal polyps is:
a) Intranasal steroid sprays
b) Systemic steroids
c) Local decongestants
d) Systemic decongestants
e) Functional endoscopic sinus surgery
140. e) Functional endoscopic sinus surgery includes removal of polyps and
other associated nasal disease. Answers (a) and (b) provide some shrinkage in
nasal polyps. Answers (c) and (d) do not shrink nasal polyps but reduce
mucosal edema.
141. The most common organism for common cold is:
a) Coronavirus
b) Adenovirus
c) Influenza virus
d) Rhinovirus
e) Respiratory syncytial virus
142. The most common complication of cold is:
a) Sinusitis
b) Asthma
c) Bronchiolitis
d) Croup
e) Otitis media
143. The best method of prevention of common cold is:
a) Influenza vaccine
b) Vitamin C
c) Echinacea
d) Good handwashing
e) Chicken soup
144. A young woman appears with recurrent pneumothorax during
menstruation. The investigation of the following organ is indicated:
a) Uterus
b) Fallopian tubes
c) Ovary
d) Vagina
e) Diaphragm
144. e) Diaphragmatic defect results in passage of intra abdominal air into the
pleural cavity. This girl has catamenial pneumothorax.
145. The most common organism isolated in patients with pneumothorax,
especially in infants is:
a) S. aureus
b) S. epidermidis
c) Streptococcus pneumoniae
d) Pseudomonas aeruginosa
e) Group B Streptococcus
145. a) Staphylococcus aureus; Mycoplasma pneumoniae infection can cause
pneumothorax
146. The most common cause of chylothorax is:
a) Chest injury
b) Metastatic lung disease
c) Child abuse
d) Thrombosis of thoracic duct
e) Rupture of thoracic duct during cardiac surgery
146. e) Rupture of thoracic duct during cardiac surgery for complex congenital
heart diseases. Answers (a), (b), (c), and (d) can cause chylothorax
147. The most of inheritance in patients with Jeune syndrome is:
a) Autosomal recessive
b) Autosomal dominant
c) X-linked recessive
d) X-linked dominant
e) Multifactorial
147. a) Autosomal recessive. Jeune syndrome is called asphyxiating thoracic
dystrophy (thoracic-pelvic-phalangeal dystrophy).
148. The most common cause of community-acquired bacterial pneumonia
is:
a) Haemophilus influenzae type b
b) Streptococcus pneumoniae
c) Staphylococcus aureus
d) Pseudomonas aeruginosa
e) E. Coli

149. The most common cause of community-acquired otitis media is:


a) Pseudomonas aeruginosa
b) E. coli
c) Haemophilus influenzae type b
d) Staphylococcus aureus
e) Streptococcus pneumonia
150. A child appears with upper respiratory tract infection and a
characteristic rash. The rash appeared within 24-48 hours after the onset
of symptoms. The rash began around the neck and subsequently involved
trunk and extremities. The rash is diffuse, finely papular, and
erythematous. The bright red discoloration of the skin, which blanches on
pressure. The skin feels rough and has goose-pimple appearance. The rash
is more prominent along the creases of the elbows, axillae, and groins. She
also has pharyngitis. The most likely diagnosis is:
a) Kawasaki disease
b) Rubella
c) Roseola
d) Toxic shock syndrome
e) Scarlet fever

151. The most common cause of bacterial pharyngitis is:


a) Group C Streptococcus
b) Group G Streptococcus
c) Group A Streptococcus
d) Corynebacterium diphtheriae
e) Arcanobacterium haemolyticum
152. The ethmoid sinuses reach their maximum size during:
a) 0-1 year of age
b) 1-3 years of age
c) 4-7 years of age
d) 7-14 years of age
e) 14-18 years of age
152. d) 7-14 years of age. All other sinuses (e.g., frontal, maxillary,
sphenoid) reach their maximum size after puberty.
153. A child develops retractions and respiratory distress after extubation
following a surgical procedure. A physical examination reveals inspiratory
stridor and wheezing in both lungs. The next step in management is:
a) Chest physiotherapy
b) Selective intubation in right lung
c) Selective intubation in left lung
d) Intravenous corticosteroids
e) Racemic epinephrine aerosols
153. e) Racemic epinephrine aerosols therapy are effective in patients
with postoperative stridor. In severe cases, reintubation is indicated.
154. A 2-hour-old baby becomes cyanotic. The infant is transferred to
NICU from regular nursery and is placed under oxyhood. The following
test can distinguish cardiac from pulmonary disease:
a) Chest x-ray
b) EKG
c) Pulmonary function test
d) Hyperoxia test
e) Chest CT scan
154. d) Hyperoxia test (i.e., infant receives 100% oxygen and arterial
blood gas is performed after 15-20 minutes). If the Pao2 is above 150
mm Hg, cyanotic cardiac anomaly is probably excluded and pulmonary
cause is most likely etiology of hypoxia. Echocardiogram is the best
diagnostic study to diagnose cardiac anomaly.
155. The hyperoxia test in a newborn reveals Pao2 is 200 mm Hg. The
most likely diagnosis is:
a) Tricuspid atresia
b) Transposition of great vessels
c) Pulmonary disease
d) Truncus arteriosus
e) Total anomalous pulmonary venous return

156. Arterial blood gas (ABG) in patients with BPD (bronchopulmonary


dyplasia) usually reveals the following result:
a) Baseline hypoxemia, elevated bicarbonate level, and reduced Pco2 level
b) Normal Po2 level, elevated bicarbonate level, and elevated Pco2 level
c) Elevated Po2 level, elevated bicarbonate level, and elevated Pco2 level
d) Baseline hypoxemia, decreased bicarbonate level, and elevated Pco2 level
e) Baseline hypoxemia, elevated bicarbonate level, and elevated Pco2 level
156. e) ABG in BPD patients usually reveals baseline hypoxemia (requiring
oxygen to maintain oxygen saturation above 90%), elevated bicarbonate level,
and elevated Pco2 level (due to chronic respiratory insufficiency).
157. The following metabolic and respiratory conditions are noted in
patients with BPD:
a) Respiratory acidosis and metabolic acidosis
b) Respiratory alkalosis and metabolic alkalosis
c) Respiratory acidosis and metabolic alkalosis
d) Absence of respiratory acidosis but presence of metabolic alkalosis
e) Presence of respiratory acidosis but absence of metabolic alkalosis
592. The treatment in patients with BPD includes all of the following
except:
a) High calorie intake
b) Supplemental oxygen
c) Adequate fluid
d) Furosemide prn
e) Treatment for gastroesophageal reflux, if present
592. c) Fluid should be restricted along with high calorie intake in
patients with BPD. Gastroesophageal reflux is common in patients with
BPD. Inhaled glucocorticoids can be used in young children with BPD.
593. An infant with BPD develops wheezing. The infant’s condition worsen
with beta-agonist therapy. Most likely cause is:
a) BPD with concomitant airway malacia
b) BPD with cardiac failure
c) BPD with bronchial smooth muscle irritation
d) BPD with bronchial smooth muscle hypertrophy
e) BPD with inflammation
593. a) BPD with concomitant airway malacia usually worsen with beta-
agonist therapy because beta-agonist relaxes bronchial smooth
muscles resulting in airway collapse due to airway malacia.
594. The following immunoprophylaxis should be given to all patients with
BPD during winter months (October to April):
a) Influenza vaccine
b) Synagis vaccine
c) Meningococcal vaccine
d) Varicella vaccine
e) Oral amoxicillin prophylaxis
594. b) Synagis vaccine should be given to prevent RSV infectins to all
premature babies with or without BPD.
595. Analysis of the pleural fluid after thoracocentesis reveals a milky fluid
containing fat, protein, and lymphocytes. The definitive test to diagnose
the chylous fluid is:
a) Examination of the fluid under microscope
b) Measurements of sodium in the fluid
c) Measurements of chloride in the fluid
d) Measurements of cholesterol in the fluid
e) Quantitative measurements of triglyceride in the fluid
595. e) Triglyceride levels are elevated in chylous fluid. Cholesterol
levels are elevated in chronic serous effusions.
596. Majority of infants younger than 1 year of age with chylothorax have
the following outcome:
a) Placement of chest tube for a prolonged period of time
b) Need surgical intervention as soon as possible
c) Develop chronic lung disease
d) Develop pleural thickening
e) Spontaneously recover
596. e) More than 50% of cases recover spontaneously. Repeated
aspirations may be needed to release the pressure. However, chyle
accumulates rapidly and repeated aspirations cause loss of calories,
protein, and lymphocytes.
597. The therapy in patients with chylothorax includes all of the following
except:
a) Low-fat diet
b) High-protein diet
c) Adequate salt
d) Diuresis
e) Increased caloric intake
597. c) Salt restrictions are required in most patients. Fat soluble
vitamins (A, D, E and K) should be added.
598. A child has chylothorax for the last 4 weeks. The child is on a
mechanical ventilator and receiving TPN. Conservative therapy failed.
The child required several pleural fluid aspirations. The preferred
definitive therapy is:
a) Pleuroperitoneal shunt placement
b) Inhalation of nitric oxide (20 ppm)
c) Subcutaneous octreotide
d) Pressure controlled ventilation with positive end-expiratory pressure
e) Surgical ligation of the thoracic duct
693. A child with BPD (bronchopulmonary dysplasia) is treated with
furosemide. Furosemide therapy can cause the
following condition:
a) Metabolic acidosis
b) Metabolic alkalosis
c) Respiratory alkalosis
d) Respiratory acidosis
e) Both respiratory and metabolic acidosis
694. The respiratory compensation for a metabolic acidosis usually takes:
a) 12-24 hours
b) 24-48 hours
c) 48-72 hours
d) 72-96 hours
e) 4-5 days
695. The metabolic compensation for a respiratory acidosis usually takes:
a) 1- 2 days
b) 2 - 3 days
c) 3-4 days
d) 4-5 days
e) 5-6 days
759. A child is on a mechanical ventilator and is receiving 80% oxygen
(FIO2 0.8). ABG (arterial blood gas) reveals Po2 of 100 mm Hg, Pco2 of
40, and pH 7.35. The alveolar-arterial oxygen gradient (A-a gradient) is:
a) 306
b) 350
c) 400
d) 420
e) 480
759. d)
A-a gradient is 420 (i.e., alveolar oxygen gradient minus arterial oxygen
gradient).
Arterial Po2 is 100 mm Hg.
Calculation of alveolar oxygen gradient: PAO2 = [FIO2 (Pb-PH2O)]-
(Paco2 / R) = [0.8 (760-47)]-(40 / 0.8) = (0.8 x 713)-(50) = 570-50 = 520
(FIO2 = 0.8 i.e., 80% oxygen, Pb is 760 i.e., barometric pressure, PH2O
is 47 i.e., water vapor pressure, Paco2 is 40, R is 0.8 i.e., respiratory
quotient, PAo2 = alveolar oxygen gradient). A-a gradient = 520-100 =
420 mm Hg. Normal A-a gradient in a healthy person is less than 10
mm Hg. If a person is receiving 100% oxygen (F102 1.0) and A-a
gradient is more than 300 mm Hg, patient needs intubation.
760. A child develops respiratory acidosis. The preferred method to
distinguish between the intrinsic lung disease and the poor respiratory
effort is:
a) Arterial pH
b) Arterial Po2
c) Arterial Pco2
d) Venous pH
e) Alveolar-arterial oxygen gradient
760. e) Alveolar-arterial oxygen gradient that is increased in intrinsic
lung disease unlike in poor respiratory effort
761. Acute respiratory alkalosis manifests with all of the following findings
except:
a) Bradycardia
b) Chest tightness
c) Circumoral numbness
d) Light headedness
e) Paresthesias of the extremities
761. a) Bradycardia is absent. Patient usually have palpitations. Less common
findings are cramps, tetany, syncope, and seizures. The lightheadedness and
syncope are due to the reduction in the cerebral blood flow. The paresthethias,
tetany, and seizures may be partially due to the decreased ionized calcium
because alkalemia causes more calcium to bind with albumin
762. The following condition produces tissue hypoxia without hypoxemia
is:
a) Bacterial pneumonia
b) Viral pneumonia
c) Diaphragmatic hernia
d) Cyanotic heart disease
e) Carbon monoxide poisoning
762. e) Carbon monoxide poisoning, severe anemia, and congestive heart
failure produce tissue hypoxia without hypoxemia.
763. The following statement is not true about pulse oximetry:
a) It measures partial pressure of oxygen
b) Accuracy depends upon the adequate tissue perfusions
c) Abnormal hemoglobin and nail polish can affect results
d) Pulse oximetry is not very sensitive in detecting a mildly low Po2
e) Pulse oximetry is not adequate to eliminate hypoxia as a cause of respiratory
alkalosis
763. a) Pulse oximetry measures real-time oxygen saturation and does
not measure partial pressure of oxygen. Only ABG (arterial blood gas)
is adequate to eliminate hypoxia as a cause of a respiratory alkalosis.
Pulse oximetry does not give direct measurements of CO2 tension,
bicarbonate level, or acid-base status.
845. Most common anatomical cause of obstructive sleep apnea and
hypoventilation (OSA/H) in children is:
a) Enlarged tongue
b) Anterior nasal stenosis
c) Micrognathia
d) Midface hypoplasia
e) Adenotonsillar hypertrophy
845. e) Adenotonsillar hypertrophy. Answer (d) is noted in Down, Cruzon, and
Apart syndromes
846. Most common functional process contributing to obstructive sleep
apnea and hypoventilation (OSA/H) is:
a) Rapid eye movement sleep
b) Generalized hypotonia
c) Birth asphyxia
d) Cerebral palsy
e) Narcotics
846. a) Rapid eye movement (REM) sleep occurs about one fourth of a
typical night sleep. During REM sleep, apnea frequency, apnea
duration, and degree of hypoxia are severe in patients with OSA/H.
Answer (b) is noted in Down syndrome
847. Risk factors for obstructive sleep apnea and hypoventilation are all of
the following except:
a) Obesity
b) Chronic rhinitis
c) Asthma
d) White race
e) Positive family history
848. Chronic hypoxia that develops due to obstructive sleep apnea and
hypoventilation (OSA/H) resulting in all of the following conditions
except:
a) Growth failure
b) Polycythemia
c) Arrythmias
d) Death
e) Left ventricular failure
848. e) Chronic hypoxia due to OSA/H causes pulmonary hypertension and
right ventricular failure.
849. Most common symptom in patients with obstructive sleep apnea and
hypoventilation is:
a) Habitual snoring
b) Chronic mouth breathing
c) Restlessness during sleep
d) Frequent awakenings during sleep
e) Abnormal position during sleep
849. a) Habitual snoring is the most common symptom in patient with OSA/H.
Please remember, not all children with snoring are at risk for the development
of OSA/H. Most children with OSA/H breathe normally while awake. Answers
(b), (c), (d), and (e) are all clinical manifestations in patients with OSA/H.
Abnormal position during sleep (e.g., hyperextended neck or prone with the
bottom up in the air) is required to maintain an upper airway.

850. A patient with Down syndrome develops pulmonary hypertension and


right ventricular failure. He has a history of habitual snoring. Physical
examination reveals obesity and pectus excavatum deformities. EKG
reveals a right ventricular hypertrophy. Most likely diagnosis is:
a) Large VSD
b) Hypoplastic lungs
c) Pleural effusion
d) Endocardial cushion defect
e) Obstructive sleep apnea / hypoventilation
851. Preferred diagnostic study in patients with obstructive sleep apnea /
hypoventilation is:
a) Arterial blood gas
b) Lateral x-ray of the neck
c) Audio / video taping during sleep
d) Home polysomnography
e) An overnight recording of multiple physiologic sensors during sleep
852. The following findings may be noted in patients with obstructive sleep
apnea and hypoventilation except:
a) Anemia
b) Right ventricular hypertrophy
c) Dysfunction in echocardiography
d) Respiratory acidosis with a metabolic alkalosis
e) Enlarged adenoid in the lateral x-ray of the neck
852. a) Anemia is not present. Polycythemia due to chronic hypoxia may be
noted. Polycythemia and answer (d) support the diagnosis but are absent in
majority of the pediatric patients.
853. Most severe complication of obstructive sleep apnea and
hypoventilation is:
a) Pulmonary hypertension
b) Daytime somnolence
c) Decreased memory
d) Decreased cognitive function
e) Increased behavioral problems
854. Preferred therapy in patients with obstructive sleep apnea and
hypoventilation (OSA/H) is:
a) Adenotonsillectomy in patients with a normal adenoid and tonsils
b) Nasal CPAP (continuous positive airway pressure)
c) Nasal steroids
d) Medroxyprogesterone acetate
e) Depends on the underlying abnormalities
854. e) Depends on the underlying abnormalities (e.g, -
- adenotonsillectomy is performed in patients with hypertrophied adenoid and
tonsils;
- maxillomandibular reconstruction surgery for children with craniofacial
disorders;
- nasal steroids can reduce snoring in some children; -
- medroxyprogesterone acetate increases ventilatory drive but is effective only
daytime hypoventilation associated with obesity-hypoventilation syndrome;
- nasal CPAP can be used in children in whom adenotonsillectomy failed;
- tracheotomy is indicated in severe cases.)
855. The adverse effect of medroxyprogesterone acetate when used in
female patients with OSA/H is:
a) Amenorrhea
b) Dysmenorrhea
c) Premenstrual syndrome
d) Pubertal development
e) Abnormal uterine bleeding
855. d) Pubertal development and growth are affected by medroxyprogesterone
acetate. This medication is also used for an excessive menstrual bleeding,
primary amenorrhea, and secondary amenorrhea.
856. A complete resolution of symptoms may not occur after
adenotonsillectomy in all of the following conditions except:
a) Down syndrome
b) Extreme obesity
c) Cerebral palsy
d) Arnold chiari malformation
e) Onset of symptoms after 2 years of age
856. e) Incomplete resolution (i.e., symptoms persist even after
adenotonsillectomy) occurs in children who had onset of symptoms
before 2 years of age.

857. A child appears with a rapid enlargement of the right tonsil. He also
has fever, weight loss, and ipsilateral lymphadenopathy. Physical
examination reveals a normal left tonsil and an abnormal right tonsil.
Most likely diagnosis is:
a) Acute tonsillitis
b) Chronic tonsillitis
c) Peritonsillar abscess
d) Tonsillar abscess
e) Lymphoma
857. e) Lymphoma; a rapid unilateral enlargement of one tonsil when
associated with fever, weight loss, and lymphadenopathy, is highly
indicative of tonsillar malignancy. Lymphoma is the most common
malignancy of tonsil in children
858. Major virulence factor of group A beta-hemolytic Streptococci
(GABHS) is:
a) M protein
b) N protein
c) A protein
d) B protein
e) C protein
858. a) M protein produces resistance to phagocytosis by neutrophils.
859. The only accurate method to diagnose sinusitis is:
a) CT scan of the sinuses
b) Plain x-ray of the sinuses
c) History and physical examination
d) Transillumination of the sinuses
e) Sinus aspirate culture
863. A child has virus-induced reactive airway disease. She has been
coughing for the last 2 weeks. Most effective therapy is:
a) Vitamin C
b) Bronchodilator
c) Guaifenesin
d) Codeine
e) Dextromethorphan hydrobromide
864. Most common site of bleeding from the nose is:
a) Nasopharynx
b) Fractured nasal bone
c) Floor of the nose
d) Nasal sinuses
e) Kieselbach plexus
864. e) Kieselbach plexus is located in the anterior part of nasal septum and
consists of branches from the internal carotid (e.g., anterior and posterior
ethmoidal arteries) and external carotid (e.g., sphenopalatine and terminal
branches of internal maxillary arteries.)
865. Most common cause of nose bleed from anterior septum is:
a) Digital trauma
b) Sinusitis
c) Foreign bodies
d) Dry air
e) URI
866. The initial treatment in patients with nose bleed is:
a) Cold compress on the nose
b) Oxymetazoline drops
c) Neo-Synephrine drops
d) Anterior nasal packing
e) The nares should be compressed, an upright position, and head tilted forward
866. e) Most nose bleeds stop spontaneously within a few minutes. The nares
should be compressed, an upright position, kept as quiet as possible, and head
titled forward to avoid blood goes back into the throat and ultimately
swallowed into the stomach. Answers (a), (b), (c), and (d) are used if the
previous therapies fail. Some patients need posterior nasal packing when the
bleeding occurs from the back of the nasal cavity. Anterior and posterior nasal
packing should be done by an ENT specialist. A cautarization by silver nitrate
may be required.
867. A child has nose bleeds during the winter months. He lives in an
apartment building. There is plenty of heat supply in the apartment. The
preferred preventive measure to avoid nose bleeds in this patient is:
a) Cold compress on the nose
b) Neo-Synephrine drops
c) Oxymetazoline drops
d) A room humidifier
e) Prophylactic anterior nasal packing
867. d) A room humidifier, saline drops, and petrolatum (vaseline) applied to
the septum may prevent epistaxis.
868. A child appears with mouth breathing and hyponasal speech.
Physical examination reveals widen bridge of the nose, eroded adjacent
bony structures, fleshy, glistening, gray, grapelike masses noted between
the nasal turbinates and the septum. Most likely diagnosis is:
a) Syphilis
b) Ethmoidal polyps
c) Sphenoidal polyps
d) Frontal encephalocele
e) Juvenile nasopharyngeal angiofibroma
868. b) This is a characteristic presentation of ethmoidal polyps. Prolonged
presence of ethmoidal polyps can cause widen bridge of the nose and erode
adjacent bony structures
869. A child appears in the ER with unilateral nasal discharge. Physical
examination reveals a small piece of crayon and mildly swollen
surrounding tissue. Preferred therapy is:
a) Keep the foreign body but start antibiotic therapy to prevent an infection
b) Remove the foreign body under general anesthesia
c) Topical steroid therapy is to reduce the swelling
d) Topical Neo-Synephrine drops
e) Remove the foreign body by forcep or nasal suction under local anesthesia
869. e) Remove the foreign body by forcep or nasal suction under local
anesthesia. General anesthesia is used if there is marked swelling,
tissue overgrowth, or bleeding. Infection improves promptly when the
foreign body is removed
870. Most common congenital anomaly of the nose is:
a) Choanal atresia
b) Hypoplastic nose
c) Single nasal cavity
d) Upturned nose
e) Long nose
871. A newborn appears cyanotic at birth. She becomes pink when crying
and again becomes cyanotic when stops crying. Most likely diagnosis is:
a) Choanal stenosis
b) Mother received morphine
c) Perinatal asphyxia
d) Subarachnoid bleeding
e) Bilateral choanal atresia
872. Internal nasal airway doubles in size by:
a) 3 months of age
b) 6 months of age
c) 9 months of age
d) 1 year of age
e) 2 years of age
872. (b). By 6 months of age. Therefore, symptoms of nasal congestion
improve after 6 months of age in many children.
873. Acute bacterial sinusitis occurs after the following condition:
a) Acute otitis media
b) Chronic otitis media
c) Periorbital cellulitis
d) Orbital cellulitis
e) Viral upper respiratory tract infection
873. e) Viral upper respiratory tract infection; first, viral rhinosinusitis
develops (i.e., edema and inflammation within the sinuses), nose blowing
propels the nasal secretions into the sinus cavities, nasopharyngeal bacteria
enter the sinuses (normally cleared readily) and grow inside due to an
inadequate sinus drainage resulting in bacterial sinusitis
879. All of the following statements are true about the treatment in
patients with an acute sinusitis except:
a) Antibiotic therapy has substantial benefit in clinically diagnosed acute
bacterial sinusitis.
b) In uncomplicated cases, initial therapy is amoxicillin (45 mg/kg/day) for an
acute bacterial sinusitis.
c) Penicillin-allergic patients should receive cefuroxime axetil, cefpodoxime,
clarithromycin, or azithromycin.
d) A “high-dose” amoxicillin-clavulanate (80-90 mg/kg/day of amoxicillin
and 6.4 mg/kg/day of clavulanate)
should be given in patients who are at high risk (e.g., younger than 2 years of
age, daycare attendance, received antibiotic in the last 1-3 months).
e) Unresponsive patients should be referred to an ENT specialist for further
evaluation and maxillary sinus aspiration and culture.
879. a) It is not clear whether antibiotic therapy has substantial benefit in
clinically diagnosed acute bacterial sinusitis. A study reveals that a 14-day
treatment with amoxicillin, amoxicillin-clavulanate, or placebo did not affect
resolution and duration of symptoms, or days missed from school. About 50-
60% children will recover without antibiotic therapy in acute bacterial sinusitis.
However, the American Academy of Pediatrics recommend antibiotic therapy
to promote resolution of symptoms and to prevent suppurative complications.
The duration of antibiotic therapy is unclear but should be given 7 days after
resolution of symptoms.
880. The preferred method of prevention in sinusitis is:
a) Frequent hand washing
b) Influenza vaccine
c) Oseltamivir
d) Zanamivir
e) Use face mask
880. a) Frequent handwashing, avoiding persons with cold; influenza vaccine
can prevent only some cases because influenza is responsible for only a small
portion of all colds. Answers (c) and (d) have complications in children.
Answer (e) does not make sense.
Prpared &Revised By
Dr.Wahid Helmi Rifahie
Consultant pediatrician
Zarka Hospital –Dymiate-Egypt

Second Revision All answered

100 Questions with


answer&Eplaination

WhatsApp
01004313142
Which one of the following is the most common cause
of congenital hydrocephalus?
A. Craniosynostosis
B. Intra uterine meningitis
C. Aqueductal stenosis
D. Malformations of great vein of Galen

Correct answer : C. Aqueductal stenosis

The most common malignant neoplasm of infancy is:


A. Malignant teratoma
B. Neuroblastoma
C. Wilms’ tumor
D. Hepatoblastoma

Correct answer : B. Neuroblastoma

ln a child, non functioning kidney is best diagnosed


by:
A. Ultrasonography
B. IVU
C. DTPA renogram
D. Creatinine clearance

Correct answer : C. DTPA renogram


The most common cause of renal scarring in a 3 year
old child is:
A. Trauma
B. Tuberculosis
C. Vesicoureteral reflux induced pyelonephritis
D. Interstitial nephritis

Correct answer : C. Vesicoureteral reflux induced


pyelonephritis

A child with recurrent urinary tract infections is most


likely to show:
A. Posterior urethral valves
B. Vesicoureteric reflux
C. Neurogenic bladder
D. Renal and ureteric calculi

Correct answer : B. Vesicoureteric reflux


Vesicoureteric reflux is the most common cause of
urinary tract infections in childhood. (upto 50%)

Which of the following is the most common renal


cystic disease in infants?
A. Polycystic kidney
B. Simple renal cyst
C. Unilateral renal dysplasia.
D. Calyceal cyst

Correct answer : C. Unilateral renal dysplasia


The most common type of total anomalous pulmonary
venous connection is:
A. Supracardiac
B. Infracardiac
C. Mixed
D. Cardiac

Correct answer : A. Supracardiac

One of the intestinal enzymes that is generally


deficient in children following an attack of severe
infectious enteritis is:
A. Lactase
B. Trypsin
C. Lipase
D. Amylase

Correct answer : A. Lactase


Even in the normal intestine, the lactase activity is
limited. Hence it is most prone to become deficient
following an attack of infectious enteritis.
Eisenmenger syndrome is characterized by all except:
A. Return of left ventricle & right ventricle to normal
size
B. Pulmonary veins not distended
C. Pruning of peripheral pulmonary arteries
D. Dilatation of central pulmonary arteries

Correct answer : A. Return of left ventricle & right


ventricle to normal size
Right ventricular hypertrophy that develops in
Eisenmenger syndrome will not return to normal size.

Diagnosis of beta Thalassemia is established by:


A. NESTROFT Test
B. HbA1c estimation
C. Hb electrophoresis
D. Target cells in peripheral smear

Correct answer : C. Hb electrophoresis


NESTROFT Test – Naked Eye Single Tube Red Cell
Osmotic Fragility Test – used for screening for
Thalassemia
HbA1c – Used to assess long term glycemic control in
diabetics (blood sugar control over past 3 months)
Target cells – A feature of thalassemia, but not
diagnostic
The coagulation profile in a 13-year old girl with
Menorrhagia having von Willebrands disease is:
A. Isolated prolonged PTT with a normal PT
B. Isolated prolonged PT with a normal PTT
C. Prolongation of both PT and PTT
D. Prolongation of thrombin time

Correct answer : A. Isolated prolonged PTT with a


normal PT
In VWF, there is impairment of intrinsic coagulation
pathway. Hence there is isolated prolonged PTT.

The most common leukocytoclastic vasculitis


affecting children is:
A. Takayasu disease
B. Mucocutaneous lymph node syndrome (Kawasaki
disease)
C. Henoch–Schönlein purpura
D. Polyarteritis nodosa

Correct answer : C. Henoch–Schönlein purpura


HSP is the most common childhood vasculitis. It is
produces leukocytoclastic vasculitis.
Bart‘s hydrops fetalis is lethal because:
A. Hb Bart’s cannot bind oxygen
B. The excess alpha globin form insoluble precipitates
C. Hb Bart’s cannot release oxygen to fetal tssues
D. Microcytic red cells become trapped in the placenta

Correct answer : C. Hb Bart’s cannot release oxygen


to fetal tssues
Hb Barts is formed of 4 gamma chains. The oxygen
affinity is so high that it releases very little oxygen
into the fetal tissues.

A child with a small head, minor anomalies of the face


including a thin upper lip, growth delay, and
developmental disability can have all of the following,
except:
A. A chromosomal syndrome
B. A teratogenic syndrome
C. A mendelian syndrome
D. A polygenic syndrome

Correct answer : D. A polygenic syndrome


The features given can occur as a part of a
chromosomal / teratogenic / mendelian syndrome.
Polygenic inheritance is the answer of exclusion. In a
polygenic inheritance, multiple genes are involved in
the phenotypic expression. Some examples of
polygenic inheritance are hypertension and

An affected male infant born to normal parents could


be an example of all of the following, except
A. An Autosomal dominant disorder
B. An Autosomal recessive disorder
C. A polygenic disorder
D. A vertically transmitted disorder

Correct answer : A. An Autosomal dominant disorder


An autosomal dominant disorder if present in the
parents will always express itself phenotypically. So it
is not possible for normal parents to have an child
affected with an autosomal dominant disorder.

The process underlying differences in expression of a


gene according to which parent has transmitted is
called:
A. Anticipation
B. Mosaicism
C. Non penetrance
D. Genomic imprinting

Correct answer : D. Genomic imprinting


In genomic imprinting, a gene / group of genes on the
paternal or maternal chromosome is selectively
inactivated. The functional allele will be provided by
the counterpart.
Example:
Angelman syndrome – deletion of maternal gene
Prader Willi syndrome – deletion of paternal gene

A malignant tumor of childhood, that metastasizes to


bones most often is:
A. Wilm’s tumor
B. Neuroblastoma
C. Adrenal gland tumors
D. Granulosa cell tumor of ovary
.

Correct answer : B. Neuroblastoma


60-70% of metastasis in case of neuroblastoma are
skeletal metastasis

The most common etiological agent for acute


bronchiolitis in infancy is:
A. Influenza virus
B. Para influenza virus
C. Rhinovirus
D. Respiratory syncytial virus

Correct answer : D. Respiratory syncytial virus (It is


responsible for more than 50% cases of bronchiolitis)

Late onset hemorrhagic disease of newborn is


characterized by all of the following features except:
A. Usually occurs in cow-milk fed babies
B. Onset occurs at 4-12 week of age
C. lntracranial hemorrhage can occur
D. Intramuscular vitamin K prophylaxis at birth has a
protective role

Correct answer : A. Usually occurs in cow-milk fed


babies
Haemorrhagic disease of newborn is more common in
breast milk fed babies. (Breast milk is a poor source
of Vitamin K)

With reference to mumps which of the following is


true?
A. Meningoencephalitis can precede parotitis
B. Salivary gland involvement is limited to the
parotids
C. The patient is not infectious prior to clinical parotid
enlargement
D. Mumps orchitis frequency leads to infertility

Correct answer : A. Meningoencephalitis can precede


parotitis
Meningoencephalitis can occur 1 week before onset of
parotitis.

The earliest indicator of response after starting iron in


a 6-year-old girl with iron deficiency is:
A. Increased reticulocyte count
B. Increased hemoglobin
C. Increased ferritin
D. Increased serum iron

Correct answer : A. Increased reticulocyte count


Sequence of changes during correction of iron
deficiency:
Clinical improvement in the child (increase in appetite,
improvement in irritability)
Inital bone marrow response
Increased reticulocyte count
Haemoglobin levels return to normal
Body iron stores return to normal (ferritin levels)
The following features are true for tetralogy of Fallot,
except:
A. Ventricular septal defect
B. Right ventricular hypertrophy
C. Atrial septal defect
D. Pulmonary stenosis

Correct answer : C. Atrial septal defect


Components of Tetralogy of Fallot:
Pulmonary stenosis
Overriding aorta
Ventricular septal defect
Right ventricular hypertrophy

Blalock and Taussig shunt is done between:


A. Aorta to pulmonary artery
B. Aorta to pulmonary vein
C. Subclavian artery to pulmonary vein
D. Subclavian vein to artery

Correct answer : A. Aorta to pulmonary artery


Blalock and Taussig shunt is used in the surgical
management of tetralogy of fallot. It is shunts blood
from the subclavian artery to the pulmonary artery.
That is not given among the list of options. As
subclavian artery is a branch of aorta, the best answer
would be ‘aorta to pulmonary artery.’

The most important determinant of prognosis in


Wilms tumor:
A. Stage of disease
B. Loss of heterozygosity of chromsome lp
C. Histology
D. Age less than one year at presentation

Correct answer : C. Histology


Histology and stage of disease are important
prognostic factors. But histology is more important.

A 1 month old boy is referred for failure to thrive. On


examination, he shows feature of congestive heart
failure. The femoral pulses are feeble as compared to
branchial pulses. The most likely clinical diagnosis is:
A. Congenital aortic stenosis
B. Coarctation of aorta
C. Patent ductus arteriosus
D. Congenital aortoiliac disease

Correct answer : B. Coarctation of aorta


Feeble femoral pulses compared to brachial pulse
indicates coarctation of aorta.
ln which of the following conditions left atrium is not
enlarged:
A. Ventricular septal defect
B. Atrial septal defect
C. Aortopulmonary window
D. Patent ductus arteriosus

Correct answer : B. Atrial septal defect


Right atrium and ventricle is enlarged in ASD. Left
atrium can enlarge once Eisenmenger’s syndrome
develops.

Which of the following haemoglobin (Hb) estimation


will be diagnostically helpful in a case of beta
thalassemia trait?
a) HbF
b) HbA1c
c) HbA2
d) HbH

Correct answer : c) HbA2


Normal hemoglobin is composed predominantly of
HbA1 – which is composed of 2 alpha chains and 2
beta chains. In beta thalassemia trait, there is
decrease in production of beta chains of hemoglobin.
But since the defect is minor, they will not have any
clinical features and the peripheral smear may be
normal. But there will be a compensatory increase in
HbA2 which is composed of 2 alpha chains and 2
delta chains. Normally HbA2 constitutes about 1.5 to
3.5% of total hemoglobin. But in beta thalassemia trait,
this increases to about 3.6 to 8%.

The sodium content of ReSoMal (rehydration solution


for malnourished children) is:
A. 90 mmol/L
B. 60 mmol/L
C. 45 mmol/L
D. 30 mmol/L

Correct answer : C. 45 mmol/L


Components of of ReSoMal:
Glucose 125 mmol/l
Sodium 45 mmol/l
Potassium 40 mmol/l
Chloride 70 mmol/l
Magnesium 3 mmol/l
Zinc 0.3 mmol/l
Copper 0.045 mmol/l
Citrate 7 mmol/l
A 15-year old female presented to the emergency
department with history of recurrent epistaxis,
hematuria and hematochezia. There was a history of
profuse bleeding from the umbilicus stump at birth.
Previous investigations revealed normal prothrombin
time, activated partial thromboplastin time, thrombin
time and fibrinogen levels. Her platelet counts as well
as platelet function tests were normal but urea clot
lysis test was positive. Which one of the following
clotting factor is most likely to be deficient?
a) Factor X
b) Factor XI
c) Factor XII
d) Factor XIII

Correct answer : d) Factor XIII


Clinical features of Factor XIII deficiency
History of prolonged bleeding from umbilical stump
Delayed bleeding
Recurrent abortions
Diagnosis (Investigations)
Prothrombin time and Activated partial thromboplastin
time are normal
Factor XIII is needed for stabilization of fibrin clot
Intrinsic and extrinsic pathways are not affected
Clot stability in 5M urea – qualitative test
Normal clot remains stable in 5M urea
But in cases of Factor XIII deficiency, the clot
dissolves
But it becomes positive only in very severe deficiency
Quantitative factor XIII assay – using photometry

The requirement of potassium is a child is:


A. 1-2 mEq/kg
B. 4-7 mEq/kg
C. 10-12 mEq/kg
D. 13-14 mEq/kg

Correct answer : A. 1-2 mEq/kg


Daily recommended intake of potassium is 1-2mEq/kg.

Which of the following malformation in a newborn is


specific for maternal insulin dependent diabetes
mellitus?
A. Transposition of great arteries
B. Caudal regression
C. Holoprosencephaly
D. Meningmyelocele
.
Correct answer : B. Caudal regression
Caudal regression is the most specific anomaly in a
child born to a diabetic mother

All of the following may occur in Down’s syndrome


except:
A. Hypothyroidism
B. Undescended testis
C. Ventricular septal defect
D. Brushfield`s spots

Correct answer : B. Undescended testis

The following are characteristic of autism except:


A. Onset after 6 years of age
B. Repetitive behavior
C. Delayed language development
D. Severe deficit in social interaction
Correct answer : A. Onset after 6 years of age
Features of autism usually appear before 3 years of
age. (OP Ghai)
Which of the following is the principal mode of heat
exchange in an infant incubator?
A. Radiation
B. Evaporation
C. Convection
D. Conduction
Correct answer : C. Convection
Which one of the following in the characteristic
feature of juvenile myoclonic epilepsy?
a) Myoclonic seizures frequently occur in the morning
b) Complete remission is common
c) Response to anticonvulsants is poor
d) Associated absence seizures are present in
majority of patients

Juvenile myoclonic epilepsy –


Clinical features, treatment
It starts in early adolescence
Bilateral myoclonic jerks are seen
Mostly in the morning
Precipitated by sleep deprivation
The patient usually remains conscious during the
episode
Associated with absence seizures and generalised
tonic clonic seizures
Benign condition
Complete remission is uncommon
Positive family history may be present
Responds well to anticonvulsants
Drug of choice – Valproate

A 12 year old Boy with hematemesis, melena and mild


splenomegaly presented to the paediatrics OPD. Examination
revealed absence of jaundice / ascites. Most probable diagnosis
is?
A. Extrahepatic Portal Venous Obstruction (EHPVO)
B. Cirrhosis
C. Non Cirrhotic Portal Fibrosis (NCPF)
D. Malaria with disseminated intravascular coagulation
Correct answer : A. Extrahepatic Portal Venous Obstruction
(EHPVO)Hematemesis, melena and splenomegaly are suggestive
of a diagnosis of portal hypertension
The first three options can cause portal hypertension
But considering the age and sex of the child, Extrahepatic Portal
Venous Obstruction (EHPVO) is the most probable diagnosis
Non Cirrhotic Portal Fibrosis (NCPF) is usually seen in adult
females in the third or fourth decade
Cirrhosis is not very common in children, and it is usually
accompanied by jaundice / ascites

Most common malignant orbital tumor in children is?


A. Acute myeloid leukemia
B. Acute lymphoblastic leukemia
C. Rhabdomyosarcoma
D. Cavernous hemangioma
Correct answer : C. Rhabdomyosarcoma
Most common malignant orbital tumor in children is
rhabdomyosarcoma.
Most common benign orbital tumor in children is dermoid cyst.

Right sided isomerism is found in association with?


A. Asplenia
B. Single spleen
C. Two spleens
D. Multiple spleens
Correct answer : A. Asplenia
Right sided isomerism
Alternate names – Ivemark syndrome / asplenia syndrome
It is a type of situs anomaly
Features : asplenia, malrotation of bowel, transverse liver, gall
bladder agenesis, imperforate anus, horseshoe adrenal gland,
urethral valves
The patients are immunocompromised due to absence of spleen,
most die before 1 year of age

Which of the following is the treatment of choice for true precocious


puberty?
A. Surgical removal of hypothalamic tumor
B. GnRH
C. Androgen blocking agents
D. LH
Correct answer : B. GnRH
Treatment of precocious puberty :
Pulsatile secretion of Gonadotropin Releasing Hormone is
responsible for the secretion of FSH and LH. By exogenous
administration of GnRH, it is possible to maintain a constant level
of GnRH in the blood. This inhibits the release of FSH and LH.

Which of the following is the most common complication of mumps in


children?
A. Orchitis
B. Conjunctivitis
C. Meningoencephalitis
D. Myocarditis

Correct answer : C. Meningoencephalitis


Meningoencephalitis is the most common complication of mumps
in children.

Which is not used for the treatment of juvenile myoclonic epilepsy?


A. Zonisamide
B. Topiramate
C. Carbamazepine
D. Valproate

Correct answer : C. Carbamazepine


Use of carbamazepine / phenytoin may increase myoclonus
in juvenile myoclonic epilepsy.

An eight year old boy presented to the casualty with high fever,
pruritic erythematous rash, joint pain and lymph node
enlargement. There is a history of upper respiratory tract infection
for which he was on cefaclor – 8 days completed of a 10 day
course. The most likely diagnosis is?
A. Serum sickness like illness
B. HSP
C. Type III hypersensitivity
D. Kawasaki disease

Correct answer : A. Serum sickness like illness


Serum sickness like reaction can occur following the use of
certain drugs, especially cefaclor in children. It presents with an
urticarial / purpuric rash, arthritis, lymphadenopathy and fever. But
unlike true serum sickness (a type III hypersensitivity response), it
is not caused by circulating immune complexes.
A one year old child presented to the OPD with the history of short
stature, tiredness and constipation. Examination revealed a
palpable goitre. Serum T4 was decreased and TSH levels were
increased. Which is the most probable diagnosis?
A. Thyroid dysgenesis
B. Thyroid Dyshormonogenesis
C. TSH receptor blocking antibody
D. Central hypothyroidism

Correct answer : B. Thyroid Dyshormonogenesis


Among the options given, only Thyroid Dyshormonogenesis
presents with a palpable goitre.

All of the following therapies may be required in a 1 hour old infant


with severe birth asphyxia except:
A. Glucose
B. Dexamethasone
C. Calcium gluconate
D. Normal saline
Correct answer : B. Dexamethasone
Steroids should not be used in management of infants with
asphyxia.

The karyotype of a patient with Androgen Insensitivity Syndrome


is:
A. 46XX
B. 46XY
C. 47XXY
D. 45XO
Correct answer : B. 46XY
Androgen insensitivity is seen in genetic males (XY) who exhibit a
female phenotype.

Which one of the following drugs is used for fetal therapy of


congenital adrenal hyperplasia?
A. Hydrocortisone
B. Prednisolone
C. Fludrocortisone
D. Dexamethasone
Correct answer : D. Dexamethasone
Dexamethasone acts by suppressing the secretion of steroids by
fetal adrenals.

Blood specimen for neonatal thyroid screening is obtained on:


A. Cord blood
B. 24 hours after birth
C. 48 hours after birth
D. 72 hours after birth
Correct answer : A. Cord blood D. 72 hours after birth
Update: The correct answer is 72 hours after birth.
Guidelines for neonatal thyroid screening – American Academy of
Pediatrics
When to screen:
Normal hospital delivery at term – Filter paper collection ideally at
2-4 days of age or at time of discharge
NICU / preterm home birth – Within 7 days of birth
Maternal history of thyroid medication / family history of
congenital hypothyroidism – cord blood for screening
Cord blood is to be used only if there is family history of
hypothyroidism. In normal delivery, blood is taken at 2-4 days of
age.

The appropriate approach to a neonate presenting with vaginal


bleeding on day 4 or life is:
A. Administration of vitamin K
B. Investigation for bleeding disorder
C. No specific therapy
D. Administration of 10 ml/kg of fresh frozen plasma over 4 hours
Correct answer : C. No specific therapy
Vaginal bleeding can occur in 3rd – 7th day after birth. It occurs due
to withdrawal of maternal hormones. It usually subsides in 2-3
days. No specific treatment is required.

In unconjugated hyperbilirubinemia, the risk of kernicterus


increases with the use of:
A. Ceftriaxone
B. Phenobarbitone
C. Ampicillin
D. Sulphonamide
Correct answer : D. Sulphonamide
Sulphonamide can displace bilirubin from albumin.
The following bacteria are most often associated with acute
neonatal meningitis except:
A. Escherichia coli
B. Streptococcus agalactiae
C. Neisseria meningitidis
D. Listeria monocytogenes
Correct answer : C. Neisseria meningitidis
Meningitis by Neisseria meningitidis is seen in 2months-12 years
of age.

Which one of the following is the most common cause of


congenital hydrocephalus?
A. Craniosynostosis
B. Intra uterine meningitis
C. Aqueductal stenosis
D. Malformations of great vein of Galen
Correct answer : C. Aqueductal stenosis

The most common malignant neoplasm of infancy is:


A. Malignant teratoma
B. Neuroblastoma
C. Wilms’ tumor
D. Hepatoblastoma
Correct answer : B. Neuroblastoma

The most common presentation of a child with Wilms tumor is:


A. An asymptomatic abdominal mass
B. Haematuria
C. Hypertension
D. Hemoptysis due to pulmonary secondary
Correct answer : A. An asymptomatic abdominal mass
ln a child, non functioning kidney is best diagnosed by:
A. Ultrasonography
B. IVU
C. DTPA renogram
D. Creatinine clearance
Correct answer : C. DTPA renogram
Isotope renogram is the best modality for renal function
estimation.

The most common cause of renal scarring in a 3 year old child is:
A. Trauma
B. Tuberculosis
C. Vesicoureteral reflux induced pyelonephritis
D. Interstitial nephritis
Correct answer : C. Vesicoureteral reflux induced pyelonephritis
A child with recurrent urinary tract infections is most likely to
show:
A. Posterior urethral valves
B. Vesicoureteric reflux
C. Neurogenic bladder
D. Renal and ureteric calculi
Correct answer : B. Vesicoureteric reflux
Vesicoureteric reflux is the most common cause of urinary tract
infections in childhood. (upto 50%)

Which of the following is the most common renal cystic disease in


infants?
A. Polycystic kidney
B. Simple renal cyst
C. Unilateral renal dysplasia
D. Calyceal cyst
Correct answer : C. Unilateral renal dysplasia

One of the intestinal enzymes that is generally deficient in children


following an attack of severe infectious enteritis is:
A. Lactase
B. Trypsin
C. Lipase
D. Amylase
Correct answer : A. Lactase
Even in the normal intestine, the lactase activity is limited. Hence it
is most prone to become deficient following an attack of infectious
enteritis.

The most common type of total anomalous pulmonary venous


connection is:
A. Supracardiac
B. Infracardiac
C. Mixed
D. Cardiac
Correct answer : A. Supracardiac

Eisenmenger syndrome is characterized by all except:


A. Return of left ventricle & right ventricle to normal size
B. Pulmonary veins not distended
C. Pruning of peripheral pulmonary arteries
D. Dilatation of central pulmonary arteries
Correct answer : A. Return of left ventricle & right ventricle to
normal size
Right ventricular hypertrophy that develops in Eisenmenger
syndrome will not return to normal size.
Diagnosis of beta Thalassemia is established by:
A. NESTROFT Test
B. HbA1c estimation
C. Hb electrophoresis
D. Target cells in peripheral smear
Correct answer : C. Hb electrophoresis
NESTROFT Test – Naked Eye Single Tube Red Cell Osmotic
Fragility Test – used for screening for Thalassemia
HbA1c – Used to assess long term glycemic control in diabetics
(blood sugar control over past 3 months)
Target cells – A feature of thalassemia, but not diagnostic

The most common leukocytoclastic vasculitis affecting children is:


A. Takayasu disease
B. Mucocutaneous lymph node syndrome (Kawasaki disease)
C. Henoch–Schönlein purpura
D. Polyarteritis nodosa
Correct answer : C. Henoch–Schönlein purpura
HSP is the most common childhood vasculitis. It is produces
leukocytoclastic vasculitis.
The coagulation profile in a 13-year old girl with Menorrhagia
having von Willebrands disease is:
A. Isolated prolonged PTT with a normal PT
B. Isolated prolonged PT with a normal PTT
C. Prolongation of both PT and PTT
D. Prolongation of thrombin time
Correct answer : A. Isolated prolonged PTT with a normal PT
In VWF, there is impairment of intrinsic coagulation pathway.
Hence there is isolated prolonged PTT.
Bart‘s hydrops fetalis is lethal because:
A. Hb Bart’s cannot bind oxygen
B. The excess alpha globin form insoluble precipitates
C. Hb Bart’s cannot release oxygen to fetal tssues
D. Microcytic red cells become trapped in the placenta
Correct answer : C. Hb Bart’s cannot release oxygen to fetal tssues
Hb Barts is formed of 4 gamma chains. The oxygen affinity is so
high that it releases very little oxygen into the fetal tissues.
An affected male infant born to normal parents could be an
example of all of the following, except
A. An Autosomal dominant disorder
B. An Autosomal recessive disorder
C. A polygenic disorder
D. A vertically transmitted disorder
Correct answer : A. An Autosomal dominant disorder
An autosomal dominant disorder if present in the parents will
always express itself phenotypically. So it is not possible for
normal parents to have an child affected with an autosomal
dominant disorder

A child with a small head, minor anomalies of the face including a


thin upper lip, growth delay, and developmental disability can have
all of the following, except:
A. A chromosomal syndrome
B. A teratogenic syndrome
C. A mendelian syndrome
D. A polygenic syndrome
Correct answer : D. A polygenic syndrome
The features given can occur as a part of a chromosomal /
teratogenic / mendelian syndrome. Polygenic inheritance is the
answer of exclusion. In a polygenic inheritance, multiple genes are
involved in the phenotypic expression. Some examples of
polygenic inheritance are hypertension and diabetes.

The process underlying differences in expression of a gene


according to which parent has transmitted is called:
A. Anticipation
B. Mosaicism
C. Non penetrance
D. Genomic imprinting
Correct answer : D. Genomic imprinting
In genomic imprinting, a gene / group of genes on the paternal or
maternal chromosome is selectively inactivated. The functional
allele will be provided by the counterpart.
Example:
Angelman syndrome – deletion of maternal gene
Prader Willi syndrome – deletion of paternal gene
A malignant tumor of childhood, that metastasizes to bones most
often is:
A. Wilm’s tumor
B. Neuroblastoma
C. Adrenal gland tumors
D. Granulosa cell tumor of ovary
Correct answer : B. Neuroblastoma
60-70% of metastasis in case of neuroblastoma are skeletal
metastasis.
Earliest sign of pathological gastroesophageal reflux in infants is?
A. Upper GI bleeding
B. Respiratory symptoms
C. Oesophageal stricture
D. Postprandial regurgitation
Correct answer : B. Respiratory symptoms
Gastroesophageal reflux in infants
Gastroesophageal reflux in infants can be of 2 types –
Physiological or Pathological.
Physiological reflux
The infant has regurgitation after feeding, but weight gain is
normal.
There are no respiratory symptoms / features of oesophagitis.
Pathological reflux (gastroesophageal reflux disease)
Regurgitation is associated with refusal to eat, excessive
vomiting, weight loss and failure to thrive.
The infant has recurrent respiratory features like like wheezing,
stridor, chronic cough, aspiration, sore throat, hoarseness and
recurrent pneumonitis.
Features of oesophagitis may be present – upper GI bleeding,
anemia and Sandifer syndrome (torticollis and dystonia associated
with GERD).
The most frequent complications are failure to thrive and recurrent
respiratory symptoms.
Investigations
Upper GI imaging series
Upper GI endoscopy
Esophageal pH monitoring
Management
Conservative management
Small frequent feeds thickened with cereal.
Burping the infant.
Hold the infant in an upright position for 20-30 minutes after
feeding.
Medical management
H2 blockers like ranitidine.
Proton pump inhibitors like omeprazole and pantoprazole.
Antacids like magnesium hydroxide.
Surgical management
Gastrostomy or fundoplication is done in a very small minority.
Differentiating an ASD from a VSD using a chest X-ray is by detection
of?
A. Enlargement of pulmonary artery
B. Enlarged left atrium
C. Dilated aorta
D. Pulmonary plethora
Correct answer : B. Enlarged left atrium

Differentiating an ASD from a VSD using a chest X-ray


Left atrium is not dilated in a case of atrial septal defect (ASD).
It is dilated in patients with ventricular septal defect. (VSD).
Pulmonary plethora is present in both ASD and VSD.
Pulmonary artery is dilated in both.
Size of the aorta is normal in both.

Least common cause of ambiguous genitalia in females?


A. 21 hydroxylase deficiency
B. 11 hydroxylase deficiency
C. WNT4 gene mutation
D. Foetal placental steroid sulfatase deficiency
Correct answer : D. Placental steroid sulfatase deficiency
Foetal placental steroid sulfatase deficiency is a rare X-linked
condition, and is seen only in males.
It is not a cause for development of ambiguous genitalia in
females.
The most common cause of ambiguous genitalia in females is
Congenital Adrenal Hyperplasia (CAH).
It is responsible for about 70% of cases.
21 hydroxylase deficiency and 11 hydroxylase deficiency are
subtypes of CAH.
Other causes include WNT4 mutation and foetal placental
aromatase deficiency.

All of the following are example of peripheral neuropathies except?


A. Abetalipoproteinemia
B. Charcot-Marie-Tooth disease
C. Werdnig Hoffman disease
D. Dejerine Sottas disease
Correct answer : C. Werdnig Hoffman disease
Abetalipoproteinemia is a progressive ataxic neuropathy with
retinitis pigmentosa and steatorrhoea.
Charcot-Marie-Tooth disease is a hereditary motor and sensory
neuropathy. It is characterised by weakness of extremities and
hammer toe.
Dejerine Sottas disease is also a hereditary motor and sensory
neuropathy. It is associated with distal weakness and ataxia.
Werdnig Hoffman disease is spinal muscular atrophy type 1. It
presents within first 6 months of life. It presents with generalised
symmetrical weakness, more in proximal muscle groups with
hypotonia.
Dr.Wahid Helmi
Pediatric Consultant
Egypt- Dymiate
&Cairo(Giza –ALharam)
01004313142
Third Revision
X-ray Dr-Wahid Helmi
1. Diagnosis?
2. What are the first two steps in treatment of hypoxic spell?
3. In a cyanotic newborn, how can you distinguish pulmonary disease from
cyanotic congenital heart disease?
4. most common neurological complications associate with disease
5. Which cardiac conditions are associated with following
Egg Shaped Heart
Snowman silhouette
Rib notching

Dr-Wahid Helmi
Answers
1.cyano c congenital heart diseseas ,most probably TOF
Pulmonary vascular markings are decreased, low PBF
RVH on with upturn of apex
A hypoplastic main pulmonary artery segment contributes to the formation
-

2.knee chest posi on , morphine

3.Hyperoxia Test

4.Cerebral thrombosis, and brain abscess.


5.X-ray appearances
Egg Shaped Heart Transposition of great arteries
Snowman silhouette TAPVD (supracardiac)
Rib notching Co-arctation of aorta (long standing)
1. What is this sign called?
2. What are the structures that cause this appearance in this condition?
3. What is the diagnosis?
4. What are the treatment?

Answer

1. i.The heart shape is ovoidDr-Wahid Helmi


(egg-shaped).
ii. The upper mediastinum is narrow.
iii. The lung fields appear normal , no VSD.
2. Transposi on of the great arteries.
The diagnosis should be confirmed by echocardiography.
A prostaglandin infusion should be started and the case should be discussed
with the nearest pediatric cardiac centre as soon as possible.
Chest X ray of baby IDMM

1. Describe the X-ray.


2. What is your diagnosis?
3. What will you do next?

Answer

1.The heart is much larger than


Dr-Wahid Helminormal.
The lung fields appear well aerated.
UAC on the left very high
UVC on the right, very high

2. A diabetic cardiomyopathy is the most likely diagnosis.


Also baby IDMM may have TGA

3.Rx Reposi on the lines.

Echocardiography.

Seek expert advice if haemodynamically unstable.


1. What is the diagnosis?
2. What is this sign called?
3. What are the structures that cause this appearance in this condition?
4. What are the clinical presentation of the infant?

Answer Dr-Wahid Helmi

1. Total anomalous pulmonary venous drainage- supracardiac type

2. Snow man appearance, figure of 8 appearance

3.Cardiomegaly with increased vascular markings


Dilatation of both the left and right innominate veins and right SVC .
enlargement of sup. mediastinum secondary to right SVC, innominate artery

and ascending vertical vein.

4. Mild cyanosis, cardiac failure, recurrent chest infection, pulmonary HT.


1. What was the lesion?
2. What was the procedure carried out?
3. What are the complications ?

Answer
Dr-Wahid Helmi

1. VSD
2. Device implant
3. Device displacement
Emboli formation
Haemolytic anaemia
1. Identify the procedure, where the device shown in this X-ray is used.
2. List two indications of this procedure.
3. What is the long-term non-cardiac complication, which may occur, if
the device is not placed in asymptomatic children?

Answer
1. ASD device closure Dr-Wahid Helmi
2. All symptomatic patients:
Asymptoma c pa ents with a Qp : Qs ra o of at least 2 : 1
3. Paradoxical (right to left shunt )
systemic embolization.
1. Identify the procedure, where the device shown in this X-ray is used.
2. List five complications, which may occur, if the device is not placed in
children with the above diagnosis.

Answer
1. PDA coil closure
2. Cardiac failure: Dr-Wahid Helmi
Infective endarteritis
Aneurysmal dilatation of the pulmonary artery
Calcification of the ductus
Non-infective thrombosis of the ductus with embolization
Paradoxical emboli
Pulmonary hypertension.
1) What is abnormalities , your diagnosis ?
2) Name three risk factor for this diagnosis?
3) What is the treatment?
4) What is the dose?

Answer Dr-Wahid Helmi


1. Respiratory distress syndrome
widespread opacification throughout both lung fields.
There are clear air bronchograms on both sides.
The heart border is not clearly defined.
The costophrenic and cardiophrenic angles are not clearly visualised.
There is an endotracheal tube.
2. Preterm, male, elective LSCS, gestational diabetes, multiple gestation, asphyxia
3. R x Antenatal corticosteroids - Betamethasone
Betamethasone 12 mg /12hour interval/I.M. /24 hours prior to delivery
Surfactant should be given if the dose has not been repeated since birth.
Ventilatory requirements are high and some centers
would consider high frequency oscillatory ventilation at this point. If this is not
available ventilation will probably need to be adjusted to improve the blood gases.
1. Describe the X ray of post term NB?
2. Diagnosis?
3. Give three recent advances in management ?

Dr-Wahid Helmi

Answer

1. Hyperinflated lung with diffused patch infiltration , and atelectasis


2. MAS
3. Lung lavage , surfactant instillation , HFOV
1) Iden fy the condi on in the CXR of an ELBW newborn
2) Give the defini on of this condi on
3) Men on the stages of this condi on in a 34 wk old
4) What are the pharmacological strategies in the management of this condition
5) Expand INSURE

Answer
1. Bronchopulmonary dysplasia (BPD)
2. Current definitions include
total dura on of oxygen supplementa on requirement for >28 days,
Dr-Wahid Helmi
degree of prematurity (<32 weeks gestational age at birth), and
Oxygen dependency at 36 weeks Postmenstrual age.
3. Stages:
a. Mild: Breathing room air at 56 days postnatal age or discharge*
b. Moderate: Need for <30% oxygen at 56 days postnatal age or discharge*
c. Severe: Need for > 30% oxygen and/or positive pressure (IMV/CPAP) at
56 days postnatal age or discharge* (* whichever comes first)
4. Pharmacological strategies
a. Vitamin A
b. Postnatal steroids
c. Superoxide dismutase
d. Furosemide
e. Intubate SURfactant Extubate
Dr-Wahid Helmi
RADIOLOGY12

1. Diagnosis?

2. What is the clinical picture?

3. What is the requirement of echo before surgery?

Dr-Wahid Helmi

Answers

1. Tracheo-esophageal fistula

2. Excessive drooling ,Respiratory distress

3. To rule out associated Congenital heart diseases and Right sided aorta
1. What is the abnormality?
2. What is the diagnosis ?
3. How is it suspected clinically ?
4. What is the management ?
5. What are three causes of respiratory distress in a baby born with this condition?

Answer
1. Bowel loops in Right hemithorax, mediastinal shift to left
2. Congenital Right diaphragmatic hernia
3. Respiratory distress
Mediastinal shift
Dr-Wahid Helmi
Bowel sounds in the thorax
Scaphoid abdomen
4. no baging using a mask and mask ventilation avoiding lung inflation prevent the
bowel from distending any more with swallowed air.
NGT on drainage
Intubation and ventilation immediately after birth till pt stabilizes
Refer to NICU
Treat PPHN
5. Surgical correction A pediatric surgical opinion should be sought.
1. a) Mechanical compression of the lungs from the herniated viscera
b) Pulmonary hypoplasia from compression of the developing lungs in utero
c) Pulmonary hypertension
1. Diagnosis ?

2. Di eren al diagnosis of this Xray ?

3. Maternal condi on associated ?

4. Prognos c factors associated with be er outcome ?

Answer

1. Diagnosis: Congenital le Dr-Wahid Helmi Hernia (CDH)


Diaphragmatic

2. Di erential diagnosis of this Xray:

-Congenital cystic adenomatoid malformation (CCAM)

-Cystic pulmonary interstitial emphysema

-Staphylococcal pneumonia with pneumatocele formation.

3. Maternal condi on associated: Polyhydramnios

4. Prognos c factors associated with better outcome:

-Hernia on a er 2nd trimester

-Absence of liver herniation

-Late onset of postnatal symptoms


Congenital left diaphragmatic hernia

Dr-Wahid Helmi
Left Congenital lobar emphysema

Hyperinflation of the left upper lobe, paucity


Dr-Wahid Helmi of vascular markings of the left
upper lobe, mediastinal shift to the right, atelectasis of the left lower lobe,
flattening of the left hemidiaphragm.

Common site of involvement In Left upper lobe.


Congenital left upper lobe emphysema.

Extension of the emphysematous lobe into the left lower lobe and its
displacement of the mediastinum toward the right.

In 50% of cases, a cause of CLE can be iden fied.


Congenital deficiency of the bronchial
Dr-Wahid Helmi cartilage
external compression by aberrant vessels
bronchial stenosis
redundant bronchial mucosal flaps
kinking of the bronchus.
Dr-Wahid Helmi
Describe chest xray ,whts Rx ?

Dr-Wahid Helmi

Huge a left tension pneumothorax with mediastinal shift to the right.

endotracheal tube that is slightly high.

There is a central venous line with the tip at the thoracic inlet.

There is an umbilical venous line with a very low tip.

Rx A chest drain must be inserted immediately


Describe chest xray ,whts Rx ?

Dr-Wahid Helmi

Large tension pneumothorax on the left.

Mediastinal shift to the right with tracheal deviation.

Transcutaneous oxygen electrode on left upper chest.

Rx Immediate drainage of the pneumothorax, needle in 2nd LT ICS then

Intubation and ventilation is very likely to be needed.


Dr-Wahid Helmi
1. What is the radiological diagnosis?

2. Name 2 risk factors for the development of this condi on

Answer Dr-Wahid Helmi

1. Right pneumothorax mediastinal shift to left with left upper zone haziness

2. Risk Factors-

Assisted ventilation (including CPAP)

MAS

RDS

Other Air Leak Syndromes (e.g. PIE)

Pulmonary hypoplasia

CHD

Idiopathic or spontaneous
1. What is the abnormality
2. Three high risk situations when this condition is imminent
3. Management

Dr-Wahid Helmi

Answer

1. Pneumothorax
2. Put a needle in second intercostal space then Intercostal drain(done)
1. Describe the X ray findings .
2. What is the diagnosis?
3. What is the treatment of choice ?
4. Write 4 life threatening complica ons of Kawasaki disease.
5. Write side effects pertaining to CVS of Digitalis

Dr-Wahid Helmi
Answer

1. X ray chest PA view with air trapped s/o


2. Pneumopericrdium
3. O2 and monitoring if severe distress prompt evacua on.
4. MI, Coranry aneurusm, Thrombosis, DIC
5. Atrial, ventricular extrasystole, heart block,AV block, VT,VF
1. Identify the abnormality?
2. Clinical presentation?
3. Treatment ?

Dr-Wahid Helmi

Answer

1. Pneumopericardium
2. Shock with weak pulses
3. Drainage
1. What is the abnormality ?
2. What is it a complication of ?

Answer Dr-Wahid Helmi

1. Pneumomediastinum
2. Forceful ventilation
Autosomal dominant. Dr-Wahid Helmi

It results from a muta on in the CBFA1 gene, which controls a key


transcription factor in osteoblast differentiation.

The anterior fontanelle often closes late , may be delayed eruption of teeth.

There can be bossing of the forehead.


A 17-year-old boy states that he is healthy, although he admits to being
treated for three cases of left lower lobe pneumonia over the past 10 years.

1. Diagnosis?
2. What is the treatment ?

Dr-Wahid Helmi

Answers

1. Pulmonary sequestration.
2. Surgical lobectomy generally is curative.
1. What is the X ray suggestive of ?
2. What is the likely organism?
3. What are the complications?
4. What is the drug of choice?

Dr-Wahid Helmi
Answer

1. Lobar Pneumonia
2. Pnemococous/Staphylococcus
3. pneumonic effusion, empyema
4. Penicillin for susceptible org and cefotaxime / vancomycin for penicillin
resistant org for 10-14 days
Consolidation in the right lower lobe , S. pneumoniae.

indications for admission in this patient?

Immunocompromised state
Toxic appearance
Moderate to severe respiratory distress
Requirement for supplementalDr-Wahidoxygen
Helmi
Complicated pneumonia
Dehydration
Vomiting or inability to tolerate oral fluids or medications
No response to appropriate oral antibiotic therapy
Social factors (e.g., inability of caregivers to administer medications at
home or follow-up appropriately)
If patient presents with fever and toxaemia.

1. give three differential diagnosis


2. Give three modalities of management

Answer Dr-Wahid Helmi


1. differential diagnosis
Lung Abcess
Infected Bronchogenic cyst
Infected Hydatid Cyst
2. management
Antibiotics (anaerobic +aerobic)
Chest physiotherapy
Percutaneous CT guided aspiration
6-year-old boy, who is presented with high fever, respiratory distress, and hypoxia. He
had an infected varicella lesion on his ear.

1. What are the CXR findings?

2. What is the most likely diagnosis?

3. What is your immediate management?

Answer Dr-Wahid Helmi


1. Complete opacifica on of the le lung field and a medias nal shi to right.

2. Empyema

3. Chest tube inser on and an bio cs.


1. What is the diagnosis?
2. plural fluid finding in bacterial pneumonia ?
3. What does VATS stand for?

Answers Dr-Wahid Helmi


1. Pleural Effusion (Right)

2. plural fluid finding / exudative pleural fluid

Proteins > 3.0 g/dL


Pleural Fluid LDH > 200 IU/L
Fluid to serum LDH ra o > 0.6
Cell count > 1000
3. Video Assisted thoracoscopy
17 month old boy brought with H/o inges ng Kerosene.
First X ray( A) was taken at 3 hours and second (B) a er few hours.
1. What is the role of gastric aspiration here on admission?
2. Ingestion of what amount is considered at risk for Pneumonitis ?
3. How long would observe this child ,if no abnormal symptoms develop.

Answer
1. Not to be done. Dr-Wahid Helmi
2. 30 ml
3. 8 12 hours
7-month-old child with worsening cough. On examination his temperature is
37.8oC. Respiratory rate 60 bpm, heart rate 130 bpm. He has moderate
intercostal recessions and his arterial oxygen satura ons are 88% on room air.
Ausculta on of his chest reveals bilateral sca ered fine crepts. weight is 7.3
kg (10th percen le), and length and head circumference are at the 95th
percentile for age.

1. Finding ?
2. Diagnosis?
3. What is the treatment (mention complete schedule)?

Dr-Wahid Helmi

Answers

1. Interstitial infiltrates beginning in the perihilar region and spreading to


the periphery. Apices spared until later in the disease.
2. Pneumocystis pneumonia.
RADIOLOGY36

4. Finding ?
5. Diagnosis?
6. What is the treatment (mention complete schedule)?

Answers
Dr-Wahid Helmi
1. reticulonodular infiltrate distributed fairly uniformly throughout the lungs
,reflect nodular interstitial spread without alveolar involvement
2. Miliary tuberculosis
3. 2HRZE + 7HR
1. Xray finding?
2. DX ?
3. DDx?

Answer

1. thin-walled, air-filled cysts within the RT lung parenchyma


2. Pneumatoceles
3. Staphylococcus Aureus Infection
Bronchogenic Cyst
Dr-Wahid Helmi
Cystic Adenomatoid Malformation
Hyperimmunoglobulinemia E (Job) Syndrome
Pneumococcal Infections
Pneumonia
Pulmonary Sequestration
Tuberculosis
Pneumatoceles are thin-walled, air-filled cysts that develop within the lung
parenchyma frequent with staphylococcal infections, and they should not be
confused with a pulmonary abscess.
Pneumatoceles have thin, smooth walls and are seen with an improving
clinical picture thought to be a form of localized pulmonary interstitial
emphysema and are self limiting with only the rare case of a large, persisting
pneumatocele needing surgery, whereas pulmonary abscesses have thick, irregular
walls with an air fluid level and the child tends to be very ill.
Dr-Wahid Helmi
1. Mention the abnormality in this CXR.
2. Men on 3 causes of this appearance.
3. What further investigations would you order?

Dr-Wahid Helmi

ANSWER

1. Mediastinal enlargement
2. Lymphoma
Thymoma
Teratoma
3. CT chest,CT guided biopsy and HPE
This the Chest X ray of an 11 year old female child with h/o recurrent lower
respiratory infections.

1)What is the diagnosis?

2)Write the a) clinical features and b) one important diagnos c clinical sign

3)Which syndrome is associated with the above condi on?

4)Write the management

5)What is inves ga on of choice

Dr-Wahid Helmi

Answer
1)Bronchiectasis
2)a) Productive cough with expectoration Hemoptysis ,FTT ,Cyanosis
b) Chest Deformities (Harrison's sulci) ,Crepitations, wheeze, crackles may
be heard on auscultation ,Clubbing
3) Kartagener's syndrome may be associated.
4) Management-
i. Treatment of underlying disorder
ii. Postural drainage
iii. Chest Physiotherapy
iv. Antibiotics
v. Surgical removal of the affected area
5) HRCT
1. What is the anatomical structure in which coin is lodged?

2. What is the location of carina with respect to thoracic vertebrae?

3. What are the anatomical areas of esophageal narrowing?

4. How can this foreign body be removed?

5. What complications you expect?

Answers

Esophagus (When foreign bodies lodgeHelmi


Dr-Wahid in the esophagus, the flat surface of
the object is seen in the AP view)
T4
Anatomic areas of esophageal narrowing
a. Cricoid
b. Tracheal bifurcation
c. Gastro-esophageal junction
4. Emergent Endoscopy

5. Airway compromise.

Esophageal rupture.

Erosion into the mediastinal structures.


Child admitted with sudden breathing problems . There was history of playing
with marbles at the time of development of marbles. X-RAY done shows ?

1. Describe X RAY
2. Diagnosis
3. Treatment

Dr-Wahid Helmi

Answer

1. X-Ray findings:-
The right lung volume is increased and has herniated across the mid-line.
The left lung is compressed by the displaced heart and mediastinum.
The left lung remains airated and normal bronchi are seen on that side.
The right main bronchus cannot be traced from its origin.
2. Rt main bronchus partially obstructed by non opaque foreign body
3. Bronchoscopy and removal
Three year old toxic boy with fever, drooling, stridor, respiratory distress.

1.what Xray finding?

2. what Diagnosis ?

3. what Treatment ?

Answer
1. Xray finding: Dr-Wahid Helmi
-Increased space between the pharyngeal air shadow and the vertebrae.
-Posterior pharyngeal wall is bulging
2. Diagnosis: Retropharyngeal Abscess
3. Treatment:
- Intravenous antibiotics with or without surgical drainage.
A third generation cephalosporin with ampicillin-sulbactam or clindamycin to
provide anaerobic coverage.
RADIOLOGY44

Scenario: this is a 5 years old child who presented to you with history of cough
and fever for 4 days who got lethargic, his vaccine history is incomplete, this
hisX-ray

Q1: Describe the finding?

Q2: What is your diagnosis?

Q3: How would you approach this child?

Q4: what is the possible organism?

Q5: What is other considera on you need to make?

Dr-Wahid Helmi

Answer
1. Lateral view of the neck with "thumb" sign
2. Epiglottitis
3. ABC, don't do throat exam, Antibiotics, consult anesthesia for possible
emergency tracheostomy, oxygen, IV fluid
4. H. Influenza
5. To give prophylaxis for the family contact
1. What is the x-ray finding?

2. What is the most likely diagnosis?

3. What are the Indica ons for hospitalization?

Answer
1. AP radiograph of the neck showing typical steeple sign (subglottic narrowing).
2. Viral croup (Laryngotracheobronchitis).Dr-Wahid Helmi
3. Indications for hospitalization include:
„h Persistent or worsening signs of respiratory distress despite therapy
„h Signs of impending or frank respiratory failure or compensated respiratory failure
„h Stridor at rest
„h Unreliable caretaker
„h Poor oral fluid intake
3 day neonate with Lethargy Feed refusal Abdominal distension (see x ray)
1. Describe the abnormalities on the X-ray ?
2. What is the diagnosis?
3. What is the radiological feature of Bell stage III NEC?
4. Name other conditions associated with pneumatosis intestinalis?

Answers
1. dilated bowel with thickened wallHelmi
Dr-Wahid no air in the rectum
widespread intramural gas ,Pneumoperitoneum
2. NEC
3. Hirschsprung's disease
Pseudomembranous enterocolitis
Neonatal ulcerative colitis
Ischemic bowel disease

Stage I (suspected NEC)


Normal or mild dilatation or ileus
Stage II (definite NEC)
Intestinal dilatation and pneumatosis intestinalis (subserosal,submucosal air)
Stage III (advanced NEC, severely ill)
Same as II with portal vein gas and pneumoperitoneum
3 day neonate with Lethargy Feed refusal abdominal distension (see x ray)
1. What you see in this film?

2. What is your diagnosis?

3. What is the most likely cause in this preterm newborn?

Dr-Wahid Helmi

Answers
1. Air under the diaphragm

2. Pneumoperitonium

3. Necro zing enterocoli s


Answer Following questions based on X Ray seen
1) What is abnormal in this X ray?
2) What is the ideal position of placement of umbilical arterial and U venous line?
2. After putting in a UA line, the right lower limb appears pale ,what would you do?
3. What is the level of the renal artery?
4. How do you maintain a UA line?

Dr-Wahid Helmi

Answers
1) Abnormally placed umbilical arterial line in the subclavian artery
2) For umbilical arterial line - High: Between T7- T10; Low: Between L2-L3
For umbilical vein - Just above the diaphragm
3) Warm the other limb; If s ll pale >1/2 hour, remove the UA line
4) L-1
5) Use heparin infusion at rate of 0.5-1.0 Unit per hour
Single bubble -Pyloric atresia

Dr-Wahid Helmi
1. What are two imp radiological abnormalities?
2. What is the diagnosis ?
3. What are the three imp investigations ?
4. What are the metabolic abnormalities expected?
5. What is the management ?

Dr-Wahid Helmi
Answer

1. Distension of stomach ,No gas in the intestines


2. Pyloric Stenosis
3. USS abdomen: pyloric thickness >4mm/ pyloric length >14 mm
S electrolytes
Ba studies

4. Hypochloremic, hypokalemic, metabolic alkalosis

5. Management of the fluid & electrolytes + Ramstedt's operation


Double bubble- Duodenal atresia

Dr-Wahid Helmi
Triple bubble- Jejunal atresia

Dr-Wahid Helmi
1. What is the abnormality?
2. What is the likely diagnosis?
Dr-Wahid Helmi
3. Delineate management.
4. Men on 3 complica ons

Answer
1. Multiple fluid levels ,gasless lower abdomen
2. Small bowel obstruction
3. Surgical correction
4. Dyselectrolytemia
Perforation
Exaggerated hyperbilirubinemia
Dysmotility syndrome
1. What is the diagnosis?
2. Describe three features seen on the X-ray of the disease?
3. What biochemical test would help clinch the diagnosis?
4. What is the treatment of the condition?

Dr-Wahid Helmi

Answers
1. Rickets
2. features seen on the X-ray
a) Cupping
b) Widening of distal end of metaphysis
c) Fraying
3. Calcium, Phosphorus, Alkaline phosphatase
4. Injec on Vitamin D 6 lac unit IM stat and PO Calcium
1. What is the diagnosis?
2. This infant is 8 months old, what is the most likely type?
3. What is the earliest sign of this disorder?
4. What is the first radiological change that occurs in response to specific Rx?
5. How could this have been prevented?
6. What are the non - specific urinary findings in this disorder ? (at least 2)

Answer
1. Rickets
Dr-Wahid Helmi
2. Vitamin D deficiency
3. Craniotabes
4. Appearance of provisional zone of calcification
5. Supplement of 400IU of vitamin D
6. Generalized aminoaciduria
> Glycosuria
> Phosphaturia
> Elevated urinary citrate
> Impaired renal acidification.
Short mother brings in her 1-year-old boy for the first time because she is

1-What is the most likely diagnosis?


2-What are the most likely serum laboratory findings?

Answer

1- Familial hypophosphatemicDr-Wahid
rickets ofHelmi
either the autosomal dominant or
sex-linked type.

2- The typical laboratory findings in this disorder are normal serum calcium
and low serum phosphate values.
Dr-Wahid Helmi
1. what is the test you will ask for?
2. what is the treatment in this case?

Dr-Wahid Helmi

Answer

1. Serum Ca+ and phosphate level


2. Vitamin D with Ca+ supplement
6 weeks infant K/C of Cholesta c jaundice (Extra-Hepatic Biliary Atresia)

1. What is the likely cause of fracture femur in this case?

2. How can this complica on be prevented?

3. How do you manage pruritus in these pa ents?

4. An infant with cholestasis, triangular facies, and a pulmonic stenosis


murmur is likely to have what syndrome?

Dr-Wahid Helmi

Answers

1. Metabolic Bone disease (secondary to Vitamin D deficiency due to


malabsorption of fat soluble vitamins)

2. Replace 5,000-8,000 U /d of D 2, or 3 -5 µg/kq/d of 25 hydroxycholecalciferol

3. Ursodeoxycholic acid 15-20 mg/kg/day

4. Alagille syndrome (Arteriohepa c dysplasia)


Give five radiological findings ?

What is the diagnosis ?

What is the management ?

ANSWER

1. Ground glass appearance


Dr-WahidofHelmi
bone
Thinned cortex
Periosteal calcification
White line of Fraenkel (well calcified cartilage)
Wimberger's sign (white ring)
2. Scurvy

3. Vit C 100-200 mg/ day + Dietary Therapy


X ray pictures of a 11 year old boy presen ng with recurrent long bone
fractures
1. Identify the condition?
2. Mode of inheritance ?
3. Underlying pathology?
4. Men on 1 di eren al diagnosis
5. Other clinical Features in this condi on? (Any 4)

Dr-Wahid Helmi

1. Pyknodysostosis
2. Autosomal recessive
3. Lysosomal disorder due to genetic deficiency of Cathepsin K, which is
important for normal osteoclast function
4. Osteopetrosis
5. Short stature, Delayed closure of cranial sutures, fronto-parietal bossing,
short broad hands with hypoplasia of nails, nasal beaking, proptosis,
obtuse mandibular gonial angle
1. Findings in this x ray ?
2. What are the two important conditions which produce similar findings?
3. How do you differentiate radiologically these two conditions ?
4. What hematological problems can occur in a child with such x-ray findings ?

Dr-Wahid Helmi

Answer
1. - Increased density of bone
- Changes suggestive of of rickets
2. - Osteopetrosis
- Pyknodysostosis
3. - Angle of mandible normal in osteopetrosis
- Increased angle of mandible in pyknodysostosis
- Distal phalanges normal in osteopetrosis
- Narrow distal phalanges in pyknodysostosis.
5. Anemia
1) Identify this condition.
2) Men on 3 treatment op ons
3) Men on 2 complica ons

Dr-Wahid Helmi

ANSWER

1. Dense bones suggestive of osteopetrosis


2. Steroids,Interferon,Bone marrow Transplantation
3. Infections,Bleeds,Loss of vision
Dr-Wahid Helmi
16-year-old boy who has a 3-month history of bilateral leg pain and lower back
pain. He also reports occasional low-grade fever and a 5- to 10-lb (2.25- to 4.5-kg)
weight loss over the last 3 months. He denies rashes or other symptoms.
His physical examination reveals loss of mobility of the lower spine when bending
forward and tenderness of both knees, hip radiograph was ordered.

1- What is the most likely diagnosis?


2- What other features you may see in this patient?
3- What finding you see in the hip radiograph?

Dr-Wahid Helmi

answer
1- Ankylosing spondylitis, one of the diseases categorized as spondyloarthropathies,
or more recently, enthesitis-related arthritides.
2- Other features, including anterior uveitis, renal involvement, and rarely aortic
insufficiency, have been described in adolescents who have this disease.
3- Hip radiographs show evidence of sacroiliitis.
Dr-Wahid Helmi
1. Describe the findings on the spine ?

2. Name the disease where this is seen?

3. Name 2 other skeletal complica ons of this condi on ?

Answer
1. Infarction affecting the central part of the vertebrae (fed by a spinal artery branch)
results in the characteristic H. vertebrae of sickle cell disease. The outer portions of
the plates are spared because of the numerous apophyseal arteries.
The lateral cxray shows multiple vertebral end-plate irregularities and depressions.
The peripheral portions of the end-plates are spared.
The appearance is due to bone infarction and subsequent collapse. This is an early
example of the classical h-shaped vertebrae seen in sickle-cell anaemia.
2. Sickle cell anemia (also seen in Gauchers disease)
3. Dactylitis, avascular necrosis of femoral head and humerus, osteomyelitis
Baby 4 wks old girl never seen her turn her head, which makes it difficult for
her to feed at the breast. They also note that her back does not appear
normal. On physical examination, her hairline appears low posteriorly. You
confirm that she does not turn her head, and when placed prone, does not
turn her head to the side. Her right scapula appears to be higher than the left,
and you note that the spine does not appear to be perfectly straight,
suggesting congenital scoliosis x ray done .
1. Diagnosis?
2. What other associated defects may be seen??

Dr-Wahid Helmi

Answers

1- The Klippel-Feil syndrome involves the fusion of cervical vertebrae and


occurs in approximately 1 in 42,000 births, with a 65% female predominance. It
is usually a sporadic event. Due to neck immobility, affected individuals are at
risk of cervical spine injury.
2-Associated defects may include deafness (conduction or
sensorineural, occurring in up to 30% of pa ents), congenital heart
defects (usually ventricular septal defect), rib defects,hemivertebrae, Sprengel
anomaly (elevation of the scapula), scoliosis, and renal anomalies.
1. Describe the lesion?
2. Give two D/D
3. What is the triad of tumor lysis syndrome?

Dr-Wahid Helmi
Answers

1. Osteolytic lesion of skull


2. Histiocytosis
Ewing`s Sarcoma
Lymphoma
Bone cyst
3. Hyperuricemia, hyperkalemia, and hyperphosphatemia
1. What is the abnormality seen on this radiograph?
2. Name 2 disorders which may give rise to this abnormality

Dr-Wahid Helmi

ANSWER

1. Xray skull lateral view showing multiple lytic lesions


2. Histiocytosis
3. Secondaries
4. Multiple myeloma
1. Describe the X-ray appearance
2. Pathogenesis of the appearance
3. Possible Diagnosis
4. Which disorder is most commonly associated with an elevated MCHC?
5. How is the corrected reticulocyte count calculated?

Dr-Wahid Helmi

Answers

1. Sunray appearance
2. Medullary widening
3. Chronic hemolytic anaemia
4. Hereditary spherocytosis
5. Corrected retic count = reticulocyte % × (patient Hct/normal Hct)
Dr-Wahid Helmi
Dr-Wahid Helmi
Dr-Wahid Helmi
Dr-Wahid Helmi
Dr-Wahid Helmi
Dr=Wahid Helmi

Revised & prepared


Dr- Wahid Helmi
Consutant Pediatrician
Pediatric Department Zarka
Hospital –Dymiate=Egypt
MCQ IN PEDIATRIC
CARDIOLOGY
Dr=Wahid Helmi

You are seeing a 10-year-old girl in the clinic for a wellchild check. During the
visit, her parents mention that she has been complaining of her heart "fluttering."
It happens sometimes while she is just watching TV and other times when she is
playing. You are able to elicit that it seems to start suddenly and stop equally
suddenly. Her father mentions that he has felt his daughter's heart beat during the
episode and that it is "too fast to count." She is otherwise fairly asymptomatic
during the episode. Of the following, what is the most likely diagnosis?
a. Sinus arrhythmia
b. Second-degree heart block (Mobitz I)
c. Second-degree heart block (Mobitz II)
d. Atrial fibrillation
e. Reentrant tachycardia.

A 3-year-old boy is brought in to the emergency department by his parents for


fatigue and decreased appetite. On examination, he has cool extremities, a gallop
rhythm, and hepatomegaly. His heart rate is 140 beats/ min, and his respiratory
rate is 54 breaths/min. His parents report that he and his older sister were sick
with cough and congestion 3 weeks ago, but he never seemed to completely
recover. You decide to obtain a chest x-ray. What radiologic finding is most likely
for this suspected diagnosis?
a. Narrowed mediastinum
b. Complete whiteout of bilateral lung fields
c. Cardiomegaly .
d. Pneumothorax
e. Rib notching
Dr=Wahid Helmi

You are seeing a 15-year-old male patient in your clinic for the first time, along
with his mother and father. His mother reports that her father and brother had
heart attacks at 5 7 and 44, respectively. There are many other members of the
family, including your patient's mother, who have been diagnosed with
hypercholesterolemia. Your patient is overweight with a BMI of 28. You send him
for a fasting lipid panel and find that his total cholesterol is slightly elevated, LDL
is normal, HDL is normal, and triglycerides are moderately elevated. What is your
next step in managing this patient?
a. Diet and exercise recommendations .
b. Statin initiation
c. Echocardiogram
d. Genetic testing
e. Liver ultrasound

Answer d.
Dr=Wahid Helmi

Answer---B

A 3-year-old boy presents to the Emergency Department with 5 days of fever and
irritability. On examination, he has cervical lymphadenopathy, his eyes appear
red, and he is noted to have a maculopapular rash on his abdomen. Laboratory
results reveal a hemoglobin of 10.3 g/dL and platelets of 600,000/ mL. The next
best step in management is:
a. Obtain an echocardiogram
b. Culture the oropharynx and order an antistreptolysin 0 (ASO) titer
c. Treat the patient with 2 g/kg of IVIG.
d. Order antibody titers to Borrelia burgdorferi e. Obtain a heterophile antibody test.

A 10-year-old boy presents to the ED with 5 days of fever. He complains of chest


pain and difficulty catching his breath. Chest radiography shows an enlarged
cardiac silhouette. On examination, he is leaning forward to catch his breath, and
his heart rate is 130 beats per minute. His blood pressure is normal without pulsus
paradoxus. The next best step to establish the diagnosis is:
a. Obtain an electrocardiogram
b. Culture the oropharynx and order an antistreptolysin 0 (ASO) titer
c. Treat the patient with 2 g/kg of IVIG
d. Order antibody titers to Borrelia burgdorferi
e. Obtain large volume blood cultures.
Dr=Wahid Helmi

A 3-year-old recent immigrant from Somalia presents for his first well-child care
visit. On examination, the child is noted to be below the 3rd percentile for weight.
AII/VI soft, vibratory systolic ejection murmur is auscultated along the left sternal
border. The murmur becomes louder when the child is laid supine. No
hepatomegaly is noted. What is the next best step in management?
a. Referral to the emergency department for urgent
echocardiogram and evaluation by a cardiologist
b. Referral to cardiology clinic
c. Well-child visit in 1 year
d. Investigation of causes of failure to thrive.
Answer--d. This child's murmur has multiple characteristics of abenign murmur of
childhood: It is soft, systolic ejection
in quality, and becomes louder when the child is supine.
The vibratory nature of the murmur is also suggestive of a
Still' s murmur, a common murmur in this age range. The
child should be evaluated for other etiologies of failure to
thrive before consultation with a cardiologist.

15-year-old girl presents for evaluation after a syncopal event during track
practice. She was approaching the finish line when observers saw her fall. By the
time bystanders reached her, she had regained consciousness and appeared to be
back to baseline. She sustained several abrasions and a bruise on her shin during
the fall.She reports having experienced dizziness with exertion in the past, but she
has no recollection for this specific event. She denies any prior history of loss of
consciousness.The next step in management is:
a. Referral to an eating disorder specialist
b. Recommendation to increase fluid intake to 2 L or
more per day
c. Referral to a cardiologist and restriction from all exercise
until she is evaluated.
d. Referral to a cardiologis
Answer--c.
Dr=Wahid Helmi

Answer---a.

A 6-week-old, full term, previously healthy infant presents to your clinic because the
parents feel that the child has not been eating well and grunts with every feed. They
also report that she occasionally appears to be sweating with feeds and has to be woken
up to breastfeed. Vital signs reveal that her weight is at the 8th percentile (birth
weight at the 45th percentile), heart rate is 163 beats/min, respiratory rate is 62
breaths/minute, and saturation is 97%. Her right arm blood pressure is 77/48 mm
Hg, and her right leg blood pressure is 84/45 mm Hg.On physical examination, she
displays increased work of breathing, and diffuse rales can be auscultated. She has
a soft, I/VI holosystolic murmur that radiates throughout the precordium. What is the
most likely cause of this child's congestive heart failure symptoms?
A. Coarctation of the aorta
B. TGA
C. TOF
D. VSD
E. Bicuspid aortic valve
Answe-- d. Congestive heart failure can have a variety of etiologies,
but most fall into the categories of ventricular dysfunction,volume overload, and
pressure overload. In the setting of a large VSD, there is unrestricted blood flow
from the left ventricle into the right ventricle, causing a volume and pressure load to the
pulmonary circulation,worsening as pulmonary vascular resistance falls.
The increased pulmonary blood flow leads to pulmonary edema and respiratory
distress.

You are called to the newborn nursery to evaluate a2-day-old infant who appears
to be in distress. After placing her on monitors, you note that her right arm
saturation is 94% and her right leg saturation is 74%. She has a heart rate of 1 7 5
beats/ min and a respiratory rate of 7 7breaths/min. Her right arm blood pressure
is 87145 mm Hg, andherrightlegbloodpressureis 62/31 mmHg. On examination,
she has diminished 1 + pedal pulses with cool extremities. Of the following, what is
the most likely diagnosis and subsequent management strategy?
A. TGA, balloon atrial septostomy
B. TOF, prostaglandin infusion
Dr=Wahid Helmi

C. Interrupted aortic arch, prostaglandin infusion.


D. Total anomalous pulmonary venous return, emergent surgery
E. VSD, oxygen
Answer ---. c. In an interrupted aortic arch, the aorta is discontinuous,
with the proximal portion usually supplying the head and neck vessels and ending at the
level of the ductus,and with the distal portion arising from the ductus directly. Because
of this, the blood supply to the lower portion of the body is completely dependent on
patency of the ductus, making it a ductal-dependent lesion

The most common cyanotic congenital heart defect seen in infants of


diabetic mothers is which of the following?
a. d-Transposition of the great arteries .
b. Hypoplastic left heart syndrome
c. Tricuspid atresia
d. Tetralogy of Fallot
Dr=Wahid Helmi

Physiologic responses to asphyxia include which of the following?


a. Tachycardia, decreased central venous pressure,vasoconstriction of the
cerebral vessels
b. Bradycardia, increased central venous pressure,vasoconstriction of the
skeletal muscle vessels.
c. Bradycardia, decreased central venous pressure,vasodilation of the
cerebral vessels
d. Tachycardia, increased central venous pressure,vasoconstriction of the
skeletal muscle vessels

A 1-week-old infant with tetralogy of Fallot is undergoing a rule-out sepsis


evaluation. The infant’s baseline saturations are 98%, with a 2/6 systolic ejection
murmur along the left sternal border. During the blood draw, the infant’s
saturations drop to 60% and the infant becomes visibly cyanotic. On auscultation,
the lung sounds remain clear, but the murmur is no longer audible. What
interventions should be performed?
a. Intubation with 100% O2
b. STAT echocardiogram
c. Arterial blood gas
d. Morphine and phenylephrine administration.
Answer----d. The infant is this scenario is having a hypercyanotic spell caused by an acute decrease in
pulmonary blood flow. These are commonly triggered by medical proce-dures such as blood draws.
Morphine and phenylephrine are first-line agents to counteract the hypercyanotic spell.
Dr=Wahid Helmi

Newborn infant was found to have interrupted aortic arch type B, with truncus
arteriosus. What other features is the infant likely to have?
a. Epicanthal folds with a single palmar crease
b. Elfin facies
c. Broad thumbs and toes
d. Absent thymus on chest x-ray and hypocalcemia.
d. Both type B interrupted arch and truncus arteriosus are associated with 22q11
deletion syndrome. Infants with 22q11 deletion syndrome frequently have hypo-
calcemia and an absent thymus.

A term infant undergoes echocardiography after birth due to a murmur.


Numerous small cardiac tumors are identified throughout the heart. Which
genetic disease is likely?
a. Down syndrome
b. Williams syndrome
c. Tuberous sclerosis.
d. Hunter syndrome
Answer---- c. Numerous cardiac tumors found in a neonate are almost always
rhabdomyomas, which are strongly asso-ciated with tuberous sclerosis

An infant was born at full term to a 29-year-old woman with lupus by cesarean
section due to persistent fetal bradycardia. The newborn appears vigorous,with
normal capillary refill, blood pressure of 80/55 mm Hg, and 2+ femoral pulses.
The heart rate, however,is 65 beats/min, with a narrow QRS complex not
associated with p waves. What is the best course of action?
a. Begin isoproterenol
b. Begin cardiopulmonary resuscitation
c. Monitor closely on telemetry.
d. Use transcutaneous pacing
c. This infant has complete heart block related to trans-placental transfer of SSA and/or
SSB antibodies related to maternal lupus. Although the heart rate is low, the infant has
adequate cardiac output, as evidenced by good capillary refill and blood pressure.
Monitoring is all that is required in this situation.
Dr=Wahid Helmi

A 3-day-old infant suddenly develops tachycardia to 240 beats/min. Adenosine is


administered, which slows the ventricular rate briefly to 140 beats/min, revealing a
sawtooth-like pattern between the QRS complexes. Transesophageal pacing is used to
terminate the tachycardia. Which drug is best to use as prophylaxis
to prevent future tachycardia?
a. Propranolol
b. No prophylactic therapy needed.
c. Digoxin
d. Amiodarone
b. This infant had an episode of atrial flutter, which is very unlikely to recur and thus
does not warrant pro-phylaxis

An infant with a large ventricular septal defect and congestive heart failure is
being medically managed with digoxin, enalapril, and furosemide (Lasix).
Spironolactone was added yesterday due to persistent hypokalemia.
Overnight, the infant developed Mobitz II heart block. This is a toxicity of which
medication?
a. Spironolactone
b. Digoxin.
c. Enalapril
d. Lasix
b. Atrioventricular block is a well-described result of digoxin toxicity. Digoxin toxicity
is more likely to occur in the setting of hypokalem

A 10-day-old infant is being treated for sepsis with antibiotics and dopamine. The
infant is requiring escalating doses of dopamine. All the interventions below can
increase the infant’s response to the current dose of dopamine, except for one.
Which intervention does not increase an infant’s sensitivity to catecholamines?
a. Beginning steroid administration
b. Normalizing pH
c. Normalizing serum calcium levels
d. Normalizing potassium levels.
Answer----d.
Dr=Wahid Helmi

1. You are called to the newborn nursery to evaluate a 2-hour-old newborn who
has suddenly become cyanotic. The oxygen saturation on room air is 69%, and the
patient is tachycardic and tachypneic. Oxygen is administered without
improvement in the patient’s oxygen saturation.On examination, you hear a loud
S2 and no murmur. A chest radiograph shows increased pulmonary vascular
markings, a narrow mediastinum, and a small heart. Which of the
following would be the next step in management?
A. Start digoxin.
B. Refer the patient to surgery for placement of a Blalock–Taussig shunt.
C. Refer the patient to surgery for repair of a ventricular septal defect.
D. Proceed with pulmonary balloon valvuloplasty.
E. Begin infusion of prostaglandin E (PGE).
This patient’s clinical presentation and physical examination are
most consistent with transposition of the great arteries (TGA).

2. A 20-month-old boy with tetralogy of Fallot is admitted for evaluation of


cyanosis that is increasing in frequency. As you conclude your history and
physical examination, you witness an episode of cyanosis when the patient’s
brother makes him cry. As the crying increases, the
patient becomes more and more cyanotic. On examination, his cardiac murmur is
now much softer than before he began crying. What is the next most appropriate
step in management?
A. Intubate and begin mechanical ventilation.
B. Administer intravenous dopamine.
C. Place the patient in a knee-chest position.
D. Administer subcutaneous epinephrine.
E. Call for a cardiology consult.

Tetralogy of Fallot (hypercyanotic or “tet”) spells are


defined as paroxysmal episodes of hyperpnea, irritability, and prolonged crying that
Dr=Wahid Helmi

result in
increasing cyanosis and decreasing intensity of the heart murmur.

3. A 10-year-old girl is seen for a routine health maintenance evaluation. Five


years ago, she underwent surgical repair of coarctation of the aorta. On
examination, the blood pressure in her right arm is 173/81 mm Hg, and her
oxygen saturation is 97% in room air. Auscultation reveals a systolic ejection
murmur audible throughout the precordium. The patient is otherwise
asymptomatic. Which of the following would be the most appropriate next step in
management?
A. Check the blood pressure in all extremities.
B. Refer the patient to a cardiac surgeon promptly.
C. Obtain an echocardiogram to rule out a bicuspid aortic valve.
D. Recheck the oxygen saturation in 100% oxygen.
E. Have the patient return in 6 months for reevaluation

Restenosis is a known complication from repair of coarctation of the aorta, and these
clinical features are consistent with restenosis. Patients with coarctation of the aorta
classically present with hypertension in the right arm, and commonly in the left
arm,and reduced blood pressures in the lower extremities.

4. A 7-year-old boy presents with a 3-day history of fever (temperature to 39.7°C


[103.5°F]),shortness of breath, and weakness. He also complains of chest pain,
which is most intense when he lies down and improves when he sits upright. His
past medical history is significant for closure of a ventricular septal defect 2 weeks
ago. Which of the following findings is consistent with the most likely diagnosis?
A. Splinter hemorrhages
B. Pulsus paradoxus
C. Heart rate of 260 beats/minute with absent P waves
D. Prolongation of his QT interval
E. Tenderness at two of his costochondral joints
Dr=Wahid Helmi

This patient’s clinical presentation is most consistent with


pericarditis. The likely cause of his pericarditis is postpericardiotomy syndrome, given
the
recent closure of his ventricular septal defect before the onset of his symptoms.

5. A 1-month-old female infant is seen in your office for a routine health


maintenance evaluation.On examination, you hear a grade 4 holosystolic murmur
at the left sternal border. Femoral pulses and oxygen saturation in room air are
normal. The infant is otherwise well and growing normally. Which of the
following statements regarding this patient’s condition is correct?
A. Without intervention, congestive heart failure will develop.
B. Eisenmenger syndrome will eventually occur.
C. Surgical closure of the patent ductus arteriosus is indicated.
D. The murmur may disappear without intervention.
E. Balloon valvuloplasty is indicated.

This patient’s murmur is consistent with a small ventricular septal


defect (VSD). With a small VSD, a patient is likely to remain asymptomatic with
normal
growth and development.
Dr=Wahid Helmi

6. You are called to the nursery to evaluate a male newborn with cyanosis. On
auscultation, you hear a single S2 but no murmur. Pulse oximetry shows an
oxygen saturation of 72% in room air. An electrocardiogram reveals left axis
deviation and left ventricular hypertrophy. What is his likely diagnosis?
A. Tetralogy of Fallot
B. Transposition of the great arteries
C. Truncus arteriosus
D. Total anomalous pulmonary venous return
E. Tricuspid atresia with intact ventricular septum
Tricuspid atresia is the only cause of cyanosis in the
newborn period that manifests with left axis deviation and left ventricular hypertrophy
on
electrocardiogram (ECG).

Patients with tetralogy of Fallot present with a systolic murmur of pulmonary


stenosis and right ventricular hypertrophy (RVH) on ECG.
Patients with tricuspid atresia without a ventricular septal defect
have a single S2 as a result of the usual coexistence of pulmonary atresia and do not
have a
murmur.
Patients with transposition of the
great arteries also have no murmur and a single S2 but will have RVH on ECG.
Similarly, RVH
is present in total anomalous pulmonary venous return, along with a systolic murmur.
Truncus arteriosus manifests as combined ventricular hypertrophy with both a systolic
and
diastolic murmur.
Dr=Wahid Helmi

7. A 4-year-old boy is in the office for a routine health maintenance evaluation. His
examination is normal except for multiple deep dental cavities. You plan on
referring him for dental evaluation and possible dental extraction. His mother
reminds you that he has a “heart condition.” Which of the following cardiac
conditions requires antibiotic prophylaxis against endocarditis?
A. Patch repair of ventricular septal defect repaired 4 months ago
B. History of uncomplicated Kawasaki disease
C. Wolff–Parkinson–White syndrome
D. Patent ductus arteriosus
E. Ostium secundum atrial septal defect

8. You see a 7-week-old male infant with cough and poor feeding. Examination
reveals a respiratory rate of 72 breaths/minute and a heart rate of 170
beats/minute. His weight is 7 pounds 6 oz, just 2 oz more than his birth weight.
You hear diffuse rales throughout the lung fields and a systolic murmur on
auscultation. The liver is 4 cm below the right costal margin.
Which of the following conditions is the most likely cause of his signs and
symptoms?
A. Large ventricular septal defect
B. Ostium secundum atrial septal defect
C. Small patent ductus arteriosus
D. Critical or severe aortic stenosis
E. Mild to moderate pulmonary stenosis
Dr=Wahid Helmi

9. A thin 5-year-old boy presents for a routine health maintenance evaluation. He


feels well and is growing normally. On examination, you hear a continuous
murmur below the right midclavicle. The murmur is loudest while the patient is
sitting and disappears while he is supine. The femoral pulses are normal. Which of
the following conditions is the most likely diagnosis?
A. Aortic stenosis with regurgitation
B. Venous hum
C. Patent ductus arteriosus
D. Still’s murmur
E. Pulmonic systolic murmur
1-This murmur is most consistent with a venous hum, an innocent heart murmur.
Aortic stenosis with 2-regurgitation presents with systolic ejection and decrescendo
diastolic murmurs.
The murmur of a patent ductus arteriosus (PDA) is generally
continuous and machinery-like and does not vary with position. Patients with PDAs
generally
also have brisk pulses.
3-Both a Still’s murmur and a pulmonic systolic murmur are innocent
systolic murmurs. A Still’s murmur is usually a grade 1–3 systolic murmur best heard
at the
mid-left sternal border.
4-A pulmonic systolic murmur is a grade 1–2 high-pitched systolic
murmur best heard at the upper left sternal border.
5-Both the Still’s murmur and the pulmonic systolic murmur are loudest supine.
Dr=Wahid Helmi

10. A 15-year-old boy complaints of chest pain that occurs during basketball
practice. He is otherwise healthy and has no history of cardiac problems.
Examination is normal except for aharsh systolic ejection murmur at the apex that
worsens with standing and the Valsalva maneuver. An electrocardiogram
demonstrates left ventricular hypertrophy and left axis
deviation. Which of the following is the most appropriate initial management at
this time regarding the likely diagnosis?
A. Start propranolol to reduce left ventricular outflow tract obstruction.
B. Admit for urgent aortic balloon valvuloplasty.
C. Reassure the patient that the murmur is innocent and allow complete athletic
participation.
D. Admit for surgical myomectomy for septal hypertrophy.
E. Begin albuterol, as the patient’s chest pain is likely caused by asthma.
Dr=Wahid Helmi

Second Part

1. The most common cyanotic heart disease manifesting as congestive cardiac


failure during first week of life is:
(a) Pulmonary stenosis
(b) Fallot’s tetralogy
(c) Tricuspid atresia
(d) Hypoplastic left heart syndrome.

2. Congenital heart disease which causes death in the first week of life
(a) VSD
(b) TOF
(c) Epsteins anomaly
(d) Hypoplastic left ventricle.

3. The most common cause of heart failure in infants is:


(a) Myocarditis .
(b) Rheumatic fever
(c) Fluid over load
(d) Cardiomyopathy

1D 2D 3A
4. Congestive cardiac failure is diagnosed in an infant by:
(a) Basal crepts
(b) Elevated JVP
(c) Pedal oedema.
(d) Liver enlargement
Dr=Wahid Helmi

5. Excessive sweating in a young infant may be manifestation of


(a) Polycythaemia
(b) Phenylketonuria
(c) Heart failure.
(d) Anxiety

6. The most consistent early clinical sign of congestive cardiac failure in infants is:
(a) Oedema of feet .
(b) Basal rates
(c) Hepatomegaly
(d) Raised jugular venous pressure.

7. Which of the following is not a characteristic of right sided failure?


(a) Pulmonary oedema.
(b) Ascites
(c) Oliguria
(d) Dependent oedema

8. Digitalis toxicity can be diagnosed if ECG shows:


(a) Prolonged PR interval.
(b) T wave inversion\
(c) Shortening of QT interval
(d) Ventricular bigeminy
9. Compared to the oral digitalizing dose parenteral dose of digoxin should be:
(a) 1/4
(b) 1/2
(c) 1/3
(d) 2/3.
Dr=Wahid Helmi

10. Carey Coombs’ murmur of rheumatic carditis is:


(a) An apical pansystolic murmur
(b) An apical mid diastolic murmur .
(c) An apical early diastolic murmur
(d) A basal ejection systolic murmur

11. Essential feature of diagnosis of a acute rheumatic fever is:


(a) Recent sore throat infection.
(b) Erythema marginatum
(c) 1 major and 2 minor Jone’s criteria
(d) Prior H/o of rheumatic fever

4 C 5 C 6 A 7 A 8 A 9 D 10 B 11 A

12. Acute rheumatic fever is characterised by


(a) Erythema marginatum
(b) Atypical rugling pansystolic murmur
(c) Tender nodule
(d) Haemoptysis

13. Major manifestations of rheumatic fever are A/E


(a) Streptococcal antibody
(b) Polyarthritis
(c) Subcutaneous nodule
(d) Chorea

14. All of following are recognized manifestation of acute rheumatic fever except
(a) Abdominal pain
(b) Epistaxis
(c) Chorea
(d) Subcutaneous nodules
Dr=Wahid Helmi

15. The diagnosis of rheumatic fever is best confirmed by:


(a) Throat swab culture
(b) Raised ESR
(c) ASLO titre
(d) ECG changes
16. Rheumatic fever is suggested by the presence of:
(a) Symmetrical polyarthritis
(b) Nail clubbing
(c) Anemia
(d) ECG evidence of prolonged PR interval

17. An 8-year-old male child is admitted with a diagnosis of rheumatic fever with
arthritis, carditis and congestive heart failure. with reference to this case, consider
the following as initial lines of management:
(1) Eradication of remnant streptococcal infection.
(2) Administration of an anti-inflammatory drug.
(3) Institution of decongestive therapy
(4) Institution of graded and gradually increasing exercise.
Which of these should actually consist of initial line of management?
(a) 1, 2 and 3 (b) 1, 2, 3 and 4 (c) 2, 3 and 4 (d) 1 and 4
12 A 13 A 14 B 15 C 16 D 17 A

18. Bacterial endocarditis is most commonly seen in:


(a) VSD .
(b) PDA
(c) ASD
(d) AS
Dr=Wahid Helmi

19. Infective endocarditis not seen in:


(a) ASD .
(b) TOF
(c) VSD
(d) MR
20. A wide and fixed split second heart sound occurs in:
(a) Mitral stenosis
(b) Atrial septal defects .
(c) VSD
(d) Coarctation of aorta
21. Which one of the following does not produce cyanosis in the first year of life
(a) Artrial septal defect .
(b) Hypoplastic left heart syndrome
(c) Truncus arteriosus
(d) Double outlet right ventricle.
22. True about ASD:
(a) Patient foramen ovale
(b) Increase pulmonary artey flow leads to left parasternal heave .
(c) S2 widen
and variable
(d) Systolic murmur due to rapid flow of blood

23. VSD shunt reversal is called:


(a) Eisenmenger’s syndrome
(b) Eisenmenger’s complex .
(c) Ebstein’s anomaly
(d) None of the above
Dr=Wahid Helmi

24. Natural course of events in untreated ventricular septal defects except:


(a) Spontaneous closure of defect
(b) Development of pulmonary insufficiency
(c) Subacute bacterial endocarditis
(d) A normal life without symptoms .

25. A young boy had developed congestive failure and was found to have
membranous VSD. He Spontaneously showed improvement in his condition. The
is most likely to be due to:
(a) Perimembranous closure of VSD
(b) Development of AR
(c) Pulmonary vascular changes .
(d) Infective endocarditis

26. Differential cyanosis and clubbing is seen in


(a) PDA and reversal of shunt .
(b) TOF
(c) VSD + ASD
(d) MS + MR
18 A 19 A 20 B 21 A 22 B 23 B 24 D 25 C 26 A

27. Pure left sided failure may be seen with:


(a) ASD
(b) Aortic atresia
(c) Patent ductus arteriosus .
(d) Pulmonary valvular obstruction
28. One-year-old child with PDA; which is true:
(a) Symptoms similar to artopulmonary window .
(b) Chances of spontaneous closure high
(c) Indomethacin may help in closure
(d) Indocarditis is rare
Dr=Wahid Helmi

29. Preterm infant with PDA is given...to effect closure of the PDA:
(a) Corticosteroids
(b) Indomethacin .
(c) Aspirin
(d) Hyperbaric oxygen

30. The great danger of patients with patent ductus arteriosus is:
(a) Syncope
(b) Convulsions
(c) Arrhythmia
(d) Bacterial endocarditis .

31. All are characteristic of Fallot’s tetralogy except:


(a) Infundibular stenosis
(b) VSD
(c) Overriding aorta
(d) Left ventricular hypertrophy.

32. Which one of the following congenital heart disease has cyanosis without
cardiomegaly and /or congestive heart
failure?
(a) Transportation of great arteries
(b) Fallot’s tetralogy .
(c) Congenital mitral regurgitation
(d) Congenital pulmonary stenosis
33. Anoxic spells in tetralogy of Fallot’s are precipitated by:
(a) Fever
(b) Exertion
(c) Crying on feeding
(d) All .
Dr=Wahid Helmi

34. The congenital heart disease that has least chance of heart failure in infants:
(a) TOF .
(b) VSD
(c) TAPVC
(d) ASD

35. Cyanosis and breathlessness in a child is seen in:


(a) Transposition of great vessels
(b) Tetralogy of Fallot .
(c) Ventricular septal defect
(d) Atrial septal defect
36. Which of the following conditions does not present with CHF in neonate:
(a) Total anomalous pulmonary venous connection
(b) Coarctation of aorta
(c) Tetralogy of Fallot.
(d) Transposition of great vessels

37. In TOF:
(a) Cynotic spells are due to arrhythmias
(b) CXR and ECG are typically normal
(c) Wide split 2nd HS on inspiration
(d) Central cyanosis with clubbing.

27 C 28 A 29 B 30 D 31 D 32 B 33 D 34 A 35 B 36 C 37 D

38. A blue infant was found to have oligemic lung fields with normal sized heart.
The diagnosis is:
(a) Transposition of great vessels
(b) Tricuspid atresia
(c) TOF .
(d) Pulmonary stenosis
Dr=Wahid Helmi

39. Which one of the following is the most common cause of cyanotic congential
heart disease?
(a) Dextrocardia
(b) Fallot’s tetralogy.
(c) Atrial septal defect
(d) Coarctation of aorta

40. Pulmonary plethora is seen in all except:


(a) VSD
(b) ASD
(c) Fallot’s tetralogy.
(d) PDA

41. Blalock Taussing’s operation involves:


(a) Right pulmonary artery with aorta
(b) Left pulmonary artery with ipsilateral subclavian A .
(c) Right
pulmonary artery with descending aorta
(d) Left pulmonary artery with ascending aorta

42. Potts shunt is running from:


(a) Right subclavian artery to right pulmonary
(b) Descending aorta to left pulmonary.
(c) Left subclavian
to left pulmonary
(d) Ascending aorta to right pulmonary
Dr=Wahid Helmi

43. Tricuspid atresia all are true except:


(a) R.V. hypoplasia
(b) Left axis deviation
(c) Split S2 in inspiration.
(d) Pulmonary oligaemia

44. A child with central cyanosis and enlarged left ventricle the probable diagnosis
is:
(a) Tricuspid atresia.
(b) Eiesenmenger’s syndrome
(c) Tetralogy of Fallot
(d) Anomalous pulmonary artery

45. Left axis deviation with left ventricular hypertrophy is seen:


(a) Tricuspid atresia .
(b) Tetralogy of Fallot
(c) Coarctation of aorta
(d) Ventricular septal defect

46. True of tricuspid atresia are A/E:


(a) Right axis deviation .
(b) Left axis deviation
(c) Left ventricular hypertrophy
(d) Severe cyanosis

47. The best position for examination of cardiac murmurs in a child is:
(a) Sitting
(b) Standing
(c) Right lateral
(d) Recumbent.
Dr=Wahid Helmi

48. Which of the following congenital cyanotic disease presents with gallop
rhythm, left parasternal murmur, pericardial
friction rub:
(a) Ebstein anomaly.
(b) TA PVC (c) TGA
(d) Eissenmenger’s complex

38 C 39 B 40 C 41 B 42 B 43 C 44 A 45 A 46 A 47 D 48 A

49. In transposition of great vessels, all are true except:


(a) Aorta arises from the right ventricle
(b) Mitral valve is continuous with the aortic valve
(c) Causes.
jaundice immediately after birth
(d) None of the above

50. Emergency treatment for TGV:


(a) Balloon septostomy .
(b) Oxygen
(c) Ventilation
(d) Digoxin

51. Congestive heart failure, LVH and systolic heart murmur in a 29-day-old child
would suggest:
(a) TGA.
(b) VSD
(c) Rheumatic fever
(d) TOF (OPG/5th413)
Dr=Wahid Helmi

52. A neonate has central cyanosis and short systolic murmur on the 2nd day of
birth. The diagnosis is:
(a) TGV.
(b) TOF
(c) VSD
(d) ASD

53. A five-day-old. Full term male infant was severely cyanotic at birth.
Prostaglandin E was administered initially and later balloon atrial septostomy was
done which showed improvement in oxygenation. The most likely
diagnosis of this infant is:
(a) Tetralogy of Fallot
(b) Transposition of great vessels .
(c) Truncus arteriosus
(d) Tricuspid atresia

54. A neonate is found to have central cyanosis and a cardiac murmur at 30 hours
of age. The most probable diagnosis is.
(a) Endocardial cushion defect
(b) Ventricular septal defect
(c) Transposition of great vessels.
(d) Patent ductus arteriosus
55. True in total anomalous pulmonary connection are A/E
(a) The total pulmonary venous blood reaching the right atrium
(b) Always associated with VSD.
(c) The oxygen saturation of the blood in the pulmonary artery is higher to that in the
aorta
(d) Infracardiac type is always obstructive
Dr=Wahid Helmi

56. Figure of eight appearance is seen in:


(a) Partial anomalous pulmonary vessels
(b) Total anomalous pulmonary vessels .
(c) Truncus arteriosus
(d) Transposition of great vessels

49 C 50 A 51 A 52 A 53 B 54 C 55 B 56 B

THIRD PART
1. What is the most common congenital heart defect with a left to right shunt
causing congestive heart failure in the pediatric age group?
a. Atrial septal defect
b. Atrioventricular canal
c. Ventricular septal defect VSD
d. Patent ductus arteriosus
e. Aortopulmonary window
ANSWER: C

2-All true regarding ASD Exept:


a. Atrial septal defect is the second most common congenital heart defect in
children and adults.
b. Patients with atrial septal defects may have an embolic stroke as the initial
presentation.
c. Most children with atrial septal defects are asymptomatic.
d. The most common yet least serious type of atrial septal defect is an ostium
secundum defect.
e. The most common yet least serious type of atrial septal defect is ostium
primum defect.
ANSWER: E

3. What is the most likely age an infant with a large ventricular septal defect
will begin manifesting symptoms of congestive heart failure?
a. 1 day
b. 1 week
c. 1 month
d. 6 months
e. 1 year
ANSWER: C
Dr=Wahid Helmi

4. What is the dominant mechanism with which infants and young children
increase their cardiac output?
a. By increasing ventricular contractility
b. By increasing heart rate
c. By increasing ventricular end-diastolic volume
d. By decreasing heart rate
e. By increasing respiratory rate
ANSWER: B

5. The earliest sign of congestive heart failure on a chest X-ray is:


a. Increased heart size.
b. Kerley B lines.
c. Central pulmonary vascular congestion.
d. Pulmonary edema.
e. Pleural effusion.
ANSWER: A

6. A two day old cyanotic infant with a grade 3/6 ejection systolic murmur is
noted to have decreased pulmonary vascular markings on
chest x-ray and left axis deviation on EKG. The most likely diagnosis is:
a. Tetralogy of Fallot
b. Transposition of Great Vessels
c. Truncus Arteriosus
d. Tricuspid Atresia.
ANSWER: D

7. A 2 year old infant is noted to have mild cyanosis who assumes a


squatting position during long walking. He is noted to have
increasing fussiness followed by increasing cyanosis, limpness and
unresponsiveness. The most likely underlying lesion is:
a. Hypoplastic left heart
b. Transposition of the Great Vessels
c. Anomalous Pulmonary Venous Return
d. Tetralogy of Fallot
e. Aspiration with obstruction to air passages
ANSWER: D

8. An infant with a marked cyanotic congenital heart defect with decreased


pulmonary vascularity should be treated with:
a. Digoxin
b. Indomethacin
c. Prostaglandin E1
d. Epinephrine
ANSWER: C
Dr=Wahid Helmi

9. Cyanosis is produced by the presence of deoxygenated hemoglobin of at


least:
a. 1-2 gm/dL
b. 3-5 gm/dL
c. 6-8 gm/dL
d. 9-10 gm/Dl
ANSWER: B

10. A "tet spell" or "blue" spell of tetralogy of Fallot is treated with all of
the following except:
a. oxygen
b. knee chest position
c. morphine
d. digoxin
e. propranolol
f. phenylephrine
g. sodium bicarbonate
ANSWER: D

11. Pulmonary vascularity is increased in all of the following except:


a. TAPVR
b. Tricuspid atresia
c. TGV
d. Hypoplastic left heart
ANSWER: B

12. Pulmonary vascularity is decreased in all of the following except:


a. Tetralogy of Fallot
b. Pulmonary atresia
c. TAPVR
d. Tricuspid atresia
ANSWER: C

Fourth Revision
The side effect of hydralazine medication is:
a) Bronchospasms
b) Bradycardia
c) Hyperglycemia
d) Drug-induced lupus
e) Orthostatic hypotension
Ans… (d)
Hydralazine cause drug–induced lupus and tachycardia. Hydralazine is an
arterial vasodilator like diazoxide,nitroprusside, and minoxidil.
Dr=Wahid Helmi

The antihypertensive medication that causes bronchospasms is:


a) Propranolol
b) Clonidine
c) Diazoxide
d) Enalaprilat
e) Captopril
Ans (a) Propranolol and labetalol cause bronchospasms.
Propranolol also causes bradycardia and vivid dreams.

The preferred therapy for patients with hypertensive emergencies is:


a) Intravenous labetalol
b) Intravenous propranolol
c) Intravenous enalaprilat
d) Intravenous hydralazine
e) Intravenous phentolamine
Ans (a) Intravenous labetalol or nitroprusside or sublingual
nifedipine is the preferred therapy in patients with hypertensive
emergencies.

All of the following therapies are indicated initially in adolescents with


an essential hypertension except:
a) Weight reduction in obese
b) Reduce sodium intake
c) Aerobic exercise
d) Avoid tobacco
e) A small amount of red-wine (15 mL per day)
Ans (e) Adolescents should be counseled to avoid alcohol
and tobacco because of their adverse effects on blood pressure.
In a patient with hypertensive crisis, the blood pressure should be
reduced to about:
a) 95% of the total planned reduction within the first 2 hours and the
remaining 5% reduction over the next 4 hours.
b) 80% of the total planned reduction within the first 30 minutes and the
remaining 20% reduction over the next 24 hours.
c) 33% of the total planned reduction within the first 6 hours and the
remaining 67% reduction over the next 48-72 hours.
d) 20% of the total planned reduction within the first 3 hours and the
remaining 80% reduction over the next 12 hours.
e) 100% within the first 3 hours in order to avoid hypertensive
encephalopathy that is a potential complication.
Ans (c) The blood pressure should be reduced about one
third of the total planned reduction during the first 6 hours and remaining
two thirds over the following 2–3 days. A rapid reduction in blood pressure
may reduce the organ perfusions.
Dr=Wahid Helmi

The following antihypertensive medication may be avoided in patients


with abnormal renal functions:
a) Nifedipine
b) Furosemide
c) Captopril
d) Propranolol
e) Nitroprusside
(c) Captopril causes proteinuria, neutropenia, chronic cough, rash, and dysgeusia
(dysfunction of the taste sensation).

All children should have blood pressure


measurements at every well child visit at the age of:
a) 1 year
b) 2 years
c) 3 years
d) 4 years
e) 5 years
Ans (c) 3 years of age

All of the following are risk factors for developing cardiovascular


diseases in children except:
a) A child with elevated lipid levels
b) A child with a high blood pressure
c) The family history of a premature cardiovascular disease in a parent
(less than 55 years of age) or grandparent
d) The family history of high blood cholesterol levels
e) A 22-month-old child with a body weight that is above 97th percentile for
his/her age.
Ans
(e) The obesity for a child should be monitored at 2 years of age.
The most common clinical manifestation of myocarditis is:
a) Cardiac failure.
b) Bradycardia
c) Heart block
d) Hypertension
e) Bounding peripheral pulses
Ans (a) Cardiac failure. Arrhythmia and sudden death may be the first
clinical sings
Dr=Wahid Helmi

The most common cause of myocarditis is:


a) Diptheria
b) Viral infection
c) Toxoplasmosis
d) Histoplasmosis
e) Bacterial infections
Ans…. (b) Viral infection; answer (a), (b), (c), (d), and (e) also
cause myocarditis.

The most common primary malignant cardiac tumors in children is:


a) Sarcomas
b) Leukemia
c) Lymphomas
d) Wilms tumor
e) Mesotheliomas
Ans…. (a) Sarcomas

Furosemide therapy can cause all of the following complications


except:
a) Hyponatremia
b) Hypokalemia
c) Acidosis
d) Hypochloremia
e) Increased extracellular fluid volume (ECF)
Ans…. (c) Chronic furosemide therapy causes ‘contraction alkalosis’ due to
contraction of ECF volume.

The most common cause of death in pediatric patients with heart


transplantation is:
a) Rejection
b) Malignancy
c) Pulmonary hypertension
d) Cytomegalovirus infection
e) Graft coronary artery disease
Ans (d) Viral infections (32% of cases), especially CMV
infections (25% of cases) are the most common causes of death. Answer
(a), (b), (c), and (e) also cause death.
Dr=Wahid Helmi

The most common tumor that occurs in patients after cardiac


transplantation is:
a) Sarcomas
b) Leukemia
c) Mesotheliomas
d) Skin cancers
e) Lymphoproliferative disease
Ans... (e) PTLD (post –transplant–lymphoproliferative disease)
is associated with Epstein–Barr virus. The children can also develop. skin
cancers.

The most common cause of hypertension in newborn is:


a) Umbilical venous catheterization
b) Umbilical artery catheterization
c) Acute glomerulonephritis
d) Acute tubular necrosis
e) Excessive administration of intravenous fluids
Ans (b) Umbilical artery catheterization and renal artery thrombosis are the most
common causes

The side effects of propranolol include all of the following except:


a) Bradycardia
b) Bronchospasm
c) Hyperglycemia
d) Hypotension
e) Heart block
Ans (c) Hypoglycemia; other side effects are congestive heart failure and loss of
concentration or
memory
.

The following therapy is contraindicated in patients with supraventricular


tachycardia (SVT):
a) Verapamil
b) Ice bag over the face
c) Phenylephrine
d) Edrophonium
e) Synchronized DC cardioversion
Ans….. Verapamil causes hypotension, bradycardia, and cardiac arrest in
infants younger than 1 year of age
Dr=Wahid Helmi

The preferred therapy in a stable patient with SVT is:


a) Propranolol
b) Quinidine
c) Procainamide
d) Submersion of the face in iced saline
e) Adenosine
Ans (e) Adenosine

The preferred therapy in patients with SVT and severe cardiac failure
is:
a) Adenosine
b) Propranolol
c) Phenylephrine
d) Synchronized DC cardioversion
e) Quinidine
Ans…. (d) Synchronized DC cardioversion

Jervell-Lange-Nielsen syndrome (JLNS) is usually associated with:


a) Cataract
b) Deafness.
c) Hydronephrosis
d) Hepatitis
e) Choanal atresia
Ans (b) JLNS is associated with congenital deafness.JLNS is an autosomal recessive
disorder. It is an uncommon form of long Q–T syndrome.A newborn is admitted to
the NICU with respiratory distress.
Dr=Wahid Helmi

The newborn was born by NSVD with Apgar scores 8


and 9 at 1 and 5 minutes respectively. The physical examination reveals a
grade 2/6 systolic vibratory murmur at the left sternal border. The newborn
received 100% oxygen. A peripheral arterial blood gas reveals pH 7.34,
PCO2 45, and Po2 170. The preductal oxygen saturation is 96% and the
postductal oxygen saturation is 97%. The all for extremities BPs are the
following: LA 60/30, LL 65/34, RA 62/32, and RL 66/35. The chest x-ray
reveals hyperinflation, fluids in the fissure, and very small pleural effusions.
The right atrial and right ventricular oxygen saturations are 75%. The left
atrial and left ventricular oxygen saturations are 95%. The left ventricular
pressure is 100/8 and the aortic pressure is 100/60. The right ventricular
pressure is 25/3 and the pulmonary artery pressure is 25/10. The oxygen
consumption is 160 L/minute. The oxygen capacity is 200 mL/L. The
systemic and pulmonary blood flows are 5 L/minute/meter square. The
most likely diagnosis is:
a) Transposition of great vessels
b) Total anomalous pulmonary venous return
c) Tetralogy of Fallot
d) Critical Pulmonic stenosis
e) Normal cardiac anatomy
Ans: . (e) Normal cardiac
anatomy because all the parameters are normal. Please remember, the all
normal values. This newborn has
the x–ray findings of TTNB (transient tachypnea of the newborn).

The most common site the anomalous pulmonary veins drain into:
a) Right atrium
b) Infracardiac
c) Coronary sinus
d) Right superior vena cava
e) Left superior vena cava.
Ans (e) Left superior vena cava (40% of case)
791. The most common obstruction type of anomalous pulmonary venous
return is:
a) Infracardiac
b) Right atrium
c) Right superior vena cava
d) Left superior vena cava
e) Coronary sinus
Ans… (a) Infracardiac (95–100% of cases)
Dr=Wahid Helmi

A newborn appears with severe tachypnea and cyanosis at birth. The


newborn was intubated and placed on a mechanical ventilator. The
newborn remains ill. The physical examination reveals absence of heart
murmur but presence of mild intercostals retractions. The chest x-ray
reveals a small heart and a perihilar pulmonary edema. The EKG reveals a
right ventricular hypertrophy and the tall and spiked P waves. The oxygen
saturation in all four chambers are almost the same. The next step in
management is:
a) Blalock-Taussig shunt
b) Switch operation
c) Fontan operation
d) Repair of the truncus arteriosus
e) Surgical correction of total anomalous pulmonary venous return
Ans… (e) Surgical correction of TAPVR (obstructive type)

A newborn appears with mild tachypnea


and cyanosis at birth. The newborn received 100% oxygen by oxyhood.
The physical examination reveals a grade 2/6 systolic murmur along the left
sternal border and a gallop rhythm. The chest x-ray reveals a normal heart
size and an increased pulmonary vascularities. The EKG reveals a right
ventricular hypertrophy and the tall and spiked P waves. The next step in
management is:
a) Prostaglandin E1 infusions.
b) Indomethacin
c) Fontan operation
d) Switch operation
e) Blalock-Taussig shunt
Ans… (a) Prostaglandin E1 infusions are indicated in patients
with TAPVR (nonobstructive type) before the surgical correction.
A newborn appears with severe cyanosis and respiratory distress at birth. The
physical examination reveals aholosystolic murmur, gallop rhythm, and multiple
clicks over the left sternal border. The chest x-ray reveals a massive
cardiomegaly. The EKG reveals a right bundle branch block and WPW
syndrome. The child is placed on amechanical ventilator and infusion of PGE1
started. The next step in
management is:
a) Aortopulmonary shunt.
b) Blalock-Taussig shunt
c) Switch operation
d) Surgical closure of VSD
e) Surgical closure of ASD
Ans . (a) Ebstein anomaly of the tricuspid valve is treated with either
aortopulmonary shunt alone or by surgical patch closure of
tricuspid valve, atrial septectomy, and aortopulmonary shunt (Starnes
procedure).
Dr=Wahid Helmi

The most common organism in patients with Lemierre disease is:


a) Group A Streptococcus
b) S. aureus
c) Haemophilus influenzae
d) Klebsiella
e) Fusobacterium necrophorum
Ans (e) Fusobacterium necrophorum (anaerobic oropharyngeal bacteria)

A child appears with nausea, vomiting,decreased appetite, lethargy, and fever.


The child had a surgical repair of VSD 7 days age. He denies chest pain. The
findings in echocardiography may reveal:
a) Recurrence of VSD
b) Mitral valve stenosis
c) Aortic valve stenosis
d) Pulmonary valve regurgitation
e) Pericardial fluid.
Ans (e) In postpericardiotomy syndrome, patients may develop pericardial
effusion and cardiac tamponade.

The preferred therapy for patients with a postpericardiotomy syndrome


is:
a) Salicylates
b) Propranolol
c) Cefuroxime
d) Furosemide
e) Temporary pace maker
Ans (a) Salicylates or indomethacin and bed rest are indicated.
Sometimes, steroid therapy or pericardiocentesis is required.

The following is a contraindication to surgical closure of VSD:


a) Severe pulmonary vascular disease with VSD.
b) Supracrystal VSD
c) Qp : Qs ratio greater than 2:1 in a patient older than 24 months of age
d) Large VSD with failure to thrive
e) Large VSD with a developing pulmonary hypertension
Ans… (a) Severe pulmonary vascular disease
Dr=Wahid Helmi

The most common cause of heart transplants in children younger than 1 year of
age is:
a) Sinus arrythmia
b) Sinus bradycardia
c) Anomalous coronary artery
d) Total anomalous pulmonary venous return
e) Hypoplastic left heart syndrome
Ans (e) Congenital heart lesions (75% of cases)

[
In a case of suspected digoxin toxicity,serum digoxin level should be obtained:
a) Immediate after a dose
b) 30 minutes after a does
c) 1 hour after a dose
d) 2 hours after a dose
e) Immediately before a dose but at minimum 4 hours after the last dose
Ans (e) Immediately before a
dose but at minimum 4 hours after the last dose

The following condition increases digitalis toxicity:


a) Hyperkalemia
b) Hypernatremia
c) Hyponatremia
d) Hypocalcemia
e) Hypokalemia
Ans….. (e) Hypokalemia and hypercalcemia increase digitalis
toxicities
.
The digoxin therapy should be discontinued in the following condition:
a) Prolongation of P-R intervals
b) Changes in ST segments
c) Changes in T-waves
d) Blood level is 3 ng/mL in infants
e) Disturbances in new rhythms
Ans (e) A new rhythm disturbance is noted. The normal
blood levels of digoxin are 2–4 ng/dL in infants and 1–2 ng/dL in older
children.
.
Dr=Wahid Helmi

Blood pressure rises steadily beginning at approximately:


a) 1 month of age
b) 1 year of age
c) 2 years of age
d) 4 years of age
e) 6 years of age
Ans….( E ) 6 years of age

A 12 year-old obese boy has a strong family history of coronary heart disease.
His plasma total cholesterol level is 200mg/dL, HDL cholesterol level is
74/dL,and triglyceride level is 110mg/dL. His plasma LDL cholesterol level:
a) 96 mg/dL
b) 100 mg/dL
c) 104 mg/dL
d) 110 mg/dL
e) 112 mg/dL
Ans…….( C ) 104 mg/dL.LDL cholesterol= total cholestero l_ [HDL cholesterol+
(triglyceride/5)]
=200 _ [ 74+(100/5)]
=200 _96
=104 mg/dL

The preferred therapy for children older than 2 years with average LDL
cholesterol level more than 110 mg/dL is:
a) Statins
b) Nicotinic acid
c) Bile acid-binding resins
d) Dietary modification
e) Reassurance
Ans….( D )
Dietary modification I.e., their daily food intake should not provide more
than 30%of total calories as fat (approximately equal amount of saturated,
monosaturated,and polysaturated fats),and no more than 100 mg
cholesterol/1,000 calories (maximum 300 mg/day).This diet is called step I
diet from American Heart Association. The ideal goal is to lower LDL
cholesterol less than 110 mg/dL, but the minimal goal is to lower it less
than 130 mg/ Id
Dr=Wahid Helmi

A dietary intervention in children with hyperlipidemia usually reveals the


following result:
a) Lowers LDL cholesterol levels by more than 50%
b) Lowers LDL cholesterol levels by more than 40%
c) Lowers LDL cholesterol levels by more than 30%
d) Lowers LDL cholesterol levels by more than 20%
e) Lowers LDL cholesterol levels by no more than 10-15%
Ans...( E )
Lowers LDL cholesterol levels by no more than 10-15 %
The dietary management of hyperlipidemia in children is safe in the following
ages except:
a) 5 months.
b) 2 1/2 years
c) 3 years
d) 3 1/2 years
e) 4 years
Ans….( A )
The dietary management is safe in children older than 2 years of age.
Children younger than 2 years need large amount of calories to maintain
their rapid growth. Some authorities have demonstrated that a lower fat diet
can maintain normal growth in children as young as 6 months old

All of the following are risk factors in patients with hyperlipidemia except:
a) Diabetes
b) Hypertension
c) Smoking
d) Physical inactivity
e) Mild obesity
Ans…..( E ) Mild obesity is not a risk factor. However, severe obesity and low HDL
cholesterol levels are also risk factors

The first-line pediatric medicine for the therapy of hypercholesterolemia in


children is:
a) Statins
b) Bile acid -binding resin
c) Nicotinic acid
d) Fibrates
e) Absolute removal of fat from diet
Ans….( B ) Bile acid- binding resins (e.g., cholestyramine, colestipol, colesevelam)
primarily reduce LDL cholesterol levels in plasma. However, they can
enhance hypertriglyceridemia in children with triglyceride levels greater
than 300 mg/dL. Answers (a), (c), and(d) are also lipid-lowering
medications
Dr=Wahid Helmi

All of the following drugs raise triglyceride levels except:


a) Oral contraceptives
b) 13-ci-retinoic acid
c) Thiazide diuretics
d) HIV protease inhibitors
e) Phenytoin.
Ans…...( E )
Phenytoin does not increase triglyceride levels. Other drugs that raise
triglyceride levels are steroids, immunosuppressants, and some
beta-adrenegic blocking agents

All of the following conditions are associated with Hyperlipidemia except:


a) Hyperthyroidism
b) Diabetes mellitus
c) Nephrotic syndrome
d) Anorexia nervosa
e) Congenital biliary atresia
Ans…..( A )
Hyperthyroidism is not associated with hyperlipidemia.Other conditions
associated with hyperlipidemia and hypothyroidism, storage diseases (e.g.,
Tay-Sachs disease, Niemann-pick disease, glycogen storage disease),
hepatitis, systemic lupus erythematosus, and excessive alcohol intake (in
teenagers)
Dr=Wahid Helmi

The following statement is not true about the drug treatment in children with
hyperlipidemia aged 10 years and older,after an adequate trial (6-12 months)of
diet therapy:
a) General pediatrician should be able to treat children who need drug
therapy
b) Drug therapy is indicated when LDL cholesterol level is greater than 190
mg/dL
c ) Drug therapy is indicated when LDL cholesterol level is greater than
160 mg/dL and family history of premature coronary heart disease
d) Drug therapy is indicated when LDL cholesterol level is greater than 160
mg/dL in children with diabetes and hypertension
e) Drug therapy is indicated when LDL cholesterol level is greater than 160
mg/dL in children with severe obesity and low HDL cholesterol level
Ans…..( A )
General pediatrician should refer all children who need drug therapy to a
specialized pediatric lipid center. Drug therapy is also indicated when LDL
cholesterol level is greater than 160 mg/dL in children with smoking and
physical inactivity.

Hypertensive encephalopathy is most commonly associated with the following


disease:
a) Renal.
b) Hepatic
c) Adrenal
d) Pituitary
e) Thyroid
Ans…..( A ) Renal disease (e.g., acute glomerulonephritis, chronic pyelonephritis,
end-stage renal disease)

A child with congenital heart disease receiving loop diuretic. His serum
bicarbonate level is 31.The preferred management is:
a) Stop loop diuretic
b) Start digoxin
c) Add potassium chloride
d) Switch to another diuretic
e) No intervention is indicated
Ans...( E ) No intervention is indicated because the patient has mild metabolic
alkalosis ( I.e.,bicarbonate level is less than 32)
Dr=Wahid Helmi

A mother is admitted at 20th week of pregnancy. The fetus developed cardiac


failure due to arrhythmia. The preferred therapy is:
a) Lasix to the mother
b) Lasix to the fetus
c) Digoxin to the mother
d) Digoxin to the fetus
e) Deliver the fetus
Ans...( C )
Digoxin should be given to the mother and it crosses the placenta to the
fetus

A child has dilated cardiomyopathy. She is receiving adequate digoxin and


furosemide therapies. Here condition improve but not significantly. The
preferred therapy is:
a) Continue digoxin and increase furosemide dose
b) Increase digoxin and furosemide doses
c) Discontinue digoxin and add ACE inhibitor along with furosemide
d) Continue digoxin and furosemide, add ACE inhibitor
e) Continue the same management
Ans….( D ) Afterload-reducing agents and ACE inhibitors ( e.g., captopril, enalapril)
are used in conjunction with digoxin and furosemide. Afterload-reducing agents
(e.g., ACE inhibitors, Hydralazine, nitroglycerin, nitroprusside, prazocin,
carvedilol) are not generally used in patient with stenotic lesions of the left
ventricular outflow tract

All of the following are mechanisms of action of ACE inhibitors (e.g., captopril )
except:
a) Arterial dilatation
b) Venous dilatation
c) Increase aldosterone production
d) Reduced afterload and preload
e) Cardiac remodeling independent on their
influence on afterload
Ans...( C ) ACE inhibitors (e.g., captopril) decrease aldosterone production and
reduce salt and water retention. These medications are used in hypertension and
cardiac failure (nonsteotic lesions).Arterial dilatation causes afterload
reduction and venous dilatation causes preload reduction.The principal
effect of ACE inhibitors is arterial dilatation
Dr=Wahid Helmi

Adverse reactions of captopril include all of the following conditions:


a) Maculopapular rash
b) Chronic cough
c) Neutropenia
d) Hypotension
e) Improves renal function
Ans…...( E ) Captopril produces renal toxicity and failure
A child has been receiving high dose intravenous nitroprusside for
the past several days. He has an uncontrolled hypertension. The
following parameter should be monitored:
a) Serum BUN
b) Serum creatinine
c) Liver function tests
d) Pancreatic enzymes
e) Blood thiocyanate level
Ans….( E ) Blood thiocyanate level (more than 10 g/dL causes
toxicity)

A child developed nausea, fatigue,disorientation, acidosis, and


muscular spasms.She is in ICU.Most likely the child is receiving the
following medication:
a) Captopril
b) Hydralazine
c) Digoxin
d) Dopamine
e) Nitroprusside
Ans…..( E )
Nitroprusside (intravenous)administration causes thiocyanate poisoning

The use of nitroprusside is contradicted in


the following condition:
a) Seriously ill patients
b) Pre-existing hypotension
c) Increased preload
d) Hypertension
e) Increased afterload
Ans….( B )
Nitroprusside should not be used in patients with pre-existing
hypotension
because it can cause sudden hypotension. Nitroprusside is used in
critically
ill patients with hypertension. It primarily causes peripheral arterial
vasodilatation and afterload reduction, but may cause venodilatation and
preload reduction by decreasing venous return to the heart
Dr=Wahid Helmi

The following statement is not true abou dopamine:


a) Predominantly alph -adrenergic agonist
b) Selective renal vasodilator effect
c) Useful in patients with a compromised renal function
d) Vasoconstriction predominates if the dose is increased above 15
microgram/kg/minute
e) Increase cardiac contractility with little peripheral vasoconstriction at a
dose 2-10 microgram/kg/minute
Ans….( A ) Dopamine is predominantly beta-adrenegic agonist

Major side effect of milrinone


(phosphodiestrase inhibitor)is:
a) Hypertension
b) Hypotension
c) Bradycardia
d) Thrombocytopenia
e) Apnea
Ans...( B ) Hypotension is secondary to a peripheral vasodilatation.
Milrinone is used
in patients with low cardiac output who are unresponsive to standard
therapy. Milrinone is used as an adjunct to dopamine or dobutamine

Preferred therapy in patients with milrinone-induced hypotension is:


a) Hydralazine
b) Decrease intravenous fluids
c) Increased intravenous fluids
d) Captroil
e) Nitroprusside
Ans...( C ) Increased administration of intravenous fluids increase the
intravascular
volume. Other choices cause hypotension
Dr=Wahid Helmi

The side effect of amrinone therapy is:


a) Thrombocytosis
b) Thrombocytopenia
c) Leukocytosis
d) Polycythemia
e) Lymphocytosis
Ans….( B )
Amrinone is a phosphodiestrase inhibitor.It causes thrombocytopenia that
depends on the rate of infusion and duration of therapy

A child was admitted to the ICU for dilated cardiomyopathy. He has


improved and the acute condition is over. The following medication is
added as a part of chronic comprehensive heart failure treatment
program:
a) Vitamin E
b) Metoprolol
c) Carvedilol
d) Milrinone
e) Vitamin A
Ans...( B )
Metoprolol (a selective beta 1-adrenergic receptor antagonist)is used
commonly. Answer (c) is currently under study .Answer (d)is used in
ICU
patients. Answers (a ) and (e) do not make sense. Metoprolol improves
exercise tolerance; decreases hospitalization and mortality
Dr=Wahid Helmi

A 3.9 kg newborn appears with cyanosis at 2 hours of age, cyanosis


progressively gets worse, also dyspnea. An examination reveals lower
extremities are less cyanotic than upper extremities, hyperactive
precordium, either no murmur or PDA murmur of grade 3/6, 2nd heart
sound single and loud, or occasionally it may be split. Chest X-ray reveals
mild cardiomegaly, normal or increased pulmonary flow, narrow
mediastinum. ABG reveals Pao2 15 to 30, pH 7.30, Pco2 35, O2 saturation
30 to 70%, base deficit – 15.0. Pao2 increases slightly after hyperoxia test
(i.e., 100% oxygen). EKG reveals biventricular hypertrophy. Newborn was
intubated and placed on mechanical ventilator. PGE1 and NaHCO3 are
given. Baby’s oxygen saturation improved up to 85 to 90%. Next
appropriate step in management:
a) Fontan procedure
b) Blalock-Taussig shunt
c) Rashkind balloon atrial septostomy.
d) Norwood operation
e) Starnes procedure
Ans….(c) Newborn is diagnosed with transposition of great
vessels, therefore Rashkind baloon atrial septostomy is the initial
procedure of choice.
Dr=Wahid Helmi

A newborn male appears with rapid heart rate. He is afebrile. In room


air, oxygen saturation is 100% recorded by pulse oximeter. Heart rate is
260 per minute, respiratory rate is 46 per minute, and temperature is
98.6°F. Management of this patient includes all of the following except:
a) Verapamil
b) Adenosine
c) Ice pack over face
d) DC cardioversion
e) Propranolol
Ans…. (a) Verapamil is not used in children less than 1 year of age
because it reduces cardiac output and produces hypotension and cardiac
arrest.

A full-term newborn boy was admitted to NICU (neonatal intensive care


unit) with mild respiratory distress. He was born by cesarean section with
Apgar scores 8 an 8 at 1 and 5 minutes respectively. Initially baby was
Dr=Wahid Helmi

placed under 40% oxygen by oxyhood. Oxygen saturation was 85% by


pulse oximeter. Physical examination reveals tachypnea but no heart
murmur. Then, oxygen was increased to 50%. Chest X-ray reveals
increased pulmonary vascular markings. ABG: pH 7.30 / Pco2 42 / Po2 65
in 40% oxygen. Umbilical arterial and venous lines were placed. Po2 is
higher in umbilical venous line than that of arterial line. Repeat CXR
reveals umbilical venous line is in the right atrium and umbilical arterial line
at T8 level. Most likely diagnosis:
a) TTNB (transient tachypnea of newborn)
b) TGA (transposition of great arteries)
c) VSD
d) ASD
e) TAPVR (total anomalous pulmonary venous return).
Ans…. (e) TAPVR because all four pulmonary veins drain into right
atrium. Pulmonary veins contain oxygenated blood to right atrium.
Therefore, umbilical catheter in right atrium has higher oxygen than that of
umbilical arterial line.
Dr=Wahid Helmi

3. A 17-year-old male athlete suddenly collapsed while playing football.


He was brought to the ER. He expired despite adequate CPR. Past
medical history was unremarkable. Autopsy was performed. He had no
history of smoking, drugs, and alcohol. Most likely cause of death:
a) Critical aortic stenosis
b) Mitral valve prolapse
c) Aortic incompetence
d) Hypoplastic left heart syndrome
e) Long QT interval syndrome.
Ans…. (e) A malignant form of ventricular arrhythmia called
torsades de pointes occurs in patients with LQT syndrome and is a cause
of syncope or sudden death.

Heart disease is most common with:


a) Trisomy 21
b) Trisomy 18.
c) Trisomy 13
d) Turner syndrome
e) Trisomy 22
Ans (b) Heart disease is most common in trisomy 18 (more than
90% of cases).
Dr=Wahid Helmi

A 3-day-old newborn infant was diagnosed with congenital cyanotic


heart disease. Cardiac surgery was performed. The cardiac output was
decreased after surgery. The clinical manifestations suggestive of
decreased cardiac output:
a) Tachycardia
b) Bradycardia
c) Hypertension
d) Hypotension and prolonged capillary refilling time.
e) Hypotension and normal capillary refilling time.
Ans... (d) Hypotension and prolonged capillary refilling time indicate
decreased cardiac output.

The onset of routine screening for hypertension should be:


a) 6 months
b) 12 months
c) 3 years
d) 6 years
e) 12 years
Ans….. (c) Hypertension screening should begin at 3 years of age.
Dr=Wahid Helmi

A 3-year-old boy was admitted to pediatric cardiac ICU after surgical


repair of a coarctation of aorta. After the surgery, physical examination
reveals grade 2/6 systolic ejection murmur in the upper right sternal border.
Most likely cause of this murmur:
a) Congenital bicuspid aortic valve
b) Persistent coarctation of aorta
c) Functional murmur
d) Anemia causing murmur
e) Atrial septal defect
Ans…. (a) Congenital bicuspid aortic valve is an associated
anomaly in coarctation of aorta.

Physical activities should be limited in the following cardiac disease:


a) VSD
b) ASD
c) PDA
d) Aortic valve stenosis
e) Pulmonary artery stenosis
Ans….. (d) Aortic valve stenosis 320. A well-developed, well-nourished infant appears
with systolic ejection murmur in the right upper sternal border. The murmur heard best
during expiration.
Dr=Wahid Helmi

The 2nd heart sound was split. Most likely diagnosis:


a) Pulmonic stenosis
b) ASD
c) VSD
d) Aortic stenosis
e) Mitral regurgitation
Ans…. (a) Pulmonic stenosis

A female child is diagnosed with hyperkalemia. Physical examination


reveals irregular and rapid heart rates. The serum K level is 7.5 mEq/L. The
rest of the SMA 6 results are normal. The first drug of choice in this patient:
a) Intravenous glucose and insulin
b) Oral kayexalate
c) Slow intravenous calcium gluconate with ECG monitoring
d) Intravenous sodium bicarbonate
e) Intravenous salbutamol
Ans… (c) IV calcium gluconate
Dr=Wahid Helmi

A preterm 26 weeks gestational age newborn required surgical ligation


for PDA (patent ductus arteriosus). All of the following are expected after
surgical ligation of PDA except:
a) Child gains weight
b) Cardiac failure disappears
c) Less respiratory infections
d) EKG becomes normal
e) Absence of a functional systolic murmur over pulmonary area.
Ans… (e) A functional systolic murmur over pulmonary area may
be present.

Most common cause of thromboembolic stroke in children:


a) Coagulation disorder
b) Polycythemia
c) CNS trauma
d) Aortic injury
e) Cardiac abnormalities
Ans…. (e) Cardiac abnormalities
Dr=Wahid Helmi

A child was brought to the ER. He was resuscitated but expired.


Mother told that the boy was scheduled to have a cardiac surgery
6 weeks from now. Autopsy was performed. Most likely diagnosis:
a) Mitral stenosis
b) Mitral valve prolapse
c) Pulmonic stenosis
d) Aortic coarctation
e) Congenital aortic stenosis
Ans…. (e) Congenital aortic stenosis.

A full-term male newborn is delivered by NSVD. His mother has


history of lupus. The most common complication:
a) Mitral stenosis
b) Aortic stenosis
c) Cardiac tumor
d) Congenital heart block.
e) VSD
Ans…. (d) Congenital heart block

Most common cardiac lesion in trisomies:


a) ASD b) VSD c) Aortic stenosis d) Mitral stenosis e) Tricuspid
regurgitation
Ans….b) VSD
Dr=Wahid Helmi

A child appeared in the ER with lethargy and palpitation. Physical


examination revealed a hypotension and irregular heart beats.
Cardiopulmonary resuscitation was failed. Mother stated that the child was
receiving a prescribed drug:
a) Azithromycin
b) Alpha methyl dopa
c) Desipramine
d) AZT
e) Prednisone
Ans... (c) Desipramine

A 5-year-old boy appears in a clinic for routine check-up. Physical


examination reveals grade 2/6, systolic, musical heart murmur in the left
sternal border. Most likely diagnosis:
a) PDA
b) VSD
c) ASD
d) Aortic stenosis
e) Innocent murmur.
Ans... (e) Functional murmur
Dr=Wahid Helmi

A 14-year-old girl is diagnosed with scoliosis. The other organ defect


associated with scoliosis:
a) Renal
b) Eye
c) Liver
d) Cardiac.
e) ENT
Ans…. (d) Cardiac, mitral valve prolapse

A foramen ovale usually closes by:


a) 1 month
b) 2 months
c) 3 months.
d) 4 months
e) 6 months
Ans (c) Foramen ovale usually close by 3 months of age

A full-term, LGA male was born by a cesarean section due to fetal


distress. Apgar scores were 8 and 9 at 1 and 5 minutes respectively. His
mother had diabetes during this pregnancy. Most common cardiac
anomaly:
a) VSD b) ASD c) Cardiac failure d) Cardiomegaly.
e) Asymmetric septal hypertrophy
Ans (d) Cardiomegaly
Dr=Wahid Helmi

A 8-year-old boy has been taking digoxin for the last 3 months. He
appears in the ER with history of vomiting, diarrhea, diplopia, yellow or
green vision, and photophobia for the last 12 hours. Physical examination
reveals
bradycardia and dry mucous membranes. EKG reveals prolonged P – R
intervals. The next step in management:
a) Intravenous hydration
b) Oral hydration
c) Stop digoxin and give digoxin-specific Fab antibodies.
d) Opthalmology consult
e) Increase the digoxin dose
Ans….c) Stop digoxin and give digoxin-specific Fab antibodies

The hypertension is a characteristic feature of:


a) 21-hydroxylase deficiency
b) Carboxylase deficiency
c) 11 beta-hydroxylase deficiency
d) Beta glucosidase deficiency
e) 3 beta-hydroxysteroid dehydrogenase defect
Ans….   (c) 11 beta-hydroxylase deficiency causes hypertension.
A full-term newborn male was intubated in the delivery room. Apgar
scores were 2, 5, 7 at 1, 5, and 10 minutes respectively. He was brought to
NICU and was placed on a mechanical ventilator. Physical examination
revealed grade 2-3/6 systolic murmur in lower sternal border and equal air
entry in both lungs. Chest X-ray revealed mild cardiomegaly. EKG revealed
Dr=Wahid Helmi

S-T and T-wave changes. Reexamination at 12 hours of age revealed


louder murmur and progressive deterioration of his clinical condition. The
ventilator settings were adjusted accordingly. After 24 hours of age clinical
condition improved and heart murmur disappeared. Most likely diagnosis:
a) Meconium aspiration
b) VSD
c) Mitral valve regurgitation
d) Tricuspid valve regurgitation
e) PDA
Ans….   (d) Tricuspid regurgitation
A 6-year-old boy had a modified Blalock-Tausig shunt for tetralogy of
Fallot. About 12 hours after surgery, the boy developed respiratory distress.
Chest X-ray revealed pleural effusion. Most likely diagnosis:
a) Hemothorax
b) Chylothorax
c) Pneuthorax
d) Hydrothorax
e) Diaphragmatic hernia
Ans….   (b) Chylothorax
A full-term male infant is delivered by an emergency cesarean section
for a fetal distress. At birth, the boy is limp, cyanotic, and has no respiratory
effort. He is dried, suctioned, and stimulated. However, no clinical
improvement is noted. His Apgar score is 1 for heart rate 60/minute. The
next step in the management:
a) Chest compression
Dr=Wahid Helmi

b) Intubation STAT
c) Free-flow oxygen at a rate of 10 L/minute
d) Vigorous stimulation
e) Bag-and-mask ventilation with a rate of at least 10 L/minute.
Ans…..   (e) The boy needs oxygen by bag and mask. The infant
usually improves with that therapy.
Most common indication of cardiac transplantation:
a) Hypoplastic left heart syndrome
b) Cardiomyopathies
c) Multiple cardiac tumors
d) Tricuspid atresia
e) Single ventricle
Ans…...   (b) Cardiomyopathies
A full term newborn male appears cyanotic at birth. He was intubated
in the delivery room and received 100% oxygen. Maternal history of type I
diabetes. The boy was placed on a mechanical ventilator with the setting of
an IMV 40, PIP 20, PEEP 5, FiO2 100%, inspiratory time 0.5 second, and
flow rate 10 L/minute. Oxygen saturation is 100%. Physical examination
reveals plethora, heart rate is 190 per minute, respiratory rate is 80 per
minute, blood pressure 58/30, grade 3/6 systolic ejection murmur in the
mid-left sternal border, and liver is palpable 5 cm below the right costal
margin. Chest X-ray reveals increased pulmonary congestion. EKG reveals
increased anterior and posterior forces. CBC reveals WBC count 21,000,
60% polymorphs, 40% lymphocytes, platelet count 210,000, hemoglobin 21
g/dL and hematocrit 63. Most likely diagnosis:
Dr=Wahid Helmi

a) VSD
b) ASD
c) Pulmonic stenosis
d) Diabetic cardiomyopathy
e) Aortic stenosis
Ans…..    (d) Diabetic cardiomyopathy
Mitral valve prolapse is found in all of the following conditions except:
a) Marfan syndrome
b) Down syndrome
c) Scoliosis
d) Pectus excavatum
e) Straight back syndrome
Ans….   (b) Down syndrome patients do not have mitral valve prolapse
Most common cause of secondary hypertension in children:
a) Aortic coarctation
b) Portal vein thrombosis
c) Umbilical arterial catheter
d) Pheochromocytoma
e) Renal abnormality
Ans….   (e) Renal abnormality
A 4-day-old 26 weeks gestational age preterm infant on a mechanical
ventilator developed PDA (patent ductus arteriosus) murmur. The fluid was
restricted. However, the murmur was audible and PDA was confirmed by
an echocardiogram. Indomethacin therapy is contraindicated when:
a) BUN value is 18 mg/dL
Dr=Wahid Helmi

b) Platelet count is 55,000


c) Creatinine level is 1.9 mg/dL
d) Necrotizing enterocolitis
e) Urine output is 1.1 mL/Kg/hour
Ans….   (c) Indomethacin is contraindicated in patients with bleeding
disorder, NEC, oliguria (less than 1 mL/kg/hr), elevated creatinine level
(more than 1.8 mg/dL), and thrombocytopenia (less than 50,000/mm3).
Most common organism causing bacterial endocarditis after a dental
procedure:
a) S. aureus
b) Streptococcus viridans
c) Group D enterococcus
d) Pseudomonas aeruginosa
e) Serratia marcescens
Ans….   (b) Streptococcus viridans
A 15-year-old girl appears with a history of occasional chest pain and
palpitations for the last 6 months. She is a very bright student. Physical
examination reveals a late systolic murmur at apex with click. The murmur
is prolonged in late systole when the patient is standing or sitting condition.
She also has a thoracic scoliosis: Most likely diagnosis:
a) Mitral valve prolapse (MVP)
b) Tricuspid valve regurgitation
c) Mitral valve stenosis
d) Tricuspid atresia
e) Mild aortic stenosis
Dr=Wahid Helmi

Ans….   a) Mitral valve prolapse (MVP


Treatment of choice for moderate-to-severe hypertension in neonate:
a) Diazoxide
b) Propranolol
c) Hydralazine
d) Furosemide
e) Methldopa
Ans….   c) Hydralazine
A 2-day-old full term female newborn appears with poor feeding and
dyspnea. Physical examination reveals blood pressure 35/20, respiratory
rate 70 per minute, temperature 98.6°F, weak peripheral pulses, grayish
blue color of skin, hepatomegaly, and right ventricular heave. Most likely
diagnosis:
a) Aortic stenosis
b) Coarctation of aorta
c) Hypoplastic right heart
d) Hypoplastic left heart syndrome
e) Bicuspid aortic valve
Ans….   d) Hypoplastic left heart syndrome
A full term female newborn is admitted in NICU with meconium
aspiration. A few hours later, she developed PPHN (persistent pulmonary
hypertension). The best diagnostic study for PPHN:
a) Chest X-ray
b) Hyperoxia test
c) Echocardiogram
Dr=Wahid Helmi

d) Pulmonary function test


e) Arterial blood gas
Ans….   c) Echocardiogram

A 16-year-old boy appears in a clinic for a routine check up. Physical


examination reveals an average blood pressure 145/90, respiratory rate 20
per minute, pulse rate 74 per minute, and rest of the findings are
unremarkable. His father and uncle have a high blood pressure. His
grandfather died of a stroke at the age of 62. His weight and height are
above 95 th percentile. Laboratory test results reveal a normal CBC, SMA
6, and urinalysis. He is aware that he should control his weight and eating
habits. After 1 month, repeat blood pressure is 140/90 (checked twice).
Most likely diagnosis:
a) Renal artery stenosis
b) Aortic coarctation
c) Essential hypertension.
d) Hyperparathyroidism
e) Normal blood pressure
Ans….   (c) Essential hypertension
A routine newborn physical examination of a newborn reveals a grade
2/6, short, harsh systolic murmur at the apex. The newborn is completely
asymptomatic. The most likely diagnosis is:
a) Atrial septal defect
b) Ventricular septal defect.
c) Mitral stenosis
d) Mitral regurgitation
Dr=Wahid Helmi

e) Patent ductus arteriosus


Ans….   b) VSD

The first retinal sign in patients with a hypertensive retinopathy is:


a) Edema
b) Cotton-wool spots
c) Flame-shaped hemorrhages
d) Papilledema.
e) Generalized constrictions of retinal arterioles.
Ans….e) Generalized constrictions and irregular narrowing of
retinal arterioles are the first signs of hypertensive retinopathy. Answers
(a), (b), (c), (d), and thickening of the retinal vessels (i.e., silver-or
copper-wire appearance) also occur in hypertensive retinopathy.

The most common cardiovascular anomaly in patients with Alagille


syndrome is:
a) Transposition of great vessels
b) VSD
c) ASD
d) Tetralogy of Fallot
e) Peripheral pulmonic stenosis.
Ans... e) Peripheral pulmonic stenosis is usually present;
sometimes TOF is present.
Dr=Wahid Helmi

A newborn is diagnosed with a Shone complex. The following features


are present in this newborn:
a) Right heart obstructive lesions
b) Left-sided cardiac obstructive lesions.
c) Right-sided cardiac nonobstructive lesions
d) Left-sided nonobstructive lesions
e) Coarctation of the aorta
Ans….b) Left-sided cardiac obstructive lesions (e.g., a combination
of mitral valve, subaortic, aortic valve, and aortic arch) are called the Shone
complex.

A surgical repair of the coarctation of aorta was performed in an infant.


The next day after surgery, the infant developed vomiting, abdominal
distension, bloody stools, hypertension, and leukocytosis. The most likely
etiology is:
a) Gastric ulcers
b) Duodenal ulcers
c) Intestinal perforations
d) Gram-negative sepsis
e) Mesenteric arteritis.
Ans... e) Mesenteric arteritis can occur in patients with
postcoarctectomy syndrome. A patient may develop bowel necrosis and
small bowel obstruction. A patient improves with antihypertensive
medication (e.g., nitroprusside, esmolol, captopril). A surgical exploration of
the abdomen is rarely indicated.
Dr=Wahid Helmi

A high-output cardiac failure occurs in the following condition:


a) Myocarditis
b) Constrictive pericarditis
c) Large systemic arteriovenous fistulas.
d) Aortic stenosis
e) Hypoplastic left heart syndrome
Ans... c) Arteriovenous fistulas. In a patient with high-output cardiac
failure, cardiac output is more than normal and myocardial abnormalities
are absent
.

The most common cause of morbidity in patients with Marfan syndrome


is:
a) Progressive aortic root dilatation
b) Scoliosis
c) Dislocated lens
d) Funnel chest
e) Progressive mitral valve prolapse.
Ans….e) Marfan syndrome patients with progressive mitral valve prolapse appear with
arrythmias, endocarditis, cardiac failure, or
thromboemboli

A surgical repair of the coarctation of aorta was performed in an infant.The next day after surgery, the infant
developed vomiting, abdominal distension, bloody stools, hypertension, and leukocytosis. The most likely
etiology is:
a) Gastric ulcers
b) Duodenal ulcers
c) Intestinal perforations
d) Gram-negative sepsis
e) Mesenteric arteritis.
Dr=Wahid Helmi

A high-output cardiac failure occurs in the following condition:


a) Myocarditis
b) Constrictive pericarditis
c) Large systemic arteriovenous fistulas.
d) Aortic stenosis
e) Hypoplastic left heart syndrome
Ans... c) Arteriovenous fistulas. In a patient with high-output cardiac
failure, cardiac output is more than normal and myocardial abnormalities
are absent.

The most common cause of morbidity in patients with Marfan syndrome


is:
a) Progressive aortic root dilatation
b) Scoliosis
c) Dislocated lens
d) Funnel chest
e) Progressive mitral valve prolapse.
Ans….e) Marfan syndrome patients with progressive mitral valve
prolapse appear with arrythmias, endocarditis, cardiac failure, or
thromboemboli
Dr=Wahid Helmi

The most common cardiac defect in patients with Marfan syndrome is:
a) Aortic root dilatation.
b) Aortic stenosis
c) Mitral valve prolapse
d) Mitral stenosis
e) Coarctation of aorta
Ans….a) Aortic root dilatation

The sudden death in children is most commonly involved the following


organ system:
a) CNS
b) Pulmonary
c) Cardiac
d) Gastrointestinal
e) Hepatic
Ans…..c) Cardiac
Dr=Wahid Helmi

The following factor reduces a risk of SIDS (sudden infant death


syndrome):
a) Breast feeding
b) Prone (and side) sleep position
c) Pacifier (dummy) use.
d) Male gender
e) Recent febrile illness
Ans….c) Pacifier (dummy) use significantly reduces the risk of SIDS in infants when used
routinely and during sleep. The mechanism is unknown. It may be due to a direct effect of
pacifier or associated infant or parental behaviors. Breast feeding is recommended for
many advantages but is not recommended to reduce the SIDS. Answers (b), (d), and (e) can
increase a risk of SIDS.

Risk of SIDS is highest between:


a) 0-1 month of age
b) 1-2 months of age
c) 2-4 months of age.
d) 4-6 months of age
e) 6-9 months of age
Ans….c) 2-4 months of age.

A child appears with a history of syncopal episode during exercise,


fright, or sudden startle response. The preferred diagnostic study is:
a) EEG
Dr=Wahid Helmi

b) Head CT scan
c) Head MRI
d) ECG
e) Hearing test
Ans….d) ECG can reveal long Q-T syndrome.
An ECG finding in a patient reveals a heart-rate corrected Q-T interval
is 0.49 second. The most likely diagnosis is:
a) First-degree heart block
b) Second-degree heart block
c) Third-degree heart block
d) Long Q-T syndrome
e) Normal
Ans….d) Long Q-T syndrome; a heart-rate corrected Q-T interval
of more than 0.47 second is highly indicative of long Q-T syndrome
and more than 0.44 second is suggestive of long Q-T syndrome.
143. A child appears with fainting spells. A physical examination reveals a
normal neurological examination and positional (supine vs erect) character
of murmur on the left sternal border near mitral area. The preferred therapy
is:
a) Prophylactic antibiotic
b) Dilatation of the mitral valve
c) Repair of VSD
d) Repair of ASD
e) Removal of myxomas
Ans….e) A patient with myxoma appears with fainting spells and
have a positional character of murmur. Myxomas should be removed
Dr=Wahid Helmi

completely.
The most common location of cardiac myxomas is:
a) Right atrium
b) Left atrium
c) Right ventricle
d) Left ventricle
e) Interventricular septum
Ans….b) Left atrium (75% of cases); these smooth, pedunculated
masses arise from an interatrial septum and protrude into the left atrium.
150. The most common cause of congenital heart disease is:
a) Chromosomal abnormalities
b) Maternal alcohol intake..
c) Single gene mutation
d) Maternal drug intake
e) Unknown
Ans….e) Unknown
A congenital heart disease is most common in the following condition:
a) Trisomy 21
b) Trisomy 18
c) Trisomy 13
d) Turner syndrome
e) Trisomy 22
Ans….b) Trisomy 18 (90% of cases); trisomy 21 (50% of cases);
Turner syndrome (40% of cases)
All of the following heart diseases are more common in girls than that
of boys except:
Dr=Wahid Helmi

a) ASD
b) VSD
c) PDA
d) Pulmonic stenosis
e) Transposition of great vessels
Ans…..e) Transposition of great vessels and left-sided obstructive
lesions (e.g., aortic stenosis, hypoplastic left heart syndrome) are
more common in boys than in girls.
A 12-year-old boy came for a routine physical examination. The boy is
asymptomatic. The blood pressure recorded in the right upper extremity is
140/90. The blood pressure recorded in the left upper extremity is 138/88.
The next step in management is:
a) Measure blood pressures in both lower extremities
b) ECG
c) Chest x-ray
d) Serum electrolytes
e) Liver function tests
Ans….a) Measure blood pressures in both lower extremities. In a
normal person, systolic blood pressure in legs is 10-20 mm Hg higher than
in arms. In patients with a coarctation of the aorta, blood pressures in lower
extremities are lower than in upper extremities.

The most common complication of patients with a supracrystal VSD is:


a) Right ventricular failure
b) Left ventricular failure
Dr=Wahid Helmi

c) Pulmonic stenosis
d) Aortic insufficiency
e) Arrythmias
Ans….d) Aortic insufficiency (50-90% of cases) is due to a
prolapse of aortic valve into a defect.
The most common clinical presentation in patients with a mild or
moderate Pulmonic stenosis is:
a) Right ventricular failure
b) Hepatic failure
c) Tachypnea
d) Tachycardia
e) Asymptomatic
Ans... e) Asymptomatic
The most common cardiac lesion in patients with a rheumatic heart
disease is:
a) Mitral stenosis
b) Aortic stenosis
c) Aortic insufficiency
d) Tricuspid stenosis
e) Mitral insufficiency
Ans... e) Mitral insufficiency

In newborn infants with a congenital heart disease, the chance that a


murmur heard at birth is:
a) 1/2
Dr=Wahid Helmi

b) 1/4
c) 1/8
d) 1/12
e) 1/25
Ans... d) 1/12 (i.e., 8.3%)
Transposition of great vessels are most commonly associated with:
a) ASD
b) VSD
c) Pulmonic stenosis
d) Aortic stenosis
e) Tricuspid regurgitation
Ans….b) VSD (about 50% of cases)
A 9-month-old appears with a respiratory distress. He was previously
admitted for recurrent pneumonia. A physical examination reveals
wheezing, intercostals retractions, mild cyanosis, hyperdynamic
precordium, and a loud to-and-fro murmur over theleft upper sternal border.
The most likely diagnosis is:
a) Pulmonic stenosis
b) Aortic stenosis
c) Aortic regurgitation
d) Truncus arteriosus
e) Congenital absence of the pulmonary valve
Ans….e) Congenital absence of the pulmonary valve produces a
syndrome that manifests with signs of an upper airway obstruction. Marked
aneurysmal dilatation of the main and right or left pulmonary artery
resulting in compression of the bronchi that causes wheezing and recurrent
Dr=Wahid Helmi

pneumonia. Cyanosis may be absent, mild, or moderate. This syndrome


may be associated with TOF
431. The following chromosome is involved in patients with a William
syndrome:
a) Chromosome 5
b) Chromosome 7
c) Chromosome 11
d) Chromosome 13
e) Chromosome 17
Ans….b) Chromosome 7
The preferred therapy for children with a moderate to severe aortic
valvular stenosis is:
a) Balloon valvuloplasty
b) Konno procedure
c) Surgical valve replacement
d) Aortopulmonary anastomosis
e) Rashkind procedure
Ans…..a) Balloon valvuloplasty
A child was diagnosed with a coarctation of aorta. The operative repair
was performed. Several months after surgery, the child developed a
systolic ejection murmur along the left sternal border. The most likely
condition that becomes apparent after a surgical procedure for coarctation
of aorta is:
a) Aortic insufficiency
b) Recurrence of aortic coarctation
c) Mitral stenosis
Dr=Wahid Helmi

d) Mitral insufficiency
e) Subvalvular aortic stenosis
Ans…..e) Subvalvular aortic stenosis may become apparent after a
successful repair of other cardiac defects (e.g., VSD, PDA, or
coarctation of aorta).
A child appears with symptoms of shortness of breath and fatigue with
a mild exertion. The private MD appreciated a systolic ejection murmur at
the sternal border. The patient was referred to a pediatric cardiologist. The
echocardiogram reveals an aortic stenosis and severe tunnel-like subaortic
obstruction. The preferred therapy is:
a) Kenno procedure
b) Balloon valvuloplasty
c) Aortopulmonary anastomosis
d) Put a stent in the obstructed area
e) Create an artificial VSD
Ans…..a) Konno procedure (i.e., obstruction of a left ventricular
outflow tract can be relieved by borrowing space anteriorly from a right
ventricular outflow tract).
Heterotaxy syndromes are associated with all of the following cardiac
conditions except:
a) Dextrocardia
b) Transposition of great arteries
c) Common atrioventricular valve
d) Single atrium
e) Total anomalous pulmonary venous return
Ans…..   d) Single atrium is not present; other cardiac conditions are
Dr=Wahid Helmi

single ventricle, pulmonic stenosis, and pulmonary atresia.


A child is diagnosed to have a congenital heart disease. The oxygen
saturation in superior vena cava is 74%. The oxygen saturation and
pressure in the right atrium are 74% and 0-3 mm Hg respectively. The
oxygen saturation and pressure in the right ventricle are 74% and 40/6 mm
Hg. The oxygen saturation and pressure in the pulmonary artery are 74%
and 5 mm Hg. The oxygen saturation in the pulmonary veins are 98%. The
oxygen saturation and pressure in the left atrium are 98% and 4-8 mm Hg.
The oxygen saturation and pressure in the left ventricle are 80% and
105/10 mm Hg. The oxygen saturation and pressure in the aorta are 80%
and 100/60 mm Hg. The preferred surgical procedure is:
a) Arterial switch (Jatene) operation
b) Norwood operation
c) Starnes procedure
d) Fontan procedure
e) Blalock-Taussig shunt
Ans….e) Blalock-Taussig shunt or total repair is performed in
patients with a tetralogy of Fallot. This patient has a high right ventricular
pressure 40/6 mm Hg (normal 25/3 mm Hg), low pulmonary artery pressure
5 mm Hg (normal 25/10 mm Hg), reduced left ventricular saturation 80%
(normal 94-100%), and reduced aortic saturation 80% (normal 94-100%).
Dr=Wahid Helmi

A child is diagnosed to have a congenital heart disease. The oxygen


saturation in superior vena cava is 74%. The oxygen saturation and
pressure in the right atrium are 74% and 0-3 mm Hg. The oxygen
saturation and pressure in the right ventricle are 74% and 100/8 mm Hg.
The oxygen saturation and pressure in the pulmonary artery are 98% and
40/2 mm Hg. The oxygen saturation in the pulmonary veins are 98%. The
oxygen saturation and pressure in the left atrium are 98% and 4-8 mm Hg.
The oxygen saturation and pressure in the left ventricle are 98% and 40/2
mm Hg. The oxygen saturation and pressure in the aorta are 74% and
100/60 mm Hg. The preferred surgical procedure is:
a) Blalock-Taussig shunt
b) Arterial switch (Jatene) operation.
c) Glenn shunt
d) Fontan procedure
e) Norwood operation
Ans... b) Arterial switch (Jatene) operation is the preferred
procedure for transposition of great vessels. However, Rashkind balloon
atrial septostomy is the initial procedure of choice. This patient has a high
right ventricular pressure 100/8 mm Hg (normal 25/3 mm/Hg), high
pulmonary artery pressure 40/2 mm Hg (normal 25/10 mm Hg), decreased
left ventricular pressure 40/2 mm Hg (normal 100/8 mm Hg).
Dr=Wahid Helmi

A child is diagnosed to have a congenital heart disease. The oxygen


saturation in superior vena cava is 74%. The oxygen saturation and
pressure in the right atrium are 74% and 0-3 mm Hg. The oxygen
saturation and pressure in the right ventricle are 80% and 35/5 mm Hg. The
oxygen saturation and pressure in the pulmonary artery are 80% and 25/10
mm Hg. The oxygen saturation in the pulmonary veins are 98%. The
oxygen saturation and pressure in the left atrium are 98% and 4-8 mm Hg.
The oxygen saturation and pressure in the left ventricle are 98% and 100/8
mm Hg. The oxygen saturation and pressure in the aorta are 98% and
100/60 mm Hg. The preferred surgical procedure is:
a) Repair of VSD
b) Repair of ASD
c) Surgical closure of PDA
d) Surgical repair of truncus arteriosus
e) Total repair of anomalous pulmonary venous return
Ans... a) Repair of VSD is indicated. This patient has an increased
right ventricular pressure 35/5mm Hg (normal 25/3mm Hg),
increased right ventricular saturation 80% (normal 74%), and increased
pulmonary artery saturation 80% (normal 74%).
Dr=Wahid Helmi

A 15-year-old boy came for a routine physical examination. He is a


good athlete. A physical examination reveals heart rate 40 beats per
minute. The rest o the physical examination is unremarkable. The next step
in management is:
a) EKG
b) Stress test
c) Cardiology consult
d) Pace maker
e) Reassurance
Ans... e) Reassurance; the heart rate can normally drop up to 40
beats per minute in athletic adolescents.

All of the following statements indicate pathologic conditions except:


a) A newborn’s heart rate is more than 200 beats per minute persistently.
b) An infant’s heart rate is more than 150 beats per minute persistently.
c) An older child’s heart rate is more than 120 beats per minute
persistently.
d) A full term infant’s heart rate is 68 beats per minute during sleep.
e) A 16-year-old athlete’s heart rate is 40 beats per minute.
Ans... e) A 16-years-old athlete’s heart rate up to 40 beats per
minute is normal
Dr=Wahid Helmi

A newborn appears with a pink right arm and blue lower extremities.
The most likely diagnosis is:
a) Tetralogy of Fallot
b) Transposition of great arteries
c) Truncus arteriosus
d) Coarctation of aorta
e) Total anomalous pulmonary venous return
Ans….d) Differential cyanosis is noted in coarctation of aorta,
interrupted aortic arch, and persistent pulmonary hypertension (PPHN).

A newborn appears with a blue upper arm and less blue lower
extremities. The most likely diagnosis is:
a) Transposition of great arteries
b) Tetralogy of Fallot
c) Truncus arteriosus
d) Tricuspid atresia
e) Total anomalous pulmonary venous return
Ans….a) Transposition of great arteries
Dr=Wahid Helmi

A newborn appears with a circumoral cyanosis. The mucous


membranes (i.e., lips and tongue) are pink. The most likely diagnosis is:
a) Cold lips
b) VSD
c) ASD
d) PDA
e) Prominent venous plexus in lips
Ans….e) Prominent venous plexus in lips

A new born appears with a blueness around the forehead. The rest of
the physical examination is unremarkable. The most likely diagnosis is:
a) Cold forehead
b) Meningocele
c) Hydrocephalus
d) Encephalocele
e) Prominent venous plexus on the forehead
Ans….e) Prominent venous plexus on the forehead
Dr=Wahid Helmi

A newborn appears with a cyanosis of both hands and feet. The rest
of the physical examination is unremarkable. The most likely diagnosis is:
a) Transposition of great vessels
b) Gram-negative septic shock
c) Patent foramen ovale
d) Unwrapped and cold extremities
e) Persistent pulmonary hypertension
Ans….d) Unwrapped and cold extremities (i.e., acrocyanosis)

The common organisms in patients with a native valve and infective


endocarditis are all of the following except:
a) Staphylococcus epidermidis
b) Staphylococcus aureus
c) Streptococcus mutans
d) Streptococcus sanguis
e) Streptococcus bovis
Ans... a) S. epidermidis is common in the presence of an indwelling
central venous catheter or prosthetic valve. For a native valve, the common
organisms are viridans group streptococci (e.g., S. mutans, S. sanguis, S.
mitis), S. aureus, group D Streptococcus (enterococcus) (S. bovis, S.
faecalis).
Dr=Wahid Helmi

The most common organism in patients with an infective endocarditis


without an underlying heart disease is:
a) S. aureus
b) Pseudomonas aeruginosa
c) Serratia marcescens
d) Streptococcus mitis
e) Streptococcus faecalis
Ans... a) S. aureus

The most common organism in patients with an infective endocarditis


after a dental procedure is:
a) Streptococcus mutans
b) S. aureus
c) Streptococcus bovis
d) Streptococcus pneumoniae
e) Streptococcus faecalis
Ans... a) Viridans group streptococci (e.g., S. mutans, S. sanguis,
S. mitis)
The most common organism in patients with an infective endocarditis
after a lower bowel surgery is:
a) Streptococcus bovis
b) Streptococcus mutans
c) Streptococcus sunguis
d) E. coli
e) Serratia marcescens
Ans….a) Group D streptococcus (enterococcus)(e.g., S. bovis, S.
Dr=Wahid Helmi

faecalis)
The most common organism in patients with an infective endocarditis
after a genitourinary intervention is:
a) Streptococcus pneumoniae
b) E. coli
c) Pseudomonas aeruginosa
d) Campylobacter fetus
e) Streptococcus faecalis
Ans….e) Group D streptococcus (enterococcus) (e.g., S. bovis, S.
faecalis)
The most common organism in patients with an infective endocarditis
and intravenous drug abusers is:
a) Streptococcus aureus
b) Pseudomonas aeruginosa
c) Streptococcus epidermidis
d) Streptococcus mitis
e) Candida albicans
Ans….b) Pseudomonas aeruginosa or Serratia marcescens
The most common organism in patients with an infective endocarditis
and indwelling central venous catheter is:
a) S. aureus
b) Serratia marcescens
c) S. epidermidis
d) Candida albicans
e) Haemophilus influenzae
Ans….c) Staphylococcus epidermidis
Dr=Wahid Helmi

The following condition is an important risk factor for developing an


infective endocarditis in children with a cyanotic heart disease:
a) Diarrhea
b) RSV infection
c) MMR vaccination
d) Poor dental hygiene
e) Arrythmias
Ans…..d) Poor dental hygiene
All of the following postoperative cardiac conditions are risk factors for
developing infective endocarditis except:
a) Replaced aortic valve
b) Norwood aortopulmonary anastomosis
c) Blalock-Taussig shunt
d) Repair a mitral valve
e) Repair of a simple atrial septal defect
Ans... e) Repair of a simple ASD or closure of PDA almost
eliminates the risk of developing endocarditis. A surgical correction of a
congenital heart disease can reduce but does not eliminate the risk of
endocarditis except the two conditions mentioned above
All of the following preoperative cardiac conditions are risk factors for
developing infective endocarditis except:
a) VSD
b) Tetralogy of Fallot
c) Aortic stenosis
d) Tricuspid regurgitation in newborns
e) Transposition of great arteries
Dr=Wahid Helmi

Ans….d) Tricuspid regurgitation (TR) is not a preoperative


condition and does not produce infective endocarditis. TR is common in
newborns with a perinatal asphyxia secondary to transient papillary
muscles dysfunction. TR is a self-limiting condition. However, mitral valve
prolapse, bicuspid aortic valves, PDA, ASD, and other anatomical cardiac
defects, especially involving the left side of the heart can develop infective
endocarditis.

The foramen ovale is functionally closed in a normal healthy newborn


by:
a) 12 hours of life
b) 24 hours of life
c) 7 days of life
d) 1 month of life
e) 3 months of life
Ans….e) 3 months of life
The ductus arteriosus is functionally closed in a normal healthy
newborn by
a) 0-5 hours of life
b) 5-10 hours of life
c) 10-15 hours of life
d) 15-20 hours of life
e) 20-25 hours of life
Ans….c) 10-15 hours of life
554. The ductus arteriosus closes when the PO2 of the blood at the ductus
is about:
Dr=Wahid Helmi

a) 45 mm Hg
b) 50 mm Hg
c) 55 mm Hg
d) 60 mm Hg
e) 65 mm Hg
Ans….b) 50 mm Hg

The most common cardiac anomaly in patients with an autosomal


dominant polycystic kidney disease is:
a) ASD
b) VSD
c) Mitral valve prolapse
d) Peripheral pulmonic stenosis
e) Tricuspid atresia
Ans….c) Mitral valve prolapse
Facio-auriculo-vertebral spectum (FAVS) is associated with the
following cardiac anomaly:
a) ASD
b) Tetralogy of Fallot
c) Tricuspid stenosis
d) Truncus arteriosus
e) Transposition of great arteries
Ans….b) TOF or VSD
Cat-eye syndrome has the following chromosomal abnormality:
a) Trisomy 9
Dr=Wahid Helmi

b) Trisomy 21
c) Trisomy 21P
d) Trisomy 13
e) Trisomy 5P
Ans….c) Trisomy 21p is cat-eye syndrome. Deletion 5p is cat-cry
syndrome (or cri du chat syndrome).

The most common cause of death in pediatric patients with a heart


transplantation is:
a) CMV infection
b) Candida albicans infection
c) Protozoal infection
d) Toxoplasma infection
e) S. aureus infection
Ans…. a) CMV infections
All of the following suggestions are useful as a prevention of high
blood pressure in pediatric population except:
a) Control obesity
b) Reduce dietary sodium intake
c) Physical exercise
d) Reduce serum cholesterol levels
e) Consume a small amount of red-wine
Ans... e) Alcohol and tobacco use cause hypertension
A 17-year-old boy for a routine physical examination. He is
asymptomatic but experienced occasional headaches. His recorded blood
Dr=Wahid Helmi

pressure is above the 95th percentile for his age. The most likely diagnosis
is:
a) Aortic coarctation
b) Renal artery stenosis
c) Pheochromocytoma
d) Hyperthyroidism
e) Suspected hypertension
Ans... e) Suspected hypertension (BP is above 95th percentile)
A 16-year-old boy appears for a routine physical examination. He is
asymptomatic. His recorded blood pressures are between 90th and 95th
percentiles. He is obese. In addition to weight loss, the next step in
management is:
a) A follow-up examination after 1 month
b) A follow-up examination after 3 months
c) A follow-up examination after 6 months
d) Furosemide
e) Aldomet
Ans... c) A follow up examination after 6 months is recommended
(BP is between 90th and 95th percentiles)
A 13-year-old boy appears for a routine physical examination. He is
asymptomatic. His recorded blood pressure is below the 90th percentile.
The next step in management is:
a) Echocardiography
b) Renal ultrasonography
c) Follow-up after 1 year
d) Low-salt diet
Dr=Wahid Helmi

e) Blood pressure should be measured on three separate occasions.


Ans…,e) BP should be measured on three separate occasions
prior to further interventions (BP is less than 90th percentile).
The lowest limit of a normal systolic blood pressure in a full-term
newborn is:
a) 45 mm Hg
b) 50 mm Hg
c) 60 mm Hg
d) 70 mm Hg
e) 80 mm Hg
Ans….c) Less than 60 mm Hg in a full-term newborn is considered
low BP
The lowest limit of a normal systolic BP from 1 month to 1 year of age
is:
a) 65 mm Hg
b) 70 mm Hg
c) 75 mm Hg
d) 80 mm Hg
e) 85 mm Hg
Ans….b) Less than 70 mm Hg from 1 month to 1 year of age is
considered low BP.
The lowest limit of a normal systolic BP in a 5-year-old child is:
a) 75 mm Hg
b) 80 mm Hg
c) 85 mm Hg
d) 90 mm Hg
Dr=Wahid Helmi

e) 95 mm Hg
Ans….b) Less than 80 mm Hg in a 5 year old child is considered
low BP; the formula:Less than 70 mm Hg + 2 x age from 1-10 years.
The lowest limit of a normal systolic BP in a child older than 10 years
of age is:
a) 80 mm Hg
b) 85 mm Hg
c) 90 mm Hg
d) 95 mm Hg
e) 100 mm Hg
Ans….c) Less than 90 mm Hg in children older than 10 years of
age is considered low BP.
A routine physical examination in a 12-year-old boy reveals heart rate
85 per minute, systolic BP 135 mm Hg, diastolic BP 85 mm Hg, respiratory
rate 18 per minute, and the rest of the examination is unremarkable.
Occasionally, he complains of headaches. The next step in management
is:
a) Exercise
b) Weight loss
c) Low-salt diet
d) Furosemide
e) Reassurance
Ans….e) Reassurance because all findings are the upper limit of
normal for a 12-year-old child. However, exercise is good for all
healthy children and weight loss is indicated in obese children.
A routine physical examination of the child reveals pulsation of the
Dr=Wahid Helmi

aorta. He is thin and asymptomatic. The next step in management is:


a) Ultrasonography of the aorta
b) Measure four extremities BP
c) Aortic angiography
d) Abdominal x-ray
e) Reassurance
Ans….e) Reassurance; this is a normal finding in this young
children.
A 2-day-old newborn male appears with cyanosis and tachypnea for
the last 2 hours. He was born by NSVD with Apgar scores are 8 and 9 at 1
and 5 minutes respectively. The physical examination reveals a grade 2/6
systolic murmur at the left sternal border. The arterial PO2 is 80 in
hyperoxia test. The oxygen saturation values are as follows: RA 58%, RV
58%, LA 100%, LV 94%, pulmonary artery 58%, and ascending aorta 80%.
The volume of blood passes through different structures are as follows: RA
3 L/min/m2, RV 3 L/min/m2, LA 2 L/min/m2, LV 2 L/min/m2, pulmonary
artery 2 L/min/m2, and ascending aorta 3 L/min/m2. The newborn is
receiving PGE1 infusions. The preferred surgical therapy is:
a) Switch operation (Jatene)
b) Modified Blalock-Taussig shunt
c) Starnes procedure
d) Norwood operation
e) Fontan procedure
Ans... b) Modified Blalock-Taussig shunt (i.e., Gore-Tex conduit
anastomosed side to side from the subclavian artery to the
homolateral branch of the pulmonary artery) is the preferred surgical
Dr=Wahid Helmi

procedure in patients with a tetralogy of Fallot


A newborn appears with cyanosis and respiratory distress within the
1st hour of life. She was born by NSVD with Apgar score of 7 and 8 at 1
and 5 minutes respectively. The physical examination reveals a soft
systolic ejection murmur at the midleft sternal border and single, loud 2nd
heart sound. The arterial PO2 is 60 in hyperoxia test. The oxygen
saturation values are as follows: both vena cava 58%, RA 73%, RV 73%,
LA 90%, LV 90%, pulmonary artery 90%, right and left pulmonary arteries
80%, pulmonary veins 100%, and ascending aorta 73%. The newborn is
placed on a mechanical ventilator and PGE1 infusions have started. The
preferred initial surgical therapy is:
a) Norwood operation
b) Glenn operation
c) Fontan procedure
d) Aortic valvuloplasty
e) Rashkind procedure
Ans….e) Rashkind balloon atrial septostomy is the initial procedure
of choice in patients with transposition of great arteries. Arterial
switch (Jatene) operation is the final procedure of choice
The pulmonary arteriovenous fistulas are most commonly present is:
a) Williams syndrome
b) Down syndrome
c) Turner syndrome
d) Prader-Willi syndrome
e) Osler-Weber-Rendu syndrome
Ans... e) Osler-Weber-Rendu syndrome (hereditary hemorrhagic
Dr=Wahid Helmi

telangiectasia type 1) is associated with angiomas of the GI tract, liver,


nasal and buccal mucous membranes
A newborn appears with bradyarrythmias. ECG reveals that the P-P
intervals are constant, the P-R intervals increase progressively until a P
wave is not conducted; following a dropped beat, the P-R interval is
shorter. The most likely diagnosis is:
a) Long Q-T syndrome
b) 1st-degree heart block
c) Mobitz type I
d) Mobitz type II
e) 3rd-degree heart block
Ans….c) Mobitz type I (Wenckebach type) is one variant of
2nd-degree heart block.
A child appears with history of syncope. He is otherwise healthy. ECG
reveals that occasional atrial beats are not conducted to the ventricles. The
most likely diagnosis is:
a) 1st-degree heart block
b) Wenckebach type heart block
c) Mobitz type II heart block
d) 3rd-degree heart block
e) Normal variant
Ans... c) Mobitz type II
A newborn female appears with edema of the dorsa of the hands and
feet, loose skinfolds at the nape of the neck, low birthweight, and
decreased length. The most common cardiac anomaly in this patient is:
a) Aortic coarctation
Dr=Wahid Helmi

b) Pulmonic stenosis
c) Aortic stenosis
d) Mitral valve prolapse
e) Nonstenotic bicuspid aortic valve
Ans….e) Nonstenotic bicuspid aortic valve (33-50% of cases) is
the most common cardiac defect in patients with Turner syndrome

Match the congenital heart diseases and surgical procedures


998. Transposition of great arteries
999. Hypoplastic left heart syndrome
1000. Tetralogy of Fallot
1001. Ebstein anomaly
1002. Aortic stenosis
a) Blalock-Taussig shunt
b) Norwood procedure
c) Balloon valvuloplasty
d) Starnes procedure
e) Arterial switch (Jatene)
Dr=Wahid Helmi

Ans.... e) Arterial switch (Jatene); Rashkind balloon atrial


septostomy is the initial procedure of choice.
999. b) Norwood procedure
1000. a) Blalock-Taussig shunt
1001. d) Starnes procedure
1002. c) Balloon valvuloplasty
Dr=Wahid Helmi

Match the following syndromes with different clinical findings (1003-1007):


1003. Aortic stenosis
1004. Pulmonic stenosis
1005. Mitral valve prolapse
1006. Total anomalous pulmonary venous return 1007. Tricuspid atresia
a) Well-developed, well-nourished child
b) Sudden death
c) “Snowman sign” (figure eight)
d) Scoliosis
e) ECG reveals characteristic left axis deviation
Ans.. 1003. b) Sudden death
1004. a) Well-developed, well-nourished child
1005. d) Scoliosis
1006. c) “Snowman sign” (figure eight)
1007. e) ECG reveals a characteristic left axis deviation that is also noted
in endocardial cushion defects.
Dr=Wahid Helmi

A child is diagnosed to have restrictive


pericardial effusion. The important clinical signs are the following:
a) Tachycardia, hypotension, and decreased oxygen saturation
b) Bradycardia, hypotension, and decreased perfusion
c) Tachycardia, hypertension, and swelling of face
d) Arrythmia, normal BP, and venous engorgement of neck
e) Fever, tachycardia, and hypertension
Ans---A
Tachycardia, hypotension, and decreased oxygen saturation

day-old male newborn appears with heart murmur. Oxygen saturation values are the
following: right atrium (72%), right ventricle (83%), left atrium (100%), left ventricle
(100%),pulmonary artery (87%), and aorta (100%). Blood flow volume
(L/minute/m2) through the heart and blood vessels is the following: right
atrium (3), right ventricle (5), left atrium (6), left ventricle (6), pulmonary
artery (5), and aorta (4). The most likely diagnosis is:
a) Tetralogy of Fallot
b) VSD
c) Pulmonary stenosis
d) Aortic stenosis
e) Transposition of great arteries
Ans….B VSD
Dr=Wahid Helmi

child had surgery for right ventricular outflow tract obstruction. The expected
postoperative complication is:
a) Pulmonic stenosis
b) Tricuspid regurgitation
c) Pulmonary valvular insufficiency
d) Tricuspid stenosis
e) Rupture interventricular septum
Ans C Pulmonary valvular insufficiency also occurs due to severe pulmonary
hypertension.

child has congenital absence of the pulmonary valve. The most common associated
anomaly is:
a) ASD
b) VSD
c) Aortic stenosis
d) Pulmonic stenosis
e) Congenital absence of tricuspid valve
AnsB VSD
Dr=Wahid Helmi

Match the different cardiac conditions with different types of murmurs (242-246):
242. Coarctation of the aorta
243. Mitral valve prolapse
244. Pulmonary valvular insufficiency
245. Congenital mitral insufficiency
246. Congenital mitral stenosis
a) A low-pitched decrescendo diastolic murmur at the upper and midleft
sternal border
b) A short systolic murmur is noted along the left sternal border at the 3rd
and 4th intercostal spaces; murmur is transmitted
to the left infrascapular area.
c) Apical murmur is late systolic and associated with a click.
d) A high-pitched (“cooing dove”), apical holosystolic murmur
e) A rumbling apical diastolic murmur is followed by loud 1st sound; 2nd
sound is loud and split.
Ans….242/ b - A short systolic murmur is noted along the left sternal border at the
3rd and 4th intercostal spaces; murmur is transmitted to the left infrascapular area.
243/ c - Apical murmur is late systolic and may be associated with a click.
244/ a - A low-pitched decrescendo diastolic murmur is noted at the upper and
midleft sternal border
245/ d - A high-pitched (“cooing dove”), apical holosystolic murmur; moderate to
severe insufficiency causes low-pitched, mid- diastolic rumbling murmur.
246/ e - A rumbling apical diastolic murmur is followed by loud 1st sound; 2nd
sound is loud and split.
Dr=Wahid Helmi

Match all the clinical findings with the different clinical conditions (451-453):
451. Sinus bradycardia in neonates
452. Sinus bradycardia after neonatal period
453. Supraventricular tachycardia in neonates
a) Heart rate is 231 beats/minute
b) Heart rate is less than 90 beats/minute.
c) Heart rate is less than 60 beats/minute.
Ans...
451/ b - In neonates with sinus bradycardia, heart rate is less than 90 beats/minute.
452/c - After neonatal period, heart rate less than 60 beats/minute represents sinus
bradycardia
453/ a - In neonates with SVT, heart rate is usually more than 230 beats/minute
with abnormal P-wave axis. In other age groups with SVT, heart rate is
above 180 and up to 300 beats/minute.

routine physical examination in a full-term newborn reveals heart rate 90 beats/minute


during sleeping. The newborn is asymptomatic. The next step in management is:
a) Cardiology consultation
b) EKG
c) Sepsis work up
d) Chest x-ray
e) Reassurance
AnsE Reassurance
Dr=Wahid Helmi

routine physical examination of anewborn infant reveals heart rate 180 beats/minute
during crying. The newborn is asymptomatic. The next step in management is:
a) EKG
b) Cardiology consultation
c) Chest x-ray
d) Sepsis work up
e) Reassurance
AnsE Reassurance

The preferred therapy for patients with re-stenosis of coarctation of the aorta after earlier
surgery is:
a) Aortic stent placement
b) Repeat surgical procedure
c) Antihypertensive agent
d) Balloon angioplasty.
e) Small dose of acetyl salicylic acid daily
Ans….D Balloon angioplasty

All of the following clinical manifestations are present in newborns with critical pulmonic
stenosis except:
a) Pulmonary edema . b) Peripheral edema
c) Hepatomegaly d) Cyanosis
e) Dyspnea
Ans ---A--- Pulmonary circulation is decreased significantly in patients with critical PS.
They appear with right-sided heart failure.
Dr=Wahid Helmi

All of the following clinical manifestations are present in newborns with critical aortic
stenosis except:
a) Pulmonary edema
b) Poor perfusion
c) Hepatomegaly
d) Peripheral edema
e) Hypertension
Ans….E
Patients with critical AS manifest with hypotension and subsequently, total
circulatory collapse. They appear with both left- sided and right-sided
cardiac failure.

The most common cardiac defect in patients with autosomal dominant Holt-Oram
syndrome is:
a) PDA
b) ASD
c) VSD
d) Aortic stenosis
e) Pulmonic stenosis
Ans ---B ---Secundum ASD (atrial septal defect)
Dr=Wahid Helmi

The most common cardiac defect in patients with Alagille syndrome is:
a) VSD
b) ASD
c) Mitral regurgitation
d) Tricuspid regurgitation
e) Pulmonary stenosis.
Ans…..E Pulmonary stenosis (e.g., pulmonary valve or branch pulmonary artery) is
common in patients with arteriohepatic dysplasia (Alagille syndrome).

PHACE syndrome includes all of the following features except:


a) Posterior brain fossa anomalies
b) Hemangiomas (facial)
c) Arterial anomalies
d) Cardiac anomalies and aortic dilatation.
e) Eye anomalies
Ans….D Cardiac anomalies and aortic coarctation
Dr=Wahid Helmi

CATCH 22 syndrome can be associated with the following cardiac defect:


a) Tetralogy of Fallot.
b) Transposition of great vessels
c) Total anomalous pulmonary venous return
d) Tricuspid atresia
e) Hypoplastic left heart syndrome
Ans—A ---CATCH 22 syndrome can be associated with tetralogy of Fallot, pulmonary
atresia with VSD (a severe form of TOF), or interrupted aortic arch.

CATCH 22 syndrome includes all of the following findings except:


a) Cardiac defects
b) Abnormal facies
c) Thymic hypoplasia
d) Cleft lip
e) Hypocalcemia
Ans….D Cleft palate
Dr=Wahid Helmi

Match all different postoperative complications with clinical manifestations (779-783):


779. Phrenic nerve injury
780. Recurrent laryngeal nerve injury
781. Sympathetic nerve chain injury
782. Hyponatremia
783. Spinal artery ischemia
a) Horner syndrome
b) Seizures
c) Diaphragmatic paralysis
d) Vocal cord paralysis
e) Paraplegia
Ans….
779/ c Phrenic nerve injury causes diaphragmatic paralysis.
780/ d Recurrent laryngeal nerve injury causes vocal cord paralysis.
781/ a Sympathetic nerve chain injury causes Horner syndrome.
782/ b Hyponatremia causes seizures that are also produced by hypoglycemia,
CNS ischemia, and emboli.
783/ e Spinal artery ischemia after repair of coarctation causes paraplegia.
Dr=Wahid Helmi

Match all of the following complications


after cardiac surgery with different manifestations (784-788):
784. Postcoarctectomy syndrome
785. Repair of TAPVR
786. ARDS postpump syndrome
787. SIADH
788. Serous postpericardiotomy syndrome
a) Hyponatremia
b) Hypertension
c) Pericardial tamponade
d) Pulmonary hypertension
e) Possible release of vasoactive substances
Ans
784/ b Postcoarctectomy syndrome causes hypertension and mesenteric arteritis.
785/ d Repair of TAPVR (total anomalous pulmonary venous return) can cause
pulmonary hypertension that can occur trisomy 21 and Norwood 1st stage
operation.
786/ e ARDS postpump syndrome can be due to possible release of vasoactive
substances but the exact mechanism is unknown.
787/ a SIADH produces hyponatremia.
788/ c Serous postpericardiotomy syndrome produces pericardial tamponade
Dr=Wahid Helmi

All of the following statements are true in patients with Wolff-Parkinson-White syndrome
except:
a) WPW syndrome may be associated with Ebstein anomaly.
b) WPW syndrome is absent in patients with a normal heart.
c) WPW syndrome is usually seen when the patient does not have
tachycardia.
d) EKG reveals a short P-R interval and slow upstroke of QRS (delta
wave).
e) The anatomic re-entrant circuit are the AV node and an accessory
preexcitation pathway consisting of a muscular bridge
connecting atrium to the ventricle.
Ans- B WPW syndrome most often present in patients with a normal heart.

Ebstein anomaly is associated with the following:


a) Tricuspid stenosis
b) Tricuspid regurgitation
c) Mitral stenosis
d) Pulmonic stenosis
e) Aortic regurgitation
Ans- (b) Tricuspid regurgitation
Dr=Wahid Helmi

Mitral valve prolapse is commonly associated with all of the following


conditions except:
a) Marfan syndrome
b) Scoliosis
c) Pectus excavatum
d) Straight back syndrome
e) Noonan syndrome
Ans- (e) Noonan syndrome

The immediate postoperative complication after surgical repair of coarctation of aorta is:
a) Rebound hypertension
b) Hypotension
c) Aortic aneurysm
d) Upper extremities hypotension
e) Anastomotic leak
Ans (a) Rebound hypertension

The treatment for postoperative hypertension after surgical repair of


coarctation of aorta is:
a) Antihypertensive medication
b) Reoperation because of operative failure
c) Renal failure
d) Cardiac failure
e) Fluid retention because of increased ADH secretion
Ans…. (a) Antihypertensive medication
Dr=Wahid Helmi

The development of recoarctation after surgical repair of coarctation of


the aorta is best treated with:
a) Repeat surgical repair
b) Reassurance
c) Aspirin
d) Vasodilator medication
e) Balloon angioplasty
Ans.... (e) Balloon angioplasty. Intravascular stents are commonly used.
Dr=Wahid Helmi

A 5-month-old girl appeared with history of respiratory infections


followed by congestive cardiac failure. She was completely asymptomatic
prior to this episode. She appeared with dyspnea, cough, and failure to
thrive. Physical examination revealed edema, hepatomegaly, and
pulmonary congestion. Chest x-ray revealed cardiomegaly and clear lung
fields. EKG reveals left atrial and left ventricular hypertrophy.
Echocardiogram revealed a bright endocardial surface and a dilated, poorly
contracting left ventricle. The most likely diagnosis is:
a) Hypoplastic left ventricle
b) Aortic stenosis
c) Tetralogy of Fallot
d) Cardiomyopathy
e) Endocardial fibroelastosis
Ans (e) Endocardial fibroelastosis

The preferred therapy for patients with end-stage endocardial


fibroelastosis is:
a) Digoxin b) Lasix
c) Pacemaker d) Removal of endocardial tissue
e) Heart transplantation
Ans (e) Heart transplantation. Initial therapy includes prevention of respiratory infections
and treatment of congestive heart failure
Dr=Wahid Helmi

A mother is pregnant with her second child. Her first child suffered from a congenital
heart disease. The incidence of heart disease in her second child is:
a) 0.8%
b) 1-2%
c) 2-6%.
d) 10-20%
e) 20-30%
Ans
(c) 2–6%; the same incidence if one parent was affected.

321. A mother is pregnant with her third child. Her first and second children
suffered from congenital heart disease. The incidence of heart disease in
her third child is:
a) 2-6%
b) 10-20%
c) 20-30%.
d) 30-50%
e) 50-100%
Ans (c) 20–30%; the same incidence if two first–degree
Dr=Wahid Helmi

relatives were affected.


The lateral thoracotomy was performed on a child with tetralogy of
Fallot. All of the following complications can occur shortly after surgery
except:
a) Chylothorax
b) Horner syndrome
c) Diaphragmatic paralysis
d) Cardiac failure
e) Hyperactive radial pulse.
Ans (e) Radial pulse may be diminished on the side of
Blalock–Taussig shunt.
An infant is diagnosed with tetralogy of Fallot and admitted for the
modified Blalock-Taussig shunt operation. Two days prior to surgery, in the
early morning the infant developed cyanosis, tachypnea, and restlessness.
The resident who was on call made the diagnosis hypoxic, ‘blue’, or ‘tet’
spells. The infant was placed on the abdomen in knee-chest position,
received oxygen, and subcutaneous injection of morphine (0.2 mg/kg). The
infant remained cyanotic and developed metabolic acidosis. The infant
received a rapid intravenous infusion of sodium bicarbonate but did not
improve. The next step in management is:
a) Sepsis work up
b) Repeat ABG
c) Repeat EKG
d) Repeat echocardiogram
e) Intravenous methoxamine.
Dr=Wahid Helmi

Ans… (e) Intravenous methoxamine or phenylephrine


improves right ventricular flow, decreases right–to–left shunt, and
improves the cyanosis and symptoms. Intravenous propranolol
(beta–adrenergie blocker) is also used.

An infant is diagnosed with a severe form of tetralogy of Fallot. She is


receiving prostaglandin E1 infusion. The preferred therapy for the infant is:
a) Prostaglandin E2 infusion
b) Continue PGE1 infusion
c) Oral propranolol
d) Iron therapy
e) Open heat surgery and total correction.
Ans… (e) Open heart surgery and total corrections are
indicated. The modified Blalock–Taussig shunt (i.e, a Gore–Tex conduit
anastomosed side to side from the subclavian artery to the homolateral
branch of pulmonary artery) can be performed to improve pulmonary
circulation.
Dr=Wahid Helmi

An infant is diagnosed with a less severe form of tetralogy of Fallot.


He is growing normally and has no cyanotic spells. The time to perform
surgical repair is:
a) Between 1 and 3 months
b) Between 4 and 12 months.
c) Between 12 and 15 months
d) Between 15 and 18 months
e) Between 18 and 24 months
Ans. (b) Between 4 and 12 months of age
An infant is diagnosed with a less severe form of tetralogy of Fallot.
She is waiting for a surgical repair but needs careful observation. All of the
following treatments are useful except:
a) Hydration
b) Iron
c) Propranolol
d) Oxygen prn
e) Sodium bicarbonate.
Ans (e) Sodium bicarbonate therapy is not indicated.
Hydration is useful to prevent hemoconcentration. Iron therapy prevents the
paroxysmal dyspneic attacks in infancy and early childhood. Propranolol
decreases the hypercyanotic spells. Oxygen is given when indicated
1
1.Questions and Answers after the questions.

1. Which of the following is not associated with esophageal webs?

A. Plummer-Vinson syndrome
B. Epidermolysis bullosa
C. Lupus
D. Psoriasis
E. Stevens-Johnson syndrome

2. An 11 year old boy complains that occasionally a bite of hotdog “gives mild
pressing pain in his chest” and that “it takes a while before he can take another
bite.” If it happens again, he discards the hotdog but sometimes he can finish it.
The most helpful diagnostic information would come from

A. Family history of Schatzki rings


B. Eosinophil counts
C. UGI
D. Time-phased MRI
E. Technetium 99 salivagram

3. 12 year old boy previously healthy with one-month history of difficulty swallowing
both solid and liquids. He sometimes complains food is getting stuck in his
retrosternal area after swallowing. His weight decreased approximately 5% from last
year. He denies vomiting, choking, gagging, drooling, pain during swallowing or
retrosternal pain. His physical examination is normal.

What would be the appropriate next investigation to perform in this


patient?

A. Upper Endoscopy
B. Upper GI contrast study
C. Esophageal manometry
D. Modified Barium Swallow (MBS)
E. Direct laryngoscopy

4. A 12 year old male presents to the ER after a recent episode of emesis. The parents
are concerned because undigested food 3 days old was in his vomit. He admits to a
sensation of food and liquids “sticking” in his chest for the past 4 months, as he
points to the upper middle chest. Parents relate a 10 lb (4.5 Kg) weight loss over the
past 3 months. Past medical history and family history are unre- markable. Vital
signs are stable, and physical exam is unremarkable. The ER physician obtains a chest
2
X-ray AP and lateral that shows dilatation of the esophagus with an air fluid level.
What is the best diagnostic test for this patient’s condition?

A. Endoscopy
B. 24 hour PH monitoring
C. Barium swallow
D. Esophageal manometry
3

5. A 26 month old female is referred to the GI clinic with a history of spitting up since 10
month of age. Mom noticed that symptoms began after the introduction of table food.
The pediatrician diagnosed GERD and started the patient on an H2 blocker.
Medication was changed to a proton pump inhibi- tor without improvement and the
patient continued to spit up and have difficulty swallowing with solids but not with
liquids. She was in the 50%ile for height and the 25%ile for weight. Her diet was
mainly liquid. The pediatrician was concerned because in the last month she did not
gain weight. Af- ter your history and physical exam you ordered a barium swallow
which showed a posterior impres- sion of the upper-middle esophagus. Which of the
followings is the next step in management?

A. Esophagoscopy
B. 24 hours PH monitoring
C. Barium swallow
D. Esophageal manometry
D. Chest MRI

6. Eosinophilic Esophagitis is associated with the following diseases except:

A. Atopic Dermatitis
B. Asthma
C. Helicobacter Pylori
D. Allergic Rhinitis

7. You are seeing an 8 year old male in clinic as a follow-up from a recent EGD you
performed for the sensation of “things getting stuck” while swallowing. A distal
esophageal biopsy showed 10 eosino- phils/HPF. The EGD was otherwise
endoscopically and histologically normal, which included a total of 6 esophageal
biopsies. What is the most appropriate next step:

A. Start oral fluticasone.


B. Start proton pump inhibitor therapy
C. Start elemental diet
D. Refer to an allergist

8. You have diagnosed a 1 year old child with eosinophilic esophagitis. All of the
following are treat- ment options except:

A. Oral fluticasone.
B. Directed food elimination diet based on food allergy testing (skin prick and patch testing)
C. 6-food elimination diet (eliminating milk, soy, egg, wheat, peanut, and fish/shellfish)
D. Elemental diet
E. Lactose-free diet

9. A 10 year old African-American female presents with complaints of several months


of intermittent symptoms including trouble keeping eyelids open, inability to brush her
hair, and trouble getting out of chairs at school. Her speech is sometimes slurred. She
4

complains of double vision occasionally. Her symptoms are usually worse in the
evening after school. On exam, she has bilateral ptosis. When asked to raise both
extended arms over her head, she can raise them only 3-4 times before tiring. On
laboratory evaluation, she is acetylcholinesterase receptor antibody positive. Which of
the following structures is most likely to be affected?

A. Duodenal villi chloride channels


B. Colonic motility
C. Bile canaliculi
D. Upper esophageal sphincter
E. Pancreatic duct
5

10. The ER calls you at 7 PM to see a 2 year old who swallowed an unknown quantity of
vanilla scented hair relaxer. You ask about the presence of facial or oral lesions and
you are told none are evident but the patient is not fully cooperative for a complete
exam. You know the endoscopy suite is only on emergency status so you

A. Request the ER attending to call the ENT service.


B. Request the ER attending to notify the endoscopy suite for an emergent
study.
C. Proceed to the ER for your own assessment and finding no lesions or
respiratory distress, you recommend sending the patient home on bismuth
subsalicylate to return for F/U in one week.
D. Proceed to the ER for your own assessment and finding no lesions you or
respiratory distress, you recommend symptomatic treatment and endoscopy
the following morning

11. The patient with achalasia may have:

A. Incomplete relaxation of the lower esophageal sphincter


B. Ineffective peristalsis
C. Absent peristalsis
D. Dysphagic chest pain
E. A and C
F. All of the above

12. Bloody emesis in a 2 day old healthy full term neonate is likely to be secondary to:

A. Mallory-Weiss tear
B. Esophageal varices
C. Foreign body aspiration
D. Swallowed maternal blood

13. Which of the following statements is true regarding reflux:

A. Thickening formula reduces reflux episodes


B. Proton pump inhibitors have been found to improve infant irritability
C. Treatment with PPI’s for three months is indicated in patients with
endoscopically proven reflux esophagitis
D. Acute life threatening events have definitively been linked to gastroesophageal
reflux disease
E. Erythromycin has been proven to be beneficial in patients with GERD

14. Nissen fundoplication is indicated for all except:

A. Institutionalization
B. Intractable pain
6

C. Recurrent bleeding
D. Recurrent aspirations
E. Neurological impairment

15. ENT complications of GERD may include all of the following except:

A. Sinusitis
B. Otalgia
C. Laryngitis hoarseness
D. Glue ear
E. Recurrent epistaxis
7

16. An older sibling finds his 18m/o brother has taken apart a small non-functioning LED
flashlight and the lithium battery is missing. He informs his parents who then bring
the toddler to the ER. The ER attending calls and informs you that by x-ray the
battery in the stomach. You proceed to the ER, review the history, examine the
child and find he is in no distress. You then:

A. Call the endoscopy suite to set up for immediate removal


B. Tell the parents since the battery size is <10mm there is no need for concern or follow-
up
C. Tell the parents a “dead” battery will not cause tissue damage
D. Arrange to follow the course of the battery with daily radiographs
E. Discharge the patient for F/U in one week but ask the parents to notify you if
pain or vomit- ing evolves and to examine diaper stools for the battery over
the next 2-4 days

17. A one week old male infant has crying after feeds that last 2 hours. He spits up and
often calms down after passing gas. He stools after each feed. He takes a standard
cows’ milk formula. Mother recently noted small flecks of blood in the stools.
The most likely etiology is

A. Malrotation
B. Pyloric stenosis
C. Hirschsprung’s Disease
D. Milk-protein intolerance
E. Mild ulcerative colitis

18. Which statement is false?

A. E. histolytica infections are asymptomatic in 90% of patients.


B. E. histolytica liver abscesses tend to occur only in
those children who develop severe dysentery
C. G. lamblia can be zoonotic.
D. B. hominis produces diarrhea, bloating and eosinophilia
E. G. lamblia is predominantly contracted through water.

19. Which statement is false?

A. Salmonella infections can result from contaminated eggs, chicken, salads and cheese.
B. Campylobacter and Shigella sp infection can be very similar in presentation.
C. Yersinia infection of the terminal ileum can mimic appendicitis
D. Bacillus cereus constitutes a major component of probiotic therapy
E. Bacillus bifidum and Streptococcus thermophilus constitutes major
components of probiotic therapy

20. Which is a common association found amongst gastric polyps?


8

A. Peutz-Jeghers syndrome and juvenile polyps


B. H. pylori infection and hyperplastic polyps
C. Juvenile polyposis syndrome and fundic gland polyps
D. Familial adenomatous polyposis syndrome and hamartomas.

21. With respect to gastric tumors in childhood, which statement is false?

A. Fundic gland polyps associated with familial adenomatous polyposis


may undergo malignant transformation
B. Fundic gland polyps associated with long-term PPI use rarely appear before six years
C. Fundic gland polyps associated with long-term PPI require
surveillance for malignant transformation
D. Nearly all patients with Peutz-Jeghers syndrome require surveillance for gastric
hamartomas
E. Gastric teratomas with fetal elements occur exclusively in females
9

22. During a fraternity initiation, a 18y/o is forced to swallowed two live minnows.
Three days later he presents to the ER with severe abdominal cramps, nausea and
blood tinged vomitus. Physical exami- nation reveals diffuse abdominal tenderness.
You decide to consult a surgeon because you suspect

A. A perforated duodenal ulcer


B. Outlet obstruction by a minnow
C. Gastritis from schistosomiasis
D. Gastritis and perforation by Eustrongylides

23. Regarding H pylori which statement is true?

A. The natural reservoir for H pylori is zoonotic


B. Once gastritis becomes chronic it is usually irreversible
C. 90% of children with duodenal ulcer have antral H pylori infection
D. Approximately 15-20% of children with GERD response to H pylori
eradication
C. H Pylori

24. Regarding H pylori infection which statement is false?

A. Patients with MALT lymphoma experience regression of the


tumor with eradication of H pylori
B. H pylori gastric “cobblestoning” is more common in children
C. Biopsy for H pylori is best obtained from the antrum
D. H pylori grows best on chocolate agar medium

25. The metabolic disturbance most typical of pyloric stenosis is:

A. Hypochloremic acidosis
B. Hyperchloremic acidosis
C. Hypochloremic alkalosis
D. Hyperchloremic alkalosis

26. Which object(s) is LEAST likely to require endoscopic removal from the stomach?

A. 8 cm metal rod
B. A toy with known lead paint
C. 4 magnet balls
D. A nickel from a 9 year old boy

27. Which items is MOST likely to need endoscopic removal from the stomach?

A. A 3 cm-long pen cap in a 14 year old boy


B. A closed safety pin
10

C. A quarter ingested 4 weeks ago.


D. A 1 cm long toy piece

28. Which of the following is matched with its major site of injury?

A. Acid ingestion  stomach


B. Alkali ingestion  stomach
C. Doxycycline  duodenal bulb
D. Ibuprofen  colon
11

29. An 8 year old female comes to the ER with 12 hours of abdominal pain, vomiting,
and low grade temperature of 100.2. On exam, she has diffuse tenderness with
guarding in the right lower quad- rant and rebound tenderness. The next best
step in management is:

A. Oder a CBC, blood culture, and urine culture


B. Order an abdominal CT without contrast
C. Order a surgical consult
D. Order a pelvic ultrasound

30. A 14 year old male has had diarrhea and vomiting for 3 days with fevers up to 102.
Today he has right-sided lower abdominal pain with rebound tenderness. Laparoscopy
only shows mild periappen- diceal involvement. Which of the following is most
likely to yield the correct etiology:

A. Blood culture for gram negative sepsis


B. Stool culture for yersinia
C. WBC to look for immunosuppression
D. Colonoscopy for inflammatory bowel disease

31. 3 month old male infant is referred to you because of secretory diarrhea and
hypoglycemia. His work up included normal CBC, and normal Immunoglobulin
levels except for elevated IgE. You are sus- pecting autoimmune enteritis (AIE) as a
diagnosis. Which one of the following is true about AIE?

A. IPEX usually presents later in childhood.


B. It only affects boys.
C. Histology is significant for intraepithelial T cell infiltration.
D. Patients usually have normal lymphocyte count.
E. Only the IPEX form has extra-intestinal manifestations.

32. In you discussion with the parents of the above patient, which of which statement is correct?

A. Most of these cases will resolve by adolescence.


B. Diarrhea will decrease with bowel rest.
C. Immune suppression with monotherapy is effective.
D. BMT helps control diabetes and eczema in patients with AIE.

33. Appendicitis is usually characterized by periumbilical pain which moves to the RLQ
within the first 12-24 hours. In what situation may a patient not experience the
“classic” RLQ pain associated with appendicitis?

A. A long appendix
B. A fecalith impacted appendix
C. A perforated appendix
12

D. A retrocecal appendix.

34. Which of these would be most likely to cause pain in the LLQ?
A. Sigmoid volvulus
B. Pancreatitis
C. Acute Cholecystitis
D. Peptic ulcer

35. A full-term neonate has nonbilious emesis after each feed since birth. An abdominal
radiograph shows a dilated stomach and you suspect possible gastric outlet
obstruction. The next diagnostic test performed should be:

A. Nuclear medicine gastric emptying study


B. Barium contrast upper gastrointestinal series
C. Magnetic resonance imaging of the abdomen
D. Nuclear medicine biliary scan
E. Left lateral decubitus radiograph
13

36. A 2 year old male undergoes an abdominal ultrasound to evaluate the kidneys after
an abnormal urinalysis is discovered. A gastric duplication cyst is an incidental
finding on the ultrasound. The cyst is not large enough to cause compression and the
child has no vomiting. The recommended timing of treatment is:

A. Emergent surgery the same day


B. Surgical excision soon after the lesion is discovered
C. Surgical excision after 5 years of age
D. Surgical excision during adolescence
E. No surgical excision because the child is asymptomatic

37. A 14 m/o male presents with a three month history of chronic diarrhea, anorexia and
a fall from the 75th to the 25th percentile in weight. His height however remains on
track. There is no vomiting, but he does have increased foul and rancid flatulence and
hydrogen sulfide eructations. This patient merits

A. Serum tTG antibodies screening


B. IgA/IgG antigliadin antibodies screening
C. Stool analysis for giardiasis
D. DQ2-DQ8 screening
E. A trial of a milk-free diet

38. With respect to celiac disease which statement is false?

A. It is the most common non-surgical cause of asplenia


B. It can be associated with Down syndrome and idiopathic pericarditis
C. The biopsy reveals intraepithelial lymphocytosis
D. African-Americans are at increased risk if they are lactose intolerant
E. Up to 6% of patients with irritable bowel syndrome turn out to have celiac
disease.

39. Most duplication cysts arise in:

A. Stomach
B. Colon
C. Ileocecal region
D. Jejunum

40. Most duplication cysts present in:

A. Early adulthood
B. Puberty
C. Neonatal period
D. Before age 2
14

41. The best treatment option for enteric duplication cysts is:

A. Percutaneous drainage
B. Medical management
C. Surgical excision
D. Establish communication with main intestinal lumen
15

42. Which of the following neonates has the greatest risk potential of
developing necrotizing Enterocolitis?

A. Stable 1 day old, full term, female infant with no significant medical
findings consuming breast milk enterally.
B. 2 week old, 32 week gestation, male infant with a birth weight of 1200 grams
on hyperos- molar formula.
C. 3 week old, large for gestational age full term male infant of a diabetic mother.
D. 3 week old, 36 week gestation, male infant with birth weight of 2000 grams
and a menin- gomyelocele

43. A 10 day old, ex- 28 week premature female is confirmed to have Stage IIB NEC.
You would expect this infant to display all of the following signs and
symptoms except:

A. Temperature instability, lethargy and abdominal distension


B. Pneumatosis intestinalis on abdominal radiograph
C. Severely perforated bowel
D. Thrombocytopenia, diminished bowel sounds and grossly bloody stool

44. Most patients with malrotation present:

A. After one year of age


B. In the first month of life
C. In adolescence
D. Immediately after birth

45. The best modality for diagnosis of malrotation in a child is:

A. Barium enema
B. Upper GI series
C. Abdominal Ultrasoound
D. Upper endoscopy

46. On an UGI series the location of the duodenal-jejunal flexure ( Ligament of Treitz) is found:

A. Upper right of the spine


B. Upper left of the spine
C. Midline
D. Lower left of the spine

47. You will be following a patient who is s/p successful surgical repair of a
gastroschisis. Your long term concerns will be for:

A. Evolving intestinal dysmotility


16

B. Cantrell pentalogy defects


C) Development of GERD
D) All of the above
E) A and C

48. Newborns with surgically corrected omphalocele or gastroschisis are both at risk for:

A. Adhesions
B. GER and dysmotility
C. Atresias
D. Bowel ischemia
E. All of the above
17

49. Which of the following laboratory findings is NOT likely to be found in a


patient presenting with Small Bowel Bacterial Overgrowth?

A. Elevated D-lactate
B. Macrocytic Anemia
C. Microcytic Anemia
D. Elevated Stool pH
E. Hypocalcemia

50. Which of the following pairings of anatomic location and bacterial concentration is
INCORRECT?

A. Colon: <1011 – 1012 CFU/mL


B. Ileum: <108-1010 CFU/mL
C. Duodenum and Proximal Small Bowel: <106-108 CFU/mL
D. Stomach: <103 CFU/mL

51. A 4 year old male presented to the ED with a 5d h/o diarrhea, which became
bloody for the past 2 days. He was admitted overnight. Labs on admission include
WBC 17,000 Hgb 12.5, Plts 195; Na 142, K 4, Creat 1.5 BUN 30. Which of the
following organisms is most likely?

A. Yersinia
B. Toxigenic E. Coli
C. Norwalk-like virus
D. C. difficile
E. E. Coli O157:H7

52. A 2yo girl, who attends daycare, is brought to your outpatient clinic with a 3 day
history of watery, nonbloody diarrhea. Mom reports that she had low-grade fever,
not checked; since yesterday the stools have become bloody. She has not received
any recent antibiotic therapy. On exam the pa- tient’s vital signs reveal temperature
40 deg C, blood pressure 85/63, pulse 110, respiratory rate 20, oxygen saturations
100% room air; she has dry mucous membranes; stool testing reveals multiple
leukocytes. The most likely cause of this girl’s illness is:

A. Salmonella typhi
B. C difficile
C. Enteroinvasive E coli
D. Shigella
E. Giardia

53. What is the most common cause of diarrhea in children worldwide?

A. Rotavirus
18

B. Norwalk virus
C. Enterotoxigenic E. Coli
D. Salmonella
E. Shigella

54. You were called to evaluate a 48-hr-old male in the newborn nursery for a 12 hr
history of bilious emesis. The patient was born full-term vaginally without
complication. A prenatal ultrasound exami- nation revealed polyhydramnios during
the third trimester. On physical exam, the patient is mildly jaundiced, with a slight
abdominal distension and hypoactive bowel sounds. There is no abdominal
tenderness, respiratory distress, or signs of dehydration. He passed meconium 24 hrs
after delivery. You order an abdominal radiograph that shows dilation of the stomach
and proximal duodenum and absence of distal gas. The most appropriate next study
for this newborn is:

A. Fetal Karyotyping
B. Upper gastrointestinal radiograph
C. Echocardiography
D. Renal Ultrasound
E. Abdominal CT scan
19

55. Which of the following statements about tropical sprue (TS) are true?

A. Tropical sprue (TS) is seen only in visitors to the tropics


B. TS does not occur in epidemic form
C. TS has a uniform geographical distribution in all tropical regions
D. Patients can present with nutritional deficiencies in the absence of diarrhea.

56. Which of the following statements about TS are true?

A. Bacterial colonization of the small intestine is rare in patients with TS


B. Colonic absorption of water remains unaffected in TS
C. Morphological changes in the mucosa in patients with TS resemble those of celiac
disease
D. Small bowel transit is reduced in TS.

57. Which of the following is true regarding Clostridium difficile?

A. It’s presence always indicates active infection


B. Primarily causes watery diarrhea
C. Prevalence increases with age
D. Retesting should be done after 2 weeks of treatment

58. Which of the following causes of colitis presents as focal lesions without surrounding
inflammation

A. Crohn’s colitis
B. Microscopic colitis
C. Eosinophilic colitis
D. Behçet ‘s disease

59. Of the following, which best describes graft vs. host disease of the gut?

A. Exclusively involved the upper gastrointestinal tract


B. Significant involvement of the lamina propria
C. Apoptosis is commonly seen
D. Precipitated by food or drug allergies

60. A 14 year old female presents with lower abdominal pain for the past 4 months,
diarrhea, weight loss and intermittent fevers. Blood work shows a hematocrit of
10.2, mean cell volume of 65%, platelet count of 525, and sedimentation rate of 45.
Colonoscopy reveals moderate chronic, active colitis and ileitis with granuloma. You
decide to begin medical therapy for Crohn’s disease. A week later, her mother calls
and informs you she has developed pain in her legs. Which of the following
medications is most likely the cause of her new symptom?
20

A. Prednisone
B. Mesalamine
C. Metronidazole
D. Infliximab
E. Lactobacillus

61. An 8 year old male presents with chronic diarrhea and abdominal pain waking him
from sleep at night. His school performance has been declining due to frequent
absences and inability to concen- trate. Upper intestinal endoscopy and colonoscopy
reveal active esophagitis and linear ulcerations in the colon. The most common
extraintestinal manifestation of this disorder is which of the following:

A. Iritis
B. Erythema nodosum
C. Arthritis
D. Arthralgia
E. Aphthous stomatitis
21

62. Which of the following medications for ulcerative colitis works by inhibition of
prostaglandin and leukotriene synthesis?

A. Azathioprine
B. Mesalamine
C. Tacrolimus
D. Infliximab
E. Cyclosporin

63. A 14 year old male comes to your office with 6 weeks of persistent diarrhea
containing streaks of blood and crampy lower abdominal pain. Perianal inspection
reveals no lesions, and occult blood testing confirms the presence of blood. His
height is at the 45 percentile for his age. You suspect ulcerative colitis. Which of the
following findings on colonoscopy would most strongly support this diagnosis?

A. Inflammatory pseudopolyps
B. Areas of normal colon mucosa between inflamed regions
C. Inflammation of the terminal ileum
D. Nodularity of the colon mucosa.
E. Linear ulcerations

64. A child with Hirschsprung’s disease would have the following finding on anorectal
manometry after rectal dilation with the balloon:

A. Increased internal sphincter tone and decreased external sphincter tone.


B. Increased internal sphincter tone and increased external sphincter tone.
C. Decreased internal sphincter tone and decreased external sphincter tone.
D. Decreased internal sphincter tone and increased external sphincter tone.

65. The most common cause of constipation in childhood is:

A. Celiac disease
B. Inflammatory bowel disease
C. Hirschsprung disease
D. Functional constipation
E. Cystic fibrosis

66. A 4 year old otherwise healthy male has large-caliber, painful bowel movements,
which occur every 5-7 days. On physical exam, there is a hard palpable mass in the
left lower quadrant of his abdo- men. His exam is otherwise normal. He is taking
no medications. Which of the following is the most appropriate next step?

A. Obtain an abdominal x-ray to confirm presence of fecal impaction


B. Start this patient on daily maintenance oral laxative
C. Treat fecal impaction with oral and/or rectal therapies, followed by a
22

maintenance regimen and close followup.


D. Encourage increased fiber in his diet and close followup.
E. Order a barium enema.

67. Encopresis typically occurs when:

A. The external anal sphincter is no longer able to function to prevent defecation


B. The internal anal sphincter is no longer able to function to prevent defecation
C. The puborectalis is no longer able to function to prevent defecation
D. The external anal sphincter is contracted.
E. The anal canal is lengthened.
23

68. Which of the following is the correct statement about hemorrhoids?

A. Internal hemorrhoids cause painful bleeding.


B. Hemorrhoids develop later in life due to chronic straining.
C. Bleeding from hemorrhoids can cause anemia.
D. Patients with portal hypertension have higher risk of developing hemorrhoids.

69. Which statement is incorrect about the management of Hemorrhoids?

A. Acute thrombosed external hemorrhoids can be excised within 24-48 hr.


B. High fiber diet helps shrink hemorrhoids and decrease bleeding.
C. Sclerotherapy is more effective than rubber banding.
D. 5-10 % of patients with hemorrhoids will need surgical treatment.

70. Which of the following statements regarding Hirschsprung disease is TRUE?

A. HD is commonly associated with other congenital malformations.


B. Presence of recto-anal inhibitory reflex is characteristic.
C. Early diagnosis is important to avoid development of enterocolitis and
subsequent toxic megacolon.
D. Surgery for HD is usually performed between 2 and 3 years of age.

71. Which of the following is a common manifestation of HSP?

A. Painful palpable purpura


B. Colicky abdominal pain and occult GI bleed
C. Arthritis of large joints in the arms
D. Proteinuria

72. What of the following is part of the pathophysiology of E coli in HUS?

A. It produces Shiga toxin


B. It produces cholera toxin
C. It produces C diff toxin
D. It is enteroinvasive and ulcerogenic

73. Which of the following is the appropriate diet for an infant with primary intestinal
lymphangiectasia?

A. Low protein and low fat diet


B. Low protein and high fat diet
C. High protein and low fat diet
D. High protein and high fat diet

74. You follow a 6 y/o boy with juvenile polyposis coli. Part of your care involves
24

A. Screening all at risk family members


B. Colonoscopy performed annually with polypectomies PRN
C. Gastroscopy of the index patient beginning at adolescence
D. Only B and C
E. A, B and C

75. A 9 year old girl with presumed Peutz Jeghers syndrome lacks the STK11 mutation
on chromosome number 11. In this patient:

A. The risk of malignancy is greatest in the small bowel over the colorectal area
B. There is no risk for precocity
C. Her risk for breast tumor is under 25%
D. All statements are true
E. All statements are false
25

76. 12 month old girl present with her third episode of rectal prolapse during the last 2
months. No constipation reported by the parents but was prescribed lactulose in
the last month with a good response. Physical examination normal.

Which investigation should be the next step?

A. colonoscopy
B. barium enema
C. sweat test
D. abdominal series

77. An 8 month-old male infant presents to your clinic with 6 weeks of crying with
stooling. The parents have noted small steaks of blood in the diaper after he stools.
His nutrition consists of breast milk and a variety of fruits, vegetables and cereal. He
has had no vomiting, signs of abdominal pain, fe- vers or diarrhea. He was passing
hard-consistency stools at the beginning of this course, but this has improved with
daily MiraLax prescribed by his primary physician. Currently, he is passing three softly
formed stools daily. On your physical exam, you note a fissure in the posterior midline,
an associated small non-inflamed skin tag and hypertonicity of the anal sphincter. In
addition to careful hygiene, what is the best management approach?

A. As 0.2% nitric oxide preparation topically three times daily


B. Daily anal dilatation at home
C. Cow’s milk restriction
D. Increased dietary fiber
E. Lateral internal sphincterotomy

78. A 2-year old female has had a three week history of decreased stooling frequency.
Her stools have become hard in consistency and require straining. Her parents have
not noted blood in her stools, but are concerned with the amount of discomfort she
has with defecation. She has become ex- tremely apprehensive with diaper changes
and is exquisitely tender to the touch in the diaper area. On your physical exam, you
note a well-demarcated and moist area of erythema in her perineum, radiating from
the anus without induration. She has not had a similar perineal rash in the past. What
is the most appropriate diagnostic test?

A. Direct GABHS antigen


B. ASLO or anti-DNase B
C. Perianal swab culture
D. Sigmoidoscopy with biopsies
E. Stool ova & parasites

79. A previously healthy 7-year-old male was diagnosed with a perirectal abscess by his
primary physi- cian. Initial management of oral antibiotics and sitz baths has been
initiated. Which of the following is considered an indication for surgical
26

management?

A. Persistent perineal pain despite antibiotics


B. Persistent drainage despite 3 months of medical management
C. Underlying inflammatory bowel disease
D. The presence of fistula in ano at initial presentation
E. Current immunosuppressive medications for JIA
27

80. A 2-year-old female presents with distended abdomen, feeding intolerance, vomiting,
and chronic severe constipation since infancy. One year ago she had been at the
40% weight for age on the CDC growth curves and she is now on the 10% weight
for age. Past medical history is significant for multiple urinary tract infections which
have required antibiotic prophylaxis. Previous abdominal ultrasound showed
megaureters and hydronephrosis. An abdominal radiograph is notable for a
distended small bowel with multiple air fluid levels and stool-filled colon. Previous
home clean-outs prescribed with osmotic and stimulant laxatives have not helped to
resolve her symptoms. She has no abnormalities in thyroid studies, metabolic
problems, celiac testing, or hematocrit. She had a previ- ous normal anorectal
manometry. Exam in your office was notable for distended abdomen which is
tympanic to percussion with mild abdominal pain on palpation throughout. Rectal
exam was normal without stool mass noted.

What next study would give the most diagnostic yield?

A. Upper gastrointestinal contrast study with small bowel follow through


B. Radio-opaque marker study
C. Colonoscopy with biopsies for histopathology
D. Antroduodenal and colonic manometry

81. The hepatic lobule is centered on

A. Bile duct
B. Central vein
C. Portal vein
D. Hepatic artery

82. You follow an eleven year old boy with ulcerative colitis and he comes in for a
routine evaluation. He claims his symptoms have been well controlled with
mesalamine and at this visit he says his stools are formed with no blood, but he has
had new tenderness in his abdomen, occasional nausea, new
fatigue and itching. He denies recent trauma, fever or new medications. On physical
examination you note mild hepatomegaly and tenderness, and subtle conjunctival
icterus. You are concerned he may have acquired hepatitis or is evolving primary
sclerosis cholangitis. You order all the following studies except one.

A. Hepatitis serologies
B. JAG-1 and NOTCH-2 mutational studies
C. Fresh liver function studies
D. Blood cultures
E. MRCP

83. Findings in a patient with Alagille syndrome may include all of the following findings except
which?
28

A. Posterior embryotoxon
B. Hemivertebra
C. Periductal hyperplasia
D. Peripheral pulmonary stenosis
E. Portal tracts with bile duct ratio of less than 0.5
29

84. During his routine one month check-up a four week old African American male is
found to have woody hepatomegaly and a conjunctiva suggestive of icterus. His
mother is concerned he is not gain- ing weight and that his suck is poor. Your initial
studies reveal conjugated hyperbilirubinemia, moder- ate increase of transaminases
but a 4-fold increase of GGT. You order an ultrasound which reveals a small
gallbladder but the sonographer is unable to measure the hepatic ducts. Hepatic
scintography shows normal uptake but delayed and diminished excretion at 24 hours.
Your next step is to order

A. A repeat ultrasound to locate the “triangular cord sign.”


B. Schedule a liver biopsy
C. Repeat scintography with a double dose of Phenobarbital
D. Order serologies for CMV, HBV, HCV and Cocksackie B
E. Order a bone marrow for hemophagocytosis

85. You are following a five month old male who is status post Kasai. His liver is firm
but not hard and he has no splenomegaly. He is slowly gaining weight and his
nutrition appears adequate. There is no diarrhea but both his stools and urine are dark.
He is on ADEK supplementation and ursodeoxycholic acid. At this visit, his total
bilirubin is 6.4 mg/dL and his stools are guaiac positive. You should at this point:

A. Schedule endoscopy for possible varices


B. Increase his dose of ADEK and ursodeoxycholic acid
C. Change his antibiotics for possible ascending cholangitis
D. Request a HIDA scan to assess Kasai function
E. Refer the child to the transplant team for preliminary assessment and
registration.

86. A 16-year-old female has a 6-month history of intermittent epigastric and right upper
quadrant ab- dominal pain. Her BMI is 35. The pain is often worse after fatty meals
and it has not improved with 8 weeks of appropriate proton pump inhibitor therapy.
Laboratory testing for pancreatitis, hepatitis, liver function and celiac disease have
been unrevealing. Her WBC is normal. An abdominal ultra- sound with focus in the
right upper quadrant showed a normal appearing gallbladder without any evidence
of gallstones or sludge. The liver parenchyma and pancreas appeared normal. The
com- mon bile duct diameter was 3mm. You suspect that she may suffer from
chronic acalculous chole- cystitis/biliary dyskinesia. Which of the following is the
best next step in establishing that diagnosis?

A. Magnetic resonance cholangiopancreatography (MRCP)


B. Endoscopic retrograde cholangiopancreatography (ERCP) with sphincter of
Oddi manometry
C. Abdominal CT scan
D. Upper endoscopy with bile sampling for analysis
E. HIDA scan with fatty meal stimulation
30

87. The risk of childhood cholelithiasis is increased in all these circumstances except:

A. Cystic fibrosis with the mutation of F508 or I507


B. Crohn’s disease limited to the terminal ileum
C. Total abdominal situs inversus
D. Thalassemia and hereditary spherocytosis
E. A pregnant Pima Indian adolescent
31

88. A 2-day-old otherwise healthy full-term male infant undergoes an abdominal


ultrasound for surveil- lance of pelviectasis initially diagnosed on a prenatal
ultrasound. The kidneys appear normal but the ultrasound reveals a 3mm, mobile
echogenic mass in the gallbladder consistent with a gallstone. No other gallbladder or
liver abnormalities are seen and there is no pericholecystic fluid noted. The child has a
transcutaneous bilirubin consistent with a value of 4.3 mg/dl and is feeding well.
What is the most appropriate next step in the management/evaluation of the
gallstone?

A. Start Ursodeoxycholic Acid therapy


B. Consult pediatric surgery for cholecystectomy
C. Order a magnetic resonance cholangiopancreatography (MRCP)
to assess for bile duct anatomy
D. No further evaluation if the gallstone does not cause symptoms
E. Technetium 99m–hepatic iminodiacetic acid (HIDA) scan

89. A 7-year-old boy with nephrotic syndrome who has been hospitalized for 3 weeks
for the manage- ment of edema develops intermittent right upper quadrant
abdominal pain. An abdominal ultra- sound shows the presence of a mobile, 6mm
stone in the gallbladder. Which of his medications is most likely to have played a
role in the development of the gallstone.

A. Hydralazine
B. Omeprazole
C. Furosemide
D. Lisinopril
E. Dalteparin

90. Which of the following is a risk factor for the development of black pigment gallstones?

A. Hereditary Spherocytosis
B. Obesity
C. Pregnancy
D. Bile infection
E. Hyperlipidemia

91. 17 year old male has ALF. He was well until 2 wks before admission, when jaundice
developed. One week later he was hospitalized because of progressive confusion. He
has slight asterixis. Labs: Hemo- globin 9.8, ALK PHOS 60, T bilirubin 40, D
bilirubin 12, UA 1.1, AST 300, ALT 170, INR 2.5. SMA and ANA neg.
Ceruloplasmin 24 (22-43). Which of the following is true?

A. Normal value for ceruloplasmin excludes Wilson


B. Slit-lamp examination should be done to look for KF rings
C. He is too ill to be considered for OLT
32

D. Liver Bx should be the next study

92. 20 year old female was admitted for “Liver Transplant”. Three months ago she
began gaining wt (16 lbs). Two mo ago she had dark urine and yellow skin. Tests
for HBsAg, HB core AB, HAV IgM, HCV AB, CMV IgM and IgM to VCA for
EBV were all negative. PE showed jaundice, shifting dullness,
hepatomegaly. LABS: AST 624, T Bilirubin 12, TP 8.5, Alb 2, INR 2, ANA 1:40.
Tests for SMA and LKM were neg. What is the next step in her treatment?

A. OLT
B. Observation for 3 mo
C. IFN and Ribavirin
D. Prednisone 60 mg daily
33

93. A 15 year old female is admitted to the ICU following intentional acetaminophen
overdose. She is unresponsive, mildly hypotensive and has no stigmata of chronic
liver disease. She is most likely to die of:

A. Cerebral herniation
B. Coagulopathy with bleeding
C. Liver synthetic failure
D. Renal failure

94. A 12 year old boy is seen for FTT, episodic irritability, lethargy, and refusal to eat
animal protein (milk, eggs, and meat). Which of the following abnormalities is most
characteristic of OTC deficiency?

A. Elevated ammonia
B. Elevated plasma citrulline
C. Metabolic acidosis
D. Aminotransaminase levels more than 1000

95. A 5 day old breastfed infant is admitted to the hospital because of lethargy and a
poor suck. The infant appears jaundiced. The total bilirubin is 12 with 25%
conjugated. The metabolic disease most likely to cause jaundice in this infant is:

A. A HFI
B. B Galactosemia
C. C Hypothyroidism
D. D PKU

96. A 2 month old infant who presents with a 1 wk h/o intermittent vomiting appears
jaundiced on PE. At 4 weeks of age the exclusively breastfed baby was gaining wt
well and was not icteric. The mother subsequently returned to work and the infant
has been receiving supplements of formula and apple juice. A urine test for
reducing substance is positive.

A. alpha-1-antitrypsin deficiency
B. Biliaryatresia
C. hereditary Fructose Intolerance
D. Cystic Fibrosis

97. Which of the following statements about pyogenic abscess of the liver is true?

A. The right lobe is more commonly involved than the left lobe.
B. Appendicitis with perforation and abscess is the most common
underlying cause of hepatic abscess.
C. Mortality is not determined by the underlying disease.
D. Mortality from hepatic abscess is currently greater than 80%.
34

98. You follow a 7 m/o cholestatic male with biopsy proven non-syndromic paucity of
bile ducts. At this visit the mother is pleased to report he is no longer scratching
himself and is gaining weight. Surveil- lance LFT reveal persistent cholestasis,
elevated AlkP, progressive increase in ALT/AST, but a progres- sive drop in GGT.
You next order

A. Urinary bile acid profile


B. Repeat liver sonography
C. Repeat liver biopsy
D. A followed by B if the profile is abnormal
E. A followed by C if the profile is abnormal
35

99. You receive a request for a second opinion on a 4 wk/o male with hepatomegaly,
increased LFTs with low GGT, evidence of steatorrhea, and a liver biopsy revealing
giant cell transformation, fibrosis and bile lakes. You suggest as a primary next
step to:

A. Seek a hematology consult to r/o a hemophagocytosis syndrome


B. Obtain a urinary bile acid profile
C. Begin an empiric trial of Phenobarbital at 3 mg/kg/day
D. Begin an empiric trial of ursodeoxycholic acid at 15 mg/kg/day
E. Seek a infectious disease consult to r/o enterovirus infection

100. A 5- month old baby presents to the ER with vomiting, diarrhea and poor weight
gain for the past month. He is a full term baby born to a healthy mother with no
prenatal or perinatal complications. There have been no sick contacts. No fevers,
rashes, or recent antibiotic use. He has been solely breastfed, and one month ago he
started eating jarred baby foods, but has not been taking them well. A deficiency in
which enzyme should be considered in this case?

A. Aldolase B
B. Galactokinase
C. Fumarylacetoacetate hydrolase
D. Glucose-6-phosphatase
D. Galactose-1-phosphate uridyl transferase

101. A 3-day old female develops a fever prior to discharge home. She undergoes a
complete sepsis evaluation including blood, urine, and CSF cultures. Urine and
blood cultures are both positive for E Coli. Which of the following should be
the initial next step for this patient?

A. Check thyroid studies


B. Obtain an abdominal ultrasound
C. Switch to a soy-based formula
D. Check PT, PTT, INR
E. Obtain chest x-ray

102. Which of the following chemotherapy medications is NOT associated with veno-occlusive
disease?

A. 6-thioguanine
B. Busulfan
C. Cytosine arabinoside
D. Dactinomycin
E. L-asparaginase

103. Ground glass cytoplasmic inclusions are typically seen in which types of drug-induced liver
36

injury?

A. Amiodarone
B. Isoniazid
C. Isotretinoin
D. Mycophenolate mofetil
E. Oral Contraceptives
37

104. A two-year old boy develops URI symptoms 2 days prior to admission. His
pediatrician diagnoses him with a viral pharyngitis. The boy drinks minimal fluids
over the next two days. On the day of admission, parents note that he is pale and
extremely sleepy. They drive him to the ER, and the boy has a seizure in the car on
the way there. In the ER, his serum glucose is 25 mg/dL. Blood gas re- veals pH
7.29, pCO2 31, and HCO3 of 15. Electrolytes reveal Na 131, K 4.4, and Cl 99.
Ammonia is elevated at 95. Urinalysis demonstrates no glucose, no protein, and 1+
ketones. The most likely diagnosis is:

A. Sepsis
B. Urea cycle defect
C. Congenital heart defect
D. Organic acidemia
E. Fatty acid oxidation disorder

105. A 27-year old woman at 33 weeks gestation presents to the emergency room with
nausea, vomiting, and abdominal pain. She is found to have a mild elevation in her
transaminases, and is subsequently admitted for IV fluid hydration and observation.
Over the next 3 days, she develops worsening elevation in her transaminases and a
coagulopathy, and progresses to fulminant liver failure. An emergency cesarean
section is scheduled and the baby is delivered. Both mom and baby go on to do well.
Screening for which of the following mutations should be considered for the
baby?

A. MCAD mutation
B. G1528C mutation
C. Trifunctional protein deficiency
D. SCAD mutation

106. A 16-year-old white female presented with a few weeks history of low-grade-fever
and arthritis. On physical exam, she was noted to have erythema nodosum on both
her shins, arthritis of her ankles, and hepatomegaly. Chest radiograph showed
bilateral hilar adenopathy. Liver biopsy revealed non- caseating granulomas mainly
in the portal tract. Skin tuberculin test was negative. What is the most likely
diagnosis:

A. Tuberculosis
B. Sarcoidosis
C. Coccidioidomycosis
D. Histoplasmosis

107. The oncology service consults you for a febrile 12 y/o with hepatomegaly,
splenomegaly, hyperbili- rubinemia, elevated alkaline phosphatase and leucocytosis
Ultrasound reveals “bull’s eye” lesions in the hepatic parenchyma most likely
caused by
38

A. Systemic candidiasis
B. Coccidiomycosis
C. Histoplasmosis
D. Coxiella infection

108. With regards to hepatic granuloma, which statement is false?

A. Up to 10% of all liver biopsies reveal granulomas


B. Brucellosis, Q-fever and Hansen’s disease may cause hepatic granulomas
C. Granulomas occur in about 10% of patients with miliary tuberculosis
D. Acid-fast bacilli can actually be seen in TB granulomas

109. All of the following are associated with NAFLD EXCEPT:

A. Increased TGF-beta
B. Increased adiponectin levels
C. Increased hepatic Fe stores
D. Increased reactive oxygen species
E. Increased free fatty acids
39

110. The most common area of presentation of PTLD in pediatric liver transplant recipients include;

A. The kidneys
B. The pancreas
C. The liver
D. The pharyngeal and cervical lymphatic chain

111. Which of the following statements are true regarding the outcome of liver transplantation in
children

A. Expected one year patient survival is approximately 99%


B. Chronic rejection results in graft loss in approximately 5% of children
C. Primary non-function has been virtually eliminated as a clinical problem
D. All of the above

112. Nephrotoxicity secondary to calcineurin inhibitors causes the following

A. Afferent arteriole vasoconstriction


B. Endothelial cell damage
C. Interstitial fibrosis
D. Tubular atrophy
E. All of the above

113. In the setting of acute liver failure it is appropriate to proceed with transplantation:

A. Before establishing the etiology of the liver injury


B. When patients exhibit clinical signs of stage II encephalopathy
C. When patients have clinical signs of irreversible neurologic injury
D. Only if some form of bridge therapy is not available
E. None of the above

114. Accepted contraindications to liver transplantation in children include all except the following:

A. Alpers syndrome
B. Cystic fibrosis
C. Advanced pulmonary hypertension
D. Uncontrolled systemic infection
E. The above are all accepted contraindications

115. As compared to patients receiving tacrolimus, patients receiving cyclosporine are less likely to:

A. Develop DM
B. Develop hypertension
C. Dyslipidemia
D. Seizures
40

116. Primary bile acid therapy with cholic acid is effective in which of these peroxisomal disorders?

A. Zellweger syndrome
B. Adrenoleukodystrophy (ALD)
C. Refsum disease
D. Methylacyl-CoA racemase deficiency
E. Rhizomelic chondrodysplasia punctata
41

117. An eight month old infant presents with cholestasis, poor growth and chronic
diarrhea. He had negative TORCH serologies, normal alpha 1 antitrypsin, and normal
metabolic screening. His labora- tory tests revealed GGT 23, Bilirubin 6.0/4.0 and
ALT 350. His most likely diagnosis is:

A. PFIC-1, FIC-1 disease


B. PFIC-2, BSEP disease
C. PFIC-3, MDR3 disease
D. BRIC

118. Match the finding or symptoms to the gene defect:

A. MDR3 disease 1. Bile duct proliferation


B. BSEP disease 2. Episodes of severe pruritus which resolve
C. FIC-1 disease 3. Associated with liver tumors
D. BRIC 4. Recurrent pancreatitis

119. The family of a child newly diagnosed with AGS asks you what is the risk of their
older child or future children also being affected with AGS. You advise them
that:

A. The majority of mutations in AGS occur for the first time in the identified
individual
B. If there is a family history then the chance is 50% of a sibling being
affected
C. The sibling could carry the same gene mutation as the newly diagnosed child
and yet have a very different range and severity of clinical symptoms
D. All of the above

120. A child with AGS is likely to have:

A. A lifetime risk of needing a Liver transplant of 90%


B. Poor growth
C. Severe itching
D. All of the above
E. B and C only

121. Graft-versus-host disease (GVHD) is caused when:

A. Host lymphocytes are depleted


B. Immunosuppressive therapy is increased
C. Donor lymphocytes attack host tissues
D. Host granulocytes attack donor tissues

122. After bone marrow transplantation, present in the graft, either as contaminants or
42

inten- tionally introduced into the host, attack the tissues of the transplant recipient
after perceiving host tissues as antigenically foreign.

A. T Cell
B. T Helper Cell
C. Cytotoxic T Cell
D. Regulatory T Cell

123. The progression of disease in GVHD goes in which order?

A. GI Tract  Liver  Skin


B. GI Tract  Skin  Liver
C. Liver  GI Tract  Skin
D. Liver  Skin  GI Tract
E. Skin  GI Tract  Liver
F. Skin  Liver  GI Tract
43

124. During an upper endoscopy where are you most likely to find
histologic changes associated with GVHD?

A. Esophagus
B. Stomach
C. Duodenum
D. Rectum
E. Liver

125. When looking at a histologic sample, what finding is most suggestive of GVHD?

A. Crypt abnormalities
B. Epithelial cell apoptosis
C. Focal fibrosis
D. Focal reactive surface epithelium
E. Lymphocytic infiltrate

126. A 14 year old boy is referred to you for conjunctival icterus. He feels well and has
no complaints. He volunteers that has noted his eyes “yellow” when he has a cold.
At the visit, there is no icterus evident, no visceromegaly and all his liver function
tests and hemogram are normal. You next:

A. Schedule a liver biopsy


B. Order an ultrasound
C. Order hepatitis serologies
D. Order a 10 day course of low dose phenobarbital
E. Request a screen from UGT1a1 mutation

127. A 15 year old boy with sickle cell disease presents with jaundice and fatigue.
Review of systems reveals intermittent dark urine and icteric conjunctiva over the last
month. Physical exam is unremark- able except for scleral conjunctiva. Differential
diagnosis includes the following except:

A. Hemolysis related to his sickle cell disease


B. Hepatocellular dysfunction
C. Obstruction due to biliary stones
D. Acute cholecystitis

128. A three month-old infant of Canadian descent presents with irritability, mild
jaundice and hepa- tomegaly. Labs are concerning for AST of 130, ALT 170, PT
28, INR 2.4, and serum glucose of 40. Which of the following tests would be most
helpful in confirming the underlying diagnosis?

A. CBC with differential


B. Urine succinylacetone
44

C. Factor 5 level
D. Serum alpha-fetoprotein level
E. Serum albumin

129. Which condition or syndrome is NOT associated with hepatoblastoma?

A. Beckwith-Wiedemann Syndrome
B. Familial Adenomatous Polyposis Syndrome
C. Glycogen Storage Disease 1a
D. Biliary Atresia
E. Cleft Palate
45

130. A 3-week old full term, otherwise healthy formula-fed infant presents with new-
onset poor feeding and fever to 38.3°C. Laboratory findings at presentation are
significant for leukocytosis, as well as a mild transaminase elevation (approximately
1.5 times upper limits of normal). Total bilirubin is el-
evated to 6.7 mg/dL with conjugated bilirubin of 2.3 mg/dL. A bacterial infection is
found on culture analysis. What is the most likely bacterial cause and route of
infection?

A. Staphylococcus aureus infection from skin


B. Streptococcus pyogenes infection from throat
C. Escherichia coli infection from urine
D. Listeria monocytogenes infection from blood

131. A 1-week old infant is noted to have fulminant liver failure with coagulopathy after
presenting with vesicular rash and seizures. Disseminated herpes simplex virus
infection is confirmed by nasopharyn- geal culture and PCR analysis of CSF.
Which of the following is true regarding this infection?

A. Parenteral acyclovir is indicated only in instances of disseminated disease


B. Liver transplantation is contraindicated due to the presence of active viral
infection.
C. Mortality rate is high even with acyclovir therapy
D. The majority of mothers of infants with disseminated HSV will have
symptomatic infection at time of delivery
E. Both C and D.

132. Which serologic profile best reflects the “immune tolerant” state of chronic Hepatitis
B infection?

A. HBsAg positive, HBsAb positive, HBeAg positive, HBeAb positive


B. HBsAg positive, HBsAb negative, HBeAg positive, HBeAb positive
C. HBsAg positive, HBsAb negative, HBeAg positive, HBeAb negative
D. HBsAg negative, HBsAb positive, HBeAg negative, HBeAb positive

133. Which of the following individuals should receive HBV immune globulin
therapy?

A. All infants and children


B. Asymptomatic children with chronic HBV infection
C. Post-HBV exposure, within 24 hours after exposure
D. All household contacts of HBV infected child

134. True or False: Serum Hepatitis C antibody (IgG) is protective and will prevent
recurrent infection upon re-exposure of HCV.
46

135. All of the following hepatitis viruses can result in a chronically infected state,
except:

A. HEV
B. HCV
C. HDV
D. HBV

136. 6 week old presents with neonatal jaundice, direct hyperbilirubinemia, markedly
elevated GGT and pale-colored stools. Possible diagnoses include all of the
following, except:

A. Biliary atresia
B. PFIC2
C. PFIC3
D. A1AT deficiency
47

137. Cholestatic infants should be monitored for deficiencies in many vitamins, including:

A. Vitamin K
B. Folic acid
C. Vitamin A
D. Iron
E. A and C
F. All of the above

138. Which of the following is true regarding autoimmune hepatitis?

A. Children represent a very small subset of patients diagnosed with autoimmune hepatitis
B. Acute hepatitis is the most common presentation
C. Giant cell transformation is a classic histological feature
D. In children, the incidence is equal in males and females
E. HLA A2 is a common association.

139. Patients with Type 1 autoimmune hepatitis:

A. Typically exhibit anti-perinuclear antibody (pANCA)


B. Present at an earlier age than do patients with Type 2 disease
C. Do not have other autoimmune disorders
D. Typically progress to liver transplant in 5-8 years
E. Often demonstrate mildly abnormal cholangiograms

140. An 8 year old girl is being treated for Type 2 AIH with a standard
prednisone/azathioprine regimen. She responded readily with ALT falling from a
high of 650 IU/l consistently. A slow prednisone taper is in progress. With the most
recent dosage decrease, ALT was noted to jump from 75 to 180 with 6-MP
metabolites in the normal range. The appropriate response should be:

A. This is a treatment failure. Consider addition of CSA


B. Consider overlap syndrome and order MRCP
C. Stop current regimen and start MMF
D. Refer for transplantation
E. Increase prednisone temporarily and resume taper when labs improve

141. Which of the following arteries contributes to the blood supply of the pancreas?

A. Left gastric artery


B. Right gastric artery
C. Proper hepatic artery
D. Superior mesenteric artery
E. Inferior mesenteric artery
48

142. The major mediator of meal-stimulated pancreatic enzyme secretion is:

A. Secretin
B. Cholecystokinin
C. Pancreatic polypeptide
D. Peptide YY
E. Somatostatin
49

143. A ten-year-old male presents to the emergency room with a one day history of
vomiting, decreased appetite, and abdominal pain after falling off his bicycle the
day before. Labs reveal an elevated lipase of 3500 and abdominal ultrasound reveals
peripancreatic fluid and pancreatic fullness. Patient is admitted with a diagnosis of
acute pancreatitis. He is initially managed with bowel rest, IV fluids, and IV pain
medications. Within 4 days, he is no longer requiring IV pain medications, is
tolerating a regular diet, and his lipase has decreased to 485. He is subsequently
discharged home. Two weeks later at his follow-up visit, he complains of mild to
moderate diffuse abdominal pain and decreased appetite. He is noted to have
epigastric fullness on physical exam. Lipase is rechecked and is now 1205. Which
of the following steps should be taken next in the evaluation of this patient?

A. Follow serum lipase on a daily basis for the next week


B. Repeat abdominal ultrasound
C. Admit the patient for bowel rest and IV fluids
D. Send genetic testing for CFTR mutations
E. Schedule for an upper endoscopy

144. A 2-year old male presents with a cough and diarrhea. Weight is below the 5th
percentile and height is at the 10th percentile on the CDC growth curve. His
mother states his stool is very foul- smelling. Stool culture for bacteria and
examination for ova and parasites are negative. The parents want the fecal elastase test
performed to rule out pancreatic insufficiency. Which statement is correct to tell the
parents?

A. No sweat test is necessary in this child.


B. A 72-hour fecal fat test is easy to perform.
C. Fecal trypsin testing is more specific than fecal elastase testing
D. Fecal elastase can be artificially lowered if the child has short bowel
syndrome.

145. You are giving a medical student lecture regarding the embryology of pancreatic
development. What statement is correct to present to the students?

A. The ventral and dorsal pancreatic segments fuse at the 4th week of
gestation.
B. Pancreatic function occurs at the 12th week of gestation.
C. Sonic hedgehog protein is the hedgehog protein necessary for
pancreatic cellular differentiation.
D. The dorsal aspect of the pancreatic segment contains the
connection to the common bile duct.

146. A 2 year old female comes to your office with failure to thrive, steatorrhea, small
teeth, nasal ab- normalities, and microcephaly. An outside lab test reveals
50

hypothyroidism. You suspect pancreatic insufficiency due to what diagnosis?

A. Johansson-Blizzard syndrome
B. Homozygous F508 mutation cystic fibrosis
C. Shwachman-Diamond syndrome
D. Congenital Rubella

147. Pancreas divisum:

A. Results from lack of fusion of the ventral and dorsal pancreatic ducts
B. Is easily diagnosed with ultrasound
C. Is associated with ectopic pancreatic tissue
D. Is treated with duodenoduodenostomy

148. Annular pancreas

A. Is best diagnosed by ultrasound


B. Results from the failure of atrophy of the ventral bud
C. Is not associated with congenital malformations
D. Occurs at the third portion of the duodenum
51

149. Which of the following statements regarding Shwachman-Diamond Syndrome is TRUE?

A. Exocrine pancreatic insufficiency is a universal manifestation in all patients with SDS


B. Exocrine pancreatic insufficiency in SDS arises from ductular inflammation
C. Few SDS patients exhibit intermittent neutropenia
D. SDS adolescents commonly have hepatomegaly

150. How would you assess for vitamin D deficiency in a child in whom you suspected
inadequate intake/ inadequate sun exposure?

A. Serum 25 (OH) D level


B. Serum phosphorus
C. Serum 1, 25 (OH) D
D. Serum calcium
E. Urinary 1, 25 dihydroxy D

151. A 4-year-old child with biliary atresia, status post Kasai, presents with a direct
bilirubin of 3.0, an ALT of 230, AST of 340 and GGT of 850. Recently his mother
noticed that he was walking ‘funny’. On examination he is jaundiced, has a large
spleen and liver. You note that his gait is wide and irregular. What vitamin
deficiency is the most likely cause of his ‘funny’ walking?

A. Vitamin A
B. Vitamin D
C. Vitamin E
D. Vitamin K
E. Carnitine

152. You are seeing a family who just moved to the US from Greenland. A family
brings in a 1 year old infant who has failure to thrive, diarrhea, and abdominal
distension. The baby has been breastfed ex- clusively till 6 months of age when solid
foods were introduced. Between 6 and 12 months of age, the weight decreased from
the 50th to the 5th percentile, while the height remained at the 50th per- centile. On
physical examination, the alert but thin infant has a distended abdomen and a perianal
rash. The stool is watery and foul-smelling and has a pH of 3. No parasites are
identified in the stool. Fecal fat and fecal alpha-1-antitrypsin measurements are both
within normal limits. Of the following, the MOST likely diagnosis is:

A. Congenital lactose intolerance


B. Cystic fibrosis
C. Hereditary fructose intolerance
D. Intestinal lymphangiectasia
E. Sucrase-isomaltase deficiency
52

153. A 15 month old female with history of failure to thrive and a mild, persistent diarrhea
is brought to your clinic for further evaluation of a 2 week history of “being
wobbly” and “running into things, especially at night”. Laboratory analysis reveals
the abnormality shown below. What is the patient’s diagnosis?

A. Acrodermatitis enterohepatica
B. Congenital chloride diarrhea
C. Abetalipoproteinemia
D. Syndromic diarrhea
E. Vitamin E deficiency

http://www.wadsworth.org
53

154. Which is the only amino acid malabsorption disorder that presents with
gastrointestinal manifesta- tions of diarrhea, failure to thrive, and possible
hyperammonemic coma with ingestion high protein diet?

A. Cystinuria
B. Lysinuric protein intolerance
C. Hartnup disease
D. Iminoglycinuria
E. Renal tubular acidosis

155. An 18 month old presents to your clinic with history of or diarrhea since about 9
months of age. Dietary history is positive for 3-4 servings of fruit daily and at least
16 to 24 ounces of juice daily. In addition, he drinks 16 ounces of whole milk and eats
a variety of food including chicken, pasta, and vegetables. Parents have noticed some
relationship to fruit and juice intake. What is likely cause of his diarrhea and the
treatment you would recommend?

A. Congenital Glucose –Galactose malabsorption requiring him to be on Ross


Carbohydrate Free formula supplemented with Fructose as his sole
source of carbohydrate
B. Acrodermatitis enterohepatica requiring treatment with zinc
C. Abetalipoproteinemia requiring supplementation with medium chain
triglycerides and fat-soluble vitamins (with especially high doses of
vitamin E)
D. Fructose malabsorption requiring them to take fructose in the form of fruit
and fruit juice out of his diet. They can reintroduce in small amounts when
he is school-aged.
E. Load dependent lactose intolerance with secondary disaccharide
deficiencies.

156. In chronic pancreatic insufficiency, which of the following is absorbed normally?

A. Fat
B. Folic acid
C. Vitamin B12
D. Protein
E. amylopectin

157. Medium chain triglycerides account for 40-50% of the fat content of formulas
fed to low-birth weight infants. Of the following, the BEST explanation for
this practice is that

A. Absorption of MCT is similar to that of butterfat


B. Low-birth weight infants have a large pool of bile acids
C. Low-birth weight infants have normal capacity to synthesize bile acids
D. MCT cause less steatorrhea
54

E. MCT require micelle formation for absorption

158. Lactose enhances the intestinal absorption of which one of the following
nutrients?

A. Calcium
B. Chloride
C. Lipid
D. Potassium
E. Sodium

159. Of the following, the most beneficial formula for patients with gastrointestinal
allergy, short gut or cystic fibrosis is:

A. Protein hydrolysate
B. Carbohydrate free
C. Lactose free
D. Low iron
E. Soy based
55

160. Of the following fatty acids, which must be added to cow milk-based infant formula?

A. Arachidonic acid
B. Linoleic acid
C. Oleic acid
D. Palmitic acid
E. Stearic acid

161. In regards to gastric emptying, which of the following statements is TRUE?

A. Solids empty faster after vagotomy


B. Proteins delay emptying more than lipids
C. Solids are emptied in an exponential pattern
D. Liquids are emptied faster than solids
E. Cholecystokinin accelerates emptying

162. An Asian-American family is concerned that their 2-month-old infant’s abdominal


distention is due to lactose intolerance. She is intermittently fussy, with frequent
vomiting and poor weight gain, but no diarrhea. She takes a standard cow’s-milk
based infant formula. Which of these options is the most appropriate first
intervention?

A. Trial of a soy-based formula


B. Trial of lactase supplementation
C. Upper GI series
D. IgE RAST testing for cow’s milk
E. Stool pH and reducing substances

163. An adolescent patient with recurrent abdominal pain has a duodenal biopsy
showing low lactase activity, but lactose breath hydrogen test is normal. What is one
likely explanation for these conflict- ing results?

A. His pain is strictly functional in nature.


B. He recently completed a course of antibiotics.
C. He suffers from sucrase-isomaltase deficiency.
D. He ate a high-fiber meal prior to breath testing.
E. He has cow’s-milk protein intolerance.

164. After three weeks of nursing a new mother develops fissured nipples and has to use a
breast pump. At the baby’s one month check-up the mother expresses concerns that
when she begins the pump- ing process the milk seems watery and she is worried
about the baby’s nutrition. You assure her that:

A. Foremilk is normally thinner and primarily serves to assure the baby’s hydration
B. The milk has the normal whey : casein ratio of 30 : 70
56

C. Consuming a diet higher in fat will increase the lipid concentration and nutrition of her
milk
D. Consuming more cow milk in her diet will increase the carbohydrate content of her
milk
E. It provides sufficient vitamin D for her baby

165. Which of the following statements regarding nutritional evaluation in children is TRUE?

A. Marasmus is defined as severe malnutrition with nutritional edema


B. Prealbumin is a sensitive marker of chronic malnutrition
C. Iron deficiency anemia is the most common nutritional deficiency in children
D. Folate and vitamin B12 deficiency cause microcytic anemia
E. The first step in management of acute moderate malnutrition is always aggressive
hydration
with intravenous fluids
57

166. A 10 year old female and her mother present to your office with dietary questions.
The patient has just recently decided to follow a vegetarian diet and her mother is
concerned that this diet will be inadequate for her. Supplementation of which
vitamin is recommended for this patient?

A. Vitamin C
B. Thiamine (Vitamin B1 )
C. Folate (Vitamin B9 )
D. Cobalamin (Vitamin B12 )
E. Niacin (Vitamin B3 )

167. A 12 year old male with Crohn’s disease obtains a chest x-ray prior to the start of
Remicade therapy. CXR reveals cardiomegaly. Patient is referred to cardiologist,
and Echo reveals cardiomyopathy. Deficiency of which of the following
micronutrients has been associated with cardiac complications?

A. Selenium
B. Iron
C. Copper
D. Vitamin C
E. Zinc

168. What is the most appropriate IV formulation within the first 24 hrs of life for a
patient born at 28 week gestation weighing 1,100 grams?

A. Normal Saline
B. 5% Dextrose with electrolytes
C. 5% Dextrose with amino acids
D. 5% Dextrose

169. What is the caloric requirement for a healthy 13 year old male?

A. 100-110kcal/kg/day
B. 70-90 kcal/kg/day
C. 20-30 kcal/kg/day
D. 45-55 kcal/kg/day

170. A 16 year old boy with a Body Mass Index (BMI) of 47, Obstructive Sleep Apnea,
and Type 2 Diabetes Mellitus is interested in a laparoscopic Roux-en-Y Gastric
Bypass surgery for weight reduction. A true statement regarding this procedure is:

A. Patients undergoing Roux-en-Y Gastric Bypass may experience paradoxical


weight gain
B. Post-surgery follow-up pediatric care should be limited to a single office visit
C. Roux-en-Y Gastric Bypass may lead to iron deficiency and other micronutrient
deficiencies
58

D. A post-surgical decrease in insulin resistance is not seen until the BMI decreases
by 30%
E. The stomach is removed completely during the Roux-en-Y Gastric Bypass

171. Which of the following statements regarding the ketogenic diet is FALSE?

A. The ketogenic diet is high in fat content and low in protein and
carbohydrates.
B. Ketones have a direct anti-seizure effect on the brain.
C. The ketogenic diet is recommended for children with disorders of fatty acid
oxidation.
D. Patients following a ketogenic diet require vitamin and mineral
supplementation.
59

172. A twelve-year-old girl who was diagnosed with cystic fibrosis and pancreatic
insufficiency as an infant presents for her annual evaluation. Her BMI has fallen from
the 60th percentile to the 40th percentile for age. She is receiving pancreatic enzyme
replacement therapy at a dose of 2,500 units of lipase per kg per meal. Which is the
most appropriate next step in management?

A. Reassure the patient and family that a BMI at the 40th percentile is adequate.
B. Provide oral nutritional supplements and conduct a full nutritional and behavioral
evaluation.
C. Increase her dose of pancreatic enzymes to 3,000 units of lipase per kg per meal.
D. Refer the patient for surgical gastrostomy tube placement.

173. A three-year-old boy presents for evaluation of diarrhea. His mother states that he
has had up to 6 watery bowel movements per day for one week. He also has a fever
and nasal congestion. Which of the following is the most appropriate
recommendation?

A. Restrict the patient’s diet to clear liquids until the diarrhea resolves.
B. Remove all sources of lactose from the patient’s diet until the diarrhea resolves.
C. Begin a diet consisting of only bananas, rice, applesauce, and toast.
D. Encourage oral rehydration with fluids followed by an unrestricted, age-appropriate
diet.

174. A 2 year old with history of short bowel syndrome (secondary to necrotizing
enterocolitis), TPN dependence, and TPN induced liver disease presents to your clinic
with a history of persistent anemia over the last 6 months. On exam, he is afebrile,
pale, and significantly jaundiced. He has hepato- splenomegaly. He has a Mickey
button in place and well-healed incision from his previous surger- ies. Labs are
significant for a total bilirubin of 8 mg/dL and a direct bilirubin of 4 mg/dL. His AST
54 units/L and ALT 68 units/L, Albumin 2.5 g/dL, INR 1.5. His hemoglobin is 7.5
g/dL and MCV 70. What micronutrient deficiency has resulted in his persistent
anemia?

A. Selenium
B. Copper
C. Niacin
D. Folate

175. A 6 year old male develops a duodenal hematoma after routine endoscopy for
evaluation of his chronic diarrhea and failure to thrive. Biopsies shows subtotal
villous atrophy, atrophic villi, enlarged crypts with large amounts of inflammatory
cells. The patient is diagnosed with celiac disease Marsh 3b. What vitamin
deficiency contributed to the formation of the duodenal hematoma during the
endoscopy?
60

A. Vitamin A
B. Vitamin D
C. Vitamin E
D. Vitamin K

176. A healthy 1 week old male presents to your clinic for evaluation of a questionable
anal abnormality noted by the pediatrician at 2 days of age. In turns out, everything
is normal – however, the mother asks you about vitamin supplementation,
specifically of vitamin D, as she is completely breastfed infant. When should
vitamin D supplementation begin and how much?

A. Immediately, 200 IU till patient is 2 months old then 400 IU.


B. Immediately, 400 IU.
C. Not till 2 months of age, 400 IU.
D. Never, as long as patient is breastfed he will get all the vitamin D he needs from mom.

177. Which of the following patients does not require folate supplementation?

A. A 10 year old male with inflammatory bowel disease on


methotrexate for maintenance therapy.
B. A 2 year male with seizure disorder on phenytoin.
C. A 6 year old female with cystic fibrosis.
D. A 12 year old female with eosinophilic colitis being tried on sulfasalazine therapy.
61

178. A 6 month old male develops a weepy, crusted dermatitis around the eyes, nose,
mouth, diaper area, hands and feeds about 4 weeks after being weaned from breast
milk to formula. He recently devel- oped a watery diarrhea and has stopped gaining
weight over the last month. On exam, he appears listless. His hair is sparse. What is
there is fine and lightly pigmented. What nutritional deficiency has resulted in this
patient’s current condition?

A. Copper
B. Aluminum
C. Zinc
D. Molybdenum

179. A 6 month old female with a history of short bowel syndrome secondary to multiple
intestinal atre- sias presents with complaints of increased work of breathing, poor
feeding, and cough over the last week. She is diagnosed with RSV. It is noted on chest
x-ray that her heart size is significant enlarged and that on exam her heart rate seems
irregular. Further evaluation is concerning for evolving car- diomyopathy. The ICU
physician is concerned that the patient receives her TPN from a new, small emerging
pharmaceutical company by her home in rural Texas. What micronutrient deficiency
is the ICU physician concerned about?

A. Selenium
B. Copper
C. Pyridoxine
D. Iodine

180. A 2 year old male with diagnosis of abetalipoproteinemia moves to your area. He
has recently been diagnosed and comes to your office for establishment of care.
The mother’s chief complaint for this visit is that he stumbles while walking despite
walking with a wide-gait. The stumbling is much worse at night. In addition to a
single supplementation with AquaDEK – which of the following vitamins should
be supplemented further that the standard dosage in AquaDEK.

A. Vitamin A
B. Vitamin D
C. Vitamin E
D. Vitamin K

181. Considered one of three major nutrient deficiencies in the world by the World Health
Organization, this deficiency is the primary cause of blindness in children in the
developing world.

A. Iodine
B. Iron
C. Zinc
62

D. Vitamin A

182. A 16 year old adolescent male has undergone resection of the terminal ileum
because of an ileal stricture. Of the following nutrients, which is MOST likely to
become deficient in this patient?

A. Folic acid
B. Thiamin
C. Pantothenic acid
D. Cyanocobalamin
E. Vitamin K
63

183. A 15 year old boy with well–controlled ulcerative colitis is noted to have a
hemoglobin of 10.2 gm%. He denies any symptoms of bowel disease. He has a good
appetite and eats a varied diet. He takes sulfasalazine and azathioprine for
maintenance therapy. Which of the following vitamin levels is likely to be
abnormally low.

A. Folate
B. Thiamine
C. Vitamin C
D. Cobalamin

184. A healthy 13 y/o male is referred to GI clinic due to elevated alkaline phosphatase
which is 2-3 times the upper limit of normal. Aspartate aminotransferase, alanine
aminotransferase, total bilirubin, and albumin are normal. What should be your next
step to assess for cholestatic disease?

A. Nothing. This is definitely not hepatobiliary disease as this patient is a rapidly


growing male and elevated alkaline phosphatase is due to increased bone
activity.
B. Liver Ultrasound
C. Obtain gamma glutamyltransferase or 5’-nucleotidase
D. Repeat Alkaline Phosphatase, AST,ALT, and total bilirubin in 6 months
E. HIDA scan

185. The endoscopic definition for hiatal hernia involves:

A. Ascension of the Z line more than 2 cm above the hiatus


B. Ascension of the Z line 3 cm above the hiatus
C. Presence of folds in the distal esophagus
D. Open an incompetent hiatus.

186. Six hours after an upper endoscopy a 6 year old develops abdominal pain
and vomiting. Upper abdomen is tender, KUB is unremarkable. Best
next step:

A. Pain medication and antiemetic


B. Imaging study
C. Surgery consult
D. Reassure mom and follow patient in 1 week

187. A 5 year old with abdominal pain and diarrhea is undergoing combined breath
hydrogen/methane testing. A standard dose of lactulose is given while the patient is
NPO. Results from the study show an increase in hydrogen production and peak 3
hours after ingestion of the test dose. No change in methane production is noted.
64

The most correct interpretation of these results is:

A. The results indicate small bowel bacterial overgrowth


B. The results indicate lactose intolerance
C. The results are normal
D. The results are inadequate for interpretation
E. The results indicate fast intestinal transit time

188. Which of the following represents the most distinctive feature of the duodenum, as
compared to the rest of the small bowel?

A. Generally taller and more slender villi


B. Increased proportion of goblet cells within the epithelium
C. Abundance of lymphoid aggregates localized within the lamina propria
D. Predominance of tubuloalveolar glands known as Brunner’s glands
throughout the mucosa and submucosa
E. The plicae circularis are the tallest and most numerous within the small bowel
65

189. Which of the following could distinguish celiac disease from autoimmune
enteropathy on small bowel biopsies?

A. The degree of villous blunting is far less in autoimmune enteropathy


B. An increased number of plasma cells in the lamina propria in autoimmune
enteropathy
C. The lack of increased intraepithelial lymphocytes in autoimmune
enteropathy
D. The patchy nature of scalloped and ulcerated mucosa seen in autoimmune
enteropathy
E. The increased presence of submucosal eosinophils in autoimmune
enteropathy

190. Fecal elastase may be used to diagnose chronic pancreatitis. Which of the following
conditions does not result in abnormal levels of fecal elastase?

A. Celiac disease
B. Crohn’s disease
C. Diarrheal illness
D. Primary sclerosing cholangitis
E. Short bowel syndrome

191. Which of the following tests can detect mild pancreatic dysfunction?

A. Lundh test meal


B. Fecal elastase
C. Secretin stimulation test
D. Serum immunoreactive trypsinogen
E. Fecal fat measurement

192. An contrast UGI reveals a corkscrewed duodenum ending blindly


suggesting the possible diagnosis of:

A. Duodenal web
B. Duodenal protein enteropathy
C. Malrotation
D. Congenital microcolon of disuse
E. Annular pancreas

193. The radiological signs of necrotizing enterocolitis can include

A. pneumatosis intestinalis, pneumoperitoneum and hepatic pneumatosis


B. Colonic “thumb printing”
C. Punctate adrenal calcification
D. A and B
66

E. A and C

194. You see in follow-up a 12 y/o boy who sustained a bicycle handlebar injury 6 weeks
prior. He reports he is feeling well with no fever, vomiting or pain, but on
physical examination, you notice he grimaces when you palpate a smooth fullness
just to the left of the umbilicus. You next order:

A. Plainfilm of the abdomen


B. Contrast UGI
C. US of the abdomen
D. CAT scan
E. Tc99m scan
67

195. A 5 year old boy is referred to your clinic for the evaluation of chronic diarrhea and
failure to thrive. You would like the parent to perform a 3 day collection for fecal
fat. Which of the following is NOT true regarding this test?

A. Steatorrhea is present in a child or adult if more than 7% of ingested fat is excreted.


B. It allows one to determine whether fat malabsorption is pancreatic or
nonpancreatic in nature.
C. The test involves the meticulous weighing of food and careful dietary
records in order to calculate mean daily fat intake.
D. Every single stool during a 3 day period should be collected.

196. Limitation of stool electrolyte testing include:

A. Can only be done on solid stool


B. Special media are needed
C. Subjects often ingest non-absorbable sugars
D. No standard exists for electrolytes in stool therefore results must be used in a
comparative values to prior or future stools
E. There are few genetic tests available for differentiation of congenital causes of diarrhea

197. A 3 year old male with a past medical history significant for small bowel resection
including the terminal ileum following necrotizing enterocolitis at 2 weeks of
age presents with perianal excoriation and diarrhea. Which antidiarrheal
agent would be most appropriate?

A. Bismuth subsalicylate
B. Cholestyramine
C. Loperamide
D. Octreotide
E. Clonidine

198. All of the following are true EXCEPT:

A. Pruritus related to cholestatic liver disease is thought to be due to centrally


mediated causes related to endogenous opioid neurotransmission.
B. Diphenhydramine can help ameliorate pruritus in cholestatic liver disease
C. Opiate antagonists is the first line treatment for pruritus associated with
cholestasis in children
D. Cholestyramine is a hydrophilic, water insoluble anion-exchange resin that
binds bile acids, preventing their absorption through the enterohepatic
circulation

199. Regarding blood transfusions in children, which statement is false?

A. Blood volume in children varies more dramatically in the first year of life than in latency
68

B. A unit of pRBCs has a hematocrit of 90%


C. Transfusion in children raises hemoglobin approximately 2 to 2.5 g/dL for
every 10 ml/kg of pRBCs given.
D. Hematocrit equilibrium post transfusion is generally evident within 24 hours
E. Newborns have a blood volume of about 85 ml/kg

200. What is the gold standard for measuring dehydration?

A. Serum electrolytes
B. BUN/Cr
C. Urine specific gravity
D. Percent loss of body weight
E. Heart rate
69

201. A 7 month old boy with biliary atresia underwent liver transplantation. His
medications include tacro- limus and prednisone. Two months postoperatively he is
noted to have elevated transaminases. An infectious work up is initiated and a liver
biopsy is performed, which shows dense periportal lympho- cytic and eosinophilic
infiltration with endotheliitis and bile duct damage. One week after starting high
dose steroids his transaminases are still elevated. Which of the following agents may
be consid- ered in this situation?

A. Tacrolimus at an increased dose


B. Cyclosporine
C. MMF
D. OKT3

202. A 13 year old girl has a liver transplantation for autoimmune hepatitis and is started
on prednisone and tacrolimus in the post operative period. Her hospital course is
complicated by a central line infec- tion and adenovirus infection. She is discharged
home on Prednisone, tacrolimus and MMF. Several months after discharge she
develops pharyngitis and is treated with an antibiotic.

One week later, her labs are as follows: Albumin - 4.1 mg/dL; total protein - 6.3
mg/dL; total biliru- bin – 2.0 mg/dL; direct bilirubin - 1.0 mg/dL; ALT – 35 IU/L,
AST – 22 IU/L, GGT – 45 IU/L; Na: 135 mEq/L; K: 6.8 mEq/L; CL – 110 mEq/L;
CO2 – 24 mEq/L; glucose – 120mg/dL, BUN – 45 mg/dL; Cr -
2.3 mg/dL; Mg – 1.1

Which antibiotic is most likely to be the cause of these abnormal laboratory


findings?

A. Penicillin
B. Amoxicillin
C. Erythromycin
D. Ceftriaxone

203. A 3 year old girl underwent liver transplantation for hepatoblastoma. Her post
operative course is complicated by biliary leak and multiple episodes of ascending
cholangitis. She receives 10 days of parenteral antibiotics with one week of
fluconazole. She is discharged home on Tacrolimus, Predni- sone and
Multivitamins.

Two weeks later laboratory assessment shows Albumin – 3.5 mg/dL; total protein -
6.8 mg/dL, total bilirubin - 5.0 mg/dL, direct bilirubin 3.2 mg/dL, ALT – 375 IU/L,
AST – 450 IU/L, ALP – 315 IU/L. A liver biopsy is suggestive of rejection.

Which of the following factors may be responsible?


70

A. Tacrolimus toxicity
B. Discontinuation of Fluconazole
C. Intercurrent viral infection
D. Incorrect HLA typing of donor

204. A twelve year old male who was diagnosed with Crohn disease 3 years ago has
been on q8 wk Inf- liximab infusions for one year. The parents reported that he
developed erythematous, vesicular, skin lesions in the right posterior rib cage area.
The patient was afebrile but complained of skin tingling sensation. According to
above history, your recommend is:

A. Observation
B. Start 3rd generation cephalosporin
C. Discontinue Infliximab
D. Start p.o acyclovir
E. Admit for IV acyclovir
71

205. A 16 year old female has been on maintenance Infliximab mono therapy for 3 years.
She was previ- ously treated with 6-mercaptopurine. She presents with night
sweats and splenomegaly.

What rare life threatening tumor must you be concerned about:

A. Burkitt’s Lymphoma
B. Hepatosplenic T-cell Lymphoma
C. Acute Myelogenous Leukemia
D. Hodgkin’s Lymphoma
E. Signet Ring Carcinoma

206. A 13-year-old female has a draining peri-rectal fistula. You choose to use infliximab
therapy to heal the fistula. What problem with using the therapy must you think
about before starting therapy?

A. Walled off abscess formation


B. Systemic infection
C. Increased pain
D. Alternative site of fistulization
E. Constipation

207. A 17-year-old male has Crohn disease and has responded to Infliximab monotherapy
for two years without complication. He has been on 5 mg/kg/dose every 8 weeks. He
develops 6 bloody stools per day at 7 weeks and a trough level is obtained. It is
undetectable. Stool studies for infection and CMV PCR of colonic tissue is
negative. Your best choice to treat this patient is:

A. Change to Adalimumab
B. Obtain a HACA
C. Increase to 10 mg/kg/dose or change interval to 4 weeks
D. Start prednisone
E. Start methotrexate or 6-mercaptpurine

208. A 7 year old female with cystic fibrosis is seen in follow up. She has not lost
weight but has had little weight gain over the last six months despite adequate
caloric intake. She has 2 stools per day, and sometime sees oil droplets. She has no
other symptoms. She takes 2000 units/kg of pancreatic enzymes with all meals.
The next best step in management is:

A. Increase the pancreatic enzyme dose to 4000 units/kg before each meal
B. Offer reassurance that no intervention is needed unless there is weight loss
C. Add loperamide to her daily medications
D. Add a PPI to her daily medications
72

209. You are interested in assessing the possibility that a child has steatorrhea. What
question would you not ask the parents?

A. Are the stools hard to wipe off during a diaper change?


B. Do the stools float?
C. Do the stools smell rancid?
D. Are the stools difficult to flush?
E. Are the stools loose and oily?

210. A 4 month old infant presents to the emergency department with a four day history
of progressive lethargy, bradycardia, loose stools, and diminished deep tendon
reflexes. She is afebrile and has no rhinorrhea or exanthema. History revealed that her
parents were mixing antacids into her bottles to help with reflux symptoms. Her
symptoms are MOST likely explained by

A. Hypermagnesemia
B. Hypomagnesemia
C. Hyperkalemia
D. Hypokalemia
E. Hypoglycemia
73

211. In a 12 year old boy with Zollinger-Ellison Syndrome, which of the


following is the BEST medical therapy?

A. Sodium bicarbonate
B. Nizatidine
C. Lansoprazole
D. Famotidine
E. Calcium carbonate-ranitidine compound

212. Probiotics have been shown to be of benefit in

A. Ulcerative colitis
B. Crohn’s disease
C. Both
D. Neither

213. In regard to the colonization of the newborn gut, which statement is false?

A. It is initially colonized by facultative maternal vaginal and fecal flora


consisting of
Streptococcus, Enterococcus and Coliform genera
B. C-section babies are initially colonized by Klebsiella, Enterobacter and
Clostridia genera
C. Breast fed babies have higher concentrations of Bifidobacterium and
Lactobacillus genera
D. Formula fed babies have higher concentrations of Bifidobacterium and
Lactobacillus genera
E. After 10 days both vaginal and C-section babies share the same flora

214. Probiotics are live microorganisms, that when consumed in adequate numbers, confer
a health ben- efit to the host. They also:

A. Lower intestinal pH and decrease the adherence of pathologic bacteria


B. Increase the production of short chain fatty acids and increase motility
C. Contribute to the production of riboflavin and folate
D. All above statements are false
E. All above statements are true

215. Misoprostol is thought to treat and prevent NSAID-associated peptic


ulcers by which of the following mechanisms:

A. Decreasing gastric acid secretion


B. Increasing mucosal resistance to injury by decreasing bicarbonate secretion
C. Providing mucosal protection by decreasing mucosal blood flow
D. Blocking histamine receptors
74

E. Activating adenylate cyclase in parietal cells.

216. Which of the following is true concerning the properties and action of Sucralfate on
gut mucosa?

A. Sucralfate is a sulfated aluminum hydroxide that selectively binds to ulcers and


erosions.
B. Sucralfate is a sulfated magnesium hydroxide that selectively binds to ulcers
and erosions.
C. Sucralfate binds to ulcers and erosions and is activated by non-acidic
conditions.
D. Sucralfate should be taken together with antacids for optimal binding
activity.

217. A dialysis patient being treated for a gastric ulcer presents with bone pain, mental
status changes and proximal muscle weakness. Which medication was
responsible for these side effects?

A. Ranitidine
B. Cimetidine
C. Sucralfate
D. Terfenadine
75

218. Which one of these drugs have reduced bioavailability when co-administered with
Sucralfate?

A. Ciprofloxacin
B. Erythromycin
C. Aspirin
D. Prednisone

219. You are called at 3 am from the ER about a 3 year old female who has a history of
severe constipa- tion. The ER physician performed a history and a physical exam and
there is no evidence of constipa- tion or any acute problem but the mother demanded
an X-ray of the abdomen to be taken since “that is how her doctor diagnosed the
constipation”. The X-ray showed scattered stool around the colon. Thereafter the
mother insisted on an admission for a clean-out. The ER physician is asking you,
would you admit this patient under your service for a Go-Lytely clean-out?

A. The abdominal X-ray and the history/physical exam are enough evidence that
this patient has constipation and needs a clean-out.
B. Parental insistence on admission may be reasonable given what appears to be a
knowledge- able parent.
C. The history/physical exam and X-ray do not support a diagnosis of
constipation. You will request history of admissions, ER visits and test
results before deciding.
D. You recommend no admission; there is no indication to admit the patient.
Refer the patient and the mother to a psychiatrist.
E. You Recommend a CT scan of the abdomen since is more reliable than X-
ray to diagnose constipation.

220. You are consulted for admission of a 5 year old male who has G-tube issues. The
patient has a his- tory of severe recurrent asthma and is followed by the pulmonology
service. He was diagnosed with GERD for which he received a Nissen-
fundoplication and G-tube. Recently, he started spitting up again at home but it has
not been witnessed by nurses. Mother believes the vomiting is caused by
fundoplication failure and a blocked G-tube and that an endoscopy is the best test to
diagnose failed fundoplication. At the end of the interview she gives you a gift and
thanks for your time and asks when you would do the scope and if you can also
change her G-tube to a G-J tube since this will help with the vomiting. The next
best course of action is:

A. Accuse mother of Munchausen Syndrome by Proxy and call hospital security


to protect the child and the mother
B. Ask the mother to go out of the room to interview the child alone and ask
him questions about child abuse
C. Immediately leave the room and call Child Protective Services to take him into
custody.
76

D. Call physicians and staff involved in his care to get details of the history.
Obtain medical records and document details of the encounter.
E. Schedule upper endoscopy and G-J tube placement as soon as possible. Consult
surgeon for repeat fundoplication.

221. Infant rumination best responds to

A. Anticholinergics
B. Soy formula
C. NG tube feedings
D. Frequent holding and social interaction
E. PPI therapy
222. Clostridium difficile colitis is found in what percentage of children with antibiotic
associated diarrhea: A. 5%-10%
B. 15%-20%
C. 25%-30%
D. Greater than 30%
77

223. Which of the following statements regarding congenital chloridorrhea is true?

A. Stool Cl- level less than 90 mmol/L


B. Results in metabolic alkalosis
C. Results in metabolic acidosis
D. First detected after the patient first gets solid food
E. Due to defective chloride-bicarbonate exchange in the colon

224. Which of the following conditions is usually suspected after protracted diarrhea
begins after the first few weeks of life?

A. Microvillous inclusion disease


B. Tufting enteropathy
C. Congenital chloridorrhea
D. Autoimmune enteropathy
E. Enterocyte heparan sulfate deficiency

225. Erythromycin when used as a prokinetic:

A. Stimulates motilin
B. If given to a one week old increased the risk of pyloric stenosis
C. Can safely be given to a nursing mother
D. All are true

226. Pill esophagitis is commonly associated with these medications except:

A. Doxycycline
B. Tetracycline
C. Bismuth subsalicylate
D. Theophylline
E. KCl

227. What is the most common finding in a 15 year old male with IgA deficiency?

A. Recurrent infections with E. histolytica


B. Celiac disease
C. Inflammatory bowel disease
D. None of the above

228. Which immunodeficiency should be suspected in a 6yo male with recurrent


infections and perianal disease with gastric granulomas?

A. Leukocyte adhesion defect


B. Severe combined immunodeficiency
C. X-linked chronic granulomatous disease
78

D. IgA deficiency

229. Regarding IgA deficiency, which statement is false?

A. IgA deficiency is the most common form of immunodeficiency.


B. Most patients with confirmed IgA deficiency are asymptomatic.
C. There is variable penetrance of the IgA deficiency defect but all cases have
<2 SD of normal IgA levels for age.
D. Sinopulmonary infection is the most common manifestation, followed by
recurrent gastroin- testinal parasitosis of various genera including protozoa,
flagellate, nematodes and platyhel- minthes.
E. The diagnosis is most accurate after the age of four years.
79

230. You are consulted for a jaundiced 9 y/o male who is 3 weeks post bone marrow
transplant. He is afebrile and has right upper quadrant tenderness. All viral serology
is negative except HBsAg. ALT is 78, AST is 59, TB is 8.2 with a direct of 3.9. AP is
181. His coagulation studies are normal. His RUQ ultrasound report is
unremarkable. You suspect hepatic GVHD and advise beginning with

A. Treatment with steroids


B. Treatment with steroids plus ursodeoxycholic acid
C. Liver biopsy to confirm the diagnosis
D. HIDA scan
E. Bone marrow aspiration for culture.

231. A 14 year old female with poorly controlled hyperthyroidism is evaluated for
diarrhea. Which is the following is not a possible cause for diarrhea is in this
patient?

A. Decreased bile output


B. Decreased trypsinogen levels
C. Small intestinal bacterial overgrowth
D. Lactase deficiency
E. Accelerated oral to cecal transit

232. Nonalcoholic steatohepatitis is associated with which one of the following conditions?

A. Autoantibodies against liver antigens


B. Chronic hepatitis C
C. Insulin-dependent diabetes mellitus
D. Iron overload
E. Copper overload

233. Which of the following is not associated with hypothyroidism?

A. Ascites
B. Esophageal compression
C. Delayed gastric emptying
D. Delayed small bowel transit
E. Pernicious anemia

234. Graft-versus-host disease (GVHD) is caused when:

A. Host lymphocytes are depleted


B. Immunosuppressive therapy is increased
C. Donor lymphocytes attack host tissues
D. Host granulocytes attack donor tissues
80

235. The ethical precepts described in the Belmont Report include adherence to the
principles of Autonomy, Beneficence, Non-maleficence and:

A. Consent
B. Self-determination
C. Justice
D. Morality

236. In human subjects research involving minors, ethical considerations include:

A. Assent must be obtained from all children.


B. Clinical trials may not involve a greater than a minor
increase over minimal risk to individual subjects.
C. Investigator conflicts of interest are permissible, as long as they are reported to the local
IRB.
D. A subpart D (Code of Federal Regulation) determination is required for all studies.
81

237. Choose the technique used to assess a statistical models assumptions:

A. Specificity
B. Cost-effective analysis
C. Sensitivity analysis
D. Validity
238. Match each word with the corresponding answer
choice: Validity A. Measure of
agreement
Reliability B. Measurement of what is intended
Accuracy C. Measure gives same results on repeated trials
Precision D. Measure of the internal consistency of
a test Kappa statistic E. How close measurement is to actual
value Cronbach’s alpha F. How close repeated measures
are to each other

239. If a researcher wants to increase the power of the study from 80% to 90% but
keep all other parameters the same, the original sample size will:

A. Decrease
B. Stay the same
C. Increase
D. Unable to tell

240. A control group compared to the intervention group should vary by:

A. Gender
B. Age
C. Education
D. None of the above

241. Analysis of variables by the original group assignment regardless if they


remained or adhered to that group is called:

A. Intention to treat analysis


B. Per protocol analysis
C. Post-hoc analysis
D. Cost-effectiveness analysis

242. The measure of how results of a study can be generalized to the population as a
whole:

A. External validity
82

B. Accuracy
C. External variation
D. Power

243. A 17-year-old boy who has evidence of Crohn’s disease in the terminal ileum
develops severe radiating inguinal and scrotal pain.

The MOST likely visceral source for the referred pain in this patient
is the

A. Appendix
B. Diaphragm
C. Gallbladder
D. Small bowel
E. Ureter
83

244. Which of the following represents the MOST likely mechanism for peptic injury
associated with chronic administration of nonsteroidal anti-inflammatory
drugs (NSAIDS)?

A. Generalized ischemia of the gut


B. Genetic predisposition
C. Inhibition of prostaglandin
D. Promotion of growth of Helicobacter pylori
E. Topical caustic injury

245. A 6 year old girl has had abdominal pain and nonbilious vomiting for 8 hours. History
reveals cough and fever for the past 3 days. Findings on physical examination include
temperature, 39°C (102.2°F); tachypnea; toxic appearance; diffuse, voluntary
guarding; and quiet bowel sounds.

Of the following, the examination MOST likely to establish the etiology of the
abdominal pain and fever in this patient is a(n):

A. Abdominal radiograph
B. Chest radiograph
C. Complete blood count
D. Rectal examination
E. Upper gastrointestinal series

246. A 9-year-old girl has the height age of a 7-year-old and the bone age of
a 6-year-old. Among the following, the MOST likely cause of her
short stature is

A. Achondroplasia
B. Hypothyroidism
C. Malnutrition
D. Normal variant short stature
E. Silver-Russell syndrome

247. An antral or pyloric web (diaphragm) is considered in the differential diagnosis of a 6-


month-old girl with failure to thrive syndrome and nonbilious vomiting.

The MOST specific study for diagnosing this condition is

A. Gastric emptying study


B. Plain abdominal radiography
C. Ultrasonography
D. Upper gastrointestinal contrast study
E. Upper gastrointestinal endoscopy
84

248. A 10-year-old child has had intermittent diarrhea and weight loss over
the past year. A TRUE statement regarding testing with guaiac or
orthotolidine for occult blood in this patient’s stool is:

A. Microscopic examination of the stool is a better test for detecting occult blood
B. Negative results exclude lower gastrointestinal bleeding
C. Positive results confirm the presence of occult blood
D. These tests detect peroxidase activity in hemoglobin
E. These tests quantitate the amount of hemoglobin in the stool
85

249. An 18-year-old girl who is taking tolmetin for juvenile rheumatoid arthritis
develops gastritis. Which of the following medications would have been MOST
likely to prevent the development of peptic disease in this patient?

A. Antacids
B. Corticosteroids
C. H2-blockers
D. Misoprostol
E. Sucralfate

250. A 3 year old boy with acute lymphoblastic leukemia in hematologic remission is
receiving vincristine, methotrexate, and 6-mercaptopurine. He develops abdominal
pain and distention and nausea with- out fever or diarrhea.

These findings are MOST likely caused by

A. Escherichia coli gastroenteritis due to granulocytopenia


B. Intestinal candidiasis due to lymphopenia
C. Intestinal mucosal ulcerations due to methotrexate toxicity
D. Necrotizing enterocolitis due to 6-mercaptopurine toxicity
E. Reduced intestinal motility due to vincristine toxicity

251. A term infant is born with gastroschisis that is repaired at birth. The infant is
placed on total parenteral nutrition.

The serum level of which of the following is likely to become abnormal


FIRST?

A. Alkaline phosphatase activity


B. Bile salt concentration
C. Direct bilirubin concentration
D. Indirect bilirubin concentration
E. Transaminase activity

252. A previously healthy 12-year-old boy is icteric. Physical examination reveals a


noncommunicative, moderately ill-appearing boy who has an enlarged, soft, tender
liver and ascites; there is no sple- nomegaly. Pitting edema of the ankles and sacral
area and scattered bruising of the extremities are noted.

Among the following, the MOST critical set of studies to include in the initial
laboratory evaluation is

A. Hepatitis A and B serologies


B. Hepatocellular enzyme activities
C. Prothrombin time and partial thromboplastin time
86

D. Total and fractionated serum bilirubin levels


E. Total serum protein and albumin concentrations

253. A 2 year old girl has a history of recurrent pneumonia, short stature, and failure to
thrive. Studies reveal absolute neutropenia and thrombocytopenia, normal sweat
chloride concentration, and me- taphyseal dysplasia of the head of the left femur.
The MOST likely diagnosis for this patient is:

A. Alagille-Watson syndrome
B. Shwachman-Diamond syndrome
C. Sideroblastic anemia
D. Trypsinogen deficiency
E. Wiskott-Aldrich syndrome
87

254. Of the following, which condition is the MOST common presentation of


infection with enterotoxigenic Escherichia coli?

A. Dysentery-like illness with fever


B. Hemolytic-uremic syndrome
C. Hemorrhagic colitis
D. Self-limited illness with watery stools and cramps
E. Severe diarrhea and dehydration

255. A 7-day-old breastfed infant born at term has had decreased appetite, irritability, and
vomiting for 24 hours. On physical examination, the infant appears listless.
Respiratory rate is 40/min; heart rate, 160/min; and blood pressure, 68/38 mm Hg. The
skin and conjunctiva are icteric but no other abnor- malities are noted. Laboratory
studies reveal: hemoglobin, 12 gm/dL; total bilirubin, 16 mg/dL; and direct bilirubin,
8 mg/dL. Urinalysis is negative for reducing substances.

Of the following, the MOST likely diagnosis is

A. Bacterial sepsis
B. Blood group incompatibility
C. Breast milk jaundice
D. Hypothyroidism
E. Intrauterine infection

256. Which of the following metabolic alterations is most commonly seen with re-feeding syndrome?

A. Hyperlipidemia secondary to increased serum ketone bodies


B. Wernicke’s encephalopathy secondary to thiamine deficiency
C. Severe hypophosphatemia affecting myocardial and respiratory function
D. Hypernatremia and hypertonic dehydration affecting mental status

257. A 29 year old male is referred from an optometrist for evaluation. The patient’s liver
profile shows AST 78 IU/L, ALT 92 IU/L, Bili 1.4 mg/dL, Alk Phos 88 IU/L, and
albumin 3.4 g/dL. The photo of the patient’s eyes is below. All of the following
statements are true except:

A. The patient’s diagnosis is Wilson’s disease if the ceruloplasmin is low.


88

B. This finding on the eye exam can be seen in primary biliary


cirrhosis and autoimmune hepatitis.
C. The pigmentation will disappear with effective therapy.
D. Neurologic symptoms typically do not occur in the absence of this finding.
E. The mechanism that causes the disorder associated with this
finding occurs as a result of over-absorption of copper.
89

258. A 10-old male with HIV on HART therapy is evaluated for elevated liver enzymes
(ALT 119 IU/L, AST 101 IU/L, bilirubin 1.3 mg/dL, Alk Phos 390 IU/L). A liver
biopsy showed numerous blood-filled cysts that do not have an endothelial lining.
This liver biopsy finding is most likely secondary to:

A. Cytomegalovirus
B. Protease inhibitors
C. Rochalimaea henselae
D. Caroli disease
E. Congenital factors (e.g., cystic Von Meyenburg complexes)

259. An infant boy does not pass stool during the first 36 hours of life. Following rectal
examination, he passes a meconium plug. During the next 2 weeks, he has
intermittent episodes of both watery and hard, pellet-like stools. Barium enema
reveals dilation of the large bowel with narrowing immediately proximal to the
rectum.

The MOST appropriate management of this patient’s problem is to

A. Add table sugar to the present formula


B. Arrange for a rectal biopsy
C. Change to a soy-based formula
D. Observe without intervention
E. Order an upper gastrointestinal series

260. Lactose enhances intestinal absorption of which ONE of the following


nutrients?

A. Calcium
B. Chloride
C. Lipid
D. Potassium
E. Sodium

261. A 2 week old boy has short-bowel syndrome following surgery for severe necrotizing
enterocolitis. Management has included total parenteral nutrition. Clinical findings
include a wasted appearance; dry, flaky skin; a poorly healing abdominal
incision; and thrombocytopenia.

The patient MOST likely has a deficiency of

A. Calories
B. Essential amino acids
C. Essential fatty acids
D. Iron
90

E. Vitamin E

262. A 14 month old African-American infant, exclusively breastfed since birth, has just
begun walking. Physical examination reveals prominence of the costochondral
junctions. Radiographs reveal widen- ing of the distal end of the radii. The laboratory
test MOST likely to confirm the diagnosis is measure- ment of the serum
concentration of:

A. Albumin
B. Lactate dehydrogenase
C. Phosphorus
D. Vitamin A
E. Vitamin C
91

263. A 6 year old boy who has had a relapse of acute lymphoblastic leukemia has had a
4.8 kg (10 lb) weight loss during a course of combination chemotherapy with
prednisone, vincristine, and high- dose methotrexate. He has had a poor
appetite but no vomiting or diarrhea.

Of the following, the MOST appropriate next course of management would be

A. Administration of anabolic steroids


B. Administration of intravenous gamma globulin
C. Discontinuation of methotrexate
D. Enteral hyperalimentation
E. Parenteral hyperalimentation

264. The basal energy or metabolic requirement for children is calculated MOST accurately by
considering

A. Body surface area


B. Creatinine-height index
C. Serum protein concentration
D. Total lymphocyte count
E. Triceps skinfold thickness

265. An infant boy born at term is delivered at home without medical supervision. At 48
hours of age, he is brought to the emergency room because of a bloody discharge from
the umbilical cord and bloody stools. Until the results of laboratory studies are
available, the BEST initial management is to adminis- ter intravenous:

A. Ampicillin and gentamicin


B. Cryoprecipitate
C. Factor VIII concentrate
D. Fresh frozen plasma
E. Vitamin K

266. A 3 1/2 year old boy with chronic diarrhea and failure to thrive is diagnosed with
cystic fibrosis. Neurologic examination reveals absent deep tendon reflexes, truncal
ataxia, and muscle weakness. A nutrient deficiency is suspected. Given this
constellation of findings, what additional physical sign is MOST likely to be present
in this child?

A. Desquamating skin lesions


B. Ophthalmoplegia
C. Positive Trousseau sign
D. Purpura
E. Stooped posture
92

267. Which of the following BEST explains why solutions containing 1.2 to 2.5%
glucose, rather than 5% glucose, are used for oral rehydration?

A. Absorption of sodium and water in the gut is maximized


B. Glomerular filtration rate is enhanced
C. Hyperglycemia and glycosuria are less likely to occur
D. Potassium absorption is decreased
E. Stomach distention with vomiting is less likely to occur

268. A TRUE statement about the sugar content of infant formulas is:

A. All lactose-containing formulas are cow milk-based


B. All cow milk-based formulas contain only simple sugars
C. All soy-based formulas are corn syrup-free
D. All soy-based formulas contain lactose
E. Proprietary formulas do not contain sucrose
93

269. Shortly after birth, a 3,500 g term newborn is found to be jittery and to have a high-
pitched cry. Physical examination reveals tachypnea and a liver edge that is palpable
several centimeters below the umbilicus. Blood glucose concentration is 14 mg/dL.
Among the following, the MOST likely cause of the hypoglycemia in this newborn
is:

A. Galactokinase deficiency
B. Glycogen storage disease
C. Insulinoma
D. Maternal diabetes mellitus
E. Prolonged maternal labor

270. A breastfed infant who appeared healthy at birth develops chronic diarrhea, failure
to thrive, and hepatomegaly during the first few weeks of life. Ultrasonography
reveals adrenal enlargement and calcification.

Of the following, the MOST likely explanation for these findings is:

A. Cystic fibrosis
B. Glucose-galactose malabsorption
C. Glycogen storage disease
D. Niemann-Pick disease
E. Wolman disease

271. Among the following, the gastrointestinal disease MOST likely to respond to
treatment with anti- cholinergic medications is:

A. Constipation
B. Dysentery
C. Gastroesophageal reflux
D. Irritable bowel syndrome
E. Poor motility

272. Examination of a developmentally normal 7-month-old boy reveals moderately


enlarged cervical lymph nodes; a hemorrhagic seborrhea-like rash on the forehead,
scalp, and trunk; and hepato- splenomegaly. Laboratory findings include:
hemoglobin, 12.0 g/dL; mean corpuscular volume, 82 fL; white blood cell count,
10,700/mm³, with 40% neutrophils and 60% lymphocytes; and platelet count,
260,000/mm³.

These findings are MOST consistent with

A. Acute lymphoblastic leukemia


B. Aplastic anemia
C. Langerhans cell histiocytosis
94

D. Neuroblastoma
E. Niemann-Pick disease

273. A 14 year old boy is being evaluated for jaundice that was first noted 1 week ago
following an upper respiratory tract infection. He reports not feeling very hungry for
the past month. Physical examina- tion reveals a firm liver, an enlarged spleen,
and an intention tremor.

Among the following, the test that would be MOST helpful for making a
definitive diagnosis in this patient is a:

A. Liver biopsy for copper content


B. Serum bilirubin concentration
C. Serum ceruloplasmin level
D. Serum transaminase activity
95

274. A 3 month old infant girl is admitted to the hospital for evaluation of recurrent
episodes of hypogly- cemia. Physical examination reveals hepatomegaly. After 2
hours of fasting, she develops hypoglyce- mia. Measurement of which of the
following would be MOST helpful in determining the etiology of this patient’s
hypoglycemia?

A. Ammonia in the serum and the arterial pH


B. Cortisol and growth hormone in the serum
C. Insulin and glucose in the serum
D. Ketones and reducing substances in the urine
E. Organic acids in the urine and lactate in the serum

275. The mother of a 3 month old infant reports that the boy is demanding frequent
feedings and has a noticeably protuberant abdomen. Physical examination reveals
doll-like facies and marked hepa- tomegaly. Laboratory findings include a serum
glucose level of 20 mg/dL and an elevated venous lactate level of 44 mg/dL
(normal, <18 mg/dL).

Of the following, the most appropriate INITIAL management of this infant is:

A. Administration of a formula that has high concentrations of fructose and galactose


B. Daily injections of glucagon
C. Insertion of a portacaval shunt
D. Nocturnal infusion of glucose via a nasogastric tube
E. Referral for liver transplantation

276. The mother of a 3 month old boy reports that he has a poor appetite and
constipation. Findings on physical examination, when compared to those 2
months ago, include poor interim growth, increased lethargy, hoarse cry, decreased
tone, large fontanelles, and a more pronounced umbilical hernia.

Of the following, the MOST likely cause of this infant’s problem is

A. Agenesis of the thyroid gland


B. Endemic goitrous hypothyroidism
C. End-organ unresponsiveness to thyroid hormone
D. Inborn error of thyroxine synthesis
E. Thyroid gland unresponsiveness to thyrotropin

277. Which of the following is the most common cause of pancreatitis in childhood?

A. Viral
B. Drug induced
C. Idiopathic
D. Familial
96

E. Abdominal trauma

278. Which of the following is not part of the Currarino triad characterizing caudal
regression syndrome which can present as infantile constipation?

A. Dysplastic sacrum
B. Anal abnormalities
C. Tethered cord
D. Pre-sacral mass

279. Which of the following is NOT associated with Wilson’s Disease:

A. Fatty liver
B. High serum uric acid
C. Low serum zinc level
D. Low serum alkaline phosphatase
E. High serum bilirubin
97

280. Which statement is false?

A. 90% of Vitamin A is stored within the liver


B. Vitamin A deficiency Xerophthalmia is irreversible
C. Can cause a hypochromic microcytic anemia with low Fe but normal Fe
stores
D. Hypervitaminosis A is associated with head aches

281. Which of the following is NOT a common feature of BOTH kwashiorkor and
marasmus:

A. Irritability
B. Decreased serum lipoproteins
C. Markedly Depressed serum albumin
D. Increased susceptibility to infection
E. Anemia

282. Tyrosinemia is associated with:

A. Boiled cabbage smell


B. Mousey smell
C. Blue Cheese Vinaigrette smell
D. Maple syrup
E. All of the above except D

283. Which statement about Hepatitis A is true?

A. Leading cause of fulminant hepatitis in Pediatrics


B. Has been associated with chronic hepatitis
C. Recurrence of the disease can occur up to 6 months after primary infection
D. Treatment for non-fulminant hepatitis A includes Lamivudine for 4 weeks
E. Severity of disease decrease with increasing age

284. Acute lower GI hemorrhage in HIV infected patients is most often caused by:

A. CMV colitis
B. Lymphoma
C. Kaposi’s sarcoma
D. Idiopathic chronic colitis
E. Nonspecific colitis

285. First line of treatment of esophageal candidiasis in HIV infected patient is:

A. Clotrimazole
B. Ketoconazole
98

C. Fluconazole
D. Amphotericin B

286. Organism causing intestinal microsporidiosis in AIDS patients is:

A. Encephalitozoon intestinalis
B. Cryptosporidium
C. Isospora Belli
D. Enterocytozoon Bieneusi
99

287. A 7 year old girl who had undergone a surgical repair for long-segment
Hirschsprung’s disease in early infancy presents with a fever, abdominal distention,
and bloody diarrhea for 2 days. Which of the following is the most likely
diagnosis?

A. Diversion colitis
B. Enterocolitis
C. Ulcerative colitis
D. Colonic stricture
E. Viral gastroenteritis

288. A 6 year old boy just arriving from Eastern Europe has had malodorous diarrhea since
early infancy, even though he was breast-fed. He is small, has some bruises from
bumping into furniture going to the bathroom at night, and has recently developed
some difficulty walking. Physical examination shows that he is small and
undernourished, with depleted subcutaneous fat. He has a protuber- ant abdomen
and 1+ edema in his lower extremities. He has no deep tendon reflexes in his lower
extremities. Which one of the following explains the finding on the small intestinal
biopsy from this patient?

A. Gluten enteropathy
B. Congenital lactase deficiency
C. Abetalipoproteinemia
D. Glucose-galactose transport defect
E. Chronic nonspecific diarrhea of childhood

289. A 5 year old boy is referred for evaluation of liver disease after presenting to his
primary physi- cian with chronic pruritus. His evaluation reveals a small child
(below the fifth percentile for height;
weight for height tenth percentile) with excoriations on his trunk and extremities. He
has no icterus. A grade 2/6 systolic murmur is heard at the left upper sternal border.
His liver is soft, about 1 cm below the right costal margin and nontender. Spleen was
not palpable. He has diminished but sym- metric deep tendon reflexes in his lower
extremities. Laboratory studies reveal:

Hemoglobin 12.8
Platelet count 239,00
0
AST 129
ALT 134
Alkaline phosphatase 678
GGTP 948
Total bilirubin 0.7
Prothrombin time 13.9
INR 1.2

Which one of the following is the most likely diagnosis?


100

A. Progressive familial intrahepatic cholestasis (e.g., Byler’s disease)


B. Sclerosing cholangitis
C. Niemann-Pick disease, type A
D. Alagille syndrome
E. Alpha-1-antitrypsin deficiency

290. All of the following statements about hepatitis E are true, except

A. Outbreaks of hepatitis E tend to be very large because of the high rate


of secondary (case-to-case) spread
B. Cases of hepatitis E in the United States are rare
C. Infection with hepatitis E virus (HEV) in pregnancy is associated with high mortality rate
D. Anti-HEV appears to be protective, and prospects for developing a vaccine are good
E. HEV is not closely related in structure or function to any of the other viral hepatitis agents
101

291. All of the following agents are effective for both induction of remission and
maintenance of remission in patients with ulcerative colitis except

A. Oral mesalamine formulations including Asacol and Pentasa


B. Sulfasalazine
C. Olsalazine (Dipentum)
D. Rectal mesalamine (Rowasa enemas and suppositories)
E. No exception

292. Which one of the following statements is false with respect to the use of
cyclosporine in patients with inflammatory bowel disease?

A. Cyclosporine administered as a continuous intravenous infusion at a high dose


of 4 mg/kg/d is effective for severely active ulcerative colitis
B. Cyclosporine administered orally at a dose of 5 mg/kg/d is ineffective for the
induction of improvement or remission in patients with active Crohn’s
disease
C. Cyclosporine administered orally at a dose of 5 mg/kg/d is ineffective for
maintenance of remission in patients with Crohn’s disease
D. Cyclosporine is slow acting and thus is not useful as a bridge therapy to other
slower acting medications such as 6-mercaptopurine, azathioprine, or
methotrexate
E. None of the above

293. A young adult with a life-long history of mild jaundice, but no bilirubinemia or
evidence of chronic hepatitis or hemolysis is likely to have a genetic defect in:

A. Sinusoidal bilirubin uptake pump


B. MRP2 (canalicular multispecific organic anion transporter)
C. Bilirubin-UGT
D. UDP glucuronic acid synthetase
E. Cholesterol 7-á-hydroxylase

294. Alpha- l-antitrypsin deficiency leads to liver injury by way of

A. Uncontrolled proteolytic enzyme activity in the portal tracts


B. Chronic pancreatitis and focal biliary cirrhosis
C. Inability to transport divalent cations into the endoplasmic reticulum
D. Accumulation of abnormal glycoprotein in the liver cells
E. Pulmonary fibrosis and the development of cardiac cirrhosis

295. Which serological test is most commonly abnormal in autoimmune hepatitis?

A. Anti-nuclear antibody (ANA)


B. Anti-smooth muscle antibody
102

C. Anti-LKM antibody
D. Anti-SLA
E. Anti-HCV

296. A patient with photosensitivity is referred to you because of abnormal


transaminases (ALT=120, AST=150). Physical examination shows pigmentation
and blisters on the dorsa of the hands. What results of laboratory testing is the
least likely to be found?

A. Positive hepatitis C antibody


B. High levels of excretion of ALA (aminolevulinic acid) and
porphobilinogen (PBG)
C. Serum ferritin of 300 ng/mL
D. Elevated MCV (mean corpuscular volume)
E. Elevated gamma glutamyl-transpeptidase
103

297. Crigler-Najjar syndrome type II is typically treated by:

A. Liver transplantation
B. Lifelong phototherapy
C. Ursodeoxycholic acid
D. Gene transfer of UGT using adenovirus vectors
E. Phenobarbital

298. A 15 year old boy with decompensated cryptogenic cirrhosis presents with a 2 week
history of increasing anorexia and weakness. Four weeks prior to presentation he
was treated for an episode of spontaneous bacterial peritonitis and was discharged
after 5 days of IV antibiotics on prophylac- tic Bactrim, spironolactone and
furosemide. Physical examination is remarkable for jaundice and ascites.
Laboratory data reveal a serum creatinine of 3.1 mg/dl (4 weeks prior: 0.8 mg/dl), and
BUN of 52 mg/dl (4 weeks prior: 14 mg/dl). At the present time, all of the following
are appropriate steps except:

A. Decrease diuretics by half, liberalize sodium intake and obtain follow-up blood studies.
B. Obtain urinalysis, urine sodium measurement, and urine eosinophil count
C. Renal ultrasound
D. Discontinue diuretics and give saline or colloid challenge
E. Repeat diagnostic paracentesis

299. Hepatitis A occurs in cyclical outbreaks in the


United States. These outbreaks spread largely
because of:

A. HAV infection among injection drug users


B. Promiscuous sexual behavior
C. Infected food handlers
D. Widespread vaccination programs
E. Close personal contact with infected but asymptomatic individuals, particularly
children

300. A previously healthy two year old boy is referred to you for elevation of liver
function tests. When a liver profile was drawn during an episode of fever, his serum
alkaline phosphatase concentration was elevated. He has no recent history of
fractures. His growth and development have been normal. He did not have neonatal
liver disease. Review of symptoms is negative for pruritus, chronic diarrhea, or acholic
stools. His physical examination is normal. Laboratory studies at your institution
confirm the biochemical findings. Serum 25-hydroxy vitamin D levels are
within the normal range.
Which of the following is the most appropriate next step to manage this child?

Patient’s results Normal range


104

Calcium 9.2 mg/dL 8.8-10.7 mg/dL


Phosphorus 4.2 mg/dL 3.0-5.0 mg/dL
Blood urea nitrogen 8 mg/dL 5-20 mg/dL
SGOT 28 IU/L 20-60 IU/L
SGPT 18 IU/L 5-45 IU/L
GGT 12 IU/L 6-20 IU/L
Conjugated bilirubin 0.1 mg/ dL < 0.3 mg/dL
Alkaline phosphatase 2800 IU/L 65-525 IU/L

A. Abdominal ultrasound
B. Liver biopsy
C. Radiographs for rickets survey
D. 1,25 dihydroxy vitamin D level
E. No further laboratory tests
105

301. The following statements regarding the management of foreign bodies in


the stomach are true EXCEPT:

The clinician should consider removing objects that are more than 2 cm in
A.
diameter or more than 5 cm in length, because they are unlikely to pass
through the duodenum.
B. In the case of battery ingestion, levels of heavy metal in the
blood and urine should be measured.
C. Batteries that have passed through the esophagus to the stomach
should always be removed.
D. Between 80% to 90% of ingested foreign bodies that reach the
stomach will pass without specific therapy.
302. Which of the following metabolic alterations is most commonly seen with re-feeding
syndrome?

A. Hyperlipidemia secondary to increased serum ketone bodies


B. Wernicke’s encephalopathy secondary to thiamine deficiency
C. Severe hypophosphatemia affecting myocardial and respiratory function
D. Hypernatremia and hypertonic dehydration affecting mental status

303. Which of the following statements concerning hereditary hemochromatosis (HH) is


false?

A. The phenotypic expression in the United States is 1/200-1/250.


B. HH is one of the most common, identified, genetic disorder in Caucasians.
C. The genetic defect causes an excessive absorption of iron.
D. Compound heterozygosity (C282Y, H63D) accounts for 3-5% of cases.
E. HH should be considered in any male patient with transferrin
saturation greater than 30 percent.

304. What is not a result of ascorbic acid deficiency?

A. Perifollicular hemorrhage
B. Subperiosteal hemorrhage
C. Hyperkeratotic hair follicles
D. Cheilosis

305. An asymptomatic duplication cyst that is found incidentally should be

A. Watched expectantly
B. Excised in a patients under the age of three years
C. Excised regardless of age
D. Undergo MRI and if it communicates with intestinal lumen, then excise
E. Undergo MRI and if it does not communicate with intestinal lumen, then
watch
106

306. With respect to candidal esophagitis which statement is false?

A. It results in dysphagia, substernal pain and odynophagia


B. It can be treated with oral fluconazole
C. It is highly unlikely in the absence of thrush
D. It can be mimicked by cryptococcosis
E. Patients with AIDS may have candidal esophagitis with thrush

307. With respect to herpetic esophagitis which statement is false?

A. It is highly unlikely in the absence of herpetic stomatitis


B. It may be accompanied by drooling, fever and generalized malaise
C. Esophageal biopsy may show multinucleated giant cells
D. It can occur in children with normal immunity
E. Ulcerations may resemble CVM esophagitis
107

308. A previously healthy 2-month-old infant suddenly develops bilious vomiting.


Physical examination reveals an ill-appearing child with abdominal distention and
diminished bowel sounds. Which of the following is the MOST likely diagnosis?

A. Antral web
B. Cholelithiasis
C. Duodenal atresia
D. Malrotation with midgut volvulus
E. Peptic ulcer disease

309. Meckel’s diverticulum

A. Can be lined by ileal mucosa with external serosa but no muscularis layers
B. Can be lined by ileal mucosa, ectopic gastric mucosa and muscularis – serosa layers (*)
C. Can be lined with total gastric mucosa
D. Often has pancreatic rest which ulcerate
E. Can be lined with non-HCL secreting gastric mucosa

310. A toddler presents with edema of the hands, feet, and scrotum. Hypoproteinemia,
lymphocytopenia, and decreased levels or serum albumin, immunoglobulins,
transferrin, and ceruloplasmin are noted. Small bowel contrast study shows thickened
mucosal folds. Characteristic histopathology will most likely reveal

A. Eosinophilia in the lamina propria


B. Dilated lymphatics in the villous tips
C. Neutrophils invading the crypts
D. Basal lymphocytosis
E. Haloed inclusions bodies within the enterocytes

311. You are consulted on a 19 m/o with corrected tricuspid atresia and moderate anasarca.
Upon hearing the history and performing a physical examination you ask for a
screening stool alpha-1-antitrypsin level because you suspect

A. Hypoproteinemia secondary to alpha-1-antitrypsin liver disease


B. Protein losing enteropathy from heart surgery
C. Pulmonary hypertension secondary to alpha-1-antitrypsin deficiency fibrosis
D. Chaperon disease of enterocytes
E. Neutrophil elastase enteropathy

312. Which of the following laboratory findings is NOT likely to be found in a patient
presenting with Small Bowel Bacterial Overgrowth?

A. Elevated D-lactate
B. Macrocytic Anemia
C. Microcytic Anemia
108

D. Elevated Stool pH
E. Hypocalcemia
109

313. A 9-year-old girl with no previous illness is admitted with RLQ abdominal pain of
one month dura- tion, worsening over the last few days. Her pain is constant, non-
radiating, moderate to severe
in intensity, and associated with nausea and vomiting. She has suffered a weight loss
of10 lb. On examination, she has RLQ tenderness without abdominal rigidity,
guarding or rebound pain. Bowel sounds are normal. She has no fever. CBC, serum
chemistries, CRP, amylase and lipase were normal except for mild normocytic
anemia and moderately elevated CRP. Abdominal CT scan with contrast reveals
mesenteric adenopathy measuring 3 cm maximum and an irregular filling defect
involving the terminal ilium. EGD and colonoscopy reveals 3 polypoidal mucosal
lesions in the cecum measuring
2.5 cm maximum. The ileocecal valve is edematous and the ileum is hard to intubate.
Biopsies reveal mild focal active cecitis and normal mucosa in the rest of the colon
and upper GI tract. The most ap- propriate next step is:

A. Small Bowel Series


B. Start Treatment with Steroids
C. Diagnostic laparoscopy
D. TB skin Test
E. Send IBD panel
F. Abdominal/Pelvic Ultrasound

314. Gastric acid secretion is inhibited by

A. Gastrin
B. Secretin
C. Motilin
D. CCK
E. Histamine
110

Answers for Board Review

1. C (Congenital Anomalies of the Esophagus)


2. C (Congenital Anomalies of the Esophagus)
3. B (Deglutition)
4. A (Dysphagia)
5. E (Dysphagia)
6. C (Eosinophilic Esophagitis).
7. B (Eosinophilic Esophagitis).
8. E (Eosinophilic Esophagitis).
9. D (Esophageal Anatomy, Development and Physiology)
10. D (Esophageal Caustic Injury)
11. F (Esophageal Motility)
12. D (Upper GI Bleed)
13. C (Gastroesophageal Reflux and Gastroesophageal Reflux Disease)
14. A (Gastroesophageal Reflux and Gastroesophageal Reflux Disease)
15. E (Gastroesophageal Reflux and Gastroesophageal Reflux Disease)
16. E (Esophageal Caustic Ingestions)
17. D milk-protein intolerance (Colic)
18. B (Food and Waterborne Illness)
19. D (Food and Waterborne Illness)
20. B (Abdominal Masses)
21. A (Abdominal Masses)
22. D (Gastritis)
23. D (H Pylori)
25. C (Pyloric Stenosis)
26. D (Gastric Foreign Body)
27. C (Gastric Foreign Body)
28. A (Gastric Foreign Body)
29. C (Appendicitis)
30. B (Appendicitis)
31. D (Autoimmune Enteropathy)
32. D (Autoimmune Enteropathy)
33. D (Abdominal Pain)
34. A (Abdominal Pain)
35. B (Stomach Congenital Anomalies)
36. B (Stomach Congenital Anomalies)
37. A (Celiac Disease)
38. D (Celiac Disease)
39. C (Small Intestine – Congenital Anomalies)
40. D (Small Intestine – Congenital Anomalies)
41. C (Small Intestine – Congenital Anomalies)
42. B (Necrotizing Enterocolitis)
43. C (Necrotizing Enterocolitis)
111

44. B (Small Intestine – Congenital Anomalies)


45. B (Small Intestine – Congenital Anomalies)
46. B (Small Intestine – Congenital Anomalies)
47. E (Small Intestine – Congenital Anomalies)
48. E (Small Intestine – Congenital Anomalies)
49. D (Small Bowel Bacterial Overgrowth)
50. C (Small Bowel Bacterial Overgrowth)
51. B (Enteric Infections)
52. D (Enteric Infections)
53. A (Enteric Infections)
54. B (Small Intestine – Obstruction)
112

55. D (Tropical Sprue)


56. C (Tropical Sprue)
57. B (Colitis not due to inflammatory bowel disease)
58. D (Colitis not due to inflammatory bowel disease)
59. C (Colitis not due to inflammatory bowel disease)
60. C (Inflammatory bowel disease – Crohn’s Disease)
61. C (Inflammatory bowel disease – Crohn’s Disease)
62. B (Inflammatory bowel disease – Ulcerative Colitis)
63. A (Inflammatory bowel disease – Ulcerative Colitis)
64. B (Colonic Motility)
65. D (Constipation)
66. C (Constipation)
67. A (Constipation)
68. C (Hemorrhoids)
69. C (Hemorrhoids).
70. C (Hirschsprung Disease)
71. B (Other inflammatory lesions of the bowel)
72. A (Other inflammatory lesions of the bowel)
73. C (Other inflammatory lesions of the bowel)
74. E (Polyps)
75. E (Polyps)
76. C (Rectal Prolapse)
77. B (Perianal Disease)
78. B (Perianal Disease)
79. B (Perianal Disease)
80. D (Pseudo Obstruction)
81. B (Biliary Tree Normal Microanatomy)
82. B (Other disorders of the bile ducts)
83. C (Other disorders of the bile ducts)
84. B (Biliary Atresia)
85. E (Biliary Atresia)
86. E (Cholecystitis)
87. C (Cholecystitis)
88. D (Cholecystitis)
89. C (Cholecystitis)
90. A (Cholecystitis)
91. B (Fulminant liver failure)
92. D (Fulminant liver failure)
93. A (Fulminant liver failure)
94. A (Fulminant liver failure)
95. B (Fulminant liver failure)
96. C (Fulminant liver failure)
97. A (Bacterial, parasitic and other infections of the liver)
98. E (Bile acid synthetic defects)
99. B (Bile acid synthetic defects)
113

100. A (Carbohydrate Metabolism)


101. C (Carbohydrate Metabolism)
102. E (Drug Induced Liver Injury)
103. D (Drug Induced Liver Injury)
104. E (Disorders of Lipid Metabolism)
105. B (Disorders of Lipid Metabolism)
106. B (Granulomatous hepatitis)
107. A (Granulomatous hepatitis)
108. C (Granulomatous hepatitis)
109. B (Non alcoholic fatty liver disease and steatohepatitis (NAFLD/NASH))
110. D (Liver transplantation)
111. B (Liver transplantation)
112. E (Liver transplantation)
113. A (Liver transplantation)
114. B (Liver transplantation)
114

115. A (Liver transplantation)


116. D (Peroxisomal Disorders)
117. A (Familial hepatocellular cholestatic disorders)
118. A = 1, B = 3, C = 4, D = 2 (Familial hepatocellular cholestatic disorders)
119. D (Familial hepatocellular cholestatic disorders)
120. E ( Familial hepatocellular cholestatic disorders)
121. C (Acute graft vs Host disease and veno occlusive disease)
122. A (Acute graft vs Host disease and veno occlusive disease)
123. F (Acute graft vs Host disease and veno occlusive disease)
124. B(Acute graft vs Host disease and veno occlusive disease)
125. B (Acute graft vs Host disease and veno occlusive disease)
126. E (Disorders of bilirubin metabolism)
127. A (Jaundice)
128. B (Disorders of amino acid metabolism)
129. D (Liver masses)
130. C (Congenital hepatic infections)
131. E (Congenital hepatic infections)
132. C (Viral hepatitis)
133. C (Viral hepatitis)
134. F (Viral hepatitis)
135. A (Viral hepatitis)
136. B (Neonatal Cholestasis)
137. E (Neonatal Cholestasis)
138. B (Chronic hepatitis - Autoimmune Hepatitis and Crossover Syndromes in Children)
139. A (Chronic hepatitis - Autoimmune Hepatitis and Crossover Syndromes in Children)
140. E (Chronic hepatitis - Autoimmune Hepatitis and Crossover Syndromes in Children)
141. D (Pancreas - Normal anatomy, development and physiology)
142. B (Pancreas - Normal anatomy, development and physiology)
143. B (Acute Pancreatitis)
144. D (Pancreas – Exocrine Function)
145. B (Pancreas – Exocrine Function)
146. A (Pancreas – Exocrine Function)
147. A (Congenital anomalies of the pancreas)
148. B (Congenital anomalies of the pancreas)
149. A (Shwachman-Diamond syndrome)
150. A (Nutritional consequences of cholestasis)
151. C (Nutritional consequences of cholestasis)
152. C (Congenital enzyme and transport defects)
153. C (Congenital enzyme and transport defects)
154. B (Congenital enzyme and transport defects)
155. D (Congenital enzyme and transport defects)
156. B (Normal digestion and absorption)
157. D (Normal digestion and absorption)
158. A (Normal digestion and absorption)
159. A (Normal digestion and absorption)
115

160. B (Normal digestion and absorption)


161. D (Normal digestion and absorption)
162. C (Disaccharidase deficiency)
163. B (Disaccharidase deficiency)
164. A (Comparison of human milk and cow-milk based formulas)
165. C (Malnutrition)
166. D (Vitamin and mineral absorption, function and deficiency states)
167. A (Vitamin and mineral absorption, function and deficiency states)
168. C (Nutritional requirements of pre-term and term infants, children and adolescents)
169. D (Nutritional requirements of pre-term and term infants, children and adolescents)
170. C (Obesity)
171. C (Nutritional Therapy)
172. B (Nutritional Therapy)
173. D (Nutritional Therapy)
174. B (Vitamin and mineral absorption, function and deficiency states)
116

175. D (Vitamin and mineral absorption, function and deficiency states)


176. B (Vitamin and mineral absorption, function and deficiency states)
177. C (Vitamin and mineral absorption, function and deficiency states)
178. C (Vitamin and mineral absorption, function and deficiency states)
179. A (Vitamin and mineral absorption, function and deficiency states)
180. C (Vitamin and mineral absorption, function and deficiency states)
181. D (Vitamin and mineral absorption, function and deficiency states)
182. D (Vitamin and mineral absorption, function and deficiency states)
183. A (Vitamin and mineral absorption, function and deficiency states)
184. C (Alkaline phosphatase)
185. A (Endoscopy)
186. B (Endoscopy)
187. C (Breath tests)
188. D (Intestinal Biopsy)
189. C (Intestinal Biopsy)
190. D (Pancreatic Function Testing)
191. C (Pancreatic Function Testing)
192. C (Radiologic Evaluations)
193. D (Radiologic Evaluations)
194. C (Radiologic Evaluations)
195. B (Stool Testing)
196. C (Stool Testing)
197. B (Anti-diarrheals)
198. C (Anti-pruritic agents)
199. B (Blood Replacement)
200. D (Fluid Therapy)
201. D (Anti-rejection and anti-inflammatory)
202. C (Anti-rejection and anti-inflammatory)
203. B (Anti-rejection and anti-inflammatory)
204. E (Biologics)
205. B (Biologics)
206. A (Biologics)
207. C (Biologics)
208. D (Pancreatic Enzymes)
209. B (Pancreatic Enzymes)
210. A (Acid Control)
211. C (Acid Control)
212. A (Prebiotics/Probiotics)
213. D (Prebiotics/Probiotics
214. E (Prebiotics/Probiotics)
215. A (Prostaglandins)
216. A (Sucralfate)
217. C (Sucralfate)
218. A (Sucralfate)
219. C (Munchausen by proxy syndrome)
117

220. D (Munchausen by proxy syndrome)


221. D (Rumination)
222. B (Chronic Diarrhea)
223. C(Chronic Diarrhea)
224. D (Chronic Diarrhea)
225. D (Drug induced bowel injury)
226. C (Drug induced bowel injury)
227. D (GI manifestations of immunodeficiency).
228. C (GI manifestations of immunodeficiency)
229. D (GI manifestations of immunodeficiency)
230. C (GI manifestations of immunodeficiency)
231. C ( GI manifestations of endocrine disorders)
232. C (GI manifestations of endocrine disorders)
233. D (GI manifestations of endocrine disorders)
234. C (Graft versus host disease)
118

235. B C (Ethics)
236. D (Ethics)
237. C (Study Design and Statistics)
238. Validity- B; Reliability- C; Accuracy- E; Precision- F;
Kappa statistic-A; Cronbach’s alpha- D (Study Design
and Statistics)
239. C (Study Design and Statistics)
240. D (Study Design and Statistics)
241. A (Study Design and Statistics)
242. A (Study Design and Statistics)
243. E (Abdominal Pain)
244. C (Prostaglandins)
245. B (Abdominal Pain)
246. B (GI Manifestations of Endocrine Disease)
247. E (Endoscopy)
248. D (Stool Testing)
249. D (Prostaglandins)
250. E (Drug Induced Bowel Injury)
251. B (TPN)
252. C (Acute Liver Failure)
253. B (Schwachman Diamond Syndrome)
254. D (Enteric Infection)
255. A (Neonatal Cholestasis)
256. C (Malnutrition)
257. E (Wilson’s Disease)
258. C (GI Manifestations of Immune Deficiency)
259. B (Hirschsprung Disease)
260. E (Normal Digestion and Absorption)
261. C (Essential Fatty Acids)
262. C (Vitamin and Mineral Absorption, function and deficiency states)
263. D (TPN)
264. E (Nutritional Assessment)
265. E (Vitamin and Mineral Absorption, function and deficiency states)
266. B (Vitamin and Mineral Absorption, function and deficiency states)
267. A (Normal Digestion and Absorption)
268. A (Infant Formula)
269. B (Glycogen Storage Disease)
270. E (Disorders of Lipid Metabolism)
271. D (Irritable Bowel Syndrome)
272. C (Hematologic Manifestations of GI Disease)
273. A (Wilson’s Disease)
274. E (Hepatomegaly)
275. D (Glycogen Storage Disease)
276. D (Constipation)
277. E (Acute Pancreatitis)
119

278. C (Constipation)
279. B (Wilson’s Disease)
280. B (Vitamin and Mineral Absorption, function and deficiency states)
281. C (Malnutrition)
282. A (Tyrosinemia)
283. C (Viral Hepatitis)
284. A (GI manifestations of Immunodeficiency)
285. C (GI manifestations of Immunodeficiency)
286. C (GI manifestations of Immunodeficiency)
287. B (Hirschsprung Disease)
288. C (Abetalipoproteinemia)
289. D (Alagille syndrome)
290. A (Viral Hepatitis)
291. E (IBD – Ulcerative Colitis)
292. D (IBD – Crohn’s Disease)
293. C (Disorders of bilirubin metabolism)
120

294. D (Alpha-1-antitrypsin deficiency)


295. A (Chronic Hepatitis)
296. B (Porphyria)
297. E (Bilirubin Metabolism disorders)
298. A (Liver Transplant)
299. E (Viral Hepatitis)
300. E (Alkaline Phosphatase)
301. B (Gastric Foreign Body)
302. C (Malnutrition)
303. E (Iron Storage Disease)
304. D (Vitamin and Mineral Absorption, function and deficiency states)
305. C (Congenital Anomalies of Small Intestine)
306. C (Infectious Esophagitis)
307. A (Infectious Esophagitis)
308. D (Congenital Anomalies of Small Intestine)
309. B (Congenital Anomalies of Small Intestine)
310. B (Protein-losing Enteropathy)
311. B (Protein-losing Enteropathy)
312. D (Small bowel bacterial Overgrowth)
313. C (Abdominal Mass)
314. B (Stomach Anatomy, Development and Physiology)
Review Questions
1. A 3-year-old girl presents to your office with 3 months of diarrhea. She is
otherwise well, and her growth has been normal. Dietary history reveals low
fiber intake and high juice intake. What is the first step in her management?
a. Initiation of a BRAT diet
b. Elimination of juice and other sugar-sweetened foods
and beverages from her diet
c. Screening laboratories
d. Stool pH and reducing substances
e. 24 hours of oral rehydration solution only
1. b. This 3-year-old girl has diarrhea but has continued to have normal growth. As a
result, a true malabsorptive or inflammatory process is less likely. Given that
she has a high amount of juice intake, this provides ahigh osmotic load to the intestine
and results in toddler's diarrhea. The first step in her management is minimally
invasive and involves a trial of eliminated sugar-sweetened foods and beverages from
her diet to see whether the diarrhea resolves. Her process is chronic, and treating it as
an acute process with 24hours of oral rehydration solution will not help long
term. A BRAT diet is no longer recommended for diarrhea,but especially not for
chronic diarrhea. Screening laboratories and stool studies can be obtained but are
likely to be low yield in this setting.
2. You have been seeing a 16-year-old girl with Crohn disease for several years. She
recently had an ileocecectomy and has remained on medical treatment after surgery.
She reports that after surgery, she has been having increased stool frequency but is
otherwise feeling well.Colonoscopy and MRE before surgery demonstrated
that the areas of active disease were limited to the ileum and cecum. What is the most
likely etiology of her diarrhea?
a. Clostridium difficile colitis
b. Short bowel syndrome
c. Bile acid diarrhea
d. Active Crohn disease
e. Pancreatitis
2. c. This patient has Crohn disease, and evaluation before surgery showed that active
disease was limited to the ileum and cecum. This portion of the bowel was
resected, and she should no longer have active Crohn disease. Only a small segment
was resected, and she should not have short bowel syndrome. Though possible,
it would be unusual for her to develop Clostridium difficile colitis or pancreatitis
immediately after surgery.Additionally, pancreatitis should not give her diarrhea.
After resection of the ileum, she has less reuptake of bile acids, which can lead to an
osmotic diarrhea. She developed diarrhea immediately after surgery, which
makes this most likely.

3. A 3-year-old boy comes to your office due to chronic


emesis. He has previously been healthy. Further history
reveals that emesis tends to occur in the morning.
His medical record notes dysmetria on examination last
week, but this was attributed to his young age. What are
your next steps?
a. Physical examination with thorough neurologic
evaluation and fundoscopic examination
b. Upper GI to rule out anatomic abnormality
c. Trial of acid suppression
d. Screening laboratories
e. Referral to neurology
3. c. This 3-year-old boy has morning emesis, which immediately should trigger
concern for possible causes of increased intracranial pressure. Neurologic
examinations can be tough on young patients, but dysmetria was recently noted. The
next best step would be a thorough neurologic examination, including a fundoscopic
examination. If there is concern for increased intracranial pressure, head imaging is
warranted. An upper GI,trial of acid suppression, and screening laboratories
may be helpful, but a neurologic examination needs to be perf armed first to help
narrow the differential for emesis. Referral to neurology seems premature, and
if there is concern for increased intracranial pressure,more emergent attention is
needed.

4. A 2-week-old male is transferred to your hospital due to


the presence of profuse diarrhea and an inability to gain
weight since birth. What is the first step in your diagnostic
evaluation?
a. Upper endoscopy with biopsies
b. Trial of pancreatic enzymes
c. Observe whether diarrhea persists or stops while
patient is fasting
d. Upper GI with small bowel follow-through to evaluate
for congenital short bowel syndrome
e. Celiac panel
4. c. This patient has congenital diarrhea, which has along list of possible etiologies.
The first and least invasive step is to see what happens to the diarrhea if the
patient is not fed. This will help distinguish between osmotic and secretory diarrhea,
which will help narrow the differential. An upper endoscopy or an upper GI
may be needed in the future. A celiac panel will not be helpful because this infant
would not be exposed to gluten to have celiac disease. A trial of pancreatic enzymes
is also less likely to be helpful until it has been established that there is a reason for
pancreatic insufficiency.
5. Which of the following results is not consistent with carbohydrate
malabsorption?
a. Stool pH <5.5
b. Positive stool-reducing substances
c. Positive hydrogen breath test after administration of
lactose
d. Stool osmotic gap <SO mOsm/L
e. Increased flatus
5. d. When carbohydrates are not absorbed, they are fermented by bacteria present in
our gastrointestinal tract, which leads to increased gas production. The increased
production of gas, including hydrogen and methane, leads to a positive hydrogen
breath test and also increased flatus. The breakdown of carbohydrates
by bacteria also results in the production of short-chain fatty acids, which decreases
stool pH to less than S.S. Stool-reducing substances are also an indicator of
carbohydrate malabsorption. Stool osmotic gap is used to distinguish osmotic and
secretory diarrhea. It does not indicate which dietary component is malabsorbed.
6. A 2-month-old baby boy presents to the emergency room
with a week of large-volume nonbilious emesis after
every meal. Physical examination reveals an olive-like
mass in the upper abdomen. His abdominal x-ray demonstrates
a distended stomach. Which of the following
electrolyte abnormalities is this baby most likely to have?
a. Hypochloremic, hypokalemic metabolic alkalosis
b. Hyperchloremic, hypokalemic metabolic acidosis
c. Hypochloremic, hyperkalemic metabolic alkalosis
d. Hypochloremic, hyperkalemic metabolic acidosis
e. Hyperchloremic, hyperkalemic metabolic alkalosis
6. a. In any form of gastric outlet obstruction, classically due to pyloric stenosis, loss
of hydrochloric acid from the stomach results in hypochloremia and a metabolic
alkalosis. In trying to compensate for volume contraction and metabolic alkalosis, the
kidneys retain sodium and excrete potassium and hydrogen ions. The patient
will have a hypochloremic, hypokalemic metabolic alkalosis.
7. A female full-term neonate has been vomiting copious
amounts of breast milk since birth. There was a maternal
history ofpolyhydramnios. What is the best next step?
a. Two-view abdominal x-ray
b. Continue to try to feed breast milk
c. Surgical consult
d. Placement of nasogastric tube for feeding
e. Abdominal ultrasound
7. a. This patient has had high-volume emesis since birth.Additionally,
polyhydramnios in utero may suggest a GI tract obstruction that did not allow the
fetus to swallow amniotic fluid. GI tract obstruction needs to be considered in this
situation, and a two-view abdominal x-ray can serve as an initial screen for this.
Continuing to feed orally or via nasogastric tube will result in more emesis. A surgical
consult is likely needed, but further assessment can be done first. An abdominal
ultrasound is not likely to be as helpful to evaluate for GI tract obstruction in this
scenario.
8. A teenage girl presents to your office with diarrhea for the past 6 weeks. She
reports that ever since she came home from a camping trip, she acutely developed
diarrhea.Upon further questioning, she tells you that she drank from a stream. She has
been having abdominal pain. There is no blood or mucus in the stool. She has
had weight loss. Her abdominal examination is notable for generalized abdominal
tenderness. What is the most likely cause of her symptoms?
a. Lactose intolerance
b. Celiac disease
c. Inflammatory bowel disease
d. Infectious gastroenteritis
e. Irritable bowel syndrome
8. d. This teenage girl presents with chronic diarrhea of acute onset after a camping
stream with exposure to contaminated drinking water. An infectious etiology,
especially Giardia, is most likely in this scenario. Celiac disease and inflammatory
bowel disease can both present as chronic diarrhea with abdominal pain, but they
tend to have a gradual onset. Irritable bowel syndrome can also present with diarrhea,
but it does not tend to cause weight loss within a short period of time.
9. A 3-month-old baby girl presents to your office with nonbilious emesis that started
soon after birth. She vomits after most feeds. She is happy, developing normally, and
gaining weight well. What are the next steps in management?
a. Upper endoscopy with biopsies
b. Change to an elemental formula
c. Abdominal ultrasound
d. Initiation of metoclopramide
e. Reassurance
9. e. This baby is a "happy spitter" and has reflux, the most common cause of emesis
in this age group. She has emesis with every feed but is feeding well and growing
well. Further testing or a formula change is not warranted because this is a problem
that should improve with time. Testing is likely to be normal.Though initiation of
ranitidine could be considered, it is not necessary. Certainly, a physician would not
start with metoclopramide for emesis due to a higher side effect profile than
ranitidine. The best answer is reassurance that the infant will outgrow the reflux with
time.

10. A 15-year-old boy presents to your office with several months of diarrhea. Upon
further questioning, he reveals that he has been having right lower quadrant
abdominal pain and blood in his stool. His energy level is low. His growth chart
demonstrates weight loss. On examination, he has focal right lower quadrant
tenderness and pallor. Diagnostic testing is most likely to reveal which of the
following:
a. Low fecal elastase
b. Elevated stool pH
c. Elevated fecal calprotectin
d. Elevated serum creatine kinase
e. Elevated serum gastrin
10. c. This patient presents with chronic diarrhea, localized right lower quadrant pain,
blood stool, weight loss, and fatigue. His clinical picture is most consistent
with inflammatory bowel disease. His fatigue is suggestive of active disease but also
anemia. His localized right lower quadrant pain suggests ileocecal disease,
Review Questions
1. A 10-week-old ex-38-week exclusively breastfed infant presents to
your outpatient pediatric office for asick visit. Her mother reports that she
has had browncolored emesis for the past 24 hours. She is otherwise
well-appearing. She has a normal physical examination in your office;
however, she has an episode of darkbrown emesis while you are in the
room. A Gastroccult test of the emesis is positive for blood. What is the
most likely etiology of this brown-colored emesis?
a. N ecrotizing enterocolitis
b. Swallowed maternal blood
c. Volvulus
d. Gastric vascular lesion
e. Milk protein allergy
1. b. An otherwise healthy breastfed infant with Gastroccult positive
emesis and a normal examination is most likely to have swallowed
maternal blood from a mother's cracked nipple during breastfeeding. Use
of an Apt test can aid in distinguishing whether the blood is the
infant's blood or her mother's blood.
2. A 6-year-old male presents to your office for evaluation of a 3-month
history of abdominal pain and a 2-week history of blood in the stool. He
is passing pellet-like stools approximately every 3 to 4 days. He is
resistant to sitting on the toilet and often cries while he is passing stool.
He reports that the abdominal pain is often relieved with stooling. His
weight for age has remained in the 50th percentile over the past 3 years.
He has a good appetite.He is not on any medications at this time. His
mother has noticed that he has had streaks of bright-red blood on top
of his stools intermittently over the past several weeks.What is the most
likely etiology for the blood in his stool?
a. Anal fissure
b. Juvenile polyp
c. Crohn disease
d. Infectious colitis
e. Intussusception
2. a. Functional constipation is a common cause of abdominal pain in a
toddler or school-aged patient.Children with constipation will often strain
to stool.Chronic straining can lead to anal fissures, which often
result in streaks of bright-red blood in the stool.
3. A 2-year-old previously healthy male presents to the emergency
department with sudden-onset brisk, painless rectal bleeding. Which of
the fallowing studies is most likely to diagnose the source of the
bleeding?
a. Abdominal x-ray
b. Abdominal ultrasound
c. Meckel scan
d. Clostridium difficile toxin testing
e. Hemoccult
3. c. Meckel diverticulum most commonly presents with brisk, painless
rectal bleeding. An abdominal ultrasound and abdominal x-ray imaging
would be unlikely to be helpful in the diagnosis of a Meckel
diverticulum.Clostridium difficile colitis is often associated with
abdominal cramping and diarrhea. Although the hemoccult would be
positive, it would not be diagnostic for this patient.

4. What is the most important initial management step if a child presents


to the emergency department with an acute gastrointestinal hemorrhage?
a. Perform an upper endoscopy
b. Perform a bleeding scan
c. Ensure hemodynamic stabilization
d. Start IV proton pump inhibitor therapy
e. Place a nasogastric tube
4. c. The most important initial management step in an acute
gastrointestinal hemorrhage is to resuscitate the patient and ensure
hemodynamic stabilization.This should be done before any additional
procedures or imaging studies. It should be noted that initial CBC
will likely not be reflective of total volume of blood loss as hemoglobin
measure will not drop until there is re-equilibration of fluid amongst the
body's compartments.Consequently, it is imperative that packed red
blood cells be administered empirically at least at the rate of blood loss,
ideally warmed and with adequate plasma components to prevent
coagulopathy (e.g.,fresh frozen plasma, cryoprecipitate).

5. You are caring for a 12-year-old patient who has a4-year history of
inflammatory bowel disease. She is currently anemic and has been having
intermittently bloody stools for the past 1 month. She also reports
intermittent cramping abdominal pain for the past 1month. She is
currently maintained on oral Pentasa for her inflammatory bowel disease
maintenance therapy.An upper endoscopy and colonoscopy were
performed by her gastroenterologist last week and were both visually
normal. You suspect that she may have a small bowel lesion, and you
would like to do additional testing at this time. What would be the next
study that you or the gastroenterologist should order?
a. Capsule endoscopy
b. Patency capsule
c. Abdominal x-ray
d. Bleeding scan
e. Upper GI endoscopy
5. b. A patient with a several-year history of inflammatory bowel disease
is at increased risk for a posb sible intestinal stricture. Before performing
a capsule endoscopy to look for a small-intestinal lesion, the patient hould
have a patency capsule test to ensure that the capsule will pass through
the intestine. If the patency capsule does not pass through with ease,
then the patient should not proceed with the capsule endoscopy.

6. A false positive hemoccult may be due to:


a. Iron supplements
b. Ascorbic acid
c. Orange juice
d. Redmeat
6. d. Red meat ingestion can yield a false positive hemoccult result. Iron
supplements do not produce a false positive test result. Ascorbic acid and
orange juice can produce a false negative hemoccult result.
7. An 8-year-old previously healthy female presents to the emergency
department after an episode of brightred vomit this morning. She has had
several days of previously nonbloody vomiting, nonbloody diarrhea,
and fever. She has been vomiting approximately four to five times per
day for the past 3 days. Several children in her class are out sick as well.
What is the most likely cause of her bloody vomit?
a. Stress ulcer
b. Esophageal varices
c. Vascular malformation
d. Toxic ingestion
e. Mallory-Weiss tear
7. e. Mallory-Weiss tear is the most common cause of sudden-onset
bloody emesis in an otherwise healthy child who has had several days of
acute vomiting.These tears typically heal spontaneously, and the children
make a full recovery.
8. "Currant jelly" stools are typical of what gastrointestinal
disease process?
a. Intussusception
b. Meckel diverticulum
c. Intestinal duplication
d. Clostridium difficile colitis
e. Inflammatory bowel disease
8. a. "Currant jelly" stools are typical of intussusception.In addition to
this mixed bloody and mucusy stool,intussusception is also often
associated with vomiting,lethargy, and intermittent abdominal pain
episodes.
9. All of the following are acceptable potential endoscopic
interventions to perform to stop a gastrointestinal
hemorrhage, except:
a. Injection of sclerosing agent
b. Injection of epinephrine
c. Thermocoagulation
d. Argon plasma coagulation
e. Injection ofBotulinum toxin
9. e. Injection of sclerosing agent (i.e., ethanolamine oleate), injection of
epinephrine, thermocoagulation,and argon plasma coagulation are all
endoscopy interventions used to stop acute gastrointestinal bleeding.
Injection of Botulinum toxin is not used in gastrointestinal bleeds. Rather,
it can be injected around the pyloric sphincter to relax the sphincter and
allow more food to pass through.

10. A 12-year-old boy presents to your pediatric outpatient office for


evaluation of lack of weight gain over the past 1 year. Upon further
questioning, he reports that he has had some intermittent bloody diarrhea
with bright-red blood mixed in with his stools for the past 3months. He
has also had daily periumbilical abdominal pain for the past several
months. His examination is significant for pallor, diffuse tenderness to
palpation throughout the abdomen with no rebound tenderness and no
guarding, and multiple skin tags around the anus. What is the most likely
diagnosis?
a. Crohn Diseas
b.Ulcerative colitis
c. Juvenile polyps
d. Anal fissure
e. Meckel diverticulum
10. a. Poor weight gain, bloody stool, abdominal pain,and perianal lesions
raise suspicion for Crohn disease.Ulcerative colitis, juvenile polyps, anal
fissure, and Meckel diverticulum are not typically associated with
perianal disease.
Dr-Wahid Helmi

Question 1:
A 14-year-old youth has had intermittent periumbilical and lower abdominal pain, some bloating,
and increased flatus for 2 years. He denies a relationship of the pain to food or drink, and a milk-free
diet did not relieve his symptoms. He has had no fevers, diarrhea, constipation, bleeding, or other
systemic problems and his growth is normal. He eats a regular diet, including diet soft drinks and
sugar-free gum. His family is intact and he denies undue stress in his school or social life. He also
denies alcohol, drug use, or smoking. Physical examination is normal, including heme occult
negative rectal examination. Screening laboratory tests including complete blood count, ESR, liver
enzymes and amylase, and urinalysis are normal. Which one of the following would be the most
likely etiology of the abdominal complaints in this teenager?
A. lactose intolerance
B. excessive sorbitol intake
C. irritable bowel syndrome
D. teenage stresses
E. acid-peptic disease

Suggested answer: B or C. Sorbitol is a FODMAP (fermentable oligo-, di-, and


monosaccharides and polyols), metabolized by colonic bacteria to produce short chain fatty
acids and gas. Sorbitol is commonly found in diet soft drinks and sugar-free gum. When taken
in excess, sorbitol produces enough gas to produce abdominal pain and bloating as in this
patient. Sorbitol can also cause a persistent, osmotic diarrhea.

Irritable bowel syndrome, the most commonly diagnosed gastrointestinal condition, is diagnosed
by the presence of 3 things: i) chronic abdominal pain, ii) altered bowel habits (constipation or
diarrhea, which is may or may not be present in this patient), and iii) no organic cause. The
etiology of IBS in unclear; however, one hypothesis is that bacteria cause IBS, by fermenting
FODMAPs in the colon. FODMAPs have been shown to produce IBS symptoms in clinical
trials. Treatment for IBS is broad, and includes a strong physician-patient relationship,
avoiding certain foods (lactose, allergens, gluten), anti-spasmodics, anti-depressants, anti-
diarrheal agents, and/or antibiotics.

This patient does not have lactose intolerance, as a milk-free diet did not relieve his symptoms.
He should resume drinking milk, or at least receive calcium and Vitamin D supplements to
support growth. In addition, the patient does not report excessive stress, and does not have
typical GERD symptoms.

Question 2:
A 9-year-old girl has been losing hair for about 2 years and now presents with a large mass in her
epigastrum. Her parents separated about 3 years ago. The patient and her siblings have been
spending part of each week at each parent's home. A plain film of the abdomen reveals a large
mass in the stomach. Endoscopy reveals a trichobezoar. What do you now recommend to the
family?
A. administer meat tenderizer orally
B. endoscopic removal of bezoar under anesthesia
C. abdominal CT scan to rule out gastric tumor
D. surgical consultation for removal of bezoar
Dr-Wahid Helmi

E. reassurance that the bezoar will pass

Suggested answer: D. Trichobezoars usually occur in young women with psychiatric disorders,
and in some cases may extend through the small bowel and even cecum (Rapunzel syndrome).
Phytobezoars (from vegetable matter) are responsive to enzymatic dissolution, including
cellulase, papain, and carbonated soda. Trichobezoars are resistant to enzymes and must be
removed manually. Endoscopy can be attempted but often fails, with the endoscopic tools
becoming snared in the strands of hair. Surgical removal is more effective. All patients should
receive psychiatric therapy to prevent the problem from reoccurring.

Question 3:
Among the hepatitis viruses, the hepatitis B virus (HBV) is unique because:
A. liver injury is mediated through the immune system
B. the presence of antibodies to HBV indicates protective immunity
C. it has similarities to human retroviruses because it is a DNA virus which replicates
through an RNA intermediate
D. it is the only agent to cause fulminant hepatic failure
E. it has a glycoprotein coat enclosing a viral nucleocapsid

Suggested answer: C. HBV is a DNA virus that enters the hepatocyte nucleus and is read by
host-cell machinery to make RNA particles. The RNA particles in turn are exported to the
cytoplasm, where HBV-encoded reverse transcriptase converts them to new viral DNA particles.
Infection can be acute and cause fulminant hepatic failure; however, chronic infections
characterized by a series of stages are more common. First, in the “immune tolerant” phase,
the host allows HBV infection and replication as seen by high HBV DNA levels and HBeAg
positivity. Second, in the “immune clearance” phase, the host mounts an immune response
against infected hepatocytes, signified by the appearance of anti-HBe, disappearance of HBe-Ag,
transaminase elevation, and hepatitis symptoms. In some cases, the host clears the virus
completely (anti-HBs appears, HBsAg disappears); in other cases, the virus becomes latent and
can reactivate/stimulate the “immune clearance” phase at unpredictable times. The virus has a
glycoprotein envelop enclosing a proteinaceous nucleocapsid.

HAV is an RNA virus that never enters the nucleus; rather, in the cytoplasm, it makes more
particles using its own RNA-dependent RNA polymerase. Infections are acute and can cause
fulminant hepatic failure. Infections have a biphasic pattern, first with viral replication (akin to
the HBV “immune tolerant” phase”) quickly followed by immune attack of virus and cells
harboring the virus (akin to the HBV “immune clearance” phase, and characterized by
appearance of anti-Hep A IgM and hepatitis symptoms). The virus does not have an envelope,
but does have a proteinaceous nucleocapsid surrounding the RNA.

HCV is an RNA virus like HAV but has an envelope. It also never enters the nucleus, but rather
enters the cytoplasm, takes over the cell’s ribosomal machinery, and converts the cell into a
factory producing viral proteins that generate more viral nucleic acid and particles. Viral
particles then bud off the cells (using the cell’s plasma membrane for an envelope) and infect
other cells. Acute infections are mild or even asymptomatic, and fulminant hepatic failure is
rare. Chronic infections are more common, and occur when the host cannot completely clear
Dr-Wahid Helmi

the infection. This long-term, low-level battle between host and virus leads to liver
inflammation, cirrhosis, and hepatocellular carcinoma.

HDV is an RNA virus with an envelope like HCV; however, the envelope has HBsAg, making
HDV replication possible only in cells also infected with HBV. HDV enters the nucleus, and
host-cell RNA polymerases use the original HDV strand to make more RNA particles. Acutely,
HDV is thought to cause cytopathic damage to hepatocytes and may be confused with HBV
reactivation (see above). Fulminant hepatic failure is possible. Chronically, the host immune
response to HDV infected cells causes hepatocyte damage.

HEV is an RNA virus without an envelope, similar to HAV. The life cycle is less well-
understood, but HEV appears to never enter the nucleus (similar to HAV). Instead, in the
cytoplasm, the virus makes more particles using its own RNA-dependent RNA polymerase.
Infections are usually acute and can cause fulminant hepatic failure. Chronic infections have
only been reported in solid-organ recipients taking post-transplant immunosuppression.

Question 4:
Common routes of the spread of hepatitis A virus include all of the following except:
A. consumption of contaminated water or food
B. close personal contact
C. infants in daycare centers
D. homosexual men by sexual contact
E. transfusion of packed red blood cells

Suggested answer: E. Hepatitis A is shed in bile, found in stools, and spread fecal-orally.
Hence, it can be spread through consumption of contaminated water/food, close personal
contact (without proper hand-washing), infants in close proximity in a day care, and other close
contact such as anal-oral sexual contact. Interestingly, infants that acquire HAV infection have
a less severe course, perhaps because of a less developed immune system. HAV is not spread
through blood transfusions.

Question 5:
Serological changes associated with being a healthy carrier of the hepatitis B virus are:
A. HBsAg positive, HBeAg positive, HBV DNA negative by hybridization assay, with
normal serum aminotransferases
B. HBsAg positive, HBV DNA negative by hybridization assay, anti-HBe positive, with
normal serum aminotransferases
C. HBsAg negative, anti-HBs positive, anti-HBe positive
D. HBsAg negative, anti-HBs negative, anti-HBe positive
E. HBsAg positive, HBV DNA positive by hybridization assay, HBeAg negative, with only
slightly elevated serum aminotransferases

Suggested answer: E. Inactive carries are those who were once infected with Hepatitis B and
mounted a successful immune response against it. During the infection or “immune tolerant”
phase, these patients had HBV antigens and DNA in their serum (HBsAg positive, HBeAg
positive, HBV DNA virus positive) but have not yet mounted an immune response (anti-HBe
Dr-Wahid Helmi

negative as well as anti-HBs negative). During the immune response or “immune clearance”
phase, these patients developed antibodies against the virus (anti-HBe positive) as well as a
clinical picture of hepatitis (elevated transaminases). Their immune response is effective but not
complete, rendering them carriers with evidence of the virus still present (HBsAg positive, HBV
DNA hybridization positive) but also evidence of a successful response (anti-HBe positive
causing HBeAg negativity). Carriers may also have slightly elevated aminotransferases. Had
their immune response been complete, the patients would be cured of the disease and be anti-
HBs positive, HBsAg negative, anti-HBe positive, HBeAg negative, and HBV DNA negative.

Question 6:
All of the following statements about hepatitis E are true, except:
A. outbreaks of hepatitis E tend to be very large because of the high rate of secondary (case-
to-case) spread
B. cases of hepatitis E in the United States are rare
C. infection with hepatitis E virus (HEV) in pregnancy is associated with high mortality rate
D. anti-HEV appears to be protective, and prospects for developing a vaccine are good
E. HEV is not closely related in structure or function to any of the other viral hepatitis
agents

Suggested answer: A. While the seroprevalence of HEV is ~20% in the US, symptomatic HEV
infection is rare (perhaps because most infections may be with the less virulent genotype 3).
HEV is acquired through contaminated water and rarely spreads person-to-person, making it
less readily transmissible than HAV. Following HEV infections, humans develop anti-HEV IgM
and then anti-HEV IgG. There are no HEV vaccines readily available yet, though some are
being developed. HEV has a unique genome and structure and is the only member of the genus
hepevirus in the family Hepeviridae.

HEV is most dangerous during pregnancy. For unclear reasons, women infected in the third
trimester have an increased risk of hepatic failure and mortality (15-25% mortality rate). This
may because the virus replicates faster in the pregnancy state. Furthermore, infants born to
infected mothers are also at risk, with some presenting with massive hepatic necrosis shortly
after birth.

Question 7:
Recognized complications of acute hepatitis A include all of the following except:
A. fulminant hepatic failure
B. relapsing hepatitis C
C. chronic hepatitis
D. cholestatic hepatitis
E. triggering autoimmune hepatitis

Suggested answer: B. HAV most commonly causes an acute, self-resolving hepatitis. However,
it has been associated with a variety of other presentations: i) fulminant hepatic failure (rare);
ii) relapsing, chronic hepatitis characterized by HAV-caused hepatitis interspersed between long
periods without symptoms (prognosis is excellent, with no reports of cirrhosis or chronic liver
disease); iii) cholestatic hepatitis characterized by prolonged jaundice, itching, and laboratory
Dr-Wahid Helmi

abnormalities, which eventually self-resolves with supportive care; and iv) triggering of
autoimmune hepatitis, which develops months after the initial HAV infection.

Question 8:
A 45-year-old woman is undergoing treatment for chronic hepatitis C with the combination of
interferon (3 three times a week) and ribavirin (1000 mg per day). Prior to therapy she was
found to have cirrhosis but there was no evidence of hepatic decompensation. Her baseline blood
count was hemoglobin 13.9 g/dL, MCV 89, total white cell count 4200/mm3, platelet count
92,000/mm3. After 6 weeks of therapy, her blood count was hemoglobin 9.9 g/dL, MCV 102,
total white cell count 2500/mm3, platelet count 47,000/mm3. The best course of action is:
A. decrease the dose of ribavirin to 600 mg/day
B. decrease the dose of interferon to 1.5 three times a week
C. administer recombinant human erythropoietin
D. decrease the dose of both ribavirin (to 600 mg/day) and interferon (to 1.5 three times a
week)
E. check serum levels of folate and vitamin B12 and correct if deficient

Suggested answer: D. This patient is being treated with interferon and ribivarin, which implies
she has either genotype 2, 3, or 4 (genotype 1 is treated with interferon, ribivarin, and a protease
inhibitor). Interferon strengthens the innate and adaptive immune response, whereas ribivarin is
a nucleoside analog.

80% of patients taking interferon and ribivarin have side-effects. Anemia usually appears in the
first 12 weeks, caused by ribivarin-induced hemolysis (ribivarin is concentrated in erythrocytes
leading to oxidative damage) and interferon-related bone marrow suppression to blunt a
compensatory response. Neutropenia and thrombocytopenia occur soon after treatment
initiation and are due to interferon-related bone marrow suppression. Ribivarin dose reduction
would treat the anemia, whereas Interferon dose reduction would treat the thrombocytopenia.
Erythropoietin is another medication that has been shown to improve the anemia, possibly
without needing to reduce the ribivarin dose.

Question 9:
An 18-year-old Asian woman is being treated for hepatitis B. Prior to therapy she was found to
have ALT 198 U/L, AST 91 U/L, normal bilirubin, albumin, and prothrombin time. Liver biopsy
results showed chronic hepatitis B, grade 3, stage 3. After 12 weeks of therapy, serum ALT is
found to have increased to 1,082 U/L, bilirubin 2.1 mg/dL but albumin and prothrombin time
remain normal. Apart from some fatigue, the patient is tolerating interferon well. The best course
of action is:
A. check for antinuclear antibodies and total immunoglobulin level in serum and consider
instituting corticosteroid therapy
B. stop interferon
C. recheck lab work again in 2 weeks time
D. add lamivudine to the regimen
E. ask the patient to skip three scheduled doses of interferon
Dr-Wahid Helmi

Suggested answer: C. Interferon and the antiviral lamivudine are used to treat HBV infection in
children. Interferon is dosed for 16 weeks, has numerous side effects, and leads to serological
conversion (presence of anti-HBe positive and HBeAg negative) in 40% (genotypes A and B) to
5-15% (genotypes C and D) cases. Lamivudine is taken indefinitely and has similar rates of
seroconversion after 2 years (~30-35%). Lamivudine, however, induces viral resistance, with
rates greater than 60% reported after 3 years of use.

This patient is taking interferon. Interferon causes a rise in AST/ALT in 30-40% of cases, likely
because it stimulates the immune system to destroy infected hepatocytes. Hence, this rise is a
sign the medication is working. Had the patient been taking lamivudine, a rise in AST/ALT could
also signify development of viral resistance. Interferon does induce autoantibodies that can be
symptomatic (i.e. hypothyroidism, hyperthyroidism) but this is less common. Interferon should
not be stopped, because the patient’s fatigue is only mild. Adding lamivudine to the interferon
regimen may decrease viral load faster, but has not been shown to produce better virological
outcomes after the 16 week course has finished. (On the other hand, combination therapy with
lamivudine plus interferon, versus lamivudine alone, may prevent development of viral
resistance.)

Question 10:
All of the following are primary cellular components of the immune system and the gut mucosa
except:
A. Peyer's patches
B. lamina propria lymphocytes
C. intrapithelial lymphocytes
D. IgG-secreting B lymphocytes
E. IgA-secreting B lymphocytes

Suggested answer: D. The gut is the largest lymphoid organ in the body. Specialized epithelial
cells, called M cells, sit above Peyer’s patches and sense luminal antigens. The M cells then
stimulate naïve B and T cells in the Peyer’s patches. The naïve cells undergo a long maturation
process, during which they take a long circular course to eventually return to the gut mucosa
(Peyer’s patches  regional lymph nodes  lymphatics  thoracic duct  systemic circulation
 exit in the lamina propria). In the lamina propria, they reside and perform their immune
surveillance functions. In addition, special memory T-cells called intraepithelial lymphocytes
are anchored to the epithelial layer. They respond to a subset of luminal antigens, and secrete
cytokines to mediate the inflammatory response.

B cells that mature into plasma cells ultimately residing in the lamina propria have special
functions. They mainly secrete IgA2 and very little IgM and IgE, but virtually no IgG. IgA2
differs from the IgA1 made by plasma cells in the circulation. IgA2 molecules are dimeric and
secreted into the luminal space only after transiting through epithelial cells (via endocytosis
followed by exocytosis). Furthermore, IgA2 molecules are essentially non-inflammatory because
they bind antigens but do not activate complement. As a result, IgA2 molecules neutralize
antigens without triggering excessive inflammation to the countless number of gut antigens.

Question 11:
Dr-Wahid Helmi

Cells from intestinal lymphoid tissues migrate to all of the following immune tissues except:
A. gastrointestinal mucosal immune tissues
B. pulmonary mucosal immune tissues
C. genitourinary tract
D. lactating mammary glands
E. skin

Suggested answer: E. The “common mucosal immune system” refers to the interconnected
mucosal organs that house IgA2 secreting plasma cells (which initially started as naïve B cells in
Peyer’s patches). These mucosal organs include the lamina propria of the small intestine, the
salivary and lacrimal glands, the lactating mammary glands, the genitourinary tract, and the
lungs. Migration to these areas is mediated by specific cell adhesion molecules.

Question 12:
Which one of the following most accurately reflects the epidemiologic features of inflammatory
bowel disease?
A. the prevalence is approximately 100 cases per 100,000 general population
B. the prevalence is approximately 1,000 cases per 100,000 general population
C. non-Jews are more likely to develop Crohn's disease than Jews
D. people who smoke are less likely to get Crohn's disease
E. people who smoke are more likely to get ulcerative colitis

Suggested answer: A. The epidemiology of inflammatory bowel disease has been well studied.
In North America, UC has prevalence of 27-246 per 100,000 persons, and Crohn disease has a
prevalence of 26-201 per 100,000 persons. There is a negative correlation between smoking and
UC, and a positive correlation between smoking and Crohn disease recurrence. People of
Jewish descent have a higher risk of developing Crohn disease compared to non-Jews,
highlighting the partly-genetic etiology of the disease.

Question 13:
Which one of the following extraintestinal manifestations of inflammatory bowel disease do not
parallel the course of intestinal inflammation and do not improve in parallel with improvement in
intestinal symptoms?
A. peripheral arthritis
B. apthous ulcers
C. spondylitis and sacroiliitis
D. erythema nodosum
E. uveitis and iritis

Suggested answer: C. Arthritis is the most common extra-intestinal complications of IBD.


Peripheral arthritis usually involves large joints, does not cause synovial destruction, and
parallels the course of intestinal symptoms. Central axial arthritis, such as ankylosing
spondylitis (characterized by back and progressive spinal stiffness) is similar to primary
sclerosing cholangitis in that it does not follow the course of intestinal disease. Treatments for
axial arthritis include NSAIDs (despite concern for inducing worsening intestinal inflammation),
Dr-Wahid Helmi

methotrexate, sulfasalazine, and/or anti-TNF therapy. Apthous ulcers, erythema nodosum,


uveitis, and iritis often (but not always) parallel intestinal disease.

Question 14:
All of the following urinary tract complications may occur as a consequence of Crohn's disease
except:
A. calcium oxalate renal stones
B. calcium phosphate renal stones
C. uric acid renal stones
D. ureteral obstruction due to retroperitoneal fibrosis
E. fistulous communication between the terminal ileum and the bladder

Suggested answer: B. There are two general mechanism by which Crohn disease affects the
renal system. First, transmural inflammation can affect the underlying ureters, causing
occlusion usually on the right side and hydronephrosis. Inflammation can also create fistulas
between inflamed bowel and the bladder, promoting cystitis and urinary tract infections.

Second, Crohn disease is associated with calcium oxalate and uric acid renal stones. Calcium
oxalate stones form when the ileum is affected, preventing proper absorption of fats.
Unabsorbed long-chain fatty acids compete with the insoluble calcium oxalate for calcium.
Without calcium, oxalate binds sodium, becomes soluble, is absorbed by the colon (“enteric
hyperoxaluria”), and eventually re-precipitates as calcium oxalate stones when excreted in the
urine. Calcium is an effective treatment. Uric acid stones form secondary to bicarbonate loss
with diarrhea. With low serum bicarbonate, the kidneys excrete acid in compensation. Uric
acid, which is soluble in alkalotic conditions, precipitates as stones in the low pH urine.

Question 15:
All of the following statements regarding the release of mesalamine (5-aminosalicylate) by the
following delivery systems are true except:
A. the dose form Pentasa releases throughout the small and large intestine beginning in the
duodenum and continuing through to the rectum
B. the dose form Asacol releases beginning in the terminal ileum and cecum and continuing
through to the rectum
C. the dose form olsalazine (Dipentum) releases throughout the colon beginning in the cecum
and extending to the rectum
D. mesalamine administered as a Rowasa suppository extends to the left colon and splenic
flexure
E. no exception

Suggested answer: A. Sulfasalazine is used to treat topical inflammatory bowel disease. It is


metabolized by colonic bacteria into two components: 5-ASA, which has anti-inflammatory
properties, and sulfapyridine, which has anti-bacterial products. More recently 5-ASA products
alone have been developed, because many patients are intolerant to the sulfapyridine component.
Sulfasalazine and 5-ASA behave similarly in trials, suggesting that the sulfapyridine anti-bacterial
function has little significance.
Dr-Wahid Helmi

Because ingested 5-ASA is rapidly absorbed in the jejunum, enemas have been used to deliver
medication directly to the colon. In addition, two delayed release preparations have been made.
The first involves coating 5-ASA with resins or microgranules, which dissolve and release 5-ASA in
settings of pH>7 (distal small bowel and colon). Pentasa and Asacol have this coating. The second
involves dimerizing 5-ASA, so that it is only released after bacterial cleavage (in the colon).
Olsalazine (Dipentum) is an example of this dimer form. Importantly, neither delivery system has
proven more efficacious over the other.

Question 16:
All of the following agents are effective for both induction of remission and maintenance of
remission in patients with ulcerative colitis except:
A. oral mesalamine formulations including Asacol and Pentasa
B. sulfasalazine
C. olsalazine (Dipentum)
D. rectal mesalamine (Rowasa enemas and suppositories)
E. no exception

Suggested answer: E. For mild to moderate ulcerative colitis, sulfasalazine and 5-ASA preparations
have been shown effective in inducing and maintaining remission (compared to placebo controls).
There were no significant differences between sulfasalazine versus 5-ASA, or among the different 5-
ASA preparations. Furthermore, for distal colitis, 5-ASA enemas achieve remission in as many as
90% of cases and maintain remission in as many as 75% of cases. When combined, oral plus enema
therapy is more efficacious in inducing and maintaining remission when compared to either agent
alone.

Question 17:
All of the following side effects associated with azathioprine and 6-mercaptopurine are idiosyncratic
reactions except:
A. pancreatitis
B. leukopenia
C. fever
D. rash

Suggested answer: B. Idiosyncratic reactions are those that occur rarely, are unpredictable, and
are dose-independent. For AZA and 6-MP use, these include pancreatitis, fever, rash, and
pneumonitis. Leukopenia, on the other hand, is an intended side-effect used to reduce host cell
inflammation. Leukopenia occurs when the 6-MP metabolites are shunted away from TPMT-
mediated production of 6-MMP (which causes hepatotoxicity). Instead, 6-MP is metabolized
through a different pathway to 6-TG. 6-TG accumulates in tissues, inhibits purine metabolism and
subsequent DNA/RNA synthesis, and prevents lymphocyte proliferation. If too much, 6-TG can
cause severe leukopenia and dangerous host immunosuppresion.

Question 18:
All of the following statements are true with respect to the treatment of inflammatory bowel
disease with azathioprine and 6-mercaptopurine except:
Dr-Wahid Helmi

A. controlled clinical trials have demonstrated that azathioprine doses of 2.0-2.5 mg/kg/d
and 6-mercaptopurine at doses of 1.0-1.5 mg/kg/d are effective
B. these agents are thought to be relatively slow acting requiring up to 3 months or more to
reach the full clinical effect
C. these agents are effective for the treatment of Crohn's disease including patients with
chronically active disease, patients with fistulas who are steroid dependent and patients
who require maintenance of remission therapy
D. these agents are not useful in patients with ulcerative colitis for maintenance of remission
E. none of the above

Suggested answer: D. Both 6-MP and its pro-drug AZA have been used successfully to treat UC
and Crohn disease. AZA’s molecular weight is >2 times that of 6-MP, and ~88% of AZA
converts to 6-MP, so dosing for AZA is ~2 times that of 6-MP. The drugs require 3-6 months to
achieve their full effect, so they are better at maintaining – rather than inducing – remission.
They have been tested numerous times in clinical trials, with results supporting their ability to
increase remission maintenance by ~40-70% and decrease corticosteroid requirements in ~70%
of patients with corticosteroid-dependent disease.

Question 19:
All of the following statements regarding the use of methotrexate for the treatment of
inflammatory bowel disease are true except:
A. methotrexate administered as a 25 mg intramuscular dose weekly is effective for inducing
remission in patients with active Crohn's disease and for steroid sparing
B. methotrexate administered at doses of 15-25 mg/week orally or intramuscularly is
effective for induction of improvement and remission in patients with active ulcerative
colitis
C. there is no controlled clinical trial data to indicate that methotrexate is effective for
maintenance of remission in patients with Crohn's disease
D. there is no controlled clinical trial data to indicate that methotrexate is effective for
maintenance of remission in patients with ulcerative colitis
E. no exception

Suggested answer: B and C. Methotrexate blocks inflammation, perhaps by inhibiting


methylation reactions vital for immune cells to function and proliferate. Methotrexate has been
best studied in Crohn disease, with results seen with the intramuscular (versus oral) form. In
Crohn disease, intramuscular methotrexate has been found to induce remission (25 mg q week)
and maintain remission (15 mg qweek) in clinical trials. Studies with oral preparations have
been less promising, and positive controlled clinical trial data for UC is lacking. Methotrexate
side effects include nausea, vomiting, and abdominal distress, relieved in part by folic acid.
Chronic methotrexate can also cause liver toxicity, but the doses given for IBD are not high
enough to warrant surveillance liver biopsies.

Question 20:
Which one of the following statements is false with respect to the use of cyclosporine in patients
with inflammatory bowel disease?
Dr-Wahid Helmi

A. cyclosporine administered as a continuous intravenous infusion at a high dose of 4


mg/kg/d is effective for severely active ulcerative colitis
B. cyclosporine administered orally at a dose of 5 mg/kg/d is ineffective for the induction of
improvement or remission in patients with active Crohn's disease
C. cyclosporine administered orally at a dose of 5 mg/kg/d is ineffective for maintenance of
remission in patients with Crohn's disease
D. cyclosporine is slow acting and thus is not useful as a bridge therapy to other slower acting
medications such as 6-mercaptopurine, azathioprine, or methotrexate
E. none of the above

Suggested answer: D. Cyclosporin binds cyclophilins to inhibit calcineurin and subsequent


transcription of pro-inflammatory cytokines. Cyclosporin has a rapid onset of action, allowing it
to be used as a bridge to 6-MP, AZA, or methotrexate therapy. In ulcerative colitis, cyclosporin
(4 mg/kg/day continuous infusion) can prevent colectomy in severe, steroid-refractory cases.
More recent data shows that lower dose infusions (2 mg/kg/day) may have similar potency with a
possible (but not proven) decrease in side-effects. On the other hand, in Cohn disease, low dose
oral cyclosporin (5 mg/kg/day) does not induce remission and higher doses and/or parenteral
administration have yet to be properly tested. Cyclosporin does have significant side effects
limiting its use, including opportunistic infections, nephrotoxicity, seizures (especially if
cholesterol levels are low), peripheral neuropathy, and anaphylaxis.

Question 21:
The gene product of the hemochromatosis locus, HFE is:
A. a component of the ferritin complex which stores iron
B. a surface molecule that associates with the transferrin receptor
C. a divalent cation transport protein
D. a component of the endoplasmic reticulum
E. a subunit of the apotransferrin molecule

Suggested answer: B. Hereditary hemochromatosis is caused by mutations in the HFE (high Fe)
locus. HFE is a transmembrane protein expressed in intestinal crypt and liver cells. It controls
liver absorption in two proposed ways:

i) expressed in crypt cells  in crypts, binds transferrin receptor and promotes uptake of
transferrin-bound iron from the circulation  enterocytes have high transferrin bound
iron  enterocytes down-regulate the apical iron transporter DMT1 (“divalent metal
transporter 1”, a divalent cation transport protein)  enterocytes migrate up to villi but
without DMT1 cannot absorb iron
ii) expressed in liver cells  increases liver hepicidin expression  liver hepicidin travels
to enterocytes and downregulate basal transporter ferroportin  without ferroportin,
iron is absorbed by villi enterocytes but cannot be transported into the circulation 
enterocytes sloughed into lumen and absorbed iron lost

Question 22:
The liver disease that results from mutations in the Wilson's disease gene is associated with
which one of the following?
Dr-Wahid Helmi

A. inability to take up copper from the plasma ceruloplasmin pool


B. dysfunction of a copper transporter that is embedded in the canalicular membrane
C. inability to transport copper from the cytosol to the trans-Golgi network
D. copper-dependent antibody mediated immune reactions
E. abnormal primary structure of the ceruloplasmin molecule

Suggested answer: C. Wilson’s disease is caused by a mutation in the copper-transporting


adenosine triphosphatase (ATPase) gene (ATP7B). ATP7B has at least two functions: i)
coordinates transport of copper from the cytoplasm, through the trans-Golgi network to vesicles,
and eventually out into the bile cannaliculi via exocytosis; and ii) promotes the binding of
copper with apoceruloplasmin to form ceruloplasmin, which is secreted into the bloodstream.
Without ATP7B, copper is neither excreted into the bile nor the circulation (via ceruloplasmin).
As a result, copper levels rise in hepatocytes, ultimately leading to oxidative damage and spilling
of copper into the circulation.

Question 23:
A young adult with a life-long history of mild jaundice, but no bilirubinemia or evidence of
chronic hepatitis or hemolysis is likely to have a genetic defect in:
A. sinusoidal bilirubin uptake pump
B. MRP2 (canalicular multispecific organic anion transporter)
C. bilirubin-UGT
D. UDP glucuronic acid synthetase
E. cholesterol 7-á-hydroxylase

Suggested answer: C. This patient has Gilbert’s syndrome, which is caused by reduced uridine
diphosphoglucuronate glucuronosyltransferase (UGT) activity. Normally, hepatocytes take up
unconjugated bilirubin through the sinusoidal space. Hepatocytes then conjugated the bilirubin
using bilirubin-UGT and excrete it into the cannalicular space. This process is independent of
bile acid secretion, so isolated defects in bilirubin production/secretion should not lead to the
liver damage often seen with defects in bile acid secretion.

The bilirubin conjugation process can be impaired at many steps. First, hepatocyte uptake of
unconjugated bilirubin can be blocked by drugs. Second, hepatocyte conjugation can be
impaired by defects in UGT activity. Gilbert’s syndrome is caused by mutations in the
regulatory elements controlling UGT expression and has mild, jaundice phenotypes. Crigler-
Najjar syndrome, on the other hand, is caused by mutations in the gene itself, and can result in
severe kernicterus-like phenotypes. Third, hepatocytes can have impaired secretion of bilirubin
after it is conjugated. Dubin-Johnson syndrome results from mutations in MRP-2, which
transports conjugated bilirubin from the cytoplasm to the cannalicular space. Rotor syndrome
results from defective hepatocyte storage of conjugated bilirubin, leading to leakage into the
cytoplasm.

Question 24:
Alagille syndrome is differentiated from other causes of cholestasis by:
A. typical facies and cardiovascular abnormalities
B. high unconjugated bilirubin levels
Dr-Wahid Helmi

C. Unusually high transaminases plus high alkaline phosphatase


D. high hepatic copper content
E. the frequent finding of coexistent pulmonary fibrosis

Suggested answer: A. Alagille’s syndrome is an autosomal dominant disease in the Notch


signaling pathway. It has an incidence of 1:40,000 to 1:100,000, with variable penetrance and
variable presentation. Common findings include paucity of bile ducts, characteristic facies
(triangular face with pointed chin and broad forehead), pulmonic stenosis, Tetrology of Fallot,
posterior embryotoxin, and butterfly vertebrate. Conjugated bilirubin, not unconjugated
bilirubin levels, can be high, with pruritis often the most bothersome symptom. High hepatic
content is more characteristic of Wilson’s disease (though Alagille’s patients may have
increased copper staining on biopsies, secondary to poor excretion), and Hepatitis C infection is
associated with increased risk of idiopathic pulmonary fibrosis.

Question 25:
Alpha- l-antitrypsin deficiency leads to liver injury by way of:
A. uncontrolled proteolytic enzyme activity in the portal tracts
B. chronic pancreatitis and focal biliary cirrhosis
C. inability to transport divalent cations into the endoplasmic reticulum
D. accumulation of abnormal glycoprotein in the liver cells
E. Pulmonary fibrosis and the development of cardiac cirrhosis

Suggested answer: D. A1AT “deficiency” refers to a deficiency of A1AT in lung tissue, leading
to unchecked elastase activity and pulmonary damage over many years. A1AT is produced in
and secreted by hepatocytes. However, in many forms of A1AT such as Z and M alleles, a
misfolded protein is made that polymerizes in the endoplasmic reticulum, cannot be secreted,
and accumulates abnormally in hepatocytes. This, in turn, can lead to liver damage.
Accumulated A1AT protein can be detected by with periodic acid-Schiff (PAS) reagent staining
on liver biopsy. Importantly, patients with deletions in the A1AT gene will not have liver
disease, because there will be no protein to abnormally accumulate.
Dr-Wahid Helmi

Question 1:
A 10 month old boy undergoes resection of hepatoblastoma limited to the right lobe of his liver. He has
no underlying liver disease and completed his chemotherapy prior to surgery without complication.

Assuming that his post-operative course is unremarkable, which of the following best describes the
regenerative response of the residual liver segment to resection.

A. The liver mass will be restored only after infusion of hepatocyte stem cells
B. The liver mass will be restored only after the remaining hepatocytes have undergone 12 rounds of cell
doubling
C. The liver mass will be stored only after the remaining hepatocytes have undergone 1-2 rounds of
replication
D. The liver mass will be restored only after infusion of recombinant hepatocyte growth factor
E. The liver mass will be restored only after infusion of TNF-

Suggested answer: C. Liver regeneration is thought to proceed through one of two routes: i) in healthy
tissue, such as following resection, proliferation of all cell types; and ii) in diseased tissue proliferation of
stellate cells, oval cells, or other cells with stem-cell properties. In the above question, the cells in the
remaining normal liver (hepatocytes, cholangiocytes, endothelial cells, etc) will divide soon after
resection to restore the missing liver mass. 1-2 rounds of replication is sufficient to double the liver
mass.

Hepatic growth factor has mitogenic, anti-apoptotic, and anti-inflammatory properties, and in
experimental settings does promote liver regeneration. TNF-alpha, among other things, promotes
hepatic endothelial cell proliferation and also has been used in experiments to promote the regenerative
program. However, neither compound has proven clinical utility in regeneration, and neither is
administered following liver resection.

For a review, see Michalopoulos GK. Liver Regeneration. J Cell Physiol. 2007 Nov; 213(2):286-300.

Question 2:
An 8 year old young girl is referred to you for evaluation of increased weight. The patient and her parents
want to know her risk for developing obesity in adulthood. She has no other complaints. Her school
performance is good and she gets along well with peers. Her mother is 34 years old and her body mass
index is 24.5. Her father is 36 years old and his body mass index is 32. Physical examination shows that
her body mass index is greater than 95th percentile for age and sex. Examination is otherwise normal and
she has no biochemical evidence of endocrine disease.

Of the following, what is the most accurate answer to the parent's question.

A. Her risk of obesity as an adult is 1 in 5, similar to that of the general population


B. She has a 2 fold increased risk of obesity by age 30 compared to her peers
C. Her increased weight likely reflects the prepubertal growth surge.
D. Weight before puberty does not correlate with risk of obesity during adulthood.
E. Since one of her parents is obese, she has an 90% likelihood of obesity by age 30

Suggested answer: B. Many childhood factors have been studied to determine whether they predict adult
obesity, including childhood obesity after age 3, parental obesity, television time, birthweight, and in
utero insults. The patient in this question has two important risk factors: she is obese (BMI > 95%) and
at least one of her parents is obese. As a result, her risk for obesity increases above that of the
Dr-Wahid Helmi

population. The exact increase may be anywhere from 2-4 times depending on the population studied. A
child having one obese child is thought to have a 50% increased chance of becoming obese as an adult.

See Whitaker RC, Wright JA, Pepe MS, Seidel KD, Dietz WH. Predicting obesity in young adulthood
from childhood and parental obesity. N Engl J Med. 1997 Sep 25;337(13):869-73.

Question 3:
The emergency room calls about a 2 year old who swallowed some toilet bowl cleaner. Mom says she
was cleaning the bathroom and thought the toddler was in another room. She turned around and saw her
son with the bottle at his lips. He did some coughing, crying, and spitting. 2 hours later, physicians in the
ED say that the lips look red and perhaps there is a burn on the posterior oropharynx. The child appears
well. The next appropriate step would be:

A. send home with follow up in GI clinic in 1 month


B. send home with UGI in 1 month and follow up as needed
C. observe in ED for 6 hours and if no symptoms send home
D. admit NPO with EGD the next morning
E. admit NPO on steroids and antibiotics with EGD the next morning.

Suggested answer: D. Toilet bowl cleaners are acidic, allowing them to dissolve stains made from
minerals found in pipe water. Acid ingestions cause coagulation necrosis. Acids pass quickly through
the GI tract, so symptoms commonly occur in the stomach and small intestine (though oropharynx and
esophageal lesions can also occur). EGD should be performed within 12-36 hours of the ingestion (the
delay allows time for some damage to occur, so that it can be visualized during endoscopy).

In contrast to acid ingestions, base ingestions cause liquefaction necrosis. The base reacts with stomach
acid to create heat, which burns the mucosa. Symptoms can present in the oropharynx; however, lack of
oropharynx findings does not mean that more distal (i.e. esophagus) mucosa is intact. Alkaline
ingestions have a 1000-fold lifetime risk of developing esophageal carcinoma.

Question 4:
A 13 year old young woman developed recurrent seizures one year after receiving a liver transplant one
year ago because of acute liver failure. She has been started on treatment with dilantin after she was
determined to have an idiopathic seizure disorder. The family is concerned that dilantin may affect the
metabolism of tacrolimus. Which of the following is the most likely effect of dilantin on tacrolimus
pharamcokinetics?

A. Decreased tacrolimus level due to competition with dilantin for intestinal absorption
B. No effect
C. Decreased tacrolimus level due to activation of the CYP3A4 by dilantin
D. Increased tacrolimus level due to inhibition of the CYP3A4 by dilantin
E. Increased tacrolimus level due to competition with dilantin for renal excretion

Suggested answer: C. Numerous medications affect the cytochrome P450 3A4 system, the major enzymes
involved in clearing Tacrolimus. Azole antifungals, calcium channel blockers, and macrolides inhibit the
P450 system, thereby increasing the Tacrolimus levels. Antiseizure agents (such as dilantin) and anti-TB
agents can induce the P450 system and decrease Tacrolimus levels.

Question 5:
Dr-Wahid Helmi

A 16 year old young woman with steroid-dependent Crohn's disease has a 3 week history of increased
diarrhea and weight loss. Her disease is well-controlled when she is taking prednisone at 40 mg each day.
When her dose of prednisone is decreased below 20 mg each day, she has an exacerbation of symptoms.
On examination, her body mass index is 17 and she has mild tenderness to palpation throughout her
abdomen. Endoscopic and radiologic evaluation show inflammatory disease primarily in the jejunum and
ileum. You recommend that treatment be started with mercaptopurine and discuss potential nutritional
intervention. Which of the following nutritional interventions is most likely to promote recovery?

A. Complete bowel rest and institution of total parenteral nutrition


B. Complete bowel rest and institution of peripheral parenteral nutrition
C. Oral zinc supplementation
D. Elemental diet delivered by nasogastric tube
E. Supplement present diet with a lactose free formula

Suggested answer D. This patient is having a flare of small bowel Crohn’s. There are two things to
consider when making a nutrition plan: i) stopping the flare, and ii) reversing the malnutrition. In terms
of stopping the flare, both TPN/bowel rest and enteral nutrition (elemental or non-elemental) have
produced remission rates up to 80% in some studies. However, relapse rates are generally higher after
enteral nutrition vs. prednisolone. In terms of reversing malnutrition, sufficient enteral nutrition is hard
to achieve via PO feedings and usually needs an NG tube. Parental nutrition via IV bypasses this, but
has increased cost, greater chance of mixing error, and risk of line infection. Because this patient has no
contraindication to using the gut, enteral feeds are preferable. No consistent differences have been found
between elemental or non-elemental formulas.

Supplements (choice E) are controversial, as they may simply displace other foods and result in no net
gain in calorie consumption.

Question 6:
A 10 month old boy with biliary atresia has developed progressive jaundice and is listed for liver
transplantation. His 27-year-old mother has been evaluated and found to be a suitable living liver
transplant donor. When discussing the procedure for transplantation with the family, the surgeon explains
that only the left lateral segment of the mother's liver will be removed and transplanted into the recipient.
Which of the following describes the segment(s) of the liver to be used for transplantation in this case?

A. Segments 5,6,7, and 8


B. Segments 2,3, and 4
C. Segment 4
D. Segment 1
E. Segments 2 and 3

Suggested answer: E. Left lateral segment transplants comprise segments 2 and 3. The left lateral
segment represents approximately 20-25% of total volume, and includes the left hepatic vein, left branch
of the portal vein, and left branch of the hepatic artery. In split liver transplantation, the left lateral
segment goes to a child, whereas the right graft (segments I and IV to VIII, including the vena cava, right
branch of the hepatic artery, and portal vein) goes to an adult.

Question 7:
In addition to the regulatory mechanisms intrinsic to the epithelium itself, electrolyte transport is
regulated by all of the following pathways except:

A. Endocrine and paracrine regulation


Dr-Wahid Helmi

B. ATP
C. Enteric neural regulation
D. Intestinal immune system

Suggested answer: A. Electrolyte transport is controlled by a number of processes. The enteric nervous
system controls electrolyte transport, as acetylcholine and vasoactive intestinal polypeptide from nerve
endings stimulate epithelial cells to secrete chloride, which in turn leads to water in the lumen for food
lubrication. Paracrine factors (from immune and other surrounding cells) also control electrolyte
transport. For example, histamine from mast cells increases chloride secretion and decreases
bicarbonate secretion; prostaglandins from myofibroblasts, on the other hand increase secretion of both.
ATP has a central role in electrolyte transport, providing the energy to create gradients of cations (i.e.
Na+/K+ ATPase). Classic endocrine hormones are not thought to be major regulators of electrolyte
transport.

Question 8:
Which of the following is not a feature of Reye’s syndrome?

A. elevated transaminases
B. cholestasis
C. encephalopathy
D. microvesicular steatosis
E. vomiting

Suggested answer: B. Reye’s syndrome involves mitochondrial defects in fatty acid oxidation, caused by
a double insult of viral infection (flu, gastroenteritis) and mitochondrial toxins (i.e. salicylates). It is best
characterized as an event occurring after aspirin is given for a viral illness, and presents as
vomiting/lethargy followed by encephalopathy and hepatocyte damage (ALT and AST 3X > normal).
Liver biopsies show microvesicular steatosis without necrosis, and electron microscopy shows
mitochondrial changes. Bilirubin levels are usually normal or only slightly increased, and if cholestasis
is present other diagnoses should also be considered.

Question 9:
What is the most sensitive indicator for renal cyclosporine toxicity?

A. cyclosporine level
B. blood pressure
C. GFR calculated using creatinine
D. Creatinine
E. GFR based on inulin

Suggested answer: E. Cyclosporine binds with cyclophilin, which in turn inhibits calcineurin. Active
calcineurin normally dephosphorylates nuclear factor of activated T cells (NFAT-1), allowing it to enter
the nucleus and promote transcription of IL-2 and other pro-T and B cell cytokines. Hence, cyclosporine
is known as a calcineurin inhibitor (Tacrolimus works in a similar way, binding to its partner FK-506
binding protein to inhibit calcineurin).

Cyclosporine causes two forms of renal toxicity: acute and chronic. Acute toxicity is characterized by
afferent and efferent arteriole vasoconstriction, secondary to endothelial cell dysfunction. This results in
a drop in GFR and acute renal failure. Chronic toxicity occurs from vasoconstriction leading to ischemia
and structural changes in the kidneys. Acute toxicity is reversible by withdrawing the medication,
whereas chronic toxicity is thought to be permanent.
Dr-Wahid Helmi

Clinically, patients with cyclosporine toxicity present with decreased GFR. They have hypertension
secondary to sodium/volume retention, as the kidneys try to preserve perfusion in the setting of arteriole
constriction. The lowered GFR also leads to an increase in BUN and creatinine. Hence, the most
sensitive test to determine toxicity would be one that identifies an impaired GFR. Inulin, an inert
substance that is freely filtered through the glomerulus and not reabsorbed, is used to accurately
determine GFR.

Question 10:
A 7 month old infant is brought to you for evaluation because of recurrent vomiting and lethargy for 3
months. Dietary history reveals that she was exclusively breastfed until 5 months and her diet now
consists of cereal, fruits, fruit juice, and vegetables. What enzyme deficiency is likely?

A. fructose-6-phosphate
B. triokinase
C. aldolase B
D. lactase
E. sucrase isomaltase

Suggested answer C. This patient tolerates lactose from breast milk but does not tolerate foods with
sucrose/fructose (lactose is a disaccharide made from glucose and galactose, whereas sucrose is a
disaccharide made from glucose and fructose). Sucrose is digested by the brush border enzyme complex
sucrose isomaltase, which has both sucrase and alpha1,6 dextrin hydrolyzing activity. Deficiencies in
this enzyme result in sugar malabsorption and diarrhea.

Fructose, once transported into the enterocytes by facilitate diffusion, becomes phosphorylated to
fructose-1-phosphate by fructokinase. Aldolase B then controls the fate of the molecule, cleaving it into
products that enter the glycolytic, gluconeogenic, or glycogen synthesis pathways. Without this enzyme,
fructose-1-phosphate accumulates. Such patients with hereditary fructose intolerance develop poor
feeding, vomiting, lethargy, hypoglycemia, failure to thrive, liver disease, and proximal renal tubular
dysfunction when fed sucrose or fructose.

Question 11:
Cobalamin (Vitamin B12) absorption may be impaired in each of the following conditions EXCEPT:

A. pernicious anemia
B. cholestatic jaundice
C. Crohn’s ileitis
D. Zollinger-Ellison syndrome
E. Small bowel bacterial overgrowth
F. Pancreatic insufficiency

Suggested answer: B. Vitamin B12 (cobalamin) comes mainly from the cobalamin-containing meats, but
gut bacteria produce small amounts that are absorbed. The absorption of B12 from foodstuffs is a well-
characterized process. First, cobalamin must make it through the acidic stomach, by binding to
haptocorrin (R binder) at low pH. Most of the gastric haptocorrin originates from saliva. In the
duodenum, pancreatic proteases activate in the presence of bicarbonate, hydrolyze haptocorrin, and
liberate cobalamin. The cobalamin in turn binds to intrinsic factor (made by gastric parietal cells) and
becomes resistant to pancreatic proteases. The cobalamin/intrinsic factor complex binds to an ileal
brush border receptor and enters enterocytes.
Dr-Wahid Helmi

Given this sequence, B12 absorption can be impaired at multiple steps. Pernicious anemia leads to low
intrinsic factor, due to autoimmune attack on parietal cells. Zollinger-Ellison syndrome causes the
duodenum to be too acidic, preventing protease activation and degradation of the haptocorrin/cobalamin
complex. In the same way, pancreatic insufficiency prevents degradation of haptocorrin/cobalamin.
Bacterial overgrowth in the small intestine disrupts the cobalamin/intrinsic factor interaction, and ileal
disease affects cobalamin/intrinsic factor binding at the ileal brush border. Cholestasis does not alter
B12 absorption. Bile does contain haptocorrin, but salivary haptocorrin is clinically the most important.

Question 12:
True statements regarding the Schilling test include each of the following EXCEPT:

A. The stage I test result is normal in patients receiving acid suppressive therapy
B. The Schilling test relies on normal renal function for adequate interpretation
C. The stage II test result is abnormal in healthy patients who have undergone total gastrectomy
D. Results may be abnormal in patients with small bowel bacterial overgrowth

Suggested answer: C. In patients with B12 deficiency, the Schilling test is used to determine which step
in the absorption process is impaired. There are 4 steps to the test. Step 1 involves feeding radiolabeled
B12 and measuring the amount excreted in the urine, as an indicator of how much is absorbed.
Unlabeled B12 is injected concurrently that saturates tissue B12 receptors, ensuring that labeled B12
passes to the urine. Step 2-4 then repeat the first step with various adjuncts: step 2 adds intrinsic factor
(if test becomes normal, then assume IF deficiency or pernicious anemia), step 3 adds antibiotic (if test
becomes normal, then assume small bowel overgrowth), and step 4 adds pancreatic enzymes (if test
becomes normal, then assume pancreatic insufficiency).

The Schilling test should be normal in patients with acid suppression, as too much acid (not too little)
disrupts B12 absorption by interfering with pancreatic protease activation. The Schilling test relies on
normal renal function to excrete radiolabeled B12, and results are impaired with bacterial overgrowth
(hence step 3 of the test). Patients without a stomach should have impaired B12 absorption because of
reduced IF levels; however, step 2 provides supplemental IF and bypasses this defect.

Question 13:
Folate supplementation is indicated in all of the following situations EXCEPT:

A. Anticonvulsant therapy with phenytoin or carbamazepine


B. Celiac sprue
C. Chronic sulfasalazine therapy
D. Pancreatic insufficiency
E. Methotrexate therapy

Suggested answer: D. Folate is absorbed via a sodium-dependent carrier, after being hydrolyzed from
polyglutamate to monoglutamate forms by brush border enzymes. Once inside epithelial cells, folic acid
becomes methylated and reduced into its metabolically active form 5-methyltetrahydrofolate. Celiac
sprue impairs brush border activity. Phenytoin impairs brush border enzyme activity converting
polyglutamate to monoglutamate forms, and sulfasalazine interferes with absorption through the carrier.
Methotrexate prevents reduction of folate that enters cells into its active form, by inhibiting dihydrofolate
reductase (DHFR). Pancreatic enzymes are not involved in folate absorption.

Question 14:
Brush border saccharidase activity is decreased by:
A. A diet rich in sucrose
Dr-Wahid Helmi

B. Fasting
C. Diabetes mellitus
D. Pancreatic exocrine insufficiency
E. Intestinal bacterial overgrowth

Suggested answer: E, There are 3 brush border saccharidases: sucrase-isomaltase, lactase-phlorizin


hydrolase, and maltase-glucoamylase. The brush border, and subsequently the saccharidases, are
destroyed by proteases created by overgrowing bacteria. Lactase-phlorizin hydrolase expression can be
increased in infants by enteral feeds, but fasting has never been proven to decrease activity. In adults, all
enzymes except lactase can be induced by substrate (lactase expression declines with age in the majority
of people, and persistence of lactase expression is an autosomal recessive trait). Pancreatic amylase is
important to start alpha1,6 digestion of starch, which is continued by sucrase-isomaltase and lactase-
phlorizin hydrolase. However, salivary amylase can compensate in settings of pancreatic insufficiency.

Question 15:
Which of the following is not a recognized disease association of hepatitis C viral infection?

A. Cryoglobulinemia
B. Porphyria cutanea tarda
C. Membranoproliferative glomerulonephritis
D. Diabetes mellitus
E. Increased risk of myocardial infarction

Suggested answer: E. There are many extrahepatic manifestations of Hepatitis C. Cryoglobulinemia


occurs when Hepatitis C-antibody complexes form in the blood. “Mixed” cryoglobulinemia refers to
complexes with IgG against the Hepatitis C antigen and IgM against the IgG. To diagnose
cryoglobulinemia, blood samples must be kept warm because the complexes precipitate when cooled.

Porphyria cutanea tarda is a common skin manifestation of Hepatitis C. As a result of liver dysfunction,
the UROD gene (uroporphyrinogen decarboxylase) is impaired, heme synthesis is halted, and porphyrins
build up. The porphyrins collect in the skin absorb visible violet light, producing free radicals that
damage nearby tissue. In the skin, the most common findings are erosions, blisters, and scarring.

Membranoproliferative glomerulonephritis is also seen with Heptitis C, perhaps from a vasculitis


produced by the antibody-antigen complexes. Diabetes mellitus (adult-onset, insulin resistant) is 4X
more likely in Hepatitis C patients for unknown reasons. Some have speculated that the antibody
response induced by Hepatitis C results in an autoimmune attack of the pancreas. No association
between Hepatitis C and myocardial infarction has been demonstrated.

Question 16:
All of the following statements regarding the pathogenesis of diarrhea in patients with Zollinger-Ellison
syndrome are true EXCEPT:
A. Hypergastrinemia leads to colonic hypersecretion of fluid and electrolytes.
B. Acid hypersecretion leads to extreme acidification of the duodenum, resulting in inactivation of
pancreatic enzymes and concomitant steatorrhea.
C. Extreme acidification of the small intestine can lead to mild disruption of enterocyte integrity,
resulting in mild malabsorption.
D. Gastric hypersecretion can lead to significant volume overload of the small intestine.

Suggested answer: A. Zollinger-Ellison syndrome is a disorder of acid hypersecretion, caused by tumors


(commonly pancreatic) secreting gastrin. Children have increased gastric secretions, as well as diarrhea
Dr-Wahid Helmi

secondary to acid osmotic load, mucosal damage, and pancreatic lipase inactivation. Most changes from
acid hypersecretion occur in the small intestine rather than the colon.

Question 17:
The following statements regarding the management of foreign bodies in the stomach are true EXCEPT:

A. The clinician should consider removing objects that are more than 2 cm in diameter or more than 5
cm in length, because they are unlikely to pass through the duodenum.
B. In the case of battery ingestion, levels of heavy metal in the blood and urine should be measured.
C. Batteries that have passed through the esophagus to the stomach should always be removed.
D. Between 80% to 90% of ingested foreign bodies that reach the stomach will pass without specific
therapy.

Suggested answer: B. When disk button batteries are swallowed, X-ray imaging must be done to
determine the location of the battery. 90% of batteries pass spontaneously within 14 days. When
batteries lodge in the esophagus, they must be removed immediately as even 1 hour of contact has been
shown to be injurious (and 4 hours of contact can erode through all esophageal layers). Injury occurs
from electrolyte leakage, liquefaction necrosis secondary to alkali leakage such as sodium or potassium
hydroxide, mercury toxicity, pressure necrosis, and direct current flow causing low voltage burns.
Batteries found in the stomach can be removed if they are not passed after 1 week. Levels of heavy metals
are not routinely measured in battery ingestion, as serum levels have never been reported to be high
enough after battery ingestion to cause toxicity.

Question 18:
All of the following are treatments for symptomatic bezoars EXCEPT:

A. Cellulase
B. Acetylcysteine
C. Atropine
D. Mechanical fragmentation at the time of endoscopy

Suggested answer: C. Bezoars refer to a mass trapped in the gastrointestinal tract (usually the stomach).
Removing the mass can be accomplished via mechanical fragmentation or enzymatic digestion.
Mechanical digestion involves breaking the bezoar into small pieces, so that it can pass through the
intestinal tract. This approach risks causing pylorous/ intestinal obstruction. For digestion, the enzymes
used depend on the constituents of the mass, and include cellulase (which digests cellulose) and
acetylcycteine (which splits disulfide bonds in proteins). Atropine does not have digestive activity.

Question 19:
All of the following are potent acid secretagogues EXCEPT:

A. Coffee
B. Decaffeinated coffee
C. Milk
D. Alcohol

Suggested answer: C. Secretagogues are substances that cause the secretion of other substances, i.e.
acid by parietal cells. Alcohol and coffee directly stimulate parietal cells through uncharacterized
mechanisms. Alcohol also impairs gastric mucosa which contributes to acid-mediated damage. Caffeine
increases cAMP levels in parietal cells leading to increased acid production. However, coffee
independent of caffeine also stimulates parietal cells, consistent with observations that both caffeinated
Dr-Wahid Helmi

and decaffeinated coffee increases acid production. Milk is not a secretagogue; rather, it is thought to
neutralize stomach acid because it has a pH higher than that in the stomach.

Question 20:
A 21 year old woman who has had type I diabetes for 12 years had a 2-year history of intermittent nausea
and vomiting that has worsened in the past 3 months. A scintigraphic scan to evaluate solid-phase gastric
emptying shows 8% emptying at 2 hours (normal, 42% to 80%). All of the following statements are true
EXCEPT:

A. This condition is usually associated with other peripheral and/or autonomic neuropathies.
B. Delays in gastric emptying correlate well with symptoms of nausea and vomiting in this condition.
C. Findings on gastrointestinal manometry may include loss of fed and fasting antral motility and
increased phasic and tonic pyloric motility.
D. A gastric acid analysis may show a reduction in acid production in response to sham feeding.
E. The delay in gastric emptying may be exacerbated by periods of worsening hyperglycemia.

Suggested answer: B. While delayed gastric motility is best-documented in long-standing diabetics, a few
case reports have documented the phenomenon in children as well. The mechanism is thought to be
diabetic neuropathy of the vagal nerve. Patients feel fullness and bloating, though symptoms of nausea
and vomiting correlate poorly with delayed gastric emptying. Furthermore, with poor vagal input, acid
secretion decreases during sham feeding, migrating motor complexes are reduced (but cells of Cajal are
normal in number), antral contractions are less frequent, and the pylorous has increased tone.
Interestingly, in addition to the chronic diabetic changes, episodes of hyperglycemia in normal and
diabetic patients can cause delayed gastric emptying.

For further discussion, see Kashyap P, Farrugia G. Diabetic gastroparesis: what we have learned and
had to unlearn in the past 5 years. Gut. 2010 Dec;59(12):1716-26.

Question 21:
Which of the following statements concerning management of the above case is TRUE?

A. Avoidance of a high fat diet might reduce symptoms.


B. Increasing the oral dose of erythromycin might worsen the nausea.
C. The efficacy of domperidone in reducing nausea may be partially explained by CNS effects on
dopamine receptors in the brain stem.
D. Endoscopic destruction of a gastric bezoar may alleviate symptoms temporarily.
E. All of the above statements are correct.

Suggested answer: E. In order to improve gastric motility, a number of approaches are available. The
first is to improve glycemic control. The second is to use motility agents, including erythromycin,
domperidone (a dopamine receptor antagonist), cisparide, and IV grehlin. Erythromycin can be titrated
up as tolerated, until side-effects such as diarrhea, nausea, or vomiting occur. Removing any bezoars
that have formed secondary to poor motility may also improve symptoms, as well as avoiding meals that
further slow gastric motility, i.e. high fat meals.

Question 22:
A one month old infant boy recovery from heart surgery develops post-prandial emesis. He was the
product of a 35 week gestation. He was found to have congenital heart disease and received prostaglandin
E1 for 21 days to maintain patency of his ductus arteriosus prior to surgery. Post-operatively he did well.
He began feeding with a standard infant formula 2 days ago.
Dr-Wahid Helmi

A. Chest X-ray
B. Upper GI
C. Upper endoscopy
D. Change to elemental formula
E. Begin treatment with H2 blocker

Suggested answer: C. Post-prandial emesis occurs for a number of reasons, including simple reflux,
malrotation, milk allergies, dysmotility, and pyloric stenosis. In the setting of heart disease, gastric
hypoperfusion must also be considered which may delay motility and/or decrease intestinal barrier
function, allowing large protein epitopes to enter the bloodstream and cause allergies. Finally, with
chronic prostaglandin use, case reports have documented antral hyperplasia and gastric outlet
obstruction. Unlike pyloric stenosis which involves muscle, this phenomenon involves the mucosa. The
condition reverses with drug withdrawal. Endoscopy could diagnose this definitively, as well as rule-out
allergic causes. (Upper GI could also suggest antral hyperplasia but could not distinguish it from pyloric
stenosis).

Question 23:
A 17-year-old girl presents with a complaint of a fatigue and jaundice for 4 weeks. She has enlargement
of her liver, scleral icterus and 5 fold elevation of serum transaminases. Her INR is 1.2. Her hepatitis A,
B, and C serologies are negative but her anti-nuclear antibody titer is 1:640. She has not taken
medications. Review of systems reveals that she has had loose stools for 6 months and that she has lost 5
pounds.

Which of the following is the most appropriate next step?

A. Observe for six months


B. Measure anti-mitochondrial antibody titers
C. Colonoscopy
D. Liver biopsy
E. UGI and small bowel follow through

Suggested answer: D. This patient has liver disease and a positive ANA titer (1:640 is lower limit of
abnormal), making type I autoimmune hepatitis likely. Her liver disease also accompanies intestinal
disease suspicious for possible inflammatory bowel disease. Indeed, approximately <1% of children with
ulcerative colitis develop autoimmune hepatitis and 3-5% develop primary sclerosing cholangitis. The
first step is to confirm he liver diagnosis with a liver biopsy, looking for mononuclear and plasma cell
infiltrates in the portal tracts. Next, a colonoscopy can be performed to further investigate the possibility
of inflammatory bowel disease.

Question 24:
The gastric mucosa is a hostile environment for most bacteria. Helicobacter pylori is highly adapted to
survival in the gastric mucosa. Which of the following is most likely explanation for the survival of
H.pylori in the acidic environment of the gastric mucosa?

A. Expression of a vacuolating cytotoxin which impairs acid secretion


B. Production of urease
C. Inhibition of histamine stimulated acid secretion
D. Inhibition of the H+/K+ pump
E. Strong adhesion to the mucosal surface
Dr-Wahid Helmi

Suggested answer: B. Helicobacter pylori are uniquely situated to handle the acidic environment of the
stomach. H. pylori has a urease, which hydrolyzes urea into carbon dioxide and ammonia, and the
ammonia in turn buffers the stomach pH. H. pylori uses adhesins to adhere to the mucosal surface, as
well as a vacuolating cytotoxin (VacA) to create channels in which nutrients can escape from mucosal
cells to feed the bacteria. Proton pump inhibitors block the H+/K+ pump, and ranitidine inhibits
histamine-induced parietal cell acid secretion.

Question 25:
What is one of the primary differences between pediatric enteral formulas and adult enteral formulas?

A. Pediatric formulas have less protein/same volume


B. Pediatric formulas utilize different sources of carbohydrates
C. Pediatric formulas are only made with hydrolyzed protein
D. Pediatric formulas utilize only LCT to enhance fat absorption

Suggested answer: A. Pediatric formulas (versus adult formulas) have more fat but decreased protein
and carbohydrates. They utilize similar carbohydrates, are made full proteins, pepstides, or amino acids,
and use various triglycerides (including long chain fatty acids to prevent essential fatty acid deficiency)
for the fat component.
Dr-Wahid Helmi

Question 1:
A 5-month-old infant has been fed only goat milk. Laboratory studies reveal: hemoglobin concentration,
9.5 g/dL; mean corpuscular volume, 98 fL; white blood cell count, 4200/mm3; and reticulocyte count,
0.2%.

For initial treatment, it would be MOST appropriate to recommend dietary supplementation with

A. ascorbic acid
B. folic acid
C. iron
D. pyridoxine
E. vitamin B12

Suggested answer: B. Goat’s milk can be used for infants suffering from cow’s milk protein allergy.
Goat’s milk contains only trace amounts of the allergenic protein alpha-S1-casein normally found in
cow’s milk (however, goat’s milk contains equivalent amounts of another allergenic protein beta-
lactoblobulin). Goat’s milk is also deficient in other vitamins, including vitamin D (supplemented in
cow’s milk and formulas), vitamin B12, and especially folate. Goat’s milk contain 6 ug/L folate, whereas
breast milk contains 45 ug/L and cow’s milk 50 ug/L. Hence, infants drinking goat milk are most
susceptible to folate deficiency and resulting megoblastic anemia.

Of note, milk from other animals have been tested on children with cow’s milk protein allergy, with better
results (see Vita et al. Ass's milk in children with atopic dermatitis and cow's milk allergy: crossover
comparison with goat's milk. Pediatr Allergy Immunol (2007) 18: 594-8).

Question 2:
The basal energy or metabolic requirement for children is calculated MOST accurately by considering

A. body surface area


B. creatinine-height index
C. serum protein concentration
D. total lymphocyte count
E. triceps skinfold thickness

Suggested answer: E. The basal metabolic rate (BMR) is the amount of energy used by an organism at
rest. In children it includes the energy of growing, and differs between boys and girls in accord with the
sex differences in the intensity and duration of the adolescent growth spurt. BMR is measured by the heat
given off per unit of time, and is expressed as calories released per kilogram of body weight (or per
square meter of body surface area) per hour. BMR is a function of an organism’s fat free mass (FFM),
as this comprises the bulk of active metabolic tissue.

BMR correlates best with body surface area. Creatinine-height index may estimate lean body mass/FFM,
and hence correlate with BMR, but it is not well validated in children. Triceps skinfold thickness
measures subcutaneous fat, not FFM. Certain methods using multiple skin fold measurements to estimate
FFM (“Dauncey” method and the “Durnin/ Womersley” method) are fraught with multiple problems.

Question 3:
Of the antibodies found in human colostrum and milk, the immunoglobulin (Ig) that is MOST likely to
prevent organisms from adhering to the infant's intestinal mucosa is

A. IgA
Dr-Wahid Helmi

B. IgD
C. IgE
D. IgG
E. IgM
Suggested answer: A. Infants passively acquire many antibodies from their mothers (“passive humoral
immunity”). During gestation, they receive maternal IgG transplacentally. After birth, they receive IgA
from colostrum and breast milk, which provides local protection in the gut but does enter the systemic
circulation. They also receive IgG from breast milk, which can bind to transepithelial Hc receptors and
enter the circulation. Infant do not receive IgE, IgM, or IgD (an immunoglobulin associated with IgM)
passively from their mothers.

Question 4:
A 3-year-old boy, who has sustained second- and third-degree burns of the esophagus from ingesting a
lye solution, requires placement of a gastrostomy tube for nutritional support. The parents are counseled
about the risks and benefits of tube feedings.

In your discussion, you tell them that the MOST frequent complication of enteral feeding is

A. clogging of the feeding tube


B. diarrhea
C. electrolyte disturbances
D. gastric irritation
E. hypoalbuminemia

Suggested answer: B. Gastrostomy tubes (G tubes) allow for nutrition in patients with oral or esophageal
disease. There are many complications with G tubes, including aspiration, electrolyte imbalances (i.e.,
refeeding syndrome), and fluid imbalances as most formulas only contain 70-80% water. Other long
term complications may include dysmotility, as the procedure attaches the stomach to the abdominal
wall. The most frequent complication is osmotic diarrhea. Diarrhea results when the rate of feeding
exceeds the gut’s capacity to absorb, leading to increased sugar delivery to the colon where it is
metabolized by bacteria into osmotically active substances.

Question 5:
A 1,220 gm infant develops necrotizing enterocolitis. After the infant recovers, the BEST type of formula
to use to initiate feedings is

A. 20 calories/oz elemental formula


B. 20 calories/oz premature formula
C. 20 calories/oz standard infant formula
D. 24 calories/oz premature formula
E. 27 calories/oz premature formula

Suggested answer: D. Breast milk has many advantages over formula, including the prevention of NEC.
Multiple studies have demonstrated that infants started on expressed breast milk (from their mother or a
donor) have lower NEC rates compared to those started on formula. He current recommendations are to
“prime” the with breast milk early. It is unclear whether the rate of milk increase affects NEC rates.

Despite the benefits of breast milk, breast milk does not have enough calcium and phosphorous for
preterm infants. Preterm formulas account for this deficiency and have adequate amounts. For the
patient in the vignette, the infant should first undergo “priming” with human milk. When a rate of 100
cc/kg is achieved, the human milk can be fortified to 24 kcal/ounce and advanced to reach a goal of 150-
Dr-Wahid Helmi

180 cc/kg/day. If human milk is unavailable, elemental formula (better) or preterm formula fortified to 24
kcal can be used (there is no data favoring 20 kcal formula over 24 kcal formula).

Question 6:
A TRUE statement regarding anthropometric measurements in the assessment of nutritional status is:

A. Acute changes in weight reflect changes in muscle mass


B. Arm circumference is the best screening tool for malnutrition
C. Single measurements are the most sensitive indicators of nutritional problems
D. Standard growth curves are equally applicable to all ethnic groups
E. Standard growth curves overestimate the early gains to be made by breastfed infants

Suggested answer: B. Mid-upper arm circumference (MUAC) is an easy way to screen for malnutrition,
applicable to all children >1 year of old or >6 months old and taller than 65 cm. To measure MUAC,
measure the circumference (to the nearest millimeter) at the midpoint of the left arm, half-way between
the tip of the shoulder (olecranon) and the tip of the elbow (acromium). MUAC compared to BMI is less
affected by edema. Despite the ease of MUAC, the most sensitive indicators of nutritional problems
account for multiple variables, such as social factors in addition to anthropometric calculations. Acute
changes in weight usually reflect fluid shifts, not changes in muscle mass.

Standard growth curves are convenient ways to compare a child’s height and weight with same-aged
children in the population. There are CDC and WHO growth curves. The CDC curves reflect growth of
US children over the last 30 years, whereas the WHO curves reflect growth of children all over the world,
including Brazil, Ghana, India, Norway, Oman, and the United States. Initially, the CDC recommended
using the CDC growth curves for all children in the US. Now, however, they recommend using the WHO
growth curves for children <2 years old, because the WHO growth curves better account for breast
feeding. When the curves for <2 years old are compared, the WHO curve (more breast fed infants) starts
off faster but plateaus quicker. As a result, the CDC curve overestimates the percentiles for breast fed
infants early and underestimates their percentiles later. The CDC recommends switching to the CDC
curve at age 2, resulting in a situation where a child may be overweight at age 2 on the WHO curve but
normal on the CDC curve.

Question 7:
An 8-month-old girl recently recovered from a gastrointestinal illness. When the mother reintroduced the
formula, cereal, and baby foods the child had been eating before the illness, the girl developed persistent,
watery diarrhea; abdominal distention; flatulence; and recurrent abdominal pain.

Of the following, the MOST likely cause of this patient's problem is

A. cow milk allergy


B. gluten sensitivity
C. lactose intolerance
D. new-onset infectious enteritis
E. persistent infectious enteritis

Suggested answer: C. This patient suffers from secondary/acquired lactose intolerance. It occurs after a
viral infection, in which intestinal brush border (and their disaccharidases such as lactase) are
destroyed. As a result, lactose will not be digested in the small intestine, will pass to the colon, and will
be metabolized by gut bacteria into gas and osmotically active substances. Lactase is the last
disaccharidase to return to normal function after injury, so secondary lactose intolerance may last for
months.
Dr-Wahid Helmi

Question 8:
In humans, vitamin E functions PRIMARILY as a:

A. coenzyme of carboxylase
B. component of rhodopsin
C. membrane antioxidant
D. methyl donor
E. regulator of calcium absorption

Suggested answer: C. Vitamin E is a fat-soluble antioxidant that stops production of reactive


oxygen species normally formed when fat undergoes oxidation. Biotin (Vitamin B7) is a
coenzyme for many carboxylases. Vitamin A is a component of rhosopsin, the photopigment
used in black/white vision. Folic acid (Vitamin B9) and colbalamin (Vitamin B12) are methyl
donors. Vitamin D is a regulator of calcium absorption.

Question 9:
Zinc is MOST easily absorbed from:

A. casein-dominant, cow milk-based formula


B. cow milk
C. human milk
D. soy-based formula
E. whey-dominant, cow milk-based formula

Suggested answer: C. Zinc bioavailability has been studies by labeling zinc with a radioisotope, mixing it
into various milk/formula preperations (human milk, cow’s milk, cow’s milk formula, and soy protein
formula), and assaying it on newborn sucking rates. Zinc bioavailability was 28% in human milk, 24%
from cow’s milk formula, 15% from cow’s milk, and 10% from soy formula.

Cow’s milk-based formula is the first choice for infants who are not breast fed. Cow’s milk has 50%
more protein, and the ratio of whey protein to casein protein is 20:80 (compared to 70:30 in human
milk). Caesin is composed of proteins held together in granular structures called micelles, precipitates
out in the acidic environment of the stomach, but is nicely digested in the alkaline environment of the
small intestine, providing a slowly absorbed source of amino acids. Whey, on the other hand, consists
mainly of beta-lactoglobulin (major component of bovine whey, and allergenic), alpha-lactalbumin
(major component of human whey), albumin, immunoglobulins, hormones, and growth factors. It is
easily digested in the acidic environment of the stomach and produces a rapid surge in plasma amino
acids. Pasteurization destroys many of the immunoprotective properties of whey.

The ratio of whey to casein in popular formulas are as follows: 100:0 in Carnation Good Start Supreme,
60:40 in Enfamil Lipil, and 48:52 in Similac Advance.

Question 10:
An infant boy born at term is delivered at home without medical supervision. At 48 hours of age, he is
brought to the emergency room because of a bloody discharge from the umbilical cord and bloody stools.

Until the results of laboratory studies are available, the BEST initial management is to administer
intravenous
Dr-Wahid Helmi

A. ampicillin and gentamicin


B. cryoprecipitate
C. factor VIII concentrate
D. fresh frozen plasma
E. vitamin K

Suggested answer: E. Newborns are vulnerable to hemorrhagic disorders secondary to Vitamin K


deficiency. Very little Vitamin K is transferred transplacentally, and the storage of Vitamin K in the
neonatal liver is also limited. Furthermore, until the gut is colonized with bacteria (especially
Bacteroides species), there is very little microbial production of Vitamin K. Breast milk also contains
little Vitamin K. To prevent bleeding, Vitamin K is administered intramuscularly immediately after birth
in the US. With a Vitamin K injection, the incidence of Vitamin K deficiency-related bleeding varies from
0.25-1.7% in the first week of life.

Question 11:
Physical examination of a 2-year-old malnourished girl reveals a "rachitic rosary."

Among the following, the indicator MOST likely to be associated with this child's problem is:

A. bronchopulmonary dysplasia
B. chronic hepatitis
C. galactosemia
D. sucrase-isomaltase deficiency
E. tyrosinemia

Suggested answer: E. “Rachitic rosary” refers to prominent knobs of bone at the costochondrial
joints. It is associated with rickets, and is caused by excessive urinary losses of calcium and
phosphate and/or defect in renal hydroxylation of 25-OH Vitamin D3 into 1,25-diOH Vitamin
D3. Tyrosinemia can involve renal tubules, leading to a Fanconi-like syndrome with normal
anion-gap, metabolic acidosis, hyperphosphaturia, hypophosphatemia, and Vitamin D resistant
rickets. Galactosemia is characterized by inability to metabolize galactose, and is caused by
mutations in one of 3 genes. It can involve renal tubular dysfunction and a Fanconi-like
syndrome, but (unlike this case) typically presents with jaundice and seizures in the first few days
of life. Sucrase-isomaltase deficiency leads to improper digestion of sucrose, resulting in
sucrose delivery to colonic gut bacteria. It is characterized by diarrhea and malnutrition.

Question 12:
The immunoglobulin (Ig) MOST abundant in human milk is:

A. IgA
B. IgD
C. IgE
D. IgG
E. IgM

Suggested answer: A. Human milk contains all the immunoglobulins (M, A, D, G, E), but secretory IgA
is the most abundant. sIgA from breast milk is an important source of passive immunity before the
neonate generates its own sIgA; furthermore, assuming the mother and child share the same environment
and flora, breast milk sIgA will confer protection against the relevant organisms. sIgA is packaged in
breast milk in acid-resistant packages, allowing it to survive the acidic stomach and makes its way to the
Dr-Wahid Helmi

intestines. Early milk contains 2 g/L IgA, 0.12 g/L IgM, and 0.34 g/L IgG. Mature milk contains 1 g/L
IgA, 0.2 g/L IgM, and 0.05 g/L IgG.

Question 13:
An infant born at term has a Coombs positive ABO blood group incompatibility. On the third day of life,
laboratory findings include: venous hemoglobin concentration, 14 gm/dL, and total serum bilirubin level,
10.8mg/dL.

Among the following, the MOST appropriate therapy is

A. blood transfusion
B. expectant follow-up
C. intravenous administration of immune globulin
D. therapeutic iron supplementation
E. vitamin K supplementation

Suggested answer: B. This child is at risk for elevated unconjugated bilirubin requiring phototherapy,
given his Coombs positive/ABO blood group incompatibility status. Currently, his total serum bilirubin
level is 10.8 mg/dL which is below the recommended phototherapy range for a healthy, full-term
newborn. His future care should include expectant follow-up, as well as at least one bilirubin
fractionation measured early to ensure that the there is no elevation in conjugated bilirubin.

Question 14:
A 1, 450 gm infant is not growing well on 24 calories/oz premature formula. The caloric density is
increased to 27 calories/oz by the addition of medium-chain triglycerides.

Of the following complications, this change in formula is MOST likely to produce:

A. bloody stools
B. diarrhea
C. gastric distention
D. increased gastric emptying time
E. steatorrhea

Suggested answer: B. This patient is being fed a formula with increased osmolarity, and is at risk for an
osmolar diarrhea. Medium-chain triglycerides (MCTs) are considered more easily absorbed than that
other fatty acids, because they are absorbed directly into the enterocyte and carried through the portal
circulation to the liver (they do not travel through lymphatics). Furthermore, MCTs do not require bile
acids, lipase, or co-lipase for their absorption. Recently, studies have challenged the benefits of MCTs,
showing infants fed long chain fatty acids grew just as well as those taking MCTs.

Question 15:
Which of the following drugs administered to the mother is MOST likely to have an adverse effect on the
infant who is breast feeding?

A. Furosemide
B. Gentamicin
C. Hydralazine
D. Metronidazole
E. Penicillin
Dr-Wahid Helmi

Suggested answer: D. Metronidazole is secreted in breast milk in small quantities, just as furosemide,
gentamicin, hydralazine, and penicillin are. None of the medications have been shown to pose a risk to
the infant. However, metronidazole poses a theoretical risk of cancer to children, as it increases the risk
of certain cancers in mice and rats. As a result, some practioners will recommend mothers to stop breast
feeding for 12 to 24 hours after taking metronidazole.

Question 16:
During the first 4 months of life, the USUAL caloric requirement (in Kcal/kg per day) for appropriate
postnatal growth in the healthy term infant who is formula fed is

A. 90
B. 105
C. 120
D. 135
E. 150

Suggested answer: B. Newborns need extra calories to support rapid growth. On average, a 0-3 month
bottle fed infant will take from 90-135 kcal/kg/day to achieve an increase in weight of 25 to 30 g/day
(135-202.5 cc/kg/day). A 3-12 month infant requires approximately 100 kcal/kg/day. Hence, on average,
during the 4 months of life a child will need 105 kcal/kg/day.

Question 17:
Medium-chain triglycerides account for 40 to 50% of the fat content of formulas fed to low-birthweight
infants.

Of the following, the BEST explanation for this practice is that:

A. absorption of medium-chain triglycerides is similar to that of butterfat


B. low-birthweight infants have a large pool of bile acids
C. low-birthweight infants have increased capacity to synthesize bile salts
D. medium-chain triglycerides cause less steatorrhea
E. medium-chain triglycerides require micelle formation for absorption

Suggested answer: D. Preterm infants have low intraluminal bile salts, and all infants have low
pancreatic lipase (their salivary lipase compensates). Medium chain triglycerides (MCTs) can be
absorbed directly into enterocytes and passed to the liver via the portal vein. Their absorption is not
dependant on bile salts, pancreatic enzymes, chylomicron formation, or lymphatic transport. Recently,
studies have challenged the benefit of MCTs over long-chain fatty acids by showing that MCTs offered no
extra growth advantage in preterm infants. MCTs carry the risk of increased urinary dicarboxylic acid
excretion, which is produced after incomplete beta oxidation of MCTs.

Question 18:
A 31/2-year-old boy with chronic diarrhea and failure to thrive is diagnosed with cystic fibrosis.
Neurologic examination reveals absent deep tendon reflexes, truncal ataxia, and muscle weakness. A
nutrient deficiency is suspected.

Given this constellation of findings, what additional physical sign is MOST likely to be present in this
child?

A. Desquamating skin lesions


B. Ophthalmoplegia
Dr-Wahid Helmi

C. Positive Trousseau sign


D. Purpura
E. Stooped posture

Suggested answer: B. Vitamin E deficiency is characterized by neurological and neuromuscular findings,


including ophthalmoplegia. Vitamin A deficiency can present as desquamating skin lesions and night
blindness. Vitamin D deficiency can present with hypocalcemia leading to a positive Trousseau sign
(spasms of the hand when blood flow if occluded via a blood pressure cuff). Vitamin D deficiency can
also lead to vertebral fractures producing a stooped posture. Vitamin K deficiency leads to bleeding seen
as petechiae (<3 mm) or purpura (3-10 mm).

Question 19:
Which of the following amino acids, present in adequate amount in cow milk-based formula, must be
added to soy-based infant formula?

A. Isoleucine
B. Leucine
C. Lysine
D. Methionine
E. Valine

Suggested answer: D. Methionine is a sulfur-containing essential amino acid, and is two times more
abundant in cow’s milk than soy milk. Methionine supplementation improves the biological quality of soy
protein, by improving weight gain, urea nitrogen excretion, and albumin synthesis. Hence,
supplementation began in the early 70s; before that, infants feeding soy formulas were at risk for severe
hypoalbuminemia and edema. In addition to methionine, carnitine (fatty acid transport), and taurine
(neurodevelopment, bile acid conjugation) are also supplemented in soy protein.

Question 20:
A 2-year-old child has undergone resection of the terminal ileum because of an ileal-ileal intussusception.
Of the following nutrients, which is MOST likely to become deficient in this patient?

A. Folic acid
B. Thiamine
C. Vitamin A
D. Vitamin B12
E. Vitamin K

Suggested answer: D. Vitamin B12 is exclusively absorbed in the terminal ileum. Vitamin B12
(cobalamin) comes mainly from cobalamin-containing meats, but gut bacteria produce small amounts
that are absorbed. The absorption of B12 from foodstuffs is a well-characterized process. First,
cobalamin must make it through the acidic stomach, by binding to haptocorrin (R binder) at low pH.
Most of the gastric haptocorrin originates from saliva. In the duodenum, pancreatic proteases activate in
the presence of bicarbonate, hydrolyze haptocorrin, and liberate cobalamin. The cobalamin in turn binds
to intrinsic factor (made by gastric parietal cells) and becomes resistant to pancreatic proteases. The
cobalamin/intrinsic factor complex binds to an ileal brush border receptor and enters enterocytes.

Question 21:
Of the following, the MOST beneficial formula for patients with gastrointestinal allergy, short gut
syndrome, or cystic fibrosis is:
Dr-Wahid Helmi

A. a protein hydrolysate
B. carbohydrate free
C. lactose free
D. low in iron
E. soy based

Suggested answer: A. A protein hydrolysate formula provides proteins in the form of small peptides. As
such, they are less allergenic and less likely (vs. full proteins) to cause allergies in patients with
gastroenterology allergies such as cow’s milk protein allergy. They are also less likely to cause allergies
in children with poor gut integrity, such as those with short gut syndrome. Protein hydrolysate formulas
requires less peptidase digestion because they are pre-digested, making them ideal for patients with
Cystic Fibrosis or patients with other reasons for pancreatic sufficiency. Sugars such as lactose have
nothing to do with allergy, but may cause an osmotic diarrhea in patients with malabsorption. Soy
proteins are both allergenic and require pancreatic enzymes to fully digest.

Question 22:
Of the following fatty acids, which MUST be added to cow milk-based infant formula?

A. Arachidic
B. Linoleic
C. Oleic
D. Palmitic
E. Stearic

Suggested answer: B. There are 2 essential fatty acids for humans: alpha-linolenic acid (ALA, an
omega-3 fatty acid) and linoleic acid (LA, an omega-6 fatty acid). LA is the only fatty acid
whose amount is required to be stated on formulas. ALA and LA are not independently added,
but rather part of the vegetable oils mixed in formulas to provide fatty acids.

ALA is converted to eicosapentaenoic acid (EPA) and docosahexaenoic acid (DHA), which are
anti-inflammatory. LA is used to make arachidonic acid (ARA) which is a precursor to
prostaglandins and other proinflammatory mediators. Both DHA and ARA are supplemented in
available formulas.

Question 23:
A 2-year-old boy with developmental delay receives daily tube feedings. Among the following, the side
effect MOST likely to result from high osmolality or high carbohydrate content of these feedings is:

A. diarrhea
B. fecal impaction
C. gastritis
D. mucosal atrophy
E. occlusion of the feeding tube

Suggested answer: A. High osmolality and high carbohydrate content formulas can both lead to
diarrhea. High osmolality results in high osmolar concentration in the colon, which draws water into the
lumen creating loose stools. High carbohydrate contents can result in excess carbohydrates reaching the
colonic gut bacteria. The bacteria ferment the carbohydrates into osmotically-active short chain fatty
acids, which also attract water in the lumen to produce diarrhea.
Dr-Wahid Helmi

Question 24:
When the composition of colostrum is compared to that of mature human milk, colostrum is found to be
LOWER in:

A. ash
B. immunoglobulin A
C. polymorphonuclear leukocytes
D. sodium
E. total fat

Suggested answer: E. Colostrum is the first stage of breast milk and lasts for many days after birth.
Colostrum is high in protein (especially immunoglobulins), fat-soluble vitamins, and minerals (“ash”).
Compared to mature milk, it has higher concentrations of protein, sodium, potassium, and chloride. It
has lower fat concentrations than that found in mature milk.

Question 25:
The RECOMMENDED daily dose of vitamin D for a healthy 1-week-old, preterm infant taking oral
feedings is (in IU):

A. 4
B. 40
C. 100
D. 400
E. 1,000

Suggested answer: D. Vitamin D is best known for its role in calcium homeostasis; however, it is also a
hormone that has far ranging effects in the host. Because infants often receive inadequate sunlight, and
because breast milk has low concentration of Vitamin D, infants are advised to supplement their
milk/formula feeds with Vitamin D supplementation. The current recommendation for preterm or term
neonates is to supplement with 400 IU/day.
Dr-Wahid Helmi

Question 1
Which of the following statements regarding parenteral nutrition (PN)-associated complications and
toxicity is true?

A. Premature infants develop protein toxicity rapidly with early introduction of PN.
B. The most common feature of neonatal PN-associated liver disease is steatosis.
C. Experimental models have demonstrated impaired pulmonary function and displacement of
bilirubin from albumin binding sites with excessive intravenous (IV) lipid administration.
D. PN-associated liver disease usually presents within one week of initiating PN.

Suggested answer: C. Parenteral nutrition has many associated complications. In general, within 1-2
weeks LFTs become elevated and micro/macro steatosis develops. Cholestasis (conjugated bili > 1.5
mg/dL), extramedullary hematopoiesis, fibrosis and then cirrhosis ensue. Moderate fibrosis is present
after approximately 40 months of parenteral nutrition. Cholestasis is the prominent initial finding in
infants (steatosis and extramedullary hematopoiesis resolves), whereas steatosis is most prominent in
adults.

Specific macronutrient complications include: i) protein toxicity with time, perhaps from amino acids
such as methionine, cysteine, and tryptophan, as documented in premature infants receiving 3.6 g/kg/day
vs. 2.5 g/kg/day; ii) glucose toxicity leading to steatosis, especially with GIRs>12.6; and iii) lipid toxicity,
possibly through phytosterols in lipid preparations. Lipids have also been shown to alter pulmonary
membrane diffusion and displace bilirubin from albumin.

Question 2
Which of the following metabolic alterations is most commonly seen with re-feeding syndrome?

A. Hyperlipidemia secondary to increased serum ketone bodies


B. Wernicke’s encephalopathy secondary to thiamine deficiency
C. Severe hypophosphatemia affecting myocardial and respiratory function
D. Hypernatremia and hypertonic dehydration affecting mental status

Suggested answer: C. Refeeding syndrome occurs within 4 days when transitioning from the starving to
the fed state. During starvation, the body metabolizes protein and fat, and uses little insulin. Protein/fat
catabolism leads to low total body phosphate stores, even if serum levels are normal. With feeding, the
body starts metabolizing sugar and generates large amounts of insulin. The insulin surge promotes
cellular uptake of phosphate, resulting in profound hypophosphatemia. The low phosphate impairs ATP
production resulting in a myriad of symptoms, from non-specific signs to rhabdomyolysis, leukocyte
dysfunction, respiratory failure, cardiac failure, hypotension, arrhythmias, seizures, coma, and even
sudden death. Hypophosphatemia can also rarely cause Wernicke’s encephalopathy, but thiamine
deficiency (not associated with refeeding syndrome) is a more likely cause.

Question 3
Which of the following statements is false concerning failure to thrive (FTT)?

A. FTT is a term used to describe infants and children whose growth deviates from that expected
for their sex and age.
B. It is a symptom not a diagnosis
C. Dysfunction of any organ system may result in FTT.
D. It is a common disorder that may affect 10-20% of outpatients.
E. The majority of patients with FTT have organic causes.
Dr-Wahid Helmi

Suggested answer: E. There is no consensus definition for FTT. It usually refers to children with
abnormally low weight for age and gender. Others suggest that it should include failure in other aspects
of development as well. Still other definitions for older children are: <75% median weight/height, <2
standard deviations below mean weight/height, or weight that crosses 2 percentiles. FTT accounts for 1-
5% of pediatric hospital referrals and 10-20% of outpatient visits. Usually an organic cause is not found,
though oro-motor dysfunction is increasingly becoming recognized as a common cause for FTT.

Question 4
A 29 year-old male is referred from an optometrist for evaluation. The patient’s liver profile shows AST
78 IU/L, ALT 92 IU/L, Bili 1.4 mg/dL, Alk Phos 88 IU/L, and albumin 3.4 g/dL. The photo of the
patient’s eyes is attached. All of the following statements are true except: 3.18.04

A. The patient’s diagnosis is Wilson’s disease if the ceruloplasmin is low.


B. This finding on the eye exam can be seen in primary biliary cirrhosis and autoimmune hepatitis.
C. The pigmentation will disappear with effective therapy.
D. Neurologic symptoms typically do not occur in the absence of this finding.
E. The mechanism that causes the disorder associated with this finding occurs as a result of over-
absorption of copper.

Suggested answer: E. Wilson’s disease, the commonest metabolic cause of ALF in children over 3,
results from defective copper trafficking out of hepatocytes. The disease is caused by a mutation in
ATB7B, which moves copper ions against concentration gradients i) into the Golgi (for formation of
ceruloplasmin) and ii) out of the cell. As a result, serum ceruloplasmin is low, though it may sometimes
be normal or high because ceruloplasmin is also a negative acute phase reactant. Other diagnostic tests
include a blood smear for hemolysis, low alkaline phosphatase, and raised urine copper before and after
pencillamine challenge. The best diagnostic test is elevated copper levels on liver biopsy.

Kayser-Fleisher rings are gold or gray/brown rings in the peripheral cornea representing copper/sulfur
deposits. They are usually (but not always) present in older patients with neurological problems, such as
mood disorders/depression (adolescents), bradykinesia (pseudo-parkinsonian Wilson’s), multiple sclerosis
with tremor (pseudo-sclerotic Wilson’s) , or dyskinesia. Treatment reverses KF rings. Some treatments
include pencillamine which binds free copper for urinary excretion, and zinc which traps copper in
enterocytes to be lost in the gut lumen when the cells are sloughed.

Serum copper is not a reliable test for Wilson’s disease. Serum copper reflects a combination of free
copper and copper bound to ceruloplasmin. Therefore, in Wilson’s disease it may be high (from free
copper released when hepatocytes lyse) or low (because of poor ceruloplasmin production).

Question 5
All of the following medications can cause acute cholestasis resembling obstructive jaundice except:
Dr-Wahid Helmi

A. Erythromycin estolate
B. Nitrofurantoin
C. Sulindac
D. Amoxicillin-clavulanic acid
E. Chlorpromazine

Suggested answer: C. Most drug-induced liver disease is cytotoxic. However, some drugs induce
hepatitis-cholestasis disease characterized by jaundice, itching, elevated alkaline phosphatase, and
histological evidence that resembles obstructive jaundice. Drugs that cause this include erythromycin,
chlorpromazine, nitrofurantoin, and amoxicillin-clavulanic acid. Eosinophils can be seen on liver
biopsies, suggesting a hypersensitivity/allergic etiology. Sulindac is an NSAID that can also cause
cholestasis through inhibiting cannilicular secretion of bile acids. Rather than an obstructive picture,
histology shows more hepatocellular damage as the retained bile damages hepatocytes.

Question 6
A 17 year-old female is gravida 21 weeks. She presents with nausea, emesis and jaundice. Her
medications are prenatal vitamins. Her laboratory exam is significant for ALT 639 IU/L, AST 459 IU/L,
bilirubin 4.8 mg/dL, Alk Phos 320 IU/L, and albumin 3.4 g/dL. The most likely cause of her jaundice is:

A. Intrahepatic cholestasis of pregnancy


B. Viral hepatitis
C. Acute fatty liver of pregnancy
D. Hyperemesis gravidarum
E. Drug-induced hepatitis

Suggested answer: B. The most common cause of jaundice in pregnancy is viral infection, with HSV and
Hepatitis E most concerning for the developing fetus. In the case of Hepatitis B infections, the newborn
needs active (vaccine) and passive (immunoglobulins) immunoprophylaxis. Intrahepatic cholestasis of
pregnancy usually has more itching than jaundice, and occurs in the 3rd trimester when hormone levels are
highest (estrogen inhibits BSEP and progesterone inhibits MDR3). Mutations in BSEP and MDR3 may
also manifest for the first time during pregnancy. Acute fatty liver of pregnancy presents more severely,
and hyperemesis gravidarum has extreme nausea and vomiting. Prenatal vitamins should not cause
hepatitis.

One algorithm is as follows: 1) rule out viral hepatitis; then 2) if ALT>1000, consider medication toxicity
(Tylenol); 3) if ALT<1000 and with renal failure or DIC, consider acute fatty liver of pregnancy; 4) if
ALT<1000 with RUQ pain, consider stones; and 5) otherwise if ALT<1000 consider hyperemesis
gravidarum or other drugs.

Question 7
A 10-old male with HIV on HART therapy is evaluated for elevated liver enzymes (ALT 119 IU/L, AST
101 IU/L, bilirubin 1.3 mg/dL, Alk Phos 390 IU/L). A liver biopsy showed numerous blood-filled cysts
that do not have an endothelial lining. This liver biopsy finding is most likely secondary to:
A. Cytomegalovirus
B. Protease inhibitors
C. Rochalimaea hensalae
D. Caroli’s disease
E. Congenital factors (e.g., cystic Von Meyenburg complexes)
Dr-Wahid Helmi

Suggested answer: C. Peliosis hepatis refers to blood filled cavities in the liver that develop from neo-
angiogenesis. Peliosis often also occurs in the spleen. The neo-angiogenesis is induced by Bartonella
spp. infection (formerly Rochalimaea, such as those that cause Cat Scratch disease) in the setting of
HIV/AIDS (CD4<100). In HIV negative individuals, hepatis peliosis can be caused by anabolic steroids,
Castleman’s disease, Hodgkin’s lymphoma, leukemia, or other malignancies. Diagnosis is made by
visualizing gram-negative bacilli in blood or biopsy specimens, and cultures are seldom positive.

Caroli’s disease and cystic von Meyenburg complexes are both developmental diseases associated with
polycystic kidney disease. Caroli’s disease has numerous dilated intrahepatic bile ducts lined with
defective cholangiocytes. Cystic von Meyenburg complexes are benign cystic tumors, also referred to as
bile duct hamartomas.

Question 8
All of the following statements regarding coagulation disorders in cirrhosis are true except:

A. Thrombopoietin is decreased in patients with cirrhosis.


B. Factor VIII is not depressed until the late stages of cirrhosis.
C. Bleeding is a common cause of death in patients with cirrhosis who undergo abdominal surgery.
D. Thrombotic complications are rare in patients with cirrhosis because of thrombocytopenia and
decreased clotting factor levels.

Suggested answer: D. Coagulopathies accompany liver disease, as hepatocytes synthesize both pro- and
anti- coagulation factors. As a result, bleeding is a common cause of death in cirrhotic patients who
undergo surgery, while thrombotic complications such as DVTs and PEs also occur. Factor VIII is high
(made in spleen in lymph nodes) in cirrhosis and is not depressed until late stages. The liver also makes
thrombopoietin, which promotes megakarocytes to make platelets. Thrombopoietin is low in cirrhosis,
and, along with platelet congestion in the spleen from portal hypertension, explains thrombocytopenia in
cirrhotic patients (liver transplant restores thrombopoietin levels to normal).

Question 9
Which of the following statements concerning esophageal varices is not true?

A. Esophageal variceal bleeding ceases spontaneously in 40% of patients.


B. The mortality from an episode of variceal hemorrhage is 30%.
C. Overall mortality is not significantly reduced with beta-blocker prophylaxis.
D. A hepatic venous portal gradient (HVPG) of greater than or equal to 20 mm HG predicts a poorer
one-year survival in a patient who has bled from varices.
E. Sclerotherapy is an effective method of preventing first time variceal bleeding and reducing
mortality.

Suggested answer: E. Variceal bleeding must be managed effectively, as the mortality from variceal
hemorrhage is 30-50%. Varices are unlikely to bleed when the HVPG is less than 12, whereas 1/3 of
patients in one study bled if the gradient was greater than 12. Prophylactic measures such as beta
blockers have been shown to prevent bleeds but have not improved mortality from variceal hemorrhage.
Prophylactic sclerotherapy or banding has not become standard practice. Prophylactic sclerotherapy did
reduce variceal bleeding but increased congestive hypertensive gastropathy in one study (no difference in
survival). In another study, 42% of patients bled after prophylactic sclerotherapy.

In the event of bleeding, only 40-50% episodes stop spontaneously. Sclerotherapy and band ligation have
proven effective measures to stop bleeding temporarily, and are considered standard measures to stabilize
a patient ahead of liver transplant.
Dr-Wahid Helmi

Question 10
All of the following drugs can cause microvesicular steatosis except:

A. Tetracycline
B. Aspirin
C. AZT
D. Alcohol
E. Troglitazone
F. Valproic acid

Suggested answer: E. Troglitazone is a PPARg agonist used to treat DM2, that has since been pulled off
the market for causing hepatitis. Histology shows hepatocellular injury, and liver failure can ensure
within 1-7 months. The other choices (tetracycline, aspirin, AZT, alcohol, and valproic acid) can all
affect mitochondria and cause mircovesicular steatosis, analogous to that seen with aspirin in Reye’s
syndrome.

Question 11
A 14 year-old female presents for evaluation of jaundice and fatigue. She has been taking phenytoin for
six years for a seizure disorder and levothyroxine for three years for hypothyroidism. Her review of
systems is positive for amenorrhea (three months). Laboratory exam shows ALT 684 IU/L, AST 388
IU/L, bilirubin 6.1 mg/dL, Alk Phos 199 IU/L, total protein 8.2 g/dL, albumin 3.1 g/dL, and PT 16
seconds. The most likely explanation for her illness is:

A. Phenytoin liver toxicity


B. Autoimmune hepatitis
C. Hyperthyroidism
D. Primary Biliary Cirrhosis
E. Choledocholithiasis

Suggested answer: B. This patient likely has autoimmune hepatitis. She has some early symptoms of
AIH, such as fatigue and amenorrhea. Other extra-hepatic symptoms include low grade fever, skin rash,
and joint pain. She has a pre-existing history of likely autoimmune thyroid disease. And her total protein
is elevated in the context of a low albumin, suggesting she is making copious immunoglobulins as would
occur in autoimmune states. Phenytoin toxicity and PBC are not associated with amenorrhea.
Hyperthyroidism usually does not elevate liver numbers. Choledocholithiasis would present with RUQ
pain.

Question 12
Which of the following statements concerning hereditary hemochromatosis (HH) is false?

A. The phenotypic expression in the United States is 1/200-1/250.


B. HH is the most common, identified, genetic disorder in Caucasians.
C. The genetic defect causes an excessive absorption of iron.
D. Compound heterozygosity (C282Y, H63D) accounts for 3-5% of cases.
E. HH should be considered in any male patient with transferrin saturation greater than 60 percent.

Suggested answer: B. Hereditary hemochromatosis is a disease of excessive gut iron absorption. Iron (in
the form of free iron or attached to heme) is processed by the brush border and then stored in enterocytes
as ferritin. The enterocytes can either pass the iron into the circulation via the transporter ferroportin or
carry the iron into the intestinal lumen and out of the body when the cells are sloughed. Iron that passes
Dr-Wahid Helmi

through the ferroportin channel is carried to the liver via the portal circulation and stored in zone 1
hepatocytes. Kupffer cells in the liver store iron from the systemic circulation, whereas reticulendothelial
cells in the spleen store iron from spent red blood cells.

Hereditary hemochromatosis is most commonly caused by the HFE (high Fe) gene. The gene controls
liver hepcidin expression. Hepcidin is a hormone which negatively regulates ferroportin. With mutant
HFE, then, hepcidin levels are low. As a result, ferroportin activity is increased, and more iron stored in
the enterocyte is allowed to enter the systemic circulation, even when excess iron is already present. The
iron becomes deposited in various end organs, where it causes a series of systemic complications. The
iron binds to its carrier transferrin, so a transferrin saturation >60% in males and >45-50% in females
should be concerning for hemochromatosis.

The genetics of HFE are complex. The incidence of HFE homozygosity in Caucasian/Celtic populations
is as high as 1:150-1:200. However, phenotypic expression ranges from 1:2 to 1:150, depending on the
what symptoms are accepted as consistent with disease (in the 1:2 study, many of the symptoms were
general and could be found in control subjects as well). As a result, the incidence of HH based on
phenotype may be as rare as 1:200 (incidence of homozygosity) X 1:150 (incidence of symptoms in
homozygous individuals) = 1:30,000. This is rarer than the 1:3200 incidence of CF, which is the most
common identified genetic disorder in Caucasians.

Question 13
All of the following statements regarding TPN liver disease are true except:

A. TPN liver disease is commonly associated with the development of gallbladder stones and sludge.
B. TPN liver disease is usually cholestatic in adults and associated with steatosis in infants.
C. TPN liver disease usually resolves following institution of enteral nutrition if cirrhosis has not
developed.
D. Low birth-weight infants are more susceptible than adults to TPN liver disease.
E. The combination of CCK and ursodiol does not reduce mortality from TPN-related liver disease.

Suggested answer: B. Liver disease is a mjor complication of TPN. Clinical cholestasis develops in 25%
of premature infants on TPN, with low birth-weight infants and those developing sepsis at highest risk.
Steatosis is transient in infants, whereas it is the defining characteristic in adults. 100% of infants on TPN
for more than 6 weeks develop calcium bilirubinate sludge (stones are much less common). Studies have
shown that CCK does not reduce gallstones, cholestasis, or mortality. Ursodiol can improve liver
numbers initially, but has not been shown to improve liver histology or mortality.

Question 14
Pellegra is associated with what vitamin deficiency?

A. Riboflavin
B. Thiamin
C. Ascorbic acid
D. Pyridoxine
E. Niacin

Suggested answer: E. Vitamin B1 (thiamine) deficiency causes “beri beri,” characterized by confusion,
peripheral paralysis, muscle weakness, tachycardia, and cardiomegaly. Vitamin B2 (riboflavin)
deficiency causes anemia, angular stomatitis, and seborrheic dermatitis. Vitamin B3 (niacin) deficiency
causes pellagra, consisting of dermatitis, diarrhea, and dementia. Vitamin B6 (pyridoxine) deficiency
causes tongue swelling, rash, and neuropathy. Vitamin B9 (folate) deficiency causes macrocytic anemia
Dr-Wahid Helmi

and macroglossia. Vitamin B12 (cyanocobalmin) deficiency also causes macrocytic anemia. Vitamin C
(ascorbic acid) deficiency causes bleeding gums, poor wound healing, and scurvy.

Question 15
A 24 month old infant who is TPN-dependent secondary to short gut syndrome has persistent anemia.
She does have TPN-cholestasis but does not have esophageal varices. There has been no evidence of
bleeding. On exam she is pale and mildly jaundiced. Her hemoglobin is 7.5 gm/dl and her MCV is 70.
What deficiency would result in her persistent anemia?

A. Carnitine
B. Selenium
C. B12
D. Copper

Suggested answer: D. Copper deficiency can occur with TPN use, especially because copper is often
omitted from formulations to avoid excessive copper accumulation (copper is secreted via bile, and
cholestasis impairs this secretion). Copper deficiency can cause iron anemia and microcytic anemia,
because copper is needed for enzymes involved in iron absorption (i.e. copper-dependent ferroxidase).
B12 deficiency causes megoblastic anemia, and can result from ileal resection in short bowel syndrome.
Selenium deficiency occurs if it is not supplemented in TPN, and leads to cardiomyopathy and Keshan’s
disease (perhaps in association with Coxsackie virus infection).. Carnitine deficiency also occurs if it is
not supplemented in TPN. Carnitine is needed to shuttle long chain fatty acids into the mitochondria, and
without it myopathy (including cardiomyopathy) develops.

Question 16
What is not a result of ascorbic acid deficiency?

A. perifollicular hemorrhage
B. subperiosteal hemorrhage
C. hyperkeratotic hair follicles
D. cheilosis

Suggested answer: D. Ascorbic acid (Vitamin C) deficiency is associated with enlargement and
hyperkeratosis of hair follicles, followed by proliferation of blood vessels around the follicle leading to
perifollicular hemorrhage. Subperiosteal hemorrhage is also possible, especially in infants.
Cheilosis/angular stomatitis is a characteristic of riboflavin (vitamin B2) deficiency.

Question 17
Vitamin A deficiency is manifested by all of the following symptoms except

A. keratomalacia
B. follicular hyperkeratosis
C. night blindness
D. xerosis
E. seborrhea dermatitis

Suggested answer: E. Vitamin A deficiency is characterized by night blindness, keratomalacia (dryness


and keratinized epithelium of the cornea), xerosis especially of the conjunctivae and cornea, and follicular
hyperkeratosis from blockage of hair follicles with keratin. Excessive vitamin A can cause seborrhea
dermatitis.
Dr-Wahid Helmi

Question 18
The standard deviation is defined as

A. The average of the squared differences between each observation and the mean.
B. Measure of the inaccuracy of the sample mean as a representative of the mean of the entire
patient population from which the sample was drawn.
C. A standardized measure of variation used to compare dispersion for variables with different units
of measurement.
D. Mean of the differences between each observation and their mean.

Suggested answer: C. Standard deviation measures how much variation there is in a data set from the
mean. It quantifies the variability of the population. It can be viewed as a standarized measure of
“dispersion,” or variation, around the mean. One standard deviation below and above the mean accounts
for 68.26% of the population, whereas two standard deviations below and above the mean accounts for
95.44% of the population.

If the data points represent the entire population, then the true mean can be calculated. In this case, the
population variance is calculated by subtracting the value of each point from the mean, squaring the
result, and then averaging it by dividing by the total number of values (choice A). The population
standard deviation is then calculated by taking the square root of the population variance.

If data points represent a sample of the population, then the mean of the data points will estimate the true
mean of the population. To account for this estimation, the variance of a sample is calculated by
subtracting the value of each point from the mean, squaring the result, and then dividing by the total
number of values minus 1. The sample standard deviation is calculated by taking the square root of the
sample variance.

When the data points represent a sample of the population, the standard error of the mean can be
calculated. It quantifies the how accurately the sample mean representing the true mean. In other words,
it quantifies the certainty/uncertaintly of the calculated mean (choice B). It is calculated by dividing the
sample standard deviation by square root of the number of data points (standard deviation of the sampling
distribution of the mean).

Question 19
The following statement is false regarding a correlation coefficient r:

A. A correlation coefficient is a measurement of the strength of the linear association between two
variables.
B. The correlation coefficient varies between -1 and +1.
C. It represents the proportion of the variability in an outcome variable that can be explained by its
linear association with a predictor variable and vice versa.
D. It can only be used when the variables of interest are continuous.

Suggested answer: C. The Pearson’s correlation coefficient r measures how well two continuous
variables vary with one another. -1 reflects perfect negative correlation, +1 perfect positive correlation,
and 0 no correlation. The square of the correlation (coefficient of determination) measures the amount of
variance in one group that can be associated with the amount of variance in the other group (choice C).
This question assumes r refers to the Pearson correlation coefficient; however, other correlation
coefficients, like the Spearman rank correlation coeficient, does not require variables to be continuous.

Question 20
Dr-Wahid Helmi

The following equation represents a linear regression line


A. r2
B. y = β1x1 +β2x2 + E
C. sd/√n
D. logit P(x) = α + βE+ γ1V1 + γ2V2 +δ1EV1

Suggested answer: B. A linear regression line models the relationship between two variables using a line.
A pre-requisite is that a correlation between two variables exists. The most common method for making
the line is the method of least-squares. This method calculates the best-fitting line for the data by
minimizing the sum of the squares of the vertical deviations from each data point to the line. Choice A is
the coefficient of determination and choice C is the standard error of the mean.

Question 21
The following statements are true regarding sensitivity

A. Probability of testing positive if the disease is truly present


B. Probability of having the disease given the results of the test
C. Should not be increased relative to specificity when the penalty associated with misdiagnosis is
high.
D. Can be increased by increasing the prevalence of disease in the screened population

Suggested answer: A. Sensitivity is the probability of a test being positive if the disease is truly present,
whereas specificity is the probability of the test being negative if the disease is truly absent. These
calculations do not depend on the prevalence of the disease. For screening tests (i.e. HIV screening tests),
in which the goal is to catch all diseased individuals even at the risk of having false positives, sensitivity
should be increased relative to specificity. In contrast to sensitivity and specificity, positive and negative
predictive values reflect the probability of having or not having the disease given a test result. These
values are influenced by the prevalence of the disease in the population.

Question 22
Using the following number set determine the mean, median, and mode

1, 2 ,3 ,4, 5 ,5 , 5

Suggested answer: Mean is (1+2+3+4+5+5+5)/7 = 3.57. Median is the middle number, or 4. Mode is the
most common value, or 5.

Question 23
The reaction of the internal anal sphincter illustrated below is mediated via which pathway?

A. Cholinergic inhibitory
B. Cholinergic excitatory
C. Adrenergic inhibitory
D. Adrenergic excitatory
E. Interstitial cells of Cahal

IAS
Pressure
(mmHg)
EAS

Rectal
balloon
Dr-Wahid Helmi

Suggested answer: E. When the rectum is distended, the IAS relaxes to allow stools to descend and the
EAS contracts to prevent defecation. The IAS relaxation is accomplished by non-adrenergic, non-
cholinergic (NANC) nerves that release nitric oxide (NO), vasoactive intestinal peptide (VIP), and
perhaps carbon monoxide. Rodent studies have also shown that Interstitial Cells of Cajal, the pacemaker
cells throughout the gut stimulating slow waves of peristalsis, are located in the IAS and may be involved
in IAS relaxation. (See Gut 2005;54:1107-1113. Interstitial cells of Cajal are involved in the afferent
limb of the rectoanal inhibitory reflex)

Question 24
The pathogenesis of Hirschsprung’s disease is thought to be related to which of the following?

A. Increased acetylcholine
B. Decreased glutamine
C. Decreased GABA
D. Decreased nitric oxide
E. Increased serotonin

Suggested answer: D. Hirschprung’s disease is caused by lack of ganglion cells in the mucosal
(Meissner’s) and myenteric (Auerbach’s) plexuses of the distal colon. These ganglion cells normally
come from migrating neural crest cells during development, and mutations in genes involved in neural
crest cell migration have been found in Hirschprung’s patients. The classic manometry finding in
Hirshprung’s disease is absent IAS relaxation when the rectum is distended. The relaxation normally is
mediated by nitric oxide, suggesting that nitric oxide-generating neurons are missing or defective in
Hirschprung’s disease.

Question 25
A 14 year old boy with biliary atresia who underwent a Kasai procedure at 5 weeks of age has progressed
to end stage liver disease and is listed for liver transplantation. He has had 2 major variceal bleeds in the
last 3 months and recently had a transjugular intrahepatic postosystemic shunt (TIPS) placed. The most
likely complication following this shunt placement is:

A. Infection
B. Encephalopathy
C. Progressive liver failure
D. Stent migration

Suggested answer: B. The most common complication of the TIPS procedure is encephalopathy, as the
procedure allows blood from the gut to bypass the liver filter and enter the systemic circulation directly.
Encephalopathy happens in as many as 25-34% adult cases, and usually occurs immediately after the
procedure. The second most common complication is shunt occlusion, which can develop within 24
hours of the procedure. Heart failure can also occur, as more blood is returned to right heart following
TIPS. Stent migration has been reported, including cases in which the stent caused atrial puncture.
Dr-Wahid Helmi

Question 1:
In the pediatric population, which of the following is the most common route of HCV transmission:

A. materno-foetal
B. IV drug use
C. blood products
D. sexual contacts
E. tattoos

Suggested answer: A. Since blood product screening started in 1991, most childhood HCV cases occur
through perinatal transmission. Adolescents and adults acquire HCV through percutaneous (i.e. IV drug use)
and non-percutaneous (i.e. sexual transmission) routes. Acute infections are often asymptomatic. Chronic
infections, caused by incomplete immune-mediated attack against infected hepatocytes, lead to fibrosis and
cirrhosis, and accounts for the most number of adult liver transplantations worldwide.

Question 2:
Which of the following is NOT associated with Wilson’s Disease:

A. Fatty liver
B. High serum uric acid
C. Low serum zinc level
D. Low serum alkaline phosphatase
E. High serum bilirubin

Suggested answer: B. Wilson’s disease is characterized by poor copper intracellular trafficking and
extracellular transport, caused by a defect in the copper-transporting adenosine triphosphatase (ATPase)
gene (ATP7B). It is the most common reason for fulminant hepatic failure in children greater than 3 years.
The earliest histological changes include microvesicular and macrovesicular fatty deposition and
glycogenylated nuclei, which progresses to inflammation and fibrosis.

Wilson’s disease can present as a wide spectrum, from modest elevations in transaminases to acute hepatitis
with high serum bilirubin levels. Wilson’s disease also has characteristically low zinc and alkaline
phosphatase levels, though the mechanism has not been completely resolved (zinc is a treatment in Wilson’s
disease, inducing metallothionein which traps copper in enterocytes). Uric acid levels are low in Wilson’s
disease, because copper accumulates in renal tubular cells. This causes a Fanconi-like syndrome and uric
acid wasting into the urine.

Question 3:
Which statement is false:

A. 90% of Vit. A is stored within the liver


B. Vit. A deficiency Xeropthalmia is irreversible
C. Can cause a hypochromic microcytic anemia with low Fe but normal Fe stores
D. Hypervitaminosis A is associated with head aches

Suggested answer: B. Vitamin A comes from plant and animal sources. Vitamin A from plant sources is in
the form of Provitamin A, which is converted to Vitamin A in a series of highly regulated steps that depend on
whole-body Vitamin A status. Vitamin A from animal sources is already in the active form, so excessive
intake can lead to Vitamin A toxicity. Once Vitamin A is absorbed, in enters the lymphatic system through
chylomicrons, then the venous system at the thoracic duct where it is metabolized into remnants such as
Dr-Wahid Helmi

apoliproteins B and E, and eventually endocytosed into the liver. 50-90% of total body retinol is stored in
stellate (Ito) cells, and transports to other organs through the serum by binding to retinol binding protein.

Vitamin A deficiency is uncommon in the developed world, but the third most common nutritional deficiency
world-wide. It presents as xerophthalmia (abnormal keratinization of conjunctiva secondary to poor lacrimal
gland secretion), poor bone growth, non-specific skin problems, and decreased humoral and cell mediated
immune function. Vitamin A deficiency can also lead to anemia, presumably by inhibiting the normal
metabolism of iron. Supplementation can reverse many of these problems; however, advanced corneal
scarring may be irreversible.

Vitamin A toxicity, on the other hand, presents with a myriad of symptoms, including dry skin, headaches,
hepatomegaly, and increased CSF pressures (pseudotumor cerebri).

Question 4:
The most common route of transmission for new onset Hepatitis C in the pediatric population in United States
and Europe is:

A. Drug Abuse
B. Male to male sex
C. Tainted blood supply
D. Vertical transmission

Suggested answer: D. Please see question1 in this set. Vertical transmission accounts for as much as 65% of
pediatric cases of HCV infection.

Question 5:
Which of the following variables DOES NOT predict a higher likelihood of response to interferon therapy in
children with chronic hepatitis B:

A. Active inflammation on liver biopsy


B. Higher interferon dose
C. Female gender
D. Low level of baseline HBV DNA
E. Elevated serum transaminase levels

Suggested answer: B. Interferon has at least two antiviral mechanisms: (1) increases expression of anti-
viral genes, and (2) stimulates the immune system to eliminate HBV. The immune-stimulatory effects of
interferon explains why responders first have a burst of elevated aminotransferases (presumably from the
host immune system attacking infected hepatocytes), followed by normalization of aminotransferases, loss
of serum HBV-DNA, and loss of viral antigens HbeAg and HbsAg.

Hence, interferon may help those children already trying to mount an immune response against infected
hepatocytes (those with active inflammation on liver biopsy and those with ALT greater than or equal to 2
times the upper limit of normal). Furthermore, interferon may promote a better response when there is
less virus to fight, i.e. in those with lower baseline serum HBV DNA levels. Interferon has also been
shown to have a better response in females. Increasing interferon doses or priming with prednisone has
not been shown to improve outcomes.

Question 6:
Which of the following is NOT a common feature of BOTH kwashiorkor and marasmus:
Dr-Wahid Helmi

A. Irritability
B. Decreased serum lipoproteins
C. Markedly Depressed serum albumin
D. Increased susceptibility to infection
E. Anemia

Suggested answer: C. There are two subtypes of severe protein-energy malnutrition (PEM): kwashiorkor and
marasmus. Kwashiorkor is characterized by muscle atrophy and increased body fat, secondary to poor
protein intake in the setting of adequate energy intake. Marasums, on the other hand, is characterized by
muscle wasting and depleted fat stores, secondary to inadequate intake of all nutrients. Severe PEM of both
types produces a number of signs and symptoms: irritability, decreased serum lipoproteins, increased
infection risk, and anemia. However, kwashiorkor classically presents with severely low serum albumin
concomitant with edema. Marasmus presents with low-normal serum albumin with wasting but no edema.

Question 7:
Tyrosinemia is associated with:

A. Boiled cabbage smell


B. Mousey smell
C. Blue Cheese Vinaigrette smell
D. Resident post call smell
E. All of the above except D

Suggested answer: A. Tyrosinemia is caused by a defect in metabolizing tyrosine into its two products,
acetoacetate (ketogenic) and fumarate (glucogenic). Five enzymes are involved in tyrosine metabolism,
and defects in fumarylacetoacetate hydrolase (FAH) catalyzing the last enzymatic step leads to the most
severe form of tyrosinemia (Hereditary tyrosinemia Type 1). Fumarylacetoacetate (FAA) accumulates,
damaging renal and liver cells. FAA is metabolized into succinylacetoacetate and succinylacetone which
are measured in the urine to make the diagnosis. Nitisinone, which blocks an earlier step in the tyrosine
metabolism pathway, is used as treatment to prevent FAA accumulation.

Patients with tyrosinemia may present with a “boiled cabbage” smell. Others describe the smell as
“rotten mushrooms” or “rancid butter.” Patient with PKU (inability to break down phenylalanine) have
a “mousy,” “musty,” “wolf-like,” “barny,” “horsey,” or “stale” smell. Isovaleric acidemia patients
have a “cheesy” “acrid” odor of “smelly feet.”

Question 8:
Which statement about Hepatitis A is true:

A. Leading cause of fulminant hepatitis in Pediatrics


B. Has been associated with chronic hepatitis
C. Recurrence of the disease can occur up to 6 months after primary infection
D. Treatment for non-fulminant hepatitis A includes Lamivudine for 4 weeks
E. Severity of disease decrease with increasing age

Suggested answer: C. Hepatitis A is an RNA virus which replicates in hepatocytes and spreads. The
virus per se does not cause symptoms; rather, cell-based immunity targeting infected hepatocytes leads to
jaundice, elevated aminotransferases, and other symptoms. Older children are more severely affected,
perhaps because they have a more developed immune response (70% under 6 years old are
Dr-Wahid Helmi

asymptomatic, whereas 70% of older children/adults are symptomatic). Treatment is largely supportive,
though vaccine and/or immunoglobulin administration to previously-unvaccinated exposed individuals
has been shown to limit symptoms. Relapse can occur, typically only once within 6-9 months after the
initial infection (“relapsing Hepatitis A”). HAV infection may also trigger autoimmune hepatitis in
predisposed children, but HAV on its own has never been to cause a chronic infection.

Fulminant hepatitis occurs when the host’s immune response is excessive. HAV infection accounts for
only 1% of pediatric acute liver failure cases in the US (but up to 60% in Latin America).

Question 9:
Symptoms and signs NOT associated with Kwashiorkor (see picture) include:

A. Flaky paint sign


B. Hanover’s sign
C. Flag sign
D. Proportional weight for height
E. Fatty liver

Suggested answer: B. Kwashiorkor is characterized by decreased protein intake but normal or above
normal energy intake, and presents with edema and low serum albumin. Kwashiorkor patients have
normal weight and height for age, anasarca, and dry, atrophic, peeling skin (“flaky-paint sign”) with
areas of hyperkeratosis and hyperpigmentation. They also have hepatomegaly (from fatty liver
infiltrates) and a distended abdomen with dilated intestinal loops. The hair of kwashiorkor patients is
dry, fargile, and hypopigmented. During periods of adequate protein intake, hair color is restored and
produces bands of normal color (the “flag sign”).

Question 10:
Acute lower GI hemorrhage in HIV infected patients is most often caused by:

a. CMV colitis
b. Lymphoma
c. Kaposi's sarcoma
d. Idiopathic chronic colitis
e. Nonspecific colitis

Suggested answer: C. HIV infected patients, especially those with low CD4 counts in the pre-HAART
era, were especially vulnerable to GI problems of infectious or malignant origin. CMV colitis presents
similarly to inflammatory bowel disease, with severe cases causing mucosal ulceration and hemorrhage.
Dr-Wahid Helmi

Additionally, HIV-related non-Hodgkin’s lymphoma frequently presents in the GI tract, with all
segments from mouth to anus including biliary system vulnerable (vs. non-HIV related GI lymphomas, in
which the stomach is most commonly affected). Severe cases may also produce rectal bleeding. The most
common cause of GI bleeding in HIV patients is Kaposi’s sarcoma. This malignancy is a vascular tumor
caused by HHV-8, and appears as hemorrhagic nodules on colonoscopy. It has been associated with
both upper and lower GI bleeds.

Question 11:
First line of treatment of esophageal candidiasis in HIV infected patient is:

A. Clotrimazole
B. Ketoconazole
C. Fluconazole
D. Amphotericin B

Suggested answer: C. Esophageal candidiasis presents with pain on swallowing, and occurs in HIV
infected patients with CD4 counts <100 cells/ul. Unlike oropharyngeal thrush which can be treated with
topical treatments such as clotrimazole, esophageal candidiasis always requires systemic antifungal
therapy. Fluconazole is the intial treatment of choice, and has been shown to be more effective than
ketoconazole in randomized trials. Amphotericin B must be given IV and has a number of side effects,
making it a poor first choice.

Question 12:
Organisms causing intestinal microsporidiosis in AIDS patients is:

A. Encephalitozoon intestinalis
B. Cryptosporidium
C. Isospora belli
D. Enterocytozoon bieneusi

Suggested answer: A and D. Microsporidiosis is one of the commonest enteropathogens in HIV/AIDS


patients. Microsporidiosis are small, spore-forming, intracellular organisms most closely related to
fungi. In enterocytes, they distort villous architecture and impair absorption without much concomitant
inflammation, leading to watery diarrhea. The most common species affecting humans is Enterocytozoon
bieneusi (normally remain local), followed by Encephalitozoon intestinalis (can spread systemically via
macrophages). Treatment is with albendazole, though it less effective for E. bieneusi.

Isospora belli and Cryptosporidium also affect immunocompromised patients. In contrast to


microsporidiosis, they are protozoa. Both cause gastric/small bowel symptoms, and cryptosporidiosis is
associated with voluminous watery diarrhea.

Question 13:
The most common non-opportunistic protozoon parasite is AIDS patients is:

A. Cryptosporidium
B. Gardia lamblia
C, Blastomyces hominis
D. Entamoeba histolytica
E. Toxoplasma gondii
Dr-Wahid Helmi

Suggested answer: D. The most common gastrointestinal pathogens in HIV patients include
Cryptosporidium parvum, Isospora belli, and Entamoeba histolytica. Of these, Cryptosporidium and
Isospora are considered opportunisitc because they require the host to be immunocompromised to cause
disease. Entamoeba histolytica, on the other hand, affects immunocompromised and immunocompetent
individuals and is considered non-opportunistic.

Blastomyces dermatitidis (not hominis) is an opporunisitic fungus that typically does not affect the GI
tract, whereas Blastocystis (not Blastomyces) hominis is a non-opportunistic protozoa whose role as a
commensal versus pathogenic organism is still unclear. Giardia lamblia is a protozoan causing
prolonged diarrhea in HIV/AIDS patients as well as immunocompetent patients. Finally, Toxoplasma
gondii is an opportunistic protozoan that presents typically with encephalitis, pneumonitis, or
chorioretinitis, but many also involve the gastrointestinal tract.

Question 14:
The most common opportunistic infection in HIV patients is:

A. Mycobacterium Avium Complex


B. Mycobacterium Tuberculosis
C. Salmonella Typhimurium
D. Clostridium Difficile

Suggested answer: B. While the statistics and definition differ in various studies (some report esophageal
candidiasis and Pneumocystitis jiroveci as the most common opportunistic infections), Mycobacterium
tuberculosis is one of the commonest opportunistic infections in HIV. Ongoing studies are investigating
the relationship between TB and HIV, with results suggesting that TB may not only increase after HIV but
lead to a more serious course of disease.

Other common opportunistic pathogens include Mycobacterium Avium Complex (MAC, comprising M.
avium and M. intracellulare) causing chronic abdominal cramps and bloating, as well as Salmonella
typhimurium causing hemtochezia, tenesmus, and lower abdominal cramps. Clostridium difficile is not
an opportunistic infection, though it is very common in HIV patients presumably secondary to frequent
antibiotic use.

Question 15:
Which of the following is NOT found in Wolman's disease (cholesterol ester storage disease)

A. Orange coloured liver


B. Lipid laden macrophages in the portal triad
C. Blueish hue to some hepatocytes
D. Diffuse steatosis
E. Inflammation

Suggested answer: C. Wolman disease is caused by a defect in lysosomal acid lipase (LAL). As a result,
lysozymes receive endocytosed lipoproteins properly, but they cannot hydrolyze the triglycerides and
cholesterol esters. The triglycerides and cholesterol esters accumulate inside cells, leading to bowel wall
thickening (accumulation in enterocytes and macrophages) and severe life-threatening diarrhea and
malnutrition. In Wolman disease, the adrenal glands are also calcified.

The liver in Wolman disease is enlarged and appears yellow/orange and greasy in appearance (the orange
comes more from the cholesterol esters than the triglycerides). There is extensive fibrosis, associated with
lymphoid infiltration and accumulation of triglycerides/cholesterol esters in hepatocytes, Kupffer’s cells, and
Dr-Wahid Helmi

portal area macrophages. In iron storage disorders, rather than Wolman disease, hepatocytes stained for
iron may have bluish-hue reflecting excess ferritin in the cytoplasm.

Question 16:
A 5-month-old girl is referred for evaluation of poor growth and irritability. The history is significant for
constant spitting up of formula. An upper gastrointestinal series reveals some gastroesophageal reflux but
is otherwise normal. Which one of the following most likely explains these findings?

A. Prone positioning after feedings


B. Stress in the home
C. Immaturity with low pressures of the lower esophageal sphincter
D. Inappropriate relaxation of the lower esophageal sphincter
E. Pyloric stenosis

Suggested answer: D. This patient has gastroesophageal reflux disease (GERD) as opposed to
uncomplicated reflux, because the reflux is causing pathological symptoms such as poor growth. She
does not have anantomical defects such as malrotation, annular pancreas, or even pyloric stenosis
because her UGI was normal. She may have cow’s milk protein allergy, which can cause similar
symptoms. However, the most common explanation for GERD is transient relaxations of the LES
separate from the normal LES relaxation that occurs with swallowing.

The prone position has actually been shown to decrease reflux events, though in most cases the risk of
SIDS outweighs risks caused by reflux. The patient is 5 months old and should not have an immature LES
with consistently low pressures. Finally, stress in the home is more often correlated with adolescent, and
adult GERD.

Question 17:
A 7-year-old girl who had undergone a surgical repair for long-segment Hirschsprung's disease in early
infancy presents with a fever, abdominal distention, and bloody diarrhea for 2 days. Which of the
following is the most likely diagnosis?

A. diversion colitis
B. enterocolitis
C. ulcerative colitis
D. colonic stricture
E. viral gastroenteritis
Suggested answer: B. This patient has Hirschsprung-associated enterocolitis (HAEC). The disease most
commonly occurs within the year after correction though may occur before or many years after repair. It
presents with diarrhea, fever, vomiting, and sometimes bloody stools. Imaging shows air-filled loops of
bowel with no air in the rectosigmoid colon (“cut-off sign”). The pathogenesis is thought to be related to
intestinal stasis and bowel overgrowth proximal to the agangiolic segment or proximal to an anastomotic
stricture. Bacteria invade the intestinal wall, leading to disease. HAEC occurs in ~1/4 of patients with
Hirschsprung’s disease, and at one time had a mortality as high as 33%.

Diversion colitis is also caused by stasis; however, this patient does not have any blind loops of bowel.
UC should have a more chronic course. Colonic (anastomotic) strictures are associated with HAEC, but
HAEC can occur without strictures and strictures can exist without HAEC. Finally, viral gastroenteritis
would not be expected to produce bloody diarrhea and the severe symptoms found in this case.
Dr-Wahid Helmi

Question 18:
A 6-year-old boy just arriving from Eastern Europe has had malodorous diarrhea since early infancy, even
though he was breast-fed. He is small, has some bruises from bumping into furniture going to the
bathroom at night, and has recently developed some difficulty walking. Physical examination shows that
he is small and undernourished, with depleted subcutaneous fat. He has a protuberant abdomen and 1+
edema in his lower extremities. He has no deep tendon reflexes in his lower extremities. Which one of the
following explains the finding on the small intestinal biopsy from this patient?

A. gluten enteropathy
B. congenital lactase deficiency
C. abetalipoproteinemia
D. glucose-galactose transport defect
E. chronic nonspecific diarrhea of childhood

Suggested answer: C. This patient has evidence of fat malabsorption (wasting), including Vitamin K
(bruising), Vitamin E (decreased deep tendon reflexes), and perhaps even Vitamin A (night blindness or
retinitis pigmentosa). Abetalipoproteinemia (ABL) can cause such symptoms.

The body uses lipoproteins to shuttle cholesterol esters and triglycerides among organs. Lipoproteins
have a core of cholesterol esters and triglycerides, surrounded by a monolayer of cholesterol,
phospholipids, and dedicated lipoprotein peptides called apolipoproteins. There are many types of
apolipoproteins, with beta-apolipoproteins being the largest. Furthermore, different lipoprotein particles
have different apolipoproteins. For example, chylomicrons (a lipoprotein particle made in enterocytes)
carry beta-apolipoprotein 48 (ApoB-48), whereas VLDL (a lipoprotein particle made in hepatocytes)
carry beta-apoliprotein 100 (ApoB-100).

In ABL, enterocytes can absorb lipids properly but cannot exocytose the chylomicon particles from their
basolateral membrane into the systemic circulation. The defect is caused by a mutation in microsomal
triglyceride transfer protein (MTP). As a result, patients with ABL have lipids trapped in their
enterocytes, leading to fat malabsorption, diarrhea, and severe malnutrition. Without chylomicrons to
deliver ingested lipids, they have low levels of lipoprotein particles and their corresponding
apolipoproteins. Perhaps the most serious consequence is Vitamin E malabsorption. By 2-6 years,
infants show profound Vitamin E deficiency manifested by symptoms such as retinopathy and
spinocellular degeneration. Treatment is a medium chain triglyceride diet and copious Vitamin E
supplementation.

Gluten enteropathy (celiac disease) would have started only after exposure to gluten, not while the child
was still exclusively breast feeding. Congenital lactase deficiency is rare and would have led to diarrhea
and malnutrition, without the extra neurological signs. Similarly, glucose-galactose transporter defects
are rare and lead to severe diarrhea and dehydration in infants. They are caused by defects in the
sodium/glucose cotransporter (SGLT1), and can be treated with fructose-based formulas.

Question 19:
An 11-year-old girl presents with a history of epigastric abdominal pain for 2 months. The pain is worse
after meals and has occasionally awakened her from sleep. She denies diarrhea or constipation, skin rash,
fever, or mouth sores. Physical examination is completely normal. An upper gastrointestinal radiograph
reveals a duodenal ulcer. Which one of the following is indicated at this time?

A. upper endoscopy with biopsy


B. Helicobacter pylori antibody titers
C. fasting serum gastrin level
Dr-Wahid Helmi

D. colonoscopy with biopsy


E. cytomegalovirus antibody titer

Suggested answer: A. This patient has a duodenal ulcer. Endoscopy is the gold-standard for diagnosing
ulcers, because radiography depends on patient compliance/flexibility and can miss up to 50% of
duodenal ulcers. Endoscopy in this case has many advantages: i) allows for therapeutics in cases where
the ulcer is bleeding/at risk for bleeding; ii) allows for visualization of other ulcers as in hypersecretory
disorders; and iii) allows for diagnosis of H. pylori.

H. pylori antibody titers are not used in pediatrics, in part because standards for IgG levels do not exist
and infections are usually too chronic for abnormal IgM levels. A fasting serum gastrin level could be
useful to diagnose Zollinger-Ellison syndrome; however, this disease usually is diagnosed in patients
between 20-50 years old. A colonoscopy would be useful if the upper endoscopy showed signs of Crohn
disease, but would not be an appropriate first test. CMV gastrointestinal disease is very rare in
immunocompetent hosts. If present, upper endoscopy with viral studies on biopsy specimens would be a
better test than measuring serum antibody titers.

Question 20:
A 9-month-old boy has irritability and regurgitation. Growth and development are normal, as is his
physical examination. Upper gastrointestinal radiograph is normal with no gastroesophageal reflux,
normal anatomy, and no obstructive lesions. Upper endoscopy is normal. The results of an esophageal
biopsy are shown in the figure (Genevay et al. Archives of Pathology and Laboratory Medicine 2010,
134: 815-825). What is the most likely mechanism that explains these findings?

A. achalasia of infancy
B. inappropriate relaxation of the lower esophageal sphincter
C. protein intolerance
D. infection of the gastrointestinal tract
E. gluten enteropathy

Suggested answer: C or B (Note: No picture was included in the original question) Picture A shows
esophageal findings that may be present in infants with protein intolerance. There are many eosinophils
with superficial layering, basal cell hyperplasia (>50% of epithelial cell thickness), and dilated
intercellular spaces. Eosinophilic abscesses are seen in the inset. Picture B shows reflux, or
Dr-Wahid Helmi

inappropriate relaxation of the LES. Basal cell hyperplasia, papillary elongation, and fewer eosinophils
are present.

Achalasia of infancy is very rare, would appear on UGI (dilated esophagus, distal taper), and usually
does not have histological findings on routine upper endoscopy biopsy. Reflux may or may not be present
on UGI, and endoscopy findings are nonspecific. Mild protein intolerance would be undetected by UGI
or endoscopy, though severe cases in older children may be seen as narrowing on imaging and furrowing
on endoscopy. Celiac disease has small intestinal, not esophageal, findings.

Question 21:
A 15-year-old girl with chronic ulcerative pancolitis complains of recent recurrence of diarrhea and
bleeding while taking Azulfidine 4.0 gm/d. Although she had a difficult course during the first year of
disease, she subsequently has not required corticosteroid therapy for over 9 years. Which one of the
following endoscopic findings has the most ominous prognosis?
A. pancolitis
B. stricture in the ascending colon
C. pseudopolyps in the rectum and sigmoid colon
D. inflammatory infiltrates noted on endoscopic ileal biopsy
E. lack of haustra in the transverse colon

Suggested answer: B. Prognosis in UC relates to the extent of disease, with 20% of adults with
proctitis/distal colitis resolving spontaneously in some studies. Pseudopolyps in the rectum and sigmoid
colon suggest relatively distal disease, whereas lack of haustra (a sign of chronic UC) in the transverse
colon identifies more proximal disease. Pancolitis is consitent with the entire colon being involved, with
some inflammatory infiltrates spilling over to the terminal ileum in 10-15% of cases (“backwash ileitis”).

Unlike in Crohn disease, in UC strictures are rare and concerning for neoplasm. UC patients most
vulnerable are those with long-standing (>7 years), more extensive (beyond the splenic flexure) disease
as in this case. Strictures should be sampled by endoscopy to rule out colon cancer. Benign strictures
can also occur in UC, usually from repeated bouts of inflammation and muscle hypertrophy.

Question 22:
A 9-year-old presents with abdominal pain, nausea, and vomiting continuing for several weeks. Upper
endoscopy reveals some erythema and a few superficial erosions in the antrum. Gastric biopsies reveal
mild chronic inflammatory infiltrates and tight ~5-7 m spiral-shaped bacteria in the mucosa. A 13C-Urea
breath test was negative. Which one of the following is the most appropriate next step in treating this
child?

A. penicillin G, single intramuscular dose


B. omeprazole, amoxicillin, and metronidazole orally for 2 weeks
C. amoxicillin orally for 6 weeks
D. H-2 blocking agent for 8 weeks
E. amoxicillin and metronidazole for 1 week

Suggested answer: B. This patient has chronic gastritis caused by Helicobacter heilmanni. Both H.
heilmanni and H. pylori are spiral-shaped; however, H. heilmanni gives a negative urea breath test. H.
heilmanni compared to H. pylori usually produces a less severe gastritis, with fewer ulcers/erosions that
are limited to the antrum. Intestinal metaplasia and MALT is less common with H. heilmanni as well.
Treatment is similar to treatment for H. pylori, and consists of a PPI plus two antibiotics, usually
Dr-Wahid Helmi

amoxicillin and clarithromycin or flagyl and clarithromycin for 2 weeks. Older regimens includes a PPI,
amoxicillin, and flagyl for 2 weeks.

Question 23:
The barium enema shown in the figure (no figure included in original question) is obtained in a young
infant with abdominal distention and vomiting. Which one of the following is the most likely diagnosis?
A. intussusception
B. Hirschsprung's disease
C. malrotation
D. distal intestinal obstruction syndrome
E. anal stenosis

Suggested answer: B. In Hirschsprung’s disease, a barium enema can identify the aganglionic segment.
The area usually shows a thin lumen and colonic dilation proximally. Barium enemas are no longer used
when there is a risk for perforation, as in intussusception and distal intestinal obstruction syndrome
(partial or complete obstruction of the ileocecal area by intestinal contents, commonly seen in CF
patients with pancreatic insufficiency and/or dysmotility). Rather, water-soluble contrast materials are
used to reduce the intussusception or diagnose DIOS/clear the obstruction. A barium swallow, not
barium enema, makes the diagnosis of malrotation, whereas no imaging is needed to diagnose of anal
stenosis.

Question 24:
A 4-month-old formula-fed girl presents for evaluation of rectal bleeding. The remainder of the history is
unremarkable, and the physical examination is normal except for heme occult positive yellow stools. The
figure (no figure included in the original question) is a photograph taken at sigmoidoscopy. Which one of
the following is the appropriate next step?
Dr-Wahid Helmi

A. remove all cow and soy protein from the infant's diet
B. begin sulfasalazine
C. obtain an abdominal CT scan
D. blood test for APC gene abnormalities
E. give reassurance only

Suggested answer: A. This patient likely has cow’s milk protein allergy in response to cow’s milk in
formula. The condition leads to mild colitis with patches of erythema in the rectum, and histology shows
eosinophils. Treatment is eliminating cow’s milk protein from the diet and using semi-elemental formulas
(soy formulas are discouraged because a number of infants will also be sensitive to proteins in soy milk).
After switching formulas, bleeding may resolve as fast as 72 hours or as long as a few weeks. Finally,
nearly all children will outgrow the condition by 1 year of age.

In cow’s milk protein allergy, the histological findings resolve with diet change and there is no need for
anti-inflammatory medication. Furthermore, the bleeding is usually mild and imaging, such as a CT
scan, is not needed. Colonic polyps and cancer are extremely rare in infancy, and without a family
history of colon cancer gene testing is not warranted. In a select group of patients – those thriving on
breast milk without any signs of anemia, whose mothers have already eliminated dairy and insist on
breast-feeding – reassurance may be appropriate next step.

Question 25:
A 5-year-old boy is referred for evaluation of liver disease after presenting to his primary physician with
chronic pruritus. His evaluation reveals a small child (below the fifth percentile for height; weight for height
tenth percentile) with excoriations on his trunk and extremities. He has no icterus. A grade 2/6 systolic
murmur is heard at the left upper sternal border. His liver is soft, about 1 cm below the right costal margin
and nontender. Spleen was not palpable. He has diminished but symmetric deep tendon reflexes in his lower
extremities. Laboratory studies reveal:

Hemoglobin 12.8

Platelet count 239,000

AST 129

ALT 134

Alkaline phosphatase 678

GGTP 948

Total bilirubin 0.7

Prothrombin time 13.9

INR 1.2

Which one of the following is the most likely diagnosis?


A. progressive familial intrahepatic cholestasis (e.g., Byler's disease)
B. sclerosing cholangitis
C. Niemann-Pick disease, type A
D. Alagille syndrome
Dr-Wahid Helmi

E. alpha-1-antitrypsin deficiency

Suggested answer: D. With pruritis and a heart defect, this patient likely has Alagille syndrome. Alagille
syndrome is characterized by paucity of bile ducts, pulmonic stenosis, butterfly vertebrate, posterior
embryotoxin, and dysmorphic facies (triangular-broad forehead with small pointed mandible). Patients often
are malnourished secondary to fat malabsorption, and show signs of fat-soluble vitamin deficiency (including
Vitamin E deficiencies causing diminished reflexes, as in this case). Some patients also experience extreme
pruritis, which may require a biliary diversion or even liver transplantation. Alagille syndrome is most
commonly caused by mutations in Jag1, which serves as a ligand for Notch receptors during cholangiocyte
cell fate specification.

PFIC1, or Byler’s disease, causes growth failure and pruritis. It is associated with diarrhea, and has coarse
granular bile on EM. However, similar to PFIC2, GGT levels are usually normal. Sclerosing cholangitis is
characterized by pruritis, can precede a diagnosis of IBD (especially UC), and is not associated with heart
findings. Niemann-Pick disease type A is caused by pathological accumulation of the phospholipid
sphingomyelin in the monocyte-macrophage system, leading to fatty accumulation in the liver, spleen, and
CNS (leading to early neurodegeneration). The disease is caused by a mutation in the lysozymal enzyme
sphingomyelinase encoded by the SMPD1 gene. Finally, A1AT deficiency has a variety of liver presentation,
with some children showing mild increases in transaminases and others progressing quickly to cirrhosis and
liver failure. Similar to PSC, A1AT is not associated with cardiac defects.
Dr-Wahid Helmi

Question 1:
A 5-month-old infant who was bottle fed cow milk-based formula had severe watery diarrhea for 4 days.
It resolved within 24 hours of beginning an oral rehydration solution. Efforts to restart the previous
formula result in a return of severe diarrhea.

At this time, you are MOST likely to recommend a 5-day course of:

A. elemental formula
B. evaporated milk
C. goat milk
D. lactose-free formula
E. oral rehydration solution

Suggested answer: D. This patient suffers from secondary lactose malabsorption. One possible
pathogenesis model is: a) viral infection blunts small intestine villi; b) without villi, there is insufficient
lactase (lactase sits on the tips of villi); c) lactose from milk cannot be digested; and d) lactose is
delivered to colonic bacteria, which ferment it into osmotically active substances, causing diarrhea.
Temporarily switching to a lactose-free formula would address the lack of lactase.

One unexplained issue is why secondary lactose malabsorption can take many weeks to resolve, even if
villi are reformed properly. Some have proposed that there is something special about the disaccharidase
lactase. Others have suggested that other factors rather than lactase deficiency may be responsible,
including chronic small intestinal infection after the initial insult.

Question 2:
In which of the following conditions is intermittent (bolus) feeding PREFERRED to continuous
nasogastric feeding?

A. Congenital heart disease with failure to thrive


B. Gastroesophageal reflux with failure to thrive
C. Inflammatory bowel disease with bloody diarrhea
D. Malabsorption syndrome with severe villous atrophy
E. Oromotor discoordination due to birth asphyxia

Suggested answer: C. Continuous feeds are problematic because they require the patient to be attached to
the pump constantly, and because they ignore normal physiological patterns of feeding and fasting.
However, they are useful when the stomach cannot handle large volumes. For example, in congenital
heart disease, large volumes may divert too much blood to the gut and away from other vital organs. In
reflux, large volumes may promote more vomiting and, if there is oromotor dysfunction, may lead to
aspiration pneumonia. In malabsorption syndromes, neither bolus nor continuous feeding would be
expected to provide adequate nutrition, and parenteral nutrition would be needed. Patients with
inflammatory bowel disease affecting the colon should have intact gastric tissue, and should be able to
handle bolus feeds.

Question 3:
In the preterm infant, supplementation of human milk or use of formulas containing increased
concentrations of fat and protein often is necessary for adequate nutrition.

Of the following, the BEST explanation for this need for supplementation is that:

A. amino acids are poorly absorbed in the preterm infant


Dr-Wahid Helmi

B. bile acid production in the preterm infant is normal but pancreas activity is deficient
C. fat in human milk is of low caloric value
D. malabsorption of up to 20% of ingested fat is common in the preterm infant
E. medium-chain triglycerides are poorly absorbed in the preterm infant

Suggested answer: D. Premature infants need increased nutrients for a number of reasons: 1) they may
have been malnourished in utero (prompting the early delivery); 2) they undergo a number of challenges
post-natally, including respiratory problems, sepsis, and temperature control; and 3) they have an
immature digestive system and impaired absorptive capacity. Premature infants have the most trouble
digesting lipids, as both bile acid and pancreatic lipase secretion are reduced. Amino acids, on the other
hand, should be absorbed well because intestinal amino acid/peptide transporters are intact.

Fat in human milk is the major calorie source. Furthermore, fat from human milk is better absorbed by
premature infants, in part because human milk also contains lipase. Human milk fat only contains
approximately 12% medium-chain triglycerides. However, because MCTs can be absorbed without bile
salts and lipase, premature formulas have MCT concentrations as high as 40%. Importantly, in infants
this increased MCT concentration in formulas has never been shown to improve absorption or growth.

Question 4:
A 6-month-old infant who has congenital heart disease has grown poorly due to insufficient caloric
intake. You recommend nasogastric feeding to enhance weight gain.

Of the following, the MOST common complication of nasogastric feeding is:

A. allergic reaction to the nasogastric tube


B. dehydration due to diarrhea
C. gastric perforation
D. gastritis
E. gastroesophageal reflux

Suggested answer: E. Children with failure to thrive require supplemental feeds. When oral intake is not
sufficient, nasogastric tube feeding can be used. NG feeds have a number of advantages, including
controlling precisely how many calories are delivered and adjusting fluid volumes according to the
patient’s needs. Complications such as gastric perforation and allergic reaction to the tube are rare,
whereas complications such as gastritis may occur but are not prohibitive. Diarrhea can occur when high-
sugar or osmolarity formulas are delivered, but dehydration usually is prevented because total fluid intake
is controlled. Reflux is a common complication, because (a) the LES is stented open by the feeding tube;
and (b) infants are often fed larger volumes than they are accustomed to, in order to promote catch-up
growth.

Question 5:
A 2-week-old boy is admitted to the hospital with sepsis due to Escherichia coli. He is being breastfed
and has been vomiting frequently. Findings include failure to thrive, lethargy, hypotonia, jaundice,
hepatomegaly, and positive nonglucose reducing substances in the urine.

Of the following, the MOST likely explanation for these findings is:

A. galactosemia
B. glycogen storage disease
C. lactose intolerance
D. maple syrup urine disease
Dr-Wahid Helmi

E. urea cycle defect

Suggested answer: A. Galactosemia classically presents in the context of E. coli sepsis, and is associated
with hypotonia, jaundice, hepatomegaly, and failure to thrive. Reducing substances (galactose) are
present in the urine. Galactosemia results from misprocessing of galactose. Normally galactose and
glucose are generated from lactose by lactase. Galactose enters the cells and is phosphorylated to
galactose-1-phosphate by galactokinase (GALK), which is then converted to uridine diphosphate
galactose by glactose-1-phosphate uridyl transferase (GALT). GALK deficiency, a mild form of
galactosemia, only causes cataracts from galactose accumulation. GALT deficiency, the severe from of
galactosemia, causes cataracts as well as the more severe symptoms present in this vignette, presumably
from galactose-1-phosphate accumulation.

Glycogen storage disease usually causes hypoglycemia when feeding intervals are lengthened, and
hepatomegaly from metabolite accumulation. Lactose intolerance causes diarrhea from malabsorbed
sugars. Maple syrup urine disease is caused by a defect in branched-chain amino acid metabolism,
leading to poor motor and feeding problems and urine with a maple syrup odor. Urea cycle defects cause
defects in ammonia disposal, generating very high levels of ammonia in newborns.

Question 6:
A 3-month-old infant girl is admitted to the hospital for evaluation of recurrent episodes of hypoglycemia.
Physical examination reveals hepatomegaly. After 2 hours of fasting, she develops hypoglycemia.

Measurement of which of the following would be MOST helpful in determining the etiology of this
patient's hypoglycemia?

A. Ammonia in the serum and the arterial pH


B. Cortisol and growth hormone in the serum
C. Insulin and glucose in the serum
D. Ketones and reducing substances in the urine
E. Organic acids in the urine and lactate in the serum

Suggested answer: D. The case vignette is most characteristic of glycogen storage disease, with
symptoms caused by impaired hepatic glycogenolysis during fasting. Glycogen remains in the liver
(causing hepatomegaly), and glucose is not deposited in the serum (causing hypoglycemia). Typically
patients will have urine ketones from fat breakdown but no reducing substances in the urine.

A number of other diseases also lead to hypoglycemia. Defects in sugar metabolism such as galactosemia
and hereditary fructose intolerance impair gluconeogenesis from galactose and fructose, respectively, and
lead to hypoglycemia after many hours of fasting. They are not associated with urine ketones, but do
have positive urine reducing substances. Defects in amino acid metabolism may cause hypoglycemia
because of concomitant liver disease resulting in poor gluconeogenesis, while defects in fatty acid
metabolism can cause hypoglycemia because of liver disease resulting in poor gluconeogenesis, as well as
because of overused and depleted sugar stores in the absence of being able to utilize fats. Fatty acid
metabolism defects can be detected by examining urine for organic acids.

Hypoglycemia may also be caused by too much sugar utilization, as in the case of hyperinsulin states.
Finally, low cortisol (Addison’s disease, congenital adrenal hyperplasia) and low growth hormone cause
hypoglycemia, presumably from poor gluconeogensis. For these children, hormone replacement is the
treatment of choice.

Question 7:
Dr-Wahid Helmi

The mother of a 3-month-old infant reports that the boy is demanding frequent feedings and has a
noticeably protuberant abdomen. Physical examination reveals doll-like facies and marked hepatomegaly.
Laboratory findings include a serum glucose level of 20 mg/dL and an elevated venous lactate level of 44
mg/dL (normal, <18 mg/dL).

Of the following, the most appropriate INITIAL management of this infant is:

A. administration of a formula that has high concentrations of fructose and galactose


B. daily injections of glucagon
C. insertion of a portacaval shunt
D. nocturnal infusion of glucose via a nasogastric tube
E. referral for liver transplantation

Suggested answer: D. This patient has glycogen storage disease type 1, or Von Gierke disease. It is
caused by a deficiency is glucose-6-phosphatase, the last step in glycogenolysis (and the final step in
gluconeogenesis). Patients have doll-like facies and hepatomegaly, and laboratory results are significant
for marked hypoglycemia and lactic acidosis. Patients usually present when they begin to sleep through
the night and have longer periods without feeds. Treatment includes frequent glucose feeds during the
day and continuous feeds at night, usually with uncooked starch that releases glucose slowly. Efforts to
increase glycogenolysis (via glucagon) or gluconeogenesis (via fructose, galactose, or amino acid
formulas) do not help. There is no evidence of portal hypertension, so a shunt is not needed, and diet
modification – not liver transplant – is the standard treatment.

Question 8:
A 5-year-old girl was treated with amoxicillin for otitis media. One week after antibiotic therapy was
completed, she developed crampy abdominal pain and has been passing six stools daily that contain both
bright red blood and mucus. Physical examination reveals a temperature of 38.5°C (101°F), abdominal
distension, and diffuse abdominal tenderness.

Among the following, the most appropriate INITIAL diagnostic study to perform is a(n):

A. barium enema
B. colonoscopy
C. culture of stool for Clostridium difficile
D. evaluation of stool for ova and parasites
E. evaluation of stool for rotavirus

Suggested answer: C. C. difficile infection typically occurs during or shortly after antibiotic use, and
presents with watery stools, abdominal pain, and leukocytosis. In more severe cases, fever, bloody stools,
and (in the most severe forms) toxic megacolon is present. Diagnosis can be made by anaerobic cultures.
However, because cultures take time to grow, various toxin assays – including cytotoxin assays,
immunoassays, and PCR tests – have been developed. Colonoscopy can also be used to make the
diagnosis, which may show a spectrum of findings from erythema to ulcers to pseudomembranes (ulcers
filled with inflammatory material). First line treatment is flagyl, with oral vancomycin used for
reoccurring cases.

Question 9:
A previously healthy 2-year-old boy presents with irritability, drooling, and refusal to eat. Results of
physical examination include: drooling; symmetric aeration with normal breath sounds; and absence of
wheezes, retractions, or rhonchi. You suspect a foreign body was ingested.
Dr-Wahid Helmi

The MOST likely location of the foreign body is the:

A. esophagus
B. oral cavity
C. right main stem bronchus
D. stomach
E. trachea

Suggested answer: A. Foreign body ingestion is a common problem in the 6 month to 3 year old age
group, occurring approximately 80,000 times each year in the US. The most important initial step is to
rule out airway involvement. Drooling in this case suggests the object has lodged in the esophagus.
Imaging can confirm the diagnosis, with most esophageal objects trapped at physiological areas of
narrowing (UES, aortic arch, LES) or areas of stricture (usually mid-esophagus). Treatment depends on
the object and symptoms. If the object is sharp, a battery, causing complete obstruction, or causing other
systemic symptoms (fever, pain), the object should be removed immediately. If the object is blunt and
not causing any other symptoms, the object may be allowed to pass into the stomach on its own.
However, waiting beyond 24 hours for the object to pass is not advised, because the object may cause
local trauma after long periods of time.

Question 10:
A 6-week-old previously well girl is brought to the emergency department because she is irritable,
jaundiced, and feeds poorly.

Of the following, the most appropriate INITIAL laboratory investigation to determine the etiology of this
child's illness is:

A. an upper gastrointestinal series


B. cultures of the blood, cerebrospinal fluid, and urine
C. complete blood count and reticulocyte count
D. determination of serum electrolyte levels
E. measurement of serum acetaminophen level

Suggested answer: B. The differential diagnosis for this patient is sepsis vs. toxin ingestion vs. metabolic
disease. After taking a history negative for toxin ingestion, a sepsis work-up is an appropriate first step.
This patient likely has galactosemia, which presents with poor feeding, jaundice, and hepatomegaly.
Galactosemia also commonly occurs alongside E. coli sepsis, explaining the irritability and sepsis-like
presentation. In addition to blood work, a urinalysis showing reducing substances would also help
establish the diagnosis.

Question 11:
Vitamins and minerals are incorporated into infant formulas in the United States to provide an essentially
complete diet.

Which of the following minerals or trace minerals must be SUPPLEMENTED in ready-to-feed infant
formulas to meet the recommended daily allowances?

A. Calcium
B. Fluoride
C. Iron
D. Magnesium
E. Selenium
Dr-Wahid Helmi

Suggested answer: B. Ready-to-feed infant formulas are pre-mixed, compared to powdered formulas that
must be mixed by the parent before feeding. Ready-to-feed formulas have certain advantages, including
avoiding mixing mistakes and preventing infectious diseases from dirty water. However, ready-to-feed
formulas are made without fluorinated water. As a result, infants drinking ready-to-feed formulas (or
living in areas with fluorinated water) should receive fluoride supplementation after 6 months of age.

Formulas (whether pre-mixed or powdered) contain many supplements. Formulas are supplemented with
calcium (milk-based ~50-55 mg/100ml and soy-based ~70 mg/100ml vs. breast milk ~33 mg/100ml).
Formulas is also supplemented with Vitamin D, while breast milk contains very little Vitamin D. As a
result, all breast fed infants, and formula fed infants drinking less than 1 liter of formula, require
supplementation to achieve 400 IU.

Iron is also supplemented in formula, because iron deficiency is the most common nutrient deficiency in
children. Children require 0.5-0.8 mg/day of iron. Breast milk provides approximately 0.3-1 mg/L of
iron and is 50% bioavailable; unsupplemented formula, on the other hand, provides 1-2 mg/L but is only
4-6% bioavailable. Supplemented formula, in contrast, contains 12 mg of iron per liter of formula. With
a 4-6% bioavailability, this gives the infant 0.48-7.2 mg absorbed iron per liter of formula consumed.

Magnesium in breast milk and formula is abundant and readily bioavailable. Magnesium is present at 4-5
mg/100ml in routine formula, 5-8 mg/100ml in soy formula, and ~3.4 mg/100ml in breast milk. Finally,
selenium is present in higher quantities in breast milk, and selenium supplementation of formula has been
shown to improve infants’ selenium status. Formula selenium supplementation may be a change seen in
the future.

Question 12:
You suspect the cause of abdominal pain in a 17-year-old child is Helicobacter pylori-associated
gastroduodenitis.

Of the following, the most appropriate INITIAL step in management is to:

A. begin an empiric 2 week course of amoxicillin therapy


B. begin a trial of omeprazole therapy
C. obtain serologic testing for anti-Helicobacter antibodies
D. obtain upper gastrointestinal series to exclude gastric adenocarcinoma
E. refer for endoscopy and gastric biopsy

Suggested answer: E. Even though H. pylori is the most common cause of uncomplicated
gastric/duodenal ulcers in patients not taking NSAIDs, other causes do exist. As a result, establishing the
diagnosis is important before starting treatment. H. pylori can be diagnosed with endoscopy, urease
breath tests, and stool antigen tests. Serological testing for anti-pylori IgG was common (most infections
are chronic, so IgM serologies are less useful), but has since fallen out of favor because of its high false
positive rate and low positive predictive value. A negative serology test, however, does suggest no
infection.

Stool antigen tests are preferred over serology, and are especially useful for pediatric population in which
serological ranges are not available. Stool antigen tests may produce false-negative results in the setting
of PPI use.

Question 13:
An 8-year-old boy recently was diagnosed as having severe ulcerative colitis.
Dr-Wahid Helmi

Of the following, the best INITIAL management is:

A. bowel resection
B. continuous nasogastric feedings
C. intravenous corticosteroids
D. oral azathioprine
E. oral 5-aminosalicylates

Suggested answer: C. This patient is having a flare, and short-term IV steroids are the treatment of choice to
reduce inflammation acutely. Bowel resection is too aggressive as a first step, though may be needed if the
disease is steroid-resistant. NG feeds are used to treat mild-to-moderate IBD in some places such as Canada,
but more severe disease requires extra therapy. Similarly, oral 5-ASAs may be effective for mild-to-moderate
disease but alone would be inadequate for a severe flare. Finally, oral azathioprine would be an appropriate
maintenance medication. However, it takes weeks to have full effect, and would be inadequate in the short-
run to improve symptoms.

Question 14:
In a child who has chronic diarrhea and weight loss, the PREFERRED method for providing nutritional
support is:

A. continuous nasogastric feedings


B. intermittent (bolus) nasogastric feedings
C. nasoduodenal feedings
D. oral feedings
E. parenteral nutrition

Suggested answer: D. For ill patients needing supplemental nutrition, the first decision is whether to feed
enterally or parenterally. Those that can absorb nutrients (i.e. no small bowel disease) should be tried on
enteral feeds first. Enteral feeds provide trophic support to the gut, and can be given by mouth (as in this
case), by NG tube (if the oropharynx or esophagus is diseased), or by ND tube (if the stomach empties
poorly). On the other hand, those with small bowel disease (Crohn disease, celiac disease, tufting
enteropathy) have poor absorptive function and may require parenteral nutrition. Parenteral nutrition has
many disadvantages including a high rate of complications such as line infections.

Question 15:
A 16-month-old boy who has leukemia is receiving prednisone, vincristine, L-asparaginase, and intrathecal
methotrexate. On the seventh day of chemotherapy, broad-spectrum antibiotics were administered for fever.
All cultures were negative, and antibiotics were stopped after 14 days. Over the past 2 weeks, abdominal
distension and pain have developed. Hematologic findings are normal.

The MOST likely etiology of the abdominal findings is:

A. candidal mucositis
B. chemotherapy-induced mucositis
C. chemotherapy-induced neurotoxicity
D. Clostridium difficile infection
E. leukemic infiltration of the bowel wall

Suggested answer: D. The most common possibilities for this case are C. difficile versus teflitis. Teflitis is a
necrotizing enterocolitis that develops after induction chemotherapy, thought to be due to a combination of
damaged mucosa from chemotherapeutics and microorganism invasion from neutropenia. It typically
Dr-Wahid Helmi

presents in neutropenic patients with RLQ abdominal pain and fever. Given the normal neutrophil count and
recent antibiotic use, C. difficile infection is the more likely. Chemotherapy-induced neurotoxicity usually
affects peripheral nerves, and damage of the enteral nervous system causing ileus would take longer to
develop. Leukemic infiltration of the bowel wall – especially after starting chemotherapy – would be rare.
Pancreatitis with ileus is another possibility, given that L-asparaginase often induces pancreatitis.

Question 16:
Among the following, the MOST specific clinical manifestation of an antral web in an infant is:

A. diarrhea
B. distension of the lower abdomen
C. extreme irritability
D. hematemesis
E. nonbilious vomiting

Suggested answer: E. Antral webs are rare, as are all non-pyloric stenosis causes of gastric outlet obstruction
in the neonate. The incidence is thought to be 1 in 100,000, and they are commonly associated with other GI
or cardiac abnormalities. Antral webs usually are located 1-2 cm proximal to the pylorus and are 1-4 mm
thick. Webs with apertures greater than 1 cm are rarely asymptomatic, whereas those with near complete
closure cause nonbilious vomiting in the first feeds after birth. The diagnosis can be made by UGI showing
two compartments to the stomach, and the definitive treatment is surgical removal. Lower GI symptoms
(diarrhea, distension of the lower quadrants) would not occur. Hematemesis could occur but nonbloody
vomiting would be more common. Finally, although patients with antral webs may be hungry and irritable,
irritability is a general sign and not specific to any particular disease.

Question 17:
An 18-month-old infant who has intestinal atresia has been maintained on total parenteral nutrition via a
central venous catheter since birth. He is brought to your office with a 1-day history of lethargy, fever, and
jaundice.

Of the following, the MOST likely cause of this infant's findings is:

A. cholelithiasis
B. fulminant hepatic failure
C. infection of the central venous catheter
D. intrahepatic cholestasis
E. viral hepatitis

Suggested answer: C. Parenteral nutrition, though life-saving, has a number of acute and chronic
disadvantages. Acutely, patients commonly develop central line infections. Infected children may have mild
symptoms initially, that rapidly progress to shock requiring resuscitation. Chronically, patients develop TPN-
associated cholestasis, likely from compounds present in the intralipid formulation. The cholestasis leads to
bile salt back-up into the liver and liver damage, which ultimately could lead to liver failure. Cholelithiasis,
secondary to poor bile flow, is another chronic complication of TPN. It could lead to cholecystitis and
produce similar symptoms, though abdominal pain would be a more prominent finding. Finally, fulminant
hepatic failure and viral hepatitis are both acute and would not be more likely to occur in a patient on
parenteral nutrition.

Question 18:
A 1-week-old boy has a seizure. Significant findings on physical examination include a "cherubic" face,
protuberant abdomen, and profound hepatomegaly. Glycogen storage disease is suspected.
Dr-Wahid Helmi

The MOST important issue in the initial management of this patient is to:

A. decrease serum lactate


B. prevent hypoglycemia
C. reduce hypercholesterolemia
D. reduce serum free fatty acids
E. treat hyperuricemia

Suggested answer: B. This patient is having hypoglycemic seizures, which can be managed by
administering sugars to return the infant to a euglycemic state. Patients with glycogen storage have a
number of other serum findings (particularly those with GSD 1/von Gierke disease, or glucose-1-
phosphatase deficiency, as in this case). They may have elevated lactate levels from massive amounts of
glucose-1-phosphate, created from the breakdown of glycogen during fasting but trapped in the liver,
undergoes glycolysis. Furthermore, in the absence of usable glucose, de novo triglyceride production
increases dramatically and leads to elevated cholesterol concentrations. This hyperlipidemia is associated
with xanthomas but not premature atherosclerosis, and, as a result, cholesterol-lowering drugs are not
recommended. Finally, the excessive glycolysis creates large amounts of phosphorylated intermediate
compounds, which inhibits re-phosphorylation of adenine nucleotides. This causes nucleic acid
degradation and produces excessive uric acid, which may lead to nephrolithiasis.

Question 19:
A 5-month-old boy with frequent vomiting is switched from human milk to formula. His symptoms
immediately worsen, and he becomes highly irritable. Shortly afterward, he becomes comatose. Physical
examination reveals a small, hypotonic child responsive only to pain. Laboratory studies reveal: increased
anion gap; metabolic acidosis; serum ammonia concentration, 150 mg/dL; and serum glucose level, 85
mg/dL.

Which of the following classes of inborn errors of metabolism is MOST likely in this patient?

A. Disorder of fatty acid oxidation


B. Glycogen storage disease
C. Lysosomal disease
D. Organic acidemia
E. Urea cycle defect

Suggested answer: Urea cycle defects should be considered in patients with an anion gap, high ammonia,
and normal glucose. Urea cycle defects lead to high ammonia, because there is a defect in converting
ammonia to the secreted product urea. High ammonia causes neurological deficits through mechanisms
incompletely understood. High ammonia also causes an anion gap, possibly by impairing brain
mitochondria, forcing anaerobic metabolism of sugar, and producing lactic acid as a by-product. This
patient showed mild symptoms with human milk (i.e. frequent vomiting), because human milk has low
protein levels (2.3 g/dL protein at birth and decreasing to 1.5-1.8 g/dL after 2-4 weeks). The symptoms
increased with formula, because formula has more protein (approximately 2.1-2.2 g/dL) which led to an
increased accumulation of ammonia.

Question 20:
Of the following, the inborn error of metabolism that exists MOST frequently in a vitamin-dependent
form is:

A. argininosuccinase deficiency
Dr-Wahid Helmi

B. branched-chain ketoaciduria
C. cytochrome C oxidase deficiency
D. homocystinuria
E. phenylketonuria

Suggested answer: D. Homocystinuria occurs with a defect in the conversion of methionine to cysteine,
leading to accumulation of the intermediate product homocysteine. Normally cystathionine beta synthase
(CBS) uses B6 to transsulfurate homocysteine to cysteine. However, in CBS deficiency, homocysteine
accumulates and causes developmental delay, osteoporosis, ocular abnormalities, thromboembolic
disease, homocystinuria, and premature atherosclerosis.. Treating with excessive B6 can drive the
reaction forward and reduce symptoms in some cases. Homocysteine can also be remethylated to
methionine, using methyl donors such as folate and B12.

Argininosuccinate synthetase/lyase deficiencies are responsible for urea cycle defects. Branched-chain
alpha-ketoacid dehydrogenase complex (BCKD) deficiency causes branched chain ketoaciduria (maple
syrup urine disease), a disease characterized by improper metabolism of branched chain amino acids.
This results in a shortage of substrates for gluconeogenesis, energy production, and fatty acid/cholesterol
synthesis. Cytochrome C oxidase deficiency is the most common respiratory chain defect causing
myopathy in newborns, resulting in poor energy production. Finally, phenylalanine hydroxylase causes
phenylketonuria because of an inability to convert phenylalanine to tyrosine. Excessive phenylalanine
accumulates and causes mental retardation.

Question 21:
A previously healthy 20-month-old boy develops gastroenteritis with anorexia, vomiting, and diarrhea; 24
hours later he develops seizures and has a decreased level of consciousness. The serum glucose level is
25 mg/dL, electrolyte concentrations are normal, and urinalysis reveals no ketones.

Among the following, the MOST likely diagnosis for this patient is

A. alcohol ingestion
B. glycogen storage disease type I
C. hereditary fructose intolerance
D. insulinoma
E. medium-chain acyl-CoA dehydrogenase deficiency

Suggested answer: E. For patients with hypoglycemia and no ketones, a disorder in fatty acid metabolism
should be suspected. Normally, with fasting or illness, the liver creates energy for the brain by beta-
oxidizing fatty acids from adipose tissue into ketones. However, in fatty acid oxidation defects such as
medium-chain acyl-CoA dehydrogenase deficiency (MCAD), the liver cannot produce ketones and the
brain is left without an energy source. Without ketones, sugars are rapidly used as the sole energy source
by all tissues, eventually creating hypoglycemia and further depriving the brain of energy.
Alcohol ingestion can create hypoglycemia, but is inconsistent with the clinical history. Glycogen storage
disease produces ketones, as fatty acids must be mobilized because glucose is limiting. Hereditary
fructose intolerance results in hypoglycemia and low ketones, but presents at the onset of fructose
ingestion with vomiting and failure to thrive. Finally, insulinomas also have hypoglycemia and low
ketones (insulin prevents fatty acid mobilization), but insulinomas are very rare in infants.

Question 22:
Dr-Wahid Helmi

The mother of a 3-month-old boy reports that he has a poor appetite and constipation. Findings on
physical examination, when compared to those 2 months ago, include poor interim growth, increased
lethargy, hoarse cry, decreased tone, large fontanelles, and a more pronounced umbilical hernia.

Of the following, the MOST likely cause of this infant's problem is

A. agenesis of the thyroid gland


B. endemic goitrous hypothyroidism
C. end-organ unresponsiveness to thyroid hormone
D. inborn error of thyroxine synthesis
E. thyroid gland unresponsiveness to thyrotropin

Suggested answer: A. Congenital hypothyroidism occurs in 1:2000-4:000 births and is the leading reason
for preventable mental retardation. Congenital hypothyroidism is most commonly caused by thyroid
gland dysgenesis (agenesis, hypoplasia, ectopy). Other causes include defects in thyroid hormone
synthesis, secretion, transport, metabolism, and/or responsiveness. Most infants appear normal at birth
because of maternal contributions of T4; hence, screening tests to measure free T4 and TSH are
administered to identify hypothyroid infants before symptoms described in the vignette develop.

Question 23:
A 3-month-old infant who has a history of gastroesophageal reflux has had increasing vomiting for 2
days. This morning she developed rapid, deep, labored breathing; lethargy; and shock. Findings include:
serum sodium, 144 mEq/L; potassium, 4.5 mEq/L; chloride, 89 mEq/L; bicarbonate, 5 mEq/L; pH, 7.16;
glucose, 48 mg/dL; ammonia, 128 mcmol/L; and ketonuria.

The MOST likely explanation for these findings is:

A. aminoacidopathy
B. ethylene glycol poisoning
C. metoclopramide toxicity
D. organic acidemia
E. urea cycle defect

Suggested answer: D. This patient has organic academia. These diseases are usually caused by defects in
enzymes that metabolize amino acids, and can also be caused by defects in fat and carbohydrate
metabolism. Infants present with poor feeing, vomiting, and lethargy in the first weeks of life, similar to
infants with sepsis. Laboratory results show a profoundly high anion gap metabolic acidosis (in this case,
the gap is 50), with increased ammonia, decreased glucose, and electrolytes consistent with dehydration.
Organic acid analysis in urine is used to confirm the diagnosis. Acute management involves rehydration
and glucose infusion; after stabilization, protein feeds can be reintroduced as long as the offending amino
acid is omitted.

Aminoacidopathies refer only to defects in amino acid metabolism; however, the patient in this vignette
could have problems with fat or sugar metabolism leading to organic acidemia. Ethylene glycol
poisoning also causes a high anion gap metabolic acidosis, but gaps at 50 would only occur very acutely
after ingestion. Metoclopramide toxicity usually causes tardive dyskinesia, which may or may not be
reversible. Finally, urea cycle defects usually have higher ammonia levels and a less severe, lactic acid
metabolic acidosis.

Question 24:
Which of the following is the most common cause of pancreatitis in childhood?
Dr-Wahid Helmi

A. viral
B. drug induced
C. idiopathic
D. familial
E. abdominal trauma

Suggested answer: A. Pancreatitis in childhood has a number of causes. Viral infections are the most
common cause (up to 13-33%), including infections by mumps, rubella, coxsackie B virus, CMV, and HIV.
Other causes include trauma, drug-induced, anatomical (pancreatic divisum, choledochal cyst), toxin-induced,
gall-stone induced, and hereditary (defects in cationic trypsinogen, CF gene).

Question 25:
Which of the following is not part of the Currarino triad characterizing caudal regression syndrome which can
present as infantile constipation?

a. dysplasic sacrum
b. anal abnormalities
c. tethered cord
d. pre-sacral mass

Suggested answer: C. Caudal regression syndrome refers to a constellation of diseases with poorly
formed caudal (sacral and lower lumbar) vertebrate. They are more common in infants of diabetic
mothers, and often is accompanied with a tethered cord. Three types of caudal regression syndromes are:
VACTERL (Vertebral, Anorectal, Cardiac, Tracheal-Esophageal fistula, Renal, and Limb abnormalities);
OEIS (Omphalocele, cloacal Exstrophy, Imperforate anus, Spinal malformation); and Currarino syndrome
(partial sacral agenesis, a pre-sacral mass, and recto-anal abnormalities).
Dr-Wahid Helmi

Question 1:
A TRUE statement regarding gastrointestinal functioning in the premature infant is:
A. Bile acid pools are elevated
B. Intestinal motility is normal
C. Lactose is digested effectively
D. Vegetable oils are digested poorly
E. Vitamins are absorbed adequately

Suggested answer: D. Premature infants have a decreased bile acid pool, for a combination of reasons:
a) bile acid synthesis proteins are not fully expressed, b) the gall bladder does not concentrate well,
thereby diluting bile acids in bile, c) the terminal ileum machinery to reabsorb bile acids is immature,
and d) the liver machinery to re-uptake bile acids from the portal circulation is also immature (explaining
why infants have high circulating bile acids for up to 6 months). All of these reasons, in combination with
low pancreatic lipase and colipase secretion in infants, lead to poor absorption of vegetable oils and fat
soluble vitamins.

Premature infants have reduced lactase expression, which may be induced with early feeding. Premature
infants also have altered motility, with poor antral contractions before 32 weeks in some studies. Other
studies report altered migrating motor complex (MMC) patterns lacking the strong contractions of phase
3 in infants less than 32 weeks old.

Question 2:
Which of the following BEST explains why solutions containing 1.2 to 2.5% glucose, rather than 5%
glucose, are used for oral rehydration?
A. Absorption of sodium and water in the gut is maximized
B. Glomerular filtration rate is enhanced
C. Hyperglycemia and glycosuria are less likely to occur
D. Potassium absorption is decreased
E. Stomach distention with vomiting is less likely to occur

Suggested answer: A. Enterocytes on villi express Na-coupled glucose transporter-1 (SGLT-1). The
transporter is located on the apical side of the cell, and uses a sodium concentration gradient to bring 2
molecules of sodium and 1 molecule of glucose into the cell. (The gradient is created by the basal Na/K
pump, which pumps sodium out of the cell). Water follows the sodium, promoting rehydration. The
glucose is either used by the enterocyte, or enters circulation via facilitated diffusion through the basal
transporter Glut2.

Excessive glucose can be harmful, because it may be unabsorbed in the small intestine. When it reaches
the large intestine, it may be metabolized by gut bacteria to create osmotically active substances.
Diarrhea, and further dehydration, could result.

Question 3:
A TRUE statement concerning a low protein diet in patients with renal insufficiency is:
A. Appetite will increase because the diet is more palatable
B. Cognitive capacity will decrease
C. Nutrient absorption will increase
D. Renal osteodystrophy will be easier to control
E. The progression of renal insufficiency will be slowed

Suggested answer: E. In renal failure, the kidneys have trouble clearing toxins related to urea. As a
result, a constellation of symptoms may develop, including decreased cognitive capacity, decreased taste/
Dr-Wahid Helmi

appetite, decreased growth, and further renal damage by increasing GFR. A low protein does lead to an
improvement of symptoms, presumably from less uremic toxins. A low protein diet is associated not only
with lower creatinine measurements, but also slower progression to renal failure. Hence, children with
renal insufficiency should be given limited amounts of high-quality protein.

Neural function such as taste/appetite and cognitive capacity may increase with less uremic toxins (but
not because the low-protein diet tastes better). Absorption will not change with a low-protein diet. Renal
osteodystrophy is related more to renal 1-hydroxylation of 25-OH Vitamin D and less to uremic toxins.

Question 4:
A 6-week-old boy with known congestive cardiomyopathy weighs 4 kg. He takes only 12 oz of formula
daily. Of the following, the MOST likely consequence of decreased intake in this infant is
A. hypocalcemia
B. hypoglycemia
C. hypokalemia
D. poor growth in length
E. poor weight gain

Suggested answer: E. Children with congestive cardiomyopathy have increased metabolic needs.
Nutritional support is complicated because these children also have strict fluid requirements. The patient
in this question is only taking 72 kcal/kg/day (assuming a 24 kcal/oz formula), which is insufficient even
for an infant without heart disease. As a result, he will have poor weight gain, and if persistent, poor
growth in length later.

Question 5:
The weight of an adolescent boy is twice ideal body weight for height. If he remains obese as an adult, he
is likely to develop each of the following EXCEPT
A. infertility
B. learning disorder
C. pulmonary insufficiency
D. sleep disorder
E. thromboembolism

Suggested answer: B. Obesity has not been shown to cause an adolescent to develop learning disorders
as he transitions to adulthood. However, some cases of obesity are related to neurological disorders (i.e.
Prader Willi Syndrome, Bardet-Biedl Syndrome), and obesity in an otherwise normal child may cause
low self-esteem and poor school performance. Obesity is associated with a number of other co-
morbidities, including infertility (possibly related to leptin abnormalities), pulmonary insufficiency,
obstruction sleep apnea, and hypercoagulation.

Question 6:
A 6-month-old infant has been fed only goat milk. Laboratory studies reveal: hemoglobin concentration,
9.5 gm/dL; mean corpuscular volume, 100; white blood cell count, 4,200/mm^3; and reticulocyte count,
0.4%. In addition to dietary counseling, you would MOST likely recommend initial supplementation
with:
A. ascorbic acid
B. folic acid
C. iron
D. pyridoxine
E. vitamin B12
Dr-Wahid Helmi

Suggested answer: B. Goat milk is an alternative for infants with cow’s milk protein allergy. However,
goat milk lacks folate, which can lead to a megoblastic anemia as seen in this patient. Folate is absorbed
in the small intestine, when the brush border enzyme folate conjugase hydrolyzes dietary polyglutamates
into folic acid. Hence, severe mucosal disease can also lead to folate deficiency. Finally, sulfasalazine
use can lead to folate deficiency, because sulfasalazine competes with enzymes of the folate absorption
pathway.

Question 7:
A 4-year-old girl who attends preschool has had diarrhea for 10 days. Several other children have had
similar symptoms, including fever and vomiting that persisted for 3 to 5 days. This child, however, has
continued to have abdominal discomfort, excessive gas with abdominal distention, and watery stools. Her
appetite is good, and she is not acting ill. Which of the following laboratory tests is MOST likely to
confirm the diagnosis?
A. Breath nitrogen test
B. Clostridium difficile toxin and antigen
C. Stool culture
D. Stool for ova and parasites
E. Stool for pH and reducing substance

Suggested answer: D. This patient has several risk factors for giardia (in preschool, watery diarrhea,
excessive gas, appears well) and sending stool for ova and parasites is appropriate. If available, sending
stool for giardia antigen may have a higher yield. Giardia is commonly treated with Flagyl
(metronidazole) or Alinia (nitozoxanide).

This patient’s symptoms are less concerning for acute gastritis, so a breath nitrogen test to detect H.
pylori would not be warranted (and if H. pylori was a concern, a stool antigen test would be better).
Clostridium difficile is less likely because there is no prior antibiotic use, stools are non-bloody,
symptoms are both upper and lower, and the symptoms seem to be rapidly contagious. Bacterial stool
cultures would be more useful to work-up acute bloody diarrhea. Finally, the patient has no evidence of
sugar malabsorption, so stool for pH and reducing substances would not be helpful.

Question 8:
Of the following, the BEST reason for administering parenteral alimentation through a peripheral vein
rather than through a central vein is:
A. inability to insert and maintain a catheter in a central vein
B. lower incidence of hyperglycemia
C. lower incidence of infection
D. need for parenteral alimentation after discharge from the hospital
E. need to deliver solutions of high osmotic load

Suggested answer: C. Lines in peripheral veins (including PICCs) have a lower incidence of
infection. Peripheral IVs, however, last only a few days and are can only deliver a limited
osmotic load. PICC and central lines last for long periods and can deliver a high osmotic load.
Central veins are placed surgically, and may be used when peripheral access is limited.
Hyperglycemia is a risk with all forms of parenteral nutrition, and involves many factors
including glucose infusion rate, the patient’s insulin production, and the patient’s insulin
sensitivity.

Question 9:
Dr-Wahid Helmi

During a routine health supervision visit, a 16-year-old girl is noted to have a hemoglobin concentration
of 10 gm/dL and mean corpuscular volume of 72. The MOST common cause for anemia in such a patient
is:
A. chronic hemolysis
B. Crohn disease
C. folic acid deficiency
D. iron deficiency
E. parasitic infestation

Suggested answer: D. For a menstruating female, the most common reason for microcytic
anemia is low iron from blood loss. Chronic hemolysis can lead to anemia but is rare. Crohn
disease can also cause microcytic anemia from poor duodenal iron absorption; however, in this
patient without Crohn disease symptoms, iron loss is much more likely. Folic acid deficiency
causes a megoblastic anemia. Finally, parasites can also cause Fe-deficiency anemia, and is
more common in the developing world.

Question 10:
The mother of a 4-year-old boy reports that he has developed hives within 1 hour after ingesting
acetaminophen syrup, brompheniramine elixir, and hard candies. He also has developed hives and
wheezing following injection of penicillin. He inadvertently was given an acetaminophen tablet but had
no reaction. Of the following, the MOST likely cause of this boy's symptoms is
A. allergy to sucrose
B. idiopathic urticaria
C. idiosyncratic reaction to artificial coloring or preservatives
D. multiple drug allergies
E. penicillin contamination of the acetaminophen syrup and brompheniramine elixir

Suggested answer: D. This patient has an allergy to penicillin. He also has an allergy to
brompheniramine elixir, because he tolerates Tylenol alone and hard candies are generally
hypoallergenic. Interestingly, brompheniramine is an anti-histamine, which in most children would be
expected to improve allergic symptoms. However, in this child with multiple drug allergies, the
brompheniramine actually induces more symptoms. The pathophysiology of multiple drug allergy
syndrome remains unknown.

Question 11:
A 33-year-old pregnant woman wishes to breastfeed her baby. She has had systemic lupus erythematosus
for 2 years and takes prednisone and hydralazine. Among the following, your BEST advice to her
regarding her current medication regimen and breastfeeding is:
A. Both prednisone and hydralazine usually are compatible with breastfeeding
B. Captopril should be substituted for hydralazine
C. Cyclophosphamide should be substituted for prednisone
D. Cyclosporine should be substituted for prednisone
E. High dose aspirin should be substituted for prednisone

Suggested answer: A. Although prednisone and hydralazine can be passed in breast milk, the absolute
amount in negligible and poses no risk to the infant. Captopril is not contraindicated during
breastfeeding, though has been associated with birth defects if taken during pregnancy.
Cyclophosphamide and cyclosporine are contraindicated in breastfeeding mothers, as they may cause
pancytopenia and other problems in the child. High dose aspirin must be used with caution in
Dr-Wahid Helmi

breastfeeding mothers, because aspirin is passed in breast milk and may cause bleeding and platelet
abnormalities in infants.

Question 12:
The daily caloric requirement in kcal/kg for a healthy full-term infant is approximately:
A. 40
B. 60
C. 80
D. 100

Suggested answer: D. The dietary recommended intake for a newborn is 102 kcal/kg/day, or
approximately 150 cc/kg/day of human milk (approximately 16.5 ounces/day for a 3.5 kg newborn).
From 4 months to approximately 3 years, the DRI is in the 80 kcal/kg/day range. From age 5-8 years old
the DRI is the 60 kcal/kg/day range, and post-pubertal adolescents generally have a DRI below 40
kcal/kg/day.

Question 13:
The MAIN advantage of enteral nutrition over parenteral nutrition is that enteral nutrition:
A. facilitates monitoring caloric intake
B. has fewer complications
C. permits maintenance of longer periods of positive nitrogen balance
D. prevents toxins from accumulating in the colon
E. provides improved caloric intake in the outpatient setting

Suggested answer: B. Enteral nutrition is favored over parenteral nutrition, because parenteral
nutrition has major complications. These include line infections, cost, TPN associated cholestasis, and
calculation errors on nutrients delivered. With enteral nutrition, it may be harder to calculate caloric
intake because everything that is given may not be absorbed. Both enteral and parenteral nutrition can
permit positive nitrogen balance, depending on how much protein is delivered. Enteral nutrition may be
associated with more toxin accumulation in the colon, as bacteria metabolize unabsorbed foods and
create short-chain fatty acids and other compounds. Finally, both enteral and parenteral nutrition can be
used in the outpatient setting.

Question 14:
A TRUE statement about the sugar content of infant formulas is:
A. All lactose-containing formulas are cow milk-based
B. All cow milk-based formulas contain only simple sugars
C. All soy-based formulas are corn syrup-free
D. All soy-based formulas contain lactose
E. Proprietary formulas do not contain sucrose

Suggested answer: A. Most cow’s milk-based infant formulas contain the disaccharide lactose and not
the disaccharide sucrose (there are exceptions). Soy-based formulas contains a combination of corn
syrup solids, sucrose, and malto-dextrin. Proprietary formulas (i.e. Alimentum, Isomil, and Alsoy) also
contain sucrose. The sucrose content in formula becomes important, especially in the work-up of
hereditary fructose intolerance. Infants consuming cow milk-based formulas generally will not be
exposed to sucrose, which is made of fructose and glucose. However, infants on soy or specialty formulas
will be exposed to sucrose and hence fructose.

Question 15:
Dr-Wahid Helmi

Continuous, rather than intermittent (bolus), tube feeding is the PREFERRED choice for providing
nutrition to the patient with:
A. a transpyloric feeding tube
B. anorexia nervosa
C. coma following head injury
D. esophageal obstruction
E. multiple facial fractures

Suggested answer: A. Bolus feedings are preferred to continuous feedings, because they a) more closely
replicate normal eating, and b) allow the patient to be disconnected from the pump for many hours
during the day. However, if feeds are administered directly to the intestine, a continuous rate is needed.
The intestine does not have the storage capacity of the stomach, and large bolus feeds would lead to
dumping syndrome.

Question 16:
A full-term infant, who weighed 2,200 gm at birth, has been breastfed for 4 months. General growth and
development appear normal. Physical examination reveals enlargement of the costochondral junctions
and subtle thickening of the wrists and ankles. The disorder that is MOST consistent with these findings
is:
A. cleidocranial dysostosis
B. rickets
C. Rubenstein-Taybi syndrome
D. Russell-Silver syndrome
E. Scurvy

Suggested answer: B. This patient’s clinical symptoms are characteristic of rickets. Vitamin-D
deficiency rickets is common between 3 months and three years, because calcium needs are high for
growth and sunlight exposure is limited. Furthermore, vitamin D in breast milk is scant. Infants may be
protected for the first 3 months, because 25-OH Vitamin D (clacidiol) is transferred via placenta from
mother to fetus. Calcidiol has a half life of 3 to 4 weeks; afterwards, serum 25-OH Vitamin D
concentration will fall unless supplemented through multivitamins, formula, or sunlight exposure.

Question 17:
A TRUE statement regarding introduction of solid foods before the age of 4 to 6 months is:
A. Delaying introduction of solid foods beyond this age increases the likelihood of food allergy later
B. Feeding of solid foods helps the infant sleep through the night
C. Feeding of solid foods improves maternal-infant bonding
D. Starches are poorly digested because of low levels of amylase in the gut
E. Supplementation with solid foods is necessary for adequate nutrition

Suggested answer: D. Solid foods should be introduced between 4 to 6 months of age. This is the ideal
time, as introducing foods may provide extra calories to support extra growth, satisfy hunger, and reduce
the occurrence of allergies. Introducing foods before this time may interfere with adequate energy intake,
overload the kidneys, promote food allergies, and place the child at risk for aspiration. In addition,
pancreatic enzyme secretion (including amylase)is immature for the first months of life, making starches
difficult to digest. When foods are introduced after 6 months, infants may suffer from decreased growth
(from decreased calorie intake), iron-deficiency anemia in breastfed infants, delayed oral motor function,
and solid food aversion.

Question 18:
Dr-Wahid Helmi

A 6-week-old infant born at term has a hemoglobin level of 11 gm/dL and is diagnosed with physiologic
anemia of the newborn. The MOST likely cause is:
A. inadequate iron stores in the bone marrow
B. inadequate serum levels of vitamin E
C. increased excretion of iron in the stool
D. low levels of serum erythropoietin
E. persistent fetal hemoglobin

Suggested answer: D. Physiologic anemia of the newborn occurs when fetal hemoglobin normally
declines at 6 to 8 weeks. Because fetal hemoglobin has a higher affinity for oxygen, serum erythropoietin
levels are initially low. Erythropoietin levels eventually rise again with time, correcting the anemia.
Some have investigated whether Vitamin E mitigate the normal hemoglobin nadir, with equivocal results.

Question 19:
Most authorities encourage the early introduction of human milk in the very-low-birthweight (VLBW)
infant. However, mothers must be informed early in the feeding process that supplementation of their
milk with protein and other nutrients may be necessary. The MOST likely explanation for why protein
supplementation of human milk often is required in the VLBW infant is that

A. human milk contains less than half the protein of cow milk formula
B. the hepatic metabolism of protein is ineffective in most preterm infants
C. the protein in preterm human milk is of poor nutritional quality
D. VLBW infants have excessive gastrointestinal losses of ingested protein
E. VLBW infants require an increased protein intake because of their high catabolic rate

Suggested answer: E. Though VLBW benefit most from human milk, VLBW infants require higher
protein intake to sustain adequate growth. They also have higher protein turnover rates. As a result, for
VLBW infants, human milk should be fortified with extra protein to meet these increased needs. VLBW
infants fed human milk without protein fortification have slower growth rates, lower BUN, and lower
serum albumin, demonstrating the importance of protein supplementation.

Human milk and formula contain almost equivalent amounts of protein; however, human milk contains
more whey protein (quickly digested) whereas many formulas contain more casein protein (more slowly
digested). There is no evidence that VLBW have increased protein losses from their gut.

Question 20:
A 3-year-old boy underwent a hepatoportoenterostomy (Kasai procedure) for extrahepatic biliary atresia
at 6 weeks of age. He has been receiving cholestyramine to treat severe pruritus for the past 2 months. Of
the following, the nutrient MOST likely to be malabsorbed because of this patients underlying liver
disease and its treatment is:
A. carbohydrate
B. fat
C. protein
D. trace elements
E. water-soluble vitamins

Suggested answer: B. This patient will have low luminal bile acids for two reasons: 1) poor bile acid
secretion into the gut, secondary to biliary atresia; and 2) poor bile acid activity in the gut, secondary to
binding with cholestyramine. Bile acids, pancreatic lipase, and pancreatic colipase work together to
digest fats efficiently. Without bile acids, fat malabsorption will occur. Neither BA nor cholestyramine
impairs duodenal or pancreatic function, so protein and carbohydrate absorption should be intact.
Dr-Wahid Helmi

Question 21:
A 5-year-old girl who has severe developmental delay is fed exclusively through a gastrostomy tube.
Within 1 day of a change in her tube feeding formula, she develops diarrhea, abdominal distress, and
flatulence. The stool pH is less than 5.0. These findings are MOST consistent with malabsorption of:
A. carbohydrate
B. fiber
C. lipid
D. protein
E. vitamins

Suggested answer: A. This child has signs of symptoms of excessive carbohydrates in her feeds. Sugars
that are not absorbed in the small intestine are delivered to gut bacteria in the colon. Here bacteria
metabolize sugars into short chain fatty acids. The byproduct of this reaction is gas (causing bloating)
and an increased colonic osmotic load (causing diarrhea). Furthermore, the short-chain fatty acids
cause the stool to be acidic.

Question 22:
A 10-month-old African-American infant who was born at 30 weeks gestation weighed 1,400 g at birth.
She has been breastfed exclusively and has received no supplemental vitamins. General growth velocity
has been slow, but development has been normal. Physical examination reveals enlargement of the
costochondral junctions and slight thickening of the wrists and ankles. This infant MOST likely has a
deficiency of:
A. folic acid
B. vitamin A
C. vitamin C
D. vitamin D
E. vitamin E

Suggested answer: D. This child has Vitamin D-deficiency rickets. Premature babies are at higher risk
for developing rickets, possibly secondary to poor maternal transfer of calcidiol through the placenta,
poor bile acid production to absorb fat-soluble vitamins, and lack of sunlight in the NICU. In addition,
breastfed infants receive inadequate Vitamin D in milk, as breast milk only contains 12-60 IU/L
(recommended dose for infants controversial, and spans 200-400 IU/day). Hence, breastfed infants must
receive supplements. Formula fed infants do not need supplements, because formula contains adequate
amounts of added Vitamin D.

Question 23:
You have volunteered to work in a refugee camp in Rwanda. You are examining a 15-month-old infant
whose diet has been a dilute carbohydrate gruel since he was weaned from the breast at 9 months of age.
Physical examination reveals pallor; apathy; thin, pale hair; a desquamating skin rash; pitting edema of
the lower extremities; and weight at the 80th percentile for age. Among the following, the MOST likely
diagnosis is:
A. kwashiorkor
B. marasmus-kwashiorkor
C. marasmus
D. vitamin A deficiency
E. vitamin C deficiency

Suggested answer: A. This patient has kwashiorkor. Marasmus occurs secondary to inadequate total
calories, and is characterized by low muscle and fat mass. Children look wasted. Kwashiorkor occurs
Dr-Wahid Helmi

secondary to adequate total calories but inadequate protein, and is characterized by low muscle mass
and normal fat mass. Children look edematous and may have a scaly dermatitis resembling flaky paint.
In the mixed picture, patients start with marasmus but then develop a pro-inflammatory state (i.e. from
infection), leading to an edematous appearance. Vitamin A deficiency causes corneal dryness
(xerophthalmia) leading to scarring and night blindness, whereas Vitamin C deficiency causes impaired
collagen synthesis and scurvy.

Question 24:
A 3-week-old boy who was born at 28 weeks gestation is being fed 100 kcal/kg per day of fortified (24
kcal/oz) breast milk by gavage. He also is receiving aminophylline for apnea and low-flow oxygen and
diuretic therapy for chronic lung disease. He is gaining weight at a rate of 5 g/day. Of the following, the
MOST likely explanation for this child’s slow weight gain is that:
A. administration of a loop diuretic causes excessive loss of glucose and protein
B. aminophylline therapy impairs fat absorption in the upper small intestine
C. multiple episodes of apnea/bradycardia increase the basal metabolic rate
D. the daily caloric intake is less than the recommended range for preterm infants
E. the metabolic rate of a growing preterm infant is twice that of a term infant

Suggested answer: D. Preterm infants have increased energy needs, in part because they have lower
body stores of fat and glycogen, expend more energy controlling their temperature, and have to support
high rates of growth relative to their body size. Their metabolic rate is more than that of a term infant
(but not twice as much), and hence they need more calories that the standard 100 kcal/kg/day needed by a
term infant.

Infants are commonly given theophylline/aminophylline (caffeine) for apnea and bradycardia, which also
increases energy expenditure and may impair growth but does not impair fat absorption. Loop diuretics
should not cause protein or glucose loss. Finally, apnea/bradycardia episodes may increase energy
expenditure but would not affect the basal metabolic rate.

Question 25:
A 6-week-old boy who was born at home and who has been exclusively breastfed has had diarrhea for 5
days. His parents bring him to the emergency department because he has multiple deep ecchymoses and
bloody stools. Until the results of laboratory studies become available, the best INITIAL management of
this patient is to administer intravenous?
A. cryoprecipitate
B. factor VIII concentrate
C. fresh frozen plasma
D. platelets
E. vitamin K

Suggested answer: E. Newborns receive Vitamin K supplementation at birth because many are vitamin
K deficient. There are many reasons for this deficiency: 1) low vitamin K stores at birth, 2) poor
placental transfer of vitamin K, 3) low levels of vitamin K in breast milk (but adequate amounts in
formula), and 4) low colonic bacteria that normally generate vitamin K. Without adequate vitamin K,
infants can develop bleeding at the umbilicus, in the mucous membranes, in the GI tract, at circumcision
sites, and at IV sites. They can also have hematomas at sites of trauma, as well as life-threatening
intracranial bleeding.

Vitamin K administration is the first treatment of choice. For active bleeding, fresh frozen plasma should
be administered. Prothrombin complex concentrates (PCC) can be given in life-threatening situations.
Dr-Wahid Helmi

Question 1:
Shortly after birth, a 3,500 g term newborn is found to be jittery and to have a high-pitched cry. Physical
examination reveals tachypnea and a liver edge that is palpable several centimeters below the umbilicus.
Blood glucose concentration is 14 mg/dL.

Among the following, the MOST likely cause of the hypoglycemia in this newborn is:

A. galactokinase deficiency
B. glycogen storage disease
C. insulinoma
D. maternal diabetes mellitus
E. prolonged maternal labor

Suggested answer: B. Hypoglycemia in an infant suggests either too much insulin (insulinoma, maternal
DM) or too little glucose release into the bloodstream in non-feeding states (GSD). MDM is much more
common than insulinomas, and neither are associated with hepatomegaly,. On the other hand, GSD is
associated with a large liver secondary to glycogen entrapment. Other signs include lactic acidosis,
slight ketosis, and hyperuricemia.

Galatosemia is caused by defects in one of two enzymes. Classic galactosemia occurs from defects in
galactose-1-phosphate uridyltransferase (GALT), which results in accumulation of galactose-1-
phosphate, galactose, and oxidative and reductive products galactitol and galactonate. Patients develop
hepatosplenomegaly and mental retardation, in addition to cataracts and galactosuria. The other
mutation is in galactokinase, which works one step ahead of GALT and is used to create galactose-1-
phosphate. Infants with mutations in this gene have cataracts and galactosuria, but do not have
hepatosplenomegaly or mental retardation (presumably because galactose-1-phosphate is not produced
so does not accumulate). Cataracts cccur in both types of galactosemia, when excessive galactose is
converted to the osmotically active galactitol in the lens/eye.

Question 2:
A breastfed infant who appeared healthy at birth develops chronic diarrhea, failure to thrive, and
hepatomegaly during the first few weeks of life. Ultrasonography reveals adrenal enlargement and
calcification.

Of the following, the MOST likely explanation for these findings is:

A. cystic fibrosis
B. glucose-galactose malabsorption
C. glycogen storage disease
D. Niemann-Pick disease
E. Wolman disease

Suggested answer: E. Wolman disease results from recessive mutations in lysosomal acid lipase. This
enzyme acts on endocytosed lipoproteins, hydrolyzing cholesteryl esters and triacylglycerols. Without the
enzyme, patients accumulate cholesteryl esters and triacylglycerols in various tissues. Patients have
severe diarrhea (presumably from intestinal involvement), malnutrition, abdominal distension,
hepatosplenomegaly, and calcification of the adrenal glands. Patients do not survive past infancy.
Niemann-Pick Types A, B, and C disease are associated with neurological changes rather than diarrhea.
Types A and B are caused by defects in acid sphingomyelinase activity. Sphingomyelin accumulates
intracellularly, producing hepatosplenomegaly, “foam cells,” and, in Type A, severe neurodegenerative
Dr-Wahid Helmi

disease and death by 3 years. Type C disease, caused by mutations in NPC1 or NPC2, involves defects in
cholesterol trafficking from lysosomes to other compartments. As a result, excessive lipids are found in
lysosomes.

Glucose-galactose malabsorption causes life-threatening diarrhea and dehydration in the first few weeks,
associated with renal (not adrenal) calcium deposits. The disease is caused by recessive mutations in the
sodium/glucose cotransporter SGLT1, which is the main transporter involved in transporting glucose and
galactose into enterocytes. As a result, glucose and galactose remain in the gut lumen and act as an
osmotic drive causing diarrhea. SGLT1 is also found in the kidney, and mutations can also lead to
glucosuria. Treatment involves fructose-based formulas that do not contain glucose or galactose.

Question 3:
A 4-week-old infant is jaundiced. Findings include weight and length at the 75th percentile for age;
icterus; hepatosplenomegaly; total bilirubin, 6.3 mg/dL; direct bilirubin, 5.5 mg/dL; alanine
aminotransferase activity, 130 U/L; aspartate aminotransferase activity, 143 U/L; and gamma-glutamyl
transpeptidase activity, 950 U/L.

Of the following, the BEST study to evaluate the excretion of bile from the liver is:

A. computed tomography of the liver


B. hepatic ultrasonography
C. hepatobiliary scintigraphy
D. measurement of galactose-1-phosphate uridyltransferase activity
E. measurement of the serum alpha1-antitrypsin level

Suggested answer: C. Bile excretion can be documented with the HIDA (hepatobiliary iminodiacetic
acid) scan. The procedure is based on monitoring a radiolabeled compound as it travels from the
hepatocyte, into the cannalicular space, through the biliary tree, and into the intestine. HIDA scans have
been shown to be more sensitive than specific for biliary atresia; another better, yet more invasive test, to
demonstrate patent bile ducts is the intraoperative cholangiogram.

The other choices do not measure excretion. CT is not needed in this case, whereas US is useful to
demonstrate presence of a choledochal cyst, polysplenia (found in approximately 10% of BA cases), the
triangular cord sign (thickening of left branch of portal vein in BA), and absence of gall bladder. GALT
activity is useful if classic galactosemia is suspected. Finally, low A1AT levels may hint at A1AT
deficiency, though PI (protease inhibitor) typing is a better test.

Question 4:
A 14-year-old boy has a 36-hour history of severe, continuous midepigastric pain radiating to the back,
persistent vomiting, and fever. Physical examination reveals: blood pressure, 70/40 mm Hg; temperature,
39.5ºC (103.1ºF); marked midepigastric tenderness, guarding, and rebound; absent bowel sounds; and
abdominal distention. You suspect acute pancreatitis.

The test or procedure that will be MOST specific in confirming the diagnosis is a(n):

A. abdominal ultrasonogram
B. endoscopic retrograde cholangiopancreatogram
C. serum alanine aminotransferase activity
D. serum amylase activity
E. white blood cell count
Dr-Wahid Helmi

Suggested answer: D. Amylase can be used to make the diagnosis of acute pancreatitis; however, lipase
has been shown to be sensitive and more specific. Amylase elevations can also occur in salivary disease,
intestinal disease, gynecological disease, and neoplasms. Abdominal ultrasound may or may not show
changes, and may be hampered in this case by air from abdominal distension. ERCP outlines the
pancreatic duct to identify stones or strictures, but does not outline the pancreatic parenchyma. ALT/AST
would be useful to identify gall-stone related common bile duct obstruction, leading to pancreatitis and
hepatitis. Finally, WBC counts could be high in a number of infectious or inflammatory diseases.

Question 5:
Enteric diseases commonly occur in young children who attend out-of-home child care facilities.

Of the following, the enteropathogen most likely to cause CHRONIC diarrhea in an immunocompetent
child is:

A. adenovirus
B. Cryptosporidium
C. Escherichia coli 0157:H7
D. rotavirus
E. Shigella

Suggested answer: D. Chronic diarrhea is diarrhea that occurs for more than 4 weeks. Viruses such as
rotavirus and adenovirus can cause “post-enteritis syndrome,” a form of chronic diarrhea thought to be
secondary to the acute infection. Post-enteritis syndrome may be caused by a number of factors: a)
disaccharide deficiency because of blunted villi, b) leaky mucosa leading to protein translocation and
sensitivity/allergy, and c) repeated enteric infections following the initial insult. Cryptosporidium causes
chronic diarrhea in immunocompromised hosts. E. coli and Shigella usually cause acute symptoms.

Question 6:
Among the following, the gastrointestinal disease MOST likely to respond to treatment with anti-
cholinergic medications is:

A. constipation
B. dysentery
C. gastroesophageal reflux
D. irritable bowel syndrome
E. peptic ulcer disease

Suggested answer: D. Cholinergic agonists are used to stimulate the parasympathetic nervous system,
thereby activating motility. Anti-cholinergic agents, then, would be expected to make dysmotility
problems such as constipation and reflux worse. Anti-cholinergics can be used to treat IBS patients
suffering from too much motility. Theoretically, they can also be used in peptic ulcer disease
(acetylcholine is one input to parietal cells), but more often proton-pump inhibitors or histamine receptor
agonists are chosen. Finally, dysentery should be treated by targeting the offending agent rather than by
altering motility though anti-cholinergic medications.

Question 7:
For the past 6 weeks, a 4-year-old boy has had painless, bright red rectal bleeding associated with bowel
movements. Examination of the abdomen and anus reveals normal findings. The rectal vault is empty, and
no blood is noted on gross inspection.
Dr-Wahid Helmi

Of the following, the MOST likely cause for the hematochezia is

A. hemolytic-uremic syndrome
B. Henoch-Schönlein purpura
C. intussusception
D. juvenile polyposis
E. Meckel diverticulum

Suggested answer: D. Juvenile polyposis is characterized by painless bright red blood associated with
bowel movements. Histology usually shows hamartomatous changes. Meckel’s diverticulum presents
with melena (if slow bleeding) or bright red rectal bleeding not associated with bowel movements (if fast
bleeding). HUS (a coagulopathy), HSP (a vasculitis), and intussusception (an anatomical defect) all
present with abdominal pain.

Question 8:
A 2-year-old boy has had bilious vomiting and bloody stools since last night. Physical examination
reveals a moderately ill, dehydrated child who has a scaphoid abdomen and absent bowel sounds. Stools
are maroon-colored and strongly positive for blood.

After stabilizing the patient, the INITIAL diagnostic study that should be performed is a(n)

A. acute abdominal radiographic series


B. barium swallow
C. computed tomogram of the abdomen
D. Meckel scan
E. upper endoscopic examination

Suggested answer: A. This patient has signs of symptoms of an acute abdomen, likely secondary to
volvulus causing bilious vomiting and ischemic gut. To confirm the diagnosis, radiographic films can be
used to document ileus and presence of free air. A barium swallow would be poorly tolerated because of
the intestinal obstruction, and may cause harm if perforation is present. CT of the abdomen would better
outline the patient’s anatomy, though may be more than what is needed before surgery. A Meckel’s
diverticulum would not cause upper GI symptoms, and an EGD would provide no diagnostic or
therapeutic benefit.

Question 9:
A 13-year-old girl is being evaluated for diarrhea, abdominal pain, and weight loss.

Of the following, the feature that BEST distinguishes Crohn disease from ulcerative colitis is:

A. development of crypt abscesses


B. hepatic involvement
C. mucosal ulcerations on endoscopy
D. noncaseating granulomas on mucosal biopsy
E. poor growth

Suggested answer: D. Although the treatments overlap, the type of IBD does play a role in diagnostic
tests and prognosis. Generally, UC is a mucosal disease which starts in the rectum. As the disease
progresses, it involves continuous parts of the colon but does not extend into the ileum. Crypt abscesses
are common, and hepatic involvement can be seen in the form of primary sclerosing cholangitis or
autoimmune hepatitis. Crohn disease, on the other hand, extends throughout the thickness of the bowel
Dr-Wahid Helmi

and can be present in skip lesions throughout the GI tract. Liver involvement is less common, though can
be present. The most characteristic pathological finding is the noncaseating granuloma, comprised of
macrophages coordinating an autoimmune reaction in the mucosa. Both UC and Crohn disease can be
associated with ulcers on endoscopy and poor growth.

Question 10:
A 6-month-old boy has chronic diarrhea. Findings include: weight less than the 5th percentile and length
at the 25th percentile for age; marked cachexia with protuberant abdomen; sodium, 125 mEq/L; chloride,
90 mEq/L; albumin, 2.5 g/dL; and total protein, 4.3 g/dL. Stool is negative for reducing sugars but
positive for neutral fats; a 72-hour fecal fat collection shows a coefficient of absorption of 45% (normal,
>93%).

These findings are MOST consistent with:

A. celiac sprue
B. cow milk-soy protein allergy
C. Crohn disease
D. cystic fibrosis
E. giardiasis

Suggested answer: C. Cystic fibrosis is associated with pancreatic insufficiency, secondary to


inspissation of pancreatic secretions and poor secretion. As a result, patients develop fat malabsorption,
protein malabsorption, chronic diarrhea, and malnutrition. Celiac disease starts when children consume
gluten-containing foods, so would not be present as a chronic disease in a 6 month old. Cow milk-soy
protein allergy presents as bloody stools, rectal eosinophilia, and possibly constipation, but does not lead
to failure to thrive. Crohn disease is less common in a 6 month old, and does not necessarily lead to fat
malabsorption. Finally, giardiasis can cause chronic diarrhea but would not be expected to cause
steatosis.

Question 11:
The clinical manifestations of cholecystitis differ depending on the age of the patient.

Which of the following findings is MORE likely to occur in an affected child than in an affected adult?

A. Fat intolerance
B. Fever
C. Jaundice
D. Pain radiating to the right scapula
E. Palpable mass in the right upper quadrant

Suggested answer: C. Cholecystitis is defined as inflammation of the gall bladder, usually secondary to
bile stasis and bacterial growth. It can be acute (from stone obstruction) or chronic (from stone
obstruction or poor gall bladder motility). Symptoms include fever and pain in the RUQ and radiating to
the right scapula. Pain intensifies with fatty meals that stimulate gall bladder contraction. Children
more commonly present with jaundice, even if a stone is not identified. This jaundice is presumably from
edema of the bile duct walls, preventing bile outflow and bilirubin back-up into the circulation.

Question 12:
A febrile 1-month-old infant has a generalized seizure. Findings include healthy appearance; weight and
length, 90th percentile; liver span, 11 cm; serum glucose, 20 mg/dL; alanine aminotransferase activity,
Dr-Wahid Helmi

123 U/L; aspartate aminotransferase activity, 153 U/L; total bilirubin, 2.0 mg/dL; and lactic acid, 4.7
mmol/L.

These findings are MOST suggestive of:

A. alpha1-antitrypsin deficiency
B. congenital hepatic fibrosis
C. galactosemia
D. glycogen storage disease
E. perinatal cytomegalovirus infection

Suggested answer: D. This patient is having a hypoglycemic seizure, likely from a glycogen storage
disease. In type Ia GSD, there is a deficiency of glucose-6-phosphatase. In the liver, glucose-6-
phosphatase converts glucose-1-phosphate (generated from stored glycogen) into glucose to be released
in the circulation during times of fasting. In addition to hypoglycemia and related seizures, patients with
GSD type I have hepatomegaly and hepatocyte damage from excessive stored glycogen.

A1AT could cause jaundice, elevated AST/ALT, and hepatomegaly; however, it is not associated with
hypoglycemia at diagnosis. Similarly, congenital hepatic fibrosis and CMV infection involve the liver but
would not lead to hypoglycemia so early. Galactosemia causes hepatosplenomegaly from galactose-1-
phosphate accumulation; however, it is not characteristically associated with hypoglycemia.

Question 13:
A 6-year-old boy who has chronic constipation has been treated with cathartic medications for 1 week.
You decide to place him on maintenance therapy with a lubricating laxative.

Among the following, the agent you are MOST likely to recommend is:

A. bisacodyl
B. docusate sodium
C. magnesium hydroxide
D. malt soup extract
E. mineral oil

Suggested answer: E. Mineral oil is a lubricating laxative, which coats the mucosa allowing stools to
slide easily and preventing colonocytes from reabsorbing water. Docusate sodium acts like a soap,
reducing the surface tension of stool and allowing more fat and water to enter. Magnesium hydroxide
(along with polyethylene glycol and lactulose) are common osmotic laxatives, whereas bisacodyl is a
stimulant laxative that increases persistalsis by irritating mucosal smooth muscle. Malt soup extract is a
bulk forming fiber laxative. Its cellulose content absorbs water from the intestine and causes stool to
become bulky and soft.

Question 14:
A 4-month-old boy regurgitates after all feedings. His weight has remained at the 10th percentile for age.
Normal findings on an upper gastrointestinal barium study have excluded anatomic abnormalities. You
suspect gastroesophageal reflux.

The best INITIAL management of this child would be:

A. administration of cisapride
B. administration of ranitidine
Dr-Wahid Helmi

C. administration of small, thickened oral feedings


D. change to an elemental formula
E. referral for fundoplication

Suggested answer: C. This patient has uncomplicated GER with no pain, arching, or failure to thrive.
The first intervention is to thicken oral feeds. The next intervention would be to try a hypoallergenic (not
elemental) formula, because cow’s milk allergy presents with symptoms similar to GER. Cisapride, a
pro-motility agent, is not available due to associations with heart arrhythmias. Ranitidine or a proton-
pump inhibitor may be good choice if the patient showed symptoms of pain, such as back-arching;
however, without these symptoms, acid suppression is not warranted. Finally, a fundoplication is
excessive for uncomplicated GER.

Question 15:
Examination of a developmentally normal 7-month-old boy reveals moderately enlarged cervical lymph
nodes; a hemorrhagic seborrhea-like rash on the forehead, scalp, and trunk; and hepatosplenomegaly.
Laboratory findings include: hemoglobin, 12.0 g/dL; mean corpuscular volume, 82 fL; white blood cell
count 10,700/mm³, with 40% neutrophils and 60% lymphocytes; and platelet count 260,000/mm³.

These findings are MOST consistent with:

A. acute lymphoblastic leukemia


B. aplastic anemia
C. Langerhans cell histiocytosis
D. neuroblastoma
E. Niemann-Pick disease

Suggested answer: C. Langerhans cell histocytosis results from abnormal proliferation of the Langerhan
cell, a dendritic cell subtype. Multisystem LCH involves the skin (rash), lymph nodes, liver (liver
dysfunction, primary sclerosing cholangitis), spleen, and various other organs. LCH is treated with
chemotherapy. In this scenario, ALL and aplastic anemia are unlikely because there is no anemia.
Neuroblastoma can present with an abdominal mass. Niemann-Pick disease involves cholesterol
trafficking, and leads to neurodegenerative changes.

Question 16:
A previously healthy 12-year-old boy presents with an upper gastrointestinal tract hemorrhage. Findings
include: hepatosplenomegaly, ascites, a prominent vascular pattern on the abdomen, thrombocytopenia,
mildly elevated aminotransferase activity, markedly elevated gammaglutamyl transpeptidase activity, and
a prolonged partial thromboplastin time.

Which of the following laboratory studies is MOST likely to provide a diagnosis?

A. Alpha1-antitrypsin level
B. Fasting blood glucose level
C. Galactose-1-phosphate uridyltransferase activity
D. Hepatobiliary scintigraphy
E. Sweat test

Suggested answer: E. This patient has chronic liver disease (as demonstrated by the many signs of portal
hypertension) caused by a biliary etiology (as suggested by high GGT levels). The most likely diagnosis
is cystic fibrosis-related liver disease. CFTR is present in biliary epithelium and promotes proper bile
flow. Without it, bile backs up and leads to liver cirrhosis. In addition, as in this case, children with CF-
Dr-Wahid Helmi

related liver disease may have minimal lung disease and no growth problems. Biliary atesia causes
similar symptoms and uses hepatobiliary scans in its work-up. BA, however, presents in infancy.

Alpha-1-antitrypsin deficiency can present with cirrhosis in adolescents, but usually AST/ALT levels are
higher secondary to misfolded A1AT protein accumulating in hepatocytes. Also, A1AT is best diagnosed
with PI typing, because alpha-1-antitrypsin levels may be falsely elevated in times of inflammation.
Fasting blood glucose may be low secondary to liver dysfunction, but would not identify the primary liver
disease. GALT deficiency leads to classic galactosemia, which presents in infancy.

Question 17:
A 14-year-old boy is being evaluated for jaundice that was first noted 1 week ago following an upper
respiratory tract infection. He reports not feeling very hungry for the past month. Physical examination
reveals a firm liver, an enlarged spleen, and an intention tremor.

Among the following, the test that would be MOST helpful for making a definitive diagnosis in this
patient is a:

A. liver biopsy for copper content


B. serum bilirubin concentration
C. serum ceruloplasmin level
D. serum transaminase activity
E. slit lamp examination of the cornea

Suggested answer: A. In an adolescent with liver disease and neurological findings, Wilson disease
should be suspected. Wilson disease is caused by a mutation in the ATP7B copper transporter.
Mutations result in abnormal copper trafficking at 2 levels: 1) impaired copper binding to
apoceruloplasmin inside the hepatocyte, and 2) decreased secretion of copper into the biliary system.
Copper accumulates in hepatocytes and other tissues (brain, kidneys, heart), leading to clinical
symptoms.

Only 50% of patients have the classical findings of Kayser-Fleischer rings (detected by slit lamp
examination) and low serum ceruloplasmin. Serum ceruloplasmin is especially tricky, because although
it should be low secondary to low serum copper levels, it may be elevated as an acute phase reactant in
the setting of chronic liver inflammation. Elevated serum transaminase activity or bilirubin
concentration may initiate the work-up but are not specific to Wilson disease. The gold standard for
diagnosing Wilson disease is hepatic copper content. 85% of patients have a hepatic copper
concentration greater than 250mcg/g of liver tissue.

Question 18:
A 4-month-old infant has had diarrhea for 6 weeks. Findings include: weight less than the 5th percentile
and length at the 10th percentile for age; cachexia; a protuberant abdomen; total protein, 4.3 g/dL;
albumin, 2.8 g/dL; stool pH, 4.5; stool for reducing substances, 2+; stool culture, negative for enteric
pathogens; and sweat chloride, 10 mEq/L.

The study that would be MOST helpful in determining the cause of this child’s diarrhea is

A. D-xylose test
B. serum trypsinogen level
C. small bowel biopsy
D. stool alpha1-antitrypsin level
E. upper gastrointestinal barium study
Dr-Wahid Helmi

Suggested answer: C. This patient has severe malnutrition, with poor weight compared to height, low
serum proteins, and sugar malabsorption. The differential diagnosis for this child’s condition is broad,
and includes infection, pancreatic insufficiency, and villi disease (i.e. tufting enteropathy, microvillus
inclusion disease). EGD to visualize the mucosa and collect samples for routine histology as well as
electron microscopy will help distinguish between some of these possibilities.

It is already clear that the child has malabsorption, so a D-xylose test is unnecessary. (In the test, D-
xylose, a substance absorbed in the SI, is given and then measured in blood and urine. Low values
suggest malabsorption). Serum trypsinogen is used to detect pancreatitis, which is unlikely in this child.
Elevated stool alpha-1-antitrypsin identifies protein loss in the gut, which in an infant raises the
possibility of congenital intestinal lymphangiectasia. However, in congenital intestinal lymphangiectasia,
lymphatics are blocked and drain backwards into the bowels, leading to fat – rather than sugar –
malabsorption. Finally, an UGI barium swallow is used to detect malrotation when the primary symptom
is vomiting.

Question 19:
Physical examination of a 13-year-old boy who is being evaluated for short stature reveals aphthoid
lesions in the mouth and skin tags and fissures around the anus.

Of the following, these findings are MOST consistent with

A. Crohn disease
B. herpes simplex virus infection
C. immunoglobulin A deficiency
D. pseudomembranous colitis
E. ulcerative colitis

Suggested answer: A. Crohn disease is characterized by involvement of all parts of the GI tract,
including mouth (aphthous ulcers), small intestine (poor growth), and perianal areas (skin tags and
fissures, present in 1/3 of cases). Ulcerative colitis, on the other hand, usually presents sub-acutely with
bloody diarrhea and symptoms restricted to the colon. HSV infection could cause mouth ulcers but would
not cause chronic growth problems and anal findings. Those with IgA deficiency are usually
asymptomatic. The 10% with symptoms may have diarrhea secondary to persistent Giardia infection.
Pseudomembranous colitis occurs acutely in response to C. difficile infection. It can be seen
endoscopically as membranes of host protein, mucus, and inflammatory cells over C. difficile-induced
mucosal ulcers.

Question 20:
A 4-week-old boy who was born at 28 weeks' gestation and weighed 800 g at birth is gaining weight at a
rate of only 5 g/day even though his caloric intake is 125 kcal/kg. The stool is poorly formed and bulky.
The infant is receiving a lactose-free formula containing 24 kcal/oz.

Which of the following interventions is MOST likely to result in a decrease in steatorrhea and improved
weight gain?

A. Increase the daily caloric intake to 150 kcal/kg


B. Increase the glucose polymers in the formula
C. Increase the protein intake
D. Increase vitamin A and vitamin E supplements
E. Substitute medium-chain triglycerides for long-chain triglycerides
Dr-Wahid Helmi

Suggested answer: E. Premature infants absorb long-chain triglycerides poorly, because they have not
yet developed the machinery to digest and absorb fats. They have poor pancreatic and bile excretion,
leading to low amounts of lipase, colipase, and bile acids in the intestinal lumen. Without these enzymes,
micelles do not form, lipids are not absorbed into the intestinal lymphatics, and, as a result, fats are
passed into the stool. Medium chain triglycerides can circumvent this problem because they are
absorbed directly into enterocytes without digestion by bile acids. Hence, changing the fat content of the
formula should lead to better absorption and better weight gain.

Question 21:
A 14-year-old female adolescent who has severe juvenile rheumatoid arthritis presents to your office with
epigastric abdominal pain. Six weeks earlier she began taking a nonsteroidal anti-inflammatory drug
(NSAID) because of worsening joint complaints.

Of the following, the most appropriate INITIAL management of her symptoms would be

A. administration of an antibiotic effective against Helicobacter pylori


B. administration of an H2-receptor antagonist
C. dietary modification
D. substitution of salicylate for NSAID
E. upper endoscopy and gastric biopsy for gastric adenocarcinoma

Suggested answer: B. NSAIDs (and aspirin) cause gastric/duodenal damage by inhibiting COX enzymes
and prostoglandin production. Prostoglandins have a number of protective roles in the upper GI tract,
including decreasing acid secretion, stimulating mucus production, stimulating bicarbonate production,
and promoting vasodilation of gastric vessels to increase oxygen delivery. To counter these effects, acid
suppression therapy with H2-receptor antagonists and proton pump inhibitors can be used. Misoprostol,
a prostaglandin E analog, seems to have even more gastroprotective effects, but also causes diarrhea and
abdominal discomfort.

H. pylori may predispose patients taking NSAIDs to develop peptic ulcer disease, so testing and treating
positive cases is appropriate. There is no evidence that this patient consumes foods likely to increase
acid secretion, so dietary modification would not be the primary intervention. Upper endoscopy may be
appropriate to confirm ulcers (not adenocarncinoma) caused by NSAID/aspirin use.

Question 22:
The essential difference between elemental formulas and casein hydrolysate formulas is that elemental
formulas are MOST likely to include:

A. a hyperosmolar composition
B. an increased concentration of vitamins and minerals
C. lactose as the predominant carbohydrate source
D. medium-chain triglycerides
E. protein in the form of amino acids rather than oligopeptides

Suggested answer: E. Cow’s milk and soy milk based formulas contain large protein epitopes which can
trigger an allergic reaction. To address this, casein hydrolysate formulas are used, consisting of smaller
peptides rather than full proteins. If the peptides still induce allergy, then elemental formulas consisting
of individual amino acids are used. Individual amino acids do not produce allergic responses.
Dr-Wahid Helmi

Elemental formulas have a slightly higher osmolarity than hydrolysate formulas, but this difference is not
as important as their differing protein contents. The two formulas have equivalent vitamins and minerals,
and use corn syrup solids, cornstarch, and in some cases sucrose (Similac Alimentum) as carbohydrate
sources. The amount of MCT oils added is brand-dependent.

Question 23:
Among the following disorders, rickets is MOST likely to develop in patients who have:

A. acute pancreatitis
B. adrenal insufficiency
C. cirrhosis
D. congenital heart disease
E. lactose intolerance

Suggested answer: C. Patients with liver disease have two reasons to be Vitamin D deficient. First, if
cholestatic, they have impaired absorption of fat soluble vitamins including Vitamin D2. (The skin can
compensate to some extent, creating Vitamin D3). Second, the Vitamin D2/D3 is converted in the liver to
25-hydroxy-Vitamin D, which then passes to the kidneys and is converted to the active 1,25-dihydroxy-
Vitamin D. Patient with cirrhosis have impaired 25-hydroxylation of Vitamin D2 and D3. Acute
pancreatitis would not cause Vitamin D deficiency; however, chronic pancreatitis and pancreatic
insufficiency would result in impaired lipase/colipase secretion, and would impact absorption of fat-
soluble vitamins.

Question 24:
A 6-month-old boy developed a weepy, crusted dermatitis around the eyes, nose, mouth, diaper area,
hands, and feet about 4 weeks after being weaned from human milk to formula. He is listless, recently
developed diarrhea, and has stopped gaining weight. In addition to the dermatitis, he has sparse hair that
is fine and lightly pigmented.

Of the following, this constellation of findings is MOST consistent with a deficiency in

A. copper
B. thiamine
C. vitamin A
D. vitamin C
E. zinc

Suggested answer: E. Acrodermatitis enteropathica, a recessive mutation in the zinc transporter


SLC39A4, is responsible for this child’s symptoms. Affected infants absorb zinc poorly, and have
dermatitis (around orifices and the limbs), alopecia, diarrhea, growth retardation, frequent infections,
neuropsychiatric problems, and delayed sexual maturation. Symptoms usually start when the infant is
weaned from breast milk to formula, suggesting that breast milk has some compound that aids in zinc
absorption. Treatment is with daily zinc supplementation.

Also on the differential in this case is Menkes syndrome. Menkes syndrome is caused by a mutation in the
X-linked gene ATP7A, which is important in copper trafficking. In the small intestine, it is responsible for
copper absorption. Symptoms arise when copper-dependent mitochondrial enzymes are impaired, and
when copper abnormally accumulates in certain tissues. Infants present with “kinky” hair, growth
failure, and neurological deterioration.
Dr-Wahid Helmi

Poor nutrition can lead to deficiencies in many vitamins and minerals. Copper deficiency is associated
with sideroblastic anemia, neutropenia, failure to thrive, and skeletal abnormalities. Thiamine deficiency
(Vitamin B1) is associated with beriberi, consisting of cardiomyopathy and polyneuritis. Vitamin A
deficiency causes night blindness, and Vitamin C deficiency causes scurvy. Zinc deficiency leads to
dermatitis around perioral and perianal areas, poor appetite, diminished taste acuity, hypogonadism, and
short stature.

Question 25:
Following an acute febrile illness, a 6-month-old infant has had diarrhea for 3 weeks. When the diarrhea
began, the mother said she fed the infant an oral electrolyte solution exclusively. As the diarrhea slowed,
the mother reintroduced cow milk infant formula, and the diarrhea recurred. The infant continues to
receive oral electrolyte solution exclusively. She appears thin but is otherwise healthy.

The BEST management at this time is to administer:

A. a gluten-free diet
B. a hypoallergenic diet
C. a lactose-free diet
D. clindamycin
E. metronidazole

Suggested answer: C. Post-enteritis syndrome refers to chronic diarrhea persisting after an initial
enteric infection. It was thought that such patients have lactose malabsorption secondary to villi damage
and poor lactase production, which then leads to an osmotic diarrhea. Patients were instructed to avoid
lactose and consume formulas with glucose polymers. If intolerance persisted, then protein
hypersensitivity was thought to be the cause, presumably from damaged mucosa allowing translocation of
large peptides. To treat this scenario, patients were put on hypo-allergenic formulas.

Recent work has shown that disaccharidase deficiency and protein sensitization are actually not that
common. Rather, the mechanism of disease appears to be from repeated enteral infections following the
initial infection. Some recommend treating with probiotics to hasten recovery times.
Dr-Wahid Helmi
Question 1:
A 12 year old girl with cirrhosis due to alpha-1-antitrypsin deficiency presents for evaluation of a 6-
month history of slowly progressive exertional dyspnea. Physical examination reveals multiple spider
angiomas, clubbing, and acrocyanosis. Chest and cardiac examination are unremarkable and chest
radiography and pulmonary function tests are normal. Arterial blood gas reveals a respiratory
alkalosis with hypoxemia. Which one of the following is the best screening test to define the
diagnosis?

A. Pulmonary angiography
B. Contrast echocardiography
C. Abdominal ultrasonography with Doppler examination of the hepatic vasculature
D. Bronchoscopy
E. Right heart catheterization to assess pulmonary arterial pressure

Suggested correct Answer: B. Alpha-1-antitrypsin deficiency is caused by mutations in the A1AT


gene. The best diagnostic test is PI (protease inhibitor) typing, which is commonly mistaken as a
genotyping test. PI typing is actually a phenotype test, in which different protein forms are identified
by Western blot. There are 3 common protein variants: M (normal), S (somewhat abnormal), Z
(abnormal). There is also a null variant, caused by a mutation that leads to no protein production.

A1AT deficiency leads to liver and lung disease. Lung disease happens in adults, from progressive
damage of lung tissue from neutrophil serine proteases (there is no A1AT to counter these enzymes).
The liver injury occurs earlier, because misfolded A1AT cannot leave the hepatocyte. It accumulates
and causes damage. ZZ and SZ phenotypes are most likely to have accumulation and hepatocyte
damage in children, whereas MZ may lead to liver issues later in life. Interestingly, the null phenotype
will have no accumulation or liver damage but will have significant lung injury.

In this question, the child likely has a ZZ or SZ phenotype causing her liver symptoms. Her
respiratory symptoms could be due to concomitant A1AT lung disease. However, lung disease in
children is very rare, and would present as emphysema on exam and pulmonary function tests. In
one study, adolescents with ZZ phenotype showed no significant abnormality in pulmonary function
tests compared to age-matched controls. Another study showed a slight degree of hyperinflation in
affected children, but no studies have conclusively shown major lung disease in the pediatric
population.

The pulmonary findings of hypoxemia and alkalosis (presumably secondary to hyperventilation) are
more likely from one of two processes related to any liver disease: hepatopulmonary syndrome vs.
portopulmonary hypertension. Hepatopulmonary syndrome consists of hypoxemia, intrapulmonary
vascular dilations, and liver disease. The vascular dilations lead to pulmonary R->L shunts detected
on contrast (bubble) echocardiography or technetium 99m-labeled microalbumin study. Classic
physical findings are clubbing, cyanosis, and spider nevi. Furthermore, because the shunts are more
often found at the lung bases, symptoms usually worsen when standing.

Portopulmonary hypertension is the opposite of hepatopulmonary syndrome, because it involves


pulmonary vasoconstriction (as opposed to dilation) and increased pulmonary pressures. It is not as
well described in children. An echocardiogram showing increased tricuspid regurgitation would
suggest this diagnosis, and right heart catherization would confirm it.

Question 2:
Hepatitis A occurs in cyclical outbreaks in the United States. These outbreaks spread largely
because of:
A. HAV infection among injection drug users
Dr-Wahid Helmi
B. Promiscuous sexual behavior
C. Infected food handlers
D. Widespread vaccination programs
E. Close personal contact with infected but asymptomatic individuals, particularly children

Suggested correct Answer: E. Hepatitis A is an RNA virus acquired by direct contact or through
contaminated water/food. Most infections are asymptomatic, and nearly 100% of 18 year olds in
developing countries (before the vaccine was available) had serological evidence of past infection.
Children appear to be the reservoirs, making answer E the correct choice.

Infants and toddlers are less likely than older children or adults to develop jaundice, with one study
showing that only 1 in 12 infants with Hepatitis A developing jaundice. Small children can present
simply with diarrhea, and may be diagnosed as general acute viral gastroenteritis (whereas jaundice
is more common in adults, diarrhea is more common in children). Finally, children may be completely
asymptomatic with Hepatitis A (“anicteric hepatitis”). Despite these mild symptoms, vaccinations are
warranted in children to prevent them from harboring the virus and ultimately spreading it to adults, in
which the infection is more severe.

Question 3:
A previously healthy two year old boy is referred to you for elevation of liver function tests. When a
liver profile was drawn during an episode of fever, his serum alkaline phosphatase concentration was
elevated. He has no recent history of fractures. His growth and development have been normal. He
did not have neonatal liver disease. Review of symptoms is negative for pruritis, chronic diarrhea, or
acholic stools. His physical examination is normal. Laboratory studies at your institution confirm the
biochemical findings. Serum 25-hydroxy vitamin D levels are within the normal range.

Patient's results Normal range


Calcium 9.2 mg/dL 8.8- 10.7 mg/dL
Phosphorus 4.2 mg/dL 3.0 -5.0 mg/dL
Blood urea nitrogen 8 mg/dL 5-20 mg/dL
SGOT 28 IU/L 20-60 IU/L
SGPT 18 IU/L 5-45 IU/L
GGT 12 IU/L 6-20 IU/L
Conjugated bilirubin 0.1 mg/ dL < 0.3 mg/dL
Alkaline phosphatase 2800 IU/L 65-525 IU/L

Which of the following is the most appropriate next step to manage this child?
A. Abdominal ultrasound
B. Liver biopsy
C. Radiographs for rickets survey
D. 1,25 dihydroxy vitamin D level
E. No further laboratory tests

Suggested correct Answer: E. Some clinicians will further fractionate the alkaline phosphatase, to
determine whether it came from bone, intestine, or liver. High bone alkaline phosphatase results from
high osteoclast activity and bone turnover, such as with calcium or vitamin D deficiency. High
intestinal alkaline phosphatase has been reported in intestinal disease, such as intestinal ulcers.
High liver alkaline phosphatase comes from cholangiocyte injury and points to bile duct disease.

High alkaline phosphatase with no other clinical signs, however, often leads to a long course of lab
work and other tests with little diagnostic yield (i.e. X-rays, CT scans, liver biopsies, endoscopies). A
few case reports have been published on the issue, showing that alkaline phosphatase levels
Dr-Wahid Helmi
eventually return to normal with time. Other names for this phenomenon are benign transient hyper-
alkaline phosphatasia or benign transient hyperphosphatasemia. It is also known as “Ulysses
syndrome,” reflecting the long journey of diagnostic tests leading the clinician to no concrete
diagnosis.

Question 4:
Mutations in the cystic fibrosis transmembrane regulator (CFTR) may cause cystic fibrosis. The
protein is present on the apical surface of epithelial cells and regulates chloride secretion. Which of
the following is the best evidence that the CFTR is a c-AMP dependent protein kinase A activated
chloride channel?

A. Demonstration of c-AMP regulated chloride channels after transfection of the CFTR gene into
cells lacking CFTR
B. The presence of mutations in the CFTR in patients with elevated sweat chloride concentrations
C. Structural similarity of the CFTR to other ATP Binding Cassette (ABC) membrane transporters
known to regulate ion transport
D. Failure of heat stable enterotoxin to stimulate chloride secretion in cell with a mutation in the
CFTR
E. Demonstration of message for the CFTR in lung, pancreas, and biliary epithelium

Suggested correct Answer: A. This is a basic science question about experimental method. One
time-tested way to study a gene or protein is to introduce it into cells without it, and then study what
changes occur in the cell. In this case, introducing CFTR into a cell line without CFTR will allow the
scientist to manipulate the system to answer specific questions about CFTR.

One way to diagnose CF is the sweat test. This technique works because children with CF have high
concentrations of sodium and chloride in their sweat. They have higher concentrations of chloride
because the CFTR channel is used to reabsorb chloride from sweat as it passed through the ducts to
the skin. In CF patients, the CFTR works improperly and chloride cannot be reabsorbed, leading to
more chloride deposited in sweat.

Question 5:
A 2 year old boy with biliary atresia underwent liver transplantation for biliary atresia 6 weeks ago. He
was CMV seronegative at transplantation and received a left lateral segment graft from his mother
who was CMV seropositive. He has developed low-grade fever and vomiting. He is referred to you
for evaluation. He is taking cyclosporine, prednisone 10 mg each day and no antiviral medications.

Based on the clinical picture, which of the following would be the most specific test to confirm the
presence of active CMV disease?

A. Positive urine for CMV antigen


B. Blood sample which is CMV PCR positive
C. Rise in CMV IgG titers
D. Presence of increased CMV IgM titers
E. In situ hybridization positive for CMV in a gastric body biopsy

Suggested correct answer: E. CMV infection in normal hosts is usually inconsequential or consists of
mononucleosis-like symptoms with mild hepatitis. On the other hand, CMV infection in
immunosuppressed hosts (such as those status-post liver transplantation) can be devastating. CMV
may come from the new organ, transfused blood, or reactivation of latent infection. It can lead to a
clinical presentation similar to that of rejection, and if allowed to persist may progress to liver failure.
Dr-Wahid Helmi
To diagnose CMV disease, the best test is to identify CMV in serum, urine, or biopsy tissues. In this
case, the child has signs of CMV gastritis (vomiting), so identifying CMV is stomach tissue would
confirm the diagnosis. Urine positive for CMV antigen and PCR positive serum documents infection
somwhere in the body, but does not confirm or refute CMV causing GI symtpoms. Furthermore, PCR
is more sensitive than specific, and may be positive in patients with latent CMV with no disease.
Antibody tests can be confusing, because of maternal contributions (in <18 months) as well as
antibodies transferred in transfusions. Treatment for CMV includes IV ganciclovir until the infection
subsides, followed by PO valganciclovir.

Question 6:
Which of the following statements regarding transient lower esophageal sphincter relaxations
(TLESRs) is false?
A. TLESRs represent the major mechanism by which acid reflux occurs.
B. TLESRs reduce or obliterate the resting pressure of the lower esophageal sphincter.
C. Gastric distension increases the frequency of TLESRs
D. TLESRs cannot be induced by pharyngeal stimulation.

Suggested correct Answer: D. The LES is tonically contracted at rest, a process mediated by vagal
cholinergic and some sympathetic fibers. The LES relaxes with the esophageal peristaltic wave. The
LES also relaxes with TLSERs which may be due to increased NO production from iNOS. TLSERs
are thought to be the mechanism allowing GER to occur, and their frequency increases with gastric
distension. Simulating the pharynx, as in swallowing, also promotes LES relaxation.

Question 7:
In a double-blinded placebo controlled trial involving 200 patients, investigators observed that drug A
did not improve pulmonary function in children with cystic fibrosis. The change in pulmonary function
was normally distributed for both groups. The investigator is concerned that that drug A improved
pulmonary function but that the effect was not detected.

If the investigator is correct, which of the following is the most likely explanation for the observation:

A. Type 1 error
B. The analysts failed to use non-parametric analysis to compare the groups
C. Type II error
D. The patients in the study changed eating behavior while on drug A
E. The analysts failed to use Chi-square analysis to compare the groups

Suggested correct Answer: C. There are two types of common errors, type I and type II. Type I
errors occur when an association is found that does not really exist (false positive), whereas type II
errors occur when an association is not found when one really does exist (false negative). In
epidemiological terms, if a null hypothesis is incorrectly rejected when it is in fact true then a type I
error occurs. When a null hypothesis is not rejected despite it being false a type II error occurs.
Alpha is used to denote type I errors, whereas Beta is used to denote type II errors.

Question 8:
A 17 year old young man presents with a 3 year history of dysphagia. There has been no weight
loss, and the patient’s nutrition is good. The findings of upper endoscopy and esophageal
manometry are consistent with a diagnosis of achalasia. Which of the following statements is true?

A. The risk of esophageal perforation with pneumatic dilation is about 1 in 500.


B. Esophageal dilation must be repeated in fewer than 10% of patients.
C. Heller myotomy affords a good to excellent response in more than 80% of cases.
Dr-Wahid Helmi
D. Acid reflux is more common after pneumatic dilation than after surgical myotomy.

Suggested correct Answer: C. Achalasia is a rare disorder in children, resulting in improper


relaxation of the lower esophageal sphincter. One proposed mechanism is less NO formation,
leading to tonic LES contraction. On physical exam, patients have trouble swallowing solids and
eventually liquids. The classic manometry findings are: (1) poor esophageal peristalsis, and (2) no
LES relaxation upon swallowing. The first finding is always present, whereas the second finding may
be present to variable degrees. Other diagnostic modalities used are barium swallow study (proximal
dilations present) and endoscopy (to rule out infection, carcinoma, and leiomyoma).

Therapies commonly used (with increasing success) include: (1) calcium channel blockers, (2) Botox
injections, (3) pneumatic dilation, and (4) Heller myotomy. Pneumatic dilations can be done by a
gastroenterologist, though they pose particular challenges. Technically they are difficult, because
they require manipulations that cut the muscular layer but leave the mucosa intact. The incidence of
perforation can be as much as 5%, and most patients require repeat dilation over time. Acid reflux is
a common in all procedures that open the LES, and sometimes reflux control is attempted with
fundoplication following myotomy.

Question 9:
Cytokines act as messengers to regulate immune responses. One of the mechanisms by which the
calcineurin inhibitors, tacrolimus and cyclosporin, promote allograft survival following liver
transplantation is inhibition of transcription of the cytokine, interleukin 2. Which of the following is the
dominant mechanism by which interleukin 2 enhances the immune response to foreign tissue?

A. Promotes activation of macrophages


B. Promotes clonal expansion of activated T-cells
C. Promotes clonal expansion of plasma cells
D. Promotes activation of eosinophils
E. Promotes clonal expansion of B-cells

Suggested correct Answer: B. IL-2 binds to the IL-2 receptor and promotes T-cell expansion. IL-2
transcription is mediated by NF-AT (nuclear factor of activated T-cells), which moves into the nucleus
and directs transcription only after it is dephosphorylated. Calcineurin is the molecule that
accomplishes this dephosphorylation in the cytoplasm, and calcineurin activation occurs only through
an increase in intracellular calcium secondary to T-cell receptor activation. Both tacrolimus and
cyclosporine form complexes with different T-cell proteins. These complexes in turn bind to
calcineurin and inhibit its dephosphorylation activity, even in the presence of high intracellular calcium
levels.

Question 10:
A four year old boy presents to you with a history of bloody diarrhea. He was well until one week ago
when he developed bloody diarrhea while on vacation with his family. His family had been traveling
by car from California to Michigan and stopped at several fast-food restaurants. He was seen in a
local emergency room and treated empirically with amoxicillin. His symptoms have persisted. Today,
he appears pale and is listless. His serum hemoglobin is 7.8 and his white blood count is 14,000. His
stool contains blood and mucus. Which of the following is the most appropriate next step in
managing this child?

A. Stool culture and sensitivity


B. Treatment with Trimethoprim-Sulfa
C. Observation and reevaluation tomorrow
D. Colonoscopy
Dr-Wahid Helmi
E. Complete blood count, renal group and urinalysis

Suggested correct answer: E. This patient has signs of hemolytic uremic syndrome (HUS), a
vasculitis characterized by microangiopathic anemia, thrombocytopenia, and uremia. In children the
majority of cases follow a diarrheal illness caused by Shiga-like toxin, usually from E. coli type
O157:H37. The bacteria cause symptoms through 2 mechanisms: (1) local effacement of villi,
leading to an early watery diarrhea, and (2) toxin secretion, producing damage in epithelial and
microvascular endothelial cells throughout the body including the intestines and kidney, leading to
renal damage and bloody diarrhea. Damage to endothelial cells results in localized clotting, which
traps platelets and shears passing red blood cells. Coagulation times are normal in HUS.

Stool cultures should be done for public health reasons, especially in the cases of severe bloody
diarrhea. However, the most appropriate immediate step is to confirm the diagnosis through a CBC
and smear, and monitor for ongoing kidney damage (HUS is a leading pediatric cause of acute renal
failure in many countries). Antibiotic treatment is considered a contraindication, because lysis of
bacteria may release more Shiga-like toxin and worsen the vasculitis.

Question 11:
A 14 year old African-American young woman is referred for evaluation of asymptomatic elevation of
serum transaminases. She has not received blood transfusions. She is not taking medications.
Examination shows that her body mass index is 34. She is anicteric and has no signs of portal
hypertension but has prominent acanthosis nigricans in her neck folds and axilla.

Laboratory evaluation

Hepatitis B surface Antigen Negative


Hepatitis B surface antibody Positive
Hepatitis C antibody Negative
AST 100 IU/l
ALT 120 IU/l
Total serum bilirubin level 0.4 mg/dl
Serum immunoglobulin level Normal

If a liver biopsy were performed, the most probable histologic findings would be:

A. Mixed portal infiltration with necrotic hepatocytes at the limiting plate


B. Macrovesicular hepatic steatosis with mild portal inflammation
C. Microvesicular hepatic steatosis
D. Cirrhosis with portal inflammation and Mallory bodies
E. Normal histology

Suggested correct answer: B. This patient likely has non-alcoholic fatty liver disease (NAFLD). The
term encompasses a wide variety of liver pathology, from steatosis to steatohepatitis to cirrhosis. The
pathophysiology of this disease centers around insulin resistance, which is probably present in this
obese adolescent with acanthosis nigricans. The diagnosis of NAFLD is suggested by
hyperglycemia, hyperlipidemia, elevated transaminases (ALT>AST), and increased echogenicity on
liver ultrasound. A recent article on the topic concluded that normal ALT ranges at most children’s
hospitals are not sensitive for abnormal ALT levels, and that an ALT greater than 25 for boys and 23
girls should raise suspicion for liver disease.

The gold-standard diagnosis for NAFLD is (1) absence of other liver disease (i.e. infectious,
autoimmune, toxin, or genetic causes, and (2) liver biopsy showing characteristic NAFLD findings.
Dr-Wahid Helmi
The three most consistent biopsy findings are: i) macrovesicular > microvesicular steatosis, ii) lobular
inflammation consisting of neutrophils and lymphocytes (“mixed”), and iii) hepatocyte ballooning.
Fibrosis (portal > perisinusoidal in children) and glycogenated nuclei may also be present. In severe
cases, extensive fibrosis leading to cirrhosis is present.

Treatment for NAFLD is evolving. Weight loss has proven to be beneficial. Other therapies being
tested include insulin sensitizing agents (i.e. metformin), anti-oxidants to prevent oxidative damage
(i.e. Vitamin E), and N-acetylcysteine, among many others. Liver transplant for patients with NAFLD-
related cirrhosis has been performed, though fatty changes can reoccur in the transplanted organ.
Bariatric surgery is also a potential new therapy, because it often improves insulin sensitivity which
may in turn treat the pathophysiology underlying NAFLD.

Question 12 and 13:


A 19-month-old female infant came to the emergency room with a 5-day history of intermittent
abdominal pain and diarrhea. She has been passing 8 to 10 brown, watery stools a day. There was
no blood or mucus in her stools. She had vomited twice (once at the onset of the illness and on the
day of presentation), and the vomitus was nonbloody and nonbilious. She had no fever or other
systemic symptoms. There was no prior antibiotic use. Her mother had diarrhea for 2 days at the
onset of her illness.

Physical examination results were within normal limits except for right lower quadrant tenderness. No
masses were palpated. Rectal examination results were normal, but her stool was guaiac positive.
Laboratory study results included a hemoglobin concentration of 11.9 g/dL and a leukocytosis of
17,400/mm 3 with a normal white blood cell differential count. Renal profile and urinalysis results were
normal. Her stool was negative for leukocytes but positive for Clostridium difficile toxin and antigen.
Abdominal radiographs were obtained (Fig 1), and when she continued to have significant abdominal
pain, an abdominal computed tomography was requested (Fig 2).

Figure 1: Figure 2:

12. Based on the findings on the imaging studies what is your diagnosis?
Dr-Wahid Helmi

A. Pseudomembranous colitis
B. Ileocolonic intussusception
C. Lymphoma
D. Colonic polyp
E. Distal intestinal obstruction syndrome

13. What is the next appropriate step in the treatment of the patient?

A. Surgical resection
B. Colonoscopy
C. Air enema
D. Sweat chloride
E. Oral Flagyl

Suggested correct answers: A, E. Clostridium difficile is a gram-positive anaerobic bacillus that can
produce an infectious colitis. This usually happens after antibiotics are administered, which allows C.
difficile to flourish by clearing the bowels of normal flora. Infections are also commonly seen in
patients receiving anti-neoplastic therapy. C. difficile elaborates Toxin A (disease causing) and Toxin
B, which are identified in ELISA (protein) or PCR (gene) assays. C. difficile usually causes a mild
limited watery diarrhea, but long-term diarrhea and/or bloody diarrhea can also occur. In
pseudomembranous colitis, the most severe form of C. difficile infection, high fever, leukocytosis, and
hypoalbuminemia are present. Imaging shows boggy, thickened areas of bowels.

Treatment consists of stopping causative antibiotics and/or starting anti-C. difficile antibiotics. Oral
flagyl is the first line choice, followed by oral vancomycin. IV Flagyl is also acceptable, though IV
vancomycin is thought to be ineffective because it does not penetrate into the gut lumen. Surgery
may be needed in especially severe cases of pseudomembranous colitis.

Question 14:
The colon is able to absorb each of the following EXCEPT:
A. Monosaccharides
B. Acetate
C. Propionate
D. Butyrate

Suggested correct answer: A. The colon’s best known functions are water absorption and fecal
storage. However, the colon is also an enormous reservoir of gut bacteria. Bacteria metabolize
carbohydrates that make it past the small intestine, including digestion-resistant starches, excess
fructose and lactose, and poorly digestible monosaccharides such as lactulose, sorbitol, and
sucrolose. Bacteria ferment these sugars, producing gas and short-chain fatty acids such as acetate,
propionate, and butyrate as a by-product. These short-chain fatty acids in turn are absorbed by
colonocytes and may have numerous benefits including trophic properties for enterocytes.

Monosaccharides are absorbed in the small intestine, and are the only form in which sugars can be
absorbed. Salivary and pancreatic amylase digests starch into oligosaccharides, which are further
digested by brush border enzymes into the monosaccharide glucose. The brush border enzyme
lactase digests lactose into the monosaccharides glucose and galactose, whereas the brush border
enzyme sucrase digests sucrose into the monosaccharides glucose and fructose. The
monosaccharides then enter enterocytes through channels, because they are too big for passive
diffusion. Glucose and galactose are co-transported with sodium driven by a sodium gradient.
Fructose has its own channel that operates via diffusion from high fructose (lumen) to low fructose
Dr-Wahid Helmi
(inside the cell) gradients. The Na-glucose transporter on villi is the bases for oral rehydration
solution containing 2 molecules of sodium for every molecule of glucose. The diffusion-driven
fructose channel explains why children who drink too much juice have excessive fructose that cannot
enter enterocytes, leading to high fructose in the lumen and subsequent osmotic diarrhea (toddler’s
diarrhea).

Question 15:
A 2-year-old boy was referred to the Pediatric Gastroenterology Unit because of a short period of
vomiting, dysphagia, and decreased appetite. He had been operated at birth for an esophageal
atresia; the postoperative course and recovery had been uneventful. He remained healthy until the
present episode of acute vomiting and dysphagia, which began 48 hours before referral to our unit.
Physical examination was unremarkable. Complete blood count and sedimentation rate were normal.
Contrast radiography of the upper gastrointestinal tract ruled out anastomotic esophageal stricture but
strongly suggested the presence of a foreign body obstructing the lower esophagus. The child
underwent esophagogastroscopy under general anesthesia. At esophagogastroscopy, no foreign
body was present, but an ulcerative lesion of the lower esophageal region was noted, corresponding
to the location of the presumed esophageal foreign body before spontaneous migration.
Esophagoscopy was followed by a systematic examination of the stomach, pylorus, and duodenum.
Surprisingly, a 10-mm diameter lesion was noted in the lumen of the prepyloric region (Fig 1); the
surrounding mucosa was normal. Biopsies of this lesion were performed.

Figure 1

This lesion most likely represents:

A. Gastric leiomyoma
B. Heterotopic pancreas
C. Peptic ulcer
D. Gastric adenocarcinoma
E. Gastric hyperplastic polyp
Dr-Wahid Helmi
Suggested correct answer: B. Heterotopic pancreas, also known as a pancreatic rest, is a congenital
anomaly characterized by pancreatic tissue in ectopic locations. The defect results from abnormal
migration of pancreatic tissue. The most common site of pancreatic rests is in prepyloric gastric
antrum, though numerous other sites have been reported such as the duodenum, ileum, gallbladder,
common bile duct, and splenic hilum. There are a few case reports of distal esophagus pancreatic
rests in patients with isolated esophageal atresia.

Pancreatic rests are usually asymptomatic and found incidentally. They appear as a round, smooth
submucosal mass with a central crater. They can be symptomatic when large (>1.5 cm), leading to
abdominal pain, dyspepsia, and bleeding. Pancreatitis has also been described in pancreatic rests.

Neoplasms such as gastric adenocarcinoma usually do not occur in such young patients. Gastric
polyps commonly come in two varieties: (1) parietal cell hyperplasia secondary to PPI use, and (2)
hamartomas such as those in Peutz-Jehger syndrome. Reflux and strictures related to reflux may
occur after repair of esophageal atresia, because an intact LES sphincter is not present. However,
stomach protective mechanisms are functional and there is no increased incidence of peptic ulcers.

Question 16:
A one month old infant boy presents to you with complaints of persistent jaundice. The infant was a
term product of an unremarkable pregnancy. Since going home, he has been breast fed and has
nursed well. His mother has a prominent forehead with deep set eyes. Examination of the infant
shows an icteric infant boy. He is afebrile. Heart examination shows an III/VI holosystolic murmur. His
liver is palpable 2 centimeters below the right costal margin in the midclavicular line and he does not
have ascites. An abdominal ultrasound showed a small gall bladder. Pertinent laboratory studies
include:

Alkaline phosphatase 650 IU/L


Total serum bilirubin 7 mg/dl
Conjugated serum bilirubin 6.2 mg/dl
AST 95 IU/L
-glutamyl transpeptidase 650 IU/L
Serum albumin 3.6 mg/dL

Which of the following is the most likely diagnosis?

A. Biliary atresia
B. Alagille Syndrome
C. Galactosemia
D. Sclerosing cholangitis
E. Cystic fibrosis

Suggested correct answer: B. This patient has neonatal cholestasis. The differential of this condition
is broad, and includes infectious, genetic, metabolic, and toxin causes. Biliary atresia must be ruled-
out, because Kasai operations performed earlier (<45 days of life) may have the best outcome.
Galactosemia, in which the child is unable to convert galactose to glucose secondary to deficiencies
in one of 3 enzymes, leads to high serum galactose levels. It presents with weight loss, diarrhea,
vomiting, lethargy, positive reducing substances (i.e. galactose) in the urine, and, commonly, E. coli
sepsis. CF can present as neonatal cholestasis but cardiac findings are not characteristic. Neonatal
sclerosing cholangitis is a rare entity that presents histologically like biliary atresia. It differs clinically
because in sclerosing cholangitis the stools are pigmented. Sclerosing cholangitis is best diagnosed
by ERCP.
Dr-Wahid Helmi
In this patient, the combination of cardiac findings (usually pulmonary stenosis), cholestasis, and
family history points to Alagille’s syndrome. The disorder is characterized by paucity of
intrahepatic/interlobular bile ducts. It is caused by defects in the Notch signaling pathway, which is
critical for proper cholangiocyte differentiation. The most common mutation in Alagille’s syndrome is
in Jagged 1, the ligand for the Notch receptor. Because the Notch pathway is used in many
developmental processes, Alagille’s syndrome patients can have numerous extra-hepatic
phenotypes. These include cardiac, skeletal (i.e. butterfly vertebrate), ocular (posterior embryotoxin),
vascular (intracranial bleeding), and renal defects (in light of all these issues, pruritis from cholestasis
is still the complaint most often made by patients). For still unclear reasons, the penetrance of Notch
mutations varies, so that individuals in the same family with the same mutation may be affected to
different degrees.

Question 17:
A 14 year old young man is referred for evaluation of abdominal pain. He had a history of
gastroesophageal reflux disease that was refractory to medical therapy. He underwent a laparoscopic
fundoplication 6 months ago. He now complains of postprandial abdominal pain and distension. The
pain is not associated with vomiting, diarrhea, dysphagia or heartburn.

Which of the following is the most-likely cause for his symptoms?

A. Non-ulcer dyspepsia
B. Biliary tract dyskinesia
C. Recurrence of gastroesophageal reflux disease
D. Gas-bloat syndrome
E. Eosinophilic esophagitis

Suggested correct answer: D. The fundoplication operation takes proximal portions of the stomach
and wraps it around the lower esophagus to recapitulate LES tone. In the past it was a mainstay for
medication-resistant reflux. Subsequent studies, however, have shown that a sizeable number of
children must return to PPI use within a few years of the surgery. Other complications include “gas
bloat syndrome,” which occurs when stomach gas cannot escape through the esophagus. Symptoms
include abdominal distension, abdominal pain, wretching, and gagging.

Question 18:
All of the following statements regarding familial pancreatitis are true EXCEPT:

A. Pattern of inheritance is autosomal recessive


B. The mutated gene encodes cationic trypsinogen
C. Males and females are equally affected
D. Usually presents before 15 years of age
E. Patients with this disorder have a 50-fold increased risk of developing pancreatic cancer

Suggested correct answer: A. Familial pancreatitis involving the cationic trypsinogen gene (PRSS1)
is autosomal dominant. Mutations cause trypsinogen to autoactivate in acinar cells, producing trypsin
which in turn activates other pancreatic proenzymes. Together, the active enzymes collectively
destroy pancreatic tissues (normally, trypsinogen is activated only in the intestinal lumen by
enterokinase on the brush border). Other genetic causes of pancreatitis include mutations in the
serine protease inhibitor Kazal type 1 (SPINK-1). Recessive mutations in the CFTR gene can also
lead to chronic pancreatitis, by thickening the composition of secretions in the duct thus causing
backflow into the pancreas. Cases of hereditary pancreatitis usually present before age 10, and
patients with hereditary pancreas have a much higher risk for developing pancreatic cancer,
presumably due to many years of chronic inflammation.
Dr-Wahid Helmi

Question 19:
A 17-year-old girl is referred to you for evaluation of persistent episodic burning epigastric pain. An
adult gastroenterologist previously saw her. She had a history of using non-steroidal anti-
inflammatory drugs once each week for headaches. Serology for H. pylori was positive and
esophagogastroduodenoscopy showed a duodenal ulcer. She was given a 14-day treatment of
omeprazole and amoxicillin. Her symptoms have persisted. A stool antigen test for H. Pylori
performed more than 4 weeks after treatment was positive.

Which of the following is the most likely cause for her persistent symptoms?
A. Unrecognized esophagitis
B. Failure to adhere to the medical regimen
C. Primary resistance of H.pylori to amoxicillin
D. Failure to use an effective treatment for H.pylori eradication
E. Peptic disease due to NSAID

Suggested correct answer: D. This patient’s symptoms could be attributed to many causes, including
H. pylori gastritis, peptic ulcer disease, and eosinophilic disease. Given the positive H. pylori test and
duodenal ulcer, H. pylori disease is the most likely. H. pylori is a gram negative rod. Stool antigen
testing is the noninvasive diagnostic test of choice (serology, as performed in this case, is not
recommended, because standard values have never been established in children). Treatment
consists of a PPI plus 2 antibiotics, such as amoxicillin and clarithromycin. Antibiotic resistance has
been identified, and current guidelines recommend susceptibility testing only if the first round of
treatment fails to eradicate the organism.

One important area of active research is determining when to test children for H. pylori. Many
children are carriers (some populations have infection rates >50%), so results may return positive
even if H. pylori is not responsible for the child’s symptoms. H. pylori should be tested when ulcers
are found endoscopically, in iron deficiency refractory to therapy, in patients with MALT lymphoma, in
those with a family history of gastric cancer, and when checking the success of eradication therapy.
H. pylori should not be checked in patients with recurrent abdominal pain/non-ulcer dyspepsia,
GERD, and asymptomatic children. Practitioners who discover H. pylori incidentally (i.e. during
foreign body removal, because a primary care physician ordered the test) must make a decision on
whether or not to treat. Many practitioners will treat, given the H. pylori-associated long terms risks of
MALT lymphoma and gastric cancer.

Question 20:
A 22-month-old boy is referred for evaluation of poor growth and chronic diarrhea since birth. His
stools are large and foul-smelling. Stool cultures and examination of stool for ova and parasites has
been negative. His past medical history is remarkable for recurrent otitis media. His physical
examination shows that his weight and height are both below the 5th percentile for age. His abdomen
is protuberant. A sweat chloride was normal. His serum trypsinogen is low. Abdominal ultrasound
shows a large echogenic pancreas.

Which of the following laboratory abnormalities is most likely to be detected in this boy?

A. Increased stool alpha-1 antitrypsin levels


B. Elevated anti-endomysial antibody titers
C. Neutropenia
D. Peripheral eosinophilia
E. Macrocytic anemia
Dr-Wahid Helmi
Suggested correct answer: C. This patient shows evidence of pancreatic insufficiency, including poor
fat absorption, failure to thrive, low serum trypsinogen, and presumably low fecal elastase. The
negative sweat test argues against CF, making Shwachman-Diamond syndrome the most likely
diagnosis. Shwachman-Diamond syndrome leads to improper development of acinar cells (vs.
pancreatic duct problems in CF). Pancreatic tissue is replaced with fatty tissue, and pancreatic
lipomatosis produces an echogenic image on ultrasound. Treatment includes pancreatic enzyme and
fat soluble vitamin replacement. With time, pancreatic function does appear, with half of the patients
demonstrating pancreatic sufficiency by age 4.

Shwachman-Diamond syndrome also has bone marrow and skeletal abnormalities. Bone marrow
problems include recurrent neutropenia, anemia, thrombocytopenia, and possible leukemia. Skeletal
problems involve long bones and the thoracic cage. The inheritance is autosomal recessive, in an
incompletely characterized gene names SBDS. The product of this gene is likely to play a
fundamental role, because it is present in all archaea and eukaryotes. Another syndrome
characterized by pancreatic insufficiency secondary to fatty replacement of the pancreas is
Johanson-Blizzard syndrome. This syndrome comes with multiple congenital abnormalities
(deafness, imperforate anus, urogenital malformations, dental anomalies, “beak-shaped” face) as well
as significant endocrine involvement (hypothyroidism, panhypopituitarism, diabetes, growth hormone
deficiency).

Question 21:
A two year old boy with biliary atresia presents with anemia and hematemesis. He underwent a
hepatic portoenterostomy when he was 5 weeks old. Today, he is pale. His abdomen is non
distended but he has prominent abdominal wall vessels and his spleen is palpable 5 cm below the left
costal margin. After hemodynamic stabilization, you plan to perform upper endoscopy and
sclerotherapy. Under which of the following conditions is antibiotic prophylaxis recommended?

A. The patient has aortic stenosis


B. The patient has a history of spontaneous bacterial peritonitis
C. The patient has a history of recurrent cholangitis
D. The patient has multiple dental caries
E. The patient has an nasojejunal feeding tube

Suggested correct answer: A. Prophylactic sclerotherapy therapy versus watchful waiting in patients
with portal hypertension and esophageal varices is an active debate (banding is impractical in young
children, because the equipment needed for banding is too large). Sclerotherapy has been shown to
reduce to incidence of first-time variceal bleeds. However, these patients also had a higher incidence
of gastric variceal bleeding if gastric varices were present, presumably secondary to congestive
hypertensive gastropathy. Survival rate was not affected by prophylactic sclerotherapy.

Sclerotherapy can lead to transient bacteremia, so patients with heart disease are at increased risk.
The patients that qualify for prophylactic antibiotics have recently been modified. High risk patients
should receive antibiotics, including those with a prosthetic valve, history of endocarditis, complex
cyanotic heart disease, and systemic-pulmonary shunts. Aortic stenosis does not fit into this
category, so antibiotic therapy will be at the discretion of the endoscopist.

In cases of active bleeding, prophylactic antibiotics for all patients are warranted. Prophylactic
antibiotics have numerous benefits: i) protects patient from bacteremia during the procedure, ii) may
prevent infection from introducing endoscope, and iii) treats infections that may have caused
increased portal pressures and promoted onset of the bleed.

Question 22:
Dr-Wahid Helmi
A 22-month-old boy is referred for evaluation of chronic diarrhea. He has had persistent diarrhea
following an episode of acute gastroenteritis 4 months ago. His stools are loose and watery and of
large volume. He has never passed gross or occult blood per rectum. Despite his diarrhea, he has
grown normally and remains active. Stool cultures and examination of stool for ova and parasites has
been negative. Because of his diarrhea, his parents stopped giving him milk and he currently drinks
more than 1 liter each day of juice or Kool-aid. His physical examination is normal. Which of the
following is the cause for his chronic diarrhea?

A. Giardiasis
B. Excessive carbohydrate intake
C. Lactose intolerance
D. Celiac disease
E. Inflammatory bowel disease

Suggested correct answer: B. This patient has “toddler’s diarrhea,” an osmotic diarrhea from
excessive sugar intake. Clues to the diagnosis include normal growth and excessive juice intake.
Toddler’s diarrhea occurs with increased luminal sucrose. In the small intestine, sucrase digests
sucrose into glucose and fructose. Glucose is readily absorbed through sodium-glucose channels, in
a process that depends on sodium gradients. Fructose, on the other hand, is absorbed by diffusion
through fructose channels. When intracellular fructose levels reach a certain point, more fructose
cannot diffuse intracellularly and instead remains in the intestinal lumen. This excess fructose
becomes an osmotic force, drawing water into the lumen and leading to a watery diarrhea. The
patient’s symptoms will improve once juice intake is returned to normal.

Question 23:
A 15-year-old girl with steroid-resistant Crohn's disease who is in remission has been started on
mercaptopurine (dose 1.5 mg/kg/day). You find that she has decreased activity of the enzyme,
thiopurine methyltransferase (TPMT) (the enzyme responsible for a major alternative catabolic
pathway of mercaptopurine).

For which of the following is the patient at risk due to decreased TPMT activity

A. Relapse of Crohn's disease


B. pancytopenia
C. opportunistic infection
D. bacterial overgrowth
E. sclerosing cholangitis

Suggested correct answer: B. 6-MP is an antimetabolite initially developed to destroy immune cells in
the setting of leukemia. It is converted to a purine analogue that disrupts DNA replication. At lower
doses, it is used as immunosuppressive therapy in IBD. Azathioprine is converted to 6-MP non-
enzymatically. 6-MP then has 3 fates: i) 6-thiouracil (inactive), ii) 6-MMP (hepatotoxic), and iii) 6-TG
(active immunosuppressive compound). TPMT, or thiopurine methyltransferase, determines how
much 6-MP is converted away from the active metabolite 6-TG to 6-MMP. Patients with low TPMT
activity, or those with mutant alleles, produce excess amounts of 6-TG leading excessive immune
suppression and bone marrow failure. For this reason, TMPT activity or genotype is often tested
before administering 6-MP. Other important side effects of 6-MP include pancreatitis, hepatotoxicity
(presumably in part from 6-MMP metabolites), and possible hematopoetic neoplasm.
Dr-Wahid Helmi

Question 24:
A 16-month-old boy received a cadaveric liver transplant at age 10 months for biliary atresia. His
immunosuppressive medications are tacrolimus and prednisone. He presents to you with complaints
of diarrhea and intermittent fever. A colonoscopy shows findings consistent with post-transplant
lymphoproliferative disease (PTLD) and biopsy of the lesions shows predominantly lymphocytic
infiltrate. Which of the following is the best way to confirm the diagnosis (EBV associated PTLD)?

A. Quantitation of peripheral blood EBV viral load by polymerase chain reaction


B. Qualitative identification of peripheral blood EBV by polymerase chain reaction
C. Identification of B-cells in the biopsy by monoclonal antibody against CD20 (a specific marker for
B-cell)
D. Flow cytometry of blood to identify activated B cells
E. In situ hybridization of biopsy with EBER-1 probe (labels EBV-encoded RNA in infected cells)

Suggested correct answer: E. PTLD is a post-transplant complication related to EBV infection and
immunosuppression. When a child has been infected with EBV prior to transplant, he or she may do
better on minimal post-transplant immunosuppression to prevent EBV reactivation while also
preventing rejection. More often, children acquire EBV infection either i) during transplant from the
donor or ii) after transplant while being heavily immunosuppressed. EBV titers can be accurately
followed using serum EBV PCR measurements.

PTLD occurs when EBV-infected B cells proliferate excessively, usually (but not always) in the setting
on increased EBV PCR titers. There are at least two broad steps to this process. First, viral genes
overtake cellular machinery and promote polyclonal proliferation (ultimately, the fittest B cell clone is
selected and the lesion becomes monoclonal). Second, immunosuppression impairs T cells, which
normally check B cell proliferation. The final result is neoplastic proliferation of B cells, diagnosed
through histology using two criteria: i) evidence of lymphoproliferation on tissue biopsy, and ii)
presence of EBV (i.e. EBV RNA detected by in situ hybridization) in tissue.

Clinically, PTLD presents with mononucleosis-type symptoms, isolated lymph node involvement,
and/or lymphocyte invasion of tissues (liver, gut, iris). Gut PTLD presents with diarrhea, GI bleeding,
and weight loss. Treatment includes reduced immunosuppression (at the risk of inducing rejection),
anti-virals, surgical excision of the lesion, localized radiation therapy, chemotherapy, monoclonal
antibodies against the B cell antigen CD20, interferon therapy, and T-cell therapy.

Question 25:
A 6 year old boy is referred to you for post-prandial abdominal pain of 9-12 months duration. He has
no associated vomiting, heartburn, dysphagia, or diarrhea. His appetite is decreased. Past medical
history reveals that he found to have a Stage II Wilm’s tumor at age 2 years. He was treated with
Dr-Wahid Helmi
resection and received 3000 rads to his abdomen. Examination shows no evidence of malnutrition.
He has mild epigastric and right lower quadrant tenderness to deep palpation.

Laboratory evaluation
Hemoglobin 14 g/dL
Serum amylase Normal

Which of the following would be the most appropriate next step?

A. CT scan of the abdomen


B. Esophagogastroduodenoscopy with small intestinal biopsy
C. Hydrogen breath test
D. Upper GI contrast study with small-bowel follow through
E. Colonoscopy with biopsy

Suggested correct answer: A. Radiation enteritis is common after radiation treatment to the intestines
in pediatric patients. Radiation treatment creates free radicals from water, which in turn damage
neoplastic and normal tissue. Acute changes include edema and inflammation, producing abdominal
pain, diarrhea, and tenesmus. Chronic changes are due to obliterative arteritis that causes ischemia,
leasing to fibrosis, stricture, and/or ulceration. Patients may have dysmotility, bacterial overgrowth,
obstruction, altered absorption, in bleeding (with colonic involvement).

The first step in diagnosing the patient’s GI problem is to obtain a CT image to look for signs of
stricture/obstruction vs. tumor recurrence (though recurrence of stage I and II Wilm’s tumor is
uncommon). An UGI/SBFT may then be used to better evaluate the caliber of the small bowel and
the presence of strictures. Colonoscopy would be performed if there were concerns for colonic
disease, such as proctitis.
Dr-Wahid Helmi

Question 1:
A 3-month-old boy has failure to thrive and chronic diarrhea. Stools are pale, foul-smelling, and
greasy. Physical examination reveals a wasted, nonicteric boy who has coarse breath sounds and
rhonchi. The abdomen is distended and tympanitic. The liver has normal texture and is palpable
at the right costal margin.

Of the following, the MOST likely diagnosis is:

A. biliary atresia
B. celiac disease
C. congenital lactase deficiency
D. cystic fibrosis
E. Schwachman-Diamond syndrome

Suggested answer: D. Cystic fibrosis has a number of GI presentations. Meconium ileus at birth
is pathognomonic for the disease. Most infants have some degree of pancreatic insufficiency,
clinically leading to diarrhea (steatorrhea, secondary to fat malabsorption) and failure to thrive.
Some infants also develop cholestasis from inspissated bile which progresses to chronic liver
damage. Finally, older children and adults can develop intermittent small bowel obstruction,
called distal intestinal obstructive syndrome (DIOS) or “meconium equivalent.”

Biliary atresia would present with jaundice and pale stools. Celiac disease only presents after
exposure to gluten, which occurs after the child starts taking foods. Congenital lactase
deficiency is a rare cause of sugar malabsorption, which is not associated with steatorrhea or
pulmonary problems. Schwachman-Diamond syndrome is a rare recessive condition
characterized by: i) pancreatic insufficiency, leading to steatorrhea; ii) hypocellular bone
marrow with some or all cell lines down, resulting in frequent infections; iii) skeletal
abnormalities; and iv) poor growth.

Question 2:
A 15-year-old girl who is being evaluated for poor school performance and acting-out behaviors
is noted to have a large, firm liver. Rings of brown pigment are seen in Descemet membrane on
ophthalmologic examination.

Determination of which of the following is MOST likely to confirm the diagnosis in this patient?

A. Hepatic copper concentration


B. Hepatic iron concentration
C. Hepatitis B antibody titers
D. Serum alpha1-antitrypsin level
E. Sweat chloride level

Suggested answer: A. Wilson’s disease should be considered in any teenager with new onset
mental status changes. The disease is caused by mutations in ATP7B, which is involved in
hepatocyte copper trafficking. Tests favoring the diagnosis of Wilson’s disease include low
serum ceruloplasmin (because the liver cannot incorporate copper into apo-ceruloplasmin to
Dr-Wahid Helmi

create ceruloplasmin), high urinary copper excretion, and presence of corneal Kayser-Fleischer
rings. Hepatic copper concentration is considered the gold standard test, with concentrations
>250 mcg/g of tissue diagnostic (normal is <50mcg/g). However, because reports exist of
patients with Wilson’s disease and normal hepatic copper concentration, all factors must be
considered when making the diagnosis.

Question 3:
A 17-year-old boy who has evidence of Crohn’s disease in the terminal ileum develops severe
inguinal pain.

The MOST likely visceral source for the referred pain in this patient is the:

A. appendix
B. diaphragm
C. gallbladder
D. small bowel
E. ureter

Suggested answer: E. Crohn disease patients, especially those with terminal ileum disease, are
susceptible to renal calcium oxalate stone formation and resulting pain. Oxalate normally binds
to calcium in the gut lumen to form an insoluble precipitate passed in stools. However, in the
setting of fat malabsorption, calcium binds to free fatty acids instead and allows oxalate to bind
sodium. Sodium oxalate is soluble and is absorbed into the circulation. The oxalate is then
delivered to the renal system, where it binds calcium and precipitates out as stones.

Question 4:
Which of the following represents the MOST likely mechanism for peptic injury associated with
chronic administration of nonsteroidal anti-inflammatory drugs (NSAIDS)?

A. Generalized ischemia of the gut


B. Genetic predisposition
C. Inhibition of prostaglandin
D. Promotion of growth of Helicobacter pylori
E. Topical caustic injury

Suggested answer: C. NSAIDs can cause gastrointestinal injury by two mechanisms. First, they
can cause topical injury. NSAIDs are carboxylic acid derivates and are still protonated in the
low pH stomach environment. As a result, they remain soluble, are absorbed, and within the
more neutral gastric epithelial cells, deprotonate and cause injury. Second NSAIDs block
cyclooxygenase (COX) activity, preventing conversion of arachidonic acid into prostaglandin
precursors. Prostaglandins protect gastric mucosa from injury through various mechanisms,
including stimulating mucin production and increasing bicarbonate excretion. Because both
oral and IV NSAIDs cause peptic injury, the prostaglandin effect is thought to be more important
than the topical effect.
Dr-Wahid Helmi

NSAIDs that reach the distal small intestine and colon can cause intestinal damage and
increased permeability through similar mechanisms. As a result, NSAIDs have been known to
aggravate IBD.

Question 5:
A 7-year-old boy who has dysentery develops self-limited generalized seizures.

Of the following, the enteric pathogen MOST likely to cause these clinical findings is:

A. Campylobacter jejuni
B. Giardia lamblia
C. rotavirus
D. Salmonella typhimurium
E. Shigella dysenteriae

Suggested answer: E. Shigella species are resistant to stomach acid, multiply in the small
intestine, and then invade colonic cells. They lead to a number of intestinal manifestations,
including obstruction, perforation, toxic megacolon, and proctitis. They also have a number of
systemic effects, including bacteremia, arthritis, and hemolytic-uremic syndrome. Shigella has
also been associated with generalized seizures usually in the setting of high fevers in children
<15 years old. A Shigella toxin is thought to cause the seizures, though the exact toxin has not
been indentified.

Campylobacter jejuni is associated with Guillain-Barre syndrome, likely caused by antibodies


made in response to C. jejuni antigens. Rotavirus infections have been associated with seizures,
but less commonly that Shigella infections. Nontyphoidal Salmonella can lead to bacteremia and
meningitis, usually in children <1 year old. Giardia manifestations are usually limited to the
intestinal tract.

Question 6:
A 2-year-old girl is brought to the emergency department immediately after she swallowed some
drain cleaner. Physical examination reveals a crying child who is drooling; there are no abnormal
findings in the mouth.

Among the following, the BEST next step in the management of this child is to:

A. admit her for observation


B. arrange for follow-up in 1 week
C. encourage her to drink milk
D. obtain a consultation for upper endoscopy
E. order a barium swallow

Suggested answer: D. All patients with caustic ingestions should be evaluated with endoscopy,
because esophageal/gastric injury can be present without oropharynx symptoms. Endoscopy is
performed between 24-48 hours, because earlier endoscopy may not detect the full extent of
damage and later endoscopy may cause perforation. Diluting the ingested substance with milk
Dr-Wahid Helmi

or water is not recommended, because the amount of liquid required may induce vomiting and
cause further caustic damage. Barium swallows are not useful in the acute setting, because they
do not adequately detect injury or risk for stricture. However, in severe ingestions, they can be
used 2 to 3 weeks post- ingestion to evaluate for strictures.

In severe esophageal injuries, a nasogastric tube is placed with endoscopy-assisted visual


guidance to prevent perforation. The tube serves two purposes: i) allows a conduit for nutrition,
and ii) prevents complete stricturing of the esophagus. Corticosteroids are not recommended,
and they have shown no benefit in reducing strictures.

Question 7:
The physical features of a newborn girl suggest trisomy 21. She spits up the first feeding and
develops bilious emesis with subsequent feedings. Findings include a scaphoid abdomen, no
evidence of an abdominal mass, and quiet bowel sounds; the double-bubble sign is noted on a
plain radiograph of the abdomen (see radiograph).

Among the following, the MOST likely cause for the findings in this patient is:

A. antral web
B. duodenal atresia
C. Hirschsprung disease
D. meconium ileus
E. tracheoesophageal fistula

Suggested answer: B. 5% of Down syndrome patients have congenital gastrointestinal


abnormalities, including: i) duodenal atresia with our without annular pancreas, ii) imperforate
anus, iii) esophageal atresia with tracheoesophageal fistula, iv) Hirschsprung disease, and v)
celiac disease. Antral webs have been reported in Down syndrome patients but are not
common. Meconium ileus is most often seen in cystic fibrosis. Duodenal atresia can be detected
by KUB, which shows the “double bubble” sign (first bubble is stomach, second bubble is
duodenum with atretic outlet distally).

Question 8:
An 18-month-old girl has had inconsolable screaming, abdominal distention, and nonbilious
vomiting for the past 9 hours. Findings include irritability alternating with lethargy, quiet bowel
Dr-Wahid Helmi

sounds, a palpable mass in the right upper quadrant, hemoccult-positive stool, and decreased
bowel gas in the right lower quadrant.

The MOST appropriate next step is to obtain a(n):

A. air reduction enema


B. computed tomogram of the abdomen
C. laparotomy
D. ultrasonogram of the abdomen
E. upper gastrointestinal series

Suggested answer: A. Intussusception is the most common abdominal emergency in childhood,


occurring most frequently in the ileo-cecal junction. When the diagnosis is uncertain, abdominal
ultrasound or plain films may be useful. However, when the diagnosis is clear by the clinical
symptoms (as in the case) and there are no clinical signs of perforation, non-operative reduction
using air or water/saline is warranted. Surgical reduction is used if perforation is present, or if
the intussusception is limited to the small intestine and is causing symptoms (most
intussusceptions limited to the small bowel are asymptomatic and self-resolve).

Question 9:

For the past 3 months a 5-year-old girl has had intermittent, painless, bright red blood per rectum
in association with bowel movements. Inspection of the anus reveals no fissures, but blood is
present on the examiner's finger following digital examination of the rectum.

Of the following, the MOST likely cause of this patient's rectal bleeding is:

A. intussusception
B. juvenile polyp
C. Meckel diverticulum
D. peptic ulcer disease
E. ulcerative colitis

Suggested answer: B. Juvenile polyps and Meckel diverticulum – as opposed to intussusception,


peptic ulcer disease, and ulcerative colitis – causes painless rectal bleeding. Juvenile polyps are
benign hemartomas (non-neoplastic) usually found in the rectosigmoid region in children ages
2-8. They bleed when injured by stool in transit, and often present as bright red blood
depending on how distal they are located. Meckel diverticulum is a remnant of the
omphalomesenteric duct that persists in 2% of the general population. They consist of ectopic
gastric mucosa, and bleeding results from acid-related ulceration of tissue. Only 4% of affected
patients ultimately develop clinical symptoms, most before the age of 20 years.

Question 10:
A 6-year-old girl has had abdominal pain and nonbilious vomiting for 8 hours. History reveals
cough and fever for the past 3 days. Findings on physical examination include temperature 39°C
(102.2°F); tachypnea; toxic appearance; diffuse, voluntary guarding; and quiet bowel sounds.
Dr-Wahid Helmi

Of the following, the examination MOST likely to establish the etiology of the abdominal pain
and fever in this patient is a(n):

A. abdominal radiograph
B. chest radiograph
C. complete blood count
D. rectal examination
E. upper gastrointestinal series

Suggested answer: B. This patient likely has pneumonia and associated abdominal pain, which
can best be diagnosed with a chest X-ray. A chest X-ray would also be able to detect air under
the diaphragm, if perforation is a concern. Abdominal pain in the setting of pneumonia is
referred, because the T9 dermatome distribution is shared by both the abdomen and lung. A
CBC could indicate infection, but would not identify which part of the body is affected. A rectal
examination would be useful if constipation causing abdominal pain is suspected. An UGI series
is best used to define anatomy; however, given that this child has no history of previous emesis,
malrotation is unlikely.

Question 11:
An 1800 g infant born at 34 weeks' gestation is being fed enterally with a 24 kcal/oz formula
developed for preterm infants. The formula contains supplemental medium-chain triglycerides.

Among the following, the BEST explanation for using medium-chain triglycerides is to

A. decrease watery stools


B. enhance absorption of calcium
C. enhance absorption of iron
D. enhance absorption of vitamin C
E. prevent malabsorption of fat

Suggested answer: E. Premature infants have impaired bile and pancreatic lipase secretion,
and, as a result, have fat malabsorption. Medium chain triglycerides can bypass these
deficiencies, because they are digested and absorbed without the need of bile acids, pancreatic
lipase, or co-lipase. In addition, MCTs enter the circulation directly from the enterocyte,
without passing through the lymphatic system.

Despite the theoretical benefits of MCTs in preterm infants, their actual benefits remain
controversial. In formulas with MCTs versus long chain triglycerides, there was no benefit in
weight gain, lipid absorption, or mineral absorption. Furthermore, MCTs may be incompletely
oxidized leading to increased urine dicarboxylic acid excretion and metabolic inefficiency.

Question 12:
A 10-year-old boy adopted from Romania has pruritus, mild icterus, and hematemesis. Physical
examination reveals an anxious boy who has normal vital signs, ascites, hepatosplenomegaly,
Dr-Wahid Helmi

and a prominent venous pattern over the abdomen. Stools are charcoal colored and guaiac
positive.

Of the following, the MOST likely cause for the hematemesis is:

A. esophageal varices
B. gastric polyp
C. peptic ulcer disease
D. posterior nasal bleeding
E. thrombocytopenia

Suggested answer: A. This patient likely has liver disease and resulting portal hypertension,
causing esophageal varices. Liver disease is suggested by pruritis (bile acid back-up into the
circulation) and icterus. Portal hypertension is suggested by splenomegaly, ascites, and a
prominent venous pattern on the abdomen. When esophageal varices bleed, they can result in
hematemesis. Blood can also transit through the GI tract and appear as black, charcoal colored
stool. Gastric lesions (such as a gastric polyp or peptic ulcer disease), posterior nasal bleeding,
or thrombocytopenia would not give extra-intestinal symptoms such as ascites or splenomegaly.

Question 13:
A 17-year-old adolescent who is receiving nonsteroidal anti-inflammatory agents for juvenile
rheumatoid arthritis has the acute onset of epigastric pain. He vomits "coffee ground" material
containing flecks of blood that clears quickly with gastric lavage.

Among the following, the MOST likely cause for these clinical findings is

A. duodenal ulcer
B. gastritis
C. Helicobacter pylori infection
D. Mallory-Weiss tear
E. posterior nasal bleeding

Suggested answer: B. NSAIDs cause gastritis, by impairing prostaglandin production by gastric


mucosa (see question 4 above). Prostaglandins exert their protective role by inducing mucin
and bicarbonate secretion. NSAIDs and aspirin can also cause duodenal ulcers, though H.
pylori infection is a more common cause. The mechanism by which NSAIDs cause duodenal
ulcers is acid dependent and prevented by acid blockade. Mallory-Weiss tears are esophageal
tears associated with vomiting, whereas posterior nasal bleeding should not cause epigastric
pain.

Question 14:
A 9-year-old girl has the height age of a 7-year-old and the bone age of a 6-year-old.

Among the following, the MOST likely cause of her short stature is

A. achondroplasia
Dr-Wahid Helmi

B. hypothyroidism
C. malnutrition
D. normal variant short stature
E. Silver-Russell syndrome

Suggested answer: B. This patient has short stature in the context of delayed bone age. Both
hypothyroidism and growth hormone deficiency are associated with short stature and
corresponding delay in bone maturation. As a result, when re-supplemented with either thyroid
hormone or growth hormone, patients demonstrate good growth potential.

Achondroplasia, caused by a dominant mutation over-activating the Fgf3 receptor, leads to


shortened bones that would produce a shorter stature than in this patient. At the growth plates,
achondroplasia patients do demonstrate delayed bone age. Severe malnutrition will stunt
growth but not affect bone age. Normal variant short stature by definition is short stature in the
presence of normal bone age. Finally, Russell-Silver syndrome patients typically have a
constellation of symptoms, including IUGR, prominent forehead, triangular face, and body
asymmetry. They have delayed bone age early with fast advancement later.

Question 15:
A 15-month-old boy who is a recent immigrant from the Caribbean area has rectal prolapse.
Stools sometimes contain mucus and blood. Physical examination reveals a child who appears
well nourished, but short for age. The mucosal prolapse is easily reducible; other findings on
rectal examination are normal.

Of the following, the MOST likely cause for this boy's rectal prolapse is:

A. celiac disease
B. chronic constipation
C. cystic fibrosis
D. rectal polyp
E. trichuriasis

Suggested answer: E. Rectal prolapse results from intussusception of the upper rectum and
sigmoid colon. There are a number of predisposing factors, including: i) increased intra-
abdominal pressure, from constipation, coughing, or vomiting; ii) diarrheal illnesses, from
parasites such as Ascaris lumbricoides and Trichuris trichiura or from malabsorptive syndromes
such as celiac disease; iii) malnutrition (hypoproteinemia leading to mucosal edema and smaller
fat pads reducing perirectal support); iv) inflammatory lesions such as polyps producing lead
points; and v) cystic fibrosis (may occur in 20% of CF patients under 3, from a combination of
coughing, malabsorptive diarrhea, and malnutrition). The patient’s clinical history is most
consistent with parasitic infection.

Question 16:
An antral or pyloric web (diaphragm) is considered in the differential diagnosis of a 6-month-old
girl with failure to thrive syndrome and nonbilious vomiting.
Dr-Wahid Helmi

The MOST specific study for diagnosing this condition is

A. gastric emptying study


B. plain abdominal radiography
C. ultrasonography
D. upper gastrointestinal contrast study
E. upper gastrointestinal endoscopy

Suggested answer: E. An antral/pyloric web is a circumferential mucosal septum in the pyloric


region, projecting into the lumen perpendicular to the long axis of the antrum. Antral webs may
have a large aperature and be clinically insignificant; alternatively, they may be nearly complete
and cause gastric outlet obstruction symptoms. Histologically, webs are made of gastric
mucosa, submucosa, and muscularis mucosae. Plain abdominal radiography may show a
dilated stomach, whereas an UGI study could show an extra compartment between the web and
antrum reminiscent of a second duodenal bulb. The most specific study would be to directly
visualize and sample the tissue through upper endoscopy.

Question 17:
A 10-year-old child has had intermittent diarrhea and weight loss over the past year.

A TRUE statement regarding testing with guaiac or orthotolidine for occult blood in this patient's
stool is:

A. Microscopic examination of the stool is a better test for detecting occult blood
B. Negative results exclude lower gastrointestinal bleeding
C. Positive results confirm the presence of occult blood
D. These tests detect peroxidase activity in hemoglobin
E. These tests quantitate the amount of hemoglobin in the stool

Suggested answer: D. The guaiac-based fecal occult blood test (gFOBT) is a qualitative test that
detects a peroxidase reaction (which turns guaiac-impregnated paper blue). Because heme from
hemoglobin has peroxidase activity, gFOBT has been used to detect the presence of a GI bleed.
However, gFOBTs can produce false positives and negatives. False positives occur because
certain foods have peroxidase activity. These include animal meats with high heme content
(beef, lamb) and raw peroxidase-rich fruits and vegetables (broccoli, cauliflower, radishes,
turnips, and some melons). False negatives occur depending on sampling. As a result, in
colorectal cancer screening, three to six gFOBTs are performed in sequence to establish a
gFOBT negative result. Microscopic examination may miss blood depending on what part of the
stool is studied.

Question 18:
An 18-year-old girl who is taking tolmetin for juvenile rheumatoid arthritis develops gastritis.

Which of the following medications would have been MOST likely to prevent the development
of peptic disease in this patient?
Dr-Wahid Helmi

A. Antacids
B. Corticosteroids
C. H2-blockers
D. Misoprostol
E. Sucralfate

Suggested answer: D. Tolmetin is an NSAID, which causes gastritis by inhibiting prostaglandin


production. Prostaglandins in turn have a number of effects, including increasing bicarbonate
secretion and mucin production. As a result, the prostaglandin E analog Misoprostol is the most
appropriate prevention strategy. Though less effective than Misoprostol, PPIs have also been
shown to be protective in preventing gastric ulcers or NSAID-related duodenal damage (NSAID-
related duodenal damage, compared to gastric damage, is more closely related to NSAIDs
ability to increase acid secretion). Antacids, H2-blockers, and sucralfate may all have some
benefit because each replicates some aspect of the protective effects of prostaglandins.
Corticosteroids promote gastritis.

Question 19:
A previously healthy 13-month-old boy has had two bloody stools in the past 6 hours. He has
had no nausea, vomiting, or diarrhea. The child appears well and has good peripheral perfusion.
The pulse is 130/min. Abdominal examination reveals normal findings except for hyperactive
bowel sounds. Maroon-colored, guaiac-positive stool is found on rectal examination.
Radiography of the abdomen shows a nonspecific bowel gas pattern.

Of the following, the MOST likely diagnosis is

A. anal fissure
B. Crohn disease
C. intussusception
D. Meckel diverticulum
E. peptic ulcer disease

Suggested answer: D. Four clinical aspects suggest a bleeding Meckel diverticulum: i) the
bleeding is painless (vs. painful in intussusception and IBD); ii) the bleeding is copious (vs. not
as much in anal fissures); iii) the patient is 13 months old and healthy (vs. usually older and
malnourished for Crohn disease); and iv) the blood is maroon, suggesting a bleed in the
proximal bowels. A Meckel scan may confirm the diagnosis, though false negatives are common.
The scan uses intravenous 99m technetium pertechnetate which is taken up by gastric mucosa.
Positive signal in the terminal ileum area indicates ectopic gastric mucosa and is consistent with
a Meckel diverticulum.

Question 20:
A 2-year-old boy has just swallowed a quarter. Physical examination reveals a quiet child in no
distress

The MOST appropriate first step in managing this patient is to


Dr-Wahid Helmi

A. administer glucagon intramuscularly


B. administer papain
C. admit for observation
D. obtain radiographs
E. perform an upper endoscopy

Suggested answer: D. The most appropriate step is to obtain radiographs for two reasons.
First, radiographs will confirm that the quarter in not in the trachea (if in the trachea, the side
instead of the face of the coin would be seen on PA films). Second, radiographs will identify the
quarter’s location in the GI tract. Coins in the esophagus that cause symptoms should be
removed immediately, whereas asymptomatic coins can be watched for 24 hours to allow
passage in the stomach. Coins in the stomach can be left to pass in the stools; however, if the
coin remains in the stomach after 4 weeks, it is unlikely to pass and should be removed
endoscopically.

Coins lodged in the esophagus for more than 24 hours should be removed via upper endoscopy,
because they may cause aspiration or other complications. There are 3 esophageal sites where
coins/foreign objects become lodged: the cricopharyngeus, mid-esophagus, and above the LES.
Glucagon, which lowers the LES, has been tried with little success in helping coins move to the
stomach. Papain has also been tried in cases of food impaction, to digest the food. Papain is
now contraindicated because it poses a risk for esophageal injury and possible perforation.

Question 21:
Of the following, the MOST common feature of chronic, nonspecific diarrhea of infancy and
childhood (toddler's diarrhea) is:

A. guaiac-positive stools
B. intermittent fever
C. lactose intolerance
D. significant weight loss
E. unimpaired growth

Suggested answer: E. Nonspecific diarrhea of infancy (toddler’s diarrhea) is defined as the


passage of three or more large, unformed stools during waking hours (not at night) for four or
more weeks. It begins in infancy or preschool years, does not cause failure to thrive, and does
not have a specific definable cause. Toddler’s diarrhea may be caused by rapid intestinal transit
secondary to uninterrupted MMCs, because food particles are often seen in the stool. Other
proposed mechanisms include dumping bile acids and hydoxyl fatty acids into the colon, as well
as excessive sugar intake promoting an osmotic diarrhea. Symptoms usually improve with
dietary modifications including increased fat and fiber intake, limited fluid intake, and avoidance
of fruit juices (especially apple, prune, and pear juice which have a high osmotic load).
Symptoms usually resolve by age 4.

Guaiac-positive stools, intermittent fever, and significant weight loss are associated with
inflammatory or infectious causes of diarrhea. Primary lactose intolerance is rare in children
and usually is accompanied by additional signs and symptoms including poor growth.
Dr-Wahid Helmi

Secondary lactose malabsorption can occur in infants following temporary villi blunting from an
infectious gastroenteritis.

Question 22:
Among the following, the LEAST important variable in the development of diarrhea is

A. gender
B. immunologic status
C. nutritional status
D. presence of chronic infection
E. presence of systemic disease

Suggested answer: A. Generally, diarrhea occurs when enterocytes are actively secreting
solutes that draw water with them (secretory diarrhea) or when they are unable to absorb
osmotically active solutes (osmotic diarrhea). There is a cyclic relationship between
immunological and nutritional status in causing diarrhea, whereby poor immunological status
leads to infection/diarrhea, which causes malnutrition, which in turn increases vulnerability to
further infection/diarrhea. Systemic diseases can also cause malnutrition, which increases
vulnerability to infection, which may in turn cause diarrhea. There is no evidence that gender is
a direct variable correlating with the development of diarrhea.

Question 23:
A 5-year-old girl has had a recent onset of postprandial emesis and epigastric pain that awakens
her at night. Physical examination reveals normal findings except for guaiac-positive stools.
There is a strong family history of peptic ulcer disease.

Among the following, the MOST specific diagnostic study for peptic ulcer disease is

A. abdominal ultrasonography
B. plain abdominal radiography
C. technetium pertechnetate scanning
D. upper gastrointestinal contrast study
E. upper gastrointestinal endoscopy

Suggested answer: E. Peptic ulcer disease is most accurately diagnosed by visualizing the
mucosa, via upper endoscopy with biopsies. UGI contrast studies have been used as a
noninvasive test to detect mucosal abnormalities; however, they have high false positive and
false negative rates. Technetium pertechnetate scanning (Meckel scan) is useful for detecting
gastric tissue especially in ectopic places, not for detecting gastritis or ulcers. Abdominal
ultrasonography and plain abdominal radiography are not used to diagnose peptic ulcer
disease.

Question 24:
A 3-year-old boy with acute lymphoblastic leukemia in hematologic remission is receiving
vincristine, methotrexate, and 6-mercaptopurine. He develops abdominal pain and distention and
nausea without fever or diarrhea.
Dr-Wahid Helmi

These findings are MOST likely caused by

A. Escherichia coli gastroenteritis due to granulocytopenia


B. intestinal candidiasis due to lymphopenia
C. intestinal mucosal ulcerations due to methotrexate toxicity
D. necrotizing enterocolitis due to 6-mercaptopurine toxicity
E. reduced intestinal motility due to vincristine toxicity

Suggested answer: E. In humans and animal models, vincristine reduces gastric motility
presumably from its neuropathic side effects. The most common motility problem with
vincristine is constipation, usually starting 3 days after the dose is given. However, other signs
of dysmotility, including abdominal distension and vomiting, have been known to occur. The
patient’s lack of diarrhea is inconsistent with enteritis. Methotrexate commonly causes
hepatotoxicity, vomiting, and mucositis, without abdominal distension. 6-MP commonly causes
vomiting and may cause elevation in liver numbers. It is also associated with a severe
hypersensitive reaction, with gastrointestinal symptoms of nausea, vomiting, diarrhea, and fever.

Question 25:
Of the following, the MOST likely cause of passage of bright red blood from the rectum in an
otherwise healthy, asymptomatic 3-year-old child is

A. intussusception
B. juvenile polyp
C. Meckel diverticulum
D. peptic ulcer
E. ulcerative colitis

Suggested answer: B. The two most common causes of rectal bleeding in the 2-5 year age group
are juvenile polyps (asymptomatic) and infectious colitis (symptomatic). A Meckel diverticulum
is also possible, but classically the majority of cases (60%) occur before age 2. Intussusception
causes pain and is accompanied by red “currant jelly” stool. Peptic ulcers usually present later
and produce black, melanotic stools. Ulcerative colitis would likely be accompanied by other
symptoms, including diarrhea.
Dr.Wahid Helmi - Dymiate -Egypt

Dr-Wahid Helmi
Pediatrics consultant
Dmiate-Egypt
#revision_6_All_Ped.
Dr.Wahid Helmi - Dymiate -Egypt

1. Which of the following is the most likely pathogen


responsible for bronchiectasis?
(A) Corynebacterium diphtheriae
(B) Streptococcus pneumoniae
(C) parainfluenza virus
(D) rhinovirus
1. (B) Bronchiectasis has numerous etiologies. Most
commonly, cultures reveal normal oral flora from the lower
respiratory tract: Streptococcus pneumoniae,
Staphylococcus aureus,Haemophilus influenzae,
Pseudomonas aeruginosa. Parainfluenza viruses typically
are responsible for croup. Corynebacterium diphtheriae is
the causative organism for diphtheria.
Rhinovirus is the most common pathogen isolated with acute
viral rhinitis or the common cold

2. A 3-month-old infant had a mild microcytic, hypochromic


anemia at birth and the screen was negative for sickle cell
disease/trait. She was started on iron therapy and presents
today for follow up. The hemoglobin (Hgb) electrophoresis
laboratory results are:
Hemoglobin 8.8 mg/dL (normal: 10.5–14.0)
Hematocrit 25% (normal: 33–42)
Mean corpuscular volume (MCV) 60 fL (normal: 70–90)
Mean corpuscularhemoglobin concentration (MCHC) 32
g/dL (normal: 33–37)
Hgb A2 27% (normal: 1.5%–4%); Hgb A1 30% (normal:
76%–99%); HgF (fetal hemoglobin)
50% (normal: 0%–20%); Bart Hgb 0% (normal: 0%)
Which of the following is the MOST likely diagnosis?
(A) heterozygous alpha thalassemia
(B) homozygous alpha thalassemia
(C) beta thalassemia major
(D) beta thalassemia minor
Dr.Wahid Helmi - Dymiate -Egypt

2. (D) The typical hemoglobin electrophoresis for beta


thalassemia minor has an elevated level of hemoglobin A2.
In a normal infant there is mainly HgF and HgA1 with
minimal amounts of A2.Bart hemoglobin is diagnostic for
the alpha thalassemias after the neonatal period is over. Beta
thalassemia major will only have fetal hemoglobin on
electrophoresis. Because of the high
incidence of false-negatives in hemoglobin screenings in the
neonatal period, it is important for the provider to do a full
work-up of microcytic, hypochromic anemias to ensure
proper diagnosis.

3. Which of the following is the most common etiologic agent


for acute tonsillitis in the United States?
(A) adenovirus
(B) group A beta-hemolytic Streptococcus Pyogenes
(C) Epstein–Barr virus
(D) Mycoplasma pneumoniae
3. (B) In children who present with symptoms of sore throat
and fever, approximately 50% to 70% of these cases are due
to a viral infection. Adenovirus is one of the most common
etiologic viral
agents. Epstein–Barr virus is the etiologic agent for
mononucleosis and while very common in the United States
it is still less than rhinoviruses and coronaviruses. The two
remaining choices are bacterial pathogens of which group A
beta-hemolytic streptococcus (GAS) is the most
common followed by the less common pathogens (group C
Streptococcus, Arcanobacterium haemolyticus, and
Streptococcus pneumoniae). As a single agent, GAS is the
most common etiology of acute tonsillitis and pharyngitis
Dr.Wahid Helmi - Dymiate -Egypt

4. In a pediatric patient with suspected congestive heart


failure, which of the following signs and symptoms would be
least likely seen on physical examination?
(A) bradycardia
(B) cardiomegaly
(C) hepatosplenomegaly
(D) tachypnea
4. (A) In left-sided congestive heart failure, the signs of
tachycardia, tachypnea, intercostals retractions, rales, and
rhonchi are found. Hepatosplenomegaly is a sign of right-
sided congestive heart failure. Bradycardia is not associated
with either left- or right-sided congestive heart
failure in the pediatric patient.

5. A 14-year-old girl presents to the office for a third visit


over the past month complaining of fatigue and pain in her
pelvic bones. Her previous evaluation included the following
laboratory tests:
Heterophile antibody test: negative
Hematocrit: 34% Since her last visit to the office, she has
lost 4 lb. Her mother reports she has a poor appetite. On
physical examination, she is noted to be pale, has several
large ecchymotic areas on her legs, and has inguinal
lymphadenopathy. Her complete blood cell count (CBC)
results are given below:
Dr.Wahid Helmi - Dymiate -Egypt

Which of the following is the MOST likely diagnosis?


(A) Hodgkin's lymphoma
(B) acute lymphoblastic leukemia
(C) acute myelogenous leukemia
(D) infectious mononucleosis/Epstein–Barr virus
5.B) Leukemia is the most common form of childhood cancer.
Acute lymphoblastic leukemia is the most common form of
leukemia in childhood, accounting for approximately 4 out of
100,000 children younger than the age of 15. The clinical
presentation is variable, ranging from severe
with a life-threatening infection to asymptomatic at a routine
well-child visit. Often, there is a 3-to 4-week history of an
illness prior to the diagnosis, with signs and symptoms including
malaise, anorexia, intermittent fever, bone tenderness, pallor,
petechiae, purpura, and abdominal pain.
Findings noted on the physical examination include pallor,
petechiae, purpura, retinal hemorrhages, lymphadenopathy
(either localized or generalized to cervical, axillary, or inguinal
areas), bone and joint tenderness (especially in the pelvis, lower
vertebral bodies, and femur،(
hepatosplenomegaly, and nephromegaly. Initially, the most
useful test is a complete blood count with differential, revealing
multiple cytopenias and leukemic blasts. The bone marrow
examination is diagnostic, revealing a homogeneous infiltration
of leukemic blasts replacing normal marrow. Acute
myelogenous leukemia typically presents with
Dr.Wahid Helmi - Dymiate -Egypt

hyperleukocytosis (WBC<
(100,000or with myeloblasts on peripheral smears and bone
marrow biopsies. It accounts for 25% of leukemias in childhood.
Patients with chronic Epstein–Barr virus infections present with
sore throat, fever, posterior cervical lymphadenopathy, and
malaise associated with atypical lymphocytosis and a positive
heterophile antibody test. Hodgkin's lymphoma typically
presents
with painless cervical adenopathy and a normal CBC. However,
typically the C-reactive protein and erythrocyte sedimentation
rates are elevated.

6. Which of the following is the most common etiologic agent


of bacterial meningitis in the pediatric population of the
United States?
(A) Streptococcus pneumoniae
(B) Haemophilus influenzae type B
(C) Listeria monocytogenes
(D) Neisseria meningitides

6. (A) Despite the increase in vaccination of infants in the


United States, Streptococcus pneumoniae remains the most
common etiologic agent for bacterial meningitis in the
pediatric population. Haemophilus influenzae type B is the
second most common, but has gone down
significantly due to the widespread vaccination of children.
Neisseria meningitides has approximately 2,400 to 3,000
cases a year. Meningitis due to Listeria monocytogenes is
typically seen in the neonatal period due to transmission
from the mother. It is present in normal
fecal matter in around 10% of the population. Its rates have
Dr.Wahid Helmi - Dymiate -Egypt

gone down due to strict guidelines for the food industry,


resulting in less than 1,000 cases per year.

7. Three weeks ago, an 8-year-old child was diagnosed with


Streptococcal pharyngitis based upon a positive throat
culture for group A beta-hemolytic streptococcus. Today,
she returns to the clinic with evidence of carditis. The
differential diagnosis includes rheumatic fever. What
additional finding would allow you to make the diagnosis of
rheumatic fever based upon the
modified Jones criteria?
(A) leukocytosis
(B) polyarthritis
(C) elevated erythrocyte sedimentation rate (ESR)
(D) erythema multiforme
7. (B) The diagnosis of rheumatic fever is based on clinical
grounds using the modified Jones criteria. Two major
manifestations or one major and two minor manifestations
in addition to supporting evidence of a preceding
streptococcal infection are needed to make the diagnosis of
rheumatic fever. The major manifestations are polyarthritis,
carditis, erythema marginatum,
subcutaneous nodules, and Sydenham chorea. The minor
manifestations are fever, arthralgia,previous rheumatic
fever or rheumatic heart disease, an elevated sedimentation
rate or Creactive protein, and a prolonged P–R interval. The
supporting evidence of a preceding streptococcal infection
includes elevated titers of antistreptolysin O or other
streptococcal
antibodies and positive throat culture for group A beta-
hemolytic streptococcus
Dr.Wahid Helmi - Dymiate -Egypt

8. In a patient who has newly diagnosed Hemophilia B,


which of the following laboratory results would be expected
on a coagulation panel?
(A) Increased aPTT (activated partial thromboplastin time),
normal PT (prothrombin time),
factor VIII deficiency
(B) + von Willebrand factor, decreased aPTT, and increased PT
(C) Increased PT, increased bleeding time, decreased platelets,
decreased fibrinogen
(D) Increased aPTT, normal PT, normal thrombin time
8. (D) Hemophilia B, also known as Christmas disease and
factor IX deficiency. Factor IX is activated on the intrinsic
side of the coagulation cascade right before the common
pathway and the result is an increased aPTT, with a normal
prothrombin time, thrombin time, and INR. It does
not affect platelets nor bleeding time.

9. A routine physical examination of a 12-year-old girl


demonstrates dark, coarse, curly pubic hair spread sparsely
over the pubic symphysis, as well as elevation of the breast
and areola without separation of their contours. According
to Tanner stages of sexual maturation, at what stage
would you assess her sexual maturity?
(A) Tanner Stage II
(B) Tanner Stage III
(C) Tanner Stage IV
(D) Tanner Stage V
9. (B) Tanner stages of sexual maturation categorize the
progression of pubertal development in girls according to
pubic hair and breast development. Menarche usually
occurs 18 to 24 months
following the onset of breast development. In female breast
development, Tanner Stage I is an absence of breast
Dr.Wahid Helmi - Dymiate -Egypt

development; Stage II is a small, raised breast bud; Stage III


shows further enlargement/elevation of breast and alveolar
tissue; Stage IV is the areola and papilla forming a
secondary mound on breast contour; and Stage V is the
mature breast with alveolar area as part of the breast
contour. For the stages of pubic hair development, Stage I is
prepubertal, an absence of hair; Stage II shows sparse, fine
hair, primarily on the border of labia; Stage III is pigmented
and curly and increases in quantity on the mons pubis; Stage
IV is increased quantity of coarser texture with labia and
mons pubis well covered; and Stage V is mature adult
distribution with spreading to medial thighs.

10. During the first year of life, what would be the expected
average growth for an infant who weighs 8 lb at birth?
(A) 7 lb at 2 weeks, 14 lb at 6 months, 21 lb at 12 months
(B) 7 lb at 2 weeks, 21 lb at 4 months, 28 lb at 12 months
(C) 8 lb at 2 weeks, 16 lb at 4 months, 24 lb at 12 months
(D) 8 lb at 2 weeks, 24 lb at 6 months, 32 lb at 12 months
10. (C) During the first year of life, the average, expected
increase in weight of a full-term infant is to regain the birth
weight by 2 weeks of age, double the birth weight by 4
months of age, and triple the birth weight by 1 year of age.

11. A 3-year-old child presents to the emergency department


(ED) with bruises on his body. His mother claims that her
son sustained these bruises when he tumbled down the stairs
3 days ago.Which of the following colors would you expect
the bruises to be if this occurred as stated?
(A) brown
(B) purple
Dr.Wahid Helmi - Dymiate -Egypt

(C) red
(D) yellow
11. (B) When evaluating children with physical injuries, the
major difficulty is distinguishing intentional injuries from
unintentional injuries. Inconsistencies between the stated
story and the injury are suspect. Discoloration caused by
healing bruises tends to follow a distinctive pattern.
On the first day, there is swelling without discoloration.
From day 1 through day 5, the bruise is purple in color. For
days 5 through 7, the bruise is green. Then, from day 7
through day 10, the bruise is yellow, followed by a brownish
color from day 10 to day 14.

12. An 8-month-old infant, whose parents elected not to


immunize, presents with a 5-day history of arunny nose in
late January. Then, over the past 3 days she has developed a
temperature of 101.2°F and vomiting (three times in 24
hours). This morning she developed watery, non-
bloody,nonmucous diarrhea. Which of the following is the
MOST likely causative organism for her illness?
(A) Clostridium difficile
(B) Giardia lamblia
(C) Shigella species
(D) rotavirus
12. (D) Rotavirus is one of the most important causes of
acute gastroenteritis in infants and young
children primarily 6 to 24 months of age. In the United
States, there are 65,000 to 70,000
hospitalizations and 200 deaths per annum. Peak incidences
occur in the fall and winter. Most
initial infections are characterized by diarrhea (watery,
nonbloody, nonmucous), fever, and vomiting. Nasal
congestion and coryza often precede the gastrointestinal
symptoms. Clostridium difficile produces a toxin that causes
Dr.Wahid Helmi - Dymiate -Egypt

a self-limited diarrhea in which symptoms characteristically


begin following the administration of antibiotics that reduce
normal bowel
flora. Giardia lamblia, a flagellated protozoa,
characteristically causes a mild diarrhea, with or without a
low-grade fever, anorexia, flatulence, and abdominal
cramps. It is not associated with vomiting nor upper
respiratory tract symptoms. Shigella gastroenteritis in young
children classically presents acutely with a high fever or
seizures along with vomiting followed by
bloody, mucoid, diarrheal stools.

13. In a 4-year-old female child who presents with “toeing


in,” which of the following is the likely etiology?
(A) femoral anteversion
(B) genu valgum
(C) genu varum
(D) tibial torsion
13. (A) “Toeing in” in children before the age of 2 is
typically due to tibial torsion; however, any “toeing in”
after the age of 2 to 3, is usually due to femoral anteversion.
The femur has more
internal rotation that results in the presentation Genu
varum is known as bowleg and genu
valgum is known as knock-kneed.

14. Which of the following is the first sign of puberty in a


normal male?
(A) appearance of axillary hair
(B) appearance of pubic hair
(C) deepening of the voice
(D) enlargement of the testes
14. (D) The first sign of pubertal development in boys is the
enlargement of testicular size and occurs at the mean age of
Dr.Wahid Helmi - Dymiate -Egypt

11.6 years. Genital stages accelerate before pubic hair


development, which occurs, on average, at 13.4 years of age.
The deepening of the voice and the development of chest and
axillary hair usually occurs in midpuberty or 2 years after
the growth of pubic hair.

15. Huntington disease has which of the following types of


genetic patterns of inheritance?
(A) autosomal dominant
(B) autosomal recessive
(C) X-linked dominant
(D) X-linked recessive
15. (A) Huntington disease is an autosomal dominant
hereditary disease. Its occurrence is between
1:5,000 and 1:20,000. It is caused by a defect on chromosome
4p16.3 that results in a repeat of
“CAG” in the “Huntington” protein gene.

16. A 9-year-old child, who was diagnosed with a viral upper


respiratory infection 2 weeks ago,returns to the clinic with a
complaint of a 2-day history of drooping of one side of her
mouth. She is afebrile with a blood pressure of 110/60 mm
Hg. Her physical examination reveals an inability to
completely close her left eye, inability to wrinkle her
forehead, and the drooping of her mouth on the left side.
Her smile is asymmetric. The remainder of her examination
is otherwise normal. Which of the following is the MOST
likely diagnosis?
(A) Bell's palsy
(B) botulism
(C) brainstem glioma
(D) Guillain–Barré syndrome
16. (A) Bell's palsy is the acquired peripheral facial
Dr.Wahid Helmi - Dymiate -Egypt

weakness (cranial nerve VII) of sudden onset and unknown


etiology. It often follows a viral illness with notable
improvement within 2 weeks and near complete recovery
within 2 months. Prednisone therapy may promote recovery
of facial strength. Guillain–Barré syndrome (acute
idiopathic polyneuritis) generally presents with symmetrical
weakness of the lower extremities, which may ascend rapidly
to the arms, trunk, and face. Nonspecific respiratory or
gastrointestinal symptoms may occur 5 to 14 days preceding
the
infection. Physical examination will yield symmetric flaccid
weakness, which is usually proximal in distribution. Rarely,
there is cranial nerve (III–VI, IX–XI) involvement. Botulism
is most often caused by the ingestion of food containing the
Clostridium botulinum toxin or rarely
from an infected wound. Children will present with blurred
or double vision, ptosis, or choking.Physical findings include
a weak swallow paralysis of accommodation and eye
movements. In this case, there was not a history of food
ingestion or wound infection to support this diagnosis.
Children with a brain stem tumor may present with facial
and extraocular muscle palsies,hemiparesis, gait
disturbances, and hydrocephalus (25%). Changes in
personality such as lethargy, irritability, and aggressive
behavior are particularly common findings. Speech and
swallowing difficulties are not unusual. Later in the illness,
patients will develop vomiting and
headaches.
Dr.Wahid Helmi - Dymiate -Egypt

17. A 5-year-old child presents to the office for a school


physical examination. His medical history is unremarkable,
including normal growth and development. His physical
examination is normal except for a grade II/VI high-pitched,
vibratory, systolic ejection murmur heard best at the left
lower sternal border with radiation to the apex. When the
child is in a supine position, the murmur is louder. Which of
the following murmurs is the MOST likely diagnosis?
(A) physiologic peripheral pulmonic stenosis murmur
(B) pulmonary ejection murmur
(C) Still's murmur
(D) venous hum
17. (C) Still's murmur is the most common innocent
murmur of early childhood and is usually appreciated in
children from 3 to 6 years of age. It is a grade I–III/VI early
systolic ejection murmur of musical or vibratory quality
heard best between the apex and the left lower sternal
border. It is loudest when the patient is in a supine position.
The murmur may diminish or disappear with inspiration,
during the Valsalva maneuver, or when the patient is
standing or seated. A physiologic peripheral pulmonic
stenosis murmur is a soft, short, high-pitched, grade I–II/VI
systolic ejection murmur. Typically, it is auscultated with
equal intensity at the left upper
sternal border, along the back, and in both axillae. It is
usually found in newborns and generally disappears by 3 to
6 months of age. A pulmonary ejection murmur is the most
common innocent murmur of later childhood and is usually
seen in children 8 to 14 years of age. It is a soft, early to
midsystolic ejection, grade I–III/VI murmur heard best
along the left upper sternal border. It is louder when the
patient is supine or with increased cardiac output. It
diminishes with standing or during the Valsalva maneuver.
A venous hum is a continuous musical, grade I–II/VI
Dr.Wahid Helmi - Dymiate -Egypt

murmur heard
at the right or left superior infraclavicular area. The
murmur is obliterated when the patient is in a supine
position, with head rotation, and with compression of the
jugular vein. It is usually auscultated in children from 3 to 6
years of age.

18. Which of the following sleeping positions for a healthy


infant should be recommended to parents during
anticipatory guidance in order to reduce the risk for sudden
infant death syndrome?
(A) prone position
(B) seated position
(C) side position
(D) supine position
18. (D) Sudden infant death syndrome (SIDS) is defined as
the sudden, unexplained death of an apparently healthy
infant that is unexpected and not adequately explained by a
comprehensive medical history, a postmortem physical, and
investigation of the death scene. SIDS is a leading cause of
death in infants between the ages of 1 month and 1 year,
second only to congenital
anomalies. The exact etiology of SIDS is unclear. Prevention
of SIDS has become a focus of public health measures. In
1994, The American Academy of Pediatrics initiated a
campaign called “Back to Sleep,” which recommended
placing infants in the supine position for sleep.Following the
institution of this campaign in the United States, the annual
death rate decreased from 1 3 per 1 000 to 0.7 per 1,000.

19. A previously healthy 12-month-old infant has been


coughing and experiencing fever on and off for 2 months. He
was diagnosed and treated for pneumonia approximately 3
Dr.Wahid Helmi - Dymiate -Egypt

months ago. On physical examination, he is noted to be in no


acute respiratory distress; however, he is tachypneic
with bibasilar rales and scattered rhonchi. Which of the
following is the MOST likely diagnosis?
(A) bronchiectasis
(B) chronic bronchitis
(C) croup
(D) bronchopulmonary dysplasia
19. (A) Bronchiectasis, meaning “dilation of the bronchi,”
results from destruction of the airway and poor drainage,
often associated with cystic fibrosis, foreign body aspiration,
or an infection. It is uncommon in the general population.
The presentation may vary from a chronic productive cough
to recurrent pneumonia with or without hemoptysis.
Persistent rhonchi, rales, and decreased breath sounds are
noted over the affected atelectatic area. Croup is an
inflammatory disease of the larynx most frequently affecting
young children during the fall and early winter months.
Typically, there is an upper respiratory tract prodrome
followed by stridor and a “barky cough”in the absence of
drooling. Subglottic narrowing with a normal epiglottis is
diagnostic on alateral neck X-ray. The most common
pathogen is parainfluenza virus. Bronchopulmonary
dysplasia is most commonly seen in infants in the neonatal
intensive care unit. It is a chronic
condition seen in patients whose clinical course included
hyaline membrane disease. These infants typically need
oxygen for a few months as they grow and some need
permanent tracheostomy and ventilation for up to 2 years.
Chronic bronchitis falls into the chronic obstructive
pulmonary disease category typically seen in older adults
and does not typically present with acute symptoms.
Dr.Wahid Helmi - Dymiate -Egypt

20. The second most common etiologic agent of otitis externa


is which of the following?
(A) Staphylococcus aureus
(B) Corynebacterium
(C) Anaerobes
(D) Streptococcus pyogenes
20. (A) Otitis externa is an infection of the auditory canal.
The most common etiologic agent is pseudomonas. However,
Staphylococcus aureus is a very close second and therefore
antibiotic treatment should provide coverage for both
organisms. Corynebacterium is part of the normal flora of
the auditory canal and does not typically cause infections.
Streptococcus pyogenes is the
most common cause of acute bacterial pharyngitis.

21. At 12 hours of age, a physical examination is performed


on a neonate who has intrauterine growth retardation. He is
noted to have microcephaly, jaundice, and
hepatosplenomegaly. Which of the following is the MOST
likely congenital viral infection in this neonate?
(A) cytomegalovirus
(B) herpes simplex virus
(C) rubella
(D) syphilis
21. (A) Cytomegalovirus (CMV) is one of the congenital
neonatal TORCH infections (toxoplasmosis, o ther [syphilis,
varicella-zoster, and parvovirus in this list], r ubella,
cytomegalovirus, and h erpes simplex/h epatitis/H IV). CMV
is the most common congenital infection. The disease-
specific manifestations for CMV include microcephaly with
Dr.Wahid Helmi - Dymiate -Egypt

periventricular calcifications, neonatal jaundice with direct


hyperbilirubinemia, and
hepatosplenomegaly. Other associated manifestations
include intrauterine growth retardation,thrombocytopenia,
and purpura. Disease-specific manifestations for herpes
simplex virus include skin/eye/mouth vesicles, encephalitis,
respiratory distress, and sepsis. Disease-specific
manifestations of rubella include congenital heart lesions
(patent ductus arteriosus, pulmonary artery stenosis, aortic
stenosis, ventricular defects), thrombocytopenic purpura
characterized by purple macular lesions (“blueberry
muffin” appearance), cataracts, retinopathy, and
sensorineural deafness. Disease-specific manifestations of
syphilis include mucocutaneous lesions (snuffles), periostitis,
osteochondritis, and hemolytic anemia. Often, these babies
are stillborn. Syphilis is caused by a spirochete, Treponema
pallidum, not a virus.

22. A previously healthy, 5-month-old infant is admitted to


the hospital due to lethargy progressing to
semiconsciousness. The physical examination reveals a
depressed mental status and bilateral retinal hemorrhages.
Which of the following is the MOST likely diagnosis?
(A) child abuse
(B) retinitis pigmentosa
(C) Reye syndrome
(D) viral encephalitis
22. (A) Approximately 40% of children who have been
physically abused showed evidence of ocular trauma. Retinal
hemorrhages are the most frequent ocular finding that
result from violent shaking. This form of child abuse is
Dr.Wahid Helmi - Dymiate -Egypt

termed shaken baby syndrome. The finding of retinal


hemorrhages in an infant without an appropriate medical
condition (eg, clotting disorder,leukemia) should raise
concerns about nonaccidental trauma. Some of the most
common presenting complaints of infants with shaken baby
syndrome are lethargy, coma, seizures,vomiting, and
respiratory distress. Retinal hemorrhages are not associated
with retinitis
pigmentosa, retinoblastoma, Reye syndrome, or viral
encephalitis. With Reye syndrome, an antecedent viral
illness is followed by vomiting and progressive lethargy. On
examination, there is usually fever, tachypnea, and stupor.
Laboratory hallmarks include elevated serum hepatocellular
enzyme assays and elevated serum ammonia. Retinitis
pigmentosa is a progressive
retinal degeneration and is characterized by pigmentary
changes, optic atrophy, and progressive impairment of
visual function. The presenting clinical manifestation is
usually an impairment of dark adaptation or night vision.
Clinical manifestations of viral encephalitis vary in severity
depending upon the etiologic organism (eg, cytomegalovirus,
mumps, echovirus). Some children will have mild symptoms
lapsing into a coma leading to death, whereas others are
febrile, with convulsions and hallucinations followed by full
recovery.

23. A 2-year-old child presents to the emergency department


via ambulance due to a seizure lasting approximately 2
minutes with jerking and somnolence. En route in the
ambulance her vital signs are: temperature 39°C rectal;
pulse 120/min; respirations 32/min; blood pressure 110/64
mm Hg.Upon further questioning, her mother claimed she
had a runny nose yesterday. On physical examination, she is
sleepy but arousable with negative Kernig and Brudzinski
Dr.Wahid Helmi - Dymiate -Egypt

signs. Which of the following seizures is the MOST likely


diagnosis?
(A) absence seizure
(B) complex partial seizure
(C) febrile seizure
(D) simple partial seizure
23. (C) A febrile seizure is a brief (less than 15 minutes),
generalized, symmetric, tonic–clonic seizure associated with
a febrile illness (temperature greater than 38.8°C) without
any central nervous system infection or neurologic cause. An
absence (petit mal) seizure is a brief (2 to 25
seconds) loss of consciousness that can occur multiple times
per day. There is no loss of tone,and frequently the only
observable behaviors are staring or minor movements such
as lip smacking and semipurposeful movements of the
hands. There is no postictal period. Complex partial seizures
(psychomotor) have varied symptoms including alterations
in consciousness,
unresponsiveness, and repetitive complex motor activities
that are purposeless. Often, at the beginning of the attack,
there is a psychoillusory phenomenon such as hallucinations,
visual distortions, visceral sensations, or feelings of intense
emotions. Simple partial seizures include
focal motor, adversive, and somatosensory seizures.
Manifestations of these seizures are varied including
hallucinatory, psychoillusory, or complex emotional
phenomena. Children will interact normally with their
environment, with the exception of those limitations imposed
by the seizure.
Following the seizure (minutes to hours), there may be
transient paralysis of the affected body
part.
Dr.Wahid Helmi - Dymiate -Egypt

24. A 6-month-old uncircumcised male infant presents with


a 2-day history of a fever (39.6°C rectal today), vomiting,
and poor feeding. Urinalysis of a catheterized specimen
reveals 50 to 100 white blood cells per high-power field and
moderate bacteria. Two days later, the urine culture results
are available. Which of the following is the MOST common
pathogen responsible for this
infant's first urinary tract infection?
(A) Enterococcus
(B) Escherichia coli .
(C) Klebsiella
(D) Staphylococcus saprophyticus
24. (B) Urinary tract infections (UTIs) are one of the most
common infections in children. Clinical features of a UTI
vary depending upon the age and sex of the child. In
newborns, the most common symptom is failure to thrive
associated with poor feeding, diarrhea, and vomiting. In
infants, the symptoms may be relatively nonspecific, such as
poor feeding, failure to gain weight,vomiting, fever, strong-
smelling urine, and irritability. As children grow older, the
initial signs and symptoms become more specific to the
urinary tract. In early infancy, males are two times more
likely than girls to have a UTI. Also, uncircumcised males
are 10 times more likely to be
affected than circumcised males. Escherichia coli is the most
common pathogen for the first UTI (80%) and of recurrent
infections (75%). Other organisms that cause infections
include Pseudomonas aeruginosa, Proteus, Enterobacter,
Klebsiella, and Enterococcus. An infection with
Staphylococcus saprophyticus, a coagulase-negative
staphylococcus, is primarily seen in
adolescents with a UTI.
Dr.Wahid Helmi - Dymiate -Egypt

25. A 6-month-old infant presents to the emergency


department with a 2-day history of vomiting and diarrhea.
Upon physical examination, she appears to be intermittently
irritable and restless with minimal tearing when crying and
dry mucous membranes. Her capillary refill is 2 to 3
seconds.Her urine sodium is less than 20 mEq/L and mildly
oliguric. On the basis of these clinical manifestations, what is
the magnitude of her dehydration?
(A) less than 3%
(B) approaching 3% to 5% (mild)
(C) approaching 6% to 10% (moderate)
(D) approaching 11% to 15% (severe)
25. (B) Dehydration is a common pathophysiologic alteration
in fluid and electrolyte balance in children. Children are at
an increased risk for dehydration because of their decreased
oral intake,especially when ill, and their higher ratio of
surface area to body weight, promoting significant
evaporative losses. Important clinical features to estimate
the degree of dehydration include postural blood pressure,
changes in heart rate, capillary refill time, skin turgor and
color, lack of tears, lack of external jugular venous filling
when supine, sunken fontanel (if present), and altered
mental status. This infant was estimated to have mild
dehydration (3% to 5% decrease in
body weight) with decreased tears, slightly longer capillary
refill time (2 to 3 seconds), and intermittent irritability and
restlessness. Severe dehydration (11% to 15% decrease in
body weight) manifests as markedly decreased skin turgor
with parched or mottled mucous membranes, absence of
tears, tachycardia, capillary refill greater than 4 seconds,
hypotension,
circulatory collapse, and anuria. Moderate dehydration (6%
to 10% decrease in body weight) manifests as decreased skin
turgor; dry mucous membranes; decreased tearing;
oliguria; and normal pulse, blood pressure, and perfusion.
Dr.Wahid Helmi - Dymiate -Egypt

26. A previously well, 15-month-old baby boy is brought to


the emergency department in the middle of the night with
increased irritability and severe paroxysmal colicky
abdominal pain followed by vomiting. On physical
examination, a tubular mass is palpated in the abdomen.
The rectal
examination reveals bloody mucus. Which of the following is
the MOST likely diagnosis?
(A) appendicitis
(B) infectious enteritis
(C) intussusception.
(D) pyloric stenosis
26. (C) Intussusception is the most common cause of
intestinal obstruction between 3 months and 6years of age. It
is twice as common in males than females. It is caused by
intestinal invagination,usually around the ileocecal valve.
The classic presentation is intermittent severe colicky
abdominal pain with legs drawn up, followed by periods of
comfort or falling asleep. Vomiting usually occurs in the
early phase, which later becomes bilious. A passage of blood
and mucus in the stool (“currant jelly stools”) occurs in
60% of the cases. Palpation of the abdomen usually
reveals a sausage-shaped mass in the right upper quadrant.
The classic presentation of pyloric stenosis is in first-born
males of 3 to 6 weeks of age, presenting with nonbilious
projectile vomiting leading to dehydration with
hypochloremia, hypokalemia, and metabolic alkalosis.
Afirm, movable, 2-cm olive-shaped mass (“olive”) is
palpable superior and to the right of the
umbilicus in the midepigastrium. In addition, peristaltic
waves may be visible on the physical examination. The
classic presentation of appendicitis presents with a period of
anorexia followed by steady periumbilical pain shifting to
Dr.Wahid Helmi - Dymiate -Egypt

the right lower quadrant; nausea and vomiting is followed


by a low-grade fever. Diarrhea (nonbloody and nonmucous),
if it occurs, is infrequent. Peritoneal signs are present. The
incidence increases with age and peaks during
adolescence. Infective enteritis usually begins with emesis
followed by crampy abdominal pain of hyperperistalsis. This
sequence of symptoms with emesis preceding pain is an
important factor in distinguishing it from intussusception.
Masses are not palpated with infective enteritis.

27. Which of the following daily maintenance fluid


requirements is the closest approximation for a24-kg child
who is refusing to eat?
(A) 1,080 mL
(B) 1,200 mL
(C) 1,580 mL.
(D) 2,000 mL

27. (C) Dehydration is a common pathophysiologic


alteration in fluid balance in children. The body has a
maintenance fluid requirement to replace daily normal
losses that occur through the skin,kidney, intestines, and
respiratory tract. The following formula can be used to
calculate the usual
amount of fluid a healthy child requires by mouth to
maintain hydration:
100 mL/kg for the first 10 kg of body weight
50 mL/kg for the next 10 kg of body weight
20 mL/kg for the weights above 20 kg
For this question, a 24-kg child would require:
100 mL/kg × 10 kg = 1,000 mL for the first 10 kg
50 mL/kg × 10 kg = 500 mL for the next 10 kg
20 mL/kg × 4 kg = 80 mL for the next 4 kg
Total = 1,580 mL 24 kg
Dr.Wahid Helmi - Dymiate -Egypt

28. Which of the following is the recommended treatment


for a 4-year-old child with presumed bacterial meningitis?
(A) cefotaxime or ceftriaxone plus ampicillin
(B) cefotaxime or ceftriaxone plus vancomycin.
(C) gentamicin plus ampicillin
(D) ampicillin plus chloramphenicol
28. (B) The most common etiologic organisms for bacterial
meningitis in children are Spneumoniae, N Meningitidis, and
H influenzae. Because of an increase in resistant
Spneumoniae, coverage with vancomycin and a third-
generation cephalosporin such as cefotaxime or ceftriaxone
is needed for best coverage. Gentamicin can be used but, as
with all aminoglycosides, caution is needed regarding
toxicity. Ampicillin, rifampin, and chloramphenicol are
alternative treatments if necessary.

29. At a 2-month-old well-child checkup, a female infant is


noted to have the following physical findings: widely open
anterior and posterior fontanels, large protruding tongue,
coarse facial features, low-set hair line, and an umbilical
hernia. In the newborn period, there was aprolongation of
physiologic icterus. The results of the newborn screening test
are abnormal.
Which of the following is the MOST likely diagnosis?
(A) congenital adrenal hyperplasia
(B) congenital hypothyroidism.
(C) Crigler–Najjar syndrome
(D) galactosemia
29. (B) Congenital hypothyroidism is one of the most
common disorders tested for in newborn screening tests,
revealing an elevated TSH (thyroid stimulating hormone)
Dr.Wahid Helmi - Dymiate -Egypt

and a decreased T4(thyroxine). Symptoms suggestive of


congenital hypothyroidism in the neonate include hypotonia,
coarse facial features, hirsute forehead, large fontanels
(anterior and posterior),
widely open sutures, umbilical hernia, protruding/large
tongue, hoarse cry, distended abdomen,and prolonged
jaundice. Signs of congenital hypothyroidism include
lethargy or hypoactivity,poor feeding, constipation, mottling,
and hypothermia. Congenital adrenal hyperplasia (CAH) is
not universally screened for in the newborn screening test,
as it is included in only 14 of the 50 states. In females with
CAH, there may be virilization with abnormalities of the
external genitalia varying from mild enlargement of the
clitoris to complete fusion of the labioscrotal folds. Signs of
adrenal insufficiency (salt loss) may present in the first few
days of life. Crigler–Najjar
syndrome is not one of the disorders tested for in the
standard newborn screening tests. It is an inherited disease
producing congenital nonobstructive, nonhemolytic,
unconjugated severe hyperbilirubinemia. The physical
findings in this infant do not correlate with Crigler–Najjar
syndrome. Galactosemia is tested for in the newborn
screening test in nearly all 50 states. The
infant may have symptoms of cataract, hepatomegaly, and
prolonged jaundice. Often, these
neonates have Escherichia coli sepsis, leading to death in the
first 2 weeks of life if not treated
promptly.

30. Within hours of birth, a healthy infant is noted to have a


superficial swelling over the right occipitoparietal region
that extends across the suture line. Which of the following
conditions is it MOST likely to be?
(A) caput succedaneum
Dr.Wahid Helmi - Dymiate -Egypt

(B) cephalohematoma
(C) craniotabes
(D) subgaleal hemorrhage
30. (A) Caput succedaneum is a result of fluid and blood
accumulation in the occipitoparietal region of the newborn's
scalp due to the vacuum effect of membrane rupture. A
cephalohematoma is afirm, tense external swelling of the
cranium that does not extend across suture lines because it is
limited to the surface of one cranial bone. It occurs most
often in the parietal area. This subperiosteal hemorrhage
usually is not present at birth, but develops within the first
24 hours of life. Craniotabes is a condition caused by the
osteoporosis of the outer table of the involved membranous
bone, generally over the temporoparietal or parietooccipital
areas, creating a “pingpong ball” sensation when gentle
pressure is applied. A subgaleal hemorrhage is a firm,
fluctuant external swelling of the cranium that does extend
across suture lines and increases in size over time.

31. A 2-year-old male child is brought to the emergency


department by his mother with a sudden onset of choking,
gagging, coughing, and wheezing. Vital signs are
temperature 37°C ; pulse 120/min; and respirations
28/min. The physical examination reveals decreased breath
sounds
over the right lower lobe with inspiratory rhonchi and
localized expiratory wheezing. The chest X-ray reveals
normal inspiratory views but expiratory views show
localized hyperinflation with mediastinal shift to the left.
Which of the following is the MOST likely diagnosis?
(A) asthma
(B) epiglottitis
(C) foreign body aspiration.
(D) pulmonary embolism
Dr.Wahid Helmi - Dymiate -Egypt

31. (C) Foreign body aspiration into the respiratory tract is


associated with an acute choking or coughing episode with
expiratory wheezing (indicative of a lower airway
obstruction) in children aged 6 months to 4 years of age.
Often, there is a history of the child playing with small toys
that are commonly aspirated. Asymmetrical physical
findings of decreased breath sounds and localized wheezing
are present with foreign body aspiration. A positive forced
expiratory chest X-ray shows a mediastinal shift away from
the affected side. Radiolucent foreign bodies such as plastic
toys may not appear on an X-ray, but there will be evidence
of this mediastinal
shift. Asthma is generally characterized by wheezing, but it
is not unilateral nor is it of sudden onset. A chest X-ray
reveals bilateral hyperinflation with flattening of the
diaphragm. Epiglottitis is a life-threatening upper airway
obstructive condition that presents with a sudden onset of
fever, dysphagia, drooling, and inspiratory retractions with
stridor. A lateral neck X-ray reveals an enlarged, indistinct
epiglottis (“thumb sign”); however, the chest X-ray is
normal. Pulmonary embolism, rare in children, presents
clinically with acute dyspnea, tachypnea, and
tachycardia.There may be mild hypoxemia, rales, and focal
wheezing. Chest X-rays may be normal, or there
may be a peripheral infiltrate, small pleural effusion, or
elevated hemidiaphragm.

32. A 16-year-old girl is brought to the emergency


department by ambulance after reportedly ingesting “a
bottle of aspirin.” Vital signs are temperature 37.8°C
oral; pulse 94/min;respirations 30/min; blood pressure
100/68 mm Hg. What would you expect the blood gases to
show that would confirm she had swallowed the aspirin?
(A) anion gap metabolic acidosis with respiratory acidosis
Dr.Wahid Helmi - Dymiate -Egypt

(B) nonanion gap metabolic acidosis with respiratory alkalosis


(C) anion gap metabolic acidosis with respiratory alkalosis.
(D) nonanion gap metabolic acidosis with respiratory acidosis
32. (C) An acute salicylate overdose (greater than 150
mg/kg) will produce symptoms of salicylate intoxication.
Chronic salicylate intoxication occurs with ingestion of
greater than 100 mg/kg/day for at least 2 days. Salicylates
affect most organ systems, leading to various metabolic
abnormalities. Because salicylates are a gastric irritant,
symptoms of vomiting and diarrhea
occur soon after the overdose, which may contribute to the
development of dehydration.Salicylates stimulate the
respiratory center leading to hyperventilation and
hyperpnea resulting in respiratory alkalosis and
compensatory alkaluria. A characteristic feature of
salicylate
intoxication is the coexistence of a respiratory alkalosis with
a widened anion gap metabolic acidosis.

33. A 16-year-old high school boy presents to the emergency


department 4 hours after sustaining an abrasion to his knee
after a fall while rollerblading on the school playground. His
school immunization record reveals that his last diphtheria,
tetanus, and pertussis (DTaP) booster was administered at
age 4. In this situation, which of the following is the MOST
appropriate plan?
(A) administer tetanus toxoid
(B) administer adult tetanus and diphtheria toxoid (Td)
(C) administer diphtheria, tetanus toxoid, and acellular pertussis
(Tdap) vaccine
(D) administer tetanus immune globulin
33. (C) Generalized tetanus (lockjaw) is a neurologic disease
caused by Clostridium tetani. Although any open wound is a
potential source for contamination with C tetani, those with
Dr.Wahid Helmi - Dymiate -Egypt

dirt,soil, feces, or saliva are at increased risk. Tetanusprone


wounds contain devitalized tissue,
especially those caused by punctures, frostbite, crush injury,
or burns. Recommendations for tetanus prophylaxis in a
child with a laceration or abrasion depend upon the number
of previous vaccinations, occurrence of last booster, type of
wound (clean or tetanus-prone), and age of child. In this
case, the patient is older than 7 years and had all of his
previous immunizations;
however, his most recent booster was greater than 10 years
ago. Thus, he should receive an adult-type diphtheria and
tetanus toxoid with acellular pertussis. In most cases, when
tetanus toxoid is required for wound prophylaxis in a child
older than 7 years, the Td instead of tetanus
toxoid alone is recommended so that diphtheria immunity is
maintained. If tetanus immunization is not up to date at the
time of wound treatment, then the immunization series
should be completed according to the primary immunization
schedule. If a child is younger than 7 years, then the
diphtheria, tetanus, acellular pertussis (DTaP) booster is
indicated, unless there is acontraindication for pertussis, in
which case the diphtheria and tetanus (DT) booster should
be administered. Tetanus immune globulin (TIG) is
recommended for treatment of tetanus. Under special
circumstances, a patient infected with the human
immunodeficiency virus (HIV) with a
tetanusprone wound should also receive TIG in addition to
the prophylactic vaccine.

34. Which of the following newborn reflexes should still be


present at the 9-month check-up?
(A) Galant reflex
(B) Landau reflex
(C) rooting reflex
(D) parachute reflex
Dr.Wahid Helmi - Dymiate -Egypt

34. (D) Normally, primitive reflexes are present at birth and


should not persist beyond the age of 6months. However, the
parachute reflex is a postural response that normally
appears around 7months of age to coincide with volitional
movement and persists for life. It occurs when an infant
is held prone by the waist over a surface and lowered with
the head downward and extends the arms and legs as a form
of protection. The rooting reflex occurs when the cheek is
stroked on the infant and they turn his/her head to feed.
Galant and Landau reflex disappear by the age of 2
months (trunk incurvation upon stroking the back) and 6
months (the baby lifts head and straightens spine upon being
held prone), respectively

35. A 3-day-old infant has bilateral copious, yellow-green eye


discharge and conjunctival inflammation. A Gram stain of
this discharge reveals gram-negative intracellular
diplococci.Which of the following antibiotics is the drug of
choice for this infection?
(A) ceftriaxone
(B) cephalexin
(C) erythromycin
(D) gentamicin
35. (A) Gonococcal ophthalmia neonatorum presents as a
unilateral or bilateral serosanguineous discharge and then
within 24 hours the discharge becomes mucopurulent,
followed by conjunctival injection and edema of the eyelids.
The usual incubation period for Neiserria gonorrhea is 2 to 5
days; however, the infection may be present at birth or
delayed greater than 5
days if there has been instillation of silver nitrate
prophylaxis. A presumptive diagnosis is made by the
Dr.Wahid Helmi - Dymiate -Egypt

demonstration of gram-negative intracellular diplococci on


Gram stain. Definitive diagnosis is made by culture.
Following a positive Gram stain and pending culture
results,treatment should be promptly initiated with
ceftriaxone (50 mg/kg/24 hours IV or IM for one dose
not to exceed 125 mg), a third-generation cephalosporin with
good coverage for gram-negative bacteria. An alternate drug
is cefotaxime (100 mg/kg/24 hours IV or IM every 12 hours
for 7days or 100 mg/kg as a single dose), which is also a
third-generation cephalosporin. Although erythromycin
drops (0.5%) are used prophylactically for N gonorrhea, this
is not an effective
treatment. Gentamicin would be used for Pseudomonas, and
Chlamydia is treated with erythromycin. Cephalexin as a
first-generation cephalosporin does not have coverage for
gramnegative
bacteria.

36. A 10-year-old boy presents to the office, complaining of a


painful, swollen area along his right jaw and neck. On
physical examination, he is noted to be febrile and has
diffuse tenderness over the right parotid gland. His
laboratory tests include an elevated serum amylase. His
parents elected not to vaccinate him. In this patient, based
on the most likely diagnosis, which of the following is a
complication of his disease?
(A) hepatitis
(B) nerve deafness.
(C) pneumonitis
(D) testicular torsion
36. (B) The most likely diagnosis in this patient is mumps. It
is endemic in most unvaccinated populations. The onset is
characterized by pain and swelling in one or both parotid
glands. The pain can be exacerbated by tasting sour liquids
Dr.Wahid Helmi - Dymiate -Egypt

such as lemon juice. An elevated serum amylase level is


common and coincides with the parotid swelling. Unilateral,
rarely bilateral, nerve
deafness is a complication of mumps that may be transient
or permanent. Other complications include
meningoencephalomyelitis, orchitis, epididymitis,
pancreatitis, arthritis, and rarely thyroiditis and
myocarditis.

37. Which of the following physical examination findings in


a newborn infant should cause the clinician to suspect a
genetic disorder?
(A) café au lait spots
(B) subconjunctival hemorrhages
(C) miliaria
(D) vernix caseosa
37. (A) Café au lait spots are brown macules that may be
found on any part of the body. The presentation of six or
more spots greater than 1.5 cm is a sign of
neurofibromatosis, a genetic disorder that results in
neurofibromas that can develop in any organ/tissue system.
Miliaria are blocked sweat gland ducts that are commonly
found on the face, scalp, or intertriginous areas.
Vernix caseosa is a normal finding in newborns and is a
whitish, greasy layering on the body— it decreases as an
infant comes to full term. Subconjunctival hemorrhages are
a common finding in infants secondary to birth trauma.

38. The newborn examination at 1 minute shows a heart rate


of 120 bpm, strong cry, some flexion in the upper
extremities, sneezing with nasal catheter suction, and bluish
hands and feet; but the remainder of the body is pink. What
is the Apgar score?
(A) 7
Dr.Wahid Helmi - Dymiate -Egypt

(B) 8.
(C) 9
(D) 10
38. (B) The Apgar score assesses the newborn at 1-minute
and 5-minute intervals to determine the need for
resuscitative care. The infant is evaluated by heart rate,
respiratory effort, muscle tone,response to catheter in
nostril, and color, and each is rated on a scale of 0, 1, or 2 for
a total
score of 10. The heart rate is scaled 0–2 for absent, less than
100 bpm (slow), and greater than 100 bpm; respiratory
effort of absent, slow/irregular, and good crying. Muscle
tone scale (0–2) consists of limp, some flexion, and active
motion; response to catheter stimulation (0–2) is scaled no
response, grimace, and cough/sneeze. Finally, color is scored
0–2 for blue/pale, body
pink with blue extremities, and completely pink.

39. Which of the following is the most common congenital


heart malformation?
(A) atrial septal defect
(B) tetralogy of Fallot
(C) ventricular septal defect
(D) transposition of the great vessels
39. (C) Ventricular septal defect, a hole between the two
ventricles, can be cyanotic or acyanotic based on the size of
the defect, and accounts for 30% of cases of congenital heart
disease. Atrial septal defect occurs in approximately 10% of
congenital heart disease cases. Transposition of great vessels
is an embryonic malformation resulting in the aorta arising
from the right ventricle
and the pulmonary artery arising from the left ventricle. It is
responsible for about 10% of all congenital malformations.
Tetralogy of Fallot, consisting of a ventricular septal
Dr.Wahid Helmi - Dymiate -Egypt

defect,overriding aorta, pulmonic/subpulmonic stenosis, and


right ventricular hypertrophy, accounts for 10% of
congential heart disease.

40. A 5-year-old male child presents to the office for his


kindergarten physical examination.Assuming that the
patient's immunizations have been up to date, which of the
following are the immunizations that the patient should
receive at the end of today's visit?
(A) hepatitis B, inactivated poliovirus (IPV), diphtheria, tetanus,
acellular pertussis (DTaP),
measles, mumps, rubella (MMR), varicella
(B) IPV, DTaP, MMR, pneumococcal (PCV)
(C) IPV, DTaP, MMR, Haemophilus influenzae type B (Hib)
(D) DTaP, IPV, MMR, varicella.
40. (D) The immunization schedule is developed biannually
by the Centers for Disease Control and Prevention.
Assuming that the child has had the appropriate
immunizations at the regularly scheduled examinations, the
recommended immunizations at the 4- to 6-year-old range
are the
DTaP (diphtheria, tetanus, acellular pertussis), IPV
(inactivated polio), and the MMR (measles,mumps, and
rubella). The hepatitis series should have been completed by
the age of 6 months and the Haemophilus influenzae type B
(Hib) should be completed by the age of 12 to 15
months.Varicella is given from 12 to 18 months and again
from 4 to 6 years; the PCV (pneumococcal)
should be finished by 12 to 15 months.

41. Which of the following is an absolute contraindication to


breastfeeding?
(A) tuberculosis of the mother
(B) methadone treatment (20 mg/d)
Dr.Wahid Helmi - Dymiate -Egypt

(C) maternal smoking


(D) infant with cystic fibrosis
41. (A) There are only two known absolute contraindications
to breastfeeding: tuberculosis of the mother and
galactosemia of the infant. The highly contagious nature of
tuberculosis makes the risk greater than the benefit, and
infants with galactosemia are unable to digest any lactose
due to an
enzyme deficiency. Infants of mothers in a methadone
program may be breastfed as long as the mother's dose is
less than 40 mg. While nicotine is transmitted in breast milk
and is therefore strongly discouraged, it is not an absolute
contraindication. As long as a breastfed infant with cystic
fibrosis is maintaining normal growth with supplemented
pancreatic enzymes,
breastfeeding is encouraged.

42. A 4-month-old infant presents to the office for her “well-


check.” The parents state that they have no concerns and think
she is doing well. She is being breastfed every 4 to 6 hours and
has four wet diapers a day and two dirty diapers a day. Her birth
weight was 7 lb 7 oz (50th percentile);she missed her 2-month
appointment and at today's visit her weight is 11 lb 5 oz (5th
percentile).
The clinician, however, is very concerned and diagnoses the
infant with which of the following?
(A) dwarfism
(B) growth deficiency
(C) lactose intolerance
(D) Beckwith–Wiedemann syndrome
42. (B) Failure to thrive is diagnosed in infants younger than
the age of 6 months with a decrease in growth velocity that
results in a decrease in two major percentile lines on the
growth chart. In the case of this patient, she was initially in
Dr.Wahid Helmi - Dymiate -Egypt

the 50th percentile and crossed the 25th and 10th percentile
and fell into the 5th percentile. Failure to thrive is also
known as growth deficiency
and may also be diagnosed if the child is younger than 6
months and has not grown for two consecutive months or if
a child is older than 6 months and has not grown for 3
consecutive months. Growth hormone deficiency/dwarfism
may present with decreased growth velocity later in
childhood; the drop in percentiles is grossly below the 5th
percentile mark. Lactose intolerance presents with varying
gastrointestinal symptoms without the marked decrease in
weight. Beckwith–Wiedemann syndrome consists of
macrosomia, macroglossia, and omphalocele and they are at
increased risk for malignancies, hypoglycemia, and
dysmorphism (usually of the ears).

43. During influenza season, a 15-year-old boy presents to


the emergency department, unresponsive.The parents state
that when they tried to wake him up in the morning he
would not get up and was barely breathing. They deny any
drug or alcohol use and state that he just had some cold
symptoms the past few days. A spinal tap shows decreased
glucose, increased pressure, and increased proteins, but
there were no cells found. The rest of the blood work shows
elevated liver enzymes, but normal serum bilirubin and
alkaline phosphatase. A liver biopsy demonstrates
microvesicular steatosis without glycogen and large
mitochondria. Which of the following is the
best treatment for this patient?
(A) high-dose steroids
(B) broad-spectrum antibiotics until the cultures come back
(C) supportive treatment, including maintenance fluids and
hyperventilation
(D) liver transplant
Dr.Wahid Helmi - Dymiate -Egypt

43. (C) This patient has presented with classical findings of


Reye syndrome—upper respiratory infection followed by
unresponsiveness. Reye syndrome is usually preceded by an
upper respiratory tract illness, which progresses into
vomiting, strange behavior, stupor, and coma.Liver
function tests (LFTs) will be markedly elevated (without
jaundice); however, the serum bilirubin and alkaline
phosphatase are normal. Unresponsive patients who have a
spinal tap will show no cells in the CSF and glucose may be
low with increased CSF pressure. If arterial blood gases are
ordered, they will show a mild respiratory alkalosis and
metabolic acidosis. A liver biopsy will show little
inflammatory changes with diffuse microvesicular steatosis
and absent
glycogen from the hepatocytes. The mitochondria of the
hepatocytes are large and polymorphic with decreased
matriceal density. Treatment for patients with Reye
syndrome is largely supportive—specifically decreasing
cerebral edema. There is no place for antibiotics or steroids.
The liver will fully recover if the cerebral edema is
decreased.

44. A 5-year-old female child presents for her kindergarten


physical examination and her mother mentions that she
thinks she looks a bit yellow to her. The clinician notes
diffuse jaundice,icterus, and Kayser–Fleischer rings. Which
of the following is the treatment of choice for this patient?
(A) alpha-interferon therapy
(B) D-penicillamine therapy
(C) methylprednisolone
(D) protease inhibitor therapy
44. (B) Wilson disease is a result in a genetic mutation on
chromosome 13 that causes decreased bile excretion of
copper and results in accumulation of copper by the liver,
Dr.Wahid Helmi - Dymiate -Egypt

specifically the ceruloplasmin. The build-up of copper causes


damage to the liver, basal ganglia, and other tissues.
Physical examination shows jaundice, hepatosplenomegaly,
Kayser–Fleischer rings (abrown band at the junction of the
iris and cornea under slit-lamp), and neurologic
manifestations later in the disease process. Laboratory tests
show marked decrease in ceruloplasmin of the liver, anemia,
hemolysis, and severely elevated bilirubin with decrease
alkaline phosphatase.Urinalysis shows severe elevation in
copper excretion, glycosuria, and aminoaciduria. Liver
biopsy is conclusive with evidence of copper greater than 250
μg/g of dry tissue. Treatment requires copper chelation
with D-penicillamine or trientine hydrochloride. Liver
transplant may be required with noncompliance and in
acute fulminant disease. Copper chelation is continued for
life with the addition of zinc (decrease copper absorption)
and vitamin B6 (decrease optic neuritis). Genetic screening
of siblings and future children should be strongly
encouraged. Alphainterferon therapy is mainly used to treat
hepatitis patients. There is no place for steroids in therapy
and protease inhibitors are antiviral medications that are
typically used in HIV patients.

45. A 24-month-old infant presents for his routine physical


examination. The parents state that he has been following all
of his developmental milestones. On examination, the
clinician hears a grade II/VI murmur along the left sternal
border, which radiates into the left axilla and the left side of
the back. The child also has decreased femoral pulses
bilaterally. The clinician orders a chest Xray.Which of the
Dr.Wahid Helmi - Dymiate -Egypt

following is the expected finding on X-ray based on the


presentation?
(A) notching or scalloping of the ribs
(B) boot-shaped heart—right ventricular hypertrophy
(C) “egg on string”—narrowed mediastinum
(D) absence of the main pulmonary artery
45. (A) The patient's presentation is consistent with findings
of coarctation of the aorta. The pathognomonic finding in
coarctation is decreased or absent femoral pulses. However,
the majority of children show no signs of coarctation in
infancy and develop signs and symptoms during childhood,
most notably unequal pulses and blood pressure between
arms and legs (arms
lower than legs). In addition, a grade II/VI ejection murmur
is heard at the aortic area and left sternal border that
radiates into the left axilla and left back. Chest X-ray shows
a normal-sized heart, a prominent aorta, indents at the level
of the coarctation, and a dilated poststenotic segment
resulting in the “figure 3” sign. Scalloping or notching of
the ribs is due to enlargement of the intercostal arteries.
Echocardiography is used to directly visualize the
coarctation and estimate the obstruction. Asymptomatic
infants and children are encouraged to have corrective
surgery prior to age 5, after which they are at increased risk
for myocardial dysfunction and hypertension, and require
exercise testing prior to participation in aerobic activities.
The bootshaped heart is seen in patients with tetralogy of
Fallot secondary to right ventricular hypertrophy; the
narrowed mediastinum finding with “egg on a string” is
typically seen in patients with transposition of the great
vessels.

46. An 8-year-old female child presents to the emergency


department with her parents. They state she has been
Dr.Wahid Helmi - Dymiate -Egypt

coughing all night the past few nights, to the point she
sounds like she is choking. On examination, the clinician
notes mild retractions at rest. Retractions worsen with the
lung examination and there is diffuse stridor on
auscultation. Pulse oximetry is 92% on room air and the
child is afebrile. Which of the following is the recommended
treatment for this patient?
(A) supportive care only—mist therapy
(B) IV (intravenous) antibiotics, with gram-negative coverage
(C) IM (intramuscular) dexamethasone
(D) nebulized racemic epinephrine and oral dexamethasone

46. (D) Viral croup usually presents with cough that may
sound like a dog or a seal barking. The
Dr.Wahid Helmi - Dymiate -Egypt

patients are usually afebrile and also present with stridor


either at rest, in severe cases, or when
agitated, in mild cases. In addition, the patient may be
cyanotic and have retractions and acute
shortness of breath. Radiologic examination of the neck
shows subglottic narrowing with a
normal epiglottis, “steeple sign.” However, X-rays are
usually not indicated in patients with the
common presenting symptoms. Treatment for viral croup
is mainly symptomatic, especially in
mild cases consisting of oral hydration and mist therapy.
Severe cases (stridor at rest) call for
oxygen in patients who have desaturated, and nebulized
racemic epinephrine and
glucocorticoids. Dexamethasone as an intramuscular
injection or oral as a one time dose is
effective in alleviating symptoms, decreasing the need for
intubation, and decreasing hospital
stays. Inhaled budesonide is also effective in decreasing
hospital stays and improving symptoms,
but dexamethasone is more cost-effective. Patients who are
unable to be stabilized need airway
maintenance either by intubation with endotracheal tube
or by tracheostomy if intubation fails.
Because it is a self-limiting disorder, unless there is a
secondary infection most children recover
in a few days.

47. A neonate presents with meconium ileus that is


successfully unobstructed. The infant returns at her 4-month
appointment with signs of failure to thrive. Which of the
following is the most likely diagnosis for this patient?
(A) cystic fibrosis
Dr.Wahid Helmi - Dymiate -Egypt

(B) Wilson disease


(C) intussusception
(D) volvulus
47. (A) Cystic fibrosis (CF) is a major cause of
gastrointestinal and pulmonary morbidity in children due to
mutations in the CF genes. The mutations lead to a
deficiency in cystic fibrosis transmembrane conductance
regulator protein that controls movement of salt and water
into and out of epithelial cells and results in production of
abnormally thick mucus. About 15% of patients
with CF present with meconium ileus at birth. This is
typically treated with enema for disimpaction and rarely
surgery. Approximately half of the infants with CF will
present with failure to thrive, which is diagnosed by lack of
growth for 2 consecutive months in patients younger than 6
months of age. They may also present with respiratory
compromise. However, not
all patients present in childhood. Diagnosis of CF is
confirmed by a sweat chloride level above 60 meq/L or with
genetic testing. Treatment for patients with CF is mainly
symptomatic therapy for obstructions of the digestive and
respiratory tract. In addition, there is pancreatic enzyme
supplementation to aid in digestion and vitamin and calorie
supplementation for deficiencies in
the diet. Gene therapy is now being looked at for future
treatment. Intussusception (telescoping of the small
intestine) typically presents in an infant with paroxysmal
abdominal pain, vomiting, and diarrhea that may progress
into bloody stools. Volvulus is normally the result of
intestinal
malrotation that causes occlusion of the superior mesenteric
artery and eventual bowel necrosis.Infants typically present
within 3 weeks of life with bile-stained vomiting and bowel
obstruction.Wilson's disease is the defect in the ability to
excrete copper in the bile that results inaccumulation of
Dr.Wahid Helmi - Dymiate -Egypt

copper in the liver.

48. Which of the following is NOT a cyanotic heart lesion?


(A) transposition of the great arteries
(B) atrioventricular septal defect
(C) hypoplastic left heart syndrome
(D) tricuspid atresia
48. (B) Cyanotic heart lesions are a result of a right-to-left
shunt. These include tetralogy of Fallot,pulmonary atresia
with and without ventricular septal defect, tricuspid atresia,
hypoplastic left heart syndrome, and transposition of the
great arteries. The right-to-left shunt results in
deoxygenated blood reaching the left ventricle, aorta, and
systemic arteries. The decreased oxygen in the blood results
in decreased oxygen to the tissue and subsequently causes
cyanosis.Atrial septal defect, ventricular septal defect,
atrioventricular septal defect, and patent ductusarteriosus
most commonly present with a left-to-right shunt.

49. A 2-week-old male infant presents for a routine checkup.


The mother complains that he nurses every hour, but vomits
(nonbilious) after every time he eats. He has only had three
bowel movements since he has been home. On examination,
the infant has not gained any weight since leaving the
hospital, and the clinician notes gastric peristaltic waves.
Which of the following is
the treatment of choice for this patient?
(A) pyloromyotomy
(B) metoclopramide
(C) laparotomy
(D) omeprazole

49. (A) This infant is presenting with signs and symptoms of


Dr.Wahid Helmi - Dymiate -Egypt

pyloric stenosis. Infants typically have vomiting (projectile


at times) after every feeding and it normally starts between
the age of 2 and 4 weeks. The infant nurses fervently and is
hungry. In addition, there may be dehydration,constipation,
weight loss, and apathy. Abdomen may be distended with
gastric peristaltic waves.
Occasionally, an olive-sized mass can be felt in the right
upper quadrant with deep palpation after the child has
vomited. Vomitus is typically nonbilious. Diagnosis is
confirmed by an upper gastrointestinal series with delayed
gastric emptying, enlarged pyloric muscle, and
characteristic semilunar impressions on the gastric antrum.
In addition, an ultrasound is needed to verify the
hypertrophic muscle. The treatment of choice for these
patients is pyloromyotomy, which can be done
laparoscopically. These patients make full recoveries and
have an excellent prognosis.

50. A 6-month-old infant presents for her checkup. Her


father mentions that they started solid foods after her 4-
month check. She has had foul-smelling diarrhea off and on
for the first month of solids; it now occurs after every meal
and looks greasy. They have tried different formulas and
different cereals without improvement. What is the
diagnostic test of choice for the most likely disorder?
(A) sweat chloride test
(B) RAST (radioallergosorbent assay test)
(C) gastrin level
(D) intestinal biopsy
50. (D) Celiac disease or gluten enteropathy typically
presents with diarrhea episodes in the first 6to 12 months of
life—when whole grains are first fed. Therefore, in strictly
breastfed babies,symptoms may not be noticed until solid
foods are begun. The diarrhea is usually intermittent at
Dr.Wahid Helmi - Dymiate -Egypt

first and then typically progresses into pale, greasy, foul-


smelling, frothy stools. Additional symptoms may be
constipation, vomiting, and abdominal pain, which may lead
the clinician to think of intestinal obstruction. Other findings
may be failure to thrive, anemia, and vitamin deficiencies.
Stool sample demonstrates excessive fecal fat excretion.
Blood tests show
hypoproteinemia and impaired carbohydrate absorption.
Intestinal biopsy is the diagnostic test of choice for celiac
disease. Results show shortened celiac mucosa, absent villi,
lengthened crypts of Lieberku$$$hn, plasma cell infiltration
of the lamina propria, and intraepithelial lymphocytes.
Treatment consists of dietary restriction of gluten—wheat,
rye, and barley. Steroids are given on an as needed basis.
Sweat chloride testing is utilized in patients suspected of
cystic fibrosis.Gastrin level is taken in patients suspected of
Zollinger–Ellison syndrome, and RAST (radioallergosorbent
assay test) is used in patients to determine different
environmental-type allergens

51. A 9-year-old child presents to the urgent care center with


her mother. The child is complaining of dark colored urine.
The mother mentions that the child was complaining of sore
throat and cold symptoms a few weeks ago. The urine shows
gross hematuria without nitrites or leukocytes.Which of the
following is the best test to help the clinician confirm the
diagnosis?
(A) monospot
(B) antistreptolysin O titer
(C) immunoglobulin electrophoresis
(D) renal biopsy
51. (B) The most likely diagnosis for this patient is
poststreptococcal glomerulonephritis. The diagnosis is
supported by a documented culture of group A beta-
Dr.Wahid Helmi - Dymiate -Egypt

hemolytic streptococcus infection. If a culture is not


available, like of the patient in this scenario, the clinician can
order
an antistreptolysin O titer. Antistreptolysin is an enzyme
released by group A streptococcus and is elevated for up to 1
month after strep infection. Glomerulonephritis presents
with gross hematuria with or without edema. Hypertension,
proteinuria, ascites, and headache may also be
present. Treatment with antibiotics is useful if infection is
still present, and, if necessary,symptomatic treatment for
renal failure is done with hemodialysis. Symptoms typically
resolve within a few weeks. The monospot is used to
diagnose infectious mononucleosis. Renal biopsy could be
performed on extreme cases of glomerulonephritis but is not
typically necessary.
Immunoglobulin electrophoresis would be utilized in
patients suspected of having immunoglobulinopathies or
IgA-mediated glomerulonephritis.

52. Upon performing a newborn examination, the clinician


notes a widened pulse pressure,paradoxical splitting of S2,
and a “machine”-like murmur heard best at the second
intercostals space, left sternal border, and inferior to the
clavicle. Which of the following is the most likely diagnosis?
(A) tetralogy of Fallot
(B) ventricular septal defect
(C) atrial septal defect
(D) patent ductus arteriosus
52. (D) Patent ductus arteriosus (PDA) is an isolated
abnormality that occurs in infants. The ductus arteriosus is
a normal fetal vessel that joins the aorta and the pulmonary
artery and spontaneously closes after 3 to 5 days. Lack of
closure results in the audible murmur that is
“machinelike” and
Dr.Wahid Helmi - Dymiate -Egypt

maximal at the second intercostal space (ICS), at the left


sternal border (LSB), and inferior to the clavicle. It is
typically a pansystolic murmur with bounding pulses and a
widened pulse pressure.There is also a paradoxical splitting
of S1 and S2. Echocardiography confirms the PDA, the
direction and degree of shunting, and the presence of lesions
for which the PDA is needed to keep. If there are no other
cardiac malformations requiring the PDA, then if the PDA is
large,surgery should be completed before 1 year of age.
Symptomatic PDAs that are relatively small may be closed
with indomethacin in preterm infants. The murmur heard in
atrial septal defect
(ASD) usually is an ejection type, systolic murmur heard
best at the LSB, second ICS with awide, fixed S2 and normal
pulses. Ventricular septal defect (VSD) presents with a
harsh,pansystolic murmur heard best at the third and fourth
ICS. With increasing size of the VSD,heaves, thrills, and lifts
are present along with radiation throughout the chest.
Tetralogy of Fallot
presents with a rough ejection, systolic murmur heard best
at the LSB and the third ICS with radiation to the back.

53. A 15-year-old boy suddenly collapses on the basketball


court; his sports physical conducted at the beginning of the
year did not elicit any abnormal findings. Basic life support
initiated at the scene, however, is unsuccessful in
resuscitation. Which of the following is the most likely
etiology of his sudden death?
(A) mitral valve prolapse
(B) surgically corrected aortic stenosis
(C) hypertrophic cardiomyopathy
(D) rheumatic heart disease
53. (C) Hypertrophic cardiomyopathy in adolescence is
typically due to familial hypertrophic cardiomyopathy with
Dr.Wahid Helmi - Dymiate -Egypt

an incidence of 1:500. Many patients are asymptomatic until


a sporting event, which may cause symptoms, specifically
sudden cardiac death. Examination may demonstrate a
palpable or audible S4, an LV (left ventricular) heave,
systolic ejection murmur
(may need to stimulate cardiac activity), and/or a left
precordial bulge. Echocardiography is the gold standard for
diagnosis but family history should be assessed. Stress
testing is indicated to assess for ischemia and arrhythmias.
Strenuous activities are prohibited for these patients. The
other cardiomyopathies (dilated and restrictive) are next but
are not as common. Congenital structural abnormalities of
the coronary arteries are the next most common cause.
Valvular disorders, including surgically repaired aortic
stenosis, are typically not causes of sudden death,but these
patients should be screened for symptoms and stress tested
as necessary.

54. A 7-year-old male child presents to the emergency


department with complaints of severe dyspnea, dysphagia,
drooling, muffled voice, and fever. The pulse oximetry is
91% on room air;lung examination shows stridor and
inspiratory retractions. Which of the following is the
expected chest X-ray finding for the suspected diagnosis?
(A) thumbprint sign
(B) Scottie dog sign
(C) steeple sign
(D) figure 3 sign
54. (A) This patient presentation describes epiglottitis.
Although there is a decreased incidence of epiglottitis
secondary to the introduction of the vaccine for
Haemophilus influenzae type B(Hib), patients still present
with sudden onset of fever, dysphagia, muffled voice,
drooling,cyanosis, inspiratory retractions, and soft stridor.
Dr.Wahid Helmi - Dymiate -Egypt

The patients are usually sitting in a tripod


position to aid their breathing. Recognition of the classic
symptoms needs to be immediate to stabilize the patient's
airway, as these patients will decompensate into respiratory
failure quickly. In the event that there is time, a lateral neck
X-ray will show the “thumb sign,” which is an
enlarged, undistinguished epiglottis. Treatment for the
patient requires intubation for airway stabilization, blood
cultures and throat/epiglottis cultures, and antibiotic
coverage for H.influenzae. The steeple sign is seen in patients
with croup and is due to a subglottic narrowing.The “figure
3” sign is seen in patients with coarctation of the aorta. The
“Scottie dog” sign is
seen in oblique lumbar films and is a normal finding
representing the pars interarticularis. Its absence signifies
spondylolysis.

55. Which of the following is one of the most common lethal


genetic disorders in the United States?
(A) trisomy 13
(B) trisomy 21
(C) cystic fibrosis
(D) neurofibromatosis
55. (C) With an incidence of 1:3,000 to 1:4,000 Caucasians,
cystic fibrosis is the most common lethal genetic disorder in
the United States. While trisomy 21 (Down syndrome) is one
of the most common genetic disorders with 1:500 newborns,
it is typically not a fatal disease. It is characterized with
mental retardation and physical malformations. Trisomy 13
is a fatal trisomy,
with most deaths occurring in early infancy or by the age of
2, but its incidence is approximately 1:12,000 live births.
Neurofibromatosis, a genetic disorder of typical autosomal
dominant inheritance, occurs in approximately 1:3,000 live
Dr.Wahid Helmi - Dymiate -Egypt

births. Most affected children have the skin lesions (café au


lait macules or neurofibromas) and other minor problems.

56. Which of the following is the initial treatment step in an


adolescent who presents to the emergency department with
status epilepticus?
(A) IV glucose
(B) stabilize airway
(C) arterial blood gas
(D) IV diazepam therapy

56. (B) Status epilepticus is a medical emergency and is


defined as seizure activity that lasts aminimum of 30
minutes. This results in hypoxia, acidosis, cerebral edema,
and structural damage.In addition, fever, respiratory
depression, hypotension, and death may occur. There are
both convulsive and nonconvulsive types of status
epilepticus. Because of its emergency status and potential
complications, the clinician needs to initiate the ABCs
(airway, breathing, circulation).
Therefore, the first line of treatment is to establish and
maintain an airway, oxygen is next, and then circulation,
which encompasses pulse, blood pressure, and IV access.
Once the IV is established, the orders should be for
administering glucose-containing fluids and IV drug therapy
with diazepam, lorazepam, or midazolam as well as
administer phenytoin and phenobarbital.Arterial blood
gases should be ordered and any abnormalities should be
corrected appropriately.Finally, the clinician should
determine the underlying cause: trauma, structural
Dr.Wahid Helmi - Dymiate -Egypt

disorder,infection, lactic acidosis, toxins, and uremia.


Maintenance drug therapy is necessary until the underlying
cause is determined and rectified.

57. A 12-year-old boy presents to the urgent care center


complaining of burning pain in his lower extremities with
weakness. On examination, the clinician notes symmetric
weakness with severely decreased active range of motion of
the lower extremities. In addition, there is decreased position
and vibratory sensation in the distal portions bilaterally.
Upon further questioning, the patient admits to being
diagnosed with mononucleosis 2 weeks ago. Which of the
following is the most likely diagnosis?
(A) poliomyelitis
(B) botulism
(C) Tick-bite paralysis
(D) Guillain–Barré syndrome
57. (D) Guillain–Barré syndrome is most likely due to a
delayed hypersensitivity with T-cell–mediated antibodies to
mycoplasma and viral infections (CMV, EBV, hepatitis B,
campylobacter jejuni). The patients may mention a
nonspecific respiratory or gastrointestinal infection 1 to 2
weeks prior to symptoms. Complaints may be paresthesias,
weakness in bilateral lower extremities with occasional
ascension into the arms, trunk, and face, and rarely ataxia
and ophthalmoplegia in the Miller–Fisher variant.
Examination findings demonstrate symmetric flaccid
weakness, with impairment of position, vibration, and touch
in the distal portions of the
extremities. If a spinal tap is performed, it may show few
polymorphonuclear neutrophils with high protein and
normal glucose. EMG is positive for decreased nerve
Dr.Wahid Helmi - Dymiate -Egypt

conduction. Laboratory tests may show high titers of


suspected infections or active infection of hepatitis/bacterial
pathogens. Guillain-Barré is normally a self-limiting
disorder within a few weeks, unless there are issues with
respiratory depression. Poliomyelitis is secondary to
polioviruses and presents with fever, paralysis, meningeal
signs, and asymmetrical weakness. Botulism secondary to
infection with Clostridium botulinum in older children
presents with blurred vision, diplopia,ptosis, choking, and
weakness. In infants, botulism presents as constipation, poor
suck and cry,apnea, lethargy, and choking. Tick-bite
paralysis presents with rapid onset with ascending flaccid
paralysis reaching upper extremities in a couple of days of
onset and patients often present with
paresthesia and pain. Finding of a tick is usually
confirmatory for these patients.

58. Which thoracic curvature is an indication for treatment


with bracing in an adolescent with scoliosis?
(A) less than 20°
(B) 20° to 40°
(C) 40° to 60°
(D) 40° with lumbar curvature of 30°
58. (B) Scoliosis is defined by lateral curvature of the spine
with rotation of vertebrae and is typically located in the
thoracic or lumbar spine in the right or left directions.
Idiopathic scoliosis most commonly presents as a right
thoracic curve in females from 8 to 10 years of age.Scoliosis
is typically asymptomatic unless curvatures are so severe
that there is pulmonary
dysfunction or there is an underlying disorder (bone or
spinal tumor) that is causing the scoliosis.X-rays need to be
taken of the entire spine to help determine the degree of
curvature. Treatment modalities are based on the degree of
Dr.Wahid Helmi - Dymiate -Egypt

curvature: 20° or less does not normally require treatment;


20° to 40° is an indication for bracing in an immature
child; and 40° and greater is resistant to bracing and
requires surgical fixation with spinal fusion, which is best
done at special centers.

59. A 6-year-old female child presents with complaints of


chronic hip pain so severe that she has not been able to walk
to the school bus. Examination shows severe tenderness at
the left hip with markedly decreased active and passive
range of motion. Radiologic examination demonstrates joint
effusion with widening. Which of the following is the most
likely diagnosis?
(A) osteochondritis dissecans
(B) slipped capital femoral epiphysis
(C) septic hip arthritis
(D) Legg–Calvé–Perthes disease
59. (D) Legg–Calvé–Perthes disease is also known as
avascular necrosis of the proximal femur. It typically occurs
in children between 4 and 8 years old and persistent hip pain
is the main symptom. On examination, the clinician notices a
limp and/or limitation of motion of the affected hip.
Radiologic examination demonstrates the necrosis with
effusion and joint space widening with a negative aspirate.
Treatment involves surgical hip replacement. Slipped capital
femoral epiphysis (SCFE) is due to the displacement of the
proximal femoral epiphysis owing to disruption of the
growth plate. The head is normally displaced medially and
posteriorly relative to the femoral neck. It typically occurs in
adolescence, specifically obese males, and can also be
associated with hypothyroidism. SCFE usually occurs after
direct trauma to the hip or a fall.Patients complain of vague
symptoms at first that progress into pain of the hip or of the
knee. On examination, there is decreased internal rotation of
Dr.Wahid Helmi - Dymiate -Egypt

the hip that can be confirmed by lateral X-ray of the hip.


Septic hip arthritis is not common in children between the
age of 5 and 12 years. The
legs are held in external rotation to minimize pain and will
have a positive aspirate.Osteochondritis dissecans typically
presents in the knee, elbow, and talus and is characterized
by a wedge-shaped necrosis of bone.

60. A 16-year-old boy presents to the office with thumb pain.


He just returned from a skiing trip. On examination, the
practitioner notes a positive ulnar collateral ligament laxity
test. What is the
most likely diagnosis?
(A) mallet finger
(B) gamekeeper thumb
(C) boxer fracture
(D) nondisplaced scaphoid fracture
60. (B) Gamekeeper thumb is a result of damage to the ulnar
collateral ligament during forced abduction of the
metacarpophalangeal joint, an injury that is most commonly
seen in skiers. An avulsed fragment may or may not be seen
on radiologic examination. If it is smaller than 2 mm,
there is no fragment, a thumb spica cast can be used as seen
in patients with no fragment. If the fragment is larger than 2
mm, surgery is required. Mallet finger is an avulsion of the
extensor tendon and occurs in ball-handling sports. Boxer
fracture is a distal neck fracture of the 5th metacarpal.
Scaphoid fractures are due to hyperextension of the wrist
injuries and present with
pain in the anatomic snuffbox and swelling.

61. A 3-year-old child is brought in by her parents to the


urgent care center stating that the child “will not bend her
arm.” They are obviously worried and distraught. The
Dr.Wahid Helmi - Dymiate -Egypt

clinician notices the elbow is held in strict pronation and


there is tenderness over the radial head. X-ray examination
shows no findings. Which of the following is the treatment of
choice for this disorder?
(A) place elbow in full supination and move from full extension
to full flexion
(B) immobilization of the elbow in a splint for 2 weeks
(C) referral to the orthopedic surgeon for suspected radial head
fracture
(D) call child protective services for suspected battery
61. (A) Nursemaid elbow is the subluxation of the radial
head due to a child or infant being lifted or pulled by the
hand. The patient will present with the elbow pronated and
painful and he or she will not bend the elbow. During the
radiologic examination, the dislocation is usually reduced by
placing the elbow in full supination and moving it slowly
from full extension to full flexion. This typically provides
immediate relief of pain and a sling may be given for
comfort for a couple of days. Otherwise, X-rays are normal.
Child protective services should be considered if this is
arecurrent problem or if there are other associated signs and
symptoms of battery. There is no
need for orthopedic referral unless reduction is not
commonly done in your setting.Immobilization of the elbow
is not recommended, because the patient then may have to
recover from frozen shoulder.

62. A 7-year-old child is brought into the office by her


mother who states that the child “is still wetting the bed at
night.” The child has already decreased liquid intake and
uses the bathroom before going to bed. The mother is
worried that there is something wrong with the child. Upon
examination there is no abnormality. Urinalysis is negative.
Dr.Wahid Helmi - Dymiate -Egypt

Which of the following is the treatment of choice for this


disorder?
(A) bed-wetting alarm
(B) desmopressin acetate (DDAVP)
(C) imipramine
(D) amitriptyline
62. (A) This patient is presenting with signs and symptoms of
primary nocturnal enuresis, which is the wetting only at
night during sleep without any sustained period of dryness.
It is mainly considered a parasomnia occurring in deep
sleep. The incidence of enuresis is higher in boys, is typically
related to a developmental delay, and most children become
continent by adolescence.
Patients need to be tested for structural abnormalities and
infections, in addition to neurologic diseases, diabetes
mellitus and insipidus, and seizure disorders. Treatment
includes limiting liquids at bedtime and routine bathroom
training during the day. If these are unsuccessful, the next
option is a bed-wetting alarm. This device is attached to the
child's undergarment and vibrates when the child is wet to
arouse the child to be aware of their need to urinate. If the
alarm is unsuccessful, then the next step is medication—
DDAVP (desmopressin acetate) or imipramine.

63. A 13-year-old boy presents with fever and blood in his


urine. Examination shows an asymptomatic mass in the left
lower quadrant. Urinalysis shows hematuria and small
leukocytes.Which of the following is the most likely
diagnosis?
(A) renal cell carcinoma
(B) intussusception
(C) volvulus
(D) nephroblastoma
63. (D) Nephroblastoma also known as Wilms tumor
Dr.Wahid Helmi - Dymiate -Egypt

typically presents with an asymptomatic abdominal mass


noticed by the parent or an increasing size of the abdomen.
On examination, the mass feels smooth and firm, is well
defined, and usually does not cross the midline. Gross
hematuria may be present, but rare, and some patients have
microscopic hematuria when tested.
Wilms tumor accounts for approximately 5% of cancers in
children younger than 15 years. Wilms tumor arises from
the kidney and the average age at diagnosis is 4 years.
Ultrasound and CT of the abdomen can be used to confirm
the presence of an intra-abdominal mass. Treatment
includes
exploratory abdominal surgery for removal and staging with
a mixture of chemotherapy.Intussusception (telescoping of
the small intestine) typically presents in an infant with
paroxysmal abdominal pain, vomiting, and diarrhea that
may progress into bloody stools.Volvulus is normally the
result of intestinal malrotation that causes occlusion of the
superior
mesenteric artery and eventual bowel necrosis. Infants
typically present within 3 weeks of life with bile-stained
vomiting and bowel obstruction.

64. A mother brings in her 20-month-old female child to the


office because she noticed pubic hair growing. On
examination, the clinician notices that the clitoris is
enlarged; the rest is unremarkable. Which of the following is
an expected laboratory finding on this patient?
(A) increased aldosterone
(B) increased estrogen
(C) increased androstenedione
(D) increased luteinizing hormone
64. (C) Infant girls presenting with signs of precocious
Dr.Wahid Helmi - Dymiate -Egypt

puberty need to be screened for congenital adrenal


hyperplasia (CAH). CAH most commonly presents with
pseudohermaphroditism in females—urogenital sinus,
enlarged clitoris, or other signs of virilization. In males,
there tends to
be isosexual precocity in older males and salt-losing crisis in
infant males. Both children show increased linear growth
and skeletal maturation. The most common type of CAH is a
deficiency in the enzyme 21-hydroxylase and laboratory tests
demonstrate increased urinary and plasma
androgens (DHEA, androstenedione). There may be elevated
progesterone, but typically there is no effect on estrogen.
There is also decreased aldosterone and elevated urinary
ketosteroids.There is also no effect on the levels of
leuteinizing hormone or follicle-stimulating
hormone.Treatment usually involves glucocorticoids,
mineralocorticoids, and reconstructive surgery, if
needed.

65. A 5-year-old child presents for her kindergarten


checkup. The clinician notes that over the past couple of
years, her height decreased from the 50th percentile to the
5th percentile. On examination, the clinician also notes
truncal adiposity. Her CBC and lead levels were
normal.Which of the following is the most likely diagnosis?
(A) growth hormone deficiency
(B) Cushing disease
(C) congenital hypothyroidism
Dr.Wahid Helmi - Dymiate -Egypt

(D) congenital adrenal hyperplasia


65. (A) Growth hormone (GH) deficiency is defined as a
decreased growth velocity, delay in skeletal maturation,
absence of other explanations for poor growth (lack of
intake), and laboratory tests demonstrating decreased GH
secretion. Etiology of GH deficiency can be congenital,
genetic, acquired, or idiopathic, which is the most common.
Infants usually have a
normal birth weight and may have a slightly decreased
length. In addition, most infants present with other
endocrine deficiencies like hypoglycemia, hypothyroidism,
and/or adrenal insufficiency. Children may present with
truncal adiposity because growth hormone promotes
lipolysis. Serum GH or intrinsic growth factor levels may or
may not be decreased. In patients who do not have a
demonstrated decrease in these hormones, a trial period
with GH is indicated.These patients and positive GH-
deficient patients receive a once-daily subcutaneous injection
of
recombinant human GH. Congenital hypothyroidism
typically presents with short stature (typically noted after
the 4-month newborn visit), delayed epiphyseal
development, delayed closure of fontanelles, and retarded
dental eruption in addition to other signs of
hypothyroidism.Cushing disease typically presents with
truncal adiposity with thin extremities, muscle
wasting,decreased growth rate, and moon facies. Laboratory
results show elevated adrenocorticosteroids both in urine
and serum, hypokalemia, eosinopenia, and lymphocytopenia.
Typically, in patients
younger than the age of 12, Cushing disease is secondary to
administration of ACTH or glucocorticoids. Congenital
adrenal hyperplasia typically presents with
pseudohermaphroditism in females or salt-losing crisis in
males with or without isosexual precocity. There is an
Dr.Wahid Helmi - Dymiate -Egypt

increased linear growth and advanced skeletal maturation.

66. An Rh-negative, 5-year-old male child presents with


acute onset of petechiae and purpura after an acute viral
illness. In addition, he has episodes of epistaxis. Which of the
following is atreatment option if his platelet count falls below
20,000/mm3, but he is not actively bleeding?
(A) platelet transfusions
(B) IV anti-D (WinRho SD) 50–70 mg/kg/dose
(C) prednisone 2.4 mg/kg/24 hours × 2 weeks
(D) splenectomy
66. (C) In patients with idiopathic thrombocytopenic
purpura, treatment options should be initiated when platelet
counts fall below 20,000, regardless of whether there is
active bleeding or not.Without active bleeding the treatment
options include prednisone 2–4 mg/kg/24 hours for 2
weeks; IV immunoglobulin 1 g/kg/24 hours for 1 to 2 days,
or IV anti-D 50–75 μg/kg/dose for Rh-positive patients.
Splenectomy is indicated for life-threatening bleeding. There
is currently no indication for platelet transfusion and none
of the above treatments are considered optimal,
because in the majority of children, it will resolve on its own
within 6 months.
Dr.Wahid Helmi - Dymiate -Egypt

67. A 9-year-old female child presents with tachycardia,


tachypnea, shortness of breath, bibasilar rales, and
distended jugular veins. Which of the following is the least
likely cause for her signs and symptoms?
(A) rheumatic heart disease
(B) sickle cell anemia
(C) viral myocarditis
(D) patent ductus arteriosus
67. (D) This patient is presenting with signs of congestive
heart failure. The most common causes of heart failure in
children/adolescents are due to acquired heart disease.
Congenital heart diseases,such as malformations of the
heart— patent ductus arteriosus and ventricular septal
defects, are
the most common causes of heart failure in infants–toddlers,
and are second to fluid overload in
neonates.

68. A 12 year-old girl patient was treated for a urinary tract


infection 3 days ago. She presents today with severe
conjunctivitis, target lesions on her trunk, and bullous
eruptions in her mouth. Which of the following medications
is the likely cause of her symptoms?
(A) ciprofloxacin
Dr.Wahid Helmi - Dymiate -Egypt

(B) erythromycin
(C) amoxicillin
(D) trimethoprim-sulfamethoxazole (TMP-SMX)
68. (D) This patient has the classic presentation of erythema
multiforme major or Stevens–Johnson syndrome. The most
common causes in children of erythema multiforme are
medications and Mycoplasma pneumoniae. Of the
antibiotics listed, the one most commonly causing Stevens–
Johnson syndrome is sulfonamide followed by penicillin and
tetracycline. The most common
medications causing SJS in children are nonsteroidal anti-
inflammatory drugs.

69. Which of the following is a complication of infection with


Parvovirus B19?
(A) aplastic crisis
(B) leukopenia
(C) aseptic meningitis
(D) congenital defects (if mother contracts during pregnancy)
69. (A) Infection with Human parvovirus B19 (also known as
fifth disease) resulting in the slapped cheek appearance, can
also cause aplastic anemia. This is because the virus infects
the precursors of erythrocytes and halts erythropoiesis.
Recovery is typically spontaneous with an occasional
transfusion for severe anemias.

70. A 9-year-old male child presents with a painful rash of


his upper extremity. His mom states it started 4 days ago
and seems like it is spreading. Physical examination
demonstrates a vesicular rash across the right upper arm
and chest but does not cross the midline. Which of the
following prescriptions would be most appropriate for this
patient at today's visit?
(A) hydration
Dr.Wahid Helmi - Dymiate -Egypt

(B) nonsteroidal anti-inflammatory drugs (NSAIDs)


(C) Varicella-Zoster immunoglobulin (VZIG)
(D) oral acyclovir
70. (D) As this patient is presenting with signs and symptoms
of herpes zoster within the appropriate time frame for
antiviral treatment, the treatment for this patient would be
oral acyclovir. NSAIDs may help with the pain associated
from zoster but will not hasten the length of the course of the
virus as acyclovir will. Varicella-Zoster immunoglobulin
(VZIG) is indicated for prophylaxis in exposed individuals
who are immunocompromised.

71. Which of the following findings would suggest a specific


child abuse diagnosis of Munchausen syndrome by proxy?
(A) fractures in various stages of healing
(B) retinal hemorrhages
(C) head or abdominal trauma
(D) recurrent polymicrobial sepsis
71. (D) Munchausen syndrome by proxy is when the
parent/caregiver is causing or complaining of signs and
symptoms of illnesses in his/her children. While it is a form
of child abuse and should be treated as such, it is also
considered a psychiatric disorder where the
parent/caregiver is
desiring to be in the sick role. The most common signs or
symptoms that should raise the level of suspicion for
Munchausen syndrome by proxy are: recurrent
polymicrobial sepsis, recurrent apnea, chronic dehydration,
or other unexplained symptoms like vomiting, diarrhea,
seizures,
failure to thrive, and hypoglycemia. The remaining signs are
seen in classical physical child
abuse.
Dr.Wahid Helmi - Dymiate -Egypt

72. Which of the following is a contraindication for the


meningococcal (Menomune) vaccine?
(A) history of Guillain-Barré
(B) complement deficiency
(C) college freshman in dormitories
(D) persons with functional asplenia
72. (A) Menomune is a tetravalent vaccine that is indicated
for prevention of meningococcemia caused by the bacterium
Neisseria meningitides. Menomune is indicated for patients
between 11and 12 years of age and at 15 years of age. It is
also indicated for college freshmen in dormitories, military
recruits, microbiologists working with the bacterium,
persons with complement deficiency and functional or
anatomic asplenia, and for those traveling to countries with
endemic disease. Guillain–Barré is a rare complication of the
Menomune vaccine, and if apatient has a history of
developing it, is the only relative contraindication other than
a known reaction to a previous administration of the vaccine
rubber latex and diphtheria toxoid severe
allergic reaction

73. A 7-year-old Caucasian female child presents to the


office with “an itchy head.” The child's mother, who is
with her, states that this has been bothering her daughter for
about a week and she has noticed a lot of “dandruff” in
the child's hair that will not come out. She also mentions
that several of her daughter's friends are having the same
Dr.Wahid Helmi - Dymiate -Egypt

problem. On the basis of the most likely diagnosis, what is


the best treatment for this patient?
(A) permethrin 1% shampoo
(B) ketoconazole cream
(C) tar-based shampoo
(D) silver sulfadiazine 1% cream
73. (A) The most likely diagnosis is pediculosis. This
parasitic infestation is most commonly seen in the young
school-aged child, and more often in female and Caucasian
children. The pediculosis louse lives in the hair and on the
scalp and intermittently “bites” into the skin to feed.
Discrete
urticarial papules or erosions may arise at the bite site. By
visualizing the live louse on the scalp,or in the hair, one can
easily make the diagnosis. However, the louse may be
difficult to see, as it is only 1 to 3 mm in size. Otherwise, nits,
or the casings of the eggs laid by the louse, can often be seen
on the proximal portion of the hair shaft. The nit adheres to
the hair shaft and is often
difficult to remove. Brown nits are representative of current
infestations and white nits past infestations. Treatment of
head lice can be difficult due to the increasing resistance to
some of the current treatment options. First-line treatment
includes permethrin (5%) and permethrin-based products.
Secondary treatment options for resistant infestations may
include Malathion (0.5%).Regardless of treatment, viable
ova should be removed by combing the patient's wetted hair
with a finely toothed comb until all are removed.
Ketoconazole cream and tar-based shampoos are utilized in
fungal and seborrheic dermatitis infections. Silver
sulfadiazine cream is a topical
antibiotic.
Dr.Wahid Helmi - Dymiate -Egypt

74. A young mother brings her 4-year-old son to the clinic


for evaluation of a rash on his umbilicus and hands. She has
been treating it with an over-the-counter ointment for about
a week, without success. She says that she has noticed that he
scratches the rash periodically, and it seems to bother him
the most at night. She also says that she noticed this same
rash on the hands on one of the other boys at his daycare
center. On examination, there are excoriated papules and
nodules on his hand and umbilicus. What is the most likely
diagnosis?
(A) herpes simplex
(B) scabies
(C) pediculosis
(D) tinea corporis

74. (B) Scabies, Sarcoptes scabiei, is the most common


arthropod infestation of children, and it is highly contagious.
However, its presentation varies widely and is dependent on
the child's age,
duration of the infestation, and immune status. Most often,
the presenting complaint is severe intermittent itching. The
linear papule or burrow commonly associated with scabies is
often difficult to identify. Instead, most children will present
with eczematous eruptions of red,
excoriated papules and nodules. Usually, the distribution of
the papules are the most diagnostic finding, and may include
the web spaces of the fingers and toes, axillae, umbilicus,
groin, penis,and the instep of the feet. Usually, in older
children and adults, the face and scalp are spared. The
treatment for scabies is a 12-hour application of permethrin
5% lotion. In addition, the parents
and all caregivers should be treated at the same time.
Clothing and bedding should be washed and dried (heat kills
scabies). The family should also be educated in the treatment
Dr.Wahid Helmi - Dymiate -Egypt

and prevention of future infestations. Moreover, they should


be advised that the itching associated with scabies
could persist for 7 to 14 days after successful treatment.
Pediculosis is an infestation of louse in the hair. Tinea
corporis is a fungal infection of the torso or “ring worm”
and presents with annual scaly plaques with central clearing
and pustules. Herpes simplex typically presents with
grouped
vesicles on erythematous base and is painful. It typically is
located in the lips, eyes, cheeks, or
hands of children.

75. A new mother brings her 3-month-old daughter to the


clinic for a rash on the infant's head. On examination, the
skin affected by the rash is thickened, yellowish white in
color, scaly, and looks waxy. In addition, it involves only the
scalp and bilateral postauricular areas. What is the most
likely diagnosis?
(A) contact dermatitis
(B) lichen planus
(C) pityriasis rosea
(D) seborrheic dermatitis
75. (D) Seborrheic dermatitis is common in all age groups.
In infants, this inflammatory skin disease is often manifested
as thickened, yellowish white, scaly, waxy appearing skin of
the scalp and commonly involves the postauricular areas
and the forehead. The more common name is “cradle
cap.” Cradle cap is a self-limiting disease of infants and
resolves by the child's first birthday. In all ages, the scalp
scale can be treated by shampooing with zinc pyrithione
(Head and Shoulders), selenium sulfide 1% to 2.5%, salicylic
acid (Tsal), or ketoconazole (Nizoral). The primary lesion in
lichen planus presents on the flexor surfaces and is
characterized by pruritic
Dr.Wahid Helmi - Dymiate -Egypt

papules that are polygonal and flat-topped. Pityriasis rosea


typically presents with the “herald
patch” that is a solitary pink, round patch with some
central clearing typically found on the torso. The rest of the
eruption is described as papulovesicular and develops a
Christmas tree pattern.
Contact dermatitis usually presents with red patches and
plaques with scales and is localized to the area exposed to the
irritant.

76. An 8-year-old male child presents with brown,


nonpruritic, annular lesions on the back of his hands and
feet. Intradermal nodules are seen on the extensor surfaces
of the elbows and knees that have been present for several
months. At today's visit, the lesions are essentially
unchanged since his last visit about a month ago. What is the
best treatment for this suspected disorder?
(A) excision and biopsy
(B) no treatment
(C) topical steroids
(D) wet to dry dressings
76. (B) This presentation is typical for granuloma annulare,
which is a benign skin disorder, and treatment is not
warranted. It is most commonly seen in children aged 6 to
10. The red to brown lesions are annular or circinate. These
asymptomatic lesions are often confused with tinea corporis.
The lesions will disappear on their own over a couple of
years.

77. Which of the following is the recommended treatment of


a 2- to 5-cm, single, nonpainful, common wart on the hand of
a 7-year-old?
(A) 40% salicylic acid plaster
(B) burning laser surgery
Dr.Wahid Helmi - Dymiate -Egypt

(C) electrocautery
(D) liquid nitrogen
77. (D) Liquid nitrogen is the treatment of choice for a single
isolated wart. Forty percent salicylic acid in a plaster
application is the most effective treatment of large and
painful warts.Electrosurgery, burning laser surgery, and
other destructive treatments should be avoided because of
the potential for scarring and subsequent problems often
associated with scars, as
well as the possible recurrence of the wart after destructive
treatment.

78. In treating uncomplicated, comedonal acne (open and


closed comedones) in adolescents, which of the following
treatments is best?
(A) topical antibiotics
(B) topical keratolytics
(C) oral retinoids
(D) systemic antibiotics
78. (B) Topical keratolytic agents applied to the skin either
as a single, once a day agent or in combination regime
(retinoic acid cream, azelaic acid, and adapalene) once a day
in the evening and benzoyl peroxide gel in the morning, will
control approximately 80% to 85% of cases of adolescent
acne. When treating inflammatory acne, papular or
pustular, a daily topical antibiotic
such as tetracycline, minocycline, or erythromycin can be
used in addition to a daily keratolytic.The oral retinoid, 13-
cis-retinoic acid (isotretinoin), Accutane is reserved for
treating nodulocystic acne (severe cystic acne). This
medication is not effective for the milder forms of acne such
as comedonal. Isotretinoin is teratogenic in women of
childbearing age and has other side effects. Therefore, strict
adherence to FDA guidelines is required
Dr.Wahid Helmi - Dymiate -Egypt

79. The most common fracture of newborns is a fracture of


the
(A) clavicle
(B) humerus
(C) radius
(D) ulna
79. (A) Clavicular factures are the most common fractures
in infants and children. In newborns, this fracture is usually
unilateral and often occurs after a difficult delivery. Many
times no treatment is required or a figure-of-eight bandage
can be used. For infants and children, a sling can be
used. The bump that can be seen after fracture consolidation
will usually resolve in a few months to a year. The next most
common fractures are of the extremities, humerus being the
most common and then the femur, but still much less
common than the clavicle.

80. A 13-year-old boy presents to the clinic for a complaint


of right knee pain that he first noticed about a year ago. It
started out as mild discomfort in the area just below the
kneecap, but has been getting progressively worse. Now, it
hurts anytime he uses his leg, even when walking. He does
not remember any injury to his knee. When you examine his
knee, you notice swelling and exquisite tenderness over the
tibial tubercle. X-rays are normal. What is the most likely
diagnosis?
(A) chondromalacia patellae
(B) Osgood–Schlatter disease
(C) patellar dislocation
(D) patellofemoral overuse syndrome
80. (B) Osgood–Schlatter disease is caused by microfractures
of the patellar ligament where it inserts into the tibial
tubercle. This condition usually occurs in the preteen and
Dr.Wahid Helmi - Dymiate -Egypt

adolescent years,and is more common in males than females.


The history of injury can be vague and the patient may not
remember a specific injury that precipitated the pain. Often,
the pain progresses to the point of interference of even
routine physical activities. X-rays may or may not show any
abnormalities. Upon X-ray, Type I disease appears normal,
but Type II will reveal fragmentation of the tibial tubercle.
Often, after healing there will be enlargement of the tibial
tubercle.
Generally, treatment consists of rest, limitation of activities,
and isometric exercises.Chondromalacia patellae can only be
diagnosed under an arthroscopic examination, not on the
basis of clinical features. Patellofemoral overuse syndrome
presents with medial knee pain and subpatellar pain.
Additional signs are swelling and crepitus in the knee and it
is more common in females than males. It is diagnosed by
increased Q-angles (anterosuperior iliac spine through
center of patella to tibial tubercle). Subluxation of the
patella or dislocation is more common in adolescent girls
and the patient presents with acute knee pain. The knee is in
flexion with a mass lateral to the knee and with absence of
the bony prominence of the patella (flat). X-ray confirms the
dislocation.

81. The eggs of this parasite are detected by microscopic


examination of clear adhesive tape that has been pressed to
the child's anus in the morning, prior to bathing. What
parasite is most likely to be identified by this test method?
(A) Ancylostoma duodenale (hookworm)
(B) Ascaris lumbricoides (ascaris)
(C) enterobiasis (pinworm)
(D) trichuriasis (whipworm)
81. (C) Enterobiasis or pinworms is a worldwide infection
that affects people of all ages and socioeconomic levels. It
Dr.Wahid Helmi - Dymiate -Egypt

especially affects children. The classic manifestation of this


problem is nocturnal anal pruritis and sleeplessness. The
sleeplessness may be secondary to the migration of female
worms to the perianal area to lay eggs, during which the
tape may pick up the larvae.
Transmission of the worms occurs when children ingest the
eggs that are present on their hands (from scratching), in the
bedclothes, or in house dust. After hatching in the stomach,
the larvae migrate to the cecum where they mature into
adults. The treatment of choice for pinworms is pyrantel
pamoate or mebendazole. Albendazole may also be used. For
eradication of this
parasite, often the entire family must be treated at once.
Ascaris is a helminthiasis infection that is ingested and
excreted in the stool. Diagnosis is made by stool examination
for the characteristic eggs. Hookworms are found in warm,
damp soil and penetrate the skin. From there the infection
can spread to the lungs where they ascend into the trachea to
be swallowed and live in the intestine. Diagnosis is made by
stool examination for the eggs. Whipworm is ingested from
the soil and lives in the intestine; detection is also made by
egg in the feces.

82. A 13-year-old boy presents with complaints of pain in


both knees and his right ankle. The pain is worse in the
morning. He denies any injuries, but does notice he tires
more easily when playing baseball. He says this has been
going on for about 8 weeks. His father admits to having
Dr.Wahid Helmi - Dymiate -Egypt

chronic low back pain, but otherwise the family medical


history is noncontributory. On the basis of this history,
which of the following is the most likely diagnosis?
(A) juvenile idiopathic arthritis
(B) Lyme arthritis
(C) psoriatic arthritis
(D) enteropathic arthritis
82. (A) Juvenile idiopathic arthritis (JIA) presents as three
distinct types. The types are based upon clinical
manifestations during the first 6 months of the illness. The
most common type is pauciarticular as presented by this 13-
year-old boy in the scenario mentioned. Second is
polyarticular disease with five or more joints being affected,
and the third is systemic onset of disease that begins with
high spiking fevers that are often associated with a rash that
comes and goes with the fever elevations. It is recommended
that patients with pauciarticular JIA have an
ophthalmologic evaluation and slit lamp examination every 3
months, if the antinuclear antibody
test (ANA) is positive and every 6 months, if the ANA is
negative, for 4 years after the JIA is identified to catch
iridocyclitis (untreated results in blindness). Lyme arthritis
usually presents with a monoarticular rash that typically
affects the larger joints, without morning
stiffness.Enteropathic arthritis is associated with
gastrointestinal symptoms occurring simultaneously as
lower extremity arthritis. It encompasses Reiter syndrome,
reactive arthritis (eg, postsalmonella,shigella), and arthritis
associated with celiac disease and inflammatory bowel
disease. Psoriatic arthritis is the arthritis accompanying the
dermatological disorder of psoriasis.The build-up of
epidermal cells over the joints causes inflammation and
thickening that results in arthralgia.
Dr.Wahid Helmi - Dymiate -Egypt

83. An 8-year-old female child presents with complaints of a


red itchy right eye with a lot of yellowish green color
discharge for 3 days. She denies any injury. Her visual
acuity is normal but she does have moderate tearing and
mild photophobia. What is the most likely diagnosis?
(A) allergic conjunctivitis
(B) bacterial conjunctivitis
(C) viral conjunctivitis
(D) reactive arthritis/Reiter syndrome
83. (B) Bacterial conjunctivitis is often unilateral and
presents with a mucopurulent discharge.Common bacterial
causes of this problem include nontypable Haemophilus,
Streptococcus pneumoniae, Moraxella catarrhalis, and
Staphylococcus aureus. These infections usually respond to
topical antibiotics such as sulfacetamide and erythromycin.
Systemic treatment is
indicated for conjunctivitis caused by chlamydia
trachomatis, Neisseria gonorrhea, or Neisseria meningitides.
Allergic conjunctivitis is usually associated with moderate to
severe itching of the eyes and clear mucoid drainage. Viral
conjunctivitis is usually associated with minimal itching,
profuse tearing, and minimal clear mucoid drainage. While
reactive arthritis
typically presents with a conjunctivitis, it is also
concomitantly present with arthritis and urethritis.

84. While seeing a 12-week-old baby girl for her well-child


checkup, it is noticed that she has tearing from her left eye.
There is a small reddened area that is swollen and she cries
when it is touched. The swollen area is just below the medial
inferior eyelid. There is also constant tearing from this same
eye. Her mother says it just started about 2 days ago and is
getting worse. What is the most likely cause of this problem?
(A) blepharitis
(B) conjunctivitis
Dr.Wahid Helmi - Dymiate -Egypt

(C) dacryocystitis
(D) anterior uveitis
84. (C) Dacryocystitis, whether acute or chronic, is usually
secondary to bacterial infections. It presents as an acutely
inflamed swelling and tender area over the lacrimal sac just
medial and inferior to the inner canthus of the eye. Because
the lacrimal sac is inflamed and blocked there is tearing and
usually purulent discharge from the eye. There may also be
an orbital cellulitis.Treatment consists of oral and topical
antibiotics and warm compresses, and surgical drainage
may also be indicated. After the acute episode and for
chronic cases, surgical correction of the nasolacrimal
obstruction is required. Anterior uveitis typically presents
with pain, photophobia,blurred vision, and injection without
exudates. Blepharitis is an inflammation of the lid margin
that presents with crusty debris along the lashes. Unless
there is a concomitant conjunctival infection there is
typically no injection noted.

85. The majority of cases of halitosis in young children can


be traced to which of the following causes?
(A) dental caries
(B) nasal foreign body
(C) poor dietary habits
(D) upper respiratory tract infection
85. (B) While halitosis can be caused by pharyngitis,
sinusitis, and poor hygiene, the most common cause of
halitosis in children is a nasal foreign body. Seeds and beads
are the leading objects inserted into the nose. If not
promptly removed, they can cause nasal obstruction,
infection,
rhinorrhea, bleeding, halitosis, or a foul smell. They are
usually easy to remove, but if there is difficulty in removing
the foreign body, the child should be referred to an
Dr.Wahid Helmi - Dymiate -Egypt

otolaryngologist for definitive care. Tobacco use in


adolescents is a common cause of halitosis. Dental disease is
the most common cause of halitosis in adults.

86. Of the following, which is the most frequent cause of


epistaxis in children?
(A) bleeding disorders
(B) choanal atresia
(C) digital trauma
(D) foreign bodies
86. (C) Most cases of epistaxis in the anterior portion of the
nose are caused by digital trauma (nose
picking) or some other mechanical cause such as nose
blowing or repeated nose rubbing. Other
causes may include incorrect use of steroid nasal sprays.
Examination of the anterior nose will
usually reveal irritation of the Kiesselbach area. Less than
5% of recurrent nosebleeds are
caused by bleeding disorders. Choanal atresia, unilateral,
usually appears as a chronic nasal
discharge that may be mistaken for chronic sinusitis.
Foreign bodies typically present with
purulent discharge instead of bleeding.

87. A 5-year-old male child in the clinic is being evaluated


for a firm, painful lump that is slightly reddened and
approximately 3 cm in diameter, in his right axilla. His
mother tells you the lump has been there for a couple of
days. The boy does not look acutely ill. The mother informs
you that they got a new kitten and puppy about a month ago
but otherwise nothing else is new at home. Which of the
following is the most likely etiology for his rash?
(A) Bartonella henselae
(B) parvovirus
Dr.Wahid Helmi - Dymiate -Egypt

(C) Hodgkin disease


(D) Osgood–Schlatter disease
87. (A) Cat scratch disease (CSD) is caused by the gram-
negative bacillus, Bartonella henselae.The disease is more
common in the fall and winter months and more males than
females are affected. Typically (approximately 90%),
patients report handling a cat or kitten and up to 70%
will report a scratch by a cat. The most common
complication of CSD is encephalitis. About half of the
patients with CSD will develop a primary cutaneous papule
at the site of inoculation, most often (approximately 50%) on
the hands or upper extremities, 3 to 10 days after the
exposure.
Regional lymphadenopathy will usually develop in about 1
to 7 weeks after the cutaneous lesions
and will affect the nodes draining the site of the scratch or
bite. The affected lymph nodes may be inflamed and are
usually tender. Occasionally, the involved nodes may
suppurate. The lymphadenopathy resolves in about 2
months, but may last as long as 4 to 8 months. Treatment is
usually not indicated for this self-resolving disease. However,
suppurative lesions may need to be aspirated for pain relief.
It has been shown that 5 days of treatment with
azithromycin has helped to speed recovery for some patients.
Because Hodgkin disease involves the lymph nodes,it should
be considered as a differential diagnosis when evaluating a
child for CSD. However, it
typically presents as a cervical lymphadenopathy. Fifth
disease (erythema infectiosum) is achildhood disease caused
by the human parvovirus. This common community-
acquired disease does not usually require treatment, but
respiratory isolation is recommended for 7 days following
the onset of symptoms. The initial stage of the disease
presents as red cheeks that appear to be “slapped” or
Dr.Wahid Helmi - Dymiate -Egypt

“slapped cheeks” with circumoral pallor. Osgood–


Schlatter disease is an orthopedic problem in children. It is
the result of repetitive microtraumas to the patellar ligament
at its point of insertion into the tibial tubercle. Usually, rest
and anti-inflammatory medications
are helpful in alleviating the pain associated with this
condition.

88. When considering infections caused by nematodes, which


of the following is most consistent with iron deficiency
anemia, abdominal discomfort, weight loss, and the presence
of ova in the
feces?
(A) ascariasis
(B) hookworm
(C) pinworms
(D) whipworm
88. (B) Hookworm (Ancylostoma duodenale and Necator
americanus) infections, if severe, can cause iron deficiency
anemia. Abdominal discomfort, weight loss, and ova in the
stool are more commonly associated with these nematodes.
Both types of human hookworms are found in tropic
and subtropical climates, which include the southeastern
United States, primarily the coastal areas. The larva of this
parasite is passed in the feces and incubates in warm, damp
soil when they hatch into larvae. The larvae penetrate
directly into the skin of humans, enter the bloodstream, and
migrate to the lungs. From the lungs they move up to the
trachea and are
swallowed. Once swallowed, they mature in the intestines.
The worms attach their mouth to the mucosal lining of the
intestine where they suck blood and shed new ova. Mild
infections are usually asymptomatic, but severe infestations
can cause anemia. Treatment for the infestation is achieved
Dr.Wahid Helmi - Dymiate -Egypt

with albendazole. In severe cases of anemia, parenteral iron


or transfusion may be
indicated. Pinworms are associated only with localized
pruritus, specifically the anus. Treatment may help
recurrent urinary tract infections in some young girls when
the pinworm has infected the urethra. Ascariasis is usually
asymptomatic; however, in severe cases it may be associated
with
anorexia, diarrhea, vomiting, weight loss, and abdominal
pain. Whipworm is also asymptomatic until the infection is
severe, with general gastrointestinal symptoms—pain,
diarrhea, and mild abdominal distention. Eosinophilia may
also be present, although slight.

89. Erythema migrans, the characteristic rash of Lyme


disease, occurs in what percent of patients with this disease?
(A) 20% to 40%
(B) 40% to 60%
(C) 60% to 80%
(D) 80% to 100%
89. (C) Appearing in 60% to 80% of cases, the characteristic
rash may not be present in all cases of acute Lyme disease.
Following the bite of a deer tick (Ixodes species), infected
with the spirochete Borrelia burgdorferi, an erythematous
ring forms around the bite site and spreads outward. The
ring may have a raised border and usually a clear center.
The ring can attain a
diameter of up to 20 cm. Multiple rings may form and they
can form at sites distal to the original bite site. If left
untreated, the rash will usually resolve within 3 weeks.
Erythema migrans is aminimally tender to nontender,
nonscaly rash that persists longer than many of the other
erythematous rashes of childhood.
Dr.Wahid Helmi - Dymiate -Egypt

90. A 2-week-old male infant is being seen in the clinic for a


profuse mucoid discharge from both eyes, with some
associated tearing. On examination, you notice both eyes are
hyperemic and the eyelids are red and swollen. Which of the
following is the most likely cause of this patient's ophthalmia
neonatorum (conjunctivitis in the newborn)?
(A) allergic
(B) gonococcal
(C) chlamydial
(D) viral
90. (C) Chlamydial infections are the most common cause of
conjunctivitis in newborns in developed countries. Other
causes of ophthalmia neonatorum include reactions to silver
nitrate prophylaxis, other bacterial infections such as
gonococcal or staphylococcal, or viral organisms such as
adenovirus or echovirus. Chlamydia trachomatis causes
conjunctivitis and pneumonia in
neonates. Treatment for chlamydial conjunctivitis should be
with systemic erythromycin to treat the conjunctivitis and as
prophylaxis against pneumonia.

91. Which of the following neurologic disorders is least likely


to be associated with Lyme disease?
(A) aseptic meningitis
(B) Bell palsy
(C) polyradiculitis
(D) seizures
91. (D) Seizures have not been associated with Lyme disease.
Neurologic manifestations occur in up to approximately
20% of patients with Lyme disease. Primarily, these are Bell
palsy,lymphocytic, aseptic meningitis, and polyradiculitis.
Dr.Wahid Helmi - Dymiate -Egypt

Cranial neuropathies, such as Guillain–Barré syndrome and


ataxias are less common. Additional neurologic
manifestations include
peripheral neuropathy, pseudotumor cerebri, and
encephalitis. If untreated, most neurological symptoms are
self-limited but some will persist or become permanent.

92. A young mother brings her 3-week-old daughter for care


of a rash in her mouth. The mother indicates the baby was
doing fine until 2 days ago when she noticed white spots in
the infant's mouth. On examination, they do not come off
easily with a tongue blade. She is bottle-feeding the infant
without any problem. Which of the following is the most
likely diagnosis of this problem?
(A) leukoplakia
(B) hand–foot–mouth disease
(C) herpangina
(D) oral candidiasis
92. (D) Oral candidiasis (thrush) is very common in the first
few weeks of infancy. The diagnosis is usually done by visual
inspection and does not usually require further laboratory
testing. On visual examination, white, creamy plaques are
found on the buccal mucosa and occasionally the
gingival and lingual mucosa. For this age group, direct
topical application of nystatin in oral suspension to the
lesions should suffice. If the lesions are resistant to treatment
or if they occur in older children, consideration should be
given to the possibility of the patient being
immunocompromised. All sources of candida, such as toys
and bottle nipples, should be
sterilized daily. Herpangina and hand–foot–mouth disease
are ulcerating lesions of the oral cavity due to viruses and
are self-limiting, but can be very painful. Leukoplakia is a
precursor lesion to oral cancer, seen most commonly in oral
Dr.Wahid Helmi - Dymiate -Egypt

tobacco users.

93. A 14-year-old boy presents for evaluation of behavior


problems that his mother reports have been present for
about a year, but have been worsening in the past few
months. She complains that her son has been having
problems in school, is not behaving, and is getting into fights.
He seems to only want to talk about science fiction movies
and occasionally seems to be talking to people who are not
really there. Sometimes, he seems really depressed and at
other times, “full of energy and happy.” On the basis of
this mother's observations, which of the following is the most
likely diagnosis?
(A) attention-deficit/hyperactivity disorder (ADHD)
(B) bipolar disorder
(C) conduct disorder
(D) depression
93. (B) Bipolar affective disorder is the most likely diagnosis
for this patient. Although ADHD,bipolar disorder, and
conduct disorder share many similarities in behavior
disorders, such as varying degrees of school and behavior
problems, defiant attitude, and distractibility, the
obsession with ideas (in this case, science fiction movies) is
not present in ADHD and conduct disorder. The mood
swings described here, as depression and elation are
consistent with bipolar disorder, which is confirmed by the
presence of hallucinations. Hallucinations, when considering
a differential diagnosis in a behavior disorder, are diagnostic
for bipolar disorder. In up to 70% of patients with bipolar
Dr.Wahid Helmi - Dymiate -Egypt

disorder, their first symptom of the disorder may be


depression.However, hallucinations are not typically a
manifestation of depression.

94. A 9-year-old male child presents in August with


complaints of a red rash on the palms of his hands, soles of
his feet, and a little on his legs. His mother states that this
rash started about 2days ago, and just before it appeared
her son had been complaining of a severe headache and
aching all over. She said he felt “hot to the touch” during
that time, as well. The child mentions he was camping in
Arkansas about 10 days ago with his dad but did not eat
anything abnormal.
On the basis of this history, what is the most likely
diagnosis?
(A) endemic typhus
Dr.Wahid Helmi - Dymiate -Egypt

(B) human ehrlichiosis


(C) Q fever
(D) Rocky Mountain spotted fever
94. (D) Rocky Mountain spotted fever (RMSF) is the most
common rickettsial infection in the United States, especially
in the eastern, southeastern, and western states, and it is
very common in 5- to9-year-old children. A known tick
exposure may or may not be documented. Most exposures to
ticks carrying Rickettsia rickettsii, the causative organism of
this disease, occur in the warmer months of April to
September when victims are most likely to participate in
outdoor activities in wooded areas. The incubation period of
RMSF is 3 to 12 days (mean 7) after a tick exposure.
The tick must be attached for 6 hours or greater in order to
transmit the disease. Clinical presentation includes fever,
often 40°C, myalgias, headache, and less characteristic,
red-rose macular or maculopapular rash. The rash usually
appears within 2 to 6 days, after the fever. The rash is
especially prevalent on the palms, soles, and extremities.
After several days, the rash,
which starts peripherally and spreads centrally, becomes
petechial. Conjunctivitis, edema,splenomegaly,
meningismus, and confusion may occur. Up to 5% to 7% of
patients with RSMF will die, and therefore, delays in
treatment should be avoided. Treatment for children is
doxycycline, regardless of age and the possible side effect of
stained teeth. In endemic areas,treatment should be started
early and is often based on suspicion alone, and prior to the
appearance of the rash. Endemic typhus (murine typhus) is
not transmitted by ticks but instead by the fleas from
infected rodents. The rash of endemic typhus differs from
that of RMSF in that it does not involve the palms and soles.
Q fever is spread by inhalation instead of ticks. The cause
of this rickettsial disease is Coxiella burnetii hosted by
Dr.Wahid Helmi - Dymiate -Egypt

domestic animals including dogs, cats,cattle, and sheep.


Unpasteurized milk from infected animals may also be a
source of this infection. One form of human monocytic
ehrlichiosis is carried by ticks that have fed on infected hosts
that may include deer, wild rodents, and sheep, most
commonly in the southeast, north, and
south central United States. The presentation is usually a
viral syndrome without any rash.Although this is usually a
self-limiting disease, deaths do occur in children; therefore,
treatment should be carried out with the antibiotic of choice,
doxycycline, regardless of side effects.

95. In young children, which of the following is the most


common cause of lower respiratory tract infections?
(A) adenovirus
(B) human parvovirus
(C) parainfluenza virus
(D) respiratory syncytial virus
95. (D) In young children, respiratory syncytial virus (RSV)
accounts for more than 70% of bronchiolitis, approximately
40% of the cases of pneumonia, and about 10% of cases of
croup. This seasonal disease occurs in the winter and early
spring months of the year. More than 50% of
children have been infected with RSV by age 1, and by the
age of 2, almost all children have been infected. Reinfection
commonly occurs but is mild. Adenovirus infections, though
common in early childhood, only account for approximately
up to 10% of all respiratory diseases. The
peak incidence of adenovirus respiratory infections occurs in
Dr.Wahid Helmi - Dymiate -Egypt

the spring, summer, and early winter. Human parvovirus


infection is typically seen in school-aged children. This
disease is characterized by the “slapped-cheek” appearing
rash on the face that appears about 10 to 17 days
following the infection. About 2 days after the appearance of
this facial rash, a similar rash appears on the extremities,
trunk, neck, and buttocks. The rash often persists for a few
days to afew weeks (average of 10 days) and often will recur
with exposure to bathing in warm water,exercise, sunlight,
and stress. Parainfluenza viruses fall into four categories
and are responsible
for the majority of cases of croup (65%), laryngitis (50%),
and tracheobronchitis (25%). Types 1to 3 occur as seasonal
outbreaks with types 1 and 2 in the fall and type 3 in the
spring and summer. Type 4 is an endemic virus. Clinical
symptoms of these viruses include laryngotracheitis
(croup), laryngitis, bronchiolitis, and less commonly
pneumonia (especially in
immunocompromised children).

96. A 3-year-old male child presents to the clinic for a cough


that occurs only after he has been running, according to his
mother. She says she first noticed this about 6 months ago,
after he had had one of his usual winter colds, and his cough
persisted for about a week. On the basis of this history, what
is the most likely diagnosis?
(A) airway foreign body
(B) asthma
(C) cystic fibrosis
(D) laryngomalacia
96. (B) Asthma, in this case exercise-induced, is the most
likely cause of this problem. The symptoms commonly
associated with acute exacerbations of asthma include
wheezing, cough,dyspnea, and chest pain. Some symptoms
Dr.Wahid Helmi - Dymiate -Egypt

that might be suggestive of asthma include exerciseinduced


cough, nighttime cough, cough after cold air exposure, and
cough after laughing. Airway
foreign bodies, though not common, are an acute problem
that may present as sudden cough,choking, and wheezing.
Cystic fibrosis (CF) is the most common, lethal, genetic
disease affecting the Caucasian population. Up to 50% of
patients with CF are diagnosed in infancy, but others
may not be diagnosed until adolescence or adulthood.
Chronic or recurrent cough should be an indicator for
consideration of CF as a differential diagnosis.
Laryngomalacia is the most common cause of stridor in
infants. It is the incomplete development of the cartilaginous
support of the
laryngoglottic structures. This congenital condition is
usually self-limiting and occurs most commonly in infants at
or just after birth. The inspiratory collapse of the epiglottis
or arytenoids cartilages is heard as stridor.

97. A 12-month-old male infant presents with his mother's


concerns that he does not seem to play with other children as
his brother and sister did at this age. She indicates she has
noticed that he does not seem to respond when she or other
children call him by name, he is indifferent to other children
or adults when they are present, and he does not seem to
know any and “just grunts.” On the basis of this history,
the most likely diagnosis for this problem is which of the
following?
(A) attention-deficit/hyperactivity disorder (ADHD)
(B) autism
(C) fragile X syndrome
(D) schizophrenia
97. (B) Autism is the most likely diagnosis for this child. The
Dr.Wahid Helmi - Dymiate -Egypt

signs of autism often present before the second year of life


such as the child's failure to respond to their name, failed
speech development, and appearing self-absorbed and
withdrawn in the presence of other children or
adults. Often in childhood, autistic children may develop
ritualistic behaviors and intense interests that if interrupted
may cause tantrums and rages. When speech does begin to
develop, it may be nonsensical: reversal of speech patterns,
echolocation, and other abnormal patterns.Goals of
treatment include early intervention to address behavior and
communication skills.
ADHD is characterized by easy distractibility, inattention,
and overactivity. Estimates for the presence of ADHD in
school-aged children range from 2% to 20%. Fragile X
syndrome is the most common cause of functional mental
retardation. This syndrome, affecting approximately 1 in
1,250 males, is caused by a trinucleotide expansion (CGG
repeated sequence) in the Fragile X Mental Retardation I
(FMR1) gene. Fragile X syndrome is characterized by a wide
range of symptoms, which may include language delay,
hyperactivity, autistic behavior, and variable
levels of mental retardation. Schizophrenia is usually
detected in adolescence, with prepubertal onset occurring
rarely. Patients may initially present with somatic or social
behavior problems.Schizophrenic children and adolescents
often have the same symptoms as adults, such as
hallucinations, bizarre thought processes, and rambling
speech.

98. When evaluating a newborn, the inability to pass a small


catheter through the nasal cavity is most
indicative of which of the following conditions?
(A) choanal atresia
(B) meconium ileus
(C) nasal infection
Dr.Wahid Helmi - Dymiate -Egypt

(D) nasal polyps


98. (A) Choanal atresia, whether unilateral or bilateral, is a
nasal obstruction that occurs relatively rarely in newborns.
If bilateral choanal atresia occurs at birth, it causes a
respiratory distress that requires immediate treatment (due
to infants being obligate nose breathers) by placing an oral
airway and subsequent surgical correction. Unilateral
choanal atresia can present as a

chronic, single-sided, nasal discharge that may not appear


until later in childhood. Meconium ileus, intestinal
obstruction secondary to inspissated meconium, occurs in
approximately 10% of newborns with cystic fibrosis. Cystic
fibrosis affects approximately 1 in 2,500 live Caucasian

births, and is a leading cause of death in young adults. Nasal


infections may occur secondary to afuruncle (infected hair
follicle) in the anterior nares or as a nasal septal abscess
following spread of a furuncle. Common causes of nasal
infections include picking at the nose and pulling out nose
hair. Nasal polyps are uncommon in children younger than
age 10, and when they do occur it is usually in older children
and adults with allergic rhinitis.
Dr.Wahid Helmi - Dymiate -Egypt

99. Anorexia nervosa is an eating disorder commonly


affecting teenage girls. Which of the following best
represents the percentage of the teenage girls affected?

(A) 1% to 5%

(B) 5% to 10%

(C) 10% to 15%

(D) 15% to 20%

99. (A) It is estimated that 1% to 5% of adolescents are affected


by anorexia nervosa. There are two types of anorexia nervosa.
The first is the nonpurging type when patients restrict their total
caloric intake and the second involves binge eating and purging
in association with the restrictive dietary habits. Otherwise,
intensive exercise regimes may be used as a means to control
weight.Anorexia nervosa occurs in boys but is more prevalent in
girls (2:1). The specific etiology of this familial problem is
unknown; there are genetic and environmental factors. DSM-IV
criteria also include refusal to keep weight at 85% of ideal
weight, intense fear of gaining weight even though underweight
amenorrhea and disturbance in the way one's body shape is
experienced.
Dr.Wahid Helmi - Dymiate -Egypt

100. Which of the following is the most common childhood


nutritional disorder in the United States?

(A) binge eating disorder

(B) folate deficiency

(C) obesity

(D) rickets

100. (C) Obesity is the number one nutritional disorder in


children in the United States. In 2004, 17% of American
children aged between 9 and 19 were considered obese. Risk
factors for obesity include other obese family members and
infants born to diabetic mothers. Associated environmental
factors include sedentary lifestyle, total caloric intake, television
watching, and computer games. All are considered contributory
factors in childhood obesity. Binge eating disorder is a relatively
new eating disorder category. It is most frequent in overweight
or obese individuals. This disorder includes recurrent episodes
of binge eating (eating more than most individuals would in a 2-
hour period) and a sense of lack of control over the impulse to
eat,marked distress over the episode at least 2 days a week, and
is not associated with regular compensatory activity such as
purging or fasting. Folate deficiency anemia (megaloblastic) can
occur in infants within a few weeks after birth. This deficiency
may be a result of malabsorption,low dietary intake such as with
goat's milk or home-prepared formulas that have been sterilized
by heating, or formulas based on pasteurized milk. Infants who
are breastfed or given supplemented cows’ milk formulas do
Dr.Wahid Helmi - Dymiate -Egypt

not have a problem with folate deficiency. In children,

rickets is most commonly a result of poor dietary intake of


vitamin D and inadequate exposure to direct sunlight. Vitamin D
sources include milk, cheese, and baby formula. Vitamin D in
humans is produced by activation of its inactive precursors in
the skin after exposure to ultraviolet light.
Dr.Wahid Helmi - Dymiate -Egypt

101. By which age do infants develop heart failure secondary


to congenital heart lesions?

(A) birth

(B) 6 months

(C) 9 months

(D) 12 months

101. (B) The symptoms for congestive heart failure in infants


are typically failure to thrive,tachycardia, and poor feeding.
These will typically not present at birth and will be identified by

102. Which of the following is the antibiotic of choice for a


patient who is diagnosed with Bordetella pertussis?

(A) erythromycin 40-50 mg/kg/24 hours in divided doses X 14


days

(B) ampicillin 100 mg/kg/34 hours in divided doses X 7 days

(C) amoxicillin 80-90 mg/kg/24 hours in divided doses X 10


days

(D) cephalexin 30 mg/kg/24 hours in divided doses X 7 days

102. (A) Bordetella pertussis is a gram-negative bacillus and,


therefore, of all the choices, the antibiotic with good gram-
Dr.Wahid Helmi - Dymiate -Egypt

negative coverage is erythromycin. The other


macrolides,azithromycin and clarithromycin may also be given
for shorter durations, however they are more expensive.
Ampicillin, amoxicillin, and cephalexin provide mainly gram-
positive coverage.

103. Which of the following is indicated for an incarcerated


inguinal hernia present for more than 12 hours?

(A) watchful waiting

(B) manual reduction

(C) surgical reduction

(D) bilateral surgical reduction

103. (C) Surgical reduction is the treatment of choice for


incarcerated hernias over 12 hours. At that point the likelihood
that the hernia will manually reduce is very small and the bowel
is becoming necrotic and needs to be removed as soon as
possible. Bilateral surgical reduction is required only in the
event of two hernias, and there is no place for prophylaxis
Dr.Wahid Helmi - Dymiate -Egypt

surgery for inguinal hernia repairs.


Dr.Wahid Helmi - Dymiate -Egypt

104. A 6-year-old female child presents with neck pain and


fever for 2 days. Her remote history consists of 2 to 3 days of
diarrhea and vomiting. She attends a local daycare where
other kids had similar nausea/vomiting, but recovered. The
LP was positive for gram-negative bacilli, decreased protein,
and increased neutrophils. Which of the following is the
most likely etiologic agent?

(A) rotavirus

(B) Salmonella species

(C) Corynebacterium diphtheria

(D) Clostridium botulinum

104. (B) Salmonella species are gram-negative bacilli that are


classified as Enterobacteriaceae,along with E Coli. While
extremely uncommon as an etiology for meningitis, salmonella
can cause lethal meningitis infections and must be watched.
While there is typically no treatment for mild to moderate
diarrhea from salmonella infections, these patients should be
monitored for complete resolution. Viral meningitis typically
does not have a positive Gram stain, unless there is
contamination. Corynebacterium and clostridium are gram-
positive bacilli
Dr.Wahid Helmi - Dymiate -Egypt

105. In an infant with highly suspected vitamin K deficiency,


which laboratory finding would be expected?

(A) prolonged PT (prothrombin time)

(B) elevated fibrinogen

(C) decreased platelet count

(D) decreased aPTT (activated partial thromboplastin time)

105. (A) Vitamin K deficiency causes hemorrhagic disease of


the newborn. Vitamin K is one of the compounds required for
conversion of prothrombin, factors VII, IX, and X of the
coagulation cascade. In addition, proteins C & S are also
Vitamin K dependent. Therefore, the result is an increased
prothrombin time and this would result in an increased aPTT.
There is no effect on platelets or fibrinogen.
Dr.Wahid Helmi - Dymiate -Egypt

106. In a 12-month-old male infant presenting with acute


onset ear pain that is disrupting his sleep, which of the
following findings on clinical examination would confirm a
diagnosis of acute otitis media?

(A) erythematous tympanic membrane

(B) tenderness upon palpation of the tragus (C) bulging


tympanic membrane

(D) flat tracing on tympanometry

106. (C) The diagnosis of otitis media requires the presence of


middle ear effusion, acute onset ofsymptoms, and signs and
symptoms of middle ear inflammation. Presence of the middle
ear effusion can be determined by the bulging of the tympanic
membrane, air-fluid levels, absent mobility of the tympanic
membrane by pneumatic otoscopy, or otorrhea from perforation.
Office tympanometry can be performed to confirm a diagnosis
of effusion. Tenderness on palpation of the tragus typically is a
sign of otitis externa.
Dr.Wahid Helmi - Dymiate -Egypt

107. Which of the following requirements for child safety


restraints is TRUE?

(A) Infants weighing less than 20 lb and longer than 20 inches


may sit in forward-facing seats.

(B) Children weighing between 20 and 30 lb may sit in upright


booster seats.

(C) Children who weigh less than 80 lb should be in a certified


child safety seat/booster.

(D) Children weighing more than 40 lb may sit in the fiont seat
in four-door vehicles.

107. (C) While different states have different requirements for


child safety restraints, the most common guidelines state that
infants must be 20 lb and 1 year of age before switching to
forwardfacing seats. Children between 20 lb and 40 lb should be
in front-facing safety seats, typically with a 5-point harness;
children between 40 and 80 lb may be in booster seats in which
the back is typically required based on the height of the child.
Lastly, children should be older than 12years of age and
typically at least 80 lb as the front air bags are dangerous.
Dr.Wahid Helmi - Dymiate -Egypt

108. Which of the following is the most common cause for


childhood gynecomastia?

(A) neoplasms

(B) medications

(C) illicit drug use

(D) idiopathic

108. (D) The most common etiology of gynecomastia is


idiopathic. Occurring in 50 % to 60% of adolescent males,
idiopathic gynecomastia typically is self-limited. Additional
uncommon etiologies of gynecomastia include liver disease,
hyperthyroidism, illicit drugs (marijuana heroin), neoplasms
(adrenal, testicular), and medications (eg, antacids,
chemotherapy)
Dr.Wahid Helmi - Dymiate -Egypt

109. In the evaluation of a child with newly diagnosed


hypertension, which of the following evaluations will help
rule in the most common etiology?

(A) urinalysis

(B) serum uric acid 7 V D tewelatdalaegltull

(D) chest radiography

109. (A) A urinalysis should be performed because renal disease


is the most common etiology of hypertension in children.
Electrocardiograms and chest radiography should be considered
as part of the evaluation for end-organ disease as well as an
initial basic metabolic panel to include serum and creatinine.
Although rare, elevated uric acid has also been shown to cause
essential hypertension in children

.
Dr.Wahid Helmi - Dymiate -Egypt

110. The Centers for Disease Control and Prevention


recommend the first lead screening for children living in
high risk areas in the United States at which age?

(A) 6 months

(B) 9 months

(C) 15 months

(D) 24 months

110. (B) The CDC recommends that there are two age ranges for
testing lead in children in the United States: 9 to 12 months and
again at 24 months. These high-risk areas include poverty-
stricken areas, use of lead paint pottery, lead painted homes
(peeling or cracking), industrial exposures,and use of diarrhea
remedies in Mexico. The CDC recommends using questions to
screen all children between 6 months and 6 years of age.

Answers and Explanations


1. (B) Bronchiectasis has numerous etiologies. Most commonly,
cultures reveal normal oral flora
from the lower respiratory tract: Streptococcus pneumoniae,
Staphylococcus aureus,
Haemophilus influenzae, Pseudomonas aeruginosa.
Parainfluenza viruses typically are
responsible for croup. Corynebacterium diphtheriae is the
causative organism for diphtheria.
Rhinovirus is the most common pathogen isolated with acute
viral rhinitis or the common cold.
Dr.Wahid Helmi - Dymiate -Egypt

2. (D) The typical hemoglobin electrophoresis for beta


thalassemia minor has an elevated level of
hemoglobin A2. In a normal infant there is mainly HgF and
HgA1 with minimal amounts of A2.
Bart hemoglobin is diagnostic for the alpha thalassemias after
the neonatal period is over. Beta
thalassemia major will only have fetal hemoglobin on
electrophoresis. Because of the high
incidence of false-negatives in hemoglobin screenings in the
neonatal period, it is important for
the provider to do a full work-up of microcytic, hypochromic
anemias to ensure proper
diagnosis.
3. (B) In children who present with symptoms of sore throat and
fever, approximately 50% to 70%
of these cases are due to a viral infection. Adenovirus is one of
the most common etiologic viral
agents. Epstein–Barr virus is the etiologic agent for
mononucleosis and while very common in
the United States it is still less than rhinoviruses and
coronaviruses. The two remaining choices
are bacterial pathogens of which group A beta-hemolytic
streptococcus (GAS) is the most
common followed by the less common pathogens (group C
Streptococcus, Arcanobacterium
haemolyticus, and Streptococcus pneumoniae). As a single
agent, GAS is the most common
etiology of acute tonsillitis and pharyngitis
4. (A) In left-sided congestive heart failure, the signs of
tachycardia, tachypnea, intercostal
retractions, rales, and rhonchi are found. Hepatosplenomegaly is
a sign of right-sided congestive
heart failure. Bradycardia is not associated with either left- or
right-sided congestive heart
Dr.Wahid Helmi - Dymiate -Egypt

failure in the pediatric patient.


5. (B) Leukemia is the most common form of childhood cancer.
Acute lymphoblastic leukemia is the
most common form of leukemia in childhood, accounting for
approximately 4 out of 100,000
children younger than the age of 15. The clinical presentation is
variable, ranging from severe
with a life-threatening infection to asymptomatic at a routine
well-child visit. Often, there is a 3-
to 4-week history of an illness prior to the diagnosis, with signs
and symptoms including malaise,
anorexia, intermittent fever, bone tenderness, pallor, petechiae,
purpura, and abdominal pain.
Findings noted on the physical examination include pallor,
petechiae, purpura, retinal
hemorrhages, lymphadenopathy (either localized or generalized
to cervical, axillary, or inguinal
areas), bone and joint tenderness (especially in the pelvis, lower
vertebral bodies, and femur),
hepatosplenomegaly, and nephromegaly. Initially, the most
useful test is a complete blood count
with differential, revealing multiple cytopenias and leukemic
blasts. The bone marrow
examination is diagnostic, revealing a homogeneous infiltration
of leukemic blasts replacing
normal marrow. Acute myelogenous leukemia typically presents
with hyperleukocytosis (WBC >
100,000) or with myeloblasts on peripheral smears and bone
marrow biopsies. It accounts for
25% of leukemias in childhood. Patients with chronic Epstein–
Barr virus infections present with
sore throat, fever, posterior cervical lymphadenopathy, and
malaise associated with atypical
Dr.Wahid Helmi - Dymiate -Egypt

lymphocytosis and a positive heterophile antibody test.


Hodgkin's lymphoma typically presents
with painless cervical adenopathy and a normal CBC. However,
typically the C-reactive protein
and erythrocyte sedimentation rates are elevated.
6. (A) Despite the increase in vaccination of infants in the
United States, Streptococcus
pneumoniae remains the most common etiologic agent for
bacterial meningitis in the pediatric
population. Haemophilus influenzae type B is the second most
common, but has gone down
significantly due to the widespread vaccination of children.
Neisseria meningitides has
approximately 2,400 to 3,000 cases a year. Meningitis due to
Listeria monocytogenes is
typically seen in the neonatal period due to transmission from
the mother. It is present in normal
fecal matter in around 10% of the population. Its rates have gone
down due to strict guidelines
for the food industry, resulting in less than 1,000 cases per year.
7. (B) The diagnosis of rheumatic fever is based on clinical
grounds using the modified Jones
criteria. Two major manifestations or one major and two minor
manifestations in addition to
supporting evidence of a preceding streptococcal infection are
needed to make the diagnosis of
rheumatic fever. The major manifestations are polyarthritis,
carditis, erythema marginatum,
subcutaneous nodules, and Sydenham chorea. The minor
manifestations are fever, arthralgia,
previous rheumatic fever or rheumatic heart disease, an elevated
sedimentation rate or Creactive
protein, and a prolonged P–R interval. The supporting evidence
of a preceding
Dr.Wahid Helmi - Dymiate -Egypt

streptococcal infection includes elevated titers of


antistreptolysin O or other streptococcal
antibodies and positive throat culture for group A beta-
hemolytic streptococcus
8. (D) Hemophilia B, also known as Christmas disease and
factor IX deficiency. Factor IX is
activated on the intrinsic side of the coagulation cascade right
before the common pathway and
the result is an increased aPTT, with a normal prothrombin time,
thrombin time, and INR. It does
not affect platelets nor bleeding time.
)
9. (B) Tanner stages of sexual maturation categorize the
progression of pubertal development in
girls according to pubic hair and breast development. Menarche
usually occurs 18 to 24 months
following the onset of breast development. In female breast
development, Tanner Stage I is an
absence of breast development; Stage II is a small, raised breast
bud; Stage III shows further
enlargement/elevation of breast and alveolar tissue; Stage IV is
the areola and papilla forming a
secondary mound on breast contour; and Stage V is the mature
breast with alveolar area as part
of the breast contour. For the stages of pubic hair development,
Stage I is prepubertal, an absence
of hair; Stage II shows sparse, fine hair, primarily on the border
of labia; Stage III is pigmented
and curly and increases in quantity on the mons pubis; Stage IV
is increased quantity of coarser
texture with labia and mons pubis well covered; and Stage V is
mature adult distribution with
spreading to medial thighs.
10. (C) During the first year of life, the average, expected
increase in weight of a full-term infant is
Dr.Wahid Helmi - Dymiate -Egypt

to regain the birth weight by 2 weeks of age, double the birth


weight by 4 months of age, and
triple the birth weight by 1 year of age.
11. (B) When evaluating children with physical injuries, the
major difficulty is distinguishing
intentional injuries from unintentional injuries. Inconsistencies
between the stated story and the
injury are suspect. Discoloration caused by healing bruises tends
to follow a distinctive pattern.
On the first day, there is swelling without discoloration. From
day 1 through day 5, the bruise is
purple in color. For days 5 through 7, the bruise is green. Then,
from day 7 through day 10, the
bruise is yellow, followed by a brownish color from day 10 to
day 14.
12. (D) Rotavirus is one of the most important causes of acute
gastroenteritis in infants and young
children primarily 6 to 24 months of age. In the United States,
there are 65,000 to 70,000
hospitalizations and 200 deaths per annum. Peak incidences
occur in the fall and winter. Most
initial infections are characterized by diarrhea (watery,
nonbloody, nonmucous), fever, and
vomiting. Nasal congestion and coryza often precede the
gastrointestinal symptoms. Clostridium
difficile produces a toxin that causes a self-limited diarrhea in
which symptoms
characteristically begin following the administration of
antibiotics that reduce normal bowel
flora. Giardia lamblia, a flagellated protozoa, characteristically
causes a mild diarrhea, with or
without a low-grade fever, anorexia, flatulence, and abdominal
cramps. It is not associated with
vomiting nor upper respiratory tract symptoms. Shigella
gastroenteritis in young children
Dr.Wahid Helmi - Dymiate -Egypt

classically presents acutely with a high fever or seizures along


with vomiting followed by
bloody, mucoid, diarrheal stools.
13. (A) “Toeing in” in children before the age of 2 is typically
due to tibial torsion; however, any
“toeing in” after the age of 2 to 3, is usually due to femoral
anteversion. The femur has more
internal rotation that results in the presentation Genu varum is
known as bowleg and genu
valgum is known as knock-kneed.
14. (D) The first sign of pubertal development in boys is the
enlargement of testicular size and
occurs at the mean age of 11.6 years. Genital stages accelerate
before pubic hair development,
which occurs, on average, at 13.4 years of age. The deepening of
the voice and the development
of chest and axillary hair usually occurs in midpuberty or 2
years after the growth of pubic hair.
15. (A) Huntington disease is an autosomal dominant hereditary
disease. Its occurrence is between
1:5,000 and 1:20,000. It is caused by a defect on chromosome
4p16.3 that results in a repeat of
“CAG” in the “Huntington” protein gene.
16. (A) Bell's palsy is the acquired peripheral facial weakness
(cranial nerve VII) of sudden onset
and unknown etiology. It often follows a viral illness with
notable improvement within 2 weeks
and near complete recovery within 2 months. Prednisone
therapy may promote recovery of facial
strength. Guillain–Barré syndrome (acute idiopathic
polyneuritis) generally presents with
symmetrical weakness of the lower extremities, which may
ascend rapidly to the arms, trunk, and
Dr.Wahid Helmi - Dymiate -Egypt

face. Nonspecific respiratory or gastrointestinal symptoms may


occur 5 to 14 days preceding the
infection. Physical examination will yield symmetric flaccid
weakness, which is usually
proximal in distribution. Rarely, there is cranial nerve (III–VI,
IX–XI) involvement. Botulism is
most often caused by the ingestion of food containing the
Clostridium botulinum toxin or rarely
from an infected wound. Children will present with blurred or
double vision, ptosis, or choking.
Physical findings include a weak swallow paralysis of
accommodation and eye movements. In
this case, there was not a history of food ingestion or wound
infection to support this diagnosis.
Children with a brain stem tumor may present with facial and
extraocular muscle palsies,
hemiparesis, gait disturbances, and hydrocephalus (25%).
Changes in personality such as
lethargy, irritability, and aggressive behavior are particularly
common findings. Speech and
swallowing difficulties are not unusual. Later in the illness,
patients will develop vomiting and
headaches.
17. (C) Still's murmur is the most common innocent murmur of
early childhood and is usually
appreciated in children from 3 to 6 years of age. It is a grade I–
III/VI early systolic ejection
murmur of musical or vibratory quality heard best between the
apex and the left lower sternal
border. It is loudest when the patient is in a supine position. The
murmur may diminish or
disappear with inspiration, during the Valsalva maneuver, or
when the patient is standing or
Dr.Wahid Helmi - Dymiate -Egypt

seated. A physiologic peripheral pulmonic stenosis murmur is a


soft, short, high-pitched, grade I–
II/VI systolic ejection murmur. Typically, it is auscultated with
equal intensity at the left upper
sternal border, along the back, and in both axillae. It is usually
found in newborns and generally
disappears by 3 to 6 months of age. A pulmonary ejection
murmur is the most common innocent
murmur of later childhood and is usually seen in children 8 to 14
years of age. It is a soft, early to
midsystolic ejection, grade I–III/VI murmur heard best along the
left upper sternal border. It is
louder when the patient is supine or with increased cardiac
output. It diminishes with standing or
during the Valsalva maneuver. A venous hum is a continuous
musical, grade I–II/VI murmur heard
at the right or left superior infraclavicular area. The murmur is
obliterated when the patient is in
a supine position, with head rotation, and with compression of
the jugular vein. It is usually
auscultated in children from 3 to 6 years of age.
18. (D) Sudden infant death syndrome (SIDS) is defined as the
sudden, unexplained death of an
apparently healthy infant that is unexpected and not adequately
explained by a comprehensive
medical history, a postmortem physical, and investigation of the
death scene. SIDS is a leading
cause of death in infants between the ages of 1 month and 1
year, second only to congenital
anomalies. The exact etiology of SIDS is unclear. Prevention of
SIDS has become a focus of
public health measures. In 1994, The American Academy of
Pediatrics initiated a campaign
Dr.Wahid Helmi - Dymiate -Egypt

called “Back to Sleep,” which recommended placing infants


in the supine position for sleep.
Following the institution of this campaign in the United States,
the annual death rate decreased
from 1 3 per 1 000 to 0.7 per 1,000.
19. (A) Bronchiectasis, meaning “dilation of the bronchi,”
results from destruction of the airway and
poor drainage, often associated with cystic fibrosis, foreign body
aspiration, or an infection. It is
uncommon in the general population. The presentation may vary
from a chronic productive cough
to recurrent pneumonia with or without hemoptysis. Persistent
rhonchi, rales, and decreased
breath sounds are noted over the affected atelectatic area. Croup
is an inflammatory disease of
the larynx most frequently affecting young children during the
fall and early winter months.
Typically, there is an upper respiratory tract prodrome followed
by stridor and a “barky cough”
in the absence of drooling. Subglottic narrowing with a normal
epiglottis is diagnostic on a
lateral neck X-ray. The most common pathogen is parainfluenza
virus. Bronchopulmonary
dysplasia is most commonly seen in infants in the neonatal
intensive care unit. It is a chronic
condition seen in patients whose clinical course included hyaline
membrane disease. These
infants typically need oxygen for a few months as they grow and
some need permanent
tracheostomy and ventilation for up to 2 years. Chronic
bronchitis falls into the chronic
obstructive pulmonary disease category typically seen in older
adults and does not typically
present with acute symptoms.
Dr.Wahid Helmi - Dymiate -Egypt

20. (A) Otitis externa is an infection of the auditory canal. The


most common etiologic agent is
pseudomonas. However, Staphylococcus aureus is a very close
second and therefore antibiotic
treatment should provide coverage for both organisms.
Corynebacterium is part of the normal
flora of the auditory canal and does not typically cause
infections. Streptococcus pyogenes is the
most common cause of acute bacterial pharyngitis.
21. (A) Cytomegalovirus (CMV) is one of the congenital
neonatal TORCH infections
(toxoplasmosis, o ther [syphilis, varicella-zoster, and parvovirus
in this list], r ubella, c
ytomegalovirus, and h erpes simplex/h epatitis/H IV). CMV is
the most common congenital
infection. The disease-specific manifestations for CMV include
microcephaly with
periventricular calcifications, neonatal jaundice with direct
hyperbilirubinemia, and
hepatosplenomegaly. Other associated manifestations include
intrauterine growth retardation,
thrombocytopenia, and purpura. Disease-specific manifestations
for herpes simplex virus include
skin/eye/mouth vesicles, encephalitis, respiratory distress, and
sepsis. Disease-specific
manifestations of rubella include congenital heart lesions (patent
ductus arteriosus, pulmonary
artery stenosis, aortic stenosis, ventricular defects),
thrombocytopenic purpura characterized by
purple macular lesions (“blueberry muffin” appearance),
cataracts, retinopathy, and
sensorineural deafness. Disease-specific manifestations of
syphilis include mucocutaneous
Dr.Wahid Helmi - Dymiate -Egypt

lesions (snuffles), periostitis, osteochondritis, and hemolytic


anemia. Often, these babies are
stillborn. Syphilis is caused by a spirochete, Treponema
pallidum, not a virus.
22. (A) Approximately 40% of children who have been
physically abused showed evidence of
ocular trauma. Retinal hemorrhages are the most frequent ocular
finding that result from violent
shaking. This form of child abuse is termed shaken baby
syndrome. The finding of retinal
hemorrhages in an infant without an appropriate medical
condition (eg, clotting disorder,
leukemia) should raise concerns about nonaccidental trauma.
Some of the most common
presenting complaints of infants with shaken baby syndrome are
lethargy, coma, seizures,
vomiting, and respiratory distress. Retinal hemorrhages are not
associated with retinitis
pigmentosa, retinoblastoma, Reye syndrome, or viral
encephalitis. With Reye syndrome, an
antecedent viral illness is followed by vomiting and progressive
lethargy. On examination, there
is usually fever, tachypnea, and stupor. Laboratory hallmarks
include elevated serum
hepatocellular enzyme assays and elevated serum ammonia.
Retinitis pigmentosa is a progressive
retinal degeneration and is characterized by pigmentary changes,
optic atrophy, and progressive
impairment of visual function. The presenting clinical
manifestation is usually an impairment of
dark adaptation or night vision. Clinical manifestations of viral
encephalitis vary in severity
depending upon the etiologic organism (eg, cytomegalovirus,
mumps, echovirus). Some children
Dr.Wahid Helmi - Dymiate -Egypt

will have mild symptoms lapsing into a coma leading to death,


whereas others are febrile, with
convulsions and hallucinations followed by full recovery.
23. (C) A febrile seizure is a brief (less than 15 minutes),
generalized, symmetric, tonic–clonic
seizure associated with a febrile illness (temperature greater than
38.8°C) without any central
nervous system infection or neurologic cause. An absence (petit
mal) seizure is a brief (2 to 25
seconds) loss of consciousness that can occur multiple times per
day. There is no loss of tone,
and frequently the only observable behaviors are staring or
minor movements such as lip
smacking and semipurposeful movements of the hands. There is
no postictal period. Complex
partial seizures (psychomotor) have varied symptoms including
alterations in consciousness,
unresponsiveness, and repetitive complex motor activities that
are purposeless. Often, at the
beginning of the attack, there is a psychoillusory phenomenon
such as hallucinations, visual
distortions, visceral sensations, or feelings of intense emotions.
Simple partial seizures include
focal motor, adversive, and somatosensory seizures.
Manifestations of these seizures are varied
including hallucinatory, psychoillusory, or complex emotional
phenomena. Children will interact
normally with their environment, with the exception of those
limitations imposed by the seizure.
Following the seizure (minutes to hours), there may be transient
paralysis of the affected body
part.
24. (B) Urinary tract infections (UTIs) are one of the most
common infections in children. Clinical
Dr.Wahid Helmi - Dymiate -Egypt

features of a UTI vary depending upon the age and sex of the
child. In newborns, the most
common symptom is failure to thrive associated with poor
feeding, diarrhea, and vomiting. In
infants, the symptoms may be relatively nonspecific, such as
poor feeding, failure to gain weight,
vomiting, fever, strong-smelling urine, and irritability. As
children grow older, the initial signs
and symptoms become more specific to the urinary tract. In
early infancy, males are two times
more likely than girls to have a UTI. Also, uncircumcised males
are 10 times more likely to be
affected than circumcised males. Escherichia coli is the most
common pathogen for the first UTI
(80%) and of recurrent infections (75%). Other organisms that
cause infections include
Pseudomonas aeruginosa, Proteus, Enterobacter, Klebsiella, and
Enterococcus. An infection
with Staphylococcus saprophyticus, a coagulase-negative
staphylococcus, is primarily seen in
adolescents with a UTI.
25. (B) Dehydration is a common pathophysiologic alteration in
fluid and electrolyte balance in
children. Children are at an increased risk for dehydration
because of their decreased oral intake,
especially when ill, and their higher ratio of surface area to body
weight, promoting significant
evaporative losses. Important clinical features to estimate the
degree of dehydration include
postural blood pressure, changes in heart rate, capillary refill
time, skin turgor and color, lack of
tears, lack of external jugular venous filling when supine,
sunken fontanel (if present), and
altered mental status. This infant was estimated to have mild
dehydration (3% to 5% decrease in
Dr.Wahid Helmi - Dymiate -Egypt

body weight) with decreased tears, slightly longer capillary refill


time (2 to 3 seconds), and
intermittent irritability and restlessness. Severe dehydration
(11% to 15% decrease in body
weight) manifests as markedly decreased skin turgor with
parched or mottled mucous
membranes, absence of tears, tachycardia, capillary refill greater
than 4 seconds, hypotension,
circulatory collapse, and anuria. Moderate dehydration (6% to
10% decrease in body weight)
manifests as decreased skin turgor; dry mucous membranes;
decreased tearing; oliguria; and
normal pulse, blood pressure, and perfusion.
26. (C) Intussusception is the most common cause of intestinal
obstruction between 3 months and 6
years of age. It is twice as common in males than females. It is
caused by intestinal invagination,
usually around the ileocecal valve. The classic presentation is
intermittent severe colicky
abdominal pain with legs drawn up, followed by periods of
comfort or falling asleep. Vomiting
usually occurs in the early phase, which later becomes bilious. A
passage of blood and mucus in
the stool (“currant jelly stools”) occurs in 60% of the cases.
Palpation of the abdomen usually
reveals a sausage-shaped mass in the right upper quadrant. The
classic presentation of pyloric
stenosis is in first-born males of 3 to 6 weeks of age, presenting
with nonbilious projectile
vomiting leading to dehydration with hypochloremia,
hypokalemia, and metabolic alkalosis. A
firm, movable, 2-cm olive-shaped mass (“olive”) is palpable
superior and to the right of the
Dr.Wahid Helmi - Dymiate -Egypt

umbilicus in the midepigastrium. In addition, peristaltic waves


may be visible on the physical
examination. The classic presentation of appendicitis presents
with a period of anorexia
followed by steady periumbilical pain shifting to the right lower
quadrant; nausea and vomiting
is followed by a low-grade fever. Diarrhea (nonbloody and
nonmucous), if it occurs, is
infrequent. Peritoneal signs are present. The incidence increases
with age and peaks during
adolescence. Infective enteritis usually begins with emesis
followed by crampy abdominal pain
of hyperperistalsis. This sequence of symptoms with emesis
preceding pain is an important factor
in distinguishing it from intussusception. Masses are not
palpated with infective enteritis.
27. (C) Dehydration is a common pathophysiologic alteration in
fluid balance in children. The body
has a maintenance fluid requirement to replace daily normal
losses that occur through the skin,
kidney, intestines, and respiratory tract. The following formula
can be used to calculate the usual
amount of fluid a healthy child requires by mouth to maintain
hydration:
100 mL/kg for the first 10 kg of body weight
50 mL/kg for the next 10 kg of body weight
20 mL/kg for the weights above 20 kg
For this question, a 24-kg child would require:
100 mL/kg × 10 kg = 1,000 mL for the first 10 kg
50 mL/kg × 10 kg = 500 mL for the next 10 kg
20 mL/kg × 4 kg = 80 mL for the next 4 kg
Total = 1,580 mL 24 kg
28. (B) The most common etiologic organisms for bacterial
meningitis in children are S
Dr.Wahid Helmi - Dymiate -Egypt

pneumoniae, N Meningitidis, and H influenzae. Because of an


increase in resistant S
pneumoniae, coverage with vancomycin and a third-generation
cephalosporin such as cefotaxime
or ceftriaxone is needed for best coverage. Gentamicin can be
used but, as with all
aminoglycosides, caution is needed regarding toxicity.
Ampicillin, rifampin, and
chloramphenicol are alternative treatments if necessary.
29. (B) Congenital hypothyroidism is one of the most common
disorders tested for in newborn
screening tests, revealing an elevated TSH (thyroid stimulating
hormone) and a decreased
T4(thyroxine). Symptoms suggestive of congenital
hypothyroidism in the neonate include
hypotonia, coarse facial features, hirsute forehead, large
fontanels (anterior and posterior),
widely open sutures, umbilical hernia, protruding/large tongue,
hoarse cry, distended abdomen,
and prolonged jaundice. Signs of congenital hypothyroidism
include lethargy or hypoactivity,
poor feeding, constipation, mottling, and hypothermia.
Congenital adrenal hyperplasia (CAH) is
not universally screened for in the newborn screening test, as it
is included in only 14 of the 50
states. In females with CAH, there may be virilization with
abnormalities of the external genitalia
varying from mild enlargement of the clitoris to complete fusion
of the labioscrotal folds. Signs
of adrenal insufficiency (salt loss) may present in the first few
days of life. Crigler–Najjar
syndrome is not one of the disorders tested for in the standard
newborn screening tests. It is an
Dr.Wahid Helmi - Dymiate -Egypt

inherited disease producing congenital nonobstructive,


nonhemolytic, unconjugated severe
hyperbilirubinemia. The physical findings in this infant do not
correlate with Crigler–Najjar
syndrome. Galactosemia is tested for in the newborn screening
test in nearly all 50 states. The
infant may have symptoms of cataract, hepatomegaly, and
prolonged jaundice. Often, these
neonates have Escherichia coli sepsis, leading to death in the
first 2 weeks of life if not treated
promptly.
30. (A) Caput succedaneum is a result of fluid and blood
accumulation in the occipitoparietal region
of the newborn's scalp due to the vacuum effect of membrane
rupture. A cephalohematoma is a
firm, tense external swelling of the cranium that does not extend
across suture lines because it is
limited to the surface of one cranial bone. It occurs most often in
the parietal area. This
subperiosteal hemorrhage usually is not present at birth, but
develops within the first 24 hours of
life. Craniotabes is a condition caused by the osteoporosis of the
outer table of the involved
membranous bone, generally over the temporoparietal or
parietooccipital areas, creating a “pingpong
ball” sensation when gentle pressure is applied. A subgaleal
hemorrhage is a firm, fluctuant
external swelling of the cranium that does extend across suture
lines and increases in size over
time.
31. (C) Foreign body aspiration into the respiratory tract is
associated with an acute choking or
coughing episode with expiratory wheezing (indicative of a
lower airway obstruction) in
Dr.Wahid Helmi - Dymiate -Egypt

children aged 6 months to 4 years of age. Often, there is a


history of the child playing with small
toys that are commonly aspirated. Asymmetrical physical
findings of decreased breath sounds
and localized wheezing are present with foreign body aspiration.
A positive forced expiratory
chest X-ray shows a mediastinal shift away from the affected
side. Radiolucent foreign bodies
such as plastic toys may not appear on an X-ray, but there will
be evidence of this mediastinal
shift. Asthma is generally characterized by wheezing, but it is
not unilateral nor is it of sudden
onset. A chest X-ray reveals bilateral hyperinflation with
flattening of the diaphragm. Epiglottitis
is a life-threatening upper airway obstructive condition that
presents with a sudden onset of
fever, dysphagia, drooling, and inspiratory retractions with
stridor. A lateral neck X-ray reveals
an enlarged, indistinct epiglottis (“thumb sign”); however, the
chest X-ray is normal. Pulmonary
embolism, rare in children, presents clinically with acute
dyspnea, tachypnea, and tachycardia.
There may be mild hypoxemia, rales, and focal wheezing. Chest
X-rays may be normal, or there
may be a peripheral infiltrate, small pleural effusion, or elevated
hemidiaphragm.
32. (C) An acute salicylate overdose (greater than 150 mg/kg)
will produce symptoms of salicylate
intoxication. Chronic salicylate intoxication occurs with
ingestion of greater than 100 mg/kg/day
for at least 2 days. Salicylates affect most organ systems,
leading to various metabolic
abnormalities. Because salicylates are a gastric irritant,
symptoms of vomiting and diarrhea
Dr.Wahid Helmi - Dymiate -Egypt

occur soon after the overdose, which may contribute to the


development of dehydration.
Salicylates stimulate the respiratory center leading to
hyperventilation and hyperpnea resulting in
respiratory alkalosis and compensatory alkaluria. A
characteristic feature of salicylate
intoxication is the coexistence of a respiratory alkalosis with a
widened anion gap metabolic
acidosis.
33. (C) Generalized tetanus (lockjaw) is a neurologic disease
caused by Clostridium tetani.
Although any open wound is a potential source for
contamination with C tetani, those with dirt,
soil, feces, or saliva are at increased risk. Tetanusprone wounds
contain devitalized tissue,
especially those caused by punctures, frostbite, crush injury, or
burns. Recommendations for
tetanus prophylaxis in a child with a laceration or abrasion
depend upon the number of previous
vaccinations, occurrence of last booster, type of wound (clean or
tetanus-prone), and age of
child. In this case, the patient is older than 7 years and had all of
his previous immunizations;
however, his most recent booster was greater than 10 years ago.
Thus, he should receive an
adult-type diphtheria and tetanus toxoid with acellular pertussis.
In most cases, when tetanus
toxoid is required for wound prophylaxis in a child older than 7
years, the Td instead of tetanus
toxoid alone is recommended so that diphtheria immunity is
maintained. If tetanus immunization
is not up to date at the time of wound treatment, then the
immunization series should be completed
according to the primary immunization schedule. If a child is
younger than 7 years, then the
Dr.Wahid Helmi - Dymiate -Egypt

diphtheria, tetanus, acellular pertussis (DTaP) booster is


indicated, unless there is a
contraindication for pertussis, in which case the diphtheria and
tetanus (DT) booster should be
administered. Tetanus immune globulin (TIG) is recommended
for treatment of tetanus. Under
special circumstances, a patient infected with the human
immunodeficiency virus (HIV) with a
tetanusprone wound should also receive TIG in addition to the
prophylactic vaccine.
34. (D) Normally, primitive reflexes are present at birth and
should not persist beyond the age of 6
months. However, the parachute reflex is a postural response
that normally appears around 7
months of age to coincide with volitional movement and persists
for life. It occurs when an infant
is held prone by the waist over a surface and lowered with the
head downward and extends the
arms and legs as a form of protection. The rooting reflex occurs
when the cheek is stroked on the
infant and they turn his/her head to feed. Galant and Landau
reflex disappear by the age of 2
months (trunk incurvation upon stroking the back) and 6 months
(the baby lifts head and
straightens spine upon being held prone), respectively
35. (A) Gonococcal ophthalmia neonatorum presents as a
unilateral or bilateral serosanguineous
discharge and then within 24 hours the discharge becomes
mucopurulent, followed by
conjunctival injection and edema of the eyelids. The usual
incubation period for Neiserria
gonorrhea is 2 to 5 days; however, the infection may be present
at birth or delayed greater than 5
days if there has been instillation of silver nitrate prophylaxis. A
presumptive diagnosis is made
Dr.Wahid Helmi - Dymiate -Egypt

by the demonstration of gram-negative intracellular diplococci


on Gram stain. Definitive
diagnosis is made by culture. Following a positive Gram stain
and pending culture results,
treatment should be promptly initiated with ceftriaxone (50
mg/kg/24 hours IV or IM for one dose
not to exceed 125 mg), a third-generation cephalosporin with
good coverage for gram-negative
bacteria. An alternate drug is cefotaxime (100 mg/kg/24 hours
IV or IM every 12 hours for 7
days or 100 mg/kg as a single dose), which is also a third-
generation cephalosporin. Although
erythromycin drops (0.5%) are used prophylactically for N
gonorrhea, this is not an effective
treatment. Gentamicin would be used for Pseudomonas, and
Chlamydia is treated with
erythromycin. Cephalexin as a first-generation cephalosporin
does not have coverage for gramnegative
bacteria.
36. (B) The most likely diagnosis in this patient is mumps. It is
endemic in most unvaccinated
populations. The onset is characterized by pain and swelling in
one or both parotid glands. The
pain can be exacerbated by tasting sour liquids such as lemon
juice. An elevated serum amylase
level is common and coincides with the parotid swelling.
Unilateral, rarely bilateral, nerve
deafness is a complication of mumps that may be transient or
permanent. Other complications
include meningoencephalomyelitis, orchitis, epididymitis,
pancreatitis, arthritis, and rarely
thyroiditis and myocarditis.
37. (A) Café au lait spots are brown macules that may be found
on any part of the body. The
Dr.Wahid Helmi - Dymiate -Egypt

presentation of six or more spots greater than 1.5 cm is a sign of


neurofibromatosis, a genetic
disorder that results in neurofibromas that can develop in any
organ/tissue system. Miliaria are
blocked sweat gland ducts that are commonly found on the face,
scalp, or intertriginous areas.
Vernix caseosa is a normal finding in newborns and is a whitish,
greasy layering on the body— it
decreases as an infant comes to full term. Subconjunctival
hemorrhages are a common finding in
infants secondary to birth trauma.
38. (B) The Apgar score assesses the newborn at 1-minute and
5-minute intervals to determine the
need for resuscitative care. The infant is evaluated by heart rate,
respiratory effort, muscle tone,
response to catheter in nostril, and color, and each is rated on a
scale of 0, 1, or 2 for a total
score of 10. The heart rate is scaled 0–2 for absent, less than 100
bpm (slow), and greater than
100 bpm; respiratory effort of absent, slow/irregular, and good
crying. Muscle tone scale (0–2)
consists of limp, some flexion, and active motion; response to
catheter stimulation (0–2) is
scaled no response, grimace, and cough/sneeze. Finally, color is
scored 0–2 for blue/pale, body
pink with blue extremities, and completely pink.
39. (C) Ventricular septal defect, a hole between the two
ventricles, can be cyanotic or acyanotic
based on the size of the defect, and accounts for 30% of cases of
congenital heart disease. Atrial
septal defect occurs in approximately 10% of congenital heart
disease cases. Transposition of
great vessels is an embryonic malformation resulting in the aorta
arising from the right ventricle
Dr.Wahid Helmi - Dymiate -Egypt

and the pulmonary artery arising from the left ventricle. It is


responsible for about 10% of all
congenital malformations. Tetralogy of Fallot, consisting of a
ventricular septal defect,
overriding aorta, pulmonic/subpulmonic stenosis, and right
ventricular hypertrophy, accounts for
10% of congential heart disease.
40. (D) The immunization schedule is developed biannually by
the Centers for Disease Control and
Prevention. Assuming that the child has had the appropriate
immunizations at the regularly
scheduled examinations, the recommended immunizations at the
4- to 6-year-old range are the
DTaP (diphtheria, tetanus, acellular pertussis), IPV (inactivated
polio), and the MMR (measles,
mumps, and rubella). The hepatitis series should have been
completed by the age of 6 months and
the Haemophilus influenzae type B (Hib) should be completed
by the age of 12 to 15 months.
Varicella is given from 12 to 18 months and again from 4 to 6
years; the PCV (pneumococcal)
should be finished by 12 to 15 months.
41. (A) There are only two known absolute contraindications to
breastfeeding: tuberculosis of the
mother and galactosemia of the infant. The highly contagious
nature of tuberculosis makes the risk
greater than the benefit, and infants with galactosemia are
unable to digest any lactose due to an
enzyme deficiency. Infants of mothers in a methadone program
may be breastfed as long as the
mother's dose is less than 40 mg. While nicotine is transmitted
in breast milk and is therefore
strongly discouraged, it is not an absolute contraindication. As
long as a breastfed infant with
Dr.Wahid Helmi - Dymiate -Egypt

cystic fibrosis is maintaining normal growth with supplemented


pancreatic enzymes,
breastfeeding is encouraged.
42. (B) Failure to thrive is diagnosed in infants younger than the
age of 6 months with a decrease in
growth velocity that results in a decrease in two major percentile
lines on the growth chart. In the
case of this patient, she was initially in the 50th percentile and
crossed the 25th and 10th
percentile and fell into the 5th percentile. Failure to thrive is also
known as growth deficiency
and may also be diagnosed if the child is younger than 6 months
and has not grown for two
consecutive months or if a child is older than 6 months and has
not grown for 3 consecutive
months. Growth hormone deficiency/dwarfism may present with
decreased growth velocity later
in childhood; the drop in percentiles is grossly below the 5th
percentile mark. Lactose
intolerance presents with varying gastrointestinal symptoms
without the marked decrease in
weight. Beckwith–Wiedemann syndrome consists of
macrosomia, macroglossia, and
omphalocele and they are at increased risk for malignancies,
hypoglycemia, and dysmorphism
(usually of the ears).
43. (C) This patient has presented with classical findings of
Reye syndrome—upper respiratory
infection followed by unresponsiveness. Reye syndrome is
usually preceded by an upper
respiratory tract illness, which progresses into vomiting, strange
behavior, stupor, and coma.
Liver function tests (LFTs) will be markedly elevated (without
jaundice); however, the serum
Dr.Wahid Helmi - Dymiate -Egypt

bilirubin and alkaline phosphatase are normal. Unresponsive


patients who have a spinal tap will
show no cells in the CSF and glucose may be low with increased
CSF pressure. If arterial blood
gases are ordered, they will show a mild respiratory alkalosis
and metabolic acidosis. A liver
biopsy will show little inflammatory changes with diffuse
microvesicular steatosis and absent
glycogen from the hepatocytes. The mitochondria of the
hepatocytes are large and polymorphic
with decreased matriceal density. Treatment for patients with
Reye syndrome is largely
supportive—specifically decreasing cerebral edema. There is no
place for antibiotics or
steroids. The liver will fully recover if the cerebral edema is
decreased.
44. (B) Wilson disease is a result in a genetic mutation on
chromosome 13 that causes decreased
bile excretion of copper and results in accumulation of copper
by the liver, specifically the
ceruloplasmin. The build-up of copper causes damage to the
liver, basal ganglia, and other
tissues. Physical examination shows jaundice,
hepatosplenomegaly, Kayser–Fleischer rings (a
brown band at the junction of the iris and cornea under slit-
lamp), and neurologic manifestations
later in the disease process. Laboratory tests show marked
decrease in ceruloplasmin of the
liver, anemia, hemolysis, and severely elevated bilirubin with
decrease alkaline phosphatase.
Urinalysis shows severe elevation in copper excretion,
glycosuria, and aminoaciduria. Liver
biopsy is conclusive with evidence of copper greater than 250
μg/g of dry tissue. Treatment
Dr.Wahid Helmi - Dymiate -Egypt

requires copper chelation with D-penicillamine or trientine


hydrochloride. Liver transplant may
be required with noncompliance and in acute fulminant disease.
Copper chelation is continued
for life with the addition of zinc (decrease copper absorption)
and vitamin B6 (decrease optic
neuritis). Genetic screening of siblings and future children
should be strongly encouraged. Alphainterferon
therapy is mainly used to treat hepatitis patients. There is no
place for steroids in
therapy and protease inhibitors are antiviral medications that are
typically used in HIV patients.
45. (A) The patient's presentation is consistent with findings of
coarctation of the aorta. The
pathognomonic finding in coarctation is decreased or absent
femoral pulses. However, the
majority of children show no signs of coarctation in infancy and
develop signs and symptoms
during childhood, most notably unequal pulses and blood
pressure between arms and legs (arms
lower than legs). In addition, a grade II/VI ejection murmur is
heard at the aortic area and left
sternal border that radiates into the left axilla and left back.
Chest X-ray shows a normal-sized
heart, a prominent aorta, indents at the level of the coarctation,
and a dilated poststenotic segment
resulting in the “figure 3” sign. Scalloping or notching of the
ribs is due to enlargement of the
intercostal arteries. Echocardiography is used to directly
visualize the coarctation and estimate
the obstruction. Asymptomatic infants and children are
encouraged to have corrective surgery
prior to age 5, after which they are at increased risk for
myocardial dysfunction and
Dr.Wahid Helmi - Dymiate -Egypt

hypertension, and require exercise testing prior to participation


in aerobic activities. The bootshaped
heart is seen in patients with tetralogy of Fallot secondary to
right ventricular
hypertrophy; the narrowed mediastinum finding with “egg on a
string” is typically seen in
patients with transposition of the great vessels.
46. (D) Viral croup usually presents with cough that may sound
like a dog or a seal barking. The
patients are usually afebrile and also present with stridor either
at rest, in severe cases, or when
agitated, in mild cases. In addition, the patient may be cyanotic
and have retractions and acute
shortness of breath. Radiologic examination of the neck shows
subglottic narrowing with a
normal epiglottis, “steeple sign.” However, X-rays are
usually not indicated in patients with the
common presenting symptoms. Treatment for viral croup is
mainly symptomatic, especially in
mild cases consisting of oral hydration and mist therapy. Severe
cases (stridor at rest) call for
oxygen in patients who have desaturated, and nebulized racemic
epinephrine and
glucocorticoids. Dexamethasone as an intramuscular injection or
oral as a one time dose is
effective in alleviating symptoms, decreasing the need for
intubation, and decreasing hospital
stays. Inhaled budesonide is also effective in decreasing hospital
stays and improving symptoms,
but dexamethasone is more cost-effective. Patients who are
unable to be stabilized need airway
maintenance either by intubation with endotracheal tube or by
tracheostomy if intubation fails.
Dr.Wahid Helmi - Dymiate -Egypt

Because it is a self-limiting disorder, unless there is a secondary


infection most children recover
in a few days.
47. (A) Cystic fibrosis (CF) is a major cause of gastrointestinal
and pulmonary morbidity in children
due to mutations in the CF genes. The mutations lead to a
deficiency in cystic fibrosis
transmembrane conductance regulator protein that controls
movement of salt and water into and
out of epithelial cells and results in production of abnormally
thick mucus. About 15% of patients
with CF present with meconium ileus at birth. This is typically
treated with enema for
disimpaction and rarely surgery. Approximately half of the
infants with CF will present with
failure to thrive, which is diagnosed by lack of growth for 2
consecutive months in patients
younger than 6 months of age. They may also present with
respiratory compromise. However, not
all patients present in childhood. Diagnosis of CF is confirmed
by a sweat chloride level above
60 meq/L or with genetic testing. Treatment for patients with CF
is mainly symptomatic therapy
for obstructions of the digestive and respiratory tract. In
addition, there is pancreatic enzyme
supplementation to aid in digestion and vitamin and calorie
supplementation for deficiencies in
the diet. Gene therapy is now being looked at for future
treatment. Intussusception (telescoping of
the small intestine) typically presents in an infant with
paroxysmal abdominal pain, vomiting, and
diarrhea that may progress into bloody stools. Volvulus is
normally the result of intestinal
malrotation that causes occlusion of the superior mesenteric
artery and eventual bowel necrosis.
Dr.Wahid Helmi - Dymiate -Egypt

Infants typically present within 3 weeks of life with bile-stained


vomiting and bowel obstruction.
Wilson's disease is the defect in the ability to excrete copper in
the bile that results in
accumulation of copper in the liver.
48. (B) Cyanotic heart lesions are a result of a right-to-left shunt.
These include tetralogy of Fallot,
pulmonary atresia with and without ventricular septal defect,
tricuspid atresia, hypoplastic left
heart syndrome, and transposition of the great arteries. The
right-to-left shunt results in
deoxygenated blood reaching the left ventricle, aorta, and
systemic arteries. The decreased
oxygen in the blood results in decreased oxygen to the tissue and
subsequently causes cyanosis.
Atrial septal defect, ventricular septal defect, atrioventricular
septal defect, and patent ductus
arteriosus most commonly present with a left-to-right shunt.
49. (A) This infant is presenting with signs and symptoms of
pyloric stenosis. Infants typically have
vomiting (projectile at times) after every feeding and it normally
starts between the age of 2 and
4 weeks. The infant nurses fervently and is hungry. In addition,
there may be dehydration,
constipation, weight loss, and apathy. Abdomen may be
distended with gastric peristaltic waves.
Occasionally, an olive-sized mass can be felt in the right upper
quadrant with deep palpation
after the child has vomited. Vomitus is typically nonbilious.
Diagnosis is confirmed by an upper
gastrointestinal series with delayed gastric emptying, enlarged
pyloric muscle, and characteristic
semilunar impressions on the gastric antrum. In addition, an
ultrasound is needed to verify the
Dr.Wahid Helmi - Dymiate -Egypt

hypertrophic muscle. The treatment of choice for these patients


is pyloromyotomy, which can be
done laparoscopically. These patients make full recoveries and
have an excellent prognosis.
50. (D) Celiac disease or gluten enteropathy typically presents
with diarrhea episodes in the first 6
to 12 months of life—when whole grains are first fed. Therefore,
in strictly breastfed babies,
symptoms may not be noticed until solid foods are begun. The
diarrhea is usually intermittent at
first and then typically progresses into pale, greasy, foul-
smelling, frothy stools. Additional
symptoms may be constipation, vomiting, and abdominal pain,
which may lead the clinician to
think of intestinal obstruction. Other findings may be failure to
thrive, anemia, and vitamin
deficiencies. Stool sample demonstrates excessive fecal fat
excretion. Blood tests show
hypoproteinemia and impaired carbohydrate absorption.
Intestinal biopsy is the diagnostic test of
choice for celiac disease. Results show shortened celiac mucosa,
absent villi, lengthened crypts
of Lieberku$$$hn, plasma cell infiltration of the lamina propria,
and intraepithelial lymphocytes.
Treatment consists of dietary restriction of gluten—wheat, rye,
and barley. Steroids are given on
an as needed basis. Sweat chloride testing is utilized in patients
suspected of cystic fibrosis.
Gastrin level is taken in patients suspected of Zollinger–Ellison
syndrome, and RAST
(radioallergosorbent assay test) is used in patients to determine
different environmental-type
allergens
Dr.Wahid Helmi - Dymiate -Egypt

51. (B) The most likely diagnosis for this patient is


poststreptococcal glomerulonephritis. The
diagnosis is supported by a documented culture of group A beta-
hemolytic streptococcus
infection. If a culture is not available, like of the patient in this
scenario, the clinician can order
an antistreptolysin O titer. Antistreptolysin is an enzyme
released by group A streptococcus and
is elevated for up to 1 month after strep infection.
Glomerulonephritis presents with gross
hematuria with or without edema. Hypertension, proteinuria,
ascites, and headache may also be
present. Treatment with antibiotics is useful if infection is still
present, and, if necessary,
symptomatic treatment for renal failure is done with
hemodialysis. Symptoms typically resolve
within a few weeks. The monospot is used to diagnose
infectious mononucleosis. Renal biopsy
could be performed on extreme cases of glomerulonephritis but
is not typically necessary.
Immunoglobulin electrophoresis would be utilized in patients
suspected of having
immunoglobulinopathies or IgA-mediated glomerulonephritis.
52. (D) Patent ductus arteriosus (PDA) is an isolated
abnormality that occurs in infants. The ductus
arteriosus is a normal fetal vessel that joins the aorta and the
pulmonary artery and spontaneously
closes after 3 to 5 days. Lack of closure results in the audible
murmur that is “machinelike” and
maximal at the second intercostal space (ICS), at the left sternal
border (LSB), and inferior to the
clavicle. It is typically a pansystolic murmur with bounding
pulses and a widened pulse pressure.
Dr.Wahid Helmi - Dymiate -Egypt

There is also a paradoxical splitting of S1 and S2.


Echocardiography confirms the PDA, the
direction and degree of shunting, and the presence of lesions for
which the PDA is needed to
keep. If there are no other cardiac malformations requiring the
PDA, then if the PDA is large,
surgery should be completed before 1 year of age. Symptomatic
PDAs that are relatively small
may be closed with indomethacin in preterm infants. The
murmur heard in atrial septal defect
(ASD) usually is an ejection type, systolic murmur heard best at
the LSB, second ICS with a
wide, fixed S2 and normal pulses. Ventricular septal defect
(VSD) presents with a harsh,
pansystolic murmur heard best at the third and fourth ICS. With
increasing size of the VSD,
heaves, thrills, and lifts are present along with radiation
throughout the chest. Tetralogy of Fallot
presents with a rough ejection, systolic murmur heard best at the
LSB and the third ICS with
radiation to the back.
53. (C) Hypertrophic cardiomyopathy in adolescence is
typically due to familial hypertrophic
cardiomyopathy with an incidence of 1:500. Many patients are
asymptomatic until a sporting
event, which may cause symptoms, specifically sudden cardiac
death. Examination may
demonstrate a palpable or audible S4, an LV (left ventricular)
heave, systolic ejection murmur
(may need to stimulate cardiac activity), and/or a left precordial
bulge. Echocardiography is the
gold standard for diagnosis but family history should be
assessed. Stress testing is indicated to
assess for ischemia and arrhythmias. Strenuous activities are
prohibited for these patients. The
Dr.Wahid Helmi - Dymiate -Egypt

other cardiomyopathies (dilated and restrictive) are next but are


not as common. Congenital
structural abnormalities of the coronary arteries are the next
most common cause. Valvular
disorders, including surgically repaired aortic stenosis, are
typically not causes of sudden death,
but these patients should be screened for symptoms and stress
tested as necessary.
54. (A) This patient presentation describes epiglottitis. Although
there is a decreased incidence of
epiglottitis secondary to the introduction of the vaccine for
Haemophilus influenzae type B
(Hib), patients still present with sudden onset of fever,
dysphagia, muffled voice, drooling,
cyanosis, inspiratory retractions, and soft stridor. The patients
are usually sitting in a tripod
position to aid their breathing. Recognition of the classic
symptoms needs to be immediate to
stabilize the patient's airway, as these patients will
decompensate into respiratory failure quickly.
In the event that there is time, a lateral neck X-ray will show the
“thumb sign,” which is an
enlarged, undistinguished epiglottis. Treatment for the patient
requires intubation for airway
stabilization, blood cultures and throat/epiglottis cultures, and
antibiotic coverage for H.
influenzae. The steeple sign is seen in patients with croup and is
due to a subglottic narrowing.
The “figure 3” sign is seen in patients with coarctation of the
aorta. The “Scottie dog” sign is
seen in oblique lumbar films and is a normal finding
representing the pars interarticularis. Its
absence signifies spondylolysis.
Dr.Wahid Helmi - Dymiate -Egypt

55. (C) With an incidence of 1:3,000 to 1:4,000 Caucasians,


cystic fibrosis is the most common
lethal genetic disorder in the United States. While trisomy 21
(Down syndrome) is one of the
most common genetic disorders with 1:500 newborns, it is
typically not a fatal disease. It is
characterized with mental retardation and physical
malformations. Trisomy 13 is a fatal trisomy,
with most deaths occurring in early infancy or by the age of 2,
but its incidence is approximately
1:12,000 live births. Neurofibromatosis, a genetic disorder of
typical autosomal dominant
inheritance, occurs in approximately 1:3,000 live births. Most
affected children have the skin
lesions (café au lait macules or neurofibromas) and other minor
problems.
56. (B) Status epilepticus is a medical emergency and is defined
as seizure activity that lasts a
minimum of 30 minutes. This results in hypoxia, acidosis,
cerebral edema, and structural damage.
In addition, fever, respiratory depression, hypotension, and
death may occur. There are both
convulsive and nonconvulsive types of status epilepticus.
Because of its emergency status and
potential complications, the clinician needs to initiate the ABCs
(airway, breathing, circulation).
Therefore, the first line of treatment is to establish and maintain
an airway, oxygen is next, and
then circulation, which encompasses pulse, blood pressure, and
IV access. Once the IV is
established, the orders should be for administering glucose-
containing fluids and IV drug therapy
with diazepam, lorazepam, or midazolam as well as administer
phenytoin and phenobarbital.
Dr.Wahid Helmi - Dymiate -Egypt

Arterial blood gases should be ordered and any abnormalities


should be corrected appropriately.
Finally, the clinician should determine the underlying cause:
trauma, structural disorder,
infection, lactic acidosis, toxins, and uremia. Maintenance drug
therapy is necessary until the
underlying cause is determined and rectified.
57. (D) Guillain–Barré syndrome is most likely due to a delayed
hypersensitivity with T-cell–
mediated antibodies to mycoplasma and viral infections (CMV,
EBV, hepatitis B, campylobacter
jejuni). The patients may mention a nonspecific respiratory or
gastrointestinal infection 1 to 2
weeks prior to symptoms. Complaints may be paresthesias,
weakness in bilateral lower
extremities with occasional ascension into the arms, trunk, and
face, and rarely ataxia and
ophthalmoplegia in the Miller–Fisher variant. Examination
findings demonstrate symmetric
flaccid weakness, with impairment of position, vibration, and
touch in the distal portions of the
extremities. If a spinal tap is performed, it may show few
polymorphonuclear neutrophils with
high protein and normal glucose. EMG is positive for decreased
nerve conduction. Laboratory
tests may show high titers of suspected infections or active
infection of hepatitis/bacterial
pathogens. Guillain-Barré is normally a self-limiting disorder
within a few weeks, unless there
are issues with respiratory depression. Poliomyelitis is
secondary to polioviruses and presents
with fever, paralysis, meningeal signs, and asymmetrical
weakness. Botulism secondary to
Dr.Wahid Helmi - Dymiate -Egypt

infection with Clostridium botulinum in older children presents


with blurred vision, diplopia,
ptosis, choking, and weakness. In infants, botulism presents as
constipation, poor suck and cry,
apnea, lethargy, and choking. Tick-bite paralysis presents with
rapid onset with ascending flaccid
paralysis reaching upper extremities in a couple of days of onset
and patients often present with
paresthesia and pain. Finding of a tick is usually confirmatory
for these patients.
58. (B) Scoliosis is defined by lateral curvature of the spine with
rotation of vertebrae and is
typically located in the thoracic or lumbar spine in the right or
left directions. Idiopathic
scoliosis most commonly presents as a right thoracic curve in
females from 8 to 10 years of age.
Scoliosis is typically asymptomatic unless curvatures are so
severe that there is pulmonary
dysfunction or there is an underlying disorder (bone or spinal
tumor) that is causing the scoliosis.
X-rays need to be taken of the entire spine to help determine the
degree of curvature. Treatment
modalities are based on the degree of curvature: 20° or less
does not normally require treatment;
20° to 40° is an indication for bracing in an immature child;
and 40° and greater is resistant to
bracing and requires surgical fixation with spinal fusion, which
is best done at special centers.
59. (D) Legg–Calvé–Perthes disease is also known as avascular
necrosis of the proximal femur. It
typically occurs in children between 4 and 8 years old and
persistent hip pain is the main
symptom. On examination, the clinician notices a limp and/or
limitation of motion of the affected
Dr.Wahid Helmi - Dymiate -Egypt

hip. Radiologic examination demonstrates the necrosis with


effusion and joint space widening
with a negative aspirate. Treatment involves surgical hip
replacement. Slipped capital femoral
epiphysis (SCFE) is due to the displacement of the proximal
femoral epiphysis owing to
disruption of the growth plate. The head is normally displaced
medially and posteriorly relative
to the femoral neck. It typically occurs in adolescence,
specifically obese males, and can also be
associated with hypothyroidism. SCFE usually occurs after
direct trauma to the hip or a fall.
Patients complain of vague symptoms at first that progress into
pain of the hip or of the knee. On
examination, there is decreased internal rotation of the hip that
can be confirmed by lateral X-ray
of the hip. Septic hip arthritis is not common in children
between the age of 5 and 12 years. The
legs are held in external rotation to minimize pain and will have
a positive aspirate.
Osteochondritis dissecans typically presents in the knee, elbow,
and talus and is characterized by
a wedge-shaped necrosis of bone.
60. (B) Gamekeeper thumb is a result of damage to the ulnar
collateral ligament during forced
abduction of the metacarpophalangeal joint, an injury that is
most commonly seen in skiers. An
avulsed fragment may or may not be seen on radiologic
examination. If it is smaller than 2 mm,
there is no fragment, a thumb spica cast can be used as seen in
patients with no fragment. If the
fragment is larger than 2 mm, surgery is required. Mallet finger
is an avulsion of the extensor
tendon and occurs in ball-handling sports. Boxer fracture is a
distal neck fracture of the 5th
Dr.Wahid Helmi - Dymiate -Egypt

metacarpal. Scaphoid fractures are due to hyperextension of the


wrist injuries and present with
pain in the anatomic snuffbox and swelling.
61. (A) Nursemaid elbow is the subluxation of the radial head
due to a child or infant being lifted or
pulled by the hand. The patient will present with the elbow
pronated and painful and he or she
will not bend the elbow. During the radiologic examination, the
dislocation is usually reduced by
placing the elbow in full supination and moving it slowly from
full extension to full flexion. This
typically provides immediate relief of pain and a sling may be
given for comfort for a couple of
days. Otherwise, X-rays are normal. Child protective services
should be considered if this is a
recurrent problem or if there are other associated signs and
symptoms of battery. There is no
need for orthopedic referral unless reduction is not commonly
done in your setting.
Immobilization of the elbow is not recommended, because the
patient then may have to recover
from frozen shoulder.
62. (A) This patient is presenting with signs and symptoms of
primary nocturnal enuresis, which is
the wetting only at night during sleep without any sustained
period of dryness. It is mainly
considered a parasomnia occurring in deep sleep. The incidence
of enuresis is higher in boys, is
typically related to a developmental delay, and most children
become continent by adolescence.
Patients need to be tested for structural abnormalities and
infections, in addition to neurologic
diseases, diabetes mellitus and insipidus, and seizure disorders.
Treatment includes limiting
Dr.Wahid Helmi - Dymiate -Egypt

liquids at bedtime and routine bathroom training during the day.


If these are unsuccessful, the next
option is a bed-wetting alarm. This device is attached to the
child's undergarment and vibrates
when the child is wet to arouse the child to be aware of their
need to urinate. If the alarm is
unsuccessful, then the next step is medication—DDAVP
(desmopressin acetate) or imipramine.
63. (D) Nephroblastoma also known as Wilms tumor typically
presents with an asymptomatic
abdominal mass noticed by the parent or an increasing size of
the abdomen. On examination, the
mass feels smooth and firm, is well defined, and usually does
not cross the midline. Gross
hematuria may be present, but rare, and some patients have
microscopic hematuria when tested.
Wilms tumor accounts for approximately 5% of cancers in
children younger than 15 years. Wilms
tumor arises from the kidney and the average age at diagnosis is
4 years. Ultrasound and CT of
the abdomen can be used to confirm the presence of an intra-
abdominal mass. Treatment includes
exploratory abdominal surgery for removal and staging with a
mixture of chemotherapy.
Intussusception (telescoping of the small intestine) typically
presents in an infant with
paroxysmal abdominal pain, vomiting, and diarrhea that may
progress into bloody stools.
Volvulus is normally the result of intestinal malrotation that
causes occlusion of the superior
mesenteric artery and eventual bowel necrosis. Infants typically
present within 3 weeks of life
with bile-stained vomiting and bowel obstruction.
Dr.Wahid Helmi - Dymiate -Egypt

64. (C) Infant girls presenting with signs of precocious puberty


need to be screened for congenital
adrenal hyperplasia (CAH). CAH most commonly presents with
pseudohermaphroditism in
females—urogenital sinus, enlarged clitoris, or other signs of
virilization. In males, there tends to
be isosexual precocity in older males and salt-losing crisis in
infant males. Both children show
increased linear growth and skeletal maturation. The most
common type of CAH is a deficiency
in the enzyme 21-hydroxylase and laboratory tests demonstrate
increased urinary and plasma
androgens (DHEA, androstenedione). There may be elevated
progesterone, but typically there is
no effect on estrogen. There is also decreased aldosterone and
elevated urinary ketosteroids.
There is also no effect on the levels of leuteinizing hormone or
follicle-stimulating hormone.
Treatment usually involves glucocorticoids, mineralocorticoids,
and reconstructive surgery, if
needed.
65. (A) Growth hormone (GH) deficiency is defined as a
decreased growth velocity, delay in
skeletal maturation, absence of other explanations for poor
growth (lack of intake), and
laboratory tests demonstrating decreased GH secretion. Etiology
of GH deficiency can be
congenital, genetic, acquired, or idiopathic, which is the most
common. Infants usually have a
normal birth weight and may have a slightly decreased length. In
addition, most infants present
with other endocrine deficiencies like hypoglycemia,
hypothyroidism, and/or adrenal
Dr.Wahid Helmi - Dymiate -Egypt

insufficiency. Children may present with truncal adiposity


because growth hormone promotes
lipolysis. Serum GH or intrinsic growth factor levels may or
may not be decreased. In patients
who do not have a demonstrated decrease in these hormones, a
trial period with GH is indicated.
These patients and positive GH-deficient patients receive a
once-daily subcutaneous injection of
recombinant human GH. Congenital hypothyroidism typically
presents with short stature
(typically noted after the 4-month newborn visit), delayed
epiphyseal development, delayed
closure of fontanelles, and retarded dental eruption in addition to
other signs of hypothyroidism.
Cushing disease typically presents with truncal adiposity with
thin extremities, muscle wasting,
decreased growth rate, and moon facies. Laboratory results
show elevated adrenocorticosteroids
both in urine and serum, hypokalemia, eosinopenia, and
lymphocytopenia. Typically, in patients
younger than the age of 12, Cushing disease is secondary to
administration of ACTH or
glucocorticoids. Congenital adrenal hyperplasia typically
presents with pseudohermaphroditism
in females or salt-losing crisis in males with or without
isosexual precocity. There is an
increased linear growth and advanced skeletal maturation.
66. (C) In patients with idiopathic thrombocytopenic purpura,
treatment options should be initiated
when platelet counts fall below 20,000, regardless of whether
there is active bleeding or not.
Without active bleeding the treatment options include
prednisone 2–4 mg/kg/24 hours for 2
Dr.Wahid Helmi - Dymiate -Egypt

weeks; IV immunoglobulin 1 g/kg/24 hours for 1 to 2 days, or


IV anti-D 50–75 μg/kg/dose for
Rh-positive patients. Splenectomy is indicated for life-
threatening bleeding. There is currently no
indication for platelet transfusion and none of the above
treatments are considered optimal,
because in the majority of children, it will resolve on its own
within 6 months.
67. (D) This patient is presenting with signs of congestive heart
failure. The most common causes of
heart failure in children/adolescents are due to acquired heart
disease. Congenital heart diseases,
such as malformations of the heart— patent ductus arteriosus and
ventricular septal defects, are
the most common causes of heart failure in infants–toddlers, and
are second to fluid overload in
neonates.
68. (D) This patient has the classic presentation of erythema
multiforme major or Stevens–Johnson
syndrome. The most common causes in children of erythema
multiforme are medications and
Mycoplasma pneumoniae. Of the antibiotics listed, the one most
commonly causing Stevens–
Johnson syndrome is sulfonamide followed by penicillin and
tetracycline. The most common
medications causing SJS in children are nonsteroidal anti-
inflammatory drugs.
69. (A) Infection with Human parvovirus B19 (also known as
fifth disease) resulting in the slapped
cheek appearance, can also cause aplastic anemia. This is
because the virus infects the
precursors of erythrocytes and halts erythropoiesis. Recovery is
typically spontaneous with an
Dr.Wahid Helmi - Dymiate -Egypt

occasional transfusion for severe anemias. (Scott, 2006, pp. 701-


702; Levin and Weinberg 2009,
pp. 1100-1101)
70. (D) As this patient is presenting with signs and symptoms of
herpes zoster within the appropriate
time frame for antiviral treatment, the treatment for this patient
would be oral acyclovir. NSAIDs
may help with the pain associated from zoster but will not
hasten the length of the course of the
virus as acyclovir will. Varicella-Zoster immunoglobulin
(VZIG) is indicated for prophylaxis in
exposed individuals who are immunocompromised.
71. (D) Munchausen syndrome by proxy is when the
parent/caregiver is causing or complaining of
signs and symptoms of illnesses in his/her children. While it is a
form of child abuse and should
be treated as such, it is also considered a psychiatric disorder
where the parent/caregiver is
desiring to be in the sick role. The most common signs or
symptoms that should raise the level of
suspicion for Munchausen syndrome by proxy are: recurrent
polymicrobial sepsis, recurrent
apnea, chronic dehydration, or other unexplained symptoms like
vomiting, diarrhea, seizures,
failure to thrive, and hypoglycemia. The remaining signs are
seen in classical physical child
abuse.
72. (A) Menomune is a tetravalent vaccine that is indicated for
prevention of meningococcemia
caused by the bacterium Neisseria meningitides. Menomune is
indicated for patients between 11
and 12 years of age and at 15 years of age. It is also indicated
for college freshmen in
dormitories, military recruits, microbiologists working with the
bacterium, persons with
Dr.Wahid Helmi - Dymiate -Egypt

complement deficiency and functional or anatomic asplenia, and


for those traveling to countries
with endemic disease. Guillain–Barré is a rare complication of
the Menomune vaccine, and if a
patient has a history of developing it, is the only relative
contraindication other than a known
reaction to a previous administration of the vaccine rubber latex
and diphtheria toxoid severe
allergic reaction
73. (A) The most likely diagnosis is pediculosis. This parasitic
infestation is most commonly seen in
the young school-aged child, and more often in female and
Caucasian children. The pediculosis
louse lives in the hair and on the scalp and intermittently
“bites” into the skin to feed. Discrete
urticarial papules or erosions may arise at the bite site. By
visualizing the live louse on the scalp,
or in the hair, one can easily make the diagnosis. However, the
louse may be difficult to see, as it
is only 1 to 3 mm in size. Otherwise, nits, or the casings of the
eggs laid by the louse, can often
be seen on the proximal portion of the hair shaft. The nit adheres
to the hair shaft and is often
difficult to remove. Brown nits are representative of current
infestations and white nits past
infestations. Treatment of head lice can be difficult due to the
increasing resistance to some of the
current treatment options. First-line treatment includes
permethrin (5%) and permethrin-based
products. Secondary treatment options for resistant infestations
may include Malathion (0.5%).
Regardless of treatment, viable ova should be removed by
combing the patient's wetted hair with
Dr.Wahid Helmi - Dymiate -Egypt

a finely toothed comb until all are removed. Ketoconazole


cream and tar-based shampoos are
utilized in fungal and seborrheic dermatitis infections. Silver
sulfadiazine cream is a topical
antibiotic.
74. (B) Scabies, Sarcoptes scabiei, is the most common
arthropod infestation of children, and it is
highly contagious. However, its presentation varies widely and
is dependent on the child's age,
duration of the infestation, and immune status. Most often, the
presenting complaint is severe
intermittent itching. The linear papule or burrow commonly
associated with scabies is often
difficult to identify. Instead, most children will present with
eczematous eruptions of red,
excoriated papules and nodules. Usually, the distribution of the
papules are the most diagnostic
finding, and may include the web spaces of the fingers and toes,
axillae, umbilicus, groin, penis,
and the instep of the feet. Usually, in older children and adults,
the face and scalp are spared. The
treatment for scabies is a 12-hour application of permethrin 5%
lotion. In addition, the parents
and all caregivers should be treated at the same time. Clothing
and bedding should be washed
and dried (heat kills scabies). The family should also be
educated in the treatment and prevention
of future infestations. Moreover, they should be advised that the
itching associated with scabies
could persist for 7 to 14 days after successful treatment.
Pediculosis is an infestation of louse in
the hair. Tinea corporis is a fungal infection of the torso or
“ring worm” and presents with annual
Dr.Wahid Helmi - Dymiate -Egypt

scaly plaques with central clearing and pustules. Herpes simplex


typically presents with grouped
vesicles on erythematous base and is painful. It typically is
located in the lips, eyes, cheeks, or
hands of children.
75. (D) Seborrheic dermatitis is common in all age groups. In
infants, this inflammatory skin disease
is often manifested as thickened, yellowish white, scaly, waxy
appearing skin of the scalp and
commonly involves the postauricular areas and the forehead.
The more common name is “cradle
cap.” Cradle cap is a self-limiting disease of infants and
resolves by the child's first birthday. In
all ages, the scalp scale can be treated by shampooing with zinc
pyrithione (Head and
Shoulders), selenium sulfide 1% to 2.5%, salicylic acid (Tsal),
or ketoconazole (Nizoral). The
primary lesion in lichen planus presents on the flexor surfaces
and is characterized by pruritic
papules that are polygonal and flat-topped. Pityriasis rosea
typically presents with the “herald
patch” that is a solitary pink, round patch with some central
clearing typically found on the torso.
The rest of the eruption is described as papulovesicular and
develops a Christmas tree pattern.
Contact dermatitis usually presents with red patches and plaques
with scales and is localized to
the area exposed to the irritant.
76. (B) This presentation is typical for granuloma annulare,
which is a benign skin disorder, and
treatment is not warranted. It is most commonly seen in children
aged 6 to 10. The red to brown
lesions are annular or circinate. These asymptomatic lesions are
often confused with tinea
Dr.Wahid Helmi - Dymiate -Egypt

corporis. The lesions will disappear on their own over a couple


of years.
77. (D) Liquid nitrogen is the treatment of choice for a single
isolated wart. Forty percent salicylic
acid in a plaster application is the most effective treatment of
large and painful warts.
Electrosurgery, burning laser surgery, and other destructive
treatments should be avoided
because of the potential for scarring and subsequent problems
often associated with scars, as
well as the possible recurrence of the wart after destructive
treatment.
78. (B) Topical keratolytic agents applied to the skin either as a
single, once a day agent or in
combination regime (retinoic acid cream, azelaic acid, and
adapalene) once a day in the evening
and benzoyl peroxide gel in the morning, will control
approximately 80% to 85% of cases of
adolescent acne. When treating inflammatory acne, papular or
pustular, a daily topical antibiotic
such as tetracycline, minocycline, or erythromycin can be used
in addition to a daily keratolytic.
The oral retinoid, 13-cis-retinoic acid (isotretinoin), Accutane is
reserved for treating
nodulocystic acne (severe cystic acne). This medication is not
effective for the milder forms of
acne such as comedonal. Isotretinoin is teratogenic in women of
childbearing age and has other
side effects. Therefore, strict adherence to FDA guidelines is
required
79. (A) Clavicular factures are the most common fractures in
infants and children. In newborns, this
fracture is usually unilateral and often occurs after a difficult
delivery. Many times no treatment
Dr.Wahid Helmi - Dymiate -Egypt

is required or a figure-of-eight bandage can be used. For infants


and children, a sling can be
used. The bump that can be seen after fracture consolidation will
usually resolve in a few months
to a year. The next most common fractures are of the
extremities, humerus being the most common
and then the femur, but still much less common than the
clavicle.
80. (B) Osgood–Schlatter disease is caused by microfractures of
the patellar ligament where it
inserts into the tibial tubercle. This condition usually occurs in
the preteen and adolescent years,
and is more common in males than females. The history of
injury can be vague and the patient
may not remember a specific injury that precipitated the pain.
Often, the pain progresses to the
point of interference of even routine physical activities. X-rays
may or may not show any
abnormalities. Upon X-ray, Type I disease appears normal, but
Type II will reveal fragmentation
of the tibial tubercle. Often, after healing there will be
enlargement of the tibial tubercle.
Generally, treatment consists of rest, limitation of activities, and
isometric exercises.
Chondromalacia patellae can only be diagnosed under an
arthroscopic examination, not on the
basis of clinical features. Patellofemoral overuse syndrome
presents with medial knee pain and
subpatellar pain. Additional signs are swelling and crepitus in
the knee and it is more common in
females than males. It is diagnosed by increased Q-angles
(anterosuperior iliac spine through
center of patella to tibial tubercle). Subluxation of the patella or
dislocation is more common in
Dr.Wahid Helmi - Dymiate -Egypt

adolescent girls and the patient presents with acute knee pain.
The knee is in flexion with a mass
lateral to the knee and with absence of the bony prominence of
the patella (flat). X-ray confirms
the dislocation.
81. (C) Enterobiasis or pinworms is a worldwide infection that
affects people of all ages and
socioeconomic levels. It especially affects children. The classic
manifestation of this problem is
nocturnal anal pruritis and sleeplessness. The sleeplessness may
be secondary to the migration of
female worms to the perianal area to lay eggs, during which the
tape may pick up the larvae.
Transmission of the worms occurs when children ingest the eggs
that are present on their hands
(from scratching), in the bedclothes, or in house dust. After
hatching in the stomach, the larvae
migrate to the cecum where they mature into adults. The
treatment of choice for pinworms is
pyrantel pamoate or mebendazole. Albendazole may also be
used. For eradication of this
parasite, often the entire family must be treated at once. Ascaris
is a helminthiasis infection that
is ingested and excreted in the stool. Diagnosis is made by stool
examination for the
characteristic eggs. Hookworms are found in warm, damp soil
and penetrate the skin. From there
the infection can spread to the lungs where they ascend into the
trachea to be swallowed and live
in the intestine. Diagnosis is made by stool examination for the
eggs. Whipworm is ingested from
the soil and lives in the intestine; detection is also made by egg
in the feces.
82. (A) Juvenile idiopathic arthritis (JIA) presents as three
distinct types. The types are based upon
Dr.Wahid Helmi - Dymiate -Egypt

clinical manifestations during the first 6 months of the illness.


The most common type is
pauciarticular as presented by this 13-year-old boy in the
scenario mentioned. Second is
polyarticular disease with five or more joints being affected, and
the third is systemic onset of
disease that begins with high spiking fevers that are often
associated with a rash that comes and
goes with the fever elevations. It is recommended that patients
with pauciarticular JIA have an
ophthalmologic evaluation and slit lamp examination every 3
months, if the antinuclear antibody
test (ANA) is positive and every 6 months, if the ANA is
negative, for 4 years after the JIA is
identified to catch iridocyclitis (untreated results in blindness).
Lyme arthritis usually presents
with a monoarticular rash that typically affects the larger joints,
without morning stiffness.
Enteropathic arthritis is associated with gastrointestinal
symptoms occurring simultaneously as
lower extremity arthritis. It encompasses Reiter syndrome,
reactive arthritis (eg, postsalmonella,
shigella), and arthritis associated with celiac disease and
inflammatory bowel
disease. Psoriatic arthritis is the arthritis accompanying the
dermatological disorder of psoriasis.
The build-up of epidermal cells over the joints causes
inflammation and thickening that results in
arthralgia.
83. (B) Bacterial conjunctivitis is often unilateral and presents
with a mucopurulent discharge.
Common bacterial causes of this problem include nontypable
Haemophilus, Streptococcus
pneumoniae, Moraxella catarrhalis, and Staphylococcus aureus.
These infections usually
Dr.Wahid Helmi - Dymiate -Egypt

respond to topical antibiotics such as sulfacetamide and


erythromycin. Systemic treatment is
indicated for conjunctivitis caused by chlamydia trachomatis,
Neisseria gonorrhea, or
Neisseria meningitides. Allergic conjunctivitis is usually
associated with moderate to severe
itching of the eyes and clear mucoid drainage. Viral
conjunctivitis is usually associated with
minimal itching, profuse tearing, and minimal clear mucoid
drainage. While reactive arthritis
typically presents with a conjunctivitis, it is also concomitantly
present with arthritis and
urethritis.
84. (C) Dacryocystitis, whether acute or chronic, is usually
secondary to bacterial infections. It
presents as an acutely inflamed swelling and tender area over
the lacrimal sac just medial and
inferior to the inner canthus of the eye. Because the lacrimal sac
is inflamed and blocked there is
tearing and usually purulent discharge from the eye. There may
also be an orbital cellulitis.
Treatment consists of oral and topical antibiotics and warm
compresses, and surgical drainage
may also be indicated. After the acute episode and for chronic
cases, surgical correction of the
nasolacrimal obstruction is required. Anterior uveitis typically
presents with pain, photophobia,
blurred vision, and injection without exudates. Blepharitis is an
inflammation of the lid margin
that presents with crusty debris along the lashes. Unless there is
a concomitant conjunctival
infection there is typically no injection noted.
85. (B) While halitosis can be caused by pharyngitis, sinusitis,
and poor hygiene, the most common
Dr.Wahid Helmi - Dymiate -Egypt

cause of halitosis in children is a nasal foreign body. Seeds and


beads are the leading objects
inserted into the nose. If not promptly removed, they can cause
nasal obstruction, infection,
rhinorrhea, bleeding, halitosis, or a foul smell. They are usually
easy to remove, but if there is
difficulty in removing the foreign body, the child should be
referred to an otolaryngologist for
definitive care. Tobacco use in adolescents is a common cause
of halitosis. Dental disease is the
most common cause of halitosis in adults.
86. (C) Most cases of epistaxis in the anterior portion of the
nose are caused by digital trauma (nose
picking) or some other mechanical cause such as nose blowing
or repeated nose rubbing. Other
causes may include incorrect use of steroid nasal sprays.
Examination of the anterior nose will
usually reveal irritation of the Kiesselbach area. Less than 5% of
recurrent nosebleeds are
caused by bleeding disorders. Choanal atresia, unilateral, usually
appears as a chronic nasal
discharge that may be mistaken for chronic sinusitis. Foreign
bodies typically present with
purulent discharge instead of bleeding.
87. (A) Cat scratch disease (CSD) is caused by the gram-
negative bacillus, Bartonella henselae.
The disease is more common in the fall and winter months and
more males than females are
affected. Typically (approximately 90%), patients report
handling a cat or kitten and up to 70%
will report a scratch by a cat. The most common complication of
CSD is encephalitis. About half
of the patients with CSD will develop a primary cutaneous
papule at the site of inoculation, most
Dr.Wahid Helmi - Dymiate -Egypt

often (approximately 50%) on the hands or upper extremities, 3


to 10 days after the exposure.
Regional lymphadenopathy will usually develop in about 1 to 7
weeks after the cutaneous lesions
and will affect the nodes draining the site of the scratch or bite.
The affected lymph nodes may be
inflamed and are usually tender. Occasionally, the involved
nodes may suppurate. The
lymphadenopathy resolves in about 2 months, but may last as
long as 4 to 8 months. Treatment is
usually not indicated for this self-resolving disease. However,
suppurative lesions may need to
be aspirated for pain relief. It has been shown that 5 days of
treatment with azithromycin has
helped to speed recovery for some patients. Because Hodgkin
disease involves the lymph nodes,
it should be considered as a differential diagnosis when
evaluating a child for CSD. However, it
typically presents as a cervical lymphadenopathy. Fifth disease
(erythema infectiosum) is a
childhood disease caused by the human parvovirus. This
common community-acquired disease
does not usually require treatment, but respiratory isolation is
recommended for 7 days following
the onset of symptoms. The initial stage of the disease presents
as red cheeks that appear to be
“slapped” or “slapped cheeks” with circumoral pallor.
Osgood–Schlatter disease is an
orthopedic problem in children. It is the result of repetitive
microtraumas to the patellar ligament
at its point of insertion into the tibial tubercle. Usually, rest and
anti-inflammatory medications
are helpful in alleviating the pain associated with this condition.
Dr.Wahid Helmi - Dymiate -Egypt

88. (B) Hookworm (Ancylostoma duodenale and Necator


americanus) infections, if severe, can
cause iron deficiency anemia. Abdominal discomfort, weight
loss, and ova in the stool are more
commonly associated with these nematodes. Both types of
human hookworms are found in tropic
and subtropical climates, which include the southeastern United
States, primarily the coastal
areas. The larva of this parasite is passed in the feces and
incubates in warm, damp soil when
they hatch into larvae. The larvae penetrate directly into the skin
of humans, enter the
bloodstream, and migrate to the lungs. From the lungs they
move up to the trachea and are
swallowed. Once swallowed, they mature in the intestines. The
worms attach their mouth to the
mucosal lining of the intestine where they suck blood and shed
new ova. Mild infections are
usually asymptomatic, but severe infestations can cause anemia.
Treatment for the infestation is
achieved with albendazole. In severe cases of anemia, parenteral
iron or transfusion may be
indicated. Pinworms are associated only with localized pruritus,
specifically the anus. Treatment
may help recurrent urinary tract infections in some young girls
when the pinworm has infected the
urethra. Ascariasis is usually asymptomatic; however, in severe
cases it may be associated with
anorexia, diarrhea, vomiting, weight loss, and abdominal pain.
Whipworm is also asymptomatic
until the infection is severe, with general gastrointestinal
symptoms—pain, diarrhea, and mild
abdominal distention. Eosinophilia may also be present,
although slight.
Dr.Wahid Helmi - Dymiate -Egypt

89. (C) Appearing in 60% to 80% of cases, the characteristic


rash may not be present in all cases of
acute Lyme disease. Following the bite of a deer tick (Ixodes
species), infected with the
spirochete Borrelia burgdorferi, an erythematous ring forms
around the bite site and spreads
outward. The ring may have a raised border and usually a clear
center. The ring can attain a
diameter of up to 20 cm. Multiple rings may form and they can
form at sites distal to the original
bite site. If left untreated, the rash will usually resolve within 3
weeks. Erythema migrans is a
minimally tender to nontender, nonscaly rash that persists longer
than many of the other
erythematous rashes of childhood.
90. (C) Chlamydial infections are the most common cause of
conjunctivitis in newborns in
developed countries. Other causes of ophthalmia neonatorum
include reactions to silver nitrate
prophylaxis, other bacterial infections such as gonococcal or
staphylococcal, or viral organisms
such as adenovirus or echovirus. Chlamydia trachomatis causes
conjunctivitis and pneumonia in
neonates. Treatment for chlamydial conjunctivitis should be
with systemic erythromycin to treat
the conjunctivitis and as prophylaxis against pneumonia.
91. (D) Seizures have not been associated with Lyme disease.
Neurologic manifestations occur in up
to approximately 20% of patients with Lyme disease. Primarily,
these are Bell palsy,
lymphocytic, aseptic meningitis, and polyradiculitis. Cranial
neuropathies, such as Guillain–
Barré syndrome and ataxias are less common. Additional
neurologic manifestations include
Dr.Wahid Helmi - Dymiate -Egypt

peripheral neuropathy, pseudotumor cerebri, and encephalitis. If


untreated, most neurological
symptoms are self-limited but some will persist or become
permanent.
92. (D) Oral candidiasis (thrush) is very common in the first few
weeks of infancy. The diagnosis is
usually done by visual inspection and does not usually require
further laboratory testing. On
visual examination, white, creamy plaques are found on the
buccal mucosa and occasionally the
gingival and lingual mucosa. For this age group, direct topical
application of nystatin in oral
suspension to the lesions should suffice. If the lesions are
resistant to treatment or if they occur in
older children, consideration should be given to the possibility
of the patient being
immunocompromised. All sources of candida, such as toys and
bottle nipples, should be
sterilized daily. Herpangina and hand–foot–mouth disease are
ulcerating lesions of the oral
cavity due to viruses and are self-limiting, but can be very
painful. Leukoplakia is a precursor
lesion to oral cancer, seen most commonly in oral tobacco users.
93. (B) Bipolar affective disorder is the most likely diagnosis for
this patient. Although ADHD,
bipolar disorder, and conduct disorder share many similarities in
behavior disorders, such as
varying degrees of school and behavior problems, defiant
attitude, and distractibility, the
obsession with ideas (in this case, science fiction movies) is not
present in ADHD and conduct
disorder. The mood swings described here, as depression and
elation are consistent with bipolar
Dr.Wahid Helmi - Dymiate -Egypt

disorder, which is confirmed by the presence of hallucinations.


Hallucinations, when considering
a differential diagnosis in a behavior disorder, are diagnostic for
bipolar disorder. In up to 70%
of patients with bipolar disorder, their first symptom of the
disorder may be depression.
However, hallucinations are not typically a manifestation of
depression.
94. (D) Rocky Mountain spotted fever (RMSF) is the most
common rickettsial infection in the United
States, especially in the eastern, southeastern, and western
states, and it is very common in 5- to
9-year-old children. A known tick exposure may or may not be
documented. Most exposures to
ticks carrying Rickettsia rickettsii, the causative organism of this
disease, occur in the warmer
months of April to September when victims are most likely to
participate in outdoor activities in
wooded areas. The incubation period of RMSF is 3 to 12 days
(mean 7) after a tick exposure.
The tick must be attached for 6 hours or greater in order to
transmit the disease. Clinical
presentation includes fever, often 40°C, myalgias, headache,
and less characteristic, red-rose
macular or maculopapular rash. The rash usually appears within
2 to 6 days, after the fever. The
rash is especially prevalent on the palms, soles, and extremities.
After several days, the rash,
which starts peripherally and spreads centrally, becomes
petechial. Conjunctivitis, edema,
splenomegaly, meningismus, and confusion may occur. Up to
5% to 7% of patients with RSMF
will die, and therefore, delays in treatment should be avoided.
Treatment for children is
Dr.Wahid Helmi - Dymiate -Egypt

doxycycline, regardless of age and the possible side effect of


stained teeth. In endemic areas,
treatment should be started early and is often based on suspicion
alone, and prior to the
appearance of the rash. Endemic typhus (murine typhus) is not
transmitted by ticks but instead by
the fleas from infected rodents. The rash of endemic typhus
differs from that of RMSF in that it
does not involve the palms and soles. Q fever is spread by
inhalation instead of ticks. The cause
of this rickettsial disease is Coxiella burnetii hosted by domestic
animals including dogs, cats,
cattle, and sheep. Unpasteurized milk from infected animals
may also be a source of this
infection. One form of human monocytic ehrlichiosis is carried
by ticks that have fed on infected
hosts that may include deer, wild rodents, and sheep, most
commonly in the southeast, north, and
south central United States. The presentation is usually a viral
syndrome without any rash.
Although this is usually a self-limiting disease, deaths do occur
in children; therefore, treatment
should be carried out with the antibiotic of choice, doxycycline,
regardless of side effects.
95. (D) In young children, respiratory syncytial virus (RSV)
accounts for more than 70% of
bronchiolitis, approximately 40% of the cases of pneumonia,
and about 10% of cases of croup.
This seasonal disease occurs in the winter and early spring
months of the year. More than 50% of
children have been infected with RSV by age 1, and by the age
of 2, almost all children have
been infected. Reinfection commonly occurs but is mild.
Adenovirus infections, though common
Dr.Wahid Helmi - Dymiate -Egypt

in early childhood, only account for approximately up to 10% of


all respiratory diseases. The
peak incidence of adenovirus respiratory infections occurs in the
spring, summer, and early
winter. Human parvovirus infection is typically seen in school-
aged children. This disease is
characterized by the “slapped-cheek” appearing rash on the
face that appears about 10 to 17 days
following the infection. About 2 days after the appearance of
this facial rash, a similar rash
appears on the extremities, trunk, neck, and buttocks. The rash
often persists for a few days to a
few weeks (average of 10 days) and often will recur with
exposure to bathing in warm water,
exercise, sunlight, and stress. Parainfluenza viruses fall into four
categories and are responsible
for the majority of cases of croup (65%), laryngitis (50%), and
tracheobronchitis (25%). Types 1
to 3 occur as seasonal outbreaks with types 1 and 2 in the fall
and type 3 in the spring and
summer. Type 4 is an endemic virus. Clinical symptoms of these
viruses include laryngotracheitis
(croup), laryngitis, bronchiolitis, and less commonly pneumonia
(especially in
immunocompromised children).
96. (B) Asthma, in this case exercise-induced, is the most likely
cause of this problem. The
symptoms commonly associated with acute exacerbations of
asthma include wheezing, cough,
dyspnea, and chest pain. Some symptoms that might be
suggestive of asthma include exerciseinduced
cough, nighttime cough, cough after cold air exposure, and
cough after laughing. Airway
Dr.Wahid Helmi - Dymiate -Egypt

foreign bodies, though not common, are an acute problem that


may present as sudden cough,
choking, and wheezing. Cystic fibrosis (CF) is the most
common, lethal, genetic disease affecting
the Caucasian population. Up to 50% of patients with CF are
diagnosed in infancy, but others
may not be diagnosed until adolescence or adulthood. Chronic
or recurrent cough should be an
indicator for consideration of CF as a differential diagnosis.
Laryngomalacia is the most common
cause of stridor in infants. It is the incomplete development of
the cartilaginous support of the
laryngoglottic structures. This congenital condition is usually
self-limiting and occurs most
commonly in infants at or just after birth. The inspiratory
collapse of the epiglottis or arytenoid
cartilages is heard as stridor.
97. (B) Autism is the most likely diagnosis for this child. The
signs of autism often present before the
second year of life such as the child's failure to respond to their
name, failed speech
development, and appearing self-absorbed and withdrawn in the
presence of other children or
adults. Often in childhood, autistic children may develop
ritualistic behaviors and intense
interests that if interrupted may cause tantrums and rages. When
speech does begin to develop, it
may be nonsensical: reversal of speech patterns, echolocation,
and other abnormal patterns.
Goals of treatment include early intervention to address
behavior and communication skills.
ADHD is characterized by easy distractibility, inattention, and
overactivity. Estimates for the
presence of ADHD in school-aged children range from 2% to
20%. Fragile X syndrome is the
Dr.Wahid Helmi - Dymiate -Egypt

most common cause of functional mental retardation. This


syndrome, affecting approximately 1 in
1,250 males, is caused by a trinucleotide expansion (CGG
repeated sequence) in the Fragile X
Mental Retardation I (FMR1) gene. Fragile X syndrome is
characterized by a wide range of
symptoms, which may include language delay, hyperactivity,
autistic behavior, and variable
levels of mental retardation. Schizophrenia is usually detected in
adolescence, with prepubertal
onset occurring rarely. Patients may initially present with
somatic or social behavior problems.
Schizophrenic children and adolescents often have the same
symptoms as adults, such as
hallucinations, bizarre thought processes, and rambling speech.
98. (A) Choanal atresia, whether unilateral or bilateral, is a nasal
obstruction that occurs relatively
rarely in newborns. If bilateral choanal atresia occurs at birth, it
causes a respiratory distress
that requires immediate treatment (due to infants being obligate
nose breathers) by placing an
oral airway and subsequent surgical correction. Unilateral
choanal atresia can present as a
chronic, single-sided, nasal discharge that may not appear until
later in childhood. Meconium
ileus, intestinal obstruction secondary to inspissated meconium,
occurs in approximately 10% of
newborns with cystic fibrosis. Cystic fibrosis affects
approximately 1 in 2,500 live Caucasian
births, and is a leading cause of death in young adults. Nasal
infections may occur secondary to a
furuncle (infected hair follicle) in the anterior nares or as a nasal
septal abscess following
spread of a furuncle. Common causes of nasal infections include
picking at the nose and pulling
Dr.Wahid Helmi - Dymiate -Egypt

out nose hair. Nasal polyps are uncommon in children younger


than age 10, and when they do
occur it is usually in older children and adults with allergic
rhinitis.
99. (A) It is estimated that 1% to 5% of adolescents are affected
by anorexia nervosa. There are two
types of anorexia nervosa. The first is the nonpurging type when
patients restrict their total
caloric intake and the second involves binge eating and purging
in association with the restrictive
dietary habits. Otherwise, intensive exercise regimes may be
used as a means to control weight.
Anorexia nervosa occurs in boys but is more prevalent in girls
(2:1). The specific etiology of
this familial problem is unknown; there are genetic and
environmental factors. DSM-IV criteria
also include refusal to keep weight at 85% of ideal weight,
intense fear of gaining weight even
though underweight amenorrhea and disturbance in the way
one's body shape is experienced.
100. (C) Obesity is the number one nutritional disorder in
children in the United States. In 2004, 17%
of American children aged between 9 and 19 were considered
obese. Risk factors for obesity
include other obese family members and infants born to diabetic
mothers. Associated
environmental factors include sedentary lifestyle, total caloric
intake, television watching, and
computer games. All are considered contributory factors in
childhood obesity. Binge eating
disorder is a relatively new eating disorder category. It is most
frequent in overweight or obese
individuals. This disorder includes recurrent episodes of binge
eating (eating more than most
Dr.Wahid Helmi - Dymiate -Egypt

individuals would in a 2-hour period) and a sense of lack of


control over the impulse to eat,
marked distress over the episode at least 2 days a week, and is
not associated with regular
compensatory activity such as purging or fasting. Folate
deficiency anemia (megaloblastic) can
occur in infants within a few weeks after birth. This deficiency
may be a result of malabsorption,
low dietary intake such as with goat's milk or home-prepared
formulas that have been sterilized
by heating, or formulas based on pasteurized milk. Infants who
are breastfed or given
supplemented cows’ milk formulas do not have a problem with
folate deficiency. In children,
rickets is most commonly a result of poor dietary intake of
vitamin D and inadequate exposure to
direct sunlight. Vitamin D sources include milk, cheese, and
baby formula. Vitamin D in humans
is produced by activation of its inactive precursors in the skin
after exposure to ultraviolet light.
101. (B) The symptoms for congestive heart failure in infants
are typically failure to thrive,
tachycardia, and poor feeding. These will typically not present at
birth and will be identified by
the 6-month well visit.
102. (A) Bordetella pertussis is a gram-negative bacillus and,
therefore, of all the choices, the
antibiotic with good gram-negative coverage is erythromycin.
The other macrolides,
azithromycin and clarithromycin may also be given for shorter
durations, however they are more
expensive. Ampicillin, amoxicillin, and cephalexin provide
mainly gram-positive coverage.
Dr.Wahid Helmi - Dymiate -Egypt

103. (C) Surgical reduction is the treatment of choice for


incarcerated hernias over 12 hours. At that
point the likelihood that the hernia will manually reduce is very
small and the bowel is becoming
necrotic and needs to be removed as soon as possible. Bilateral
surgical reduction is required
only in the event of two hernias, and there is no place for
prophylaxis surgery for inguinal hernia
repairs.
104. (B) Salmonella species are gram-negative bacilli that are
classified as Enterobacteriaceae,
along with E Coli. While extremely uncommon as an etiology
for meningitis, salmonella can
cause lethal meningitis infections and must be watched. While
there is typically no treatment for
mild to moderate diarrhea from salmonella infections, these
patients should be monitored for
complete resolution. Viral meningitis typically does not have a
positive Gram stain, unless there
is contamination. Corynebacterium and clostridium are gram-
positive bacilli
105. (A) Vitamin K deficiency causes hemorrhagic disease of
the newborn. Vitamin K is one of the
compounds required for conversion of prothrombin, factors VII,
IX, and X of the coagulation
cascade. In addition, proteins C & S are also Vitamin K
dependent. Therefore, the result is an
increased prothrombin time and this would result in an increased
aPTT. There is no effect on
platelets or fibrinogen.
106. (C) The diagnosis of otitis media requires the presence of
middle ear effusion, acute onset of
symptoms, and signs and symptoms of middle ear inflammation.
Presence of the middle ear
Dr.Wahid Helmi - Dymiate -Egypt

effusion can be determined by the bulging of the tympanic


membrane, air-fluid levels, absent
mobility of the tympanic membrane by pneumatic otoscopy, or
otorrhea from perforation. Office
tympanometry can be performed to confirm a diagnosis of
effusion. Tenderness on palpation of
the tragus typically is a sign of otitis externa.
107. (C) While different states have different requirements for
child safety restraints, the most
common guidelines state that infants must be 20 lb and 1 year of
age before switching to forwardfacing
seats. Children between 20 lb and 40 lb should be in front-facing
safety seats, typically
with a 5-point harness; children between 40 and 80 lb may be in
booster seats in which the back
is typically required based on the height of the child. Lastly,
children should be older than 12
years of age and typically at least 80 lb as the front air bags are
dangerous. 108. (D) The most common etiology of
gynecomastia is idiopathic. Occurring in 50 % to 60% of
adolescent males, idiopathic gynecomastia typically is self-
limited. Additional uncommon
etiologies of gynecomastia include liver disease,
hyperthyroidism, illicit drugs (marijuana
heroin), neoplasms (adrenal, testicular), and medications (eg,
antacids, chemotherapy)
109. (A) A urinalysis should be performed because renal disease
is the most common etiology of
hypertension in children. Electrocardiograms and chest
radiography should be considered as part
of the evaluation for end-organ disease as well as an initial basic
metabolic panel to include
serum and creatinine. Although rare, elevated uric acid has also
been shown to cause essential
hypertension in children.
Dr.Wahid Helmi - Dymiate -Egypt

110. (B) The CDC recommends that there are two age ranges for
testing lead in children in the United
States: 9 to 12 months and again at 24 months. These high-risk
areas include poverty-stricken
areas, use of lead paint pottery, lead painted homes (peeling or
cracking), industrial exposures,
and use of diarrhea remedies in Mexico. The CDC recommends
using questions to screen all
children between 6 months and 6 years of age.
th
7 revision (Infectious diseases)
Dr.Wahid Helmi
Consultant Pediatrician
(Egypt-Zarka City –Dymiate)
First PART
1-A 12-yr-old boy with cystic fibrosis experiences an acute exacerbation and is
admitted for intravenous antibiotic therapy. Sputum culture reveals
Pseudomonas aeruginosa.The antibiotic recommended for treatment is:
A-Ampicillin-sulbactam
B-Ceftazidime
C-Cefotetan
D-Ceftriaxone
E-Ciprofloxacin
Answer: B
Ceftazidime
Appropriate single agents for treatment of Pseudomonas aeruginosa infection include
ceftazidime, cefoperazone, ticarcillin-clavulanate, and
piperacillintazobactam.Gentamicin or another aminoglycoside may be used
concomitantly for synergistic
effect.

2-A 2 year old girl presented with a 12-hour history of fever and poor feeding.
The family owned two cats. Her temperature was 39.5°C, and she was irritable,
with no localising signs or skin lesions. A full septic screen was performed.
Cerebrospinal fluid (CSF) showed a neutrophilic pleocytosis and gram-negative
coccobacilli. She was treated with intravenous cefotaxime and gentamicin.
Within 24 hours both CSF and blood cultures showed growth of gram-negative
bacilli.
What is the likely infective organism?
A. Legionella pneumophilia
B. Haemophilus influenzae
C. Pasteurella multicoda
D. E coli
E. Pseudomonas aeruginosa.
Answer: C
(Pasteurella multicoda)
Pasteurella multicoda is an oral commensal of domestic pets known to be an
opportunistic human pathogen after traumatic animal contact.
The most common infections in humans are skin and pulmonary
infections.Pasteurella meningitis occurs at extremes of age (infants), in the
immunecompromised (associated with liver cirrhosis, renal disease and
haematological malignancies) and after traumatic head injury. .

3-Your 13 month old had a high fever for 4 days and then broke out in a red
rash all over once the fever broke.He likely has.
a) Impetigo
b) Herpes
c) Chickenpox
d) Roseola
e) Robella
f) Measles
Answer: D
(Roseola)
Although all of these conditions cause a rash and are caused by infections, only
roseola has the typical rash that begins once the fever breaks.Two common and
closely related viruses can cause roseola: human herpesvirus (HHV) type 6 and
possibly type 7. These viruses belong to the same family as the better-known herpes
simplex viruses (HSV).

4-All of the following conditions can present with spherocytosis in peripheral


blood smear except:
1-ABO incompatibility
2- Thermal injury
3- Wilson disease
4-Autoimmune hemolytic anemia
5- Pneumococcal sepsis.
Answer: 5
(Pneumococcal sepsis)
Pneumococcal sepsis does not cause spherocytosis. However, clostridial
septicemia with exotoxemia can cause spherocytosis.

5-A 6-yr-old child with meningococcal meningitis develops a swollen left knee on
the fifth day of antibiotic treatment. Which of the following is the most likely
etiology of this finding?
A) A-Hemorrhage into the joint occurring as a result of disseminated intravascular
coagulation
B) Progression of septic arthritis that was unrecognized at the onset of illness
C) Nonspecific edema from progressive sepsis-related capillary leak
D) Immune complex deposition resulting from production of anti-meningococcal
antibodies
E) Secondary bacterial infection from the immunosuppressive effects of
meningococcal infection.
Answer: D
Immune complex deposition resulting from production of anti-meningococcal
patients with meningococcal disease showed one or more of the three allergic
complications: arthritis, cutaneous vasculitis, and episcleritis.
These complications, which were often multiple, occurred six to nine days after the
beginning of the illness and three to six days after the start of successful antibiotic
therapy.
Those patients with severe systemic disease were prone to the complications.
Histological and bacteriological study of the arthritis and vasculitis showed that
these lesions were probably not due to persisting infection and suggested that they
might be due to immune complex disease.

6-An 18m old child presents to the emergency center having a brief, generalized
tonic-clonic seizure. He is now postictal and has a temperature of 40°C (104°F).
During the lumbar puncture (which ultimately proves to be normal), he has a
large,watery stool that has both blood and mucus in it.
Which of the following is the most likely diagnosis in this patient?
a. Salmonella
b. Enterovirus
c. Rotavirus
d. Campylobacter
e. Shigella
Answer: E
Clinical manifestations of shigellosis range from watery stools for several days to
severe infection with high fever, abdominal pain, and generalized seizures. In
addition, there is a rare and fatal “toxic encephalopathy” seen with Shigella
infection known as Ekiri syndrome.
In general, about 50% of infected children have emesis, greater than two-thirds
have fever, 10% to 35% have seizures, and 40% have blood in their stool.
Often, the seizure precedes diarrhea and is the complaint that brings the family to
the physician.
Fever usually lasts about 72 hours, and the diarrhea resolves within 1 to 2 weeks.
Presumptive diagnosis can be made on the clinical history; confirmation is through
stool culture.
Supportive care, including adequate fluid and electrolyte support, is the mainstay
of therapy.
Antibiotic treatment is problematic; resistance to trimethoprim sulfamethoxazole
and ampicillin is common, necessitating therapy with third-generation
cephalosporins in many cases.
As always, knowledge of the susceptibility patterns of the bacteria in your area is
the key to using the right antibiotic.
7-Scimitar syndrome:
A) Is usually associated with a hypoplastic left lung
B) Can be palliated by coil occlusion in the cardiac catheter laboratory
C) Will show dextrocardia on the X-ray as a result of situs inversus
D) Is associated with abnormal pulmonary arterial supply
E) Is usually associated with abnormal radiology
Answer: B,D
Scimitar syndrome is a form of anomalous pulmonary venous drainage in which
the veins from the lower right lung drain into the inferior vena cava.
The right lung itself is hypoplastic, and there is an associated dextrocardia due to
the heart moving over to the right side of the chest, but with normal situs.
Situs is the orientation of the organs, situs solitus being normal, and situs inversus
being mirror image. The arterial supply to the lung is from branches of the
descending aorta.The right upper lobe pulmonary vein draining into the inferior vena
cava may be seen as a vertical line on a chest X-ray and is known as the ‘Scimitar
sign’.There may be an atrial septal defect, and children can suffer with recurrent chest
infections, which may require right lower lobectomy.

8-In response to social overtures (being held, hugged, kissed, talked to), a febrile
infant does not smile, has a dull, expressionless face, and is not alert to stimuli.
The most appropriate approach to management is to:
A) A-Administer ceftriaxone IM after a blood culture and have the parent and child
return to the office in the morning
B) If the child is older than 6 mo, obtain a blood culture and have the parents
return to the office if the patient remains febrile
C) Administer acetaminophen and reassess after the infant is no longer febrile
D) Administer ceftriaxone after obtaining a blood, urine, and CSF specimens for
culture and admit the child to the hospital
E) Administer a normal saline bolus of 20 mL/kg and reevaluate in 1 hr
Answer: D
Administer ceftriaxone after obtaining a blood, urine, and CSF specimens for
culture and admit the child to the hospital
Clinical observation of young patients is critical in helping you to evaluate and
distinguish the degree of risk of infection and physiologic impairment. In addition
to observing color, tone, grunting, or a bulging fontanel, the response to social
stimuli is valuable. This 3-mo-old had pneumococcal meningitis.

9-The mother of an 8-yr-old boy with acute streptococcal tonsillitis calls to


report that now, within 15 min after the first dose of oral penicillin V that you
prescribed,he is complaining of itching and has developed hives.
Which of the following should you recommend?
A) A dose of oral Benadryl, with instructions to call again if he has not improved
within 30 min
B) Immediate return to your office or the nearest emergency department
C) Careful monitoring at home, with instructions to return to your office or the
nearest emergency department if he becomes short of breath or loses
consciousness
D) Schedule a visit for a laboratory test to determine serum trypticase level
E) Substitution of erythromycin for penicillin
Answer: B
Immediate return to your office or the nearest emergency department.
The urticarial reaction described in the question may develop into anaphylaxis; the
latter requires emergency treatment. In addition, the penicillin V should be stopped
and a substitute non penicillin antibiotic chosen.

10-All of the following may be associated with neutropenia except:


A-Leukocyte adhesion deficiency
B-Shwachman-Diamond syndrome
C-Cartilage-hair hypoplasia
D-Chédiak-Higashi syndrome
E-Glycogen storage type Ibdisease
Answer: A
Leukocyte adhesion deficiency
Persons with leukocyte adhesion deficiency have impaired trans-endothelial
migration.
The circulating neutrophil count with infection is typically above 30,000/mm3 and
can surpass 100,000 per mm3, with a paucity of neutrophils in the infected tissues.

11-A 6-m-old child presents with recurrent cellulitis and bacteremia due to
Staph.aureus. The white blood cell count is 2500/mm3 with 5% neutrophils,
10% eosinophils, 35% monocytes, and 50% lymphocytes. The platelet count is
650,000/mm3. A brother and a female cousin died at the ages of 18 m and 2 yr,
respectively.
The most likely diagnosis is:
A-AIDS
B-Severe combined immunodeficiency
C-Kostmann disease
D-Cyclic neutropenia
E-Chronic granulomatous disease
Answer: C
Kostmann disease
An autosomal recessive severe infantile form of Agranulocytosis, manifests with
persistently low absolute neutrophil counts (<200/mm3) and severe, recurrent, and
at times lethal (by age 3 Y) infection.

12-A 9-yr-old boy presents with fever >39°C for 4 days, myalgias, watery
diarrhea,conjunctival infection, diffuse erythroderma, strawberry tongue, blood
pressure of 105/45 mm Hg, and moderately elevated hepatic transaminases.
The most likely diagnosis is:
A-Staphylococcal scalded skin syndrome
B-Kawasaki disease
C-Toxic shock syndrome
D-Stevens-Johnson syndrome
E-Toxic epidermal necrolysis.
Answer: C
Toxic shock syndrome
Toxic shock syndrome and Kawasaki disease share many features, but Kawasaki
disease is not accompanied by hypotension and shock. Kawasaki disease also
typically occurs in children younger than 5 yr. Toxic shock syndrome can
complicate focal infections caused by TSST-1-producing strains of Staph. aureus.

13-A 26-m-old child with sickle cell disease appears in your office for the first
time.He has a history of a prior hospitalization for pneumococcal bacteremia.
Currently the child is on penicillin prophylaxis, but he has not received either
the conjugate or polysaccharide pneumococcal vaccine.Which of the following is
the optimal means for protecting this child from developing another episode of
invasive pneumococcal disease?
A) A-Give one dose of the pneumococcal conjugate vaccine and continue
penicillin prophylaxis.
B) Give one dose of the pneumococcal polysaccharide vaccine and continue
penicillin prophylaxis.
C) Give one dose of the pneumococcal conjugate vaccine followed one month later
by one dose of the pneumococcal polysaccharide vaccine, and continue
penicillin prophylaxis.
D) Give one dose of the pneumococcal conjugate vaccine and switch from
penicillin prophylaxis to Augmentin prophylaxis.
E) Give one dose of pneumococcal conjugate vaccine and switch to intramuscular
benzathine penicillin prophylaxis
Answer: C
Give one dose of the pneumococcal conjugate vaccine followed one month later by
one dose of the pneumococcal polysaccharide vaccine, and continue penicillin
prophylaxis.Immunization with the conjugate polysaccharide vaccine is
recommended for all infants. High-risk children >2 yr of age, such as those with
asplenia, sickle cell disease, some types of immune deficiency (e.g., antibody
deficiencies), HIV infection, or chronic lung, heart, or kidney disease (including
nephrotic syndrome),may benefit also from the 23-valent pneumococcal
polysaccharide vaccine.

14-Of the following diagnostic tests, which is least useful in establishing a


diagnosis of mesenteric lymphadenitis due to Yersinia pseudotuberculosis?
A-Stool culture
B-Mesenteric lymph node histology
C-Abdominal ultrasonography
D-Mesenteric lymph node culture
E-Endoscopy
Answer: A
Stool culture
Many patients affected by Y. pseudotuberculosis do not have diarrhea, and thus a
stool culture is not even considered as part of the diagnostic evaluation. If the
extent of infection is limited to the mesenteric lymph nodes, the stool culture
results may be negative.

15-A 12-yr-old boy with cystic fibrosis experiences an acute exacerbation and
is admitted for intravenous antibiotic therapy. Sputum culture reveals
Pseudomonas aeruginosa.
The antibiotic recommended for treatment is:
A-Ampicillin-sulbactam
B-Ceftazidime
C-Cefotetan
D-Ceftriaxone
E-Ciprofloxacin.
Answer: B
Ceftazidime
Appropriate single agents for treatment of Pseudomonas aeruginosa infection
include ceftazidime, cefoperazone, ticarcillin-clavulanate, and
piperacillintazobactam.Gentamicin or another aminoglycoside may be used
concomitantly for synergistic effect.

16-A 2-mo-old infant is suspected of having infant botulism. Which is the best means to confirm
the diagnosis?
A-Lumbar puncture
B-CT scan
C-Muscle biopsy
D-Electromyography
E-Fecal specimen
Answer: E
Fecal specimen
The diagnosis of botulism is unequivocally established by demonstrating the
presence of botulinum toxin in serum or of C.botulinum toxin or organisms in
wound material or feces

17-A 2-mo-old infant is suspected of having infant botulism. Which is the best
means to confirm the diagnosis?
A-Lumbar puncture
B-CT scan
C-Muscle biopsy
D-Electromyography
E-Fecal specimen
Answer: E
Fecal specimen
The diagnosis of botulism is unequivocally established by demonstrating the
presence of botulinum toxin in serum or of C.botulinum toxin or organisms in
wound material or feces

18-All of the following statements regarding C.difficile-associated diarrhea are


true except:
A) A-Antibiotic-associated diarrhea is often related to production of a toxin by
C.difficile.
B) Newborn and young infants are commonly colonized by C.difficile.
C) The antibiotic that most commonly causes C.difficile colitis is gentamicin.
D) Most children with antibiotic-associated diarrhea will improve without specific
antibiotic treatment.
E) Treatment of C.difficile colitis should include stopping antibiotics whenever
feasible.
Answer: C
The antibiotic that most commonly causes C.difficile colitis is gentamicin.
Virtually all known antibiotics have been implicated; penicillins, broad-spectrum
cephalosporins, and clindamycin are the most frequent offenders.

19-A previously healthy 6-m-old child who just completed antibiotic for acute
otitis media and now is healthy and asymptomatic undergoes a stool culture to
check for antibiotic-associated diarrhea.
The stool culture grows C. difficile.
The preferred antibiotic treatment is:
A-Oral clindamycin
B-Oral vancomycin
C-Intravenous vancomycin
D-Oral metronidazole
E-None of the above.
Answer: E
None of the above: antibiotic treatment is not indicated
Difficile is frequently isolated from the stool of healthy infants.
The interpretation of a positive stool culture or toxin requires clinical correlation.
Treatment would not be indicated for an asymptomatic infant.

20- The recommended treatment for active pulmonary tuberculosis in children


is:
A) Isoniazid for 9 m
B) Isoniazid and rifampin for 6 m
C) Isoniazid, rifampin, and pyrazinamide for 6-9 m
D) Isoniazid and rifampin for 6 m, with pyrazinamide during the first 2 m
E) Isoniazid and rifampin for 6 m, with ethambutol during the first 2 mo.
Answer: D
Isoniazid and rifampin for 6 m, with pyrazinamide during the first 2 mo
The American Academy of Pediatrics and the Centers for Disease Control and
Prevention recommend that children with pulmonary tuberculosis be treated with

21- All of the following are characteristic manifestations of congenital rubella


syndrome except:
A) Snuffles
B) Intrauterine growth retardation
C) Cataracts
D) Structural cardiac defects
E) Sensorineural hearing loss.
Answer: A
Snuffles
Congenital rubella affects virtually all organ systems.
Snuffles is a sign of congenital syphilis

22- Which of the following statements concerning measles and vitamin A is true?
A) There is not a confirmed relationship of vitamin A to the prognosis of
measles
B) Measles causes vitamin A deficiency
C) Treatment with vitamin A reduces measles severity for children in developing
countries
D) Subacute sclerosing panencephalitis (SSPE) is more likely in children with
underlying vitamin A deficiency
E) Treatment with vitamin A reduces the incidence of SSPE
Answer: C
Treatment with vitamin A reduces measles severity & complications for children in
developing countries
There is an apparent correlation between retinol concentration and measles
severity.
Treatment with vitamin A reduces morbidity and mortality in children with severe
measles in developing countries and is recommended for selected children with
severe measles in the United States

The most common complication of mumps in childhood is:


A-Arthritis
B-Meningoencephalitis
C-Myocarditis
D-Orchitis
E-Pancreatitis
Answer: B
Meningoencephalitis
Meningoencephalitis is the most common complication of mumps in childhood,
but appears to occur in more than two-thirds of cases, with clinical symptoms in
more than 10% of cases

A 14-yr-old boy and his twin sister each developed mumps. Their parents had
repeatedly declined for them to receive MMR vaccination. They ask about the
prognosis for this disease in their children, especially the possibility of infertility.
Which of the following statements most accurately describes the relationship
between mumps and infertility?
A-Infertility occurs as a sequela of mumps only among prepubertal children
B-Infertility is more common among females than among males
C-Infertility rate among males with mumps orchitis is approximately 30- 40%
D-Infertility among males is rare even with bilateral orchitis
E-Infertility rate among females with mumps oophoritis is approximately 30-40%
Answer: D
Infertility among males is rare even with bilateral orchitis
Infertility among males with mumps is rare, even with bilateral orchitis. There
is
no evidence of impairment of fertility among women with mumps oophoritis.
Mumps orchitis and oophoritis are rare among prepubescent boys and girls, and
are
more common among postpubertal men (14-35%) and women (7%).

Which of the following features distinguishes paralytic polio from Guillain-


Barré syndrome?
A-Pleocytosis is uncommon in paralytic polio
B-Paralysis is usually asymmetric in paralytic polio
C-The paralysis of polio is usually spastic
D-Sensory changes are common in paralytic polio
E-Paralytic polio only occurs in unimmunized persons
Answer: B
Paralysis is usually asymmetric in paralytic polio
Paralytic polio is characterized by aseptic meningitis accompanied by asymmetric
flaccid paralysis without sensory loss.
In Guillain-Barré syndrome, the paralysis is characteristically symmetric,
and sensory changes (paresthesias) are common.
Pleocytosis is common in polio, whereas the cerebrospinal fluid in Guillain-Barré
syndrome usually shows only elevated protein and occasionally a few cells.

All of the following statements regarding (HSV) infections in neonates are true except:
A) Most cases are caused by HSV type 2
B) Women with primary HSV genital tract infection are more likely to transmit
infection to their offspring than women with recurrent HSV infection
C) Most mothers of newborns with perinatal HSV infection have a history of
genital HSV infection
D) Most mothers of newborns with perinatal HSV infection are asymptomatic at
delivery
E) Most cases are transmitted at delivery and are not true congenital infections
Answer: C
Most mothers of newborns with perinatal HSV infection have a history of genital
HSV infection
Only 15-20% of mothers of newborns with perinatal HSV have a history of
obvious HSV infection, and only about 25% have any relevant symptoms at birth

Recommended management for a mother with active genital HSV infection during
labor is:
A) Culture of blood from the newborn, with treatment based on culture results
B) Culture of blood from the newborn, with empirical acyclovir therapy
C) Intravenous acyclovir treatment for the mother
D) Cesarean section within 4 hr of rupture of membranes
E) Intravenous acyclovir treatment for the mother and cesarean section within 4 hr
of rupture of membranes
Answer: D
Cesarean section within 4 hr of rupture of membranes
Both the American Academy of Pediatrics and the American College of Obstetrics
and Gynecology recommend cesarean section if primary, first-episode, or recurrent
HSV lesions are present on the mother at the onset of labor.
Only 15-20% of mothers of newborns with perinatal HSV have a history of HSV
infection
A 3-yr-old boy presents with a 7-day history of fever, cervical lymphadenopathy, foul breath, and
painful oral lesions on his tongue, gums, and lips. For the past 3 days he has had a red, painful swollen
area about the nail of his right thumb with an area of fluid by the nail bed, unresponsive to warm
soaks and a first- generation cephalosporin. The most likely etiologic agent is:
A-Staphylococcus aureus
B-Mucocutaneous candidiasis
C-Coxsackievirus
D-Adenovirus
E-Herpes simplex virus
Answer: E
Herpes simplex virus
The child has herpetic gingiva-stomatitis.
He has auto-inoculated his thumb by sucking, and herpetic whitlow has developed

A 9-day-old neonate, born by vaginal delivery to a 21-yr-old healthy woman, is


presented with fever, lethargy, and poor feeding. There are no diagnostic
findings on physical examination, and results of sepsis evaluation, including a
CBC and cerebrospinal fluid studies, are unremarkable. Ampicillin and
cefotaxime are begun. Two days later all bacterial cultures are negative but the
child's clinical condition worsens, with falling blood pressure, decreased level of
consciousness,thrombocytopenia, and elevated liver enzymes.
The indicated change in treatment is:
A-Addition of amphotericin
B-Addition of vancomycin
C-Addition of ribavirin
D-Addition of acyclovir
E-Substitution with meropenem and amikacin
Answer: D
Addition of acyclovir
The history, inability to confirm bacterial infection, and worsening clinical
condition are suggestive of neonatal herpes simplex virus infection. Acyclovir
therapy should be initiated. Repeat cerebrospinal fluid analysis or MRI might also
be recommended

All of the following tumors are associated with Epstein-Barr virus except:
A-Burkitt lymphoma
B-Kaposi sarcoma
C-Leiomyosarcoma
D-Lymphoproliferative disease in immunocompromised persons
E-Nasopharyngeal carcinoma
Answer: B
Kaposi sarcoma
EBV is associated with several malignancies, including nasopharyngeal carcinoma,
Burkitt lymphoma, Hodgkin disease, and lymphoproliferative diseases and
leiomyosarcomas in immune-compromised persons.
Kaposi sarcoma is associated with HHV-8.

10-mo-old child presents to the emergency department with a 4-day history of


fever, with temperatures to 40.3°, and watery diarrhea, and has just experienced
ageneralized seizure.
The most likely cause of this syndrome is:
A- Salmonella gastroenteritis
B- Aeromonas gastroenteritis
C- Shigella gastroenteritis
D- Rotavirus gastroenteritis
E- Drug ingestion
Answer: C
Shigella gastroenteritis
Shigella usually causes diarrhea and fever and sometimes, particularly in young
infants, seizures can occur.
The diarrhea may be watery and of large volume initially, evolving into frequent
small-volume, bloody mucoid stools; most children (>50%) never progress to the
stage of bloody diarrhea, whereas in others the 1st stools are bloody.
Neurologic findings are among the most common extraintestinal manifestations of
bacillary dysentery, occurring in as many as 40% of hospitalized infected children.
Enteroinvasive E. coli can cause similar neurologic toxicity.
Convulsions, headache, lethargy, confusion, nuchal rigidity, or hallucinations may
be present before or after the onset of diarrhea. The cause of these neurologic
findings is not understood. In the past, these symptoms were attributed to the
neurotoxicity of Shiga toxin, but it is clear that this explanation is wrong since the
organisms isolated from children with Shigella-related seizures are usually not
Shiga toxin producers.
Seizures sometimes occur when little fever is present, suggesting that simple
febrile convulsions do not explain their appearance.
Hypocalcemia or hyponatremia may be associated with seizures in a small number
of patients.Although symptoms often suggest central nervous system infection, and
cerebrospinal fluid pleocytosis with minimally elevated protein levels can occur,
Shigella meningitis is rare.

A 20-m-old child develops hemolytic anemia, anuria, azotemia, and


thrombocytopenia after a bout of febrile bloody diarrhea.
The most likely etiologic agent of this illness is:
A- Campylobacter jejuni
B- Salmonella typhi
C- Entero-hemorrhagic Escherichia coli
D- Aeromonas
E- Non-typhi Salmonella
Answer: C
Entero-hemorrhagic Escherichia coli
Shiga toxin-producing E. coli O157:H7 is an entero-hemorrhagic pathogen and is
responsible for most episodes of hemolytic-uremic syndrome.
Shigella dysenteriae is occasionally responsible for the hemolytic-uremic
syndrome (HUS)

Examination of the cerebrospinal fluid of an 8-year-old, mildly febrile child with


nuchal rigidity and intermittent stupor shows the following:
WBCs 100/μL (all lymphocytes), negative Gram stain, protein 150 mg/dL, and
glucose 15 mg/dL.
The most likely diagnosis is
A. Tuberous sclerosis
B. Tuberculous meningitis
C. Stroke
D. Acute bacterial meningitis
E. Pseudotumor cerebri.
Answer: B
Included among those things that can cause the clinical picture are viral meningitis,
TB meningitis, meningeal leukemia, and medulloblastoma, all of which can cause
pleocytosis as well as elevated protein and lowered glucose concentrations in
(CSF). Of the four diseases, tuberculous meningitis is associated with the lowest
glucose levels in CSF.
The cellular response to viral meningitis eventually will be predominantly
lymphocytic.
Cells found in the CSF of a child who has meningeal leukemia most commonly are
lymphocytes or lymphoblasts.
Children who have a medulloblastoma generally present with the signs and
symptoms caused by a mass in the posterior cranial fossa; their pleocytotic CSF
contains unusual-appearing cells of the monocytic variety.
The decrease in the glucose concentration of CSF associated with these disorders
has been attributed to a disturbance of glucose transport as a result of meningeal
irritation.
In pseudo tumor cerebri, the constituents of CSF are generally normal except for a
low protein content in some instances.
Acute bacterial disease typically causes PMN cells and positive Gram stains.
Neither tuberous sclerosis nor stroke typically causes these findings on CFS
examination.
Acute bacterial disease typically causes PMN cells and positive Gram stains

Potential cardiac problems in patients with MPS include all of the following
except:
A-Coronary artery disease
B-Mitral regurgitation
C-Pericarditis
D-Aortic valve disease
E-Cardiomyopathy
F-Endocarditis
Answer: C
Pericarditis
Storage in endothelial cells can cause coronary artery disease and even myocardial
infarction.
Cardiomyopathy may be present in infancy and is associated with endocardial
fibroelastosis.
Valve disease necessitating valve replacement usually occurs in older children.

Which of the following is associated with a poorer prognosis for persons


presenting with meningococcal disease?
A-Presence of petechiae for <12 hr
B-Meningitis
C-Thrombocytosis
D-Leukocytosis
E-Low circulating levels of tumor necrosis factor
Answer: A
Poor prognostic signs for meningococcal disease include:
 Hypotension and shock,
 Purpura fulminans,
 Seizures,
 Leukopenia,
 Thrombocytopenia,
 High circulating levels of endotoxin and tumor necrosis factor.
The presence of petechiae for less than 12 hr before admission, hyperpyrexia, and
the absence of meningitis reflect rapid clinical progression and poorer prognosis.

A 9 month old boy presents with a 24 hr. history of fever and poor feeding. He has
become less responsive and has developed a purple rash over his arms and trunk.
No neonatal problems, No drugs, fully immunized, No F/H or social history of
note.On examination temperature is 38.2°C, RR 50/min (mild recession, HR
150/min.Cool peripheries, with capillary refill time of 4 seconds. Non-blanching
purple spots 3-27mm over the arms and trunk.What is the most likely diagnosis?
1-meningococcal infection
2-Thrombocytopaenia
3-Trauma/ child abuse
4-Vasculitis (HSP)
5-Viral infection
Answer: 1
Meningococcal infection.
The history is brief, with a sick, shocked child with purpura.
The diagnosis is purpura fulminans, probably due to septicaemia.
The 3 commonest organisms which cause this are meningococcus type B,A&C,
pneumococcus and Haemophilus influenza type B.
The incidence of Meningitis C and Hib have

Which of the following is true of Koplik's spots?


1) Are diagnostic of Measles
2) Located opposite the incisor teeth.
3) Only appear when fever is over 39°C
4) They appear as red papules on the plamar surface of the hands
5) Typically appear two days after the rash.
Answer: 1
Are diagnostic of Measles.
Koplik spots are a prodromic viral enanthem of measles manifesting two days
before the measles rash itself.
They are characterized as clustered, white lesions on the buccal mucosa near each
Stenson's duct (opposite the premolars) and are pathognomonic for measles.
The textbook description of Koplik spots is ulcerated mucosal lesions marked by
necrosis, neutrophilic exudate, and neovascularization.
They are described as "grains of salt on a wet background" and often fade as the
macular rash develops.

A 2-year-old boy is brought into the emergency room with a C/O fever for 6 days
and development of a limp. On examination, he is found to have an erythematous
macular exanthem over his body, ocular conjunctivitis, dry and cracked lips, a red
throat, and cervical LN.
There is a grade II/VI vibratory systolic ejection murmur at the lower left
sternal border.
A white blood cell count and differential show predominant neutrophils with
increased platelets on smear.
The most likely diagnosis is
a. Scarlet fever
b. Rheumatic fever
c. Kawasaki disease
d. Juvenile rheumatoid arthritis
e. Infectious mononucleosis.
Answer: c
Kawasaki disease (KD)
Many conditions can be associated with prolonged fever, a limp caused by
arthralgia, exanthem, adenopathy, and pharyngitis. Conjunctivitis, however, is
suggestive of Kawasaki disease.
The fissured lips, although common in KD, could occur after a long period of fever
from any cause if the child became dehydrated.
The predominance of neutrophils and high ESR are common to all.
An increase in platelets within this constellation of symptoms, however, is found
only in KD.
KD presents as prolonged fever, rash, epidermal peeling on the hands and feet
(especially around the fingertips), conjunctivitis, lymphadenopathy,
fissured lips, oropharyngeal mucosal erythema, and arthralgia or arthritis.
The diagnosis is still possible in the absence of one or two of these physical
findings.
Coronary artery aneurysms can develop, as can aneurysms in other areas.
Initial treatment is typically IVIG and high-dose aspirin. The child will usually
defervesce shortly after the infusion. Aspirin is typically kept at a higher dose until
the platelet count begins to decrease, and then is continued at a lower dose for
several weeks.
While bacterial infection is in the differential diagnosis for this patient’s
presentation and blood cultures are usually part of the evaluation, IV vancomycin
should be reserved.

A 6-year-old boy has brown urine and healing impetigo


lesions. He presents with hypertension, dyspnea, periorbital edema, and
hepatomegaly.
The most likely cause of his problem is
a. IgA nephropathy
b. Poststreptococcal glomerulonephritis
c. Idiopathic hypercalciuria
d. Pyelonephritis
e. Sexually transmitted disease.
Answer: b
Acute post-streptococcal glomerulonephritis follows a skin or throat infection
with certain nephritogenic strains of group A, β-hemolytic streptococci.
Hematuria often colors the urine dark, and decreased urinary output can result in
circulatory congestion from volume overload with pulmonary edema, periorbital
edema, tachycardia, and hepatomegaly. This can be avoided by fluid restriction.
Acute hypertension is common and can be associated with headache, vomiting, and
encephalopathy with seizures.
Pyelonephritis and sexually transmitted diseases can cause bloody urine, but rarely
the other symptoms of impetigo as described in the question.
IgA nephropathy is rare in black children and rarely presents with
hypertension.
Idiopathic hypercalciuria can present

A 2-year-old boy develops bloody diarrhea shortly after eating in a fast food
restaurant. A few days later, he develops pallor and lethargy; his
face looks swollen and his mother reports that he has been urinating very
little. Laboratory evaluation reveals low hematocrit and platelet count and
positive blood and protein in the urine.
Which of the following diagnoses is likely to explain these symptoms?
a. Henoch-Schönlein purpura
b. IgA nephropathy
c. Intussusception
d. Meckel diverticulum
e. Hemolytic-uremic syndrome.
Answer: e
Hemolytic-uremic syndrome: is Triad of:
 Acute micro-angiopathic hemolytic anemia,
 Thrombocytopenia from increased platelet utilization,
 Renal insufficiency from vascular endothelial injury and local fibrin deposition.
Ischemic changes result in renal cortical necrosis and damage to other organs
such as colon, liver, heart, brain, and adrenal.
Laboratory findings associated with HUS include
 Low hemoglobin level, decreased platelet count, hypoalbuminemia, and
evidence of hemolysis on peripheral smear (burr cells, helmet cells,
schistocytes).
 Urinalysis reveals hematuria and proteinuria.
 A marked reduction of renal function leads to oliguria and rising levels of blood
urea nitrogen (BUN) and creatinine.
 Gastrointestinal bleeding and obstruction, ascites,
Central nervous system findings such as somnolence, convulsions, and coma can
occur.
In the past decade, infection by the verotoxin-producing E.coli 0157:H7 has been
implicated as a cause of hemolytic-uremic syndrome. This organism is epizootic in
cattle. Outbreaks associated with undercooked contaminated hamburgers have
been reported in several states. Roast beef, cow’s milk, and fresh apple cider have
been implicated as well.
The Coombs test is not positive in this type of hemolytic anemia.

The newborn pictured was born at home and has puffy, tense eyelids; red
conjunctivae; a copious amount of purulent ocular discharge; and ecchymosis 2
days after birth.
The most likely diagnosis is
a. Dacryocystitis
b. Chemical conjunctivitis
c. Pneumococcal ophthalmia
d. Gonococcal ophthalmia
e. Chlamydial conjunctivitis.
Answer: d
Gonococcal Ophthalmia
The time of onset of symptoms is somewhat helpful in the diagnosis of
Ophthalmia Neonatorum.
Gonococcal conjunctivitis has its onset within 2-5 days after birth and is the most
serious of the bacterial infections.
Prompt and aggressive topical treatment and systemic antibiotics are indicated to
prevent serious complications such as corneal ulceration, perforation, and resulting
blindness.
Parents should be treated to avoid the risk to the child of reinfection.
Chemical conjunctivitis is a self-limited condition that presents within 6-12 h of
birth as a consequence of silver nitrate or erythromycin prophylaxis.
Silver nitrate is believed by some to be ineffective prophylaxis against chlamydial
conjunctivitis, which occurs 5-14 days after birth, but is no longer made for ocular
prophylaxis.
To avoid the risk of chlamydial pneumonia, treatment with systemic antibiotics is
indicated for the infant as well as both parents in cases of chlamydial
conjunctivitis.

A 2 year old girl presented with a 12-hour history of fever and poor feeding. The
family owned two cats. Her temperature was 39.5°C, and she was irritable, with no
localizing signs or skin lesions. A full septic screen was performed. Cerebrospinal
fluid (CSF) showed a neutrophilic pleocytosis and gram-negative coccobacilli. She
was treated with IV cefotaxime and gentamicin. Within 24 hours both CSF and
blood cultures showed growth of gram-negative bacilli.
What is the likely infective organism?
A. Legionella pneumophilia
B. Haemophilus influenza
C. Pasteurella multicoda
D. E coli
E. Pseudomonas aeruginosa.
Answer: C
Pasteurella multicoda.
Pasteurella multicoda is an oral commensal of domestic pets known to be an
opportunistic human pathogen after traumatic animal contact.
The most common infections in humans are skin and pulmonary infections.
Pasteurella meningitis occurs at extremes of age (infants), in the
immunocompromised (associated with liver cirrhosis, renal disease and
haematological malignancies) and after traumatic head injury.

A previously healthy 6-m-old child who just completed antibiotic treatment for
acute otitis media and now is healthy and asymptomatic undergoes a stool
culture to check for antibiotic-associated diarrhea. The stool culture grows
C.difficile.
The preferred antibiotic treatment is:
A-Oral clindamycin
B-Oral vancomycin
C-Intravenous vancomycin
D-Oral metronidazole
E-None of the above.
Answer: E
None of the above: antibiotic treatment is not indicated
C.difficile is frequently isolated from the stool of healthy infants.
The interpretation of a positive stool culture or toxin requires clinical correlation.
Treatment would not be indicated for an asymptomatic infant.

A12-year-old Chinese-American boy complains of gas, bloating, and diarrhea for


the past 3 months. He has no nocturnal symptoms and denies weight loss. His
recent history is significant only for a family trip to mainland China to visit his
grandparents 2 months ago. Physical exam demonstrates a well-developed,
wellnourished
adolescent. The remainder of the examination is unremarkable, except
for a non tender, tympanitic, slightly distended abdomen with active bowel sounds.
His stool is watery, however occult blood test results are negative and the pH: 5.0
Of the following, the test that you are MOST likely to recommend is a
A. 72-hour fecal fat
B. Lactose breath test
C. Lactulose breath test
D. Small intestinal biopsy for disaccharidase levels
E. Stool for ova and parasites
Answer: B
The boy described has a 3-month history of watery diarrhea in the absence of
weight loss or nocturnal or other constitutional symptoms.
Although a recent trip abroad suggests the possibility of an acquired enteric
infection, his symptoms began before he left the United States.
Examination of a stool specimen demonstrates an acidic pH of 5.0 and no occult
blood loss. In this scenario, a diarrheal state occurring as a consequence of dietary
causes, especially carbohydrate malabsorption.

A 17 year old male from India presents with fever of 4 months duration and
splenomegaly.What is the most likely diagnosis?
A. Coccidiomycosis
B. Giardiasis
C. Tropical sprue
D. Typhoid
E. Visceral leishmaniasis
Answer: E
Visceral leishmaniasis (Kala-azar): is an endemic disease in several regions of
India and sub-Saharan Africa. It is caused by the parasite Leishmania donovani and
spread by Phlebotomus sand-flies. Leishmaniasis is common in immunesuppressed
patients, particularly those infected with HIV.
There has recently been a substantial increase of cases in the Mediterranean region.
It has been estimated that 15% of HIV positive drug users in Spain are infected
with Leishmania donovani infantum.
Giardiasis and tropical sprue present with gastrointestinal symptoms and
malabsorption.
Typhoid is an acute illness.
Coccidiomycosis is largely confined to the Americas.
Most patients present with pulmonary symptoms although disseminated
disease can occur particularly in the immune-suppressed.

newborn is noted to be quite jaundiced at 3 days of age.


Which of the following factors is associated with an increased risk of neurologic
damage in a jaundiced newborn?
a) Metabolic alkalosis
b) Increased attachment of bilirubin to binding sites caused by drugs such as
sulfisoxazole
c) Hyperalbuminemia
d) Neonatal sepsis
e) Maternal ingestion of phenobarbital during pregnancy.
Answer: d
Neonatal sepsis
Significant unconjugated serum bilirubin levels in full-term newborn infants can
lead to diffusion of bilirubin into brain tissue and to neurologic damage.
Sulfisoxazole and other drugs compete with bilirubin for binding sites on albumin;
therefore, the presence of these drugs can cause dislocation, not increased affinity,
of bilirubin to tissues. Metabolic acidosis also reduces binding of bilirubin, and
neonatal sepsis interrupts the blood-brain barrier, thus allowing diffusion of
bilirubin into the brain.
Administration of phenobarbital has been used to induce glucuronyl transferase in
newborn infants and can reduce, rather than exacerbate, neonatal jaundice.
Other factors that reduce the amount of unconjugated bilirubin bound to albumin
(and therefore cause an increase in free unconjugated bilirubin) include
hypoalbuminemia and certain compounds (e.g., non-esterified fatty acids, which
are elevated during cold stress) that compete with bilirubin for albumin binding
sites.
One of the surgical wards notes an outbreak of Methicillin-resistant Staph. aureus
(MRSA) infections.
What is the best mechanism for reducing further transmission of this infection?
A) Cleaning the floors and walls of the ward with chlorhexidine
B) Close the ward for one month
C) Encourage regular hand washing by ward staff
D) Screen ward staff using nasal swabs and exclude those with positive cultures
for MRSA
E) Treatment of culture-positive patients with vancomycin
Answer: C
Cross-infection via hands of medical and nursing staff is a very important vehicle
of transmission of MRSA. Hand washing before and after contact with patients is
the single most effective measure to control hospital spread of this organism.
Screening of ward staff is appropriate only in certain situations and should not be
carried out unless recommended by the hospital infection control team.
Vancomycin should never be used for MRSA decolonization.
The hospital infection control policy should outline which patients should be
screened and when decolonization should be attempted.

1-Question 1. In babies born to a mother infected with HIV, which of the following
management strategies are recommended?
(a) Antiretroviral therapy during pregnancy and for 6 months after birth
(b) Vaginal delivery unless the fetus becomes distressed
(c) Breastfeeding the baby to boost immunity
(d) Early start of solid feeds to prevent failure to thrive
(e) None of the above.
The correct answer is (e)

None of the statements are correct. Antiretroviral therapy is given in pregnancy to


reduce the viral load, and zidovudine is given in labour and for only 4 weeks after
birth. Delivery should be by caesarean section to reduce vertical transmission and
breast-feeding should be avoided in countries where safe water cannot be relied upon.
Although weight faltering can be a presenting feature of HIV in young children early
weaning is inadvisable in any baby.

Question 2. Which of the following statements is correct regarding pyrexia of


unknown origin (PUO)
a) PUO is defined as fever of >2 week in young children
b) Blood cultures should be taken when the fever breaks
c) PUO is usually due to an atypical presentation of a common illness
d) Absence of fever and sweating is a sign of occult disease
e) Antipyretics should be given as the fever rises to make the patient more
comfortable

he correct answer is (c)

PUO is usually due to an atypical presentation of a common illness such as urine


infection or pneumonia. In adolescents PUO is defined as a fever for more than 2–3
weeks, but in young children the definition is fever for more than one week. The
absence of fever and sweating suggests factitious fever rather than organic disease.
Blood cultures should be obtained at the peak of fever as the yield is then higher.
During the investigation phase of PUO antipyretics should be avoided as they will
obscure the pattern of the fever.

Question 3. The following features in a history or physical examination would alert


you to the possibility of infective endocarditis as a cause for PUO except:
(a) Congenital heart disease
(b) An indwelling catheter for parenteral nutrition
(c) Splinter haemorrhages and clubbing
(d) Disappearance of a pre-existing heart murmur
(e) Microscopic haematuria

The correct answer (d)

All of the answers are correct except for d. A pre-existing heart murmur may change
in character in infectious endocarditis but will not disappear. Other features of
endocarditis include splenomegaly and preceding dental or other surgery.
SECOND
PART
1 1. A 5 year boy presents to you at a poor-infrastructure rural hospital with history of
dog bite 12 hrs ago. A category III exposure was noted. What is the best first thing
you would like to advise :
Immediate immunoglobulin injection
Refer to speciality care as soon as possible
Both immunoglobulin infiltration and anti-rabies immunization
Wound toilet with soap and water.

Q. 2 Whom you should consider least infectious_?


Immunocompetent patient with thoracolumbar zoster
Immunosuppressed patient with localized zoster
Immunocompetent patient with ophthalmic zoster
Disseminated zoster in immunocompetent.

Q. 3 3. Hepatitis C virus {HCV} assay of a child shows HCV-RNA positive but


Anti-HCV antibody is negative. What could be this case_?
Early acute HCV infection
Chronic hepatitis C infection in immunosuppressed
False-positive HCV-RNA test
Any of the above

Q. 4 False regarding rabies management:


10 days observational period applies for cats and dogs only
Wild animal bite is category III always
Antibody titre of 0.5 IU, mL serum or more is protective
d} Essen schedule administers 5 doses of intradermal anti-rabies vaccine.

Q. 5 Entamoeba histolytica DNA can be detected in the following sample from a


patient with amoebic liver abscess:
Aspirated pus from liver
Urine
Saliva
All of the above

Q6. A 10 year boy from north-eastern part of India presented with productive cough,
low grade prolonged fever, headache and inability to move his left arm for last one
month. Blood tests showed: Hemoglobin-11gm percent, total leucocyte count-8800
cells, cumm {polymorphs 58 percent, lymphocytes 25 percent, eosinophils 17
percent}. CT brain shows multiple ring-shaped shadows in right cerebral hemisphere.
Sputum report reveals absence of acid fast bacilli {AFB} but presence of some
operculated structures. What is the case_?
Tuberculoma
Staphylococcal brain abscess
Neurocysticercosis
Cerebral Paragonimiasis

Q. 7 (Not attempt) Glomerulonephritis can be an extra-intestinal manifestation of


infection with:
Shigella
Yersinia
Campylobacter
All of the above

Q. 8 Extra-intestinal manifestation of Shigella infection includes:


Glomerulonephritis
Hemolytic Uremic syndrome
Reactive arthritis
All of the above

Q. 9 (Not attempt) Surgical complications of Shigellosis include:


Intestinal obstruction
Appendicitis
Perforation
All of the above.

Q. 10 Sign that is less frequent in meningococcal meningitis in children as compared


to other causes of bacterial meningitis
Seizure
Photophobia
Vomiting
Lethargy
THIRD PART
Q 1 :An 8-year-old girl presents with a 4-day history of fever, headache, and
abdominal pain. Her mother states that they live in a rural area and have
multiple pets, including dogs, cats, horses,cows, and a pet raccoon. There is no
history of tick bites. On physical examination, the girl appears mildly toxic, has
a temperature of 102.2°F (39°C), and has a grade II/VI systolic ejection murmur
best heard on the left side of the sternal border. Her right upper quadrant is
tender to palpation, but there is no hepatosplenomegaly. Findings on her skin
and extremity examination are normal. A complete blood count reveals a white
blood cell count of 1.2x103/mcL (1.2x109/L) with 90% neutrophils and 10%
lymphocytes. Her hemoglobin is 10 g/dL (100 g/L), and her platelet count is
50x103/mcL (50x109/L). Her alanine aminotransferase is 600 U/L, and her
aspartate aminotransferase is 450 U/L. Her amylase and lipase values are
normal. Serum sodium is 133 mEq/L (133 mmol/L), but the remainder of her
electrolyte values are normal.
Of the following, the MOST likely diagnosis is

A. human monocytic ehrlichiosis


B. Lyme disease
C. Rocky Mountain spotted fever
D. tularemia
E. typhus

Answer :A
Explanation:-

Human monocytic ehrlichiosis (HME) is a rickettsial disease caused by Ehrlichia


chaffeensis,which is transmitted to humans by the bite of a tick.
Clinically, the ehrlichioses are nonspecific illnesses. Fever (~100%) and
headache (~75%) are most common, but many patients also report myalgias,
anorexia, nausea, and vomiting.
With HME, rash is more common in children (nearly 66%) than in adults
(33%). The rash is usually macular or maculopapular, but petechial lesions can
occur.
Photophobia, conjunctivitis, pharyngitis, arthralgias, and lymphadenopathy are
less consistent features.

Hepatomegaly and splenomegaly are detected in nearly 50% of children with


ehrlichiosis.

Edema of the face, hands, and feet occurs more commonly in children than in adults,
but arthritis is uncommon in both groups.

A rash is described in approximately two thirds of children and one third of adults
and starts as maculopapular but may progress into petechial/purpuric.

Meningoencephalitis with a lymphocyte-predominant CSF pleocytosis is an


uncommon but potentially severe complication of HME

HME is clinically indistinguishable from Rocky Mountain spotted fever (RMSF).

Laboratory abnormalities common to both infections include thrombocytopenia and


hyponatremia, but
patients who have HME are more likely to have elevated liver function test results
and
leukopenia with lymphopenia. Approximately 50% to 75% of patients have no
history of a tick
bite.

Patients who have Lyme disease typically do not appear toxic or have the laboratory
abnormalities described in the vignette.

Although there is a typhoidal form of illness due to Francisella tularensis (tularemia),


it is extremely rare, and most affected children present with glandular or
ulceroglandular disease. Typhus can be endemic or epidemic.

Epidemic typhus is due to the bite of the human louse, and endemic typhus is caused
by a mite bite. Although both of these rickettsial diseases can present with fever and a
headache, patients usually are not
toxic and do not have laboratory abnormalities such as those reported for the girl in
the vignette.

Q2 :A 4-year-old boy presents to your office for evaluation of a 3-day history of


fever (temperature to 38.5°C), congestion, and sore throat. Physical examination
of the well-appearing child shows only rhinorrhea and pharyngeal erythema.
His mother and 6-year-old sister have had colds over the past week.
Of the following, the MOST appropriate treatment for this child, pending the
results of the throat culture, is
A. amoxicillin
B. azithromycin
C. nasal saline drops
D. prednisone
E. pseudoephedrine

Answer :C
The congestion and sore throat described for the boy in the vignette, combined with
the history of upper respiratory tract infections in the family, strongly suggest that he
has a viral illness. Supportive therapy such as nasal saline drops to relieve congestion
is appropriate.

Cough and cold remedies, including those containing the decongestant


pseudoephedrine, have not been demonstrated to be effective in treating viral upper
respiratory tract infection symptoms, and based on potential toxicities in young
children, the American Academy of Pediatrics and United States Food and Drug
Administration have advised against their use in children younger than 6 years of age.

There is no indication for prednisone in this setting. However, high-dose, short-term


corticosteroid therapy may be beneficial in the treatment of the patient who has
marked pharyngitis and impending airway obstruction associated with acute
infectious mononucleosis.

Antibiotics are not indicated to treat a viral illness and do not prevent development of
possible secondary bacterial infections (eg, otitis media, sinusitis).

Increased use of antibiotics has been associated with increased rates of carriage of
resistant bacteria (eg, penicillin-resistant Streptococcus pneumoniae, beta-lactamase-
positive Haemophilus influenzae, methicillin-resistant Staphylococcus aureus).

Other common illnesses that generally do not require antibiotic therapy in children
include bronchitis, middle ear effusion of short duration, mucopurulent rhinitis of less
than 10 days’ duration, and most cases of acute pharyngitis (unless group A
streptococcal infection is confirmed).

Bronchitis in children is an acute cough illness that is generally self-limited and


caused by viruses.

If the child in the vignette has a positive diagnostic test result (rapid antigen detection
or throat culture), antibiotic treatment would be appropriate.
Penicillin V is the drug of choice for streptococcal pharyngitis, although amoxicillin
often is used instead as first-line treatment.

A firstgeneration cephalosporin (eg, cephalexin or cefadroxil) also may be used.


Broader-spectrum agents (eg, amoxicillin-clavulanate, second- or third-generation
cephalosporins) are not indicated routinely for this infection.

Azithromycin should be reserved for treating streptococcal pharyngitis in the patient


who is allergic to penicillins and cephalosporins.

Streptococcal serogroups C and G rarely have been associated with symptomatic


pharyngitis. They have not
been associated with rheumatic fever, but antibiotic therapy (same agents as for group
A streptococcal infection) may be considered in the symptomatic patient who has a
positive culture and no other cause determined for the pharyngitis.

Q3 :A 12-year-old boy presents with a 5-day history of sore throat, fever, and
progressive rightsided neck pain and swelling. On physical examination, his
temperature is 40.0°C, he has trismus, the right side of his neck is swollen and
tender to palpation, and his chest is clear to auscultation. His white blood cell
count is 30.0x103/mcL (30.0x109/L), with 80% polymorphonuclear leukocytes,
15% lymphocytes, and 5% monocytes. Computed tomography scan of the neck
reveals a deep parapharyngeal abscess (Item Q141).

Of the following, the MOST appropriate antimicrobial to include in his therapy


is

A. ampicillin-sulbactam
B. azithromycin
C. clarithromycin
D. gentamicin
E. trimethoprim-sulfamethoxazole

Answer :A
The boy described in the vignette has an abscess in the deep tissues of the neck.
Streptococci, including S pyogenes, and Staphylococcus aureus are the most
common pathogens associated with infections of the parapharyngeal space.
However, oral anaerobic bacteria also are found frequently in these infections
because the primary portals of entry for organisms into the parapharyngeal
space are the oropharynx, lower molars, nasopharynx, paranasal sinuses, and
mastoid. The most common anaerobic bacteria isolated from parapharyngeal
infections are Bacteroides, Peptostreptococcus, and Fusobacterium. Most of
these infections are polymicrobial. Because the parapharyngeal space is
contiguous with the retropharyngeal, submandibular, and peritonsillar spaces,
infection may spread in any number of directions and lead to a variety of clinical
manifestations and complications.Ampicillin-sulbactam is a beta-lactamase-
resistant semisynthetic penicillin that has activity against anaerobes, susceptible
aerobic gram-positive organisms, and respiratory tract gramnegative
pathogens, making it an appropriate initial drug for the patient described in the
vignette.

Because group A streptococci are becoming increasingly resistant to macrolide


antibiotics such as azithromycin and clarithromycin and to trimethoprim-
sulfamethoxazole, these drugs are not appropriate.In addition, macrolide
antibiotics have less activity than ampicillin-sulbactam against B fragilis and
Fusobacterium.Gentamicin is not useful because aerobic enteric gram-negative
rods do not play a significant role in parapharyngeal infections.

Q 4:A five year old boy is admitted to the paediatric ward with a two day history
of fever, myalgia and jaundice. His family live on a canal barge and have been
moving around the country on a regular basis. He has many scabs on his knees
and elbows, which his parents say result from his playing on the canal banks.
Observations show temperature 38.7 °C,heart rate 150 beats per minute,
respiratory rate 35. He looks unwell but is fully conscious. He has conjunctival
suffusion and scleral icterus. Blood tests show: Haemoglobin 10.5 g/dL, White
cell count 22.5 x109/L,Neutrophils 19x109/L, Platelets 150x109/L,
Urea22.5mmol/L, Creatinine 250 micromol/L, Bilirubin 150 micromol/L,
Aspartate Amino-Transferase 350 U/L.
The mostlikely diagnosis is:
A- Hepatitis B
B- Hepatitis A
C- Leptospirosis
D- Haemolytic uraemic syndrome
E- Reye’s syndrome

Answer :C
Leptospirosis is caused by a spirochete organism, of which there are many
serovars. It is contracted by contact with water contaminated with the urine or
carcasses of infected animals eg rats. There have been cases in the UK associated
with rats around waterways. It may cause asymptomatic infection, or an
influenza like illness which may progress to severe disease with jaundice and
renal impairment (Weil’s disease). Conjunctival suffusion is characteristic but
not always present. Viral hepatitis is characterised by a prodromal phase with
fever in those who are symptomatic, followed by hepatitis after the fever
declined. Hepatitis A is frequently asymptomatic in children, and hepatitis B
rarely causes acute hepatitis. Haemolytic uraemic syndrome typically follows a
gastrointestinal disorder with bloody diarrhoea. The commonest aetiologic agent
is E Coli 0157:H7. Reye’s syndrome is an acute and often fatal encephalopathy
associated with hepatic failure. It is becoming increasingly rare.

Q 5:A 5-year-old boy is hospitalized in January with fever and seizures. Lumbar
puncture reveals clear cerebrospinal fluid that has a white blood cell count of
47/cu mm, all of which are lymphocytes. On physical examination, he appears
obtunded but arouses with painful stimuli. Neurologic examination reveals no
focal findings.
Of the following, the diagnostic test that is MOST likely to reveal the etiology of
this child's illness is:

A. bacterial culture of cerebrospinal fluid


B. polymerase chain reaction test of cerebrospinal fluid for herpes simplex
C. Streptococcus pneumoniae bacterial antigen test of cerebrospinal fluid
D. viral culture of cerebrospinal fluid
E. viral culture of nasopharyngeal and rectal swabs

Answer :B
The boy described in the vignette has symptoms suggestive of encephalitis. These
symptoms, combined with the cerebrospinal fluid (CSF) findings, are most
consistent with a viral etiology. The most likely pathogen in a sporadic case of
viral encephalitis is herpes simplex virus (HSV).

In the past, HSV encephalitis was diagnosed by culture or direct fluorescence


testing of brain biopsy tissue. More recently, polymerase chain reaction (PCR)
testing of CSF for HSV DNA has become the preferred diagnostic modality.

Viral cultures of the CSF for herpes are rarely positive in HSV encephalitis
beyond the neonatal period, and the virus is not found in cultures of sites outside
the central nervous system.

Bacterial culture of CSF or use of antigen detection tests for Streptococcus


pneumoniae are not likely to be positive in a child whose findings are consistent
with encephalitis.
Q 6 :A 3-year-old child is brought to the emergency department with a fever of
103.1°F (39.5°C) and diarrhea of acute onset. The stool is guaiac-positive and
contains leukocytes. There is no history of foreign travel, and the child has not
received antibiotics recently.Of the following, the organism that is MOST likely
to be isolated from this child's stool is:

A. Clostridium difficile

B. Giardia lamblia

C. rotavirus

D. Salmonella enteritidis

E. Vibrio cholerae

Answer:D
Infectious diarrhea is a common illness among children and is caused by a wide
variety of pathogens. The clinical presentation of the child can aid in identifying
the likely pathogen.
Children who have viral diarrheas usually have low-grade fever; vomiting; and
large, loose, watery stools. Dehydration commonly accompanies rotavirus
infection, which is the most common of the viral diarrheas.
The symptoms exhibited by the child in the vignette are most consistent with a
bacterial diarrhea, such as those caused by Salmonella or Shigella sp. Patients
who have these infections often present with high fevers and small, frequent
stools that contain mucus or blood. Stool cultures reveal the pathogen, and
susceptibility testing of the isolate is useful because many Salmonella and
Shigella isolates are resistant to ampicillin and trimethoprim-sulfamethoxazole.
Although antibiotic treatment is indicated for Shigella infections, Salmonella
gastroenteritis is self-limited in immunocompetent patients, and antibiotic treatment
usually is withheld because it may prolong carriage of the organism.
Clostridium difficile is most common in the setting of antibiotic-induced colitis.
Vibrio cholerae is acquired from contaminated seafood or water and rarely is seen in
the United States. Infection with Giardia lamblia is more likely to result in chronic or
persistent diarrhea with malabsorption
Q 7 :A 2-year-old girl presents with a swollen, tender, erythematous knee. Two
weeks ago she had fever and bloody diarrhea that lasted 4 days.

Of the following, the MOST likely organism to be associated with arthritis in


this patient is:
A. Escherichia coli

B. Giardia lamblia

C. Norwalk virus

D. rotavirus

E. Shigella flexneri

Answer :E
Postinfectious or reactive arthritis often occurs several weeks or months after an
acute infection.

Reactive arthritis frequently follows enteric infections with Shigella, Salmonella,


Yersinia, and Campylobacter sp.

As described in the vignette, affected children initially develop bloody diarrhea,


followed by the onset of arthritis, typically 1 to 2 weeks after the triggering
infection.

Reactive arthritides are usually acute and self-limited, resolving within weeks or
months.

There is no specific treatment for reactive arthritis. The patient may need
analgesics for pain relief. Of the choices listed, Shigella would be the most likely
organism to cause bloody diarrhea and arthritis.

Other important examples of reactive arthritis include postvenereal


reactive arthritis (especially with Chlamydia trachomatis) and virus-related
arthritis.

A variety of viruses have been associated with reactive arthritis, including


rubella, hepatitis B, mumps, parvoviruses, and herpesviruses.
Poststreptococcal reactive arthritis and acute rheumatic fever (ARF) are two
other examples of reactive arthritis.

Reactive arthritis does not typically follow infections with Escherichia


coli, Giardia lamblia, Norwalk virus, or rotavirus.

Q8;-A 12-year-old girl who has systemic lupus erythematosus was exposed to
varicella 24 hours ago. She has been receiving prednisone 40 mg bid for 9 weeks
because of an exacerbation of nephropathy. She has not had varicella or
received varicella immunization.
Of the following, the MOST appropriate next step is to:
A. administer varicella vaccine

B. administer varicella-zoster immune globulin

C. begin prophylactic doses of acyclovir

D. discontinue the prednisone

E. provide stress doses of prednisone

---

Q 9:-A 5-year-old girl complains of perianal pruritus. Results of a clear adhesive


tape test are positive.Of the following, the drug of CHOICE for this infection is:
A. iodoquinol
B. ivermectin
C. mebendazole
D. praziquantel
E. thiabendazole

Answer:-C
Perianal pruritus is a common symptom of infection with Enterobius
vermicularis (pinworms). Although infection may appear in all age groups and
socioeconomic levels, it is most prevalent in preschool and school-age children.
Typically, embryonated eggs are ingested and migrate to the duodenum, where
they hatch and undergo sexual maturation before reaching the cecum. Adult
pinworms reside in the cecum, emerge at night through the anus, and migrate to
the perianal region, where gravid females deposit their eggs and die. The eggs
cause anal pruritus, which leads to scratching and accumulation under the
fingernails, thereby promoting autoinfection and spread to close contacts. The
eggs remain infective for 2 to 3 weeks. Aberrant migration of the adult worm
from the perineum rarely may give rise to urethritis, vaginitis, salpingitis, or
pelvic peritonitis.

Some physicians treat the infestation based only on the history, but a
definitive diagnosis should be made. Eggs are detected easily on clear adhesive
tape that is applied to the perianal area early in the morning on awakening. The
tape is applied to a slide and viewed under a low-power microscopic lens.
Repeated examinations on successive mornings may be necessary. Because
Enterobius vermicularis eggs are not excreted in the stool, examination of feces
is not a useful test.

The drugs of choice for treatment of enterobiasis are either mebendazole


(100 mg regardless of weight), pyrantel pamoate (11 mg/kg, not to exceed 1 g), or
albendazole (400 mg) administered as a single dose. Because none of these drugs
is completely effective against eggs or developing larvae, a second treatment 2
weeks after the first is recommended. Frequently, all family members are
treated in an attempt to break the cycle of reinfection.
Because pinworm infection often carries substantial unwarranted social stigma,
reassurance of families that this infection is very common, often recurs, and does
not reflect uncleanliness is an important component of therapy.

Reinfection with pinworms occurs easily. Measures that may reduce egg
contamination of the local environment are helpful and include:
• having the infected person bathe in the morning, which removes a large
proportion of the eggs;
• frequent changing of the infected person’s underclothes, bed clothes,
and bedsheets;
• hygienic measures such as washing hands prior to eating or preparing
food, keeping fingernails short, and avoiding nail biting.
Measures such as cleaning or vacuuming the entire house or washing bed
clothes and bedsheets daily are not necessary.

Mebendazole also is an effective treatment for other roundworm


infections, such as ascariasis, capillariasis, hookworm infections, trichinosis,
whipworm infections, and visceral larva migrans. Iodoquinol is used to eradicate
intestinal carriage of Entamoeba histolytica. Ivermectin is recommended for
treatment of cutaneous larva migrans, river blindness (infection with
Onchocerca volvulus), and strongyloidiasis. Praziquantel is the drug of choice
for treatment of fluke and tapeworm infections, such as schistosomiasis and
cysticercosis. Thiabendazole is effective in treating strongyloidiasis and
cutaneous larva migrans.
Q 10;-A 2-year-old boy presents with rales, pallor, chronic failure to thrive,
recurrent thrush, diarrhea, and oxygen saturation of 84% on room air.
Echocardiography demonstrates an enlarged left ventricle with diminished
systolic function.
Of the following, the blood test MOST likely to establish the diagnosis in this
child is:

A. antibody testing for Epstein-Barr virus


B. antibody testing for human immunodeficiency virus
C. antibody testing for human parvovirus
D. serum carnitine level
E. serum selenium level

Answer B
It is now appreciated that varying degrees of myocardial dysfunction are
common in human immunodeficiency virus (HIV) infection in children,
especially when the infection has reached the point of clinical immunodeficiency.
Pallor is common in affected children from the combination of anemia and
congestive heart failure. Diarrhea probably is related more to the acquired
immunodeficiency syndrome (AIDS) than to the cardiomyopathy. The unusually
low oxygen saturation may be explained by interstitial pneumonitis, sometimes
due to Pneumocystis jiroveci (carinii) infection. Although maternal HIV
screening and treatment have decreased significantly the number of children
who present in the first few years of life with cardiomyopathy and frank AIDS
symptoms, cases still do occur in clinical practice.

It has been proposed that the dilated cardiomyopathy of childhood AIDS


is due primarily to chronic viral myocarditis from coxsackievirus, adenovirus,
or cytomegalovirus that is not cleared effectively by the damaged immune
system.
Epstein-Barr viral infection of the myocardium has bee
n diagnosed by polymerase chain reaction analysis of myocardial biopsy in some
children who have dilated cardiomyopathy. Clinical signs of Epstein-Barr virus-
related myocarditis are not specific and include cardiomegaly, poor systolic left
ventricular function, and physical signs of congestive heart failure.
Human parvovirus may cause a number of clinical illnesses, including
erythema infectiosum (“fifth disease”) or papulopurpuric “gloves and socks”
syndrome. It does not have any important association with myocarditis.
Serum carnitine levels may be normal or decreased in children who have
cardiomyopathy from a variety of causes. Low serum carnitine concentrations
do not define or suggest a single specific etiology in cardiomyopathy.

Selenium deficiency is a rare mineral deficiency disorder believed to be


associated with cardiomyopathy. Some investigators believe that selenium
deficiency is common in AIDS and postulate a role for it in the cardiac
dysfunction of AIDS infection. However, as noted previously, others believe that
chronic viral infection

Q 11.A 10 year old child has just been diagnosed with meningococcal
meningitis.
In discussing chemoprophylaxis with his family, you are MOST likely to include
the statement that rifampin:

A. causes a reactive arthritis


B. causes discoloration of body fluids
C. decreases the reliability of depot medroxyprogesterone
D. is contraindicated if she has asthma
E. is safely used during pregnancy

Answer B
Rifampin penetrates the central nervous system and is found in most body
fluids. It can cause orange-colored secretions, including urine, sweat, and tears.
Patients should be advised that contact lenses may be stained orange.

Rifampin is metabolized by the liver and excreted in bile and urine. It


can alter the serum concentrations of many drugs and possibly interfere with
the efficacy of oral contraceptives. The reliability of intramuscular
medroxyprogesterone is not altered with rifampin use. Neither rifampin nor
ciprofloxacin is recommended for use during pregnancy. A single intramuscular
dose of ceftriaxone is the recommended prophylaxis during pregnancy.

Rifampin therapy is not contraindicated for patients who have asthma,


although its use may decrease the efficacy of corticosteroids. Reactive arthritis is
not a common adverse reaction associated with rifampin.
Q 11;-A child who has acute myelogenous leukemia is being treated for
Pseudomonas bacteremia with intravenous doses of piperacillin and gentamicin.
Gentamicin levels are measured after 2 days of therapy.How long after
completing a 30-minute infusion should blood for peak gentamicin
concentrations be drawn?:
A. 30 minutes
B. 60 minutes
C. 90 minutes
D. 120 minutes
E. 150 minutes

Answer B
Therapeutic drug monitoring is used to prevent or decrease the risk of toxic
effects of medication. Monitoring serum concentrations of most antibiotics is
unnecessary because these drugs are effective over a wide range of serum levels,
therapeutic levels are achieved easily, and levels associated with toxicity rarely
are encountered when standard dosing schedules are employed and patients
have normal clearance mechanisms. However, certain antibiotics, especially
chloramphenicol, vancomycin, and the aminoglycosides, have narrow
therapeutic windows and are associated with potential adverse reactions.
Therefore, careful monitoring of serum concentrations of these drugs is critical.
Measurement of serum drug levels can help determine the dose and
frequency of administration that allow for maximum therapeutic benefit with
minimum toxicity. Appropriately timed blood samples are essential for accurate
interpretation of serum drug levels. The best times to obtain blood samples for
most parenterally administered antibiotics is 30 minutes after a 20- to 30-minute
intravenous infusion, when the level is presumed to be highest (peak level), and
immediately before the next dose, when the level is presumed to be lowest
(trough level). For oral antibiotics, peak levels should be obtained 30 minutes to
1 hour after oral liquid or 1.5 hours following oral capsule administration.

The principles of therapeutic drug monitoring are based on two


pharmacokinetic parameters: volume of distribution (Vd) and half-life (t1/2). Vd
is the hypothetical volume within which the drug is distributed and is used to
determine the dose required to maximize activity. The t1/2 reflects the rate of
drug elimination and, thus, is used to determine the most appropriate frequency
of dosing. The blood sample obtained 1 hour after completing the infusion
provides information about the Vd after the drug has begun to be dispersed
through the body but before significant amounts have been eliminated. The
trough level, drawn immediately before the next dose, helps to determine
elimination kinetics and t1/2.

A level drawn 30 minutes after completing a gentamicin infusion will not


be a reliable indicator of Vd because not enough time has passed for drug
distribution to begin. Serum samples drawn 90, 120, or 150 minutes after
completing the infusion are not as reliable as a sample obtained 1 hour after
completing the infusion because drug elimination will have begun.

Aminoglycoside antibiotics (eg, gentamicin, tobramycin, amikacin) have


a high profile of toxic side effects, such as nephrotoxicity and ototoxicity.
Although aminoglycoside-induced renal injury usually is reversible, ototoxicity,
characterized by both auditory and vestibular nerve damage, is not. Individual
risk factors may contribute to the development of toxicity, but the major
association with organ damage is elevated peak and trough serum drug
concentrations. Sustained peak serum gentamicin concentrations of more than
12 to 14 mg/L and trough serum concentrations of more than 2 mg/L have been
associated with a significantly increased risk of both toxicities.

Monitoring of serum aminoglycoside peak and trough concentrations has


been shown to decrease the incidence of nephrotoxicity, although these toxicities
still can occur in patients whose serum concentrations are in the desired
therapeutic range. Thus, regular monitoring of levels is recommended to assure
the adequacy of the dosing regimen and to monitor for drug accumulation and
potential toxicity. Serial trough concentrations correlate better than peak levels
with the rising tissue accumulation of drug during a course of treatment.

Peak and trough serum concentrations should be measured following the


fifth or sixth dose of the aminoglycoside. If these levels are appropriate, serial
trough concentrations should be obtained every 4 to 7 days, depending on the
clinical status of the patient. Sustained elevation of the trough concentration in
excess of 25% over a 2- to 4-day period has been found to place patients at
measurable risk for aminoglycoside-induced toxicity.

Q12;-A child is bitten on the hand by a neighbor's dog. Within 24 hours there is
erythema, pain, and swelling at the site of the bite. The child is taken to the
emergency department where cultures are taken of sanguinopurulent drainage
from the wound.Of the following, the MOST likely organism infecting the
wound is:

A. Eikenella corrodens
B. Francisella tularensis
C. Pasteurella multocida
D. Staphylococcus aureus
E. Streptococcus pyogenes

Answer:-C
Pasteurella multocida is the organism most likely to infect animal bite wounds.
Clinical infection with P multocida is characterized by the rapid evolution of an
intense inflammatory response, with substantial pain and swelling developing
within 24 hours of the initial injury in 70% of cases and by 48 hours in 90% of
patients who develop an infection. P multocida infection has resulted in abscess
formation, septic arthritis, osteomyelitis, sepsis, meningitis, endocarditis, and
pneumonia. Infections usually exhibit localized cellulitis and purulent discharge.
Fever, regional adenopathy, and lymphangitis are seen in fewer than 20% of
patients.

The drug of choice for treatment of P multocida infections is penicillin. Other


effective agents include ampicillin, amoxicillin-clavulanate, cefuroxime,
cefpodoxime, trimethoprim-sulfamethoxazole, and tetracycline. For patients
allergic to beta-lactam agents, tetracycline is effective, but it should not be
administered to children younger than 8 years of age
Fourth PART
1. A 13-day-old male infant presents with a fever (temperature up to 100.6°F
[38.1°C]), mild irritability, and diminished appetite. His parents report no
change in the number of wet diapers. Which of the following statements
regarding this patient’s management or prognosis
is correct?
A. Careful observation at home is appropriate because of the relatively low fever and
normal urine output.
B. The risk of bacteremia in this patient is approximately 25%.
C. Intramuscular ceftriaxone and close home monitoring are appropriate after
evaluation with a complete blood count, blood culture, urinalysis, and urine culture.
D. Irrespective of the results of initial laboratory testing, management should include
intravenous antibiotics and hospitalization.
E. Bacteria likely to cause fever in this patient include Streptococcus pneumoniae and
Haemophilus influenzae type b.
Fever in an infant younger than 28 days of age must be
taken very seriously because the neonate’s immune system is immature. As a result,
the current appropriate management for any neonate with fever (temperature
>100.4°F [>38°C]) includes a complete workup for serious bacterial infection (SBI)
that includes evaluation of the blood, urine, and cerebrospinal fluid for evidence of
bacterial infection; administration of empiric intravenous antibiotics; and
hospitalization. The risk of SBI in a nontoxic infant younger than 3 months of age is
approximately 1–7%. Usual bacteria resulting in infection in this age group include
group B streptococcus, Escherichia coli, and Listeria monocytogenes.

2. A 10-month-old female infant with up-to-date immunizations presents with a


fever (temperature up to 103.5°F [39.7°C]) for the past 3 days. She was
previously healthy. Her parents report no symptoms other than the fever. On
examination, she is well hydrated and appears nontoxic, and no focus of
infection is identified. Which of the following is the next appropriate
management step?
A. Complete blood count (CBC) and blood culture, and empiric oral antibiotics to
cover likely causes of bacteremia
B. Urine culture and urinalysis, and if suggestive of a urinary tract infection, empiric
antibiotics to cover likely urinary pathogens
C. No laboratory studies are indicated because the patient appears nontoxic.
D. Hospitalization and empiric intravenous cefotaxime
E. CBC, blood culture, urinalysis, urine culture, lumbar puncture, and chest
radiograph;intramuscular ceftriaxone should be given because of the high risk of
bacteremia.

2. The answer is B [II.D.3 and Table 7-1]. Because of the patient’s elevated fever,
evaluation for a
urinary tract infection, including urine culture and urinalysis, is indicated. Because
the infant
is completely immunized, risk of bacteremia is low enough not to warrant blood tests.
If she
were incompletely immunized (had not received her 2-, 4-, and 6-month
vaccinations), a
complete blood count and blood culture would be indicated and intramuscular
ceftriaxone
may be given either empirically, or only if the white blood count is ≥15,000
cells/mm3.
Hospitalization is generally not required unless the patient is toxic in appearance, is
dehydrated, or has poor ability to return to the physician for follow-up. Neither
evaluation of
spinal fluid nor a chest radiograph is indicated in this nontoxic patient without
respiratory
signs or symptoms. Neither intravenous antibiotics nor hospitalization is indicated
because
the infant is nontoxic and well hydrated.

3. A 2-year-old girl presents with fever. On examination, she has exudative


pharyngitis, enlarged
posterior cervical lymph nodes, and splenomegaly. Which of the following
statements
regarding her evaluation and management is correct?
A. Amoxicillin should be prescribed after performing a throat culture for suspected
“strep throat.”
B. Monospot testing is highly sensitive and is the best test to make a diagnosis in this
case.
C. Human immunodeficiency virus is the most likely cause of this infection .
D. Amoxicillin may result in a pruritic rash in this patient.
E. The patient should be administered corticosteroids that will lead to rapid
improvement and resumption of full activity.

3. The answer is D This patient’s clinical presentation with fever, lymphadenopathy,


pharyngitis, and splenomegaly is most consistent with infectious mononucleosis. If a
child with infectious mononucleosis is given amoxicillin, a diffuse pruritic rash may
develop.Monospot testing is highly sensitive in older children, but heterophile
antibodies do not reliably form in children younger than 4 years of age. Antibody
titers are therefore the preferred diagnostic test in such young children. The most
common cause of infectious mononucleosis is Epstein–Barr virus. Although
supportive care is most appropriate,corticosteroids may be indicated for treatment of
infectious mononucleosis complicated by airway compromise, but are not routinely
recommended. Splenomegaly is not consistent with the diagnosis of streptococcal
pharyngitis.
4. A 6-year-old girl is sent home from summer camp with a temperature of
101.3°F (38.5°C), stiff neck, photophobia, and headache. Lumbar puncture in
the emergency department reveals the
following results: white blood count 380 cells/mm3, with 65%
polymorphonuclear cells and
35% lymphocytes; normal protein and glucose; and negative Gram stain. Which
of the
following pathogens is the most likely cause of her meningitis?
A. Neisseria meningitidis
B. Streptococcus pneumoniae
C. Enterovirus
D. Borrelia burgdorferi
E. Mycobacterium tuberculosis
4. The answer is C . This cerebrospinal fluid (CSF) evaluation is
most consistent with aseptic meningitis, specifically viral meningitis. Enteroviruses
are the most common cause of viral meningitis and most often occur during the
summer and fall.Early in viral meningitis, the white blood cell (WBC) count in the
CSF may demonstrate apolymorphonuclear cell predominance that shifts to a
lymphocyte predominance within 24–48 hours. The normal protein and glucose and
negative Gram stain are also consistent with viral meningitis. Meningitis caused by
Neisseria meningitidis or Streptococcus pneumoniae would be reflected by a higher
CSF WBC count, lower glucose, and higher protein. Although patients
with Lyme meningitis, which is caused by Borrelia burgdorferi, may present with an
aseptic CSF profile, the onset is not as acute as in this patient. Patients with
Mycobacterium tuberculosis

5. A previously healthy 18-month-old girl is admitted to the hospital with fever


(temperature up to 102.8°F [39.3°C]), vomiting, and lethargy. She was well until
2 days ago, when she was diagnosed with a viral upper respiratory infection.
Lumbar puncture to evaluate the cerebrospinal fluid shows the following
results: white blood cells 3050 cells/mm3, with 98% polymorphonuclear cells;
very low glucose; and elevated protein. Gram stain shows Grampositive
diplococci. Initial management should include which of the following?
A. Vancomycin and third-generation cephalosporin
B. Third-generation cephalosporin alone
C. Ampicillin and third-generation cephalosporin
D. Third-generation cephalosporin and acyclovir
E. Third-generation cephalosporin and corticosteroids
5. The answer is A. Empiric therapy of presumed bacterial meningitis should include
athird-generation cephalosporin and the addition of vancomycin until sensitivities are
available, because of the high level of pneumococcal antibiotic resistance in many
communities. Ampicillin is not indicated because this child is out of the age range at
which Listeria infection occurs. Acyclovir is not indicated because the cerebrospinal
fluid profile is most consistent with bacterial meningitis, not viral meningitis.
Acyclovir is used to treat neonates with presumed herpes simplex meningitis, or
encephalitis or in older children with encephalitis. Corticosteroids are effective in
reducing the incidence of hearing loss in Haemophilus influenzae type b meningitis
but have not been shown to be effective for other
bacterial pathogens.
6. A 25-year-old woman is pregnant with her first child. The woman has human
immunodeficiency virus (HIV) infection that was diagnosed 2 years before this
pregnancy.Which of the following has been shown to increase her risk of
transmitting HIV to her infant?
A. Treatment with highly active antiretroviral therapy during pregnancy and before
delivery
B. Exclusive bottle formula feeding
C. Prolonged rupture of membranes
D. Birth by cesarean section
E. Orally administered zidovudine given to the infant after birth
6. The answer is C. Factors that increase the risk of HIV transmission from mother to
infant include high maternal viral load (measured by RNA copy number) at delivery,
concomitant chorioamnionitis or other genital tract infections, primary or advanced
maternal
HIV infection, premature birth, and prolonged rupture of membranes. Transmission
may also
occur through breast milk. Transmission is decreased through the use of maternal
antiretroviral therapy, newborn prophylaxis with antiretroviral agents (e.g.,
zidovudine), birth
by cesarean section, and low maternal viral load.

7. An 8-year-old girl presents with sore throat, fever, and a rough sandpaper-
like rash over her trunk and extremities. A throat culture is positive for group A
β-hemolytic streptococcus.Treatment of her infection with antibiotics will
prevent which of the following complications?
A. Reactive arthritis
B. Rheumatic fever
C. Poststreptococcal glomerulonephritis
D. Guillain–Barré syndrome
7. The answer is B .This patient’s clinical presentation of a sandpaper-like rash
associated with pharyngitis and fever is consistent with scarlet fever, caused by
erythrogenic
toxin–producing strains of group A β–hemolytic streptococcus (GABHS). Although
there are
multiple complications of GABHS infection, including rheumatic fever,
glomerulonephritis,
and reactive arthritis, only rheumatic fever will be prevented by treatment with
antibiotics.

8. A 1-year-old girl presents with weight loss and a 2-week history of large,
bulky, nonbloody, foul-smelling stools. She has been attending day care and
recently received amoxicillin for an ear infection. Which of the following is the
most likely cause of her diarrhea?
A. Entamoeba histolytica
B. Enterotoxigenic Escherichia coli
C. Clostridium difficile
D. Giardia lamblia
E. Norwalk virus
8. The answer is D. Infection with the protozoan Giardia lamblia is
associated with bulky, foul-smelling stools, weight loss, and day care attendance.
Entamoeba
histolytica and Clostridium difficile generally cause bloody diarrhea. Escherichia coli
infection
generally results in short-term watery diarrhea. Day care attendance is also associated
with
Norwalk virus; however, symptoms of Norwalk virus infection generally last only
48–
72 hours.

9. A 19-year-old boy, a college sophomore, presents with high fever, headache,


cough,conjunctivitis, and a diffuse macular rash over his trunk and face. He is
unsure of his immunization status. You suspect measles infection. Which of the
following is correct regarding this diagnosis?
A. Vitamin A may improve his outcome.
B. Koplik spots would likely be present on examination of his mouth.
C. Mortality is most commonly caused by measles encephalitis.
D. Diagnosis is based on culture and direct fluorescent antigen testing.
E. Corticosteroids will decrease symptoms and improve outcome.
9. The answer is A. This patient’s presentation is most consistent
with measles infection. Management includes supportive care, and vitamin A
therapy may
also be beneficial. Koplik spots are transient, and by the time the rash is present,
Koplik spots are no longer appreciated. Bacterial pneumonia is the most
common complication of measles infection and is the most common cause of
mortality. Diagnosis is based on confirmation by serologic testing in the presence
of typical clinical features. Corticosteroids do not play a role
in the therapy of measles.
The response options for statements 10–14 are the same. You will be required to
select one answer for each statement in the set.
A. Malaria Plasmodium species
B. Toxoplasma gondii
C. Giardia lamblia
D. Entamoeba histolytica
E. Coccidioides immitis
F. Cryptococcus neoformans
G. Aspergillus fumigatus
H. Candida albicans
For each clinical description, select the most likely cause.
1. At birth, a term infant is noted to have hydrocephalus and intracranial
calcifications on computed tomography of the head. Eye examination reveals bilateral
chorioretinitis.
(Answer is BThe triad of intracranial calcification, hydrocephalus, and
chorioretinitis is consistent with congenital toxoplasmosis, which is caused by
Toxoplasma gondii.)

2. A 5-year-old boy is admitted with a fever of unknown origin. An abdominal


computed tomographic scan reveals a large hepatic abscess.
(Answer isD Entamoeba histolytica may result in asymptomatic infection or
colitis. The most common extraintestinal complication is a
liver abscess. .)

3. A 12-year-old girl with cystic fibrosis has an exacerbation of her disease and
presents with wheezing, pulmonary infiltrates, and eosinophilia.
(Answer G Aspergillus infection may result in invasive disease or in noninvasive
allergic disease characterized by wheezing, eosinophilia, and pulmonary
infiltrates. This can occur in patients with severe asthma or with cystic fibrosis.
.)
4. A 16-year-old boy is admitted to the hospital for a workup of cyclical fevers after a
trip to India. His illness began with flulike symptoms.
(Answer is AMalaria classically presents with a flulike illness followed by the
development of high fevers that cycle in 48- to 72-hour paroxysms. )
5. An 18-month-old girl and three of her day care classmates present with 2 weeks of
watery diarrhea and some weight loss.
(CGiardia lamblia typically presents with bulky, large-volume, watery stools that
eventually lead)

6. A 2-year-old boy has a positive tuberculin skin test that measures 12 mm. It was
placed during a routine well-child care visit. He is well, without fever, chills, cough,
weight loss, or night sweats. No known tuberculosis contacts are identified. Which of
the following statements regarding this patient’s management is correct?
A. A chest radiograph should be ordered because the tuberculin test is positive.
B. He should be placed into respiratory isolation immediately because he is likely to
spread tuberculosis to others.
C. Isoniazid is not indicated because this tuberculin skin test is negative.
D. Triple drug therapy for tuberculosis should be started immediately.
E. Gastric aspirates should be ordered.
(The answer is A tuberculin skin test is considered positive depending on a
patient’s specific risk factors for acquisition of tuberculosis. A tuberculin skin test
≥10 mm is considered positive if the patient is younger than 4 years of age or if the
patient resides or has lived in an area endemic for tuberculosis. Therefore, given that
the tuberculin skin test is positive in this patient, a chest radiograph to evaluate for
pulmonary tuberculosis is indicated.Children younger than 10 years of age with
tuberculosis are unlikely to be contagious because of minimal cough and pulmonary
involvement. Medications for tuberculosis disease (e.g.,rifampin, isoniazid,
pyrazinamide, ethambutol regimen) are indicated if the patient has signs
and symptoms of tuberculosis. Gastric aspirates are indicated only if the chest
radiograph reveals pulmonary disease.)

The response options for statements 16–20 are the same. You will be required to
select one answer for each statement in the set.
A. Salmonella species
B. Shigella sonnei
C. Yersinia enterocolitica
D. Clostridium difficile
E. Campylobacter jejuni
F. Vibrio cholerae
G. Enterotoxigenic Escherichia coli
H. E. coli 0157:H7
Match the clinical description with the likely causative organism.
1. While visiting Monterey, Mexico, a 16-year-old boy develops watery, nonbloody
diarrhea, without fever.
(Answer is G Enterotoxigenic Escherichia coli is
the major cause of traveler’s diarrhea and results in nonbloody watery stools. Bloody
stools may result from infection with Salmonella, Shigella, Yersinia, Campylobacter,
enterohemorrhagic E. coli, and Clostridium difficile.)
2. A 3-year-old boy presents with an acute onset of high fevers, bloody diarrhea, and
ageneralized tonic-clonic seizure. The stool Wright stain reveals sheets of white
blood cells.
(Answer is B Shigella may be associated with seizures caused by the release
of a neurotoxin.)
3. An 8-year-old girl presents with a 1-week history of diarrhea and low-grade fever.
The family reports that they have recently acquired a pet turtle.
(Answer is A Salmonella may be acquired by ingestion of contaminated poultry or by
exposure to turtles and lizards that carry the organism.)
4. A 10-year-old boy is admitted to the hospital and taken directly to the operating
room for suspected acute appendicitis. Surgeons discover a normal appendix but
enlarged mesenteric lymph nodes.
( Answer is C Yersinia may result in mesenteric
adenitis that causes pain mimicking acute appendicitis. )
5. A group of travelers to Bangladesh suddenly develop massive, watery, nonbloody
diarrhea that results in severe dehydration and electrolyte imbalance.
(Answer is F Infection with Vibrio cholerae generally
occurs in developing countries and causes massive fluid loss from the gut.).

The response options for statements 21–24 are the same. You will be required to
select one answer for each statement in the set.
A. Buccal cellulitis
B. Impetigo
C. Necrotizing fasciitis
D. Erysipelas
E. Staphylococcal scalded skin syndrome
F. Toxic shock syndrome
Match the clinical description with the likely diagnosis.
1. A 9-month-old girl with mild facial eczema has fever and a facial skin rash.
The skin lesion is weepy with a honey-colored crust.
(Answer is B Impetigo typically presents with honeycrusted lesions on the face; it is
caused by infection with Staphylococcus aureus and group A β-hemolytic
streptococcus (GABHS). )

2. An unvaccinated 4-month-old boy has a blue color to his cheeks and a positive
blood culture for Haemophilus influenzae type b.
(Answer is A Buccal cellulitis is characterized by a bluish color to the cheeks of a
young child; this condition is typically caused by infection with Haemophilus
influenzae type b, which is identified on blood culture.)
3. An infant boy has fever, an erythematous skin rash, and a positive Nikolsky
sign.
(Answer is E---- Staphylococcal scalded skin syndrome is manifested by Nikolsky
sign or the extension of bullae with lateral pressure applied to the skin. Fever, tender
skin, and widespread bullae are present. )

4. A 7-year-old girl develops fever and a rapidly expanding tender skin rash
with a well demarcated border.
(Answer is D --- Necrotizing fasciitis is a severe skin infection that involves multiple
layers of tissue. Toxic shock presents with fever, sunburn rash, and multiorgan
failure. Both of these complications are a result of toxin produced by the
GABHS. Necrotizing fasciitis and toxic shock syndrome are complications of
infection with GABHS. Erysipelas is characterized by tender, erythematous skin, but
the border is well demarcated.

13. The answers are B, A, E, and D, respectively .


nfectious Diseases
Fifth Part
QUESTIONS
1. Approximately 80% of blood cultures that will be positive are identified at
A. 1st 24 hr from incubation
B. 25 – 48 hr from incubation
C. aGer 3 day from incubation
D. aGer 5 day from incubation
E. aGer 7 day from incubation
1.(A).

2. The op mal amount of blood to collect from a pediatric pa ent for blood
culture
depend on
A. age
B. weight
C. length
D. BMI
E. laboratory standards
2.(B).

3. The IgM response occurs earlier in the illness, generally peaking at 7-10 days
after infection, and usually disappears within a few weeks, but for some infections
it can persist for months such as
A. measles
B. West Nile Virus
C. rubella
D. mumps
E. varicella
3.( B). Hepatitis A and West Nile Virus.
4. Which of the following is whole inactivated microorganisms vaccine?
A. hepatitis A
B. hepatitis B
C. pneumococcal.
D. varicella.
E. diphtheria.
4.(A-B). Vaccines can consist of whole inactivated microorganisms (e.g., polio and
hepatitis A), parts of the organism (e.g., acellular pertussis, HPV, and hepatitis B),
polysaccharide capsules (e.g., pneumococcal and meningococcal polysaccharide
vaccines), polysaccharide capsules conjugated to protein carriers (e.g., Hib,
pneumococcal, and meningococcal conjugate vaccines), live attenuated
microorganisms (measles, mumps, rubella, varicella, rotavirus, and live-attenuated
influenza vaccines), and toxoids (tetanus and diphtheria).

5. Which of the following is a T-lymphocyte independent vaccine?


A. hepatitis A
B. hepatitis B
C. pneumococcal
D. varicella
E. diphtheria
5.(C). T-lymphocyte independent vaccines are associated with poor immune responses
in children <2 yr of age, short-term immunity, and absence of an enhanced or booster
response on repeat exposure to the antigen. With some polysaccharide vaccines,
repeat doses actually are associated with reduced responses, as measured by antibody
concentra ons, compared to 1st doses (i.e., hyporesponsive). To overcome problems
of plain polysaccharide vaccines, polysaccharides have been conjugated, or covalently
linked, to protein carriers, converting the vaccine to a T-lymphocyte dependent
vaccine. In contrast to plain polysaccharide vaccines, conjugate vaccines induce higheravidity
antibody, immunologic memory leading to booster responses on repeat
exposure to the antigen, long-term immunity, and herd protection by decreasing
carriage of the organism.
6. The minimum age for the first dose of Rotavirus is
A. 11 wk
B. 13 wk
C. 6 weeks
D. 17 wk
E. 19 wk
6.(C).

7. The final dose of rotavirus vaccine must be administered no later than


A. 6 mo of age
B. 8 mo of age
C. 10 mo of age
D. 12 mo of age
E. 18 mo of age
7.(B). Two rotavirus vaccines are available, RotaTeq (RV5) and Rotarix (RV1). With both
vaccines, the 1st dose can be administered as early as 6 wk of age and must be
administered by 14 wk 6 days. The final dose in the series must be administered no
later than 8 mo of age. The RV5 vaccine is administered in 3 doses at least 4 wk apart.
The RV1 vaccine is administered in 2 doses at least 4 wk apart. Immuniza on should
not be ini ated for infants 15 wk of age and older as stated in the immuniza on
schedule
8. Immuniza on is one of the most beneficial and cost-effective disease prevention measures. As
a result of effective and safe vaccines, which of the following diseases has been eradicated?
A. smallpox
B. polio
C. measles
D. rubella
E. pertussis

8.(A). As a result of effective and safe vaccines, smallpox has been eradicated; polio is
close to worldwide eradication.
9. Infants, children, and adolescents in Iraq are routinely immunized against
A. 10 diseases
B. 11 diseases
C. 12 diseases
D. 13 diseases
E. 14 diseases
9.(B). Infants, children, and adolescents in the Iraq rou nely are immunized against 11
diseases: BCG, poliomyelitis, hepatitis B, diphtheria, tetanus, pertussis, H. influenzae
type b (Hib) disease, rotavirus, measles, mumps and rubella.
10. The MOST common adverse reaction to intramuscular immunoglobulin is
A. pain at the injection site
B. flushing
C. headache
D. chills
E. nausea
10.(A). The most common adverse reaction to immunoglobulin is pain and discomfort
at the injection site and, less commonly, flushing, headache, chills, and nausea.

11. All the following are major recommended indica ons for IVIG EXCEPT
A. replacement therapy for primary immunodeficiency disorders
B. Kawasaki disease
C. hepatitis A prophylaxis
D. immune-mediated thrombocytopenia
E. prophylaxis of infection following bone marrow transplantation
11.(C). In hepatitis A prophylaxis, intramuscular IG is recommended.
12. Serious reac ons to IVIG include all the following EXCEPT
A. anaphylactoid events
B. thromboembolic disorders
C. aseptic meningitis
D. carditis
E. renal insufficiency
12.(D).
13. A toxoid is a modified bacterial toxin that is made non-toxic but still able to induce
an active immune response against the toxin. Which of
the following vaccine is a toxoid?
A. hepatitis A
B. hepatitis B
C. pneumococcal
D. varicella
E. diphtheria
13.(E). Toxoids are (tetanus and diphtheria).

14. Hepa s A vaccine, licensed for administra on to children 12 mo of age and


older.
The 2 doses in the series should be separated by at least
A. 2 mo
B. 4 mo
C. 6 mo
D. 1 yr
E. 2 yr
14.(C).

15. The minimum interval between the 2 doses of MMR is


A. 2 wk
B. 4 wk
C. 2 mo
D. 6 mo
E. 1 yr
15.(B).
16. The minimum age for the last dose of hepa s B vaccine is
A. 16 weeks
B. 20 weeks
C. 24 weeks
D. 28 weeks
E. 32 weeks
16.(C). The minimum interval between 2nd and 3rd dose is 8 weeks andat least 16 weeks
aGer 1st dose. Minimum age for the last dose is 24 weeks.
17. Preterm infant, weight 1600 gm, should not receive the following vaccine at
birth
A. BCG
B. hepatitis B, if born to a HBs Ag negative mother
C. polio
D. DPT
E. MMR
17.(B). Preterm infants generally can be vaccinated at the same chronologic age as fullterm
infants according to the recommended childhood immunization schedule. An
exception is the birth dose of hepatitis B vaccine. Infants weighing ≥2 kg and who are
stable may receive a birth dose. However, hepatitis B vaccination should be deferred in
infants weighing <2 kg at birth un l 30 days of age, if born to a HBs Ag nega ve
mother. All preterm, low birth weight infants born to HBs Ag positive mothers should
receive hepa s B immunoglobulin and hepa s B vaccine within 12 hr of birth.
However, such infants should receive an addi onal 3 doses of vaccine star ng at 30
days of age.
18. Which of the following vaccines is contraindicated for a pa ent with X-linked
agammaglobulinemia?
A. BCG
B. hepatitis B
C. DPT
D. MMR
E. varicella
18.(A). Oral polio, smallpox, live attenuated influenza, and BCG all these vaccines are
contraindicated for this patient.
19. Which of the following vaccines is contraindicated for a pa ent with chronic renal
disease?
A. pneumococcal
B. hepatitis B
C. live attenuated influenza
D. varicella
E. hepatitis A

19.(C).

20. Waterless hand hygiene products are effec ve in killing most microbes but do
not
remove dirt or debris and are ineffective against
A. Pseudomonas
B. hepatitis A
C. Salmonella
D. C. difficile
E. S. aureus
20.(D). Waterless hand hygiene products increase hand hygiene compliance and save
time; these agents are the preferred agents for routine hand hygiene when hands are
not visibly soiled. These products are effective in killing most microbes but do not
remove dirt or debris. However, they are ineffective against C. difficile spores, requiring
the use of other cleansing products during hospital C. difficile outbreaks.

21. All the following are recognized skin infec ons or infesta ons in children in
childcare EXCEPT
A. impetigo
B. pediculosis
C. scabies
D. erythrasma
E. tinea corporis
21.(D).
22. A 3-year-old child diagnosed with shigella infection, he can return to his daycare
A. 2 days aGer ini a on of treatment
B. after diarrhea resolves
C. after diarrhea resolves and results of a single stool culture is negative for these
organisms
D. aGer diarrhea resolves and results of 2 stool cultures are nega ve for these
organisms
E. aGer diarrhea resolves and results of 3 stool cultures are nega ve for these
Organisms

22.(D).

23. Oral rehydra on is the mainstay of treatment for pediatric traveler’s diarrhea and
the drug of choice is
A. metronidazol
B. azithromycin
C. amoxicillin
D. trimethoprim-sulfamethoxazol
E. erythromycin

23.(B). The drug of choice is azithromycin (10 mg/kg once daily for up to 3 days, with
maximum daily dose of 500 mg). Ciprofloxacin (10 mg/kg per dose twice a day for up to
3 days, maximum dose of 500 mg twice a day) is an alterna ve for children >1 yr of age.
Amoxicillin, trimethoprim-sulfamethoxazole (cotrimoxazole), and erythromycin should
not be prescribed for self-treatment of traveler’s diarrhea, because of widespread
resistance among diarrheal pathogens.

24. Fever is defined as a rectal temperature of


A. ≥37.5°C
B. ≥37.6°C
C. ≥38°C
D. ≥38.1°C
E. ≥38.5°C
24.(C). Fever is defined as a rectal temperature ≥38 ° C (100.4°F) and a value >40°C
(104°F) is called hyperpyrexia.
25. Three mechanisms can produce fever: pyrogens, heat produc on exceeding loss,
and defective heat loss. Endogenous pyrogens include
A. antigen–antibody complexes
B. complement components
C. lymphocyte products
D. interferons β and γ
E. androgenic steroid metabolites

25.(D). Endogenous pyrogens include the cytokines interleukins 1 and 6, tumor necrosis
factor α, and interferons β and γ. Some substances produced within the body are not
pyrogens but are capable of stimulating endogenous pyrogens. Such substances include
antigen–antibody complexes in the presence of complement, complement
components, lymphocyte products, bile acids, and androgenic steroid metabolites.

26. Drugs that are known to cause fever include


A. chlorpheniramine
B. allopurinol
C. diphenhydramine
D. acetazolamide
E. adenosine
26.(B). Drugs that are known to cause fever include vancomycin, amphotericin B, and
allopurinol.

27. Inges on of dirt is an important clue to infec on with


A. amebiasis
B. giardiasis
C. malaria
D. trichinosis
E. toxoplasmosis
27.(E). Ingestion of dirt is a particularly important clue to infection with Toxocara canis
(visceral larva migrans) or Toxoplasma gondii (toxoplasmosis).

28. Bulbar conjunc vi s in a child with FUO suggests


A. leptospirosis
B. coxsackievirus infection
C. tuberculosis
D. infectious mononucleosis
E. lymphogranuloma venereum
28.(A). Palpebral conjunctivitis in a febrile patient may be a clue to measles,
coxsackievirus infection, tuberculosis, infectious mononucleosis,
lymphogranulomavenereum, or cat-scratch disease. In contrast, bulbar conjunctivitis in
a child with FUO suggests Kawasaki disease or leptospirosis.
29. Mul ple blood cultures may be required to detect bacteremia associated with
A. malaria
B. pyelonephritis
C. brucellosis
D. osteomyelitis
E. pneumonia
29.(D). Multiple or repeated blood cultures may be required to detect bacteremia
associated with infective endocarditis, osteomyelitis, or deep-seated abscesses.

30. Primary immunodeficiencies are compromised states that result from genetic
defects affec ng 1 or more arms of the immune system while secondary
immunodeficiencies result from infection, malignancy, or as an adverse effect of
immunomodulating or immunosuppressing medications.
Which of the following represent secondary immune deficiency?
A. Shwachman-Diamond syndrome
B. cystic fibrosis
C. Chédiak-Higashi syndrome
D. Omenn syndrome
E. ataxia-telangiectasia
30.(B). All other distractors represent primary immune deficiency.

31. Which type of fever is persistent and varies by more than 0.5°c (0.9°f)/day?
A. intermittent
B. hectic
C. sustained
D. remittent
E. relapsing
31.(D). Intermittent fever is an exaggerated circadian rhythm that includes a period of
normal temperatures on most days; extremely wide fluctuations may be termed septic
or hec c fever. Sustained fever is persistent and does not vary by more than 0.5°C
(0.9°F)/day. Remi ent fever is persistent and varies by more than 0.5°C (0.9°F)/day.
Relapsing fever is characterized by febrile periods that are separated by intervals of
normal temperature.
32. All the following are causes of very high temperatures (>41°C) EXCEPT
A. malignant hyperthermia
B. malignant neuroleptic syndrome
C. infection
D. drug fever
E. heat stroke
32.(C). Temperatures in excess of 41°C (105.8°F) are most oGen associated with a
noninfectious cause.
33. Rela ve bradycardia (when the pulse rate remains low in the presence of fever) can
accompany all the following conditions EXCEPT
A. typhoid fever
B. brucellosis
C. leptospirosis
D. visceral leishmaniasis
E. drug fever

33.(D).
34. The MOST common serious bacterial infec on in infant aged 1-3 mo is
A. pyelonephritis
B. meningitis
C. pneumonia
D. septic arthritis
E. osteomylitis
34.(A). Pyelonephritis is the most common serious bacterial infection in this age group
and is also more common in uncircumcised infant boys and infants with urinary tract
anomalies. E. coliis the most common pathogen identified in bacteremic infants, the
majority having pyelonephritis.

35. Neutropenia is defined as an absolute neutrophil count of less than


A. 1,000 cells/mm3
B. 1,500 cells/mm3
C. 2,000 cells/mm3
D. 2,500 cells/mm3
E. 3,000 cells/mm3
35.(A). Neutropenia is defined as an absolute neutrophil count of <1,000 cells/mm3
and can be associated with significant risk for developing severe bacterial and fungal disease.
36. Leukocyte adhesion defects are caused by defects in the β chain of integrin (CD18),
which is required for the normal process of neutrophil aggregation and
attachment to
endothelial surfaces.It is characterized by all the following EXCEPT
A. delayed cord separation
B. recurrent infections
C. ecthyma gangrenosum
D. neutropenia
E. survival is usually <10 yr

36.(D). Because the defect involves leukocyte migration and adherence, the neutrophil
count in the peripheral blood is usually extremely elevated but pus is not found at the
site of infection
37. Penicillins are the drugs of choice for pediatric infections caused by the following
EXCEPT
A. group A Streptococcus
B. H .influenzae
C. Treponema pallidum
D. L. monocytogenes
E. N. meningitides

37.(B). Although there has been ever-increasing emergence of resistance to penicillins


but they remain the drugs of choice for pediatric infections caused by group A and
group B Streptococcus, Treponema pallidum (syphilis), L. monocytogenes, and N.
meningitidis.
38. Cefixime is a third-generation cephalosporin active against all the following
EXCEPT
A. Streptococci
B. Staphylococci
C. H. influenza
D. Neisseria gonorrhoeae
E. Proteus vulgaris
38.(B). Active against streptococci, H. influenzae, M. catarrhalis, Neisseria gonorrhoeae,
Serratia marcescens, and Proteus vulgaris. No antistaphylococcal or antipseudomonal
ac vity. Dose: 8 mg/kg/24 hr divided q 12-24 hr PO.
39. Cephalexin is a cephalosporin active against S. aureus, Streptococcus, E. coli, Klebsiella, and
Proteus. To which generation of cephalosporins it belongs?
A. 1st
B. 2nd
C. 3rd
D. 4th
E. 5th
39.(A). Dose: 25-100 mg/kg/24 hr divided q 6-8 hr PO.

40. Meropenem is a carbapenem an bio c with broad-spectrum activity but has no


activity against
A. P. aeruginosa
B. S. maltophilia
C. L. monocytogenes
D. S. aureus
E. N. meningitides
40.(B). Carbapenem antibiotic with broad-spectrum activity against Gram-positive cocci
and Gram-negative bacilli, including P. aeruginosa and anaerobes. No activity against S.
maltophilia.
41. Metronidazole is highly effective in the treatment of infections caused by anaerobes. It can increase
the level of which of the following drugs if given at the same
time
A. carbamazepine
B. rifampin
C. phenobarbital
D. phenytoin
E. vaiproic acid

41.(D). It may increase levels of warfarin, phenytoin, and lithium.

42. Cephalosporins are widely used in pediatric prac ce, both in oral and
parenteral
formula ons. Which of the following is a 2nd genera on cephalosporins?
A. cefazolin
B. cephalexin
C. cefuroxime
D. ceftazidime
E. ceftaroline
42.(C). The first-generation cephalosporins (e.g., cefazolin, a parenteral formulation,
and cephalexin, an oral equivalent) are commonly used for management of skin and
soft-tissue infections caused by susceptible strains of S. aureus and group A
Streptococcus. The second-generation cephalosporins (e.g., cefuroxime, cefoxitin) have
better activity against Gram-negative bacterial infections than do first-generation
cephalosporins and are used to treat respiratory tract infections, urinary tract
infections, and skin and soft-tissue infections. A variety of orally administered secondgeneration
agents (cefaclor, cefprozil, loracarbef, cefpodoxime) are commonly used in
the outpatient management of sinopulmonary infections and otitis media. The thirdgeneration
cephalosporins (cefotaxime, ceftriaxone, and ceftazidime) are typically used
for serious pediatric infections, including meningitis and sepsis. A fourth generation
cephalosporin, called cefepime, has activity against P. aeruginosa and retains good
activity against methicillin-susceptible staphylococcal infections. A fifth generation
cephalosporin, ceftaroline has been licensed.

43. Which cephalosporin should not be mixed or recons tuted with a calciumcontaining
product, such as Ringer solution or parenteral nutrition containing calcium?
A. cefazolin
B. cefotaxime
C. ceftriaxone
D. ceftazidime
E. cefepime
43.(C). Ceftriaxone should not be mixed or reconstituted with a calcium-containing
product, because particulate formation can result. Cases of fatal reactions with
ceftriaxone–calcium precipitates in lungs and kidneys in neonates have been reported.
44. Toxic shock syndrome is an acute and poten ally severe illness characterized by all
the following EXCEPT
A. desquamation on the hands and feet
B. myalgias
C. focal neurologic abnormalities
D. conjunctival hyperemia
E. strawberry tongue

44.(C). Toxic shock syndrome (TSS) is an acute and potentially severe illness
characterized by fever, hypotension, erythematous rash with subsequent
desquamation on the hands and feet, and multisystem involvement, including
vomiting, diarrhea, myalgias, nonfocal neurologic abnormalities, conjunctival
hyperemia, and strawberry tongue.

45. Which of the following represent a major criterion for the diagnosis of staphylococcal
toxic shock syndrome?
A. rash
B. conjunctival hyperemia
C. myalgia
D. thrombocytopenia
E. vomiting
45.(A).
MAJOR CRITERIA (ALL REQUIRED)
1. Acute fever; temperature >38.8°C (101.8°F)
2. Hypotension
3. Rash (erythroderma with convalescent desquamation)
MINOR CRITERIA (ANY 3 OR MORE)
1. Mucous membrane inflammation (vaginal, oropharyngeal or conjunctival
hyperemia, strawberry tongue)
2. Vomiting, diarrhea
3. Liver abnormalities (bilirubin or transaminase greater than twice upper limit of
normal)
4. Renal abnormalities (urea nitrogen or creatinine greater than twice upper limit
of normal, or greater than 5 white blood cells per high-power field)
5. Muscle abnormalities (myalgia or creatinine phosphokinase greater than twice
upper limit of normal)
6. Central nervous system abnormalities (alteration in consciousness without
focal neurologic signs)
7. Thrombocytopenia (100,000/mm3 or less)
46. Kawasaki disease closely resembles toxic shock syndrome clinically. However, many
of the clinical features of toxic shock syndrome are usually absent or rare in
Kawasaki
disease like
A. fever unresponsive to antibiotics
B. hyperemia of mucous membranes
C. erythematous rash
D. desquamation
E. diffuse myalgia

46.(E). Many of the clinical features of TSS are usually absent or rare in Kawasaki
disease, including diffuse myalgia, vomiting, abdominal pain, diarrhea, azotemia,
hypotension, acute respiratory distress syndrome, and shock.

47. S. pneumoniae is the most frequent cause of bacteremia, bacterial pneumonia,


otitis media, and bacterial meningitis in children. Children at increased risk of
pneumococcal infections include those with the following conditions EXCEPT
A. megaloplastic anemia
B. deficiencies in humoral immunity
C. HIV infection
D. cerebrospinal fluid leak
E. cochlear implants
47.(A). Children at increased risk of pneumococcal infections include those with sickle
cell disease, asplenia, deficiencies in humoral (B cell) and complement-mediated
immunity, HIV infection, certain malignancies (e.g., leukemia, lymphoma), chronic
heart, lung, or renal disease (particularly nephrotic syndrome), cerebrospinal fluid leak,
and cochlear implants.

48. Group A streptococcus can be subdivided into more than 220 serotypes on the
basis of the M protein antigen, M serotyping is valuable for epidemiologic studies;
specific group A streptococcus diseases tend to be associated with certain M
types.
Of the following, the M type associated with glomerulonephritis is
A. 1
B. 6
C. 12
D. 18
E. 29
48.(C). A pharyngeal strains (e.g., M type 12) are associated with glomerulonephri s;
skin strains (e.g., M types 49, 55, 57, and 60) are considered nephritogenic. Several
pharyngeal serotypes (e.g., M types 1, 3, 5, 6, 18, 29), but no skin strains, are
associated with acute rheumatic fever.
49. Scarlet fever is an upper respiratory tract infection associated with a characteristic
rash.Of the following, which statement is TRUE?
A. it is caused by an infection with pyrogenic endotoxin producing group A
streptococcus
B. the rash appears 96 hr aGer onset of symptoms
C. the rash begins to fade aGer 1-2 weeks
D. before desquamation, the reddened papillae are prominent, giving the tongue
a strawberry appearance
E. the milder form can be confused with Kawasaki disease

49.(E). It is caused by an infection with pyrogenic exotoxin (erythrogenic toxin)–


producing GAS. The rash appears within 24-48 hr aGer onset of symptoms, although it
may appear with the first signs of illness. AGer 3-4 days, the rash begins to fade and is
followed by desquamation. After desquamation, the reddened papillae are prominent,
giving the tongue a strawberry appearance. The milder form can be confused with viral
exanthems, Kawasaki disease, and drug eruptions.
50. Impe go (or pyoderma) has traditionally been classified into 2 clinical forms: bullous and
nonbullous.Of the following, which statements are TRUE??
A. nonbullous impetigo is more common.
B. nonbullous lesions are most common on the trunk and perineum
C. nonbullous impetigo is generally not accompanied by fever.
D. regional lymphadenitis is commonly associated with nonbullous lesions.
E. bullous impetigo usually involve the face and extremeties
50.(A.C.D). Nonbullous impetigo is the more common form. The lesions may occur
anywhere but are most common on the face and extremities. Regional lymphadenitis is
common. Nonbullous impetigo is generally not accompanied by fever or other systemic
signs or symptoms. Bullous impetigo is less common and occurs most often in neonates
and young infants. The usual distribution involves the face, buttocks, trunk, and
perineum.51. In which of the following circumstances the diagnosis of acute rheuma c
fever can
be made without strict adherence to Jones criteria?
A. when chorea occurs as the only major manifestation of acute rheumatic fever
B. when indolent carditis is the only manifestation months after the apparent
onset of acute rheumatic fever
C. in a limited number of patients with recurrences of acute rheumatic fever in
particularly high-risk populations
D. all of the above
E. none of the above
52. Because no clinical or laboratory finding is pathognomonic for acute rheumatic
fever, 5 major and 4 minor criteria with evidence of recent group A streptococcus
infection are required for the diagnosis. Migratory polyarthritis represent one of the
major criteria, which of the following statements is TRUE regarding this major
criterion?
A. occurs in approximately 50% of pa ents
251
B. typically involves small joints
C. the pain can precede and can appear to be disproportionate to the objective
findings
D. rheumatic arthritis is almost deforming
E. there is often a proportional relationship between the severity of arthritis and
the severity of cardiac involvement
53. Acute rheumatic carditis usually presents as cardiac murmurs and
A. tachycardia
B. chest pain
C. dyspnea
D. syncopal attack
E. tachypnea
54. Patients with acute rheumatic carditis and more than minimal cardiomegaly
should
receive prednisone 2 mg/kg/day in 4 divided doses for
A. 7-10 days
B. 2-3 weeks
C. 6-8 weeks
D. 2-3 months
E. 4-6 months
55. The drug of choice for Sydenham chorea is
A. aspirin
B. prednisone
C. phenobarbital
D. haloperidol
E. chlorpromazine
56. To prevent first attack of acute rheumatic fever after acute group A
streptococcus
pharyngitis, appropriate antibiotic therapy should be instituted before
A. 3rd day of illness
B. 5th day of illness
C. 7th day of illness
D. 9th day of illness
E. 11th day of illness
57. Prophylaxis for people who have had acute rheumatic fever with carditis but
without residual heart disease persist for
A. 5 yr or un l 21 yr of age, whichever is longer
B. 10 yr or un l 21 yr of age, whichever is longer
C. 5 yr or un l 40 yr of age, whichever is longer
252
D. 10 yr or un l 40 yr of age, whichever is longer
E. Lifelong
58. Group B streptococcus (GBS), or Streptococcus agalactiae, is a major cause of
neonatal bacterial sepsis.
Of the following, the MOST common syndrome associated with childhood GBS
disease
beyond early infancy is
A. bacteremia without a focus
B. meningitis
C. ventriculitis
D. septic arthritis
E. pneumonia
59. Recommended duration of therapy for manifestations of group B streptococcus
are
as follow
A. bacteremia without a focus: 7 days
B. meningi s: 3-6 weeks
C. ventriculi s: at least 8 weeks
D. sep c arthri s: 3-4 weeks
E. osteomyeli s: 6-8 weeks
60. Diphtheria is an acute toxic infec on; toxic cardiomyopathy occurs in 10-25% of
pa ents with respiratory diphtheria and is responsible for 50-60% of deaths.
Of the following, the mainstay of therapy is
A. antitoxin
B. penicillins
C. erythromycin
D. clindamycin
E. rifampin
61. A painless, slow-growing, hard mass producing cutaneous fistulas, a condition
commonly known as lumpy jaw is usually caused by
A. Staphylococcus
B. Actinomyces
C. Nocardia
D. Yersinia
E. Leptospira
62. The MOST common secondary site involved in cases of pulmonary nocardiosis
is
A. brain
B. skin
C. kidney
253
D. liver
E. bone
63. Which of the following is a gram negative bacterium?
A. Staphylococcus aureus
B. Streptococcus pneumoniae
C. Actinomyces
D. Haemophilus influenza
E. Nocardia
64. Regarding epidemiology of Neisseria meningitides, all the following are true
EXCEPT
A. meningococci are transmitted during close contact via aerosol droplets or
exposure to respiratory secretions
B. meningococci survive for long periods in the environment
C. smoking and respiratory viral infection are associated with increased rates of
carriage and disease
D. the highest rate of meningococcal disease occurs in infancy
E. most cases of meningococcal disease are sporadic
65. The MOST common clinical manifestation of meningococcal infection is
A. asymptomatic carriage
B. meningococcal meningitis
C. bacteremia without sepsis
D. meningococcal septicemia
E. pneumonia
66. In meningococcal purpura, necrotic skin lesions are less common among children
treated with
A. penicillin G
B. ampicillin
C. cefotaxime
D. ceftriaxone
E. meropenem
67. The MOST common complication of acute severe meningococcal septicemia is
A. arthritis
B. focal skin infarction
C. endocarditis
D. pneumonia
E. peritonitis
68. A poor prognostic factor for invasive meningococcal disease on presentation is
254
A. hypertension
B. leukocytosis
C. alkalosis
D. meningitis
E. normal erythrocyte sedimentation rate
69. Of the following, the MOST effective agent for prophylaxis of meningococcal
disease is
A. ceftriaxone
B. rifampin
C. ampicillin
D. penicillin
E. amoxicillin
70. Gonorrhea is manifested by a spectrum of clinical presentations from
asymptomatic
carriage, to the characteristic localized urogenital infections, to disseminated
systemic
infection.
Regarding disseminated gonococcal infection, the following statement is TRUE
A. hematogenous dissemina on occurs in 10-30% of all gonococcal infections
B. men account for the majority of cases
C. meningitis and osteomyelitis are the most common manifestations
D. skin lesions found in75% of patients
E. acute endocarditis is an uncommon but often fatal manifestation
71. All patients who are presumed or proven to have gonorrhea should be evaluated
for concurrent presence of all the following infections EXCEPT
A. syphilis
B. hepatitis B
C. HIV
D. C. trachomatis
E. HSV2
72. Children who have bacteremia or arthritis caused by gonococcal infections
should
be treated with ceGriaxone (50 mg/kg/day; maximum: 1 g/day if weighs <45 kg) for
a
minimum of
A. 3 days
B. 5 days
C. 7 days
D. 14 days
E. 21 days
73. The MOST common etiology of joint and bone infections in young children is
255
A. Staphylococcus aureus
B. Streptococcus pneumoniae
C. Kingellakingae
D. Haemophilus influenza
E. Neisseria meningitidis
74. Diagnosis of chancroid in infants and children is a strong evidence of sexual
abuse.
Chancroid is caused by
A. Haemophilus ducreyi
B. Syphilis
C. C. trachomatis
D. Kingellakingae
E. Neisseria gonorrhea
75. The MOST important clinical manifestation of M.catarrhalis infection in children
is
A. pneumonia
B. bronchitis
C. otitis media
D. pharyngitis
E. cystitis
76. The MOST common reason for which children receive antibiotics is
A. pneumonia
B. bronchitis
C. otitis media
D. pharyngitis
E. cystitis
77. Infants younger than 3 mo of age with suspected pertussis usually are admi ed
to
hospital, as are many between 3 and 6 mo of age unless witnessed paroxysms are
not
severe, as well as are patients of any age if significant complications occur.
Typical paroxysms that are not life threatening have the following feature
A. dura on >45 sec
B. blue color change
C. bradycardia <60 beats/min in infants
D. oxygen desaturation that spontaneously resolves at the end of the paroxysm
E. post-tussive unresponsiveness
78. All the following regarding assessment and care of infants with pertussis are true
EXCEPT
A. infants with potentially fatal pertussis may appear well between episodes
256
B. a paroxysm must be witnessed before a decision is made between hospital and
home care
C. suctioning of nose, oropharynx, or trachea should be performed on a
preventive schedule
D. feeding in the period following a paroxysm may be more successful than after
napping
E. family education, recruitment as part of the team, and continued support after
discharge are essential
79. Hospital discharge of infants with pertussis is appropriate in all the following
circumstances EXCEPT
A. over a 24-hr period disease severity is unchanged or diminished
B. intervention is not required during paroxysms
C. nutrition is adequate
D. no complication has occurred
E. parents are adequately prepared for care at home
80. The preferable antibiotic in neonate with pertussis is
A. azithromycin
B. erythromycin
C. clarithromycin
D. trimethoprim-sulfamethoxazole
E. amoxycilline
81. Echocardiography should be performed in critically ill infants with pertussis to
detect presence of
A. myocarditis
B. heart failure
C. congenital heart defect
D. pulmonary hypertension
E. right atrial dilatation
82. The least common clinical feature of typhoid fever in children is
A. diarrhea
B. abdominal pain
C. pallor
D. splenomegaly
E. headache
83. All the following are clinical features of typhoid fever EXCEPT
A. incuba on period is usually 30-45 days
B. macular or maculopapular rash (rose spots) may be visible around the 7th-10th
257
day of the illness
C. if complications not occur, the symptoms and physical findings gradually resolve
within 2-4 wk
D. typhoid fever usually manifests as high-grade fever with a wide variety of
associated features
E. relative bradycardia, neurologic manifestations, and gastrointestinal bleeding,
are rare in children
84. Regarding diagnosis of typhoid fever, one of the following is TRUE
A. blood cultures are posi ve in 65-80% of the patients
B. urine culture results become posi ve within the 1st wk
C. thrombocytosis may be a marker of severe illness
D. diagnosis by Widal test alone is prone to error
E. leukocytosis is rare in young children
85. All the following are clinical features of shigellosis EXCEPT
A. an incuba on period of 12 hr to several days
B. most children never progress to the stage of bloody diarrhea
C. untreated diarrhea can last more than 4 weeks
D. neurologic findings are the most common extraintestinal manifestations
E. neonatal shigellosis is rare
86. Cholera is a dehydrating diarrheal disease caused by Vibrio cholerae, of more
than
200 serogroups; the serogroups that have been associated with epidemics are
A. O131
B. O139
C. O151
D. O159
E. O165
87. Persons with which blood group are at increased risk for developing severe
disease
of cholera?
A. A
B. B
C. AB
D. O
E. Blood group not related to the severity of the disease
88. Seizure in cholera is commonly due to
A. hypocalcemia
B. hyponatremia
C. hypernatremia
258
D. hypoglycemia
E. fever
89. Complications of campylobacter jejuni can include acute and late onset
complications that may present after the acute infection has resolved.
Of the following, the MOST common late-onset complication is
A. reactive arthritis
B. immunoglobulin A nephropathy
C. immune complex glomerulonephritis
D. hemolytic anemia
E. carditis
90. MOST Campylobacter isolates are susceptible to
A. aminoglycosides
B. cephalosporins
C. rifampin
D. penicillins
E. trimethoprim
91. The genus Yersinia is a member of the family Enterobacteriaceae and comprises
more than 14 named species, 3 of which are established as human pathogens.
Which of the following is MOST often associated with mesenteric lymphadenitis?
A. Yersinia enterocolitica
B. Yersinia pseudotuberculosis
C. Yersinia pestis
D. Yersinia mollaretii
E. Yersinia rohdei
92. Patients with conditions leading to iron overload are at higher risk of developing
infections with
A. Aeromonas
B. Pseudomonas aeruginosa
C. Yersinia
D. Francisella tularensis
E. Campylobacter
93. The MOST common complication of Y. enterocolitica infection in younger
children
is
A. reactive arthritis
B. erythema multiforme
C. hemolytic anemia
D. thrombocytopenia
259
E. septicemia
94. Tularemia is a zoonotic infection caused by the gram-negative bacterium
Francisella
tularensis.
Of the following, the MOST common forms of tularemia diagnosed in children is
A. ulceroglandular
B. pneumonia
C. oropharyngeal
D. oculoglandular
E. typhoidal
95. Human brucellosis is caused by organisms of the genus Brucella and continues to
be
a major public health problem worldwide.
All the following are true EXCEPT
A. symptoms can be acute or insidious
B. most patients present with fever, arthralgia/arthritis, and hepatosplenomegaly
C. some present as a fever of unknown origin
D. variable fever pattern
E. invasion of the nervous system occurs in approximately 20% of cases
96. Which antibiotics should be avoided in patients with botulism?
A. penicillin
B. cephalosporin
C. macrolide
D. aminoglycoside
E. trimethoprim-sulfamethoxazole
97. Tetanus is an acute spastic paralytic illness historically called lock-jaw that is
caused
by the neurotoxin produced by Clostridium tetani.
Of the following, the TRUE statement is
A. tetanus is most often localized
B. incuba on period typically is 2-6 weeks
C. patient remains conscious and there is no pain
D. tetanic paralysis becomes more severe in the 4th wk aGer onset
E. cephalic tetanus occurs in association with chronic otitis media
98. Worsening of acne in adolescent female on anti-tuberculous treatment is caused
by
A. isoniazid
B. rifampin
C. pyrazinamide
D. ethambutol
260
E. streptomycin
99. Isoniazid is accompanied by significant drug–drug interactions, which of the
following is TRUE?
A. aluminum salts increase absorption of isoniazid
B. isoniazid increase toxicity of carbamazepine
C. rifampin decreased hepatotoxicity of isoniazid
D. isoniazid decrease level of warfarin
E. prednisolone decreased isoniazid metabolism
100. The ideal agent for treating fungal urinary tract infections is
A. amphotericin B
B. fluconazole
C. voriconazole
D. micafungin
E. caspofungin
101. Candida is a common cause of oral mucous membrane infections (thrush) and
perineal skin infections (Candida diaper dermatitis) in young infants.
All the following are true EXCEPT
A. candida species are the third most common cause of bloodstream infection in
premature infants
B. up to 10% of full-term infants are colonized as the result of vertical
transmission from the mother at birth
C. histamine-2 blockers facilitate Candida colonization and overgrowth
D. significant risk factors for neonatal invasive candidiasis include the presence of
a central venous catheter
E. the cumula ve incidence is <0.3% among infants <750 g birthweight admi ed
to the NICU
102. NICUs with a high incidence of invasive candidiasis should consider prophylaxis
with fluconazole in infants with a birthweight of
A. <750 g
B. <1,000 g
C. <1,500 g
D. <2,000 g
E. <2,500 g
103. Antiviral chemotherapy typically requires a delicate balance between targeting
critical steps in viral replication without interfering with host cellular function.
Which of the following an viral blocks M2 protein ion channel?
A. acyclovir
261
B. amantadine
C. ganciclovir
D. foscarnet
E. vidarabine
104. In neonates with HSV infection including CNS involvement, to improve
neurodevelopmental outcome, suppressive therapy with oral acyclovir should be
used
for
A. 3 mo
B. 6 mo
C. 9 mo
D. 12 mo
E. 18 mo
105. Acyclovir is a safe and effective therapy for herpes simplex virus (HSV)
infections.
All the following are true EXCEPT
A. activity against CMV is less pronounced
B. activity against Epstein-barr virus is modest, both in vitro and clinically
C. acyclovir therapy in a nursing mother is not a contraindication to breastfeeding
D. main route of elimination is hepatic
E. high doses of acyclovir are associated with neurotoxicity
106. Ribavirin is a guanosine analog that has broad-spectrum activity against a
variety
of viruses, particularly RNA viruses.
All the following are true EXCEPT
A. its precise mechanism of action is incompletely understood
B. aerosolized ribavirin effective for parainfluenza, influenza, and measles
infections
C. ribavirin is generally nontoxic, particularly when administered by aerosol
D. ribavirin and its metabolites concentrate in hepatocytes
E. conjunctivitis and bronchospasm have been reported following exposure to
aerosolized drug
107. The portal of entry of measles virus is through the respiratory tract or
conjunctivae following contact with large droplets or small-droplet aerosols in which
the virus is suspended.
Patients with measles are infectious
A. 1 day before to up to 4-6 days aGer the onset of rash
B. 2 days before to up to 4-6 days aGer the onset of rash
C. 3 days before to up to 4-6 days aGer the onset of rash
D. 4 days before to up to 4-6 days aGer the onset of rash
E. 5 days before to up to 4-6 days aGer the onset of rash
108. All the following are true about Measles EXCEPT
A. incuba on period is 8-12 days
B. Koplik spots have been reported in 50-70% of measles cases
C. the rash begins on the forehead, behind the ears, and on the upper neck
D. the rash fades over about 3 days in the same progression as it evolved
E. in more severe cases, generalized lymphadenopathy may be present
109. The MOST common complication of measles is
A. aseptic meningitis
B. pharyngitis
C. otitis media
D. pneumonia
E. cystitis
110. Subacute sclerosing panencephalitis (SSPE) is a chronic complication of measles
with a delayed onset and an outcome that is nearly always fatal.
SSPE is characterized by
A. females are affected twice as often as males
B. clinical manifesta ons begin insidiously 3-6 yr aGer primary measles infection
C. the hallmark of the 1st stage is massive myoclonus
D. the 3rd stage is characterized by loss of cri cal centers
E. clinical trials using isoprinosine suggest significant benefit
111. The MOST common finding among infants with congenital rubella syndrome is
A. psychomotor retardation
B. cataracts
C. deafness
D. patent ductus arteriosus
E. neonatal purpura
112. Mumps virus is in the family Paramyxoviridae and the genus Rubulavirus.
Mumps is characterized by
A. incuba on period ranges from 7-11 days
B. unilateral parotitis rarely becomes bilateral
C. pale opening of the Stensen duct
D. paro d swelling peaks in approximately 7 days
E. a morbilliform rash is rarely seen
113. The MOST common complication of mumps is
A. meningitis
263
B. conjunctivitis
C. optic neuritis
D. pneumonia
E. thrombocytopenia
114. Intrauterine mumps infection have been associated with
A. glaucoma
B. limb anomalies
C. pulmonary stenosis
D. renal agenesis
E. no fetal malformations
115. Mumps virus is neurotropic and is thought to enter the CNS via the choroid
plexus
and infect the choroidal epithelium and ependymal cells. CNS involvement is
characterized by
A. symptoma c CNS involvement occurs in 40-60% of infected individuals
B. encephali s most commonly manifests 10 days aGer the parotitis
C. CNS symptoms usually resolve in 3 days
D. CSF protein content is usually elevated
E. facial palsy is a less-common CNS complication
116. Hematogenous dissemination of HSV to the central nervous system occur in
A. neonates
B. individuals with eczema
C. severely malnourished children
D. infants on steroids
E. all the above
117. Herpes whitlow is HSV infection of the
A. eye
B. mouth
C. fingers
D. ear
E. lip
118. The MOST common cause of recurrent aseptic meningitis is
A. mumps
B. EBV
C. adenoviruses
D. HSV
E. influenza virus
264
119. In neonatal HSV encephalitis, skin vesicles occur in approximately
A. 20%
B. 40%
C. 60%
D. 80%
E. 100%
120. In pa ents with neonatal HSV infec on who receive IV treatment for 2-3 weeks
then suppressive treatment for 6 months, they should be monitored by
A. absolute neutrophil count ANC
B. liver function tests
C. renal function tests
D. platelet count
E. Hb%
121. Varicella-zoster virus (VZV) causes primary, latent, and recurrent infections.
All the following statements are true EXCEPT
A. varicella is a serious disease in young infants
B. within households, transmission of VZV occurs at a rate of 65-86%
C. Herpes zoster is more common in winter
D. the life me risk for herpes zoster for individuals with a history of varicella is 20-
30%
E. Herpes zoster is very rare in healthy children younger than 10 yr of age
122. Varicella is an acute febrile rash illness that is common in Iraq.
Of the following, the TRUE statement is
A. it usually begins 4-6 days aGer exposure
B. subclinical varicella is common
C. mild abdominal pain may occur 24-48 hr before the rash appears
D. temperature elevation usually as high as 41.1°c
E. lesions often appear first on the extremities
123. All the following are true about varicella in unvaccinated individuals EXCEPT
A. simultaneous presence of lesions in the same stages of evolution
B. distribution of the rash is predominantly central
C. many children have vesicular lesions on the eyelids
D. exanthem may be much more extensive in children with skin disorders
E. hypopigmentation or hyperpigmentation of lesion sites persists for days to
weeks in some children
124. Encephalitis and acute cerebellar ataxia are well-described neurologic
complications of varicella.
265
All the following are true EXCEPT
A. morbidity from central nervous system complications is highest among patients
younger than 5 yr and older than 20 yr
B. nuchal rigidity, altered consciousness, and seizures characterize
meningoencephalitis
C. patients with cerebellar ataxia have a gradual onset of gait disturbance,
nystagmus, and slurred speech
D. neurologic symptoms usually begin 2-6 days aGer the onset of the rash but may
occur during the incubation period or after resolution of the rash
E. clinical recovery is typically occurring aGer 96 hr, and is usually gradual
125. Oral therapy with acyclovir (20 mg/kg/dose; maximum: 800 mg/dose) given as
4
doses/day for 5 days can be used to treat uncomplicated varicella in individuals at
increased risk for moderate to severe varicella.
It includes all the following individuals EXCEPT
A. nonpregnant individuals older than 12 yr of age
B. individuals older than 12 mo of age with chronic cutaneous disorders
C. individuals receiving short-term corticosteroid therapy
D. individuals receiving long-term salicylate therapy
E. individuals with chronic hematological disorders
126. Epstein-Barr virus (EBV) is shed in oral secretions after acute infection for
A. 2 wk
B. 4 wk
C. 2 mo
D. 4 mo
E. 6 mo
127. What percent of the world’s population infected by Epstein-Barr virus (EBV)?
A. 5%
B. 25%
C. 50%
D. 75%
E. 95%
128. Infectious mononucleosis is the best-known clinical syndrome caused by
Epstein-
Barr virus (EBV). It is characterized by
A. symptomatic hepatitis
B. elevated liver enzymes
C. massive splenic enlargement
D. huge hepatomegaly
E. occasional palatal petechiae
266
129. The MOST common long-term sequela associated with congenital CMV
infection is
A. renal failure
B. hearing loss
C. heart failure
D. vision loss
E. hepatic failure
130. Human herpesvirus 6 (HHV-6A and HHV-6B) and human herpesvirus 7 (HHV-7)
cause infection in infancy and early childhood. HHV-6B is responsible for the
majority of
cases of roseola infantum (exanthema subitum or sixth disease).
All the following are true EXCEPT
A. 95% of children being infected with HHV-6 by 2 yr of age
B. peak age of primary HHV-6B infec on is 6-9 mo of life
C. congenital infection with HHV-6 occurs in 1% of newborns
D. congenital infection with HHV-7 is well recognized
E. breast milk does not play a role in transmission of either HHV-6 or HHV-7
131. Amantadine and rimantadine are effective only against influenza A viruses and
are
not approved for use in children younger than
A. 6 mo
B. 18 mo
C. 3 yr
D. 5 yr
E. 10 yr
132. Typically, the first sign of infection in infants with respiratory syncytial virus
(RSV)
is
A. rhinorrhea
B. cough
C. low-grade fever
D. increased respiratory rate
E. subcostal retractions
133. For military services, vaccines are available for human adenoviruses types
A. 1 and 4
B. 2 and 5
C. 3 and 6
D. 4 and 7
E. 5 and 8
134. Respiratory tract infections are common manifestations of human
adenoviruses
(HAdV) infections in children and adults.
267
All the following are true EXCEPT
A. HAdVs cause 5-10% of all childhood respiratory disease
B. primary infections in infants may manifest as bronchiolitis or pneumonia
C. HAdV pneumonia may manifest as features more typical of bacterial disease
D. pharyngitis typically includes symptoms of coryza, sore throat, and fever
E. HAdV can be identified in <5% of children with isolated pharyngitis
135. Rotaviruses are in the Reoviridae family and cause disease in virtually all
mammals
and birds.
All the following statements are true EXCEPT
A. infec on typically begins aGer an incuba on period of <48 hr (range: 1-7 days)
B. fever, vomi ng and frequent watery stools are present in about 50-60% of
cases
C. vomi ng and fever typically abate aGer the 4th day of illness
D. dehydration may develop and progress rapidly particularly in infants
E. most severe disease typically occurs among children 4-36 mo of age
136. Common adverse effects of nitazoxinide include
A. diarrhea
B. flatulence
C. increased appetite
D. fever
E. pruritus
137. One of the following statements regarding tinidazole is TRUE
A. it is FDA approved for treatment of trichomoniasis
B. it can be in infancy
C. it should be given in 2 divided doses
D. it is excreted via urine only
E. it carries a pregnancy category A
138. All the following can cause human amoebic meningoencephalitis EXCEPT
A. Naegleria
B. Acanthamoeba
C. Balamuthia
D. Entamoeba histolytica
E. Sappinia

139. One of the following statements regarding Giardia lamblia is TRUE


A. life cycle is composed of 3 stages
B. each ingested cyst produces 2 trophozoites in the caecum
C. trophozoites contain 4 oval nuclei anteriorly
D. cyst viability is not affected by the usual concentrations of chlorine used to
purify water for drinking
E. Giardia genotypes difference lead to wide spectrum of clinical manifestations
139.(D). The life cycle of G. lamblia (also known as Giardia intestinalis or Giardia
duodenalis) is composed of 2 stages: trophozoites and cysts. Each ingested cyst
produces 2 trophozoites in the duodenum. Giardia trophozoites contain 2 oval
nuclei anteriorly, a large ventral disk, a curved median body posteriorly, and 4
pairs of flagella.Studies suggest that different Giardia genotypes may cause unique
clinical manifestations, but these findings appear to vary according to the
geographic region tested.
140. All the following statements regarding giardiasis are true EXCEPT
A. incuba on period of Giardia infec on usually is 1-2 wk
B. most infections are asymptomatic
C. symptomatic infections occur more frequently in adults than in children
D. stools do not contain blood, mucus, or fecal leukocytes
E. malabsorption of sugars, fats, and fat-soluble vitamins is well documented
140.(C).

141. Giardiasis should be considered in children who have the following


presentations
EXCEPT
A. acute dysenteric diarrhea
B. persistent diarrhea
C. failure to thrive
D. malabsorption
E. chronic crampy abdominal pain
141.(A). Giardiasis should be considered in children who have acute nondysenteric
diarrhea, persistent diarrhea, intermittent diarrhea and constipation, malabsorption,
chronic crampy abdominal pain and bloating, failure to thrive, or weight loss.

142. Stool analysis of 5years old child contains giardia cyst, your treatment is
A. reassurance, no treatment
B. metronidazole15 mg/kg/day in 3 divided doses for 5-7 days
C. nitazoxinide 200 mg bid for 3 days
D. inidazole 50 mg\kg once
E. albendazole 400 mg once a day for 5 days
142.( B). Asymptomatic excreters generally are not treated except in specific instances
such as outbreak control, prevention of household transmission by toddlers to
pregnant women and patients with hypogammaglobulinemia or cystic fibrosis, and
situations requiring oral antibiotic treatment where Giardia may produce
malabsorp on of the an bio c.Tinidazole not recommended below 3 years old as
albendazole not recommended below age of 6 years. Nitazoxanide1-3 yr: 100 mg bid
for 3 days
143. The largest protozoan that parasi zes human is
A. Entamoeba histolytica
B. Giardia lamblia
C. Balantidium coli.
D. Isospora belli
E. Cryptosporidium
143.(C).

144. The leading protozoal cause of diarrhea in children worldwide is


A. Entamoeba histolytica
B. Giardia lamblia
C. Balantidium coli
D. Isospora belli
E. Cryptosporidium.
144.(E). Cryptosporidium is recognized as a leading protozoal cause of diarrhea in
children worldwide and is a common cause of outbreaks in childcare centers; it is also a
significant pathogen in immunocompromised patients.

145. bleeding is acause of death in


A. severe anemia
B. kala azar.
C. secondary bacterial infection
D. hepatic failure
E. renal failure
145.( B). The late stage of the illness is often complicated by secondary bacterial
infections, which frequently are a cause of death.
146. coxsackievirus A16 is acause of .
A. coxsackievirus A6
B. Hand-foot-mouth disease.
C. enterovirus 71
D. coxsackievirus B2
E. coxsackievirus B12

146.(B). Hand-foot-and-mouth disease, one of the more distinctive rash


syndromes, is most frequently caused by coxsackievirus A16, some mes in large
outbreaks, and can also be caused by enterovirus 71; coxsackie A viruses 5, 6, 7, 9,
and 10; coxsackie Bviruses 2 and 5; and some echoviruses.
Answers of questions
from 51-139 (fifth part)
51.(D).
52.(C). Arthritis occurs in approximately 75% of patients with acute rheumatic fever
and typically involves larger joints, particularly the knees, ankles, wrists, and elbows.
Involvement of the spine, small joints of the hands and feet, or hips is uncommon.
Rheumatic arthritis is almost never deforming. There is often an inverse relationship
between the severity of arthritis and the severity of cardiac involvement.
53.(A). Acute rheumatic carditis usually presents as tachycardia and cardiac
murmurs,
with or without evidence of myocardial or pericardial involvement.
54.(B). Patients with carditis and more than minimal cardiomegaly and/or congestive
heart failure should receive corticosteroids. The usual dose of prednisone is 2
mg/kg/day in 4 divided doses for 2-3 wk followed by half the dose for 2-3 wk and
then
tapering of the dose by 5 mg/24 hr every 2-3 days. When prednisone is being tapered,
aspirin should be started at 50 mg/kg/day in 4 divided doses for 6 wk to prevent
rebound of inflammation.
55.(C). Phenobarbital (16-32 mg every 6-8 hr PO) is the drug of choice. If
phenobarbital
is ineffective, then haloperidol (0.01-0.03 mg/kg/24 hr divided bid PO) or
chlorpromazine (0.5 mg/kg every 4-6 hr PO) should be initiated. Some patients may
benefit from a few week courses of corticosteroids.
56.(D).
57.(B).
o Rheumatic fever without carditis : 5 yr or until 21 yr of age, whichever is longer
275
o Rheumatic fever with carditis but without residual heart disease : 10 yr or until
21 yr of age,whichever is longer
o Rheumatic fever with carditis and residual heart disease: 10 yr or until 40 yr of
age, whichever is longer; sometimes lifelong prophylaxis
58.(A). Early-onset neonatal GBS disease presents as sepsis; pneumonia and
meningitis.
Late-onset neonatal GBS disease most commonly manifests as bacteremia (45-65%)
and meningitis (25-35%). Invasive GBS disease in children beyond early infancy is
uncommon. Bacteremia without a focus is the most common syndrome associated
with childhood GBS disease beyond early infancy.
59.(D).
o bacteremia without a focus 10 days
o meningitis 2-3 wk
o ventriculitis at least 4 wk
o septic arthritis 3-4 wkase
o osteomyelitis 3-4 wkas
60.(A). Specific antitoxin is the mainstay of therapy and should be administered on
the
basis of clinical diagnosis. The role of antimicrobial therapy is to halt toxin
production,
treat localized infection, and prevent transmission of the organism to contacts.
61.(B). Three major forms of actinomycosis—cervicofacial, abdominal and pelvic,
and
pulmonary. In the patient with cervicofacial actinomycosis, there is often a history of
oral trauma, oral surgery, dental procedures, or caries, facilitating entry of organisms
into cervicofacial tissues. Cervicofacial actinomycosis usually manifests as a
painless,
slow-growing, hard mass and can produce cutaneous fistulas, a condition commonly
known as lumpy jaw. Less frequently, cervicofacial actinomycosis manifests
clinically as
an acute pyogenic infection with a tender, fluctuant mass with trismus, firm swelling,
and fistulas with drainage containing the characteristic sulfur granules.
62.(A). The brain is the most common secondary site and is involved in 15-40% of
cases
of pulmonary nocardiosis. Brain abscess is the most common presentation, and
meningitis is the second most common presentation.
63.(D). Actinomyces, staphylococcus, streptococcus, diphtheria, enterococcus,
listeria
and nocardia are gram positive.
64.(B).
65.(A). The most common clinical manifestation of meningococcal infection is
asymptomatic carriage of the organism in the nasopharynx.
66.(D).
67.(B). The most common complication of acute severe meningococcal septicemia is
focal skin infarction, which most commonly affects the lower limbs and can lead to
substantial scarring and require skin grafting.
68.(E). Poor prognostic factors on presentation include hypothermia or extreme
hyperpyrexia, hypotension or shock, purpura fulminans, seizures, leukopenia,
thrombocytopenia (including disseminated intravascular coagulation), acidosis, and
high circulating levels of endotoxin and tumor necrosis factor-α. The presence of
276
petechiae for <12 hr before admission, absence of meningitis, and low or normal
erythrocyte sedimentation rate indicate rapid, fulminant progression and poorer
prognosis.
69.(A). Ceftriaxone and ciprofloxacin are the most effective agents for prophylaxis,
the
latter being the drug of choice in some countries. Rifampin is most widely used but
fails
to eradicate colonization in 15% of cases. Neither penicillin nor ampicillin treatment
eradicates nasopharyngeal carriage and should not be routinely used for prophylaxis.
70.(E). Hematogenous dissemination occurs in 1-3% of all gonococcal infections,
more
frequently after asymptomatic primary infections than symptomatic infections.
Women
account for the majority of cases, with symptoms beginning 7-30 days after infection
and within 7 days of menstruation. The most common manifestations are asymmetric
arthralgia, petechial or pustular acral skin lesions, tenosynovitis, suppurative arthritis,
and, rarely, carditis, meningitis, and osteomyelitis. Only 25% of patients complain of
skin lesions.
71.(E). All patients who are presumed or proven to have gonorrhea should be
evaluated for concurrent syphilis, hepatitis B, HIV, and C. trachomatis infection.
72.(C). Children who have bacteremia or arthritis should be treated with ceftriaxone
(50 mg/kg/day; maximum: 1 g/day if weighs <45 kg) for a minimum of 7 days.
Meningitis should be treated for 10-14 days, and endocarditis for a minimum of 28
days, with ceftriaxone (50 mg/kg/ dose q12h with maximum of 1-2 g IV q12h).
Neonatal
gonococcal ophthalmia is treated effectively with a single dose of ceftriaxone (50
mg/kg IM, not to exceed 125 mg); a single dose of cefotaxime (100 mg/kg IM) is an
acceptable alternative. The conjunctivae should be irrigated frequently with
physiologic
saline solution.
73.(C). Kingellakingae is being increasingly recognized as the most common
etiology of
joint and bone infections in young children.
74.(A). Chancroid is caused by Haemophilus ducreyi, a fastidious Gram negative
bacillus.
75.(C). The most important clinical manifestation of M.catarrhalis infection in
children
is otitis media. Colonization and infection with M. catarrhalis are increasing in
countries
in which pneumococcal conjugate vaccines are used widely.
76.(C). Otitis media is the most common reason for which children receive
antibiotics.
On the basis of culture of middle ear fluid obtained by tympanocentesis, the
predominant causes of acute otitis media are Streptococcus pneumoniae, H.
influenzae, and M. catarrhalis.
77.(D). Typical paroxysms that are not life threatening have the following features:
duration <45 sec; red but not blue color change; tachycardia, bradycardia (not <60
beats/min in infants), or oxygen desaturation that spontaneously resolves at the end of
the paroxysm; whooping or strength for brisk self-rescue at the end of the paroxysm;
self-expectorated mucus plug; and post-tussive exhaustion but not unresponsiveness.
78.(C). Suctioning of nose, oropharynx, or trachea should not be performed on a
“preventive” schedule.
277
79.(A). Over a 48-hr period disease severity is unchanged or diminished.
80.(A).
o Azithromycin: Recommended agent 10 mg/kg/day in a single dose for 5 days
o Erythromycin: Not preferred
o Clarithromycin: Not recommended
o trimethoprim-sulfamethoxazole: Contraindicated for infants <2 mo of age
81.(D). Progressive pulmonary hypertension in very young infants and secondary
bacterial pneumonia are severe complications of pertussis and are the usual causes of
death. Echocardiography should be performed in critically ill infants with pertussis to
detect presence of pulmonary hypertension and to intervene expeditiously.
82.(E). Diarrhea 36%, abdominal pain 21%, pallor 20%, splenomegaly 17% and
headache 4%.
83.(A). The incubation period of typhoid fever is usually 7-14 days but depends on
the
infecting dose and ranges between 3 and 30 days.
84.(D). Results of blood cultures are positive in 40-60% of the patients seen early in
the
course of the disease, and stool and urine culture results become positive after the 1st
wk. Thrombocytopenia may be a marker of severe illness and may accompany
disseminated intravascular coagulopathy. Blood leukocyte counts are frequently low
in
relation to the fever and toxicity, there is a wide range in counts; in younger children
leukocytosis is common and may reach 20,000-25,000 cells/μL.
85.(C). Untreated diarrhea can last 7-10 days; only approximately 10% of patients
have
diarrhea persisting for longer than 10 days. Persistent diarrhea occurs in
malnourished
infants, children with AIDS, and occasionally previously normal children.
86.(B). Of the more than 200 serogroups, only serogroups O1 and O139 have been
associated with epidemics, although some non-O1, non-O139 V.choleraestrains (e.g.,
O75 and O141) are pathogenic and can cause small outbreaks.
87.(D). Persons with blood group O, decreased gastric acidity, malnutrition,
immunocompromised state, and absence of local intestinal immunity (prior exposure
by infection or vaccination) are at increased risk for developing severe disease.
88.(D).
89.(A). The most common late-onset complications include reactive arthritis and
Guillain-Barré syndrome.
90.(A). Most Campylobacter isolates are susceptible to macrolides, fluoroquinolones,
aminoglycosides, chloramphenicol, tetracyclines, and clindamycin, and are resistant
to
cephalosporins, rifampin, penicillins, trimethoprim, and vancomycin.
91.(B). Yersinia enterocolitica is by far the most common Yersinia species causing
human disease and it produces fever, abdominal pain that can mimic appendicitis,
and
diarrhea. Yersinia pseudotuberculosis is most often associated with mesenteric
lymphadenitis. Yersinia pestis is the agent of plague and most commonly causes an
acute febrile lymphadenitis (bubonic plague) and less commonly occurs as
septicemic,
pneumonic, pharyngeal, or meningeal plague. Other Yersinia organisms are
uncommon
causes of infections of humans.
278
92.(C). Patients with conditions leading to iron overload are at higher risk of
developing
Yersinia infections.
93.(E). Septicemia is more common in younger children, and reactive arthritis is
more
common in older patients.
94.(A). Ulceroglandular and glandular disease are the 2 most common forms of
tularemia diagnosed in children. The most common glands involved are the cervical
or
posterior auricular nodes owing to a tick bite on the head or neck. If an ulcer is
present,
it is erythematous and painful and may last from 1-3 wk. The ulcer is located at the
portal of entry. After the ulcer develops, regional lymphadenopathy ensues.
95.(E). Headache, mental inattention, and depression may be demonstrated in
patients
with brucellosis, invasion of the nervous system occurs in only approximately 1% of
cases.
96.(D). Aminoglycoside antibiotics should be avoided because they may potentiate
the
blocking action of botulinum toxin at the neuromuscular junction.
97.(E). Tetanus is most often generalized but may also be localized. The incubation
period typically is 2-14 days but may be as long as months after the injury. Because
tetanus toxin does not affect sensory nerves or cortical function, the patient
unfortunately remains conscious, in extreme pain, and in fearful anticipation of the
next tetanic seizure. The tetanic paralysis usually becomes more severe in the 1st wk
after onset, stabilizes in the 2nd wk, and ameliorates gradually over the ensuing 1-4
wk.
98.(A). Major adverse events include hepatotoxicity in 1% of children and
approximately 3% of adults (increasing with age) and dose-related peripheral
neuropathy. Minor adverse events include rash, worsening of acne, epigastric pain
with
occasional nausea and vomiting, decreased vitamin D levels, and dizziness.
99.(B). Aluminum salts decrease absorption of isoniazid, rifampin increase
hepatotoxicity of isoniazid, isoniazid increase level of warfarin and prednisolone
increased isoniazid metabolism.
100.(B). Concentrations of fluconazole are 10-20– fold higher in the urine than blood,
making it an ideal agent for treating fungal urinary tract infections.
101.(E). The cumulative incidence is <0.3% among infants >2,500 g birthweight
admitted to the NICU. The cumulative incidence increases to 8% for infants <750 g
birthweight.
102.(B). Twice weekly fluconazole at 3 and 6 mg/kg/dose decreases rates of both
colonization with Candidaspecies and invasive fungal infections.
103.(B). While the others inhibits viral DNA polymerase.
104.(B). The use of suppressive therapy with oral acyclovir for 6 mo has been
demonstrated to improve neurodevelopmental outcome.
105.(D). The main route of elimination is renal, and dosage adjustments are necessary
for renal insufficiency. Toxicity is observed typically only in exceptional
circumstances:
for example, if administered by rapid infusion to a dehydrated patient or a patient
with
underlying renal insufficiency, acyclovir can crystallize in renal tubules and produce
a
reversible obstructive uropathy.
279
106.(D). Ribavirin and its metabolites concentrate in red blood cells and can persist
for
several weeks and, in rare instances, may be associated with anemia.
107.(C).
108.(D). The rash fades over about 7 days in the same progression as it evolved, often
leaving a fine desquamation of skin in its wake.
109.(C). Acute otitis media is the most common complication of measles and
pneumonia is the most common cause of death in measles.
110.(E). Males are affected twice as often as females, and there appear to be more
cases reported from rural than urban populations. Clinical manifestations of SSPE
begin
insidiously 7-13 yr after primary measles infection. The hallmark of the 2nd stage is
massive myoclonus, which coincides with extension of the inflammatory process site
to
deeper structures in the brain, including the basal ganglia. The 4th stage is
characterized by loss of critical centers that support breathing, heart rate, and blood
pressure. Death soon ensues.
111.(C). Nerve deafness is the single most common finding among infants with
congenital rubella syndrome.
112.(E). The incubation period for mumps ranges from 12-25 days but is usually 16-
18
days. Parotitis may be unilateral initially but becomes bilateral in approximately 70%
of
cases. The opening of the Stensen duct may be red and edematous. The parotid
swelling peaks in approximately 3 days and then gradually subsides over 7 days.
113.(A). The most common complications of mumps are meningitis, with or without
encephalitis, and gonadal involvement. Uncommon complications include
conjunctivitis, optic neuritis, pneumonia, nephritis, pancreatitis, and
thrombocytopenia.
114.(E). No fetal malformations have been associated with intrauterine mumps
infection.
115.(E). Symptomatic CNS involvement occurs in 10-30% of infected individuals,
but
CSF pleocytosis has been found in 40-60% of patients with mumps parotitis. The
meningoencephalitis may occur before, along with, or following the parotitis. It most
commonly manifests 5 days after the parotitis. In typical cases, symptoms resolve in
7-
10 days. The CSF protein content is normal or mildly elevated. Less-common CNS
complications of mumps include transverse myelitis, aqueductal stenosis, and facial
palsy.
116.(A). Hematogenous dissemination of virus to the central nervous system appears
to only occur in neonates.
117.(C). Herpes whitlow is a term generally applied to HSV infection of fingers or
toes,
although strictly speaking it refers to HSV infection of the paronychia.
118.(D). HSV is the most common cause of recurrent aseptic meningitis (Mollaret
meningitis).
119.(C). Infants with encephalitis typically present at 8-17 days of life with clinical
findings suggestive of bacterial meningitis, including irritability, lethargy, poor
feeding,
280
poor tone, and seizures. Fever is relatively uncommon, and skin vesicles occur in
only
approximately 60% of cases.
120.(A). The absolute neutrophil count should be measured at weeks 2 and 4 after
initiation treatment and then monthly.
121. (C). Herpes zoster is caused by the reactivation of latent VZV. It is not common
in
childhood and shows no seasonal variation in incidence.
122.(C). The illness usually begins 14-16 days after exposure, although the
incubation
period can range from 10-21 days. Subclinical varicella is rare. Temperature
elevation is
usually 37.8-38.9°C but may be as high as 41.1°C. Varicella lesions often appear first
on
the scalp, face, or trunk.
123.(A). The simultaneous presence of lesions in various stages of evolution is
characteristic of varicella.
124.(E). Clinical recovery is typically rapid, occurring within 24-72 hr, and is usually
complete.
125.(E).
126.(E). EBV is shed in oral secretions consistently for more than 6 mo after acute
infection and then intermittently for life.
127.(E). EBV infects more than 95% of the world’s population.
128.(B). The classic physical examination findings are generalized lymphadenopathy
(90% of cases), splenomegaly (50% of cases), and hepatomegaly (10% of cases).
Epitrochlear lymphadenopathy is particularly suggestive of infectious mononucleosis.
Symptomatic hepatitis or jaundice is uncommon, but elevated liver enzymes are very
common. Splenomegaly to 2-3 cm below the costal margin is typical (15-65% of
cases)
and is seen in most cases by ultrasonography; massive enlargement is uncommon.
Palatal petechiae at the junction of the hard and soft palate are frequently seen.
129.(B). Hearing loss is the most common long-term sequela associated with
congenital CMV infection, the failure of an infant to pass a newborn hearing
screening
exam should raise the possibility of congenital CMV infection. Hearing loss in the
older
infant and young child should also alert the clinician to the possibility of congenital
CMV infection, as approximately 50% of infants with hearing loss associated with
congenital CMV infection will pass an initial hearing screening exam but develop
hearing loss in later infancy and early childhood.
130.(D). Congenital infection with HHV-7 has not been demonstrated.
131.(D).
132.(A).
133.(D). Vaccines are available for HAdV types 4 and 7, but are used only for
military
populations.
134.(E). The virus can be identified in 15-20% of children with isolated pharyngitis,
mostly in preschool children and infants.
135.(C). Vomiting and fever typically abate during the 2nd day of illness, but
diarrhea
often continues for 5-7 days.
281
136.(A). Common adverse effects include abdominal pain, diarrhea, and nausea. Rare
side effects include anorexia, flatulence, increased appetite, fever, pruritus, and
dizziness.
137.(A). It is FDA approved for treatment of trichomoniasis and for giardiasis and
amebiasis in children 3 yr of age and older. In the treatment of giardiasis, it has the
advantages of very few side effects and only requiring a single dose. It is excreted via
urine and feces. Tinidazole carries a pregnancy category C classification and can be
detected in breast milk. Breastfeeding should be interrupted during treatment and for
3 days after treatment.
138.(D). Naegleria, Acanthamoeba, Balamuthia, and Sappiniaare small, free living
amebas that cause human amebic meningoencephalitis.
Dr.Wahid Helmi-Egypt

Dr.Wahid Helmi Your text here 1

Consultant Pediatrician
Zarka-Dymiate – Egypt
Neonatal x-ray (Revision)
REFERENCES
1 . Bell MJ , Ternberg JL , Fagin RD et al . Neonatal necrotizing enterocolitis. Therapeutic decisions
based upon clinical staging . Ann Surg 1978; 187 : 1 – 7
2 . Blakely ML , Tyson JE , Lally KP . Laparotomy versus peritoneal drainage for necrotizing enterocolitis
or isolated intestinal perforation in extremely low birth weight infants: outcomes through
18 months adjusted age . Pediatrics 2006; 117 : e680 – e687
3 . Moss RL , Dimmitt RA , Barnhart DC et al . Laparotomy versus peritoneal drainage for necrotizing
enterocolitis and perforation . N Engl J Med 2006; 354 : 2225 – 34
4 . Shohat M , Levy G . Transient tachypnoea of the newborn and asthma . Arch Dis Child
1989 ; 64 : 277 – 79
Dr.Wahid Helmi-Egypt

1. A term baby is noted to be grunting shortly after


birth. The baby is tachypnoeic and there is marked
recession. The following chest X-ray has been obtained.

a. Describe the abnormalities on the X-ray.


b . W hat is the diagnosis?
c . W hat is the management of the baby?

1. a. i. The heart is displaced to the left of the chest.


ii. There are bowel loops visible in the chest cavity on the right.

b. Right-sided diaphragmatic hernia. These are much less


common than left-sided diaphragmatic hernias. The baby is not
ventilated (no endotra-cheal tube visible on CXR) and this
therefore suggests that this defect was not recognised antenatally.
The optimal management for these babies is intubation and
ventilation immediately after birth avoiding lung infl a-tion using
a mask.

c. This baby should be ventilated to prevent the bowel from


distending any more with swallowed air. A paediatric surgical
opinion should be sought
Dr.Wahid Helmi-Egypt

2 . A 28 week baby is 10 days old. Abdominal distension has


been noted and the following X-ray is obtained.

a . D escribe the abnormalities on the X-ray.


b . W hat is the diagnosis?

2. a. i. There is a nasogastric tube in situ.


ii. The bowel loops are dilated.
iii. The bowel wall is thickened. iv. There is no air in the rectum.
v. There is widespread intramural gas.

b. The diagnosis is necrotising enterocolitis. There are staging


criteria for NEC described by Bell. The features described in the
above X-ray are consistent with Bell stage 2 NEC.
Dr.Wahid Helmi-Egypt

3. The same baby as in question 2 is reviewed a few hours


later. His abdomen has become more distended and
discoloured. The following abdominal X-ray is obtained.

a. Describe the abnormalities on the X-ray.


b . W hat is the diagnosis?
c . W hat is the management of this baby?

3. a. The lateral abdominal X-ray shows free air. There are


loops of bowel extending into the gas fi lled area.

b. The baby has a perforation. This would increase the


stage of NEC to Bell stage 3.

c. The baby should be reviewed by a paediatric surgeon. If


ventilation is compromised a drain should be inserted
through the abdominal wall to release the pressure from
build up of free gas in the abdomen. There have been
recent studies to compare drain insertion with laparotomy
for NEC.
These have not demonstrated superiority of one technique
over the other, but have confi rmed the poor outlook of
babies requiring surgery for perforated NEC.
Dr.Wahid Helmi-Egypt

4. A 37 week gestation baby is noted to have frequent small


vomits. At 24 hours of age he is noted to be tachypnoeic and
he is admitted to the neonatal unit. A chest X-ray is obtained.

a. Describe the abnormalities on the X-ray.


b. What are the diagnoses?
c. What is the management of the baby?

4. a. i. There is a large patch of opacifi cation in the mid


zone on the right side of the chest. There is some patchy
shadowing elsewhere.
ii. There is a nasogastric tube in place.
iii. The nasogastric tube is coiled in a pouch in the thorax,
behind the heart.

b. This baby has a sliding para-oesophageal (hiatus) hernia.


The prevalence in newborns is unknown but it is thought to
be relatively uncommon. It can be discovered on a routine
X-ray but usually presents with features of gastro-
oesophageal refl ux. In this case the changes in the lung
would be compatible with aspiration in association with
reflux.

c. No interventions are needed if the child is asymptomatic.


However, in this case it would be appropriate to commence
antibiotics and consider anti-refl ux treatment if vomiting
remains problematic.
Dr.Wahid Helmi-Egypt

5 . A 36 week baby is admitted to the neonatal unit after a


choking episode. The following chest X-ray is obtained.

a. Describe the abnormalities on the X-ray.


b . W hat is the diagnosis?
c . W hat is the management of the baby?

5. a. With the exception of chest leads and a nasogastric


tube there are no
abnormalities on this X-ray. It is normal.

b. Normal.

c. The chest X-ray does not reveal anything that needs


treatment.
Dr.Wahid Helmi-Egypt

6. A 27 week infant is ventilated for RDS. Ventilatory


requirements have increased over the last hour. A chest
X-ray is obtained.

a . D escribe the abnormalities on the X-ray.


b . W hat is the management of the baby?

6. a. i. The endotracheal tube is down the right main bronchus.


ii. The left lung is inadequately aerated.
iii. There is a nasogastric tube in situ.

b. The endotracheal tube should be withdrawn and the correct


position confi rmed with a repeat chest X-ray.
Dr.Wahid Helmi-Egypt

7 . A term baby is noted to have a degree of frontal


bossing, as does his mother. A chest X-ray is
performed.

a . Describe the abnormalities on the X-ray.


b . W hat is the diagnosis?
c . W hat is the inheritance of this condition?

7. a. The clavicles are absent.

b. Cleidocranial dysostosis. In this condition, the clavicle is


either hypoplastic or absent and the ribs are short. The
anterior fontanelle often closes late and there may be
delayed eruption of teeth. There can be bossing of the
forehead.

c. Autosomal dominant. It results from a mutation in the


CBFA1 gene,which controls a key transcription factor in
osteoblast differentiation.
Dr.Wahid Helmi-Egypt

8. A 29 week gestation baby has been on CPAP for


moderate respiratory distress and has been stable in
35% oxygen. He is active and has been noted to have
quite marked intercostal recession on occasion. He
suddenly deteriorates with
persistent recession and an increase in oxygen
requirements to 95%. A chest X-ray is taken.

a. Describe the abnormalities on the X-ray.


b . W hat is the management of the baby?

8. a. i. Large tension pneumothorax on the left.


ii. Mediastinal shift to the right with tracheal deviation.
iii. Transcutaneous oxygen electrode on left upper chest.
iv. Surprisingly the infant is not intubated.

b. i. Immediate drainage of the pneumothorax.


ii. Intubation and ventilation is very likely to be needed.
Dr.Wahid Helmi-Egypt

9 . A 27 week gestation infant is ventilated for respiratory distress.


Ventilation requirements have been moderately high and two doses
of surfactant have been given. Oxygen requirements have steadily
risen from 55% to 95%. In response to a profound bradycardia and
desaturation he receives ventilation down his
endotracheal tube using a resuscitation bag. He deteriorates further
and a chest X-ray is obtained.

a . D escribe the abnormalities on the X-ray.


b . W hat is the management of the baby?
9. a. i. Intubated infant.
ii. Right pneumothorax.
iii. Pneumopericardium.
iv. Umbilical arterial catheter.
v. Bilateral hyperinfl ation of the chest.
vi. There is abnormal shadowing at the right base which is diffi cult to
assess without other X-rays. This turned out to be a cystic adenomatoid
malformation.

b. The pneumothorax should be drained. The decision to drain the


pneumopericardium will be dictated by the clinical condition of the infant,
specifi cally with respect to cardiac function.
Dr.Wahid Helmi-Egypt

10. A 31 week gestation infant is admitted to the intensive


care unit. Mother booked late and there has been minimal
antenatal care. The baby needs resuscitation at delivery
and is transferred to the unit ventilated. Ventilation proves
very difficult and a chest X-ray is obtained.

a . D escribe the abnormalities on the X-ray.


b . W hat is the differential diagnosis?
c . W hat is the management of the baby?

10. a. i. Intubated infant.


ii. High UAC.
iii. Large cystic mass in right lung with a septum running across the
middle.

b. These appearance would be consistent with a large cystic


adenomatoid malformation (which was the case in this infant) or
lobar emphysema. The latter is unlikely as this condition usually
develops after birth as opposed to presenting with problems at birth.
There is debate about the use of the word ‘congenital’ as some
authorities maintain that the lobar emphysema develops as a result
of airway problems that develop postnatally.

c. The management of these conditions is extremely diffi cult. Close


co-operation with paediatric surgeons is essential. Direct puncture
has a high chance of leading to formation of a broncho-pleural fi
stula. The infant may be too sick to tolerate thoracotomy, yet
respiratory compromise may become so severe that intervention is
necessary.
Dr.Wahid Helmi-Egypt

11. A 32 week gestation infant is stable at birth but


aproblem is noted at admission and a chest X-ray
obtained.

a . D escribe the abnormalities on the X-ray.


b . W hat is the management of the baby?

11. a. There is a large bore (Replogle) nasogastric tube,


partially coiled in the upper oesophagus. There is a shadow
of air in the upper oesophagus but none lower down,
suggesting an oesophageal atresia. There is evidence of
gas in the gastrointestinal tract, suggesting a tracheo-
oesophageal fi stula.

b. Management is complex and requires close collaboration


with paediatric surgeons. If the infant is mature and clinically
stable, surgery should be considered in the relatively near
future. If the infant is very sick or immature, then a decision
may be made to defer surgery until a more stable state is
achieved, or the baby has grown.
Dr.Wahid Helmi-Egypt

12. A term baby is thought to have an absent Moro refl ex and


limited movementc of the arm. An X-ray is taken. In view of these
changes a chest X-ray is then performed.

a. Describe the abnormalities on the arm X-ray.


b . D escribe the abnormalities on the chest X-ray.
c . W hat is the most likely diagnosis?
d . W hat management would you consider?
12. a. i. The X-ray shows a fracture of the left humerus.
ii. There is marked osteopenia of all bones, especially marked in the bones of
the forearm.
b. i. Marked rotation of the film.
ii. There is thinning of the ribs with what are almost certainly fractures with
callous formation.
iii. Osteopenia of all bones.
c. Osteogenesis imperfecta.

d. Pamidronate infusions have been used to improve bone density and reduce
the risk of further fractures. Such management should only be undertaken in
specialist centres as general experience of this management is extremely
limited.
Dr.Wahid Helmi-Egypt

13. A 27 week gestation infant is now 4 weeks old. She needs 0.4 L/min
supplementary oxygen but this has recently risen to 0.8 L/min. A chest X-
ray
is performed.
a . D escribe the abnormalities on the X-ray.
b . W hat management has this baby had in the past that is clearly evident
on
the X-ray?
c . W hat management will you initiate in response to the abnormality on
this
chest X-ray?

13. a. i. Clip in left upper chest.


ii. Abnormal separation of left 4th and 5th ribs.
iii. Widespread patchy opacifi cation of both lungs.
iv. Large bulla in right lung base.
v. Nasogastric tube in situ.

b. The PDA has been clipped.

c. None. This infant is not intubated and ventilated and


unless there are signs of severe respiratory distress, no
action is needed. The infant has what appears to be quite
severe chronic lung disease and would not benefit from re-
ventilation. Although the bulla looks fairly large on X-ray,it is
only occupying a relatively small proportion of the intra-
thoracic space. The majority resolve spontaneously over
time, as was the case with this infant.
Dr.Wahid Helmi-Egypt

14. Antenatal ultrasounds have suggested a growth


retarded fetus. At birth, the baby has obvious limb
abnormalities and requires ventilation. Limb and chest X-
rays are performed.

a. Describe the abnormalities on the limb X-ray.


b . D escribe the abnormalities on the chest X-ray.
c . W hat is the most likely diagnosis?
d . W hat is the prognosis?
14. a. i. Short long bones (compare to size of cord clamp).
ii. Marked bowing of bones, particularly noticeable in femurs.
b. The chest is very narrow with abnormal ribs that are short and thin
with splayed ends. The vertebral bodies are fl attened with wide
intervertebral spaces.
c. The appearance is that of a congenital skeletal dysplasia. In this
case the most likely diagnosis is thanatophoric dysplasia as
indicated by the extremely short limbs with classic ‘telephone
receiver ’ shaped femurs.
d. Thanatophoric dysplasia is invariably fatal, usually shortly after
birth.
Dr.Wahid Helmi-Egypt

15. A baby is born at term and is noted to be cyanosed at rest.


There is moderate recession. There is bilateral lower limb
oedema. Femoral pulses are weak. The anterior fontanelle is
bulging and a loud bruit can be heard when listening over it.A
chest X-ray is performed.
a. What abnormalities are there?
b . W hat possible explanation do you have?
c . W hat initial further investigations would you consider?

15. a. The heart is grossly enlarged in all dimensions.

b. The enlarged heart does not in itself point to a specifi c


diagnosis and conditions such as a cardiomyopathy,
structural heart defect, storage disorder, maternal diabetes
and heart failure are all possibilities. In this case the
combination of a large heart, peripheral oedema and a bruit
over the bulging anterior fontanelle should raise the
possibility of an intracranial arteriovenous malformation
resulting in high output cardiac failure
(as was the case in this baby).

c. i. Echocardiogram.
ii. Cranial ultrasound, CT scan or MRI.
iii. Four limb BP and oxygen saturations.
iv. Check maternal history and antenatal ultrasounds.
Dr.Wahid Helmi-Egypt

1 6. A baby is born at 26 weeks and requires ventilation from


birth. Surfactant was given on delivery suite. Ventilation has
steadily increased and the baby is in 95% oxygen at pressures
of 28/6 at 24 hours of age. Blood gases are poor. A chest X-ray
is performed.
a . W hat abnormalities are there?
b . W hat is the most likely diagnosis?
c . W hat treatment would you consider?

16. a. i. There is widespread opacifi cation throughout both lung


fields.
ii. There are clear air bronchograms on both sides.
iii. The heart border is not clearly defi ned.
iv. The costophrenic and cardiophrenic angles are not clearly
visualised.
v. There is an endotracheal tube.
vi. There is a central venous line entering by the left arm and with a
tip at the junction of the SVC and right atrium.

b. Respiratory distress syndrome is the most likely diagnosis.

c. Surfactant should be given if the dose has not been repeated


since birth.Ventilatory requirements are high and some centres
would consider high frequency oscillatory ventilation at this point. If
this is not available ventilation will probably need to be adjusted to
improve the blood gases.
Dr.Wahid Helmi-Egypt

17. A term infant is born following a severe asphyxial insult. There is thick
meconium present and the oro-pharynx and trachea are suctioned under
direct vision and meconium is removed. Ventilation is required and
requirements steadily increase. At 8 hours of age the pressures are 32/4
and 100% oxygen is required. The blood gas shows a pH of 7.01, PO 2 of
3.4 and PCO 2 of 7.2.
Figure 7.17
a. What does the X-ray show?
b. What is the diagnosis and what complication may be present?
c. What action could be taken?

17. a. i. There is patchy opacifi cation that is most marked in the perihilar region.
ii. There are basal bullae, most apparent on the right.
iii. The lungs are relatively hyperexpanded, 11 posterior rib ends are showing.
iv. There is a transcutaneous oxygen electrode, chest leads and a
nasogastric tube.
v. There is an endotracheal tube.
b. The history and chest X-ray appearances are typical of meconium aspiration
syndrome. It is very possible that a signifi cant degree of persistent pulmonary
hypertension is contributing to the clinical picture.
c. There is evidence that both high frequency ventilation and nitric oxide may be
useful in this condition, and are particularly useful in combination.In severe
cases extracorporeal membrane oxygenation may be life saving and referral
should be made early. There is some evidence from animal studies and limited
human data to support the use of surfactant,either by bolus administration of by
surfactant lavage.
Dr.Wahid Helmi-Egypt

18. A 36 week gestation infant has mild respiratory distress at birth.


He requires 30% oxygen and there is mild recession. As symptoms
persist an X-ray is performed at 6 hours of age.
a . W hat does the X-ray show?
b . I f this is the only problem with the baby, what is the most likely
diagnosis?
c . I f the baby is asymptomatic, what should you do?
d . I f the baby is symptomatic, what would you consider?
e . W hat other investigations would be warranted?

18. a. i. There is a nasogastric tube in situ.


ii. There is loss of the costophrenic angle on the right and
increased
shadowing in the lower peripheral right chest.

b. Right pleural effusion. Pleural effusions can occur for


many reasons. If diagnosed antenatally, they are associated
with chromosomal or congenital abnormalities. Congenital
viral infections can also cause effusions.Isolated effusions
can be a chylothorax or idiopathic.

c. If asymptomatic and there are no other concerns it is


appropriate to simply observe the infant. Idiopathic effusions
may resolve while a chylothorax.
Dr.Wahid Helmi-Egypt

1 9. A term infant is born by elective caesarean section because


of previous caesarean sections. Shortly after birth the baby is
noted to be mildly dusky and grunting and there is mild
recession. A septic screen is performed and a chest X-ray
obtained. Antibiotics are started.
a. What abnormality is seen on this X-ray?
b . W hat is the most likely diagnosis?
c . W hat management steps do you need to take?
d . W hat is the prognosis for the baby?

19. a. There is fl uid in the horizontal fi ssure and some peri-hilar


streaking.

b. The most likely diagnosis is transient tachypnoea of the newborn.


However due to the diffi culty in accurately diagnosing infection it is
common to consider this as a possible differential diagnosis.

c. Close observation and respiratory support such as CPAP may be


needed
for a short while. Affected infants rarely require more than 40%
oxygen
or respiratory support for more than three days. Antibiotics should be
given until infection has been excluded.

d. The prognosis is good. Most infants recover over 24 hours. There


is
debate regarding whether infants with TTN are more prone to
wheezing
in childhood.
Dr.Wahid Helmi-Egypt

20. A preterm infant has chronic lung disease. Her condition worsens
and following investigations a surgical procedure is performed. The
following day a chest X-ray
is performed.
a . D escribe the X-ray.
b . W hat surgical procedure was performed?
c . W hat treatment may be indicated?

20. a. i. There is an endotracheal tube in situ.


ii. There is a nasogastric tube in situ.
iii. Both lungs show infl ammatory and cystic change consistent with
bronchopulmonary dysplasia.
iv. The lungs appear hyperinfl ated – note the bulging pleura on the
right.
v. There is loss of the left costophrenic angle and obvious outlining
of the lungs on the left side with fl uid.
vi. There is abnormal separation of the upper ribs on the left.
vii. There is a long line in situ in the left subclavian vein. The position
of the tip seems satisfactory.
viii. There is marked oedema visible most obviously outside the
chest.

b. The separation of the ribs on the left side of the chest is highly
suggestive of a ductal ligation in the recent past. The chylothorax
which is present is a well recognised complication of the procedure.

c. This infant has bronchopulmonary dysplasia and is still ventilated.


The treatment options available at this stage would be a short course
of diuretics or steroids. Diuretics can improve oxygenation and lung
compliance,but it is prudent to give a short course to assess for
response before embarking on long-term diuretic use due to the
high incidence of side effects.
Dr.Wahid Helmi-Egypt

21. A 26 week infant is in 0.5 L /min of supplementary oxygen at


36 weeks corrected age. He has mild recession and capillary
gases show a fully compensated respiratory acidosis. A chest
X-ray is taken as part of a work-up for chronic lung disease.
a . Describe the X-ray.
b . W hat is the likely diagnosis?

21. a. i. Generalised patchy opacification throughout the lung fields.


ii. Small volume lungs.
iii. Tracheal deviation to the right.
iv. Rib fractures on the right with callus formation.
Dr.Wahid Helmi-Egypt

22. A baby born at 24 weeks gestation is 2 weeks old. He has been stable
on low ventilation pressures but has not tolerated CPAP. Attempts have
been made to start nasogastric feeds on several occasions but he does
not appear to tolerate them as there are reasonable volume gastric
aspirates on most occasions.Over the last 48 hours the aspirates have
become increasingly bilious and an abdominal X-ray is taken.
a . D escribe the X-ray abnormalities.
b . W hat is the diagnosis?
c . W hat is this diagnosis commonly associated with?
d . W hat is your immediate management plan?
e . W hat is the likely outcome for this baby?

22. a. There is a ‘double bubble ’ with a dilated stomach and duodenum.


b. Duodenal atresia.
c. Additional problems may be Down syndrome, cardiac abnormalities and
malrotation.
d. A large-bore nasogastric tube must be inserted and left on free drainage;
electrolyte and blood gas anomalies must be corrected.
e. Isolated duodenal atresia can be successfully corrected by surgery
although experience at such early gestation is very limited. Other anomalies
such as cardiac defects (which are much commoner with duodenal atresia) will
affect the prognosis.
Dr.Wahid Helmi-Egypt

23. A 26 week infant has been ventilated for RDS. He was weaned
onto CPAP after 16 days and was initially stable. Six days after
starting CPAP his oxygen requirements started to rise (from
35% to 65%) and he was commenced on a course of
dexamethasone; 36 hours after starting dexamethasone he
deteriorates substantially. An X-ray is taken.
a. What abnormalities are seen on this X-ray?
b. What is the diagnosis?
c. What is your management plan?

Your text here 1

23. a. i. There is a naso-jejunal tube in situ.


ii. There is a very large translucency covering much of the abdomen.
This ‘football sign’ indicates free air in the peritoneal cavity.
iii. There is a white line running to the right of and almost parallel to the
vertebral column. This is the falciform ligament and it only becomes
visible when there is free intraperitoneal gas.
iv. The lung fields are patchy throughout.They are extremely low volume
yet appear hyperinflated with marked upward slanting of the ribs. It is
difficult to interpret how many of these changes are due to intrinsic
lung disease and how much to compression from abdominal
distension.
b. There is obvious chronic lung disease and definite gastrointestinal perfo-
ration.The latter is quite possibly related to the administration of steroids.
c. Management will depend upon the severity of illness. If his condition has
become unstable and gases have deteriorated, ventilation will be
required. If the abdominal distension compromises ventilation an
abdominal drain may well be required. Surgical review and consideration
for a laparotomy will be needed early in the process.
Dr.Wahid Helmi-Egypt

24. A term baby becomes cyanosed 6 hours after birth. Increasing the inspired
oxygen concentration only partially treats this. On examination the chest is
clear and there are no abnormal sounds. A chest X-ray is obtained.
a. Describe the chest X-ray.
b. What is your diagnosis?
c. What will you do next?

24. a. i. The heart shape is ovoid (egg-shaped).


ii. The upper mediastinum is narrow.
iii. The lung fields appear normal.
b. Transposition of the great arteries.
c. The diagnosis should be confirmed by echocardiography. A
prostaglan- din infusion should be started and the case should be
discussed with the nearest paediatric cardiac centre as soon as
possible.
Dr.Wahid Helmi-Egypt
25. A 28 week gestation infant has been born. A chest and abdominal
X-ray is taken at 4 hours of age.
a. Describe the X-ray.
b. What is your diagnosis?
c. What will you do next?

25. a. i. Both lung fields show a diffuse granular appearance.


ii. There are bilateral air bronchograms.
iii. The end of the endotracheal tube is high.
iv. The tip of the UAC appears to be somewhere close to the right
subclavian artery.
b. Respiratory distress with incorrectly placed UAC and ET tube.
c. i. The UAC will need to be withdrawn into a more appropriate
position.
ii. The endotracheal tube will need to be inserted further.
iii. A nasogastric tube should be inserted and the intestines
decompressed.
iv. Surfactant administration should be considered.
Dr.Wahid Helmi-Egypt
26. A term infant is born to a mother who is known to have insulin
dependent diabetes. He is grunting from birth and oxygen
saturations are poor. A chest X-ray is obtained.
a. Describe the X-ray.
b. What is your diagnosis?
c. What will you do next?

26. a. i. The heart is much larger than normal.


ii. The lung fields appear well aerated.
iii. Pleura bulging on both sides.
iv. UAC on the left – very high (probably – this cannot be confirmed to
be the UAC until the abdomen is X-rayed and the caudal loop is
seen).
v. UVC on the right, very high (position confirmation needed as
above).
b. A diabetic cardiomyopathy is the most likely diagnosis. Although
septal hypertrophy is the best described association, biventricular
hypertrophy may develop with significant and serious reduction in
stroke volume.
c. i. Reposition the lines.
ii. Echocardiography.
iii. Seek expert advice if haemodynamically unstable.
Dr.Wahid Helmi-Egypt
27. A 32 week gestation infant has been born. There has been a 10-week
history of oligohydramnios. The baby has required ventilation from
birth. Blood gases are poor at high ventilation pressures. A chest X-
ray is taken at 3 hours of age.
a. Describe the X-ray.
b. What is your diagnosis?

a. i. The chest is small.


27.
ii. The ribs are thin and slope downwards steeply.
iii. There is a UVC about 1 cm above the diaphragm.
iv. The UAC tip is at T9.
v. The endotracheal tube tip is very high and is only just visible on
the X-ray.
b. Pulmonary hypoplasia secondary to oligohydramnios.
Dr.Wahid Helmi-Egypt
A term baby has not fed well since birth. Bilious vomiting has been
28.
noted, as has mild abdominal distension. At 48 hours the
distension becomes much worse and an abdominal X-ray is taken.
a. What does the X-ray show?
b. What do you think has happened?
c. What will you do next?

28. a. i. There is free gas in the peritoneal cavity.


ii. There are dilated loops of bowel, several of which appear to have
very irregularly thickened walls.
iii. The right diaphragm appears thickened.
iv. There is no gas in the rectum.
b. There is intestinal perforation which has probably been present some
time to result in the degree of bowel wall thickening that is apparent.
The presence of irritant material is also probably responsible for thick-
ening of the diaphragm. It is not clear why the bowel has perforated but
the absence of rectal gas may be suggestive of an obstruction. In this
baby’s case there was colonic atresia.
c. The free gas will need drain insertion, unless a laparotomy is performed
immediately. Surgical review is essential and the baby should be
stabilised as dictated by clinical condition.
Dr.Wahid Helmi-Egypt
29. A 27 week gestation infant has had a central venous line inserted. A
chest X-ray is taken to confirm line position.
a. What does the X-ray show?
b. Is the line position acceptable?
c. What will you do next?

29. a. i. The film is extremely rotated and it is difficult to accurately place


anything in the picture.
ii. There is a line to the right of the vertebral column that could be a UAC,
but this cannot be stated with any certainty.
iii. There is an endotracheal tube which is slightly high.
iv. There is a central venous line coming in from the right arm. It is very
likely to be actually ending lateral to the chest.
b. The line position is almost certainly unsatisfactory.
c. The line should be removed. A further non-rotated film could be obtained
but the position of the line is likely to be more lateral if this is done.
Dr.Wahid Helmi-Egypt30. A 28 week gestation infant has been ventilated for 12 days. He is now
6 weeks old and requires 0.6 L/min of oxygen. A chest X-ray is
taken.
a. What does the X-ray show?
b. What action will you take?

30. a. i. The lungs show diffuse patchy opacities consistent with chronic
lung disease.
ii. The lungs are relatively small volume and hyperinflated, with upward
sloping ribs.
iii. There are vertebral abnormalities.
b. No action is indicated although it is assumed that a detailed examination has
already been performed to exclude any other anomalies that may be associated with the
vertebral abnormalities.
Dr.Wahid Helmi-Egypt
A 25 week gestation infant has been ventilated since birth and is
31.
now 9 days old. A chest X-ray is taken.
a. What does the X-ray show?
b. What is responsible for these changes?

a. i. The lungs are hyperinflated.


31.
ii. There is a nasogastric feeding tube in place.
iii. There is what is probably a UAC, with the tip at around T8.
iv. There is an endotracheal tube.
v. There is widespread patchy shadowing which appears consistent with
cystic changes throughout both lung fields.
b. These apparently cystic changes are compatible with chronic lung dis- ease.
Although the diagnosis of CLD (or BPD) is dependent on a pro- longed oxygen requirement
changes may be present well before then and can be seen histologically in infants who have
died only a few days after birth. The distribution and pattern of the changes is different from
that expected with pulmonary interstitial emphysema, which also gives widespread cystic
changes.
Dr.Wahid Helmi-Egypt
32. A 26 week gestation infant has received one dose of surfactant in the delivery
room and a further dose 12 hours later. A chest X-ray is taken at 24 hours of age.

a. What does the X-ray show?


b. What do you think has happened?
c. What will you do next?

32. a. i. There is an endotracheal tube.


ii. There is a nasogastric tube.
iii. There is opacification of the left lung, consistent with RDS.
iv. The right lung has cystic change throughout, consistent with pul- monary
interstitial emphysema.
v. There is a large bulla at the base of the right lung.
b. The presence of pulmonary interstitial emphysema in the right lung and what
appears to be quite severe RDS in the other lung suggests that the
endotracheal tube may have been in the right main bronchus at the time of
the first surfactant administration. As one lung is now well expanded and the
other solid, any further surfactant is likely to also go into the right lung.
c. Ventilatory management of this baby will be difficult.The left lung needs
aeration and thus recruitment of alveoli. The ideal way to do this is to use
higher ventilatory pressures to both recruit and retain alveolar expan- sion.
Unfortunately the right lung is showing signs of cystic change and over-
distension with a bulla in the right base, and high pressures will result in
further damage. High frequency may be useful and anecdotally it has been
suggested that paralysing the baby and ventilating with the PIE side down
may minimise damage to the PIE side and facilitate infla- tion of the
atelectatic side.
Dr.Wahid Helmi-Egypt
33. A baby is born at 36 weeks to a mother with insulin dependent diabetes.
He does not tolerate feeds and abdominal distension is noted. A chest and
abdominal X-ray is taken at 36 hours of age.
a. What does the X-ray show?
b. What do you think has happened?
c. What will you do next?

33. a. i. There is a nasogastric tube.


ii. There are loops of dilated bowel but no evidence of inflammatory
changes, thus making NEC unlikely.
iii. There is no air in the rectum.
b. Either structural or functional bowel obstruction is likely. There is noth-
ing on the X-ray that allows differentiation between possible causes.
Maternal diabetes is associated with a small left colon in the newborn
baby. This results in a functional obstruction that usually resolves with
time (as it did with this infant).
c. A contrast study is probably indicated. Further investigations will depend
on the result of this. The baby should not be fed, intravenous fluids should
be given and a nasogastric tube should be left on free drainage.
Dr.Wahid Helmi-Egypt
34. A term infant shows minimal respiratory effort and requires
ventilation. A chest X-ray is taken. On talking to the parents you notice
that the mother shows little expression in her facial movements.

a. Describe the X-ray?


b. What do you think may be the explanation for these appearances?
c. What will you do?
34. a. i. There is extremely poor lung inflation.
ii. The diaphragms are very high.
iii. The ribs are thin, particularly on the posterior aspect.
b. Myotonic dystrophy.
c. The baby will need ventilating. There is normally some improvement
over time, although it may be a long time before breathing is strong
enough to allow extubation.
Dr.Wahid Helmi-Egypt
35. A 25 week gestation infant has been ventilated since birth. Three doses of
surfactant have been given with little effect. A chest X-ray is taken at 48
hours of age.

What does the X-ray show?

35. a. i. There is an endotracheal tube in place.


ii. There is a line with a tip at approximately T11. It is not clear what
this line is.
iii. There is an endotracheal tube at an appropriate position.
iv. There is what appears to be a central venous line with the tip below
the sternal end of the left clavicle.
v. Both lungs are very poorly aerated.
vi. There is a fracture of the left arm. The bones do not appear particularly
osteopenic and this is presumably traumatic in origin.
Dr.Wahid Helmi-Egypt
36.A 28 week gestation infant has developed moderate RDS and is
stable on CPAP. Abdominal distension is noted and an abdominal
X-ray is requested. By mistake a chest X-ray is taken instead.

a. What does the X-ray show?


b. What action will you take?

a. i. There is a nasogastric tube with the end in the mid-thoracic


36.
oesophagus.
ii. The lungs have diffuse patchy opacification consistent with
chronic lung disease.
b. The nasogastric tube needs to be repositioned and as much gas
removed from the intestines as possible.
Dr.Wahid Helmi-Egypt
37. A 24 week infant requires ventilation from birth and is now 4 weeks old.
He is known to have duodenal atresia but the surgeons do not want to
operate until he is a lot bigger. He has been to theatre for a surgical
procedure. A chest X-ray is taken to check that the procedure has been
performed correctly. The appearance of the right lung is as it has been
since birth.

a. What does the X-ray show?


b. What was the surgical procedure?
c. What is the explanation for the appearances of the right lung?
37. a. i. There is an endotracheal tube and nasogastric tube.
ii. There is a fine-bore central venous catheter with the tip in the
tho- racic inlet.
iii. There is a large-bore central venous catheter on the right with the
tip at the junction of the SVC and right atrium.
iv. The right lung is opaque with no areas of aeration.
b. Surgical placement of a central venous catheter. The large-bore
catheter is a Broviac line.
c. There is aplasia/hypoplasia of the right lung. The fact that the lung
has always had this appearance and the infant is now 4 weeks old is
strongly against collapse or consolidation as the cause of this
appearance.
Dr.Wahid Helmi-Egypt
38. A 28 week gestation infant has been born and has needed relatively
little ventilatory support. Feeds are introduced on day 3 and
increased slowly. On day 5 he deteriorates and there is obvious
abdominal distension. An X-ray is obtained.

a. What does the X-ray show?


b. What do you think has happened?
c. What will you do next?
38. a. i. There is free gas above the liver in this lateral X-ray.
b. Gastrointestinal perforation.
c. Surgical referral. A drain may need to be inserted pending laparotomy.
Dr.Wahid Helmi-Egypt
39. A term infant is born in poor condition following an ante-partum
haemorrhage. The infant takes a few deep gasps and then stops breathing.
Heart rate is 57 bpm. Bag and mask ventilation is commenced. There is
poor chest movement and there is little response to ventilation.
Transillumination is performed and there is no difference between the two
sides. An endotracheal tube is inserted and ventilation does not improve.
A butterfly needle is inserted into the right chest. A few bubbles are seen
but this then stops. An X-ray is taken.
a. What does the X-ray show?
b. How do you explain the clinical story?
c. What will you do next?

39. a. i. There are bilateral pneumothoraces.


ii. There is an endotracheal tube in position.
b. Pneumothoraces may have been present at birth, following the gasping
respirations or following initial ventilation. Bilateral pneumothoraces may
be difficult to diagnose because all signs are equal on both sides. Although
a needle drain may allow re-inflation the lung will expand onto the needle
which can easily become blocked.
c. Immediate insertion of bilateral chest drains.
Dr.Wahid Helmi-Egypt
40. A 36 week gestation infant has been born and is in poor condition. She is
very growth retarded and there is a widespread petechial rash. An X-ray
is taken at 6 hours of age.

a. What does the X-ray show?


b. How might these appearances link with the clinical history?
c. What treatment would you consider?
40. a. i. There is an endotracheal tube, the tip of which is too high.
ii. There is a nasogastric tube.
iii. There is patchy opacification of both lungs.
iv. There is enlargement of both liver and spleen.
b. The combination of growth retardation, a petechial rash and hepato-
splenomegaly is highly suggestive of a congenital infection. Cyto-
megalovirus is the most likely candidate. A pneumonitis may develop
with CMV infection although it is rarely present at birth and most com-
monly associated with perinatally acquired infection.
c. i. Blood needs to be sent for CMV testing.
ii. Further investigations should look for other evidence of CMV dam-
age – an echocardiogram to look for congenital heart defects and
cerebral ultrasound, CT or MRI to look for intracranial calcification
and evidence of lissencephaly or polymicrogyria that may occur with
early infection.
iii. If the diagnosis is confirmed, treatment with anti-CMV chemother-
apy should be considered. There is some encouraging information
Dr.Wahid Helmi-Egypt
41. A 26 week gestation infant is 4 days old. A central venous line has been
inserted into the left long saphenous vein and shortly afterwards
abdominal distension is noted. An X-ray is performed.
a. List three major abnormalities in the intestines.
b. How is the position of the central line relevant?
c. What are the underlying diagnoses?

41. a. i. There is dilated bowel with thickened walls, mainly on the right side
of the abdomen.
ii. There are areas of bowel with intramural gas.
iii. The gut distribution is opposite to that expected. The stomach (see NGT
position) is on the right, liver on the left.
b. The central line is passing to the left of the spine.
c. The appearances suggest necrotising enterocolitis in a baby with situs
inversus. The appearances are typical of NEC. The distribution of the
abdominal contents and the left sided position of the inferior can only be
explained by situs inversus. The very small amount of heart visible appears
to be on the left but further investigation would be needed to exclude
dextrocardia.
Dr.Wahid Helmi-Egypt
42.A 26 week gestation infant has had severe RDS and has not
responded well to surfactant. Because of deteriorating gases at high
peak pressures high frequency oscillation is commenced. He initially
improves and his inspired oxygen falls from 100% to 45%. After 24
hours he starts to deteriorate again and inspired oxygen increases to
90%. A chest X-ray is taken at 4 hours of age.
a. What does the X-ray show?
b. What action will you take?

a. i. There is an endotracheal tube with a high tip.


42.
ii. There is a central line which is probably an umbilical arterial line with the
tip at T8.
iii. There is patchy opacification throughout both lungs.
iv. The lungs are hyperinflated with 11 posterior rib ends showing and with
flat diaphragms.
v. The heart appears moderately compressed.
b. Mean airway pressure should be reduced or ventilation should be changed to a
different modality.
43. A 27 week gestation infant has had moderate RDS and ventilation is
Dr.Wahid Helmi-Egypt
weaning fairly rapidly. Extubation to CPAP is being considered when she
deteriorates and abdominal distension is seen. NEC is suspected and a chest
and abdominal X-ray is requested.
a. What does the X-ray show?
b. What is the reason for her deterioration?
c. What action will you take?

43. a. i. There is a central venous catheter entering from the right arm.
ii. There is patchy shadowing of both lungs.
iii. There is a nasogastric tube, the end of which cannot be seen.
iv. There is gaseous distension of the stomach and intestines.
v. The endotracheal tube is not in the trachea – it is almost certainly in the
oesophagus.
b. Ventilation of the gastrointestinal tract.
c. Remove the ET tube and either re-intubate or try CPAP. Gas should be
sucked out of the gastrointestinal tract if possible and the nasogastric tube
should then be put on free drainage.
Dr.Wahid Helmi-Egypt
44. A 26 week infant is born in poor condition and needs high pressure
ventilation from the outset. The baby has caused considerable concern to
the fetal medicine abdominal X-ray is obtained shortly after birth.
a. What does the X-ray show?
b. What intra-uterine procedure was performed?
c. What do you think the underlying diagnosis is?

44. a. i. There is an endotracheal tube in the correct position.


ii. There is a central line which is probably an umbilical arterial cath-
eter with the tip at T11–12.
iii. The lungs are very small volume.
iv. There are pleural effusions, more marked on the right than the left.
v. There are two echodensities over the right lung field and one over
the left. Each has the appearance of a small cylinder.
b. In-utero drainage of pleural effusions using drains with pigtails on both
ends.The echodense bodies are markers – one on each end of the
drains.
c. Pleural effusions of this severity are uncommon. In the absence of
other signs of hydrops, pulmonary lymphangiectasia is a definite
possibility.
Dr.Wahid Helmi-Egypt

45. A 28 week gestation infant has mild RDS. There was signifi cant
growth retardation and Dopplers showed reversed end-diastolic fl ow.
The abdomen looks moderately distended and stage 1 NEC is
suspected. A decision has been taken not to feed for at least for 10
days. A chest X-ray is obtained.
a. What does the X-ray show?
b. What action will you take?

45. a. i. There is an endotracheal tube that is high.


ii. There is a nasogastric tube in position.
iii. The lungs show patchy shadowing in both fields.
iv. There is a central venous line which appears to have been inserted through
a scalp or neck vein.The end is resting within the right atrium.
b. The central venous line needs to be withdrawn to lie outside the heart.
Dr.Wahid Helmi-Egypt
46. In the same infant as in question 45 the procedure is repeated. A
further chest X-ray is taken.
a. What does the X-ray show?
b. What will you do next?

46. a.i. There is an endotracheal tube.


ii. There is a nasogastric tube.
iii. There is a central venous line which appears to have been inserted
through a vein in the right arm. It has passed in a cephalad direction.
The tip is not visible but probably lies in either the neck or the head.
b. The line needs to be withdrawn until the tip lies in the region of the thoracic inlet.
Dr.Wahid Helmi-Egypt
47. A 26 week gestation infant has required ventilation from birth. A chest and
abdominal X-ray is taken at 4 hours of age.
a. What does the X-ray show?
b. What action will you take?

a. i. There is an endotracheal tube that is very high.


47.
ii. There is a nasogastric tube with the tip in the stomach.
iii. There is an umbilical arterial line with the tip at T5.
iv. There is an umbilical venous catheter which deviates to the right
and appears to lie within the liver.
b. Umbilical venous and arterial lines and the endotracheal tube need to be
repositioned.
Dr.Wahid Helmi-Egypt 48. A 25 week gestation infant has required ventilation from birth and
has needed high pressures. She is now 2 weeks old and there has
been a sudden deterioration. The cause is not obvious on
examination and a chest and abdominal X-ray is requested.
a. What does the X-ray show?
b. Why was this not detected on examination?
c. What action will you take?

48. a. i. There is an endotracheal tube that is slightly high.


ii. There is a central venous line with the tip at the thoracic inlet.
iii. There is an umbilical venous line with a very low tip.
iv. There is a large amount of oedema, very obvious on the sides of the
chest.
v. The lungs show dense shadowing and there may be some parenchy- mal
cysts compatible with pulmonary interstitial emphysema.
vi. There is a left tension pneumothorax with mediastinal shift to the right.
b. Although a pneumothorax of this size should be easily detected by trans-
illumination the oedema around the chest will have resulted in a diffuse
brightness that may well have concealed the pneumothorax.
c. A chest drain must be inserted immediately.
Dr.Wahid Helmi-Egypt
49. A 26 week infant has been unwell since birth. At 6 days of age abdominal
distension is noted. Three abdominal X-rays are obtained at 24-hour
intervals.

a. What do the X-rays show?


b. What is the diagnosis?
c. What actions should be taken?

49. a. There are fixed loops of dilated bowel that do not change from one
X-ray to another.
b. This unchanging appearance is highly suggestive of dead bowel. NEC
is the most likely underlying cause.
c. Surgical referral is needed. A laparotomy will be required at some
point and an abdominal drain may be required as an interim measure.
Dr.Wahid Helmi-Egypt
50.A 29 week gestation infant deteriorates rapidly after birth and
requires ventilation. A chest and abdominal X-ray is taken at 24
hours of age.
a. What does the X-ray show?
b. What is the explanation?
c. What will you do next?

50. a. i. There is an endotracheal tube that is high.


ii. There is a central venous line entering from the right arm that is slightly too
long.
iii. There is what is probably an umbilical arterial line, with the tip at T8/9.
iv. The lung fields are poorly aerated.
v. There is a nasogastric tube which is passing down into the right lung.
vi. There is gas in the stomach.
b. There is a tracheo-oesophageal fistula or cleft. Although it is possible to pass a
nasogastric tube into the trachea it should not be possible to do so when an
endotracheal tube is in place as is the case here. The nasogastric tube is gaining
access to the trachea through a fistula or cleft lower down.
c. The nasogastric tube needs to be removed.The endotracheal tube should be
inserted further to see if it can block a fistula, otherwise ventilation will be very
difficult. A surgical opinion should be sought.
Dr.Wahid Helmi-Egypt

٥٢
Neonatoloy Revision

Dr.Wahid Helmi
Egypt
Consultant Pediatrician
Consultant Dr.Wahid Helmi Domiate Egypt ( Neonatoloy Revision First note)

Neonatology revision First part


Dr-Wahid Helmi
Consultant Pediatrician
5-1. Which of the following statements regarding vitamin K administration in the
newborn period is correct?
a. Two 1-mg doses of intramuscular vitamin K are required in the first 48 hours of age.
b. Vitamin K cannot be given safely to neonates as an oral preparation.
c. Neonates receiving a 2-mg dose of oral vitamin K require no additional dosing in
infancy.
d. Oral vitamin K administration dosing has been standardized with internationally
accepted guidelines.
e. Parental nonadherence with additional doses of oral vitamin K is a major factor in
oral vitamin K failures.

5-2. A 4130-g male infant is born via vaginal delivery after 40 6/7 weeks’ gestation. The
mother recently migrated from Ecuador and received no prenatal care. In the
delivery room, the baby developed mild tachypnea and intercostal retractions. Pulse
oximetry from the infant’s left foot registers 95% saturation in room air. Physical
examination findings include a flat abdomen and audible bowel sounds over the left
side of the infant’s chest. Clear breath sounds are noted over the right side of the
chest. Heart sounds are best auscultated at the left sternal border.
The most appropriate initial step in management of this neonate is:
a. Needle decompression of the right hemithorax
b. Positive pressure ventilation using a bag and mask
c. Decompression of the stomach with a nasogastric tube
d. Positive pressure ventilation with a T-piece resuscitator
e. Transillumination of the right hemithorax
Consultant Dr.Wahid Helmi Domiate Egypt ( Neonatoloy Revision First note)

5-3. A 4550-g male neonate is delivered vaginally after a 37 6/7 weeks’ gestation
complicated by gestational diabetes. One application of a vacuum extractor to the
neonate’s head was required to facilitate delivery. On examination at 12 hours of
age, the baby has a well- circumscribed, fluctuant mass over the parietal skull that
does not cross suture lines. The overlying skin is erythematous but intact. He is
active and alert, and the rest of his examination is unremarkable. His hematocrit
(measured from a capillary sample) is 45%.
Which of the following steps is next indicated in the management of this
neonate?
a. Administration of intravenous ampicillin and cefotaxime
b. Monitoring serum and/or transcutaneous bilirubin levels
c. Careful aspiration of the mass with a tuberculin syringe
d. Application of topical bacitracin zinc ointment to the mass
e. Transfusion of packed red blood cells

5-4. An obstetric colleague asks you to provide prenatal counseling for a new patient
to his practice. The woman, currently at 34 3/7 weeks’ gestation, transferred her
medical care from an out-of-state obstetrical
group. Her past medical history is notable for poorly controlled type 1 diabetes.
The mother asks about fetal growth and neonatal body habitus in pregnancies
complicated by diabetes.
Which of the following statements regarding growth in fetuses and infants of
diabetic mothers (IDMs) is true?
a. Growth hormone (GH) is the primary anabolic growth factor in the fetus.
b. If a mother has advanced diabetes-related vascular disease (ie, diabetic retinopathy),
fetal growth is not affected.
c. IDMs have excess fat distribution in the extremities
d. Hypertrophic cardiomyopathy (HCM) in IDMs may result in ventricular outflow tract
obstruction.
e. IDMs have equally increased weight, length, and head circumference percentiles at
birth.

Consultant Dr.Wahid Helmi Domiate Egypt


Consultant Dr.Wahid Helmi Domiate Egypt ( Neonatoloy Revision First note)

5-5. A 3840-g female infant is delivered by cesarean section after a 39 5/7 weeks’
gestation. In the delivery room, she develops tachypnea and cyanosis when quiet.
Physical examination reveals clear bilateral breath sounds when the infant is crying.
However, the nurse is unable to pass a 5 French suction catheter through the nares
into the posterior pharynx.
The most appropriate initial step in management of this neonate is:
a. Oral airway placement
b. Intranasal dexamethasone instillation
c. Nasal cannula oxygen administration
d. Supine positioning
e. Nasal continuous positive airway pressure (CPAP)

5-6. A 9-week-old female infant is seen for a follow-up appointment with her
pediatrician. The baby weighed 4330 g at birth, and was delivered vaginally (from
the breech presentation) at 40 weeks’ gestation. In the nursery, hip examination
was unremarkable bilaterally on Ortolani and Barlow maneuvers. Physical
examination at this visit shows normal movement of the lower extremities.
The appropriate screening strategy for developmental dysplasia of the hip (DDH) in
this infant at this time is:
a. Repeat Ortolani and Barlow maneuvers only
b. Measurement and comparison of the lower extremity length (from the anterior superior
iliac crest to the heel)
c. Ultrasonography of both hips
d. Radiographs of both hips
e. Magnetic resonance imaging of both hips

5-7. A 1200-g male infant is delivered vaginally after a 28 3/7 weeks’ gestation complicated by preterm
rupture of membranes and preterm labor. On arrival to the delivery room table, the neonate receives
positive pressure ventilation (PPV) after the neonatal nurse dries the skin and suctions the oropharynx.
After 3 minutes of PPV, the nurse notes the baby has a heart rate of 110 beats/min with cyanosis and
intercostal retractions. The pediatric resident decides to intubate the infant. Brief visual inspection of
the mouth, mandible, and tongue reveals no abnormalities.
Which of the following combinations of laryngoscope blade and endotracheal tube is indicated for
intubating this patient?
a. Number 0 blade, 3.0 mm endotracheal tube
b. Number 00 blade, 4.0 mm endotracheal tube
c. Number 1 blade, 3.5 mm endotracheal tube
d. Number 00 blade, 3.5 mm endotracheal tube
e. Number 0 blade, 4.0 mm endotracheal tube
Consultant Dr.Wahid Helmi Domiate Egypt ( Neonatoloy Revision First note)

5-8. An infant born at 26 weeks’ gestation with a birth weight of 900 g is now 42
weeks postmenstrual age (PMA). He had respiratory distress syndrome (RDS) in
his early neonatal intensive care unit (NICU) course and continues to have a
supplemental oxygen requirement.
Which of the following characteristics of his past or current neonatal course would
support the diagnosis of bronchopulmonary dysplasia (BPD) in this infant?
a. He was born due to maternal chorioamnionitis and was mechanically ventilated for
21 days.
b. He was mechanically ventilated for 2 weeks, and now requires an FiO 2 of 0.5 via
CPAP (5 cm H2O) to keep his saturations 97%.
c. He requires 2 liters per minute (LPM) of nasal cannula oxygen with an FiO 2 of
0.6 to maintain oxygen saturations of 98% to 100%.
d. He developed Bacteroides fragilis sepsis at 2 weeks of age and required mechanical
ventilation until 34 weeks PMA.
e. He requires 1 LPM nasal cannula oxygen with an FiO2 of 0.45 to maintain oxygen
saturations of 91% to 93%.

5-9. A 600-g male infant was born at 24 1/7 weeks’ gestation to a 26-year-old, gravida
2, para 0020 woman with a history of cervical incompetence. His hospital course
was complicated by cholestatic jaundice, feeding intolerance, and chronic
respiratory failure requiring intubation from birth until his death from E. coli
pneumonia and bacteremia at 105 days of age. The family has requested a complete
autopsy to better understand his lung disease.
Which of the following features would you expect to see on pathologic examination
of this infant’s lungs?
a. Lung development consistent with his postmenstrual age
b. Uniform lung tissue with no evidence of emphysematous changes
c. Localized elastin fibers at the branch points of the alveoli
d. Increased alveolar diameter with decreased number of alveoli
e. Normal pulmonary vasculature with evidence of pulmonary edema

Consultant Dr.Wahid Helmi Domiate Egypt


Consultant Dr.Wahid Helmi Domiate Egypt ( Neonatoloy Revision First note)

5-10. A 30-year-old, gravida 1 woman presents to the Labor and Delivery unit at 26 6/7
weeks’ gestation. She reports uterine contractions every 5 minutes and leakage
of clear amniotic fluid 1 hour prior to her arrival. Physical examination confirms
rupture of her amniotic membranes as well as 9.5 cm of cervical dilation.
She is afebrile and has a normal heart rate and blood pressure. The obstetricians
page the neonatal intensive care unit (NICU) team to the Labor and Delivery unit,
anticipating an imminent delivery.
After delivery, which of the following management strategies in the NICU would
decrease the risk of this infant developing bronchopulmonary dysplasia (BPD)?
a. Initiation of parenteral erythromycin within 24 hours of birth
b. Application of positive pressure ventilation in the delivery room, using high
inspiratory pressures of 40 to 60 cm water after the first 5 infant breaths
c. Initiation of high-dose corticosteroids if the neonate requires more than 7 days of
mechanical ventilation.
d. Supplementation with enteral ascorbic acid (vitamin C) when the infant is on full-
volume gastric feeds
e. Use of supplemental oxygen to keep saturations within gestational-age-specific
target parameters while in the NICU

5-11. A 26-year–old, gravida 3 female presents to your office at 25 4/7 weeks’ gestation for
a prenatal consult. She tells you that the fetus, a female, has a “rectal mass.” The
prenatal ultrasound report notes the mass to be most consistent with a
sacrococcygeal teratoma (SCT). The mother asks about the possibility of resecting
the mass prior to delivery.
Which of the following statements regarding fetal surgery for a SCT is correct?
a. The primary goal of fetal surgery is prevention of fetal hydrops.
b. The removal of pelvic components of the teratoma is accomplished prenatally.
c. The development of hydrops fetalis is an insignificant concern in this case.
d. The anorectal sphincter complex should be removed during fetal surgery.
e. The surgical intervention should occur only after complete hydrops fetalis occurs.
Consultant Dr.Wahid Helmi Domiate Egypt ( Neonatoloy Revision First note)

5-12. Which of the following pathophysiologic mechanisms is associated with neonatal


hypoxic-ischemic brain injury?
a. Loss of high-energy phosphorylated compounds
b. Reduction in cellular oxidative stress
c. Accelerated reuptake of glutamate at the synaptic cleft
d. Decrease in intracellular calcium levels
e. Sustained inhibition of neuronal apoptosis

5-13. A 3210-g female neonate was delivered by emergent cesarean section due to a
maternal seizure and cardiorespiratory arrest after a 37 6/7 weeks’ gestation. The
infant required mechanical ventilation and chest compressions in the delivery
room. Her Apgars were 1, 2, and 4 at 1, 5, and 10 minutes of age, respectively. Her
initial arterial blood gas at 45 minutes of age showed a pH of 6.78 and a base
deficit of 20mmol/L.
On admission to the NICU, the radiant warmer was not turned on in anticipation
of hypothermia therapy to prevent hypoxic ischemic encephalopathy (HIE). Her
rectal temperature was 35.1°C at 60 minutes of age.
Which of the following statements regarding therapeutic hypothermia in this
infant is correct?
a. If the infant had severe abnormalities on amplitude- integrated
electroencephalography (aEEG) on NICU admission, selective head cooling would
provide the most neuroprotection.
b. The minimum length of therapeutic hypothermia for prevention of HIE in this infant is
12 hours.
c. Therapeutic hypothermia would accelerate free radical production in this patient.
d. Based on clinical trial data, whole body hypothermia would reduce the risk of death or
moderate to severe disability in this infant at 12 to 18 months of age.
e. Hypothermia will preserve executive functioning and psychosocial development
in this infant at 15 years of age.

5-14. Which of the following statements regarding neonatal venous strokes is correct?
a. Based on randomized clinical trials, low-molecular- weight heparin is the
anticoagulant of choice for treatment of venous thromboses.
b. Venous infarctions may be associated with neonatal group B Streptococcus sepsis.
c. Venous infarctions are 3 times as likely as arterial strokes in neonates.
d. Hemorrhagic venous infarcts in neonates usually spare the deep gray matter.
e. Trials of supratherapeutic doses of erythropoietin in humans have demonstrated
benefit in treating neonatal strokes.
Consultant Dr.Wahid Helmi Domiate Egypt ( Neonatoloy Revision First note)

5-15. A 4-day-old male infant born at 37 2/7 weeks’ gestation presents for his first well-
child care visit. He weighed 4000 g at birth and his length was 55.5 cm (both greater
than the 90th percentile for gestational age). His newborn course was notable for
episodes of “low sugar,” per the mother; the nursery pediatrician suggested the
mother feed the infant every 2 hours because he was a “large baby.” The mother had
a normal oral glucose tolerance test during pregnancy.On physical examination, the
infant has a prominent tongue and bilateral ear creases. Abdominal examination is
notable for slight separation of the rectus abdominis muscles but without herniation
of intestinal contents. He is alert and interactive. A serum glucose taken in the office
is 39 mg/dL. He takes 30 cm3 of formula in the office in 15 minutes without
respiratory distress or diaphoresis.The etiology of hypoglycemia in this infant is
related to:
a. Congestive heart failure
b. Impaired fatty acid oxidation
c. Pancreatic hypertrophy
d. Swallowing dysfunction
e. Impaired glycogenolysis

5-16. You meet a 23-year-old primigravida female student, who presents for a routine
fetal ultrasound at 20 2/7 weeks’ gestation. She is a postbaccalaureate student who
will be applying to graduate school. She has an interest in fetal circulation,
particularly the oxygen concentration in the heart and blood vessels. She asks you,
“Which of the following sites has the highest oxygen concentration in the fetus?”
What is the best answer?
a. Inferior vena cava
b. Umbilical vein
c. Umbilical artery
d. Right ventricle
e. Ductus arteriosus
5-17. A 1200-g white male infant is born (at 29 1/7 weeks’ gestation) to a 25-year-old, gravida 2, para
1001 woman who presented to the hospital in preterm labor. The infant has an uncomplicated
delivery and requires only routine drying, warming, and tactile stimulation. He is admitted to the
NICU secondary to prematurity. He requires no respiratory support initially, but at 3 hours of life,
he is started on nasal continuous positive airway pressure (CPAP) due to tachypnea, grunting,
nasal flaring, and sternal retractions.
What is the most likely cause of his symptoms?
a. Persistent pulmonary hypertension of the newborn
b. Transient tachypnea of the newborn
c. Respiratory distress syndrome (RDS)
d. Meconium aspiration syndrome
e. Congenital heart disease
Consultant Dr.Wahid Helmi Domiate Egypt ( Neonatoloy Revision First note)

5-18. A 2100-g infant was born at 33 1/7 weeks’ gestation by repeat cesarean section to a
28-year-old primigravida woman. At delivery, the male infant was noted to
have respiratory distress characterized by grunting, retractions, and hypoxia. The
infant is intubated and given a dose of surfactant. The father, a graduate student
in biochemistry, asks you about the properties of surfactant and its production
and role in the normal lung.
Which of the following statements about surfactant is correct?
a. Surfactant lipids and proteins are synthesized in the alveolar epithelial type II cells.
b. Phosphatidylglycerol (PG) is the major component responsible for decreasing
alveolar surface tension.
c. Congenital surfactant protein A (SP-A) deficiency causes fatal respiratory distress in
affected full-term infants.
d. Surfactant synthesis and storage begins at 28 to 38 weeks’ gestation.
e. Antenatal testing of amniotic fluid after 34 weeks’ gestation will show a lecithin–
sphingomyelin (L/S) ratio of 1:1 in infants with mature lungs.
5-19. A 2500-g female infant was born at 34 3/7 weeks’ gestation to a 30-year-old,
gravida 3, para 1102 woman. Shortly after birth, the infant developed tachypnea
(respiratory rate of 60 breaths/min), and increased work in breathing (manifested
by intercostal retractions).She is brought to the special care nursery for evaluation
and management of her respiratory distress. Physical examination shows coarse
bilateral breath sounds, no cardiac murmur, and strong, equal peripheral pulses.
She is alert and active.
What is the most appropriate initial management strategy for this infant?
a. Intubate the infant; immediately begin high- frequency oscillatory ventilation
(HFOV).
b. Begin continuous positive airway pressure (CPAP) with supplemental oxygen and
nitric oxide (iNO).
c. Intubate the infant and begin intermittent mechanical ventilation (IMV).
d. Support the infant with CPAP and supplemental oxygen to maintain appropriate
oxygen saturations for gestational age.
e. Place the infant under a hood and titrate nitrogen (N 2) and oxygen (O2) into the
hood to generate an FiO2 of 0.16.
Consultant Dr.Wahid Helmi Domiate Egypt ( Neonatoloy Revision First note)
5-20. A 36 2/7–week, 1800-g male infant is born to a32-year-old, gravida 1 woman with
pregnancy-induced hypertension. At 30 seconds of age, he is spontaneously crying on the
delivery room table, and has a heart rate of 110 beats/min.
Which of the following steps is most important in the
initial management of this infant?
a. Administer positive pressure ventilation (PPV) via a self-inflating bag.
b. Provide continuous positive airway pressure (CPAP) using a T-piece resuscitator.
c. Administer a 10 cm3/kg intravenous bolus of normal saline via an umbilical venous
catheter.
d. Dry the infant with warm towels and suction his mouth and nose.
e. Deliver a 2 cm3/kg intravenous bolus of 10% dextrose via an umbilical venous
catheter.

5-21. A 2200-g infant was born at 39 6/7 weeks’ gestation (via spontaneous vaginal
delivery) to a 34-year-old woman with a history of chronic hypertension. In the
nursery at 2 hours of age, the baby’s blood glucose was 24 mg/dL before feeding.
After taking 20 mL of age- appropriate formula, the blood glucose increased to 50
mg/dL.
What is the most likely cause for the initial hypoglycemia in this infant?
a. Increased gluconeogenesis
b. Decreased neonatal insulin sensitivity
c. Increased placental transport of maternal insulin in utero
d. Increased serum epinephrine levels
e. Decreased glycogen stores

5-22. A 1500-g female infant is born at 34 weeks’ gestation to a 40-year-old, gravida 4,


para 3003 woman by cesarean section due to reverse end-diastolic flow noted on
fetal sonography. The pregnancy was complicated by intrauterine growth
restriction (IUGR) and pregnancy- induced hypertension. Due to respiratory
distress in the delivery room, the baby is transferred to the NICU for intravenous
nutrition and respiratory support.
The infant is at high risk for developing which of the following?
a. Low plasma concentrations of fatty acid and triglycerides if she receives
intravenous lipids
b. Low basal oxygen consumption and total energy expenditure (relative to an
appropriate-for- gestational-age [AGA] infant)
c. Increased intestinal protein losses with enteral nutrition
d. Higher serum insulin levels in the neonatal period
e. High immunoglobulin levels during infancy
Consultant Dr.Wahid Helmi Domiate Egypt ( Neonatoloy Revision First note)
5-23. Which of the following statements regarding neonatal glucose metabolism is
correct?
a.Postnatal increases in catecholamines and glucagon modulate neonatal glucose
concentrations shortly after birth.
b.Neonatal hepatic glycogen content may remain elevated up to 72 hours after birth
without exogenous glucose delivery.
c. Neonatal insulin levels surge immediately after delivery.
d.Basal glucose production in a neonate is approximately 10% to 15% the basal rate of
an adult.
e.Inhibition of long-chain fatty acid oxidation increases circulating glucose
concentrations.
5-24. A 4050-g male neonate was delivered by emergent cesarean section due to fetal
bradycardia after a 39 2/7 weeks’ gestation. He required mechanical ventilation,
chest compressions, and intravenous
epinephrine in the delivery room. The baby’s Apgars were 1, 1, and 6 at 1, 5, and 10
minutes of age, respectively. His initial arterial blood gas at 45 minutes of age showed
a pH of 6.97 and a base deficit of 17 mmol/L. He developed seizures at 30 minutes of
age, and was placed on whole-body hypothermia (for prevention of hypoxic ischemic
encephalopathy) at 75 minutes of age. He also was placed on a high- frequency
oscillator and started on maintenance intravenous fluids of 85 cm 3/kg/day containing
10% dextrose via an umbilical venous catheter.At 24 hours of age, the baby’s total
urine output for the day is 0.2 cm3/kg/h. He has 2-second capillary refill and strong
peripheral pulses. A set of serum chemistries at that time demonstrates the following:

Sodium 122 mEq/L


Potassium 6.2 mEq/L
Chloride 89 mEq/L
CO2 14 mEq/L
Blood urea nitrogen 45 mg/dL
(BUN)
Creatinine 2.6 mg/dL
Calcium 6.9 mg/dL
Glucose 67 mg/dL
Which of the following steps is appropriate in the management of this infant?
a. Administration of furosemide, 0.5 mg/kg intravenous
b. Administration of normal saline, 10 cm3/kg intravenous, over 30 minutes
c. Administration of bumetanide, 0.1 mg/kg intravenous
d. Reduction of intravenous fluids to 30 cm3/kg/day
e. Increase of dextrose in the intravenous fluids to 15%
Consultant Dr.Wahid Helmi Domiate Egypt

5-25. A 3700-g male infant, born vaginally at 39 2/7 weeks’ gestation, required intubation in the
delivery room for apnea. The mother is a 31-year-old, gravida
3, para 2002 woman with no significant medical history. She received routine prenatal care during
this uncomplicated pregnancy. She developed more frequent uterine contractions at home (4 hours
prior to birth) and, subsequently, her amniotic membranes ruptured(1 hour prior to birth). On
arrival to Labor and Delivery, her cervix was fully dilated, and she delivered the baby shortly
thereafter.
The pediatric resuscitation team arrived at 3 minutes of life to find a limp, cyanotic infant
under the radiant warmer receiving stimulation and blow-by oxygen.The baby’s heart rate was
greater than 100 beats/min, but he demonstrated little respiratory effort. He was intubated
easily by the pediatric intern and brought to the NICU. Apgars were 4 and 4 (2 points each for
color and heart rate) at 5 and 10 minutes, respectively. Cord blood was not sent for blood gas
measurement, but the infant’s initial arterial blood gas in the NICU at 30 minutes of age
revealed a pH of 6.98 and base deficit of 14 mmol/L.
At 45 minutes of age, he continues to be hypotonic and apneic (despite stimulation from NICU
caregivers), with pupils that are nonreactive to light.
Which of the following statements is correct, and should be considered in the management of
this infant?
a. There is no report of a prenatal or antenatal event that would cause hypoxia–ischemia, making this
infant’s presentation most consistent with an inborn error of metabolism.
b. Administration of prophylactic phenobarbital is indicated for seizure prophylaxis in this infant for a
minimum of 72 hours after birth.
c. If the neurologic examination does not improve, this infant will be a candidate for whole-body
hypothermia, which should begin at 12 hours of life.
d. Results of MRI imaging and electroencephalogram will be useful for discussing long-term
prognosis.
e. Tracking degree of clinical encephalopathy is useful for predicting duration of hospitalization, but
does not contribute additional information when predicting later prognosis.
5-26. A 34-year-old, gravida 1 woman presents at 18 weeks for a level 2 screening ultrasound. The
fetus is noted to have a hypoplastic nasal bone, increased nuchal fold, and short humeri and
femora.What profile of -fetoprotein (-FP), human chorionic gonadotropin (hCG), and
unconjugated estriol (uE) would have been most likely seen on the woman’s triple screen prior
to the ultrasound?
a. Low -FP, low hCG, low uE
b. Low -FP, high hCG, low uE
c. Low -FP, low hCG, high uE
d. High -FP, high hCG, high uE
e. High -FP, high hCG, low uE
Consultant Dr.Wahid Helmi Domiate Egypt ( Neonatoloy Revision First note)

5-27. A 25-year-old, gravida 2, para 1001 woman presents at 38 6/7 weeks’ gestation in
labor. She is placed on a tocodynamometer. The following tracing is obtained:
Which of the following is the most likely cause of the above findings?
a.Low fetal oxygen tension
b. Intermittent umbilical cord compression
c.Fetal hiccups (singultus)
d.Fetal head compression
e.Normal fetal movement

Fetal heart rate

Uterine
contractio
n pattern

(Reproduced, with permission, from Parer JT. Fetal heart rate. In:
Creasy R, Resnick R, eds. Maternal- Fetal Medicine: Principles
and Practice. 3rd ed. Philadelphia: Saunders; 1994.)

5-28. A 29-year-old, gravida 4, para 1203 woman presents to her obstetrician for a routine
appointment at 27 6/7 weeks’ gestation. Her urine dipstick demonstrates 4 protein, and her
blood pressure is 170/100 mm Hg. Her baseline (prepregnancy) blood pressure is 116/70 mm Hg,
and her blood pressure at 24 2/7 weeks’ gestation was 140/88 mm Hg. She denies headache, visual
changes, or abdominal pain, and has no facial or extremity edema on physical examination. She
has normal urine output and a serum creatinine of 1.1 mg/dL. Her obstetrician admits his
patients to a tertiary care obstetrical center.
Which is the most appropriate initial step in management of this patient?
a. Admission to Labor and Delivery for an emergent cesarean section delivery
b. Admission to Labor and Delivery for hemodialysis and delivery within the next 48 hours
c. Admission to Labor and Delivery and intravenous magnesium sulfate administration only
d. Admission to Labor and Delivery for intravenous magnesium sulfate and corticosteroid administration
e. Admission to Labor and Delivery for corticosteroid administration therapy only
Consultant Dr.Wahid Helmi Domiate Egypt ( Neonatoloy Revision First note)

5-29. A 10-day-old male infant born at 28 1/7 weeks’ gestation develops hypotension,
tachypnea, and an increasing number of desaturation and bradycardia episodes. The
infant was born vaginally after the mother presented to the hospital with a 4-day
history of ruptured amniotic membranes. She received a course of antenatal
betamethasone. He was intubated in the delivery room, given 3 doses of surfactant
over his initial NICU course for respiratory distress syndrome, and extubated at 4
days of age to CPAP (FiO2 0.3). The infant also received 48 hours of antibiotics
(beginning at birth) for presumed sepsis. Enteral feeds were initiated at 5 days of
age. Physical examination is notable for a holosystolic murmur. An echocardiogram
demonstrates a large patent ductus arteriosus (PDA) with left-to-right flow. His
complete blood count shows a white blood cell count of 4000/mm3 with 20% band
forms.The most likely reason for presentation of this symptomatic PDA at this time
is:
a. Extubation on day 4, which decreased pulmonary vascular resistance.
b. Failure to receive indomethacin prophylaxis for intraventricular hemorrhage in the
first 48 hours of life.
c. Inadequate antibiotic treatment for presumed sepsis, resulting in prostaglandin
release.
d. Failure of the mother to receive antenatal steroids due to precipitous delivery.
e. Advances in enteral feeding caused shunting to mesenteric circulation.

5-30. How many grams per deciliter of hemoglobin is deoxygenated when cyanosis
becomes apparent in a neonate?
a. 0.5 to 1.0 b. 3 to 5 c. 9 to 11 d. 15 to 20 e. 25 to 30

5-31. A 27-year-old woman presented to the emergency department of a local


community hospital in active labor. She has had no prenatal care and does not
remember her last menstrual period. She was transferred to the Labor and
Delivery suite, where a bedside ultrasound dated the fetus at approximately 40
weeks’ gestation. Three hours later, the woman vaginally delivered a 4700-g male
infant. Physical examination of the baby by the pediatric nurse practitioner at 15
minutes of age shows unlabored respirations but “dusky” lips and tongue. He is
given blow-by oxygen (FiO2 1.0), but his oxygen saturations never increase above
85%.
What is the most appropriate next step in management of this neonate?
a. Intubate the infant and ventilate him with an anesthesia bag.
b. Administer a bolus of 10% dextrose intravenously.
c. Admit infant to full-term nursery.
d. Obtain a computerized tomogram (CT) of the chest.
Consultant Dr.Wahid Helmi Domiate Egypt ( Neonatoloy Revision First note)
5-32. A 3800-g male infant was born at 41 3/7 weeks’ gestation to a 26-year-old woman.
She had unremarkable cultures and serologies except for a positive vaginal group
B Streptococcus culture (at 36 3/7 weeks’ gestation), for which she received
intrapartum antibiotics. Amniotic membranes were ruptured at delivery, and the
amniotic fluid was noted to be meconium stained. At birth, the infant was initially
apneic and hypotonic with a heart rate
of 90 beats/min. He was intubated easily on the first attempt, and a moderate
amount of meconium was aspirated from the trachea. He began to breathe when the
endotracheal tube was removed, but subsequently developed tachypnea and
intercostal retractions. He was admitted to the NICU and placed on supplemental
oxygen (FiO2 1.0) via an oxygen hood.
Which of the following conditions is the most likely
cause of the baby’s respiratory distress?
a. Mechanical obstruction of the airways
b. Bacterial pneumonia
c. Decreased surfactant production
d. Pulmonary vasodilation
e. Tracheal edema
5-33. A 4500-g female infant was born at 37 6/7 weeks’ gestation to a 30-year-old
woman with poorly controlled gestational diabetes. The delivery occurred
vaginally (with forceps assistance) and was complicated by shoulder dystocia.
Three hours after birth, the infant develops respiratory distress and needs
supplemental oxygen (FiO2 0.35) to keep her saturations greater than 95%.
Physical examination of the infant is notable for shallow respirations but clear
bilateral breath sounds. The baby has limited movement of the left arm on
elicitation of the Moro reflex. The intern orders a chest radiograph.
Which of the following findings is most likely to be seen on the radiograph?
a.Herniation of the stomach and small bowel into the left hemithorax
b.A decrease in pulmonary vascular markings bilaterally
c. Elevation of the left hemidiaphragm by 3 intercostal spaces (relative to the right
hemidiaphragm)
d.A left pleural fluid density compressing the lung
e.Multiple cystic lesions in the upper lobe of the left lung
Consultant Dr.Wahid Helmi Domiate Egypt ( Neonatoloy Revision First note)

5-34. A 2700-g female infant is admitted to the NICU after delivery by cesarean
section at 36 0/7 weeks’gestation. The mother is a 25-year-old, gravida 1 who
developed severe preeclampsia (after an uncomplicated pregnancy) and was
treated with a magnesium sulfate drip.Which of the following findings on initial
physical examination is normal given the clinical scenario?
a. Bursts of 8 to 10 sucks on a pacifier within 5 seconds, followed by pauses lasting 5 to
10 seconds
b. Visual fixation on a bull’s eye with sustained tracking
c. Strong flexor tone in the upper extremities and semiflexion in the lower
extremities
d. Absence of the Moro and palmar grasp reflexes
e. Movement of the right elbow to the left shoulder when the right arm is pulled across
the chest to the left
Consultant Dr.Wahid Helmi Domiate Egypt ( Neonatoloy Revision First note)

5-35. The Pediatrics team was called to a delivery by the Obstetrics team due to fetal
bradycardia. On arrival to the delivery room, the Labor and Delivery nurse told the
pediatricians that the mother was a 27-year-old, gravida 4, para 2103 woman at 39
0/7 weeks’ gestation. The mother had regular prenatal care, and her labor was
progressing normally. Rupture of membranes occurred 5 hours ago (clear amniotic
fluid).
At delivery, the baby was cyanotic, hypotonic, and apneic, but had an appropriate
response to stimulation and positive pressure ventilation delivered by the pediatric
respiratory therapist. The pediatric resident performed the initial newborn
examination 30 minutes after birth.
Which of the following findings on physical examination of this baby would raise
suspicion for central nervous system injury?
a. An intermittent disconjugate gaze when he is not fixating or tracking an object
b. Unsustained clonus when his ankle jerk reflex is elicited
c. Changes in the upper and lower extremity tones with rotation of his head from left to
right
d. Truncal extensor tone that exceeds his flexor tone
e. Rapid grasp when the palm of his hand is pressed

5-36. A 2700-g female infant was born vaginally (with vacuum assistance) after induction of labor at
36 6/7 weeks’ gestation (due to maternal preeclampsia). Apgar scores were 8 (1 color, 1 tone)
and 9 (1 color) at 1 and 5 minutes, respectively. The baby was permitted to stay with mother
and receive couplet care. While holding her at 48 hours of age, the mother felt the baby moving
but “not breathing” and called the postpartum nurse. The baby recovered quickly but was
transferred from couplet care to the NICU for monitoring. During the workup, a noncontrast
CT scan of the brain is performed. Before you can review the CT results, the radiology
technologist mentions to the baby’s nurse that the scan “… looks like a stroke.”
Which of the following statements regarding the brain injury in this infant is correct?
a. The timing of the seizure indicates that the insult occurred after delivery, and mother should be
questioned about having dropped the baby.
b. The timing of the seizure indicates that the insult
c.The stroke is most likely venous in origin, and evaluation for a hypercoagulable state is a high
priority.
d. Anticoagulation is effective therapy for neonatal thrombotic stroke regardless of the origin
(arterial vs venous).
e. This baby’s perinatal risk factors for stroke include maternal preeclampsia and vacuum-assisted
delivery.
Consultant Dr.Wahid Helmi Domiate Egypt ( Neonatoloy Revision First note)

5-37. A 2600-g male infant, born to a 16-year-old, gravida


1 female who did not receive prenatal care, was admitted to the neonatal intensive
care unit with mild tachypnea. At 3 days of age, his respiratory distress has resolved,
and a developmental pediatrician is asked to help determine his gestational age. On
his examination, the baby has very weak, intermittent finger flexion when his palm
is pressed, and no arm flexion when traction is placed on his arms. He has interest
in a pacifier, and will suck on it 4 to 5 times before requiring a 10-second pause.
These findings are most consistent with an infant of what gestational age?
a. Less than 30 weeks’ gestation
b. 30 to 31 weeks’ gestation
c. 32 to 33 weeks’ gestation
d. 34 to 35 weeks’ gestation
e. Greater than 36 weeks’ gestation

5-38. A 7-day-old female infant was born at 30 weeks’gestation by cesarean section for
maternal preeclampsia.She required CPAP for 3 days for respiratory distress syndrome,
and received 48 hours of antibiotics before blood cultures were negative. Today, the
pediatric intern noted the baby is tachypneic with an active precordium. On examination,
he hears a continuous cardiac murmur and palpates full peripheral pulses.Echocardiogram
reveals a large patent ductus arteriosus with left-to-right flow. Which of the following
factors could have contributed to the infant’s clinical status?
a. Elevated cortisol concentrations due to antenatal
betamethasone administration
b. Decreased sensitivity of premature ductal tissue to
prostaglandins
c. Antenatal treatment of maternal headache with
ibuprofen
d. . Elevated serum concentration of prostaglandin E 2
(PGE 2 )
e. Omission of surfactant administration after delivery
Consultant Dr.Wahid Helmi Domiate Egypt ( Neonatoloy Revision First note)

5-39. Which of the following statements about necrotizing enterocolitis (NEC) in near-
term and term infants is true?
a. NEC in near-term and term infants is usually preceded by a perinatal infectious risk
factor such as maternal chorioamnionitis or prolonged rupture of membranes.
b. NEC in near-term and term infants typically involves the proximal small bowel, rather
than the colon.
c. NEC typically occurs sooner after birth in near-term and term infants when compared
with premature infants.
d. Near-term and term infants with NEC are more likely to require surgical
intervention than premature infants.
e. Near-term and term infants rarely develop a spontaneous intestinal perforation (SIP) if
there are no risk factors for intestinal hypoperfusion, such as indomethacin exposure.

5-40. Which of the following statements is correct based on the studies currently
available about prevention of necrotizing enterocolitis?
a. Exposure to antenatal steroids is associated with a decreased risk of necrotizing
enterocolitis in premature infants.
b. Intermittent increases in metabolic demand from bolus feeding put very-low-birth-
weight infants at risk for necrotizing enterocolitis.
c. When compared with formula, feeding with maternal breast milk protects against
necrotizing enterocolitis, but feeding with donor breast milk confers a risk similar to
feeding with formula.
d. Rapid advancement of enteral feeding in premature, very-low-birth-weight infants
increases the risk of necrotizing enterocolitis.
e. Dosing of probiotic strains for the prevention of NEC has been standardized
internationally and requires a minimum of 1.0  1010 colony-forming units/day.
Consultant Dr.Wahid Helmi Domiate Egypt ( Neonatoloy Revision First note)

5-41. Which of the following interventions decreases an infant’s risk of


developing germinal matrix/ intraventricular hemorrhage (IVH)?
a. Antenatal administration of phenobarbital
b. Postnatal administration of glucocorticoids
c. Postnatal infusion of fresh frozen plasma
d. Treatment of maternal chorioamnionitis with antibiotics
e. Rapid correction of hypotension with fluid boluses

5-42. A newborn female presents for a weight check a few days after discharge from the
neonatal intensive care unit. She was born at 35 4/7 weeks’ gestation, weighing
2570 g. The mother, a 27-year-old unemployed woman, had good prenatal care.
She developed preterm labor on the day of her daughter’s birth, and delivered
vaginally shortly after rupture of membranes. She was admitted to the neonatal
intensive care unit for transient tachypnea of the newborn, which resolved by 12
hours of life. Antibiotics were started on admission, but discontinued after 48
hours when blood cultures were negative. She was initially slow with oral feeding,
but took an adequate amount prior to discharge.
The mother has been reading about prematurity and neurodevelopment on the
Internet, and she asks the general pediatrician what to expect for her daughter.
Which of the following statements is accurate regarding outcomes of premature
infants later in life?
a. Since her infant was almost 36 weeks’ gestation at delivery, she does not need to
worry about neurodevelopmental problems.
b. Determination of her daughter’s need for special assistance in school (due to
intellectual disability) will be evident by her daughter’s third birthday.
c. Her daughter is at risk for risk-taking behavior as a teenager due to her prematurity.
d. If the baby’s birth weight had been less than 1500 g, she would have been extremely
unlikely to graduate from high school.
e. Preterm children are more likely to have language disorders, reading disability, and
difficulty with arithmetic.
Consultant Dr.Wahid Helmi Domiate Egypt ( Neonatoloy Revision First note)

5-43. The Obstetrics attending asks the Neonatology attending to counsel a 36-year-old
female who presented to the Labor and Delivery (L&D) suite in active labor. The
patient reported to the L&D nurse that she is “at full term.” The patient received
prenatal care in a community clinic, including a dating ultrasound
at 10 weeks’ gestation. After a normal ultrasound at
20 weeks’ gestation, she was told that she didn’t require additional follow-up. The
patient reports that she has been taking her prenatal vitamins and that her fetus
has remained active. The obstetrician is concerned that the patient’s fundal height is
only 33 cm and the fetus must be more premature than 36 weeks’ gestation or
significantly growth restricted.
Which of the following statements is correct and should be included in the
consultation with the mother?
a. For a low-birth-weight infant, the likelihood of developing serious neonatal
complications is primarily related to size at delivery, not gestational age.
b. If there has been growth restriction in utero that has affected the body, but has spared
the head, her child’s cognitive outcome should not be adversely affected.
c. If there has been growth restriction due to uteroplacental insufficiency, her child will
have a greater risk for developing hypertension and diabetes as an adult.
d. Since third-trimester intrauterine growth restriction (IUGR) stimulates a response
that interferes with fetal lung maturation, her baby must be intubated in the delivery
room.
e. If sent to a pathologist within 6 hours of delivery, the placenta, on gross examination,
will provide definitive information about the reason for IUGR.

5-44. A 4325-g term infant born with meconium aspiration syndrome has severe hypoxic
respiratory failure.
Despite aggressive management with high-frequency ventilation, inhaled nitric oxide,
and phosphodiesterase inhibitors, stable oxygenation cannot be achieved. The NICU
team decides to utilize extracorporeal membrane oxygenation (ECMO) for this
infant.
Which of the following statements regarding the outcomes of neonates treated with
ECMO is correct?
a. The increased frequency of treatment of neonatal hypoxic respiratory failure with
ECMO has resulted in a decrease in ECMO complications and better survival.
b. Neonates who require treatment with ECMO for neonatal hypoxic respiratory failure
have a risk of neurodevelopmental impairment that is significantly greater than
neonates who respond to conventional therapy (including high-frequency ventilation)
and inhaled nitric oxide.
c. The most common cause of mortality in infants treated with ECMO for hypoxic
respiratory failure is hemorrhage.
d. The risk of hearing impairment in extremely premature infants is significantly greater
than in term infants with respiratory and other organ failure.
e. Survival rate for infants treated with ECMO in the United States is less than 50%.
Consultant Dr.Wahid Helmi Domiate Egypt ( Neonatoloy Revision First note)
5-45. Which of the following statements is correct about neurodevelopmental
impairment in premature infants?
a. It is possible to diagnose a major neurodevelopmental impairment prior to NICU
discharge based on brain MRI and abnormalities on serial neurologic examinations.
b. Detection of abnormalities on magnetic resonance imaging (MRI) of the brain is the
“gold standard” for predicting major neurodevelopmental disabilities.
c. More than half of the premature survivors who were born at the limit of viability,
prior to 26 weeks’ gestation, develop intellectual disability, cerebral palsy, or both.
d. . Complications of prematurity including chronic lung disease, sepsis, and necrotizing
enterocolitis confer additional risk for problems with neurodevelopment above the
risk of prematurity.
e. The most common type of cerebral palsy in premature survivors born at the limit of
viability is spastic hemiplegia.
5-46. Which of the following statements regarding the cardiorespiratory transition
from intrauterine to extrauterine life is correct?
a. Compliance in the neonatal lung decreases due to fluid shifts.
b. Surfactant is released into the alveolar space via lung inflation and increased blood
catecholamine levels.
c. Pulmonary vascular resistance increases in response to increased arterial oxygen
tension.
d. Clearance of fetal lung fluid occurs primarily through egress via the trachea.
e. Neonatal blood flow pattern is unchanged from the fetal blood flow pattern.

5-47. A premature male infant born at 26 1/7 weeks’ gestation has a cranial ultrasound
performed at 4 days of age.
The study shows a hemorrhage limited to the right germinal matrix.
Which of the following statements is correct regarding this patient’s
management/outcome?
a. Since the hemorrhage is limited to the germinal matrix/subependymal area, additional
brain imaging is not necessary for several weeks.
b. Male infants with germinal matrix/intraventricular hemorrhages are less likely to have
neurologic sequelae than female infants.
c. Since the hemorrhage is limited to the germinal matrix/subependymal area, the infant’s
development should be normal.
d. Cranial ultrasound should be repeated within the first week of life due to an
increased risk of a
primary hemorrhage into the left germinal matrix.
e. Cranial ultrasound should be repeated within the first week of life, as the hemorrhage
in the right germinal matrix may extend into the ventricles.
Consultant Dr.Wahid Helmi Domiate Egypt ( Neonatoloy Revision First note)

5-48. Which of the following infants is at highest risk of developing a germinal


matrix–intraventricular hemorrhage (GM-IVH)?
a. A term male infant born by vacuum-assisted vaginal delivery who required intubation,
positive pressure ventilation, and chest compressions in the delivery room
b. A 20-day-old male infant born at 24 weeks’ gestation who develops hypotension from a
patent ductus arteriosus
c. A female infant born at 26 weeks’ gestation by precipitous vaginal delivery who
develops hyperoxia and hypocarbia after intubation and surfactant administration in the
delivery room
d. A male infant born by cesarean section at 34 weeks’ gestation who required positive
pressure ventilation in the delivery room
e. A female infant born at 28 weeks’ gestation (due to maternal chorioamnionitis) who
develops a right- sided tension pneumothorax after reintubation at 17 days of age
5-49. A male infant born at 25 2/7 weeks’ gestation (birth weight 800 g) develops
tachycardia and abdominal distension prior to passing a blood-tinged stool. His
physical examination reveals decreased tone and activity, tenderness on
palpation of the abdomen, and a paucity of bowel sounds. His abdominal x-ray
shows diffuse pneumatosis without pneumoperitoneum or portal venous air. His
parents arrive during initial stabilization of their infant and ask to speak to the
attending neonatologist.Which of the following statement regarding this infant’s
condition is correct?
a. Because of his abdominal findings, the infant likely has hypersensitivity to mother’s
breast milk, and she should stop pumping.
b. Unless intestinal perforation occurs, their infant’s risk for growth failure is not
significantly increased as a result of this condition.
c. Their infant is unlikely to develop an intestinal stricture if there is no bowel
perforation.
d. This condition puts their infant at increased risk for neurodevelopmental impairment.
e. The incidence of this condition is inversely related to gestational age and birth
weight, but these characteristics do not increase the risk of mortality.
5-50. A 1460-g male infant is delivered by cesarean section after a 30 1/7 weeks’
gestation to a 29-year-old woman with acute lymphoblastic leukemia. The infant is
placed on the delivery room table. Drying the skin and suctioning the oropharynx
is initiated, followed by positive pressure ventilation (PPV). After 20 seconds of
PPV, the neonatal nurse notes that the infant has a
heart rate of 40 beats/min and no chest wall movement.
Which of the following interventions is indicated at this time in the resuscitation?
a. Flexion of the head and neck
b. Confirmation of an appropriate seal of the face mask
c. Intravenous infusion of 10 cm3/kg of normal saline
d. External cardioversion
e. Intravenous infusion of 0.1 mg/kg of 1:10,000 epinephrine solution
Consultant Dr.Wahid Helmi Domiate Egypt ( Neonatoloy Revision First note)

5-51. A 3690-g male infant was born vaginally after a


38 6/7 weeks’ gestation. The pregnancy was notable for polyhydramnios. He
required only suctioning,
drying, and stimulation in the delivery room. He passed meconium on the delivery
room table. Apgar scores were 8 and 9 at 1 and 5 minutes, respectively.
At 6 hours of age, the mother tells the postpartum nurse that the baby is “spitty”
with feeding. The nurse notes the infant has copious clear secretions from the
mouth. The nasopharynx and oropharynx appear normal on physical examination.
A large-bore nasogastric tube is placed; a subsequent chest x-ray shows the tube
ending at the level of the sixth cervical vertebrae.
The most appropriate initial step in management of this neonate is:
a. Intravenous antibiotics
b. Barium enema
c. Oral airway placement
d. Suction applied to the nasogastric tube
e. Contrast study of the upper gastrointestinal tract
5-52. Which of the following statements regarding fetal cortisol is correct?
a. Steroidogenic enzymes present in the adult adrenal gland are absent in the fetal adrenal
gland.
b. Fetal cortisol is necessary for normal intrauterine development.
c. Circulating fetal cortisol peaks during the middle of the first trimester of development.
d. Most of the circulating fetal cortisol is derived from the maternal adrenal gland.
e. Elevated fetal cortisol levels in the third trimester assist in neonatal respiratory
adaptation.

5-53. A 4950-g female neonate is delivered vaginally after a 38 6/7 weeks’ gestation
complicated by poorly controlled type 2 diabetes. The obstetrician reported
shoulder dystocia during the delivery. On examination at 4 hours of age, the baby’s
left arm is internally rotated, and the forearm is extended and pronated. The baby
does not move her left arm when the examiner assesses the Moro reflex.
Which of the following physical examination findings is most associated with the
findings in the arm?
a. An absent left thumb
b. A hyperpigmented patch on the left chest
c. A 2-vessel umbilical cord
d. A constricted left pupil
e. An enlarged left fifth finger
Consultant Dr.Wahid Helmi Domiate Egypt ( Neonatoloy Revision First note)

5-54. Which of the following neonates should receive hepatitis B immune globulin
within 12 hours after birth?
a. A 38 weeks’ gestation neonate (birth weight 3600 g) whose mother is hepatitis B surface
antigen (HBsAg) negative
b. A 39 weeks’ gestation neonate (birth weight 3750 g) whose mother has HBsAg
serology that is unknown, but will be confirmed within 24 hours after delivery
c. A 37 weeks’ gestation neonate (birth weight 3550 g) whose mother is positive for
hepatitis C antibody only
d. A 34 weeks’ gestation neonate (birth weight 2500 g) whose mother is HBsAg negative
but developed chorioamnionitis in labor
e. A 36 weeks’ gestation neonate (birth weight 1850 g) whose mother has HBsAg
serology that cannot be resulted until 36 hours after delivery
5-55. A 4885-g female infant was delivered via cesarean section due to cephalopelvic
disproportion after a 40 3/7 weeks’ gestation. Maternal serologies were
unremarkable, and cultures (including a cervical culture
for group B Streptococcus at 36 weeks’ gestation) were negative. The mother has
a 10-year history of poorly controlled type 2 diabetes mellitus.
The baby only required brief blow-by oxygen in the delivery suite. The infant’s
Apgar scores were 8 and 9 at 1 and 5 minutes, respectively. Her initial newborn
examination was normal.
The neonate fed well in the term nursery after birth.
At 60 hours of age, she developed rhythmic, generalized jerking of her head and
extremities. After the ictal event, vital signs and the physical examination were
unremarkable, and a capillary glucose was 69 mg/dL.
Which of the following tests should be ordered to evaluate this infant?
a. Serum calcium level
b. Serum sodium level
c. Serum C-reactive protein
d. Computerized tomography (CT) scan of the head
e. Magnetic resonance imaging (MRI) of the head

5-56. A 3440-g male neonate is delivered vaginally after a 39 6/7 weeks’ gestation
complicated by fetal bradycardia. On arrival at the delivery room table, the
baby has no respiratory effort or spontaneous movement. He is covered in
particulate meconium. The nurse palpates the pulse in the umbilical cord stump
and detects 5 beats in 6 seconds.
Which of the following steps is next indicated in resuscitation of this neonate?
a. Initiation of chest compressions
b. Placement of a 5 French nasogastric tube
c. Suctioning of the mouth and nose with a bulb syringe
d. Catheterization of the umbilical vein
e. Intubation and suctioning the trachea
Consultant Dr.Wahid Helmi Domiate Egypt ( Neonatoloy Revision First note)

5-57. A 2-day-old term female neonate is undergoing her discharge physical


examination in the nursery. On exam, you note the liver edge 1 cm below the right
costal margin and the spleen tip is not palpable. The umbilical stump is dry. Bowel
sounds are audible in the abdomen. When the infant cries, you note that there is a
slight distention of the abdominal space between the rectus abdominis muscles.
Which of the following steps is indicated for this infant?
a. Order a liver and gallbladder ultrasound
b. Discharge the baby home with her parents
c. Obtain a peripheral blood smear
d. Order a voiding cystourethrogram
e. Consult a pediatric surgeon
Consultant Dr.Wahid Helmi Domiate Egypt ( Neonatoloy Revision First note)

5-58. A 27-year-old woman brings her 3-year-old son into your office for a well-child care
visit. The child had documented intrauterine growth restriction (IUGR), and
weighed 1990 g at birth (at 39 1/7 weeks’ gestation). The mother is currently 8 weeks
pregnant and wants to decrease the risk of fetal growth restriction during her
current gestation. She has no acute or chronic medical problems. She smokes 0.5
pack of cigarettes per week.
Which of the following interventions would reduce the likelihood of fetal growth
restriction during this pregnancy?
a. Consumption of 4 oz of red wine daily
b. Avoiding food rich in -carotene
c. Initiation of a strict vegan diet
d. Dietary supplementation with folic acid
e. Cessation of cigarette smoking

5-59. Which of the following statements regarding neonatal and infant mortality is
correct?
a. Infant mortality rates are higher for infants of pregnancies where prenatal care
commences in the first trimester.
b. Asian or Pacific Islander ethnicity is a risk factor for infant mortality.
c. The introduction of surfactant therapy and antenatal steroids has reduced neonatal
mortality.
d. Female sex is a risk factor for neonatal mortality.
e. Neonatal mortality for low-birth-weight infants is reduced in high-care-level, low-
volume neonatal intensive care units (NICUs).

5-60. Which of the following neonates is most likely to experience a normal anion gap
metabolic acidosis?
a. A 2-day-old term male with weak femoral pulses (relative to brachial pulses) and a
serum lactate of
5.9 mmol/L
b. A 3-day-old female born at 24 4/7 weeks’ gestation with bounding palmar pulses and a
serum creatinine of 1.8 mg/dL
c. . A 1-day-old male born at 28 1/7 weeks’ gestation with mild respiratory distress
syndrome and an initial urine pH of 7.5
d. A 4-day-old term female male with lethargy and a serum lactate of 1.2 mmol/L
e. A 5-day-old infant born at 32 2/7 weeks’ gestation with hypotension and a blood
culture positive for gram-negative rods
Consultant Dr.Wahid Helmi Domiate Egypt ( Neonatoloy Revision First note)

5-61. A 28-day-old male presents to his pediatrician’s office with a 1-week history of
intermittent vomiting and alternating irritability and lethargy. He was delivered
vaginally (with assistance of forceps) after a 40 4/7 weeks’ gestation complicated by
fetal bradycardia. His discharge physical was notable for marked bruising. On
examination, he is afebrile with a normal cardiorespiratory examination. He has
well-circumscribed, indurated nodules over the cheeks at the site of forceps
placement.
Which of the following laboratory values is most associated with this infant’s
presentation?
a. A serum glucose of 29 mg/dL
b. A serum ammonia of 135 mol/L
c. A serum lactate of 5.0 mmol/L
d. A serum calcium of 13.1 mg/dL
e. A serum urea nitrogen of 1.8 mg/dL

5-62. A 28 3/7–week male was born by cesarean section due to worsening maternal
preeclampsia. Due to respiratory distress in the delivery room, he is intubated and
placed on mechanical ventilation.
At 6 hours of age, the infant develops mottling of the skin, tachycardia, and
desaturations (measured by bedside pulse oximetry). A stat echocardiogram shows a
structurally normal heart with a patent ductus arteriosus.
Which of the following interventions would improve the infant’s blood pressure?
a. Initiation of an intravenous prostaglandin E1 (PGE1) infusion
b. Increase in the infant’s peak end-expiratory pressure (PEEP) on the ventilator
c. Increase in the infant’s peak inspiratory pressure (PIP) on the ventilator
d. Initiation of an intravenous immune globulin infusion
e. Initiation of an intravenous dopamine infusion
5-63. A 1900-g male infant is delivered by C-section after a 29 1/7 weeks’ gestation
secondary to fetal supraventricular tachycardia (SVT) and progressive hydrops
fetalis. The infant is placed on the delivery room table, and positive pressure
ventilation (PPV) begins after drying the skin and suctioning the
oropharynx. Physical examination is notable for diffuse anasarca. After 4 minutes
of PPV, the nurse notes the neonate has a heart rate of 190 beats/min, central
cyanosis, and intercostal retractions.
Which of the following steps is indicated at this point in the resuscitation?
a. Infusion of 0.3 mg/kg of 1:10,000 epinephrine solution via an umbilical venous
catheter
b. Endotracheal intubation and ventilation using a T-piece resuscitator
c. Weaning the infant to blow-by oxygen
d. Infusion of 0.1 mg/kg of adenosine via an umbilical venous catheter
e. Weaning the infant to nasal cannula
Consultant Dr.Wahid Helmi Domiate Egypt ( Neonatoloy Revision First note)

5-64. Which of the following statements regarding thermoregulation in neonate is


correct?
a. Heat production in the neonate (controlling for body weight) is greater than heat
production in the adult.
b. Heat production in the neonate (relative to body weight) must be lower in the neonate
to maintain a normal body temperature.
c. The capacity for neonatal adaptation to cold stress is similar to that of an adult.
d. Brown fat stores are less abundant in the neonate than in the adult.
e. Circulating free fatty acids serve as an acute source of energy during cold stress in the
neonate.

5-65. You are asked by the obstetrical service to counsel a 25-year-old primigravida
woman who is experiencing preterm labor. Her pregnancy is notable for a twin
gestation.
Which of the following statements is true regarding multiple births?
a. Twin–twin transfusion syndrome usually occurs in dizygotic pregnancies as
compared with monozygotic pregnancies.
b. Most infants born following multifetal gestations are premature.
c. The rate of multiple births has remained stable over recent decades.
d. Fetuses of twin pregnancies are more likely to be born in a vertex position as compared
with fetuses of singleton pregnancies.
e. Monozygotic twins account for two thirds of all spontaneous twin births.

5-66. A newborn infant is born at 26 6/7 weeks’ gestation following a spontaneous vaginal
delivery. Resuscitative efforts include warming, and drying the infant followed
by clearing and positioning the airway. The infant developed spontaneous respiratory effort
within 1 minute of life. However, he develops intercostals retractions, minimal air entry on
auscultation, grunting,and decreased tone. He then develops apnea, cyanosis,and bradycardia
(heart rate of 80 beats/min). Positive pressure ventilation is given via self-inflating bag and face
mask, but the infant remains apneic.Which is the most appropriate next step in the
management of this infant?
a. Administration of continuous positive airway
pressure (CPAP) via nasal prongs
b. Needle decompression of the right hemithorax
c. Provision of supplemental oxygen via nasal cannula
d. . Endotracheal intubation for mechanical ventilation and surfactant administration

e. Administration of inhaled nitric oxide


Consultant Dr.Wahid Helmi Domiate Egypt ( Neonatoloy Revision First note)

5-67. A 30-day-old infant was born at 24 4/7 weeks’ gestation.Until today, she had been
intubated and ventilated using the synchronized intermittent mandatory ventilation
(SIMV) mode. Based on improved blood gases, she was extubated 12 hours ago to room
air. In the last 20 minutes, she has developed nasal flaring and intermittent subcostal
retractions. Her current blood gas shows a pH of 7.34 and a pco 2 of 56 mm Hg.
Her pulse oxygen saturations have decreased to 82%.Her chest radiograph reveals a
normal cardiothymic silhouette, decreased lung expansion, and air bronchograms.
What is the most appropriate next step in the management of this patient?
a. Endotracheal intubation and synchronized intermittent mandatory ventilation
(SIMV)
b. Provision of supplemental oxygen via nasal cannula
c. Administration of continuous positive airway pressure (CPAP) via nasal prongs
d. Endotracheal intubation and high-frequency ventilation (HFV)
e. Repeat blood gas sampling in 1 hour

5-68. A 4-day-old term neonate with congenital pneumonia is being mechanically


ventilated for respiratory failure.He is ventilated using the synchronized intermittent
mandatory ventilation (SIMV) mode. His settings are:Positive inspiratory pressure—19
mm Hg Positive end-expiratory pressure—5 mm Hg Rate—25 breaths/min For the past
several hours you notice that the ventilator is displaying wide variability in measured tidal
volumes.His most recent arterial blood gas shows a pH of 7.5 and p co 2 of 30 mm Hg. On
the same ventilator settings,the previous arterial blood gas had a pH of 7.3 and
p co 2 of 51 mm Hg.What is the most appropriate next step in the management of this
patient?
a. Change the mode to intermittent mandatory
ventilation.
b. Add pressure support of 10 mm Hg to his current
ventilator settings.
c. Change the mode to assist control ventilation.
d. Change the mode to high-frequency oscillation
ventilation.
e. . Change the mode to volume-targeted ventilation.
Consultant Dr.Wahid Helmi Domiate Egypt ( Neonatoloy Revision First note)

5-69. A term 3-day-old infant was born through meconium- stained amniotic fluid.
Following aspiration of meconium, the infant developed respiratory failure. On
current examination, he has a systolic murmur and a loud second heart sound. His
chest radiograph reveals a hyperlucent background in addition to patchy areas of
opacity. Despite the use of high-frequency oscillation and an inspired oxygen fraction
of 1.0, the postductal pulse oximeter does not read above 90%.
Which of the following treatment options is most likely to be effective in
addressing the underlying pathophysiology?
a. Permissive hypercarbia
b. Diuretic therapy
c. Inhaled nitric oxide
d. Opiate therapy
e. Albuterol

5-70. A 1-week-old infant born at 23 6/7 weeks’ gestation has developed sudden onset of
hypotension and tachycardia. She is mechanically ventilated, has been NPO since
birth, and recently completed a course
of antibiotics and hydrocortisone for septic shock. Prenatal history is notable for
intrauterine growth restriction and perinatal indomethacin exposure for preterm
labor. On examination, she has abdominal distention and guarding, lethargy, and
hypoperfusion. Chest and abdominal x-rays reveal pulmonary edema and a
pneumoperitoneum. There is no pneumatosis intestinalis or portal venous gas noted.
Which of the following is the most likely etiology of the infant’s acute
decompensation?
a. Necrotizing enterocolitis (NEC)
b. Cow’s milk protein allergy
c. Spontaneous intestinal perforation (SIP)
d. Volvulus
e. Duodenal atresia
Consultant Dr.Wahid Helmi Domiate Egypt ( Neonatoloy Revision First note)

5-71. A term infant was born to a 26-year-old, gravida


7 woman at home following a precipitous delivery. When the emergency medical
team arrived, the infant was vigorous but still attached to the placenta. The
infant was transported to a local hospital where his physical examination was
normal. Several hours later, the infant develops irritability and poor feeding. On
examination, he is tachypneic and lethargic. The remainder of the examination is
within normal limits.
Which of the following tests should be ordered first in evaluating this infant?
a. Hematocrit
b. Cranial sonogram
c. Thyroid-stimulating hormone level
d. Stool occult blood test
e. Chest radiograph

5-72. You are seeing a 3-day-old Korean male infant in your clinic for the first time
since his hospital discharge.
He was born by spontaneous vaginal delivery at
36 2/7 weeks’ gestation to a 25-year-old woman with negative prenatal serologies.
The pregnancy, delivery, and postpartum admission were uncomplicated. He has
been breast-feeding every 1 to 2 hours, but his weight on your examination has
decreased 12% from birth weight. The remainder of his examination is
unremarkable.
Which screening test is most important to perform at this visit?
a. Hemoglobin level
b. Vitamin D level
c. C-reactive protein
d. Serum lead level
e. Total serum bilirubin level
Consultant Dr.Wahid Helmi Domiate Egypt ( Neonatoloy Revision First note)

5-73. A 2-day-old infant born at 35 1/7 weeks’ gestation to a diabetic mother presents to your
office with jaundice. She is breast-feeding every 2 hours, has wet diapers with every feed,
and has passed at least 1 meconium stool. Vital signs include weight decreased 2% from
birth weight, a rectal temperature of 37.4°C, heart rate of 147 beats/min, respirations of 50
breaths/min, and blood pressure of 45/28 mm Hg. Her anterior fontanelle is soft and flat.
She is alert with appropriate tone, and
is icteric to her mid-calf. Her total serum bilirubin (TSB) level is 14.0 mg/dL, and direct
bilirubin level is
0.4 mg/dL at 48 hours of life.
Which of the following is the appropriate management of this infant’s hyperbilirubinemia?
a. Administer intravenous immunoglobulin (IV Ig).
b. . Initiate phototherapy, continue breast-feeding, and recheck TSB level in 4 hours.
c. Initiate phototherapy, replace breast-feeding with formula feeding, and recheck TSB in
4 hours.
d. Initiate phototherapy, discontinue breast-feeding, initiate IV hydration, and recheck TSB
in 4 hours.
e. Place central arterial and venous catheters for exchange transfusion.

5-74. A 24-year-old, gravida 2, para 1 Hispanic woman is being seen for the first time
for prenatal care. Her previous delivery occurred in El Salvador, and she has
recently immigrated to the United Stated for this delivery. Based on her last
menstrual period, the pregnancy is dated at 34 0/7 weeks’ gestation.
Ultrasound examination of the fetus reveals thickened subcutaneous tissue and fluid
in the pleural cavity.Sampling of the amniotic fluid is significant for bile pigment,
and percutaneous umbilical blood sampling shows fetal anemia.
Which of the following is the most likely cause of this infant’s presentation?
a. Human parvovirus B19
b. Cytomegalovirus
c. . Rh antibody exposure
d. Complex congenital heart disease
e. Supraventricular tachycardia
Consultant Dr.Wahid Helmi Domiate Egypt ( Neonatoloy Revision First note)

5-75. A 2-week-old infant presents to the pediatrician with a large reddish blue mass
under the lower middle aspect of the abdomen, measuring 3 cm  4 cm.
According to her mother, the lesion was noticed after discharge home from the
nursery, but has enlarged over the past 2 weeks. The mass is nontender and warm,
and a bruit is heard on auscultation of the overlying skin. The infant is vigorous with
normal vital signs and strong lower extremity pulses. There is no
hepatosplenomegaly, pallor, or petechiae. The results of her complete blood count
are:White blood cells—15,000/L Hemoglobin—11.5 g/dL Platelets—19,000/L
Which of the following syndromes is the most likely cause of thrombocytopenia in
this infant?
a. Trisomy 13
b. Trisomy 18
c. Fanconi anemia
d. Thrombocytopenia absent radii (TAR) syndrome
e. Kasabach-Merritt syndrome

5-76. A male infant was born at 36 4/7 weeks’ gestation by emergent cesarean section
for fetal bradycardia.
There was no evidence of labor prior to delivery. Clear amniotic fluid is noted on
rupture of membranes during the operative delivery. The infant’s Apgars were 9 and
9 at 1 and 5 minutes, respectively. However,he developed grunting and tachypnea
approximately 30 minutes after birth. On current examination, his respiratory rate
is 70 breaths/min and the preductal and postductal pulse oximeters read 97% while
he is breathing room air. His physical examination is otherwise normal. A chest
radiograph demonstrates prominent vascular markings, an opacity in the transverse
fissure, and flattening of the diaphragms. An arterial blood gas reveals:
pH—7.46
pco2—34 mm Hg Pao2—89 mm Hg HCO3—24 mEq/L
Which of the following mechanisms is most likely responsible for this infant’s
respiratory distress?
a. Evolving chemical pneumonitis
b. Delayed sodium transport across the alveolar apical and basolateral cell membranes
c. Ascending infection through the birth canal
d. Increased pulmonary vascular tone
e. Increased alveolar surface tension
Consultant Dr.Wahid Helmi Domiate Egypt ( Neonatoloy Revision First note)

5-77. A 2-week-old female infant was born at 24 1/7 weeks’ gestation. She has
significant respiratory insufficiency secondary to surfactant deficiency and is
currently intubated and on high-frequency mechanical ventilation. Chest x-rays
have persistently demonstrated diffuse pulmonary interstitial emphysema. Early
this morning, she abruptly became tachypneic and agitated. On examination, she is
tachycardic and hypotensive, and the cardiac impulse (PMI) is shifted to the right,
with diminished breath sounds over the left chest. Her chest x-ray is shown below:

(Reproduced, with permission, from Rudolph CD, Rudolph A, Lister G, First L, Gershon A. Rudolph’s
Pediatrics. 22nd ed. New York: McGraw-Hill, 2011.)

Which of the following is the most appropriate next step in the management of this
patient?
a. Decompressing the left hemithorax with an angiocatheter, followed by chest tube
placement
b. Repositioning the endotracheal tube
c. Inserting a nasogastric tube and aspirating gastric air and fluid
d. Increasing mean airway pressure on the ventilator
e. Performing an emergent pericardiocentesis
Consultant Dr.Wahid Helmi Domiate Egypt ( Neonatoloy Revision First note)

5-78. A 23-year-old primigravida woman presented to Labor and Delivery today for
decreased fetal movement. Her pregnancy history was significant for chronic
ibuprofen use to treat low back pain in the third trimester.
Nonstress testing revealed nonreassuring fetal heart tones, and the woman
underwent an emergent cesarean section. Thirty minutes following delivery, the baby
has tachypnea, nasal flaring, and grunting. His preductal and postductal pulse
oximeter readings are 97% and 86%, respectively. His chest x-ray shows hyperlucent
lung fields and an enlarged cardiothymic silhouette. A postductal arterial blood gas
on room air demonstrates the following values:
pH—7.30
pco2—37 mm Hg Pao2—55 mm Hg HCO3—19 mEq/L
Which of the following tests is the preferred method to confirm this patient’s
diagnosis?
a. Echocardiography with Doppler studies
b. Serial blood gas sampling
c. Pulse oximetry
d. Tracheal aspirate for bacterial culture
e. Arterial blood sample for bacterial culture
Consultant Dr.Wahid Helmi Domiate Egypt ( Neonatoloy Revision First note)

5-79. In which of the following scenarios is pulmonary hemorrhage most frequently


associated?
a. A term 2-day-old born following maternal cocaine use
b. A term 2-day-old with sepsis
c. A term 2-day-old born with perinatal depression
d. A preterm 2-day-old with a patent ductus arteriosis
e. A term 1-month-old with congenital heart disease

5-80. A 3-day-old infant born at 25 weeks’ gestation is undergoing high-frequency


oscillatory ventilation for respiratory insufficiency. Her x-rays since birth have
demonstrated a diffuse reticulogranular pattern with low lung volumes.
Approximately 2 hours after
administration of exogenous surfactant, she developed pallor, bradycardia, and a
bloody nasal discharge. A chest x-ray now reveals patchy infiltrates throughout the
lung fields. Abruptly, her pulse oximeter saturations decline.
What is the first step in the management of this patient?
a. Suction and secure the airway.
b. Transfuse packed red blood cells.
c. Transfuse platelets.
d. Increase the mean airway pressure.
e. Administer indomethacin.
ANSWERS
effect will be transient and the dose must be repeated in early
infancy. Guidelines for sequential oral dosing of vitamin K
Answer 5-1. e vary. Some physicians recommend a second dose at 6 to 8
Early vitamin K deficiency bleeding (VKDB), previously months of age; others recommend weekly dosing while
known breastfeeding.
as hemorrhagic disease of the newborn (HDN), is caused Failure of vitamin K therapy to prevent VKDB has
by inadequate levels of active vitamin K–dependent clotting been reported, and is associated with noncompliance with
factors (factors II, VII, IX, and X) and has an incidence that therapy (Srehle, 2010; Doran, 1995). In addition, newborns
has been reported as high as 1.7% (AAP, 2003). VKDB can receiving incomplete oral prophylaxis have a higher risk of
result in intracranial, gastrointestinal, or generalized bleeding developing VKDB.
in the neonate. Due to limited neonatal stores at birth and In 1993, the Vitamin K Ad Hoc Task Force of the American
supplies from enteral sources (including breast milk), vitamin Academy of Pediatrics (AAP) reviewed concerns about the
K association of intramuscular vitamin K injection and the
supplementation is necessary to prevent neonatal hemorrhage. incidence of childhood leukemia. The committee concluded
A single 1-mg dose of intramuscular vitamin K in the first that there was no association between the intramuscular
few hours of life will prevent VKDB. Some parents request administration of vitamin K and childhood leukemia or
oral vitamin K administration (in lieu of an intramuscular other cancers. This conclusion has been supported by several
injection). Oral administration of the intramuscular
subsequent case–control studies (AAP, 2003).
formulation by mouth is safe. However, although a single 2-mg
oral dose of vitamin K will prevent VKDB for a few days, the
Answer 5-2. c Caput
Cephalohematoma
The neonate in this vignette has physical examination Subgaleal hemorrhage
findings consistent with a congenital diaphragmatic hernia
Extradural hemorrhage
(CDH). This is a rare condition with an overall incidence of Skin
1 in 2500 births. The large majority of cases are detected by Epicranial
routine prenatal ultrasound. The majority of CDHs (roughly aponeurosis
Periosteum
85%) occur on the left side through a posteriolateral defect Skull
(Bochdalek hernia). Dura
Infants with CDH develop respiratory failure due to
pulmonary hypoplasia (decreased lung volume on the
affected side) and pulmonary hypertension (due to abnormal (Reproduced, with permission, from Hay W, Levin M, Deterding R, Abzug M.
pulmonary vascular development). Other examination findings Current Diagnosis & Treatment Pediatrics. 21st ed. New York: McGraw-Hill,
2012.)
include a scaphoid abdomen (due to displacement of the
peritoneal contents into the thorax) and displacement of the
Answer 5-4. d
heart to the midline.
Initial management involves decompression of the Most IDMs are macrosomic, which is defined as a birth weight
stomach with a nasogastric tube (to reduce gastric distention greater than the 90th percentile for gestational age or over
impacting thoracic and mediastinal contents [heart and great 4000 g. This increased body weight is a result of deposition
vessels]). Subsequent respiratory management should include of fuel sources, resulting in increased body fat and mass of
endotracheal intubation. Positive pressure ventilation with a visceral organs, including the heart.
T-piece resuscitator (a flow-controlled resuscitation device HCM involves overgrowth of the intraventricular septum
with adjustable peak inspiratory and positive end-expiratory and 1 or both ventricular walls, leading to ventricular outflow
pressures) or a bag and mask will insufflate the stomach and tract obstruction. Due to this ventricular outflow tract
may impair respiratory function. Transillumination or needle obstruction coupled with decreased myocardial function, the
decompression of a normal right hemithorax is not indicated neonate with HCM has decreased cardiac output and may
in this case. develop congestive heart failure.
Insulin, not GH, represents the primary anabolic fetal
growth factor. Excess glycogen and fat deposition occurs in
the presence of insulin and high glucose levels, and this fat is
Answer 5-3. b primarily deposited in the abdomen and subscapular areas,
putting these infants at risk for shoulder dystocia. However,
The infant in this vignette has a cephalohematoma, which
this excess growth does not affect skeletal muscle, leading
is a subperiostial hemorrhage often associated with trauma
IDMs to have higher weight percentiles at birth relative to
during labor (see figure). The hemorrhage presents hours
length and head circumference.
after delivery as a fluctuant mass over the affected skull bone
Interestingly, not all IDMs are macrosomic. In women
(often the parietal or occipital bone). The hemorrhage is
with pregestational diabetes and associated nephropathy,
limited by the periosteum of the skull bone and, as a result,
retinopathy, or chronic hypertension, placental insufficiency
does not extend beyond the suture lines. Blood loss is minimal,
may occur. The placental insufficiency limits nutrient and
and affected infants usually do not require transfusion of
oxygen delivery, which can result in a small-for-gestational-age
blood products. The most common neonatal complication is
(SGA) infant.
hyperbilirubinemia due to the breakdown of the extravasated
blood. In the absence of localized or systemic infection,
antibiotics should not be administered. Needle aspiration of
the hematoma is not indicated and may introduce skin flora Answer 5-5. a
into the hematoma, resulting in infection of the mass and
The neonate in this vignette has examination findings
surrounding skin.
consistent with bilateral choanal atresia. The etiology is
A cephalohematoma should be distinguished from 2 other
due to a persistence of a bony septum (90%) or a soft tissue
entities (see figure). A caput succedaneum is edema of the
membrane (10%) (see figure below).
scalp and tends to cross suture lines. A subgaleal hemorrhage
This condition is an emergency situation in neonates (who
is bleeding between the galea aponeurotica and the skull.
are obligate nose breathers). Placement of an oral airway and/
This condition can lead to extensive hemorrhage and shock.
or putting the baby in a prone position at the time of clinical
As a result, this is an emergency situation requiring close
presentation will bypass the nasal obstruction and allow for air
monitoring, as well as potential replacement of blood and
exchange.
clotting products.
Neither supplemental oxygen and pressure (via nasal
cannula or CPAP) nor intranasal steroids will overcome the
anatomic obstruction. While unilateral atresia can generally be
delayed until the child is 1 to 2 years of age, bilateral choanal
atresia requires prompt repair.

Bony
obstruction

FIGURE 215-1. The Barlow test for developmental dislocation of the hip in
a neonate. (A) With the infant supine, the examiner holds both of the child’s
knees and gently adducts 1 hip and pushes posteriorly. (B) When the exami-
nation is positive, the examiner will feel the femoral head make a small jump
(arrow) out of the acetabulum (Barlow sign). When the pressure is released, the
head is felt to slip back into place. (Reprinted with permission from Herring JA,
(Page 256, Section 5: Newborn) ed. Tachdjian’s Pediatric Orthopaedics. 4th ed. Philadelphia: Saunders; 2007.)

Answer 5-6. c
Although her physical examination findings at birth were
unremarkable, the infant presented in this vignette is at risk
for DDH due to a history of breech presentation. As DDH
can develop any time in the first year of life, the Ortolani
and Barlow maneuvers (see Figures 215-1 and 215-2) may
initially be negative. Risk factors increasing the incidence of A
this condition in infants include female sex and birth from
the breech position. In this infant with 2 risk factors, current
guidelines for DDH screening recommend hip ultrasonography
at 6 weeks of age. Radiographs are not reliable until 4 to
6 months of age, when there is greater ossification of the
femoral head (AAP, 2000).
Leg length discrepancy or gluteal/leg fold asymmetry
may be significant in infants with DDH, but these findings
are not diagnostic. Magnetic resonance imaging is not the
recommended imaging modality for diagnosis of DDH.
B
Reexamination of the hips should be performed at every
well-child care visit during the first year of life but should not FIGURE 215-2. The Ortolani test for developmental dislocation of the hip
replace screening hip imaging in at-risk infants. in a neonate. (A) The examiner holds the infant’s knees and gently abducts
the hip while lifting up on the greater trochanter with 2 fingers. (B) When the
test is positive, the dislocated femoral head will fall back into the acetabulum
(arrow) with a palpable (but not audible) “clunk” as the hip is abducted
(Ortolani sign). When the hip is adducted, the examiner will feel the head
redislocate posteriorly. (Reprinted with permission from Herring JA, ed.
Tachdjian’s Pediatric Orthopaedics. 4th ed. Philadelphia: Saunders; 2007.)
practice among clinicians, supplemental oxygen use and
target saturations vary widely in clinical practice. Walsh
and colleagues developed an oxygen-need test to make the
TABLE 42-4. Laryngoscope Blade Size, Endotracheal Tube Size, and diagnosis of BPD more uniform. The infant in this vignette
Depth of Insertion for Babies of Various Weights and would meet the “Walsh criteria” for BPD if he met the
Estimated Gestational Age (EGA) conditions of choice e (saturations between 90% and 96%
Blade Tube Size EGA Depth of while receiving an FiO2 of over 0.3). If maintained on an
Size (mm) Weight (g) (Weeks) Insertion (cm) FiO2 of greater than 0.3 to keep saturations greater than 96%,
No. 00 2.5 Below 750 Below 27 6.5 he would require a room air challenge (with demonstration
of oxygen saturations of 90% or less) to be diagnosed with
No. 0 2.5 750-1000 27-28 7.0 BPD. During a room air challenge, those infants who cannot
No. 0 3.0 1000-2000 28-34 7.0-8.0 maintain saturations 90% during weaning and in room air for
No. 1 3.5 2000-3000 34-38 8.0-9.0 over 30 minutes were also diagnosed with BPD.
The diagnostic definition of BPD as oxygen need at
No. 1 3.5-4.0 3000 38 9.0-10.0 36 weeks does not require antecedent exposures (eg, RDS,
Adapted from Kattwinkel J. Textbook of Neonatal Resuscitation. 5th ed. Elk Grove mechanical ventilation), abnormalities on a chest radiograph,
Village, IL: American Academy of Pediatrics and American Heart Association; 2006. antecedent infectious exposures (antenatal chorioamnionitis or
postnatal sepsis), or any laboratory test.

Answer 5-7. a (Page 253, Section 5: Newborn)

Endotracheal intubation may be performed at any point during Answer 5-9. d


neonatal resuscitation. The infant in this vignette requires
intubation based on worsening respiratory distress despite Bronchopulmonary dysplasia (BPD) is a disease due to
prolonged PPV. immature lung development (see Figure 59-1). Infants who
Using data in Table 42-4, an infant weighing 1200 g should die from BPD show pathologic changes consistent with arrest
be intubated with a 3.0 mm endotracheal tube, and the airway of lung development. The lungs have an emphysematous
should be visualized using a number 0 laryngoscope blade. appearance due to decreased alveolarization. In addition,
The presence of anatomic variations in the trachea (ie, airway alveoli present in these lungs have large diameters and contain
masses, stenosis) may necessitate use of a smaller endotracheal dysplastic type II cells.
tube. Other pathologic findings include decreased pulmonary
microvasculature, an interrupted collagen network, and
(Page 168, Section 5: Newborn) abnormal elastin localization (ie, distribution away from fibers
where alveolar septations should later occur). The lungs may
Answer 5-8. e also demonstrate acute inflammatory changes due to the
BPD is defined as the need for supplemental oxygen at 36 bacterial infection at the time of the infant’s demise.
weeks postconceptional age. However, due to inconsistent (Page 254, Section 5: Newborn)

Hits to fetal lung Hits during transition Postnatal hits

Oxygen PDA
Initiation of
Chorioamnionitis ventilation Ventilation Sepsis

Fetal lung Preterm lung BPD

Corticosteroids Corticosteroids Nutrition

FIGURE 59-1. Risk factors for bronchopulmonary dysplasia, a disease with a single root cause: a very immature lung. Multiple factors then contribute to injury
and interfere with lung development. Fetal exposures to chorioamnionitis and corticosteroids, preterm birth with lung injury during resuscitation, and postnatal
care (ventilatory and oxygen) and diseases such as sepsis and patent ductus arteriosus represent a series of “hits” to the preterm lung. (Reproduced, with permis-
sion, from Rudolph CD, Rudolph A, Lister G, First L, Gershon A. Rudolph’s Pediatrics. 22nd ed. New York: McGraw-Hill, 2011. <http://www.accesspediatrics.com>.)
Answer 5-10. e Answer 5-13. d
BPD has a multifactorial etiology, and no single intervention Hypothermia (defined as a reduction in core temperature by
has been demonstrated to reduce its incidence. Lung 1°C-6°C) has been demonstrated to reduce the deleterious
immaturity is a major factor in the pathogenesis of BPD; cellular effects of brain ischemia in experimental animal
therefore, preventing preterm delivery would have the most models, including excitatory neurotransmitter release,
substantial impact on decreasing the incidence of BPD. microglial activation, and free radical production. Data
However, as in this vignette, interruption of progressive from 2 large randomized control trials and 1 large pilot trial
preterm labor (and, ultimately, preterm delivery) is not of therapeutic hypothermia in neonates were published in
always possible. Therefore, treatment strategies should be 2005. In all of these studies, hypothermia was initiated within
implemented in preterm neonates to reduce BPD. Supplemental 6 hours of birth and maintained for 48 to 72 hours.
oxygen use (based on gestation-specific guidelines) and low Both the NICHD randomized trial and the pilot trial
ventilatory pressures during resuscitation and mechanical by Eicher et al used whole-body hypothermia; newborns
ventilation can reduce the initiation and progression of BPD. with HIE receiving this therapy had decreased death or
Postnatal, high-dose corticosteroids may facilitate extubation of moderate-to-severe disability at 12 months (Eicher) or
ventilator-dependent infants, but do not decrease the incidence 18 months (NICHD) relative to HIE infants who were
of BPD. Furthermore, antioxidant therapy (with drugs such kept normothermic. The CoolCap Trial, which provided
as vitamin C) and reducing Ureaplasma colonization from selective head cooling to infants with moderate-to-severe
birth (with parenteral erythromycin) have not been shown to encephalopathy, demonstrated protective effects (decreased
decrease the incidence or prevent BPD. death or disability in survivors at 18 months) in infants
with HIE who demonstrated less severe aEEG abnormalities
at admission to the study. Recent data from subsequent
Answer 5-11. a studies (the European Network study, the UK Total Body
Hypothermia [TOBY] trial, and the Australasian Infant
Antenatal removal of a SCT is one of the few indications for Cooling Evaluation [ICE] study) have further advanced the
open fetal surgery. This tumor, thought to be a remnant of the understanding of the benefits of hypothermia. However, data
primitive streak in early development, affects only 1 in 25,000 on neurodevelopmental outcomes in adolescents treated with
live births. However, the consequences of a rapidly expanding hypothermia as neonates have not been ascertained.
teratoma (preterm labor, polyhydramnios, hydrops, fetal
congestive heart failure) are significant to the mother and fetus.
The primary goal of fetal surgery for SCT is interruption of
the large vascular connections between the tumor and the fetus Answer 5-14. b
that result in nonimmune hydrops. Successful fetal surgery for
SCT is accomplished at the time of initial onset of hydrops, Venous cranial infarcts are less common than arterial strokes
not late in the course. Preservation of the anorectal sphincter in neonates. Factors that may predispose neonates to venous
complex, if possible, is attempted, and removal of pelvic strokes include hypovolemia, polycythemia, decreased blood
components is deferred until after the infant delivers. flow (in the setting of preeclampsia), and infection. Magnetic
resonance imaging (MRI) may demonstrate injury to the deep
gray matter and thalamus. Additionally, concomitant magnetic
venography may show occlusion or recanulation of the
Answer 5-12. a affected vessel as well as development of collateral circulation.
The cellular processes associated with neonatal hypoxic At present, there are no definite anticoagulant therapies
ischemic encephalopathy can be divided into 2 separate available for management of neonatal stoke based on data
pathophysiologic phases. Primary energy failure occurs from large randomized clinical trials. Additionally, trials of
secondary to decreases in cerebral blood flow, resulting in erythropoietin in the prevention of neonatal strokes in humans
reduced substrate (ie, glucose) and oxygen delivery to brain are in progress and have not demonstrated clinical benefit in
tissue. Cellular derangements include loss of high-energy reducing effects of stroke.
compounds (such as adenosine triphosphate [ATP] and
phosphocreatine), excessive release or failure of reuptake of
excitatory neurotransmitters (such as glutamate), disturbed
ionic homeostasis (including increases in intracellular calcium), Answer 5-15. c
and decreased cellular protein synthesis. Secondary energy
The neonate in this vignette has findings consistent with
failure, whose severity is related to the degree of primary energy
Beckwith-Wiedemann syndrome (BWS), an overgrowth
failure, results in a constellation of neurodegenerative processes
disorder associated with neonatal hypoglycemia.
(including oxidative injury, neuronal apoptosis, accumulation of
Visceromegaly, including pancreatic hypertrophy, occurs in
excitatory neurotransmitters, and inflammation).
these infants, which is associated with hyperinsulinemia and
hypoglycemia; however, other factors may be involved.
Symptoms may extend beyond the immediate neonatal Answer 5-17. c
period. Hypoglycemia in these infants may improve with more
Neonatal RDS or hyaline membrane disease (HMD) is the
frequent feedings or supplemental glucose administration.
most common cause of respiratory failure in the first days
Fatty acid oxidation defects and impaired glycogenolysis can
after birth, occurring in 1% to 2% of newborn infants. Until
result in neonatal hypoglycemia but are not associated with
about 25 years ago, approximately 50% of infants with this
BWS. The infant’s feeding trial in the office does not indicate
condition died. However, improved methods of treatment
congestive heart failure or swallowing dysfunction as a cause of
over the past 3 decades have markedly reduced mortality
increased caloric demand or impaired calorie intake.
from this condition. RDS occurs mainly in premature infants
and is more common in white infants than in black infants.
The characteristic clinical features of infants with RDS are
expiratory grunting, tachypnea, retractions (involving the
intercostal and sternal muscles), and central cyanosis.
Answer 5-16. b Persistent pulmonary hypertension of the newborn and
transient tachypnea of the newborn are more common causes of
Umbilical venous blood, which is delivered from the placenta,
respiratory distress in near-term (born at 34 0/7-36 6/7 weeks’
has the highest oxygen concentration in fetal blood (see
gestation) and term (born at 37 0/7 weeks’ gestation or later)
figure). This blood is streamed across the right atrium,
infants. Meconium passage in utero before 32 weeks’ gestation is
through the foramen ovale, and into the left atrium, where
rare, making meconium aspiration syndrome less likely in this
this well-oxygenated blood is delivered to the head and upper
infant. Congenital heart disease could present at 6 hours of life,
extremities. Less-oxygenated blood from the vena cava is
but is less common than RDS in an infant of this gestational age.
streamed through the right atrium and ventricle, and then
through the ductus arteriosus to the descending aorta and the
umbilical arteries.
Answer 5-18. a
Pulmonary surfactant, a mixture of proteins and
phospholipids, is synthesized in alveolar epithelial type II cells.
The phospholipid dipalmitoylphosphatidylcholine makes up
Ductus
arteriosus about 45% to 50% of the mass of surfactant and is the main
surfactant component that lowers surface tension. The ratio
of amniotic fluid concentrations of surfactant phospholipids
Superior vena cava (lecithin ([L] and sphingomyelin [S]) has been used to
determine fetal lung maturity.
LA An L:S ratio of 2:1 suggests a lower risk of the fetus
Foramen ovale developing respiratory distress syndrome after birth. Of the 4
RA
surfactant apoproteins (surfactant proteins A, B, C, and D), a
LV congenital deficiency in surfactant protein B production has
RA
Inferior vena cava been associated with fatal respiratory failure in term neonates.
Ductus
Synthesis and storage of surfactant begin around 16 weeks’
venosus gestation, but it is not secreted into the amniotic fluid until 28
to 38 weeks’ gestation, at approximately the same time alveolar
development begins.
Portal Aorta
sinus
Portal v.
Answer 5-19. d
Umbilical aa.
Neonates with respiratory distress should receive the least
invasive therapy that supports their pulmonary needs.
Umbilical v. The infant in this vignette is a late preterm infant with
Hypogastric
aa. clinical evidence of respiratory distress syndrome (RDS).
Oxygenated Physiologically, the infant has atelectasis and hypoxia, which
Mixed can be reversed with positive pressure and oxygen support.
Deoxygenated
Since she is not apneic, the baby should receive respiratory
Placenta support with CPAP and supplemental oxygen to maintain age-
(Reproduced, with permission, from Strange GR, Ahrens WR, Schafermeyer appropriate oxygen saturations. Intubation and mechanical
RW, Wiebe RA, eds. Pediatric Emergency Medicine. 3rd ed. New York: ventilation (either IMV or HFOV) should be started only if
McGraw-Hill, 2009.) she develops worsening respiratory distress (with respiratory
acidosis apnea, or hypoxia) while on CPAP. Nitric oxide is a these infants have higher oxygen consumption and higher
pulmonary vasodilator used for conditions causing respiratory resting energy expenditure due to increased heat loss. Finally,
failure in term newborns (such as meconium aspiration SGA infants have impaired insulin secretion (leading to poor
syndrome) but is not indicated for routine use in premature anabolic growth) and impaired immunologic function during
infants with RDS. Titrating nitrogen into the oxyhood would infancy and childhood (resulting in subtherapeutic response to
decrease the FiO2 and potentially worsen the infant’s respiratory vaccinations).
distress.

Answer 5-23. a
Answer 5-20. d Establishment of glucose homeostasis in the newborn occurs
Small-for-gestational-age (SGA) infants lose heat rapidly via a series of hormonal and enzymatic changes initiated with
because of their large surface area relative to body weight and clamping of the umbilical cord. Basal glucose production in
their scant subcutaneous fat stores. To prevent hypothermia, a neonate is approximately 2- to 3-fold greater than adult
this infant should be dried quickly and completely, placed production, which is related to the large consumption of
under a radiant warmer, and protected from drafts with glucose by the neonatal brain. Hepatic glycogen stores are
warmed blankets. He is crying and has a heart rate above converted to glucose in response to increases in circulating
100 beats/min; therefore, administering PPV, CPAP, or catecholamines and glucagon at delivery as well as decreases
intravenous volume would not be appropriate for his in serum insulin levels. The glycogen stores rapidly decline in
resuscitation in the delivery room. Although SGA infants are the term infant and are nearly exhausted by 12 hours of age,
at risk for hypoglycemia, administration of dextrose in the requiring gluconeogenesis or delivery of exogenous glucose to
delivery room is not indicated. achieve acceptable glucose levels.
Gluconeogenesis is regulated by increases in cytosolic
phosphoenylpyruvate kinase (triggered by delivery) and the
oxidation of free fatty acids (in particular, medium-chain
Answer 5-21. e
triglycerides).
Hypoglycemia is common in small-for-gestational-age
(SGA) infants, including the infant in this vignette. Of note,
hypoglycemia increases with the severity of intrauterine growth
Answer 5-24. d
restriction. The risk of hypoglycemia is greatest during the
first 3 postnatal days, but fasting hypoglycemia may occur The infant in this vignette has developed hyponatremia
repetitively for several days after birth. This early hypoglycemia secondary to inappropriate antidiuretic hormone secretion
usually results from insufficient hepatic and skeletal muscle and renal failure after perinatal depression. Of the choices
glycogen content and is exacerbated by the lack of alternative listed, reduction in intravenous fluid infusion (as well as
energy sources (because of scant adipose tissue and decreased serial monitoring of electrolytes) is the correct choice for
lactate concentrations). Early enteral feeding usually can management of this infant. Administration of additional fluid
prevent hypoglycemia. Less commonly, hyperinsulinemia (a normal saline bolus) is not indicated in a well-perfused
and/or increased sensitivity to insulin may also contribute to infant with intrinsic renal failure (oliguria with a BUN/
hypoglycemia. This insulin is fetally/neonatally derived, as creatinine ratio of 10-20). Diuretic administration (either
maternal insulin does not cross the placenta. Finally, deficient furosemide or bumetanide) is not indicated in infants with
catecholamine responses to low blood sugar levels (seen in intrinsic renal failure; furthermore, diuretics increase renal
SGA infants) may also result in persistent hypoglycemia. sodium losses and may worsen hyponatremia. Increasing the
Increased gluconeogenesis would decrease, not increase, the dextrose in maintenance fluids of a normoglycemic infant may
incidence of hypoglycemia in this infant. cause hyperglycemia, which introduces free water into the
vascular space and may worsen hyponatremia.

Answer 5-22. c
Answer 5-25. d
Small-for-gestational-age (SGA) infants have multiple
metabolic abnormalities in the neonatal period. Protein This infant’s presentation is most consistent with neonatal
digestion and absorption is impaired in these infants due to encephalopathy (probably hypoxic ischemic encephalopathy
decreased intestinal size and function. SGA infants have lower [HIE]). There is no history of a prenatal or intrapartum
muscle masses relative to AGA infants, and although muscle event that would compromise fetal blood flow, but in many
accretion is a priority in these babies, their tolerance of high cases of neonatal encephalopathy, such an event cannot be
protein/amino acid administration may be limited. identified. Information that can be useful in the first hours
SGA babies also have increased plasma triglyceride of life to assist in management of these infants includes
concentrations due to deficient cellular uptake. Additionally, serial neurodevelopmental assessments, neuroimaging,
and electroencephalography. Based on this infant’s initial Answer 5-28. d
presentation (moderate-to-severe encephalopathy with
A complication of 7% to 10% of all pregnancies, preeclampsia
metabolic acidosis), strong consideration should be given to
results from vasoregulatory abnormalities in the placenta
initiating hypothermia.
and the gravid woman. Mild preeclampsia is diagnosed by an
This therapy has shown efficacy if initiated within the first
increase in blood pressure (greater than 140/90 mm Hg) and
6 hours of life (not 12 hours of life), and can be initiated before
proteinuria. The woman in this vignette, however, has evidence
obtaining neuroimaging or an electroencephalogram, especially
of severe preeclampsia (with blood pressure of greater than
if transport to another hospital is required. Neonatal seizures
160/90 mm Hg and 3+-4+ proteinuria).
can occur as a result of HIE, and may exacerbate the brain
Emergent cesarean section is not indicated presently, as
injury. Empiric phenobarbital therapy has been considered
more data should be collected regarding maternal and fetal
for neonates with HIE; however, this strategy has not been
demonstrated to reduce cerebral palsy or mental retardation in status. Medical management of the pregnancy at this time
includes hospital admission, intravenous magnesium sulfate
these infants. Clinicians cannot predict definitively neurologic
administration (to decrease the risk of seizures and provide
outcomes for infants with HIE. However, certain neonatal MRI
findings (as described on page 225), in addition to clinical and fetal neuroprotection), antihypertensive therapy (-blockers,
electroencephalographic evidence of ongoing encephalopathy calcium channel blockers) to control blood pressure, and
(over the first week of life), are poor prognostic signs for monitoring maternal blood pressure, urine output, and end-
neurodevelopment in these neonates. organ (renal, hepatic, hematologic, and neurologic) function.
Fetal well-being, amniotic fluid index, and estimated fetal
weight should also be assessed.
Ultimately, however, if the preeclampsia worsens, delivery
Answer 5-26. b of the fetus would be indicated. If preeclampsia occurs at a
The features of the ultrasound are suggestive of Down gestational age earlier than 34 weeks, maternal corticosteroid
syndrome. The risk of Down syndrome increases with therapy should be given, with delay of delivery for 48 hours
maternal age. Women at high risk of having a fetus with Down (if possible). Antenatal steroids decrease the risk and severity
syndrome will have low serum -FP, high hCG, and low uE of certain conditions seen in premature infants, including
levels adjusted for gestational age. If the maternal plasma respiratory distress syndrome and necrotizing enterocolitis.
tests indicate increased risk, a level 2 ultrasound can identify Hemodialysis is not an appropriate therapy in this patient.
structural findings in the fetus frequently associated with
Down syndrome (increased nuchal fold, nasal bone hypoplasia,
and decreased fetal extremity length). The diagnosis can be Answer 5-29. c
confirmed by karyotype of fetal cells from amniotic fluid. During fetal development, the ductus arteriosus diverts blood
away from the fetal lungs to the descending aorta and placenta
(see Figure 494-3). Postnatally, the ductus arteriosus closes
Answer 5-27. a through vasoconstriction and remodeling. The risk of PDA is
inversely associated with gestational age.
Fetal heart rate and uterine contraction monitoring are used Multiple interacting factors, including increased sensitivity
routinely throughout labor to assess fetal status and the labor to prostaglandins, influence patency of the ductus arteriosus
pattern. While fetal monitoring is not highly predictive of in premature infants. During episodes of sepsis or necrotizing
outcomes, some fetuses at high risk for birth depression can be enterocolitis, circulating prostaglandin E2 concentrations can
identified. The above tocodynamometer tracing shows a fall reach pharmacologic levels in premature infants, causing
in fetal heart rate after the onset of uterine contractions, with vasodilation of the ductus. Under these circumstances,
a gradual return to baseline after the contractions have ceased. responsiveness of ductal tissue to inhibitors of prostaglandin
This pattern is consistent with late decelerations and suggests synthesis (indomethacin or ibuprofen) is limited. Levels
decreased fetal oxygenation due to reduced placental perfusion of indomethacin on day 8, following early administration
(which occurs with uterine contractions). With intermittent of intraventricular hemorrhage prophylaxis, would not be
cord compression, variable decelerations may occur, which adequate to promote PDA closure.
present as abrupt dips in the fetal heart rate (usually a brief Antenatal steroid therapy is associated with a decreased
acceleration followed by a deceleration). (not increased) risk of PDA. Decreased peak end-expiratory
Vagal stimulation due to fetal head compression would pressure after extubation is unlikely to play a major role in
cause an early deceleration (as opposed to a late deceleration). the maintenance of ductal patency. Enteral feeding results in
An early deceleration is manifested as a decrease in fetal heart shunting of blood to the mesenteric circulation. This could
rate associated with the contraction. Normal fetal movement decrease the degree of left-to-right shunting, which could result
would result in an acceleration of the fetal heart rate. Fetal in less pulmonary overcirculation but not increase the risk of
hiccups (singultus) do not decrease the fetal heart rate. prolonged PDA patency.
cardiac anomalies in this infant but is not the most appropriate
initial management step. Finally, administering dextrose may
SVC RPA Aorta be necessary if this large-for-gestational-age (LGA) infant is
PDA
RPV hypoglycemic but is not the most important intervention for
this infant at this time.
LPA
RA LA LPV
Answer 5-32. a
Approximately 10% to 15% of all live births are associated with
meconium-stained amniotic fluid, although only 4% to 5%
of these infants will develop meconium aspiration syndrome.
IVC
LV
Meconium represents the contents of the fetal intestine and
consists of a variety of substances, including bile-containing
intestinal secretions, blood, and amniotic fluid. Meconium
can injure the lung through multiple mechanisms, including
mechanical obstruction of the airways, chemical pneumonitis,
inactivation of endogenous surfactant, and vasoconstriction of
pulmonary vessels, all of which prevent adequate ventilation
and oxygenation in the immediate postnatal period. The ease
FIGURE 494-3. Patent ductus arteriosus. (Reproduced, with permission, of intubation of the baby in the vignette would make in airway
from Rudolph CD, Rudolph A, Lister G, First L, Gershon A. Rudolph’s Pediatrics. edema a less likely answer. Bacterial pneumonia or primary
22nd ed. New York: McGraw-Hill, 2011. <http://www.accesspediatrics.com>.)
surfactant deficiency also would be a less likely cause of this
neonate’s respiratory distress.

Answer 5-30. b
Cyanosis is due to the presence of deoxygenated hemoglobin Answer 5-33. c
in vessels visible on the skin surface and mucosa. Cyanosis in
the neonate tends to become apparent when there is about 3 This infant with a history of shoulder dystocia and a left-sided
to 5 g/dL of deoxygenated hemoglobin, but detection varies Erb’s palsy likely has a brachial plexus injury. In addition to left
widely depending on lighting, observer differences, and arm weakness, the injury in this infant will result in limited
pigmentation of the skin, among other factors. The oxygen left diaphragm movement (due to phrenic nerve injury). A
binding capacity of fetal hemoglobin in the newborn (which is chest radiograph would demonstrate the left hemidiaphragm
higher than the capacity in an adult) also decreases the degree higher than the right hemidiaphragm. Based on the physical
of desaturation at a given Pao2. examination findings, this infant is less likely to have a congenital
diaphragmatic hernia (indicated radiographically by peritoneal
contents in the left hemithorax), right-sided heart congenital
disease or pulmonary hypertension (evidenced by decreased
Answer 5-31. e pulmonary vascular markings), a cystic congenital adenomatoid
The infant in this vignette has central cyanosis with no malformation (CCAM, with cystic lesions in the lung), or a
respiratory distress and a minimal response to supplemental pleural effusion (fluid in the pleural space creating mass effect).
oxygen. These findings suggest cyanotic congenital heart
disease in this neonate. At this point, the caregiver should seek
advice from a cardiologist and transfer the baby to a tertiary
Answer 5-34. a
care center for an echocardiogram and careful monitoring
in a neonatal intensive care unit (NICU). It is important to Fetal neuromaturation prior to term typically progresses in
remember that oxygen may promote ductus arteriosus closure a standard manner with some individual variation. The late
and increase pulmonary vasodilation, which may exacerbate preterm infant in this vignette is likely to have a coordinated
the hypoxia in children with left-sided obstructive lesions (eg, pattern of sucking with discrete pauses for breathing, although
hypoplastic left heart syndrome). Thus, titration of oxygen (to her suck may be weaker than the suck of a term infant.
an FiO2 0.4-0.6) in this infant is appropriate until his cardiac Additionally, she should fixate on a bull’s eye (ability present at
lesion can be defined. 34 weeks’ gestation) with minimal, unsustained tracking that
Admitting a cyanotic infant to a full-term nursery at any will develop over subsequent weeks.
hospital is not appropriate. Additionally, in the absence of In the third trimester of pregnancy, flexor tone increases
respiratory distress, intubation and ventilation is not necessary initially in the lower extremities and progresses in a cephalad
in this infant. A chest CT may be useful in pulmonary and/or direction. Therefore, this infant should not have good flexor
tone in the upper extremities in the absence of full flexion compromised blood flow to the fetus, also increases the risk of
of the lower extremities. Some tone in the shoulders and perinatal stroke in this case.
upper extremities, however, exists by 36 weeks’ gestation, Investigation of neonatal brain injury should always include
and would prevent the infant from stretching her right elbow trauma as a part of the differential diagnosis. In this scenario,
to the contralateral shoulder (the anterior scarf sign). The however, the timing of the seizure and the perinatal risk factors
degree of upper extremity tone will increase over subsequent are most consistent with trauma having occurred around the
weeks, keeping the elbow from reaching the midline when this time of delivery, not due to a postnatal injury.
maneuver is attempted at term. Postnatal strokes can be related to congenital heart
Antenatal magnesium exposure can temporarily alter the lesions and repair, as blood flow can bypass the pulmonary
neurologic examination, and cause decreased tone. However, vasculature. However, postnatal seizure is unlikely to be the
despite magnesium exposure, the primitive reflexes should be presenting symptom.
present in this neonate. Approximately two thirds of neonatal strokes are arterial
in origin, and the remainder are due to sinovenous thrombus.
(Page 180-182, Chapter 5: The Newborn)
This injury is, therefore, less likely to be venous in origin.
Answer 5-35. d Recent work has raised questions of the value of a workup for a
hypercoagulable state in the evaluation of a neonatal stroke.
The resuscitation scenario above raises concern for perinatal There are no large randomized clinical trials of
depression in the baby, which can result in hypoxic-ischemic anticoagulation therapy for neonatal strokes, and there are no
injury to the brain. The baby’s rapid response to positive pressure definitive recommendations for such therapy in this case.
ventilation, however, suggests minimal perinatal depression.
However, in this situation, the infant’s initial examination is (Page 225-226, Section 5: The Newborn)
important for assessment of his baseline neurologic function.
One component of the neurologic examination is assessment of Answer 5-37. d
primitive reflexes, which are normal findings on the newborn Neurologic development of the fetus over the third trimester of
examination. Examples of these reflexes include the palmar pregnancy progresses in a sequence that is generally predictable,
grasp (rapid grasp of the examiner’s finger with pressure on the allowing for estimation of gestational age. The assessment
palm of the baby’s hand) and the asymmetric tonic neck reflex gives the more accurate results after the first few days of life,
(lateralized changes in the baby’s extremity tone with rotation allowing the neonate to recover from perinatal complications or
of his head). Other normal findings on the examination include exposures, and the effect of in utero positioning.
very responsive and brisk deep tendon reflexes (that can produce The infant described in the scenario has weak finger grasp
a few beats of clonus) and intermittent disconjugate gaze if the (also referred to as palmar grasp), which typically emerges at
infant is not focused on an object. As the cerebral cortex matures 34 to 35 weeks’ gestation. Coordinated breathing and sucking
over the first 6 to 9 months of age, the primitive reflexes and typically begins to organize around 32 weeks’ gestation with
periods of dysconjugate gaze should diminish and disappear, and slow sucks, little negative pressure, and brief sucking bursts
then deep tendon reflexes should relax. Newborns can exhibit (fewer than 3 sucks) alternating with 15- to 20-second periods
brief periods of pronounced active trunk extension (when of breathing. This infant demonstrates a sucking pattern more
crying, eg), but in the normal newborn, trunk extensor is not consistent with an infant born at 34 to 35 weeks’ gestation, with
stronger than trunk flexor tone. When evaluated in the quiet, longer bursts, a greater number of more rapid strong sucks,
alert state, extensor tone should be minimal to absent, and flexor and shorter pauses for breathing. Finally, the baby has no flexor
tone should be present. tone elicited in the upper extremities with traction, which is
(Page 182, Section 5: Newborn) typically present by 36 weeks’ gestation.
(Page 181, Examination of the Newborn Infant)
Answer 5-36. e
Signs and symptoms of a neonatal stroke can present shortly Answer 5-38. d
after birth, or later in the neonatal period. When presenting
Closure of the ductus arteriosus in infants is regulated by
symptoms occur days after delivery, perinatal causes should
several factors that change during in utero development. In the
still be considered, especially when there are noted risk factors.
term infant, factors promoting closure of the ductus arteriosus
Instrumented vaginal delivery using forceps or vacuum is
include an increase in arterial blood oxygen tension (Pao2),
associated with increased risk of birth trauma. Because this
a decrease in ductal luminal blood pressure, a decrease in
baby in this vignette required vacuum assistance for delivery,
she has an increased risk of traumatic bleeding that could circulating PGE2, and a decrease in PGE2 receptors.
In contrast, factors that facilitate ductal patency in
result in an intracranial hemorrhage. A perinatal hemorrhage
premature infants include:
may evolve and progress in size hours to days after delivery,
increasing pressure on adjacent normal brain tissue. The 1. Continued synthesis of the dominant PGE2 receptor (E4),
resultant ischemia may result in later clinical evidence of rendering the ductus sensitive to the vasodilatory effects
a stroke (ie, seizures). Preeclampsia, which can result in of PGE2.
2. Higher prostaglandin levels due to less efficient At present, the following factors and strategies have been
metabolism and clearance of circulating prostaglandins identified as protective in premature infants:
and increased PGE2 production due to increased
1. Antenatal corticosteroids (protective mechanism unclear).
expression of the COX-2 isoform.
2. Feeding with human milk (maternal or donor breast
3. The production of increased amounts of other
milk) when compared with feeding with formula.
vasodilators (such as nitric oxide and interleukin-6).
3. Fluid restriction.
4. Absence of the normal fetal cortisol surge, which occurs
4. Administration of enteral antibiotics (not recommended
in the third trimester and initiates physiologic changes
due to concern for the development of resistant organisms).
that are necessary for survival in the extrauterine
5. Treatment with specific probiotic strains (in infants
environment (including ductus closure). Preterm infants
with birth weight less than 1000 g) based on the results
whose mothers receive antenatal betamethasone have a
of a variety of clinical trials. However, there has been
decreased risk of prolonged ductal patency.
no consistency in strain, dose, or regimen in any of the
As pulmonary vascular resistance decreases in premature reported studies, to date.
infants, increased left-to-right shunting and resultant
Other strategies studied (ie, different enteral feeding styles,
pulmonary edema can exacerbate respiratory compromise.
administration of immunoglobulins or amino acid supplements)
Intermittent hypoxemia and additional respiratory support
have not changed the incidence of necrotizing enterocolitis.
that may be needed for management will contribute to lung
injury and the risk of chronic lung disease. In this scenario,
early administration of surfactant would have improved lung
compliance, and the PDA may have become symptomatic Answer 5-41. d
sooner. Surfactant administration, however, does not decrease The most effective prophylaxis against IVH is prolongation of
the likelihood of having a PDA. pregnancy, as the risk of IVH decreases with increased birth
Maternal use of ibuprofen (or other nonsteroidal anti- weight and gestational age. In addition, timely interventions
inflammatory drugs) is associated with intrauterine closure of to treat complications that can lead to preterm delivery are
the ductus arteriosus. associated with lower risk of IVH including administration of
antibiotics to women in preterm labor. Although premature
delivery may occur, antenatal antibiotic therapy may result in a
Answer 5-39. c less severe inflammatory response (a hypothesized component
of the pathophysiologic pathway of IVH), and better
Characteristics of NEC are different in near-term and term hemodynamic stability.
neonates than in premature neonates. An episode of NEC in a Although hypotension is an important risk factor
term or near-term newborn commonly involves the presence for intraventricular hemorrhage, rapid correction with
of a risk factor such as congenital heart disease, gastroschisis, intravenous fluid boluses may compound the risk (due to
intrauterine growth restriction, perinatal depression, or germinal matrix reperfusion injury). Antenatal administration
polycythemia. Additionally, NEC typically presents earlier in of betamethasone is associated with decreased incidence
these babies (first week of life) than in more premature infants of intraventricular hemorrhage. In contrast, postnatal
(second or third week of life). In addition, the area of intestine glucocorticoid administration does not influence the risk
affected is different in more mature infants when compared of intraventricular hemorrhage and can have a detrimental
with premature infants. NEC is more likely to affect the colon in effect on neurodevelopment. Studies that have evaluated other
near-term and term neonates. In premature infants, NEC occurs potential prophylactic therapies such as phenobarbital or fresh
primarily in the jejunum and ileum; the most common site is frozen plasma have not shown any benefit in these agents
the distal ileum. Finally, near-term and term infants with NEC preventing IVH.
have a lower risk of requiring surgery than premature infants.
SIPs are more likely to occur in premature neonates.
However, when present in a more mature neonate, a SIP
occurs earlier (typically 0-3 days of age), and is not usually Answer 5-42. e
accompanied by 1 of the typical risk factors (ie, indomethacin Extremely premature and very-low-birth-weight infants have a
therapy) associated with a SIP in a more premature neonate. significant risk of neurodevelopmental impairment, including
intellectual disability and cerebral palsy. Studies following
very-low-birth-weight survivors into adolescence and
adulthood have revealed more functional limitations in this
Answer 5-40. a
group. These problems include lower academic achievement
Although it has been difficult to elucidate the specific scores (relative to peers born at term), an increased risk
pathophysiology leading to necrotizing enterocolitis in of emotional, behavioral, and attention problems, and
premature infants, multiple studies have aimed to identify risk neurosensory impairment. However, parents of low-birth-
factors and test potential prophylactic measures/therapies. weight neonates report less risk-taking behavior and the
majority of children (74%-82%) graduate from high school and The inability to identify the cause of IUGR may lead to a
pursue productive lives. presumptive diagnosis of uteroplacental insufficiency. Certain
In general, major neurodevelopmental impairments such as findings on placental pathology, such as an infarct noted on
cerebral palsy and intellectual disability can be diagnosed by gross examination, or an inflammatory response seen on
3 years of age. However, more subtle problems with cognition microscopic examination may provide additional information.
and motor function may not present until school age, when However, despite data from placental pathology, the definitive
more complex functioning is necessary. Preterm children with cause(s) of IUGR may not be determined.
no neurologic problems or intellectual disability are more likely
to have language disorders, reading disability, and difficulty
with arithmetic than their peers who were born at term. In
Answer 5-44. c
addition, they have a greater prevalence of visual perceptual
problems, executive dysfunction, and behavior problems than ECMO is a therapy with significant risks that is reserved for
children born at term. Finally, children born to parents with infants with the most profound respiratory or cardiorespiratory
lower socioeconomic status are more likely to have cognitive failure. The overall survival rate for infants who are treated with
impairments as opposed to cerebral palsy. ECMO is 80% in the United States. The most common cause of
Recent studies have better elucidated the risks of death for infants treated with ECMO is hemorrhage. The risk of
neurodevelopmental impairment for late preterm infants neurodevelopmental problems, including cognitive and motor
(who may not be low birth weight). For example, infants born impairment and sensorineural hearing loss, is not considered to
at 33 to 36 weeks’ gestation have a significantly greater risk be greater with ECMO than the risk conferred by the underlying
for reading and spelling problems than infants born at 39 to illness and conventional therapies. Hearing impairment is more
40 weeks’ gestation. Thus, the absence of risk factors such as common in term infants with respiratory and other organ failure
extreme prematurity or low birth weight does not guarantee a than in extremely premature infants. Because less invasive
normal outcome. therapies are being used more effectively for neonatal respiratory
failure, the use of ECMO has decreased in recent years.

Answer 5-43. c
Answer 5-45. d
IUGR can occur at different times in pregnancy. Timing and
Infants born prematurely have a significantly increased risk
etiology of growth restriction can affect the intrauterine growth
of mortality when compared with infants born at term, and
pattern, often manifested in the anthropometric measures of
premature survivors have a significantly increased risk of
the newborn. The relative sparing of head growth in a growth-
neurodevelopmental problems. In addition, the risk of major
restricted fetus (“head-sparing IUGR”) is thought to be a
neurodevelopmental impairments (intellectual disability,
fetal adaptation to uteroplacental insufficiency; as nutritional
cerebral palsy, visual impairment, hearing impairment) increases
supply is limited, somatic growth is sacrificed to allow for
with decreasing birth weight and gestational age at delivery.
brain growth. If nutrient supply is further decreased relative to
High rates of neurodevelopmental impairment occur
fetal needs, accelerated pulmonary and neurologic maturation
primarily in the survivors born at the limit of viability (prior
prepare the fetus for extrauterine survival. However, sparing
to 25 completed weeks’ gestation). Data currently available
of head growth does not completely protect the brain
show that as many as half of these infants have intellectual
from inadequate intrauterine nutrition and subsequent
disability, and up to one quarter of survivors develop cerebral
neurodevelopmental sequelae in childhood. Children who had
palsy (most commonly, spastic diplegia). These infants also
evidence of head-sparing IUGR at delivery have demonstrated
are at the highest risk for complications of prematurity (such
lower cognitive scores on testing when compared with
as chronic lung disease, necrotizing enterocolitis, retinopathy
controls who were born at a size appropriate for gestational
of prematurity, and bacterial sepsis), which represent
age. Furthermore, the acceleration of lung maturity may not
additional risk factors for abnormal neurodevelopment. It is
prevent respiratory complications in this infant, including
important to focus on risk factors in discussing outcomes of
respiratory distress syndrome (RDS).
premature infants with parents, since diagnosis or exclusion
Low birth weight at delivery has been identified as a risk
of neurodevelopmental impairments is not possible prior
factor for various health outcomes in infancy, childhood, and
to NICU discharge. The impact of abnormal radiographic
later life, including the risk of hypertension, diabetes, and
or physical findings in the NICU on neurologic function
cardiovascular disease in adulthood. However, gestational
is typically not apparent until after infancy. Furthermore,
age, which represents the degree of physiologic maturity, is
although MRI of the brain can be helpful for identifying
predictive neonatal mortality and morbidity. In fact, prior to 30
abnormalities that predict neurodevelopmental impairments,
to 32 weeks’ gestation, an infant born small for gestational age
no single neuroimaging study has been recognized as a clinical
does not have a greater risk for mortality, morbidity, or later
practice standard.
neurodevelopmental disability above that conferred by degree
of prematurity.
Answer 5-46. b When cerebral blood flow subsequently increases, additional
reperfusion injury can result in rupture of the germinal matrix
Surfactant production increases in the fetal lung during the
vessels, causing a GM-IVH. In premature infants, germinal
later stages of intrauterine development. Stimulated by the
matrix hemorrhages primarily occur within the first few days
catecholamine surge that accompanies birth and inflation
of life, which may be due to a later adaptation to extrauterine
of the lungs, surfactant is released into the alveolar space,
life that provides stability to this vascular structure. Despite
decreasing surface tension and preventing collapse of the
risk factors such as hemodynamic and respiratory instability
distal air spaces with expiration. Additionally, neonatal lung
that are present in choices b and e, it is less likely for a
compliance is increased due to sustained, regular respirations premature infant to have a primary GM-IVH after 5 days
at birth and resorption of fetal lung fluid across the pulmonary
of age. The infant in choice d had respiratory distress in the
epithelium via transcellular sodium movement. Circulatory
delivery room, but his gestational age (greater than 32 weeks’
adaptations at birth include a reduction in pulmonary vascular
gestation) decreases his risk for a GM-IVH. Furthermore, the
resistance and a change in neonatal blood flow from a fetal
germinal matrix has involuted completely in the majority of
pattern (pulmonary and systemic circulations in parallel) to an
term infants, making putting the infant in choice a at low risk
adult pattern (pulmonary and systemic circulations in series).
for GM-IVH but at higher risk for another form of intracranial
hemorrhage (ie, subgaleal hemorrhage).

Answer 5-47. e
Premature infants in whom a germinal matrix hemorrhage Answer 5-49. d
has occurred require close evaluation, especially during the Necrotizing enterocolitis (NEC) occurs most commonly in
days following the primary bleed (when the hemorrhage is premature infants. The incidence of NEC is inversely related
most likely to extend into the ventricle and cause additional to 2 important risk factors: birth weight and gestational age.
complications). Serial evaluations should include daily In addition, the risk of mortality from NEC is higher for the
measurement of head circumference, and repeat cranial smallest infants born at lower gestational ages. Mortality is also
ultrasound at the end of the first week of life, or sooner if there related to the need for surgical intervention, and the extent
is clinical suspicion of more bleeding (ie, rapid increase in head of bowel involvement, with survival directly related to the
circumference, drop in hematocrit, seizure). length of remaining bowel after surgical resection. Even in the
After the first few days of age, their risk of bleeding on absence of surgical intervention, NEC increases the incidence
the contralateral side does not increase. In the absence of of other prematurity-related complications, including chronic
profound coagulopathy, hemodynamic instability, and/or mass lung disease, growth failure, neurodevelopmental delays, and
effect from the initial bleed, the risk of a primary bleed on the hospital-acquired infections. Additionally, infants with NEC
contralateral side should decrease. who require surgery have a greater risk of neurodevelopmental
Although bleeding that is limited to the germinal matrix impairment (2-3 times greater risk than infants managed
has not been associated with an increased risk of major with medical therapy alone). There is no evidence to suggest
developmental problems, at this stage, it is possible that that hypersensitivity to breast milk is involved in NEC
this infant’s bleed will progress into the ventricles, resulting pathophysiology. Strictures may occur weeks to months after
in posthemorrhagic hydrocephalus, and various related an episode of NEC, even in the absence of bowel perforation.
complications, including a greater risk of neurodevelopmental
impairment. In addition, male infants are more likely to have
neurologic sequelae from intraventricular hemorrhage.
Answer 5-50. b
The infant in this vignette has received inadequate PPV during
the initial steps of resuscitation, resulting in bradycardia.
Answer 5-48. c
Inadequate airway positioning and poor seal with the neonatal
There are many factors that can contribute to the development face mask are the most common causes of a lack of a response
of a GM-IVH as outlined in Figure 58-1. The final common to PPV (which include chest rise and a rise in the neonatal
pathway is their relationship to, or effect on, the delicate heart rate). The first steps in evaluating a poor response to PPV
germinal matrix, the site of origin of this type of intracranial are repositioning the airway (with mild extension of the head
hemorrhage. Perinatal asphyxia and vigorous resuscitation are and neck) and ensuring an appropriate seal between the mask
risk factors for intraventricular hemorrhage in a premature and the infant’s face (see Table 42-3).
baby, as cerebral hemodynamics are influenced by the Of note, the mask should rest snugly around the mouth and
respiratory condition of an infant. Establishing a stable be supported by the chin and the bridge of the nose. Volume
respiratory status in a premature infant can result in hyperoxia infusion and epinephrine administration may be necessary
and hypocarbia, which can cause a decrease in cerebral (to treat hypovolemia and bradycardia, respectively) but are
blood flow (ischemia) through the germinal matrix vessels. not indicated at this point in the resuscitation. Cardioversion
with curling of the tube in the proximal esophageal pouch.
Initial management involves decompression of the proximal
pouch by suctioning the nasogastric tube (to prevent pooling
TABLE 42-3. Strategies to Achieve Chest Rise During Bag–Mask of oral secretions).
Ventilation (MR SOPII) Subsequent steps in management may include contrast
M (mask) Check the seal of your mask imaging of the proximal pouch and esophagus (to delineate
Make sure the infant is truly in the open airway anatomy) and intravenous antibiotics (if pneumonia is suspected
R (reposition)
(mild extension) position due to aspiration of secretions). A barium enema is not indicated
for isolated esophageal atresia. An oral airway is not needed in
S (suction) Remove obstructing secretions this infant with normal pharyngeal findings.
O (open the Sometimes in an effort to get a good seal,
mouth) the mouth is accidentally closed. The higher
resistance of the nasal passages will limit
effective ventilation Answer 5-52. e
P (pressure) Try increasing the inflation pressure if possible Fetal cortisol concentrations increase during development,
I (inflation time) Increase IT to 1-2 seconds for 2-3 breaths in an beginning at the end of the first trimester and rapidly
effort to insure a functional residual capacity is increasing during the final weeks of the third trimester. This
established increase in cortisol facilitates the development of molecular
pathways in several organs, including mechanisms for lung
I (intubate) If all previous steps have failed to achieve chest
fluid clearance at birth and surfactant production. The majority
rise, it is time to intubate!
of fetal cortisol is derived from the fetal adrenal gland, and the
synthetic capacity of the fetal adrenal gland equals the capacity
Reproduced, with permission, from Rudolph CD, Rudolph A, Lister G, First L,
Gershon A. Rudolph’s Pediatrics. 22nd ed. New York: McGraw-Hill, 2011. of the adult adrenal gland. However, placental and/or maternal
steroids may allow for normal development in situations where
fetal steroidogenesis is insufficient.
to correct a neonatal arrhythmia is not performed during the
initial neonatal resuscitation.
Answer 5-53. d
The infant in this vignette has evidence of an Erb’s palsy, an
Answer 5-51. d
acquired injury of the upper portion (cervical roots 5 and 6)
The neonate in this vignette has findings consistent with of the brachial plexus secondary to avulsion or lateral traction
isolated esophageal atresia, which can present prenatally during delivery. This phenomenon often occurs in infants
with polyhydramnios (due to inability of the fetus to swallow during deliveries associated with shoulder dystocia (ie, deliveries
amniotic fluid). Postnatal findings include excessive oral of macrosomic babies). The injury leads to paralysis of the
secretions and respiratory distress. Radiographic findings deltoid, biceps, and brachialis muscles of the affected arm
include failure of a nasoenteric tube to pass to the stomach, and shoulder, resulting in positioning of the affected arm
as described in the question. Associated findings include
ptosis, miosis, and enophthalmos of the ipsilateral eye due
to cervical sympathetic nerve injury (Horner syndrome) and
diaphragmatic paralysis (due to ipsilateral phrenic nerve
injury). The other physical findings are congenital anomalies
not associated with Erb’s palsy.
10-Gauge
Catheter

Blind Upper
Answer 5-54. e
Trachea Esophageal Vertical transmission of hepatitis B infection can be prevented
Segment
with appropriate antenatal serologic screening of pregnant
women. For infants born to HBsAg seropositive mothers,
appropriate immunoprophylaxis includes administration of
FIGURE 392-2. Diagnosis of esophageal atresia is confirmed by inability to hepatitis B vaccine and hepatitis B immune globulin.
pass a 10 French orogastric tube beyond 10 cm from the gums. (Reproduced For a variety of reasons, including inability to access
from Beasley SW. Esophageal atresia and tracheoesophageal fistula. In: maternal records, maternal hepatitis B serostatus may be
Oldham KT, Colombani PM, Foglia RP, et al, eds. Principles and Practice of unknown at the time of delivery. In these cases, HBsAg
Pediatric Surgery. Philadelphia: Lippincott Williams & Wilkins; 2005:1040.) serology on the mother should be drawn immediately on
admission for delivery, and the hepatitis B vaccine should be bradycardic after tracheal aspiration and ventilation. Nasogastric
administered to the neonate within 12 hours of age. Guidelines tube placement would be indicated to decompress the stomach
for administration of hepatitis B immune globulin are based after prolonged bag–mask ventilation.
on infant weight (using a weight cutoff of 2000 g) as well as the
timing of determination of maternal serology status.
Because of the variable immune response to the hepatitis
Answer 5-57. b
B vaccine in neonates born under 2000 g, hepatitis B immune
globulin should be given to these infants if maternal serology The infant in this vignette has a normal abdominal
status cannot be determined within 12 hours. For infants with examination and, in the absence of any other abnormal
a birth weight over 2000 g, immune globulin administration physical findings, should be discharged home with her parents.
may be delayed if the maternal serology status can be The finding of a liver edge 1-2 cm below the costal margin
determined within 7 days. Infants born to mothers who are is unremarkable. The umbilical stump in a neonate should
HBsAg negative should only receive the hepatitis B vaccine. be dry; moisture around the umbilical stump may be due to
Maternal chorioamnionitis and hepatitis C seropositive status a patent urachus or a mucosal cyst. The lower poles of each
are not indications for administration of hepatitis B immune kidney may be palpable, with the right kidney higher than
globulin. the left kidney. The baby has a diastasis recti, a gap in the
abdominal rectus muscles that is common in the neonatal
period and does not require surgical intervention.
Answer 5-55. a (Page 178-179, Chapter 5: The Newborn)
The infant of a diabetic mother in this vignette most likely is
experiencing seizures due to hypocalcemia. Calcium is actively Answer 5-58. e
transported across the placenta from the mother to the fetus, Cigarette smoking remains a major cause of prenatal, perinatal,
with a corresponding decrease in fetal parathyroid hormone and infant morbidity and mortality, including IUGR. Initiation
(PTH). At birth, neonatal PTH levels surge to account for the of a smoking cessation program is an appropriate step to reduce
decreased transplacental calcium delivery. However, relative to the risk of IUGR in this mother. Prenatal alcohol exposure
normal infants, infants of diabetic mothers have reduced PTH increases the risk of growth restriction, including microcephaly,
levels as well as decreased end-organ sensitivity to PTH. The and should be avoided in pregnancy. Folic acid supplementation
resultant hypocalcemia manifests clinically at 48 to 72 hours reduces development of congenital spine and brain defects but
of age with neurologic changes (irritability, tremors, twitching, does not reduce the risk of fetal growth restriction. -Carotene
and/or seizures) and cardiac arrhythmias. Prematurity, promotes fetal growth and immune development and should be
perinatal depression, and concomitant hypomagnesemia may consumed in appropriate amounts in pregnancy. Consumption
worsen the hypocalcemia. Serum calcium levels should be of a strict vegan diet may place the fetus at risk for poor growth;
monitored in infants of diabetic mothers daily during the first appropriate supplementation of the diet with nonmeat-derived
72 to 96 hours of age. protein (ie, legumes, soy) is required in pregnancy.
A complete blood count and C-reactive protein are not
indicated in this infant with low risk factors for perinatal
bacterial infection and no other clinical signs of sepsis. Since
Answer 5-59. c
the infant is feeding well, hyponatremia is less likely to be a
cause of her seizures. Imaging of the head in an infant with a Neonatal and infant mortality rates have steadily declined over
nonfocal neurologic examination is not warranted. the past 10 years (see Figure 41-1), largely due to advances in
neonatal care. The advent of surfactant therapy and antenatal
corticosteroids to decrease the incidence of respiratory distress
syndrome has reduced significantly neonatal mortality.
Answer 5-56. e
Furthermore, mortality among low-birth-weight infants has
The nonvigorous infant in this vignette requires intubation decreased in NICU centers providing high levels of care and
and aspiration of the trachea for aspirated meconium from the treating high volumes of these at-risk infants. However, certain
amniotic fluid. He demonstrates signs of secondary apnea at birth factors, including male sex, African American ethnicity, late
(apnea, hypotonia, and bradycardia [heart rate 50 beats/min]), (after the first trimester of pregnancy) or no prenatal care,
putting him at risk for meconium aspiration syndrome. This multiple gestation, and low birth weight, increase the risk of
baby requires endotracheal intubation and suctioning on arrival neonatal and infant mortality.
at the delivery room table. Suctioning of the mouth and nose
of a meconium-stained infant would be performed only if the
infant was vigorous (ie, demonstrating normal respiratory
Answer 5-60. c
effort, muscle tone, and heart rate [over 100 beats/min]). Chest
compressions and umbilical vein catheterization (for epinephrine Metabolic acidosis is defined as an increase in serum
administration) are indicated only if the infant remains hydrogen (H+) ion or a decrease in serum bicarbonate,
resulting in a decrease in serum pH. A normal ion Answer 5-63. b
gap metabolic acidosis occurs in the setting of renal or
The infant in this vignette developed anasarca as a result
gastrointestinal bicarbonate losses and is characterized by
of an in utero tachyarrhythmia. However, delivery room
hyperchloremia. Preterm neonates (as in case C) often develop
management of this infant does not differ from the
a mild, proximal renal tubular acidosis, with urine bicarbonate
management of other infants with ineffective ventilation. This
wasting and a low serum pH. In contrast, an increased ion
infant may have respiratory failure due to respiratory distress
gap metabolic acidosis develops due to production of lactate
syndrome and pleural effusions (secondary to arrhythmia-
(from sepsis, hypoxia–ischemia, tissue damage, or a congenital
induced congestive heart failure). Thus, intubation and
heart defect) or another anion (from an inborn error of
mechanical ventilation are the appropriate steps at this point
metabolism [IEM]). The other infants have conditions (aortic
in the resuscitation. Neither blow-by nor nasal cannula oxygen
coarctation, patent ductus arteriosus, IEM, and bacterial
will sufficiently support the degree of respiratory distress in
sepsis, respectively) that will produce a serum anion gap and
this neonate. Epinephrine only should be administered after
metabolic acidosis.
adequate ventilation and chest compressions fail to improve
neonatal bradycardia. Infusion of adenosine (to convert
SVT into a sinus rhythm) is not indicated in delivery room
Answer 5-61. d resuscitation of a neonate, including those with fetal SVT.
The neonate in this vignette has an abnormal neurologic
examination, which may occur secondary to abnormalities
in levels of serum electrolytes, glucose, or amino acids. Answer 5-64. a
However, of the values listed, the serum calcium level is the
In general, due to the increased surface area of the newborn
abnormal laboratory value most associated with this neonate’s
relative to a newborn’s body weight, heat production in the
presentation. His hypercalcemia has occurred secondary to
neonate is greater than heat production in the adult.
subcutaneous fat necrosis (in the indurated cheek nodules)
Nonshivering thermogenesis is generally accepted as the
associated with perinatal trauma. Granulomata in the
major source of heat generation in neonates. An important
necrotic fat cause parathyroid hormone (PTH)–independent
protein involved in nonshivering thermogenesis is UCP1, an
overproduction of 1,25-dihydroxyvitamin D. Presentation of
ion transport protein that generates heat through entry of
the hypercalcemia may range from mild neurologic changes
seen here to hypertension, seizures, respiratory distress, and protons into mitochondria. Brown fat, the major tissue involved
in nonshivering thermogenesis, also is relatively more abundant
nephrocalcinosis. Management includes hydration, increasing
in the neonate than in the adult. Energy generation in neonates
urinary calcium excretion, and restriction of dietary calcium
is also augmented by intracellular free fatty acids, which are
intake.
produced by elevated levels of lipoprotein lipase at birth.
Circulating free fatty acids replenish depleted intracellular
energy stores but do not serve as an acute source of energy.
Answer 5-62. e The neonate, however, has thermogenic disadvantages
relative to adults. Due to an increased surface area relative to
The premature neonate in this vignette has signs of systemic
body weight, newborns must generate more heat to maintain
hypotension. The cardiovascular system of the preterm infant
a normal body temperature. Additionally, the absolute extent
has adapted in utero to the low-resistance state of the placenta.
to which a neonate can maintain a normal body temperature
With the clamping of the umbilical cord, the premature
during cold stress is limited.
myocardium is exposed to a high-resistance ex utero state, and
is generally unable to readily adapt to the change. Initiation
of an inotrope (in this case, dopamine) will provide cardiac
stimulation and vasopressor effects that will improve blood Answer 5-65. b
pressure and perfusion.
Although preterm delivery can occur in both singleton and
The myocardium of the preterm infant is impacted by
multifetal pregnancies, the preterm birth rates for multifetal
positive pressure ventilation; increases in the PIP and PEEP
gestations are significantly greater. Recent data have indicated
will increase intrathoracic pressure, decreasing cardiac output
that most multifetal gestations are born prematurely, with
and perfusion. Sepsis is a common cause of hypotension and
preterm birth rates for twins and triplets reported as 60.5%
hypoperfusion in the neonate, but immune globulin has not
and 93.7%, respectively (see Table 46-1). These preterm
been shown to reverse the circulatory collapse associated
infants are at risk for developing all of the complications
with early onset or late-onset infections. Maintaining the
typically associated with prematurity, including poor long-
patency of the ductus arteriosus with a PGE1 infusion will
term neurodevelopmental outcomes. Twin–twin transfusion
allow left-to-right shunting of blood during the cardiac cycle,
syndrome typically occurs in monozygotic twinning when
lowering systemic blood pressure.
a vascular connection between twins develops in the shared
placenta. Dizygotic twins account for two thirds of all
spontaneous twin births. In association with the use of assisted is not indicated. Supplemental oxygen may improve the infant’s
fertilization and advanced maternal age, there has been an pulse oxygen saturations but would not provide consistent and
overall increase in the incidence of multifetal gestations since adequate positive pressure to achieve airway patency.
the 1980s. Fetuses of multifetal gestations are more likely to
have breech or other malpresentations at delivery, contributing
to a higher rate of cesarean deliveries.
Answer 5-68. e
The infant in this vignette has lung injury secondary to
congenital pneumonia. As his pulmonary infection resolves,
Answer 5-66. d his lung compliance changes, leading to variable tidal volumes
The infant is this vignette has become apneic secondary to despite stable ventilator settings. With pressure-limited
ventilatory insufficiency. Surfactant deficiency and increased ventilation, the volume of gas delivered depends on the
chest wall compliance have led to atelectasis and diminished underlying lung compliance (ie, highly compliant lungs will
gas exchange, which are hallmarks of respiratory distress receive a greater volume of gas at a given pressure). Volume-
syndrome. Positive pressure ventilation is necessary until targeted ventilation attempts to deliver consistent volumes
the infant’s own respirations can support gas exchange. of gas into the lungs despite changing lung compliance. For
Endotracheal intubation will also allow for the administration example, if lung compliance abruptly increases, the ventilator
of surfactant. Exogenous surfactant administration can will maintain the same inhaled volume of gas by reducing
reduce the surface tension of the alveoli and improve the pressure used during inspiration. Intermittent mandatory
functional residual capacity. CPAP and nasal cannula ventilation, high-frequency oscillation, and pressure support all
oxygen would not be appropriate modes of ventilation for an are forms of pressure-limited ventilation that would not be able
apneic patient as they rely on the patient to spontaneously to respond to changes in lung compliance.
breathe for gas exchange. Tension pneumothoraces can be
a complication of mechanical ventilation and should be
suspected when asymmetric breath sounds, asymmetric
Answer 5-69. c
chest wall contours, hypotension, and hypoxia develop
acutely. Inhaled nitric oxide, a vasodilator used to reduce The infant in this vignette developed persistent pulmonary
the pulmonary vascular tone in infants with persistent hypertension of the newborn (PPHN) secondary to
pulmonary hypertension of the newborn, is not indicated meconium aspiration syndrome. PPHN is characterized
during delivery room resuscitation. by persistently elevated pulmonary vascular tone causing
intrapulmonary or extrapulmonary right-to-left shunting.
The overall goals of therapy are to optimize lung inflation
and lower pulmonary vascular resistance. High-frequency
Answer 5-67. c oscillation is being used in this vignette to improve lung
The infant in this vignette is a preterm infant with respiratory inflation. Techniques to reduce pulmonary vascular tone
distress syndrome (RDS) and subsequent CO2 retention. include avoiding hypercarbia, hypoxia, and acidosis. Another
Additionally, she has developed radiographic changes in her therapy is inhaled nitric oxide, a selective pulmonary
lung fields and prolonged ventilator dependency. Over time, vasodilator. It increases the activity of soluble guanylate
infants with RDS develop areas of inflammation, mucous cyclase, leading to increases in cyclic GMP (cGMP), an
plugging, and airway narrowing. Such infants may experience important promoter of smooth muscle relaxation. Inhaled
worsening of respiratory symptoms following extubation, as nitric oxide has become an important therapy in the
removal of the positive pressure can reduce airway patency. management of PPHN. Diuretics that block transporters in
Without intervention, the infant may develop worsening the nephron (chlorothiazide to block the Na+–Cl− symporter
atelectasis and severe respiratory distress. CPAP delivered or furosemide to block the Na+–K+–2Cl− cotransporter)
noninvasively (via nasal prongs or mask) at pressures of 4 to have a limited role in PPHN. Opiates, which activate mu
8 cm H2O can often provide enough support to distend the opioid receptors, may be used for sedating patients who have
airways and reduce alveolar atelectasis. difficulty tolerating high-frequency ventilation but are not
Common indications for use of CPAP in neonates specifically therapeutic in PPHN. Albuterol, a 2-adrenergic
include supporting mild RDS or narrowing of the airways, agonist, is used for patients with airway hyperresponsiveness
transitioning from mechanical ventilation, and decreasing and has a role in the management of chronic lung disease.
the incidence or severity of apnea and bradycardia in some
preterm infants. If CPAP did not resolve the infant’s symptoms
(or if symptoms worsened) over a defined period of time, then
Answer 5-70. c
endotracheal intubation and mechanical ventilation would
be appropriate management. As the infant’s lung disease was Infants with SIP clinically present with pneumoperitoneum,
improving with SIMV mode, ventilation with high frequency making the differentiation between SIP and advanced NEC
challenging. The infant in this vignette has several features iron deficiency anemia with a hemoglobin level is done at 9 to
and risk factors to suggest that the intestinal perforation is 12 months of age. Vitamin D supplementation is often needed
a consequence of a SIP, including timing of the perforation to reach the recommended amount of vitamin D intake,
(7-10 days) after indomethacin and glucocorticoid exposure, 400 IU daily; however, screening serum vitamin D levels is not
poor intrauterine growth, and extremely low birth weight. recommended. C-reactive protein can be used as a screening
Conversely, infants who develop NEC typically have received test for sepsis in the neonatal period. Screening for lead toxicity
enteral feeds prior to presentation and may demonstrate is performed between 1 and 2 years of age for infants with risk
radiographic features such as portal venous gas or pneumatosis of environmental lead exposure.
intestinalis.
(Page 231, Section 5: Newborn)
The mucosal margins of the intestinal perforation appear
healthy in a SIP (as opposed to the coagulation necrosis
observed in NEC). This histopathologic difference also helps Answer 5-73. b
to distinguish the 2 entities. Additionally, infants with SIP have Factors such as prematurity and being the infant of a diabetic
decreased mortality compared with infants with NEC-related mother contribute to increased bilirubin production in this
intestinal perforation. Volvulus and duodenal atresia in the infant, leading to neonatal jaundice. Based on the AAP
newborn period require surgical management; however, they practice guidelines for infants with hyperbilirubinemia (see
typically present with symptoms of intestinal obstruction, Figure 53-3), phototherapy should be initiated in this infant.
including vomiting and bilious aspirates. As with NEC, cow’s Phototherapy converts the bilirubin to lumirubin for excretion.
milk protein allergy can present with hematochezia and If the bilirubin level rises too quickly (greater than 0.2 mg/
feeding intolerance, but this illness does not involve intestinal dL/h) despite intensive phototherapy, IV Ig or exchange
perforation. transfusion may be needed to decrease any circulating
antibodies that cause hemolysis and/or reduce the serum
bilirubin concentration. The infant is well hydrated, voiding,
stooling, and does not have excessive weight loss; therefore, IV
Answer 5-71. a hydration or discontinuation of breast-feeding is unnecessary
The infant experienced delayed cord clamping during his at this time.
home delivery, which allowed for the transfusion of excess red
blood cell volume. Polycythemia, defined as a central venous
hematocrit of greater than 65%, can result in hyperviscosity
syndrome, or organ dysfunction due to impaired blood Answer 5-74. c
flow. Symptoms include neurologic changes (as described This infant’s fetal anemia, bilious amniotic fluid, and features
in the vignette), respiratory impairment, feeding difficulty, of hydrops suggest erythroblastosis fetalis, resulting from
and thrombocytopenia. A cranial sonogram can be used the hemolysis of fetal red blood cells by maternally derived
to diagnose intracranial causes of neonatal irritability Rh antigens. Mothers who are Rh-negative may become
and lethargy such as acute hemorrhage and sinus venous sensitized to the most significant Rh protein, anti-D antigen, if
thrombosis. Congenital hypothyroidism can present with Rh-positive fetal blood cells enter into the maternal circulation.
hypotonia, lethargy, and poor feeding, but the symptoms often However, Rh sensitization has been largely decreased in the
present days to weeks after birth. A stool occult blood test United States thanks to the use of anti-D immune globulin
can help determine a source of bleeding in an anemic infant given to Rh-negative mothers at 28 weeks’ gestation and after
but is not a useful test for polycythemia. A chest radiograph the birth of an Rh-positive infant.
would be helpful to rule out alternative causes of the infant’s Viruses such as parvovirus and cytomegalovirus are
respiratory distress but would not be helpful for the diagnosis significant causes of fetal hydrops, but the associated anemia
of polycythemia. results from decreased erythrocyte production rather
than hemolysis. Complex congenital heart disease and
supraventricular tachycardia may cause fetal hydrops but do
not typically involve hemolytic anemia.
Answer 5-72. e
Given the risk of kernicterus in the newborn period, significant
efforts should be made to identify all infants with pathologic
Answer 5-75. e
serum bilirubin levels. Risk factors for hyperbilirubinemia
include a family history of neonatal jaundice, East Thrombocytopenia in the neonate can result from either
Asian ethnicity, male gender, blood extravasation (ie, decreased production or increased destruction of platelets.
cephalohematoma), prematurity, exclusive breast-feeding, and Kasabach-Merritt syndrome, also known as giant hemangioma
postnatal weight loss. The infant in this vignette has multiple syndrome, is characterized by the presence of a vascular
risk factors for hyperbilirubinemia, making a total serum tumor that sequesters platelets, leading to significant
bilirubin level an appropriate screening test. Screening for thrombocytopenia. In this case, the trapping of platelets
and consumptive coagulaopathy end in platelet destruction. radiographic findings (bilateral granular lung fields) make RDS
There may also be a microangiopathic hemolytic anemia less likely in this case.
associated with the presence of the mass. Trisomies 13 and 18,
Fanconi syndrome, and TAR syndrome all may present with
thrombocytopenia but are not associated with the presence of Answer 5-77. a
vascular malformations.
The infant in this vignette has a pneumothorax, one type
of air leak syndrome. Pulmonary air leaks can result from
any lung disease in newborns, particularly in those who
Answer 5-76. b require mechanical ventilation. The size of the pneumothorax
at presentation can vary, as does the degree of clinical
When labor begins, sodium–potassium exchange occurs at
compromise. Pneumothoraces can cause significant respiratory
the basolateral membrane of the lung epithelium allowing for
distress, hypoxemia, hypercarbia, or hypotension, warranting
active transport of sodium across the apical surface via sodium
emergent decompression. If the infant’s clinical deterioration
channels. Subsequently, water is drawn from the air spaces into
the lung interstitium, clearing fetal lung fluid in anticipation precludes chest tube placement in a controlled setting, needle
of air exchange. This process is activated by a surge of decompression of the affected hemithorax should be performed
catecholamines and other hormones during labor. immediately as a temporizing measure. Increasing the mean
airway pressure will not resolve the infant’s pneumothorax and
The infant in this vignette was born by cesarean section
may contribute to worsening air leak.
without a period of labor, which increases his risk for transient
tachypnea of the newborn (TTN). The mild respiratory Pericardiocentesis is indicated for a pneumopericardium
alkalosis and radiographic changes noted in this vignette (air collection in the pericardial space) associated with cardiac
support this diagnosis. Chemical pneumonitis can occur as tamponade, which this infant does not demonstrate based on
a result of meconium aspiration syndrome, which did not clinical examination and radiographic data. Stabilization of the
occur in this case. Ascending infection through the birth canal airway, drainage of gastric contents, and surgical consultation
are initial steps in the management of congenital diaphragmatic
can cause congenital pneumonia, which is less likely in this
hernia, which is demonstrated radiographically by bowel (or
neonate due to the rupture of amniotic membranes at delivery.
other abdominal contents) in the thorax. Intubation of the
Increased pulmonary vascular tone results in persistent
right main stem bronchus may cause decreased left-sided
pulmonary hypertension of the newborn, which is less likely
in this case given the normal pulse oximetry readings. High breath sounds and respiratory insufficiency; however, this
alveolar surface tension due to surfactant deficiency can infant’s endotracheal tube is appropriately placed as visualized
on the x-ray.
result in respiratory distress syndrome (RDS). However, the
absence of a supplemental oxygen requirement and traditional

Lumen

Cl C l–
Na+ Na+ Na+
H2O H2O

2K +

K+
ATPase

3Na+

Interstitium

FIGURE 50-1. Model of fetal lung fluid absorption by epithelial cells. Fluid absorption results from vectorial transport of Na+, driven by Na+/K+-ATPase. The
resultant electrochemically increased gradient leads to passive Na+ absorption via apical Na+-permeant channels that is extruded by Na+/K+-ATPase out of the cell.
Cl and water passively follow the Na+ ions through paracellular or intracellular pathway. (Reproduced, with permission, from Rudolph CD, Rudolph A, Lister G,
First L, Gershon A. Rudolph’s Pediatrics. 22nd ed. New York: McGraw-Hill, 2011.)
Answer 5-78. a hemorrhage, as increased pressures are transmitted to the lung
capillaries and generate injury. Conditions that precipitate
The infant in this vignette has developed idiopathic persistent
left heart failure such as cocaine exposure, sepsis, asphyxia,
pulmonary hypertension of the newborn (PPHN). Constriction
and congenital heart disease are associated with pulmonary
of the fetal ductus arteriosus by nonsteroidal anti-inflammatory
hemorrhage. However, preterm infants with persistence of
drug exposure is 1 possible cause of idiopathic PPHN. Findings
a large patent ductus arteriosus (PDA) are at particular risk
of idiopathic PPHN include cyanosis and respiratory distress
for pulmonary hemorrhage. In neonates with large PDAs,
within hours of birth, differential cyanosis or pulse oximetry
exposure of the lungs to high pressure and high blood flow
indicating right-to-left shunt, and reduced arterial oxygenation
results in microvascular injury that leads to hemorrhage.
despite relatively normal pco2 and/or metabolic acidosis.
X-rays of the chest may reveal undervascularized lung fields (Page 203-204, Section 5: Newborn)
with hyperlucency (depending on the presence or absence of
additional lung pathology). Echocardiography with Doppler Answer 5-80. a
flow studies can confirm features of PPHN such as intracardiac
The infant in this vignette has developed a pulmonary
right-to-left shunts and tricuspid valve regurgitation, which
hemorrhage following the administration of surfactant,
suggest elevated right-sided pressures. Serial blood gas
which is a recognized risk factor. As pulmonary hemorrhage
sampling and pulse oximetry are important clues to the
is a potentially fatal event, rapid measures should be taken
diagnosis of PPHN and are useful in ongoing management,
to stabilize the patient and correct any known causes of the
but are nonspecific and do not estimate pulmonary vascular
hemorrhage. Specific measures to treat pulmonary hemorrhage
resistance. Tracheal aspirates and blood cultures may identify
include maintaining adequate mean airway pressure to
an infectious etiology for an infant with PPHN but do not
support lung volume and gas exchange, restoring circulating
assess pulmonary vascular pressures.
blood volume, and ameliorating coagulopathy. Management
of predisposing factors also should be considered, such as
closure of a large patent ductus arteriosus with indomethacin.
Answer 5-79. d However, without a patent airway none of these interventions
will sufficiently provide adequate cardiorespiratory support for
Pulmonary hemorrhage, a potentially fatal complication, is
the patient. Therefore, the first step in the management of this
a chief cause of death in 9% of neonatal autopsies. It most
infant is to suction and secure the airway.
commonly occurs between days 2 and 4 of life. Left ventricular
failure is thought to be an inciting factor in pulmonary
Neonatoloy Revision
Dr.Wahid Helmi Egypt
Consultant Pediatrician
1) Hepatitis B infection all false except:-
A. Increase incidence of malformed & stillbirth
B. Increase the risk of prematurity
C. Hepatitis B immunization & vaccine should be given in 1 st week
D. 25% babies born to mother with HBsAg positive are HBsAg negative
E. Presence of HBeAg has no role in increase transplacentalcross infection
Ans:-B

2) The following should be investigated in five day old baby:-


A. Erythema Toxicum
B. Cloudy cornea
C. Divarication of rectii
D. Subconjunctival hemorrhage
E. Preauricular skin tags
Ans:-BE
3) Regarding Tetracyclin all are true except:-
A. It is selectively concentrated in the teeth by chelating calcium
B. It effect decidual teeth
C. It effect permanent teeth
D. Has no effect on eye ( CAUSE CATARACT)
E. It depress skeletal growth
Ans:-D

4) Necrotizing enterocloitis is recognized complication of:-


A. Polycythemia
B. Umbilical arterial catheterization
C. Parenteral feeding
D. Maternal Crohn's disease
E. Birth asphyxia in term neonate
Ans:- ABE
5) The following is true regarding changes in the fetal cardiovascular system after
birth:
A. There is normally immediate closure of the ductus arteriosus
B. Hypoxia-induced vasoconstriction is the mechanism of closure of the ductus
arteriosus
C. The ligamentum teres is the remnant of the umbilical vein
D. Regression of right ventricular hypertrophy occurs postnatally
E. Inferior vena caval pressure falls after birth
Ans:-CDE
Occlusion of the umbilical cord removes this low resistance capillary bed from the
circulation;breathing results in a marked decrease in pulmonary vascular resistance, hence
there is increased pulmonary blood flow returning to the left atrium raising the pressure in
the left atrium causing the foramen ovale to close. As pressure in the systemic circulation
rises, shunt through the ductus arteriosus reverses. As the pO2 rises, synthesis of
bradykinin and prostacyclins is inhibited, thus causing closure of the ductus arteriosus.
The ductus arteriosus can take up to 3 months to close in normal neonates.
6) Neonatal RDS:-
A. Seen in most babies of birth weight < 2.5 kg.
B. More common in infants of diabetic mothers.
C. Associated with prolonged rupture of membrane.
D. Less sever in babies of Afro-Caribiean origin than Caucasians.
E. Exacerbated by hypothermia.
Ans:-BDE
7) The following are causes of generalized hypotonia in 2 days old infants:-
A. Prematurity
B. Hypothyroidism
C. Myotonic dystrophy
D. Spinal dysraphyism
E. Anterior horn cell disease
Ans:-ACE
8) Regarding surfactant:-
A. Production begins at 30 weeks
B. It is produced by Type II pneumocytes in the walls of the bronchi
C. Testosterone stimulates surfactant production
D. Production is increased during a stressful event like hypothermia
E. Betamathasone given to the mother improves surfactant production in the
premature baby
Ans:- E
Surfactant production begins at 20-22 weeks. It is produced by Type II pneumocytes
which are in the walls of the alveoli. The hormones testosterone and insulin inhibit
surfactant production;hence hyaline membrane disease is more common in males than
females and more common in infants of diabetic mothers. Surfactant production is
suppressed if the baby is hypothermic,hypoxic, acidotic or hypoglycemic.Although
dexamethasone is more commonly used, betamethasone has an identical effect on lung
maturation
9) Concerning fetal lung development:-
A. Type ΙΙ pneumocytes are present at 24 week gestation
B. Cuboidal cells are capable of gas transfer in utero
C. There is virtually no smooth muscle in the terminal & respiratory bronchioles at
6 month of age
D. The large airways are formed at 16 week gestation
E. Alveoli are completely formed at birth
Ans:-AD
10) Congenital CMV infection:-
A. Only 10% of affected pregnancies have resulting long term sequel at birth
B. Diagnosis is by viral isolation from the urine
C. Hearing loss can develop gradually over the first 5 years
D. The affected newborn should be treated with ganciclovir
E. Intracranial calcifications are seen in a periventricular distribution
Ans:- ABCE
Congenital CMV occurs in approximately 1% of all live births and only 10% of these
infections result in clinical symptoms. Severe clinical disease is associated with primary
maternal infection in pregnancy. Infection in early gestation carries a far greater risk of
severe fetal disease. In CMV intra-cranial calcifications are in a periventricular
distribution. Ganciclovir is only used if there is CNS involvement, chorioretinitis or
pneumonitis.
11) The following conditions will present with cyanosis in the first week of life:
A. Aortic stenosis
B. Transposition of the great vessels
C. Hypoplastic left heart syndrome
D. Fallot's tetralogy
E. Fallot's pentalogy
Ans:- B
Any cardiac lesion which allows a mixing of blood along with a right to left flow or any
cardiac lesion wherein pulmonary perfusion is impaired results in cyanosis. Left heart
problems or outflow tract obstructions present as cardiac failure. Fallot's pentalogy
includes an ASD along with the tetrad of infundibular pulmonary stenosis, RVH, over-
riding of the aorta and a VSD.Babies with tetralogy of Fallot usually have a patent ductus
arteriosus at birth that provides additional pulmonary blood flow, so severe cyanosis is
rare early after birth.As the ductus arteriosus closes, as it typically will in the first days of
life, cyanosis can develop or
become more severe.The degree of cyanosis is proportional to lung blood flow and thus
depends upon the degree of narrowing of the outflow tract to the pulmonary arteries.
12) Pulmonary surfactant
A. Is partly recycled by endocytosis into the synthesizing cell
B. Is produced by type Ι alveolar cells
C. Reduction in pulmonary flow can cause a decrease in surfactant production
D. Synthesis is inhibited by thyroxine
E. Synthesis is stimulated by glucocorticoids
Ans:- ACE 5Dipalmityl- phosphotidyl choline is the main component of surfactant and is
produced by Type-ΙΙ alveolar cells (granular pneumocytes). Its half-life is 14 hours and its
main function is to reduce the surface tension of the alveoli.
13) Lung surfactant
A. Decreases the surface tension within an alveolus
B. Causes an increase in chest wall compliance
C. Is a glycoprotein
D. Maintains the same surface tension for different sized alveoli
E. Appears only after the 1st week of life
Ans:- A
Surfactant is a dipalmitoyl-phosphatidyl choline and is a phospholipid, which prevents
alveolar collapse by reducing alveoli surface tension. It is produced by type-II
pneumocytes and is seen at about 24 weeks gestation. It causes an increase in lung
compliance only (not chest wall compliance).
14) The following organisms cause conjunctivitis:-
A. Epstein Barr virus
B. Chlamydia trachomatis
C. Adenovirus
D. Haemophilus influenzae
E. Neisseria gonorrhoeae
Ans:- BCDE
Chlamydia trachomatis causes conjunctivitis in 30-50% of neonates born to mothers with
cervicitis. It is a purulent conjunctivitis, which develops 5-14 days after birth and is
indistinguishable from gonococcal infection. It is diagnosed on a swab scraped over the
lower eyelid (to allow cells to be collected – don’t forget it is an intracellular organism) by
direct fluorescent antibody, ELISA or PCR. Tetracycline ointment topically is combined
with oral erythromycin – the oral antibiotic is to prevent relapse after ointment is
discontinued and to prevent progression to pneumonia. Gonococcal conjunctivitis presents
earlier than chlamydial disease (usually within 2 days), is diagnosed on gram stain and
culture and should be treated with IV penicillin and chloramphenicol eye drops. Don’t
forget sexual health screening for the mother and informing public health of ophthalmia
neonatorum. Adenovirus causes conjunctivitis in summer outbreaks; enterovirus,
coxsackie and herpes simplex are other viral causes.
15) Concerning blood flow in the fetus:-
A. Blood flow from right to left through the foramen ovale
B. Blood in the ascending aorta has higher oxygen content than in the descending
aorta
C. The ductus arteriosus is closed
D. Pulmonary pressure equal systemic pressure
E. Hemoglobin may be 20 gm/dl
Ans:-ABE
16) -In a healthy baby the transition from fetal to neonatal circulation involves:-
A. Functional closure of the foramen ovale in the first 24 hours
B. Blood flow in the ductus arteriosus continues from right to left until its closure
C. Decrease in pulmonary artery resistance following closure of the ductus arteriosus
D. The ductus arteriosus closes in response to decreased oxygen concentrations
E. The umbilical artery is a branch of the common iliac artery
Ans:- A
Functional closure of the ductus arteriosus occurs soon after birth but anatomical closure
can take upto one week. As pulmonary pressures fall after birth, blood flow in the ductus
is reversed ie from left to right. The umbilical artery is a branch of the internal iliac
artery.Factors influencing closure of the ductus include:-
1. Increased oxygen concentrations
2. Decreased prostaglandin levels
3. Drop in pulmonary artery pressures
N.B. Prostaglandin E2 keeps the ductus open.
17) A 10-day old male presents with bilious emesis. What is the most likely diagnosis?
A. Appendicitis
B. Pyloric stenosis
C. Malrotation with midgut volvulus
D. Feeding intolerance
Ans:- C
18) A term newborn is delivered by emergent cesarean section because of
intrauterine growth restriction, oligohydramnios, and nonreassuring fetal heart rate
monitoring in labor. Delivery room resuscitation includes endotracheal intubation
and assisted ventilation with 100% oxygen, chest compressions, intravenous
epinephrine, and volume expansion. Apgar scores are 1, 2, and 3 at 1, 5, and 10
minutes, respectively. An umbilical cord arterial blood gas measurement documents
a pH of 6.9 and a base deficit of 20mmol/L. At 12 hours of age, the infant
demonstrates tonic-clonic convulsive activity of the arms and legs with a concomitant
decrease in heart rate and bedside pulse oximetry saturation. Of the following, the
MOST likely cause for this infant's seizure is:-
A.Hypercalcemia.
B.Hypercarbia.
C.Hyperglycemia.
D.Hypomagnesemia.
E.Hypoxia.
Preferred Response: E
Seizures are the most frequent sign of central nervous system injury in the newborn. When
seizures occur in a newborn who has depressed neuromotor tone, reflexes, and
cardiopulmonary function at birth that requires assisted ventilation, perinatal asphyxia is
likely. In this event, Apgar scores typically are depressed to less than 3 at 5 or more
minutes after birth,and there is a severely acidotic umbilical cord arterial pH (<7.0), with
evidence of metabolic acidemia. Poor tolerance of labor and asphyxia are more common in
fetuses that have experienced intrauterine growth restriction. Because the infant in the
vignette has the previously described features, hypoxic-ischemic encephalopathy (HIE)
must be considered as a cause for the seizures.HIE is the most common cause of seizures
occurring in the first 24 hours of postnatal life CHILDREN HOSPITAL –BENGHAZI
ABDULRAHMAN BASHIR 7and accounts for up to 67% of early neonatal seizures. Other
causes of neonatal seizure include intracranial hemorrhage, cerebrovascular accidents
(stroke), or hemorrhagic infarction (10% to 15%); intracranial malformation (<10%);
transient hypoglycemia or hypocalcemia (<10%); drug withdrawal (<5%); and inborn
errors of metabolism (<5%).When seizures occur beyond the first 24 hours after birth,
especially in the absence of any history of fetal or neonatal asphyxia, the evaluation should
focus on potential causes other than HIE. An additional cause for later seizures is infection
(meningitis, encephalitis).Asphyxia may result in hypocalcemia and hypoglycemia;
hyperglycemia and hypercalcemia are not associated with HIE and do not typically cause
seizures. Hypomagnesemia may accompany hypocalcemia in the infant of a diabetic
mother, but it is not common following asphyxia and is not associated with neonatal
seizures. Hypercarbia may occur in the depressed newborn who has inadequate
ventilation, but it is not associated with seizures unless there is corresponding hypoxia.
19) A 2-week-old-male presents with lethargy and vomiting. His electrolytes reveal
sodium of 121 meq/L, potassium of 7.0-meq/l and blood glucose of 40 mg/dl. What is
the most likely diagnosis?
A. Dehydration
B. Congenital adrenal hyperplasia
C. Inborn error of metabolism
D. Pyloric stenosis
Ans:- B
20) The following cause the onset of persistent vomiting in 3 week old child:-
A. Disaccharidase intolerance
B. Duodenal atresia
C. Pyloric stenosis
D. Hiatus hernia
E. Lactose intolerance
Ans:-C
21) The drug of choice for treating a newborn with presumed ductal-dependant
cyanotic
congenital heart disease is:
A. Morphine
B. Dobutamine
C. Prostaglandin E1
D. Indomethacin
Ans:- C
The drug of choice for ductal dependent cyanotic heart disease is prostaglandin E1- with
astarting infusion of 0.05 - 1ug/kg/min. There is a risk of apnea associated with its use so
be prepared to intubate, other complications include seizures and fever. Patients with
congenital heart disease present with poor feeding, sweating with feeds, tachypnea, sudden
onset of cyanosis or pallor that may worsen with crying, lethargy, or failure to thrive.
Patients with cyanotic congenital heart disease are hypoxic but typically have a minimal
response to oxygen therapy; whereas patients with a pulmonary process causing hypoxia
will have an increase in oxygen saturation when oxygen is administered. Indomethacin is
used to close a patent ductus.

22) How should a neonate with lethargy and a blood sugar of 20mg/dl be treated?
A. Oral feeds with apple juice
B. 25% dextrose solution
C. 10% dextrose solution
D. 50% dextrose solution
Ans:- C
Newborns with hypoglycemia should be treated with D10W solution with a range of 2-
10cc/kg. Higher concentrations should not be used as they can cause vein sclerosis and
intracranial hemorrhage. Infants and young children should be treated with D25 2-4cc/kg.
23) Neonatal convulsion can be caused by:-
A. Maternal hyperparathyroidism
B. Subdural hematoma
C. Birth asphyxia
D. Hyponatremia
E. Wilson's disease
Ans:-ABCD
24) A 2-day-old female presents with abdominal distension and vomiting. She has not
yet passed a meconium stool. What is the most likely diagnosis?
A. Hirschsprung Disease
B. Malrotation with midgut volvulus
C. Necrotizing enterocolitis
D. Constipation
Ans:-A
Suspect Hirschsprung Disease in a newborn who has not yet passed a meconium stool.
Other possibilities include an imperforate anus or meconium plugging. Older children
present with ahistory of chronic constipation. Hirschsprung disease is the absence of
intramural ganglion cells in the rectum which extends to the sigmoid colon in 77% of
patients and involves the entire colon in 15% of patients. The incidence is 1/5,000 live-
births, with a male to female ratio of 4:1. The diagnosis should be suspected if the patient
presents with lack of meconium stool within the first 24 to 48 hours of life. Vomiting and
abdominal distension may also be present.
25) A 3-week-old female presents with persistent seizures despite aggressive
management with benzodiazepines and phenobarbital. The mother reports giving
her daughter some water to "stop her from getting dehydrated." What is the most
likely cause of her status epilepticus?
A. Hypoglycemia
B. Diabetes insipidus
C. Hyponatremia
D. Hypokalemi
Ans:-C
Excessive free water intake can result in hyponatremic seizures. Infants less than 6 months
of age are particularly susceptible to these types of seizures and commonly have
intractable seizures requiring intubation and hypothermia. Immediate treatment includes
the administration of 3% saline 4cc/kg.\
26) A 1-week-old male presents with some mild erythema around his umbilicus
extending onto the abdominal wall. Which of the following is the correct
management for this patient?
A. Reassurance and continue with alcohol wipes of umbilicus
B. Topical antibiotic ointment and recheck the patient the next day
C. Discharge on cephalexin and recheck the next day
D. Perform a full septic workup and admit the patient
Ans:- D
This patient has Omphalitis and should undergo a full septic evaluation, administration of
antibiotics and hospital admission. Surgical debridement may be required for severe
cases.Omphalitis is inflammation and infection surrounding the umbilicus that can spread
to the liver or peritoneum. Patients can present with symptoms ranging from mild
erythema to necrosing lesions
surrounding the umbilicus on the abdominal wall. Fever may be present
27) A 5-day old, well-appearing male is brought to the ED by his mother today
because she noted that he has a cluster of vesicles on his scalp. Which of the following
should be the management approach?
A. Skin biopsy
B. IV acyclovir and a full septic workup
C. Oral acyclovir
D. Discharge, with next day follow up
Ans:-B
This patient is at risk for herpes encephalitis and should undergo a complete septic workup
and IV acyclovir should be initiated in the ED. Begin acyclovir (20mg/kg every 8 hours
IV) if there is apositive maternal history of herpes, a vesicular rash, focal neurologic
findings, CSF pleocytosis or elevated CSF protein without organisms on gram stain.
28) The following maternal condition can cause disease in the fetus/newborn:-
A. Hyperparathyroidism
B. ITP
C. Myasthenia gravis
D. Diabetus mellitus
E. Thyrotoxicosis
Ans:-ABCDE
29) Which of the following heart rates is most suggestive of supraventricular
tachycardia in
a newborn?
A. 180 BPM
B. 230 BPM
C. 150 BPM
D. 210 BPM
Ans:-B
PALS defines SVT in infants as a heart rate of greater than 220 BPM. In older children the
heart rate for SVT is greater than 180 BPM. The ECG demonstrates a narrow complex
tachycardia without discernible p waves or beat-to-beat variability. In the stable patient,
vagal maneuvers are the first treatment of choice (ice to the face, or blowing through an
occluded straw in older children). Adenosine given as centrally as possible is the first drug
of choice (0.1mg/kg up to 6mg for the first dose and then 0.2mg/kg for the second dose up
to 12 mg) If this is not successful, then amiodarone 5mg/kg given over 20-60 minutes or
procainamide 15mg/kg given over 30-60 minutes are the next drugs of choice. Unstable
patients should undergo cardioversion with 0.5-1J/kg followed by 2J/kg. If an IV is
accessible, a dose of adenosine can be given while setting up for the cardioversion.
30) Vaginal bleeding in a 3-day-old female is:
A. Is always indicative of child abuse
B. May be due to withdrawal of maternal hormones
C. Is suspicious for gonorrhea
D. Is most commonly due to a vaginal foreign body-such as baby wipes
Ans:- B
31) Cyanosis in the first week of life can be caused by:-
A. Tetralogy of Fallot
B. Pulmonary stenosis
C. Eisenmenger syndrome
D. TAPVD
E. Ebistein's anomaly
Ans:-ABDE
32) A 3-week old male presenting to the emergency department with vomiting and
altered mental status and acidosis. What additional laboratory test should be
included in your
evaluation?
A. Ammonia level
B. Cortisol level
C. Serum acetone
D. Thyroid function test
Ans:- A
Suspect an inborn error of metabolism in patients who have an altered level of
consciousness.These patients may or may not be acidotic depending on the type of inborn
error that is present.Patients with a urea cycle defect typically have a normal blood gas but
an elevated ammonia level. Patients with organic acidemias will be acidotic but may or
may not have an elevated ammonia level. Patients with galactosemia will have a normal
blood gas and ammonia level but will have reducing substances in the urine.
33) Which of the following are causes of shock in the newborn?
A. Infection
B. Inborn errors of metabolism
C. Child abuse
D. Thyrotoxicosis
E. All of the above
Ans:- E
34) In neonatal RDS (respiratory distress syndrome):
A. Surfactant is useful in the treatment.
B. It is rare in infants below 28 weeks gestation.
C. Antenatal steroids are beneficial.
D. Maternal opiate abuse increases the risk.
E. Maternal diabetes increases the risk.
Ans:- ACE
35) The risk of neonatal jaundice is increase by:
A. Prematurity.
B. Trisomy 21.
C. Elective caesarean section.
D. Congenital hypothyroidism.
E. Cephalahaematoma.
Ans:-ABDE
36) At birth the blood volume is approximately:-
A. 65 ml/kg body weight
B. 85 ml/kg body weight
C. 110 ml/kg body weight
D. 125 ml/kg body weight
E. 150 ml/kg body weight
Ans:- B
37) Newborn infants commonly have:-
A. Papulovesicles over the trunk.
B. Posterior fusion of the labia minora.
C. An adherent foreskin
D. Breast enlargement
E. Shallow sacral dimple
Ans:-ACDE
38) The following conditions signify disease in the newborn;-
A. Peeling of the skin of the hands and feet
B. Blanched on one side of the body and pink on the opposite side
C. Pinhead lesion on the nose ( milia)
D. Peripheral cyanosis
E. Oedema of one arm
Ans:- All false
39) Apreviously healthy full term infant have several episode of duskiness and
feeding difficulties during the second day of life . She is noted to have increasing
jaundice, which of the following tests will be the least helpful in making diagnosis:-
A. CSF
B. Urine C/ S
C. Total bilirubin
D. Endotracheal aspirate C/S
E. Venous blood
Ans:- D
40) Difference between infant born to heroin-abusing mothers and infant born to
phenobarbitone-abusing mothers is that infant in the later group:-
A. Don't have withdrawal symptoms
B. Have withdrawal symptoms appearing earlier than heroin withdrawal
C. Don't develop tremor
D. Have high incidence of jaundice
E. Are usually term and full size
Ans:- E
41) Which of the following organisms is the most frequent causes of neonatal
meningitis:-
A. GBS
B. E . coli
C. L . monocytogenes
D. H .influenza
E. S. pneumoniae
Ans:- A
42) One should be concerned about term infant who has not passed meconium stool:-
A. During the process of birth
B. Within few min of birth
C. By 1-2 hour of life
D. By 6-12 hour of life
E. By 24 hour of life
Ans:- E
43) The initial lesion of incontinentia pigmenti are:-
A. Deeply pigmented
B. Scaly
C. Waxy papules
D. Inflammatory bullae
E. Small vesicles
Ans:- D
44) Factor that appear to lower threshold for neurologic damage and kernicterus
from unconjugated hyperbilirubinemia include all of the following except:-
A. Acidosis
B. Asphyxia
C. Sepsis
D. Postmaturity
E. Hypothermia
Ans:- D
45) Infant born to diabetic mothers are at risk of all of the following except:-
A. Polycythemia
B. Hyperglycemic dehydration
C. Hypocalcemia
D. Congenital malformation
E. Cardiomyopathy
Ans:- B
46) The Apt test is used for what purpose:-
A. Crude test for carbon monoxide poisoning
B. Semiquantitative test for lead poisoning
C. Qualitative test for fetal hemoglobin
D. Screening test for S hemoglobin
E. Test for blood viscosity
Ans:- C
47) Birth injury account for the majority of the following conditions detected in early
infancy:-
A. Intraventricular hemorrhage
B. Cephalohematoma
C. Hydrocephalus
D. Facial nerve palsy
E. Pneumothorax
Ans:-BD
48) In birth trauma:-
A. Erb's palsy involve C6,C7 & C8
B. Klumpke's palsy involve C7,C8 &T1
C. Facial nerve palsy lead to persistently closed eye
D. Sciatic nerve involvement is common
E. Cephalohematoma is characteristically present at birth
Ans:- B
In Erb-Duchenne paralysis, the injury is limited to the 5th and 6th cervical nerves. The
infant loses the power to abduct the arm from the shoulder, rotate the arm externally, and
supinate the forearm. The characteristic position consists of adduction and internal rotation
of the arm with pronation of the forearm. Klumpke paralysis is a rare form of brachial
palsy; injury to the 7thand 8th cervical nerves and the 1st thoracic nerve produces a
paralyzed hand and ipsilateral ptosis,anhidrosis and miosis (Horner syndrome) if the
sympathetic fibers of the 1st thoracic root are also injured (which reflects damage to the
stellate ganglion adjacent to T1.)
49) Birth injury:-
A. Paralysis of the upper arm has better prognosis than paralysis of the lower
B. In nerve injury, neuroplasty is advised at the end of first year of life
C. In phrenic nerve paralysis, spontaneous recovery is expected
D. Facial nerve paralysis will result from the nuclear agenesis of the facial nerve
Ans:-AC
If the paralysis persists without improvement for 3–6 mo, neuroplasty, neurolysis, end-to-
end anastomosis, and nerve grafting offer hope for partial recovery. function usually
returns in a few months. Total disruption of nerves (neurotmesis) or root avulsion is the
most severe, especially if it involves C5–T1; microsurgical repair may be indicated.
Fortunately, most (75%) injuries are at the root level C5–C6, involve neurapraxia and
axonotmesis, and should heal spontaneously.Botulism toxin may be used to treat biceps-
triceps co-contractions.
50) The following are useful in the assessment of gestational age in preterm:-
A. Presence of palmar creases
B. Breast size
C. Sacral edema
D. The scarf sign
E. Muscle ton
Ans:-BDE
51) Cephalohematoma:-
A. Must be differentiated from subperiosteal hemorrhage
B. Is usually visible at birth
C. May calcify
D. May be associated with underling fracture of the skull
E. Should be managed surgically
Ans:-CD
52) Caput succedaneum is characterized by all of the following except:-
A. Diffuse edematous swelling of the soft tissues of the scalp involving the
portion presenting during vertex delivery
B. It may extend across the midline
C. It may extend across the suture lines
D. Edema usually disappear within 2-3 months
E. The scalp overlying the area may show mild bruising
Ans:- D
53) In newborn with oral moniliasis the most common primary source of infection
A. Maternal source ( vaginal )
B. Contaminated fomites
C. Following use of AgNO2
D. Contact by hospital carriers
E. Systemic antibiotic therapy
Ans:- A
54) Meconium impaction is associated with:-
A. Cretinism B. Cystic fibrosis
C. Thrush D. HMD
E. Trisomy 21 syndromes
Ans;- B
55) Premature infant is delivered precipitiously and appear asphyxiated . The infant
is cyanotic, there are no respiratory efforts,and the heart rate is 80 / min .The infant
is meconium stained and thick particulate meconium is noted in the amniotic fluid
and in infant mouth. At this point you should:-
A. Pass an umbilical artery catheter to measure the PH & PO2
B. Start bag-mask ventilation with 100% oxygen
C. Suction the oropharynx & trachea with ETT to remove the meconium
D. Intubate the trachea & begin the ventilation with 100 % oxygen
E. Establish monitoring with ECG & pulse oximeter
Ans:- C

56) Hypoglycemia has been observed in all of the following except;-


A. With LBW and RDS
B. With anoxic injury
C. With hypothermia
D. Who are SGA
E. With high PO2
Ans:- E
57) All of the following are usually associated with cretinism except:-
A. Constipation
B. Prolonged jaundice
C. Lethargy
D. Tetany
E. Hypotonia
Ans:- D
58) All of the following are characteristic of single umbilical artery except:-
A. Presence in about 5 of 1000 birth
B. About 1/3 of such infants have congenital abnormalities
C. 21 trisomy is frequently found
D. Among twin ,the rate of occurrence is 35 / 1000
E. The associated congenital abnormalities may involve the genitourinary tract
Ans:- C
59) Two minutes after normal term delivary:-
A. The ductus venosus will be closed
B. The pulmonary arterial pressure will have fallen
C. The pressure in the left atrium will have fallen
D. The arterial oxygen tension will have risen
E. Regular breathing will have begun
Ans :-ABDE
60) Established neonatal resuscitation procedures include:-
A. Directing cold stream of oxygen at the nose
B. Administration of drugs with respiratory stimulant properties
C. Oropharyngeal suction
D. Bag and face mask ventilation
E. Prompt cooling
Ans:-ACD
61) The Apgar score :-
A. At 1 min is reliable measure of asphyxia
B. At 1 min is reliable measure of respiratory failure
C. At 10 min is strongly correlated with later neurological deficit
D. Includes the infant response to pharyngeal suction catheter
E. Isn't application after 10 min of age
Ans:- BCD
62) Criteria used in the Apgar score include :-
A. Core temperature
B. Heart rate
C. Respiratory rate
D. Skin thickness
E. Muscle tone
Ans :-BE
63) Pink newborn with HR of 88/min is actively gasping , he has good muscle tone
and
respond to nasal catheter stimulation with facial grimace, the apgar score is:-
A. 5
B. 6
C. 7
D. 8
E. 9
Ans:- D
64) Mongolian spots are characterized by all of the following except:-
A. They are permanent
B. They are usually of a slate blue pigmentation
C. They are generally observed over the buttocks
D. The area of pigmentation is well demarcated
E. They aren't associated with trisomy syndromes
Ans:-A
65) All of the following physical signs may be useful in estimating the gestational age
at birth except:-
A. There are only one or two transverse skin creases on the sole of the foot until
36 week of gestation
B. The breast nodule is usually not palpable at 33 or 34 weeks
C. The breast nodule is usually 4-10 mm in term infant
D. The testes are descending and rugae cover the entire scrotal surface by 34 weeks
E. The texture of scalp hair
Ans:- D
The testes are usually not completely descended until after 36 weeks & scrotal rugae are
few and limited the anterior and inferior aspect of relatively small scrotum . By 34 weeks
the areola become raised and between 36-37 weeks the breast bud is 1 – 2 mm reaching
size of 4-10 mm at term. The transversr foot creases develop at 31 – 32 weeks .By 36
weeks creases cover the anterior two-third of the sole .
66) Newborn infant who remain centrally cyanosed after intubation and IPPV may
have:-
A. Diaphragmatic hernia
B. Choanal atresia
C. Tension pneumothorax
D. Drug induced respiratory center impairment
E. Profound anaemia
Ans:-AC
67) The following statement about pulmonary hypertension are true :-
A. It recognized complication of group B streptococcal sepsis
B. Hyperventilation is an effective treatment
C. Tolazoline is potent pulmonary vasoconstriction
D. Radial arterial PaO2 is lower than umbilical artery PaO2
E. Birth asphyxia is a risk factor
Ans:-ABE
68) Concerning air leak syndromes in the newborn:-
A. Underwater seal drain is only required if the pneumothorax is under tension
B. In term baby with small pneumothorax giving oxygen at high concentration
can worsen it
C. Increasing the I; E ratio in ventilated baby decrease the risk of pneumothorax
D. Pneumomediastenum is usually fatal
E. They can be asymptomatic
Ans;- E
69) Recognized problem of infants born at term SGA include:-
A. Hypothermia
B. Sepsis
C. Polycythemia
D. Hypoglycemia
E. Retinopathy of prematurity
Ans:-ABCD
70) Complication of steroid therapy in the newborn include:-
A. Leucopenia
B. Hypoglycemia
C. Cataract
D. Sepsis
E. Gastric perforation
Ans:-CDE
71) Concerning NEC :-
A. Exchange transfusion is predisposing factor
B. Clostridium welchii is implicated in the pathogenesis
C. It is most common in infants born less than 1500 gm
D. Oral antibiotics are useful
E. Complications include short bowel syndrome
Ans:-ACE
72) The following congenital condition require immediate ( within first week)
treatment after birth:-
A. TEF
B. Cleft lip
C. Spina bifida
D. Exomphalos
E. Hydrcephalus
Ans:-ACD
73) Preterm infant at increased risk from :-
A. Conjugated hyperbilirubinemia
B. Meconium aspiration
C. Periventricular leucomalacia
D. Necrotizing enterocloitis
E. Child abuse
Ans:- CDE
74) Peri- or intraventricular cerebral hemorrhage
A. Occur in less than 10% of VLBW infant
B. Arise most commonly in the first 72 hour after delivary
C. Is direct result of impaired vitamin K supply
D. Is the single most common cause of congenital cerebral palsy
E. May result in rapidly evolving hydrocephalus
Ans:-BE
75) Feature typical of physiological jaundice include:-
A. Recognizable jaundice in the first 48 hours
B. Peak plasma bilirubin at 4-5 days
C. Persistent beyond first week
D. Irritability
E. Pale stool
Ans:- B
76) Jaundice on day 1 is often caused by ;-
A. Metabolic disorder
B. TORCH infection
C. Gastrointestinal obstruction
D. Hemolysis
E. Physiological factor
Ans:-BD
77) Persistant, conjugated hyperbilirubinaemia may be caused by:
A. Alpha-1-antitrypsin deficiency.
B. Hypothyroidism.
C. Haemolytic disease.
D. Cytomegalovirus infection.
E. Cystic fibrosis.
Ans:-ABDE
Explanation:- all cases need further investigation.
Causes of conjugated hyperbilirubinaemia in a neonte are:
1)-Intrahepatic cholestasis -Infections.-Congenital infections (STORCH-syphlis,
toxoplasma, rubella,cytomegalovirus, hepatitis, herpes virus infection)
-Acquired infections- septicaemia, UTI.-Metabolic disorders –cystic fibrosis, alpha-1-
antitrypsin deficiency, galactosaemia,fructosaemia, lysosomal storage disorders,
peroxisomal disorders.-Endocrine disorders – hypothyroidism, hypopituitarism,
hypoadrenalism.-Anatomical disorders – intrahepatic:bile duct hypoplasia.
-Miscellaneous – idiopathic neonatal hepatitis, chromosomal abnormalities, trisomy 21, 18
and 13.
2)-Extrahepatic cholestasis:- biliary atresia, choledochal cyst, spontaneous bile duct
perforation, inspissated bile syndrome.
Unconjugated neonatal jaundice is normally physiological and resolves spontaneously.

78) All of the following are characteristic of jaundice associated with breast feeding
except :-
A. Significant elevation of unconjugated bilirubin
B. Rapid fall in serum bilirubin after discontinuation of nursing
C. Nursing can be resumed after several days without return of hyperbilirbinemia
D. Significant elevation of conjugated bilirubin
E. Kernicterus has never been reported to occur as result of breast milk jaundice alone
Ans :- D
79) Persistent jaundice during the first month of life may associated with all of the
following except:-
A. Cytomegalic inclusion disease
B. Congenital atresia of the bile duct
C. Galactosemia
D. Rh incompatibility
E. Penicillin treatment
Ans:- E
80) A 26-week gestation preterm infant is now 6 weeks old and weighs 1,250 g. He is
receiving full-volume enteral nutrition. The only significant finding on physical
examination is pallor. He has anemia (hematocrit of 28% [0.28]; reticulocyte count of
8% [0.08]) and receives iron supplementation. He is receiving a formula that is high
in polyunsaturated fatty acids.Of the following, the MOST correct statement about
his need for vitamin E is that it
A. does not need to be supplemented in infancy
B. has no effect on anemia
C. needs to be supplemented now
D. will be needed when the infant is 3 months old
E. will prevent anemia
Preferred Response: C
Historically, inadequate vitamin E, a high level of polyunsaturated fatty acids (PUFAs) in
infant formula, and exposure to the oxidizing effects of iron supplementation contributed
to a hemolytic anemia seen in preterm infants. In the United States and many developed
countries, infant formulas now provide an adequate vitamin E:PUFA ratio to eliminate this
risk. However, preterm infants continue to have low vitamin E levels due to limited stores,
especially when the birth is extremely premature, and limited enteral feedings early in
their postnatal neonatal intensive care unit course.The neonate who has anemia and is
receiving iron supplementation, such as the infant described in the vignette, requires the
antioxidant effect of vitamin E to reduce red blood cell hemolysis. A
total daily requirement of 10 to 25 IU of vitamin E meets the infant’s needs, only 50% of
which is provided by dietary formula. Of note, human milk also is an incomplete source of
vitamin E.
81) Amniocentesis is useful in establishing the prenatal diagnosis of:-
A. Down syndrome
B. Meningomyelocele
C. Erythroblastosis fetalis
D. Achondroplasia
Ans:-ABC
82) USS can be used during pregnancy to:-
A. Determine crown rump length
B. Determine fetal sex
C. Determine biparietal diameter
D. Accurately determine fetal weight
Ans ;-AC
83) Gestation that produce multiple births:-
A. Are classified as high risk
B. Are always delivered by cesarean section
C. Can produce infant with discordance in body size at birth
D. Aren't associated with the premature onset of labour
Ans:-AC
84) For newborn infant, the least important factor to consider in assessing the risk of
kernicterus is:-
A. Breast feeding
B. Acidosis
C. Sepsis
D. Albumin level
E. Moxalactam, cephalosporin type antibiotic
Ans:- A
85) The following maternal conditions are known to cause adverse effects on the
neonate:
A. Diabetes insipidus.
B. Toxoplasmosis.
C. Chronic myeloid leukaemia.
D. SLE.
E. Hyperthyroidism.
Ans:-BCDE
Explanation:- Maternal SLE is a risk factor for neonatal lupus syndrome. The neonate
presents with clinical features of SLE due to transplacental passage of maternal Ab. The
skin is frequently involved with malar rashes and there can be haematological and cardiac
abnormalities. The most frequent heart abnormality is congenital heart block – 90% of
mothers whose infants have congenital heart block are anti-Ro(SSA)Ab positive.
Toxoplasmosis leads to congenital infection by transplacental transmission. The rate of
transmission is +/- 60% of third trimester infections and 20%-30% during the first two
trimesters.The ewborn presents with the classic triad of hydrocephalus, chorioretinitis, and
cerebral calcification.Maternal chronic myeloid leukaemia can have adverse effect on
pregnancy – fetal/neonatal mortality is 16-38%. Pathology can be secondary to placental
leukaemic infiltrates, anaemia and infectious complications. Splenomegaly can restrict
intrauterine growth and lead to premature delivery.If hyperthyroidism is due to Grave’s
disease or Hashimoto thyroiditis, the neonate may present with thyrotoxicosis due to the
transplacental passage of TSI. These symptoms are frequentely short-lived i.e as long as
the circulating antibodies persist in the baby’s circulation.
86) Each of the following statement about GBS infection in the newborn is true
except:-
A. Incidence is correlates inversely with the presence of maternal antibodies
B. The exotoxin has powerful pulmonary vasoconstrictive effects
C. The disease has two distinct pattern; early & late onset
D. Both the incidence & severity of the disease can be lessened by pretreatment of the
mother who is colonized
E. The risk of invasive disease isn't related to the amount of inoculum received by the
infant
Ans:- E
87) Newborn infant may present with bile stained vomiting and abdominal distension
as the result of
A. Oesophageal atresia
B. Duodenal atresia
C. Birth asphyxia
D. Electrolyte disturbance
E. Cystic fibrosis
Ans:-BCDE
88) The following feature are consistent with newborn infant having oesophageal
atresia and tracheo-oesophageal fistula:-
A. Maternal polyhydramniose
B. Passage of wide bore orogastric catheter into the stomach
C. Plain x-ray evidence of air in the stomach and small bowel
D. Plain x-ray evidence of hemivertebra
E. Excessive mucus in the nostrils or mouth.
Ans:-ACDE
89) Vitamin K
A. Is an essential cofactor for the synthesis of coagulation factor II , VII, IC, C
B. Is readily transported across the placenta
C. Is present in the breast milk at higher concentration than in cow milk.
D. Given in single oral dose after delivary effectively prevent s hemorrhagic disease
E. Related hemorrhage in the newborn is commoner when mother have taken
anticonvulsant during pregnancy
Ans:-AE
90) The following are recognizable causes of neonatal convulsion:-
A. Birth asphyxia
B. Hypoglycemia
C. Hypothermia
D. Opiate withdrawal
E. Hypernatremia
Ans:-ABDE
91) With regard to Apgar score:-
A. 2 points given to pulse of 88/min
B. 1 point is given for irregular gasps
C. An initial satisfactory score gurarantees an eventual perinatal period
D. Score of 2 at 10 min carries worse prognosis than score of 2 at 5 min
E. The lowest possible score is 1
Ans:-BD
92) Intrauterine posture is commonly responsible for:-
A. Congenital dislocation of the hip
B. Plagiocephaly
C. Sternomastoid shortening
D. Syndactyly
E. Mandibular asymmetry
Ans:-ABCE
93) Established neonatal screening tests include:-
A. Umbilical cord blood analysis to detect phenylketonuria
B. Umbilical cord blood analysis to detect galactose
C. Umbilical cord blood analysis to detect sickle cell disease
D. Capillary blood analysis at 6-8 days to detect elevated TSH
E. Capillary blood analysis at 6-8 days to detect elevated immunoreactive trypsin
Ans:-CDE
94) Maternal condition that may have effect in the neonatal period include:-
A. ITP
B. Multiple sclerosis
C. DM
D. Varicella zoster
E. Bornholm disease
Ans:-ACDE
95) Neonatal polycythemia :-
A. Occur in small for date infant as response to placental insufficiency
B. Has an increased incidence if maternal diabetes is poorly controlled
C. Carry an increased risk of cerebral venous sinus thrombosis
D. Is recognizable feature of congenital hypothyroidism
E. May occur as result of feto-maternal transfusion
Ans:-ABCD
96) Polycythemia on the newborn is characterized by all of the following except:-
A. Increased the incidence in IDM
B. Placental transfusion decreased by late clamping of the umbilical cord
C. Clinical presentation that includes jitteriness & poor feeding
D. Complication including heart failure & NEC
E. Elevated viscosity of whole blood
Ans;- B
97) Causes of neonatal polycythemia include:-
A. Congenital rubella infection
B. Pre-eclampsia
C. Maternal diabtus mellitus
D. Delayed clamping of umbilical cord
E. Congenital adrenal hyperplasia
Ans:- BCDE
98) Meconium aspiration pneumonia:-
A. Occur with equal frequency in term and preterm infant
B. In infant required ventilation for this condition, combination of high PEEP and
rapid rate is advisable
C. Has high risk of developing even if liquor is only thinly stained
D. High dose steroid are the mainstay of treatment
E. Antibiotic treatment is an important part of treatment
Ans :- all False
99) The following statement are true regarding hemolytic disease of newborn ( HDN
):_
A. Hemolytic disease should be suspected if jaundice is noted in the first 24 hour of
life
B. HDN may occur if mother is group A +ve and the baby is group O+ve
C. If due to Rh –incompatibility the severity of the hemolysis typically increase with
each affected pregnancy
D. HDN due to ABO incompatibility can be detected at 36 week gestation by
amniocentesis
E. As long as the level of unconjugated bilirubin never rises above20 mg/dl there is no
dangerous of kernicterus
Ans:-AC
100) Full term infant has tachypnea with grunting , chest X ray show well expanded
lung with streaky shadows radiating from the bilateral hilar region , the most likely
diagnosis is :-
A. TTN
B. Congenital pneumonia
C. BPD
D. Meconium aspiration
E. Aspiration pneumonia
Ans:- A
101) Condition associated with polyhydramniose include:-
A. Oesophageal atresia
B. Down's syndrome
C. Renal agenesis
D. Cord around the neck
E. CNS malformation
Ans:- ABE
102) The following are causes of polyhydramnios:
A. Maternal diabetes mellitus.
B. Potter syndrome.
C. Anencephaly.
D. Oesophageal atresia.
E. Polycystic kidneys.
Ans:-ACD
103) The following are causes of polyhydramniose:-
A. Pottrer syndrome
B. TEF and oesophageal atresia
C. Rh – incompatibility
D. Anencephaly
E. Maternal diabetus
Ans:-BDE
104) For necrotizing enterocloitis:-
A. Term baby are particularly at risk
B. Perinatal asphyxia is risk factor
C. Expressed breast milk from milk bank confer no protection
D. Failure of temperature control is late sign
E. High platelet count is frequently seen
Ans:- B
105) You diagnose necrotizing enterocolitis in a preterm neonate who has abdominal
distention and blood in the stool. You decide that this infant should be placed on a
14-day regimen of parenteral nutrition.Of the following, the micronutrient for which
weekly monitoring is MOST recommended during this infant’s parenteral nutrition
therapy is
A. copper
B. iron
C. phosphorus
D. selenium
E. zinc
Preferred Response: C
Parenteral nutrition (PN), the intravenous administration of carbohydrates, lipids, amino
acids,and micronutrients, is an important component in the management of a variety of
chronic disorders, including surgical conditions (eg, omphalocele, gastroschisis,
diaphragmatic hernia,short bowel syndrome), inflammatory conditions (eg, Crohn disease,
ulcerative colitis,pseudomembranous colitis, pancreatitis, graft versus host disease),
hypermetabolic states (eg,burns, trauma), and intestinal motility disorders (eg, pseudo-
obstruction). PN is especially important in the support of very low-birthweight infants,
who frequently have increased caloric requirements, decreased oral intake, and immature
intestinal motility.In addition to glucose, amino acids, sodium, potassium, and chloride,
PN provides additional mineral supplements, including calcium, phosphorus, magnesium,
zinc, copper, selenium,chromium, manganese, molybdenum, and iodide. In preterm
infants, the administration of both calcium and phosphorus is important to prevent
metabolic bone disease.However, the amount of calcium and phosphorus that can be
administered in PN must be limited because of the risk of precipitation in the PN and
formation of calcium-phosphorus complexes that could embolize. It generally is thought
that the chance of precipitation is high if the product of the concentrations (in mmol/L) of
the calcium and phosphorus in PN is greater than 40.Because hypophosphatemia can have
significant consequences (including impaired cardiac function, muscle weakness, and
hemolysis), it is recommended that serum phosphorus concentrations be assessed at least
once weekly for patients receiving PN. The serum concentrations of copper, iron,
selenium, and zinc, which are trace elements, require only periodic assessment (eg, once a
month or less).
106) An 1,800-g preterm infant is recovering from surgery for gastroschisis. Of the
following, the MOST correct statement regarding this infant’s immediate daily
nutritional requirements is that he
A. can meet energy needs from fat through enteral feeding
B. can meet protein needs for growth through enteral feeding
C. requires lysine supplementation to promote protein synthesis
D. requires more protein per kilogram than a term infant
E. requires 2.5 g/kg per day of protein
Preferred Response: D
Milk remains the principal source of nutrition for infants, who consume 120 to 150 mL/kg
per day in the newborn period.Human milk has 67 kcal/100 mL, and most term infant
formulas have a similar composition. The energy derived from ingesting milk comes
primarily from fat calories (3.8 g/100 mL = 34kcal/100 mL), followed by carbohydrates
(lactose, 7 g/100 mL = 28 kcal/100 mL) and minimally from protein (1.3 g/100 mL = 5
kcal/100 mL). Conditions such as prematurity, lung disease, or surgery may increase both
caloric and specific nutrient requirements for newborns.The newborn described in the
vignette can only take limited enteral nutrition and has an increased need for both protein
and calories to facilitate healing and growth. The normal 2 to 3 g/kg per day of protein
ingested by the term infant who is either formula- or breastfed is inadequate for this low-
birthweight, preterm newborn. He will grow and heal best receiving 3.5 to 4.0 g/kg per
day of protein. The potential benefits of single amino-acid supplementation, such
as lysine, remain unknown.
107) Hypoxic-ischemic encephalopathy all are correct except:-
A. Result from excessive use of oxytocin
B. IUGR is be first indication of fetal hyoxia
C. Associated with increased beat to beat variability
D. Prognosis depend on gestational age
E. Persistent of abnormal neurological sign at two week indicate poor prognosis
Ans:- C
108) polyhydramniose:-
A. Is defined as an amniotic fluid volume of more than 500 ml
B. Occur with increased frequency in diabetic pregnancy
C. Is associated with renal agenesis
D. Is associated with tracheo-oesophageal fistula
E. Is associated with increased risk of premature labor
Ans :-BDE
109) Polyhydramnios is associated with all of the following condition except:-
A. Amniotic fluid volume between 500-2000 ml
B. Maternal diabetes
C. Twins
D. Erythroblastosis fetalis
E. Down syndrome
Ans:- A
110) Condition associated with oligohydramnios include:-
A. Infantile polycystic kidney
B. Congenital heart disease
C. IUGR
D. High intestinal obstruction
E. Posterior urethral valve
Ans:- ACE
111) Regarding hyaline membrane disease ( HMD)
A. Can occur in infants of diabetic mother of 37-40 week gestation
B. Infant born to mothers who are heroin addicts are at increased risk of HMD
C. Light-for –date infant of 33 weeks gestation has greater risk of developing
HMD than 33 week infant of appropriate weight
D. Administration of artificial surfactant is curative
E. Chest x-ray finding are markedly different between cases of HMD and Group
B streptococcal pneumonia
Ans:- A
112) Hyaline membrane disease:-
A. Is more common in babies of diabetic mother
B. Is due to surfactant deficiency
C. Is not seen in term babies
D. Occur most commonly at 12 hour post-delivary
E. Always require ventilation
Ans:- AB
113) The following symptoms in an infants in the first month of life should alert one
to possibility of hypothyroidism:
A. Prolonged jaundice
B. Vomiting
C. Diarrhea
D. Hoarse cry
E. Voracious appetite
Ans:- AD

114) The following malformation can occur in children born to mother who have
IDDM:
A. Cleft lip/ palate
B. Caudal regression syndrome
C. Femoral hypoplasia
D. Holopresencaphaly
E. Polydactyly
Ans:- BCDE
115) With regard to RDS of the newborn
A. Meconium aspiration pneumonia is especially likely to be the cause if the infant is
preterm
B. HMD is unlikely to be the cause if the prepartum lecithin sphingomylin (L/S) ratio
in amniotic fluid is > 2;1
C. Patchy opacities on chest X-ray are evidence in favour of diagnosis HMD
D. In the artificial ventilation of infant with HMD , inflation pressure must never
exceed 30 cm of water
E. CPAP is an effective in treatment for Pneumothorax
Ans : B
116) In RDS due to meconium aspiration
A. Antibiotic therapy is of crucial importance
B. Steroid therapy improve the prognosis
C. There is high risk of Pneumothorax
D. Chest X-ray typically shows ground glass opacity
E. The infant may also suffer from cerebral oedema
Ans:-C
117) Recognized association of small-for-date babies include
A. Maternal smoking in pregnancy
B. Hemolytic disease due to ABO incompatibility
C. Pre-eclamptic toxaemia
D. Congenital rubella infection
E. Fetal alcohol syndrome
Ans:- ACDE
118) Prolonged neonatal jaundice is recognized feature in infants with
A. CMV infection
B. Congenital hypothyroidism
C. Untreated urinary tract infection
D. Tracheo-oesophageal fistula
E. Galactosemia
Ans:- ABCE
119) In baby of 32 week gestation who has tachypnea and sternal recession at 4 hour
of age:
A. The ductus arteriosus is likely to be patent
B. Meconium aspiration pneumonia is likely to be diagnosis
C. The L/S ratio is likely to be low
D. Fluid level on chest X-ray suggest pneumonia due to Group B streptococcus
E. The presence of bowel shadows on the left side of the chest on X-ray is
diagnostic of Tracheo-oesophageal fistula
Ans :-AC
120) The following condition characteristically cause jaundice within the first 24
hour of life:-
A. G6PD deficiency
B. Congenital hypothyroidism
C. Sever congenital CMV infection
D. Choledochal cyst
E. Primary tyrosinaemia
Ans:- AC
121) Hypothermia can cause the following in LBW infant:-
A. Decrease synthesis of surfactant
B. Hypernatremia
C. Hypoglycemia
D. Increased oxygen consumption
E. Hypercalcemia
Ans :- ACD
122) The following substance are freely transmitted across the placenta:-
A. Carbimazole
B. Diazepam
C. Warfarin
D. Pethidine
E. IgM antibodies
Ans:-ABCD
123) In congenital diaphragmatic hernia:-
A. Hernia most commonly occur on the left
B. Pulmonary hypoplasia is the major cause of death
C. Persistent fetal circulation occurs uncommonly
D. Associated congenital anomalies are common
E. Most present between 12-24 hours of life
Ans:- AB
The incidence of CDH is between 1/2,000 and 1/5,000 live births, with females affected
twice as often as males. Defects are more common on the left (85%) and are occasionally
(<5%) bilateral.Pulmonary hypoplasia and malrotation of the intestine are part of the
lesion, not associated anomalies. Most cases of CDH are sporadic, but familial cases have
been reported. Associated anomalies have been reported in up to 30% of cases; these
include central nervous system lesions,esophageal atresia, omphalocele, and
cardiovascular lesions. CDH is recognized as part of several chromosomal syndromes:
Trisomy 21, Trisomy 13, Trisomy 18, Fryn, Brachmann–de Lange,
Pallister-Killian, and Turner. Relative predictors of a poor prognosis include an associated
major anomaly, symptoms before 24 hr of age, severe pulmonary hypoplasia, herniation to
the contralateral lung, and the need for ECMO.
124) In the normal newborn infant in the first 24 hour of life:-
A. Many significant heart defect may be clinically undetectable
B. Normal arterial PaO2 help the ductus to close
C. Hypoxia causes pulmonary artery vasoconstriction
D. All children with murmur heard in the first 24 hour should be followed up for
at least 6 months
E. The systolic blood pressure is between 40-80 mmHg
Ans:-ABCE
125) A light –for-date full term baby is at particular risk from the following
condition:-
A. HMD B. Physiological jaundice
C. Milk aspiration D. Hypoglycemia
E. Apnoeic attacke
Ans:- D
126) Recurrent apnoea of prematurity:-
A. Characteristically develop within the first 24 hour of life
B. Is more likely to occur in infant of < 32 week gestation
C. Usually respond to naloxone
D. Should be treated with 100% oxygen during attack
E. May be accentuated by the presence of nasogastric tube
Ans:- BE
127) In the management of HMD:
A. Tolazoline may lead to systemic hypertension
B. Early use of CPAP may reduce the need for subsequent ventilatory support
C. Antibiotic improve ventilation- perfusion ratio
D. The illness may be expected to increase in the severity for the first 5 days
E. Corticosteroid used postnatally have beneficial effect on the course of the
disease
Ans:- B
128) Characteristic finding in preterm baby include:-
A. Chin reaching only to the tip of shoulder
B. Full wrist flexion
C. Flat on couch when lying prone
D. Incomplete ankle dorsiflexion
E. Incomplete knee extension with hip fully flexed
Ans:- C
129) Drug effects on the fetus:-
A. Isotretinion can lead to CNS defects mainly if given in the third trimester
B. Phenytoin can lead to cleft lip, finger and toe abnormalities mainly if given in
the second trimester
C. Carbimazole cause goiter, mainly if given in the third trimester
D. Warfarine can lead to neonatal hemorrhage mainly if given in the first
trimester
E. Valproate can lead to neural tube defects mainly if given in the third trimester
Ans:- C

130) The external criteria for the Dubowitz score for gestational age include:-
A. Breast size
B. Skin texture
C. External genitalia
D. Langue hair
E. Nose firmness
Ans:-ABCD
131) Problem of babies born to mothers with poorly controlled diabetus mellitus:
A. Hypermagnesemia
B. Polycythemia
C. Hypocalcemia
D. Sacral agenesis
E. Respiratory distress
Ans:-BCDE
132) The following are recognised associations of maternal diabetes:
A. Sacral agenesis
B. Intrauterine growth retardation
C. Macrosomia
D. Hyaline membrane disease
E. Hypertrophic cardiomyopathy
Ans:-ABCDE
Comments:
Overall, malformations occur in 6%, with an increased incidence of cardiac
malformations, sacral agenesis, hyperplastic left colon. Intrauterine growth retardation is 3
times as common due to small vessel disease in the mother. More common is
macrosomia, and this is related to the degree of maternal hyperglycaemia. The glucose
crosses the placenta while the insulin does not, so the fetus increases its production of
insulin. This results in increased cell number and size. 25% of IDM are greater than 4kg
compared with only 8% of non-diabetics. This gives problems with delivery such as CPD,
shoulder dystosia resulting in an increased incidence of birth asphyxia and trauma.
In the neonatal period, hypoglycaemia, respiratory distress and reversible hypotrophic
cardiomyopathy and polychthaemia are all more common. Gestational diabetes is when
carbohydrate intolerance occurs only during pregnancy. It is commonest in obese women
and those from Afro-Caribbean and Asian ethnic groups. In these women there is no
increase in congenital malformations, thoug macrosomia and other complications remain
similar in frequency.
133) Characteristic finding in full term baby are:-
A. Full knee extension with hips fully flexed
B. Momentary neck extension when held sitting
C. Chin reach beyond tip of shoulder
D. Hip abducts and legs flat on couch when lying supine
E. Full ankle dorsiflexion
Ans:- BE
134) The external criteria for the Dubowitz score for gestational age include:-
A. Nipple formation
B. Ear form & firmness
C. Skin colour & opacity
D. Protruding tongue
E. Planter creases
Ans:- ABCE
135) In fetal circulation:-
A. 30% of the fetal cardiac output goes through the placenta
B. Oxygenated blood from the placenta passes through the ductus arteriosus toward
the right atrium
C. The oxygen saturation of blood in umbilical arteries is approximately 60%
D. Blood entering the heart from the inferior vena cava is diverted directly to the left
atrium via patent foramen ovale
E. There is one umbilical vein
Ans:- CDE
136) The following disease can now be diagnosed prenatally:-
A. Phenylketonuria
B. Homocystinuria
C. Sever combined immunodeficiency
D. Fanconi's anaemia
E. Lesh-Nyhan syndrome
Ans:- ABCDE
137) Resuscitation of neonate with coarctation of aorta may require the use of :-
A. Frusemide
B. Bicarbonate
C. Indomethacin
D. Dopamine
E. Prostaglandin E
Ans:- ABDE

138) Recognized causes of the floppy baby include:-


A. Trisomy 21
B. Zellweger syndrome
C. Becker muscular dystrophy
D. Spinal muscular atrophy
E. Hypothyroidism
Ans:- ABDE
139) Which of the following may cause apnoea in preterm infant
A. Hypocalcemia
B. Hypoglycemia
C. RSV infection
D. Caffeine
E. Intraventricular hemorrahage
Ans:- ABCE

140) Failure of resuscitation of newborn may be due to :-


A. PDA
B. VSD
C. Congenital diaphragmatic hernia
D. Pulmonary hypoplasia
E. HMD
Ans:-CD
141) Feature of IUGR include:-
A. Neutropenia
B. Hypoglycemia
C. Necrotizing Enterocolitis
D. Weight loss > 10% of birth weight in first week
E. Thermal instability
Ans:- ABCE
142) Newborn infant has breathing difficulty with central cyanosis .Nitrogen washout
test raised the PO2 to 15 kPa possible diagnosis include:-
A. Fallot's tetralogy
B. VSD
C. Pneumonia
D. Tricuspid atresia
E. HMD
Ans:- CE
143) Newborn infant has central cyanosis and fit with slow & shallow breathing ,
Nitogen
washout test produce slight raise in the PO2 , the most likely cause of cyanosis is:-
A. Methemoglobinemia
B. Cerebral disorder
C. Persistent fetal circulation
D. Congenital cyanotic heart disease
E. Lung disease
Ans:- B
144) The following disease can now be diagnosed prenatally:-
A. Hypercholesterolemia
B. Polycystic kidney disease
C. Cystinosis
D. Gaucher's disease
E. Beta thalassaemia
Ans:-ABCDE
145) Common causes of seizure in the neonatal period include:-
A. Intracranial hemorrhage
B. Electrolyte disturbance
C. Infection
D. Drug withdrawal
E. Febrile convulsion
Ans:- ABCD
146) Neonatal convulsion may be caused by :-
A. Hypomagnesaemia
B. Hyperkalamia
C. Pyridoxine dependency
D. Cephalohematoma
E. HSV infection
Ans:- ACE
147) The following disease can now be diagnosed prenatally:-
A. Sickle cell disease
B. Maple syrup urine disease
C. Retinoblastoma
D. Hypophosphaturia
E. Von-Willebrand disease
Ans:- ABCDE
148) Which of the following statement regarding surfactant therapy is correct :-
A. The incidence of Pneumothorax is reduced
B. The incidence of intraventricular hemorrhage is reduced
C. Early therapy ( within 4 hour) is more beneficial than later administration ( after
12 hour)
D. Prolonged courses of surfactant therapy confer advantages
E. The incidence of chronic lung disease is increased
Ans:- ABC
149) Which of the following statement regarding periventricular hemorrhage are
correct:
A. Most hemorrhage occur after the fourth day of life
B. Over 80% of hemorrhage progress to ventricular dilatation
C. Most cases of posthemorrhagic hydrocephalous are communicating
D. Early ventricular tapping improve the neurological outcome
E. Hemorrhage is usually caused by hypoglycemia
Ans:- C
150) Complication of phototherapy include:-
A. Diarrhea
B. Erythematosus rash
C. Hyponatremia
D. Skin discoloration
E. Peripheral desqumation
Ans:- ABD
151) Which of the following are characteristic haemodynamic changes encountered
at birth:-
A. Increase the pulmonary vascular pressure
B. Right to left flow through the ductus arteriosus
C. Closure of the ductus arteriosus
D. Increased right ventricular end diastolic pressure
E. Reversal of flow across the foramen ovale
Ans:- C

152) Feature suggestive of TEF with oesophageal atresia include:-


A. FTT
B. Recurrent pneumonia
C. Oligohydramniose
D. Large amount of mucus in the pharynx at delivary
E. Slow to establish feed
Ans:- D
153) The development of Pneumothorax is associated with :-
A. Surfactant therapy
B. Artificial paralysis
C. Patient triggered ventilation
D. Meconium aspiration
E. Pulmonary interstitial emphysema
Ans:- DE
154) Cyanotic congenital heart disease in the newborn include:-
A. TGA
B. Pulmonary stenosis
C. Fallot's tetrallogy
D. VSD
E. Tricuspid atresia
Ans:- AE
155) The following congenital disorder require immediate ( within first week)
treatment after
birth:-
A. Hemangioma
B. Imperforate anus
C. Choanal atresia
D. Hypospadius
E. Congenital diaphragmatic hernia
Ans:-BCE
156) Causes of neonatal hypoglycemia include;-
A. Erythroblastosis fetalis
B. Glycogen storage disease type I
C. Maternal treatment with sodium valproate
D. Galactosemia
E. Congenital adrenal hypoplasia
Ans:- ABDE
Plasma glucose level of <40 mg/dL, During gestation, glucose is freely transferred across
the placenta by the process of facilitated diffusion. However, after birth, the infant must
adjust to the sudden withdrawal of this transplacental supply. In all infants, there is a nadir
in blood sugar between 1 and 3 hours of life. During the first 12-24 hours of life, newborns
are at increased risk for hypoglycemia because gluconeogenesis and especially ketogenesis
are incompletely developed. These factors are accentuated in preterm infants, infants of
diabetic mothers, infants with erythroblastosis fetalis, asphyxiated infants, and infants who
are small or large for gestational age.
Transient hypoglycemia causes:- prematurity, hypothermia, Birth asphyxia, sepsis,
erythroblastosis fetalis, infants with Beckwith-Wiedemann syndrome, maternal diabetes,
maternal glucose infusion in labor, and intrauterine growth restriction (IUGR)and maternal
drugs( b sympathomimetic and chlorpropamide).
Persistent hypoglycemia may be due to: - inborn error of metabolism (Glycogen storage
disease type I , P & IV, maple syrup urine disease, Nesidioblastosis, galactosemia,and
mitochondrial fatty acid oxidation defects such as MCAD deficiency. Hormonal
deficiency suc as congenital hypopituitrism,congenital glucagons deficiency and cortisol
deficiency states including congenital adrenal hypoplasia.
Features on physical examination suggest the etiology of hypoglycemia:-
· Macrosomia: This occurs in infants of diabetic mothers, infants with severe congenital
hyperinsulinism, and infants with Beckwith-Wiedemann syndrome; recall that insulin is a
growth factor and that hyperinsulinism leads to macrosomia.
· Midline defects: Congenital pituitary deficiency can be associated with midline defects
such as cleft lip, cleft palate, single central incisor, and micro-ophthalmia.
· Micropenis: Congenital gonadotropin deficiency and possible pituitary abnormalities
cause this condition.
· Hepatomegaly: This is associated with glycogen storage diseases and fatty acid
oxidation disorders.
157) Healthy term neonate differ from adult in the following way:-
A. Less complement
B. Decrease IgG level
C. Fewer B lymphocytes
D. Lower level of secretory IgA
E. Higher level of C reactive protein
Ans:- AD
158) You are examining a newborn who is the product of an uneventful pregnancy,
labor, and delivery. Apgar scores were 9 at both 1 and 5 minutes. Findings on the
initial physical examination are unremarkable except for the presence of
vesicopustules and frecklelike macules (Item Q33A), some of which have a collarette
of surrounding scale. Wright stain of a smear of the vesicopustular contents reveals a
predominance of polymorphonuclear neutrophils.Of the following, the MOST likely
diagnosis is
A. congenital candidiasis
B. erythema toxicum neonatorum
C. infantile acropustulosis
D. miliaria rubra
E. transient neonatal pustular melanosis
Preferred Response: E
Characteristic lesions of transient neonatal pustular melanosis (TNPM) may be present at
birth as vesicles, pustules, or ruptured vesicles or pustules that have a collarette of
surrounding scale.Pigmented macules (Item C33A) often develop at the sites of resolving
pustules or vesicles.Primary lesions usually disappear by 5 days of age; the secondary
pigmented lesions may remain up to 3 months. TNPM occurs more commonly in African-
American infants.Lesions can occur on palms and soles. Pustular contents reveal a
predominance of neutrophils on Wright stain examination, as reported for the newborn in
the vignette.Infants who have congenital cutaneous candidiasis may present with scaling,
erythematous papules and pustules (Item C33B) at birth. Candida albicans can penetrate
through the amnion and chorion to cause congenital infections. Scrapings from lesions
prepared with potassium hydroxide document pseudohyphae (Item C33C) or budding
yeast.Term infants who have erythema toxicum neonatorum exhibit vesicopustular
lesions (Item C33D) that usually overlie erythematous macules. Lesions of erythema
toxicum rarely are present at birth, and Wright stain of smears of pustular contents reveals
a predominance of eosinophils.Infantile acropustulosis presents as pustules or vesicles
(Item C33E) localized to the hands and feet. It may be present at birth but more commonly
develops in the first weeks and months after birth, possibly continuing or recurring
throughout infancy and early childhood. Lesions are very similar to those of infantile
scabies infestation. Pustular contents may reveal prominent neutrophils and occasional
eosinophils without evidence of the mites, ova, or feces seen in scabies. An absence of
hyperpigmentation in resolving lesions and a prolonged or recurring course distinguishes
infantile acropustulosis from TNPM.Miliaria rubra (prickly heat or heat rash) is caused
by intraepidermal obstruction of the sweat ducts. A secondary local inflammatory response
is responsible for the erythema (Item C33F) associated with the papules and vesicles.
Miliaria rubra occurs later than miliaria crystallina,usually beyond thefirst postnatal week.
Hyperpigmented, frecklelike lesions are not expected in miliaria rubra.

159) NEC is associated with:-


A. Epidemic
B. Thrombocytopenia
C. Malabsorption
D. Bile stained aspirate
E. Air in the portal tree on abdominal X –ray
Ans:- ABCDE
160) The presenting feature of NEC include:-
A. Apnoea
B. Bradycardia
C. Abdominal distension
D. Intramural gas on abdominal X ray
E. Bloody stool
Ans:- ABCDE
161) A 3-week-old breastfed infant presents to the emergency department with
irritability,fever, jaundice, and hepatomegaly. A laboratory evaluation shows a
normal complete blood count and a bilirubin concentration of 6.5 mg/dL (111.2
mcmol/L). A urinalysis is positive for reducing substances. A blood culture is positive
for Escherichia coli. You initiate antibiotic therapy Of the following, the MOST
appropriate dietary management of this patient is to
A. continue breastfeeding
B. switch to a cow milk-based formula
C. switch to a soy-based formula
D. switch to a whey hydrolysate formula
E. switch to an elemental formula
Preferred Response: C
The clinical features of jaundice, hepatomegaly, and invasive Escherichia coli infection
described for the neonate in the vignette suggest the possible diagnosis of galactosemia.
Galactosemia is an autosomal recessive disorder most commonly caused by a deficiency
of the enzyme galactose-1-phosphate uridyltransferase. The reducing substances in the
urine represent the accumulation of galactose. In addition to recognizing and treating the
gram-negative infection in the infant, it is important to remove lactose, which is comprised
of glucose and galactose,from the diet as soon as the diagnosis is suspected.
Soy protein formulas are the first choice of nutrition for infants who have suspected or
proven galactosemia because the carbohydrate source in these formulas is sucrose or corn
syrup rather than lactose. Protein hydrolysate and elemental formulas also contain other
carbohydrates than lactose, but they can be more expensive and less readily available than
soy formulas. Lactose is the primary carbohydrate in human milk, cow milk-based
formulas, and most whey hydrolysate formulas.
162) Hydrops fetalis may be associated with :-
A. Rhesus isoimmunization
B. Paroxysmal supraventricular tachycardia
C. CMV infection
D. Achondroplasia
E. Renal vein thrombosis
Ans:- ABCDE
163) The following drugs are correctly paired with their potential teratogenic effect:-
A. Alcohol & macrocephaly with congenital heart disease
B. Phenytoin & meningomyelocele
C. Isotretinoic acid & cutis laxis syndrome
D. Penicillamine & facial abnormalities with pinna defects
E. Lithium & Ebtein's anomaly
Ans:-BCDE
Warfarine Hypoplastic nasal bridge ,chrondroplasi puncta
Isotretinoic acid facial , ear , cardiovascular abnormality
Phenytoin Hypoplastic nail, IUGR, typical facies & may be associated with
neural tube defect Tetracyclin Enamel hypoplasia
Sodium Valproate Neural tube defect
164) The neonate of mother with SLE may demonstrate:-
A. Polythycemia
B. Rash
C. Neutropenia
D. Atrial fibrillation
E. Anti-Ro antibodies
Ans:- BCE
165) Pulmonary hypoplasia is consequence of :-
A. Congenital varicella zoster
B. Anencephaly
C. Posterior urethral valve
D. Congenital diaphragmatic hernia
E. Exomphalos
Ans:- BCDE
166) The following conditions may present with bile-stained vomiting in the first
week of life:-
A. Duodenal atresia
B. Cystic fibrosis
C. Inguinal hernia
D. GER
E. NEC
Ans:- ABCE
167) Renal immaturity in normal neonate born at term is manifested as:-
A. Reduced number of nephrons
B. Decreased glucose reabsorption
C. Increased glomerular filteration rate
D. Decreased renal bicarbonate reabsorption
E. Decreased urea excretion
Ans:- BDE
168) The following maternal factors increase the incidence of surfactant deficient
RDS:-
A. Steroid therapy
B. Opiate
C. Placental insufficiency leading to IUGR
D. Diabetus
E. Alcoholism
Ans:- D
169) Bacteria commonly isolated in case of neonatal meningitis include:-
A. Escherichia coli
B. Haemophilus infuenzae
C. Group B streptococcus
D. Staphylococcu epidermidis
E. Neisseria meningitis
Ans:- AC
170) In the infant of diabetic mother :-
A. The infants brain size is increased beyond normal
B. The infant's liver size is increased beyond normal
C. He can be small for gestational age
D. There is an increased incidence of polycythemia
E. He has an increased incidence of hypertrophic Cardiomyopathy
Ans:- BCDE
171) Galactosaemia:-
A. Is caused by deficiency of the enzyme galactokinase
B. Causes jaundice in the newborn
C. May present with cataract at birth
D. Is associated with Escherichia coli septicemia
E. Is diagnosed as result of screening in the majority of cases.
Ans:- BCD
172) SVT in neonate:-
A. Is the most common abnormal tachycardia
B. Reflect underlying congenital heart disease in the majority of cases
C. Show regular rate of 160-220 beats /min on the ECG
D. Recurrent episodes usually persist into adulthood
E. May be stopped with rapid I/V bolus of adenosine
Ans:- A E
173) Neonates suffering withdrawal from in utero exposure to narcotics may show
sign of
A. Irritability
B. Vomiting
C. Photophobia
D. Hypotonia
E. Diarrhea
Ans:- ABCE
174) Clinical feature of congenital hypothyroidism diagnosed in the newborn period
include:-
A. Large tongue
B. Presence of third fontenalle
C. Umbilical hernia
D. Loose stools
E. High incidence of mental retardation
Ans:- A C
175) Causes of persistent neonatal unconjugated hyperbilirubinemia after 2 weeks
include:-
A. Rhesus incompatibility
B. Hypothyroidism
C. Breast milk jaundice
D. Rotor syndrome
E. Sepsis
Ans:- ABCE

176) Growth retardation babies are at increased risk of:-


A. Polycythemia
B. HMD
C. Hypoglycemia
D. Group B streptococcal infection
E. Sudden intrapartum death
Ans:- ACE
177) Oesophageal atresia is associated with:-
A. Maternal polyhydramniose
B. Vertebral anomalies
C. Diaphragmatic hernia
D. Low birthweight
E. Duplex collecting system
Ans:- ABDE
178) Congenital Rubella:-
A. Is associated with cerebral calcification
B. Frequently lead to cataract
C. Is associated with VSD
D. Rarely occur following maternal infection in the third trimester
E. Should be prevented by vaccinating women found to be seronegative during the
first trimester
Ans:- B D
179) Human breast milk contain:-
A. Secretory IgA
B. Macrophages
C. Lysozyme
D. Vitamin c
E. Zinc
Ans:- ABCDE
180) The following are recommended daily requirement for 1 month old child /hg of
B wt:-
A. Fluids 150-180 ml orally
B. Calories 90-115 kcal
C. Sodium 1.25-2.5 mmol
D. Protein 2.2-3.5 gm
E. Potassium 2.0-3.5 mmol
Ans:- ABCDE
181) Compared with cow's milk, human breast milk contains:
A. Less sodium.
B. Less calcium.
C. Less protein.
D. Less fat.
E. Less carbohydrate.
Ans:-A B C

182) UK infants are routinely immunised against:


A. Meningococcus type B.
B. Diphtheria.
C. Cholera.
D. Polio.Pertussis.
Ans:-B D E
183) Mature breast milk contain per 100 ml;-
A. 60-75 kcal
B. 8-10 gm of protein
C. 0.1-0.5 gm of fat
D. 0.1-0.5 gm of carbohydrate
E. 5.2 mg of folic acid
Ans:- AE
carbohydrate MATURE BREAST MILK 7.4 COWS MILK gm 4.6 gm
Fat MATURE BREAST MILK 4.2 gm COWS MILK 3.9 gm
protein MATURE BREAST MILK 1.1 gm COWS MILK 3.4 gm
Calories MATURE BREAST MILK 70 COWS MILK 67
folic acid MATURE BREAST MILK 5.2 mg COWS MILK 3.7 mg
184) Treatment of PDA with Indomethacin:-
A. Should be first line therapy
B. Is safe in renal failure
C. Should not be given if there is thrombocythenia
D. Should be avoided in jaundiced babies
E. Should not be given in the presence of IVH
Ans:- D E
185) Retinopathy of prematurity ;-
A. Develop in the first week of life
B. Is more likely to occur in VLBW infants
C. Is recognized complication of hypoglycemia
D. Rarely resolve spontaneously
E. May be treated effectively with Laser therapy
Ans:- B E
186) Alpha fetoprotein level from maternal serum may be helpful in diagnosing:-
A. Congenital cardiac malformation
B. Prune-belly syndrome
C. Cleft lip & palate
D. Down syndrome
E. Fetal alcohol syndrome
Ans:- D
187) A 2-week-old infant is jaundiced. Findings include weight and length at the 75th
percentile for age; icterus; with hepatosplenomegaly; total bilirubin, 6.3 mg/dL;
direct bilirubin, 5.5 mg/dL; alanine aminotransferase activity, 130 U/L; aspartate
aminotransferase activity, 143 U/L; and gamma-glutamyl transpeptidase activity, 950
U/L.Of the following, the BEST study to evaluate the excretion of bile from the liver
is
A. computed tomography of the liver
B. hepatic ultrasonography
C. hepatobiliary scintigraphy
D. measurement of galactose-1-phosphate uridyltransferase activity
E. measurement of the serum alpha1-antitrypsin level
Ans:- C
188) Which of the following constellations of features BEST describes the fetal
alcohol syndrome?
A. Elfin facies, irritability, and supravalvular aortic stenosis
B. Growth deficiency with microcephaly, developmental delay, and short palpebral
fissures
C. Intrauterine growth retardation, triangular-shaped face, and clinodactyly of the
fifth finger
D. Short stature, webbed neck, and pulmonic stenosis
E. Weakness, club feet, immobile face, and inadequate respirations
Ans:- B
189) A female infant born to a 24-year-old woman has been diagnosed clinically as
having Down syndrome. The mother is concerned about her risk of having another
child who has achromosomal abnormalityThe statement that you are MOST likely to
include in your discussion is that her risk
A. can be estimated by determination of maternal serum alpha-fetoprotein in all
future pregnancies
B. cannot be estimated until her infant's chromosome complement has been
determined
C. is increased for Down syndrome, but not for any other chromosomal abnormality
D. is no greater than that of any other woman her age
E. is not increased until she reaches the age of 35
Ans:- B

190) During delivery of an infant who has an estimated gestational age of 42 weeks,
you note that the amniotic fluid looks like pea soup and contains thick particles of
meconium.Of the following, the MOST important initial step in resuscitation of the
infant is to :-
A. aspirate the gastric contents
B. determine the Apgar score initiate tracheal intubation
C. provide positive pressure ventilation
D. suction the hypopharynx
Ans:- D
191) In addition to irritability, sweating, and difficulty breathing with feeding, the
symptom that is MOST indicative of congestive heart failure in a 3-week-old infant is
A. ascites
B. cough
C. cyanosis
D. diminished feeding volume
E. pretibial edema
Ans:- D
192) The decreased incidence of enteric infections noted in breastfed infants
compared with formula-fed infants is MOST likely due to the
A. more alkaline stool pH in breastfed infants
B. nutritional benefits of human milk on the infant's immune system
C. predominance of Bacteroides and Clostridium in the gut of breastfed infants
D. presence of protective antibodies against enteric infection in human milk
E. sterility of human milk
Ans:- D
193) A newborn who weighs 600 g and whose estimated gestational age is 24 weeks at
birth is admitted to the neonatal intensive care unit after successful resuscitation in
the delivery room. Arterial blood gas measurements on room air are: pH, 7.35;
PCO2, 42 mm Hg; PO2,68 mm Hg; base deficit, 2 mEq/L. Of the following, the
MOST appropriate initial management is to
A. begin intravenous vancomycin
B. begin phototherapy
C. initiate enteral feeding
D. provide bicarbonate infusion
E. provide glucose infusion
Ans:- E
194) A 3-day-old infant presents to the emergency department with vomiting,
lethargy,hypotonia, and jaundice. Physical examination reveals hepatomegaly and
neurologic depression. A full sepsis evaluation is undertaken, and the Gram stain of
the cerebrospinal fluid reveals gram-negative organisms. Of the following, the BEST
additional laboratory test to obtain is
A. erythrocyte galactose-1-phosphate
B. liver glycogen content
C. plasma insulin level
D. plasma very long-chain fatty acids.
E. stool porphyrins
Ans:- A
195) You are examining a term newborn in the nursery. His weight is 3.27 kg (50th
percentile), and his length is 50.5 cm (50th percentile). The pregnancy, labor, and
delivery were unremarkable. There are no significant findings on physical
examination. The MOST likely head circumference in this child, if it is consistent
with his other growth parameters, is
A. 31 cm
B. 33 cm
C. 35 cm
D. 37 cm
E. 39 cm
Ans:- D
196) previously healthy 5-day-old male who was born at home develops bruising and
melena.The pregnancy, delivery, and postnatal course were unremarkable. The
infant is breastfeeding vigorously every 2 hours. Findings on physical examination
are unremarkable except for several large bruises. Laboratory testing reveals:
hemoglobin, 81 g/L (8.1 g/dL);white blood cell count, 9.4 x 109/L (9,400/mm3);
prothrombin time, 37 seconds; partial thromboplastin time, 98 seconds; platelet
count, 242 x 109/L (242,000/mm3); and fibrinogen,2.34 g/L (234 mg/dL). Of the
following, the MOST likely cause of the bleeding is
A. disseminated intravascular coagulation
B. factor VIII deficiency hemophilia
C. liver disease
D. vitamin K deficiency
E. von Willebrand disease
Ans:- D
197) A newborn whose estimated gestational age is 42 weeks is stained with
meconium.Tracheal intubation reveals meconium in the hypopharynx as well as
below the vocal cords.The infant has respiratory distress. A chest radiograph is
obtained. Of the following, the MOST likely radiographic finding is
A. coarse infiltrates
B. decreased lung volumes
C. mediastinal shift
D. pleural effusion
E. reticulogranular pattern
Ans:- A
198) A 20-year-old primigravida at 30 weeks of gestation has a blood pressure of
160/112 mm Hg, serum total bilirubin level of 44.5 mcmol/L (2.6 mg/dL), serum
alanine aminotransferase level of 150 U/L, and platelet count of 75 x 109/L
(75,000/mm3). She is hospitalized for observation and electronic fetal heart rate
monitoring. Of the following, the MOST ominous sign of fetal distress during
monitoring would be
A. early decelerations
B. increased beat-to-beat variability
C. late decelerations
D. spontaneous accelerations
E. variable decelerations
Ans:- C
199) A term newborn presents with bilious vomiting shortly after birth. Her
abdomen is distended slightly, and facial features are characteristic of Down
syndrome. She has passed a normal meconium stool. The pregnancy was complicated
by polyhydramnios. Of the following, the MOST likely diagnosis is
A. duodenal atresia
B. Hirschsprung disease
C. meconium ileus
D. midgut volvulus
E. pyloric stenosis
Ans:- A
200) An 18-year-old primigravida at 32 weeks' gestation has a blood pressure of
148/96 mm
Hg, proteinuria, oliguria, and visual disturbances. Labor is induced, and the infant is
delivered. His weight is 850 g (<10th percentile), crown-heel length is 38 cm (10th
percentile), and head circumference is 30 cm (50th percentile). Of the following, the
MOST
likely complication in this infant is
A. anemia of prematurity
B. hyaline membrane disease
C. hyperglycemia
D. meconium aspiration
E. perinatal asphyxia
Ans:- E
201) An infant is born at 27 weeks' gestation following a pregnancy complicated by
preterm labor that progressed despite administration of a tocolytic agent. Of the
following, the most appropriate INITIAL management is to
A. measure transcutaneous oxygen saturation
B. perform endotracheal intubation
C. place an umbilical arterial catheter
D. place the infant in an open bed warmer
E. provide nasal continuous positive airway pressure
Ans:- D
202) Of the following, the MOST important determinant of neurodevelopmental
outcome of VLBW infants is
A. antenatal obstetric management
B. infant gender
C. length of gestation
D. maternal education
E. socioeconomic status
Ans:- C
203) You are called to the newborn nursery to examine an infant who appears
dysmorphic.On physical examination, the baby is normally grown and vigorous. You
note overfolded pinnae, deviation of the nose to one side (Item Q25A), and a small
chin. The feet are maintained in dorsiflexion (Item Q25B), but can be corrected
passively. You review the pregnancy history. Of the following pregnancy
complications, the one MOST likely to be associated with this baby’s features is
A. maternal hypertension
B. polyhydramnios
C. preeclampsia
D. transverse lie
E. vaginal bleeding
Preferred Response: D
The infant described in the vignette displays the characteristic features of a deformation
sequence. Unlike malformations, which occur due to intrinsic problems within a
developing structure,deformations are due to mechanical forces acting on an otherwise
normally developing embryo or fetus. Causes of fetal deformation are many and include
oligohydramnios, prolonged breech positioning, a small or malformed uterus, fibroid
tumors of the uterus, and multiple gestations.Often, the affected infant has a pugilistic
facies, with deviation of the nose to one side. Limb positioning defects are common.
Barring any association with malformations or disruptions, the appearance of the affected
infant typically normalizes over time.Although maternal hypertension and preeclampsia
can be associated with placental insufficiency and, ultimately, decreased fetal movement,
typically they are not associated with fetal deformation. Polyhydramnios usually is
associated with unrestricted fetal movement,and fetuses are not deformed. Vaginal
bleeding has multiple potential causes, most of which are not associated with deformation.
CHILDREN HOSPITAL –BENGHAZI ABDULRAHMAN BASHIR 45Transverse lie is the only
pregnancy complication listed that is likely to be associated with deformation due to
unusual fetal position
204) A 10-day-old infant who weighed 1,750 g at birth and whose gestational age was
34 weeks is jaundiced. His total serum bilirubin concentration is 10.0 mg/dL and the
direct fraction is 0.8 mg/dL. He is receiving intermittent orogastric feeding of
expressed human milk and supplemental parenteral nutrition. Of the following, the
MOST likely explanation for these findings is
A. Crigler-Najjar syndrome
B. jaundice due to parenteral nutrition
C. neonatal hepatitis
D. physiologic jaundice
E. pyloric stenosis
Ans:- D
205) Early hospital discharge is defined as the discharge of a newborn earlier than 48
hours following vaginal delivery or 96 hours following cesarean delivery. Of the
following, the MOST common reason for readmission to the hospital within 7 days
following an early discharge is
A. bacterial sepsis
B. congenital heart disease
C. gastrointestinal malformation
D. hyperbilirubinemia
E. metabolic disorders
Ans:- D
206) A newborn infant is delivered by emergent cesarean section at 41 weeks'
gestation following a pregnancy complicated by a prolapsed umbilical cord and
meconium-stained amniotic fluid. At 6 hours of age, the infant has a generalized
tonic-clonic seizure. Of the following, the MOST likely explanation for this seizure is
A. hyponatremia
B. hypoxic-ischemic encephalopathy
C. intracranial hemorrhage
D. meningitis
E. pyridoxine dependency
Ans:- B
207) A 4-hour-old newborn has copious oral secretions and episodes of coughing,
choking, and cyanosis. The pregnancy was complicated by polyhydramnios. You
suspect esophageal atresia with tracheoesophageal fistula.Of the following, the
MOST helpful test to confirm the diagnosis is to
A. inject a contrast medium through an orogastric catheter and obtain a neck
radiograph
B. obtain computed tomography of the neck
C. perform flexible bronchoscopy
D. place an endotracheal tube and examine the endotracheal fluid
E. place an orogastric suction catheter and obtain a chest radiograph
Ans:- E
208) A newborn is delivered by emergent cesarean section because of fetal distress
following acute abruption of the placenta. The infant is resuscitated and transferred
to the nursery.On physical examination, she appears pale, and her extremities are
cold to touch. The capillary refill is 8 seconds. Results of an arterial blood gas
analysis show a Po2 of 48 mm Hg. Of the following, the MOST likely additional
finding is
A. decreased bicarbonate concentration
B. hypercalcemia
C. hyperglycemia
D. hyponatremia
E. normal anion gap
Ans:- A
209) Of the following, erythromycin prophylaxis is MOST likely to prevent ocular
infection due to
A. Chlamydia trachomatis
B. group B streptococci
C. Neisseria gonorrhoeae
D. Staphylococcus aureus
E. Trichomonas vaginalis
Ans:- C
210) A 1,300 g infant who is born at 34 weeks' gestation has a head circumference of
27 cm and crown-heel length of 40 cm. At 48 hours of age, she is irritable, tremulous,
and inconsolable. Her systolic blood pressure is 65 mm Hg and heart rate is 180
beats/min. Her face appears normal, and her cry is high-pitched. Cranial
ultrasonography reveals bilateral echo densities suggestive of periventricular
leukomalacia. Of the following, the MOST likely explanation for the findings in this
infant is maternal exposure to
A. alcohol
B. barbiturates
C. cocaine
D. marijuana
E. opiates
Ans:- C
211) A 4.3 kg infant is delivered to a woman whose diabetes mellitus is poorly
controlled. Of the following, the MOST likely neonatal manifestation of maternal
diabetes is
A. diabetic ketoacidosis
B. Hirschsprung disease
C. hypercalcemia
D. polycythemia
E. renal vein thrombosis
Ans:- D
212) Of the following, the MOST helpful finding to distinguish GBS pneumonia from
RDS is
A. a normal C-reactive protein level
B. an elevated erythrocyte sedimentation rate
C. diffuse alveolar infiltrates on chest radiography
D. increased ratio of bands to segmented neutrophils
E. persistent hypoxemia on blood gas analysis
Ans:- D
213) An 18-hour-old infant of a diabetic mother develops abdominal distension.
Physical examination reveals a protuberant, firm, but nontender abdomen; patent
anus; and no grossly visible anomalies. The infant has passed no meconium stool
since birth. A supine abdominal radiograph reveals multiple dilated loops of
intestine.Of the following, a contrast enema would MOST likely confirm a diagnosis
of
A. atresia of the colon
B. Hirschsprung disease
C. hypoplastic left colon syndrome
D. meconium ileus
E. midgut volvulus with malrotation
Ans:- C
214) You are attending the emergency delivery by cesarean section of a primiparous
woman.The gestation was complicated by pregnancy-induced hypertension. Deep
variable fetal
heart rate decelerations were noted during labor. At delivery, the infant is
acrocyanotic with
poor tone; spontaneous movement and minimal respiratory effort are present. Of the
following, your INITIAL management is to
A. ascertain the heart rate and assign a 1-minute Apgar score
B. begin tactile stimulation and provide blow-by oxygen supplementation
C. dry all skin surfaces and clear the oropharynx
D. initiate bag-mask ventilation
E. insert an umbilical catheter and administer naloxone
Ans:-C
215) A vigorous, normal-appearing term male newborn has not voided by 18 hours
after delivery. Perinatal history is negative for maternal illness or medications.
Amniotic fluid volume was reportedly normal, and the delivery was uneventful, with
Apgar scores of 6 and 9 at 1 and 5 minutes, respectively. Of the following, the MOST
likely reason why this 18-hour-old infant has not voided is
A. bilateral ureteropelvic junction obstruction
B. intravascular volume depletion
C. neurogenic bladder
D. posterior urethral valve
E. undocumented void in the delivery room
Ans:- E
216) A 2-week-old neonate who was born at 32 weeks’ gestation has recovered from
respiratory distress syndrome. He has been tolerating increasing volumes of enteral
feedings via gavage. Over the past several feedings, abdominal distension, gastric
residuals, and stools that are positive for blood have been noted. Of the following, the
radiographic finding MOST supportive of the diagnosis of necrotizing enterocolitis is
A. absence of luminal bowel gas
B. generalized bowel distension
C. intraperitoneal fluid
D. pneumatosis intestinalis
E. thickening of the bowel wall
Ans:- D
217) A term infant is placed under a radiant warmer, the skin is dried, and the
oropharynx and nose are suctioned. After tactile stimulation, there is minimal
respiratory effort, dusky color, and a heart rate of 86 beats/min. Bag/mask
ventilation is performed for 30 seconds with 100% oxygen at a rate of 40 to 60
breaths/min. The heart rate increases to 100beats/min. Of the following, the NEXT
best step is to:
A. administer sodium bicarbonate
B. continue bag/mask ventilation at a rate of 20 to 40 breaths/min
C .continue ventilation and begin chest compressions
D. observes for spontaneous respiration and discontinues ventilation
E. perform endotracheal intubation
Ans:-D
218) A 900 g male infant is delivered vaginally to a woman who had no prenatal care.
Of the following, the physical finding that is MOST consistent with prematurity
rather than intrauterine growth restriction is
A. creases over entire sole of foot
B. descended testes with deep rugae of the scrotum
C. formed and firm pinna with instant recoil
D. gelatinous translucent skin
E. raised areola and 3 mm breast buds\
Ans:-D
219) A 1-day-old infant develops bilious vomiting and gastric distension. She has
been afebrile and has been passing meconium-laden stools. Of the following, the most
appropriate INITIAL step in the management of this infant is
A. abdominal radiography to look for the “double-bubble” sign
B .culture of a catheterized urine specimen
C .insertion of a rectal tube for decompression
D .placement of a nasogastric tube and initiation of intravenous fluid therapy
E. upper gastrointestinal radiographic series to look for malrotation of the small bowel
Ans:- D
220) While performing ultrasonography on a 31-week fetus, an obstetrician notes
that the fetal heart rate ranges from 62 to 66 beats/min. Fetal growth appears
normal, and no structural cardiac anomalies are identified. Fetal echocardiography
reveals that the fetal atria appear to be contracting at 140 beats/min, with a
ventricular rate of 65 beats/min. Of the following, the NEXT step in the management
of this infant is to:-
A. administer beta-agonist drug therapy to the mother
B. assess the cardiac status of the infant following labor and delivery
C. counsel the parents that intrauterine fetal death is likely
D. perform amniocentesis to confirm lung maturity and if mature, perform immediate
cesarean section
E. repeat the fetal echocardiography and fetal ultrasonography in 1 week
Ans:-E
222) Term infant is cyanotic and requires intubation. Findings include: heart rate,
175beats/min; blood pressure, 60/30 mm Hg; increased right ventricular activity;
single S2;short systolic murmur; and equal arm and leg pulses; chest radiography,
normal heart size and pulmonary congestion. Arterial blood gases (right radial
artery on 100% FIO2): pH,7.31; PO2, 43 torr; PCO2, 48 torr. Of the following, the
MOST likely diagnosis is:-
A. hyaline membrane disease
B. hypoplastic left heart
C. intrauterine constriction of the ductus arteriosus
D. tetralogy of Fallot
E. total anomalous pulmonary venous connection
Ans:-E
223) A 7-day-old infant has copious purulent discharge from both eyes. The 17-year-
old mother currently complains of a yellowish vaginal discharge. The only
medications received by the infant were vitamin K and topical erythromycin
prophylaxis following delivery.Giemsa stain of a conjunctival scraping reveals
intracytoplasmic inclusions. After obtaining appropriate diagnostic studies, the
BEST management includes treatment with :-
A. oral erythromycin
B. oral penicillin
C. topical erythromycin
D. topical gentamicin
E. topical sulfonamide
Ans:-A
224) Of the following, the condition that is MOST likely to present with seizures
during the
first 24 hours of life is
A .fetal alcohol syndrome
B .herpes simplex infection
C. hypoxic-ischemic encephalopathy
D. organic acidemia
E. urea cycle defect
Ans:-C
225) Maternal serum alpha fetoprotein measured at 12 weeks gestation
A. If normal then neural tube defect are excluded
B. Is increased in twin pregnancy
C. Is reduced in Turner syndrome
D. Is increased in Down syndrome
E. AFT is the main blood protein fraction in the first trimester
Ans :- BE
Serum a fetoprotein is usually measured at 16-18 week gestation, normal screening would
not rule out neural tube defect or any other lesion such as twins, Turner's syndrome,
Exomphalus,which can all have raised value .
226) A newborn presents with an early onset of dyspnea with chest retractions,
expiratory grunting and cyanosis following an uneventful normal preterm labor. On
examination no cardiac murmurs are heard and the lungs appear clear. On a plain
X-Ray there is evidence of prominent pulmonary vascular markings and fluid lines
in the fissures. The cyanosis improves with minimal oxygen. The most probable
diagnosis is :-
a) Meconium aspiration syndrome
b) Fetal aspiration syndrome
c) Transient tachypnea of the newborn
d) Persistent fetal circulation
e) Hyaline membrane disease
Ans:- C
EXPLANATION
The clinical case given above indicates the likely diagnosis of transient tachypnea of the
newborn,(Choice C) which is otherwise called Respiratory distress syndrome type II. It is
characterized by the early onset of dyspnea with chest retractions, expiratory grunting and
cyanosis. Neonates usually recover within 3 days.The syndrome is believed to be
secondary to slow absorption of fetal lung fluid resulting in decreased pulmonary
compliance and tidal volume.The distinctive feature of transient tachypnea of the newborn
from hyaline membrane disease (Choice E) are the infant’s sudden recovery and the
absence of a radiographic reticulogranular pattern.In meconium aspiration syndrome
(Choice A) the chest X-ray is characterized by patchy infiltrate,coarsestreaking of both
lung fields and flattening of diaphragm. A normal chest X-ray in an infant with severe
hypoxia and no cardiac malformation suggests the diagnosis of persistent fetal
circulation. ( Choice D).Fetal aspiration syndrome(Choice B) is very common during
prolonged labor and difficult deliveries and may initiate vigorous respiratory movements
in-utero because of interference with oxygen supply
227) A neonate on the 5th day of life was diagnosed to have developed severe
meningitis.Serology of the CSF revealed that the causative organism was
Haemophilus influenza.The neonate was started on chloramphenicol. After 36hrs the
neonate started vomiting,and became flaccid. The baby was also found to be
hypothermic. These symptoms are result of the neonate having an immature
a) hepatic function
b) immune system
c) renal tubule system
d) hypothalamic regulatory system
e) peripheral neurological system
Ans: A
EXPLANATION
Chloramphenicol is the drug of choice in Haemophilus influenza due to its high efficacy
against H.influenza.Gray baby syndrome is the name given to chloramphenicol toxicity
seen in characteristically neonates even at low therapeutic levels. The cause for this is that
neonates have an immature hepatic function and lack an effective glucoronic acid
conjugation mechanism for degradation and detoxification of chloramphenicol. So even at
low therapeutic levels the drug accumulates in the body causing toxicity which is
manifested by vomiting, flaccidity, hypothermia, gray color and ultimately shock and
collapse.
1. Indomethacin give to preterm baby may cause :-
A. Hypernatremia
B. Hyperkalemia
C. Polyuria
D. Hypoglycemia
Ans:- B
Contraindications to indomethacin include thrombocytopenia (<50,000/mm3),
bleeding disorders, oliguria (<1 mL/kg/hr), necrotizing enterocolitis, isolated
intestinal perforation, and an elevated plasma creatinine level (>1.8 mg/dL).
2. Na- Valproate used in early pregnancy may result in fetal:-
A. Cardiac anomaly
B. Platelet dysfunction
C. Neural tube defect
Ans :-C

3. α fetoprotein increased in :-
A. Turner syndrome
B. Trisomy 13
C. Rh isoimmunization
D. Spontaneous abortion
Ans:- A
4. During management of patient with sever meconium aspiration the following can
be used:-
A. IPPV
B. CPAP
C. Nitroprusside
D. Dexamethasone
Ans:-A
5. Hypoxic-ischemic encephalopathy all are correct except:-
A. Result from excessive use of oxytocin
B. IUGR is be first indication of fetal hypoxia
C. Associated with increased beat to beat variability
D. Prognosis depend on gestational age
E. Persistent of abnormal neurological sign at two week indicate poor prognosis
Ans:- C
6. Apgar score include the following except :-
A. Muscle tone
B. Response to catheter in nostrils
C. Heart rate
D. Respiratory rate
Ans:- D

7. Intracranial hemorrhage :
A. Primary hemorrhage disorder give rise to intraventricular hemorrhage
B. intraventricular hemorrhage is symptomatic immediately after birth
C. neurosurgical intervention is indicated early in the ?course of the condition
D. benefit of reducing intracranial pressure have not been established
8. HMD:-
A. Pathological finding are rarely seen in infant dying early than 8 hour after birth
B. In sever cases grunting is absent
C. Death is rare in the 1st day of illness
D. X- ray finding are pathognomic
Ans:- AB C
Death is rare on the 1st day of illness, usually occurs between days 2 and 7, and is
associated with alveolar air leaks (interstitial emphysema, pneumothorax), pulmonary
hemorrhage, or IVH, On xray,the lungs may have a characteristic, but not pathognomonic
appearance that includes a finereticular granularity of the parenchyma and air
bronchograms, which are often more prominent early in the left lower lobe because of
superimposition of the cardiac shadow
9. About HMD all correct except :-
A. Occur in 60% of infant (28 week of age)
B. Increased frequency is associated with previous history of affected infant
C. High oxygen lead to decreased surfactant
D. Atelectasis lead to ventilated but not perfused alveoli
E. HMD is accompanied by Rt → Lt shunt
Ans:- D
10. All of the following are true except:-
A. TTN isn't seen in normal delivered infant
B. Patient with TTN may have pleural effusion
C. Meconium stained amniotic fluid is seen in 10% of live birth
D. Meconium aspiration syndrome may lead to metabolic acidosis
E. Patient with meconium aspiration may get beneficial from ECMO
Ans:- A
11. natural thermal environment for 1-2 kg at 12 hour of age is :-
A. 37
B. 36
C. 34
D. 38
E. 32
Ans:- C
12. To increase the Hb of newborn by 1gm/dl the amount of packed cell required will
be :-
A. 2 cc/kg
B. 4 cc/kg
C. 5 cc/kg
D. 9 cc/kg
E. 15 cc/kg
Ans:- A
13. About birth injury all are correct except
A. C1-C2 level is the commonest site of injury
B. In spinal injury , the patient may die before appearance of neurological sign
C. Erb's palsy infant loss power to abduction
D. Erb's palsy infant, Bicep's reflex is absent
E. Erb's palsy infant, preserve of hand grasp is favorable prognosis
Ans: A
14. The following associated with polyhydramniose except;-
A. Indomethacin administration
B. Diaphragmatic hernia
C. Achondroplasia
D. Adenomatoid cystic lung
E. Trisomy 18
Ans:- A
15. Birth injury:-
A. Paralysis of the upper arm has better prognosis than paralysis of the lower
B. In nerve injury, neuroplasty is advised at the end of first year of life
C. In phrenic nerve paralysis, spontaneous recovery is expected
D. Facial nerve paralysis will result from the muscular agenesis of the facial nerve
Ans:-A C
16. All statement are true except:-
A. IVH is uncommon in infant > 34 week gestation
B. The incidence of VSD is high in VLBW infant ( PT > PDA)
C. Hypoglycemia seen more commonly in patient with SGA
D. Hypercarbia is risk factor for …..
E. Decrease vitamin E may cause hemolytic anaemia
17. Breast milk composition all true except:-
A. Protein 2gm/dl
B. Carbohydrate 7 gm /dl
C. Ca/Po4 33/15 mg/dl
D. Fat 4 gm/dl
E. Osmolality 290-300 mosm
Ans:- A
18. All statement are true except:-
A. Prematurity is predisposing factor for birth asphyxia
B. Cephalohematoma is accompanied with discoloration of the scalp
C. Sensation of central depression suggesting fracture accompanied
cephalohematoma
D. Occipital bone fracture is fatal
E. Subconjunctival hemorrhage is due to increase intrathoracic pressure
Ans:- B
19. Sign of ICH include all of the following except:-
A. Enlarged head
B. Fixed dilated pupil
C. Neck stiffness
D. Persistent cyanosis
E. Sweating
Ans:- E
20. Exchange transfusion in preterm baby may be complicated by:-
A. Hemoglobinuria
B. Hemoglobinemia
C. Thrombocytopenia
D. IVH ??
E. GVHD
Ans:-
21. For treatment of hyperbilirubinemia all are true except:-
A. Phototherpy ( 250-320 nm) ??
B. Intravenouse immunoglobuline
C. Metalloporphyrins (Tin (Sn)-protoporphyrin ).
D. Activated charcoal
Ans:- A
Clinical jaundice and indirect hyperbilirubinemia are reduced on exposure to a high
intensity of light in the visible spectrum. Bilirubin absorbs light maximally in the blue
range (420–470 nm).
22. Necrotizing enterocloitis :-
A. USS detect microbabbules of gas in the GIT wall is used recently
B. Vitamine E is used in the treatment of NEC
C. Maternal intake of cocaine may predispose to NEC
D. Breast milk protect against NEC
Ans:- C D
23. All of the following are correct except:-
A. Generalized oedema occur with Hurler syndrome
B. Cyanosis can be mask with pale
C. Postmature tend to have pale skin than term
D. Periodic respiration is common in the first 24 hour
E. Wormian bone associated with cliedo dystosis
Ans:-D
24. The following fact give false positive ( low) Apgar score except:-
A. Mg sulfate
B. Congenital myopathy
C. High fetal calcitonine level
D. Immaturity
E. Airway obstructed
Ans:-C
25. IDM:-
A. Increased insulin level in infant born to mother with gestational diabetus
B. Decrease level of FFA
C. Decrease Epinephrine & Glucagons response
D. Cortisol & human growth hormone are normal
Ans:- A BC D
26. Regarding thyroid problem during neonatal period all are false except:_
A. Maternal thyrotoxicosis during pregnancy can be treated safely with propranolol
& thiouracil
B. Mother should be kept mildly hypothyroidism
C. Breast milk may mask neonatal hypothyroidism
D. If the mother on thyroxine should stop breast feeding
E. Carbimazole can be used safely in nursing mother
Ans:-E
27. During morning rounds in the newborn nursery, you examine a healthy infant
who has blotchy erythematous macules that are 2 to 3 cm in diameter. The macules
are scattered over the trunk, face, and proximal extremities; the palms and soles are
spared. Each macule has a 1- to 3-mm central vesicle or pustule.Of the following, the
MOST likely additional finding in this patient is
A. pigmented macules located at sites of resolving pustules
B. presence of lesions at birth
C. pustules coalescing into bullae
D. Wright stain of a smear of the vesicopustular contents revealing a predominance of
eosinophils
E. Wright stain of a smear of the vesicopustular contents revealing a predominance of
polymorphonuclear neutrophils
Preferred Response: D
The infant described in the vignette exhibits the classic presentation of erythema toxicum
neonatorum. Tiny vesicles or pustules arise from blotchy erythematous macules, with
lesions characteristically appearing at 24 to 48 hours after birth. The pustules do not
coalesce into bullae. Wright staining of the pustular contents reveals a predominance of
eosinophils, not neutrophils. Because the lesions are seen in healthy infants, it has been
suggested that this benign condition be renamed; suggested names include “benign
neonatal rash” or “benign erythema neonatorum.”Transient neonatal pustular melanosis
(TNPM) is another well-recognized benign dermal eruption of infancy in which pustular
lesions spontaneously resolve into transient pigmented macules . TNPM may be present at
birth, and examination of pustular contents reveals apredominance of neutrophils.Infantile
acropustulosis is a chronic or recurring benign condition manifested by intensely pruritic
pustules on hands and feet. Characteristic papules and pustules may coalesce into
bullae.Infantile acropustulosis frequently is confused with scabies infestation. The lesions
resolve spontaneously at 1 to 2 years of age.
28. Maternal vaccination during pregnancy can be given safely except:-
A. BCG
B. HBV
C. Cholera vaccine
D. Tetanus
E. Meningococcal vaccine
Ans:- A
29. Newborn infants commonly have:-
A. Capillary hemangioma on the forehead.
B. Posterior cranial fontenalle.
C. Metopic sutures.
D. Impalpable coronal sutures.
E. Skin tag in front of the ear.
Ans:- ABC
Thyroid Disorders
1. Which of the following statements is true regarding the potential effects of maternal
Graves disease on the fetus/neonate?
a. Maternal T3 is inactivated by the placenta, and therefore there is no risk of fetal
hyperthyroidism.
b. The fetus/neonate will be at increased risk of hypothyroidism.
c. There is no risk to the fetus/neonate in mothers who had definitive therapy for Graves
with thyroidectomy
prior to conception.
d. Maternal antithyroid therapy with methimazole or propylthiouracil (PTU) should be
discontinued during pregnancy because of the risk of birth defects.
1. b. Graves disease is associated with a mix of both TSH receptor stimulating and blocking
antibodies, therefore the fetus/neonate is at risk of both hyper- and hypo-thyroidism. T3 is
inactivated, however TSH antibodies do cross the placenta. Antibodies can persist even after
definitive therapy, so the fetus/neonate is at risk even if mother has had surgery/ablation.
Untreated mater-nal hyperthyroidism results in increased risk of miscar-riage, premature
labor, intrauterine growth restriction, hyper- and hypothyroidism. PTU is generally
considered first-line treatment for Graves disease in USA, though methimazole is used in
other countries and it is unclear whether the risk of birth defects is actually higher in PTU
versus methimazole. Either way, the goal of therapy is to maintain maternal free T4 levels in
the upper end of the normal range on the lowest possible antithyroid dose.
2. A full-term, newborn male is found to have an abnormal thyroid screen with
elevated TSH of 500 mIU/mL and low T4 of 2 micrograms/dL. Which of the following
is not a possible cause of this child’s hypothyroidism?
a. Activating mutation in the TSH receptor
b. Overexpression of deiodinase 3
c. Inactivating mutation in the NIS gene
d. Presence of a lingual thyroid gland
2. a. An activating mutation in the TSH receptor gene leads to hyperthyroidism and is a rare
form of neonatal thyrotoxicosis. Consumptive hypothyroidism is due to overexpression of
deiodinase, typically hepatic hem-angiomas, that leads to excessive inactivation of thyroid
hormones. Inactivating mutations in the sodium-iodine-symporter prevent iodine from
entering the follicular cell and are a form of thyroid hormone dyshormonogenesis.
Ectopically located thyroid glands are a common cause of congenital hypothyroidism, often
they are located near the base of the tongue.
3. A 36 WGA infant born to a 16-year-old first-time mother has a hypoglycemic seizure
on DOL 4. Brain imaging reveals absence of corpus callosum and hypoplasia of optic
nerves. Which pattern of thyroid hormone tests is most likely to be present?
a. Elevated TSH, low T4, low free T4
b. Normal TSH, low T4, normal free T4
c. Normal TSH, low T4, low free T4
d. Low TSH, elevated T4, elevated free T4
3. c. The infant likely has septo-optic dysplasia with clini-cal evidence of pituitary hormone
deficiency (hypogly-cemic seizure) and is therefore at risk for having central
hypothyroidism. Central hypothyroidism is marked by low thyroid hormone levels; TSH can
be low, normal, or even slightly elevated–as such, states that employ a TSH only newborn
screen may miss infants with cen-tral hypothyroidism. An elevated TSH with low thyroid
hormone levels is diagnostic of primary hypothyroid-ism. A low T4 with normal TSH and
free T4 is consis-tent with TBG deficiency. A pattern of low TSH with elevated thyroid
hormone levels is seen in hyperthy-roidism.
4. All of the following statements are true regarding fetal goiter except:
a. A finding of fetal goiter on prenatal ultrasound is
pathognomonic for the presence of thyroid dyshormonogenesis in the fetus.
b. It is associated with increased risk of fetal upper airway obstruction at delivery.
c. There is increased incidence of fetal goiter in areas
where iodine deficiency is endemic.
d. Fetal thyroid testing can be performed via percutaneous umbilical vein sampling after 20
weeks if fetal thyroid status is uncertain.
4. a. A finding of fetal goiter can be a sign of either fetal hypo- or hyperthyroidism. Large
goiters can cause airway obstruction and subsequent respiratory fail-ure at birth. Fetal
goiters due to hyperthyroidism that result from placental passage of TSH receptor stimulat-
ing antibodies can be treated with maternal antithy-roid medication (methimazole or PTU)
administration, therefore fetal blood sampling may be indicated if there is a significant goiter
and the thyroid status of the fetus is not apparent. Fetal blood sampling is associated with
1%–2% risk of miscarriage, so risks and benefits must be weighed on a case-by-case basis.
5. You are notified that an otherwise healthy term infant had an elevated TSH and low
T4 on the newborn screen.All of the following statements are true regarding the
management of congenital hypothyroidism in this infant except:
a. The starting dose of levothyroxine will be 15 micrograms/kg/day.
b. The infant should be discharged on an oral liquid preparation of levothyroxine because it
is easier to administer.
c. Serum T4 levels should be maintained in the upper one-half of the normal range for the
first year of life.
d. Feeding intolerance with poor weight gain at the first follow-up visit may be a sign of
overtreatment.
5. b. In most situations, infants should be managed with levothyroxine tablets due to
variability in the potency and stability of liquid preparations. Initial dosing is weight-based,
and then titrated based upon biochemi-cal findings and clinical response. Infants with
congeni-tal hypothyroidism therefore require frequent clinical and biochemical monitoring
during the first year of life. The goal of therapy is to normalize TSH within two weeks and
maintain T4 levels in the upper one-half of the normal range. Overtreatment is possible; the
infant may demonstrate signs/symptoms of hyperthyroid-ism including feeding intolerance,
poor weight gain or weight loss, irritability, jitters. Labs will show a sup-pressed TSH value
with an elevated T4 (or free T4).
6. An otherwise healthy appropriate for gestational age (AGA) 34 WGA male is found
to have an abnormal thyroid newborn screen on a blood spot obtained on DOL 2.You
live in a state with a primary T4 screening strategy.Follow up venous blood sample on
DOL 4 showed a normal TSH for age, low T3, low T4, normal free T4, normal free T3.
Which of the following statements is true about the most common form of his
condition?
a. He will be at risk for cognitive impairment if not started on levothyroxine immediately.
b. His lab abnormalities are due to the fact that he was born prematurely.
c. Evaluation of maternal serum would reveal elevated TRAb.
d. He will be able to pass on his condition to a daughter,but not to a son.
6. d. Labs are most consistent with TBG deficiency, a benign condition characterized by low
TBG. Treatment with levothyroxine is not indicated as free thyroid hormone levels are
normal. The inherited form of this condition is due to mutations in SERPIN7A. This gene is
located on the X-chromosome, therefore a father can only pass an abnormal copy to a
daughter. TBG deficiency due to SERPIN7A can be partial (with one abnormal copy in a
female) or complete (with no normal copies in either a male or female). Acquired forms of
TBG deficiency are also possible in the setting of protein-losing conditions. THOP is
characterized by low levels of all thyroid hor-mones; elevated TRAb levels can be seen in
mothers with Graves disease.
7. An infant with which of the following conditions would be most likely to have
elevated reverse T3 levels?
a. Congenital hypothyroidism due to athyreosis
b. Consumptive hypothyroidism
c. TBG deficiency due to SERPIN7A mutation
d. Transient hypothyroxinemia of prematurity
7. b. Consumptive hypothyroidism will result in elevated reverse T3 levels due to
overexpression of deiodinase 3. Reverse T3 levels are most likely to be low in congeni-tal
hypothyroidism and normal in TBG deficiency and THOP. The etiology of
hypothyroxinemia in premature infants is not well understood; defective TSH surge and
diminished iodide stores have been suggested as possible contributors, but definitive data is
lacking.
8. Which of the following statements is true regarding the treatment of neonatal
Graves disease?
a. All infants born to mothers with Graves disease should be started on prophylactic
antithyroid therapy
with methimazole at birth.
b. Prednisone dosed at 1 mg/kg/day is the first-line therapy for neonatal thyrotoxicosis.
c. Infants with suppressed TSH and elevated T4 levels at birth will require lifelong therapy.
d. Treatment with Lugol solution is limited to 1–2 weeks because of “escape” from the
Wolff-Chaikoff effect.
8. d. Only 1%–10% of infants born to mothers with Graves disease are affected, therefore
treatment is initiated only with clinical and biochemical evidence of hyperthyroid-ism.
Therapy includes management of symptoms of hyperthyroidism with a beta-blocker and
reduction in thyroid hormone production with an antithyroid medi-cation (typically
methimazole). High dose iodine (as provided in Lugol solution) transiently inhibits thyroid
hormone production by blocking iodide organification (Wolff-Chaikoff effect). The thyroid
gland escapes this effect in 7–10 days, possibly due to decreased transport of iodine across
the sodium-iodine symporter. Neonatal Graves is transient and usually resolves by 6 months
of age or sooner.
9. A term infant male with neonatal seizures is found to have abnormal thyroid studies,
including elevated T3 and free T3 but low-normal T4 and free T4 levels. Which of the
following statements is true regarding this condition?
a. High-dose levothyroxine (T4) treatment should be instituted.
b. High-dose Cytomel (T3) treatment should be instituted.
c. The clinical manifestations of this condition result from low intracellular concentrations of
T3 in neurons.
d. It is inherited in an autosomal recessive manner.
9. c. This patient most likely has defective thyroid hor-mone transport due to a mutation in
MCT8 (also known as SLC16A2). This transporter is required for transport of T3 into
neurons, in its absence low intra-cellular T3 levels result in abnormal neurologic find-ings
including seizure, hypotonia, and developmental delay. Other body cells are able to transport
T4 across the cell membrane with intracellular conversion into T3. Therefore treatment with
either T4 or T3 can result in a hypermetabolic state in conjunction with hyper-thyroidism.
MCT8 deficiency is inherited in an X-linked recessive pattern.
Adrenal Disorders
1. Which of the following hormones is not produced in
the adrenal cortex?
a. Cortisol
b. Dopamine
c. Aldosterone
d. Dehydroepiandrosterone
e. All of the above are produced in the adrenal cortex
1. b. Dopamine is a catecholamine synthesized in the adrenal medulla
2. Which of the following statements is true regarding
fetal adrenal steroid hormone production?
a. The adrenal glands develop and begin secreting hormones at 12 weeks.
b. The histopathologic organization of the adrenal gland in the fetus is identical to that of an
adult.

c. The fetal adrenal gland does not express 3β-hydroxysteroid dehydrogenase and is

therefore dependent on the placenta in order to synthesize


cortisol.
d. The cells of the adrenal cortex are derived from neuroectodermal precursors.
2. c. The fetal zone of the adrenal cortex produces preg-nenolone, 17-hydroxypregnenolone,
and DHEA which then must be converted into progesterone, 17-hydroxyprogesterone, and
androstenedione by the placenta which expresses 3β-HSD in abundance.
3. Which of the following genes (enzymes) is not expressed in the adrenal gland?
a. CYP19 (aromatase)
b. CYP21 (21-hydroxylase)

c. CYP11B1 (11β-hydroxylase)

d. CYP11B2 (11β-hydroxylase)

e. All of the above enzymes are expressed and active in


the adrenal gland
3. a. Aromatase, because the conversion of androgens into estrogens is expressed primarily
in gonads but is also found in other tissues such as adipose, liver, skin, and brain.
4. A patient is born with ambiguous genitalia and found to have a genetic defect
leading to loss of function of CYP11A1 (cholesterol side-chain cleavage enzyme).Which
of the following is likely true about the patient’s
karyotype?
a. The karyotype is most likely XX, and the patient is overvirilized because of excess ACTH
production.

b. The karyotype is most likely XY, and the patient is undervirilized because of elevated 5α-

reductase activity.
c. The karyotype is mostly likely XX, and the patient is overvirilized because of decreased
aromatase activity.
d. The karyotype is most likely XY, and the patient is undervirilized because of insufficient
substrate for androgen hormone biosynthesis.
4. d. CYP11A1 mutation blocks the ability of cholesterol to be transported to the inner
mitochondrial mem-brane; therefore synthesis of all steroid hormones will be diminished
and will typically result in an underviril-ized male.
5. You are alerted that one of your patients admitted for transient tachypnea of the
newborn had an abnormal newborn screen for CAH. The patient is now a 5-day-old
infant, born 40 weeks gestational age(WGA) via scheduled cesarean delivery, and has
normal appearing male genitalia. Patient is clinically improving with decreasing
fraction of inspired oxygen(fio2) requirement, normal blood pressure, and is tolerating
feeds. The newborn screen specimen was drawn at 36 hours of age, and
electrolytes drawn on DOL 1 showed normal sodium and potassium. What is the most
appropriate next step?
a. Immediately start hydrocortisone only at a dose of 15 mg/m2/day.
b. Immediately start hydrocortisone at a dose of 15mg/m2/day and fludrocortisone at a dose
of 0.1 mg/day.
c. Obtain a venous blood sample for measurement of 17-hydroxyprogesterone.
d. Obtain a venous blood sample for measurement of 17-hydroxyprogesterone and
electrolytes.
5. d. In a well appearing infant without clinical signs of adrenal insufficiency or salt wasting,
it is appropriate to assess a venous blood sample for 17-hydroxyporo-gesterone and
electrolytes as the next step. Perinatal stress is one possible source of a false-positive
newborn screen for CAH. Because clinical signs and electrolyte abnormalities of salt
wasting may not manifest in the first few days of life, it is also prudent to repeat electro-lytes
at this time.
6. You are caring for a 1-week-old former 27 WGA infant with birth weight of 1000
grams transferred to your neonatal intensive care unit(NICU) from a community
hospital on DOL 1. Patient is phenotypically male,intubated, and has required multiple
fluid boluses, inotrope,and vasopressor support to maintain adequate perfusion. You
receive a copy of the newborn screen report sent from the community hospital
reporting a17-hydroxyprogesterone value of 15,000 ng/dL. What
is the appropriate next step?
a. Do nothing; abnormal newborn screen result is
likely the result of prematurity.
b. Obtain a venous blood sample for measurement
of 17-hydroxyprogesterone; wait until results are
back to start hydrocortisone.
c. Obtain a venous blood sample for measurement of
17-hydroxyprogesterone, and start hydrocortisone
100 mg/m2/day.
d. Obtain a venous blood sample for measurement of
17-hydroxyprogesterone, and start hydrocortisone
15 mg/m2/day.
6. c. Based on the clinical picture and the finding of a 17-hydroxyprogesterone level
greater than 10,000 ng/dL, this patient most likely has 21-hydroxylase deficiency CAH.
Prematurity and timing of sample before 48 hours of age increase the possibility of a
false-positive newborn screen result, however typically not to levels >10,000 ng/dL. A
venous 17-hydroxyprogesterone sample should be obtained, but treatment should not
be delayed as the clinical picture is concerning for adrenal crisis. During times of crisis,
stress doses of hydrocortisone at 50–100 mg/m2/day should be used.
7. A newborn is found to have ambiguous genitalia including partial fusion of the
labioscrotal folds and clitoromegaly. No testes are palpable on exam. Which of the
following scenarios is NOT likely to be responsible
for the clinical presentation in this case?
a. 21-hydroxylase deficiency with karyotype XX
b. 3-beta-hydroxysteroid dehydrogenase with karyotypeXX
c. 17-hydroxylase deficiency with karyotype XX
d. 11-beta-hydroxylase deficiency with karyotype XX
7. c. 17-hydroxylase deficiency may cause ambiguous genitalia in XY males due to
undervirilization as a result of decreased androgen synthesis. All other listed enzymatic
defects can lead to ambiguous genitalia in XX females due to overvirilization as a result of
excess androgen production.
8. An 11-month-old infant male in your unit is found to be hypertensive. Review of his
medical records shows that blood pressures have been gradually increasing over the
past several weeks to months. Which of the
following is true regarding hypertension in congenital adrenal hyperplasia?
a. Hypertension in 21-hydroxylase deficiency CAH is
a sign that the Florinef dose needs to be increased.
b. Serum aldosterone levels are typically low.
c. The patient cannot have 17-hydroxlase deficiency
because he has normal appearing genitalia.
d. The majority of patients who will develop hypertension
due to 11-beta-hydroxylase deficiency are identified on the CAH newborn screen
8. b. Forms of CAH associated with early-onset hyper-tension include 11-beta-hydroxylase
CAH and 17-hydroxylase CAH. Both conditions are marked by dramatically elevated serum
levels of 11-deoxycorti-costerone. 11-deoxycortisone has mineralocorticoid activity, when
present at high levels leads to vol-ume-expansion with resultant suppression of renin and
aldosterone. Hypertension is a sign of overtreat-ment with Florinef in 21-hydroxylase CAH,
the dose of which typically needs to be weaned after infancy. The clinical presentation of all
forms of CAH is vari-able, therefore normal virilization does not rule out 17-hydroxylase
CAH in an XY male. Some patients with severe 11-beta-hydroxylase deficiency may have
17-hydroxyprogesterone levels high enough to trig-ger an abnormal newborn screen result,
although the majority will be diagnosed later. An elevated 11-deoxycorticosone level on
ACTH stimulation is the gold-standard diagnostic technique.
9. Which of the following assessments is NOT standardly used to assess adequacy of
medical treatment in apatient with classic 21-hydroxylase CAH?
a. Linear growth
b. Radiograph of left hand for bone age
c. Androstenedione
d. ACTH
e. Plasma renin activity
9. d. ACTH is not a reliable marker for the adequacy of hydrocortisone dose. Linear growth
should be moni-tored for signs of undertreatment (increased growth velocity) and
overtreatment (suppressed growth velocity). Bone age is checked every 1–2 years for
evidence of androgen-induced skeletal maturation. Androstenedione should be followed and
maintained in the normal range for age. Plasma renin activity is used to assess adequacy of
mineralocorticoid replace-ment.

10. You are asked to provide prenatal counseling to amother who previously gave birth
to a virilized female with 21-hydroxylase CAH. Which of the following statements
should you provide to the mother?
a. The incidence of CAH is the same in males and females.
b. She has a one in four chance of having another virilized female with CAH.
c. There is a decreased incidence of virilization in female infants with CAH born to mothers
who take maintenance doses of hydrocortisone during pregnancy.
d. Fetal sexual differentiation does not occur until the third trimester so she would only have
to take dexamethasone for the last few weeks of her pregnancy to reduce the risk of
virilization for an affected female.
10. a. 21-hydroxylase is an autosomal recessive disor-der that affects males and females
equally. Statisti-cally, assuming that both mother and father are carriers, there is a one in
four chance of giving birth to an affected child with each pregnancy, so the risk of having
another affected female is one in eight. Prenatal treatment with dexamethasone to reduce
virilization of affected females is controversial; exist-ing protocols suggest starting treatment
at 6–8 weeks gestational age, around the time that external sexual differentiation occurs.
Hydrocortisone is not effective because it is metabolized by the placenta.

Normal and Abnormal Sexual Differentiation


1. A term infant is noted to have bilateral (BL) undescended testes palpable in the
inguinal canal. Penis is morphologically normal with stretched penile length of 3.2 cm.
Which of the following is true about the management of this condition?
a. Immediate surgical consultation is necessary because this infant is at increased risk of
germ cell malignancy
b. Immediate endocrinologist evaluation is necessary because this infant likely has a disorder
of sexual development
c. Spontaneous descent is likely; surgery should be delayed until after mini-puberty of
infancy
d. Spontaneous descent is likely; surgery should be delayed until after onset of puberty
1. c. The majority of palpable undescended testes will descend spontaneously after the
testosterone surge that accompanies the mini-puberty of infancy which occurs within the
first few months of age. Spontaneous descent is unlikely after 4 months of age. There is an
increased risk of germ cell malignancy in undescended testes, but malignancy is uncommon
in patients who had orchiopexy prior to 2 years of age. Endocrine eval-uation for DSD
should be performed in patients with BL nonpalpable testes or those with undescended testes
and hypospadias or micropenis.
2. A term infant born with cleft palate and large atrial septal defect has a
morphologically normal penis with stretched penile length of 1.5 cm. Which of the
following hormone defects is the most likely cause of his micropenis?
a. Hypothyroidism due to impaired synthesis of thyroid-stimulating hormone
b. Growth hormone deficiency

c. Testosterone deficiency due to 17β hydroxysteroid deficiency

d. Dihydrotestosterone deficiency due to 5α-reductase deficiency


2. b. There is a high suspicion that micropenis in a patient with multiple congenital midline
anomalies is due to deficient pituitary secretion of LH (leading to testoster-one deficiency)
or growth hormone. Central hypothy-roidism is also possible but does not cause micropenis.
Most infants with 17-beta hydroxysteroid deficiency or 5α-reductase deficiency will have a
more severe phe-notype with ambiguous external genitalia or external genitalia that appear
female. Neither condition is com-monly associated with other midline defects
.3. Which of the following conditions is typically associated with significant clinical
virilization at the time of puberty?

a. XY with 5α-reductase deficiency

b. XY with complete androgen insensitivity


c. XX with 21-hydroxylase CAH

d. XY with 17β-HSD CAH

3. a. XY males with 5alpha-reductase may have severe undervirilization at birth owing to


lack of 5-DHT pro-duction, the androgen responsible for the majority of external genitalia in
the male virilization fetus. At puberty, testosterone levels increase dramatically and because
the androgen receptor is present will cause virilization. There is no function androgen
receptor in complete androgen insensitivity, so increased testos-terone has no effect.
Individuals with partial AI will likely undergo some virilization at puberty. Virilization in an
XX female is due to excess androgen production in setting of cortisol deficiency and will not
be apprecia-bly affected by female puberty. XY males with 17β-HSD CAH lack an enzyme
necessary to produce testosterone so will not have significant virilization at puberty.
4. Who should make the final decision regarding gender assignment for an infant with
ambiguous genitalia?
a. The surgeon, who can best discuss functional and cosmetic outcomes of genitoplasty
b. The endocrinologist, who can best discuss past and future hormone exposure and will be
guiding any sex-hormone replacement at the time of puberty
c. The neonatologist, after reviewing the consults
from the surgeon, endocrinologist, geneticist, and psychologist
d. The family, after hearing recommendations of the full multidisciplinary team
4. d. Gender assignment remains complex and there usu-ally is not a “right” or “wrong”
decision. It is essential that families are involved in the discussion and make the ultimate
decision. Some people with DSDs have argued that gender assignment and surgical interven-
tion should be put off until the patient is old enough to decide and provide consent. This is
currently not stan-dard practice owing to the perceived difficulty in rais-ing a child with an
indeterminate gender and because in some cases surgical outcomes may be better when
performed earlier. Although given the changing cul-tural perceptions on gender it is likely
that counseling will evolve to allow for delaying gender assignment as a reasonable
possibility.
5. Which of the following statements regarding hormones and their actions in sexual
differentiation is not correct?
a. The production of estrogen by granulosa cells in an
XX female causes regression of the primordial male
internal genitalia structures.
b. The production of anti-müllerian hormone
by Sertoli cells in an XY male causes regression
of the primordial female internal genitalia
structures.
c. The production of testosterone by Leydig cells in an
XY male results in differentiation of internal structures
into the epididymis, vas deferens, and seminal
vesicle.
d. The conversion of testosterone into dihydrotestosterone
results in the virilization of external genitalia
in an XY male.
5. a. Estrogen plays no known role in fetal sexual differ-entiation, the Wolffian ducts
(primordial male inter-nal structures) regress spontaneously in the absence of testosterone.
Normal male virilization requires the coordinated production of several different hormones
as described in answers b–d.
6. Which of the following clinical presentations of ambiguous
genitalia is associated with high risk of adrenal
insufficiency?
a. Testes palpable in a normally formed scrotum
BL, microphallus with stretched penile length of
1 cm
b. Testes palpable in a bifid scrotum BL, urethral
opening at the ventral penile-scrotum junction
c. Partially fused scrotum, phallic structure 1.5 cm in
length, no palpable gonads
d. Testis palpable on one side of scrotum, stretched
penile length of 2.7 cm
6. c. Adrenal insufficiency should be suspected and urgently evaluated in any case of
ambiguous geni-talia with nonpalpable gonads. This presentation is commonly seen in
patients with mutations in adre-nal hormone biosynthesis resulting in congenital adrenal
hyperplasia. In XX females with CAH caus-ing adrenal insufficiency and ambiguous
genitalia, there will be no testes; virilization is due to excessive androgen production. In XY
males with CAH caus-ing adrenal insufficiency and ambiguous genitalia, undervirilization
will be due to absent/impaired testosterone production and testes will have not descended.
Micropenis (a) and hypospadias (b) in the setting of descended testes are most likely due to
structural developmental defects, though the former can be the result of growth hormone
deficiency and the infant should be monitored for hypoglycemia. Unilateral cryptorchidism
(d) with otherwise normal genitalia is relatively common and unlikely to be associated with
CAH.
7. A term infant is noted to have ambiguous genitalia.Karyotype was found to be XY,
labs drawn on DOL 2 showed undetectable testosterone, normal AMH, normal
androgen precursors including androstenedione and dehydroepiandrosterone (DHEA).
HCG stimulation test showed no increase in testosterone levels.
What is the most likely diagnosis?
a. 17β-HSD deficiency

b. Androgen insensitivity syndrome


c. Leydig cell hypoplasia

d. 5α-reductase deficiency

7. c. Laboratory workup confirms isolated testosterone deficiency (normal AMH shows that
testes/Sertoli cells are present), the absence of elevated androgen hor-mone precursors rules
out CAH. 17β-HSD deficiency (a) would show elevated androstenedione and DHEA, AIS
(b) and 5AR (d) would show normal/elevated tes-tosterone levels. Patients with Leydig cell
hypoplasia can be treated with a short course of testosterone in infancy for penile
enlargement and will develop nor-mal secondary sexual characteristics with testoster-one
hormone replacement starting at time of puberty
8. A term infant is found to have ambiguous genitalia.Rapid karyotype is XX, repeated
androgen precursor hormones including 17-hydroxyprogesterone are normal.All of the
following are appropriate next steps in
the evaluation, except
a. Questioning of mother to determine if any performance
enhancing drugs used during early phases
of pregnancy
b. Questioning/examination of mother for new-onset
hirsutism
c. Genetic testing for presence of SRY
d. Whole exome sequencing
8. d. Presence of virilization in an XX female with absence of laboratory findings of CAH
suggests pos-sibility of exposure to maternal endogenous/exog-enous androgens or presence
of an ovotesticular DSD. Maternal use of androgens as athletically perfor-mance enhancing
drugs early in pregnancy (in some case before pregnancy was recognized) can cause vir-
ilization. Development of maternal hirsutism during pregnancy suggests possibility of
placental aromatase deficiency resulting in accumulation of excess andro-gens in both
maternal and fetal circulation. Ovotestic-ular DSD is a condition associated with both
ovarian and testicular tissue; degree of virilization depends upon amount of testicular tissue
and testosterone production. In many cases these patients are mosaics for chromosomes
(possessing both XX and XY) which can be missed on initial karyotype but will be detected
with DNA testing for SRY. In the absence of SRY, pur-suit of other genetic abnormalities
(such as RSPO-1) with tests such as WES can be considered, but that would not be the next
step in the case here owing to expense/delay in results.

9. Which of the following statements is true regarding gonadal dysgenesis?


a. A karyotype showing XY sex chromosomes rules out gonadal dysgenesis
b. Patients with gonadal dysgenesis are at increased risk of gonadoblastoma
c. All patients with XY gonadal dysgenesis should be assigned male gender at birth
d. Wolffian duct differentiation is unaffected in patients with XY gonadal dysgenesis
9. b. Patients with dysgenetic gonads, including XY gonadal dysgenesis, are at increased risk
of gonadoblas-toma and germ cell malignancy; therefore dysgenetic gonads should be
identified and removed. Presence of XY karyotype dose not rule out gonadal dysgenesis;
mutations of SRY and other genes involved in early sex-ual differentiation can be
responsible for the clinical phe-notype. There is wide clinical phenotype in XY gonadal
dysgenesis ranging from completely feminized internal and external genitalia (complete
gonadal dysgenesis) to a range of undervirilization of internal (i.e., Wolffian ducts) and/or
external genitalia depending upon extent of androgen production (partial gonadal
dysgenesis)
10. An infant with ambiguous genitalia has been found to have an XY karyotype. At
which of the following timepoints will a serum testosterone level be informative?
a. 1 day of life
b. 1 week of life
c. 2 months of age
d. 6 months of age
e. 16 years of age
f. a, b, c
g. a, c, e
h. All of the above
10. g. Testosterone production is present at birth and then rapidly declines in the first several
days until rising again at 20–30 days of life (“mini-puberty of infancy”). Testosterone
production in infancy peaks between 2–3 months of age, before falling to non-detectable
levels by 6 months of age. Testosterone production again rises at the onset of puberty (range
9–14 years of age). Assessment of unstimulated tes-tosterone outside of these times will not
provide any useful clinical information; however, an hCG stimula-tion test can be performed
to assess for the capability of testosterone production.
Abnormal Renal Development
1. You are asked to consult on a woman at 21 weeks of her pregnancy for an abnormal
prenatal U/S at 20weeks. The ultrasound showed kidneys measuring 3.4
cm each with increased echogenicity. The bladder was not able to be seen. There is no
family history of kidney disease. What is the likely cause of these findings?
a. ADPKD
b. MCDK
c. Renal dysplasia
d. ARPKD
e. Beckwith-Wiedemann
1. d. The normal size of fetal kidneys at 20 weeks is 2.7 cm. The bladder can be seen by 15
weeks. The combination of large, bright kidneys with presumably no urine (can’t visualize
the bladder because it is empty) is very sugges-tive of ARPKD. ADPKD can present in
neonates but a severe presentation such as this with anuria is very rare. More often there is
just large kidneys or cysts seen with normal renal function. The lack of family history argues
more for ARPKD than ADPKD, although new mutations comprise 10% of people affected
by ADPKD. Bilateral MCDK would be associated with anuria but large cysts should be seen
in the kidneys. Dysplastic kidneys are usually (but not always) small. The kidneys in
Beckwith-Wiedemann can be large but severe impairment of func-tion in utero would be
atypical
2. You are asked to do another prenatal consult on awoman who was noted at her 20-
week U/S to be carrying a fetus with abnormal kidneys. The left kidney has
multiple cysts of similar size and is of normal size. The right kidney is of normal
echogenicity and shape with good corticomedullary differentiation and no cysts. It
measures 3 cm. No hydronephrosis or hydroureter is seen. The amniotic fluid volume is
normal. Family history is negative for renal disease. What is the MOST
likely diagnosis?
a. MCDK
b. ADPKD
c. Left-sided UPJ
d. Left UVJ obstruction
e. ARPKD
2. a. MCDK has a characteristic ultrasound picture of multiple cysts of similar size giving an
appearance of a “cluster of grapes.” They have no function. Often the contralateral kidney
will show compensatory hypertro-phy in utero as this has. With the normal kidney on the
right, you would expect normal amniotic fluid levels. ADPKD is possible but rarely presents
in neonates with ultrasound findings. ARPKD rarely presents with visible cysts and affects
both kidneys. A UPJ obstruction leads to hydronephrosis while a UVJ obstruction would
lead to hydroureter.
3. You provide a prenatal consult to a couple with a baby suspected to have a posterior
urethral valve who are considering prenatal intervention. Which of these
statements regarding prenatal intervention for suspected bladder obstruction is false?
a. The most common intervention performed is
a vesicoamniotic shunt.
b. Urine Na < 100 mEq/L on serial taps suggests a good
renal prognosis.
c. Intervention has been shown to improve both renal
and lung function long term.
d. Serial bladder taps are preferred when assessing
fetal renal function.
e. Urine Osm < 200 mOsm/L predicts good renal function.
3. c. Intervention of suspected fetal bladder outlet obstruction has not definitely been shown
to improve long-term function of lungs or kidneys. When an inter-vention is done, the
vesicoamniotic shunt is the most common procedure. Serial bladder taps are preferred to
obtain fresh urine to analyze the urine electrolytes. Urine Na < 100 and Urine Osm > 200 are
predictive of preserved renal function. Other tests predictive of good renal function are:
urine Ca < 8 mg/dL, urine protein < 40 mg/dL, urine Beta-2 microglobulin < 4 mg/L.

4. A 6-day-old ex-24 week infant is noted by his bedside nurse to have decreased pulses
in his lower extremities.His blood pressure is also noted to be trending up.
His urine has a pink tinge. Urine output is normal. His respiratory status is unchanged.
He has a UAC in place.U/S shows a small unilateral renal artery clot. What is the most
likely long-term outcome?
a. End stage renal disease as an infant
b. Necrosis of toes bilaterally
c. Persistent hematuria
d. Hypertension
e. Small bowel necrosis
4. d. The baby likely has a renal artery clot due to the UAC. Clots are thought to form due to
disruption of the endothelium during line placement. A low-lying UAC is more likely to
have a renal artery clot as compared to a high lying UAC. The findings of decreased LE per-
fusion, hypertension, and hematuria without oliguria, congestive heart failure, or multi-organ
failure suggest a “minor” clot. The treatment would be removal of the catheter and
symptomatic therapy. Hypertension is common following a renal artery clot. The hyperten-
sion should resolve but this may take months or years. ESRD is seen with bilateral renal
artery clots. Loss of toes or bowel ischemia would be expected with a more severe clot
involving the aorta
.5. A 4-week-old ex-30 week infant undergoes a renal ultrasound for work-up of UTI.
The U/S shows a small left kidney with some echogenic spots. His past medical
history includes treatment for culture-negative sepsis with ampicillin and gentamicin
for which he had aUVC (umbilical venous catheter) in place. He was noted
to have a transient drop in platelets shortly before his antibiotics finished. What is the
most likely etiology of the U/S findings?
a. Kidney stones
b. Dysplasia
c. ARPKD
d. Old renal vein thrombosis
e. Nephrocalcinosis
5. d. The baby has several risk factors for renal vein thrombosis: sepsis, prematurity,
presence of UVC. The classic clinical findings suggestive of RVT are: flank mass, low
platelets, and hypertension. The presence of all three findings is actually rare. While preterm
kid-neys are more likely to have stones or nephrocalcino-sis, the affected kidney would not
be small. Dysplastic kidneys are often small but should not have calcifica-tions. Kidneys
affected by ARPKD are large and dif-fusely echogenic
.6. A full-term baby with congestive heart failure is noted to have a rise in creatinine
from a baseline of 0.7 to 1.6 by day of life 3. Urine output remains more than 1 mL/
kg/hour. What stage of AKI is the baby in using the KDIGO system?
a. Stage 0
b. Stage 1
c. Stage 2
d. Stage 3
6. c. A rise in serum creatinine between 2 and 2.9 times the lowest creatinine obtained in the
prior 7 days or UOP > 0.3 and ≤ 0.5 mL/kg/hour defines stage 2 AKI by KDIGO definition.
7. A 5-day-old full term infant is admitted to the NICU after seeing his pediatrician for
a weight check. He was noted in the office to have a 15% decrease in weight since
birth.On exam he has dry mucus membranes, a sunken fontanelle,and capillary refill
of 3 seconds. His creatinine is 1.6. He receives fluid resuscitation with significant
clinical improvement. Over the next 2 days his creatinine falls to 1.2. His vitals are
normal for age. He appears euvolemic on exam. His renal U/S is normal. His urine
output is 1.1 mL/kg/hour. What should be your next
step in management?
a. NS (normal saline) bolus
b. Start renal dose dopamine
c. Kidney biopsy
d. Insert a bladder catheter
e. Observation
7. e. You have assessed him as euvolemic so further fluid boluses are unlikely to help. If his
hydration was in question though, a NS bolus could be trialed. Renal dose dopamine has not
been shown to hasten renal recovery. Since there is no hydronephrosis or abnormal bladder
on U/S, placement of a blad-der catheter would not improve his renal function. Kidney
biopsies are rarely required in neonates to make a diagnosis of AKI. The baby likely has
ATN from prolonged renal hypoperfusion. There is no specific treatment for this other than
ensuring good hydration, avoiding further renal insults, and man-aging any electrolytes
issues or hypertension. ATN should resolve over days/weeks depending on its severity.
8. A cGA 30-week infant weighing 1340 gm develops stage 3 AKI using the neonatal
KDIGO definition. His daily fluid intake has been severely restricted due to oliguria
and thus he is receiving next to no nutrition. His potassium is 6.2, bicarbonate 15, BUN
42, phosphorus 8. Which statement is incorrect regarding dialysis
options for this baby?
a. PD can be performed by his NICU nurses.
b. PD does not require any anticoagulation.
c. PD removes immunoglobulins.
d. HD involves rapid volume shifts that may not be tolerated.
e. CRRT fluid goals are set on the machine once every
24 hours and cannot be adjusted in between.
8. e. PD has several advantages in the NICU; it is com-paratively technically simple
compared to HD or CRRT. It is performed with manual exchanges of fluid done by NICU
nurses. The baby is not antico-agulated with PD. It may be easier to get a PD cath-eter
inserted than obtain reliable vascular access for HD or CRRT. PD’s disadvantages include
the removal of beneficial proteins like immunoglobulins, its slow efficiency at solute and
fluid removal, and risk of peritonitis. It is contraindicated in babies with abdominal wall
defects or VP shunts. HD can quickly remove solutes but rapid fluid removal is usually not
tolerated well. One of CRRT’s principal advantages is slow, hourly removal of fluid that can
be adjusted each hour.
9. A 28-week-old infant is receiving daily Lasix for fluid overload. His response to the
Lasix dose seems to be decreasing despite increasing from 1 to 2 mg/kg/day.
What strategy can be used next to optimize diuresis?
a. Increase the frequency to q8 dosing
b. Add spironolactone
c. Increase the dose to 3 mg/kg
d. Add a thiazide diuretic
e. Space dosing to q48 hours
9. d. Preterm infants have slower metabolism of furose-mide so should not be dosed multiple
times per day. Spironolactone is a weak diuretic used mostly as a potassium sparing agent in
the NICU. The baby is likely not responding as well to the Lasix because the tubule
segments past the loop of Henle are avidly reabsorbing the sodium due to a state of volume
depletion. The dis-tal nephron’s Na-Cl cotransporter can be blocked by a thiazide diuretic,
which should increase diuresis. Care must be taken though to not induce AKI from intravas-
cular depletion. Close attention to electrolytes is also necessary.
10. The parents of a 10-week-old ex-24 week infant are nervous about his medications
causing long-term damage. They ask what the side effects of his diuretics
may be. Which of these is a true statement?
a. Long-term use of thiazides is linked to bone fractures.
b. Lasix has not been associated with hearing loss.
c. Loop diuretics can lead to nephrocalcinosis.
d. Thiazides are associated with increased numbers of
kidney stones.
e. Spironolactone causes hypokalemia
10. c. Thiazides can cause increased reabsorption of cal-cium from the urine. In contrast,
loop diuretics are associated with hypercalciuria, kidney stones, and nephrocalcinosis. They
are also associated with an increased risk of bone fractures and with hearing loss.
Spironolactone is a potassium-sparing diuretic used most often in the NICU in conjunction
with a thiazide to mitigate potassium loss in the urine.
Water-Electrolyte Metabolism and Acid-Base Balance
1. A 24-week, 600 gm infant is admitted to the NICU. The baby is intubated and
sedated. Due to being quite unstable and requiring frequent hands-on care, she
is under a radiant warmer. You estimate her baseline insensible fluid loss as 100
mL/kg/d. Based on the above information, what will you write her total fluids
for the next 24 hours?
a. 75 mL/kg/d
b. 110 mL/kg/d
c. 140 mL/kg/d
d. 175 mL/kg/d
e. 200 mL/kg/d
1. c. The baby is intubated and receiving 100% humidi-fied air which will reduce/ eliminate its insensible
losses (15%–30%) from respiration. Sedation also decreases the insensible fluid loss by an additional 5%–
25%. Being under a radiant warmer; however, will increase insensible loss by 50%–100%. Combining these
factors will lead to a net increase of 15%–45% above the base-line value (-15 plus -5 plus 50 = 25 to -30
plus -25 + 100 = 45). Therefore total fluids would be 115–145 mL/kg/day as a rough estimate. Because this
is only an estimate, electrolytes, particularly sodium, ins/outs, and weights should be followed closely.
2. A full-term infant is being cared for in your NICU following
a difficult delivery. He was initially prescribed
D5W at 70 mL/kg/d for dol 0–2. On dol 3 he is noted
to have a sodium of 130, a potassium of 4.8, and a creatinine
of 0.7. His weight has been stable since birth.
He appears euvolemic on exam. Urine lytes show a
sodium of 40 mEq/L. What is the most likely etiology of
the hyponatremia?
a. AKI
b. Bartter syndrome
c. Inadequate intake of sodium
d. SIADH
e. Pseudohypoaldosteronism
2. d. SIADH can be triggered by birth asphyxia. To make this diagnosis, renal, thyroid, and
adrenal function tests must be normal. Confirmation of the diagnosis is made by the finding
of urine osmolality higher than serum osmolality and urine sodium should be greater than
20. A finding of urine sodium less than 20 would be suspicious for intravascular volume
depletion lead-ing to an increase in urine sodium reabsorption. In such cases, ADH release is
appropriate to preserve volume even at the expense of plasma osmolality. A creatinine of 0.7
in a 3-day-old infant would be in the normal range, making AKI unlikely to be the cause of
the hyponatremia. Bartter syndrome is a tubular disor-der leading to salt wasting, polyuria,
and dehydration. Infants typically do not need sodium supplementation until day of life 3.
Pseudohypoaldosteronism presents with low sodium and high potassium levels.
3. An ex-25-week-old infant is now 11 days old. Her
serum sodium has trended down from 137 to 126. Her
creatinine is 0.8. Her TF are written as 130 mL/kg/d.
Her TPN has 3 mEq/kg/d of sodium with an additional
1 mEq/kg/d provided in her carrier fluids. You obtain
urine lytes that show a sodium of 80 and a creatinine
of 10. What is her FENa and how does it explain her
serum sodium?
a. 12.6%, tubular wasting from immature tubule
b. 7.9%, tubular wasting from acute tubular necrosis
c. 5%, tubular wasting from prematurity
d. 2.5%, prerenal AKI
e. 0.05%, inadequate Na intake
3. c. The baby’s FENa is calculated as [(UNa x SCr)/(UCr x SNa)] x 100 = (80 × 0.8)/(10 ×
126) = 0.05 × 100 = 5%. While term babies conserve sodium well with FENa < 1 and often
closer to 0.5%, preterm infants’ imma-ture tubules lead to sodium wasting and high FENa. A
creatinine of 0.8 in an 11-day old ex-25 week infant would fall in the normal range so AKI is
unlikely
.4. The lab calls with a critical potassium value of 7.2 in a4-day-old ex-26-week infant.
His weight is 15% below birth weight. The rest of the chemistry panel shows a
Na of 151 and a bicarbonate of 10. Which of the following
mechanisms would not perpetuate hyperkalemia?
a. Avid sodium reabsorption by the proximal tubule
secondary to volume depletion
b. Lack of Maxi-K channels in neonates
c. Inhibition of ROMK channels by acidosis
d. Neonates’ blunted response to aldosterone
e. Acidosis stimulating Na-K ATPase, which increases
the intracellular to urine gradient.
4. e. Alkalosis stimulates the Na-K ATPase on the basolat-eral membrane. This will increase
intracellular potas-sium levels and stimulate potassium exit to the urine space. Alkalosis also
increases the amount of time the luminal potassium channels are open further encour-aging
urine potassium secretion and leading to hypo-kalemia. Avid sodium reabsorption in the
proximal tubules as seen in volume depleted states leads to less sodium delivery to the distal
tubule. The reabsorption of sodium by ENaC in this segment leaves the lumen
electronegative which allows for secretion of potas-sium. Neonates do initially lack the
potassium Maxi-K channels (although these are stimulated by high urine flow which would
not be present in the baby in the sce-nario). The potassium channel ROMK is inhibited by
acidosis which would lead to less potassium secretion to the urine. Although aldosterone
levels are high in infants, they exhibit less of a response to it.

5. A 33-week cGA infant has been on thiazides for about


6 weeks for CLD. She develops acute vomiting. A chemistry
panel shows a bicarbonate of 32 with potassium
of 3.0 and creatinine of 1.1. A blood gas confirms metabolic
alkalosis. What mechanism is not involved in
stimulating or maintaining the alkalosis?
a. Vomiting leads to loss of protons.
b. Vomiting leads to decrease in intravascular volume,
which results in a decrease in GFR.
c. Hypokalemia limits the shift of H+ out of cells.
d. Intravascular depletion stimulates proximal Na
reabsorption, which leads to increased distal H+
secretion.
e. Intravascular depletion stimulates RAS driving further
potassium loss.
5. d. In a state of volume depletion, alkalosis will persist by the following three mechanisms:
(1) a decrease in GFR leads to less filtered bicarbonate; (2) the proximal tubule avidly
reabsorbs Na which leads to increased bicarbonate reabsorption; and (3) the volume
depletion stimulates RAS which leads to an increase in aldoste-rone. The aldosterone leads
to increased activity of the epithelial Na channel (ENaC), making the lumen more
electronegative. This favors both H+ and K+ secretion. Typically in alkalosis, H+ would
shift out of the cell in exchange for K+. Hypokalemia inhibits this process.
6. A 1-week-old ex-37-week infant is admitted with the
following chemistry: Na 127, K 7.3, bicarbonate 15.
His mother reports he is not very interested in feeding
but has been making a normal amount of wet diapers.
His weight is 12% below his birth weight. His father
reports that he has been told he was often admitted to
the hospital as a baby but he doesn’t know any details.
He is currently well and not on any medications. What
advice could be given to the family about what to
expect as their child grows up?
a. Resolution of symptoms by age 2
b. A chronic cough
c. Persistent rashes
d. Growth failure
e. A need for salt supplementation throughout life
6. a. The baby has pseudohypoaldosteronism type 1, autosomal dominant type. PHA type 1
presents with vomiting, FTT, hyponatremia, hyperkalemia, and aci-dosis with high urine
sodium. Renin and aldosterone levels are high. It can be inherited as an autosomal recessive
or dominant trait. The dominant form is clini-cally outgrown, usually by age 2. It is limited
to renal findings. In contrast, the recessive form is a life-long, systemic illness involving the
skin, lungs, and kidney predominantly.
7. A 950-gram infant is noted to have a 6% loss of weight
from birth weight. His UOP has increased from dol 1 to
2. His electrolytes and creatinine are normal for age.
So far, he has been on 90 mL/kg/d TF without any Na
or K in the TPN. Which statement below regarding his
fluid/electrolytes management is correct?
a. Increase TF to 130 mL/kg/d since he is losing too
much weight.
b. He is at a decreased risk of a PDA based on the numbers
above.
c. His risk of NEC is higher than an infant of similar
BW who did not lose weight.
d. Adding 1 mEq/kg/d potassium to the TPN would be
dangerous.
e. Your management has increased his risk of chronic
lung disease.

7. b. A weight loss of up to 15% is normal in premature infants. Studies have shown that not
allowing this weight loss to occur is associated with increased rates of PDA, NEC, and CLD.
Potassium should be withheld from the TPN until urine output is established. The baby in
the above scenario is urinating normally and has a normal creatinine, so standard of care
would be to add potassium to his TPN. His TF should be increased to allow more nutrition,
not because of the expected and normal weight loss
.8. A FT infant with bilateral pneumothoraces develops respiratory acidosis. Regarding
neonatal respiratory acidosis, which statement is true?
a. Preterm infants can respond to respiratory acidosis
by increasing their tidal volume.
b. Providing a dose of bicarbonate using the following

formula is helpful: base deficit × BW × 0.3.

c. Renal compensation is as efficient as in older children.


d. Intubation may be needed.
e. Administered bicarbonate stimulates hyperventilation.
8. e. Term infants increase both respiratory rate and tidal volume, but preterm infants
increase only their respi-ratory rate. Bicarbonate is not helpful to treat respi-ratory acidosis
because it decreases respiratory rate. Preterm infants have a decreased ability to reabsorb
filtered bicarbonate
.9. A 1-month-old ex-33-week infant has been having
difficulty gaining weight. Her feeds have been fortified
to 28 kcal/oz but she still gains <5 g/d. Her mother
reports her son had a similar issue with weight gain.
You obtain a chemistry panel that shows: Na 137, K
4.1, bicarbonate 12, BUN 14, creatinine 0.3, glucose
85, Ca 9.4, phosphorus 6.0. A urine sample has a specific
gravity of 1.010, pH 5.4, and is negative for blood,
protein, LE, or nitrates. What is the baby’s diagnosis?
a. Distal RTA
b. Type IV RTA
c. Hemolyzed blood sample
d. Proximal RTA
e. Pseudohypoaldosteronism
9. d. Proximal RTA is most commonly seen in children as part of the renal Fanconi
syndrome with wasting of glucose, amino acids, and phosphorus. However, rare genetic
causes have been reported. With proximal RTA the defect lies in the reabsorption of the
filtered bicar-bonate. The distal acidification mechanism is intact. When the serum
bicarbonate is low, the distal acidifi-cation results in a low urine pH. When alkali therapy is
started, the filtered bicarbonate load increases which raises the urine pH. Therapy of
proximal RTA requires large amounts of alkali, often 10–15 mEq/kg/day. Dis-tal RTA is
due to an acidification defect. It is suggested by a normal AG acidosis with urine pH > 6.5.
Hyper-calciuria is often present. Type IV RTA presents with mild acidosis (bicarbonate
levels in high teens) and hyperkalemia. Technical difficulties obtaining blood samples can
lead to factitiously low bicarbonate levels but hyperkalemia and hyperphosphatemia should
also be seen in that case. Pseudohypoaldosteronism can have a mild metabolic acidosis but
hyperkalemia and hyponatremia are major features of the disease.
10. You are treating an ex-27-week infant for a moderate
sized PDA. The baby has developed worsening
pulmonary edema despite restricting fluids. You start
indomethacin. After the second dose, the baby’s urine
output is noted to drop from 1 to 0.6 mL/kg/hr. Which
statement explains the drop in UOP?
a. Release of ANP has caused dilation of the efferent
arteriole.
b. Indomethacin has disrupted the intrarenal production
of prostaglandins.
c. Ang II is constricting the afferent arteriole only.
d. The indomethacin has closed the PDA resulting in
shunting of blood away from the kidneys.
e. Dopamine is stimulating NA-K ATPase.
10. b. Renal prostaglandins play a crucial role in main-taining GFR when the kidney is not
receiving normal blood flow due to volume depletion, hemorrhage, etc., or when angiotensin
II levels are high. NSAIDs disrupt the production of prostaglandins and can lead to AKI.
ANP is released by stretch of the atrium. It acts on the kidney to increase GFR by dilating
the afferent arteriole and constricting the efferent arteriole. Angiotensin II constricts both the
afferent and efferent arterioles. If the PDA closed, blood would stop shunting away from the
kidneys. Dopamine is natriuretic by decreasing the activity of Na-K ATPase which provides
the driving force for Na reabsorption.
Parenteral Nutrition
1. Which of the following statements regarding TPN and enteral neonatal nutrition is
FALSE?
a. Normal fetal metabolic and growth rates and nutritional requirements stop with birth,
precluding routine use of IV nutrition.
b. The smaller and more preterm and less developed the infant, the less body stores (protein,
fat, glycogen) are available to provide nutrients for metabolic needs.
c. The metabolic and thus nutrient requirements of the newborn are equal to or greater than
those of the fetus of the same gestational age.
d. First-week protein and energy intakes are associated with improved 18-month
developmental outcomes in preterm infants.
1. a. Normal fetal metabolic and growth rates and nutritional requirements stop with birth,
precluding routine use of IV nutrition. This is False. There is no reduction in metabo-lism or
potential for growth with birth, and thus the infant will require the same amount of nutrients.
If the infant is very small, there will be insufficient nutrient stores to maintain normal
metabolism or growth, requiring early IV nutrition and early and continued enteral nutrition.
2. Which of the following statements regarding TPN is FALSE?
a. The amino acid composition of current neonatal parenteral amino acid solutions is based
on providing plasma concentrations similar to those of term, fully
breastfed infants, which is appropriate for preterm infants who are growing at much faster
rates than term infants.
b. A standard parenteral infusion of 3 g/kg/day amino acids, 10% dextrose, and 3
grams/kg/day lipid at 100mL/kg/day would provide 82 kcal/kg/day, according

to the following calculations: 3 grams amino acids × 4 kcal/gram = 12 kcal; 10 grams of

dextrose × 4kcal/gram = 40 kcal; 3 grams/kg/day of lipid would

add 30 kcal/kg/day.
c. Cholestasis or parenteral nutrition associated
liver disease (PNALD) is more common in infants
(extremely preterm infants, and those with short gut
syndromes such as gastroschisis or those with severe
enteral feeding intolerance) who receive parenteral
nutrition exclusively and for prolonged periods
(weeks, vs. days).
d. The principal metabolic complication of parenteral
nutrition is hyperglycemia.

2. a. The amino acid composition of current neonatal paren-teral amino acid solutions are
based on providing plasma concentrations similar to those of term, fully breast fed infants,
which is appropriate for preterm infants who are growing at much faster rates than term
infants. This is False. Term infants do not have as high a fractional pro-tein synthetic rate or
growth rate as the earlier gestation fetus does and thus the preterm infant of the same gesta-
tional age should. Fetal amino acid concentrations are generally higher than those of term
infants fed mature mother’s milk, an indication of their need for more amino acid and
protein intake to meet their greater requirements.
Enteral Nutrition
1. Which of the following is a TRUE statement regarding human milk being the ideal
enteral diet for human neonates?
a. It provides sufficient energy, protein, fat, carbohydrate,micronutrients, and water for
normal metabolism,growth, and development in term infants.
b. It does not require supplements of protein and certain minerals.
c. It is enriched in nutrients when it comes from a milk bank as mature donor milk.
d. It is associated with delayed neurodevelopmental outcomes compared with term formulas.
1. a. True, for term infants. Human milk provides sufficient energy, protein, fat,
carbohydrate, micronutrients, and water for normal metabolism, growth, and development in
term infants. Human milk has been the result of sev-eral million years of evolutionary
development to pro-duce the optimal nutrition of human newborn infants.
2. Which of the following about mother’s own milk produced at home or in the NICU
and stored is TRUE?
a. Varying degrees of nutrient loss may occur with long-term storage, particularly in frozen
conditions.
b. Vitamin C loss does not occur, even during the process
of feeding freshly expressed human milk by bottle.
c. For multiple human milk components, significant
degradation generally occurs with even very shortterm
storage and multiple freeze-thaw cycles.
d. Donated human breast milk does not contain pathogenic
microorganisms that could be transferred to
the infants who consume it.
2. a. Varying degrees of nutrient loss many occur with long term storage, particularly in
frozen conditions. This is true, especially when there are multiple freeze-thaw cycles, and is
one reason why fresh mother’s milk is superior.
3. Which of the following statements about heat treatment
(pasteurization) of human milk are FALSE?
a. Heat treatment (pasteurization) of human milk may reduce the concentration and
functional capacity of many bioactive components.
b. Holder pasteurization does not adversely affect the protein content of donor milk (average
value of about −4%).
c. Total lipid generally is increased (as much as by 60%), but free fatty acids are reduced,
which might increase their nutritive potential.
d. No significant reductions in lactose have been noted,both as free molecules and as part of
biologically active compounds such as oligosaccharides.

Minerals, Vitamins, and Trace Minerals


1. Which of the following is TRUE about calcium and phosphorus
supplements for preterm infants?
a. Calcium and phosphorus are readily absorbed from
the preterm infant GI tract and do not need to be supplemented.
b. Calcium and phosphorus are easily soluble in IV nutrition solutions.
c. Recommended intravenous intakes of calcium are
higher than enteral requirements, ranging from 150 to 220 mg/kg/day for calcium.
d. Risk factors in preterm infants for calcium and phosphorus
deficiency and subsequent rickets include:
gestational age <27 weeks or birth weight <1000
grams, long-term parenteral nutrition (>4–5 weeks),
severe BPD requiring diuretics and fluid restriction,
long-term steroid treatment, a history of necrotizing
enterocolitis (NEC), and intolerance to enteral formula
or human milk.
1. d. True. Risk factors in preterm infants for calcium and phosphorus deficiency and
subsequent rickets include: gestational age <27 weeks or birth weight <1000 grams, long-
term parenteral nutrition (<4-5 weeks), severe BPD requiring diuretics and fluid restriction,
long-term steroid treatment, a history of necrotizing enterocolitis (NEC), and intolerance to
enteral formula or human milk. All of these are true and self-explana-tory. Calcium and
phosphorus nutrition of very preterm infants is deficient for all of these reasons.
2. The following statement about potential manganese (Mn) toxicity is true:
a. Potential nephrotoxicity.
b. Mn supplementation should be stopped with any signs of hepatic dysfunction or
cholestasis.
c. The contaminant levels of Mn in parenteral nutrient solutions are too low to meet
requirements without additional supplementation.
d. Parenteral calcium gluconate to prevent osteopenia does not contribute to Mn
contamination and potential toxicity.
2. b. Mn supplementation should be stopped with any signs of hepatic dysfunction or
cholestasis. This is true, as excess Mn damages the liver.
Nutrition and Growth: The Fetus
1. During the third trimester of fetal growth, the largest fractional increase in body
composition is the result of:
a. water
b. fat
c. brain
d. lean body mass
1. d. True. Lean body mass, including structural compo-nents of cells that have been
proliferating, muscle, and bone account for the largest fractional increase in body weight,
indicating the large nutritional requirements for protein during this period of growth.
2. Essential amino acids are:
a. used for oxidation
b. used for protein synthesis
c. used for growth of the brain
d. required as part of nutrient supplies
e. all of the above
2. e.
3. Which of the following regarding neonatal nutrition is false?
a. Rates of protein synthesis and the requirements for amino acid and protein supply to meet
them are the same in preterm infants as in normally growing fetuses of the same gestational
age.
b. The critical amount of amino acid or protein nitrogen for preventing growth faltering in
very preterm infants <30–32 weeks is less than 1.5 g/kg/day.
c. There is no advantage to protein metabolism or growth (body weight or composition) with
more than 3.5–4.0 g/kg/day parenterally or 4.0–4.5 g/kg/day enterally in very preterm
infants.
d. Excess amino acid or protein intake always produces higher blood urea and ammonia
concentrations but these seldom are in toxic ranges.
3. b. False. The critical amount of amino acid or protein nitro-gen for preventing growth
faltering in very preterm infants <30-32 weeks is less than 1.5 g/kg/day. This is false, as
numerous studies have shown that growth cannot occur until at least 1.5 g/kg/day of amino
acids is provided.

Inborn Errors of Metabolism

1. A 3-day-old girl began feeding poorly on day two of life. She now has depressed
mental status, clonus, and respiratory alkalosis. Which test should be sent for
emergently?
a. Very long-chain fatty acids
b. 7-dehydrocholesterol
c. Ammonia
d. Lysosomal enzymes
e. Cholesterol
1. c. Infants with urea cycle defects usually become symp-tomatic in the first days to months
of life. Clinical pre-sentation includes: lethargy, hypothermia, poor feeding, tachynea,
irritability, vomiting, coma. Differential diag-nosis includes sepsis or an organic acidemia.
Hyperventi-lation due to cerebral edema causes respiratory alkalosis.
2. A newborn male has profound hypotonia and features suggestive of trisomy 21. He is
found to have hepatomegaly,hyperbilirubinemia, and synthetic liver dysfunction.
Which diagnosis should be considered?
a. Zellweger syndrome
b. Mucopolysaccharidosis
c. Smith-Lemli-Opitz syndrome
d. Pyruvate carboxylase deficiency
e. Krabbe disease
2. a. Zellweger spectrum disorder (ZSD), is a disorder of peroxisomal biogenesis caused by
a defect in the PEX gene. A newborn with ZSD presents with dysmorphic features which
includes; flattened face, epicanthal folds, upslanting palpebral fissures, broad nasal bridge.
and hypoplastic supraorbital ridges with severe hypotonia and hepatic dysfunction.
3. A breast-fed newborn girl has high anion-gap acidosis,hypoglycemia and altered
mental status. Urinalysis reveals 4+ ketones. The ketosis is suggestive of:
a. Normal response to hypoglycemia
b. Fatty acid oxidation defect
c. Peroxisomal defect
d. Organic acidemia
e. Lysosomal storage disease
3. d.
4. Poorly treated maternal phenylketonuria puts a fetus at risk for:
a. having PKU
b. cataracts
c. intracranial calcifications
d. limb defects
e. microcephaly
4. e. Uncontrolled maternal phenylketonuria leads to com-plications in the offspring. These
complications include microcephaly, intellectual disabilities, congenital heart disease, and
intrauterine growth restriction. Dietary management prior to conception and during
pregnancy is important to decrease complication risks in the offspring.
Evaluation of Infants With Congenital Anomalies
1. Several studies have shown that there is a small but definite
increase in congenital malformations in neonates
following artificial reproductive technology (ART) conception.
In which type of ART is an increased incidence
of urogenital abnormalities, specifically hypospadias,
likely to occur?
a. In vitro fertilization (IVF)
b. Gamete intrafallopian tube transfer (GIFT)
c. Intracytoplasmic sperm injection (ICSI)
d. Embryo cryopreservation
e. Blastocyst culture
1. c. Current evidence suggest an association between ART and small but definite (more
than 1.3 times than in spontaneous conception) incidence of congenital malformations. This
includes: congenital heart defects, neural tube defects, facial cleft, gastrointes-tinal
malformations, genitourinary malformations, and imprinting disorders. The rates of
congenital malformations are similar for each type of ART, except for increased urogenital
abnormalities with ICSI. The cause for the increase in malformations with ART are yet to be
determined.
2. Amniotic band sequence is caused by:
a. deformation
b. disruption
c. malformation
d. syndrome
e. association
2. b. Amniotic band sequence is an example of disrup-tion. Disruption defects are due to
destruction or interruption of a normal developmental process. It usually affects a body part,
rather than a specific organ

3. Deformations most commonly occur during:


a. the zygote period
b. the blastula period
c. the embryo period
d. the fetal period
e. the birth process
3. d. Deformations are defects caused by abnormal mechanical forces on morphologically
normal tissue in utero. They are associated with multiple gestations, uterine malformations,
and oligohydramnios
4. Arthrogryposis (congenital joint contractures) can be described as a:
a. dysplasia
b. syndrome
c. disruption
d. deformation
e. none of the above
4. d. Deformations result when normal tissue is exposed to abnormal mechanical forces in
utero.

Nongenetic Etiologies for Congenital Defects


1. The diagnosis of partial fetal alcohol syndrome requires:
a. short palpebral fissures
b. smooth philtrum
c. thin upper lip
d. central nervous system abnormalities
e. evidence of prenatal alcohol exposure
1. e. Diagnosis of partial fetal alcohol syndrome (PFAS) requires evidenced of prenatal
alcohol exposure.

Patterns of Congenital Disorders


1. A 38-weeks-gestation newborn infant at birth is noted to be small for gestational age,
have microcephaly, cleft lip and palate, narrow hyperconvex fingernails, developed
central apnea shortly after birth, and an echocardiogram showed ventricular septal
defect (VSD). This infant most likely has:
a. Trisomy 21
b. Fetal alcohol syndrome
c. Trisomy 13
d. Trisomy 18
e. Prader-Willi syndrome
1. c. Trisomy 13 (Patau syndrome), incidence is 1 in 5,000–10,000 live births, with 95% of
trisomy 13 conceptions resulting in spontaneous abortion; 90% born live die within the first
year of life. Abnormalities include: cleft lip and palate, polydactyly, narrow hyperconvex
fingernails, colobomas, umbilical or inguinal hernia, cryptorchi-dism, microcephaly,
holoprosencephaly, seizures, cardiac defects (VSD), and apnea.
2. A full-term, appropriate for gestational age (AGA) male infant has persistent
hypoglycemia despite adequate calorie feeds, orally. He is also noted on physical
examination to have a large tongue and exophthalmos. What is the mode of inheritance
of this infant’s condition?
a. Autosomal dominant
b. Uniparental disomy
c. Autosomal recessive
d. X-linked recessive
e. X-linked dominant
2. b. 20% of cases of Beckwith-Wiedemann syndrome is caused by paternal uniparental
disomy (UPD).

Molecular Genetics
1. A male infant has hypotonia, poor feeding, and cryptorchidism.You suspect Prader-
Willi syndrome (PWS).Evaluation of the 15q11-13 region does not reveal any
deletions. You should next evaluate for:
a. point mutations in the PWS critical region
b. duplication of the 15q11-13 region
c. uniparental disomy of the 15q11-13 region
d. other causes of neonatal hypotonia
1. c. Uniparental disomy in the region of chromosome 15q11-13. PWS is caused by lack of
the paternally inherited genes in the 15q11-13 region. PWS can occur secondary to deletion
of the 15q11-13 region on the paternally inherited chromosome. Alternatively, PWS can be
due to inheritance of both copies of 15q11-13 from the mother (uniparental disomy)

2. Diseases with this mode of inheritance are almost exclusively transmitted from the
mother:
a. imprinted
b. autosomal recessive
c. trinucleotide repeats
d. mitochondrial
2. d. Mitochondria are organelles that have their own chromosome which encodes several
genes essential for mitochondrial function. Mitochondrial DNA is inherited almost
exclusively from the mother.
3. You are asked to evaluate an infant for hypotonia. You note that the infant has
significant hypotonia and feeding difficulties. The mother is well-appearing, but has
difficulty releasing your handshake. Both the mother and infant are subsequently
diagnosed with myotonic dystrophy. The phenomenon described here, where
the infant is more severely affected than the mother, is
known as:
a. anticipation
b. imprinting
c. mitochondrial inheritance
d. uniparental disomy
3. a. Anticipation occurs in trinucleotide repeat diseases. Trinucleotide repeats tend to be
unstable, with their size increasing with each subsequent generation, resulting in a more
severe phenotype.
4. You are called to evaluate an infant with tetralogy of Fallot, cleft palate, and
hypocalcemia. After your initial evaluation, you suspect the child may have DiGeorge
syndrome. What is the most appropriate initial test to order?
a. Karyotype
b. Fluorescence in situ hybridization (FISH)
c. Whole exome sequencing (WES)
d. Sequencing of the TBX1 gene
4. b. Of the choices listed, FISH for 22q11 is the most appropriate test to evaluate for
suspected DiGeorge syn-drome. Although not listed, aCGH would also have been an
appropriate choice. A karyotype does not have suf-ficient resolution to reliably detect the
22q11 microdele-tion. A small fraction (<5%) of patients with DiGeorge syndrome will have
normal cytogenetic studies. In those patients, targeted evaluation of DiGeorge locus genes,
such as TBX1, may be indicated. WES is not indicated in the evaluation for DiGeorge
syndrome.
5. Which of the following is a limitation of newborn screening?
a. Preterm or critically ill neonates often have falsepositive
results.
b. Current screening techniques can only evaluate for a
few metabolites on each sample.
c. Cystic fibrosis is not reliably detected by most newborn
screening programs.
d. Many states do not screen for sickle cell disease.
5. a. Preterm or critically ill term infants are more likely to have false positive newborn
screening results. Most newborn screening programs utilize tandem mass spectrometry,
which can analyze multiple metabolites, simultaneously enabling screening for many
disorders with a single blood sample. Newborn screening panels do vary from state to state
but all states screen for sickle cell disease and cystic fibrosis.

Assisted Ventilation , ECMO,and Pharmacologic Agents


1. Which of the following are clinical situations for which
ECMO is indicated?
a. A neonate with transposition of the great arteries who has undergone a
successful arterial switch palliation but is unable to maintain adequate blood
pressure and oxygenation immediately postoperatively
b. A 4-day-old with trisomy 21 and overwhelming sepsis
and pneumonia on maximal ventilatory support,bilateral pneumothoraces, and an
oxygenation index of 60 in spite of maximal medical therapy
c. A 30-week-old, 1200-g infant with severe RDS unresponsive
to multiple doses of surfactant on optimal ventilator support with an oxygenation
index of 50
d. A full-term infant with rupture of membranes at 18weeks and persistent severe
oligohydramnios whose parents chose to continue the pregnancy, who presents
in the delivery room with severe increased work of breathing. The infant remains
hypoxic with pO2 20 mm Hg and pCO2 110 mm Hg in spite of aggressive
maximal cardiorespiratory support. A cardiac ECHO reveals normal anatomy, right
ventricular dysfunction and significant right to left shunting at the ductal and atrial
level. CXR suggests pulmonary hypoplasia.
1. a and b. ECMO is used for reversible conditions unre-sponsive to less invasive treatment
options. Given the need to anticoagulate and the technical limitations related to cannulation,
it is not standard of care to attempt ECMO in infants < 34 weeks and < 1800–2000 g. It is
also important to assess if a condition is reversible before initiating ECMO. Case (d)
describes an infant with pulmonary hypertension related to severe pulmonary hypoplasia
which developed as a result of prolonged anhydramnios during an important stage of
pulmonary development early in pregnancy. The pulmonary hyper-tension is related to
structural maldevelopment of the pulmonary capillary bed, resulting in a poor prognosis.

2. You are considering initiating iNO therapy for a37-week-old infant of a


diabetic mother with severe RDS. Which of the following is not indicated prior
to initiation of iNO?
a. Cardiac ECHO
b. CXR
c. Optimized lung expansion and ventilator support
d. Surfactant replacement
e. Steroid therapy
2. e. Infants of diabetic mothers can have delayed matu-ration of the surfactant, resulting in
increased inci-dence of RDS at an older gestational age. Cardiac ECHO is indicated to rule
out cardiac defect and confirm diag-nosis of pulmonary hypertension. CXR assesses lung
expansion and possible presence of pneumothorax. Surfactant replacement is indicated
before iNO when treating RDS. Steroid therapy has not been shown to benefit infants with
RDS, with or without pulmonary hypertension.
Apnea of Prematurity and Neonatal Respiratory Depression
1. A 32-year-old pregnant mother at 28 weeks’ gestation is referred to you for a
neonatology prenatal consult.According to the obstetrician, the fetus has reduced fetal
movements, thin bones, and polyhydramnios. The ultrasound also notes the near total
absence of fetal breathing. How will you explain the significance of the
lack of fetal breathing to the mother?
a. Fetal breathing is not significant indicator of fetal health
b. Lack of fetal breathing implies severe periodic breathing in neonate after birth
c. Regular fetal breathing at the rate of 30–40/minute
is essential for establishment of neonatal breathing
d. Fetal breathing is essential for lung development
1. d. Fetal breathing is essential for lung development. Fetal breathing movements begin in
the first trimester and are initially “tonic” with irregular tonic contrac-tion of diaphragm and
prolonged periods of apnea. The breathing becomes phasic in the third trimester but is still
associated with apneic periods. Lack of fetal breath-ing due to neuromuscular diseases or
ineffective fetal breathing movements (e.g., when the anterior chest wall is experimentally
replaced with a silicone membrane) are associated with pulmonary hypoplasia. The fetus in
the question is likely to have significant pulmonary hypopla-sia which will complicate the
postnatal ventilatory man-agement and may not be compatible with prolonged life.
2. You are doing teaching rounds with medical students and are demonstrating
periodic breathing in a 28-week preterm infant who is 2 weeks old. How will you best
describe the response of this infant to hypoxia?
a. Immediate and persistent hyperventilation
b. Biphasic response, early ventilatory depression and later hyperventilation
c. Biphasic response, early hyperventilation and later ventilatory depression
d. No predictable response
2. c. Biphasic response to hypoxia is characteristically noted in preterm infants, especially in
those with apnea of prematurity. Adults normally respond to hypoxia with sustained increase
in rate and depth of breathing. In preterm infants, there is an initial increase in the rate of
breathing which is thought to be the result of periph-eral chemoreceptor stimulation. After
1–2 minutes, though, a period of respiratory depression follows. This is primarily due to
reduced rate rather than reduced tidal volume and is thought to be due to hypoxia mediated
central ventilatory depression. The mediators for such ventilatory depression include
adenosine, GABA, and endorphins. Such inhibition of ventilation is noticed in the fetus too,
leading to the hypothesis that the purpose of the inhibition is to reduce the metabolic rate,
which, in a fetus dependent on placental oxygen delivery, will lead to reduce oxygen
demand.
Pleural Disorders and Additional Causes of Respiratory Distress
1. A term infant has inspiratory stridor noticed in the normal nursery a few hours after
birth. The stridor was most noticeable when the infant cried. There were mild
retractions but no oxygen desaturation. Mother had gestational
diabetes during pregnancy. Infant was born by spontaneous vaginal delivery at 39
weeks after forcepsassisted delivery with a birth weight of 4.3 kg. What is the most
likely diagnosis:
a. unilateral vocal cord paralysis
b. subglottic stenosis
c. laryngomalacia
d. hemangioma
1. a. Unilateral vocal cord paralysis is the second most cause of stridor in infants. This is
more common on the left side following damage to the recurrent laryngeal nerve. Surgery in
the neck or the chest is the most com-mon cause of the palsy, but trauma around birth, espe-
cially with the use of forceps, is also an important cause. Subglottic stenosis is usually
acquired and is not pres-ent at birth while laryngomalacia tends to present after the first few
days of life with the symptoms peaking at around 4–6 weeks of age. Similarly, hemangioma
also presents after first few weeks once the tumor enters the proliferative phase.

2. A newborn was born at 36 weeks following spontaneous labor. Mother had late onset
of prenatal care and the fetus was noted to have hydrops. At birth, the infant was noted
to have poor respiratory effort and hydrops with skin edema and abdominal distention.
Resuscitation included drainage of fluid from the chest and abdominal cavities and the
infant was admitted to the NICU. Examination in the NICU reveals dysmorphic
features, downsloping palpebral fissures, webbed neck, dysplastic pulmonary valve,
and cryptorchidism. Analysis of pleural fluid revealed serosanguinous fluid with
lymphocyte predominant cell count. What is the likely diagnosis?
a. Congenital empyema
b. Immune hydrops
c. Cardiac failure
d. Chylothorax
2. d. Infant likely has Noonan syndrome with chylotho-rax. Noonan syndrome is associated
with lymphatic abnormalities and is a common cause of congenital chylothorax. Fluid in
congenital chylothorax is sero-sanguinous prior to initiation of feeds and may turn milky
with onset of feeding. Diagnosis of chyle includes demonstration of lymphocyte
preponderance (contrib-utes to milky color), chylomicrons in the fluid, fluid triglycerides
>110 mg/dL and pleural fluid/serum cholesterol <1. Cardiac failure associated with anemia
(immune and nonimmune) and cardiomyopathy can also present with fetal hydrops and
serosanguinous fluid, but usually do not have lymphocyte preponder-ance of cell count.
Empyema has high neutrophil count and protein in the fluid and can be caused by intra-
amniotic shunt placements.
Aspiration, Pneumonia,and Persistent Pulmonary Hypertension
1. A 41-week-old infant is born after induction of labor.There was thick particulate
meconium at delivery. The neonatal resuscitation team that received the infant did
initial resuscitation by stimulating the infant, and when there was no respiratory effort,
initiated positive pressure ventilation (PPV). The infant eventually required
endotracheal intubation and admission to the NICU. Achest x-ray revealed coarse
bilateral opacities. Which of the following strategies is recommended in the further
management of this infant?
a. Lung lavage with dilute surfactant
b. Exogenous surfactant at standard doses
c. Respiratory alkalosis using high ventilator rates
d. Routine use of muscle relaxants
1. b. Lung lavage with dilute surfactant does not have clear evidence to support its routine
use. MAS is associated with inactivation of surfactant and use of surfactant has been
associated with less use of ECMO. Use of high ven-tilator rates can increase air-trapping and
increase the risk for pneumothorax. Further, hypocarbia can be det-rimental to cerebral
circulation. Muscle relaxants may be useful to improve oxygenation in severe MAS with
PPHN, but routine use of muscle relaxants is not recom-mended.
2. A preterm infant born at 26 weeks’ gestation is now 3 days old and you are
considering initiating trophic enteral feeds. Mom is known to be seropositive for
cytomegalovirus (CMV) and she asks you about the risk of transmission of CMV
through breast milk and the risk of pneumonia due to CMV. What is your response?
a. CMV is eliminated by freeze-thawing process.
b. CMV only rarely causes pneumonia, postnatally.
c. CMV can cause interstitial pneumonia in preterm
infants but can be eliminated by short-term pasteurization.
d. Provide preterm formula as the risk of pneumonia is very high.
2. c. CMV transmission and disease is rare in term infants as mother will likely have
transferred passive immunity to the infant during pregnancy. In preterm infants, pas-sive
transmission of immunity is incomplete and cannot be relied upon. In a prospective cohort
study of very low birth weight infants born to CMV seropositive mothers, the cumulative
incidence of CMV infection at 12 weeks was 6.9%. About 17% of the infected infants
developed disease. Sepsis-like syndrome, interstitial pneumonia, hepatitis, and necrotizing
enterocolitis are important clinical presentation of CMV disease in preterm infants. Holder
pasteurization (62.5°C [144.5°F] for 30 minutes) and short-term pasteurization (72°C
[161.6°F] for 5 sec-onds) of expressed breast milk can inactivate CMV. It is likely the short-
term pasteurization will preserve more nutrients than the Holder method. Freezing of
expressed milk does not reliably eliminate CMV.

Respiratory Distress Syndrome


1. Surfactant proteins B and C are essential components for surfactant replacement
because they:
a. Increase surfactant spreading at the air-liquid interface.
b. Coat bacteria and viruses to increase innate immunity.
c. Detoxify oxygen radicals and reduce injury to alveolar type II cells.
d. All of the above.
1. a. Increase surfactant spreading at air liquid inter-face. Surfactant proteins are categorized
as either hydrophobic (SP-B and SP-C) or hydrophilic (SP-A and SP-D). The hydrophobic
proteins are secreted with surfactant in lamellar bodies and are required for production of
lamellar bodies and for the normal spreading of surfactant phospholipids into a mono-layer
at the alveolar air-liquid interface. SP-B and SP-C are developmentally regulated. SP-A and
SP-D are collectins that bind to carbohydrates on the sur-faces of bacteria, fungi and viruses.
They are develop-mentally regulated and are important components of the lung host defense
system.
2. Which of the following recommendations about postnatal glucocorticoid use in BPD
are contained in the 2010 AAP policy statement?
a. Therapy with high-dose dexamethasone cannot be recommended.
b. There is insufficient evidence to make a recommendation regarding treatment with low-
dose dexamethasone.
c. Early hydrocortisone treatment may be beneficial in a specific population of patients;
however, there is insufficient evidence to recommend its use for all infants at risk of BPD.
d. All of the above.
2. d. All of the above. The 2010 AAP policy statement on use of glucocorticoids to mitigate
BPD does not recommend high dose dexamethasone because of the potential for adverse
neurologic outcome. No subsequent policy has been issued. Both low dose dexamethasone
and low dose hydrocortisone need more randomized controlled trials, particularly focusing
on neurodevelopmental out-come, before they can be recommended. Short courses of low
dose glucocorticoids are commonly used to treat BPD in ventilator- dependent patients.
Their use should balance the potential benefits of these agents with poten-tial harm they may
cause. The decision to use these drugs should be individualized for selected patients.

Pharmacologic Therapy of Heart Disease


1. An infant with a large ventricular septal defect and congestive heart failure is being
medically managed with digoxin, enalapril, and furosemide (Lasix). Spironolactone
was added yesterday due to persistent hypokalemia.Overnight, the infant developed
Mobitz II heart block. This is a toxicity of which medication?
a. Spironolactone
b. Digoxin
c. Enalapril
d. Lasix

1. b. Atrioventricular block is a well-described result of digoxin toxicity. Digoxin toxicity is


more likely to occur in the setting of hypokalemia
2. A 10-day-old infant is being treated for sepsis with antibiotics and dopamine. The
infant is requiring escalating doses of dopamine. All the interventions below can
increase the infant’s response to the current dose of dopamine, except for one. Which
intervention does not increase an infant’s sensitivity to catecholamines?
a. Beginning steroid administration
b. Normalizing pH
c. Normalizing serum calcium levels
d. Normalizing potassium levels
2. d.

Electrocardiography,Electrophysiology, andDysrhythmias

1. An infant was born at full term to a 29-year-old woman with lupus by cesarean
section due to persistent fetal bradycardia. The newborn appears vigorous,with normal
capillary refill, blood pressure of 80/55 mm Hg, and 2+ femoral pulses. The heart rate,
however,is 65 beats/min, with a narrow QRS complex not associated with p waves.
What is the best course of action?
a. Begin isoproterenol
b. Begin cardiopulmonary resuscitation
c. Monitor closely on telemetry
d. Use transcutaneous pacing
1. c. This infant has complete heart block related to trans-placental transfer of SSA and/or
SSB antibodies related to maternal lupus. Although the heart rate is low, the infant has
adequate cardiac output, as evidenced by good capillary refill and blood pressure.
Monitoring is all that is required in this situation.
2. A 3-day-old infant suddenly develops tachycardia to 240 beats/min. Adenosine is
administered, which slows the ventricular rate briefly to 140 beats/min,
revealing a sawtooth-like pattern between the QRS complexes. Transesophageal pacing
is used to terminate the tachycardia. Which drug is best to use as prophylaxis
to prevent future tachycardia?
a. Propranolol
b. No prophylactic therapy needed
c. Digoxin
d. Amiodarone
2. b. This infant had an episode of atrial flutter, which is very unlikely to recur and thus does
not warrant pro-phylaxis.

Cardiopulmonary Dysfunction
1. A term infant undergoes echocardiography after birth due to a murmur. Numerous
small cardiac tumors are identified throughout the heart. Which genetic disease is
likely?
a. Down syndrome
b. Williams syndrome
c. Tuberous sclerosis
d. Hunter syndrome
1. c. Numerous cardiac tumors found in a neonate are almost always rhabdomyomas, which
are strongly asso-ciated with tuberous sclerosis.

Presentation of Congenital Heart Disease


1. A 1-week-old infant with tetralogy of Fallot is undergoing
a rule-out sepsis evaluation. The infant’s baseline
saturations are 98%, with a 2/6 systolic ejection
murmur along the left sternal border. During the blood
draw, the infant’s saturations drop to 60% and the
infant becomes visibly cyanotic. On auscultation, the
lung sounds remain clear, but the murmur is no longer
audible. What interventions should be performed?
a. Intubation with 100% O2
b. STAT echocardiogram
c. Arterial blood gas
d. Morphine and phenylephrine administration
1. d. The infant is this scenario is having a hypercyanotic spell caused by an acute decrease
in pulmonary blood flow. These are commonly triggered by medical proce-dures such as
blood draws. Morphine and phenylephrine are first-line agents to counteract the
hypercyanotic spell.
2. A newborn infant was found to have interrupted aortic
arch type B, with truncus arteriosus. What other features
is the infant likely to have?
a. Epicanthal folds with a single palmar crease
b. Elfin facies
c. Broad thumbs and toes
d. Absent thymus on chest x-ray and hypocalcemia
2. d. Both type B interrupted arch and truncus arteriosus are associated with 22q11 deletion
syndrome. Infants with 22q11 deletion syndrome frequently have hypo-calcemia and an
absent thymus
Cardiovascular Physiology
1. Physiologic responses to asphyxia include which of the following?
a. Tachycardia, decreased central venous pressure,
vasoconstriction of the cerebral vessels
b. Bradycardia, increased central venous pressure,
vasoconstriction of the skeletal muscle vessels
c. Bradycardia, decreased central venous pressure,
vasodilation of the cerebral vessels
d. Tachycardia, increased central venous pressure,
vasoconstriction of the skeletal muscle vessels
1. b.
Cardiac Development
1. An infant was born with truncus arteriosus. What embryologic event led to this
anomaly?
a. Failure of the conotruncus to septate
b. Failure of the second heart field cells to migrate
c. Failure of the conotruncus to rotate
d. Abnormal development of the 6th arch
1. a. Truncus arteriosus results from failure of the conotruncus to separate.
2. The most common cyanotic congenital heart defect seen in infants of diabetic
mothers is which of the following?
a. d-Transposition of the great arteries
b. Hypoplastic left heart syndrome
c. Tricuspid atresia
d. Tetralogy of Fallot
2. a.

Asphyxia and Resuscitation


Asphyxia
1. You are called to the delivery of a 39-week infant, with no known maternal
complications of pregnancy. Meconium was noted at the time of delivery. You arrive in
the delivery room ∼1 minute after birth and find the infant is on a warmer, apneic, and
being vigorously stimulated by nursing staff. The HR is noted to be less than 100
bpm. Of the following, which is the most appropriate next step to improve the patient’s
heart rate?
a. Continue vigorous stimulation of the patient
b. Deep-suction the posterior pharynx
c. Provide positive-pressure ventilation via bag-mask ventilation
d. Intubate the patient
e. Obtain umbilical access and administer intravenous epinephrine
1. c. This patient is likely in secondary apnea. The appro-priate next step is to provide
effective positive-pressure ventilation. Continuous stimulation is unlikely to induce
spontaneous respirations and improvement in heart rate in this infant. Deep suctioning may
fur-ther exacerbate bradycardia by inducing a vagal nerve stimulatory response. If the
patient does not respond to effective bag-mask ventilation, obtaining a more secure airway
and administration of epinephrine may become necessary, but not before a period of
effective positive-pressure ventilation and chest compressions has been attempted.

2. Based on current evidence and practice guidelines,which of the following describes


an infant who would most benefit from therapeutic whole-body cooling?
a. 33-week infant born due to maternal preeclampsia,required intubation at birth, Apgar
scores—1 at 1minute, 3 at 5 minutes, 8 at 10 minutes

b. 37-week infant born after prolonged shoulder dystocia,Apgar scores—1 at 1 minute, 3 at


5 minutes, 8at 10 minutes—and an arterial cord gas of pH 6.98/CO2 88 mmHG/PaO2 24
mmHg/ Bicarbonate 20mmol/L/base excess–17 who at 1 hour of age appears
hyperalert, with mildly increased tone, a weak suck,
HR of 190 beats/min, and respiratory rate (RR) of 75breaths/min.

c. 39-week infant born after uterine rupture, Apgar scores—1 at 1 minute, 3 at 5 minutes, 8
at 10minutes—and an arterial cord gas of pH 6.98/CO288 mmHG/PaO2 24 mmHg/
Bicarbonate 20 mmol/L/base excess–17 who at 1 hour of age appears comatose,
remains intubated, is lethargic, with decreased activity, hypotonia, incomplete Moro reflex
and suck,and intermittent spontaneous respirations on the ventilator.

d. 37-week infant, now 12 hours old, found in mother’s


room to be unresponsive and received CPR with two
doses of epinephrine before spontaneous return of circulation.
2. c. The infant in choice c presents with signs of moder-ate encephalopathy after a known
perinatal hypoxic event and is most likely to have a neurologic benefit from whole-body
cooling. The infant in (b) has also experienced a significant hypoxic event at birth but shows
signs of only mild encephalopathy on examina-tion and therefore does not meet the criteria
for whole-body cooling. The infant in (a) is preterm; cooling in this population has not yet
been established to be beneficial. The infant in (d) had an unwitnessed hypoxic event 12
hours after birth; whole-body cooling for neonates who experience this type of arrest is not
the standard of care.
3. Which of the following is the preferred dose of epinephrine to be given during a
neonatal resuscitation?
a. 0.01-mL/kg/dose of a 1:10,000 solution IV
b. 0.5-mL/kg/dose of a 1:1000 solution via ETT
c. 0.0-mL/kg/dose of a 0.1-mg/mL solution (1:10,000)IV
d. 1-mL/kg/dose of a 1:10,000 solution via ETT
3. c. This is the preferred dosage and route of administra-tion for epinephrine during a
neonatal resuscitation.
4. Which of the following is an advantage of a self-inflating bag over a T-piece
resuscitator?
a. It can administer PPV without a source of compressed air or oxygen
b. It can provide PEEP when applied continuously to the face
c. It provides consistent PIP and PEEP with each breath,with minimal variation between
breaths
d. It can be used to deliver oxygen concentrations > Fio2 21%
4. a. Of the most commonly used devices used in neona-tal resuscitation, the self-inflating
bag is the only device that can deliver positive-pressure breaths without being attached to a
source of compressed air or flow. A self-inflat-ing bag cannot deliver PEEP unless
additional valves or mechanisms are attached to the device, whereas a T-piece resuscitator
can. Additionally, the benefit of a T-piece resuscitator is that it will deliver consistent PIP
and PEEP to the patient, provided that a good seal is made between the mask and the
patient’s face and/or the ETT remains in place. Both devices can deliver PPV with an
increased Fio2concentration if attached to an oxygen source.
5. The hypercapnia, hypoxemia, and acidosis that result from asphyxia will initially
cause a redistribution of blood flow to which organs?
a. Heart, kidneys, and adrenal glands
b. Heart, intestines, and brain
c. Heart, brain, and adrenal glands
d. Brain, intestines, and kidneys
e. Brain, kidneys, and heart
5. c. The hypercapnia, hypoxemia and acidosis that result from asphyxia will initially cause
a redistribution of blood flow to the most vital organs, the heart, brain and adrenal glands.
6. Which of the following is a feature of primary energy failure from hypoxic brain
injury?
a. Occurs 6−48 hours after hypoxic injury
b. Necrotic cell death
c. Apoptosis
d. Decreased glutamate reuptake by damaged cells
e. Increases in cerebral ATP stores
6. b. Necrotic cell death occurs after primary energy failure due to depletion of cerebral ATP
stores and inactivation of the Na/K membrane pumps. Secondary energy failure occurs 6−48
hours after the initial hypoxic injury and results in decreased glutamate reuptake, which
leads to the induction of apoptosis

Labor and Delivery Questions


1. For which of these clinical situations would you recommend a prompt, planned
cesarean delivery?
a. 30-year-old G3P2002 at 39 weeks; cesarean delivery × 1 (breech presentation)
b. 20-year-old G1P0 at 39 weeks; fetal gastroschisis
c. 24-year-old G2P1001 at 39 weeks; fetal hypoplastic left heart, unrestrictive atrial septum
d. 23-year-old G1P0 at 36 weeks; fetal sacrococcygeal teratoma with signs of fetal hydrops
e. 35-year-old G4P1112 with maternal dilated cardiomyopathy who has a left ventricular
ejection fraction (LVEF) of 40%
1. d. Large fetal masses are at risk of rupture or dystocia during attempted vaginal deliver;
thus, cesarean deliv-ery is recommended. Hydrops is an indication for deliv-ery near term.
Women with two or less low-transverse cesareans may consider a trial of labor. Cesarean
deliv-ery is not required for fetal gastroschisis. Labor and vagi-nal delivery are well
tolerated by fetuses with congenital heart disease; cesarean delivery is considered only in
extreme cases. Most women with cardiac disease can have safe vaginal deliveries.
2. A 31-year-old G4P3003 presents at 35 weeks with heavy vaginal bleeding and a
known placenta previa.She previously received a steroid course for an episode of
vaginal bleeding. She had two prior cesarean deliveries.An ultrasound at 18 weeks
showed placental lacunae and abnormal bladder vascularity. Maternal vital signs are
stable. Fetal NST is reactive. The most appropriate next step in management is:
a. Prompt cesarean hysterectomy
b. Repeat steroid course followed by cesarean hysterectomy
c. Repeat steroid course followed by cesarean delivery
d. Expectant management until term
e. Expectant management until documentation of fetal lung maturity
2. a. History and sonographic features are suggestive of placenta accreta. Management
involves planned cesarean hysterectomy at 34–36 weeks due to ongo-ing risk of massive
hemorrhage. Because this patient presented with heavy bleeding, prompt delivery is
indicated. Repeat steroid courses are not given after 34 weeks of gestation; thus,

(b) and (c) are incor-rect. Also,

(c) does not call for cesarean hysterec-tomy, which is the definitive management of placenta
accreta. Expectant management is not appropriate, given active hemorrhage. Testing for
fetal lung matu-rity is not recommended because delivery is indicated by the maternal
condition.

Fetal Assessment and Treatment


1. A patient at 32 weeks is having fetal growth surveillance for renal disease. The EFW
is at the seventh percentile, with a normal amniotic fluid volume. The umbilical
artery S/D ratio is elevated. Twice-weekly NSTs and weekly assessments of amniotic
fluid volume are started for surveillance. Which of the following findings require
delivery?
a. Fetal EFW < 10th percentile on the next growth assessment
b. There is interval development of oligohydramnios
c. Umbilical artery velocimetry shows reversed enddiastolic
flow
d. Intermittent variable decelerations are noted on reactive NST
1. c. Umbilical artery Doppler velocimetry defines a group of fetuses that is at high risk of
adverse perinatal out-comes. Reversed end-diastolic flow should prompt deliv-ery after 32
weeks. An EFW < 10th percentile defines growth restriction, but fetal tests of well-being are
used to make delivery decisions. Oligohydramnios does not prompt delivery until term.
Intermittent variable decel-erations on a reactive NST do not portend imminent compromise.
2. A woman with diabetes is having a biophysical profile assessment at 34 weeks. Fetal
NST shows more than two accelerations in the fetal heart rate, which is 10 beats/min
above the baseline. Fetal breathing was seen. The single
deepest vertical pocket (SDP) of amniotic fluid exceeds 2 cm. Many fetal body
movements are seen, including at least one kick, with return to flexion. What is the
BPP score, and what additional testing do you recommend?
a. 6; no further testing required at this time
b. 6; continuous electronic fetal monitoring
c. 8; no further testing required at this time
d. 8; consider a repeat biophysical profile within 24 hours
e. 10; no further testing required at this time

2. c. This fetus receives 2 points each for amniotic fluid vol-ume, movement, tone, and
breathing (total of 8 points). No points are received for the NST because the fetus is not
reactive (15 beat accelerations are required at 34 weeks). A BPP score of 8 or 10 is
predictive of normal fetal oxygenation, and no further testing is required. A BPP score of 6
is an equivocal test and requires follow-up testing (BPP at 8 and 24 hours) to determine if
delivery is required
Pre &perinatal problems
1. Which of the following is true regarding maternal adaptation
to pregnancy and the impact on chronic disease?
a. Serial echocardiography should be done in mothers with cardiac disease because
pregnancy-induced cardiovascular changes may worsen cardiac dysfunction.
b. Peak flow monitoring cannot be used for asthma management because peak expiratory
flow and FEV1 are altered by pregnancy.
c. Increased doses of antihypertensives are required in pregnancy because of the increase in
systemic vascular resistance.
d. Pregestational diabetics require less insulin during pregnancy because of lower renal
clearance of insulin.
e. Free T4 levels cannot be used to monitor thyroid disease because these values are
increased in pregnancy.
1. a. Increased cardiovascular demand may cause or worsen cardiac dysfunction
in women with cardiac diseases. Echocardiography is recommended in each trimester
or with new cardiac symptoms. Peak flow monitoring should be used because
expiratory respi-ratory indices are unchanged in pregnancy. Systemic vascular
resistance decreases in pregnancy; thus, anti-hypertensive doses often can be lowered
or eliminated. Insulin dose increases are common because of progres-sive insulin
resistance in pregnancy. Although total T4and T3 increase in pregnancy, steroid-
binding globulin also increases, so free levels are unchanged.
2. An Ashkenazi Jewish couple with a child affected by Tay-Sachs disease presents for
preconception counseling.Their child’s genetic analysis has identified a causative
mutation that is detected on routine screening panels.Neither partner has had prior
genetic testing. The wife has chronic hypertension treated with lisinopril. All the
following are recommended except:
a. Folic acid supplementation prior to conception
b. Discontinuance of lisinopril and labetalol started before conception
c. Expanded carrier screening for Eastern and Central European Jewish descent
d. An interpregnancy interval of less than 18 months
e. Gamete donation from a known Tay-Sachs negative donor to conceive the next pregnancy
2. d. An interpregnancy interval of at least 18 months is advised. Folic acid
supplementation is recommended for all women to prevent neural tube defects.
Lisino-pril use is contraindicated in pregnancy; transition to a preferred medication
should be carried out. Gamete donation can prevent genetic disease in the offspring
if parents are known carriers. Expanded carrier screening is offered to all patients of Eastern
and Central European Jewish descent. Although this couple may be presumed to be Tay-
Sachs carriers, mutation analysis will confirm recurrence risk and screen for other at-risk
conditions.
3. A 30-year-old G3P1011 woman presents at 12 weeks with known Kell sensitization
(red blood cell alloimmunization).She had an uncomplicated pregnancy 4
years ago and a 24-week fetal demise that occurred last year. She had a motor vehicle
accident 5 years ago that required multiple blood transfusions. She is wondering
if Kell sensitization caused the fetal demise. How would you answer her question?
a. It definitely caused the fetal demise.
b. It cannot cause fetal demise unless there was bleeding
in the pregnancy.
c. It did not cause the fetal demise because her first pregnancy
was uncomplicated.
d. It may have caused the demise; paternal antigen testing
is required to determine if other pregnancies are
at risk.
3. d. Alloimmunization may have caused demise if the fetus was Kell-positive.
Absence of bleeding does not rule out Kell sensitization; anemia develops from trans-
placental passage of maternal antibodies. Sensitized patients may have
uncomplicated pregnancies if a fetus is antigen-negative. Paternal antigen testing will
show if a pregnancy is at risk—no fetal risk if the father is Kell-negative, but
possibly so if the father is Kell-positive (may be heterozygous or homozygous).

4. Which of the following is not routinely recommended in the management of


pregestational diabetes in pregnancy?
a. Nutrition consult to review dietary and lifestyle modifications
b. Fetal echocardiography at 20 weeks due to increased risk of congenital heart defects
c. Frequent blood glucose checks and oral hypoglycemics or insulin as needed
d. Cesarean delivery if the EFW > 4500 g, given the high risk of shoulder dystocia
e. All patients should be delivered at 37 weeks due to an increased risk of stillbirth,
4. e. Stillbirth risk is increased with DM; fetal surveillance is recommended from 32
weeks. If glycemic control is poor, then 37- to 38-week delivery is considered.
Delivery is delayed until 39 weeks in well-controlled patients to allow time for fetal
maturity. Nutrition con-sultation is recommended in all diabetic pregnancies. Fetal
echocardiography is advised due to the increased risk of anomalies. Frequent blood
glucose checks are necessary to guide dosing of medications. Metformin, glyburide,
and insulin may all be used to achieve glyce-mic control.

5. A 27-year-old G1P0 at 29 weeks is admitted with new severe range hypertension.


Antihypertensives and a steroid course are ordered. Laboratory evaluation
shows 1400 mg/day proteinuria, creatinine of 1.0 mg/dL, AST of 106 mg/dL, platelet
count of 95,000/μL, and hematocrit of 28%. Other laboratory tests are normal. A
reactive NST was obtained.Fetal growth is normal. The patient has no symptoms.
What is the diagnosis, and what is the most appropriate next step in management?
a. Gestational hypertension; discharge from the hospital with close outpatient follow-up
b. Preeclampsia without severe features; expectant management until 37 weeks
c. HELLP syndrome; delivery after completion of steroid course
d. Preeclampsia with severe features; delivery after confirmation of fetal lung maturity
e. Preeclampsia with severe features; expectant management until 34 weeks of gestation of
controlled hypertension, stable laboratory results, and reassuring fetal status
5. c. This patient has HELLP syndrome. If maternal and fetal statuses are
reassuring, delivery should take place after completion of a steroid course. Severe
features of a preeclampsia syndrome (HELLP syndrome and severe hypertension) are
present; thus, a and b are incorrect. If the only severe feature is controllable
hypertension, then expectant management detailed in (e) is appropri-ate. Fetal lung
maturity testing is not advised because delivery is indicated regardless of fetal maturity.

6. A patient has a normal fetal anatomy survey at 18weeks. She decides to have
maternal serum quad testing for aneuploidy screening, which shows an elevated risk
of trisomy 21. Which of the following is true?
a. The anatomy survey must have overlooked features of trisomy 21 because the false-
positive rate of the quad screen is low.
b. The quad screen must be a false-positive because no sonographic features of trisomy 21
were detected.
c. Diagnostic testing for trisomy 21 should be offered.
d. The fetus has trisomy 21.

6. c. Many, but not all, fetuses with trisomy 21 have struc-tural anomalies or soft marker
ultrasound findings. The lack of anomalies or markers does not rule out a trisomy 21
diagnosis. Maternal serum quad testing is a screening test for aneuploidy; a positive
screening result should not be considered diagnostic. Although the detection rate of the
serum quad test is high (75%), the false-positive rate is also high.
Glucose Metabolism

1. A term infant is readmitted to the neonatal intensive 1. c. The patient most likely has glycogen storage disease
care unit (NICU) at 3 weeks of age in shock with car- 1 (GSD-1) due to glucose 6-phosphatase deficiency.
diorespiratory failure and oliguric renal failure. Labs This mutation blocks gluconeogenesis from glycoge-
at presentation showed plasma glucose of 25 mg/dL, nolysis during fasting and presents with severe lactic
metabolic acidosis with elevated lactate. Hepatomegaly acidosis. Often presents at a few months of age when
is noted on physical exam. Parents state that the infant infants begin to decrease feeding frequency and sleep
had not been feeding well for the past 3 days and slept through the night. Glycogen synthase deficiency
through the night last night without waking to feed. (GSD-0) leads to inability to synthesize glycogen and
What is the most likely diagnosis? therefore does not present with hepatomegaly. Neither
a. Congenital hyperinsulinism due to K ATP channel congenital hyperinsulinism nor fatty acid oxidation
mutation
b. Glycogen synthase deficiency
c. Glucose 6-phosphatase deficiency
d. Disorder of fatty acid oxidation

2. Laboratory results of a critical sample show venous 2. d. The labs are most c/w hyperinsulinism. Neither
glucose of 45 mg/dL, cortisol 2 μ/dL (normal stressed adrenal nor growth hormone deficiency can be diag-
value >18 micrograms/dL), growth hormone 12 ng/ nosed based upon a critical sample alone. An ACTH or
mL, free fatty acids 0.1 mmol/L (normal <1 mmol/L), CRH stimulation to test confirm normal hypothalamic-
β-hydroxybutyrate 0.2 mmol/L (normal <1.5 pituitary-adrenal axis would be appropriate in this
mmol/L). Plasma glucose increased to 80 mg/dL 30 case; GH was normal so growth hormone stimulation
minutes after administration of 1 mg glucagon Which testing is not needed. If clinically appropriate, the next
of the following is an appropriate next step? step in the management of hyperinsulinism would be
a. Start hydrocortisone 15 mg/m 2/BSA per day try diazoxide. It can take up to 5 days to see full effect
b. Perform an arginine/clonidine growth hormone of treatment; infant will continue to need intravenous
stimulation test or continuous enteral dextrose support, target BG > 70
c. Resume intravenous dextrose infusion to maintain mg/dL.
plasma glucose >60 mg/dL
d. Trial diazoxide at dose of 15 mg/kg/day

3. A full term, LGA infant female has required GIR of 25 3. c. Based upon the clinical history provided and the
mg/kg/min throughout the first week of life to prevent high GIR, this infant most likely has hyperinsulinism.
plasma glucoses levels from falling to below 30 mg/dL. While there is controversy regarding the definition of
Treatment should target what plasma glucose level in
a normal blood glucose range in healthy and preterm
this female?
infants, it is essential that all infants with suspected
a. >50 mg/dL
or confirmed congenital hypoglycemia be maintained
b. >60 mg/dL
at plasma glucose levels > 70 mg/dL. Normally, as
c. >70 mg/dL
plasma glucose levels fall below 70, counterregulatory
d. >80 mg/dL
mechanisms kick in to provide the brain with alternate
fuel sources (i.e., ketones) and the body with substrate
for gluconeogenesis (fatty acids, amino acids). In HI,
ketogenesis and lipolysis are blocked, leading to more
rapid and more severe neurologic damage because of
the lack of an alternate fuel source.

4. Which of the following is true regarding the use of 4. b. Diazoxide can cause fluid retention and may be asso-
diazoxide in neonates? ciated with development of pulmonary hypertension,
a. All infants with congenital hyperinsulinism will therefore it must be used with caution in patients with
respond to diazoxide. congenital heart disease with frequent monitoring of
b. Diazoxide should be used with caution in patients cardiac status. Most infants with stress-induced HI will
with congenital heart disease. respond to diazoxide, while only some forms of con-
c. Diazoxide causes hair loss. genital HI (GCK-HI, SCHAD-HI, GDH-AD AD-HI) are
d. Neonates on diazoxide need to have regular moni- responsive; patients with mutations in the KATP chan-
toring of CBCs for development of polycythemia. nel will need surgery. Diazoxide causes hypertrichosis
(reversible when discontinued) and can cause cytope-
5. Which of the following conditions of neonatal glucose nia.
disturbance is not associated with increased risk of dia- 5. d. Patients with transient neonatal diabetes typically
betes as an adult? have a remission period during childhood where insu-
a. Transient neonatal diabetes due to chromosome lin is not needed, but permanent insulin requirement is
6q24 mutation likely to return in adulthood. Diffuse HI d/t KATP chan-
b. Permanent neonatal diabetes due to activating nel will require near-total pancreatectomy; there is an
mutation in KATP channel increased risk of diabetes and insulin requirement after
c. Diffuse hyperinsulinism due to inactivating muta- this surgery. Hypopituitarism is not associated with
tion in K ATP channel increased risk of diabetes.
d. Neonatal panhypopituitarism
6. Which of the following statements is not true regarding 6. c. Glucose is the main source of energy for the brain.
energy utilization by the brain? During times of prolonged fasting, ketone bodies and
a. Glucose is the main source of ATP used by the lactate can be used as alternate fuel sources. Free fatty
brain. acids do not cross the blood brain barrier so are unable
b. Glucose is transported into brain cells via GLUT to be utilized directly by the brain for energy.
3.
c. Fatty acids cross the blood-brain barrier and can be
utilized as a source of ATP.
d. Ketone bodies cross the blood-brain barrier and can
be utilized as a source of ATP.

7. A 3-week-old AGA term male has persistent pre-fed


plasma glucose levels between 45–60 mg/dL. Other 7. d. Labs and clinical findings are strongly suggestive
notable clinical findings include partial cleft lip and of neonatal hypopituitarism, however GH and ACTH
palate and absent corpus callosum. Critical sample or CRH stimulation tests should be done to confirm
shows plasma glucose 47 mg/dL, cortisol of < 1 mcg/ the diagnosis. The patient will most likely require life-
dL, growth hormone < 1 ng/dL, β-hydroxybutyrate < long hormone replacement. He is also at risk of cen-
0.1 mmol/L, free-fatty acids < 0.1 mmol/L. Which of tral hypothyroidism, which could have been missed
the following statements is not consistent with these on newborn screen in states that use a primary TSH
laboratory findings? screening program. Uncooked cornstarch is used to
a. The patient will likely need lifelong hydrocortisone manage patients with glycogen storage disease and
replacement. has no role here.
b. Thyroid function with total or free T4 should be
checked ASAP.
c. Growth hormone therapy should be started imme-
diately once diagnosis is confirmed.
d. Uncooked cornstarch can be used to help manage
his hypoglycemia.
8. A 10-month-old former 24 weeks gestational age 8. a. The patient most likely has late-dumping syndrome.
(WGA) male infant has recently developed intermit- This condition is characterized by postprandial hypo-
tent episodes of hypoglycemia with plasma glucose glycemia, most commonly in patients with gastrostomy
levels as low as 30 mg/dL. He was receiving continu- and history of Nissen fundoplication. The etiology is
ous feeds via a G-tube; recently you have been work- not entirely understood, but appears to be a condition
ing to condense his feeds, and he is now receiving of excessive insulin secretion as a result of rapid glu-
daytime bolus feeds every 4 hours. Which of the fol- cose absorption and altered incretin hormone signal-
lowing is true about his condition? ing. These patients do not have fasting hypoglycemia
a. Critical sample is likely to show inappropriately nor- and hypoglycemia will not persist once oral feeds have
mal insulin and suppressed β-hydroxybutyrate. been established. In some cases, hypoglycemia can be
b. This condition is characterized by fasting hypogly- prevented by lowering rate of bolus feeds and tapering
cemia. rate over the last 30 minutes; acarbose given with each
c. The patient will have lifelong hypoglycemia. bolus feed is also effective.
d. Hepatomegaly is an associated clinical finding.

9. Which of the following hormones is not involved in 9. c. The secretion and action of a number of hormones
intermediary metabolism? contribute to intermediary metabolism. As plasma
a. Glucagon glucose levels fall below 70 mg/dL, insulin secre-
b. Insulin tion decreases while secretion of glucagon increases.
c. Thyroid hormone
Further declines in glucose levels induce secretion of
d. Growth hormone
epinephrine, growth hormone, and cortisol. Thyroid
hormone is not known to play a role in intermediary
metabolism.
10. Which of the following tests is most helpful in deter-
mining if source of hyperinsulinism is endogenous (i.e.,
congenital hyperinsulinism) or exogenous (i.e., surrep-
titious in Munchausen by proxy)?
a. Insulin level
b. β-hydroxybutyrate level
c. Glucose response to glucagon administration
d. C peptide level

10. d. Distinguishing between endogenous and exogenous


hyperinsulinism can be difficult. Biochemically, labo-
ratory results at time of critical sample and glucagon
stimulation test will be identical, with the exception
of C-peptide, which is expected to be elevated or inap-
propriately elevated with endogenous insulin over-
secretion but suppressed in the setting of exogenous
administration.
Calcium, Phosphorus, and Magnesium Metabolism
1. Which statement is true regarding skeletal develop- 1. c. Mutations in the alkaline phosphatase gene cause
ment? hypophosphatasia, a type of early onset osteoporosis
a. The femurs are formed via intramembranous ossifi- characterized by absent/diminished skeletal miner-
cation. alization. The femurs, like all long bones, are formed
b. A genetic defect resulting in impaired chondrocyte by endochondral ossification. Defects in chondro-
maturation would be expected to cause early onset cyte function lead to a number of skeletal dysplasias
arthritis but would not negatively affect the skeleton. because of impaired endochondral ossification. PTHrP
c. A genetic defect resulting in reduced alkaline phos- is an important regulator of calcium status and osteo-
phatase activity would negatively affect bone min- blast function prenatally, but has a limited role after
eral density during infancy. birth.
d. PTHrP is the primary regulator of osteoblast-
mediated bone formation in neonates.

2. Neonatal magnesium status affects calcium metabo-


lism in which way?
2. c. Normal magnesium levels are required for PTH. Both
a. Hypermagnesemia results in hypercalcemia due to high and low magnesium levels inhibit PTH secretion,
excessive secretion of PTH the mechanism is not fully understood. Both calcium
b. Hypermagnesemia results in hypocalcemia by bind- and magnesium form divalent cations, so magnesium
ing calcium in the GI tract and thereby decreasing would not be expected to bind to calcium and decrease
calcium availability for GI absorption availability for absorption (as is seen with phosphate
c. Hypomagnesemia results in hypocalcemia via anions).
decreased PTH secretion
d. Hypomagnesemia leads to hypercalcemia because
magnesium is required for renal calcium excretion 3. b. DiGeorge syndrome results in hypocalcemia
because of defective parathyroid gland development.
3. What is the mechanism of hypocalcemia in DiGeorge In many cases the hypoparathyroidism is transient,
(22q11.2 deletion) syndrome? though can be exacerbated by illness and acute stress.
a. Inactivating mutation in CYP27B1 Mutations in CPY27B1 cause vitamin D-dependent
b. Hypoplasia/aplasia of parathyroid glands rickets type 1A and can result in hypocalcemia due to
c. Renal resistance to parathyroid hormone decrease synthesis of active vitamin D. PTH resistance
d. Activating mutation in PHEX with associated as seen in pseudohypoparathyroidism due to muta-
hyperphosphatemia tions in GNAS. Inactivating mutations in PHEX cause
hypophosphatemia in X-linked hypophosphatemic
4. All of the following are signs/symptoms of hypocalce- rickets.
mia EXCEPT
a. Shortened QTc interval
b. Laryngospasm 4. a. Severe hypocalcemia can result in prolonged QTc
c. Involuntary muscle contraction interval and resultant cardiac dysfunction.
d. Seizure

5. A 5.1 kg, 39 Weeks gestational age (WGA) infant suf- 5. b. Transient hypoparathyroidism due to hypomagne-
fers a hypocalcemic seizure on DOL 2. What is the most semia. Large for gestational age is suggestive of infant
likely etiology? of diabetic mother, which is a risk factor for early neo-
a. Hyperparathyroidism due to inactivation mutation natal hypocalcemia related to low magnesium levels
in CaSR and impaired parathyroid hormone release. Inactivat-
b. Transient hypoparathyroidism due to hypomagne- ing mutations in the CaSR causing NSHPT and FHH
semia cause hypercalcemia and typically do not present with
c. Familial hypocalciuric hypercalcemia seizure. Infants born with DiGeorge (22q11.2 deletion)
d. Permanent hypoparathyroidism due to 22q11.2
deletion
6. A 32 WGA infant develops respiratory failure shortly 6. b. Alkaline phosphatase. This child most likely has
after birth. Radiographs show marked underminer- congenital rickets due to hypophosphatasia. This
alization of bones and multiple rib fractures. Labs are condition is caused by mutations in the ALP gene and
notable for a serum calcium of 12.5 mg/dL, serum results in markedly low serum alkaline phosphatase
phosphorus of 8.5 mg/dL, urine calcium to creatinine levels. Urine phosphorus is most helpful in differen-
ratio of 2.5. Which of the following tests will be most tiating malabsorptive from renal hypophosphatemic
helpful in confirming the diagnosis? rickets, which is not present based upon the elevated
a. Urine phosphorus serum phosphorus. 25-OHD would be the test of
b. 25-OH vitamin D choice if vitamin D deficiency rickets is suspected,
c. Alkaline phosphatase which would not typically present with this severity
d. Intact PTH and would be associated with low/normal serum cal-
cium and phosphorus. An elevated intact PTH would,
in the setting of hypercalcemia, confirm diagnosis of
hyperparathyroidism, which would not present with
rickets as described here.

7. Which of the following is not a risk factor for osteo- 7. b. Large for gestational age and/or infant of diabetic
penia of prematurity? mother are not risk factors for osteopenia of prema-
a. Postnatal furosemide exposure for CHD turity, though they may be associated with other
b. Large for gestational age due to maternal diabetes disorders of bone mineral homeostasis such as early
c. History of necrotizing enterocolitis postnatal hypocalcemia.
d. Birth weight of 1250 grams
8. Which of the following is true regarding the clinical
management of osteopenia of prematurity? 8. c. The amount of calcium/phosphorus that can be
a. Duration of TPN should be extended because this provided in TPN is limited; transition to full enteral
provides more phosphorus for bone mineralization feeds should be encouraged as soon as feasible. DXA
compared to enteral feeds. scans are capable of assessing bone mineral density,
b. DXA scans should be obtained every 4 weeks to but there is limited data in neonates so they are not
screen for osteopenia in infants born <26 WGA. currently a recommended procedure in the monitor-
c. The goal calcium intake from enteral feeds in infants ing of osteopenia of prematurity. Premature infants
at risk of osteopenia of prematurity should be 150– should be supplemented with 200–400 IU of vitamin
200 mg/kg/day. D, and increased as needed for confirmed vitamin D
d. All infants with birth weights <1500 grams should deficiency.
be supplemented with 1000 IU of vitamin D daily.

9. A 2-week-old, term male infant is re-admitted to the


NICU with weight loss and dehydration. Labs are 9. b. Based upon finding of elevated calcium and 1,25-
notable for serum calcium of 13.5, undetectable PTH, OH vitamin D with appropriately suppressed PTH, the
25-OH vitamin D of 32 ng/mL, 1,25-OH vitamin D 175 patient most likely has infantile hypercalcemia due to
pg/mL (normal range 31–87). He has been breastfed 24-hydroxylase mutation. This condition is associated
and is receiving 400 IU of vitamin D3 daily. Which of with hypercalciuria and risk of nephrocalcinosis and
the following statements about the patient’s underly- kidney stones in late childhood/early adulthood. Vita-
ing condition is false? min D intoxication would be associated with elevated
a. He will be at increased risk of kidney stones in young 25-OH vitamin D (typically above 70–100 ng/mL).
adulthood. Low calcium formulas can be helpful in managing this
b. Initial therapy should include administration of condition; he should also avoid vitamin D supplemen-
normal saline at 1.5- to 2-times maintenance. tation and sun exposure.
c. His condition is iatrogenic due to excess vitamin D
supplementation.
d. A low calcium formula may be needed to manage
his condition during infancy.
10. A 12-month-old, former 26 WGA female infant with 10. b. The patient most likely has nutritional hypophospha-
short gut maintained on elemental formula for the temic rickets related to exposure to elemental formula.
past 6 months is found on AM rounds to have acute Labs will show low serum phosphorus, undetectable
onset swelling and decreased movement of left femur. urine phosphorus, elevated alkaline phosphatase.
Radiograph confirms a mid-shaft femur fracture and Treatment includes oral phosphate supplementation
cupping and fraying of the distal femoral and proximal and requires close laboratory monitoring for the devel-
tibia metaphysis. Which of the following is true regard- opment of hypocalcemia as a result of exuberant gas-
ing the patient’s underlying diagnosis? trointestinal phosphorus absorption. This condition is
a. Urine labs will show elevated phosphorus self-limited and will resolve with appropriate phospho-
b. Hypocalcemia is a potential side effect of the treat- rus supplements.
ment for her condition
c. Her condition is expected to be lifelong
d. Alkaline phosphatase will be low
Thermoregulation

1. Which of the following hormones is the primary driver 1. c. Norepinephrine is the key counterregulatory hor-
of the acute physiologic response to cold stress? mone released in response to acute cold stress. Actions
a. Cortisol include peripheral vasoconstriction to reduce heat loss
b. Thyroid hormone and stimulation of brown adipose tissue for thermogen-
c. Norepinephrine esis. Thyroid hormone secretion ± cortisol secretion
d. Growth hormone play a role in the long-term adaption to cold, but are
not thought to play a crucial role in the acute response
to cold stress. Growth hormone is not known to have a
regulatory role in the response to short- or long-term
cold exposure.

Which of the following statements is true regarding


2. brown adipose tissue? -2. a. Free fatty acids, released via lipolysis, form the sub-
a. Fatty acids are the substrates for thermogenesis. strate for thermogenesis by brown adipose tissue. Ther-
b. Thermogenesis occurs in the nucleus of brown adi mogenesis occurs in the mitochondria of the brown
pose tissue. adipocyte and is “nonshivering” as heat is generated
c. Results in heat generation as a result of shivering. in the absence of muscle contraction. Brown adipose
d. Infants have less brown adipose tissue, in proportion tissue is more prominent in newborns than adults and
to body size, compared to adults. was once thought to be an embryonic remnant, but is
now recognized as being present throughout the life
span.
3. Which of the following is not a mechanism of heat loss in 3. c. Neonates can lose heat through conduct convec-
premature infants? tive, conductive, radiant, and evaporative heat loss.
a. Convective heat loss due to cooler air environment in Premature infants are at greater risk for convective,
delivery room compared to womb conductive, and radiant loss as a result of diminished
b. Evaporative heat loss due to thin epidermis subcutaneous fat that acts as an insulator and greater
c. Excessive nonshivering thermogenesis risk of evaporative heat loss because of thinner skin.
d. Conductive heat loss to resuscitation bed because of Nonshivering thermogenesis is a primary physiologic
decreased subcutaneous fat response to generate heat in response to cold stress.

4. Which of the following exposures is not a potential 4. a. Skin-to-skin contact with the mother is the optimal
source of heat loss to a neonate? heat source for stable infants immediately following
a. Mother’s skin delivery. Air and oxygen used for respiratory support
b. Inspired air via nasal cannula can be a source of both convective and evaporative
c. North-facing window in the neonatal intensive care heat loss; therefore, warmed or humidified air is recom-
unit (NICU) mended, if available. Windows or other cool objects can
d. Blanket be a source of radiant heat loss. Nonwarmed blankets
can be a source of conductive heat loss.
Immune System

1. An infant is successfully resuscitated following vaginal 1. b. Extremely low-gestational-age infants are highly
delivery at 23{0/7} weeks due to intrauterine infec- susceptible to infection for multiple reasons. Many
tion. You are concerned that this patient could quickly natural defenses, including skin barriers, ciliary clear-
develop overwhelming sepsis due to her extreme prema- ance, and gastric pH are naturally underdeveloped and
turity and exposure to infection. Which of the follow- also decreased due to medical interventions such as
ing immune-related factors does not contribute to this percutaneous catheters. Interferon-γ, a cytokine cen-
patient’s increased susceptibility to infection? tral to initiating and propagating many protective (and
a. An underdeveloped stratum corneum increases per- inflammatory) immune mechanisms, is suppressed in
meability of skin to pathogens. neonatal innate and adaptive immune cells. There are
b. Definitive hematopoiesis (bone marrow-derived) three pathways to complement activation, including
begins at 28 weeks’ gestation, so extremely prema- the classic (antibody-activated), lectin (lectin and IgA
ture infants rely on primitive hematopoietic immune activated), and alternative (spontaneous hydrolysis).
lineages for protection. Generally speaking, all three pathways have dimin-
c. There is impaired interferon-γ production by T cells ished activity in neonates compared to adults, and the
and monocytes in neonates. classic and lectin pathways, due to lower maternal
d. Preterm infants and full-term neonates have antibody transfer, are more affected in preterm infants
decreased lectin-mediated complement activation. born prior to 28 weeks’ gestation. Primitive hematopoi-
esis, which takes place in the fetal yolk sac, transitions
to definitive hematopoiesis by 5–8 weeks’ gestation.
1. In accordance with guidelines, a newborn screen was 1. b. Testing for SCID is currently included in the Recom-
carried out prior to the first blood transfusion in an mended Uniform Screening Panel of Core Conditions,
infant born at 23{0/7}weeks’ gestational age. The compiled by the Secretary of the Department of Health
screening test is positive for severe combined immuno- and Human Services. The SCID screen is considered
deficiency (SCID), showing low, but not absent, T cell positive when the TREC content falls below a threshold
receptor excision circles (TRECs). The most appropriate value that varies by laboratory. T cell receptor excision
next step for this patient is which of the following? circles are circular episomal DNA fragments present
a. Urgent hematology and oncology consult to assess in recent thymic emigrants that are formed during T
for bone marrow transplantation candidacy cell receptor rearrangement. Cell TREC content is nor-
b. Newborn screen repeated until patient reaches mally diluted through cell division, but overall content
term-equivalent gestational age in the blood is kept relatively steady in the healthy term
c. Antimicrobial prophylaxis started with sulfa- newborn by her or his high rate of new T cell release.
methoxazole-trimethoprim (Bactrim) Preterm infants have a lower TREC content develop-
d. Blood specimen sent to laboratory for measurement mentally, but due to the many comorbid conditions that
of lymphocyte subsets by flow cytometry can blunt their thymic function, there are no published

2. Vaccines against Haemophilus influenzae type b and 2. a. Conjugate vaccines contain a linked polysaccharide-pro-
Pneumococcus infections are highly successful in reduc- tein antigen and are tailored to improve the B cell response
ing the incidence of invasive disease. The vaccine plat- to polysaccharide (encapsulated bacteria) antigens. A
form against these pathogens is well-tailored for the major source of protection against encapsulated bacteria is
neonatal immune system in that: antibody generated by B cells in a T-independent fashion.
a. The polysaccharide antigen is T-independent, and T-independent responses bypass normal antigen process-
the protein toxin enables T cell help in inducing B cell ing and presentation by antigen-presenting cells (APC’s)
memory differentiation and antibody class switching. and interact directly with B cell receptors. Splenic marginal
b. They contain monophosphoryl lipid A adjuvant, zone B cells (MZBs) are highly active against T-indepen-
which boosts a newborn’s naïve adaptive response.
c. The antigens are recombinant proteins, which more
specifically target antigen-specific naïve T cells and
therefore generate more effective memory.
d. The polysaccharide antigen is conjugated to a DNA
fragment, which provides costimulation of antigen-
specific T cells through Toll-like receptors.
3. a. IgG is actively transported from the mother to the
3. A term newborn’s antibody concentration at birth is fetus via IgG-specific receptors that are expressed on
which of the following? the placenta during the late second trimester. By term
a. Higher in IgG compared to maternal circulation due gestation, the neonate’s IgG concentration is higher
to active transport across the placenta than the mother’s. IgA is produced mainly at mucosal
b. High in IgA due to mucosal immune responses surfaces following expos1. ure to commensal microbes,
c. High in maternal IgM in the setting of maternal which does not occur to a substantial degree until after
infection with Toxoplasmosis delivery. IgM forms pentamers, which cannot cross the
d. Similar in IgE due to passive transport across the placenta, and there are no IgM-specific receptors to
placenta facilitate transport. IgE also does not cross the placenta
but may be similar between mother and fetus due to a
genetic predisposition to allergy.

Abnormal Immune System Development

1. You are asked to evaluate a term infant born 4 hours


ago by scheduled cesarean section for persistent central
cyanosis. The infant is male, appropriate weight for
gestational age, and appears in no distress. Vital signs
are within normal limits, with the exception of oxygen
saturation,
which measures 81% preductal and postductal.
Your examination reveals a hyperdynamic precordium
with a single S2 and a regurgitant murmur.
You obtain an x-ray, which shows dextroposition of the
heart and a midline liver and stomach. You determine
that once stable, this infant may ultimately require antimicrobial
prophylaxis with which of the following?
a. Bactrim due to high risk for Pneumocystis pneumonia
b. IVIG due to low antibody levels
c. Amoxicillin due to high risk for infection with encapsulated
bacteria
d. FFP to restore complement levels
1. c. This patient presents with heterotaxy syndrome,
including inappropriate cardiac and abdominal situs.
The midline liver and stomach suggests that there is a
right-sided isomerism, which is associated with asple-
nia or hyposplenia. Splenic anatomy can be evaluated
by ultrasound, and diminished function can be assessed
by detection of Howell-Jolly bodies or pitted erythro-
cytes or by nuclear imaging modalities. Patients with
asplenia are at high risk for infection with encapsu-
lated bacteria, such as Klebsiella pneumonia, Haemophi-
lus influenzae type b, and Streptococcus pneumoniae. The
spleen is enriched with marginal zone B cells, which
rapidly produce neutralizing IgM antibodies against
polysaccharides. The spleen also plays a central role in
generating memory B cells against T-independent anti-
gens, such as polysaccharides and haptens.
2. A term infant is admitted to the NICU for evaluation and
management of her prenatally diagnosed pulmonary
stenosis. On examination, she is found to have hypertelorism,
hooded lids, a short philtrum, and micrognathia.
Her oral examination reveals a bifid uvula. Her
postnatal echocardiogram shows anatomy consistent
with tetralogy of Fallot, and there is adequate flow
through the outflow tract, without prostaglandins.
Her vital signs remain stable on the monitors. Electrolytes
remain within normal limits, with the exception
of a low ionized calcium level detected on day of life 4.
The tests most likely to reveal a contraindication to live
vaccine
administration in this patient include which of
the following?
a. Lymphocyte subsets by flow cytometry
b. Complement levels
c. Complete blood count with differential
d. Immunoglobulin levels

2. a. This infant’s presentation (conotruncal cardiac


anomaly, craniofacial findings, and hypocalcemia)
is most consistent with 22q11 deletion syndrome
(22q11DS), which affects pharyngeal pouch-derived
structures. Patients with complete 22q11DS syndrome
may suffer from athymia. The thymus is essential for
common lymphoid progenitors to differentiate into
naïve T cells, and athymic newborns will have low
or absent naïve T cells in their circulation. Live vac-
cines, therefore, are contraindicated in athymic indi-
viduals who have not received immune replacement
therapy. Enumeration of naïve T cell subsets is best
accomplished by flow cytometry. A complete blood
count with differential quantifies lymphocytes, which
combines T cells, B cells, and NK cells. T cells may be
diminished, even when the absolute lymphocyte count
falls within the normal range. Furthermore, maternal
memory T cell engraftment can occur in the absence
of an endogenous T cell population in the newborn
and will falsely elevate the neonate’s absolute lympho-
cyte count on a CBC. Flow cytometry can be used to
differentiate memory and naïve T cell subsets and can
therefore distinguish maternal from fetal T cells. Older
athymic patients often have diminished immunoglob-
ulin levels, but immunoglobulin in a term neonate is
largely derived by placental transfer and is likely to be
normal at birth. Complement levels are typically nor-
3. A term infant presents to the special care nursery with
a 24-hour total serum bilirubin of 13 (11.9 indirect,
1.1 direct). The mother’s blood type is O, Rh+. The
infant’s blood type is B, Rh+, and her hematocrit and
reticulocyte counts are 38 and 9.2%, respectively.
In this setting, the likely mechanism of intravenous
immunoglobulin’s (IVIG) therapeutic effect is which of
the following?
a. IVIG inhibits NK cell activation.
b. IVIG saturates B-antigen sites on the erythrocytes,
thereby blocking maternal antibodies from attaching.
c. IVIG binds to Fc portion of anti-B maternal antibodies,
rendering bound erythrocytes “invisible” to the
infant’s immune system.
d. IVIG saturates Fc receptor on infant’s phagocytes,
thereby blocking recognition and destruction of
antibody-
bound erythrocytes.

3. d. IVIG is pooled donor, concentrated human immuno-


globulin. IVIG contains mostly IgG, but some IgA and
IgM is present. IVIG modulates immune system activ-
ity through multiple mechanisms, including inhibitory
complement activation, inhibition of cytotoxic T cells,
and inhibition of phagocytosis. The antibody structure
includes a variable portion, which binds to its specific
antigen. Once bound, the constant portion (Fc) is avail-
able to bind to Fc receptors on phagocytes, such as splenic
macrophages, and signals internalization and destruc-
tion of bound erythrocytes. IVIG neutralizes macrophage
activity by saturating their Fc receptors, which then

4. A healthy-appearing term infant presents with a newborn


screen that is positive for severe combined immunodeficiency
(SCID). The laboratory report details that the
T cell receptor excision circle content is “absent.” Lymphocyte
subsets by flow cytometry reveal a T-negative,
B-positive, NK-positive SCID phenotype. The most likely
genetic defect explaining this patient’s T cell deficiency is
which of the following?
a. Absent IL-7 ÿ-receptor expression, leading to T cell
maturation defects
b. RAG1 deficiency (Omenn syndrome), causing defective
T cell receptor production
c. Low CD40L expression, causing T cell cycle arrest
d. Low adenosine deaminase (ADA) expression, leading
to accelerated apoptosis of T cells
4. a. The lymphocytes affected (T, B, NK) for a given SCID
syndrome are dependent on the differential effects
that molecular pathways have on the development of
each lymphocyte subset. IL-7 signaling is critical dur-
ing thymic T cell development and, in its absence, T
cells fail to mature. IL-7 is not an essential cytokine
for either NK or B cell development, and therefore B
and NK cells are unaffected in IL-7rα deficiency. The
RAG complex is necessary for V(D)J recombination,
which occurs in both T and B cells, and a deficiency
therefore leads to failed T and B cell development. T
cell CD40L binds to CD40 on B cells and signals anti-
body class switching and affinity maturation. Absent
expression causes X-linked hyper IgM, or failure to
produce mature, high-affinity antibodies. T, B, and
NK cell numbers are intact in CD40L deficiency. ADA
is involved in purine salvage, and intracellular accu-
mulation of purines in ADA SCID causes apoptosis of
5. A 26-week-old infant presents at birth in shock from
E. coli sepsis and is found to have an absolute neutrophil
count of 500/mL, which falls to 200/mL on a subsequent
CBC. Which of the following is most accurate regarding
medical therapy for sepsis-induced neutropenia?
a. Dexamethasone increases the absolute neutrophil
count by stimulating bone marrow production.
b. IVIG improves sepsis outcomes by blocking complement-
induced cytolysis and neutrophil consumption.
c. Granulocyte-colony stimulating factor (G-CSF)
increases bone marrow monocyte and neutrophil
production.
d. G-CSF will improve neutrophil count by stimulating
bone marrow neutrophil production.

5. d. G-CSF is a growth factor that selectively stimu-


lates neutrophil production in the bone marrow and
improves survival in preterm infants with sepsis-
induced neutropenia. Granulocyte-macrophage col-
ony-stimulating factor (GM-CSF) is a growth factor
for multiple hematopoietic cell lineages, including
both neutrophils and macrophages. Dexamethasone
inhibits the adhesion and margination of neutrophils,
which increases the number of measured circulating
neutrophils in a blood sample without increasing the
total body number. IVIG can dampen complement-
induced inflammatory pathways, but does not directly
improve neutrophil number. IVIG has not been shown
to improve survival in neonatal bacterial sepsis.
6. A term infant is admitted to the NICU for tachypnea and
increased work of breathing after delivery complicated
by preeclampsia. On physical examination, he is found
to have a narrow chest, but otherwise appears normal.
A chest radiograph reveals a narrow chest, increased
vascular markings, and fluid in the minor fissure. Some
long bones also have metaphyseal sclerosis. A CBC
with differential shows an absolute neutrophil count of
500 cells/mL, absolute lymphocyte count of 700/mL,
hemoglobin of 9.4 g/dL, and platelets of 110 × 103/mL.
His blood cultures remain negative, the absolute neutrophil
count (ANC) improves to >1500 cells/mL, his
respiratory symptoms resolve, and he is discharged to
home. He is seen repeatedly at the pediatrician for foulsmelling,
oily stools, recurrent respiratory infections,
and persistent growth failure. His CBC again shows
anemia
(hemoglobin, Hgb, 7.6 g/dL) and neutropenia
(400/mL). This patient’s most likely diagnosis is which
of the following?
a. Cystic fibrosis
b. Asphyxiating thoracic dystrophy
c. Shwachman-Diamond syndrome
d. Hemophagocytic lymphohistiocytosis

6. c. This patient presents with pancytopenia, a pro-


nounced neutropenia, skeletal abnormalities, and
signs of pancreatic insufficiency, consistent with a
diagnosis of Shwachman-Diamond syndrome (SDS).
The bone marrow failure of SDS places patients at risk
for life-threatening infections and blood cell dyscra-
sias and may require bone marrow transplantation for
treatment. Cystic fibrosis is the most common cause of
pancreatic insufficiency in children, but does not typi-
cally present with skeletal abnormalities. Asphyxiating
thoracic dystrophy presents with respiratory failure
due to poor development of the thoracic ribs, but does
not include pancreatic insufficiency or cytopenias.
Hemophagocytic lymphohistiocytosis (HLH) can also
present with cytopenias, but is usually associated with
immune system activation, including hepatomegaly,
lymphadenopathy, rash and fever. HLH is not associ-
ated with skeletal anomalies.
Infections of Organ Systems

1. What are the major causes of neonatal early-onset 1. a.


sepsis?
a. Group B beta-hemolytic streptococci, Escherichia
coli, & Listeria monocytogenes
b. Candida albicans, coagulase-negative staphylococci
& Staphylococcus aureus
c. Enteroviruses and herpes simplex viruses

2. What are the major causes of neonatal late-onset sepsis? 2. d.


a. Group B beta-hemolytic streptococci
b. Escherichia coli, & Listeria monocytogenes
c. coagulase-negative staphylococci & group D entero-
cocci
d. all of the above

Prevention of Infections and Immunization

1. Intrapartum antibiotic prophylaxis has significantly 1. c.


reduced the incidence of which of the following infec-
tions in the U.S.?
a. Late-onset sepsis from group B streptococci (GBS)
b. Both early-onset and late-onset sepsis from GBS
c. Early-onset sepsis from GBS
d. Syphilis
Dr.Wahid Helmi-Egypt

Dr.Wahid Helmi Your text here 1

Consultant Pediatrician
Zarka-Dymiate – Egypt
Neonatal x-ray (Revision)
REFERENCES
1 . Bell MJ , Ternberg JL , Fagin RD et al . Neonatal necrotizing enterocolitis. Therapeutic decisions
based upon clinical staging . Ann Surg 1978; 187 : 1 – 7
2 . Blakely ML , Tyson JE , Lally KP . Laparotomy versus peritoneal drainage for necrotizing enterocolitis
or isolated intestinal perforation in extremely low birth weight infants: outcomes through
18 months adjusted age . Pediatrics 2006; 117 : e680 – e687
3 . Moss RL , Dimmitt RA , Barnhart DC et al . Laparotomy versus peritoneal drainage for necrotizing
enterocolitis and perforation . N Engl J Med 2006; 354 : 2225 – 34
4 . Shohat M , Levy G . Transient tachypnoea of the newborn and asthma . Arch Dis Child
1989 ; 64 : 277 – 79
Dr.Wahid Helmi-Egypt

1. A term baby is noted to be grunting shortly after


birth. The baby is tachypnoeic and there is marked
recession. The following chest X-ray has been obtained.

a. Describe the abnormalities on the X-ray.


b . W hat is the diagnosis?
c . W hat is the management of the baby?

.1 . a. i. The heart is displaced to the left of the chest.


ii. There are bowel loops visible in the chest cavity on the right.

b. . Right-sided diaphragmatic hernia. These are much less


common than left-sided diaphragmatic hernias. The baby is not
ventilated (no endotra-cheal tube visible on CXR) and this
therefore suggests that this defect was not recognised antenatally.
The optimal management for these babies is intubation and
ventilation immediately after birth avoiding lung infl a-tion using
a mask.

c. This baby should be ventilated to prevent the bowel from


distending any more with swallowed air. A paediatric surgical
opinion should be sought
Dr.Wahid Helmi-Egypt

2 . A 28 week baby is 10 days old. Abdominal distension has


been noted and the following X-ray is obtained.

a . D escribe the abnormalities on the X-ray.


b . W hat is the diagnosis?

2. . a. i. There is a nasogastric tube in situ.


i. . The bowel loops are dilated.
ii. . The bowel wall is thickened. iv. There is no air in the
rectum. v. There is widespread intramural gas.

b. The diagnosis is necrotising enterocolitis. There are staging


criteria for NEC described by Bell. The features described in the
above X-ray are consistent with Bell stage 2 NEC.
Dr.Wahid Helmi-Egypt

3. The same baby as in question 2 is reviewed a few hours


later. His abdomen has become more distended and
discoloured. The following abdominal X-ray is obtained.

a. Describe the abnormalities on the X-ray.


b . W hat is the diagnosis?
c . W hat is the management of this baby?

3
. . a. The lateral abdominal X-ray shows free air. There are
loops of bowel extending into the gas fi lled area.

b. . The baby has a perforation. This would increase the


stage of NEC to Bell stage 3.

c. . The baby should be reviewed by a paediatric surgeon. If


ventilation is compromised a drain should be inserted
through the abdominal wall to release the pressure from
build up of free gas in the abdomen. There have been
recent studies to compare drain insertion with laparotomy
for NEC.
These have not demonstrated superiority of one technique
over the other, but have confi rmed the poor outlook of
babies requiring surgery for perforated NEC.
Dr.Wahid Helmi-Egypt

.4 A 37 week gestation baby is noted to have frequent small


vomits. At 24 hours of age he is noted to be tachypnoeic and
he is admitted to the neonatal unit. A chest X-ray is obtained.

a. Describe the abnormalities on the X-ray.


b. What are the diagnoses?
c. What is the management of the baby?

4
. . a. i. There is a large patch of opacifi cation in the mid
zone on the right side of the chest. There is some patchy
shadowing elsewhere.
.i . There is a nasogastric tube in place.
.i . The nasogastric tube is coiled in a pouch in the thorax,
behind the heart.

b
. . This baby has a sliding para-oesophageal (hiatus) hernia.
The prevalence in newborns is unknown but it is thought to
be relatively uncommon. It can be discovered on a routine
-ray but usually presents with features of gastro-
oesophageal refl ux. In this case the changes in the lung
would be compatible with aspiration in association with
reflux.

.c . No interventions are needed if the child is


asymptomatic. H owever, in this case it would be
appropriate to commence a ntibiotics and consider anti-refl
ux treatment if vomiting
remains problematic.
Dr.Wahid Helmi-Egypt

5 . A 36 week baby is admitted to the neonatal unit after a


choking episode. The following chest X-ray is obtained.

a. Describe the abnormalities on the X-ray.


b . W hat is the diagnosis?
c . W hat is the management of the baby?

5
. . a. With the exception of chest leads and a nasogastric
tube there are no
abnormalities on this X-ray. It is normal.

b. . Normal.

c. . The chest X-ray does not reveal anything that needs


treatment.
Dr.Wahid Helmi-Egypt

6
. A 27 week infant is ventilated for RDS. Ventilatory
requirements have increased over the last hour. A chest
X-ray is obtained.

a . D escribe the abnormalities on the X-ray.


b . W hat is the management of the baby?

.6 . a. i. The endotracheal tube is down the right main


bronchus. ii. The left lung is inadequately aerated.
iii. There is a nasogastric tube in situ.

b. The endotracheal tube should be withdrawn and the correct


position confi rmed with a repeat chest X-ray.
Dr.Wahid Helmi-Egypt

7 . A term baby is noted to have a degree of frontal


bossing, as does his mother. A chest X-ray is
performed.

a . Describe the abnormalities on the X-ray.


b . W hat is the diagnosis?
c . W hat is the inheritance of this condition?

7. . a. The clavicles are absent.

b
. . Cleidocranial dysostosis. In this condition, the clavicle
is either hypoplastic or absent and the ribs are short. The
anterior fontanelle often closes late and there may be
delayed eruption of teeth. There can be bossing of the
forehead.

.c . Autosomal dominant. It results from a mutation in


the C BFA1 gene,which controls a key transcription
factor in osteoblast differentiation.
Dr.Wahid Helmi-Egypt

8. A 29 week gestation baby has been on CPAP for


moderate respiratory distress and has been stable in
35% oxygen. He is active and has been noted to have
quite marked intercostal recession on occasion. He
suddenly deteriorates with
persistent recession and an increase in oxygen
requirements to 95%. A chest X-ray is taken.

a. Describe the abnormalities on the X-ray.


b . W hat is the management of the baby?

8
. . a. i. Large tension pneumothorax on the left.
.i . Mediastinal shift to the right with tracheal deviation.
.i . Transcutaneous oxygen electrode on left upper
chest. iv. Surprisingly the infant is not intubated.

b. . i. Immediate drainage of the pneumothorax.


.i . Intubation and ventilation is very likely to be needed.
Dr.Wahid Helmi-Egypt

9 . A 27 week gestation infant is ventilated for respiratory distress.


Ventilation requirements have been moderately high and two doses
of surfactant have been given. Oxygen requirements have steadily
risen from 55% to 95%. In response to a profound bradycardia and
desaturation he receives ventilation down his
endotracheal tube using a resuscitation bag. He deteriorates further
and a chest X-ray is obtained.

a . D escribe the abnormalities on the X-ray.


b . W hat is the management of the baby?
9. . a. i. Intubated
infant. ii. Right
pneumothorax.
.i . Pneumopericardium.
iv. . Umbilical arterial catheter.
v. . Bilateral hyperinfl ation of the chest.
vi. i. There is abnormal shadowing at the right base which is diffi cult to
assess without other X-rays. This turned out to be a cystic adenomatoid
malformation.

b. The pneumothorax should be drained. The decision to drain the


p neumopericardium will be dictated by the clinical condition of the infant,
specifi cally with respect to cardiac function.
Dr.Wahid Helmi-Egypt

1
0
. A 31 week gestation infant is admitted to the intensive
care unit. Mother booked late and there has been minimal
antenatal care. The baby needs resuscitation at delivery
and is transferred to the unit ventilated. Ventilation proves
very difficult and a chest X-ray is obtained.

a . D escribe the abnormalities on the X-ray.


b . W hat is the differential diagnosis?
c . W hat is the management of the baby?

.0
1 0. a. i. Intubated
infant. ii. High UAC.
iii. Large cystic mass in right lung with a septum running across the
middle.

.b . These appearance would be consistent with a large cystic


adenomatoid malformation (which was the case in this infant) or
lobar emphysema. The latter is unlikely as this condition usually
develops after birth as opposed to presenting with problems at birth.
There is debate about the use of the word ‘congenital’ as some
authorities maintain that the lobar emphysema develops as a result
of airway problems that develop postnatally.

.c . The management of these conditions is extremely diffi cult.


Close co-operation with paediatric surgeons is essential. Direct
puncture
has a high chance of leading to formation of a broncho-pleural fi
stula. The infant may be too sick to tolerate thoracotomy, yet
respiratory compromise may become so severe that intervention is
necessary.
Dr.Wahid Helmi-Egypt

1. A 32 week gestation infant is stable at birth but


aproblem is noted at admission and a chest X-ray
obtained.

a . D escribe the abnormalities on the X-ray.


b . W hat is the management of the baby?

1
. 1. a. There is a large bore (Replogle) nasogastric tube,
partially coiled in the upper oesophagus. There is a shadow
of air in the upper oesophagus but none lower down,
suggesting an oesophageal atresia. There is evidence of
gas in the gastrointestinal tract, suggesting a tracheo-
oesophageal fi stula.

b. Management is complex and requires close collaboration


with paediatric surgeons. If the infant is mature and clinically
stable, surgery should be considered in the relatively near
future. If the infant is very sick or immature, then a decision
may be made to defer surgery until a more stable state is
achieved, or the baby has grown.
Dr.Wahid Helmi-Egypt

.2
1 A term baby is thought to have an absent Moro refl ex and
limited movementc of the arm. An X-ray is taken. In view of these
changes a chest X-ray is then performed.

a. Describe the abnormalities on the arm X-ray.


b . D escribe the abnormalities on the chest X-ray.
c . W hat is the most likely diagnosis?
d . W hat management would you consider?
1
2
. 2. a. i. The X-ray shows a fracture of the left humerus.
ii. There is marked osteopenia of all bones, especially marked in the bones of
the forearm.
b. . i. Marked rotation of the film.
i. . There is thinning of the ribs with what are almost certainly fractures
with callous formation.
i. . Osteopenia of all bones.
c. . Osteogenesis imperfecta.

d. . Pamidronate infusions have been used to improve bone density and


reduce the risk of further fractures. Such management should only be
undertaken in
specialist centres as general experience of this management is extremely
limited.
Dr.Wahid Helmi-Egypt

1
3
. A 27 week gestation infant is now 4 weeks old. She needs 0.4 L/min
supplementary oxygen but this has recently risen to 0.8 L/min. A chest X-
ray
is performed.
a . D escribe the abnormalities on the X-ray.
b . W hat management has this baby had in the past that is clearly evident
on
the X-ray?
c . W hat management will you initiate in response to the abnormality on
this
chest X-ray?

1
3
. 3. a. i. Clip in left upper chest.
.i . Abnormal separation of left 4th and 5th ribs.
.i . Widespread patchy opacifi cation of both
lungs. iv. Large bulla in right lung base.
v. Nasogastric tube in situ.

b
. . The PDA has been clipped.

.c . None. This infant is not intubated and ventilated and


unless there are signs of severe respiratory distress, no
action is needed. The infant has what appears to be quite
severe chronic lung disease and would not benefit from re-
ventilation. Although the bulla looks fairly large on X-ray,it is
only occupying a relatively small proportion of the intra-
thoracic space. The majority resolve spontaneously over
time, as was the case with this infant.
Dr.Wahid Helmi-Egypt

1
4
. Antenatal ultrasounds have suggested a growth
retarded fetus. At birth, the baby has obvious limb
abnormalities and requires ventilation. Limb and chest X-
rays are performed.

a. Describe the abnormalities on the limb X-ray.


b . D escribe the abnormalities on the chest X-ray.
c . W hat is the most likely diagnosis?
d . W hat is the prognosis?
.4
1 4. a. i. Short long bones (compare to size of cord clamp).
ii. Marked bowing of bones, particularly noticeable in femurs.
b. . The chest is very narrow with abnormal ribs that are short and thin
with splayed ends. The vertebral bodies are fl attened with wide
intervertebral spaces.
c. . The appearance is that of a congenital skeletal dysplasia. In
this case the most likely diagnosis is thanatophoric dysplasia as
indicated by the extremely short limbs with classic ‘telephone
receiver ’ shaped femurs.
d. . Thanatophoric dysplasia is invariably fatal, usually shortly
after birth.
Dr.Wahid Helmi-Egypt

15. A baby is born at term and is noted to be cyanosed at rest.


There is moderate recession. There is bilateral lower limb
oedema. Femoral pulses are weak. The anterior fontanelle is
bulging and a loud bruit can be heard when listening over it.A
chest X-ray is performed.
a. What abnormalities are there?
b . W hat possible explanation do you have?
c . W hat initial further investigations would you consider?

1
5
. 5. a. The heart is grossly enlarged in all dimensions.

b. . The enlarged heart does not in itself point to a specifi


c diagnosis and conditions such as a cardiomyopathy,
structural heart defect, storage disorder, maternal diabetes
and heart failure are all possibilities. In this case the
combination of a large heart, peripheral oedema and a bruit
over the bulging anterior fontanelle should raise the
possibility of an intracranial arteriovenous malformation
resulting in high output cardiac failure
(as was the case in this baby).

c. . i. Echocardiogram.
.i . Cranial ultrasound, CT scan or MRI.
.i . Four limb BP and oxygen saturations.
vi . . Check maternal history and antenatal ultrasounds.
Dr.Wahid Helmi-Egypt

1 6. A baby is born at 26 weeks and requires ventilation from


birth. Surfactant was given on delivery suite. Ventilation has
steadily increased and the baby is in 95% oxygen at pressures
of 28/6 at 24 hours of age. Blood gases are poor. A chest X-ray
is performed.
a . W hat abnormalities are there?
b . W hat is the most likely diagnosis?
c . W hat treatment would you consider?

.6
1 6. a. i. There is widespread opacifi cation throughout both
lung fields.
ii. There are clear air bronchograms on both sides.
iii. The heart border is not clearly defi ned.
iv. . The costophrenic and cardiophrenic angles are not
clearly visualised.
v. . There is an endotracheal tube.
vi. i. There is a central venous line entering by the left arm and with
a tip at the junction of the SVC and right atrium.

.b . Respiratory distress syndrome is the most likely diagnosis.

.c . Surfactant should be given if the dose has not been


repeated since birth.Ventilatory requirements are high and
some centres
would consider high frequency oscillatory ventilation at this point. If
this is not available ventilation will probably need to be adjusted to
improve the blood gases.
Dr.Wahid Helmi-Egypt

1
7
. A term infant is born following a severe asphyxial insult. There is thick
meconium present and the oro-pharynx and trachea are suctioned under
direct vision and meconium is removed. Ventilation is required and
requirements steadily increase. At 8 hours of age the pressures are 32/4
and 100% oxygen is required. The blood gas shows a pH of 7.01, PO 2 of
3.4 and PCO 2 of 7.2.
Figure 7.17
a. What does the X-ray show?
b. What is the diagnosis and what complication may be present?
c. What action could be taken?

17
. 7. a. i. There is patchy opacifi cation that is most marked in the perihilar
region. ii. There are basal bullae, most apparent on the right.
iii. The lungs are relatively hyperexpanded, 11 posterior rib ends are showing.
iv. There is a transcutaneous oxygen electrode, chest leads and a
nasogastric tube.
v. There is an endotracheal tube.
b. . The history and chest X-ray appearances are typical of meconium
aspiration syndrome. It is very possible that a signifi cant degree of persistent
pulmonary
hypertension is contributing to the clinical picture.
c. . There is evidence that both high frequency ventilation and nitric oxide may
be useful in this condition, and are particularly useful in combination.In severe
cases extracorporeal membrane oxygenation may be life saving and referral
should be made early. There is some evidence from animal studies and limited
human data to support the use of surfactant,either by bolus administration of by
surfactant lavage.
Dr.Wahid Helmi-Egypt

18. A 36 week gestation infant has mild respiratory distress at birth.


He requires 30% oxygen and there is mild recession. As symptoms
persist an X-ray is performed at 6 hours of age.
a . W hat does the X-ray show?
b . I f this is the only problem with the baby, what is the most likely
diagnosis?
c . I f the baby is asymptomatic, what should you do?
d . I f the baby is symptomatic, what would you consider?
e . W hat other investigations would be warranted?

1
8
. 8. a. i. There is a nasogastric tube in situ.
.i . There is loss of the costophrenic angle on the right
and increased
shadowing in the lower peripheral right chest.

b
. . Right pleural effusion. Pleural effusions can occur for
many reasons. If diagnosed antenatally, they are associated
with chromosomal or congenital abnormalities. Congenital
viral infections can also cause effusions.Isolated effusions
can be a chylothorax or idiopathic.

.c . If asymptomatic and there are no other concerns it is


appropriate to simply observe the infant. Idiopathic effusions
may resolve while a chylothorax.
Dr.Wahid Helmi-Egypt

1 9. A term infant is born by elective caesarean section because


of previous caesarean sections. Shortly after birth the baby is
noted to be mildly dusky and grunting and there is mild
recession. A septic screen is performed and a chest X-ray
obtained. Antibiotics are started.
a. What abnormality is seen on this X-ray?
b . W hat is the most likely diagnosis?
c . W hat management steps do you need to take?
d . W hat is the prognosis for the baby?

.9
1 9. a. There is fl uid in the horizontal fi ssure and some peri-
hilar streaking.

b. The most likely diagnosis is transient tachypnoea of the newborn.


However due to the diffi culty in accurately diagnosing infection it is
common to consider this as a possible differential diagnosis.

.c . Close observation and respiratory support such as CPAP may


be n eeded
for a short while. Affected infants rarely require more than 40%
oxygen
or respiratory support for more than three days. Antibiotics should be
given until infection has been excluded.

.d . The prognosis is good. Most infants recover over 24 hours. There


is
debate regarding whether infants with TTN are more prone to
wheezing
in childhood.
Dr.Wahid Helmi-Egypt

.0
2 A preterm infant has chronic lung disease. Her condition worsens
and following investigations a surgical procedure is performed. The
following day a chest X-ray
is performed.
a . D escribe the X-ray.
b . W hat surgical procedure was performed?
c . W hat treatment may be indicated?

.0
2 0. a. i. There is an endotracheal tube in
situ. ii. There is a nasogastric tube in situ.
i. . Both lungs show infl ammatory and cystic change consistent
with bronchopulmonary dysplasia.
iv. . The lungs appear hyperinfl ated – note the bulging pleura on
the right.
v. . There is loss of the left costophrenic angle and obvious
outlining o f the lungs on the left side with fl uid.
vi. i. There is abnormal separation of the upper ribs on the left.
vi. i. There is a long line in situ in the left subclavian vein. The
position o f the tip seems satisfactory.
vi. i. There is marked oedema visible most obviously outside
the c hest.

.b . The separation of the ribs on the left side of the chest is highly
suggestive of a ductal ligation in the recent past. The chylothorax
which is present is a well recognised complication of the procedure.

c. . This infant has bronchopulmonary dysplasia and is still ventilated.


The treatment options available at this stage would be a short course
of diuretics or steroids. Diuretics can improve oxygenation and lung
compliance,but it is prudent to give a short course to assess for
response before embarking on long-term diuretic use due to the
high incidence of side effects.
Dr.Wahid Helmi-Egypt

.1
2 A 26 week infant is in 0.5 L /min of supplementary oxygen at
36 weeks corrected age. He has mild recession and capillary
gases show a fully compensated respiratory acidosis. A chest
X-ray is taken as part of a work-up for chronic lung disease.
a . Describe the X-ray.
b . W hat is the likely diagnosis?

.1
2 1. a. i. Generalised patchy opacification throughout the lung
fields. ii. Small volume lungs.
i. . Tracheal deviation to the right.
iv. . Rib fractures on the right with callus formation.
Dr.Wahid Helmi-Egypt

2
. A baby born at 24 weeks gestation is 2 weeks old. He has been stable
on low ventilation pressures but has not tolerated CPAP. Attempts have
been made to start nasogastric feeds on several occasions but he does
not appear to tolerate them as there are reasonable volume gastric
aspirates on most occasions.Over the last 48 hours the aspirates have
become increasingly bilious and an abdominal X-ray is taken.
a . D escribe the X-ray abnormalities.
b . W hat is the diagnosis?
c . W hat is this diagnosis commonly associated with?
d . W hat is your immediate management plan?
e . W hat is the likely outcome for this baby?

2. 2. a. There is a ‘double bubble ’ with a dilated stomach and


duodenum. b. Duodenal atresia.
c. . Additional problems may be Down syndrome, cardiac abnormalities and
malrotation.
d
. . A large-bore nasogastric tube must be inserted and left on free
drainage; electrolyte and blood gas anomalies must be corrected.
e
. . Isolated duodenal atresia can be successfully corrected by surgery
although experience at such early gestation is very limited. Other anomalies
such as cardiac defects (which are much commoner with duodenal atresia) will
affect the prognosis.
Dr.Wahid Helmi-Egypt

2
3
. A 26 week infant has been ventilated for RDS. He was weaned
onto CPAP after 16 days and was initially stable. Six days after
starting CPAP his oxygen requirements started to rise (from
35% to 65%) and he was commenced on a course of
dexamethasone; 36 hours after starting dexamethasone he
deteriorates substantially. An X-ray is taken.
a. What abnormalities are seen on this X-ray?
b. What is the diagnosis?
c. What is your management plan?

Your text here 1

23. 3. a. i. There is a naso-jejunal tube in situ.


i. . There is a very large translucency covering much of the abdomen.
This ‘football sign’ indicates free air in the peritoneal cavity.
ii. . There is a white line running to the right of and almost parallel to the
vertebral column. This is the falciform ligament and it only becomes
visible when there is free intraperitoneal gas.
iv. . The lung fields are patchy throughout.They are extremely low volume
yet appear hyperinflated with marked upward slanting of the ribs. It is
difficult to interpret how many of these changes are due to intrinsic
lung disease and how much to compression from abdominal
d istension.
b. There is obvious chronic lung disease and definite gastrointestinal perfo-
ration.The latter is quite possibly related to the administration of steroids.
c. Management will depend upon the severity of illness. If his condition has
become unstable and gases have deteriorated, ventilation will be
required. If the abdominal distension compromises ventilation an
abdominal drain may well be required. Surgical review and consideration
for a laparotomy will be needed early in the process.
Dr.Wahid Helmi-Egypt

24. A term baby becomes cyanosed 6 hours after birth. Increasing the inspired
oxygen concentration only partially treats this. On examination the chest is
clear and there are no abnormal sounds. A chest X-ray is obtained.
a. Describe the chest X-ray.
b. What is your diagnosis?
c. What will you do next?

24. 4. a. i. The heart shape is ovoid (egg-shaped).


i. . The upper mediastinum is
narrow. iii. The lung fields appear
normal.
b. Transposition of the great arteries.
c. The diagnosis should be confirmed by echocardiography. A
prostaglan- din infusion should be started and the case should be
discussed with the nearest paediatric cardiac centre as soon as
possible.
Dr.Wahid Helmi-Egypt
25.A 28 week gestation infant has been born. A chest and abdominal
X-ray is taken at 4 hours of age.
a. Describe the X-ray.
b. What is your diagnosis?
c. What will you do next?

25. 5. a. i. Both lung fields show a diffuse granular


appearance. ii. There are bilateral air bronchograms.
ii. . The end of the endotracheal tube is high.
iv. . The tip of the UAC appears to be somewhere close to the right
s ubclavian artery.
b. Respiratory distress with incorrectly placed UAC and ET tube.
c. i. The UAC will need to be withdrawn into a more appropriate
position.
i. . The endotracheal tube will need to be inserted further.
ii. . A nasogastric tube should be inserted and the intestines
decompressed.
iv. . Surfactant administration should be considered.
Dr.Wahid Helmi-Egypt
26. A term infant is born to a mother who is known to have insulin
dependent diabetes. He is grunting from birth and oxygen
saturations are poor. A chest X-ray is obtained.
a. Describe the X-ray.
b. What is your diagnosis?
c. What will you do next?

26. 6. a. i. The heart is much larger than


normal. i i. The lung fields appear well
aerated. i ii. Pleura bulging on both
sides.
iv. . UAC on the left – very high (probably – this cannot be confirmed to
be the UAC until the abdomen is X-rayed and the caudal loop is
s een).
v. . UVC on the right, very high (position confirmation needed as
above).
b. A diabetic cardiomyopathy is the most likely diagnosis. Although
s eptal hypertrophy is the best described association, biventricular
h ypertrophy may develop with significant and serious reduction in
stroke volume.
c. i. Reposition the lines.
i. . Echocardiography.
ii. . Seek expert advice if haemodynamically unstable.
Dr.Wahid Helmi-Egypt
27. A 32 week gestation infant has been born. There has been a 10-week
history of oligohydramnios. The baby has required ventilation from
birth. Blood gases are poor at high ventilation pressures. A chest X-
ray is taken at 3 hours of age.
a. Describe the X-ray.
b. What is your diagnosis?

27. 7. a.
i. The chest is small.
i. . The ribs are thin and slope downwards steeply.
ii. . There is a UVC about 1 cm above the
diaphragm. iv. The UAC tip is at T9.
v. The endotracheal tube tip is very high and is only just visible on
the X-ray.
b. Pulmonary hypoplasia secondary to oligohydramnios.
Dr.Wahid Helmi-Egypt
28. A term baby has not fed well since birth. Bilious vomiting has been
noted, as has mild abdominal distension. At 48 hours the
distension becomes much worse and an abdominal X-ray is taken.
a. What does the X-ray show?
b. What do you think has happened?
c. What will you do next?

28. a. i. There is free gas in the peritoneal cavity.


i. . There are dilated loops of bowel, several of which appear to have
very irregularly thickened walls.
ii. . The right diaphragm appears
thickened. iv. There is no gas in the
rectum.
b. There is intestinal perforation which has probably been present some
time to result in the degree of bowel wall thickening that is apparent.
The presence of irritant material is also probably responsible for thick-
ening of the diaphragm. It is not clear why the bowel has perforated but
the absence of rectal gas may be suggestive of an obstruction. In this
baby’s case there was colonic atresia.
c. The free gas will need drain insertion, unless a laparotomy is performed
immediately. Surgical review is essential and the baby should be
stabilised as dictated by clinical condition.
Dr.Wahid Helmi-Egypt
29. A 27 week gestation infant has had a central venous line inserted. A
chest X-ray is taken to confirm line position.
a. What does the X-ray show?
b. Is the line position acceptable?
c. What will you do next?

29. a. i. The film is extremely rotated and it is difficult to accurately place


anything in the picture.
ii. There is a line to the right of the vertebral column that could be a UAC,
but this cannot be stated with any certainty.
iii. There is an endotracheal tube which is slightly high.
iv. There is a central venous line coming in from the right arm. It is very
likely to be actually ending lateral to the chest.
b. The line position is almost certainly unsatisfactory.
c. The line should be removed. A further non-rotated film could be obtained b
ut the position of the line is likely to be more lateral if this is done.
Dr.Wahid Helmi-Egypt30. A 28 week gestation infant has been ventilated for 12 days. He is now
6 weeks old and requires 0.6 L/min of oxygen. A chest X-ray is
taken.
a. What does the X-ray show?
b. What action will you take?

30. a. i. The lungs show diffuse patchy opacities consistent with chronic
lung disease.
i. . The lungs are relatively small volume and hyperinflated, with upward
s loping ribs.
ii. . There are vertebral abnormalities.
b. No action is indicated although it is assumed that a detailed examination has
a lready been performed to exclude any other anomalies that may be associated with the
vertebral abnormalities.
Dr.Wahid Helmi-Egypt
A 25 week gestation infant has been ventilated since birth and is
31.
now 9 days old. A chest X-ray is taken.
a. What does the X-ray show?
b. What is responsible for these changes?

31.a. i. The lungs are hyperinflated.


1.
i. There is a nasogastric feeding tube in place.
ii. . There is what is probably a UAC, with the tip at around
T8. iv. There is an endotracheal tube.
. There is widespread patchy shadowing which appears consistent with
c ystic changes throughout both lung fields.
b. These apparently cystic changes are compatible with chronic lung dis- ease.
A lthough the diagnosis of CLD (or BPD) is dependent on a pro- longed oxygen requirement
changes may be present well before then and can be seen histologically in infants who have
died only a few days after birth. The distribution and pattern of the changes is different from
that expected with pulmonary interstitial emphysema, which also gives widespread cystic
changes.
Dr.Wahid Helmi-Egypt
32. A 26 week gestation infant has received one dose of surfactant in the delivery
room and a further dose 12 hours later. A chest X-ray is taken at 24 hours of age.

a. What does the X-ray show?


b. What do you think has happened?
c. What will you do next?

32. 2. a. i. There is an endotracheal tube.


i. . There is a nasogastric tube.
ii. . There is opacification of the left lung, consistent with RDS.
iv. . The right lung has cystic change throughout, consistent with pul- monary
interstitial emphysema.
v. . There is a large bulla at the base of the right lung.
b. . The presence of pulmonary interstitial emphysema in the right lung and what
appears to be quite severe RDS in the other lung suggests that the
endotracheal tube may have been in the right main bronchus at the time of
the first surfactant administration. As one lung is now well expanded and the
other solid, any further surfactant is likely to also go into the right lung.
.c Ventilatory management of this baby will be difficult.The left lung needs
aeration and thus recruitment of alveoli. The ideal way to do this is to use
higher ventilatory pressures to both recruit and retain alveolar expan- sion.
U nfortunately the right lung is showing signs of cystic change and over-
distension with a bulla in the right base, and high pressures will result in
further damage. High frequency may be useful and anecdotally it has been
s uggested that paralysing the baby and ventilating with the PIE side down
may minimise damage to the PIE side and facilitate infla- tion of the
atelectatic side.
Dr.Wahid Helmi-E
33gypbtaby is born at 36 weeks to a mother with insulin dependent diabetes.
.A
He does not tolerate feeds and abdominal distension is noted. A chest and
abdominal X-ray is taken at 36 hours of age.
a. What does the X-ray show?
b. What do you think has happened?
c. What will you do next?

3. 3. a. i. There is a nasogastric tube.


i. . There are loops of dilated bowel but no evidence of inflammatory
changes, thus making NEC unlikely.
ii. . There is no air in the rectum.
b. Either structural or functional bowel obstruction is likely. There is noth-
ing on the X-ray that allows differentiation between possible causes.
M aternal diabetes is associated with a small left colon in the newborn
baby. This results in a functional obstruction that usually resolves with
time (as it did with this infant).
c. A contrast study is probably indicated. Further investigations will depend
on the result of this. The baby should not be fed, intravenous fluids should
be given and a nasogastric tube should be left on free drainage.
Dr.Wahid Helmi-Egypt
34. A term infant shows minimal respiratory effort and requires
ventilation. A chest X-ray is taken. On talking to the parents you notice
that the mother shows little expression in her facial movements.

a. Describe the X-ray?


b. What do you think may be the explanation for these appearances?
c. What will you do?
34. 4. a. i. There is extremely poor lung inflation.
i. . The diaphragms are very high.
ii. . The ribs are thin, particularly on the posterior aspect.
b. Myotonic dystrophy.
c. The baby will need ventilating. There is normally some improvement
o ver time, although it may be a long time before breathing is strong
enough to allow extubation.
Dr.Wahid Helmi-Eg35y. pAt 25 week gestation infant has been ventilated since birth. Three doses of
surfactant have been given with little effect. A chest X-ray is taken at 48
hours of age.

What does the X-ray show?

35. 5. a. i. There is an endotracheal tube in place.


.i . There is a line with a tip at approximately T11. It is not clear
what this line is.
.i . There is an endotracheal tube at an appropriate position.
iv. . There is what appears to be a central venous line with the tip
below the sternal end of the left clavicle.
v. . Both lungs are very poorly aerated.
vi. i. There is a fracture of the left arm. The bones do not appear
particularly o steopenic and this is presumably traumatic in origin.
Dr.Wahid Helmi-Egypt
36.A 28 week gestation infant has developed moderate RDS and is
stable on CPAP. Abdominal distension is noted and an abdominal
X-ray is requested. By mistake a chest X-ray is taken instead.

a. What does the X-ray show?


b. What action will you take?

36. a.i.
There is a nasogastric tube with the end in the mid-thoracic o
esophagus.
ii. The lungs have diffuse patchy opacification consistent with
chronic lung disease.
b. The nasogastric tube needs to be repositioned and as much gas
removed from the intestines as possible.
Dr.Wahid Helmi-Egypt
37. A 24 week infant requires ventilation from birth and is now 4 weeks old.
He is known to have duodenal atresia but the surgeons do not want to
operate until he is a lot bigger. He has been to theatre for a surgical
procedure. A chest X-ray is taken to check that the procedure has been
performed correctly. The appearance of the right lung is as it has been
since birth.

a. What does the X-ray show?


b. What was the surgical procedure?
c. What is the explanation for the appearances of the right lung?
37. 7. a. i. There is an endotracheal tube and nasogastric tube.
i. . There is a fine-bore central venous catheter with the tip in the
tho- racic inlet.
ii. . There is a large-bore central venous catheter on the right with the
tip at the junction of the SVC and right atrium.
iv. . The right lung is opaque with no areas of aeration.
b. Surgical placement of a central venous catheter. The large-bore
catheter is a Broviac line.
c. There is aplasia/hypoplasia of the right lung. The fact that the lung
has always had this appearance and the infant is now 4 weeks old is
strongly against collapse or consolidation as the cause of this
appearance.
Dr.Wahid Helmi-Egypt
38. A 28 week gestation infant has been born and has needed relatively
little ventilatory support. Feeds are introduced on day 3 and
increased slowly. On day 5 he deteriorates and there is obvious
abdominal distension. An X-ray is obtained.

.a . What does the X-ray show?


b. . What do you think has
happened? c. What will you do
next?
38. 8. a. i. There is free gas above the liver in this lateral X-ray.
b. Gastrointestinal perforation.
c. Surgical referral. A drain may need to be inserted pending laparotomy.
Dr.Wahid Helmi-Egypt
39. A term infant is born in poor condition following an ante-partum
haemorrhage. The infant takes a few deep gasps and then stops breathing.
Heart rate is 57 bpm. Bag and mask ventilation is commenced. There is
poor chest movement and there is little response to ventilation.
Transillumination is performed and there is no difference between the two
sides. An endotracheal tube is inserted and ventilation does not improve.
A butterfly needle is inserted into the right chest. A few bubbles are seen
but this then stops. An X-ray is taken.
a. What does the X-ray show?
b. How do you explain the clinical story?
c. What will you do next?

39. 9. a. i. There are bilateral pneumothoraces.


i. . There is an endotracheal tube in position.
b. Pneumothoraces may have been present at birth, following the gasping
respirations or following initial ventilation. Bilateral pneumothoraces may
be difficult to diagnose because all signs are equal on both sides. Although
a needle drain may allow re-inflation the lung will expand onto the needle
which can easily become blocked.
c. Immediate insertion of bilateral chest drains.
Dr.Wahid Helmi-Egypt
40. A 36 week gestation infant has been born and is in poor condition. She is
very growth retarded and there is a widespread petechial rash. An X-ray
is taken at 6 hours of age.

.a . What does the X-ray show?


b. . How might these appearances link with the clinical
history? c. What treatment would you consider?
40. 0. a. i. There is an endotracheal tube, the tip of which is too
high. ii. There is a nasogastric tube.
ii. . There is patchy opacification of both lungs.
iv. . There is enlargement of both liver and spleen.
b. The combination of growth retardation, a petechial rash and hepato-
s plenomegaly is highly suggestive of a congenital infection. Cyto-
megalovirus is the most likely candidate. A pneumonitis may develop
w ith CMV infection although it is rarely present at birth and most com-
monly associated with perinatally acquired infection.
c. i. Blood needs to be sent for CMV testing.
i. . Further investigations should look for other evidence of CMV dam-
age – an echocardiogram to look for congenital heart defects and
cerebral ultrasound, CT or MRI to look for intracranial calcification
and evidence of lissencephaly or polymicrogyria that may occur with
early infection.
ii. . If the diagnosis is confirmed, treatment with anti-CMV chemother-
apy should be considered. There is some encouraging information
Dr.Wahid Helmi-Egypt
41. A 26 week gestation infant is 4 days old. A central venous line has been
inserted into the left long saphenous vein and shortly afterwards
abdominal distension is noted. An X-ray is performed.
a. List three major abnormalities in the intestines.
b. How is the position of the central line relevant?
c. What are the underlying diagnoses?

41.a. i. There is dilated bowel with thickened walls, mainly on the right side
of the abdomen.
i. . There are areas of bowel with intramural gas.
ii. . The gut distribution is opposite to that expected. The stomach (see NGT
position) is on the right, liver on the left.
b. . The central line is passing to the left of the spine.
.c The appearances suggest necrotising enterocolitis in a baby with situs
inversus. The appearances are typical of NEC. The distribution of the
abdominal contents and the left sided position of the inferior can only be
e xplained by situs inversus. The very small amount of heart visible appears
to be on the left but further investigation would be needed to exclude
dextrocardia.
Dr.Wahid Helmi-Egypt
42. A 26 week gestation infant has had severe RDS and has not
responded well to surfactant. Because of deteriorating gases at high
peak pressures high frequency oscillation is commenced. He initially
improves and his inspired oxygen falls from 100% to 45%. After 24
hours he starts to deteriorate again and inspired oxygen increases to
90%. A chest X-ray is taken at 4 hours of age.
a. What does the X-ray show?
b. What action will you take?

42. a. i. There is an endotracheal tube with a high tip.


2.
ii. There is a central line which is probably an umbilical arterial line with the
tip at T8.
iii. There is patchy opacification throughout both lungs.
iv. The lungs are hyperinflated with 11 posterior rib ends showing and with
flat diaphragms.
v. The heart appears moderately compressed.
b. Mean airway pressure should be reduced or ventilation should be changed to a
different modality.
43. A 27 week gestation infant has had moderate RDS and ventilation is
Dr.Wahid Helmi-Egypt
weaning fairly rapidly. Extubation to CPAP is being considered when she
deteriorates and abdominal distension is seen. NEC is suspected and a chest
and abdominal X-ray is requested.
a. What does the X-ray show?
b. What is the reason for her deterioration?
c. What action will you take?

43. 3.a. i. There is a central venous catheter entering from the right
arm. ii. There is patchy shadowing of both lungs.
iii. There is a nasogastric tube, the end of which cannot be seen.
iv. There is gaseous distension of the stomach and intestines.
v. . The endotracheal tube is not in the trachea – it is almost certainly in the
oesophagus.
b. Ventilation of the gastrointestinal tract.
c. Remove the ET tube and either re-intubate or try CPAP. Gas should be
sucked out of the gastrointestinal tract if possible and the nasogastric tube
s hould then be put on free drainage.
Dr.Wahid Helmi-Egypt
4. A 26 week infant is born in poor condition and needs high pressure
ventilation from the outset. The baby has caused considerable concern to
the fetal medicine abdominal X-ray is obtained shortly after birth.
a. What does the X-ray show?
b. What intra-uterine procedure was performed?
c. What do you think the underlying diagnosis is?

44. a. i. There is an endotracheal tube in the correct position.


i. . There is a central line which is probably an umbilical arterial cath-
eter with the tip at T11–12.
ii. . The lungs are very small volume.
iv. . There are pleural effusions, more marked on the right than the
left. v . There are two echodensities over the right lung field and one
over
the left. Each has the appearance of a small cylinder.
b. In-utero drainage of pleural effusions using drains with pigtails on both
ends.The echodense bodies are markers – one on each end of the
drains.
c. Pleural effusions of this severity are uncommon. In the absence of
other signs of hydrops, pulmonary lymphangiectasia is a definite
possibility.
Dr.Wahid Helmi-Egypt

45. A 28 week gestation infant has mild RDS. There was signifi cant
growth retardation and Dopplers showed reversed end-diastolic fl ow.
The abdomen looks moderately distended and stage 1 NEC is
suspected. A decision has been taken not to feed for at least for 10
days. A chest X-ray is obtained.
a. What does the X-ray show?
b. What action will you take?

45. a. i. There is an endotracheal tube that is high.


i. . There is a nasogastric tube in position.
ii. . The lungs show patchy shadowing in both fields.
iv. . There is a central venous line which appears to have been inserted through
a scalp or neck vein.The end is resting within the right atrium.
b. The central venous line needs to be withdrawn to lie outside the heart.
Dr.Wahid Helmi-Egypt
46. In the same infant as in question 45 the procedure is repeated. A
further chest X-ray is taken.
a. What does the X-ray show?
b. What will you do next?

46. a.i. There is an endotracheal tube.


i. . There is a nasogastric tube.
ii. . There is a central venous line which appears to have been inserted
through a vein in the right arm. It has passed in a cephalad direction.
The tip is not visible but probably lies in either the neck or the head.
b. The line needs to be withdrawn until the tip lies in the region of the thoracic inlet.
Dr.Wahid Helmi-Egypt
47. A 26 week gestation infant has required ventilation from birth. A chest and
abdominal X-ray is taken at 4 hours of age.
a. What does the X-ray show?
b. What action will you take?

47. 7. a.
i. There is an endotracheal tube that is very high.
ii. There is a nasogastric tube with the tip in the stomach.
iii. There is an umbilical arterial line with the tip at T5.
iv. There is an umbilical venous catheter which deviates to the right
and appears to lie within the liver.
b. Umbilical venous and arterial lines and the endotracheal tube need to be
repositioned.
Dr.Wahid Helmi-Egypt 48. A 25 week gestation infant has required ventilation from birth and
has needed high pressures. She is now 2 weeks old and there has
been a sudden deterioration. The cause is not obvious on
examination and a chest and abdominal X-ray is requested.
a. What does the X-ray show?
b. Why was this not detected on examination?
c. What action will you take?

48. a. i. There is an endotracheal tube that is slightly high.


ii. There is a central venous line with the tip at the thoracic inlet.
iii. There is an umbilical venous line with a very low tip.
iv. . There is a large amount of oedema, very obvious on the sides of the
chest.
v. . The lungs show dense shadowing and there may be some parenchy- mal
c ysts compatible with pulmonary interstitial emphysema.
vi. i. There is a left tension pneumothorax with mediastinal shift to the right.
b. Although a pneumothorax of this size should be easily detected by trans-
illumination the oedema around the chest will have resulted in a diffuse
brightness that may well have concealed the pneumothorax.
c. A chest drain must be inserted immediately.
Dr.Wahid Helmi-Egypt
49. A 26 week infant has been unwell since birth. At 6 days of age abdominal
distension is noted. Three abdominal X-rays are obtained at 24-hour
intervals.

.a . What do the X-rays


show? b. What is the
diagnosis?
c. What actions should be taken?

49. There are fixed loops of dilated bowel that do not change from one
9. a.
-ray to another.
b. This unchanging appearance is highly suggestive of dead bowel. NEC
is the most likely underlying cause.
c. S urgical referral is needed. A laparotomy will be required at some
point and an abdominal drain may be required as an interim measure.
Dr.Wahid Helmi-Egypt
50. A 29 week gestation infant deteriorates rapidly after birth and
requires ventilation. A chest and abdominal X-ray is taken at 24
hours of age.
a. What does the X-ray show?
b. What is the explanation?
c. What will you do next?

50. a. i. There is an endotracheal tube that is high.


i. . There is a central venous line entering from the right arm that is slightly too
long.
ii. . There is what is probably an umbilical arterial line, with the tip at
T8/9. iv. The lung fields are poorly aerated.
v. There is a nasogastric tube which is passing down into the right lung.
vi. There is gas in the stomach.
b. There is a tracheo-oesophageal fistula or cleft. Although it is possible to pass a
nasogastric tube into the trachea it should not be possible to do so when an
e ndotracheal tube is in place as is the case here. The nasogastric tube is gaining
access to the trachea through a fistula or cleft lower down.
c. The nasogastric tube needs to be removed.The endotracheal tube should be
inserted further to see if it can block a fistula, otherwise ventilation will be very
difficult. A surgical opinion should be sought.
Dr.Wahid Helmi-Egypt

٢٥
Dr-Wahid Helmi Consultant Pediatrician Egypt

THE MOST IMPORTANT MCQ IN


INBORN ERROR OF METABOLISM
!‫خطأ‬

!‫خطأ‬
Dr-Wahid Helmi Consultant Pediatrician Egypt
Dr-Wahid Helmi Consultant Pediatrician Egypt
Dr-Wahid Helmi Consultant Pediatrician Egypt
Dr-Wahid Helmi Consultant Pediatrician Egypt
Dr-Wahid Helmi Consultant Pediatrician Egypt
Dr-Wahid Helmi Consultant Pediatrician Egypt
Dr-Wahid Helmi Consultant Pediatrician Egypt
Dr-Wahid Helmi Consultant Pediatrician Egypt
Dr-Wahid Helmi Consultant Pediatrician Egypt
Dr-Wahid Helmi Consultant Pediatrician Egypt
Dr-Wahid Helmi Consultant Pediatrician Egypt
Dr-Wahid Helmi Consultant Pediatrician Egypt
Dr-Wahid Helmi Consultant Pediatrician Egypt
Dr-Wahid Helmi Consultant Pediatrician Egypt
Dr-Wahid Helmi Consultant Pediatrician Egypt
Dr-Wahid Helmi Consultant Pediatrician Egypt
Dr-Wahid Helmi Consultant Pediatrician Egypt
Dr-Wahid Helmi Consultant Pediatrician Egypt
Dr-Wahid Helmi Consultant Pediatrician Egypt
Dr-Wahid Helmi Consultant Pediatrician Egypt
Dr-Wahid Helmi Consultant Pediatrician Egypt
Dr-Wahid Helmi Consultant Pediatrician Egypt
Dr-Wahid Helmi Consultant Pediatrician Egypt
Dr-Wahid Helmi Consultant Pediatrician Egypt
Dr.Wahid Helmi Domiate Egypt

#Neonatology_fellowship
First Note Revision
Dr.Wahid Helmi
Egypt
Dr.Wahid Helmi Domiate Egypt
Glucose Metabolism

1. A term infant is readmitted to the neonatal intensive 1. c. The patient most likely has glycogen storage disease
care unit (NICU) at 3 weeks of age in shock with car- 1 (GSD-1) due to glucose 6-phosphatase deficiency.
diorespiratory failure and oliguric renal failure. Labs This mutation blocks gluconeogenesis from glycoge-
at presentation showed plasma glucose of 25 mg/dL, nolysis during fasting and presents with severe lactic
metabolic acidosis with elevated lactate. Hepatomegaly acidosis. Often presents at a few months of age when
is noted on physical exam. Parents state that the infant infants begin to decrease feeding frequency and sleep
had not been feeding well for the past 3 days and slept through the night. Glycogen synthase deficiency
through the night last night without waking to feed. (GSD-0) leads to inability to synthesize glycogen and
What is the most likely diagnosis? therefore does not present with hepatomegaly. Neither
a. Congenital hyperinsulinism due to KATP channel congenital hyperinsulinism nor fatty acid oxidation
mutation
b. Glycogen synthase deficiency
c. Glucose 6-phosphatase deficiency
d. Disorder of fatty acid oxidation

2. Laboratory results of a critical sample show venous 2. d. The labs are most c/w hyperinsulinism. Neither
glucose of 45 mg/dL, cortisol 2 μ/dL (normal stressed adrenal nor growth hormone deficiency can be diag-
value >18 micrograms/dL), growth hormone 12 ng/ nosed based upon a critical sample alone. An ACTH or
mL, free fatty acids 0.1 mmol/L (normal <1 mmol/L), CRH stimulation to test confirm normal hypothalamic-
β-hydroxybutyrate 0.2 mmol/L (normal <1.5 pituitary-adrenal axis would be appropriate in this
mmol/L). Plasma glucose increased to 80 mg/dL 30 case; GH was normal so growth hormone stimulation
minutes after administration of 1 mg glucagon Which testing is not needed. If clinically appropriate, the next
of the following is an appropriate next step? step in the management of hyperinsulinism would be
a. Start hydrocortisone 15 mg/m2/BSA per day try diazoxide. It can take up to 5 days to see full effect
b. Perform an arginine/clonidine growth hormone of treatment; infant will continue to need intravenous
stimulation test or continuous enteral dextrose support, target BG > 70
c. Resume intravenous dextrose infusion to maintain mg/dL.
plasma glucose >60 mg/dL
d. Trial diazoxide at dose of 15 mg/kg/day

3. A full term, LGA infant female has required GIR of 25 3. c. Based upon the clinical history provided and the
high GIR, this infant most likely has hyperinsulinism.
mg/kg/min throughout the first week of life to prevent
While there is controversy regarding the definition of
plasma glucoses levels from falling to below 30 mg/dL.
Treatment should target what plasma glucose level in
a normal blood glucose range in healthy and preterm
this female?
infants, it is essential that all infants with suspected
a. >50 mg/dL
or confirmed congenital hypoglycemia be maintained
b. >60 mg/dL
at plasma glucose levels > 70 mg/dL. Normally, as
c. >70 mg/dL
plasma glucose levels fall below 70, counterregulatory
d. >80 mg/dL
mechanisms kick in to provide the brain with alternate
fuel sources (i.e., ketones) and the body with substrate
for gluconeogenesis (fatty acids, amino acids). In HI,
ketogenesis and lipolysis are blocked, leading to more
rapid and more severe neurologic damage because of
the lack of an alternate fuel source.

4. Which of the following is true regarding the use of 4. b. Diazoxide can cause fluid retention and may be asso-
diazoxide in neonates? ciated with development of pulmonary hypertension,
a. All infants with congenital hyperinsulinism will therefore it must be used with caution in patients with
respond to diazoxide. congenital heart disease with frequent monitoring of
b. Diazoxide should be used with caution in patients cardiac status. Most infants with stress-induced HI will
with congenital heart disease. respond to diazoxide, while only some forms of con-
c. Diazoxide causes hair loss. genital HI (GCK-HI, SCHAD-HI, GDH-AD AD-HI) are
d. Neonates on diazoxide need to have regular moni- responsive; patients with mutations in the KATP chan-
toring of CBCs for development of polycythemia. nel will need surgery. Diazoxide causes hypertrichosis
(reversible when discontinued) and can cause cytope-
5. Which of the following conditions of neonatal glucose nia.
disturbance is not associated with increased risk of dia- 5. d. Patients with transient neonatal diabetes typically
betes as an adult? have a remission period during childhood where insu-
a. Transient neonatal diabetes due to chromosome lin is not needed, but permanent insulin requirement is
6q24 mutation likely to return in adulthood. Diffuse HI d/t KATP chan-
b. Permanent neonatal diabetes due to activating nel will require near-total pancreatectomy; there is an
mutation in KATP channel increased risk of diabetes and insulin requirement after
c. Diffuse hyperinsulinism due to inactivating muta- this surgery. Hypopituitarism is not associated with
tion in KATP channel increased risk of diabetes.
d. Neonatal panhypopituitarism
Dr.Wahid Helmi Domiate Egypt
6. Which of the following statements is not true regarding 6. c. Glucose is the main source of energy for the brain.
energy utilization by the brain? During times of prolonged fasting, ketone bodies and
a. Glucose is the main source of ATP used by the lactate can be used as alternate fuel sources. Free fatty
brain. acids do not cross the blood brain barrier so are unable
b. Glucose is transported into brain cells via GLUT to be utilized directly by the brain for energy.
3.
c. Fatty acids cross the blood-brain barrier and can be
utilized as a source of ATP.
d. Ketone bodies cross the blood-brain barrier and can
be utilized as a source of ATP.

7. A 3-week-old AGA term male has persistent pre-fed


plasma glucose levels between 45–60 mg/dL. Other 7. d. Labs and clinical findings are strongly suggestive
notable clinical findings include partial cleft lip and of neonatal hypopituitarism, however GH and ACTH
palate and absent corpus callosum. Critical sample or CRH stimulation tests should be done to confirm
shows plasma glucose 47 mg/dL, cortisol of < 1 mcg/ the diagnosis. The patient will most likely require life-
dL, growth hormone < 1 ng/dL, β-hydroxybutyrate < long hormone replacement. He is also at risk of cen-
0.1 mmol/L, free-fatty acids < 0.1 mmol/L. Which of tral hypothyroidism, which could have been missed
the following statements is not consistent with these on newborn screen in states that use a primary TSH
laboratory findings? screening program. Uncooked cornstarch is used to
a. The patient will likely need lifelong hydrocortisone manage patients with glycogen storage disease and
replacement. has no role here.
b. Thyroid function with total or free T4 should be
checked ASAP.
c. Growth hormone therapy should be started imme-
diately once diagnosis is confirmed.
d. Uncooked cornstarch can be used to help manage
his hypoglycemia.
8. A 10-month-old former 24 weeks gestational age 8. a. The patient most likely has late-dumping syndrome.
(WGA) male infant has recently developed intermit- This condition is characterized by postprandial hypo-
tent episodes of hypoglycemia with plasma glucose glycemia, most commonly in patients with gastrostomy
levels as low as 30 mg/dL. He was receiving continu- and history of Nissen fundoplication. The etiology is
ous feeds via a G-tube; recently you have been work- not entirely understood, but appears to be a condition
ing to condense his feeds, and he is now receiving of excessive insulin secretion as a result of rapid glu-
daytime bolus feeds every 4 hours. Which of the fol- cose absorption and altered incretin hormone signal-
lowing is true about his condition? ing. These patients do not have fasting hypoglycemia
a. Critical sample is likely to show inappropriately nor- and hypoglycemia will not persist once oral feeds have
mal insulin and suppressed β-hydroxybutyrate. been established. In some cases, hypoglycemia can be
b. This condition is characterized by fasting hypogly- prevented by lowering rate of bolus feeds and tapering
cemia. rate over the last 30 minutes; acarbose given with each
c. The patient will have lifelong hypoglycemia. bolus feed is also effective.
d. Hepatomegaly is an associated clinical finding.

9. Which of the following hormones is not involved in 9. c. The secretion and action of a number of hormones
intermediary metabolism? contribute to intermediary metabolism. As plasma
a. Glucagon glucose levels fall below 70 mg/dL, insulin secre-
b. Insulin tion decreases while secretion of glucagon increases.
c. Thyroid hormone Further declines in glucose levels induce secretion of
d. Growth hormone epinephrine, growth hormone, and cortisol. Thyroid
hormone is not known to play a role in intermediary
metabolism.
Dr.Wahid Helmi Domiate Egypt

10. Which of the following tests is most helpful in deter-


mining if source of hyperinsulinism is endogenous (i.e.,
congenital hyperinsulinism) or exogenous (i.e., surrep-
titious in Munchausen by proxy)?
a. Insulin level
b. β-hydroxybutyrate level
c. Glucose response to glucagon administration
d. C peptide level

10. d. Distinguishing between endogenous and exogenous


hyperinsulinism can be difficult. Biochemically, labo-
ratory results at time of critical sample and glucagon
stimulation test will be identical, with the exception
of C-peptide, which is expected to be elevated or inap-
propriately elevated with endogenous insulin over-
secretion but suppressed in the setting of exogenous
administration. 
Dr.Wahid Helmi Domiate Egypt

Calcium, Phosphorus, and Magnesium Metabolism


1. Which statement is true regarding skeletal develop­ 1. c. Mutations in the alkaline phosphatase gene cause
ment? hypophosphatasia, a type of early onset osteoporosis
a. The femurs are formed via intramembranous ossifi­ characterized by absent/diminished skeletal miner-
cation. alization. The femurs, like all long bones, are formed
b. A genetic defect resulting in impaired chondrocyte by endochondral ossification. Defects in chondro-
maturation would be expected to cause early onset cyte function lead to a number of skeletal dysplasias
arthritis but would not negatively affect the skeleton. because of impaired endochondral ossification. PTHrP
c. A genetic defect resulting in reduced alkaline phos­ is an important regulator of calcium status and osteo-
phatase activity would negatively affect bone min­ blast function prenatally, but has a limited role after
eral density during infancy. birth.
d. PTHrP is the primary regulator of osteoblast­
mediated bone formation in neonates.

2. Neonatal magnesium status affects calcium metabo­


lism in which way? 2. c. Normal magnesium levels are required for PTH. Both
a. Hypermagnesemia results in hypercalcemia due to high and low magnesium levels inhibit PTH secretion,
excessive secretion of PTH the mechanism is not fully understood. Both calcium
b. Hypermagnesemia results in hypocalcemia by bind­ and magnesium form divalent cations, so magnesium
ing calcium in the GI tract and thereby decreasing would not be expected to bind to calcium and decrease
calcium availability for GI absorption availability for absorption (as is seen with phosphate
c. Hypomagnesemia results in hypocalcemia via anions).
decreased PTH secretion
d. Hypomagnesemia leads to hypercalcemia because
magnesium is required for renal calcium excretion 3. b. DiGeorge syndrome results in hypocalcemia
because of defective parathyroid gland development.
3. What is the mechanism of hypocalcemia in DiGeorge In many cases the hypoparathyroidism is transient,
(22q11.2 deletion) syndrome? though can be exacerbated by illness and acute stress.
a. Inactivating mutation in CYP27B1 Mutations in CPY27B1 cause vitamin D-dependent
b. Hypoplasia/aplasia of parathyroid glands rickets type 1A and can result in hypocalcemia due to
c. Renal resistance to parathyroid hormone decrease synthesis of active vitamin D. PTH resistance
d. Activating mutation in PHEX with associated as seen in pseudohypoparathyroidism due to muta-
hyperphosphatemia tions in GNAS. Inactivating mutations in PHEX cause
hypophosphatemia in X-linked hypophosphatemic
4. All of the following are signs/symptoms of hypocalce­ rickets.
mia EXCEPT
a. Shortened QTc interval
b. Laryngospasm 4. a. Severe hypocalcemia can result in prolonged QTc
c. Involuntary muscle contraction interval and resultant cardiac dysfunction.
d. Seizure

5. A 5.1 kg, 39 Weeks gestational age (WGA) infant suf­ 5. b. Transient hypoparathyroidism due to hypomagne-
fers a hypocalcemic seizure on DOL 2. What is the most semia. Large for gestational age is suggestive of infant
likely etiology? of diabetic mother, which is a risk factor for early neo-
a. Hyperparathyroidism due to inactivation mutation natal hypocalcemia related to low magnesium levels
in CaSR and impaired parathyroid hormone release. Inactivat-
b. Transient hypoparathyroidism due to hypomagne­ ing mutations in the CaSR causing NSHPT and FHH
semia cause hypercalcemia and typically do not present with
c. Familial hypocalciuric hypercalcemia seizure. Infants born with DiGeorge (22q11.2 deletion)
d. Permanent hypoparathyroidism due to 22q11.2
deletion
Dr.Wahid Helmi Domiate Egypt
6. A 32 WGA infant develops respiratory failure shortly 6. b. Alkaline phosphatase. This child most likely has
after birth. Radiographs show marked underminer­ congenital rickets due to hypophosphatasia. This
alization of bones and multiple rib fractures. Labs are condition is caused by mutations in the ALP gene and
notable for a serum calcium of 12.5 mg/dL, serum results in markedly low serum alkaline phosphatase
phosphorus of 8.5 mg/dL, urine calcium to creatinine levels. Urine phosphorus is most helpful in differen-
ratio of 2.5. Which of the following tests will be most tiating malabsorptive from renal hypophosphatemic
helpful in confirming the diagnosis? rickets, which is not present based upon the elevated
a. Urine phosphorus serum phosphorus. 25-OHD would be the test of
b. 25­OH vitamin D choice if vitamin D deficiency rickets is suspected,
c. Alkaline phosphatase which would not typically present with this severity
d. Intact PTH and would be associated with low/normal serum cal-
cium and phosphorus. An elevated intact PTH would,
in the setting of hypercalcemia, confirm diagnosis of
hyperparathyroidism, which would not present with
rickets as described here.

7. Which of the following is not a risk factor for osteo­ 7. b. Large for gestational age and/or infant of diabetic
penia of prematurity? mother are not risk factors for osteopenia of prema-
a. Postnatal furosemide exposure for CHD turity, though they may be associated with other
b. Large for gestational age due to maternal diabetes disorders of bone mineral homeostasis such as early
c. History of necrotizing enterocolitis postnatal hypocalcemia.
d. Birth weight of 1250 grams

8. Which of the following is true regarding the clinical


management of osteopenia of prematurity? 8. c. The amount of calcium/phosphorus that can be
a. Duration of TPN should be extended because this provided in TPN is limited; transition to full enteral
provides more phosphorus for bone mineralization feeds should be encouraged as soon as feasible. DXA
compared to enteral feeds. scans are capable of assessing bone mineral density,
b. DXA scans should be obtained every 4 weeks to but there is limited data in neonates so they are not
screen for osteopenia in infants born <26 WGA. currently a recommended procedure in the monitor-
c. The goal calcium intake from enteral feeds in infants ing of osteopenia of prematurity. Premature infants
at risk of osteopenia of prematurity should be 150– should be supplemented with 200–400 IU of vitamin
200 mg/kg/day. D, and increased as needed for confirmed vitamin D
d. All infants with birth weights <1500 grams should deficiency.
be supplemented with 1000 IU of vitamin D daily.

9. A 2­week­old, term male infant is re­admitted to the


NICU with weight loss and dehydration. Labs are 9. b. Based upon finding of elevated calcium and 1,25-
notable for serum calcium of 13.5, undetectable PTH, OH vitamin D with appropriately suppressed PTH, the
25­OH vitamin D of 32 ng/mL, 1,25­OH vitamin D 175 patient most likely has infantile hypercalcemia due to
pg/mL (normal range 31–87). He has been breastfed 24-hydroxylase mutation. This condition is associated
and is receiving 400 IU of vitamin D3 daily. Which of with hypercalciuria and risk of nephrocalcinosis and
the following statements about the patient’s underly­ kidney stones in late childhood/early adulthood. Vita-
ing condition is false? min D intoxication would be associated with elevated
a. He will be at increased risk of kidney stones in young 25-OH vitamin D (typically above 70–100 ng/mL).
adulthood. Low calcium formulas can be helpful in managing this
b. Initial therapy should include administration of condition; he should also avoid vitamin D supplemen-
normal saline at 1.5­ to 2­times maintenance. tation and sun exposure.
c. His condition is iatrogenic due to excess vitamin D
supplementation.
d. A low calcium formula may be needed to manage
his condition during infancy.
Dr.Wahid Helmi Domiate Egypt

10. A 12­month­old, former 26 WGA female infant with 10. b. The patient most likely has nutritional hypophospha-
short gut maintained on elemental formula for the temic rickets related to exposure to elemental formula.
past 6 months is found on AM rounds to have acute Labs will show low serum phosphorus, undetectable
onset swelling and decreased movement of left femur. urine phosphorus, elevated alkaline phosphatase.
Radiograph confirms a mid­shaft femur fracture and Treatment includes oral phosphate supplementation
cupping and fraying of the distal femoral and proximal and requires close laboratory monitoring for the devel-
tibia metaphysis. Which of the following is true regard­ opment of hypocalcemia as a result of exuberant gas-
ing the patient’s underlying diagnosis? trointestinal phosphorus absorption. This condition is
a. Urine labs will show elevated phosphorus self-limited and will resolve with appropriate phospho-
b. Hypocalcemia is a potential side effect of the treat­ rus supplements. 
ment for her condition
c. Her condition is expected to be lifelong
d. Alkaline phosphatase will be low
Dr.Wahid Helmi Domiate Egypt
Thermoregulation

1. Which of the following hormones is the primary driver 1. c. Norepinephrine is the key counterregulatory hor-
of the acute physiologic response to cold stress? mone released in response to acute cold stress. Actions
a. Cortisol include peripheral vasoconstriction to reduce heat loss
b. Thyroid hormone and stimulation of brown adipose tissue for thermogen-
c. Norepinephrine esis. Thyroid hormone secretion ± cortisol secretion
d. Growth hormone play a role in the long-term adaption to cold, but are
not thought to play a crucial role in the acute response
to cold stress. Growth hormone is not known to have a
regulatory role in the response to short- or long-term
cold exposure.

2. Which of the following statements is true regarding


brown adipose tissue? -2. a. Free fatty acids, released via lipolysis, form the sub-
a. Fatty acids are the substrates for thermogenesis. strate for thermogenesis by brown adipose tissue. Ther-
b. Thermogenesis occurs in the nucleus of brown adi mogenesis occurs in the mitochondria of the brown
pose tissue. adipocyte and is “nonshivering” as heat is generated
c. Results in heat generation as a result of shivering. in the absence of muscle contraction. Brown adipose
d. Infants have less brown adipose tissue, in proportion tissue is more prominent in newborns than adults and
to body size, compared to adults. was once thought to be an embryonic remnant, but is
now recognized as being present throughout the life
span.
3. Which of the following is not a mechanism of heat loss in 3. c. Neonates can lose heat through conduct convec-
premature infants? tive, conductive, radiant, and evaporative heat loss.
a. Convective heat loss due to cooler air environment in Premature infants are at greater risk for convective,
delivery room compared to womb conductive, and radiant loss as a result of diminished
b. Evaporative heat loss due to thin epidermis subcutaneous fat that acts as an insulator and greater
c. Excessive nonshivering thermogenesis risk of evaporative heat loss because of thinner skin.
d. Conductive heat loss to resuscitation bed because of Nonshivering thermogenesis is a primary physiologic
decreased subcutaneous fat response to generate heat in response to cold stress.

4. Which of the following exposures is not a potential 4. a. Skin-to-skin contact with the mother is the optimal
source of heat loss to a neonate? heat source for stable infants immediately following
a. Mother’s skin delivery. Air and oxygen used for respiratory support
b. Inspired air via nasal cannula can be a source of both convective and evaporative
c. North-facing window in the neonatal intensive care heat loss; therefore, warmed or humidified air is recom-
unit (NICU) mended, if available. Windows or other cool objects can
d. Blanket be a source of radiant heat loss. Nonwarmed blankets
can be a source of conductive heat loss. 
Dr.Wahid Helmi Domiate Egypt
Immune System

1. An infant is successfully resuscitated following vaginal 1. b. Extremely low-gestational-age infants are highly
delivery at 23{0/7} weeks due to intrauterine infec- susceptible to infection for multiple reasons. Many
tion. You are concerned that this patient could quickly natural defenses, including skin barriers, ciliary clear-
develop overwhelming sepsis due to her extreme prema- ance, and gastric pH are naturally underdeveloped and
turity and exposure to infection. Which of the follow- also decreased due to medical interventions such as
ing immune-related factors does not contribute to this percutaneous catheters. Interferon-γ, a cytokine cen-
patient’s increased susceptibility to infection? tral to initiating and propagating many protective (and
a. An underdeveloped stratum corneum increases per- inflammatory) immune mechanisms, is suppressed in
meability of skin to pathogens. neonatal innate and adaptive immune cells. There are
b. Definitive hematopoiesis (bone marrow-derived) three pathways to complement activation, including
begins at 28 weeks’ gestation, so extremely prema- the classic (antibody-activated), lectin (lectin and IgA
ture infants rely on primitive hematopoietic immune activated), and alternative (spontaneous hydrolysis).
lineages for protection. Generally speaking, all three pathways have dimin-
c. There is impaired interferon-γ production by T cells ished activity in neonates compared to adults, and the
and monocytes in neonates. classic and lectin pathways, due to lower maternal
d. Preterm infants and full-term neonates have antibody transfer, are more affected in preterm infants
decreased lectin-mediated complement activation. born prior to 28 weeks’ gestation. Primitive hematopoi-
esis, which takes place in the fetal yolk sac, transitions
to definitive hematopoiesis by 5–8 weeks’ gestation. 
1. In accordance with guidelines, a newborn screen was 1. b. Testing for SCID is currently included in the Recom-
carried out prior to the first blood transfusion in an mended Uniform Screening Panel of Core Conditions,
infant born at 23{0/7}weeks’ gestational age. The compiled by the Secretary of the Department of Health
screening test is positive for severe combined immuno- and Human Services. The SCID screen is considered
deficiency (SCID), showing low, but not absent, T cell positive when the TREC content falls below a threshold
receptor excision circles (TRECs). The most appropriate value that varies by laboratory. T cell receptor excision
next step for this patient is which of the following? circles are circular episomal DNA fragments present
a. Urgent hematology and oncology consult to assess in recent thymic emigrants that are formed during T
for bone marrow transplantation candidacy cell receptor rearrangement. Cell TREC content is nor-
b. Newborn screen repeated until patient reaches mally diluted through cell division, but overall content
term-equivalent gestational age in the blood is kept relatively steady in the healthy term
c. Antimicrobial prophylaxis started with sulfa- newborn by her or his high rate of new T cell release.
methoxazole-trimethoprim (Bactrim) Preterm infants have a lower TREC content develop-
d. Blood specimen sent to laboratory for measurement mentally, but due to the many comorbid conditions that
of lymphocyte subsets by flow cytometry can blunt their thymic function, there are no published

2. Vaccines against Haemophilus influenzae type b and 2. a. Conjugate vaccines contain a linked polysaccharide-pro-
Pneumococcus infections are highly successful in reduc- tein antigen and are tailored to improve the B cell response
ing the incidence of invasive disease. The vaccine plat- to polysaccharide (encapsulated bacteria) antigens. A
form against these pathogens is well-tailored for the major source of protection against encapsulated bacteria is
neonatal immune system in that: antibody generated by B cells in a T-independent fashion.
a. The polysaccharide antigen is T-independent, and T-independent responses bypass normal antigen process-
the protein toxin enables T cell help in inducing B cell ing and presentation by antigen-presenting cells (APC’s)
memory differentiation and antibody class switching. and interact directly with B cell receptors. Splenic marginal
b. They contain monophosphoryl lipid A adjuvant, zone B cells (MZBs) are highly active against T-indepen-
which boosts a newborn’s naïve adaptive response.
c. The antigens are recombinant proteins, which more
specifically target antigen-specific naïve T cells and
therefore generate more effective memory.
d. The polysaccharide antigen is conjugated to a DNA
fragment, which provides costimulation of antigen-
specific T cells through Toll-like receptors.
Dr.Wahid Helmi Domiate Egypt 3. a. IgG is actively transported from the mother to the
3. A term newborn’s antibody concentration at birth is fetus via IgG-specific receptors that are expressed on
which of the following? the placenta during the late second trimester. By term
a. Higher in IgG compared to maternal circulation due gestation, the neonate’s IgG concentration is higher
to active transport across the placenta than the mother’s. IgA is produced mainly at mucosal
b. High in IgA due to mucosal immune responses surfaces following expos1. ure to commensal microbes,
c. High in maternal IgM in the setting of maternal which does not occur to a substantial degree until after
infection with Toxoplasmosis delivery. IgM forms pentamers, which cannot cross the
d. Similar in IgE due to passive transport across the placenta, and there are no IgM-specific receptors to
placenta facilitate transport. IgE also does not cross the placenta
but may be similar between mother and fetus due to a
genetic predisposition to allergy. 

Abnormal Immune System Development

1. You are asked to evaluate a term infant born 4 hours


ago by scheduled cesarean section for persistent central
cyanosis. The infant is male, appropriate weight for
gestational age, and appears in no distress. Vital signs
are within normal limits, with the exception of oxygen
saturation,
which measures 81% preductal and postductal.
Your examination reveals a hyperdynamic precordium
with a single S2 and a regurgitant murmur.
You obtain an x-ray, which shows dextroposition of the
heart and a midline liver and stomach. You determine
that once stable, this infant may ultimately require antimicrobial
prophylaxis with which of the following?
a. Bactrim due to high risk for Pneumocystis pneumonia
b. IVIG due to low antibody levels
c. Amoxicillin due to high risk for infection with encapsulated
bacteria
d. FFP to restore complement levels
1. c. This patient presents with heterotaxy syndrome,
including inappropriate cardiac and abdominal situs.
The midline liver and stomach suggests that there is a
right-sided isomerism, which is associated with asple-
nia or hyposplenia. Splenic anatomy can be evaluated
by ultrasound, and diminished function can be assessed
by detection of Howell-Jolly bodies or pitted erythro-
cytes or by nuclear imaging modalities. Patients with
asplenia are at high risk for infection with encapsu-
lated bacteria, such as Klebsiella pneumonia, Haemophi-
lus influenzae type b, and Streptococcus pneumoniae. The
spleen is enriched with marginal zone B cells, which
rapidly produce neutralizing IgM antibodies against
polysaccharides. The spleen also plays a central role in
generating memory B cells against T-independent anti-
gens, such as polysaccharides and haptens.
Dr.Wahid Helmi Domiate Egypt
2. A term infant is admitted to the NICU for evaluation and
management of her prenatally diagnosed pulmonary
stenosis. On examination, she is found to have hypertelorism,
hooded lids, a short philtrum, and micrognathia.
Her oral examination reveals a bifid uvula. Her
postnatal echocardiogram shows anatomy consistent
with tetralogy of Fallot, and there is adequate flow
through the outflow tract, without prostaglandins.
Her vital signs remain stable on the monitors. Electrolytes
remain within normal limits, with the exception
of a low ionized calcium level detected on day of life 4.
The tests most likely to reveal a contraindication to live
vaccine
administration in this patient include which of
the following?
a. Lymphocyte subsets by flow cytometry
b. Complement levels
c. Complete blood count with differential
d. Immunoglobulin levels

2. a. This infant’s presentation (conotruncal cardiac


anomaly, craniofacial findings, and hypocalcemia)
is most consistent with 22q11 deletion syndrome
(22q11DS), which affects pharyngeal pouch-derived
structures. Patients with complete 22q11DS syndrome
may suffer from athymia. The thymus is essential for
common lymphoid progenitors to differentiate into
naïve T cells, and athymic newborns will have low
or absent naïve T cells in their circulation. Live vac-
cines, therefore, are contraindicated in athymic indi-
viduals who have not received immune replacement
therapy. Enumeration of naïve T cell subsets is best
accomplished by flow cytometry. A complete blood
count with differential quantifies lymphocytes, which
combines T cells, B cells, and NK cells. T cells may be
diminished, even when the absolute lymphocyte count
falls within the normal range. Furthermore, maternal
memory T cell engraftment can occur in the absence
of an endogenous T cell population in the newborn
and will falsely elevate the neonate’s absolute lympho-
cyte count on a CBC. Flow cytometry can be used to
differentiate memory and naïve T cell subsets and can
therefore distinguish maternal from fetal T cells. Older
athymic patients often have diminished immunoglob-
ulin levels, but immunoglobulin in a term neonate is
largely derived by placental transfer and is likely to be
normal at birth. Complement levels are typically nor-
Dr.Wahid Helmi Domiate Egypt
3. A term infant presents to the special care nursery with
a 24-hour total serum bilirubin of 13 (11.9 indirect,
1.1 direct). The mother’s blood type is O, Rh+. The
infant’s blood type is B, Rh+, and her hematocrit and
reticulocyte counts are 38 and 9.2%, respectively.
In this setting, the likely mechanism of intravenous
immunoglobulin’s (IVIG) therapeutic effect is which of
the following?
a. IVIG inhibits NK cell activation.
b. IVIG saturates B-antigen sites on the erythrocytes,
thereby blocking maternal antibodies from attaching.
c. IVIG binds to Fc portion of anti-B maternal antibodies,
rendering bound erythrocytes “invisible” to the
infant’s immune system.
d. IVIG saturates Fc receptor on infant’s phagocytes,
thereby blocking recognition and destruction of
antibody-
bound erythrocytes.

3. d. IVIG is pooled donor, concentrated human immuno-


globulin. IVIG contains mostly IgG, but some IgA and
IgM is present. IVIG modulates immune system activ-
ity through multiple mechanisms, including inhibitory
complement activation, inhibition of cytotoxic T cells,
and inhibition of phagocytosis. The antibody structure
includes a variable portion, which binds to its specific
antigen. Once bound, the constant portion (Fc) is avail-
able to bind to Fc receptors on phagocytes, such as splenic
macrophages, and signals internalization and destruc-
tion of bound erythrocytes. IVIG neutralizes macrophage
activity by saturating their Fc receptors, which then

4. A healthy-appearing term infant presents with a newborn


screen that is positive for severe combined immunodeficiency
(SCID). The laboratory report details that the
T cell receptor excision circle content is “absent.” Lymphocyte
subsets by flow cytometry reveal a T-negative,
B-positive, NK-positive SCID phenotype. The most likely
genetic defect explaining this patient’s T cell deficiency is
which of the following?
a. Absent IL-7 ÿ-receptor expression, leading to T cell
maturation defects
b. RAG1 deficiency (Omenn syndrome), causing defective
T cell receptor production
c. Low CD40L expression, causing T cell cycle arrest
d. Low adenosine deaminase (ADA) expression, leading
to accelerated apoptosis of T cells
4. a. TheHelmi
Dr.Wahid lymphocytes
Domiateaffected
Egypt (T, B, NK) for a given SCID
syndrome are dependent on the differential effects
that molecular pathways have on the development of
each lymphocyte subset. IL-7 signaling is critical dur-
ing thymic T cell development and, in its absence, T
cells fail to mature. IL-7 is not an essential cytokine
for either NK or B cell development, and therefore B
and NK cells are unaffected in IL-7rα deficiency. The
RAG complex is necessary for V(D)J recombination,
which occurs in both T and B cells, and a deficiency
therefore leads to failed T and B cell development. T
cell CD40L binds to CD40 on B cells and signals anti-
body class switching and affinity maturation. Absent
expression causes X-linked hyper IgM, or failure to
produce mature, high-affinity antibodies. T, B, and
NK cell numbers are intact in CD40L deficiency. ADA
is involved in purine salvage, and intracellular accu-
mulation of purines in ADA SCID causes apoptosis of
5. A 26-week-old infant presents at birth in shock from
E. coli sepsis and is found to have an absolute neutrophil
count of 500/mL, which falls to 200/mL on a subsequent
CBC. Which of the following is most accurate regarding
medical therapy for sepsis-induced neutropenia?
a. Dexamethasone increases the absolute neutrophil
count by stimulating bone marrow production.
b. IVIG improves sepsis outcomes by blocking complement-
induced cytolysis and neutrophil consumption.
c. Granulocyte-colony stimulating factor (G-CSF)
increases bone marrow monocyte and neutrophil
production.
d. G-CSF will improve neutrophil count by stimulating
bone marrow neutrophil production.

5. d. G-CSF is a growth factor that selectively stimu-


lates neutrophil production in the bone marrow and
improves survival in preterm infants with sepsis-
induced neutropenia. Granulocyte-macrophage col-
ony-stimulating factor (GM-CSF) is a growth factor
for multiple hematopoietic cell lineages, including
both neutrophils and macrophages. Dexamethasone
inhibits the adhesion and margination of neutrophils,
which increases the number of measured circulating
neutrophils in a blood sample without increasing the
total body number. IVIG can dampen complement-
induced inflammatory pathways, but does not directly
improve neutrophil number. IVIG has not been shown
to improve survival in neonatal bacterial sepsis.
Dr.Wahid Helmi Domiate Egypt
6. A term infant is admitted to the NICU for tachypnea and
increased work of breathing after delivery complicated
by preeclampsia. On physical examination, he is found
to have a narrow chest, but otherwise appears normal.
A chest radiograph reveals a narrow chest, increased
vascular markings, and fluid in the minor fissure. Some
long bones also have metaphyseal sclerosis. A CBC
with differential shows an absolute neutrophil count of
500 cells/mL, absolute lymphocyte count of 700/mL,
hemoglobin of 9.4 g/dL, and platelets of 110 × 103/mL.
His blood cultures remain negative, the absolute neutrophil
count (ANC) improves to >1500 cells/mL, his
respiratory symptoms resolve, and he is discharged to
home. He is seen repeatedly at the pediatrician for foulsmelling,
oily stools, recurrent respiratory infections,
and persistent growth failure. His CBC again shows
anemia
(hemoglobin, Hgb, 7.6 g/dL) and neutropenia
(400/mL). This patient’s most likely diagnosis is which
of the following?
a. Cystic fibrosis
b. Asphyxiating thoracic dystrophy
c. Shwachman-Diamond syndrome
d. Hemophagocytic lymphohistiocytosis

6. c. This patient presents with pancytopenia, a pro-


nounced neutropenia, skeletal abnormalities, and
signs of pancreatic insufficiency, consistent with a
diagnosis of Shwachman-Diamond syndrome (SDS).
The bone marrow failure of SDS places patients at risk
for life-threatening infections and blood cell dyscra-
sias and may require bone marrow transplantation for
treatment. Cystic fibrosis is the most common cause of
pancreatic insufficiency in children, but does not typi-
cally present with skeletal abnormalities. Asphyxiating
thoracic dystrophy presents with respiratory failure
due to poor development of the thoracic ribs, but does
not include pancreatic insufficiency or cytopenias.
Hemophagocytic lymphohistiocytosis (HLH) can also
present with cytopenias, but is usually associated with
immune system activation, including hepatomegaly,
lymphadenopathy, rash and fever. HLH is not associ-
ated with skeletal anomalies. 
Dr.Wahid Helmi Domiate Egypt

Infections of Organ Systems

1. What are the major causes of neonatal early-onset 1. a.


sepsis?
a. Group B beta-hemolytic streptococci, Escherichia
coli, & Listeria monocytogenes
b. Candida albicans, coagulase-negative staphylococci
& Staphylococcus aureus
c. Enteroviruses and herpes simplex viruses

2. What are the major causes of neonatal late-onset sepsis? 2. d. 


a. Group B beta-hemolytic streptococci
b. Escherichia coli, & Listeria monocytogenes
c. coagulase-negative staphylococci & group D entero-
cocci
d. all of the above

Prevention of Infections and Immunization

1. Intrapartum antibiotic prophylaxis has significantly 1. c. 


reduced the incidence of which of the following infec-
tions in the U.S.?
a. Late-onset sepsis from group B streptococci (GBS)
b. Both early-onset and late-onset sepsis from GBS
c. Early-onset sepsis from GBS
d. Syphilis
Dr.Wahid Helmi Domiate Egypt
Dr.Wahid Helmi Domiate Egypt

Thyroid Disorders
1. Which of the following statements is true regarding the potential effects of maternal
Graves disease on the fetus/neonate?
a. Maternal T3 is inactivated by the placenta, and therefore there is no risk of fetal
hyperthyroidism.
b. The fetus/neonate will be at increased risk of hypothyroidism.
c. There is no risk to the fetus/neonate in mothers who had definitive therapy for Graves
with thyroidectomy
prior to conception.
d. Maternal antithyroid therapy with methimazole or propylthiouracil (PTU) should be
discontinued during pregnancy because of the risk of birth defects.
1. b. Graves disease is associated with a mix of both TSH receptor stimulating and blocking
antibodies, therefore the fetus/neonate is at risk of both hyper- and hypo-thyroidism. T3 is
inactivated, however TSH antibodies do cross the placenta. Antibodies can persist even after
definitive therapy, so the fetus/neonate is at risk even if mother has had surgery/ablation.
Untreated mater-nal hyperthyroidism results in increased risk of miscar-riage, premature
labor, intrauterine growth restriction, hyper- and hypothyroidism. PTU is generally
considered first-line treatment for Graves disease in USA, though methimazole is used in
other countries and it is unclear whether the risk of birth defects is actually higher in PTU
versus methimazole. Either way, the goal of therapy is to maintain maternal free T4 levels in
the upper end of the normal range on the lowest possible antithyroid dose.
2. A full-term, newborn male is found to have an abnormal thyroid screen with
elevated TSH of 500 mIU/mL and low T4 of 2 micrograms/dL. Which of the following
is not a possible cause of this child’s hypothyroidism?
a. Activating mutation in the TSH receptor
b. Overexpression of deiodinase 3
c. Inactivating mutation in the NIS gene
d. Presence of a lingual thyroid gland
2. a. An activating mutation in the TSH receptor gene leads to hyperthyroidism and is a rare
form of neonatal thyrotoxicosis. Consumptive hypothyroidism is due to overexpression of
deiodinase, typically hepatic hem-angiomas, that leads to excessive inactivation of thyroid
hormones. Inactivating mutations in the sodium-iodine-symporter prevent iodine from
Dr.Wahid Helmi Domiate Egypt

entering the follicular cell and are a form of thyroid hormone dyshormonogenesis.
Ectopically located thyroid glands are a common cause of congenital hypothyroidism, often
they are located near the base of the tongue.
3. A 36 WGA infant born to a 16-year-old first-time mother has a hypoglycemic seizure
on DOL 4. Brain imaging reveals absence of corpus callosum and hypoplasia of optic
nerves. Which pattern of thyroid hormone tests is most likely to be present?
a. Elevated TSH, low T4, low free T4
b. Normal TSH, low T4, normal free T4
c. Normal TSH, low T4, low free T4
d. Low TSH, elevated T4, elevated free T4
3. c. The infant likely has septo-optic dysplasia with clini-cal evidence of pituitary hormone
deficiency (hypogly-cemic seizure) and is therefore at risk for having central
hypothyroidism. Central hypothyroidism is marked by low thyroid hormone levels; TSH can
be low, normal, or even slightly elevated–as such, states that employ a TSH only newborn
screen may miss infants with cen-tral hypothyroidism. An elevated TSH with low thyroid
hormone levels is diagnostic of primary hypothyroid-ism. A low T4 with normal TSH and
free T4 is consis-tent with TBG deficiency. A pattern of low TSH with elevated thyroid
hormone levels is seen in hyperthy-roidism.
4. All of the following statements are true regarding fetal goiter except:
a. A finding of fetal goiter on prenatal ultrasound is
pathognomonic for the presence of thyroid dyshormonogenesis in the fetus.
b. It is associated with increased risk of fetal upper airway obstruction at delivery.
c. There is increased incidence of fetal goiter in areas
where iodine deficiency is endemic.
d. Fetal thyroid testing can be performed via percutaneous umbilical vein sampling after 20
weeks if fetal thyroid status is uncertain.
4. a. A finding of fetal goiter can be a sign of either fetal hypo- or hyperthyroidism. Large
goiters can cause airway obstruction and subsequent respiratory fail-ure at birth. Fetal
goiters due to hyperthyroidism that result from placental passage of TSH receptor stimulat-
ing antibodies can be treated with maternal antithy-roid medication (methimazole or PTU)
administration, therefore fetal blood sampling may be indicated if there is a significant goiter
Dr.Wahid Helmi Domiate Egypt

and the thyroid status of the fetus is not apparent. Fetal blood sampling is associated with
1%–2% risk of miscarriage, so risks and benefits must be weighed on a case-by-case basis.
5. You are notified that an otherwise healthy term infant had an elevated TSH and low
T4 on the newborn screen.All of the following statements are true regarding the
management of congenital hypothyroidism in this infant except:
a. The starting dose of levothyroxine will be 15 micrograms/kg/day.
b. The infant should be discharged on an oral liquid preparation of levothyroxine because it
is easier to administer.
c. Serum T4 levels should be maintained in the upper one-half of the normal range for the
first year of life.
d. Feeding intolerance with poor weight gain at the first follow-up visit may be a sign of
overtreatment.
5. b. In most situations, infants should be managed with levothyroxine tablets due to
variability in the potency and stability of liquid preparations. Initial dosing is weight-based,
and then titrated based upon biochemi-cal findings and clinical response. Infants with
congeni-tal hypothyroidism therefore require frequent clinical and biochemical monitoring
during the first year of life. The goal of therapy is to normalize TSH within two weeks and
maintain T4 levels in the upper one-half of the normal range. Overtreatment is possible; the
infant may demonstrate signs/symptoms of hyperthyroid-ism including feeding intolerance,
poor weight gain or weight loss, irritability, jitters. Labs will show a sup-pressed TSH value
with an elevated T4 (or free T4).
6. An otherwise healthy appropriate for gestational age (AGA) 34 WGA male is found
to have an abnormal thyroid newborn screen on a blood spot obtained on DOL 2.You
live in a state with a primary T4 screening strategy.Follow up venous blood sample on
DOL 4 showed a normal TSH for age, low T3, low T4, normal free T4, normal free T3.
Which of the following statements is true about the most common form of his
condition?
a. He will be at risk for cognitive impairment if not started on levothyroxine immediately.
b. His lab abnormalities are due to the fact that he was born prematurely.
c. Evaluation of maternal serum would reveal elevated TRAb.
d. He will be able to pass on his condition to a daughter,but not to a son.
Dr.Wahid Helmi Domiate Egypt

6. d. Labs are most consistent with TBG deficiency, a benign condition characterized by low
TBG. Treatment with levothyroxine is not indicated as free thyroid hormone levels are
normal. The inherited form of this condition is due to mutations in SERPIN7A. This gene is
located on the X-chromosome, therefore a father can only pass an abnormal copy to a
daughter. TBG deficiency due to SERPIN7A can be partial (with one abnormal copy in a
female) or complete (with no normal copies in either a male or female). Acquired forms of
TBG deficiency are also possible in the setting of protein-losing conditions. THOP is
characterized by low levels of all thyroid hor-mones; elevated TRAb levels can be seen in
mothers with Graves disease.
7. An infant with which of the following conditions would be most likely to have
elevated reverse T3 levels?
a. Congenital hypothyroidism due to athyreosis
b. Consumptive hypothyroidism
c. TBG deficiency due to SERPIN7A mutation
d. Transient hypothyroxinemia of prematurity
7. b. Consumptive hypothyroidism will result in elevated reverse T3 levels due to
overexpression of deiodinase 3. Reverse T3 levels are most likely to be low in congeni-tal
hypothyroidism and normal in TBG deficiency and THOP. The etiology of
hypothyroxinemia in premature infants is not well understood; defective TSH surge and
diminished iodide stores have been suggested as possible contributors, but definitive data is
lacking.
8. Which of the following statements is true regarding the treatment of neonatal
Graves disease?
a. All infants born to mothers with Graves disease should be started on prophylactic
antithyroid therapy
with methimazole at birth.
b. Prednisone dosed at 1 mg/kg/day is the first-line therapy for neonatal thyrotoxicosis.
c. Infants with suppressed TSH and elevated T4 levels at birth will require lifelong therapy.
d. Treatment with Lugol solution is limited to 1–2 weeks because of “escape” from the
Wolff-Chaikoff effect.
Dr.Wahid Helmi Domiate Egypt

8. d. Only 1%–10% of infants born to mothers with Graves disease are affected, therefore
treatment is initiated only with clinical and biochemical evidence of hyperthyroid-ism.
Therapy includes management of symptoms of hyperthyroidism with a beta-blocker and
reduction in thyroid hormone production with an antithyroid medi-cation (typically
methimazole). High dose iodine (as provided in Lugol solution) transiently inhibits thyroid
hormone production by blocking iodide organification (Wolff-Chaikoff effect). The thyroid
gland escapes this effect in 7–10 days, possibly due to decreased transport of iodine across
the sodium-iodine symporter. Neonatal Graves is transient and usually resolves by 6 months
of age or sooner.
9. A term infant male with neonatal seizures is found to have abnormal thyroid studies,
including elevated T3 and free T3 but low-normal T4 and free T4 levels. Which of the
following statements is true regarding this condition?
a. High-dose levothyroxine (T4) treatment should be instituted.
b. High-dose Cytomel (T3) treatment should be instituted.
c. The clinical manifestations of this condition result from low intracellular concentrations of
T3 in neurons.
d. It is inherited in an autosomal recessive manner.
9. c. This patient most likely has defective thyroid hor-mone transport due to a mutation in
MCT8 (also known as SLC16A2). This transporter is required for transport of T3 into
neurons, in its absence low intra-cellular T3 levels result in abnormal neurologic find-ings
including seizure, hypotonia, and developmental delay. Other body cells are able to transport
T4 across the cell membrane with intracellular conversion into T3. Therefore treatment with
either T4 or T3 can result in a hypermetabolic state in conjunction with hyper-thyroidism.
MCT8 deficiency is inherited in an X-linked recessive pattern.
Dr.Wahid Helmi Domiate Egypt

Adrenal Disorders
1. Which of the following hormones is not produced in
the adrenal cortex?
a. Cortisol
b. Dopamine
c. Aldosterone
d. Dehydroepiandrosterone
e. All of the above are produced in the adrenal cortex
1. b. Dopamine is a catecholamine synthesized in the adrenal medulla
2. Which of the following statements is true regarding
fetal adrenal steroid hormone production?
a. The adrenal glands develop and begin secreting hormones at 12 weeks.
b. The histopathologic organization of the adrenal gland in the fetus is identical to that of an
adult.

c. The fetal adrenal gland does not express 3β-hydroxysteroid dehydrogenase and is

therefore dependent on the placenta in order to synthesize


cortisol.
d. The cells of the adrenal cortex are derived from neuroectodermal precursors.
2. c. The fetal zone of the adrenal cortex produces preg-nenolone, 17-hydroxypregnenolone,
and DHEA which then must be converted into progesterone, 17-hydroxyprogesterone, and
androstenedione by the placenta which expresses 3β-HSD in abundance.
3. Which of the following genes (enzymes) is not expressed in the adrenal gland?
a. CYP19 (aromatase)
b. CYP21 (21-hydroxylase)

c. CYP11B1 (11β-hydroxylase)

d. CYP11B2 (11β-hydroxylase)

e. All of the above enzymes are expressed and active in


the adrenal gland
Dr.Wahid Helmi Domiate Egypt

3. a. Aromatase, because the conversion of androgens into estrogens is expressed primarily


in gonads but is also found in other tissues such as adipose, liver, skin, and brain.
4. A patient is born with ambiguous genitalia and found to have a genetic defect
leading to loss of function of CYP11A1 (cholesterol side-chain cleavage enzyme).Which
of the following is likely true about the patient’s
karyotype?
a. The karyotype is most likely XX, and the patient is overvirilized because of excess ACTH
production.

b. The karyotype is most likely XY, and the patient is undervirilized because of elevated 5α-

reductase activity.
c. The karyotype is mostly likely XX, and the patient is overvirilized because of decreased
aromatase activity.
d. The karyotype is most likely XY, and the patient is undervirilized because of insufficient
substrate for androgen hormone biosynthesis.
4. d. CYP11A1 mutation blocks the ability of cholesterol to be transported to the inner
mitochondrial mem-brane; therefore synthesis of all steroid hormones will be diminished
and will typically result in an underviril-ized male.
5. You are alerted that one of your patients admitted for transient tachypnea of the
newborn had an abnormal newborn screen for CAH. The patient is now a 5-day-old
infant, born 40 weeks gestational age(WGA) via scheduled cesarean delivery, and has
normal appearing male genitalia. Patient is clinically improving with decreasing
fraction of inspired oxygen(fio2) requirement, normal blood pressure, and is tolerating
feeds. The newborn screen specimen was drawn at 36 hours of age, and
electrolytes drawn on DOL 1 showed normal sodium and potassium. What is the most
appropriate next step?
a. Immediately start hydrocortisone only at a dose of 15 mg/m2/day.
b. Immediately start hydrocortisone at a dose of 15mg/m2/day and fludrocortisone at a dose
of 0.1 mg/day.
c. Obtain a venous blood sample for measurement of 17-hydroxyprogesterone.
d. Obtain a venous blood sample for measurement of 17-hydroxyprogesterone and
electrolytes.
Dr.Wahid Helmi Domiate Egypt

5. d. In a well appearing infant without clinical signs of adrenal insufficiency or salt wasting,
it is appropriate to assess a venous blood sample for 17-hydroxyporo-gesterone and
electrolytes as the next step. Perinatal stress is one possible source of a false-positive
newborn screen for CAH. Because clinical signs and electrolyte abnormalities of salt
wasting may not manifest in the first few days of life, it is also prudent to repeat electro-lytes
at this time.
6. You are caring for a 1-week-old former 27 WGA infant with birth weight of 1000
grams transferred to your neonatal intensive care unit(NICU) from a community
hospital on DOL 1. Patient is phenotypically male,intubated, and has required multiple
fluid boluses, inotrope,and vasopressor support to maintain adequate perfusion. You
receive a copy of the newborn screen report sent from the community hospital
reporting a17-hydroxyprogesterone value of 15,000 ng/dL. What
is the appropriate next step?
a. Do nothing; abnormal newborn screen result is
likely the result of prematurity.
b. Obtain a venous blood sample for measurement
of 17-hydroxyprogesterone; wait until results are
back to start hydrocortisone.
c. Obtain a venous blood sample for measurement of
17-hydroxyprogesterone, and start hydrocortisone
100 mg/m2/day.
d. Obtain a venous blood sample for measurement of
17-hydroxyprogesterone, and start hydrocortisone
15 mg/m2/day.
6. c. Based on the clinical picture and the finding of a 17-hydroxyprogesterone level
greater than 10,000 ng/dL, this patient most likely has 21-hydroxylase deficiency CAH.
Prematurity and timing of sample before 48 hours of age increase the possibility of a
false-positive newborn screen result, however typically not to levels >10,000 ng/dL. A
venous 17-hydroxyprogesterone sample should be obtained, but treatment should not
be delayed as the clinical picture is concerning for adrenal crisis. During times of crisis,
stress doses of hydrocortisone at 50–100 mg/m2/day should be used.
Dr.Wahid Helmi Domiate Egypt

7. A newborn is found to have ambiguous genitalia including partial fusion of the


labioscrotal folds and clitoromegaly. No testes are palpable on exam. Which of the
following scenarios is NOT likely to be responsible
for the clinical presentation in this case?
a. 21-hydroxylase deficiency with karyotype XX
b. 3-beta-hydroxysteroid dehydrogenase with karyotypeXX
c. 17-hydroxylase deficiency with karyotype XX
d. 11-beta-hydroxylase deficiency with karyotype XX
7. c. 17-hydroxylase deficiency may cause ambiguous genitalia in XY males due to
undervirilization as a result of decreased androgen synthesis. All other listed enzymatic
defects can lead to ambiguous genitalia in XX females due to overvirilization as a result of
excess androgen production.
8. An 11-month-old infant male in your unit is found to be hypertensive. Review of his
medical records shows that blood pressures have been gradually increasing over the
past several weeks to months. Which of the
following is true regarding hypertension in congenital adrenal hyperplasia?
a. Hypertension in 21-hydroxylase deficiency CAH is
a sign that the Florinef dose needs to be increased.
b. Serum aldosterone levels are typically low.
c. The patient cannot have 17-hydroxlase deficiency
because he has normal appearing genitalia.
d. The majority of patients who will develop hypertension
due to 11-beta-hydroxylase deficiency are identified on the CAH newborn screen
8. b. Forms of CAH associated with early-onset hyper-tension include 11-beta-hydroxylase
CAH and 17-hydroxylase CAH. Both conditions are marked by dramatically elevated serum
levels of 11-deoxycorti-costerone. 11-deoxycortisone has mineralocorticoid activity, when
present at high levels leads to vol-ume-expansion with resultant suppression of renin and
aldosterone. Hypertension is a sign of overtreat-ment with Florinef in 21-hydroxylase CAH,
the dose of which typically needs to be weaned after infancy. The clinical presentation of all
forms of CAH is vari-able, therefore normal virilization does not rule out 17-hydroxylase
CAH in an XY male. Some patients with severe 11-beta-hydroxylase deficiency may have
Dr.Wahid Helmi Domiate Egypt

17-hydroxyprogesterone levels high enough to trig-ger an abnormal newborn screen result,


although the majority will be diagnosed later. An elevated 11-deoxycorticosone level on
ACTH stimulation is the gold-standard diagnostic technique.
9. Which of the following assessments is NOT standardly used to assess adequacy of
medical treatment in apatient with classic 21-hydroxylase CAH?
a. Linear growth
b. Radiograph of left hand for bone age
c. Androstenedione
d. ACTH
e. Plasma renin activity
9. d. ACTH is not a reliable marker for the adequacy of hydrocortisone dose. Linear growth
should be moni-tored for signs of undertreatment (increased growth velocity) and
overtreatment (suppressed growth velocity). Bone age is checked every 1–2 years for
evidence of androgen-induced skeletal maturation. Androstenedione should be followed and
maintained in the normal range for age. Plasma renin activity is used to assess adequacy of
mineralocorticoid replace-ment.

10. You are asked to provide prenatal counseling to amother who previously gave birth
to a virilized female with 21-hydroxylase CAH. Which of the following statements
should you provide to the mother?
a. The incidence of CAH is the same in males and females.
b. She has a one in four chance of having another virilized female with CAH.
c. There is a decreased incidence of virilization in female infants with CAH born to mothers
who take maintenance doses of hydrocortisone during pregnancy.
d. Fetal sexual differentiation does not occur until the third trimester so she would only have
to take dexamethasone for the last few weeks of her pregnancy to reduce the risk of
virilization for an affected female.
10. a. 21-hydroxylase is an autosomal recessive disor-der that affects males and females
equally. Statisti-cally, assuming that both mother and father are carriers, there is a one in
four chance of giving birth to an affected child with each pregnancy, so the risk of having
another affected female is one in eight. Prenatal treatment with dexamethasone to reduce
Dr.Wahid Helmi Domiate Egypt

virilization of affected females is controversial; exist-ing protocols suggest starting treatment


at 6–8 weeks gestational age, around the time that external sexual differentiation occurs.
Hydrocortisone is not effective because it is metabolized by the placenta.

Normal and Abnormal Sexual Differentiation


1. A term infant is noted to have bilateral (BL) undescended testes palpable in the
inguinal canal. Penis is morphologically normal with stretched penile length of 3.2 cm.
Which of the following is true about the management of this condition?
a. Immediate surgical consultation is necessary because this infant is at increased risk of
germ cell malignancy
b. Immediate endocrinologist evaluation is necessary because this infant likely has a disorder
of sexual development
c. Spontaneous descent is likely; surgery should be delayed until after mini-puberty of
infancy
d. Spontaneous descent is likely; surgery should be delayed until after onset of puberty
1. c. The majority of palpable undescended testes will descend spontaneously after the
testosterone surge that accompanies the mini-puberty of infancy which occurs within the
first few months of age. Spontaneous descent is unlikely after 4 months of age. There is an
increased risk of germ cell malignancy in undescended testes, but malignancy is uncommon
in patients who had orchiopexy prior to 2 years of age. Endocrine eval-uation for DSD
should be performed in patients with BL nonpalpable testes or those with undescended testes
and hypospadias or micropenis.
2. A term infant born with cleft palate and large atrial septal defect has a
morphologically normal penis with stretched penile length of 1.5 cm. Which of the
following hormone defects is the most likely cause of his micropenis?
a. Hypothyroidism due to impaired synthesis of thyroid-stimulating hormone
b. Growth hormone deficiency

c. Testosterone deficiency due to 17β hydroxysteroid deficiency

d. Dihydrotestosterone deficiency due to 5α-reductase deficiency


Dr.Wahid Helmi Domiate Egypt

2. b. There is a high suspicion that micropenis in a patient with multiple congenital midline
anomalies is due to deficient pituitary secretion of LH (leading to testoster-one deficiency)
or growth hormone. Central hypothy-roidism is also possible but does not cause micropenis.
Most infants with 17-beta hydroxysteroid deficiency or 5α-reductase deficiency will have a
more severe phe-notype with ambiguous external genitalia or external genitalia that appear
female. Neither condition is com-monly associated with other midline defects
.3. Which of the following conditions is typically associated with significant clinical
virilization at the time of puberty?

a. XY with 5α-reductase deficiency

b. XY with complete androgen insensitivity


c. XX with 21-hydroxylase CAH

d. XY with 17β-HSD CAH

3. a. XY males with 5alpha-reductase may have severe undervirilization at birth owing to


lack of 5-DHT pro-duction, the androgen responsible for the majority of external genitalia in
the male virilization fetus. At puberty, testosterone levels increase dramatically and because
the androgen receptor is present will cause virilization. There is no function androgen
receptor in complete androgen insensitivity, so increased testos-terone has no effect.
Individuals with partial AI will likely undergo some virilization at puberty. Virilization in an
XX female is due to excess androgen production in setting of cortisol deficiency and will not
be apprecia-bly affected by female puberty. XY males with 17β-HSD CAH lack an enzyme
necessary to produce testosterone so will not have significant virilization at puberty.
4. Who should make the final decision regarding gender assignment for an infant with
ambiguous genitalia?
a. The surgeon, who can best discuss functional and cosmetic outcomes of genitoplasty
b. The endocrinologist, who can best discuss past and future hormone exposure and will be
guiding any sex-hormone replacement at the time of puberty
c. The neonatologist, after reviewing the consults
from the surgeon, endocrinologist, geneticist, and psychologist
d. The family, after hearing recommendations of the full multidisciplinary team
Dr.Wahid Helmi Domiate Egypt

4. d. Gender assignment remains complex and there usu-ally is not a “right” or “wrong”
decision. It is essential that families are involved in the discussion and make the ultimate
decision. Some people with DSDs have argued that gender assignment and surgical interven-
tion should be put off until the patient is old enough to decide and provide consent. This is
currently not stan-dard practice owing to the perceived difficulty in rais-ing a child with an
indeterminate gender and because in some cases surgical outcomes may be better when
performed earlier. Although given the changing cul-tural perceptions on gender it is likely
that counseling will evolve to allow for delaying gender assignment as a reasonable
possibility.
5. Which of the following statements regarding hormones and their actions in sexual
differentiation is not correct?
a. The production of estrogen by granulosa cells in an
XX female causes regression of the primordial male
internal genitalia structures.
b. The production of anti-müllerian hormone
by Sertoli cells in an XY male causes regression
of the primordial female internal genitalia
structures.
c. The production of testosterone by Leydig cells in an
XY male results in differentiation of internal structures
into the epididymis, vas deferens, and seminal
vesicle.
d. The conversion of testosterone into dihydrotestosterone
results in the virilization of external genitalia
in an XY male.
5. a. Estrogen plays no known role in fetal sexual differ-entiation, the Wolffian ducts
(primordial male inter-nal structures) regress spontaneously in the absence of testosterone.
Normal male virilization requires the coordinated production of several different hormones
as described in answers b–d.
Dr.Wahid Helmi Domiate Egypt

6. Which of the following clinical presentations of ambiguous


genitalia is associated with high risk of adrenal
insufficiency?
a. Testes palpable in a normally formed scrotum
BL, microphallus with stretched penile length of
1 cm
b. Testes palpable in a bifid scrotum BL, urethral
opening at the ventral penile-scrotum junction
c. Partially fused scrotum, phallic structure 1.5 cm in
length, no palpable gonads
d. Testis palpable on one side of scrotum, stretched
penile length of 2.7 cm
6. c. Adrenal insufficiency should be suspected and urgently evaluated in any case of
ambiguous geni-talia with nonpalpable gonads. This presentation is commonly seen in
patients with mutations in adre-nal hormone biosynthesis resulting in congenital adrenal
hyperplasia. In XX females with CAH caus-ing adrenal insufficiency and ambiguous
genitalia, there will be no testes; virilization is due to excessive androgen production. In XY
males with CAH caus-ing adrenal insufficiency and ambiguous genitalia, undervirilization
will be due to absent/impaired testosterone production and testes will have not descended.
Micropenis (a) and hypospadias (b) in the setting of descended testes are most likely due to
structural developmental defects, though the former can be the result of growth hormone
deficiency and the infant should be monitored for hypoglycemia. Unilateral cryptorchidism
(d) with otherwise normal genitalia is relatively common and unlikely to be associated with
CAH.
7. A term infant is noted to have ambiguous genitalia.Karyotype was found to be XY,
labs drawn on DOL 2 showed undetectable testosterone, normal AMH, normal
androgen precursors including androstenedione and dehydroepiandrosterone (DHEA).
HCG stimulation test showed no increase in testosterone levels.
What is the most likely diagnosis?

a. 17β-HSD deficiency

b. Androgen insensitivity syndrome


Dr.Wahid Helmi Domiate Egypt

c. Leydig cell hypoplasia

d. 5α-reductase deficiency

7. c. Laboratory workup confirms isolated testosterone deficiency (normal AMH shows that
testes/Sertoli cells are present), the absence of elevated androgen hor-mone precursors rules
out CAH. 17β-HSD deficiency (a) would show elevated androstenedione and DHEA, AIS
(b) and 5AR (d) would show normal/elevated tes-tosterone levels. Patients with Leydig cell
hypoplasia can be treated with a short course of testosterone in infancy for penile
enlargement and will develop nor-mal secondary sexual characteristics with testoster-one
hormone replacement starting at time of puberty
8. A term infant is found to have ambiguous genitalia.Rapid karyotype is XX, repeated
androgen precursor hormones including 17-hydroxyprogesterone are normal.All of the
following are appropriate next steps in
the evaluation, except
a. Questioning of mother to determine if any performance
enhancing drugs used during early phases
of pregnancy
b. Questioning/examination of mother for new-onset
hirsutism
c. Genetic testing for presence of SRY
d. Whole exome sequencing
8. d. Presence of virilization in an XX female with absence of laboratory findings of CAH
suggests pos-sibility of exposure to maternal endogenous/exog-enous androgens or presence
of an ovotesticular DSD. Maternal use of androgens as athletically perfor-mance enhancing
drugs early in pregnancy (in some case before pregnancy was recognized) can cause vir-
ilization. Development of maternal hirsutism during pregnancy suggests possibility of
placental aromatase deficiency resulting in accumulation of excess andro-gens in both
maternal and fetal circulation. Ovotestic-ular DSD is a condition associated with both
ovarian and testicular tissue; degree of virilization depends upon amount of testicular tissue
and testosterone production. In many cases these patients are mosaics for chromosomes
(possessing both XX and XY) which can be missed on initial karyotype but will be detected
with DNA testing for SRY. In the absence of SRY, pur-suit of other genetic abnormalities
Dr.Wahid Helmi Domiate Egypt

(such as RSPO-1) with tests such as WES can be considered, but that would not be the next
step in the case here owing to expense/delay in results.

9. Which of the following statements is true regarding gonadal dysgenesis?


a. A karyotype showing XY sex chromosomes rules out gonadal dysgenesis
b. Patients with gonadal dysgenesis are at increased risk of gonadoblastoma
c. All patients with XY gonadal dysgenesis should be assigned male gender at birth
d. Wolffian duct differentiation is unaffected in patients with XY gonadal dysgenesis
9. b. Patients with dysgenetic gonads, including XY gonadal dysgenesis, are at increased risk
of gonadoblas-toma and germ cell malignancy; therefore dysgenetic gonads should be
identified and removed. Presence of XY karyotype dose not rule out gonadal dysgenesis;
mutations of SRY and other genes involved in early sex-ual differentiation can be
responsible for the clinical phe-notype. There is wide clinical phenotype in XY gonadal
dysgenesis ranging from completely feminized internal and external genitalia (complete
gonadal dysgenesis) to a range of undervirilization of internal (i.e., Wolffian ducts) and/or
external genitalia depending upon extent of androgen production (partial gonadal
dysgenesis)
10. An infant with ambiguous genitalia has been found to have an XY karyotype. At
which of the following timepoints will a serum testosterone level be informative?
a. 1 day of life
b. 1 week of life
c. 2 months of age
d. 6 months of age
e. 16 years of age
f. a, b, c
g. a, c, e
h. All of the above
10. g. Testosterone production is present at birth and then rapidly declines in the first several
days until rising again at 20–30 days of life (“mini-puberty of infancy”). Testosterone
production in infancy peaks between 2–3 months of age, before falling to non-detectable
levels by 6 months of age. Testosterone production again rises at the onset of puberty (range
Dr.Wahid Helmi Domiate Egypt

9–14 years of age). Assessment of unstimulated tes-tosterone outside of these times will not
provide any useful clinical information; however, an hCG stimula-tion test can be performed
to assess for the capability of testosterone production.
Dr.Wahid Helmi Domiate Egypt

Abnormal Renal Development


1. You are asked to consult on a woman at 21 weeks of her pregnancy for an abnormal
prenatal U/S at 20weeks. The ultrasound showed kidneys measuring 3.4
cm each with increased echogenicity. The bladder was not able to be seen. There is no
family history of kidney disease. What is the likely cause of these findings?
a. ADPKD
b. MCDK
c. Renal dysplasia
d. ARPKD
e. Beckwith-Wiedemann
1. d. The normal size of fetal kidneys at 20 weeks is 2.7 cm. The bladder can be seen by 15
weeks. The combination of large, bright kidneys with presumably no urine (can’t visualize
the bladder because it is empty) is very sugges-tive of ARPKD. ADPKD can present in
neonates but a severe presentation such as this with anuria is very rare. More often there is
just large kidneys or cysts seen with normal renal function. The lack of family history argues
more for ARPKD than ADPKD, although new mutations comprise 10% of people affected
by ADPKD. Bilateral MCDK would be associated with anuria but large cysts should be seen
in the kidneys. Dysplastic kidneys are usually (but not always) small. The kidneys in
Beckwith-Wiedemann can be large but severe impairment of func-tion in utero would be
atypical
2. You are asked to do another prenatal consult on awoman who was noted at her 20-
week U/S to be carrying a fetus with abnormal kidneys. The left kidney has
multiple cysts of similar size and is of normal size. The right kidney is of normal
echogenicity and shape with good corticomedullary differentiation and no cysts. It
measures 3 cm. No hydronephrosis or hydroureter is seen. The amniotic fluid volume is
normal. Family history is negative for renal disease. What is the MOST
likely diagnosis?
a. MCDK
b. ADPKD
c. Left-sided UPJ
d. Left UVJ obstruction
Dr.Wahid Helmi Domiate Egypt

e. ARPKD
2. a. MCDK has a characteristic ultrasound picture of multiple cysts of similar size giving an
appearance of a “cluster of grapes.” They have no function. Often the contralateral kidney
will show compensatory hypertro-phy in utero as this has. With the normal kidney on the
right, you would expect normal amniotic fluid levels. ADPKD is possible but rarely presents
in neonates with ultrasound findings. ARPKD rarely presents with visible cysts and affects
both kidneys. A UPJ obstruction leads to hydronephrosis while a UVJ obstruction would
lead to hydroureter.
3. You provide a prenatal consult to a couple with a baby suspected to have a posterior
urethral valve who are considering prenatal intervention. Which of these
statements regarding prenatal intervention for suspected bladder obstruction is false?
a. The most common intervention performed is
a vesicoamniotic shunt.
b. Urine Na < 100 mEq/L on serial taps suggests a good
renal prognosis.
c. Intervention has been shown to improve both renal
and lung function long term.
d. Serial bladder taps are preferred when assessing
fetal renal function.
e. Urine Osm < 200 mOsm/L predicts good renal function.
3. c. Intervention of suspected fetal bladder outlet obstruction has not definitely been shown
to improve long-term function of lungs or kidneys. When an inter-vention is done, the
vesicoamniotic shunt is the most common procedure. Serial bladder taps are preferred to
obtain fresh urine to analyze the urine electrolytes. Urine Na < 100 and Urine Osm > 200 are
predictive of preserved renal function. Other tests predictive of good renal function are:
urine Ca < 8 mg/dL, urine protein < 40 mg/dL, urine Beta-2 microglobulin < 4 mg/L.

4. A 6-day-old ex-24 week infant is noted by his bedside nurse to have decreased pulses
in his lower extremities.His blood pressure is also noted to be trending up.
Dr.Wahid Helmi Domiate Egypt

His urine has a pink tinge. Urine output is normal. His respiratory status is unchanged.
He has a UAC in place.U/S shows a small unilateral renal artery clot. What is the most
likely long-term outcome?
a. End stage renal disease as an infant
b. Necrosis of toes bilaterally
c. Persistent hematuria
d. Hypertension
e. Small bowel necrosis
4. d. The baby likely has a renal artery clot due to the UAC. Clots are thought to form due to
disruption of the endothelium during line placement. A low-lying UAC is more likely to
have a renal artery clot as compared to a high lying UAC. The findings of decreased LE per-
fusion, hypertension, and hematuria without oliguria, congestive heart failure, or multi-organ
failure suggest a “minor” clot. The treatment would be removal of the catheter and
symptomatic therapy. Hypertension is common following a renal artery clot. The hyperten-
sion should resolve but this may take months or years. ESRD is seen with bilateral renal
artery clots. Loss of toes or bowel ischemia would be expected with a more severe clot
involving the aorta
.5. A 4-week-old ex-30 week infant undergoes a renal ultrasound for work-up of UTI.
The U/S shows a small left kidney with some echogenic spots. His past medical
history includes treatment for culture-negative sepsis with ampicillin and gentamicin
for which he had aUVC (umbilical venous catheter) in place. He was noted
to have a transient drop in platelets shortly before his antibiotics finished. What is the
most likely etiology of the U/S findings?
a. Kidney stones
b. Dysplasia
c. ARPKD
d. Old renal vein thrombosis
e. Nephrocalcinosis
5. d. The baby has several risk factors for renal vein thrombosis: sepsis, prematurity,
presence of UVC. The classic clinical findings suggestive of RVT are: flank mass, low
platelets, and hypertension. The presence of all three findings is actually rare. While preterm
Dr.Wahid Helmi Domiate Egypt

kid-neys are more likely to have stones or nephrocalcino-sis, the affected kidney would not
be small. Dysplastic kidneys are often small but should not have calcifica-tions. Kidneys
affected by ARPKD are large and dif-fusely echogenic
.6. A full-term baby with congestive heart failure is noted to have a rise in creatinine
from a baseline of 0.7 to 1.6 by day of life 3. Urine output remains more than 1 mL/
kg/hour. What stage of AKI is the baby in using the KDIGO system?
a. Stage 0
b. Stage 1
c. Stage 2
d. Stage 3
6. c. A rise in serum creatinine between 2 and 2.9 times the lowest creatinine obtained in the
prior 7 days or UOP > 0.3 and ≤ 0.5 mL/kg/hour defines stage 2 AKI by KDIGO definition.
7. A 5-day-old full term infant is admitted to the NICU after seeing his pediatrician for
a weight check. He was noted in the office to have a 15% decrease in weight since
birth.On exam he has dry mucus membranes, a sunken fontanelle,and capillary refill
of 3 seconds. His creatinine is 1.6. He receives fluid resuscitation with significant
clinical improvement. Over the next 2 days his creatinine falls to 1.2. His vitals are
normal for age. He appears euvolemic on exam. His renal U/S is normal. His urine
output is 1.1 mL/kg/hour. What should be your next
step in management?
a. NS (normal saline) bolus
b. Start renal dose dopamine
c. Kidney biopsy
d. Insert a bladder catheter
e. Observation
7. e. You have assessed him as euvolemic so further fluid boluses are unlikely to help. If his
hydration was in question though, a NS bolus could be trialed. Renal dose dopamine has not
been shown to hasten renal recovery. Since there is no hydronephrosis or abnormal bladder
on U/S, placement of a blad-der catheter would not improve his renal function. Kidney
biopsies are rarely required in neonates to make a diagnosis of AKI. The baby likely has
ATN from prolonged renal hypoperfusion. There is no specific treatment for this other than
Dr.Wahid Helmi Domiate Egypt

ensuring good hydration, avoiding further renal insults, and man-aging any electrolytes
issues or hypertension. ATN should resolve over days/weeks depending on its severity.
8. A cGA 30-week infant weighing 1340 gm develops stage 3 AKI using the neonatal
KDIGO definition. His daily fluid intake has been severely restricted due to oliguria
and thus he is receiving next to no nutrition. His potassium is 6.2, bicarbonate 15, BUN
42, phosphorus 8. Which statement is incorrect regarding dialysis
options for this baby?
a. PD can be performed by his NICU nurses.
b. PD does not require any anticoagulation.
c. PD removes immunoglobulins.
d. HD involves rapid volume shifts that may not be tolerated.
e. CRRT fluid goals are set on the machine once every
24 hours and cannot be adjusted in between.
8. e. PD has several advantages in the NICU; it is com-paratively technically simple
compared to HD or CRRT. It is performed with manual exchanges of fluid done by NICU
nurses. The baby is not antico-agulated with PD. It may be easier to get a PD cath-eter
inserted than obtain reliable vascular access for HD or CRRT. PD’s disadvantages include
the removal of beneficial proteins like immunoglobulins, its slow efficiency at solute and
fluid removal, and risk of peritonitis. It is contraindicated in babies with abdominal wall
defects or VP shunts. HD can quickly remove solutes but rapid fluid removal is usually not
tolerated well. One of CRRT’s principal advantages is slow, hourly removal of fluid that can
be adjusted each hour.
9. A 28-week-old infant is receiving daily Lasix for fluid overload. His response to the
Lasix dose seems to be decreasing despite increasing from 1 to 2 mg/kg/day.
What strategy can be used next to optimize diuresis?
a. Increase the frequency to q8 dosing
b. Add spironolactone
c. Increase the dose to 3 mg/kg
d. Add a thiazide diuretic
e. Space dosing to q48 hours
Dr.Wahid Helmi Domiate Egypt

9. d. Preterm infants have slower metabolism of furose-mide so should not be dosed multiple
times per day. Spironolactone is a weak diuretic used mostly as a potassium sparing agent in
the NICU. The baby is likely not responding as well to the Lasix because the tubule
segments past the loop of Henle are avidly reabsorbing the sodium due to a state of volume
depletion. The dis-tal nephron’s Na-Cl cotransporter can be blocked by a thiazide diuretic,
which should increase diuresis. Care must be taken though to not induce AKI from intravas-
cular depletion. Close attention to electrolytes is also necessary.
10. The parents of a 10-week-old ex-24 week infant are nervous about his medications
causing long-term damage. They ask what the side effects of his diuretics
may be. Which of these is a true statement?
a. Long-term use of thiazides is linked to bone fractures.
b. Lasix has not been associated with hearing loss.
c. Loop diuretics can lead to nephrocalcinosis.
d. Thiazides are associated with increased numbers of
kidney stones.
e. Spironolactone causes hypokalemia
10. c. Thiazides can cause increased reabsorption of cal-cium from the urine. In contrast,
loop diuretics are associated with hypercalciuria, kidney stones, and nephrocalcinosis. They
are also associated with an increased risk of bone fractures and with hearing loss.
Spironolactone is a potassium-sparing diuretic used most often in the NICU in conjunction
with a thiazide to mitigate potassium loss in the urine.
Dr.Wahid Helmi Domiate Egypt

Water-Electrolyte Metabolism and Acid-Base Balance


1. A 24-week, 600 gm infant is admitted to the NICU. The baby is intubated and
sedated. Due to being quite unstable and requiring frequent hands-on care, she
is under a radiant warmer. You estimate her baseline insensible fluid loss as 100
mL/kg/d. Based on the above information, what will you write her total fluids
for the next 24 hours?
a. 75 mL/kg/d
b. 110 mL/kg/d
c. 140 mL/kg/d
d. 175 mL/kg/d
e. 200 mL/kg/d
1. c. The baby is intubated and receiving 100% humidi-fied air which will reduce/ eliminate its insensible
losses (15%–30%) from respiration. Sedation also decreases the insensible fluid loss by an additional 5%–
25%. Being under a radiant warmer; however, will increase insensible loss by 50%–100%. Combining these
factors will lead to a net increase of 15%–45% above the base-line value (-15 plus -5 plus 50 = 25 to -30
plus -25 + 100 = 45). Therefore total fluids would be 115–145 mL/kg/day as a rough estimate. Because this
is only an estimate, electrolytes, particularly sodium, ins/outs, and weights should be followed closely.
2. A full-term infant is being cared for in your NICU following
a difficult delivery. He was initially prescribed
D5W at 70 mL/kg/d for dol 0–2. On dol 3 he is noted
to have a sodium of 130, a potassium of 4.8, and a creatinine
of 0.7. His weight has been stable since birth.
He appears euvolemic on exam. Urine lytes show a
sodium of 40 mEq/L. What is the most likely etiology of
the hyponatremia?
a. AKI
b. Bartter syndrome
c. Inadequate intake of sodium
d. SIADH
e. Pseudohypoaldosteronism
Dr.Wahid Helmi Domiate Egypt

2. d. SIADH can be triggered by birth asphyxia. To make this diagnosis, renal, thyroid, and
adrenal function tests must be normal. Confirmation of the diagnosis is made by the finding
of urine osmolality higher than serum osmolality and urine sodium should be greater than
20. A finding of urine sodium less than 20 would be suspicious for intravascular volume
depletion lead-ing to an increase in urine sodium reabsorption. In such cases, ADH release is
appropriate to preserve volume even at the expense of plasma osmolality. A creatinine of 0.7
in a 3-day-old infant would be in the normal range, making AKI unlikely to be the cause of
the hyponatremia. Bartter syndrome is a tubular disor-der leading to salt wasting, polyuria,
and dehydration. Infants typically do not need sodium supplementation until day of life 3.
Pseudohypoaldosteronism presents with low sodium and high potassium levels.
3. An ex-25-week-old infant is now 11 days old. Her
serum sodium has trended down from 137 to 126. Her
creatinine is 0.8. Her TF are written as 130 mL/kg/d.
Her TPN has 3 mEq/kg/d of sodium with an additional
1 mEq/kg/d provided in her carrier fluids. You obtain
urine lytes that show a sodium of 80 and a creatinine
of 10. What is her FENa and how does it explain her
serum sodium?
a. 12.6%, tubular wasting from immature tubule
b. 7.9%, tubular wasting from acute tubular necrosis
c. 5%, tubular wasting from prematurity
d. 2.5%, prerenal AKI
e. 0.05%, inadequate Na intake
3. c. The baby’s FENa is calculated as [(UNa x SCr)/(UCr x SNa)] x 100 = (80 × 0.8)/(10 ×
126) = 0.05 × 100 = 5%. While term babies conserve sodium well with FENa < 1 and often
closer to 0.5%, preterm infants’ imma-ture tubules lead to sodium wasting and high FENa. A
creatinine of 0.8 in an 11-day old ex-25 week infant would fall in the normal range so AKI is
unlikely
Dr.Wahid Helmi Domiate Egypt

.4. The lab calls with a critical potassium value of 7.2 in a4-day-old ex-26-week infant.
His weight is 15% below birth weight. The rest of the chemistry panel shows a
Na of 151 and a bicarbonate of 10. Which of the following
mechanisms would not perpetuate hyperkalemia?
a. Avid sodium reabsorption by the proximal tubule
secondary to volume depletion
b. Lack of Maxi-K channels in neonates
c. Inhibition of ROMK channels by acidosis
d. Neonates’ blunted response to aldosterone
e. Acidosis stimulating Na-K ATPase, which increases
the intracellular to urine gradient.
4. e. Alkalosis stimulates the Na-K ATPase on the basolat-eral membrane. This will increase
intracellular potas-sium levels and stimulate potassium exit to the urine space. Alkalosis also
increases the amount of time the luminal potassium channels are open further encour-aging
urine potassium secretion and leading to hypo-kalemia. Avid sodium reabsorption in the
proximal tubules as seen in volume depleted states leads to less sodium delivery to the distal
tubule. The reabsorption of sodium by ENaC in this segment leaves the lumen
electronegative which allows for secretion of potas-sium. Neonates do initially lack the
potassium Maxi-K channels (although these are stimulated by high urine flow which would
not be present in the baby in the sce-nario). The potassium channel ROMK is inhibited by
acidosis which would lead to less potassium secretion to the urine. Although aldosterone
levels are high in infants, they exhibit less of a response to it.

5. A 33-week cGA infant has been on thiazides for about


6 weeks for CLD. She develops acute vomiting. A chemistry
panel shows a bicarbonate of 32 with potassium
of 3.0 and creatinine of 1.1. A blood gas confirms metabolic
alkalosis. What mechanism is not involved in
stimulating or maintaining the alkalosis?
a. Vomiting leads to loss of protons.
Dr.Wahid Helmi Domiate Egypt

b. Vomiting leads to decrease in intravascular volume,


which results in a decrease in GFR.
c. Hypokalemia limits the shift of H+ out of cells.
d. Intravascular depletion stimulates proximal Na
reabsorption, which leads to increased distal H+
secretion.
e. Intravascular depletion stimulates RAS driving further
potassium loss.
5. d. In a state of volume depletion, alkalosis will persist by the following three mechanisms:
(1) a decrease in GFR leads to less filtered bicarbonate; (2) the proximal tubule avidly
reabsorbs Na which leads to increased bicarbonate reabsorption; and (3) the volume
depletion stimulates RAS which leads to an increase in aldoste-rone. The aldosterone leads
to increased activity of the epithelial Na channel (ENaC), making the lumen more
electronegative. This favors both H+ and K+ secretion. Typically in alkalosis, H+ would
shift out of the cell in exchange for K+. Hypokalemia inhibits this process.
6. A 1-week-old ex-37-week infant is admitted with the
following chemistry: Na 127, K 7.3, bicarbonate 15.
His mother reports he is not very interested in feeding
but has been making a normal amount of wet diapers.
His weight is 12% below his birth weight. His father
reports that he has been told he was often admitted to
the hospital as a baby but he doesn’t know any details.
He is currently well and not on any medications. What
advice could be given to the family about what to
expect as their child grows up?
a. Resolution of symptoms by age 2
b. A chronic cough
c. Persistent rashes
d. Growth failure
e. A need for salt supplementation throughout life
Dr.Wahid Helmi Domiate Egypt

6. a. The baby has pseudohypoaldosteronism type 1, autosomal dominant type. PHA type 1
presents with vomiting, FTT, hyponatremia, hyperkalemia, and aci-dosis with high urine
sodium. Renin and aldosterone levels are high. It can be inherited as an autosomal recessive
or dominant trait. The dominant form is clini-cally outgrown, usually by age 2. It is limited
to renal findings. In contrast, the recessive form is a life-long, systemic illness involving the
skin, lungs, and kidney predominantly.
7. A 950-gram infant is noted to have a 6% loss of weight
from birth weight. His UOP has increased from dol 1 to
2. His electrolytes and creatinine are normal for age.
So far, he has been on 90 mL/kg/d TF without any Na
or K in the TPN. Which statement below regarding his
fluid/electrolytes management is correct?
a. Increase TF to 130 mL/kg/d since he is losing too
much weight.
b. He is at a decreased risk of a PDA based on the numbers
above.
c. His risk of NEC is higher than an infant of similar
BW who did not lose weight.
d. Adding 1 mEq/kg/d potassium to the TPN would be
dangerous.
e. Your management has increased his risk of chronic
lung disease.

7. b. A weight loss of up to 15% is normal in premature infants. Studies have shown that not
allowing this weight loss to occur is associated with increased rates of PDA, NEC, and CLD.
Potassium should be withheld from the TPN until urine output is established. The baby in
the above scenario is urinating normally and has a normal creatinine, so standard of care
would be to add potassium to his TPN. His TF should be increased to allow more nutrition,
not because of the expected and normal weight loss
.8. A FT infant with bilateral pneumothoraces develops respiratory acidosis. Regarding
neonatal respiratory acidosis, which statement is true?
Dr.Wahid Helmi Domiate Egypt

a. Preterm infants can respond to respiratory acidosis


by increasing their tidal volume.
b. Providing a dose of bicarbonate using the following

formula is helpful: base deficit × BW × 0.3.

c. Renal compensation is as efficient as in older children.


d. Intubation may be needed.
e. Administered bicarbonate stimulates hyperventilation.
8. e. Term infants increase both respiratory rate and tidal volume, but preterm infants
increase only their respi-ratory rate. Bicarbonate is not helpful to treat respi-ratory acidosis
because it decreases respiratory rate. Preterm infants have a decreased ability to reabsorb
filtered bicarbonate
.9. A 1-month-old ex-33-week infant has been having
difficulty gaining weight. Her feeds have been fortified
to 28 kcal/oz but she still gains <5 g/d. Her mother
reports her son had a similar issue with weight gain.
You obtain a chemistry panel that shows: Na 137, K
4.1, bicarbonate 12, BUN 14, creatinine 0.3, glucose
85, Ca 9.4, phosphorus 6.0. A urine sample has a specific
gravity of 1.010, pH 5.4, and is negative for blood,
protein, LE, or nitrates. What is the baby’s diagnosis?
a. Distal RTA
b. Type IV RTA
c. Hemolyzed blood sample
d. Proximal RTA
e. Pseudohypoaldosteronism
9. d. Proximal RTA is most commonly seen in children as part of the renal Fanconi
syndrome with wasting of glucose, amino acids, and phosphorus. However, rare genetic
causes have been reported. With proximal RTA the defect lies in the reabsorption of the
filtered bicar-bonate. The distal acidification mechanism is intact. When the serum
bicarbonate is low, the distal acidifi-cation results in a low urine pH. When alkali therapy is
started, the filtered bicarbonate load increases which raises the urine pH. Therapy of
Dr.Wahid Helmi Domiate Egypt

proximal RTA requires large amounts of alkali, often 10–15 mEq/kg/day. Dis-tal RTA is
due to an acidification defect. It is suggested by a normal AG acidosis with urine pH > 6.5.
Hyper-calciuria is often present. Type IV RTA presents with mild acidosis (bicarbonate
levels in high teens) and hyperkalemia. Technical difficulties obtaining blood samples can
lead to factitiously low bicarbonate levels but hyperkalemia and hyperphosphatemia should
also be seen in that case. Pseudohypoaldosteronism can have a mild metabolic acidosis but
hyperkalemia and hyponatremia are major features of the disease.
10. You are treating an ex-27-week infant for a moderate
sized PDA. The baby has developed worsening
pulmonary edema despite restricting fluids. You start
indomethacin. After the second dose, the baby’s urine
output is noted to drop from 1 to 0.6 mL/kg/hr. Which
statement explains the drop in UOP?
a. Release of ANP has caused dilation of the efferent
arteriole.
b. Indomethacin has disrupted the intrarenal production
of prostaglandins.
c. Ang II is constricting the afferent arteriole only.
d. The indomethacin has closed the PDA resulting in
shunting of blood away from the kidneys.
e. Dopamine is stimulating NA-K ATPase.
10. b. Renal prostaglandins play a crucial role in main-taining GFR when the kidney is not
receiving normal blood flow due to volume depletion, hemorrhage, etc., or when angiotensin
II levels are high. NSAIDs disrupt the production of prostaglandins and can lead to AKI.
ANP is released by stretch of the atrium. It acts on the kidney to increase GFR by dilating
the afferent arteriole and constricting the efferent arteriole. Angiotensin II constricts both the
afferent and efferent arterioles. If the PDA closed, blood would stop shunting away from the
kidneys. Dopamine is natriuretic by decreasing the activity of Na-K ATPase which provides
the driving force for Na reabsorption.
Dr.Wahid Helmi Domiate Egypt

Parenteral Nutrition
1. Which of the following statements regarding TPN and enteral neonatal nutrition is
FALSE?
a. Normal fetal metabolic and growth rates and nutritional requirements stop with birth,
precluding routine use of IV nutrition.
b. The smaller and more preterm and less developed the infant, the less body stores (protein,
fat, glycogen) are available to provide nutrients for metabolic needs.
c. The metabolic and thus nutrient requirements of the newborn are equal to or greater than
those of the fetus of the same gestational age.
d. First-week protein and energy intakes are associated with improved 18-month
developmental outcomes in preterm infants.
1. a. Normal fetal metabolic and growth rates and nutritional requirements stop with birth,
precluding routine use of IV nutrition. This is False. There is no reduction in metabo-lism or
potential for growth with birth, and thus the infant will require the same amount of nutrients.
If the infant is very small, there will be insufficient nutrient stores to maintain normal
metabolism or growth, requiring early IV nutrition and early and continued enteral nutrition.
2. Which of the following statements regarding TPN is FALSE?
a. The amino acid composition of current neonatal parenteral amino acid solutions is based
on providing plasma concentrations similar to those of term, fully
breastfed infants, which is appropriate for preterm infants who are growing at much faster
rates than term infants.
b. A standard parenteral infusion of 3 g/kg/day amino acids, 10% dextrose, and 3
grams/kg/day lipid at 100mL/kg/day would provide 82 kcal/kg/day, according

to the following calculations: 3 grams amino acids × 4 kcal/gram = 12 kcal; 10 grams of

dextrose × 4kcal/gram = 40 kcal; 3 grams/kg/day of lipid would

add 30 kcal/kg/day.
c. Cholestasis or parenteral nutrition associated
liver disease (PNALD) is more common in infants
Dr.Wahid Helmi Domiate Egypt

(extremely preterm infants, and those with short gut


syndromes such as gastroschisis or those with severe
enteral feeding intolerance) who receive parenteral
nutrition exclusively and for prolonged periods
(weeks, vs. days).
d. The principal metabolic complication of parenteral
nutrition is hyperglycemia.

2. a. The amino acid composition of current neonatal paren-teral amino acid solutions are
based on providing plasma concentrations similar to those of term, fully breast fed infants,
which is appropriate for preterm infants who are growing at much faster rates than term
infants. This is False. Term infants do not have as high a fractional pro-tein synthetic rate or
growth rate as the earlier gestation fetus does and thus the preterm infant of the same gesta-
tional age should. Fetal amino acid concentrations are generally higher than those of term
infants fed mature mother’s milk, an indication of their need for more amino acid and
protein intake to meet their greater requirements.
Dr.Wahid Helmi Domiate Egypt

Enteral Nutrition
1. Which of the following is a TRUE statement regarding human milk being the ideal
enteral diet for human neonates?
a. It provides sufficient energy, protein, fat, carbohydrate,micronutrients, and water for
normal metabolism,growth, and development in term infants.
b. It does not require supplements of protein and certain minerals.
c. It is enriched in nutrients when it comes from a milk bank as mature donor milk.
d. It is associated with delayed neurodevelopmental outcomes compared with term formulas.
1. a. True, for term infants. Human milk provides sufficient energy, protein, fat,
carbohydrate, micronutrients, and water for normal metabolism, growth, and development in
term infants. Human milk has been the result of sev-eral million years of evolutionary
development to pro-duce the optimal nutrition of human newborn infants.
2. Which of the following about mother’s own milk produced at home or in the NICU
and stored is TRUE?
a. Varying degrees of nutrient loss may occur with long-term storage, particularly in frozen
conditions.
b. Vitamin C loss does not occur, even during the process
of feeding freshly expressed human milk by bottle.
c. For multiple human milk components, significant
degradation generally occurs with even very shortterm
storage and multiple freeze-thaw cycles.
d. Donated human breast milk does not contain pathogenic
microorganisms that could be transferred to
the infants who consume it.
2. a. Varying degrees of nutrient loss many occur with long term storage, particularly in
frozen conditions. This is true, especially when there are multiple freeze-thaw cycles, and is
one reason why fresh mother’s milk is superior.
Dr.Wahid Helmi Domiate Egypt

3. Which of the following statements about heat treatment


(pasteurization) of human milk are FALSE?
a. Heat treatment (pasteurization) of human milk may reduce the concentration and
functional capacity of many bioactive components.
b. Holder pasteurization does not adversely affect the protein content of donor milk (average
value of about −4%).
c. Total lipid generally is increased (as much as by 60%), but free fatty acids are reduced,
which might increase their nutritive potential.
d. No significant reductions in lactose have been noted,both as free molecules and as part of
biologically active compounds such as oligosaccharides.

Minerals, Vitamins, and Trace Minerals


1. Which of the following is TRUE about calcium and phosphorus
supplements for preterm infants?
a. Calcium and phosphorus are readily absorbed from
the preterm infant GI tract and do not need to be supplemented.
b. Calcium and phosphorus are easily soluble in IV nutrition solutions.
c. Recommended intravenous intakes of calcium are
higher than enteral requirements, ranging from 150 to 220 mg/kg/day for calcium.
d. Risk factors in preterm infants for calcium and phosphorus
deficiency and subsequent rickets include:
gestational age <27 weeks or birth weight <1000
grams, long-term parenteral nutrition (>4–5 weeks),
severe BPD requiring diuretics and fluid restriction,
long-term steroid treatment, a history of necrotizing
enterocolitis (NEC), and intolerance to enteral formula
or human milk.
1. d. True. Risk factors in preterm infants for calcium and phosphorus deficiency and
subsequent rickets include: gestational age <27 weeks or birth weight <1000 grams, long-
Dr.Wahid Helmi Domiate Egypt

term parenteral nutrition (<4-5 weeks), severe BPD requiring diuretics and fluid restriction,
long-term steroid treatment, a history of necrotizing enterocolitis (NEC), and intolerance to
enteral formula or human milk. All of these are true and self-explana-tory. Calcium and
phosphorus nutrition of very preterm infants is deficient for all of these reasons.
2. The following statement about potential manganese (Mn) toxicity is true:
a. Potential nephrotoxicity.
b. Mn supplementation should be stopped with any signs of hepatic dysfunction or
cholestasis.
c. The contaminant levels of Mn in parenteral nutrient solutions are too low to meet
requirements without additional supplementation.
d. Parenteral calcium gluconate to prevent osteopenia does not contribute to Mn
contamination and potential toxicity.
2. b. Mn supplementation should be stopped with any signs of hepatic dysfunction or
cholestasis. This is true, as excess Mn damages the liver.
Dr.Wahid Helmi Domiate Egypt

Nutrition and Growth: The Fetus


1. During the third trimester of fetal growth, the largest fractional increase in body
composition is the result of:
a. water
b. fat
c. brain
d. lean body mass
1. d. True. Lean body mass, including structural compo-nents of cells that have been
proliferating, muscle, and bone account for the largest fractional increase in body weight,
indicating the large nutritional requirements for protein during this period of growth.
2. Essential amino acids are:
a. used for oxidation
b. used for protein synthesis
c. used for growth of the brain
d. required as part of nutrient supplies
e. all of the above
2. e.
3. Which of the following regarding neonatal nutrition is false?
a. Rates of protein synthesis and the requirements for amino acid and protein supply to meet
them are the same in preterm infants as in normally growing fetuses of the same gestational
age.
b. The critical amount of amino acid or protein nitrogen for preventing growth faltering in
very preterm infants <30–32 weeks is less than 1.5 g/kg/day.
c. There is no advantage to protein metabolism or growth (body weight or composition) with
more than 3.5–4.0 g/kg/day parenterally or 4.0–4.5 g/kg/day enterally in very preterm
infants.
d. Excess amino acid or protein intake always produces higher blood urea and ammonia
concentrations but these seldom are in toxic ranges.
Dr.Wahid Helmi Domiate Egypt

3. b. False. The critical amount of amino acid or protein nitro-gen for preventing growth
faltering in very preterm infants <30-32 weeks is less than 1.5 g/kg/day. This is false, as
numerous studies have shown that growth cannot occur until at least 1.5 g/kg/day of amino
acids is provided.

Inborn Errors of Metabolism

1. A 3-day-old girl began feeding poorly on day two of life. She now has depressed
mental status, clonus, and respiratory alkalosis. Which test should be sent for
emergently?
a. Very long-chain fatty acids
b. 7-dehydrocholesterol
c. Ammonia
d. Lysosomal enzymes
e. Cholesterol
1. c. Infants with urea cycle defects usually become symp-tomatic in the first days to months
of life. Clinical pre-sentation includes: lethargy, hypothermia, poor feeding, tachynea,
irritability, vomiting, coma. Differential diag-nosis includes sepsis or an organic acidemia.
Hyperventi-lation due to cerebral edema causes respiratory alkalosis.
2. A newborn male has profound hypotonia and features suggestive of trisomy 21. He is
found to have hepatomegaly,hyperbilirubinemia, and synthetic liver dysfunction.
Which diagnosis should be considered?
a. Zellweger syndrome
b. Mucopolysaccharidosis
c. Smith-Lemli-Opitz syndrome
d. Pyruvate carboxylase deficiency
e. Krabbe disease
2. a. Zellweger spectrum disorder (ZSD), is a disorder of peroxisomal biogenesis caused by
a defect in the PEX gene. A newborn with ZSD presents with dysmorphic features which
includes; flattened face, epicanthal folds, upslanting palpebral fissures, broad nasal bridge.
and hypoplastic supraorbital ridges with severe hypotonia and hepatic dysfunction.
Dr.Wahid Helmi Domiate Egypt

3. A breast-fed newborn girl has high anion-gap acidosis,hypoglycemia and altered


mental status. Urinalysis reveals 4+ ketones. The ketosis is suggestive of:
a. Normal response to hypoglycemia
b. Fatty acid oxidation defect
c. Peroxisomal defect
d. Organic acidemia
e. Lysosomal storage disease
3. d.
4. Poorly treated maternal phenylketonuria puts a fetus at risk for:
a. having PKU
b. cataracts
c. intracranial calcifications
d. limb defects
e. microcephaly
4. e. Uncontrolled maternal phenylketonuria leads to com-plications in the offspring. These
complications include microcephaly, intellectual disabilities, congenital heart disease, and
intrauterine growth restriction. Dietary management prior to conception and during
pregnancy is important to decrease complication risks in the offspring.
Dr.Wahid Helmi Domiate Egypt

Evaluation of Infants With Congenital Anomalies


1. Several studies have shown that there is a small but definite
increase in congenital malformations in neonates
following artificial reproductive technology (ART) conception.
In which type of ART is an increased incidence
of urogenital abnormalities, specifically hypospadias,
likely to occur?
a. In vitro fertilization (IVF)
b. Gamete intrafallopian tube transfer (GIFT)
c. Intracytoplasmic sperm injection (ICSI)
d. Embryo cryopreservation
e. Blastocyst culture
1. c. Current evidence suggest an association between ART and small but definite (more
than 1.3 times than in spontaneous conception) incidence of congenital malformations. This
includes: congenital heart defects, neural tube defects, facial cleft, gastrointes-tinal
malformations, genitourinary malformations, and imprinting disorders. The rates of
congenital malformations are similar for each type of ART, except for increased urogenital
abnormalities with ICSI. The cause for the increase in malformations with ART are yet to be
determined.
2. Amniotic band sequence is caused by:
a. deformation
b. disruption
c. malformation
d. syndrome
e. association
Dr.Wahid Helmi Domiate Egypt

2. b. Amniotic band sequence is an example of disrup-tion. Disruption defects are due to


destruction or interruption of a normal developmental process. It usually affects a body part,
rather than a specific organ

3. Deformations most commonly occur during:


a. the zygote period
b. the blastula period
c. the embryo period
d. the fetal period
e. the birth process
3. d. Deformations are defects caused by abnormal mechanical forces on morphologically
normal tissue in utero. They are associated with multiple gestations, uterine malformations,
and oligohydramnios
4. Arthrogryposis (congenital joint contractures) can be described as a:
a. dysplasia
b. syndrome
c. disruption
d. deformation
e. none of the above
4. d. Deformations result when normal tissue is exposed to abnormal mechanical forces in
utero.

Nongenetic Etiologies for Congenital Defects


1. The diagnosis of partial fetal alcohol syndrome requires:
a. short palpebral fissures
b. smooth philtrum
c. thin upper lip
d. central nervous system abnormalities
e. evidence of prenatal alcohol exposure
Dr.Wahid Helmi Domiate Egypt

1. e. Diagnosis of partial fetal alcohol syndrome (PFAS) requires evidenced of prenatal


alcohol exposure.

Patterns of Congenital Disorders


1. A 38-weeks-gestation newborn infant at birth is noted to be small for gestational age,
have microcephaly, cleft lip and palate, narrow hyperconvex fingernails, developed
central apnea shortly after birth, and an echocardiogram showed ventricular septal
defect (VSD). This infant most likely has:
a. Trisomy 21
b. Fetal alcohol syndrome
c. Trisomy 13
d. Trisomy 18
e. Prader-Willi syndrome
1. c. Trisomy 13 (Patau syndrome), incidence is 1 in 5,000–10,000 live births, with 95% of
trisomy 13 conceptions resulting in spontaneous abortion; 90% born live die within the first
year of life. Abnormalities include: cleft lip and palate, polydactyly, narrow hyperconvex
fingernails, colobomas, umbilical or inguinal hernia, cryptorchi-dism, microcephaly,
holoprosencephaly, seizures, cardiac defects (VSD), and apnea.
2. A full-term, appropriate for gestational age (AGA) male infant has persistent
hypoglycemia despite adequate calorie feeds, orally. He is also noted on physical
examination to have a large tongue and exophthalmos. What is the mode of inheritance
of this infant’s condition?
a. Autosomal dominant
b. Uniparental disomy
c. Autosomal recessive
d. X-linked recessive
Dr.Wahid Helmi Domiate Egypt

e. X-linked dominant
2. b. 20% of cases of Beckwith-Wiedemann syndrome is caused by paternal uniparental
disomy (UPD).

Molecular Genetics
1. A male infant has hypotonia, poor feeding, and cryptorchidism.You suspect Prader-
Willi syndrome (PWS).Evaluation of the 15q11-13 region does not reveal any
deletions. You should next evaluate for:
a. point mutations in the PWS critical region
b. duplication of the 15q11-13 region
c. uniparental disomy of the 15q11-13 region
d. other causes of neonatal hypotonia
1. c. Uniparental disomy in the region of chromosome 15q11-13. PWS is caused by lack of
the paternally inherited genes in the 15q11-13 region. PWS can occur secondary to deletion
of the 15q11-13 region on the paternally inherited chromosome. Alternatively, PWS can be
due to inheritance of both copies of 15q11-13 from the mother (uniparental disomy)

2. Diseases with this mode of inheritance are almost exclusively transmitted from the
mother:
a. imprinted
b. autosomal recessive
c. trinucleotide repeats
d. mitochondrial
2. d. Mitochondria are organelles that have their own chromosome which encodes several
genes essential for mitochondrial function. Mitochondrial DNA is inherited almost
exclusively from the mother.
Dr.Wahid Helmi Domiate Egypt

3. You are asked to evaluate an infant for hypotonia. You note that the infant has
significant hypotonia and feeding difficulties. The mother is well-appearing, but has
difficulty releasing your handshake. Both the mother and infant are subsequently
diagnosed with myotonic dystrophy. The phenomenon described here, where
the infant is more severely affected than the mother, is
known as:
a. anticipation
b. imprinting
c. mitochondrial inheritance
d. uniparental disomy
3. a. Anticipation occurs in trinucleotide repeat diseases. Trinucleotide repeats tend to be
unstable, with their size increasing with each subsequent generation, resulting in a more
severe phenotype.
4. You are called to evaluate an infant with tetralogy of Fallot, cleft palate, and
hypocalcemia. After your initial evaluation, you suspect the child may have DiGeorge
syndrome. What is the most appropriate initial test to order?
a. Karyotype
b. Fluorescence in situ hybridization (FISH)
c. Whole exome sequencing (WES)
d. Sequencing of the TBX1 gene
4. b. Of the choices listed, FISH for 22q11 is the most appropriate test to evaluate for
suspected DiGeorge syn-drome. Although not listed, aCGH would also have been an
appropriate choice. A karyotype does not have suf-ficient resolution to reliably detect the
22q11 microdele-tion. A small fraction (<5%) of patients with DiGeorge syndrome will have
normal cytogenetic studies. In those patients, targeted evaluation of DiGeorge locus genes,
such as TBX1, may be indicated. WES is not indicated in the evaluation for DiGeorge
syndrome.
5. Which of the following is a limitation of newborn screening?
a. Preterm or critically ill neonates often have falsepositive
results.
b. Current screening techniques can only evaluate for a
Dr.Wahid Helmi Domiate Egypt

few metabolites on each sample.


c. Cystic fibrosis is not reliably detected by most newborn
screening programs.
d. Many states do not screen for sickle cell disease.
5. a. Preterm or critically ill term infants are more likely to have false positive newborn
screening results. Most newborn screening programs utilize tandem mass spectrometry,
which can analyze multiple metabolites, simultaneously enabling screening for many
disorders with a single blood sample. Newborn screening panels do vary from state to state
but all states screen for sickle cell disease and cystic fibrosis.

Assisted Ventilation , ECMO,and Pharmacologic Agents


1. Which of the following are clinical situations for which
ECMO is indicated?
a. A neonate with transposition of the great arteries who has undergone a
successful arterial switch palliation but is unable to maintain adequate blood
pressure and oxygenation immediately postoperatively
b. A 4-day-old with trisomy 21 and overwhelming sepsis
and pneumonia on maximal ventilatory support,bilateral pneumothoraces, and an
oxygenation index of 60 in spite of maximal medical therapy
c. A 30-week-old, 1200-g infant with severe RDS unresponsive
to multiple doses of surfactant on optimal ventilator support with an oxygenation
index of 50
d. A full-term infant with rupture of membranes at 18weeks and persistent severe
oligohydramnios whose parents chose to continue the pregnancy, who presents
in the delivery room with severe increased work of breathing. The infant remains
hypoxic with pO2 20 mm Hg and pCO2 110 mm Hg in spite of aggressive
maximal cardiorespiratory support. A cardiac ECHO reveals normal anatomy, right
ventricular dysfunction and significant right to left shunting at the ductal and atrial
level. CXR suggests pulmonary hypoplasia.
Dr.Wahid Helmi Domiate Egypt

1. a and b. ECMO is used for reversible conditions unre-sponsive to less invasive treatment
options. Given the need to anticoagulate and the technical limitations related to cannulation,
it is not standard of care to attempt ECMO in infants < 34 weeks and < 1800–2000 g. It is
also important to assess if a condition is reversible before initiating ECMO. Case (d)
describes an infant with pulmonary hypertension related to severe pulmonary hypoplasia
which developed as a result of prolonged anhydramnios during an important stage of
pulmonary development early in pregnancy. The pulmonary hyper-tension is related to
structural maldevelopment of the pulmonary capillary bed, resulting in a poor prognosis.

2. You are considering initiating iNO therapy for a37-week-old infant of a


diabetic mother with severe RDS. Which of the following is not indicated prior
to initiation of iNO?
a. Cardiac ECHO
b. CXR
c. Optimized lung expansion and ventilator support
d. Surfactant replacement
e. Steroid therapy
2. e. Infants of diabetic mothers can have delayed matu-ration of the surfactant, resulting in
increased inci-dence of RDS at an older gestational age. Cardiac ECHO is indicated to rule
out cardiac defect and confirm diag-nosis of pulmonary hypertension. CXR assesses lung
expansion and possible presence of pneumothorax. Surfactant replacement is indicated
before iNO when treating RDS. Steroid therapy has not been shown to benefit infants with
RDS, with or without pulmonary hypertension.
Dr.Wahid Helmi Domiate Egypt

Apnea of Prematurity and Neonatal Respiratory Depression


1. A 32-year-old pregnant mother at 28 weeks’ gestation is referred to you for a
neonatology prenatal consult.According to the obstetrician, the fetus has reduced fetal
movements, thin bones, and polyhydramnios. The ultrasound also notes the near total
absence of fetal breathing. How will you explain the significance of the
lack of fetal breathing to the mother?
a. Fetal breathing is not significant indicator of fetal health
b. Lack of fetal breathing implies severe periodic breathing in neonate after birth
c. Regular fetal breathing at the rate of 30–40/minute
is essential for establishment of neonatal breathing
d. Fetal breathing is essential for lung development
1. d. Fetal breathing is essential for lung development. Fetal breathing movements begin in
the first trimester and are initially “tonic” with irregular tonic contrac-tion of diaphragm and
prolonged periods of apnea. The breathing becomes phasic in the third trimester but is still
associated with apneic periods. Lack of fetal breath-ing due to neuromuscular diseases or
ineffective fetal breathing movements (e.g., when the anterior chest wall is experimentally
replaced with a silicone membrane) are associated with pulmonary hypoplasia. The fetus in
the question is likely to have significant pulmonary hypopla-sia which will complicate the
postnatal ventilatory man-agement and may not be compatible with prolonged life.
Dr.Wahid Helmi Domiate Egypt

2. You are doing teaching rounds with medical students and are demonstrating
periodic breathing in a 28-week preterm infant who is 2 weeks old. How will you best
describe the response of this infant to hypoxia?
a. Immediate and persistent hyperventilation
b. Biphasic response, early ventilatory depression and later hyperventilation
c. Biphasic response, early hyperventilation and later ventilatory depression
d. No predictable response
2. c. Biphasic response to hypoxia is characteristically noted in preterm infants, especially in
those with apnea of prematurity. Adults normally respond to hypoxia with sustained increase
in rate and depth of breathing. In preterm infants, there is an initial increase in the rate of
breathing which is thought to be the result of periph-eral chemoreceptor stimulation. After
1–2 minutes, though, a period of respiratory depression follows. This is primarily due to
reduced rate rather than reduced tidal volume and is thought to be due to hypoxia mediated
central ventilatory depression. The mediators for such ventilatory depression include
adenosine, GABA, and endorphins. Such inhibition of ventilation is noticed in the fetus too,
leading to the hypothesis that the purpose of the inhibition is to reduce the metabolic rate,
which, in a fetus dependent on placental oxygen delivery, will lead to reduce oxygen
demand.
Pleural Disorders and Additional Causes of Respiratory Distress
1. A term infant has inspiratory stridor noticed in the normal nursery a few hours after
birth. The stridor was most noticeable when the infant cried. There were mild
retractions but no oxygen desaturation. Mother had gestational
diabetes during pregnancy. Infant was born by spontaneous vaginal delivery at 39
weeks after forcepsassisted delivery with a birth weight of 4.3 kg. What is the most
likely diagnosis:
a. unilateral vocal cord paralysis
b. subglottic stenosis
c. laryngomalacia
d. hemangioma
Dr.Wahid Helmi Domiate Egypt

1. a. Unilateral vocal cord paralysis is the second most cause of stridor in infants. This is
more common on the left side following damage to the recurrent laryngeal nerve. Surgery in
the neck or the chest is the most com-mon cause of the palsy, but trauma around birth, espe-
cially with the use of forceps, is also an important cause. Subglottic stenosis is usually
acquired and is not pres-ent at birth while laryngomalacia tends to present after the first few
days of life with the symptoms peaking at around 4–6 weeks of age. Similarly, hemangioma
also presents after first few weeks once the tumor enters the proliferative phase.

2. A newborn was born at 36 weeks following spontaneous labor. Mother had late onset
of prenatal care and the fetus was noted to have hydrops. At birth, the infant was noted
to have poor respiratory effort and hydrops with skin edema and abdominal distention.
Resuscitation included drainage of fluid from the chest and abdominal cavities and the
infant was admitted to the NICU. Examination in the NICU reveals dysmorphic
features, downsloping palpebral fissures, webbed neck, dysplastic pulmonary valve,
and cryptorchidism. Analysis of pleural fluid revealed serosanguinous fluid with
lymphocyte predominant cell count. What is the likely diagnosis?
a. Congenital empyema
b. Immune hydrops
c. Cardiac failure
d. Chylothorax
2. d. Infant likely has Noonan syndrome with chylotho-rax. Noonan syndrome is associated
with lymphatic abnormalities and is a common cause of congenital chylothorax. Fluid in
congenital chylothorax is sero-sanguinous prior to initiation of feeds and may turn milky
with onset of feeding. Diagnosis of chyle includes demonstration of lymphocyte
preponderance (contrib-utes to milky color), chylomicrons in the fluid, fluid triglycerides
>110 mg/dL and pleural fluid/serum cholesterol <1. Cardiac failure associated with anemia
(immune and nonimmune) and cardiomyopathy can also present with fetal hydrops and
serosanguinous fluid, but usually do not have lymphocyte preponder-ance of cell count.
Empyema has high neutrophil count and protein in the fluid and can be caused by intra-
amniotic shunt placements.
Aspiration, Pneumonia,and Persistent Pulmonary Hypertension
Dr.Wahid Helmi Domiate Egypt

1. A 41-week-old infant is born after induction of labor.There was thick particulate


meconium at delivery. The neonatal resuscitation team that received the infant did
initial resuscitation by stimulating the infant, and when there was no respiratory effort,
initiated positive pressure ventilation (PPV). The infant eventually required
endotracheal intubation and admission to the NICU. Achest x-ray revealed coarse
bilateral opacities. Which of the following strategies is recommended in the further
management of this infant?
a. Lung lavage with dilute surfactant
b. Exogenous surfactant at standard doses
c. Respiratory alkalosis using high ventilator rates
d. Routine use of muscle relaxants
1. b. Lung lavage with dilute surfactant does not have clear evidence to support its routine
use. MAS is associated with inactivation of surfactant and use of surfactant has been
associated with less use of ECMO. Use of high ven-tilator rates can increase air-trapping and
increase the risk for pneumothorax. Further, hypocarbia can be det-rimental to cerebral
circulation. Muscle relaxants may be useful to improve oxygenation in severe MAS with
PPHN, but routine use of muscle relaxants is not recom-mended.
2. A preterm infant born at 26 weeks’ gestation is now 3 days old and you are
considering initiating trophic enteral feeds. Mom is known to be seropositive for
cytomegalovirus (CMV) and she asks you about the risk of transmission of CMV
through breast milk and the risk of pneumonia due to CMV. What is your response?
a. CMV is eliminated by freeze-thawing process.
b. CMV only rarely causes pneumonia, postnatally.
c. CMV can cause interstitial pneumonia in preterm
infants but can be eliminated by short-term pasteurization.
d. Provide preterm formula as the risk of pneumonia is very high.
2. c. CMV transmission and disease is rare in term infants as mother will likely have
transferred passive immunity to the infant during pregnancy. In preterm infants, pas-sive
transmission of immunity is incomplete and cannot be relied upon. In a prospective cohort
study of very low birth weight infants born to CMV seropositive mothers, the cumulative
incidence of CMV infection at 12 weeks was 6.9%. About 17% of the infected infants
Dr.Wahid Helmi Domiate Egypt

developed disease. Sepsis-like syndrome, interstitial pneumonia, hepatitis, and necrotizing


enterocolitis are important clinical presentation of CMV disease in preterm infants. Holder
pasteurization (62.5°C [144.5°F] for 30 minutes) and short-term pasteurization (72°C
[161.6°F] for 5 sec-onds) of expressed breast milk can inactivate CMV. It is likely the short-
term pasteurization will preserve more nutrients than the Holder method. Freezing of
expressed milk does not reliably eliminate CMV.

Respiratory Distress Syndrome


1. Surfactant proteins B and C are essential components for surfactant replacement
because they:
a. Increase surfactant spreading at the air-liquid interface.
b. Coat bacteria and viruses to increase innate immunity.
c. Detoxify oxygen radicals and reduce injury to alveolar type II cells.
d. All of the above.
1. a. Increase surfactant spreading at air liquid inter-face. Surfactant proteins are categorized
as either hydrophobic (SP-B and SP-C) or hydrophilic (SP-A and SP-D). The hydrophobic
proteins are secreted with surfactant in lamellar bodies and are required for production of
lamellar bodies and for the normal spreading of surfactant phospholipids into a mono-layer
at the alveolar air-liquid interface. SP-B and SP-C are developmentally regulated. SP-A and
SP-D are collectins that bind to carbohydrates on the sur-faces of bacteria, fungi and viruses.
They are develop-mentally regulated and are important components of the lung host defense
system.
2. Which of the following recommendations about postnatal glucocorticoid use in BPD
are contained in the 2010 AAP policy statement?
a. Therapy with high-dose dexamethasone cannot be recommended.
Dr.Wahid Helmi Domiate Egypt

b. There is insufficient evidence to make a recommendation regarding treatment with low-


dose dexamethasone.
c. Early hydrocortisone treatment may be beneficial in a specific population of patients;
however, there is insufficient evidence to recommend its use for all infants at risk of BPD.
d. All of the above.
2. d. All of the above. The 2010 AAP policy statement on use of glucocorticoids to mitigate
BPD does not recommend high dose dexamethasone because of the potential for adverse
neurologic outcome. No subsequent policy has been issued. Both low dose dexamethasone
and low dose hydrocortisone need more randomized controlled trials, particularly focusing
on neurodevelopmental out-come, before they can be recommended. Short courses of low
dose glucocorticoids are commonly used to treat BPD in ventilator- dependent patients.
Their use should balance the potential benefits of these agents with poten-tial harm they may
cause. The decision to use these drugs should be individualized for selected patients.

Pharmacologic Therapy of Heart Disease


1. An infant with a large ventricular septal defect and congestive heart failure is being
medically managed with digoxin, enalapril, and furosemide (Lasix). Spironolactone
was added yesterday due to persistent hypokalemia.Overnight, the infant developed
Mobitz II heart block. This is a toxicity of which medication?
a. Spironolactone
b. Digoxin
c. Enalapril
d. Lasix

1. b. Atrioventricular block is a well-described result of digoxin toxicity. Digoxin toxicity is


more likely to occur in the setting of hypokalemia
2. A 10-day-old infant is being treated for sepsis with antibiotics and dopamine. The
infant is requiring escalating doses of dopamine. All the interventions below can
increase the infant’s response to the current dose of dopamine, except for one. Which
intervention does not increase an infant’s sensitivity to catecholamines?
a. Beginning steroid administration
b. Normalizing pH
Dr.Wahid Helmi Domiate Egypt

c. Normalizing serum calcium levels


d. Normalizing potassium levels
2. d.

Electrocardiography,Electrophysiology, andDysrhythmias

1. An infant was born at full term to a 29-year-old woman with lupus by cesarean
section due to persistent fetal bradycardia. The newborn appears vigorous,with normal
capillary refill, blood pressure of 80/55 mm Hg, and 2+ femoral pulses. The heart rate,
however,is 65 beats/min, with a narrow QRS complex not associated with p waves.
What is the best course of action?
a. Begin isoproterenol
b. Begin cardiopulmonary resuscitation
c. Monitor closely on telemetry
d. Use transcutaneous pacing
1. c. This infant has complete heart block related to trans-placental transfer of SSA and/or
SSB antibodies related to maternal lupus. Although the heart rate is low, the infant has
adequate cardiac output, as evidenced by good capillary refill and blood pressure.
Monitoring is all that is required in this situation.
2. A 3-day-old infant suddenly develops tachycardia to 240 beats/min. Adenosine is
administered, which slows the ventricular rate briefly to 140 beats/min,
revealing a sawtooth-like pattern between the QRS complexes. Transesophageal pacing
is used to terminate the tachycardia. Which drug is best to use as prophylaxis
to prevent future tachycardia?
a. Propranolol
Dr.Wahid Helmi Domiate Egypt

b. No prophylactic therapy needed


c. Digoxin
d. Amiodarone
2. b. This infant had an episode of atrial flutter, which is very unlikely to recur and thus does
not warrant pro-phylaxis.

Cardiopulmonary Dysfunction
1. A term infant undergoes echocardiography after birth due to a murmur. Numerous
small cardiac tumors are identified throughout the heart. Which genetic disease is
likely?
a. Down syndrome
b. Williams syndrome
c. Tuberous sclerosis
d. Hunter syndrome
1. c. Numerous cardiac tumors found in a neonate are almost always rhabdomyomas, which
are strongly asso-ciated with tuberous sclerosis.

Presentation of Congenital Heart Disease


1. A 1-week-old infant with tetralogy of Fallot is undergoing
a rule-out sepsis evaluation. The infant’s baseline
saturations are 98%, with a 2/6 systolic ejection
murmur along the left sternal border. During the blood
draw, the infant’s saturations drop to 60% and the
infant becomes visibly cyanotic. On auscultation, the
lung sounds remain clear, but the murmur is no longer
audible. What interventions should be performed?
a. Intubation with 100% O2
Dr.Wahid Helmi Domiate Egypt

b. STAT echocardiogram
c. Arterial blood gas
d. Morphine and phenylephrine administration
1. d. The infant is this scenario is having a hypercyanotic spell caused by an acute decrease
in pulmonary blood flow. These are commonly triggered by medical proce-dures such as
blood draws. Morphine and phenylephrine are first-line agents to counteract the
hypercyanotic spell.
2. A newborn infant was found to have interrupted aortic
arch type B, with truncus arteriosus. What other features
is the infant likely to have?
a. Epicanthal folds with a single palmar crease
b. Elfin facies
c. Broad thumbs and toes
d. Absent thymus on chest x-ray and hypocalcemia
2. d. Both type B interrupted arch and truncus arteriosus are associated with 22q11 deletion
syndrome. Infants with 22q11 deletion syndrome frequently have hypo-calcemia and an
absent thymus
Cardiovascular Physiology
1. Physiologic responses to asphyxia include which of the following?
a. Tachycardia, decreased central venous pressure,
vasoconstriction of the cerebral vessels
b. Bradycardia, increased central venous pressure,
vasoconstriction of the skeletal muscle vessels
c. Bradycardia, decreased central venous pressure,
vasodilation of the cerebral vessels
d. Tachycardia, increased central venous pressure,
vasoconstriction of the skeletal muscle vessels
1. b.
Cardiac Development
1. An infant was born with truncus arteriosus. What embryologic event led to this
anomaly?
Dr.Wahid Helmi Domiate Egypt

a. Failure of the conotruncus to septate


b. Failure of the second heart field cells to migrate
c. Failure of the conotruncus to rotate
d. Abnormal development of the 6th arch
1. a. Truncus arteriosus results from failure of the conotruncus to separate.
2. The most common cyanotic congenital heart defect seen in infants of diabetic
mothers is which of the following?
a. d-Transposition of the great arteries
b. Hypoplastic left heart syndrome
c. Tricuspid atresia
d. Tetralogy of Fallot
2. a.

Asphyxia and Resuscitation


Asphyxia
1. You are called to the delivery of a 39-week infant, with no known maternal
complications of pregnancy. Meconium was noted at the time of delivery. You arrive in
the delivery room ∼1 minute after birth and find the infant is on a warmer, apneic, and
being vigorously stimulated by nursing staff. The HR is noted to be less than 100
bpm. Of the following, which is the most appropriate next step to improve the patient’s
heart rate?
a. Continue vigorous stimulation of the patient
b. Deep-suction the posterior pharynx
c. Provide positive-pressure ventilation via bag-mask ventilation
d. Intubate the patient
e. Obtain umbilical access and administer intravenous epinephrine
1. c. This patient is likely in secondary apnea. The appro-priate next step is to provide
effective positive-pressure ventilation. Continuous stimulation is unlikely to induce
spontaneous respirations and improvement in heart rate in this infant. Deep suctioning may
fur-ther exacerbate bradycardia by inducing a vagal nerve stimulatory response. If the
patient does not respond to effective bag-mask ventilation, obtaining a more secure airway
Dr.Wahid Helmi Domiate Egypt

and administration of epinephrine may become necessary, but not before a period of
effective positive-pressure ventilation and chest compressions has been attempted.

2. Based on current evidence and practice guidelines,which of the following describes


an infant who would most benefit from therapeutic whole-body cooling?
a. 33-week infant born due to maternal preeclampsia,required intubation at birth, Apgar
scores—1 at 1minute, 3 at 5 minutes, 8 at 10 minutes

b. 37-week infant born after prolonged shoulder dystocia,Apgar scores—1 at 1 minute, 3 at


5 minutes, 8at 10 minutes—and an arterial cord gas of pH 6.98/CO2 88 mmHG/PaO2 24
mmHg/ Bicarbonate 20mmol/L/base excess–17 who at 1 hour of age appears
hyperalert, with mildly increased tone, a weak suck,
HR of 190 beats/min, and respiratory rate (RR) of 75breaths/min.

c. 39-week infant born after uterine rupture, Apgar scores—1 at 1 minute, 3 at 5 minutes, 8
at 10minutes—and an arterial cord gas of pH 6.98/CO288 mmHG/PaO2 24 mmHg/
Bicarbonate 20 mmol/L/base excess–17 who at 1 hour of age appears comatose,
remains intubated, is lethargic, with decreased activity, hypotonia, incomplete Moro reflex
and suck,and intermittent spontaneous respirations on the ventilator.

d. 37-week infant, now 12 hours old, found in mother’s


room to be unresponsive and received CPR with two
doses of epinephrine before spontaneous return of circulation.
2. c. The infant in choice c presents with signs of moder-ate encephalopathy after a known
perinatal hypoxic event and is most likely to have a neurologic benefit from whole-body
cooling. The infant in (b) has also experienced a significant hypoxic event at birth but shows
signs of only mild encephalopathy on examina-tion and therefore does not meet the criteria
for whole-body cooling. The infant in (a) is preterm; cooling in this population has not yet
been established to be beneficial. The infant in (d) had an unwitnessed hypoxic event 12
hours after birth; whole-body cooling for neonates who experience this type of arrest is not
the standard of care.
Dr.Wahid Helmi Domiate Egypt

3. Which of the following is the preferred dose of epinephrine to be given during a


neonatal resuscitation?
a. 0.01-mL/kg/dose of a 1:10,000 solution IV
b. 0.5-mL/kg/dose of a 1:1000 solution via ETT
c. 0.0-mL/kg/dose of a 0.1-mg/mL solution (1:10,000)IV
d. 1-mL/kg/dose of a 1:10,000 solution via ETT
3. c. This is the preferred dosage and route of administra-tion for epinephrine during a
neonatal resuscitation.
4. Which of the following is an advantage of a self-inflating bag over a T-piece
resuscitator?
a. It can administer PPV without a source of compressed air or oxygen
b. It can provide PEEP when applied continuously to the face
c. It provides consistent PIP and PEEP with each breath,with minimal variation between
breaths
d. It can be used to deliver oxygen concentrations > Fio2 21%
4. a. Of the most commonly used devices used in neona-tal resuscitation, the self-inflating
bag is the only device that can deliver positive-pressure breaths without being attached to a
source of compressed air or flow. A self-inflat-ing bag cannot deliver PEEP unless
additional valves or mechanisms are attached to the device, whereas a T-piece resuscitator
can. Additionally, the benefit of a T-piece resuscitator is that it will deliver consistent PIP
and PEEP to the patient, provided that a good seal is made between the mask and the
patient’s face and/or the ETT remains in place. Both devices can deliver PPV with an
increased Fio2concentration if attached to an oxygen source.
5. The hypercapnia, hypoxemia, and acidosis that result from asphyxia will initially
cause a redistribution of blood flow to which organs?
a. Heart, kidneys, and adrenal glands
b. Heart, intestines, and brain
c. Heart, brain, and adrenal glands
d. Brain, intestines, and kidneys
e. Brain, kidneys, and heart
Dr.Wahid Helmi Domiate Egypt

5. c. The hypercapnia, hypoxemia and acidosis that result from asphyxia will initially cause
a redistribution of blood flow to the most vital organs, the heart, brain and adrenal glands.
6. Which of the following is a feature of primary energy failure from hypoxic brain
injury?
a. Occurs 6−48 hours after hypoxic injury
b. Necrotic cell death
c. Apoptosis
d. Decreased glutamate reuptake by damaged cells
e. Increases in cerebral ATP stores
6. b. Necrotic cell death occurs after primary energy failure due to depletion of cerebral ATP
stores and inactivation of the Na/K membrane pumps. Secondary energy failure occurs 6−48
hours after the initial hypoxic injury and results in decreased glutamate reuptake, which
leads to the induction of apoptosis

Labor and Delivery Questions


1. For which of these clinical situations would you recommend a prompt, planned
cesarean delivery?
a. 30-year-old G3P2002 at 39 weeks; cesarean delivery × 1 (breech presentation)
b. 20-year-old G1P0 at 39 weeks; fetal gastroschisis
c. 24-year-old G2P1001 at 39 weeks; fetal hypoplastic left heart, unrestrictive atrial septum
d. 23-year-old G1P0 at 36 weeks; fetal sacrococcygeal teratoma with signs of fetal hydrops
e. 35-year-old G4P1112 with maternal dilated cardiomyopathy who has a left ventricular
ejection fraction (LVEF) of 40%
1. d. Large fetal masses are at risk of rupture or dystocia during attempted vaginal deliver;
thus, cesarean deliv-ery is recommended. Hydrops is an indication for deliv-ery near term.
Women with two or less low-transverse cesareans may consider a trial of labor. Cesarean
deliv-ery is not required for fetal gastroschisis. Labor and vagi-nal delivery are well
tolerated by fetuses with congenital heart disease; cesarean delivery is considered only in
extreme cases. Most women with cardiac disease can have safe vaginal deliveries.
Dr.Wahid Helmi Domiate Egypt

2. A 31-year-old G4P3003 presents at 35 weeks with heavy vaginal bleeding and a


known placenta previa.She previously received a steroid course for an episode of
vaginal bleeding. She had two prior cesarean deliveries.An ultrasound at 18 weeks
showed placental lacunae and abnormal bladder vascularity. Maternal vital signs are
stable. Fetal NST is reactive. The most appropriate next step in management is:
a. Prompt cesarean hysterectomy
b. Repeat steroid course followed by cesarean hysterectomy
c. Repeat steroid course followed by cesarean delivery
d. Expectant management until term
e. Expectant management until documentation of fetal lung maturity
2. a. History and sonographic features are suggestive of placenta accreta. Management
involves planned cesarean hysterectomy at 34–36 weeks due to ongo-ing risk of massive
hemorrhage. Because this patient presented with heavy bleeding, prompt delivery is
indicated. Repeat steroid courses are not given after 34 weeks of gestation; thus,

(b) and (c) are incor-rect. Also,

(c) does not call for cesarean hysterec-tomy, which is the definitive management of placenta
accreta. Expectant management is not appropriate, given active hemorrhage. Testing for
fetal lung matu-rity is not recommended because delivery is indicated by the maternal
condition.

Fetal Assessment and Treatment


1. A patient at 32 weeks is having fetal growth surveillance for renal disease. The EFW
is at the seventh percentile, with a normal amniotic fluid volume. The umbilical
artery S/D ratio is elevated. Twice-weekly NSTs and weekly assessments of amniotic
fluid volume are started for surveillance. Which of the following findings require
delivery?
a. Fetal EFW < 10th percentile on the next growth assessment
b. There is interval development of oligohydramnios
c. Umbilical artery velocimetry shows reversed enddiastolic
Dr.Wahid Helmi Domiate Egypt

flow
d. Intermittent variable decelerations are noted on reactive NST
1. c. Umbilical artery Doppler velocimetry defines a group of fetuses that is at high risk of
adverse perinatal out-comes. Reversed end-diastolic flow should prompt deliv-ery after 32
weeks. An EFW < 10th percentile defines growth restriction, but fetal tests of well-being are
used to make delivery decisions. Oligohydramnios does not prompt delivery until term.
Intermittent variable decel-erations on a reactive NST do not portend imminent compromise.
2. A woman with diabetes is having a biophysical profile assessment at 34 weeks. Fetal
NST shows more than two accelerations in the fetal heart rate, which is 10 beats/min
above the baseline. Fetal breathing was seen. The single
deepest vertical pocket (SDP) of amniotic fluid exceeds 2 cm. Many fetal body
movements are seen, including at least one kick, with return to flexion. What is the
BPP score, and what additional testing do you recommend?
a. 6; no further testing required at this time
b. 6; continuous electronic fetal monitoring
c. 8; no further testing required at this time
d. 8; consider a repeat biophysical profile within 24 hours
e. 10; no further testing required at this time

2. c. This fetus receives 2 points each for amniotic fluid vol-ume, movement, tone, and
breathing (total of 8 points). No points are received for the NST because the fetus is not
reactive (15 beat accelerations are required at 34 weeks). A BPP score of 8 or 10 is
predictive of normal fetal oxygenation, and no further testing is required. A BPP score of 6
is an equivocal test and requires follow-up testing (BPP at 8 and 24 hours) to determine if
delivery is required
Dr.Wahid Helmi Domiate Egypt

Pre &perinatal problems


1. Which of the following is true regarding maternal adaptation
to pregnancy and the impact on chronic disease?
a. Serial echocardiography should be done in mothers with cardiac disease because
pregnancy-induced cardiovascular changes may worsen cardiac dysfunction.
b. Peak flow monitoring cannot be used for asthma management because peak expiratory
flow and FEV1 are altered by pregnancy.
c. Increased doses of antihypertensives are required in pregnancy because of the increase in
systemic vascular resistance.
d. Pregestational diabetics require less insulin during pregnancy because of lower renal
clearance of insulin.
e. Free T4 levels cannot be used to monitor thyroid disease because these values are
increased in pregnancy.
1. a. Increased cardiovascular demand may cause or worsen cardiac dysfunction
in women with cardiac diseases. Echocardiography is recommended in each trimester
or with new cardiac symptoms. Peak flow monitoring should be used because
expiratory respi-ratory indices are unchanged in pregnancy. Systemic vascular
resistance decreases in pregnancy; thus, anti-hypertensive doses often can be lowered
or eliminated. Insulin dose increases are common because of progres-sive insulin
resistance in pregnancy. Although total T4and T3 increase in pregnancy, steroid-
binding globulin also increases, so free levels are unchanged.
Dr.Wahid Helmi Domiate Egypt

2. An Ashkenazi Jewish couple with a child affected by Tay-Sachs disease presents for
preconception counseling.Their child’s genetic analysis has identified a causative
mutation that is detected on routine screening panels.Neither partner has had prior
genetic testing. The wife has chronic hypertension treated with lisinopril. All the
following are recommended except:
a. Folic acid supplementation prior to conception
b. Discontinuance of lisinopril and labetalol started before conception
c. Expanded carrier screening for Eastern and Central European Jewish descent
d. An interpregnancy interval of less than 18 months
e. Gamete donation from a known Tay-Sachs negative donor to conceive the next pregnancy
2. d. An interpregnancy interval of at least 18 months is advised. Folic acid
supplementation is recommended for all women to prevent neural tube defects.
Lisino-pril use is contraindicated in pregnancy; transition to a preferred medication
should be carried out. Gamete donation can prevent genetic disease in the offspring
if parents are known carriers. Expanded carrier screening is offered to all patients of Eastern
and Central European Jewish descent. Although this couple may be presumed to be Tay-
Sachs carriers, mutation analysis will confirm recurrence risk and screen for other at-risk
conditions.
3. A 30-year-old G3P1011 woman presents at 12 weeks with known Kell sensitization
(red blood cell alloimmunization).She had an uncomplicated pregnancy 4
years ago and a 24-week fetal demise that occurred last year. She had a motor vehicle
accident 5 years ago that required multiple blood transfusions. She is wondering
if Kell sensitization caused the fetal demise. How would you answer her question?
a. It definitely caused the fetal demise.
b. It cannot cause fetal demise unless there was bleeding
in the pregnancy.
c. It did not cause the fetal demise because her first pregnancy
was uncomplicated.
d. It may have caused the demise; paternal antigen testing
is required to determine if other pregnancies are
at risk.
Dr.Wahid Helmi Domiate Egypt

3. d. Alloimmunization may have caused demise if the fetus was Kell-positive.


Absence of bleeding does not rule out Kell sensitization; anemia develops from trans-
placental passage of maternal antibodies. Sensitized patients may have
uncomplicated pregnancies if a fetus is antigen-negative. Paternal antigen testing will
show if a pregnancy is at risk—no fetal risk if the father is Kell-negative, but
possibly so if the father is Kell-positive (may be heterozygous or homozygous).

4. Which of the following is not routinely recommended in the management of


pregestational diabetes in pregnancy?
a. Nutrition consult to review dietary and lifestyle modifications
b. Fetal echocardiography at 20 weeks due to increased risk of congenital heart defects
c. Frequent blood glucose checks and oral hypoglycemics or insulin as needed
d. Cesarean delivery if the EFW > 4500 g, given the high risk of shoulder dystocia
e. All patients should be delivered at 37 weeks due to an increased risk of stillbirth,
4. e. Stillbirth risk is increased with DM; fetal surveillance is recommended from 32
weeks. If glycemic control is poor, then 37- to 38-week delivery is considered.
Delivery is delayed until 39 weeks in well-controlled patients to allow time for fetal
maturity. Nutrition con-sultation is recommended in all diabetic pregnancies. Fetal
echocardiography is advised due to the increased risk of anomalies. Frequent blood
glucose checks are necessary to guide dosing of medications. Metformin, glyburide,
and insulin may all be used to achieve glyce-mic control.

5. A 27-year-old G1P0 at 29 weeks is admitted with new severe range hypertension.


Antihypertensives and a steroid course are ordered. Laboratory evaluation
shows 1400 mg/day proteinuria, creatinine of 1.0 mg/dL, AST of 106 mg/dL, platelet
count of 95,000/μL, and hematocrit of 28%. Other laboratory tests are normal. A
reactive NST was obtained.Fetal growth is normal. The patient has no symptoms.
What is the diagnosis, and what is the most appropriate next step in management?
a. Gestational hypertension; discharge from the hospital with close outpatient follow-up
b. Preeclampsia without severe features; expectant management until 37 weeks
Dr.Wahid Helmi Domiate Egypt

c. HELLP syndrome; delivery after completion of steroid course


d. Preeclampsia with severe features; delivery after confirmation of fetal lung maturity
e. Preeclampsia with severe features; expectant management until 34 weeks of gestation of
controlled hypertension, stable laboratory results, and reassuring fetal status
5. c. This patient has HELLP syndrome. If maternal and fetal statuses are
reassuring, delivery should take place after completion of a steroid course. Severe
features of a preeclampsia syndrome (HELLP syndrome and severe hypertension) are
present; thus, a and b are incorrect. If the only severe feature is controllable
hypertension, then expectant management detailed in (e) is appropri-ate. Fetal lung
maturity testing is not advised because delivery is indicated regardless of fetal maturity.

6. A patient has a normal fetal anatomy survey at 18weeks. She decides to have
maternal serum quad testing for aneuploidy screening, which shows an elevated risk
of trisomy 21. Which of the following is true?
a. The anatomy survey must have overlooked features of trisomy 21 because the false-
positive rate of the quad screen is low.
b. The quad screen must be a false-positive because no sonographic features of trisomy 21
were detected.
c. Diagnostic testing for trisomy 21 should be offered.
d. The fetus has trisomy 21.

6. c. Many, but not all, fetuses with trisomy 21 have struc-tural anomalies or soft marker
ultrasound findings. The lack of anomalies or markers does not rule out a trisomy 21
diagnosis. Maternal serum quad testing is a screening test for aneuploidy; a positive
screening result should not be considered diagnostic. Although the detection rate of the
serum quad test is high (75%), the false-positive rate is also high.
Consultant Dr.Wahid Helmi Domiate Egypt ( Neonatoloy Revision First note)

Neonatology revision First part


Dr-Wahid Helmi
Consultant Pediatrician
5-1. Which of the following statements regarding vitamin K administration in the
newborn period is correct?
a. Two 1-mg doses of intramuscular vitamin K are required in the first 48 hours of age.
b. Vitamin K cannot be given safely to neonates as an oral preparation.
c. Neonates receiving a 2-mg dose of oral vitamin K require no additional dosing in
infancy.
d. Oral vitamin K administration dosing has been standardized with internationally
accepted guidelines.
e. Parental nonadherence with additional doses of oral vitamin K is a major factor in
oral vitamin K failures.

5-2. A 4130-g male infant is born via vaginal delivery after 40 6/7 weeks’ gestation. The
mother recently migrated from Ecuador and received no prenatal care. In the
delivery room, the baby developed mild tachypnea and intercostal retractions. Pulse
oximetry from the infant’s left foot registers 95% saturation in room air. Physical
examination findings include a flat abdomen and audible bowel sounds over the left
side of the infant’s chest. Clear breath sounds are noted over the right side of the
chest. Heart sounds are best auscultated at the left sternal border.
The most appropriate initial step in management of this neonate is:
a. Needle decompression of the right hemithorax
b. Positive pressure ventilation using a bag and mask
c. Decompression of the stomach with a nasogastric tube
d. Positive pressure ventilation with a T-piece resuscitator
e. Transillumination of the right hemithorax
Consultant Dr.Wahid Helmi Domiate Egypt ( Neonatoloy Revision First note)

5-3. A 4550-g male neonate is delivered vaginally after a 37 6/7 weeks’ gestation
complicated by gestational diabetes. One application of a vacuum extractor to the
neonate’s head was required to facilitate delivery. On examination at 12 hours of
age, the baby has a well- circumscribed, fluctuant mass over the parietal skull that
does not cross suture lines. The overlying skin is erythematous but intact. He is
active and alert, and the rest of his examination is unremarkable. His hematocrit
(measured from a capillary sample) is 45%.
Which of the following steps is next indicated in the management of this
neonate?
a. Administration of intravenous ampicillin and cefotaxime
b. Monitoring serum and/or transcutaneous bilirubin levels
c. Careful aspiration of the mass with a tuberculin syringe
d. Application of topical bacitracin zinc ointment to the mass
e. Transfusion of packed red blood cells

5-4. An obstetric colleague asks you to provide prenatal counseling for a new patient
to his practice. The woman, currently at 34 3/7 weeks’ gestation, transferred her
medical care from an out-of-state obstetrical
group. Her past medical history is notable for poorly controlled type 1 diabetes.
The mother asks about fetal growth and neonatal body habitus in pregnancies
complicated by diabetes.
Which of the following statements regarding growth in fetuses and infants of
diabetic mothers (IDMs) is true?
a. Growth hormone (GH) is the primary anabolic growth factor in the fetus.
b. If a mother has advanced diabetes-related vascular disease (ie, diabetic retinopathy),
fetal growth is not affected.
c. IDMs have excess fat distribution in the extremities
d. Hypertrophic cardiomyopathy (HCM) in IDMs may result in ventricular outflow tract
obstruction.
e. IDMs have equally increased weight, length, and head circumference percentiles at
birth.

Consultant Dr.Wahid Helmi Domiate Egypt


Consultant Dr.Wahid Helmi Domiate Egypt ( Neonatoloy Revision First note)

5-5. A 3840-g female infant is delivered by cesarean section after a 39 5/7 weeks’
gestation. In the delivery room, she develops tachypnea and cyanosis when quiet.
Physical examination reveals clear bilateral breath sounds when the infant is crying.
However, the nurse is unable to pass a 5 French suction catheter through the nares
into the posterior pharynx.
The most appropriate initial step in management of this neonate is:
a. Oral airway placement
b. Intranasal dexamethasone instillation
c. Nasal cannula oxygen administration
d. Supine positioning
e. Nasal continuous positive airway pressure (CPAP)

5-6. A 9-week-old female infant is seen for a follow-up appointment with her
pediatrician. The baby weighed 4330 g at birth, and was delivered vaginally (from
the breech presentation) at 40 weeks’ gestation. In the nursery, hip examination
was unremarkable bilaterally on Ortolani and Barlow maneuvers. Physical
examination at this visit shows normal movement of the lower extremities.
The appropriate screening strategy for developmental dysplasia of the hip (DDH) in
this infant at this time is:
a. Repeat Ortolani and Barlow maneuvers only
b. Measurement and comparison of the lower extremity length (from the anterior superior
iliac crest to the heel)
c. Ultrasonography of both hips
d. Radiographs of both hips
e. Magnetic resonance imaging of both hips

5-7. A 1200-g male infant is delivered vaginally after a 28 3/7 weeks’ gestation complicated by preterm
rupture of membranes and preterm labor. On arrival to the delivery room table, the neonate receives
positive pressure ventilation (PPV) after the neonatal nurse dries the skin and suctions the oropharynx.
After 3 minutes of PPV, the nurse notes the baby has a heart rate of 110 beats/min with cyanosis and
intercostal retractions. The pediatric resident decides to intubate the infant. Brief visual inspection of
the mouth, mandible, and tongue reveals no abnormalities.
Which of the following combinations of laryngoscope blade and endotracheal tube is indicated for
intubating this patient?
a. Number 0 blade, 3.0 mm endotracheal tube
b. Number 00 blade, 4.0 mm endotracheal tube
c. Number 1 blade, 3.5 mm endotracheal tube
d. Number 00 blade, 3.5 mm endotracheal tube
e. Number 0 blade, 4.0 mm endotracheal tube
Consultant Dr.Wahid Helmi Domiate Egypt ( Neonatoloy Revision First note)

5-8. An infant born at 26 weeks’ gestation with a birth weight of 900 g is now 42
weeks postmenstrual age (PMA). He had respiratory distress syndrome (RDS) in
his early neonatal intensive care unit (NICU) course and continues to have a
supplemental oxygen requirement.
Which of the following characteristics of his past or current neonatal course would
support the diagnosis of bronchopulmonary dysplasia (BPD) in this infant?
a. He was born due to maternal chorioamnionitis and was mechanically ventilated for
21 days.
b. He was mechanically ventilated for 2 weeks, and now requires an FiO2 of 0.5 via
CPAP (5 cm H2O) to keep his saturations 97%.
c. He requires 2 liters per minute (LPM) of nasal cannula oxygen with an FiO2 of
0.6 to maintain oxygen saturations of 98% to 100%.
d. He developed Bacteroides fragilis sepsis at 2 weeks of age and required mechanical
ventilation until 34 weeks PMA.
e. He requires 1 LPM nasal cannula oxygen with an FiO2 of 0.45 to maintain oxygen
saturations of 91% to 93%.

5-9. A 600-g male infant was born at 24 1/7 weeks’ gestation to a 26-year-old, gravida
2, para 0020 woman with a history of cervical incompetence. His hospital course
was complicated by cholestatic jaundice, feeding intolerance, and chronic
respiratory failure requiring intubation from birth until his death from E. coli
pneumonia and bacteremia at 105 days of age. The family has requested a complete
autopsy to better understand his lung disease.
Which of the following features would you expect to see on pathologic examination
of this infant’s lungs?
a. Lung development consistent with his postmenstrual age
b. Uniform lung tissue with no evidence of emphysematous changes
c. Localized elastin fibers at the branch points of the alveoli
d. Increased alveolar diameter with decreased number of alveoli
e. Normal pulmonary vasculature with evidence of pulmonary edema

Consultant Dr.Wahid Helmi Domiate Egypt


Consultant Dr.Wahid Helmi Domiate Egypt ( Neonatoloy Revision First note)

5-10. A 30-year-old, gravida 1 woman presents to the Labor and Delivery unit at 26 6/7
weeks’ gestation. She reports uterine contractions every 5 minutes and leakage
of clear amniotic fluid 1 hour prior to her arrival. Physical examination confirms
rupture of her amniotic membranes as well as 9.5 cm of cervical dilation.
She is afebrile and has a normal heart rate and blood pressure. The obstetricians
page the neonatal intensive care unit (NICU) team to the Labor and Delivery unit,
anticipating an imminent delivery.
After delivery, which of the following management strategies in the NICU would
decrease the risk of this infant developing bronchopulmonary dysplasia (BPD)?
a. Initiation of parenteral erythromycin within 24 hours of birth
b. Application of positive pressure ventilation in the delivery room, using high
inspiratory pressures of 40 to 60 cm water after the first 5 infant breaths
c. Initiation of high-dose corticosteroids if the neonate requires more than 7 days of
mechanical ventilation.
d. Supplementation with enteral ascorbic acid (vitamin C) when the infant is on full-
volume gastric feeds
e. Use of supplemental oxygen to keep saturations within gestational-age-specific
target parameters while in the NICU

5-11. A 26-year–old, gravida 3 female presents to your office at 25 4/7 weeks’ gestation for
a prenatal consult. She tells you that the fetus, a female, has a “rectal mass.” The
prenatal ultrasound report notes the mass to be most consistent with a
sacrococcygeal teratoma (SCT). The mother asks about the possibility of resecting
the mass prior to delivery.
Which of the following statements regarding fetal surgery for a SCT is correct?
a. The primary goal of fetal surgery is prevention of fetal hydrops.
b. The removal of pelvic components of the teratoma is accomplished prenatally.
c. The development of hydrops fetalis is an insignificant concern in this case.
d. The anorectal sphincter complex should be removed during fetal surgery.
e. The surgical intervention should occur only after complete hydrops fetalis occurs.
Consultant Dr.Wahid Helmi Domiate Egypt ( Neonatoloy Revision First note)

5-12. Which of the following pathophysiologic mechanisms is associated with neonatal


hypoxic-ischemic brain injury?
a. Loss of high-energy phosphorylated compounds
b. Reduction in cellular oxidative stress
c. Accelerated reuptake of glutamate at the synaptic cleft
d. Decrease in intracellular calcium levels
e. Sustained inhibition of neuronal apoptosis

5-13. A 3210-g female neonate was delivered by emergent cesarean section due to a
maternal seizure and cardiorespiratory arrest after a 37 6/7 weeks’ gestation. The
infant required mechanical ventilation and chest compressions in the delivery
room. Her Apgars were 1, 2, and 4 at 1, 5, and 10 minutes of age, respectively. Her
initial arterial blood gas at 45 minutes of age showed a pH of 6.78 and a base
deficit of 20mmol/L.
On admission to the NICU, the radiant warmer was not turned on in anticipation
of hypothermia therapy to prevent hypoxic ischemic encephalopathy (HIE). Her
rectal temperature was 35.1°C at 60 minutes of age.
Which of the following statements regarding therapeutic hypothermia in this
infant is correct?
a. If the infant had severe abnormalities on amplitude- integrated
electroencephalography (aEEG) on NICU admission, selective head cooling would
provide the most neuroprotection.
b. The minimum length of therapeutic hypothermia for prevention of HIE in this infant is
12 hours.
c. Therapeutic hypothermia would accelerate free radical production in this patient.
d. Based on clinical trial data, whole body hypothermia would reduce the risk of death or
moderate to severe disability in this infant at 12 to 18 months of age.
e. Hypothermia will preserve executive functioning and psychosocial development
in this infant at 15 years of age.

5-14. Which of the following statements regarding neonatal venous strokes is correct?
a. Based on randomized clinical trials, low-molecular- weight heparin is the
anticoagulant of choice for treatment of venous thromboses.
b. Venous infarctions may be associated with neonatal group B Streptococcus sepsis.
c. Venous infarctions are 3 times as likely as arterial strokes in neonates.
d. Hemorrhagic venous infarcts in neonates usually spare the deep gray matter.
e. Trials of supratherapeutic doses of erythropoietin in humans have demonstrated
Consultant Dr.Wahid Helmi Domiate Egypt
Consultant Dr.Wahid Helmi Domiate Egypt ( Neonatoloy Revision First note)

benefit in treating neonatal strokes.


Consultant Dr.Wahid Helmi Domiate Egypt ( Neonatoloy Revision First note)

5-15. A 4-day-old male infant born at 37 2/7 weeks’ gestation presents for his first well-
child care visit. He weighed 4000 g at birth and his length was 55.5 cm (both greater
than the 90th percentile for gestational age). His newborn course was notable for
episodes of “low sugar,” per the mother; the nursery pediatrician suggested the
mother feed the infant every 2 hours because he was a “large baby.” The mother had
a normal oral glucose tolerance test during pregnancy.On physical examination, the
infant has a prominent tongue and bilateral ear creases. Abdominal examination is
notable for slight separation of the rectus abdominis muscles but without herniation
of intestinal contents. He is alert and interactive. A serum glucose taken in the office
is 39 mg/dL. He takes 30 cm3 of formula in the office in 15 minutes without
respiratory distress or diaphoresis.The etiology of hypoglycemia in this infant is
related to:
a. Congestive heart failure
b. Impaired fatty acid oxidation
c. Pancreatic hypertrophy
d. Swallowing dysfunction
e. Impaired glycogenolysis

5-16. You meet a 23-year-old primigravida female student, who presents for a routine
fetal ultrasound at 20 2/7 weeks’ gestation. She is a postbaccalaureate student who
will be applying to graduate school. She has an interest in fetal circulation,
particularly the oxygen concentration in the heart and blood vessels. She asks you,
“Which of the following sites has the highest oxygen concentration in the fetus?”
What is the best answer?
a. Inferior vena cava
b. Umbilical vein
c. Umbilical artery
d. Right ventricle
e. Ductus arteriosus
5-17. A 1200-g white male infant is born (at 29 1/7 weeks’ gestation) to a 25-year-old, gravida 2, para
1001 woman who presented to the hospital in preterm labor. The infant has an uncomplicated
delivery and requires only routine drying, warming, and tactile stimulation. He is admitted to the
NICU secondary to prematurity. He requires no respiratory support initially, but at 3 hours of life,
he is started on nasal continuous positive airway pressure (CPAP) due to tachypnea, grunting,
nasal flaring, and sternal retractions.
What is the most likely cause of his symptoms?
a. Persistent pulmonary hypertension of the newborn
b. Transient tachypnea of the newborn

Consultant Dr.Wahid Helmi Domiate Egypt


Consultant Dr.Wahid Helmi Domiate Egypt ( Neonatoloy Revision First note)

c. Respiratory distress syndrome (RDS)


d. Meconium aspiration syndrome
e. Congenital heart disease
5-18. A 2100-g infant was born at 33 1/7 weeks’ gestation by repeat cesarean section to a
28-year-old primigravida woman. At delivery, the male infant was noted to
have respiratory distress characterized by grunting, retractions, and hypoxia. The
infant is intubated and given a dose of surfactant. The father, a graduate student
in biochemistry, asks you about the properties of surfactant and its production
and role in the normal lung.
Which of the following statements about surfactant is correct?
a. Surfactant lipids and proteins are synthesized in the alveolar epithelial type II cells.
b. Phosphatidylglycerol (PG) is the major component responsible for decreasing
alveolar surface tension.
c. Congenital surfactant protein A (SP-A) deficiency causes fatal respiratory distress in
affected full-term infants.
d. Surfactant synthesis and storage begins at 28 to 38 weeks’ gestation.
e. Antenatal testing of amniotic fluid after 34 weeks’ gestation will show a lecithin–
sphingomyelin (L/S) ratio of 1:1 in infants with mature lungs.

5-19. A 2500-g female infant was born at 34 3/7 weeks’ gestation to a 30-year-old,
gravida 3, para 1102 woman. Shortly after birth, the infant developed tachypnea
(respiratory rate of 60 breaths/min), and increased work in breathing (manifested
by intercostal retractions).She is brought to the special care nursery for evaluation
and management of her respiratory distress. Physical examination shows coarse
bilateral breath sounds, no cardiac murmur, and strong, equal peripheral pulses.
She is alert and active.
What is the most appropriate initial management strategy for this infant?
a. Intubate the infant; immediately begin high- frequency oscillatory ventilation
(HFOV).
b. Begin continuous positive airway pressure (CPAP) with supplemental oxygen and
nitric oxide (iNO).
c. Intubate the infant and begin intermittent mechanical ventilation (IMV).
d. Support the infant with CPAP and supplemental oxygen to maintain appropriate
oxygen saturations for gestational age.
e. Place the infant under a hood and titrate nitrogen (N2) and oxygen (O2) into the
hood to generate an FiO2 of 0.16.
Consultant Dr.Wahid Helmi Domiate Egypt ( Neonatoloy Revision First note)

5-20. A 36 2/7–week, 1800-g male infant is born to a32-year-old, gravida 1 woman with
pregnancy-induced hypertension. At 30 seconds of age, he is spontaneously crying on the
delivery room table, and has a heart rate of 110 beats/min.
Which of the following steps is most important in the
initial management of this infant?
a. Administer positive pressure ventilation (PPV) via a self-inflating bag.
b. Provide continuous positive airway pressure (CPAP) using a T-piece resuscitator.
c. Administer a 10 cm3/kg intravenous bolus of normal saline via an umbilical venous
catheter.
d. Dry the infant with warm towels and suction his mouth and nose.
e. Deliver a 2 cm3/kg intravenous bolus of 10% dextrose via an umbilical venous
catheter.

5-21. A 2200-g infant was born at 39 6/7 weeks’ gestation (via spontaneous vaginal
delivery) to a 34-year-old woman with a history of chronic hypertension. In the
nursery at 2 hours of age, the baby’s blood glucose was 24 mg/dL before feeding.
After taking 20 mL of age- appropriate formula, the blood glucose increased to 50
mg/dL.
What is the most likely cause for the initial hypoglycemia in this infant?
a. Increased gluconeogenesis
b. Decreased neonatal insulin sensitivity
c. Increased placental transport of maternal insulin in utero
d. Increased serum epinephrine levels
e. Decreased glycogen stores

Consultant Dr.Wahid Helmi Domiate Egypt


Consultant Dr.Wahid Helmi Domiate Egypt ( Neonatoloy Revision First note)

5-22. A 1500-g female infant is born at 34 weeks’ gestation to a 40-year-old, gravida 4,


para 3003 woman by cesarean section due to reverse end-diastolic flow noted on
fetal sonography. The pregnancy was complicated by intrauterine growth
restriction (IUGR) and pregnancy- induced hypertension. Due to respiratory
distress in the delivery room, the baby is transferred to the NICU for intravenous
nutrition and respiratory support.
The infant is at high risk for developing which of the following?
a. Low plasma concentrations of fatty acid and triglycerides if she receives
intravenous lipids
b. Low basal oxygen consumption and total energy expenditure (relative to an
appropriate-for- gestational-age [AGA] infant)
c. Increased intestinal protein losses with enteral nutrition
d. Higher serum insulin levels in the neonatal period
e. High immunoglobulin levels during infancy

5-23. Which of the following statements regarding neonatal glucose metabolism is


correct?
a. Postnatal increases in catecholamines and glucagon modulate neonatal glucose
concentrations shortly after birth.
b.Neonatal hepatic glycogen content may remain elevated up to 72 hours after birth
without exogenous glucose delivery.
c. Neonatal insulin levels surge immediately after delivery.
d.Basal glucose production in a neonate is approximately 10% to 15% the basal rate of
an adult.
e. Inhibition of long-chain fatty acid oxidation increases circulating glucose
concentrations.
Consultant Dr.Wahid Helmi Domiate Egypt ( Neonatoloy Revision First note)

5-24. A 4050-g male neonate was delivered by emergent cesarean section due to fetal
bradycardia after a 39 2/7 weeks’ gestation. He required mechanical ventilation,
chest compressions, and intravenous
epinephrine in the delivery room. The baby’s Apgars were 1, 1, and 6 at 1, 5, and 10
minutes of age, respectively. His initial arterial blood gas at 45 minutes of age showed
a pH of 6.97 and a base deficit of 17 mmol/L. He developed seizures at 30 minutes of
age, and was placed on whole-body hypothermia (for prevention of hypoxic ischemic
encephalopathy) at 75 minutes of age. He also was placed on a high- frequency
oscillator and started on maintenance intravenous fluids of 85 cm3/kg/day containing
10% dextrose via an umbilical venous catheter.At 24 hours of age, the baby’s total
urine output for the day is 0.2 cm3/kg/h. He has 2-second capillary refill and strong
peripheral pulses. A set of serum chemistries at that time demonstrates the following:

Sodium 122 mEq/L


Potassium 6.2 mEq/L
Chloride 89 mEq/L
CO2 14 mEq/L
Blood urea nitrogen 45 mg/dL
(BUN)
Creatinine 2.6 mg/dL
Calcium 6.9 mg/dL
Glucose 67 mg/dL

Which of the following steps is appropriate in the management of this infant?


a. Administration of furosemide, 0.5 mg/kg intravenous
b. Administration of normal saline, 10 cm3/kg intravenous, over 30 minutes
c. Administration of bumetanide, 0.1 mg/kg intravenous
d. Reduction of intravenous fluids to 30 cm3/kg/day
e. Increase of dextrose in the intravenous fluids to 15%
Consultant Dr.Wahid Helmi Domiate Egypt

5-25. A 3700-g male infant, born vaginally at 39 2/7 weeks’ gestation, required intubation in the
delivery room for apnea. The mother is a 31-year-old, gravida
3, para 2002 woman with no significant medical history. She received routine prenatal care during
this uncomplicated pregnancy. She developed more frequent uterine contractions at home (4 hours
prior to birth) and, subsequently, her amniotic membranes ruptured(1 hour prior to birth). On
arrival to Labor and Delivery, her cervix was fully dilated, and she delivered the baby shortly
thereafter.
The pediatric resuscitation team arrived at 3 minutes of life to find a limp, cyanotic infant
under the radiant warmer receiving stimulation and blow-by oxygen.The baby’s heart rate was
greater than 100 beats/min, but he demonstrated little respiratory effort. He was intubated
easily by the pediatric intern and brought to the NICU. Apgars were 4 and 4 (2 points each for
color and heart rate) at 5 and 10 minutes, respectively. Cord blood was not sent for blood gas
measurement, but the infant’s initial arterial blood gas in the NICU at 30 minutes of age
revealed a pH of 6.98 and base deficit of 14 mmol/L.
At 45 minutes of age, he continues to be hypotonic and apneic (despite stimulation from NICU
caregivers), with pupils that are nonreactive to light.
Which of the following statements is correct, and should be considered in the management of
this infant?
a. There is no report of a prenatal or antenatal event that would cause hypoxia–ischemia, making this
infant’s presentation most consistent with an inborn error of metabolism.
b. Administration of prophylactic phenobarbital is indicated for seizure prophylaxis in this infant for a
minimum of 72 hours after birth.
c. If the neurologic examination does not improve, this infant will be a candidate for whole-body
hypothermia, which should begin at 12 hours of life.
d. Results of MRI imaging and electroencephalogram will be useful for discussing long-term
prognosis.
e. Tracking degree of clinical encephalopathy is useful for predicting duration of hospitalization, but
does not contribute additional information when predicting later prognosis.
5-26. A 34-year-old, gravida 1 woman presents at 18 weeks for a level 2 screening ultrasound. The
fetus is noted to have a hypoplastic nasal bone, increased nuchal fold, and short humeri and
femora.What profile of -fetoprotein (-FP), human chorionic gonadotropin (hCG), and
unconjugated estriol (uE) would have been most likely seen on the woman’s triple screen prior
to the ultrasound?
a. Low -FP, low hCG, low uE
b. Low -FP, high hCG, low uE
c. Low -FP, low hCG, high uE
d. High -FP, high hCG, high uE
e. High -FP, high hCG, low uE
Consultant Dr.Wahid Helmi Domiate Egypt ( Neonatoloy Revision First note)

5-27. A 25-year-old, gravida 2, para 1001 woman presents at 38 6/7 weeks’ gestation in
labor. She is placed on a tocodynamometer. The following tracing is obtained:
Which of the following is the most likely cause of the above findings?
a.Low fetal oxygen tension
b. Intermittent umbilical cord compression
c.Fetal hiccups (singultus)
d.Fetal head compression
e.Normal fetal movement

Fetal heart rate

Uterine
contractio
n pattern

(Reproduced, with permission, from Parer JT. Fetal heart rate. In:
Creasy R, Resnick R, eds. Maternal- Fetal Medicine: Principles
and Practice. 3rd ed. Philadelphia: Saunders; 1994.)

5-28. A 29-year-old, gravida 4, para 1203 woman presents to her obstetrician for a routine
appointment at 27 6/7 weeks’ gestation. Her urine dipstick demonstrates 4 protein, and her
blood pressure is 170/100 mm Hg. Her baseline (prepregnancy) blood pressure is 116/70 mm Hg,
and her blood pressure at 24 2/7 weeks’ gestation was 140/88 mm Hg. She denies headache, visual
changes, or abdominal pain, and has no facial or extremity edema on physical examination. She
has normal urine output and a serum creatinine of 1.1 mg/dL. Her obstetrician admits his
patients to a tertiary care obstetrical center.
Which is the most appropriate initial step in management of this patient?
a. Admission to Labor and Delivery for an emergent cesarean section delivery
b. Admission to Labor and Delivery for hemodialysis and delivery within the next 48 hours
c. Admission to Labor and Delivery and intravenous magnesium sulfate administration only
d. Admission to Labor and Delivery for intravenous magnesium sulfate and corticosteroid administration
e. Admission to Labor and Delivery for corticosteroid administration therapy only
Consultant Dr.Wahid Helmi Domiate Egypt ( Neonatoloy Revision First note)

5-29. A 10-day-old male infant born at 28 1/7 weeks’ gestation develops hypotension,
tachypnea, and an increasing number of desaturation and bradycardia episodes. The
infant was born vaginally after the mother presented to the hospital with a 4-day
history of ruptured amniotic membranes. She received a course of antenatal
betamethasone. He was intubated in the delivery room, given 3 doses of surfactant
over his initial NICU course for respiratory distress syndrome, and extubated at 4
days of age to CPAP (FiO2 0.3). The infant also received 48 hours of antibiotics
(beginning at birth) for presumed sepsis. Enteral feeds were initiated at 5 days of
age. Physical examination is notable for a holosystolic murmur. An echocardiogram
demonstrates a large patent ductus arteriosus (PDA) with left-to-right flow. His
complete blood count shows a white blood cell count of 4000/mm3 with 20% band
forms.The most likely reason for presentation of this symptomatic PDA at this time
is:
a. Extubation on day 4, which decreased pulmonary vascular resistance.
b. Failure to receive indomethacin prophylaxis for intraventricular hemorrhage in the
first 48 hours of life.
c. Inadequate antibiotic treatment for presumed sepsis, resulting in prostaglandin
release.
d. Failure of the mother to receive antenatal steroids due to precipitous delivery.
e. Advances in enteral feeding caused shunting to mesenteric circulation.

5-30. How many grams per deciliter of hemoglobin is deoxygenated when cyanosis
becomes apparent in a neonate?
a. 0.5 to 1.0
b. 3 to 5
c. 9 to 11
d. 15 to 20
e. 25 to 30
Consultant Dr.Wahid Helmi Domiate Egypt ( Neonatoloy Revision First note)

5-31. A 27-year-old woman presented to the emergency department of a local


community hospital in active labor. She has had no prenatal care and does not
remember her last menstrual period. She was transferred to the Labor and
Delivery suite, where a bedside ultrasound dated the fetus at approximately 40
weeks’ gestation. Three hours later, the woman vaginally delivered a 4700-g male
infant. Physical examination of the baby by the pediatric nurse practitioner at 15
minutes of age shows unlabored respirations but “dusky” lips and tongue. He is
given blow-by oxygen (FiO2 1.0), but his oxygen saturations never increase above
85%.
What is the most appropriate next step in management of this neonate?
a. Intubate the infant and ventilate him with an anesthesia bag.
b. Administer a bolus of 10% dextrose intravenously.
c. Admit infant to full-term nursery.
d. Obtain a computerized tomogram (CT) of the chest.
e. Consult a cardiologist at a tertiary care children’s hospital.

5-32. A 3800-g male infant was born at 41 3/7 weeks’ gestation to a 26-year-old woman.
She had unremarkable cultures and serologies except for a positive vaginal group
B Streptococcus culture (at 36 3/7 weeks’ gestation), for which she received
intrapartum antibiotics. Amniotic membranes were ruptured at delivery, and the
amniotic fluid was noted to be meconium stained. At birth, the infant was initially
apneic and hypotonic with a heart rate
of 90 beats/min. He was intubated easily on the first attempt, and a moderate
amount of meconium was aspirated from the trachea. He began to breathe when the
endotracheal tube was removed, but subsequently developed tachypnea and
intercostal retractions. He was admitted to the NICU and placed on supplemental
oxygen (FiO2 1.0) via an oxygen hood.
Which of the following conditions is the most likely
cause of the baby’s respiratory distress?
a. Mechanical obstruction of the airways
b. Bacterial pneumonia
c. Decreased surfactant production
d. Pulmonary vasodilation
e. Tracheal edema
Consultant Dr.Wahid Helmi Domiate Egypt ( Neonatoloy Revision First note)

5-33. A 4500-g female infant was born at 37 6/7 weeks’ gestation to a 30-year-old
woman with poorly controlled gestational diabetes. The delivery occurred
vaginally (with forceps assistance) and was complicated by shoulder dystocia.
Three hours after birth, the infant develops respiratory distress and needs
supplemental oxygen (FiO2 0.35) to keep her saturations greater than 95%.
Physical examination of the infant is notable for shallow respirations but clear
bilateral breath sounds. The baby has limited movement of the left arm on
elicitation of the Moro reflex. The intern orders a chest radiograph.
Which of the following findings is most likely to be seen on the radiograph?
a. Herniation of the stomach and small bowel into the left hemithorax
b.A decrease in pulmonary vascular markings bilaterally
c. Elevation of the left hemidiaphragm by 3 intercostal spaces (relative to the right
hemidiaphragm)
d. A left pleural fluid density compressing the lung
e. Multiple cystic lesions in the upper lobe of the left lung

5-34. A 2700-g female infant is admitted to the NICU after delivery by cesarean
section at 36 0/7 weeks’gestation. The mother is a 25-year-old, gravida 1 who
developed severe preeclampsia (after an uncomplicated pregnancy) and was
treated with a magnesium sulfate drip.Which of the following findings on initial
physical examination is normal given the clinical scenario?
a. Bursts of 8 to 10 sucks on a pacifier within 5 seconds, followed by pauses lasting 5 to
10 seconds
b. Visual fixation on a bull’s eye with sustained tracking
c. Strong flexor tone in the upper extremities and semiflexion in the lower
extremities
d. Absence of the Moro and palmar grasp reflexes
e. Movement of the right elbow to the left shoulder when the right arm is pulled across
the chest to the left
Consultant Dr.Wahid Helmi Domiate Egypt ( Neonatoloy Revision First note)

5-35. The Pediatrics team was called to a delivery by the Obstetrics team due to fetal
bradycardia. On arrival to the delivery room, the Labor and Delivery nurse told the
pediatricians that the mother was a 27-year-old, gravida 4, para 2103 woman at 39
0/7 weeks’ gestation. The mother had regular prenatal care, and her labor was
progressing normally. Rupture of membranes occurred 5 hours ago (clear amniotic
fluid).
At delivery, the baby was cyanotic, hypotonic, and apneic, but had an appropriate
response to stimulation and positive pressure ventilation delivered by the pediatric
respiratory therapist. The pediatric resident performed the initial newborn
examination 30 minutes after birth.
Which of the following findings on physical examination of this baby would raise
suspicion for central nervous system injury?
a. An intermittent disconjugate gaze when he is not fixating or tracking an object
b. Unsustained clonus when his ankle jerk reflex is elicited
c. Changes in the upper and lower extremity tones with rotation of his head from left to
right
d. Truncal extensor tone that exceeds his flexor tone
e. Rapid grasp when the palm of his hand is pressed

5-36. A 2700-g female infant was born vaginally (with vacuum assistance) after induction of labor at
36 6/7 weeks’ gestation (due to maternal preeclampsia). Apgar scores were 8 (1 color, 1 tone)
and 9 (1 color) at 1 and 5 minutes, respectively. The baby was permitted to stay with mother
and receive couplet care. While holding her at 48 hours of age, the mother felt the baby moving
but “not breathing” and called the postpartum nurse. The baby recovered quickly but was
transferred from couplet care to the NICU for monitoring. During the workup, a noncontrast
CT scan of the brain is performed. Before you can review the CT results, the radiology
technologist mentions to the baby’s nurse that the scan “… looks like a stroke.”
Which of the following statements regarding the brain injury in this infant is correct?
a. The timing of the seizure indicates that the insult occurred after delivery, and mother should be
questioned about having dropped the baby.
b. The timing of the seizure indicates that the insult
c. The stroke is most likely venous in origin, and evaluation for a hypercoagulable state is a high
priority.
d. Anticoagulation is effective therapy for neonatal thrombotic stroke regardless of the origin
(arterial vs venous).
e. This baby’s perinatal risk factors for stroke include maternal preeclampsia and vacuum-assisted
delivery.
Consultant Dr.Wahid Helmi Domiate Egypt ( Neonatoloy Revision First note)

5-37. A 2600-g male infant, born to a 16-year-old, gravida


1 female who did not receive prenatal care, was admitted to the neonatal intensive
care unit with mild tachypnea. At 3 days of age, his respiratory distress has resolved,
and a developmental pediatrician is asked to help determine his gestational age. On
his examination, the baby has very weak, intermittent finger flexion when his palm
is pressed, and no arm flexion when traction is placed on his arms. He has interest
in a pacifier, and will suck on it 4 to 5 times before requiring a 10-second pause.
These findings are most consistent with an infant of what gestational age?
a. Less than 30 weeks’ gestation
b. 30 to 31 weeks’ gestation
c. 32 to 33 weeks’ gestation
d. 34 to 35 weeks’ gestation
e. Greater than 36 weeks’ gestation

5-38. A 7-day-old female infant was born at 30 weeks’gestation by cesarean section for
maternal preeclampsia.She required CPAP for 3 days for respiratory distress syndrome,
and received 48 hours of antibiotics before blood cultures were negative. Today, the
pediatric intern noted the baby is tachypneic with an active precordium. On examination,
he hears a continuous cardiac murmur and palpates full peripheral pulses.Echocardiogram
reveals a large patent ductus arteriosus with left-to-right flow. Which of the following
factors could have contributed to the infant’s clinical status?
a. Elevated cortisol concentrations due to antenatal
betamethasone administration
b. Decreased sensitivity of premature ductal tissue to
prostaglandins
c. Antenatal treatment of maternal headache with
ibuprofen
d. Elevated serum concentration of prostaglandin E 2
(PGE 2 )
e. Omission of surfactant administration after delivery
Consultant Dr.Wahid Helmi Domiate Egypt ( Neonatoloy Revision First note)

5-39. Which of the following statements about necrotizing enterocolitis (NEC) in near-
term and term infants is true?
a. NEC in near-term and term infants is usually preceded by a perinatal infectious risk
factor such as maternal chorioamnionitis or prolonged rupture of membranes.
b. NEC in near-term and term infants typically involves the proximal small bowel, rather
than the colon.
c. NEC typically occurs sooner after birth in near-term and term infants when compared
with premature infants.
d. Near-term and term infants with NEC are more likely to require surgical
intervention than premature infants.
e. Near-term and term infants rarely develop a spontaneous intestinal perforation (SIP) if
there are no risk factors for intestinal hypoperfusion, such as indomethacin exposure.

5-40. Which of the following statements is correct based on the studies currently
available about prevention of necrotizing enterocolitis?
a. Exposure to antenatal steroids is associated with a decreased risk of necrotizing
enterocolitis in premature infants.
b. Intermittent increases in metabolic demand from bolus feeding put very-low-birth-
weight infants at risk for necrotizing enterocolitis.
c. When compared with formula, feeding with maternal breast milk protects against
necrotizing enterocolitis, but feeding with donor breast milk confers a risk similar to
feeding with formula.
d. Rapid advancement of enteral feeding in premature, very-low-birth-weight infants
increases the risk of necrotizing enterocolitis.
e. Dosing of probiotic strains for the prevention of NEC has been standardized
internationally and requires a minimum of 1.0  1010 colony-forming units/day.
Consultant Dr.Wahid Helmi Domiate Egypt ( Neonatoloy Revision First note)

5-41. Which of the following interventions decreases an infant’s risk of


developing germinal matrix/ intraventricular hemorrhage (IVH)?
a. Antenatal administration of phenobarbital
b. Postnatal administration of glucocorticoids
c. Postnatal infusion of fresh frozen plasma
d. Treatment of maternal chorioamnionitis with antibiotics
e. Rapid correction of hypotension with fluid boluses

5-42. A newborn female presents for a weight check a few days after discharge from the
neonatal intensive care unit. She was born at 35 4/7 weeks’ gestation, weighing
2570 g. The mother, a 27-year-old unemployed woman, had good prenatal care.
She developed preterm labor on the day of her daughter’s birth, and delivered
vaginally shortly after rupture of membranes. She was admitted to the neonatal
intensive care unit for transient tachypnea of the newborn, which resolved by 12
hours of life. Antibiotics were started on admission, but discontinued after 48
hours when blood cultures were negative. She was initially slow with oral feeding,
but took an adequate amount prior to discharge.
The mother has been reading about prematurity and neurodevelopment on the
Internet, and she asks the general pediatrician what to expect for her daughter.
Which of the following statements is accurate regarding outcomes of premature
infants later in life?
a. Since her infant was almost 36 weeks’ gestation at delivery, she does not need to
worry about neurodevelopmental problems.
b. Determination of her daughter’s need for special assistance in school (due to
intellectual disability) will be evident by her daughter’s third birthday.
c. Her daughter is at risk for risk-taking behavior as a teenager due to her prematurity.
d. If the baby’s birth weight had been less than 1500 g, she would have been extremely
unlikely to graduate from high school.
e. Preterm children are more likely to have language disorders, reading disability, and
difficulty with arithmetic.
Consultant Dr.Wahid Helmi Domiate Egypt ( Neonatoloy Revision First note)
Consultant Dr.Wahid Helmi Domiate Egypt ( Neonatoloy Revision First note)

5-43. The Obstetrics attending asks the Neonatology attending to counsel a 36-year-old
female who presented to the Labor and Delivery (L&D) suite in active labor. The
patient reported to the L&D nurse that she is “at full term.” The patient received
prenatal care in a community clinic, including a dating ultrasound
at 10 weeks’ gestation. After a normal ultrasound at
20 weeks’ gestation, she was told that she didn’t require additional follow-up. The
patient reports that she has been taking her prenatal vitamins and that her fetus
has remained active. The obstetrician is concerned that the patient’s fundal height is
only 33 cm and the fetus must be more premature than 36 weeks’ gestation or
significantly growth restricted.
Which of the following statements is correct and should be included in the
consultation with the mother?
a. For a low-birth-weight infant, the likelihood of developing serious neonatal
complications is primarily related to size at delivery, not gestational age.
b. If there has been growth restriction in utero that has affected the body, but has spared
the head, her child’s cognitive outcome should not be adversely affected.
c. If there has been growth restriction due to uteroplacental insufficiency, her child will
have a greater risk for developing hypertension and diabetes as an adult.
d. Since third-trimester intrauterine growth restriction (IUGR) stimulates a response
that interferes with fetal lung maturation, her baby must be intubated in the delivery
room.
e. If sent to a pathologist within 6 hours of delivery, the placenta, on gross examination,
will provide definitive information about the reason for IUGR.

5-44. A 4325-g term infant born with meconium aspiration syndrome has severe hypoxic
Consultant Dr.Wahid Helmi Domiate Egypt ( Neonatoloy Revision First note)

respiratory failure.
Despite aggressive management with high-frequency ventilation, inhaled nitric oxide,
and phosphodiesterase inhibitors, stable oxygenation cannot be achieved. The NICU
team decides to utilize extracorporeal membrane oxygenation (ECMO) for this
infant.
Which of the following statements regarding the outcomes of neonates treated with
ECMO is correct?
a. The increased frequency of treatment of neonatal hypoxic respiratory failure with
ECMO has resulted in a decrease in ECMO complications and better survival.
b. Neonates who require treatment with ECMO for neonatal hypoxic respiratory failure
have a risk of neurodevelopmental impairment that is significantly greater than
neonates who respond to conventional therapy (including high-frequency ventilation)
and inhaled nitric oxide.
c. The most common cause of mortality in infants treated with ECMO for hypoxic
respiratory failure is hemorrhage.
d. The risk of hearing impairment in extremely premature infants is significantly greater
than in term infants with respiratory and other organ failure.
e. Survival rate for infants treated with ECMO in the United States is less than 50%.

5-45. Which of the following statements is correct about neurodevelopmental


impairment in premature infants?
a. It is possible to diagnose a major neurodevelopmental impairment prior to NICU
discharge based on brain MRI and abnormalities on serial neurologic examinations.
b. Detection of abnormalities on magnetic resonance imaging (MRI) of the brain is the
“gold standard” for predicting major neurodevelopmental disabilities.
c. More than half of the premature survivors who were born at the limit of viability,
prior to 26 weeks’ gestation, develop intellectual disability, cerebral palsy, or both.
d. Complications of prematurity including chronic lung disease, sepsis, and necrotizing
enterocolitis confer additional risk for problems with neurodevelopment above the
risk of prematurity.
e. The most common type of cerebral palsy in premature survivors born at the limit of
viability is spastic hemiplegia.
Consultant Dr.Wahid Helmi Domiate Egypt ( Neonatoloy Revision First note)

5-46. Which of the following statements regarding the cardiorespiratory transition


from intrauterine to extrauterine life is correct?
a. Compliance in the neonatal lung decreases due to fluid shifts.
b. Surfactant is released into the alveolar space via lung inflation and increased blood
catecholamine levels.
c. Pulmonary vascular resistance increases in response to increased arterial oxygen
tension.
d. Clearance of fetal lung fluid occurs primarily through egress via the trachea.
e. Neonatal blood flow pattern is unchanged from the fetal blood flow pattern.

5-47. A premature male infant born at 26 1/7 weeks’ gestation has a cranial ultrasound
performed at 4 days of age.
The study shows a hemorrhage limited to the right germinal matrix.
Which of the following statements is correct regarding this patient’s
management/outcome?
a. Since the hemorrhage is limited to the germinal matrix/subependymal area, additional
brain imaging is not necessary for several weeks.
b. Male infants with germinal matrix/intraventricular hemorrhages are less likely to have
neurologic sequelae than female infants.
c. Since the hemorrhage is limited to the germinal matrix/subependymal area, the infant’s
development should be normal.
d. Cranial ultrasound should be repeated within the first week of life due to an
increased risk of a
primary hemorrhage into the left germinal matrix.
e. Cranial ultrasound should be repeated within the first week of life, as the hemorrhage
in the right germinal matrix may extend into the ventricles.
5-48. Which of the following infants is at highest risk of developing a germinal
matrix–intraventricular hemorrhage (GM-IVH)?
a. A term male infant born by vacuum-assisted vaginal delivery who required intubation,
positive pressure ventilation, and chest compressions in the delivery room
b. A 20-day-old male infant born at 24 weeks’ gestation who develops hypotension from a
patent ductus arteriosus
c. A female infant born at 26 weeks’ gestation by precipitous vaginal delivery who
develops hyperoxia and hypocarbia after intubation and surfactant administration in the
delivery room
d. A male infant born by cesarean section at 34 weeks’ gestation who required positive
pressure ventilation in the delivery room
e. A female infant born at 28 weeks’ gestation (due to maternal chorioamnionitis) who
develops a right- sided tension pneumothorax after reintubation at 17 days of age
Consultant Dr.Wahid Helmi Domiate Egypt ( Neonatoloy Revision First note)

5-49. A male infant born at 25 2/7 weeks’ gestation (birth weight 800 g) develops
tachycardia and abdominal distension prior to passing a blood-tinged stool. His
physical examination reveals decreased tone and activity, tenderness on
palpation of the abdomen, and a paucity of bowel sounds. His abdominal x-ray
shows diffuse pneumatosis without pneumoperitoneum or portal venous air. His
parents arrive during initial stabilization of their infant and ask to speak to the
attending neonatologist.Which of the following statement regarding this infant’s
condition is correct?
a. Because of his abdominal findings, the infant likely has hypersensitivity to mother’s
breast milk, and she should stop pumping.
b. Unless intestinal perforation occurs, their infant’s risk for growth failure is not
significantly increased as a result of this condition.
c. Their infant is unlikely to develop an intestinal stricture if there is no bowel
perforation.
d. This condition puts their infant at increased risk for neurodevelopmental impairment.
e. The incidence of this condition is inversely related to gestational age and birth
weight, but these characteristics do not increase the risk of mortality.
5-50. A 1460-g male infant is delivered by cesarean section after a 30 1/7 weeks’
gestation to a 29-year-old woman with acute lymphoblastic leukemia. The infant is
placed on the delivery room table. Drying the skin and suctioning the oropharynx
is initiated, followed by positive pressure ventilation (PPV). After 20 seconds of
PPV, the neonatal nurse notes that the infant has a
heart rate of 40 beats/min and no chest wall movement.
Which of the following interventions is indicated at this time in the resuscitation?
a. Flexion of the head and neck
b. Confirmation of an appropriate seal of the face mask
c. Intravenous infusion of 10 cm3/kg of normal saline
d. External cardioversion
e. Intravenous infusion of 0.1 mg/kg of 1:10,000 epinephrine solution
Consultant Dr.Wahid Helmi Domiate Egypt ( Neonatoloy Revision First note)

5-51. A 3690-g male infant was born vaginally after a


38 6/7 weeks’ gestation. The pregnancy was notable for polyhydramnios. He
required only suctioning,
drying, and stimulation in the delivery room. He passed meconium on the delivery
room table. Apgar scores were 8 and 9 at 1 and 5 minutes, respectively.
At 6 hours of age, the mother tells the postpartum nurse that the baby is “spitty”
with feeding. The nurse notes the infant has copious clear secretions from the
mouth. The nasopharynx and oropharynx appear normal on physical examination.
A large-bore nasogastric tube is placed; a subsequent chest x-ray shows the tube
ending at the level of the sixth cervical vertebrae.
The most appropriate initial step in management of this neonate is:
a. Intravenous antibiotics
b. Barium enema
c. Oral airway placement
d. Suction applied to the nasogastric tube
e. Contrast study of the upper gastrointestinal tract
Consultant Dr.Wahid Helmi Domiate Egypt ( Neonatoloy Revision First note)

5-52. Which of the following statements regarding fetal cortisol is correct?


a. Steroidogenic enzymes present in the adult adrenal gland are absent in the fetal adrenal
gland.
b. Fetal cortisol is necessary for normal intrauterine development.
c. Circulating fetal cortisol peaks during the middle of the first trimester of development.
d. Most of the circulating fetal cortisol is derived from the maternal adrenal gland.
e. Elevated fetal cortisol levels in the third trimester assist in neonatal respiratory
adaptation.

5-53. A 4950-g female neonate is delivered vaginally after a 38 6/7 weeks’ gestation
complicated by poorly controlled type 2 diabetes. The obstetrician reported
shoulder dystocia during the delivery. On examination at 4 hours of age, the baby’s
left arm is internally rotated, and the forearm is extended and pronated. The baby
does not move her left arm when the examiner assesses the Moro reflex.
Which of the following physical examination findings is most associated with the
findings in the arm?
a. An absent left thumb
b. A hyperpigmented patch on the left chest
c. A 2-vessel umbilical cord
d. A constricted left pupil
e. An enlarged left fifth finger

5-54. Which of the following neonates should receive hepatitis B immune globulin
within 12 hours after birth?
a. A 38 weeks’ gestation neonate (birth weight 3600 g) whose mother is hepatitis B surface
antigen (HBsAg) negative
b. A 39 weeks’ gestation neonate (birth weight 3750 g) whose mother has HBsAg
serology that is unknown, but will be confirmed within 24 hours after delivery
c. A 37 weeks’ gestation neonate (birth weight 3550 g) whose mother is positive for
hepatitis C antibody only
d. A 34 weeks’ gestation neonate (birth weight 2500 g) whose mother is HBsAg negative
but developed chorioamnionitis in labor
e. A 36 weeks’ gestation neonate (birth weight 1850 g) whose mother has HBsAg
serology that cannot be resulted until 36 hours after delivery
Consultant Dr.Wahid Helmi Domiate Egypt ( Neonatoloy Revision First note)

5-55. A 4885-g female infant was delivered via cesarean section due to cephalopelvic
disproportion after a 40 3/7 weeks’ gestation. Maternal serologies were
unremarkable, and cultures (including a cervical culture
for group B Streptococcus at 36 weeks’ gestation) were negative. The mother has
a 10-year history of poorly controlled type 2 diabetes mellitus.
The baby only required brief blow-by oxygen in the delivery suite. The infant’s
Apgar scores were 8 and 9 at 1 and 5 minutes, respectively. Her initial newborn
examination was normal.
The neonate fed well in the term nursery after birth.
At 60 hours of age, she developed rhythmic, generalized jerking of her head and
extremities. After the ictal event, vital signs and the physical examination were
unremarkable, and a capillary glucose was 69 mg/dL.
Which of the following tests should be ordered to evaluate this infant?
a. Serum calcium level
b. Serum sodium level
c. Serum C-reactive protein
d. Computerized tomography (CT) scan of the head
e. Magnetic resonance imaging (MRI) of the head

5-56. A 3440-g male neonate is delivered vaginally after a 39 6/7 weeks’ gestation
complicated by fetal bradycardia. On arrival at the delivery room table, the
baby has no respiratory effort or spontaneous movement. He is covered in
particulate meconium. The nurse palpates the pulse in the umbilical cord stump
and detects 5 beats in 6 seconds.
Which of the following steps is next indicated in resuscitation of this neonate?
a. Initiation of chest compressions
b. Placement of a 5 French nasogastric tube
c. Suctioning of the mouth and nose with a bulb syringe
d. Catheterization of the umbilical vein
e. Intubation and suctioning the trachea
Consultant Dr.Wahid Helmi Domiate Egypt ( Neonatoloy Revision First note)

5-57. A 2-day-old term female neonate is undergoing her discharge physical


examination in the nursery. On exam, you note the liver edge 1 cm below the right
costal margin and the spleen tip is not palpable. The umbilical stump is dry. Bowel
sounds are audible in the abdomen. When the infant cries, you note that there is a
slight distention of the abdominal space between the rectus abdominis muscles.
Which of the following steps is indicated for this infant?
a. Order a liver and gallbladder ultrasound
b. Discharge the baby home with her parents
c. Obtain a peripheral blood smear
d. Order a voiding cystourethrogram
e. Consult a pediatric surgeon
Consultant Dr.Wahid Helmi Domiate Egypt ( Neonatoloy Revision First note)

5-58. A 27-year-old woman brings her 3-year-old son into your office for a well-child care
visit. The child had documented intrauterine growth restriction (IUGR), and
weighed 1990 g at birth (at 39 1/7 weeks’ gestation). The mother is currently 8 weeks
pregnant and wants to decrease the risk of fetal growth restriction during her
current gestation. She has no acute or chronic medical problems. She smokes 0.5
pack of cigarettes per week.
Which of the following interventions would reduce the likelihood of fetal growth
restriction during this pregnancy?
a. Consumption of 4 oz of red wine daily
b. Avoiding food rich in -carotene
c. Initiation of a strict vegan diet
d. Dietary supplementation with folic acid
e. Cessation of cigarette smoking

5-59. Which of the following statements regarding neonatal and infant mortality is
correct?
a. Infant mortality rates are higher for infants of pregnancies where prenatal care
commences in the first trimester.
b. Asian or Pacific Islander ethnicity is a risk factor for infant mortality.
c. The introduction of surfactant therapy and antenatal steroids has reduced neonatal
mortality.
d. Female sex is a risk factor for neonatal mortality.
e. Neonatal mortality for low-birth-weight infants is reduced in high-care-level, low-
volume neonatal intensive care units (NICUs).

5-60. Which of the following neonates is most likely to experience a normal anion gap
metabolic acidosis?
a. A 2-day-old term male with weak femoral pulses (relative to brachial pulses) and a
serum lactate of
5.9 mmol/L
b. A 3-day-old female born at 24 4/7 weeks’ gestation with bounding palmar pulses and a
serum creatinine of 1.8 mg/dL
c. A 1-day-old male born at 28 1/7 weeks’ gestation with mild respiratory distress
syndrome and an initial urine pH of 7.5
d. A 4-day-old term female male with lethargy and a serum lactate of 1.2 mmol/L
e. A 5-day-old infant born at 32 2/7 weeks’ gestation with hypotension and a blood
culture positive for gram-negative rods
Consultant Dr.Wahid Helmi Domiate Egypt ( Neonatoloy Revision First note)

5-61. A 28-day-old male presents to his pediatrician’s office with a 1-week history of
intermittent vomiting and alternating irritability and lethargy. He was delivered
vaginally (with assistance of forceps) after a 40 4/7 weeks’ gestation complicated by
fetal bradycardia. His discharge physical was notable for marked bruising. On
examination, he is afebrile with a normal cardiorespiratory examination. He has
well-circumscribed, indurated nodules over the cheeks at the site of forceps
placement.
Which of the following laboratory values is most associated with this infant’s
presentation?
a. A serum glucose of 29 mg/dL
b. A serum ammonia of 135 mol/L
c. A serum lactate of 5.0 mmol/L
d. A serum calcium of 13.1 mg/dL
e. A serum urea nitrogen of 1.8 mg/dL

5-62. A 28 3/7–week male was born by cesarean section due to worsening maternal
preeclampsia. Due to respiratory distress in the delivery room, he is intubated and
placed on mechanical ventilation.
At 6 hours of age, the infant develops mottling of the skin, tachycardia, and
desaturations (measured by bedside pulse oximetry). A stat echocardiogram shows a
structurally normal heart with a patent ductus arteriosus.
Which of the following interventions would improve the infant’s blood pressure?
a. Initiation of an intravenous prostaglandin E1 (PGE1) infusion
b. Increase in the infant’s peak end-expiratory pressure (PEEP) on the ventilator
c. Increase in the infant’s peak inspiratory pressure (PIP) on the ventilator
d. Initiation of an intravenous immune globulin infusion
e. Initiation of an intravenous dopamine infusion
Consultant Dr.Wahid Helmi Domiate Egypt ( Neonatoloy Revision First note)

5-63. A 1900-g male infant is delivered by C-section after a 29 1/7 weeks’ gestation
secondary to fetal supraventricular tachycardia (SVT) and progressive hydrops
fetalis. The infant is placed on the delivery room table, and positive pressure
ventilation (PPV) begins after drying the skin and suctioning the
oropharynx. Physical examination is notable for diffuse anasarca. After 4 minutes
of PPV, the nurse notes the neonate has a heart rate of 190 beats/min, central
cyanosis, and intercostal retractions.
Which of the following steps is indicated at this point in the resuscitation?
a. Infusion of 0.3 mg/kg of 1:10,000 epinephrine solution via an umbilical venous
catheter
b. Endotracheal intubation and ventilation using a T-piece resuscitator
c. Weaning the infant to blow-by oxygen
d. Infusion of 0.1 mg/kg of adenosine via an umbilical venous catheter
e. Weaning the infant to nasal cannula
Consultant Dr.Wahid Helmi Domiate Egypt ( Neonatoloy Revision First note)

5-64. Which of the following statements regarding thermoregulation in neonate is


correct?
a. Heat production in the neonate (controlling for body weight) is greater than heat
production in the adult.
b. Heat production in the neonate (relative to body weight) must be lower in the neonate
to maintain a normal body temperature.
c. The capacity for neonatal adaptation to cold stress is similar to that of an adult.
d. Brown fat stores are less abundant in the neonate than in the adult.
e. Circulating free fatty acids serve as an acute source of energy during cold stress in the
neonate.

5-65. You are asked by the obstetrical service to counsel a 25-year-old primigravida
woman who is experiencing preterm labor. Her pregnancy is notable for a twin
gestation.
Which of the following statements is true regarding multiple births?
a. Twin–twin transfusion syndrome usually occurs in dizygotic pregnancies as
compared with monozygotic pregnancies.
b. Most infants born following multifetal gestations are premature.
c. The rate of multiple births has remained stable over recent decades.
d. Fetuses of twin pregnancies are more likely to be born in a vertex position as compared
with fetuses of singleton pregnancies.
e. Monozygotic twins account for two thirds of all spontaneous twin births.

5-66. A newborn infant is born at 26 6/7 weeks’ gestation following a spontaneous vaginal
delivery. Resuscitative efforts include warming, and drying the infant followed
by clearing and positioning the airway. The infant developed spontaneous respiratory effort
within 1 minute of life. However, he develops intercostals retractions, minimal air entry on
auscultation, grunting,and decreased tone. He then develops apnea, cyanosis,and bradycardia
(heart rate of 80 beats/min). Positive pressure ventilation is given via self-inflating bag and face
mask, but the infant remains apneic.Which is the most appropriate next step in the
management of this infant?
a. Administration of continuous positive airway
pressure (CPAP) via nasal prongs
b. Needle decompression of the right hemithorax
c. Provision of supplemental oxygen via nasal cannula
d. Endotracheal intubation for mechanical ventilation and surfactant administration

e. Administration of inhaled nitric oxide


Consultant Dr.Wahid Helmi Domiate Egypt ( Neonatoloy Revision First note)

5-67. A 30-day-old infant was born at 24 4/7 weeks’ gestation.Until today, she had been
intubated and ventilated using the synchronized intermittent mandatory ventilation
(SIMV) mode. Based on improved blood gases, she was extubated 12 hours ago to room
air. In the last 20 minutes, she has developed nasal flaring and intermittent subcostal
retractions. Her current blood gas shows a pH of 7.34 and a pco2 of 56 mm Hg.
Her pulse oxygen saturations have decreased to 82%.Her chest radiograph reveals a
normal cardiothymic silhouette, decreased lung expansion, and air bronchograms.
What is the most appropriate next step in the management of this patient?
a. Endotracheal intubation and synchronized intermittent mandatory ventilation
(SIMV)
b. Provision of supplemental oxygen via nasal cannula
c. Administration of continuous positive airway pressure (CPAP) via nasal prongs
d. Endotracheal intubation and high-frequency ventilation (HFV)
e. Repeat blood gas sampling in 1 hour

5-68. A 4-day-old term neonate with congenital pneumonia is being mechanically


ventilated for respiratory failure.He is ventilated using the synchronized intermittent
mandatory ventilation (SIMV) mode. His settings are:Positive inspiratory pressure—19
mm Hg Positive end-expiratory pressure—5 mm Hg Rate—25 breaths/min For the past
several hours you notice that the ventilator is displaying wide variability in measured tidal
volumes.His most recent arterial blood gas shows a pH of 7.5 and p co 2 of 30 mm Hg. On
the same ventilator settings,the previous arterial blood gas had a pH of 7.3 and
p co 2 of 51 mm Hg.What is the most appropriate next step in the management of this
patient?
a. Change the mode to intermittent mandatory
ventilation.
b. Add pressure support of 10 mm Hg to his current
ventilator settings.
c. Change the mode to assist control ventilation.
d. Change the mode to high-frequency oscillation
ventilation.
e. Change the mode to volume-targeted ventilation.
Consultant Dr.Wahid Helmi Domiate Egypt ( Neonatoloy Revision First note)

5-69. A term 3-day-old infant was born through meconium- stained amniotic fluid.
Following aspiration of meconium, the infant developed respiratory failure. On
current examination, he has a systolic murmur and a loud second heart sound. His
chest radiograph reveals a hyperlucent background in addition to patchy areas of
opacity. Despite the use of high-frequency oscillation and an inspired oxygen fraction
of 1.0, the postductal pulse oximeter does not read above 90%.
Which of the following treatment options is most likely to be effective in
addressing the underlying pathophysiology?
a. Permissive hypercarbia
b. Diuretic therapy
c. Inhaled nitric oxide
d. Opiate therapy
e. Albuterol

5-70. A 1-week-old infant born at 23 6/7 weeks’ gestation has developed sudden onset of
hypotension and tachycardia. She is mechanically ventilated, has been NPO since
birth, and recently completed a course
of antibiotics and hydrocortisone for septic shock. Prenatal history is notable for
intrauterine growth restriction and perinatal indomethacin exposure for preterm
labor. On examination, she has abdominal distention and guarding, lethargy, and
hypoperfusion. Chest and abdominal x-rays reveal pulmonary edema and a
pneumoperitoneum. There is no pneumatosis intestinalis or portal venous gas noted.
Which of the following is the most likely etiology of the infant’s acute
decompensation?
a. Necrotizing enterocolitis (NEC)
b. Cow’s milk protein allergy
c. Spontaneous intestinal perforation (SIP)
d. Volvulus
e. Duodenal atresia
Consultant Dr.Wahid Helmi Domiate Egypt ( Neonatoloy Revision First note)

5-71. A term infant was born to a 26-year-old, gravida


7 woman at home following a precipitous delivery. When the emergency medical
team arrived, the infant was vigorous but still attached to the placenta. The
infant was transported to a local hospital where his physical examination was
normal. Several hours later, the infant develops irritability and poor feeding. On
examination, he is tachypneic and lethargic. The remainder of the examination is
within normal limits.
Which of the following tests should be ordered first in evaluating this infant?
a. Hematocrit
b. Cranial sonogram
c. Thyroid-stimulating hormone level
d. Stool occult blood test
e. Chest radiograph

5-72. You are seeing a 3-day-old Korean male infant in your clinic for the first time
since his hospital discharge.
He was born by spontaneous vaginal delivery at
36 2/7 weeks’ gestation to a 25-year-old woman with negative prenatal serologies.
The pregnancy, delivery, and postpartum admission were uncomplicated. He has
been breast-feeding every 1 to 2 hours, but his weight on your examination has
decreased 12% from birth weight. The remainder of his examination is
unremarkable.
Which screening test is most important to perform at this visit?
a. Hemoglobin level
b. Vitamin D level
c. C-reactive protein
d. Serum lead level
e. Total serum bilirubin level
Consultant Dr.Wahid Helmi Domiate Egypt ( Neonatoloy Revision First note)

5-73. A 2-day-old infant born at 35 1/7 weeks’ gestation to a diabetic mother presents to your
office with jaundice. She is breast-feeding every 2 hours, has wet diapers with every feed,
and has passed at least 1 meconium stool. Vital signs include weight decreased 2% from
birth weight, a rectal temperature of 37.4°C, heart rate of 147 beats/min, respirations of 50
breaths/min, and blood pressure of 45/28 mm Hg. Her anterior fontanelle is soft and flat.
She is alert with appropriate tone, and
is icteric to her mid-calf. Her total serum bilirubin (TSB) level is 14.0 mg/dL, and direct
bilirubin level is
0.4 mg/dL at 48 hours of life.
Which of the following is the appropriate management of this infant’s hyperbilirubinemia?
a. Administer intravenous immunoglobulin (IV Ig).
b. Initiate phototherapy, continue breast-feeding, and recheck TSB level in 4 hours.
c. Initiate phototherapy, replace breast-feeding with formula feeding, and recheck TSB in
4 hours.
d. Initiate phototherapy, discontinue breast-feeding, initiate IV hydration, and recheck TSB
in 4 hours.
e. Place central arterial and venous catheters for exchange transfusion.

5-74. A 24-year-old, gravida 2, para 1 Hispanic woman is being seen for the first time
for prenatal care. Her previous delivery occurred in El Salvador, and she has
recently immigrated to the United Stated for this delivery. Based on her last
menstrual period, the pregnancy is dated at 34 0/7 weeks’ gestation.
Ultrasound examination of the fetus reveals thickened subcutaneous tissue and fluid
in the pleural cavity.Sampling of the amniotic fluid is significant for bile pigment,
and percutaneous umbilical blood sampling shows fetal anemia.
Which of the following is the most likely cause of this infant’s presentation?
a. Human parvovirus B19
b. Cytomegalovirus
c. Rh antibody exposure
d. Complex congenital heart disease
e. Supraventricular tachycardia
Consultant Dr.Wahid Helmi Domiate Egypt ( Neonatoloy Revision First note)

5-75. A 2-week-old infant presents to the pediatrician with a large reddish blue mass
under the lower middle aspect of the abdomen, measuring 3 cm  4 cm.
According to her mother, the lesion was noticed after discharge home from the
nursery, but has enlarged over the past 2 weeks. The mass is nontender and warm,
and a bruit is heard on auscultation of the overlying skin. The infant is vigorous with
normal vital signs and strong lower extremity pulses. There is no
hepatosplenomegaly, pallor, or petechiae. The results of her complete blood count
are:White blood cells—15,000/L Hemoglobin—11.5 g/dL Platelets—19,000/L
Which of the following syndromes is the most likely cause of thrombocytopenia in
this infant?
a. Trisomy 13
b. Trisomy 18
c. Fanconi anemia
d. Thrombocytopenia absent radii (TAR) syndrome
e. Kasabach-Merritt syndrome

5-76. A male infant was born at 36 4/7 weeks’ gestation by emergent cesarean section
for fetal bradycardia.
There was no evidence of labor prior to delivery. Clear amniotic fluid is noted on
rupture of membranes during the operative delivery. The infant’s Apgars were 9 and
9 at 1 and 5 minutes, respectively. However,he developed grunting and tachypnea
approximately 30 minutes after birth. On current examination, his respiratory rate
is 70 breaths/min and the preductal and postductal pulse oximeters read 97% while
he is breathing room air. His physical examination is otherwise normal. A chest
radiograph demonstrates prominent vascular markings, an opacity in the transverse
fissure, and flattening of the diaphragms. An arterial blood gas reveals:
pH—7.46
pco2—34 mm Hg Pao2—89 mm Hg HCO3—24 mEq/L
Which of the following mechanisms is most likely responsible for this infant’s
respiratory distress?
a. Evolving chemical pneumonitis
b. Delayed sodium transport across the alveolar apical and basolateral cell membranes
c. Ascending infection through the birth canal
d. Increased pulmonary vascular tone
e. Increased alveolar surface tension
Consultant Dr.Wahid Helmi Domiate Egypt ( Neonatoloy Revision First note)

5-77. A 2-week-old female infant was born at 24 1/7 weeks’ gestation. She has
significant respiratory insufficiency secondary to surfactant deficiency and is
currently intubated and on high-frequency mechanical ventilation. Chest x-rays
have persistently demonstrated diffuse pulmonary interstitial emphysema. Early
this morning, she abruptly became tachypneic and agitated. On examination, she is
tachycardic and hypotensive, and the cardiac impulse (PMI) is shifted to the right,
with diminished breath sounds over the left chest. Her chest x-ray is shown below:

(Reproduced, with permission, from Rudolph CD, Rudolph A, Lister G, First L, Gershon A. Rudolph’s
Pediatrics. 22nd ed. New York: McGraw-Hill, 2011.)

Which of the following is the most appropriate next step in the management of this
patient?
a. Decompressing the left hemithorax with an angiocatheter, followed by chest tube
placement
b. Repositioning the endotracheal tube
c. Inserting a nasogastric tube and aspirating gastric air and fluid
d. Increasing mean airway pressure on the ventilator
e. Performing an emergent pericardiocentesis
Consultant Dr.Wahid Helmi Domiate Egypt ( Neonatoloy Revision First note)

5-78. A 23-year-old primigravida woman presented to Labor and Delivery today for
decreased fetal movement. Her pregnancy history was significant for chronic
ibuprofen use to treat low back pain in the third trimester.
Nonstress testing revealed nonreassuring fetal heart tones, and the woman
underwent an emergent cesarean section. Thirty minutes following delivery, the baby
has tachypnea, nasal flaring, and grunting. His preductal and postductal pulse
oximeter readings are 97% and 86%, respectively. His chest x-ray shows hyperlucent
lung fields and an enlarged cardiothymic silhouette. A postductal arterial blood gas
on room air demonstrates the following values:
pH—7.30
pco2—37 mm Hg Pao2—55 mm Hg HCO3—19 mEq/L
Which of the following tests is the preferred method to confirm this patient’s
diagnosis?
a. Echocardiography with Doppler studies
b. Serial blood gas sampling
c. Pulse oximetry
d. Tracheal aspirate for bacterial culture
e. Arterial blood sample for bacterial culture
Consultant Dr.Wahid Helmi Domiate Egypt ( Neonatoloy Revision First note)

5-79. In which of the following scenarios is pulmonary hemorrhage most frequently


associated?
a. A term 2-day-old born following maternal cocaine use
b. A term 2-day-old with sepsis
c. A term 2-day-old born with perinatal depression
d. A preterm 2-day-old with a patent ductus arteriosis
e. A term 1-month-old with congenital heart disease

5-80. A 3-day-old infant born at 25 weeks’ gestation is undergoing high-frequency


oscillatory ventilation for respiratory insufficiency. Her x-rays since birth have
demonstrated a diffuse reticulogranular pattern with low lung volumes.
Approximately 2 hours after
administration of exogenous surfactant, she developed pallor, bradycardia, and a
bloody nasal discharge. A chest x-ray now reveals patchy infiltrates throughout the
lung fields. Abruptly, her pulse oximeter saturations decline.
What is the first step in the management of this patient?
a. Suction and secure the airway.
b. Transfuse packed red blood cells.
c. Transfuse platelets.
d. Increase the mean airway pressure.
e. Administer indomethacin.
100

ANSWERS
effect will be transient and the dose must be repeated in early
infancy. Guidelines for sequential oral dosing of vitamin K
Answer 5-1. e vary. Some physicians recommend a second dose at 6 to 8
Early vitamin K deficiency bleeding (VKDB), previously months of age; others recommend weekly dosing while
known breastfeeding.
as hemorrhagic disease of the newborn (HDN), is caused Failure of vitamin K therapy to prevent VKDB has
by inadequate levels of active vitamin K–dependent clotting been reported, and is associated with noncompliance with
factors (factors II, VII, IX, and X) and has an incidence that therapy (Srehle, 2010; Doran, 1995). In addition, newborns
has been reported as high as 1.7% (AAP, 2003). VKDB can receiving incomplete oral prophylaxis have a higher risk of
result in intracranial, gastrointestinal, or generalized bleeding developing VKDB.
in the neonate. Due to limited neonatal stores at birth and In 1993, the Vitamin K Ad Hoc Task Force of the American
supplies from enteral sources (including breast milk), vitamin Academy of Pediatrics (AAP) reviewed concerns about the
K association of intramuscular vitamin K injection and the
supplementation is necessary to prevent neonatal hemorrhage. incidence of childhood leukemia. The committee concluded
A single 1-mg dose of intramuscular vitamin K in the first that there was no association between the intramuscular
few hours of life will prevent VKDB. Some parents request administration of vitamin K and childhood leukemia or
oral vitamin K administration (in lieu of an intramuscular other cancers. This conclusion has been supported by several
injection). Oral administration of the intramuscular
subsequent case–control studies (AAP, 2003).
formulation by mouth is safe. However, although a single 2-mg
oral dose of vitamin K will prevent VKDB for a few days, the
84

Answer 5-2. c Caput


Cephalohematoma
The neonate in this vignette has physical examination Subgaleal hemorrhage
findings consistent with a congenital diaphragmatic hernia Extradural hemorrhage
(CDH). This is a rare condition with an overall incidence of Skin
1 in 2500 births. The large majority of cases are detected by Epicranial
aponeurosis
routine prenatal ultrasound. The majority of CDHs (roughly
Periosteum
85%) occur on the left side through a posteriolateral defect Skull
(Bochdalek hernia). Dura
Infants with CDH develop respiratory failure due to
pulmonary hypoplasia (decreased lung volume on the
affected side) and pulmonary hypertension (due to abnormal (Reproduced, with permission, from Hay W, Levin M, Deterding R, Abzug M.
pulmonary vascular development). Other examination findings Current Diagnosis & Treatment Pediatrics. 21st ed. New York: McGraw-Hill,
2012.)
include a scaphoid abdomen (due to displacement of the
peritoneal contents into the thorax) and displacement of the
Answer 5-4. d
heart to the midline.
Initial management involves decompression of the Most IDMs are macrosomic, which is defined as a birth weight
stomach with a nasogastric tube (to reduce gastric distention greater than the 90th percentile for gestational age or over
impacting thoracic and mediastinal contents [heart and great 4000 g. This increased body weight is a result of deposition
vessels]). Subsequent respiratory management should include of fuel sources, resulting in increased body fat and mass of
endotracheal intubation. Positive pressure ventilation with a visceral organs, including the heart.
T-piece resuscitator (a flow-controlled resuscitation device HCM involves overgrowth of the intraventricular septum
with adjustable peak inspiratory and positive end-expiratory and 1 or both ventricular walls, leading to ventricular outflow
pressures) or a bag and mask will insufflate the stomach and tract obstruction. Due to this ventricular outflow tract
may impair respiratory function. Transillumination or needle obstruction coupled with decreased myocardial function, the
decompression of a normal right hemithorax is not indicated neonate with HCM has decreased cardiac output and may
in this case. develop congestive heart failure.
Insulin, not GH, represents the primary anabolic fetal
growth factor. Excess glycogen and fat deposition occurs in
the presence of insulin and high glucose levels, and this fat is
Answer 5-3. b primarily deposited in the abdomen and subscapular areas,
putting these infants at risk for shoulder dystocia. However,
The infant in this vignette has a cephalohematoma, which
this excess growth does not affect skeletal muscle, leading
is a subperiostial hemorrhage often associated with trauma
IDMs to have higher weight percentiles at birth relative to
during labor (see figure). The hemorrhage presents hours
length and head circumference.
after delivery as a fluctuant mass over the affected skull bone
Interestingly, not all IDMs are macrosomic. In women
(often the parietal or occipital bone). The hemorrhage is
with pregestational diabetes and associated nephropathy,
limited by the periosteum of the skull bone and, as a result,
retinopathy, or chronic hypertension, placental insufficiency
does not extend beyond the suture lines. Blood loss is minimal,
may occur. The placental insufficiency limits nutrient and
and affected infants usually do not require transfusion of
oxygen delivery, which can result in a small-for-gestational-age
blood products. The most common neonatal complication is
(SGA) infant.
hyperbilirubinemia due to the breakdown of the extravasated
blood. In the absence of localized or systemic infection,
antibiotics should not be administered. Needle aspiration of
the hematoma is not indicated and may introduce skin flora Answer 5-5. a
into the hematoma, resulting in infection of the mass and
The neonate in this vignette has examination findings
surrounding skin.
consistent with bilateral choanal atresia. The etiology is
A cephalohematoma should be distinguished from 2 other
due to a persistence of a bony septum (90%) or a soft tissue
entities (see figure). A caput succedaneum is edema of the
membrane (10%) (see figure below).
scalp and tends to cross suture lines. A subgaleal hemorrhage
This condition is an emergency situation in neonates (who
is bleeding between the galea aponeurotica and the skull.
are obligate nose breathers). Placement of an oral airway and/
This condition can lead to extensive hemorrhage and shock.
or putting the baby in a prone position at the time of clinical
As a result, this is an emergency situation requiring close
presentation will bypass the nasal obstruction and allow for air
monitoring, as well as potential replacement of blood and
exchange.
clotting products.
Neither supplemental oxygen and pressure (via nasal
cannula or CPAP) nor intranasal steroids will overcome the
85

anatomic obstruction. While unilateral atresia can generally be


delayed until the child is 1 to 2 years of age, bilateral choanal
atresia requires prompt repair.

Bony
obstruction

FIGURE 215-1. The Barlow test for developmental dislocation of the hip in
a neonate. (A) With the infant supine, the examiner holds both of the child’s
knees and gently adducts 1 hip and pushes posteriorly. (B) When the exami-
nation is positive, the examiner will feel the femoral head make a small jump
(arrow) out of the acetabulum (Barlow sign). When the pressure is released, the
head is felt to slip back into place. (Reprinted with permission from Herring JA,
(Page 256, Section 5: Newborn) ed. Tachdjian’s Pediatric Orthopaedics. 4th ed. Philadelphia: Saunders; 2007.)

Answer 5-6. c
Although her physical examination findings at birth were
unremarkable, the infant presented in this vignette is at risk
for DDH due to a history of breech presentation. As DDH
can develop any time in the first year of life, the Ortolani
and Barlow maneuvers (see Figures 215-1 and 215-2) may
initially be negative. Risk factors increasing the incidence of A
this condition in infants include female sex and birth from
the breech position. In this infant with 2 risk factors, current
guidelines for DDH screening recommend hip ultrasonography
at 6 weeks of age. Radiographs are not reliable until 4 to
6 months of age, when there is greater ossification of the
femoral head (AAP, 2000).
Leg length discrepancy or gluteal/leg fold asymmetry
may be significant in infants with DDH, but these findings
are not diagnostic. Magnetic resonance imaging is not the
recommended imaging modality for diagnosis of DDH.
B
Reexamination of the hips should be performed at every
well-child care visit during the first year of life but should not FIGURE 215-2. The Ortolani test for developmental dislocation of the hip
replace screening hip imaging in at-risk infants. in a neonate. (A) The examiner holds the infant’s knees and gently abducts
the hip while lifting up on the greater trochanter with 2 fingers. (B) When the
test is positive, the dislocated femoral head will fall back into the acetabulum
(arrow) with a palpable (but not audible) “clunk” as the hip is abducted
(Ortolani sign). When the hip is adducted, the examiner will feel the head
redislocate posteriorly. (Reprinted with permission from Herring JA, ed.
Tachdjian’s Pediatric Orthopaedics. 4th ed. Philadelphia: Saunders; 2007.)
86

practice among clinicians, supplemental oxygen use and


target saturations vary widely in clinical practice. Walsh
and colleagues developed an oxygen-need test to make the
TABLE 42-4. Laryngoscope Blade Size, Endotracheal Tube Size, and diagnosis of BPD more uniform. The infant in this vignette
Depth of Insertion for Babies of Various Weights and would meet the “Walsh criteria” for BPD if he met the
Estimated Gestational Age (EGA) conditions of choice e (saturations between 90% and 96%
Blade Tube Size EGA Depth of while receiving an FiO2 of over 0.3). If maintained on an
Size (mm) Weight (g) (Weeks) Insertion (cm) FiO2 of greater than 0.3 to keep saturations greater than 96%,
No. 00 2.5 Below 750 Below 27 ∼6.5 he would require a room air challenge (with demonstration
of oxygen saturations of 90% or less) to be diagnosed with
No. 0 2.5 750-1000 27-28 ∼7.0 BPD. During a room air challenge, those infants who cannot
No. 0 3.0 1000-2000 28-34 ∼7.0-8.0 maintain saturations >90% during weaning and in room air for
No. 1 3.5 2000-3000 34-38 ∼8.0-9.0 over 30 minutes were also diagnosed with BPD.
The diagnostic definition of BPD as oxygen need at
No. 1 3.5-4.0 >3000 >38 ∼9.0-10.0
36 weeks does not require antecedent exposures (eg, RDS,
Adapted from Kattwinkel J. Textbook of Neonatal Resuscitation. 5th ed. Elk Grove mechanical ventilation), abnormalities on a chest radiograph,
Village, IL: American Academy of Pediatrics and American Heart Association; 2006. antecedent infectious exposures (antenatal chorioamnionitis or
postnatal sepsis), or any laboratory test.

Answer 5-7. a (Page 253, Section 5: Newborn)

Endotracheal intubation may be performed at any point during Answer 5-9. d


neonatal resuscitation. The infant in this vignette requires
intubation based on worsening respiratory distress despite Bronchopulmonary dysplasia (BPD) is a disease due to
prolonged PPV. immature lung development (see Figure 59-1). Infants who
Using data in Table 42-4, an infant weighing 1200 g should die from BPD show pathologic changes consistent with arrest
be intubated with a 3.0 mm endotracheal tube, and the airway of lung development. The lungs have an emphysematous
should be visualized using a number 0 laryngoscope blade. appearance due to decreased alveolarization. In addition,
The presence of anatomic variations in the trachea (ie, airway alveoli present in these lungs have large diameters and contain
masses, stenosis) may necessitate use of a smaller endotracheal dysplastic type II cells.
tube. Other pathologic findings include decreased pulmonary
microvasculature, an interrupted collagen network, and
(Page 168, Section 5: Newborn) abnormal elastin localization (ie, distribution away from fibers
where alveolar septations should later occur). The lungs may
Answer 5-8. e also demonstrate acute inflammatory changes due to the
BPD is defined as the need for supplemental oxygen at 36 bacterial infection at the time of the infant’s demise.
weeks postconceptional age. However, due to inconsistent (Page 254, Section 5: Newborn)

Hits to fetal lung Hits during transition Postnatal hits

Oxygen PDA
Initiation of
Chorioamnionitis ventilation Ventilation Sepsis

Fetal lung Preterm lung BPD

Corticosteroids Corticosteroids Nutrition

FIGURE 59-1. Risk factors for bronchopulmonary dysplasia, a disease with a single root cause: a very immature lung. Multiple factors then contribute to injury
and interfere with lung development. Fetal exposures to chorioamnionitis and corticosteroids, preterm birth with lung injury during resuscitation, and postnatal
care (ventilatory and oxygen) and diseases such as sepsis and patent ductus arteriosus represent a series of “hits” to the preterm lung. (Reproduced, with permis-
sion, from Rudolph CD, Rudolph A, Lister G, First L, Gershon A. Rudolph’s Pediatrics. 22nd ed. New York: McGraw-Hill, 2011. <http://www.accesspediatrics.com>.)
87

Answer 5-10. e Answer 5-13. d


BPD has a multifactorial etiology, and no single intervention Hypothermia (defined as a reduction in core temperature by
has been demonstrated to reduce its incidence. Lung 1°C-6°C) has been demonstrated to reduce the deleterious
immaturity is a major factor in the pathogenesis of BPD; cellular effects of brain ischemia in experimental animal
therefore, preventing preterm delivery would have the most models, including excitatory neurotransmitter release,
substantial impact on decreasing the incidence of BPD. microglial activation, and free radical production. Data
However, as in this vignette, interruption of progressive from 2 large randomized control trials and 1 large pilot trial
preterm labor (and, ultimately, preterm delivery) is not of therapeutic hypothermia in neonates were published in
always possible. Therefore, treatment strategies should be 2005. In all of these studies, hypothermia was initiated within
implemented in preterm neonates to reduce BPD. Supplemental 6 hours of birth and maintained for 48 to 72 hours.
oxygen use (based on gestation-specific guidelines) and low Both the NICHD randomized trial and the pilot trial
ventilatory pressures during resuscitation and mechanical by Eicher et al used whole-body hypothermia; newborns
ventilation can reduce the initiation and progression of BPD. with HIE receiving this therapy had decreased death or
Postnatal, high-dose corticosteroids may facilitate extubation of moderate-to-severe disability at 12 months (Eicher) or
ventilator-dependent infants, but do not decrease the incidence 18 months (NICHD) relative to HIE infants who were
of BPD. Furthermore, antioxidant therapy (with drugs such kept normothermic. The CoolCap Trial, which provided
as vitamin C) and reducing Ureaplasma colonization from selective head cooling to infants with moderate-to-severe
birth (with parenteral erythromycin) have not been shown to encephalopathy, demonstrated protective effects (decreased
decrease the incidence or prevent BPD. death or disability in survivors at 18 months) in infants
with HIE who demonstrated less severe aEEG abnormalities
at admission to the study. Recent data from subsequent
Answer 5-11. a studies (the European Network study, the UK Total Body
Hypothermia [TOBY] trial, and the Australasian Infant
Antenatal removal of a SCT is one of the few indications for Cooling Evaluation [ICE] study) have further advanced the
open fetal surgery. This tumor, thought to be a remnant of the understanding of the benefits of hypothermia. However, data
primitive streak in early development, affects only 1 in 25,000 on neurodevelopmental outcomes in adolescents treated with
live births. However, the consequences of a rapidly expanding hypothermia as neonates have not been ascertained.
teratoma (preterm labor, polyhydramnios, hydrops, fetal
congestive heart failure) are significant to the mother and fetus.
The primary goal of fetal surgery for SCT is interruption of
the large vascular connections between the tumor and the fetus Answer 5-14. b
that result in nonimmune hydrops. Successful fetal surgery for
SCT is accomplished at the time of initial onset of hydrops, Venous cranial infarcts are less common than arterial strokes
not late in the course. Preservation of the anorectal sphincter in neonates. Factors that may predispose neonates to venous
complex, if possible, is attempted, and removal of pelvic strokes include hypovolemia, polycythemia, decreased blood
components is deferred until after the infant delivers. flow (in the setting of preeclampsia), and infection. Magnetic
resonance imaging (MRI) may demonstrate injury to the deep
gray matter and thalamus. Additionally, concomitant magnetic
venography may show occlusion or recanulation of the
Answer 5-12. a
affected vessel as well as development of collateral circulation.
The cellular processes associated with neonatal hypoxic At present, there are no definite anticoagulant therapies
ischemic encephalopathy can be divided into 2 separate available for management of neonatal stoke based on data
pathophysiologic phases. Primary energy failure occurs from large randomized clinical trials. Additionally, trials of
secondary to decreases in cerebral blood flow, resulting in erythropoietin in the prevention of neonatal strokes in humans
reduced substrate (ie, glucose) and oxygen delivery to brain are in progress and have not demonstrated clinical benefit in
tissue. Cellular derangements include loss of high-energy reducing effects of stroke.
compounds (such as adenosine triphosphate [ATP] and
phosphocreatine), excessive release or failure of reuptake of
excitatory neurotransmitters (such as glutamate), disturbed
ionic homeostasis (including increases in intracellular calcium), Answer 5-15. c
and decreased cellular protein synthesis. Secondary energy
The neonate in this vignette has findings consistent with
failure, whose severity is related to the degree of primary energy
Beckwith-Wiedemann syndrome (BWS), an overgrowth
failure, results in a constellation of neurodegenerative processes
disorder associated with neonatal hypoglycemia.
(including oxidative injury, neuronal apoptosis, accumulation of
Visceromegaly, including pancreatic hypertrophy, occurs in
excitatory neurotransmitters, and inflammation).
these infants, which is associated with hyperinsulinemia and
hypoglycemia; however, other factors may be involved.
88

Symptoms may extend beyond the immediate neonatal Answer 5-17. c


period. Hypoglycemia in these infants may improve with more
Neonatal RDS or hyaline membrane disease (HMD) is the
frequent feedings or supplemental glucose administration.
most common cause of respiratory failure in the first days
Fatty acid oxidation defects and impaired glycogenolysis can
after birth, occurring in 1% to 2% of newborn infants. Until
result in neonatal hypoglycemia but are not associated with
about 25 years ago, approximately 50% of infants with this
BWS. The infant’s feeding trial in the office does not indicate
condition died. However, improved methods of treatment
congestive heart failure or swallowing dysfunction as a cause of
over the past 3 decades have markedly reduced mortality
increased caloric demand or impaired calorie intake.
from this condition. RDS occurs mainly in premature infants
and is more common in white infants than in black infants.
The characteristic clinical features of infants with RDS are
expiratory grunting, tachypnea, retractions (involving the
intercostal and sternal muscles), and central cyanosis.
Answer 5-16. b Persistent pulmonary hypertension of the newborn and
transient tachypnea of the newborn are more common causes of
Umbilical venous blood, which is delivered from the placenta,
respiratory distress in near-term (born at 34 0/7-36 6/7 weeks’
has the highest oxygen concentration in fetal blood (see
gestation) and term (born at 37 0/7 weeks’ gestation or later)
figure). This blood is streamed across the right atrium,
infants. Meconium passage in utero before 32 weeks’ gestation is
through the foramen ovale, and into the left atrium, where
rare, making meconium aspiration syndrome less likely in this
this well-oxygenated blood is delivered to the head and upper
infant. Congenital heart disease could present at 6 hours of life,
extremities. Less-oxygenated blood from the vena cava is
but is less common than RDS in an infant of this gestational age.
streamed through the right atrium and ventricle, and then
through the ductus arteriosus to the descending aorta and the
umbilical arteries.
Answer 5-18. a
Pulmonary surfactant, a mixture of proteins and
phospholipids, is synthesized in alveolar epithelial type II cells.
The phospholipid dipalmitoylphosphatidylcholine makes up
Ductus
arteriosus about 45% to 50% of the mass of surfactant and is the main
surfactant component that lowers surface tension. The ratio
of amniotic fluid concentrations of surfactant phospholipids
Superior vena cava (lecithin ([L] and sphingomyelin [S]) has been used to
determine fetal lung maturity.
LA An L:S ratio of 2:1 suggests a lower risk of the fetus
Foramen ovale developing respiratory distress syndrome after birth. Of the 4
RA
surfactant apoproteins (surfactant proteins A, B, C, and D), a
LV congenital deficiency in surfactant protein B production has
RA
Inferior vena cava been associated with fatal respiratory failure in term neonates.
Ductus Synthesis and storage of surfactant begin around 16 weeks’
venosus gestation, but it is not secreted into the amniotic fluid until 28
to 38 weeks’ gestation, at approximately the same time alveolar
development begins.
Portal Aorta
sinus
Portal v.
Answer 5-19. d
Umbilical aa.
Neonates with respiratory distress should receive the least
invasive therapy that supports their pulmonary needs.
Umbilical v. The infant in this vignette is a late preterm infant with
Hypogastric
aa. clinical evidence of respiratory distress syndrome (RDS).
Oxygenated Physiologically, the infant has atelectasis and hypoxia, which
Mixed can be reversed with positive pressure and oxygen support.
Deoxygenated Since she is not apneic, the baby should receive respiratory
Placenta support with CPAP and supplemental oxygen to maintain age-
(Reproduced, with permission, from Strange GR, Ahrens WR, Schafermeyer appropriate oxygen saturations. Intubation and mechanical
RW, Wiebe RA, eds. Pediatric Emergency Medicine. 3rd ed. New York: ventilation (either IMV or HFOV) should be started only if
McGraw-Hill, 2009.) she develops worsening respiratory distress (with respiratory
89

acidosis apnea, or hypoxia) while on CPAP. Nitric oxide is a these infants have higher oxygen consumption and higher
pulmonary vasodilator used for conditions causing respiratory resting energy expenditure due to increased heat loss. Finally,
failure in term newborns (such as meconium aspiration SGA infants have impaired insulin secretion (leading to poor
syndrome) but is not indicated for routine use in premature anabolic growth) and impaired immunologic function during
infants with RDS. Titrating nitrogen into the oxyhood would infancy and childhood (resulting in subtherapeutic response to
decrease the FiO2 and potentially worsen the infant’s respiratory vaccinations).
distress.

Answer 5-23. a
Answer 5-20. d
Establishment of glucose homeostasis in the newborn occurs
Small-for-gestational-age (SGA) infants lose heat rapidly via a series of hormonal and enzymatic changes initiated with
because of their large surface area relative to body weight and clamping of the umbilical cord. Basal glucose production in
their scant subcutaneous fat stores. To prevent hypothermia, a neonate is approximately 2- to 3-fold greater than adult
this infant should be dried quickly and completely, placed production, which is related to the large consumption of
under a radiant warmer, and protected from drafts with glucose by the neonatal brain. Hepatic glycogen stores are
warmed blankets. He is crying and has a heart rate above converted to glucose in response to increases in circulating
100 beats/min; therefore, administering PPV, CPAP, or catecholamines and glucagon at delivery as well as decreases
intravenous volume would not be appropriate for his in serum insulin levels. The glycogen stores rapidly decline in
resuscitation in the delivery room. Although SGA infants are the term infant and are nearly exhausted by 12 hours of age,
at risk for hypoglycemia, administration of dextrose in the requiring gluconeogenesis or delivery of exogenous glucose to
delivery room is not indicated. achieve acceptable glucose levels.
Gluconeogenesis is regulated by increases in cytosolic
phosphoenylpyruvate kinase (triggered by delivery) and the
oxidation of free fatty acids (in particular, medium-chain
Answer 5-21. e
triglycerides).
Hypoglycemia is common in small-for-gestational-age
(SGA) infants, including the infant in this vignette. Of note,
hypoglycemia increases with the severity of intrauterine growth
Answer 5-24. d
restriction. The risk of hypoglycemia is greatest during the
first 3 postnatal days, but fasting hypoglycemia may occur The infant in this vignette has developed hyponatremia
repetitively for several days after birth. This early hypoglycemia secondary to inappropriate antidiuretic hormone secretion
usually results from insufficient hepatic and skeletal muscle and renal failure after perinatal depression. Of the choices
glycogen content and is exacerbated by the lack of alternative listed, reduction in intravenous fluid infusion (as well as
energy sources (because of scant adipose tissue and decreased serial monitoring of electrolytes) is the correct choice for
lactate concentrations). Early enteral feeding usually can management of this infant. Administration of additional fluid
prevent hypoglycemia. Less commonly, hyperinsulinemia (a normal saline bolus) is not indicated in a well-perfused
and/or increased sensitivity to insulin may also contribute to infant with intrinsic renal failure (oliguria with a BUN/
hypoglycemia. This insulin is fetally/neonatally derived, as creatinine ratio of 10-20). Diuretic administration (either
maternal insulin does not cross the placenta. Finally, deficient furosemide or bumetanide) is not indicated in infants with
catecholamine responses to low blood sugar levels (seen in intrinsic renal failure; furthermore, diuretics increase renal
SGA infants) may also result in persistent hypoglycemia. sodium losses and may worsen hyponatremia. Increasing the
Increased gluconeogenesis would decrease, not increase, the dextrose in maintenance fluids of a normoglycemic infant may
incidence of hypoglycemia in this infant. cause hyperglycemia, which introduces free water into the
vascular space and may worsen hyponatremia.

Answer 5-22. c
Answer 5-25. d
Small-for-gestational-age (SGA) infants have multiple
metabolic abnormalities in the neonatal period. Protein This infant’s presentation is most consistent with neonatal
digestion and absorption is impaired in these infants due to encephalopathy (probably hypoxic ischemic encephalopathy
decreased intestinal size and function. SGA infants have lower [HIE]). There is no history of a prenatal or intrapartum
muscle masses relative to AGA infants, and although muscle event that would compromise fetal blood flow, but in many
accretion is a priority in these babies, their tolerance of high cases of neonatal encephalopathy, such an event cannot be
protein/amino acid administration may be limited. identified. Information that can be useful in the first hours
SGA babies also have increased plasma triglyceride of life to assist in management of these infants includes
concentrations due to deficient cellular uptake. Additionally, serial neurodevelopmental assessments, neuroimaging,
90

and electroencephalography. Based on this infant’s initial Answer 5-28. d


presentation (moderate-to-severe encephalopathy with
A complication of 7% to 10% of all pregnancies, preeclampsia
metabolic acidosis), strong consideration should be given to
results from vasoregulatory abnormalities in the placenta
initiating hypothermia.
and the gravid woman. Mild preeclampsia is diagnosed by an
This therapy has shown efficacy if initiated within the first
increase in blood pressure (greater than 140/90 mm Hg) and
6 hours of life (not 12 hours of life), and can be initiated before
proteinuria. The woman in this vignette, however, has evidence
obtaining neuroimaging or an electroencephalogram, especially
of severe preeclampsia (with blood pressure of greater than
if transport to another hospital is required. Neonatal seizures
160/90 mm Hg and 3+-4+ proteinuria).
can occur as a result of HIE, and may exacerbate the brain
Emergent cesarean section is not indicated presently, as
injury. Empiric phenobarbital therapy has been considered
more data should be collected regarding maternal and fetal
for neonates with HIE; however, this strategy has not been
status. Medical management of the pregnancy at this time
demonstrated to reduce cerebral palsy or mental retardation in
includes hospital admission, intravenous magnesium sulfate
these infants. Clinicians cannot predict definitively neurologic
administration (to decrease the risk of seizures and provide
outcomes for infants with HIE. However, certain neonatal MRI
fetal neuroprotection), antihypertensive therapy (β-blockers,
findings (as described on page 225), in addition to clinical and
calcium channel blockers) to control blood pressure, and
electroencephalographic evidence of ongoing encephalopathy
monitoring maternal blood pressure, urine output, and end-
(over the first week of life), are poor prognostic signs for
organ (renal, hepatic, hematologic, and neurologic) function.
neurodevelopment in these neonates.
Fetal well-being, amniotic fluid index, and estimated fetal
weight should also be assessed.
Ultimately, however, if the preeclampsia worsens, delivery
Answer 5-26. b of the fetus would be indicated. If preeclampsia occurs at a
The features of the ultrasound are suggestive of Down gestational age earlier than 34 weeks, maternal corticosteroid
syndrome. The risk of Down syndrome increases with therapy should be given, with delay of delivery for 48 hours
maternal age. Women at high risk of having a fetus with Down (if possible). Antenatal steroids decrease the risk and severity
syndrome will have low serum α-FP, high hCG, and low uE of certain conditions seen in premature infants, including
levels adjusted for gestational age. If the maternal plasma respiratory distress syndrome and necrotizing enterocolitis.
tests indicate increased risk, a level 2 ultrasound can identify Hemodialysis is not an appropriate therapy in this patient.
structural findings in the fetus frequently associated with
Down syndrome (increased nuchal fold, nasal bone hypoplasia,
and decreased fetal extremity length). The diagnosis can be Answer 5-29. c
confirmed by karyotype of fetal cells from amniotic fluid. During fetal development, the ductus arteriosus diverts blood
away from the fetal lungs to the descending aorta and placenta
(see Figure 494-3). Postnatally, the ductus arteriosus closes
Answer 5-27. a through vasoconstriction and remodeling. The risk of PDA is
inversely associated with gestational age.
Fetal heart rate and uterine contraction monitoring are used Multiple interacting factors, including increased sensitivity
routinely throughout labor to assess fetal status and the labor to prostaglandins, influence patency of the ductus arteriosus
pattern. While fetal monitoring is not highly predictive of in premature infants. During episodes of sepsis or necrotizing
outcomes, some fetuses at high risk for birth depression can be enterocolitis, circulating prostaglandin E2 concentrations can
identified. The above tocodynamometer tracing shows a fall reach pharmacologic levels in premature infants, causing
in fetal heart rate after the onset of uterine contractions, with vasodilation of the ductus. Under these circumstances,
a gradual return to baseline after the contractions have ceased. responsiveness of ductal tissue to inhibitors of prostaglandin
This pattern is consistent with late decelerations and suggests synthesis (indomethacin or ibuprofen) is limited. Levels
decreased fetal oxygenation due to reduced placental perfusion of indomethacin on day 8, following early administration
(which occurs with uterine contractions). With intermittent of intraventricular hemorrhage prophylaxis, would not be
cord compression, variable decelerations may occur, which adequate to promote PDA closure.
present as abrupt dips in the fetal heart rate (usually a brief Antenatal steroid therapy is associated with a decreased
acceleration followed by a deceleration). (not increased) risk of PDA. Decreased peak end-expiratory
Vagal stimulation due to fetal head compression would pressure after extubation is unlikely to play a major role in
cause an early deceleration (as opposed to a late deceleration). the maintenance of ductal patency. Enteral feeding results in
An early deceleration is manifested as a decrease in fetal heart shunting of blood to the mesenteric circulation. This could
rate associated with the contraction. Normal fetal movement decrease the degree of left-to-right shunting, which could result
would result in an acceleration of the fetal heart rate. Fetal in less pulmonary overcirculation but not increase the risk of
hiccups (singultus) do not decrease the fetal heart rate. prolonged PDA patency.
91

cardiac anomalies in this infant but is not the most appropriate


initial management step. Finally, administering dextrose may
SVC RPA Aorta be necessary if this large-for-gestational-age (LGA) infant is
PDA
RPV hypoglycemic but is not the most important intervention for
this infant at this time.
LPA
RA
PT LA LPV
Answer 5-32. a
Approximately 10% to 15% of all live births are associated with
meconium-stained amniotic fluid, although only 4% to 5%
of these infants will develop meconium aspiration syndrome.
IVC
LV Meconium represents the contents of the fetal intestine and
consists of a variety of substances, including bile-containing
intestinal secretions, blood, and amniotic fluid. Meconium
can injure the lung through multiple mechanisms, including
mechanical obstruction of the airways, chemical pneumonitis,
inactivation of endogenous surfactant, and vasoconstriction of
pulmonary vessels, all of which prevent adequate ventilation
and oxygenation in the immediate postnatal period. The ease
FIGURE 494-3. Patent ductus arteriosus. (Reproduced, with permission, of intubation of the baby in the vignette would make in airway
from Rudolph CD, Rudolph A, Lister G, First L, Gershon A. Rudolph’s Pediatrics. edema a less likely answer. Bacterial pneumonia or primary
22nd ed. New York: McGraw-Hill, 2011. <http://www.accesspediatrics.com>.)
surfactant deficiency also would be a less likely cause of this
neonate’s respiratory distress.

Answer 5-30. b
Cyanosis is due to the presence of deoxygenated hemoglobin Answer 5-33. c
in vessels visible on the skin surface and mucosa. Cyanosis in
the neonate tends to become apparent when there is about 3 This infant with a history of shoulder dystocia and a left-sided
to 5 g/dL of deoxygenated hemoglobin, but detection varies Erb’s palsy likely has a brachial plexus injury. In addition to left
widely depending on lighting, observer differences, and arm weakness, the injury in this infant will result in limited
pigmentation of the skin, among other factors. The oxygen left diaphragm movement (due to phrenic nerve injury). A
binding capacity of fetal hemoglobin in the newborn (which is chest radiograph would demonstrate the left hemidiaphragm
higher than the capacity in an adult) also decreases the degree higher than the right hemidiaphragm. Based on the physical
of desaturation at a given Pao2. examination findings, this infant is less likely to have a congenital
diaphragmatic hernia (indicated radiographically by peritoneal
contents in the left hemithorax), right-sided heart congenital
disease or pulmonary hypertension (evidenced by decreased
Answer 5-31. e pulmonary vascular markings), a cystic congenital adenomatoid
The infant in this vignette has central cyanosis with no malformation (CCAM, with cystic lesions in the lung), or a
respiratory distress and a minimal response to supplemental pleural effusion (fluid in the pleural space creating mass effect).
oxygen. These findings suggest cyanotic congenital heart
disease in this neonate. At this point, the caregiver should seek
advice from a cardiologist and transfer the baby to a tertiary
Answer 5-34. a
care center for an echocardiogram and careful monitoring
in a neonatal intensive care unit (NICU). It is important to Fetal neuromaturation prior to term typically progresses in
remember that oxygen may promote ductus arteriosus closure a standard manner with some individual variation. The late
and increase pulmonary vasodilation, which may exacerbate preterm infant in this vignette is likely to have a coordinated
the hypoxia in children with left-sided obstructive lesions (eg, pattern of sucking with discrete pauses for breathing, although
hypoplastic left heart syndrome). Thus, titration of oxygen (to her suck may be weaker than the suck of a term infant.
an FiO2 0.4-0.6) in this infant is appropriate until his cardiac Additionally, she should fixate on a bull’s eye (ability present at
lesion can be defined. 34 weeks’ gestation) with minimal, unsustained tracking that
Admitting a cyanotic infant to a full-term nursery at any will develop over subsequent weeks.
hospital is not appropriate. Additionally, in the absence of In the third trimester of pregnancy, flexor tone increases
respiratory distress, intubation and ventilation is not necessary initially in the lower extremities and progresses in a cephalad
in this infant. A chest CT may be useful in pulmonary and/or direction. Therefore, this infant should not have good flexor
92

tone in the upper extremities in the absence of full flexion compromised blood flow to the fetus, also increases the risk of
of the lower extremities. Some tone in the shoulders and perinatal stroke in this case.
upper extremities, however, exists by 36 weeks’ gestation, Investigation of neonatal brain injury should always include
and would prevent the infant from stretching her right elbow trauma as a part of the differential diagnosis. In this scenario,
to the contralateral shoulder (the anterior scarf sign). The however, the timing of the seizure and the perinatal risk factors
degree of upper extremity tone will increase over subsequent are most consistent with trauma having occurred around the
weeks, keeping the elbow from reaching the midline when this time of delivery, not due to a postnatal injury.
maneuver is attempted at term. Postnatal strokes can be related to congenital heart
Antenatal magnesium exposure can temporarily alter the lesions and repair, as blood flow can bypass the pulmonary
neurologic examination, and cause decreased tone. However, vasculature. However, postnatal seizure is unlikely to be the
despite magnesium exposure, the primitive reflexes should be presenting symptom.
present in this neonate. Approximately two thirds of neonatal strokes are arterial
in origin, and the remainder are due to sinovenous thrombus.
(Page 180-182, Chapter 5: The Newborn)
This injury is, therefore, less likely to be venous in origin.
Answer 5-35. d Recent work has raised questions of the value of a workup for a
hypercoagulable state in the evaluation of a neonatal stroke.
The resuscitation scenario above raises concern for perinatal There are no large randomized clinical trials of
depression in the baby, which can result in hypoxic-ischemic anticoagulation therapy for neonatal strokes, and there are no
injury to the brain. The baby’s rapid response to positive pressure definitive recommendations for such therapy in this case.
ventilation, however, suggests minimal perinatal depression.
However, in this situation, the infant’s initial examination is (Page 225-226, Section 5: The Newborn)
important for assessment of his baseline neurologic function.
One component of the neurologic examination is assessment of Answer 5-37. d
primitive reflexes, which are normal findings on the newborn Neurologic development of the fetus over the third trimester of
examination. Examples of these reflexes include the palmar pregnancy progresses in a sequence that is generally predictable,
grasp (rapid grasp of the examiner’s finger with pressure on the allowing for estimation of gestational age. The assessment
palm of the baby’s hand) and the asymmetric tonic neck reflex gives the more accurate results after the first few days of life,
(lateralized changes in the baby’s extremity tone with rotation allowing the neonate to recover from perinatal complications or
of his head). Other normal findings on the examination include exposures, and the effect of in utero positioning.
very responsive and brisk deep tendon reflexes (that can produce The infant described in the scenario has weak finger grasp
a few beats of clonus) and intermittent disconjugate gaze if the (also referred to as palmar grasp), which typically emerges at
infant is not focused on an object. As the cerebral cortex matures 34 to 35 weeks’ gestation. Coordinated breathing and sucking
over the first 6 to 9 months of age, the primitive reflexes and typically begins to organize around 32 weeks’ gestation with
periods of dysconjugate gaze should diminish and disappear, and slow sucks, little negative pressure, and brief sucking bursts
then deep tendon reflexes should relax. Newborns can exhibit (fewer than 3 sucks) alternating with 15- to 20-second periods
brief periods of pronounced active trunk extension (when of breathing. This infant demonstrates a sucking pattern more
crying, eg), but in the normal newborn, trunk extensor is not consistent with an infant born at 34 to 35 weeks’ gestation, with
stronger than trunk flexor tone. When evaluated in the quiet, longer bursts, a greater number of more rapid strong sucks,
alert state, extensor tone should be minimal to absent, and flexor and shorter pauses for breathing. Finally, the baby has no flexor
tone should be present. tone elicited in the upper extremities with traction, which is
(Page 182, Section 5: Newborn) typically present by 36 weeks’ gestation.
(Page 181, Examination of the Newborn Infant)
Answer 5-36. e
Signs and symptoms of a neonatal stroke can present shortly Answer 5-38. d
after birth, or later in the neonatal period. When presenting
Closure of the ductus arteriosus in infants is regulated by
symptoms occur days after delivery, perinatal causes should
several factors that change during in utero development. In the
still be considered, especially when there are noted risk factors.
term infant, factors promoting closure of the ductus arteriosus
Instrumented vaginal delivery using forceps or vacuum is
include an increase in arterial blood oxygen tension (Pao2),
associated with increased risk of birth trauma. Because this
a decrease in ductal luminal blood pressure, a decrease in
baby in this vignette required vacuum assistance for delivery,
circulating PGE2, and a decrease in PGE2 receptors.
she has an increased risk of traumatic bleeding that could
In contrast, factors that facilitate ductal patency in
result in an intracranial hemorrhage. A perinatal hemorrhage
premature infants include:
may evolve and progress in size hours to days after delivery,
increasing pressure on adjacent normal brain tissue. The 1. Continued synthesis of the dominant PGE2 receptor (E4),
resultant ischemia may result in later clinical evidence of rendering the ductus sensitive to the vasodilatory effects
a stroke (ie, seizures). Preeclampsia, which can result in of PGE2.
93

2. Higher prostaglandin levels due to less efficient At present, the following factors and strategies have been
metabolism and clearance of circulating prostaglandins identified as protective in premature infants:
and increased PGE2 production due to increased
1. Antenatal corticosteroids (protective mechanism unclear).
expression of the COX-2 isoform.
2. Feeding with human milk (maternal or donor breast
3. The production of increased amounts of other
milk) when compared with feeding with formula.
vasodilators (such as nitric oxide and interleukin-6).
3. Fluid restriction.
4. Absence of the normal fetal cortisol surge, which occurs
4. Administration of enteral antibiotics (not recommended
in the third trimester and initiates physiologic changes
due to concern for the development of resistant organisms).
that are necessary for survival in the extrauterine
5. Treatment with specific probiotic strains (in infants
environment (including ductus closure). Preterm infants
with birth weight less than 1000 g) based on the results
whose mothers receive antenatal betamethasone have a
of a variety of clinical trials. However, there has been
decreased risk of prolonged ductal patency.
no consistency in strain, dose, or regimen in any of the
As pulmonary vascular resistance decreases in premature reported studies, to date.
infants, increased left-to-right shunting and resultant
Other strategies studied (ie, different enteral feeding styles,
pulmonary edema can exacerbate respiratory compromise.
administration of immunoglobulins or amino acid supplements)
Intermittent hypoxemia and additional respiratory support
have not changed the incidence of necrotizing enterocolitis.
that may be needed for management will contribute to lung
injury and the risk of chronic lung disease. In this scenario,
early administration of surfactant would have improved lung
compliance, and the PDA may have become symptomatic Answer 5-41. d
sooner. Surfactant administration, however, does not decrease The most effective prophylaxis against IVH is prolongation of
the likelihood of having a PDA. pregnancy, as the risk of IVH decreases with increased birth
Maternal use of ibuprofen (or other nonsteroidal anti- weight and gestational age. In addition, timely interventions
inflammatory drugs) is associated with intrauterine closure of to treat complications that can lead to preterm delivery are
the ductus arteriosus. associated with lower risk of IVH including administration of
antibiotics to women in preterm labor. Although premature
delivery may occur, antenatal antibiotic therapy may result in a
Answer 5-39. c less severe inflammatory response (a hypothesized component
of the pathophysiologic pathway of IVH), and better
Characteristics of NEC are different in near-term and term hemodynamic stability.
neonates than in premature neonates. An episode of NEC in a Although hypotension is an important risk factor
term or near-term newborn commonly involves the presence for intraventricular hemorrhage, rapid correction with
of a risk factor such as congenital heart disease, gastroschisis, intravenous fluid boluses may compound the risk (due to
intrauterine growth restriction, perinatal depression, or germinal matrix reperfusion injury). Antenatal administration
polycythemia. Additionally, NEC typically presents earlier in of betamethasone is associated with decreased incidence
these babies (first week of life) than in more premature infants of intraventricular hemorrhage. In contrast, postnatal
(second or third week of life). In addition, the area of intestine glucocorticoid administration does not influence the risk
affected is different in more mature infants when compared of intraventricular hemorrhage and can have a detrimental
with premature infants. NEC is more likely to affect the colon in effect on neurodevelopment. Studies that have evaluated other
near-term and term neonates. In premature infants, NEC occurs potential prophylactic therapies such as phenobarbital or fresh
primarily in the jejunum and ileum; the most common site is frozen plasma have not shown any benefit in these agents
the distal ileum. Finally, near-term and term infants with NEC preventing IVH.
have a lower risk of requiring surgery than premature infants.
SIPs are more likely to occur in premature neonates.
However, when present in a more mature neonate, a SIP
occurs earlier (typically 0-3 days of age), and is not usually Answer 5-42. e
accompanied by 1 of the typical risk factors (ie, indomethacin Extremely premature and very-low-birth-weight infants have a
therapy) associated with a SIP in a more premature neonate. significant risk of neurodevelopmental impairment, including
intellectual disability and cerebral palsy. Studies following
very-low-birth-weight survivors into adolescence and
adulthood have revealed more functional limitations in this
Answer 5-40. a
group. These problems include lower academic achievement
Although it has been difficult to elucidate the specific scores (relative to peers born at term), an increased risk
pathophysiology leading to necrotizing enterocolitis in of emotional, behavioral, and attention problems, and
premature infants, multiple studies have aimed to identify risk neurosensory impairment. However, parents of low-birth-
factors and test potential prophylactic measures/therapies. weight neonates report less risk-taking behavior and the
94

majority of children (74%-82%) graduate from high school and The inability to identify the cause of IUGR may lead to a
pursue productive lives. presumptive diagnosis of uteroplacental insufficiency. Certain
In general, major neurodevelopmental impairments such as findings on placental pathology, such as an infarct noted on
cerebral palsy and intellectual disability can be diagnosed by gross examination, or an inflammatory response seen on
3 years of age. However, more subtle problems with cognition microscopic examination may provide additional information.
and motor function may not present until school age, when However, despite data from placental pathology, the definitive
more complex functioning is necessary. Preterm children with cause(s) of IUGR may not be determined.
no neurologic problems or intellectual disability are more likely
to have language disorders, reading disability, and difficulty
with arithmetic than their peers who were born at term. In
Answer 5-44. c
addition, they have a greater prevalence of visual perceptual
problems, executive dysfunction, and behavior problems than ECMO is a therapy with significant risks that is reserved for
children born at term. Finally, children born to parents with infants with the most profound respiratory or cardiorespiratory
lower socioeconomic status are more likely to have cognitive failure. The overall survival rate for infants who are treated with
impairments as opposed to cerebral palsy. ECMO is 80% in the United States. The most common cause of
Recent studies have better elucidated the risks of death for infants treated with ECMO is hemorrhage. The risk of
neurodevelopmental impairment for late preterm infants neurodevelopmental problems, including cognitive and motor
(who may not be low birth weight). For example, infants born impairment and sensorineural hearing loss, is not considered to
at 33 to 36 weeks’ gestation have a significantly greater risk be greater with ECMO than the risk conferred by the underlying
for reading and spelling problems than infants born at 39 to illness and conventional therapies. Hearing impairment is more
40 weeks’ gestation. Thus, the absence of risk factors such as common in term infants with respiratory and other organ failure
extreme prematurity or low birth weight does not guarantee a than in extremely premature infants. Because less invasive
normal outcome. therapies are being used more effectively for neonatal respiratory
failure, the use of ECMO has decreased in recent years.

Answer 5-43. c
Answer 5-45. d
IUGR can occur at different times in pregnancy. Timing and
Infants born prematurely have a significantly increased risk
etiology of growth restriction can affect the intrauterine growth
of mortality when compared with infants born at term, and
pattern, often manifested in the anthropometric measures of
premature survivors have a significantly increased risk of
the newborn. The relative sparing of head growth in a growth-
neurodevelopmental problems. In addition, the risk of major
restricted fetus (“head-sparing IUGR”) is thought to be a
neurodevelopmental impairments (intellectual disability,
fetal adaptation to uteroplacental insufficiency; as nutritional
cerebral palsy, visual impairment, hearing impairment) increases
supply is limited, somatic growth is sacrificed to allow for
with decreasing birth weight and gestational age at delivery.
brain growth. If nutrient supply is further decreased relative to
High rates of neurodevelopmental impairment occur
fetal needs, accelerated pulmonary and neurologic maturation
primarily in the survivors born at the limit of viability (prior
prepare the fetus for extrauterine survival. However, sparing
to 25 completed weeks’ gestation). Data currently available
of head growth does not completely protect the brain
show that as many as half of these infants have intellectual
from inadequate intrauterine nutrition and subsequent
disability, and up to one quarter of survivors develop cerebral
neurodevelopmental sequelae in childhood. Children who had
palsy (most commonly, spastic diplegia). These infants also
evidence of head-sparing IUGR at delivery have demonstrated
are at the highest risk for complications of prematurity (such
lower cognitive scores on testing when compared with
as chronic lung disease, necrotizing enterocolitis, retinopathy
controls who were born at a size appropriate for gestational
of prematurity, and bacterial sepsis), which represent
age. Furthermore, the acceleration of lung maturity may not
additional risk factors for abnormal neurodevelopment. It is
prevent respiratory complications in this infant, including
important to focus on risk factors in discussing outcomes of
respiratory distress syndrome (RDS).
premature infants with parents, since diagnosis or exclusion
Low birth weight at delivery has been identified as a risk
of neurodevelopmental impairments is not possible prior
factor for various health outcomes in infancy, childhood, and
to NICU discharge. The impact of abnormal radiographic
later life, including the risk of hypertension, diabetes, and
or physical findings in the NICU on neurologic function
cardiovascular disease in adulthood. However, gestational
is typically not apparent until after infancy. Furthermore,
age, which represents the degree of physiologic maturity, is
although MRI of the brain can be helpful for identifying
predictive neonatal mortality and morbidity. In fact, prior to 30
abnormalities that predict neurodevelopmental impairments,
to 32 weeks’ gestation, an infant born small for gestational age
no single neuroimaging study has been recognized as a clinical
does not have a greater risk for mortality, morbidity, or later
practice standard.
neurodevelopmental disability above that conferred by degree
of prematurity.
95

Answer 5-46. b When cerebral blood flow subsequently increases, additional


reperfusion injury can result in rupture of the germinal matrix
Surfactant production increases in the fetal lung during the
vessels, causing a GM-IVH. In premature infants, germinal
later stages of intrauterine development. Stimulated by the
matrix hemorrhages primarily occur within the first few days
catecholamine surge that accompanies birth and inflation
of life, which may be due to a later adaptation to extrauterine
of the lungs, surfactant is released into the alveolar space,
life that provides stability to this vascular structure. Despite
decreasing surface tension and preventing collapse of the
risk factors such as hemodynamic and respiratory instability
distal air spaces with expiration. Additionally, neonatal lung
that are present in choices b and e, it is less likely for a
compliance is increased due to sustained, regular respirations
premature infant to have a primary GM-IVH after 5 days
at birth and resorption of fetal lung fluid across the pulmonary
of age. The infant in choice d had respiratory distress in the
epithelium via transcellular sodium movement. Circulatory
delivery room, but his gestational age (greater than 32 weeks’
adaptations at birth include a reduction in pulmonary vascular
gestation) decreases his risk for a GM-IVH. Furthermore, the
resistance and a change in neonatal blood flow from a fetal
germinal matrix has involuted completely in the majority of
pattern (pulmonary and systemic circulations in parallel) to an
term infants, making putting the infant in choice a at low risk
adult pattern (pulmonary and systemic circulations in series).
for GM-IVH but at higher risk for another form of intracranial
hemorrhage (ie, subgaleal hemorrhage).

Answer 5-47. e
Premature infants in whom a germinal matrix hemorrhage Answer 5-49. d
has occurred require close evaluation, especially during the Necrotizing enterocolitis (NEC) occurs most commonly in
days following the primary bleed (when the hemorrhage is premature infants. The incidence of NEC is inversely related
most likely to extend into the ventricle and cause additional to 2 important risk factors: birth weight and gestational age.
complications). Serial evaluations should include daily In addition, the risk of mortality from NEC is higher for the
measurement of head circumference, and repeat cranial smallest infants born at lower gestational ages. Mortality is also
ultrasound at the end of the first week of life, or sooner if there related to the need for surgical intervention, and the extent
is clinical suspicion of more bleeding (ie, rapid increase in head of bowel involvement, with survival directly related to the
circumference, drop in hematocrit, seizure). length of remaining bowel after surgical resection. Even in the
After the first few days of age, their risk of bleeding on absence of surgical intervention, NEC increases the incidence
the contralateral side does not increase. In the absence of of other prematurity-related complications, including chronic
profound coagulopathy, hemodynamic instability, and/or mass lung disease, growth failure, neurodevelopmental delays, and
effect from the initial bleed, the risk of a primary bleed on the hospital-acquired infections. Additionally, infants with NEC
contralateral side should decrease. who require surgery have a greater risk of neurodevelopmental
Although bleeding that is limited to the germinal matrix impairment (2-3 times greater risk than infants managed
has not been associated with an increased risk of major with medical therapy alone). There is no evidence to suggest
developmental problems, at this stage, it is possible that that hypersensitivity to breast milk is involved in NEC
this infant’s bleed will progress into the ventricles, resulting pathophysiology. Strictures may occur weeks to months after
in posthemorrhagic hydrocephalus, and various related an episode of NEC, even in the absence of bowel perforation.
complications, including a greater risk of neurodevelopmental
impairment. In addition, male infants are more likely to have
neurologic sequelae from intraventricular hemorrhage.
Answer 5-50. b
The infant in this vignette has received inadequate PPV during
the initial steps of resuscitation, resulting in bradycardia.
Answer 5-48. c
Inadequate airway positioning and poor seal with the neonatal
There are many factors that can contribute to the development face mask are the most common causes of a lack of a response
of a GM-IVH as outlined in Figure 58-1. The final common to PPV (which include chest rise and a rise in the neonatal
pathway is their relationship to, or effect on, the delicate heart rate). The first steps in evaluating a poor response to PPV
germinal matrix, the site of origin of this type of intracranial are repositioning the airway (with mild extension of the head
hemorrhage. Perinatal asphyxia and vigorous resuscitation are and neck) and ensuring an appropriate seal between the mask
risk factors for intraventricular hemorrhage in a premature and the infant’s face (see Table 42-3).
baby, as cerebral hemodynamics are influenced by the Of note, the mask should rest snugly around the mouth and
respiratory condition of an infant. Establishing a stable be supported by the chin and the bridge of the nose. Volume
respiratory status in a premature infant can result in hyperoxia infusion and epinephrine administration may be necessary
and hypocarbia, which can cause a decrease in cerebral (to treat hypovolemia and bradycardia, respectively) but are
blood flow (ischemia) through the germinal matrix vessels. not indicated at this point in the resuscitation. Cardioversion
96

with curling of the tube in the proximal esophageal pouch.


Initial management involves decompression of the proximal
pouch by suctioning the nasogastric tube (to prevent pooling
TABLE 42-3. Strategies to Achieve Chest Rise During Bag–Mask of oral secretions).
Ventilation (MR SOPII) Subsequent steps in management may include contrast
M (mask) Check the seal of your mask imaging of the proximal pouch and esophagus (to delineate
Make sure the infant is truly in the open airway anatomy) and intravenous antibiotics (if pneumonia is suspected
R (reposition)
(mild extension) position due to aspiration of secretions). A barium enema is not indicated
for isolated esophageal atresia. An oral airway is not needed in
S (suction) Remove obstructing secretions this infant with normal pharyngeal findings.
O (open the Sometimes in an effort to get a good seal,
mouth) the mouth is accidentally closed. The higher
resistance of the nasal passages will limit
effective ventilation Answer 5-52. e
P (pressure) Try increasing the inflation pressure if possible Fetal cortisol concentrations increase during development,
I (inflation time) Increase IT to 1-2 seconds for 2-3 breaths in an beginning at the end of the first trimester and rapidly
effort to insure a functional residual capacity is increasing during the final weeks of the third trimester. This
established increase in cortisol facilitates the development of molecular
pathways in several organs, including mechanisms for lung
I (intubate) If all previous steps have failed to achieve chest
fluid clearance at birth and surfactant production. The majority
rise, it is time to intubate!
of fetal cortisol is derived from the fetal adrenal gland, and the
synthetic capacity of the fetal adrenal gland equals the capacity
Reproduced, with permission, from Rudolph CD, Rudolph A, Lister G, First L,
Gershon A. Rudolph’s Pediatrics. 22nd ed. New York: McGraw-Hill, 2011. of the adult adrenal gland. However, placental and/or maternal
steroids may allow for normal development in situations where
fetal steroidogenesis is insufficient.
to correct a neonatal arrhythmia is not performed during the
initial neonatal resuscitation.
Answer 5-53. d

Answer 5-51. d The infant in this vignette has evidence of an Erb’s palsy, an
acquired injury of the upper portion (cervical roots 5 and 6)
The neonate in this vignette has findings consistent with of the brachial plexus secondary to avulsion or lateral traction
isolated esophageal atresia, which can present prenatally during delivery. This phenomenon often occurs in infants
with polyhydramnios (due to inability of the fetus to swallow during deliveries associated with shoulder dystocia (ie, deliveries
amniotic fluid). Postnatal findings include excessive oral of macrosomic babies). The injury leads to paralysis of the
secretions and respiratory distress. Radiographic findings deltoid, biceps, and brachialis muscles of the affected arm
include failure of a nasoenteric tube to pass to the stomach, and shoulder, resulting in positioning of the affected arm
as described in the question. Associated findings include
ptosis, miosis, and enophthalmos of the ipsilateral eye due
to cervical sympathetic nerve injury (Horner syndrome) and
diaphragmatic paralysis (due to ipsilateral phrenic nerve
injury). The other physical findings are congenital anomalies
not associated with Erb’s palsy.
10-Gauge
Catheter

Blind Upper
Answer 5-54. e
Trachea Esophageal Vertical transmission of hepatitis B infection can be prevented
Segment
with appropriate antenatal serologic screening of pregnant
women. For infants born to HBsAg seropositive mothers,
appropriate immunoprophylaxis includes administration of
FIGURE 392-2. Diagnosis of esophageal atresia is confirmed by inability to hepatitis B vaccine and hepatitis B immune globulin.
pass a 10 French orogastric tube beyond 10 cm from the gums. (Reproduced For a variety of reasons, including inability to access
from Beasley SW. Esophageal atresia and tracheoesophageal fistula. In: maternal records, maternal hepatitis B serostatus may be
Oldham KT, Colombani PM, Foglia RP, et al, eds. Principles and Practice of unknown at the time of delivery. In these cases, HBsAg
Pediatric Surgery. Philadelphia: Lippincott Williams & Wilkins; 2005:1040.) serology on the mother should be drawn immediately on
97

admission for delivery, and the hepatitis B vaccine should be bradycardic after tracheal aspiration and ventilation. Nasogastric
administered to the neonate within 12 hours of age. Guidelines tube placement would be indicated to decompress the stomach
for administration of hepatitis B immune globulin are based after prolonged bag–mask ventilation.
on infant weight (using a weight cutoff of 2000 g) as well as the
timing of determination of maternal serology status.
Because of the variable immune response to the hepatitis
Answer 5-57. b
B vaccine in neonates born under 2000 g, hepatitis B immune
globulin should be given to these infants if maternal serology The infant in this vignette has a normal abdominal
status cannot be determined within 12 hours. For infants with examination and, in the absence of any other abnormal
a birth weight over 2000 g, immune globulin administration physical findings, should be discharged home with her parents.
may be delayed if the maternal serology status can be The finding of a liver edge 1-2 cm below the costal margin
determined within 7 days. Infants born to mothers who are is unremarkable. The umbilical stump in a neonate should
HBsAg negative should only receive the hepatitis B vaccine. be dry; moisture around the umbilical stump may be due to
Maternal chorioamnionitis and hepatitis C seropositive status a patent urachus or a mucosal cyst. The lower poles of each
are not indications for administration of hepatitis B immune kidney may be palpable, with the right kidney higher than
globulin. the left kidney. The baby has a diastasis recti, a gap in the
abdominal rectus muscles that is common in the neonatal
period and does not require surgical intervention.
Answer 5-55. a (Page 178-179, Chapter 5: The Newborn)
The infant of a diabetic mother in this vignette most likely is
experiencing seizures due to hypocalcemia. Calcium is actively Answer 5-58. e
transported across the placenta from the mother to the fetus, Cigarette smoking remains a major cause of prenatal, perinatal,
with a corresponding decrease in fetal parathyroid hormone and infant morbidity and mortality, including IUGR. Initiation
(PTH). At birth, neonatal PTH levels surge to account for the of a smoking cessation program is an appropriate step to reduce
decreased transplacental calcium delivery. However, relative to the risk of IUGR in this mother. Prenatal alcohol exposure
normal infants, infants of diabetic mothers have reduced PTH increases the risk of growth restriction, including microcephaly,
levels as well as decreased end-organ sensitivity to PTH. The and should be avoided in pregnancy. Folic acid supplementation
resultant hypocalcemia manifests clinically at 48 to 72 hours reduces development of congenital spine and brain defects but
of age with neurologic changes (irritability, tremors, twitching, does not reduce the risk of fetal growth restriction. β-Carotene
and/or seizures) and cardiac arrhythmias. Prematurity, promotes fetal growth and immune development and should be
perinatal depression, and concomitant hypomagnesemia may consumed in appropriate amounts in pregnancy. Consumption
worsen the hypocalcemia. Serum calcium levels should be of a strict vegan diet may place the fetus at risk for poor growth;
monitored in infants of diabetic mothers daily during the first appropriate supplementation of the diet with nonmeat-derived
72 to 96 hours of age. protein (ie, legumes, soy) is required in pregnancy.
A complete blood count and C-reactive protein are not
indicated in this infant with low risk factors for perinatal
bacterial infection and no other clinical signs of sepsis. Since
Answer 5-59. c
the infant is feeding well, hyponatremia is less likely to be a
cause of her seizures. Imaging of the head in an infant with a Neonatal and infant mortality rates have steadily declined over
nonfocal neurologic examination is not warranted. the past 10 years (see Figure 41-1), largely due to advances in
neonatal care. The advent of surfactant therapy and antenatal
corticosteroids to decrease the incidence of respiratory distress
syndrome has reduced significantly neonatal mortality.
Answer 5-56. e
Furthermore, mortality among low-birth-weight infants has
The nonvigorous infant in this vignette requires intubation decreased in NICU centers providing high levels of care and
and aspiration of the trachea for aspirated meconium from the treating high volumes of these at-risk infants. However, certain
amniotic fluid. He demonstrates signs of secondary apnea at birth factors, including male sex, African American ethnicity, late
(apnea, hypotonia, and bradycardia [heart rate 50 beats/min]), (after the first trimester of pregnancy) or no prenatal care,
putting him at risk for meconium aspiration syndrome. This multiple gestation, and low birth weight, increase the risk of
baby requires endotracheal intubation and suctioning on arrival neonatal and infant mortality.
at the delivery room table. Suctioning of the mouth and nose
of a meconium-stained infant would be performed only if the
infant was vigorous (ie, demonstrating normal respiratory
Answer 5-60. c
effort, muscle tone, and heart rate [over 100 beats/min]). Chest
compressions and umbilical vein catheterization (for epinephrine Metabolic acidosis is defined as an increase in serum
administration) are indicated only if the infant remains hydrogen (H+) ion or a decrease in serum bicarbonate,
98

resulting in a decrease in serum pH. A normal ion Answer 5-63. b


gap metabolic acidosis occurs in the setting of renal or
The infant in this vignette developed anasarca as a result
gastrointestinal bicarbonate losses and is characterized by
of an in utero tachyarrhythmia. However, delivery room
hyperchloremia. Preterm neonates (as in case C) often develop
management of this infant does not differ from the
a mild, proximal renal tubular acidosis, with urine bicarbonate
management of other infants with ineffective ventilation. This
wasting and a low serum pH. In contrast, an increased ion
infant may have respiratory failure due to respiratory distress
gap metabolic acidosis develops due to production of lactate
syndrome and pleural effusions (secondary to arrhythmia-
(from sepsis, hypoxia–ischemia, tissue damage, or a congenital
induced congestive heart failure). Thus, intubation and
heart defect) or another anion (from an inborn error of
mechanical ventilation are the appropriate steps at this point
metabolism [IEM]). The other infants have conditions (aortic
in the resuscitation. Neither blow-by nor nasal cannula oxygen
coarctation, patent ductus arteriosus, IEM, and bacterial
will sufficiently support the degree of respiratory distress in
sepsis, respectively) that will produce a serum anion gap and
this neonate. Epinephrine only should be administered after
metabolic acidosis.
adequate ventilation and chest compressions fail to improve
neonatal bradycardia. Infusion of adenosine (to convert
SVT into a sinus rhythm) is not indicated in delivery room
Answer 5-61. d resuscitation of a neonate, including those with fetal SVT.
The neonate in this vignette has an abnormal neurologic
examination, which may occur secondary to abnormalities
in levels of serum electrolytes, glucose, or amino acids. Answer 5-64. a
However, of the values listed, the serum calcium level is the
In general, due to the increased surface area of the newborn
abnormal laboratory value most associated with this neonate’s
relative to a newborn’s body weight, heat production in the
presentation. His hypercalcemia has occurred secondary to
neonate is greater than heat production in the adult.
subcutaneous fat necrosis (in the indurated cheek nodules)
Nonshivering thermogenesis is generally accepted as the
associated with perinatal trauma. Granulomata in the
major source of heat generation in neonates. An important
necrotic fat cause parathyroid hormone (PTH)–independent
protein involved in nonshivering thermogenesis is UCP1, an
overproduction of 1,25-dihydroxyvitamin D. Presentation of
ion transport protein that generates heat through entry of
the hypercalcemia may range from mild neurologic changes
protons into mitochondria. Brown fat, the major tissue involved
seen here to hypertension, seizures, respiratory distress, and
in nonshivering thermogenesis, also is relatively more abundant
nephrocalcinosis. Management includes hydration, increasing
in the neonate than in the adult. Energy generation in neonates
urinary calcium excretion, and restriction of dietary calcium
is also augmented by intracellular free fatty acids, which are
intake.
produced by elevated levels of lipoprotein lipase at birth.
Circulating free fatty acids replenish depleted intracellular
energy stores but do not serve as an acute source of energy.
Answer 5-62. e The neonate, however, has thermogenic disadvantages
relative to adults. Due to an increased surface area relative to
The premature neonate in this vignette has signs of systemic
body weight, newborns must generate more heat to maintain
hypotension. The cardiovascular system of the preterm infant
a normal body temperature. Additionally, the absolute extent
has adapted in utero to the low-resistance state of the placenta.
to which a neonate can maintain a normal body temperature
With the clamping of the umbilical cord, the premature
during cold stress is limited.
myocardium is exposed to a high-resistance ex utero state, and
is generally unable to readily adapt to the change. Initiation
of an inotrope (in this case, dopamine) will provide cardiac
stimulation and vasopressor effects that will improve blood Answer 5-65. b
pressure and perfusion.
Although preterm delivery can occur in both singleton and
The myocardium of the preterm infant is impacted by
multifetal pregnancies, the preterm birth rates for multifetal
positive pressure ventilation; increases in the PIP and PEEP
gestations are significantly greater. Recent data have indicated
will increase intrathoracic pressure, decreasing cardiac output
that most multifetal gestations are born prematurely, with
and perfusion. Sepsis is a common cause of hypotension and
preterm birth rates for twins and triplets reported as 60.5%
hypoperfusion in the neonate, but immune globulin has not
and 93.7%, respectively (see Table 46-1). These preterm
been shown to reverse the circulatory collapse associated
infants are at risk for developing all of the complications
with early onset or late-onset infections. Maintaining the
typically associated with prematurity, including poor long-
patency of the ductus arteriosus with a PGE1 infusion will
term neurodevelopmental outcomes. Twin–twin transfusion
allow left-to-right shunting of blood during the cardiac cycle,
syndrome typically occurs in monozygotic twinning when
lowering systemic blood pressure.
a vascular connection between twins develops in the shared
placenta. Dizygotic twins account for two thirds of all
99

spontaneous twin births. In association with the use of assisted is not indicated. Supplemental oxygen may improve the infant’s
fertilization and advanced maternal age, there has been an pulse oxygen saturations but would not provide consistent and
overall increase in the incidence of multifetal gestations since adequate positive pressure to achieve airway patency.
the 1980s. Fetuses of multifetal gestations are more likely to
have breech or other malpresentations at delivery, contributing
to a higher rate of cesarean deliveries.
Answer 5-68. e
The infant in this vignette has lung injury secondary to
congenital pneumonia. As his pulmonary infection resolves,
Answer 5-66. d his lung compliance changes, leading to variable tidal volumes
The infant is this vignette has become apneic secondary to despite stable ventilator settings. With pressure-limited
ventilatory insufficiency. Surfactant deficiency and increased ventilation, the volume of gas delivered depends on the
chest wall compliance have led to atelectasis and diminished underlying lung compliance (ie, highly compliant lungs will
gas exchange, which are hallmarks of respiratory distress receive a greater volume of gas at a given pressure). Volume-
syndrome. Positive pressure ventilation is necessary until targeted ventilation attempts to deliver consistent volumes
the infant’s own respirations can support gas exchange. of gas into the lungs despite changing lung compliance. For
Endotracheal intubation will also allow for the administration example, if lung compliance abruptly increases, the ventilator
of surfactant. Exogenous surfactant administration can will maintain the same inhaled volume of gas by reducing
reduce the surface tension of the alveoli and improve the pressure used during inspiration. Intermittent mandatory
functional residual capacity. CPAP and nasal cannula ventilation, high-frequency oscillation, and pressure support all
oxygen would not be appropriate modes of ventilation for an are forms of pressure-limited ventilation that would not be able
apneic patient as they rely on the patient to spontaneously to respond to changes in lung compliance.
breathe for gas exchange. Tension pneumothoraces can be
a complication of mechanical ventilation and should be
suspected when asymmetric breath sounds, asymmetric
Answer 5-69. c
chest wall contours, hypotension, and hypoxia develop
acutely. Inhaled nitric oxide, a vasodilator used to reduce The infant in this vignette developed persistent pulmonary
the pulmonary vascular tone in infants with persistent hypertension of the newborn (PPHN) secondary to
pulmonary hypertension of the newborn, is not indicated meconium aspiration syndrome. PPHN is characterized
during delivery room resuscitation. by persistently elevated pulmonary vascular tone causing
intrapulmonary or extrapulmonary right-to-left shunting.
The overall goals of therapy are to optimize lung inflation
and lower pulmonary vascular resistance. High-frequency
Answer 5-67. c oscillation is being used in this vignette to improve lung
The infant in this vignette is a preterm infant with respiratory inflation. Techniques to reduce pulmonary vascular tone
distress syndrome (RDS) and subsequent CO2 retention. include avoiding hypercarbia, hypoxia, and acidosis. Another
Additionally, she has developed radiographic changes in her therapy is inhaled nitric oxide, a selective pulmonary
lung fields and prolonged ventilator dependency. Over time, vasodilator. It increases the activity of soluble guanylate
infants with RDS develop areas of inflammation, mucous cyclase, leading to increases in cyclic GMP (cGMP), an
plugging, and airway narrowing. Such infants may experience important promoter of smooth muscle relaxation. Inhaled
worsening of respiratory symptoms following extubation, as nitric oxide has become an important therapy in the
removal of the positive pressure can reduce airway patency. management of PPHN. Diuretics that block transporters in
Without intervention, the infant may develop worsening the nephron (chlorothiazide to block the Na+–Cl− symporter
atelectasis and severe respiratory distress. CPAP delivered or furosemide to block the Na+–K+–2Cl− cotransporter)
noninvasively (via nasal prongs or mask) at pressures of 4 to have a limited role in PPHN. Opiates, which activate mu
8 cm H2O can often provide enough support to distend the opioid receptors, may be used for sedating patients who have
airways and reduce alveolar atelectasis. difficulty tolerating high-frequency ventilation but are not
Common indications for use of CPAP in neonates specifically therapeutic in PPHN. Albuterol, a β2-adrenergic
include supporting mild RDS or narrowing of the airways, agonist, is used for patients with airway hyperresponsiveness
transitioning from mechanical ventilation, and decreasing and has a role in the management of chronic lung disease.
the incidence or severity of apnea and bradycardia in some
preterm infants. If CPAP did not resolve the infant’s symptoms
(or if symptoms worsened) over a defined period of time, then
Answer 5-70. c
endotracheal intubation and mechanical ventilation would
be appropriate management. As the infant’s lung disease was Infants with SIP clinically present with pneumoperitoneum,
improving with SIMV mode, ventilation with high frequency making the differentiation between SIP and advanced NEC
100

challenging. The infant in this vignette has several features iron deficiency anemia with a hemoglobin level is done at 9 to
and risk factors to suggest that the intestinal perforation is 12 months of age. Vitamin D supplementation is often needed
a consequence of a SIP, including timing of the perforation to reach the recommended amount of vitamin D intake,
(7-10 days) after indomethacin and glucocorticoid exposure, 400 IU daily; however, screening serum vitamin D levels is not
poor intrauterine growth, and extremely low birth weight. recommended. C-reactive protein can be used as a screening
Conversely, infants who develop NEC typically have received test for sepsis in the neonatal period. Screening for lead toxicity
enteral feeds prior to presentation and may demonstrate is performed between 1 and 2 years of age for infants with risk
radiographic features such as portal venous gas or pneumatosis of environmental lead exposure.
intestinalis.
(Page 231, Section 5: Newborn)
The mucosal margins of the intestinal perforation appear
healthy in a SIP (as opposed to the coagulation necrosis
observed in NEC). This histopathologic difference also helps Answer 5-73. b
to distinguish the 2 entities. Additionally, infants with SIP have Factors such as prematurity and being the infant of a diabetic
decreased mortality compared with infants with NEC-related mother contribute to increased bilirubin production in this
intestinal perforation. Volvulus and duodenal atresia in the infant, leading to neonatal jaundice. Based on the AAP
newborn period require surgical management; however, they practice guidelines for infants with hyperbilirubinemia (see
typically present with symptoms of intestinal obstruction, Figure 53-3), phototherapy should be initiated in this infant.
including vomiting and bilious aspirates. As with NEC, cow’s Phototherapy converts the bilirubin to lumirubin for excretion.
milk protein allergy can present with hematochezia and If the bilirubin level rises too quickly (greater than 0.2 mg/
feeding intolerance, but this illness does not involve intestinal dL/h) despite intensive phototherapy, IV Ig or exchange
perforation. transfusion may be needed to decrease any circulating
antibodies that cause hemolysis and/or reduce the serum
bilirubin concentration. The infant is well hydrated, voiding,
stooling, and does not have excessive weight loss; therefore, IV
Answer 5-71. a hydration or discontinuation of breast-feeding is unnecessary
The infant experienced delayed cord clamping during his at this time.
home delivery, which allowed for the transfusion of excess red
blood cell volume. Polycythemia, defined as a central venous
hematocrit of greater than 65%, can result in hyperviscosity
syndrome, or organ dysfunction due to impaired blood Answer 5-74. c
flow. Symptoms include neurologic changes (as described This infant’s fetal anemia, bilious amniotic fluid, and features
in the vignette), respiratory impairment, feeding difficulty, of hydrops suggest erythroblastosis fetalis, resulting from
and thrombocytopenia. A cranial sonogram can be used the hemolysis of fetal red blood cells by maternally derived
to diagnose intracranial causes of neonatal irritability Rh antigens. Mothers who are Rh-negative may become
and lethargy such as acute hemorrhage and sinus venous sensitized to the most significant Rh protein, anti-D antigen, if
thrombosis. Congenital hypothyroidism can present with Rh-positive fetal blood cells enter into the maternal circulation.
hypotonia, lethargy, and poor feeding, but the symptoms often However, Rh sensitization has been largely decreased in the
present days to weeks after birth. A stool occult blood test United States thanks to the use of anti-D immune globulin
can help determine a source of bleeding in an anemic infant given to Rh-negative mothers at 28 weeks’ gestation and after
but is not a useful test for polycythemia. A chest radiograph the birth of an Rh-positive infant.
would be helpful to rule out alternative causes of the infant’s Viruses such as parvovirus and cytomegalovirus are
respiratory distress but would not be helpful for the diagnosis significant causes of fetal hydrops, but the associated anemia
of polycythemia. results from decreased erythrocyte production rather
than hemolysis. Complex congenital heart disease and
supraventricular tachycardia may cause fetal hydrops but do
not typically involve hemolytic anemia.
Answer 5-72. e
Given the risk of kernicterus in the newborn period, significant
efforts should be made to identify all infants with pathologic
Answer 5-75. e
serum bilirubin levels. Risk factors for hyperbilirubinemia
include a family history of neonatal jaundice, East Thrombocytopenia in the neonate can result from either
Asian ethnicity, male gender, blood extravasation (ie, decreased production or increased destruction of platelets.
cephalohematoma), prematurity, exclusive breast-feeding, and Kasabach-Merritt syndrome, also known as giant hemangioma
postnatal weight loss. The infant in this vignette has multiple syndrome, is characterized by the presence of a vascular
risk factors for hyperbilirubinemia, making a total serum tumor that sequesters platelets, leading to significant
bilirubin level an appropriate screening test. Screening for thrombocytopenia. In this case, the trapping of platelets
101

and consumptive coagulaopathy end in platelet destruction. radiographic findings (bilateral granular lung fields) make RDS
There may also be a microangiopathic hemolytic anemia less likely in this case.
associated with the presence of the mass. Trisomies 13 and 18,
Fanconi syndrome, and TAR syndrome all may present with
thrombocytopenia but are not associated with the presence of Answer 5-77. a
vascular malformations.
The infant in this vignette has a pneumothorax, one type
of air leak syndrome. Pulmonary air leaks can result from
any lung disease in newborns, particularly in those who
Answer 5-76. b require mechanical ventilation. The size of the pneumothorax
When labor begins, sodium–potassium exchange occurs at at presentation can vary, as does the degree of clinical
the basolateral membrane of the lung epithelium allowing for compromise. Pneumothoraces can cause significant respiratory
active transport of sodium across the apical surface via sodium distress, hypoxemia, hypercarbia, or hypotension, warranting
channels. Subsequently, water is drawn from the air spaces into emergent decompression. If the infant’s clinical deterioration
the lung interstitium, clearing fetal lung fluid in anticipation precludes chest tube placement in a controlled setting, needle
of air exchange. This process is activated by a surge of decompression of the affected hemithorax should be performed
catecholamines and other hormones during labor. immediately as a temporizing measure. Increasing the mean
The infant in this vignette was born by cesarean section airway pressure will not resolve the infant’s pneumothorax and
without a period of labor, which increases his risk for transient may contribute to worsening air leak.
tachypnea of the newborn (TTN). The mild respiratory Pericardiocentesis is indicated for a pneumopericardium
alkalosis and radiographic changes noted in this vignette (air collection in the pericardial space) associated with cardiac
support this diagnosis. Chemical pneumonitis can occur as tamponade, which this infant does not demonstrate based on
a result of meconium aspiration syndrome, which did not clinical examination and radiographic data. Stabilization of the
occur in this case. Ascending infection through the birth canal airway, drainage of gastric contents, and surgical consultation
can cause congenital pneumonia, which is less likely in this are initial steps in the management of congenital diaphragmatic
neonate due to the rupture of amniotic membranes at delivery. hernia, which is demonstrated radiographically by bowel (or
Increased pulmonary vascular tone results in persistent other abdominal contents) in the thorax. Intubation of the
pulmonary hypertension of the newborn, which is less likely right main stem bronchus may cause decreased left-sided
in this case given the normal pulse oximetry readings. High breath sounds and respiratory insufficiency; however, this
alveolar surface tension due to surfactant deficiency can infant’s endotracheal tube is appropriately placed as visualized
result in respiratory distress syndrome (RDS). However, the on the x-ray.
absence of a supplemental oxygen requirement and traditional

Lumen

Cl– Cl–
Na+ Na+ Na+
H2O H 2O

2K+

K+
ATPase

3Na+
Interstitium
FIGURE 50-1. Model of fetal lung fluid absorption by epithelial cells. Fluid absorption results from vectorial transport of Na+, driven by Na+/K+-ATPase. The
resultant electrochemically increased gradient leads to passive Na+ absorption via apical Na+-permeant channels that is extruded by Na+/K+-ATPase out of the cell.
Cl and water passively follow the Na+ ions through paracellular or intracellular pathway. (Reproduced, with permission, from Rudolph CD, Rudolph A, Lister G,
First L, Gershon A. Rudolph’s Pediatrics. 22nd ed. New York: McGraw-Hill, 2011.)
102

Answer 5-78. a hemorrhage, as increased pressures are transmitted to the lung


capillaries and generate injury. Conditions that precipitate
The infant in this vignette has developed idiopathic persistent
left heart failure such as cocaine exposure, sepsis, asphyxia,
pulmonary hypertension of the newborn (PPHN). Constriction
and congenital heart disease are associated with pulmonary
of the fetal ductus arteriosus by nonsteroidal anti-inflammatory
hemorrhage. However, preterm infants with persistence of
drug exposure is 1 possible cause of idiopathic PPHN. Findings
a large patent ductus arteriosus (PDA) are at particular risk
of idiopathic PPHN include cyanosis and respiratory distress
for pulmonary hemorrhage. In neonates with large PDAs,
within hours of birth, differential cyanosis or pulse oximetry
exposure of the lungs to high pressure and high blood flow
indicating right-to-left shunt, and reduced arterial oxygenation
results in microvascular injury that leads to hemorrhage.
despite relatively normal pco2 and/or metabolic acidosis.
X-rays of the chest may reveal undervascularized lung fields (Page 203-204, Section 5: Newborn)
with hyperlucency (depending on the presence or absence of
additional lung pathology). Echocardiography with Doppler Answer 5-80. a
flow studies can confirm features of PPHN such as intracardiac
The infant in this vignette has developed a pulmonary
right-to-left shunts and tricuspid valve regurgitation, which
hemorrhage following the administration of surfactant,
suggest elevated right-sided pressures. Serial blood gas
which is a recognized risk factor. As pulmonary hemorrhage
sampling and pulse oximetry are important clues to the
is a potentially fatal event, rapid measures should be taken
diagnosis of PPHN and are useful in ongoing management,
to stabilize the patient and correct any known causes of the
but are nonspecific and do not estimate pulmonary vascular
hemorrhage. Specific measures to treat pulmonary hemorrhage
resistance. Tracheal aspirates and blood cultures may identify
include maintaining adequate mean airway pressure to
an infectious etiology for an infant with PPHN but do not
support lung volume and gas exchange, restoring circulating
assess pulmonary vascular pressures.
blood volume, and ameliorating coagulopathy. Management
of predisposing factors also should be considered, such as
closure of a large patent ductus arteriosus with indomethacin.
Answer 5-79. d However, without a patent airway none of these interventions
will sufficiently provide adequate cardiorespiratory support for
Pulmonary hemorrhage, a potentially fatal complication, is
the patient. Therefore, the first step in the management of this
a chief cause of death in 9% of neonatal autopsies. It most
infant is to suction and secure the airway.
commonly occurs between days 2 and 4 of life. Left ventricular
failure is thought to be an inciting factor in pulmonary
Dr.Wahid Helmi Egypt

DR.WAHID HELMI RIFAHIE


PEDIATRIC CONSULTANT –EGYPT
Pediatric Nephrology
1. In acute renal failure, the urine-to-plasma creatinine ratio is:
A <5
B <10
C <15
D >40
E <20
1. Answer: E
The urine-to-plasma creatinine ratio is >40 in prerenal failure. In acute renal failure, the urine sodium
is >40 mmol/l, the fractional excretion of filtered sodium is >1, urine osmolarity is <350 mosmol/l and
the urine-to-plasma urea nitrogen ratio is <3.

2. The following all cause sustained hypertension in children except:


A Renal artery stenosis
B Conn’s disease
C Nephrotic syndrome
D Essential hypertension
E Polycystic kidneys
2. Answer: C
Hypertension In children, this can be caused by renal nephropathies, renal vascular disease,aortic
coarctation, glomerulonephritis, corticosteroid excess (e.g. Conn’s disease, Cushing’s syndrome and
congenital adrenal hyperplasia), low renin,haemolytic uraemic syndrome, phaeochromocytoma,
neuroblastoma and nephroblastoma. There are also idiopathic cases
3. The following drugs can cause haematuria except:
A Sulfonamides
B Aspirin
C Ceftriaxone
D Indometacin (indomethacin)
E Amitriptyline
3. Answer: C
Haematuria
Haematuria can be caused by glomular problems such as thin basement
membrane diseases, Alport’s syndrome, IgA nephropathy, haemolytic uraemic
syndrome, post-infectious glomerulonephritis, membranoproliferative
glomerulonephritis, lupus nephritis and anaphylactoid purpura (Henoch–
Sch¨ onlein purpura). Other non-glomerular causes include fever, strenuous
exercise, drugs/toxins, foreign bodies, mechanical trauma, urinary tract
infection, hypercalciuria/urolithiasis, sickle cell disease/trait, coagulopathy,
tumours, anatomical abnormalities (hydronephrosis, polycystic kidney disease
and vascular malformations), masturbation and menstruation.

1
Dr.Wahid Helmi Egypt

4. A 4 month old baby girl was born at 28/40. Urea is 40 mmol/l,creatinine 350 mmol/l and urine
output 1 ml/kg/h. Her sodium is 127 mmol/l and potassium 5.7 mmol/l. Her clotting is normal,
and conjugated bilirubin is 30 mmol/l, with normal ALT. The urine osmolarity is <400 mosmol/l,
and urinary sodium 75 mmol/l. The child is very oedematous, with a mean arterial blood
pressure of 26 mmHg. What is the most likely cause of her oedema?
A Congenital nephrotic syndrome
B Acute renal failure
C Chronic renal failure
D Hypothyroidism
E Hepatic failure
4. Answer: C
Chronic renal failure (CRF)
CRF in children can be progressive and be caused by systemic hypertension, acute insults from
nephrotoxins or decreased perfusion, proteinuria,increased renal ammoniagenesis with interstitial
injury, hyperlipidaemia,hyperphosphataemia with calcium phosphate deposition, and decreased levels
of nitrous oxide. The manifestation of CRF is hyperkalaemia, which usually develops when the
glomerular filtration rate GFR falls to less than 20–25 cm3/min because of the decreased ability of the
kidneys to excrete potassium. It can be observed sooner in patients who ingest a potassiumrich diet or if
serum aldosterone levels are low. Metabolic acidosis is often mixed, with the anion gap generally not
higher than 20 mEq/l. In CRF, the kidneys are unable to produce enough ammonia in the proximal
tubules to excrete the endogenous acid into the urine in the form of ammonium ion. In very advanced
CRF, accumulation of phosphates, sulfates and other organic anions causes the small anion gap.
Extracellular volume expansion and total-body volume overload result from failure of sodium and free-
water excretion. These usually intensify when the GFR falls to <10–15 cm3/min and when all
compensatory mechanisms have become exhausted. Patients usually present with peripheral, rather
than pulmonary, oedema. This can happen even with a higher GFR, as an excess of sodium and water
intake can result in a similar picture if the ingested amounts of sodium and water exceed the potential
excretion of the usual amount. Normochromic normocytic anaemia principally develops from
decreased renal synthesis of erythropoietin, which is responsible for stimulation of bone marrow for red
cell production. As the GFR decreases, the condition becomes more difficult to treat, and no
reticulocyte response occurs. RBC lifespan is reduced, and the tendency to bleed increases as a result of
uraemia-induced platelet dysfunction.Secondary hyperparathyroidism develops because of
hypocalcaemia and hyperphosphataemia. The latter is a result of the inability of the kidneys to excrete
the excess dietary intake, while the hypocalcaemia is caused primarily by decreased intestinal calcium
absorption because of low plasma calcitriol levels and possibly as a result of calcium binding to
elevated serum levels of phosphate. Renal osteodystrophy will develop because of all these changes.
These lesions develop in patients with severe CRF and are common in those with end-stage renal
disease (ESRD). Osteomalacia and dynamic bone disease are the two other lesions observed. Dialysis-
related amyloidosis fromβ2-microglobulin accumulation in patients who have required chronic dialysis
for at least 8–10 years is another form of bone disease that manifests
with cysts at the ends of long bones. Other manifestations of uraemia in ESRD are pericarditis,
encephalopathy that can progress to coma and death,peripheral neuropathy, restless leg syndrome, GI
symptoms such as anorexia,nausea, vomiting and diarrhoea, skin manifestations such as dry skin,
pruritus and ecchymosis, fatigue, increased somnolence, failure to thrive, malnutrition,erectile
dysfunction, decreased libido, amenorrhea, and platelet dysfunction
with a tendency to bleeding.

2
Dr.Wahid Helmi Egypt

5. In metabolic acidosis, decreased acid excretion can be caused by all of the following except:
A Early salicylate poisoning
B Distal renal tubular acidosis
C Acute renal failure
D Chronic renal failure
E Carbonic anhydrase inhibitors
5. Answer: A
Salicylate poisoning
Salicylates stimulate the respiratory centre, leading to hyperventilation and respiratory alkalosis.
However, respiratory alkalosis may be transient in children, so that metabolic acidosis may occur early
in the course.Salicylates also interfere with the Krebs cycle, limit production of ATP and
increase lactate production, leading to ketosis and a wide-anion-gap metabolic acidosis. Patients with
mixed acid–base disturbances have been found to have normal-anion-gap metabolic acidosis; therefore,
normal-anion-gap acidosis does not exclude salicylates
6. The most common causes of polyuria and polydipsia in children
include the following except:
A Chronic renal failure
B Diabetes mellitus
C Psychogenic
D Behavioural
E Urinary tract infection
6. Answer: A
Polyuria and polydipsia
Polyuria and polydipsia are associated with diabetes insipidus (DI). There are idiopathic and familial
forms of DI (both of which are very rare). It can be inherited as an autosomal dominant disorder, and
mutations involving AVP/neurophysin gene have been identified as occurring after neurosurgery or
trauma. Primary intracranial tumours causing DI include craniopharyngioma
and pineal tumours. Possible causes also include other malignancies (e.g. lung cancer, lymphoma and
leukaemia), hypoxic encephalopathy, infiltrative disorders (histocytosis X and sarcoidosis), anorexia
nervosa, and vascular lesions such as arteriovenous malformations or aneurysms
7. A 10-year-old girl presents with soreness of her vulval area, itching and dysuria. There is no
growth on urine culture, nor is there any vaginal discharge. Her stool is negative for infection
and blood. Her parents say that there is a strong unpleasant smell from the child’s underpants.
What is the most likely diagnosis?
A Urinary tract infection
B Child sexual abuse
C Vulvovaginitis
D Eczema
E None of the above
7. Answer: C
Vulvovaginitis
Vulvovaginitis will produce local pain, burning or itching, and external dysuria with a strong
unpleasant smell from the genital area or from the child’s underpants. A UTI can produce a similar
strong smell. During the examination, the child must be relaxed and an explanation should be given for
the examination. NAI, foreign bodies and threadworms should be ruled out. Poor hygiene is the most
common cause, along with chemicals,nylon, eczema, scabies, fungal infections and trauma. Swabs

3
Dr.Wahid Helmi Egypt

must be taken.Avoidance of any irritants, good hygiene and any barrier creams will help.There is no
need for steroids.
8. In IgA nephropathy, all of the following are true except:
A The serum C3 level is normal.
B The immune complex is positive on electron microscopy.
C The serum C4 level is high.
D Haematuria is common.
E IgA serum level may be elevated.
8. Answer: C
IgA nephropathy
The pathogenesis of IgA nephropathy remains unclear. The characteristic pathological findings are
immunofluorescence microscopy of granular deposits of IgA and complement 3 (C3) in the glomular
mesangium. These findings suggest that this disease occurs because of the deposition of circulating
immune complexes, leading to activation of the complement cascade.
In some cases there is an association of IgA nephropathy with syndromes that affect the respiratory or
gastrointestinal tracts, such as coeliac disease, suggesting that IgA nephropathy is a disease of the
mucosal immune system. Haematuria worsens during or after upper respiratory tract or gastrointestinal
tract infections, which support its origin being mucosal.IgA antibodies cannot activate complement
through the classic pathway;studies have shown that the alternative pathway can activate
complement.Serum IgA levels are elevated in approximately 50% of patients with IgA nephropathy,
but that increase is unlikely to play a role in the pathogenesis of the disease because markedly elevated
IgA levels are observed in patients with AIDS who do not have IgA nephropathy. However, IgA is
probably accumulated and deposited because of a systemic abnormality rather than a defect intrinsic to
the kidney.

9. A 7-year-old boy is admitted with acute diarrhoea and vomiting over the last 48 hours. He is
10% dehydrated, and his urine output is less than 0.5 ml/kg/h over the last 24 hours. His
potassium is 7 mmol/l,urea 45 mmol/l and creatinine 300 mmol/l. His acute renal failure is
worsening and his potassium is rising. Which of the following should be excluded in the
management of his hyperkalaemia before commencing
dialysis?
A IV infusion of glucose and insulin
B IV infusion of 7.5% bicarbonate
C IV corticosteroids
D IV infusion of salbutamol
E Rectal ion exchange resins
9. Answer: C
Management of hyperkalaemia
10% calcium gluconate and dialysis can be used to lower potassium in acute renal failure.
.
10. Anaemia associated with chronic renal failure is caused by:
A Reduced red cell production
B Increase in production of testosterone
C Iron depletion
D Iron toxicity from excessive transfusion
E Splenomegaly
10. Answer: A
Anaemia in chronic renal failure (CRF)
Anaemia in CRF is due to decreased red cell survival time and reduced red cell production as a result
of a deficiency in renal erythropoietin. Splenic sequestration for RBCs can also cause anaemia. There is

4
Dr.Wahid Helmi Egypt

no role for iron supplements for patients with anaemia secondary to CRF unless iron deficiency
anaemia has been proved by laboratory testing. Regular transfusions,
splenectomy and exogenous testosterone for postpubertal patients can be used to treat anaemia, with
other measures to treat chronic renal failure,including renal transplantation
11. A 3-year-old child presents with microscopic haematuria and dysuria. Renal ultrasound
shows one calculi in his bladder. He has passed three stones in his urine in the past. His brother
suffered
from recurrent renal stones, but responded very well to treatment with D-penicillamine. What is
the most likely diagnosis?
A Cystinosis
B Cystinuria
C Primary hyperparathyroidism
D Idiopathic hypocalcaemia
E Xanthinuria

11. Answer: B
Cystinuria

Cystinuria is characterized by excessive secretion of the dibasic amino acids cystine, ornithine, arginine
and lysine, with the same transport effect being found in the gastrointestinal tract.

It is inherited as an autosomal recessive disorder.

Stones can be prevented by maintaining a high urine flow and alkalizing the urine.

If all fails and the patient still forms stones,

D-penicillamine is effective in preventing further stones.

12. The following are all true about infantile polycystic renal disease in
older children except:
A There is progressive renal failure.
B It is an autosomal recessive disorder.
C It is associated with severe hypertension.
D Portal hypertension is a well-recognized feature.
E There are respiratory problems.
12. Answer: E
Infantile polycystic renal disease in older children
Renal failure usually develops in the 2nd year of life, with renal enlargement and vomiting. Blood
pressure will be high and may lead to congestive heart
failure. Portal hypertension will not present before the age of 5 years.

13. Infants with horseshoe kidneys are likely to have:


A Hypertension
B Renal colic
C Features of Turner’s syndrome
D Renal calculi
E Renal artery stenosis
13. Answer: C
Horseshoe kidneys

5
Dr.Wahid Helmi Egypt

Horseshoe kidneys are more common in males, and are associated with Turner’s syndrome. The
condition is usually asymptomatic. It can sometimes present as an infection, reflux or obstructive
uropathy. Those affected are prone to hydronephrosis.

14. In newborn babies, acute urinary obstruction is due to all of the


following except:
A Posterior urethral valve
B Spinal cord compression
C Pelvic teratoma
D Rhabdomyosarcoma
E Urethrocele
14. Answer: D

Acute urinary obstruction


The causes of acute urinary obstruction in the newborn period also include traumatic delivery and
prolapsing urethrocele in females; it may also be temporary,with no obvious cause.

In older children, it can be due to a posterior urethral valve in males, trauma in both males and females,
rhabdomyosarcoma in males, prolapsing urethrocele is both males and females, and meatal polyps in
males.

15. A 3-year-old girl presents with abdominal pain, dysuria, bed-wetting, haematuria and a
urinary tract infection. MCUG shows no reflux. DTPA shows delay in excretion of isotopes from
the left kidney, with an increase in flow in response to furosemide (frusemide).The possible
diagnosis is:-
A Renal vein thrombosis
B Nephrotic syndrome
C Acute glomerulonephritis
D Urticaria
E Renal failure
F PUJ obstruction

15. F: PUJ obstruction is the most common cause of hydronephrosis;it is more common in males. In
newborn babies, it can present as an abdominal mass, with vomiting and failure to thrive. Haematuria
and hypertension may be associated symptoms.Abdominal ultrasound and intravenous pyelography are
the investigations to confirm the diagnosis. Surgical intervention
can be performed early to preserve kidney function.

16. A 3-year-old boy has had increased swelling of his legs,abdomen and face over the last 10
days. The urine shows proteinuria and haematuria, and the blood pressure is 110/70 mmHg. He
has gained 3 kg in the last 10 days. The possible diagnosis is
A Renal vein thrombosis
B Nephrotic syndrome
C Acute glomerulonephritis
D Urticaria
E Renal failure
F PUJ obstruction

16. C Acute glomerulonephritis:

6
Dr.Wahid Helmi Egypt

The presentation of acute glomerulonephritis can be typical with facial swelling followed by lower
limb swelling and passage of dark red urine.
Hypertension may be noted, and is present

in the majority of hospitalized children. Some may present with tachypnoea and dyspnoea due to
pulmonary congestion or the presence of pleural effusion. Urine will show numerous red and
white cells and less proteinuria.

There will be red cell casts,which are considered as glomerular leaking in these cases. The C3 level
will be low, and C4 can be low (but not in the majority
of cases). Bed rest, furosemide (frusemide) and control of hypertension are used as management tools,
after consultation with nephrologists.

17. A 10-year-old child presents with macroscopic haematuria for the second time in the last 3
months. Urine culture is negative and renal ultrasound is normal. C3 and C4 levels and urine
microscopy for casts and protein are negative.His renal biopsy shows mesangial deposition of IgA
and a normal basement membrane. The possible diagnosis is
A Renal vein thrombosis
B Nephrotic syndrome
C Acute glomerulonephritis
D Urticaria
E Berger’s disease

17. E Berger’s disease: Berger’s disease (mesangial IgA nephritis) is more common in boys and can
be familial. It commonly presents as haematuria followed by an upper respiratory tract infection.
Abdominal pain is common, while other patients just have red urine. Haematuria can start as
microscopic then become macroscopic. Only asmall group progress to renal nephritis. The renal biopsy
finding on immunofluorescence is of granular deposits of IgA in the mesangium of the glomerulus,
which is associated with IgG and C3.

18. A 3-week-old male infant has a history of diarrhoea for 3 days. His urine shows gross
haematuria, with one plus protein and urine output less than 1 ml/kg/h. He is diagnosed as
having acute renal failure from a biochemical test,and management of renal failure is started. His
left kidney is palpable with a normal appearance. The possible diagnosis is
A Renal vein thrombosis
B Nephrotic syndrome
C Acute glomerulonephritis
D Left renal vein thrombosis
E Renal failure
F PUJ obstruction
G Polycystic kidney disease

18. D Left renal vein thrombosis: More than three-quarters of cases of left renal vein thrombosis
occur in the first month of life, with normal kidneys at birth. It is associated with gross haematuria,
oliguria, renal failure and a history of diarrhoea. Proteinuria is rare, and thrombocytopenia may present.
The infants of diabetic mothers with hypernatraemic dehydration and hypoxic ischaemic
encephalopathy will be at high risk of getting renal vein thrombosis. Children can be managed
conservatively, and a nephrectomy is not indicated.

7
Dr.Wahid Helmi Egypt

19. A 6-year-old boy is referred with a history of bed-wetting.His urine shows no growth, and he
is doing well at school. The GP organized an ultrasound scan of his kidneys, which was reported
as normal. He lives with his mother and 10-year-old brother. His father is in prison and his
mother has a new partner. The most appropriate management plan is
A Refer to an enuresis clinic
B Start on treatment (desmopressin nasal spray)
C Repeat renal ultrasound and urine analysis
D Refer to a psychologist
E Start star achievement cards

19. E: The management of enuresis starts with star achievement cards. If these do not work, use oral or
nasal desmopressin. An alarm can also be used before turning on medication. Specialist
enuresis clinics supported by specialist nurses, paediatricians and psychologists are sometimes also
needed.

20. A 4-year-old child presents with a diagnosis of passing calculi in his urine on many occasions.
He is developmentally delayed, with thick upper lips, a depressed nasal bridge and prominent
maxilla. He is very sociable and is very popular in the nursery. His ultrasound shows multiple
small renal calculi,and his father says that he has problems with his heart on the right side
(William’s Syndrome). The most appropriate management plan is
A Refer to an enuresis clinic
B Start on treatment (desmopressin nasal spray)
C Repeat renal ultrasound and urine analysis
D Refer to a psychologist
E Start star achievement cards
F Low calcium intake

20. F: Hypercalcaemia can be managed by low calcium and vitamin D intake – but watch out for
rickets. Intravenous calcitonin can be used in patients with hypocalcaemia secondary to malignancy.

21. A 2-year-old boy presents with lethargy and fever and is sleepy. His urine output is less than
0.5 ml/kg/h and his serum potassium 6.9 mmol/l.
A Refer to an enuresis clinic
B Intravenous salbutamol)
C Repeat renal ultrasound and urine analysis
D Intravenous insulin
E Start star achievement cards

21. B, D:- Hyperkalaemia can be managed with 10% calcium gluconate IV; 7.5% sodium bicarbonate
IV can also be used.

Salbutamol infusions are used most frequently.

IV insulin with
glucose should be used if the others fail. Dialysis will be the last resort if the child’s health deteriorates

8
Dr.Wahid Helmi Egypt

22.A 7-year-old boy presents with generalised oedema. Urinalysis shows marked
albuminuria.Blood tests reveal hypoalbuminaemia and hyperlipidaemia. A renal biopsy appears
normal on light microscopy.What would be the most likely finding on electron microscopy?

A Deposition of electron-dense material on the capillary basement membrane

B Splitting of the capillary basement membrane

e Fusion of foot processes of the glomerular epithelial cells

D Thinning of the capillary basement membrane

E Fibrils of amyloid protein in the mesangium

Explanation

The most common cause of nephrotic syndrome in children is minimal-change nephropathy.Loss of


foot processes may be seen in other proteinuric states such as membranous glomerulonephritis and
diabetic nephropathy, but light microscopic changes would also be evident in these conditions.
Splitting of the capillary basement membrane is seen in mesangiocapillary glomerulonephritis, while
thinning is noted in thin glomerular basement membrane disease.

23.A 2-year-old boy has vitamin D-resistant rickets. Investigations show: serum calcium 2.6
mmol/l, phosphate 0.5 mmol/| and alkaline phosphatase 1040 U/I|. Parathyroid hormone and
bicarbonate levels are normal.

What is the most probable diagnosis?


A Distal renal tubular acidosis

B Hypopnosphataemic rickets

Ce Vitamin D-dependent rickets

D Proximal renal tubular acidosis

E Hyperparathyroidism

24.A 6-week-old baby is vomiting excessively due to pyloric stenosis.


What significant finding would you expect in a blood test?

A Increased serum chloride levels

9
Dr.Wahid Helmi Egypt

B Hypernatraemia

c. Hypokalaemia

D Decreased serum bicarbonate level

E Plasma pH > 8.0


Explanation
The loss of gastric fluid rich in hydrochloric acid results in a metabolic alkalosis because bicarbonate
generated during the production of gastric acid returns to the circulation.
Although sodium and potassium loss in the gastric juice is variable, the obligate urinary loss of these
cations is intensified by bicarbonaturia, which occurs during disequilibrium. Aplasma PH of 8.0 is
incompatible with life.
The mortality rate is 80% when the pH is greater than 7.65.

25.A 1-year-old boy presents with a poor urinary stream since birth.
Which of the following investigations would be diagnostic in this case?

A Micturating cystourethrography

B Ultrasound of the bladder

S Excretion urography

D Uroflowmetry

E Computed tomography

Previous Question Skip Question

Explanation .
A poor urinary stream suggests a urinary tract obstruction (usually infravesical). The most Re.common
cause in a male child is posterior urethral valves. The best diagnostic method is amicturating
cystourethrography. The other option is endoscopy. Both these investigations would clearly show the
site of obstruction.

26.A 4-year-old boy complains of abdominal pain and inability to pass urine in a good stream.An
ultrasound scan of the abdomen shows both kidneys are enlarged, irregular, and cystic.Further
investigation shows that they are dysplastic.

What is the most likely cause of this condition?

A Posterior urethral valve

B Bladder extrophy
ce Pelvi-ureteric junction obstruction

D Recurrent urinary tract infection

10
Dr.Wahid Helmi Egypt

E Vesicoureteric reflux

Explanation

Renal dysplasia occurs due to an abnormality in metanephric differentiation. It is


characterised histologically by the persistence in the kidney of abnormal structures such as cartilage,
undifferentiated mesenchyme and immature collecting ducts and by abnormal lobar organisation. Renal
dysplasia is usually associated with posterior urethral valves. The latter should always be considered in
a boy with poor urinary stream or a thick-walled trabeculated bladder on ultrasound.

27.A 3-year-old child is brought to the clinic with a history of lethargy, failure to thrive,excessive
thirst and constant bed-wetting. A blood test shows hypokalaemia and metabolic alkalosis. There
is hyperplasia of the juxtaglomerular apparatus seen on renal biopsy.What is the most probable
diagnosis?
A Cystinosis
B Bartter’s syndrome
c. Minimal-change nephropathy
D Nephroblastoma
E Medullary cystic disease

Explanation
Bartter’s syndrome consists of metabolic hypokalaemia, alkalosis, hyopercalciuria, normal blood
pressure and elevated plasma renin and aldosterone.

It is an autosomal-recessive condition leading to tubular defects in sodium chloride transport and


increased intrarenal production of PGE?.

A renal biopsy is diagnostic.Cystinosis, another autosomal-recessive disorder, causes hyperchloraemic


acidosis and rickets.

Medullary cystic disease (juvenile neophronophthisis) is also inherited in an autosomal-recessive


manner. Diagnosis is based on the family history and renal biopsy,which shows interstitial
inflammation, tubular atrophy and medullary cysts.Minimal-change nephropathy usually occurs in
older children and presents with proteinuria.Hypokalaemia and alkalosis do not occur. Glomeruli
appear normal on light microscopy.Fusion of the foot processes of epithelial cells is seen on electron
microscopy.Nephroblastoma (Wilms’ tumour) is seen mainly within the first 4 years of life and may be
bilateral. It presents as an abdominal mass. Metabolic changes are not seen in this disease. =

28.A 15-year-old boy presents with dark discolouration of urine. There is a history of upper
respiratory tract infection two weeks earlier. If untreated, he may go on to develop:
A Nephrotic syndrome
B Immune deficiency with normal complement (C3) levels
c Acute renal failure
D Fits or a stroke
E Nephroblastoma

Explanation

11
Dr.Wahid Helmi Egypt

Bacterial infections, usually subacute, and typically Group A Streptococcal, may cause avariety of
histological patterns of glomerulonephritis, but usually with plentiful immunoglobulin deposition and
often with evidence of complement consumption (low C3).A low C3 level is indicative of poor
prognosis in nephrotic syndrome, but is common in glomerulonephritis. Renal failure occurs in less
than 1% of cases of post-streptococcal nephritis. Hypertensive encephalopathy may occur and result in
fits, coma and neurological events like stroke.

12
Dr.Wahid Helmi Egypt

29.A 15-year-old girl presents with red spots on her buttocks and legs, joint pains,
oedema,hypertension and proteinuria. She had an upper respiratory tract infection 2-3 weeks
ago.What is the most likely diagnosis?

A Proliferative glomerulonephritis

B Goodpasture’s syndrome

c. Wegener’s granulomatosis

D Henoch-Schonlein purpura

e Systemic lupus erythematosus

Explanation

This girl most probably has Henoch-Schénlein purpura. It presents with a purpuric rash over the
buttocks and extensor surfaces. Some one-third of patients have nephritis. Joint pains and acute
abdominal pain are common. The fault lies in the vasculature; the platelets are normal. It often follows
an acute respiratory tract infection and usually follows a benign course over weeks or months.
Complications include massive gastrointestinal haemorrhage,ileus and, rarely, renal failure.

Wegener's granulomatosis would have additional features such as nasal ulceration, rhinitis,otitis media
and pulmonary symptoms, such as haemoptysis. Variable shadows may be seen on chest X-ray,
particularly multipole nodules. Lung involvement (pulmonary haemorrhage) is seen in Goodpasture’s
syndrome: the chest X-ray may show diffuse infiltrates in the lower zones. Renal involvement in
systemic lupus erythematosus occurs in 50% of patients but is seldom so rapidly progressive as to
cause oedema, hypertension and proteinuria in 2-3weeks time. Proliferative glamerulonephritis would
present with macroscopic haematuria,but purpura is not a feature of this condition.

30.Primary vesicoureteric reflux is most commonly found in which patient population?

A Girls aged 3-10 years

B Prepubertal boys

C Newborn girls

D Teenage girls

e Boys over 15 years of age

Explanation
Vesicoureteric reflux refers to the retrograde flow of urine from the bladder to the upper Urinary tract.
This may occur due to incompetence of the valve at the vesicoureteric junction. It is most commonly
detected the earliest in newborn girls.

13
Dr.Wahid Helmi Egypt

31.A 4-year-old child is brought to see you as his mother noticed that his urine is pink. He has
been complaining of headaches. His blood pressure is 130/88 mmHg. Blood tests reveal: Na+130
mmol/L, K+ 4.9 mmol/L, urea (Ur) 12 mmol/L, creatinine (Cr) 96 umol/L. Complement C3 is
reduced but C4 is normal. Aloumin is 29 g/L.
A Bartter syndrome

B Factitious illness

C Haemolytic uraemic syndrome (HUS)


D Henoch-Schonlein purpura (HSP) nephritis
E Mesangial IgA nephropathy

F Nephrotic syndrome

G Post-streptococcal glomerulonephritis.
H Reflux nephropathy

Red urine probably indicates haematuria. He is hypertensive and is having symptomatic headaches;
these two features fit with a nephritis. A low C3 and normal C4 are typical for post-streptococcal
glomerular nephritis.

32.A 13-year-old child presents with blood in the urine. He reports having an upper respiratory
tract infection (URTI) 3 days ago. His blood pressure is 120/60 mmHg. Urine analysis reveals
only blood 3+, protein 2+. Renal function is normal. Complement levels are normal.
A Bartter syndrome

B Factitious illness

C Haemolytic uraemic syndrome (HUS)


D Henoch-Schonlein purpura (HSP) nephritis
E Mesangial IgA nephropathy

F Nephrotic syndrome

Though he has had a recent URTI, the onset of the symptoms is too soon after the URTI for
post-streptococcal glomerular nephritis (which is normally more than a week after).

In addition his blood pressure is normal (which doesn’t fit with a nephritis.
33.A 6-year-old child who has had diarrhoea for the past week is brought to the Emergency
Department. Her baseline bloods reveal: Na+ 133 mmol/L, K+ 5.4 mmol/L, Cr 90 umol/L, Ur42
mmol/L, Hb 6.3 g/dL, white cell count (WCC) 2.3 x 10°/L, platelets (plts) 95 x 10°/L.
A Bartter syndrome
B Factitious illness
C Haemolytic uraemic syndrome (HUS).
D Henoch-Schonlein purpura (HSP) nephritis

Any renal impairment following a diarrhoeal episode should raise the suspicion of HUS. This is
confirmed by a very high urea, low Hb and low platelets. Examination of a blood film would show a
microangiopathic haemolytic anaemia. HUS is the most common cause of acute renal failure in
childhood. It is caused by verotoxins produced by Escherichia coli 0157(a cause of bloody diarrhoea).

14
Dr.Wahid Helmi Egypt

Over half of the cases require dialysis.

34-Gross hematuria in a child may be due to all the following except


A-Infection
B-Porphyria
C- Glomerulonephritis
D- Schönlein-Henoch purpura
E- Hypercalciuria
34-(B) Pigmenturia (porphyria, urate, beets, drugs).

35-Polyuria can be caused by all the following except


A- Nephrogenic diabetes insipidus
B-Hyperkalemia
C-Hypercalcemia
D-Sickle disease/trait
E-Polyuric renal failure
35-(B) Polyuria is caused by diabetes mellitus, central and nephrogenic diabetes insipidus, obstruction,
dysplasia, hypokalemia, hypercalcemia,psychogenic polydipsia, sickle disease/trait, polyuric renal failure, and
diuretic abuse.

36-Functions of the proximal tubule include the following except


A-Reabsorb two thirds of the filtered volume, sodium, and chloride
B-Glucose and amino acids are almost completely reabsorbed
C-Approximately 75% of filtered bicarbonate is reabsorbed
D-Primary site of antidiuretic hormone (vasopressin) response
E- Calcitriol is produced by proximal tubular cells
36-(D) The collecting duct is the primary site of antidiuretic hormone
(vasopressin) response, which leads to urine concentration.

37-The following are secondary systemic causes of renal disease in


children
A-Minimal change nephrotic syndrome
B-Membranous nephropathy
C-Galactosemia
D-Alport syndrome
E-Polycystic kidney disease
37-(C) Minimal change nephrotic syndrome, membranous nephropathy,
Alport syndrome and polycystic kidney disease are primary causes of
renal disease.

38-Ultrasound reliably assesses the following except


A-Kidney structure and function
B-Kidney presence and size
C-Degree of pelvic dilation
D-Differentiates cortex and medulla
E-Bladder visualization
38-(A) Computed tomography (CT) and magnetic resonance imaging (MRI) have mostly replaced the
intravenous (IV) pyelogram to evaluate kidney structure and function.

15
Dr.Wahid Helmi Egypt

39-Membranoproliferative glomerulonephritis is characterized by the


following except
A-Hypocomplementemia
B-Signs of glomerular renal disease
C-Comprise 5% to 15% of children with primary nephritic syndromeNS
D-Seen most commonly with systemic infections
E-Likelihood of progression to renal failure
39-(D) Membranous nephropathy is seen most commonly in adolescents and children with systemic infections,
such as hepatitis B, syphilis,malaria, and toxoplasmosis, or who are receiving drug therapy (gold salts,
penicillamine).

40-The most common presentation for children with nephrotic


Syndrome (NS) is
A-Anorexia
B- Malaise
C-Abdominal pain
D-Pitting edema
E-Respiratory distress
40-(D) The sudden onset of dependent pitting edema or ascites is the most
common presentation for children with NS.

41-Typical minimal change nephrotic syndrome (MCNS) is characterized by


A-Heavy protienuria
B- Hematuria
C- Renal insufficiency
D-Hypertension
E- Hypocomplementemia
41-(A) Typical MCNS is characterized by the absence of hematuria as well
as renal insufficiency, hypertension, and hypocomplementemia.

42-Transient proteinuria can be seen after the following conditions


except
A-Vigorous exercise
B-Fever
C-Significant dehydration
D-Seizures
E- Adrenergic antagonist therapy
42-(E) Adrenergic agonist therapy.

43-Tubular proteinuria is typically suspected in conditions associated


with the following except
A-Pyelonephritis
B-Hemolytic uremic syndrome [HUS]
C- Structural renal disorders
D-Polycystic kidney disease
E-Tubular toxins
43-(B) hemolytic uremic syndrome [HUS] is a cause of glomerular proteinuria.

16
Dr.Wahid Helmi Egypt

44-One of the following is a major complication in children with NS


A-Hypovolemia
B- Peritonitis
C-Thromboembolism
D- Atherosclerotic vascular disease
E-Diarrhea
44-(B) Infection is a major complication in children with NS. An increased incidence of serious infections,
particularly bacteremia and peritonitis (particularly Streptococcus pneumoniae, Escherichia coli, or Klebsiella),
is due to urinary loss of immunoglobulins and complement.

45-Red urine, dipstick blood, but no red blood cells (RBCs) on urine
microscopy suggest
A-Poststreptococcal acute glomerulonephritis
B-Alport syndrome
C-Hemolytic uremic syndrome
D-Acute intravascular hemolysis
E-Tubulointerstitial nephritis (TIN)
12-(D) Red urine, dipstick blood, but no red blood cells (RBCs) on
(prompt) microscopy suggest: Ingested foods, medications, or chemicals
or the presence of free hemoglobin or myoglobin.
46-The following are complications of acute postinfectious GN except
A-Heart failure
B-Seizures
C-Encephalopathy
D-Renal insufficiency
E- Thromboembolism
13-(E) Thromboembolism is a complication of nephritic syndrome.

47-The most common identifiable cause of hematuria in children is


A-Sickle cell trait
B-Strenuous exercise
C-Urinary tract infection (UTI)
D-Urolithiasis
E-Renal trauma
47-(C) Urinary tract infection (UTI) is the most common identifiable cause of hematuria in children.

48-IgA nephropathy (IgA GN) is characterized by the following except


A-Microscopic hematuria or recurrent gross hematuria
B-Usually follow upper respiratory infection
C-More frequent in boys
D-Usually progress to end-stage renal disease
E-Chronic steroid therapy and anti-inflammatory strategies (fish oil,
vitamin E) show promising results
48-(D) The course of IgA GN is usually benign, and occurs more frequently in boys and in school age children
and young adults. IgA GN may progress to end-stage renal disease (ESRD) in up to 30% of children,
particularly in patients with proteinuria or renal insufficiency at presentation.

17
Dr.Wahid Helmi Egypt

49.- Classic hemolytic uremic syndrome D+HUS is characterized by the following except
A- Microangiopathic hemolytic anemia
B-Thrombocytopenia
C-Renal injury
D- Prodromal diarrheal illness
E- Typically occurs in school age children

49-(E) The disease typically occurs in children between 6 months and 4years of age.

50-Lab.investigations in hemolytic uremic syndrome usually reveals the following except


A- Schistocytes, helmet, and burr cells in blood smear
B- Positive coombs test
C- Hematuria and proteinuria
D- Elevated reticulocyte count
E- Low plasma haptoglobin

50-(B) Coombs test is negative.

51-Therapy for hemolytic uremic syndrome HUS includes the following except
A-Early dialysis
B-Red blood cell transfusions
C-Antibiotics
D-Platelet transfusions
E-Volume repletion

51-(C) Antibiotics for the prodromal diarrhea may increase the risk of
developing HUS. Antidiarrheal agents prolong exposure to VT-producing bacteria and should be avoided.

52-Nonoliguric acute renal failure (ARF) is characterized by the


following except
A-Normal urine output
B-Easily missed
C-Electrolyte disturbances
D-Uremia
E-Urine osmolality is typically higher than serum osmolality

52-(E) Nonoliguric ARF can be easily missed. Despite normal urine output,
electrolyte disturbances and uremia may become significant. Urine
osmolality is typically similar to serum osmolality in such patients.

53-The following are postrenal causes of acute renal failure


A-Tumor lysis syndrome
B-Acute tubular necrosis
C-Nephrotoxins (drugs)
D-Acute cortical necrosis
E-Disseminated intravascular coagulation

53-(A) Acute tubular necrosis, nephrotoxins (drugs), acute cortical


necrosis, and disseminated intravascular coagulation are intrinsic causes

18
Dr.Wahid Helmi Egypt

of acute renal failure.

54-The following laboratory findings are seen in a child with prerenal insufficiency except
A-Urine Na+ (mEq/L) <20
B-FENa (%) = [(urine sodium/plasma sodium) ÷ (urine creatinine/plasma
creatinine)] × 100<1
C-Urine osmolality (mOsm/L) >500
D-BUN/serum Cr ratio>20
E-Urinalysis reveals RBCs/WBCs, protein, and casts

54-(E) Urinalysis is normal.

55-In children with acute renal failure ARF, dialysis is indicated in the
following conditions except
A-Hypervolemia
B-Hyperkalemia unresponsive to medical therapy
C-Acidosis unresponsive to medical therapy
D-Blood urea more than 200 mg/dl
E-Uncontrolled hypertension

55-(A) Hypervolemia unresponsive to fluid restriction or diuretics.

56-Growth failure in children with chronic kidney disease (CKD) is due


to the following except
A-Poor nutrition
B-Renal osteodystrophy (ROD)
C-Chronic diarrhea
D-Acidosis
E-Hormonal abnormalities
56-(C) The factors associated with growth failure in children with CKD include poor nutrition, renal
osteodystrophy (ROD), acidosis, anemia,hormonal abnormalities, medication toxicity (steroids), and the growth
hormone/IGF-1 resistance seen in uremia.

57-The major complications of kidney transplants are the following except


A- Side effects of medications
B- Infections
C-Cardiac complications
D-High rejection rate
E-Increase in malignancies

57-(D) Kidney transplants have an excellent success rate. More than 90%
of transplants in children (living or deceased donor) function 1 year after
transplant; more than 50% still function 20 years later.

58-Congenital renal causes of hypertension include


A-Glomerulonephritis
B-Obstructive uropathy
C-Hemolytic uremic syndrome
D-Reflux nephropathy
E-Wilms tumor

19
Dr.Wahid Helmi Egypt

58-(B) Glomerulonephritis, hemolytic uremic syndrome, reflux nephropathy, and Wilms tumor are acquired
renal causes of hypertension.

59-Regarding hypertension HTN in children, the following are true except


A-Risk for sequelae of HTN increases as blood pressure increases
B-Obese children are more likely to develop essential HTN
C-Essential HTN is now the most common cause of HTN
D-Endocrine disease is the most common cause of secondary HTN
E-Most children with HTN have no symptoms
59-(D) Renal disease is the most common cause of secondary HTN in children.

60-Risk factors for the development of vesicoureteral reflux (VUR)


include the following except
A- Acquired bladder obstruction
B- Cystitis
226
C-Neurogenic bladder
D- Reflux nephropathy
E- Congenital incompetence of the ureterovesical (UV) junction
60-(D) Reflux nephropathy refers to development and progression of
renal scarring. This is a particular risk if VUR is associated with infection
or obstruction.
61-The most common cause of urethral obstruction in male is
A-Posterior valves
B-Diverticula
C- Strictures
D-Atresia
E-Ectopic ureter
61-(A) Posterior urethral valves are the most common cause of bladder
outlet obstruction in males, present in 1 in 50,000 boys.
62-One of the following is the most common inherited kidney disease
A-Unilateral renal agenesis
B-Renal hypoplasia/dysplasia
C-Multicystic renal dysplasia (MCD)
D-Autosomal recessive polycystic kidney disease
E-Autosomal dominant polycystic kidney disease
62-(E) Autosomal dominant PKD, due to defects in polycystin 1 or 2,
occurs in 1 in 1000 individuals, making it the most common inherited
kidney disease.
63-One of the following is not a feature of autosomal recessive
polycystic kidney disease PKD
A- Marked bilateral renal enlargement
B- Hepatic fibrosis
C- Bile duct ectasia
D- Pneumothorax
E- Cerebral aneurysms
63-(E) Cerebral aneurysm is a feature in autosomal dominant polycystic
kidney disease PKD.
64-Renal ultrasound (RUS) allows identification of the following except
A-Renal agenesis

20
Dr.Wahid Helmi Egypt

B-Renal scars
C- Renal hypoplasia
D- Renal cysts
E-Urinary tract dilation
64-(B) Nuclear renal scanning best identifies renal scars.

65-Metabolic causes of urinary tract calculi include the following except


A-Idiopathic familial hypercalciuria (IHC)
B-Uric acid disorders
C-Proximal renal tubular acidosis
D-Cystinuria
E- Primary hyperparathyroidism
65-(C) Metabolic causes include idiopathic familial hypercalciuria (IHC),hyperoxaluria, uric acid disorders, distal
renal tubular acidosis, cystinuria,hypercalcemic hypercalciuria, and primary hyperparathyroidism.

66-The following definitions are true except


A-Primary enuresis: Child has never had a prolonged (usually > 3 month) span of night-time continence
B- Retractile testes: Cryptorchidism
C- Hypospadias: Urethral meatus is located ventrally and proximal to its
normal position
D- Phimosis: Inability to retract the prepuce
E- Paraphimosis: The prepuce has been retracted behind the coronal
sulcus and cannot resume its normal position

66-(B) Retractile testes are normal testes that retract into the inguinal
canal from an exaggerated cremasteric reflex.

21
Neonatoloy Revision
Dr.Wahid Helmi Egypt
Consultant Pediatrician

1) Hepatitis B infection all false except:-


A. Increase incidence of malformed & stillbirth
B. Increase the risk of prematurity
C. Hepatitis B immunization & vaccine should be given in 1st week
D. 25% babies born to mother with HBsAg positive are HBsAg negative
E. Presence of HBeAg has no role in increase transplacentalcross infection
Ans:-B

2) The following should be investigated in five day old baby:-


A. Erythema Toxicum
B. Cloudy cornea
C. Divarication of rectii
D. Subconjunctival hemorrhage
E. Preauricular skin tags
Ans:-BE
3) Regarding Tetracyclin all are true except:-
A. It is selectively concentrated in the teeth by chelating calcium
B. It effect decidual teeth
C. It effect permanent teeth
D. Has no effect on eye ( CAUSE CATARACT)
E. It depress skeletal growth
Ans:-D

4) Necrotizing enterocloitis is recognized complication of:-


A. Polycythemia
B. Umbilical arterial catheterization
C. Parenteral feeding
D. Maternal Crohn's disease
E. Birth asphyxia in term neonate
Ans:- ABE
5) The following is true regarding changes in the fetal cardiovascular system after
birth:
A. There is normally immediate closure of the ductus arteriosus
B. Hypoxia-induced vasoconstriction is the mechanism of closure of the ductus
arteriosus
C. The ligamentum teres is the remnant of the umbilical vein
D. Regression of right ventricular hypertrophy occurs postnatally
E. Inferior vena caval pressure falls after birth
Ans:-CDE
Occlusion of the umbilical cord removes this low resistance capillary bed from the
circulation;breathing results in a marked decrease in pulmonary vascular resistance, hence
there is increased pulmonary blood flow returning to the left atrium raising the pressure in
the left atrium causing the foramen ovale to close. As pressure in the systemic circulation
rises, shunt through the ductus arteriosus reverses. As the pO2 rises, synthesis of
bradykinin and prostacyclins is inhibited, thus causing closure of the ductus arteriosus.
The ductus arteriosus can take up to 3 months to close in normal neonates.
6) Neonatal RDS:-
A. Seen in most babies of birth weight < 2.5 kg.
B. More common in infants of diabetic mothers.
C. Associated with prolonged rupture of membrane.
D. Less sever in babies of Afro-Caribiean origin than Caucasians.
E. Exacerbated by hypothermia.
Ans:-BDE
7) The following are causes of generalized hypotonia in 2 days old infants:-
A. Prematurity
B. Hypothyroidism
C. Myotonic dystrophy
D. Spinal dysraphyism
E. Anterior horn cell disease
Ans:-ACE
8) Regarding surfactant:-
A. Production begins at 30 weeks
B. It is produced by Type II pneumocytes in the walls of the bronchi
C. Testosterone stimulates surfactant production
D. Production is increased during a stressful event like hypothermia
E. Betamathasone given to the mother improves surfactant production in the
premature baby
Ans:- E
Surfactant production begins at 20-22 weeks. It is produced by Type II pneumocytes
which are in the walls of the alveoli. The hormones testosterone and insulin inhibit
surfactant production;hence hyaline membrane disease is more common in males than
females and more common in infants of diabetic mothers. Surfactant production is
suppressed if the baby is hypothermic,hypoxic, acidotic or hypoglycemic.Although
dexamethasone is more commonly used, betamethasone has an identical effect on lung
maturation
9) Concerning fetal lung development:-
A. Type ΙΙ pneumocytes are present at 24 week gestation
B. Cuboidal cells are capable of gas transfer in utero
C. There is virtually no smooth muscle in the terminal & respiratory bronchioles at
6 month of age
D. The large airways are formed at 16 week gestation
E. Alveoli are completely formed at birth
Ans:-AD
10) Congenital CMV infection:-
A. Only 10% of affected pregnancies have resulting long term sequel at birth
B. Diagnosis is by viral isolation from the urine
C. Hearing loss can develop gradually over the first 5 years
D. The affected newborn should be treated with ganciclovir
E. Intracranial calcifications are seen in a periventricular distribution
Ans:- ABCE
Congenital CMV occurs in approximately 1% of all live births and only 10% of these
infections result in clinical symptoms. Severe clinical disease is associated with primary
maternal infection in pregnancy. Infection in early gestation carries a far greater risk of
severe fetal disease. In CMV intra-cranial calcifications are in a periventricular
distribution. Ganciclovir is only used if there is CNS involvement, chorioretinitis or
pneumonitis.
11) The following conditions will present with cyanosis in the first week of life:
A. Aortic stenosis
B. Transposition of the great vessels
C. Hypoplastic left heart syndrome
D. Fallot's tetralogy
E. Fallot's pentalogy
Ans:- B
Any cardiac lesion which allows a mixing of blood along with a right to left flow or any
cardiac lesion wherein pulmonary perfusion is impaired results in cyanosis. Left heart
problems or outflow tract obstructions present as cardiac failure. Fallot's pentalogy
includes an ASD along with the tetrad of infundibular pulmonary stenosis, RVH, over-
riding of the aorta and a VSD.Babies with tetralogy of Fallot usually have a patent ductus
arteriosus at birth that provides additional pulmonary blood flow, so severe cyanosis is
rare early after birth.As the ductus arteriosus closes, as it typically will in the first days of
life, cyanosis can develop or
become more severe.The degree of cyanosis is proportional to lung blood flow and thus
depends upon the degree of narrowing of the outflow tract to the pulmonary arteries.
12) Pulmonary surfactant
A. Is partly recycled by endocytosis into the synthesizing cell
B. Is produced by type Ι alveolar cells
C. Reduction in pulmonary flow can cause a decrease in surfactant production
D. Synthesis is inhibited by thyroxine
E. Synthesis is stimulated by glucocorticoids
Ans:- ACE 5Dipalmityl- phosphotidyl choline is the main component of surfactant and is
produced by Type-ΙΙ alveolar cells (granular pneumocytes). Its half-life is 14 hours and its
main function is to reduce the surface tension of the alveoli.
13) Lung surfactant
A. Decreases the surface tension within an alveolus
B. Causes an increase in chest wall compliance
C. Is a glycoprotein
D. Maintains the same surface tension for different sized alveoli
E. Appears only after the 1st week of life
Ans:- A
Surfactant is a dipalmitoyl-phosphatidyl choline and is a phospholipid, which prevents
alveolar collapse by reducing alveoli surface tension. It is produced by type-II
pneumocytes and is seen at about 24 weeks gestation. It causes an increase in lung
compliance only (not chest wall compliance).
14) The following organisms cause conjunctivitis:-
A. Epstein Barr virus
B. Chlamydia trachomatis
C. Adenovirus
D. Haemophilus influenzae
E. Neisseria gonorrhoeae
Ans:- BCDE
Chlamydia trachomatis causes conjunctivitis in 30-50% of neonates born to mothers with
cervicitis. It is a purulent conjunctivitis, which develops 5-14 days after birth and is
indistinguishable from gonococcal infection. It is diagnosed on a swab scraped over the
lower eyelid (to allow cells to be collected – don’t forget it is an intracellular organism) by
direct fluorescent antibody, ELISA or PCR. Tetracycline ointment topically is combined
with oral erythromycin – the oral antibiotic is to prevent relapse after ointment is
discontinued and to prevent progression to pneumonia. Gonococcal conjunctivitis presents
earlier than chlamydial disease (usually within 2 days), is diagnosed on gram stain and
culture and should be treated with IV penicillin and chloramphenicol eye drops. Don’t
forget sexual health screening for the mother and informing public health of ophthalmia
neonatorum. Adenovirus causes conjunctivitis in summer outbreaks; enterovirus,
coxsackie and herpes simplex are other viral causes.
15) Concerning blood flow in the fetus:-
A. Blood flow from right to left through the foramen ovale
B. Blood in the ascending aorta has higher oxygen content than in the descending
aorta
C. The ductus arteriosus is closed
D. Pulmonary pressure equal systemic pressure
E. Hemoglobin may be 20 gm/dl
Ans:-ABE
16) -In a healthy baby the transition from fetal to neonatal circulation involves:-
A. Functional closure of the foramen ovale in the first 24 hours
B. Blood flow in the ductus arteriosus continues from right to left until its closure
C. Decrease in pulmonary artery resistance following closure of the ductus arteriosus
D. The ductus arteriosus closes in response to decreased oxygen concentrations
E. The umbilical artery is a branch of the common iliac artery
Ans:- A
Functional closure of the ductus arteriosus occurs soon after birth but anatomical closure
can take upto one week. As pulmonary pressures fall after birth, blood flow in the ductus
is reversed ie from left to right. The umbilical artery is a branch of the internal iliac
artery.Factors influencing closure of the ductus include:-
1. Increased oxygen concentrations
2. Decreased prostaglandin levels
3. Drop in pulmonary artery pressures
N.B. Prostaglandin E2 keeps the ductus open.
17) A 10-day old male presents with bilious emesis. What is the most likely diagnosis?
A. Appendicitis
B. Pyloric stenosis
C. Malrotation with midgut volvulus
D. Feeding intolerance
Ans:- C
18) A term newborn is delivered by emergent cesarean section because of
intrauterine growth restriction, oligohydramnios, and nonreassuring fetal heart rate
monitoring in labor. Delivery room resuscitation includes endotracheal intubation
and assisted ventilation with 100% oxygen, chest compressions, intravenous
epinephrine, and volume expansion. Apgar scores are 1, 2, and 3 at 1, 5, and 10
minutes, respectively. An umbilical cord arterial blood gas measurement documents
a pH of 6.9 and a base deficit of 20mmol/L. At 12 hours of age, the infant
demonstrates tonic-clonic convulsive activity of the arms and legs with a concomitant
decrease in heart rate and bedside pulse oximetry saturation. Of the following, the
MOST likely cause for this infant's seizure is:-
A.Hypercalcemia.
B.Hypercarbia.
C.Hyperglycemia.
D.Hypomagnesemia.
E.Hypoxia.
Preferred Response: E
Seizures are the most frequent sign of central nervous system injury in the newborn. When
seizures occur in a newborn who has depressed neuromotor tone, reflexes, and
cardiopulmonary function at birth that requires assisted ventilation, perinatal asphyxia is
likely. In this event, Apgar scores typically are depressed to less than 3 at 5 or more
minutes after birth,and there is a severely acidotic umbilical cord arterial pH (<7.0), with
evidence of metabolic acidemia. Poor tolerance of labor and asphyxia are more common in
fetuses that have experienced intrauterine growth restriction. Because the infant in the
vignette has the previously described features, hypoxic-ischemic encephalopathy (HIE)
must be considered as a cause for the seizures.HIE is the most common cause of seizures
occurring in the first 24 hours of postnatal life CHILDREN HOSPITAL –BENGHAZI
ABDULRAHMAN BASHIR 7and accounts for up to 67% of early neonatal seizures. Other
causes of neonatal seizure include intracranial hemorrhage, cerebrovascular accidents
(stroke), or hemorrhagic infarction (10% to 15%); intracranial malformation (<10%);
transient hypoglycemia or hypocalcemia (<10%); drug withdrawal (<5%); and inborn
errors of metabolism (<5%).When seizures occur beyond the first 24 hours after birth,
especially in the absence of any history of fetal or neonatal asphyxia, the evaluation should
focus on potential causes other than HIE. An additional cause for later seizures is infection
(meningitis, encephalitis).Asphyxia may result in hypocalcemia and hypoglycemia;
hyperglycemia and hypercalcemia are not associated with HIE and do not typically cause
seizures. Hypomagnesemia may accompany hypocalcemia in the infant of a diabetic
mother, but it is not common following asphyxia and is not associated with neonatal
seizures. Hypercarbia may occur in the depressed newborn who has inadequate
ventilation, but it is not associated with seizures unless there is corresponding hypoxia.
19) A 2-week-old-male presents with lethargy and vomiting. His electrolytes reveal
sodium of 121 meq/L, potassium of 7.0-meq/l and blood glucose of 40 mg/dl. What is
the most likely diagnosis?
A. Dehydration
B. Congenital adrenal hyperplasia
C. Inborn error of metabolism
D. Pyloric stenosis
Ans:- B
20) The following cause the onset of persistent vomiting in 3 week old child:-
A. Disaccharidase intolerance
B. Duodenal atresia
C. Pyloric stenosis
D. Hiatus hernia
E. Lactose intolerance
Ans:-C
21) The drug of choice for treating a newborn with presumed ductal-dependant
cyanotic
congenital heart disease is:
A. Morphine
B. Dobutamine
C. Prostaglandin E1
D. Indomethacin
Ans:- C
The drug of choice for ductal dependent cyanotic heart disease is prostaglandin E1- with
astarting infusion of 0.05 - 1ug/kg/min. There is a risk of apnea associated with its use so
be prepared to intubate, other complications include seizures and fever. Patients with
congenital heart disease present with poor feeding, sweating with feeds, tachypnea, sudden
onset of cyanosis or pallor that may worsen with crying, lethargy, or failure to thrive.
Patients with cyanotic congenital heart disease are hypoxic but typically have a minimal
response to oxygen therapy; whereas patients with a pulmonary process causing hypoxia
will have an increase in oxygen saturation when oxygen is administered. Indomethacin is
used to close a patent ductus.

22) How should a neonate with lethargy and a blood sugar of 20mg/dl be treated?
A. Oral feeds with apple juice
B. 25% dextrose solution
C. 10% dextrose solution
D. 50% dextrose solution
Ans:- C
Newborns with hypoglycemia should be treated with D10W solution with a range of 2-
10cc/kg. Higher concentrations should not be used as they can cause vein sclerosis and
intracranial hemorrhage. Infants and young children should be treated with D25 2-4cc/kg.
23) Neonatal convulsion can be caused by:-
A. Maternal hyperparathyroidism
B. Subdural hematoma
C. Birth asphyxia
D. Hyponatremia
E. Wilson's disease
Ans:-ABCD
24) A 2-day-old female presents with abdominal distension and vomiting. She has not
yet passed a meconium stool. What is the most likely diagnosis?
A. Hirschsprung Disease
B. Malrotation with midgut volvulus
C. Necrotizing enterocolitis
D. Constipation
Ans:-A
Suspect Hirschsprung Disease in a newborn who has not yet passed a meconium stool.
Other possibilities include an imperforate anus or meconium plugging. Older children
present with ahistory of chronic constipation. Hirschsprung disease is the absence of
intramural ganglion cells in the rectum which extends to the sigmoid colon in 77% of
patients and involves the entire colon in 15% of patients. The incidence is 1/5,000 live-
births, with a male to female ratio of 4:1. The diagnosis should be suspected if the patient
presents with lack of meconium stool within the first 24 to 48 hours of life. Vomiting and
abdominal distension may also be present.
25) A 3-week-old female presents with persistent seizures despite aggressive
management with benzodiazepines and phenobarbital. The mother reports giving
her daughter some water to "stop her from getting dehydrated." What is the most
likely cause of her status epilepticus?
A. Hypoglycemia
B. Diabetes insipidus
C. Hyponatremia
D. Hypokalemi
Ans:-C
Excessive free water intake can result in hyponatremic seizures. Infants less than 6 months
of age are particularly susceptible to these types of seizures and commonly have
intractable seizures requiring intubation and hypothermia. Immediate treatment includes
the administration of 3% saline 4cc/kg.\
26) A 1-week-old male presents with some mild erythema around his umbilicus
extending onto the abdominal wall. Which of the following is the correct
management for this patient?
A. Reassurance and continue with alcohol wipes of umbilicus
B. Topical antibiotic ointment and recheck the patient the next day
C. Discharge on cephalexin and recheck the next day
D. Perform a full septic workup and admit the patient
Ans:- D
This patient has Omphalitis and should undergo a full septic evaluation, administration of
antibiotics and hospital admission. Surgical debridement may be required for severe
cases.Omphalitis is inflammation and infection surrounding the umbilicus that can spread
to the liver or peritoneum. Patients can present with symptoms ranging from mild
erythema to necrosing lesions
surrounding the umbilicus on the abdominal wall. Fever may be present
27) A 5-day old, well-appearing male is brought to the ED by his mother today
because she noted that he has a cluster of vesicles on his scalp. Which of the following
should be the management approach?
A. Skin biopsy
B. IV acyclovir and a full septic workup
C. Oral acyclovir
D. Discharge, with next day follow up
Ans:-B
This patient is at risk for herpes encephalitis and should undergo a complete septic workup
and IV acyclovir should be initiated in the ED. Begin acyclovir (20mg/kg every 8 hours
IV) if there is apositive maternal history of herpes, a vesicular rash, focal neurologic
findings, CSF pleocytosis or elevated CSF protein without organisms on gram stain.
28) The following maternal condition can cause disease in the fetus/newborn:-
A. Hyperparathyroidism
B. ITP
C. Myasthenia gravis
D. Diabetus mellitus
E. Thyrotoxicosis
Ans:-ABCDE
29)Which of the following heart rates is most suggestive of supraventricular
tachycardia in
a newborn?
A. 180 BPM
B. 230 BPM
C. 150 BPM
D. 210 BPM
Ans:-B
PALS defines SVT in infants as a heart rate of greater than 220 BPM. In older children the
heart rate for SVT is greater than 180 BPM. The ECG demonstrates a narrow complex
tachycardia without discernible p waves or beat-to-beat variability. In the stable patient,
vagal maneuvers are the first treatment of choice (ice to the face, or blowing through an
occluded straw in older children). Adenosine given as centrally as possible is the first drug
of choice (0.1mg/kg up to 6mg for the first dose and then 0.2mg/kg for the second dose up
to 12 mg) If this is not successful, then amiodarone 5mg/kg given over 20-60 minutes or
procainamide 15mg/kg given over 30-60 minutes are the next drugs of choice. Unstable
patients should undergo cardioversion with 0.5-1J/kg followed by 2J/kg. If an IV is
accessible, a dose of adenosine can be given while setting up for the cardioversion.
30) Vaginal bleeding in a 3-day-old female is:
A. Is always indicative of child abuse
B. May be due to withdrawal of maternal hormones
C. Is suspicious for gonorrhea
D. Is most commonly due to a vaginal foreign body-such as baby wipes
Ans:- B
31) Cyanosis in the first week of life can be caused by:-
A. Tetralogy of Fallot
B. Pulmonary stenosis
C. Eisenmenger syndrome
D. TAPVD
E. Ebistein's anomaly
Ans:-ABDE
32) A 3-week old male presenting to the emergency department with vomiting and
altered mental status and acidosis. What additional laboratory test should be
included in your
evaluation?
A. Ammonia level
B. Cortisol level
C. Serum acetone
D. Thyroid function test
Ans:- A
Suspect an inborn error of metabolism in patients who have an altered level of
consciousness.These patients may or may not be acidotic depending on the type of inborn
error that is present.Patients with a urea cycle defect typically have a normal blood gas but
an elevated ammonia level. Patients with organic acidemias will be acidotic but may or
may not have an elevated ammonia level. Patients with galactosemia will have a normal
blood gas and ammonia level but will have reducing substances in the urine.
33)Which of the following are causes of shock in the newborn?
A. Infection
B. Inborn errors of metabolism
C. Child abuse
D. Thyrotoxicosis
E. All of the above
Ans:- E
34) In neonatal RDS (respiratory distress syndrome):
A. Surfactant is useful in the treatment.
B. It is rare in infants below 28 weeks gestation.
C. Antenatal steroids are beneficial.
D. Maternal opiate abuse increases the risk.
E. Maternal diabetes increases the risk.
Ans:- ACE
35) The risk of neonatal jaundice is increase by:
A. Prematurity.
B. Trisomy 21.
C. Elective caesarean section.
D. Congenital hypothyroidism.
E. Cephalahaematoma.
Ans:-ABDE
36) At birth the blood volume is approximately:-
A. 65 ml/kg body weight
B. 85 ml/kg body weight
C. 110 ml/kg body weight
D. 125 ml/kg body weight
E. 150 ml/kg body weight
Ans:- B
37) Newborn infants commonly have:-
A. Papulovesicles over the trunk.
B. Posterior fusion of the labia minora.
C. An adherent foreskin
D. Breast enlargement
E. Shallow sacral dimple
Ans:-ACDE
38) The following conditions signify disease in the newborn;-
A. Peeling of the skin of the hands and feet
B. Blanched on one side of the body and pink on the opposite side
C. Pinhead lesion on the nose ( milia)
D. Peripheral cyanosis
E. Oedema of one arm
Ans:- All false
39) Apreviously healthy full term infant have several episode of duskiness and
feeding difficulties during the second day of life . She is noted to have increasing
jaundice, which of the following tests will be the least helpful in making diagnosis:-
A. CSF
B. Urine C/ S
C. Total bilirubin
D. Endotracheal aspirate C/S
E. Venous blood
Ans:- D
40) Difference between infant born to heroin-abusing mothers and infant born to
phenobarbitone-abusing mothers is that infant in the later group:-
A. Don't have withdrawal symptoms
B. Have withdrawal symptoms appearing earlier than heroin withdrawal
C. Don't develop tremor
D. Have high incidence of jaundice
E. Are usually term and full size
Ans:- E
41)Which of the following organisms is the most frequent causes of neonatal
meningitis:-
A. GBS
B. E . coli
C. L . monocytogenes
D. H .influenza
E. S. pneumoniae
Ans:- A
42) One should be concerned about term infant who has not passed meconium stool:-
A. During the process of birth
B. Within few min of birth
C. By 1-2 hour of life
D. By 6-12 hour of life
E. By 24 hour of life
Ans:- E
43) The initial lesion of incontinentia pigmenti are:-
A. Deeply pigmented
B. Scaly
C. Waxy papules
D. Inflammatory bullae
E. Small vesicles
Ans:- D
44) Factor that appear to lower threshold for neurologic damage and kernicterus
from unconjugated hyperbilirubinemia include all of the following except:-
A. Acidosis
B. Asphyxia
C. Sepsis
D. Postmaturity
E. Hypothermia
Ans:- D
45) Infant born to diabetic mothers are at risk of all of the following except:-
A. Polycythemia
B. Hyperglycemic dehydration
C. Hypocalcemia
D. Congenital malformation
E. Cardiomyopathy
Ans:- B
46) The Apt test is used for what purpose:-
A. Crude test for carbon monoxide poisoning
B. Semiquantitative test for lead poisoning
C. Qualitative test for fetal hemoglobin
D. Screening test for S hemoglobin
E. Test for blood viscosity
Ans:- C
47) Birth injury account for the majority of the following conditions detected in early
infancy:-
A. Intraventricular hemorrhage
B. Cephalohematoma
C. Hydrocephalus
D. Facial nerve palsy
E. Pneumothorax
Ans:-BD
48) In birth trauma:-
A. Erb's palsy involve C6,C7 & C8
B. Klumpke's palsy involve C7,C8 &T1
C. Facial nerve palsy lead to persistently closed eye
D. Sciatic nerve involvement is common
E. Cephalohematoma is characteristically present at birth
Ans:- B
In Erb-Duchenne paralysis, the injury is limited to the 5th and 6th cervical nerves. The
infant loses the power to abduct the arm from the shoulder, rotate the arm externally, and
supinate the forearm. The characteristic position consists of adduction and internal rotation
of the arm with pronation of the forearm. Klumpke paralysis is a rare form of brachial
palsy; injury to the 7thand 8th cervical nerves and the 1st thoracic nerve produces a
paralyzed hand and ipsilateral ptosis,anhidrosis and miosis (Horner syndrome) if the
sympathetic fibers of the 1st thoracic root are also injured (which reflects damage to the
stellate ganglion adjacent to T1.)
49) Birth injury:-
A. Paralysis of the upper arm has better prognosis than paralysis of the lower
B. In nerve injury, neuroplasty is advised at the end of first year of life
C. In phrenic nerve paralysis, spontaneous recovery is expected
D. Facial nerve paralysis will result from the nuclear agenesis of the facial nerve
Ans:-AC
If the paralysis persists without improvement for 3–6 mo, neuroplasty, neurolysis, end-to-
end anastomosis, and nerve grafting offer hope for partial recovery. function usually
returns in a few months. Total disruption of nerves (neurotmesis) or root avulsion is the
most severe, especially if it involves C5–T1; microsurgical repair may be indicated.
Fortunately, most (75%) injuries are at the root level C5–C6, involve neurapraxia and
axonotmesis, and should heal spontaneously.Botulism toxin may be used to treat biceps-
triceps co-contractions.
50) The following are useful in the assessment of gestational age in preterm:-
A. Presence of palmar creases
B. Breast size
C. Sacral edema
D. The scarf sign
E. Muscle ton
Ans:-BDE
51) Cephalohematoma:-
A. Must be differentiated from subperiosteal hemorrhage
B. Is usually visible at birth
C. May calcify
D. May be associated with underling fracture of the skull
E. Should be managed surgically
Ans:-CD
52) Caput succedaneum is characterized by all of the following except:-
A. Diffuse edematous swelling of the soft tissues of the scalp involving the
portion presenting during vertex delivery
B. It may extend across the midline
C. It may extend across the suture lines
D. Edema usually disappear within 2-3 months
E. The scalp overlying the area may show mild bruising
Ans:- D
53) In newborn with oral moniliasis the most common primary source of infection
A. Maternal source ( vaginal )
B. Contaminated fomites
C. Following use of AgNO2
D. Contact by hospital carriers
E. Systemic antibiotic therapy
Ans:- A
54)Meconium impaction is associated with:-
A. Cretinism B. Cystic fibrosis
C. Thrush D. HMD
E. Trisomy 21 syndromes
Ans;- B
55) Premature infant is delivered precipitiously and appear asphyxiated . The infant
is cyanotic, there are no respiratory efforts,and the heart rate is 80 / min .The infant
is meconium stained and thick particulate meconium is noted in the amniotic fluid
and in infant mouth. At this point you should:-
A. Pass an umbilical artery catheter to measure the PH & PO2
B. Start bag-mask ventilation with 100% oxygen
C. Suction the oropharynx & trachea with ETT to remove the meconium
D. Intubate the trachea & begin the ventilation with 100 % oxygen
E. Establish monitoring with ECG & pulse oximeter
Ans:- C

56) Hypoglycemia has been observed in all of the following except;-


A. With LBW and RDS
B. With anoxic injury
C. With hypothermia
D. Who are SGA
E. With high PO2
Ans:- E
57) All of the following are usually associated with cretinism except:-
A. Constipation
B. Prolonged jaundice
C. Lethargy
D. Tetany
E. Hypotonia
Ans:- D
58) All of the following are characteristic of single umbilical artery except:-
A. Presence in about 5 of 1000 birth
B. About 1/3 of such infants have congenital abnormalities
C. 21 trisomy is frequently found
D. Among twin ,the rate of occurrence is 35 / 1000
E. The associated congenital abnormalities may involve the genitourinary tract
Ans:- C
59) Two minutes after normal term delivary:-
A. The ductus venosus will be closed
B. The pulmonary arterial pressure will have fallen
C. The pressure in the left atrium will have fallen
D. The arterial oxygen tension will have risen
E. Regular breathing will have begun
Ans :-ABDE
60) Established neonatal resuscitation procedures include:-
A. Directing cold stream of oxygen at the nose
B. Administration of drugs with respiratory stimulant properties
C. Oropharyngeal suction
D. Bag and face mask ventilation
E. Prompt cooling
Ans:-ACD
61) The Apgar score :-
A. At 1 min is reliable measure of asphyxia
B. At 1 min is reliable measure of respiratory failure
C. At 10 min is strongly correlated with later neurological deficit
D. Includes the infant response to pharyngeal suction catheter
E. Isn't application after 10 min of age
Ans:- BCD
62) Criteria used in the Apgar score include :-
A. Core temperature
B. Heart rate
C. Respiratory rate
D. Skin thickness
E. Muscle tone
Ans :-BE
63) Pink newborn with HR of 88/min is actively gasping , he has good muscle tone
and
respond to nasal catheter stimulation with facial grimace, the apgar score is:-
A. 5
B. 6
C. 7
D. 8
E. 9
Ans:- D
64)Mongolian spots are characterized by all of the following except:-
A. They are permanent
B. They are usually of a slate blue pigmentation
C. They are generally observed over the buttocks
D. The area of pigmentation is well demarcated
E. They aren't associated with trisomy syndromes
Ans:-A
65) All of the following physical signs may be useful in estimating the gestational age
at birth except:-
A. There are only one or two transverse skin creases on the sole of the foot until
36 week of gestation
B. The breast nodule is usually not palpable at 33 or 34 weeks
C. The breast nodule is usually 4-10 mm in term infant
D. The testes are descending and rugae cover the entire scrotal surface by 34 weeks
E. The texture of scalp hair
Ans:- D
The testes are usually not completely descended until after 36 weeks & scrotal rugae are
few and limited the anterior and inferior aspect of relatively small scrotum . By 34 weeks
the areola become raised and between 36-37 weeks the breast bud is 1 – 2 mm reaching
size of 4-10 mm at term. The transversr foot creases develop at 31 – 32 weeks .By 36
weeks creases cover the anterior two-third of the sole .
66) Newborn infant who remain centrally cyanosed after intubation and IPPV may
have:-
A. Diaphragmatic hernia
B. Choanal atresia
C. Tension pneumothorax
D. Drug induced respiratory center impairment
E. Profound anaemia
Ans:-AC
67) The following statement about pulmonary hypertension are true :-
A. It recognized complication of group B streptococcal sepsis
B. Hyperventilation is an effective treatment
C. Tolazoline is potent pulmonary vasoconstriction
D. Radial arterial PaO2 is lower than umbilical artery PaO2
E. Birth asphyxia is a risk factor
Ans:-ABE
68) Concerning air leak syndromes in the newborn:-
A. Underwater seal drain is only required if the pneumothorax is under tension
B. In term baby with small pneumothorax giving oxygen at high concentration
can worsen it
C. Increasing the I; E ratio in ventilated baby decrease the risk of pneumothorax
D. Pneumomediastenum is usually fatal
E. They can be asymptomatic
Ans;- E
69) Recognized problem of infants born at term SGA include:-
A. Hypothermia
B. Sepsis
C. Polycythemia
D. Hypoglycemia
E. Retinopathy of prematurity
Ans:-ABCD
70) Complication of steroid therapy in the newborn include:-
A. Leucopenia
B. Hypoglycemia
C. Cataract
D. Sepsis
E. Gastric perforation
Ans:-CDE
71) Concerning NEC :-
A. Exchange transfusion is predisposing factor
B. Clostridium welchii is implicated in the pathogenesis
C. It is most common in infants born less than 1500 gm
D. Oral antibiotics are useful
E. Complications include short bowel syndrome
Ans:-ACE
72) The following congenital condition require immediate ( within first week)
treatment after birth:-
A. TEF
B. Cleft lip
C. Spina bifida
D. Exomphalos
E. Hydrcephalus
Ans:-ACD
73) Preterm infant at increased risk from :-
A. Conjugated hyperbilirubinemia
B. Meconium aspiration
C. Periventricular leucomalacia
D. Necrotizing enterocloitis
E. Child abuse
Ans:- CDE
74) Peri- or intraventricular cerebral hemorrhage
A. Occur in less than 10% of VLBW infant
B. Arise most commonly in the first 72 hour after delivary
C. Is direct result of impaired vitamin K supply
D. Is the single most common cause of congenital cerebral palsy
E. May result in rapidly evolving hydrocephalus
Ans:-BE
75) Feature typical of physiological jaundice include:-
A. Recognizable jaundice in the first 48 hours
B. Peak plasma bilirubin at 4-5 days
C. Persistent beyond first week
D. Irritability
E. Pale stool
Ans:- B
76) Jaundice on day 1 is often caused by ;-
A. Metabolic disorder
B. TORCH infection
C. Gastrointestinal obstruction
D. Hemolysis
E. Physiological factor
Ans:-BD
77) Persistant, conjugated hyperbilirubinaemia may be caused by:
A. Alpha-1-antitrypsin deficiency.
B. Hypothyroidism.
C. Haemolytic disease.
D. Cytomegalovirus infection.
E. Cystic fibrosis.
Ans:-ABDE
Explanation:- all cases need further investigation.
Causes of conjugated hyperbilirubinaemia in a neonte are:
1)-Intrahepatic cholestasis -Infections.-Congenital infections (STORCH-syphlis,
toxoplasma, rubella,cytomegalovirus, hepatitis, herpes virus infection)
-Acquired infections- septicaemia, UTI.-Metabolic disorders –cystic fibrosis, alpha-1-
antitrypsin deficiency, galactosaemia,fructosaemia, lysosomal storage disorders,
peroxisomal disorders.-Endocrine disorders – hypothyroidism, hypopituitarism,
hypoadrenalism.-Anatomical disorders – intrahepatic:bile duct hypoplasia.
-Miscellaneous – idiopathic neonatal hepatitis, chromosomal abnormalities, trisomy 21, 18
and 13.
2)-Extrahepatic cholestasis:- biliary atresia, choledochal cyst, spontaneous bile duct
perforation, inspissated bile syndrome.
Unconjugated neonatal jaundice is normally physiological and resolves spontaneously.

78) All of the following are characteristic of jaundice associated with breast feeding
except :-
A. Significant elevation of unconjugated bilirubin
B. Rapid fall in serum bilirubin after discontinuation of nursing
C. Nursing can be resumed after several days without return of hyperbilirbinemia
D. Significant elevation of conjugated bilirubin
E. Kernicterus has never been reported to occur as result of breast milk jaundice alone
Ans :- D
79) Persistent jaundice during the first month of life may associated with all of the
following except:-
A. Cytomegalic inclusion disease
B. Congenital atresia of the bile duct
C. Galactosemia
D. Rh incompatibility
E. Penicillin treatment
Ans:- E
80) A 26-week gestation preterm infant is now 6 weeks old and weighs 1,250 g. He is
receiving full-volume enteral nutrition. The only significant finding on physical
examination is pallor. He has anemia (hematocrit of 28% [0.28]; reticulocyte count of
8% [0.08]) and receives iron supplementation. He is receiving a formula that is high
in polyunsaturated fatty acids.Of the following, the MOST correct statement about
his need for vitamin E is that it
A. does not need to be supplemented in infancy
B. has no effect on anemia
C. needs to be supplemented now
D. will be needed when the infant is 3 months old
E. will prevent anemia
Preferred Response: C
Historically, inadequate vitamin E, a high level of polyunsaturated fatty acids (PUFAs) in
infant formula, and exposure to the oxidizing effects of iron supplementation contributed
to a hemolytic anemia seen in preterm infants. In the United States and many developed
countries, infant formulas now provide an adequate vitamin E:PUFA ratio to eliminate this
risk. However, preterm infants continue to have low vitamin E levels due to limited stores,
especially when the birth is extremely premature, and limited enteral feedings early in
their postnatal neonatal intensive care unit course.The neonate who has anemia and is
receiving iron supplementation, such as the infant described in the vignette, requires the
antioxidant effect of vitamin E to reduce red blood cell hemolysis. A
total daily requirement of 10 to 25 IU of vitamin E meets the infant’s needs, only 50% of
which is provided by dietary formula. Of note, human milk also is an incomplete source of
vitamin E.
81) Amniocentesis is useful in establishing the prenatal diagnosis of:-
A. Down syndrome
B. Meningomyelocele
C. Erythroblastosis fetalis
D. Achondroplasia
Ans:-ABC
82) USS can be used during pregnancy to:-
A. Determine crown rump length
B. Determine fetal sex
C. Determine biparietal diameter
D. Accurately determine fetal weight
Ans ;-AC
83) Gestation that produce multiple births:-
A. Are classified as high risk
B. Are always delivered by cesarean section
C. Can produce infant with discordance in body size at birth
D. Aren't associated with the premature onset of labour
Ans:-AC
84) For newborn infant, the least important factor to consider in assessing the risk of
kernicterus is:-
A. Breast feeding
B. Acidosis
C. Sepsis
D. Albumin level
E. Moxalactam, cephalosporin type antibiotic
Ans:- A
85) The following maternal conditions are known to cause adverse effects on the
neonate:
A. Diabetes insipidus.
B. Toxoplasmosis.
C. Chronic myeloid leukaemia.
D. SLE.
E. Hyperthyroidism.
Ans:-BCDE
Explanation:- Maternal SLE is a risk factor for neonatal lupus syndrome. The neonate
presents with clinical features of SLE due to transplacental passage of maternal Ab. The
skin is frequently involved with malar rashes and there can be haematological and cardiac
abnormalities. The most frequent heart abnormality is congenital heart block – 90% of
mothers whose infants have congenital heart block are anti-Ro(SSA)Ab positive.
Toxoplasmosis leads to congenital infection by transplacental transmission. The rate of
transmission is +/- 60% of third trimester infections and 20%-30% during the first two
trimesters.The ewborn presents with the classic triad of hydrocephalus, chorioretinitis, and
cerebral calcification.Maternal chronic myeloid leukaemia can have adverse effect on
pregnancy – fetal/neonatal mortality is 16-38%. Pathology can be secondary to placental
leukaemic infiltrates, anaemia and infectious complications. Splenomegaly can restrict
intrauterine growth and lead to premature delivery.If hyperthyroidism is due to Grave’s
disease or Hashimoto thyroiditis, the neonate may present with thyrotoxicosis due to the
transplacental passage of TSI. These symptoms are frequentely short-lived i.e as long as
the circulating antibodies persist in the baby’s circulation.
86) Each of the following statement about GBS infection in the newborn is true
except:-
A. Incidence is correlates inversely with the presence of maternal antibodies
B. The exotoxin has powerful pulmonary vasoconstrictive effects
C. The disease has two distinct pattern; early & late onset
D. Both the incidence & severity of the disease can be lessened by pretreatment of the
mother who is colonized
E. The risk of invasive disease isn't related to the amount of inoculum received by the
infant
Ans:- E
87) Newborn infant may present with bile stained vomiting and abdominal distension
as the result of
A. Oesophageal atresia
B. Duodenal atresia
C. Birth asphyxia
D. Electrolyte disturbance
E. Cystic fibrosis
Ans:-BCDE
88) The following feature are consistent with newborn infant having oesophageal
atresia and tracheo-oesophageal fistula:-
A. Maternal polyhydramniose
B. Passage of wide bore orogastric catheter into the stomach
C. Plain x-ray evidence of air in the stomach and small bowel
D. Plain x-ray evidence of hemivertebra
E. Excessive mucus in the nostrils or mouth.
Ans:-ACDE
89) Vitamin K
A. Is an essential cofactor for the synthesis of coagulation factor II , VII, IC, C
B. Is readily transported across the placenta
C. Is present in the breast milk at higher concentration than in cow milk.
D. Given in single oral dose after delivary effectively prevent s hemorrhagic disease
E. Related hemorrhage in the newborn is commoner when mother have taken
anticonvulsant during pregnancy
Ans:-AE
90) The following are recognizable causes of neonatal convulsion:-
A. Birth asphyxia
B. Hypoglycemia
C. Hypothermia
D. Opiate withdrawal
E. Hypernatremia
Ans:-ABDE
91) With regard to Apgar score:-
A. 2 points given to pulse of 88/min
B. 1 point is given for irregular gasps
C. An initial satisfactory score gurarantees an eventual perinatal period
D. Score of 2 at 10 min carries worse prognosis than score of 2 at 5 min
E. The lowest possible score is 1
Ans:-BD
92) Intrauterine posture is commonly responsible for:-
A. Congenital dislocation of the hip
B. Plagiocephaly
C. Sternomastoid shortening
D. Syndactyly
E. Mandibular asymmetry
Ans:-ABCE
93) Established neonatal screening tests include:-
A. Umbilical cord blood analysis to detect phenylketonuria
B. Umbilical cord blood analysis to detect galactose
C. Umbilical cord blood analysis to detect sickle cell disease
D. Capillary blood analysis at 6-8 days to detect elevated TSH
E. Capillary blood analysis at 6-8 days to detect elevated immunoreactive trypsin
Ans:-CDE
94)Maternal condition that may have effect in the neonatal period include:-
A. ITP
B. Multiple sclerosis
C. DM
D. Varicella zoster
E. Bornholm disease
Ans:-ACDE
95) Neonatal polycythemia :-
A. Occur in small for date infant as response to placental insufficiency
B. Has an increased incidence if maternal diabetes is poorly controlled
C. Carry an increased risk of cerebral venous sinus thrombosis
D. Is recognizable feature of congenital hypothyroidism
E. May occur as result of feto-maternal transfusion
Ans:-ABCD
96) Polycythemia on the newborn is characterized by all of the following except:-
A. Increased the incidence in IDM
B. Placental transfusion decreased by late clamping of the umbilical cord
C. Clinical presentation that includes jitteriness & poor feeding
D. Complication including heart failure & NEC
E. Elevated viscosity of whole blood
Ans;- B
97) Causes of neonatal polycythemia include:-
A. Congenital rubella infection
B. Pre-eclampsia
C. Maternal diabtus mellitus
D. Delayed clamping of umbilical cord
E. Congenital adrenal hyperplasia
Ans:- BCDE
98)Meconium aspiration pneumonia:-
A. Occur with equal frequency in term and preterm infant
B. In infant required ventilation for this condition, combination of high PEEP and
rapid rate is advisable
C. Has high risk of developing even if liquor is only thinly stained
D. High dose steroid are the mainstay of treatment
E. Antibiotic treatment is an important part of treatment
Ans :- all False
99) The following statement are true regarding hemolytic disease of newborn ( HDN
):_
A. Hemolytic disease should be suspected if jaundice is noted in the first 24 hour of
life
B. HDN may occur if mother is group A +ve and the baby is group O+ve
C. If due to Rh –incompatibility the severity of the hemolysis typically increase with
each affected pregnancy
D. HDN due to ABO incompatibility can be detected at 36 week gestation by
amniocentesis
E. As long as the level of unconjugated bilirubin never rises above20 mg/dl there is no
dangerous of kernicterus
Ans:-AC
100) Full term infant has tachypnea with grunting , chest X ray show well expanded
lung with streaky shadows radiating from the bilateral hilar region , the most likely
diagnosis is :-
A. TTN
B. Congenital pneumonia
C. BPD
D. Meconium aspiration
E. Aspiration pneumonia
Ans:- A
101) Condition associated with polyhydramniose include:-
A. Oesophageal atresia
B. Down's syndrome
C. Renal agenesis
D. Cord around the neck
E. CNS malformation
Ans:- ABE
102) The following are causes of polyhydramnios:
A. Maternal diabetes mellitus.
B. Potter syndrome.
C. Anencephaly.
D. Oesophageal atresia.
E. Polycystic kidneys.
Ans:-ACD
103) The following are causes of polyhydramniose:-
A. Pottrer syndrome
B. TEF and oesophageal atresia
C. Rh – incompatibility
D. Anencephaly
E. Maternal diabetus
Ans:-BDE
104) For necrotizing enterocloitis:-
A. Term baby are particularly at risk
B. Perinatal asphyxia is risk factor
C. Expressed breast milk from milk bank confer no protection
D. Failure of temperature control is late sign
E. High platelet count is frequently seen
Ans:- B
105) You diagnose necrotizing enterocolitis in a preterm neonate who has abdominal
distention and blood in the stool. You decide that this infant should be placed on a
14-day regimen of parenteral nutrition.Of the following, the micronutrient for which
weekly monitoring is MOST recommended during this infant’s parenteral nutrition
therapy is
A. copper
B. iron
C. phosphorus
D. selenium
E. zinc
Preferred Response: C
Parenteral nutrition (PN), the intravenous administration of carbohydrates, lipids, amino
acids,and micronutrients, is an important component in the management of a variety of
chronic disorders, including surgical conditions (eg, omphalocele, gastroschisis,
diaphragmatic hernia,short bowel syndrome), inflammatory conditions (eg, Crohn disease,
ulcerative colitis,pseudomembranous colitis, pancreatitis, graft versus host disease),
hypermetabolic states (eg,burns, trauma), and intestinal motility disorders (eg, pseudo-
obstruction). PN is especially important in the support of very low-birthweight infants,
who frequently have increased caloric requirements, decreased oral intake, and immature
intestinal motility.In addition to glucose, amino acids, sodium, potassium, and chloride,
PN provides additional mineral supplements, including calcium, phosphorus, magnesium,
zinc, copper, selenium,chromium, manganese, molybdenum, and iodide. In preterm
infants, the administration of both calcium and phosphorus is important to prevent
metabolic bone disease.However, the amount of calcium and phosphorus that can be
administered in PN must be limited because of the risk of precipitation in the PN and
formation of calcium-phosphorus complexes that could embolize. It generally is thought
that the chance of precipitation is high if the product of the concentrations (in mmol/L) of
the calcium and phosphorus in PN is greater than 40.Because hypophosphatemia can have
significant consequences (including impaired cardiac function, muscle weakness, and
hemolysis), it is recommended that serum phosphorus concentrations be assessed at least
once weekly for patients receiving PN. The serum concentrations of copper, iron,
selenium, and zinc, which are trace elements, require only periodic assessment (eg, once a
month or less).
106) An 1,800-g preterm infant is recovering from surgery for gastroschisis. Of the
following, the MOST correct statement regarding this infant’s immediate daily
nutritional requirements is that he
A. can meet energy needs from fat through enteral feeding
B. can meet protein needs for growth through enteral feeding
C. requires lysine supplementation to promote protein synthesis
D. requires more protein per kilogram than a term infant
E. requires 2.5 g/kg per day of protein
Preferred Response: D
Milk remains the principal source of nutrition for infants, who consume 120 to 150 mL/kg
per day in the newborn period.Human milk has 67 kcal/100 mL, and most term infant
formulas have a similar composition. The energy derived from ingesting milk comes
primarily from fat calories (3.8 g/100 mL = 34kcal/100 mL), followed by carbohydrates
(lactose, 7 g/100 mL = 28 kcal/100 mL) and minimally from protein (1.3 g/100 mL = 5
kcal/100 mL). Conditions such as prematurity, lung disease, or surgery may increase both
caloric and specific nutrient requirements for newborns.The newborn described in the
vignette can only take limited enteral nutrition and has an increased need for both protein
and calories to facilitate healing and growth. The normal 2 to 3 g/kg per day of protein
ingested by the term infant who is either formula- or breastfed is inadequate for this low-
birthweight, preterm newborn. He will grow and heal best receiving 3.5 to 4.0 g/kg per
day of protein. The potential benefits of single amino-acid supplementation, such
as lysine, remain unknown.
107) Hypoxic-ischemic encephalopathy all are correct except:-
A. Result from excessive use of oxytocin
B. IUGR is be first indication of fetal hyoxia
C. Associated with increased beat to beat variability
D. Prognosis depend on gestational age
E. Persistent of abnormal neurological sign at two week indicate poor prognosis
Ans:- C
108) polyhydramniose:-
A. Is defined as an amniotic fluid volume of more than 500 ml
B. Occur with increased frequency in diabetic pregnancy
C. Is associated with renal agenesis
D. Is associated with tracheo-oesophageal fistula
E. Is associated with increased risk of premature labor
Ans :-BDE
109) Polyhydramnios is associated with all of the following condition except:-
A. Amniotic fluid volume between 500-2000 ml
B. Maternal diabetes
C. Twins
D. Erythroblastosis fetalis
E. Down syndrome
Ans:- A
110) Condition associated with oligohydramnios include:-
A. Infantile polycystic kidney
B. Congenital heart disease
C. IUGR
D. High intestinal obstruction
E. Posterior urethral valve
Ans:- ACE
111) Regarding hyaline membrane disease ( HMD)
A. Can occur in infants of diabetic mother of 37-40 week gestation
B. Infant born to mothers who are heroin addicts are at increased risk of HMD
C. Light-for –date infant of 33 weeks gestation has greater risk of developing
HMD than 33 week infant of appropriate weight
D. Administration of artificial surfactant is curative
E. Chest x-ray finding are markedly different between cases of HMD and Group
B streptococcal pneumonia
Ans:- A
112) Hyaline membrane disease:-
A. Is more common in babies of diabetic mother
B. Is due to surfactant deficiency
C. Is not seen in term babies
D. Occur most commonly at 12 hour post-delivary
E. Always require ventilation
Ans:- AB
113) The following symptoms in an infants in the first month of life should alert one
to possibility of hypothyroidism:
A. Prolonged jaundice
B. Vomiting
C. Diarrhea
D. Hoarse cry
E. Voracious appetite
Ans:- AD

114) The following malformation can occur in children born to mother who have
IDDM:
A. Cleft lip/ palate
B. Caudal regression syndrome
C. Femoral hypoplasia
D. Holopresencaphaly
E. Polydactyly
Ans:- BCDE
115) With regard to RDS of the newborn
A. Meconium aspiration pneumonia is especially likely to be the cause if the infant is
preterm
B. HMD is unlikely to be the cause if the prepartum lecithin sphingomylin (L/S) ratio
in amniotic fluid is > 2;1
C. Patchy opacities on chest X-ray are evidence in favour of diagnosis HMD
D. In the artificial ventilation of infant with HMD , inflation pressure must never
exceed 30 cm of water
E. CPAP is an effective in treatment for Pneumothorax
Ans : B
116) In RDS due to meconium aspiration
A. Antibiotic therapy is of crucial importance
B. Steroid therapy improve the prognosis
C. There is high risk of Pneumothorax
D. Chest X-ray typically shows ground glass opacity
E. The infant may also suffer from cerebral oedema
Ans:-C
117) Recognized association of small-for-date babies include
A. Maternal smoking in pregnancy
B. Hemolytic disease due to ABO incompatibility
C. Pre-eclamptic toxaemia
D. Congenital rubella infection
E. Fetal alcohol syndrome
Ans:- ACDE
118) Prolonged neonatal jaundice is recognized feature in infants with
A. CMV infection
B. Congenital hypothyroidism
C. Untreated urinary tract infection
D. Tracheo-oesophageal fistula
E. Galactosemia
Ans:- ABCE
119) In baby of 32 week gestation who has tachypnea and sternal recession at 4 hour
of age:
A. The ductus arteriosus is likely to be patent
B. Meconium aspiration pneumonia is likely to be diagnosis
C. The L/S ratio is likely to be low
D. Fluid level on chest X-ray suggest pneumonia due to Group B streptococcus
E. The presence of bowel shadows on the left side of the chest on X-ray is
diagnostic of Tracheo-oesophageal fistula
Ans :-AC
120) The following condition characteristically cause jaundice within the first 24
hour of life:-
A. G6PD deficiency
B. Congenital hypothyroidism
C. Sever congenital CMV infection
D. Choledochal cyst
E. Primary tyrosinaemia
Ans:- AC
121) Hypothermia can cause the following in LBW infant:-
A. Decrease synthesis of surfactant
B. Hypernatremia
C. Hypoglycemia
D. Increased oxygen consumption
E. Hypercalcemia
Ans :- ACD
122) The following substance are freely transmitted across the placenta:-
A. Carbimazole
B. Diazepam
C. Warfarin
D. Pethidine
E. IgM antibodies
Ans:-ABCD
123) In congenital diaphragmatic hernia:-
A. Hernia most commonly occur on the left
B. Pulmonary hypoplasia is the major cause of death
C. Persistent fetal circulation occurs uncommonly
D. Associated congenital anomalies are common
E. Most present between 12-24 hours of life
Ans:- AB
The incidence of CDH is between 1/2,000 and 1/5,000 live births, with females affected
twice as often as males. Defects are more common on the left (85%) and are occasionally
(<5%) bilateral.Pulmonary hypoplasia and malrotation of the intestine are part of the
lesion, not associated anomalies. Most cases of CDH are sporadic, but familial cases have
been reported. Associated anomalies have been reported in up to 30% of cases; these
include central nervous system lesions,esophageal atresia, omphalocele, and
cardiovascular lesions. CDH is recognized as part of several chromosomal syndromes:
Trisomy 21, Trisomy 13, Trisomy 18, Fryn, Brachmann–de Lange,
Pallister-Killian, and Turner. Relative predictors of a poor prognosis include an associated
major anomaly, symptoms before 24 hr of age, severe pulmonary hypoplasia, herniation to
the contralateral lung, and the need for ECMO.
124) In the normal newborn infant in the first 24 hour of life:-
A. Many significant heart defect may be clinically undetectable
B. Normal arterial PaO2 help the ductus to close
C. Hypoxia causes pulmonary artery vasoconstriction
D. All children with murmur heard in the first 24 hour should be followed up for
at least 6 months
E. The systolic blood pressure is between 40-80 mmHg
Ans:-ABCE
125) A light –for-date full term baby is at particular risk from the following
condition:-
A. HMD B. Physiological jaundice
C. Milk aspiration D. Hypoglycemia
E. Apnoeic attacke
Ans:- D
126) Recurrent apnoea of prematurity:-
A. Characteristically develop within the first 24 hour of life
B. Is more likely to occur in infant of < 32 week gestation
C. Usually respond to naloxone
D. Should be treated with 100% oxygen during attack
E. May be accentuated by the presence of nasogastric tube
Ans:- BE
127) In the management of HMD:
A. Tolazoline may lead to systemic hypertension
B. Early use of CPAP may reduce the need for subsequent ventilatory support
C. Antibiotic improve ventilation- perfusion ratio
D. The illness may be expected to increase in the severity for the first 5 days
E. Corticosteroid used postnatally have beneficial effect on the course of the
disease
Ans:- B
128) Characteristic finding in preterm baby include:-
A. Chin reaching only to the tip of shoulder
B. Full wrist flexion
C. Flat on couch when lying prone
D. Incomplete ankle dorsiflexion
E. Incomplete knee extension with hip fully flexed
Ans:- C
129) Drug effects on the fetus:-
A. Isotretinion can lead to CNS defects mainly if given in the third trimester
B. Phenytoin can lead to cleft lip, finger and toe abnormalities mainly if given in
the second trimester
C. Carbimazole cause goiter, mainly if given in the third trimester
D. Warfarine can lead to neonatal hemorrhage mainly if given in the first
trimester
E. Valproate can lead to neural tube defects mainly if given in the third trimester
Ans:- C

130) The external criteria for the Dubowitz score for gestational age include:-
A. Breast size
B. Skin texture
C. External genitalia
D. Langue hair
E. Nose firmness
Ans:-ABCD
131) Problem of babies born to mothers with poorly controlled diabetus mellitus:
A. Hypermagnesemia
B. Polycythemia
C. Hypocalcemia
D. Sacral agenesis
E. Respiratory distress
Ans:-BCDE
132) The following are recognised associations of maternal diabetes:
A. Sacral agenesis
B. Intrauterine growth retardation
C. Macrosomia
D. Hyaline membrane disease
E. Hypertrophic cardiomyopathy
Ans:-ABCDE
Comments:
Overall, malformations occur in 6%, with an increased incidence of cardiac
malformations, sacral agenesis, hyperplastic left colon. Intrauterine growth retardation is 3
times as common due to small vessel disease in the mother. More common is
macrosomia, and this is related to the degree of maternal hyperglycaemia. The glucose
crosses the placenta while the insulin does not, so the fetus increases its production of
insulin. This results in increased cell number and size. 25% of IDM are greater than 4kg
compared with only 8% of non-diabetics. This gives problems with delivery such as CPD,
shoulder dystosia resulting in an increased incidence of birth asphyxia and trauma.
In the neonatal period, hypoglycaemia, respiratory distress and reversible hypotrophic
cardiomyopathy and polychthaemia are all more common. Gestational diabetes is when
carbohydrate intolerance occurs only during pregnancy. It is commonest in obese women
and those from Afro-Caribbean and Asian ethnic groups. In these women there is no
increase in congenital malformations, thoug macrosomia and other complications remain
similar in frequency.
133) Characteristic finding in full term baby are:-
A. Full knee extension with hips fully flexed
B. Momentary neck extension when held sitting
C. Chin reach beyond tip of shoulder
D. Hip abducts and legs flat on couch when lying supine
E. Full ankle dorsiflexion
Ans:- BE
134) The external criteria for the Dubowitz score for gestational age include:-
A. Nipple formation
B. Ear form & firmness
C. Skin colour & opacity
D. Protruding tongue
E. Planter creases
Ans:- ABCE
135) In fetal circulation:-
A. 30% of the fetal cardiac output goes through the placenta
B. Oxygenated blood from the placenta passes through the ductus arteriosus toward
the right atrium
C. The oxygen saturation of blood in umbilical arteries is approximately 60%
D. Blood entering the heart from the inferior vena cava is diverted directly to the left
atrium via patent foramen ovale
E. There is one umbilical vein
Ans:- CDE
136) The following disease can now be diagnosed prenatally:-
A. Phenylketonuria
B. Homocystinuria
C. Sever combined immunodeficiency
D. Fanconi's anaemia
E. Lesh-Nyhan syndrome
Ans:- ABCDE
137) Resuscitation of neonate with coarctation of aorta may require the use of :-
A. Frusemide
B. Bicarbonate
C. Indomethacin
D. Dopamine
E. Prostaglandin E
Ans:- ABDE

138) Recognized causes of the floppy baby include:-


A. Trisomy 21
B. Zellweger syndrome
C. Becker muscular dystrophy
D. Spinal muscular atrophy
E. Hypothyroidism
Ans:- ABDE
139) Which of the following may cause apnoea in preterm infant
A. Hypocalcemia
B. Hypoglycemia
C. RSV infection
D. Caffeine
E. Intraventricular hemorrahage
Ans:- ABCE

140) Failure of resuscitation of newborn may be due to :-


A. PDA
B. VSD
C. Congenital diaphragmatic hernia
D. Pulmonary hypoplasia
E. HMD
Ans:-CD
141) Feature of IUGR include:-
A. Neutropenia
B. Hypoglycemia
C. Necrotizing Enterocolitis
D. Weight loss > 10% of birth weight in first week
E. Thermal instability
Ans:- ABCE
142) Newborn infant has breathing difficulty with central cyanosis .Nitrogen washout
test raised the PO2 to 15 kPa possible diagnosis include:-
A. Fallot's tetralogy
B. VSD
C. Pneumonia
D. Tricuspid atresia
E. HMD
Ans:- CE
143) Newborn infant has central cyanosis and fit with slow & shallow breathing ,
Nitogen
washout test produce slight raise in the PO2 , the most likely cause of cyanosis is:-
A. Methemoglobinemia
B. Cerebral disorder
C. Persistent fetal circulation
D. Congenital cyanotic heart disease
E. Lung disease
Ans:- B
144) The following disease can now be diagnosed prenatally:-
A. Hypercholesterolemia
B. Polycystic kidney disease
C. Cystinosis
D. Gaucher's disease
E. Beta thalassaemia
Ans:-ABCDE
145) Common causes of seizure in the neonatal period include:-
A. Intracranial hemorrhage
B. Electrolyte disturbance
C. Infection
D. Drug withdrawal
E. Febrile convulsion
Ans:- ABCD
146) Neonatal convulsion may be caused by :-
A. Hypomagnesaemia
B. Hyperkalamia
C. Pyridoxine dependency
D. Cephalohematoma
E. HSV infection
Ans:- ACE
147) The following disease can now be diagnosed prenatally:-
A. Sickle cell disease
B. Maple syrup urine disease
C. Retinoblastoma
D. Hypophosphaturia
E. Von-Willebrand disease
Ans:- ABCDE
148) Which of the following statement regarding surfactant therapy is correct :-
A. The incidence of Pneumothorax is reduced
B. The incidence of intraventricular hemorrhage is reduced
C. Early therapy ( within 4 hour) is more beneficial than later administration ( after
12 hour)
D. Prolonged courses of surfactant therapy confer advantages
E. The incidence of chronic lung disease is increased
Ans:- ABC
149) Which of the following statement regarding periventricular hemorrhage are
correct:
A. Most hemorrhage occur after the fourth day of life
B. Over 80% of hemorrhage progress to ventricular dilatation
C. Most cases of posthemorrhagic hydrocephalous are communicating
D. Early ventricular tapping improve the neurological outcome
E. Hemorrhage is usually caused by hypoglycemia
Ans:- C
150) Complication of phototherapy include:-
A. Diarrhea
B. Erythematosus rash
C. Hyponatremia
D. Skin discoloration
E. Peripheral desqumation
Ans:- ABD
151) Which of the following are characteristic haemodynamic changes encountered
at birth:-
A. Increase the pulmonary vascular pressure
B. Right to left flow through the ductus arteriosus
C. Closure of the ductus arteriosus
D. Increased right ventricular end diastolic pressure
E. Reversal of flow across the foramen ovale
Ans:- C

152) Feature suggestive of TEF with oesophageal atresia include:-


A. FTT
B. Recurrent pneumonia
C. Oligohydramniose
D. Large amount of mucus in the pharynx at delivary
E. Slow to establish feed
Ans:- D
153) The development of Pneumothorax is associated with :-
A. Surfactant therapy
B. Artificial paralysis
C. Patient triggered ventilation
D. Meconium aspiration
E. Pulmonary interstitial emphysema
Ans:- DE
154) Cyanotic congenital heart disease in the newborn include:-
A. TGA
B. Pulmonary stenosis
C. Fallot's tetrallogy
D. VSD
E. Tricuspid atresia
Ans:- AE
155) The following congenital disorder require immediate ( within first week)
treatment after
birth:-
A. Hemangioma
B. Imperforate anus
C. Choanal atresia
D. Hypospadius
E. Congenital diaphragmatic hernia
Ans:-BCE
156) Causes of neonatal hypoglycemia include;-
A. Erythroblastosis fetalis
B. Glycogen storage disease type I
C. Maternal treatment with sodium valproate
D. Galactosemia
E. Congenital adrenal hypoplasia
Ans:- ABDE
Plasma glucose level of <40 mg/dL, During gestation, glucose is freely transferred across
the placenta by the process of facilitated diffusion. However, after birth, the infant must
adjust to the sudden withdrawal of this transplacental supply. In all infants, there is a nadir
in blood sugar between 1 and 3 hours of life. During the first 12-24 hours of life, newborns
are at increased risk for hypoglycemia because gluconeogenesis and especially ketogenesis
are incompletely developed. These factors are accentuated in preterm infants, infants of
diabetic mothers, infants with erythroblastosis fetalis, asphyxiated infants, and infants who
are small or large for gestational age.
Transient hypoglycemia causes:- prematurity, hypothermia, Birth asphyxia, sepsis,
erythroblastosis fetalis, infants with Beckwith-Wiedemann syndrome, maternal diabetes,
maternal glucose infusion in labor, and intrauterine growth restriction (IUGR)and maternal
drugs( b sympathomimetic and chlorpropamide).
Persistent hypoglycemia may be due to: - inborn error of metabolism (Glycogen storage
disease type I , P & IV, maple syrup urine disease, Nesidioblastosis, galactosemia,and
mitochondrial fatty acid oxidation defects such as MCAD deficiency. Hormonal
deficiency suc as congenital hypopituitrism,congenital glucagons deficiency and cortisol
deficiency states including congenital adrenal hypoplasia.
Features on physical examination suggest the etiology of hypoglycemia:-
· Macrosomia: This occurs in infants of diabetic mothers, infants with severe congenital
hyperinsulinism, and infants with Beckwith-Wiedemann syndrome; recall that insulin is a
growth factor and that hyperinsulinism leads to macrosomia.
· Midline defects: Congenital pituitary deficiency can be associated with midline defects
such as cleft lip, cleft palate, single central incisor, and micro-ophthalmia.
· Micropenis: Congenital gonadotropin deficiency and possible pituitary abnormalities
cause this condition.
· Hepatomegaly: This is associated with glycogen storage diseases and fatty acid
oxidation disorders.
157) Healthy term neonate differ from adult in the following way:-
A. Less complement
B. Decrease IgG level
C. Fewer B lymphocytes
D. Lower level of secretory IgA
E. Higher level of C reactive protein
Ans:- AD
158) You are examining a newborn who is the product of an uneventful pregnancy,
labor, and delivery. Apgar scores were 9 at both 1 and 5 minutes. Findings on the
initial physical examination are unremarkable except for the presence of
vesicopustules and frecklelike macules (Item Q33A), some of which have a collarette
of surrounding scale. Wright stain of a smear of the vesicopustular contents reveals a
predominance of polymorphonuclear neutrophils.Of the following, the MOST likely
diagnosis is
A. congenital candidiasis
B. erythema toxicum neonatorum
C. infantile acropustulosis
D. miliaria rubra
E. transient neonatal pustular melanosis
Preferred Response: E
Characteristic lesions of transient neonatal pustular melanosis (TNPM) may be present at
birth as vesicles, pustules, or ruptured vesicles or pustules that have a collarette of
surrounding scale.Pigmented macules (Item C33A) often develop at the sites of resolving
pustules or vesicles.Primary lesions usually disappear by 5 days of age; the secondary
pigmented lesions may remain up to 3 months. TNPM occurs more commonly in African-
American infants.Lesions can occur on palms and soles. Pustular contents reveal a
predominance of neutrophils on Wright stain examination, as reported for the newborn in
the vignette.Infants who have congenital cutaneous candidiasis may present with scaling,
erythematous papules and pustules (Item C33B) at birth. Candida albicans can penetrate
through the amnion and chorion to cause congenital infections. Scrapings from lesions
prepared with potassium hydroxide document pseudohyphae (Item C33C) or budding
yeast.Term infants who have erythema toxicum neonatorum exhibit vesicopustular
lesions (Item C33D) that usually overlie erythematous macules. Lesions of erythema
toxicum rarely are present at birth, and Wright stain of smears of pustular contents reveals
a predominance of eosinophils.Infantile acropustulosis presents as pustules or vesicles
(Item C33E) localized to the hands and feet. It may be present at birth but more commonly
develops in the first weeks and months after birth, possibly continuing or recurring
throughout infancy and early childhood. Lesions are very similar to those of infantile
scabies infestation. Pustular contents may reveal prominent neutrophils and occasional
eosinophils without evidence of the mites, ova, or feces seen in scabies. An absence of
hyperpigmentation in resolving lesions and a prolonged or recurring course distinguishes
infantile acropustulosis from TNPM.Miliaria rubra (prickly heat or heat rash) is caused
by intraepidermal obstruction of the sweat ducts. A secondary local inflammatory response
is responsible for the erythema (Item C33F) associated with the papules and vesicles.
Miliaria rubra occurs later than miliaria crystallina,usually beyond thefirst postnatal week.
Hyperpigmented, frecklelike lesions are not expected in miliaria rubra.

159) NEC is associated with:-


A. Epidemic
B. Thrombocytopenia
C. Malabsorption
D. Bile stained aspirate
E. Air in the portal tree on abdominal X –ray
Ans:- ABCDE
160) The presenting feature of NEC include:-
A. Apnoea
B. Bradycardia
C. Abdominal distension
D. Intramural gas on abdominal X ray
E. Bloody stool
Ans:- ABCDE
161) A 3-week-old breastfed infant presents to the emergency department with
irritability,fever, jaundice, and hepatomegaly. A laboratory evaluation shows a
normal complete blood count and a bilirubin concentration of 6.5 mg/dL (111.2
mcmol/L). A urinalysis is positive for reducing substances. A blood culture is positive
for Escherichia coli. You initiate antibiotic therapy Of the following, the MOST
appropriate dietary management of this patient is to
A. continue breastfeeding
B. switch to a cow milk-based formula
C. switch to a soy-based formula
D. switch to a whey hydrolysate formula
E. switch to an elemental formula
Preferred Response: C
The clinical features of jaundice, hepatomegaly, and invasive Escherichia coli infection
described for the neonate in the vignette suggest the possible diagnosis of galactosemia.
Galactosemia is an autosomal recessive disorder most commonly caused by a deficiency
of the enzyme galactose-1-phosphate uridyltransferase. The reducing substances in the
urine represent the accumulation of galactose. In addition to recognizing and treating the
gram-negative infection in the infant, it is important to remove lactose, which is comprised
of glucose and galactose,from the diet as soon as the diagnosis is suspected.
Soy protein formulas are the first choice of nutrition for infants who have suspected or
proven galactosemia because the carbohydrate source in these formulas is sucrose or corn
syrup rather than lactose. Protein hydrolysate and elemental formulas also contain other
carbohydrates than lactose, but they can be more expensive and less readily available than
soy formulas. Lactose is the primary carbohydrate in human milk, cow milk-based
formulas, and most whey hydrolysate formulas.
162) Hydrops fetalis may be associated with :-
A. Rhesus isoimmunization
B. Paroxysmal supraventricular tachycardia
C. CMV infection
D. Achondroplasia
E. Renal vein thrombosis
Ans:- ABCDE
163) The following drugs are correctly paired with their potential teratogenic effect:-
A. Alcohol & macrocephaly with congenital heart disease
B. Phenytoin & meningomyelocele
C. Isotretinoic acid & cutis laxis syndrome
D. Penicillamine & facial abnormalities with pinna defects
E. Lithium & Ebtein's anomaly
Ans:-BCDE
Warfarine Hypoplastic nasal bridge ,chrondroplasi puncta
Isotretinoic acid facial , ear , cardiovascular abnormality
Phenytoin Hypoplastic nail, IUGR, typical facies & may be associated with
neural tube defect Tetracyclin Enamel hypoplasia
Sodium Valproate Neural tube defect
164) The neonate of mother with SLE may demonstrate:-
A. Polythycemia
B. Rash
C. Neutropenia
D. Atrial fibrillation
E. Anti-Ro antibodies
Ans:- BCE
165) Pulmonary hypoplasia is consequence of :-
A. Congenital varicella zoster
B. Anencephaly
C. Posterior urethral valve
D. Congenital diaphragmatic hernia
E. Exomphalos
Ans:- BCDE
166) The following conditions may present with bile-stained vomiting in the first
week of life:-
A. Duodenal atresia
B. Cystic fibrosis
C. Inguinal hernia
D. GER
E. NEC
Ans:- ABCE
167) Renal immaturity in normal neonate born at term is manifested as:-
A. Reduced number of nephrons
B. Decreased glucose reabsorption
C. Increased glomerular filteration rate
D. Decreased renal bicarbonate reabsorption
E. Decreased urea excretion
Ans:- BDE
168) The following maternal factors increase the incidence of surfactant deficient
RDS:-
A. Steroid therapy
B. Opiate
C. Placental insufficiency leading to IUGR
D. Diabetus
E. Alcoholism
Ans:- D
169) Bacteria commonly isolated in case of neonatal meningitis include:-
A. Escherichia coli
B. Haemophilus infuenzae
C. Group B streptococcus
D. Staphylococcu epidermidis
E. Neisseria meningitis
Ans:- AC
170) In the infant of diabetic mother :-
A. The infants brain size is increased beyond normal
B. The infant's liver size is increased beyond normal
C. He can be small for gestational age
D. There is an increased incidence of polycythemia
E. He has an increased incidence of hypertrophic Cardiomyopathy
Ans:- BCDE
171) Galactosaemia:-
A. Is caused by deficiency of the enzyme galactokinase
B. Causes jaundice in the newborn
C. May present with cataract at birth
D. Is associated with Escherichia coli septicemia
E. Is diagnosed as result of screening in the majority of cases.
Ans:- BCD
172) SVT in neonate:-
A. Is the most common abnormal tachycardia
B. Reflect underlying congenital heart disease in the majority of cases
C. Show regular rate of 160-220 beats /min on the ECG
D. Recurrent episodes usually persist into adulthood
E. May be stopped with rapid I/V bolus of adenosine
Ans:- A E
173) Neonates suffering withdrawal from in utero exposure to narcotics may show
sign of
A. Irritability
B. Vomiting
C. Photophobia
D. Hypotonia
E. Diarrhea
Ans:- ABCE
174) Clinical feature of congenital hypothyroidism diagnosed in the newborn period
include:-
A. Large tongue
B. Presence of third fontenalle
C. Umbilical hernia
D. Loose stools
E. High incidence of mental retardation
Ans:- A C
175) Causes of persistent neonatal unconjugated hyperbilirubinemia after 2 weeks
include:-
A. Rhesus incompatibility
B. Hypothyroidism
C. Breast milk jaundice
D. Rotor syndrome
E. Sepsis
Ans:- ABCE

176) Growth retardation babies are at increased risk of:-


A. Polycythemia
B. HMD
C. Hypoglycemia
D. Group B streptococcal infection
E. Sudden intrapartum death
Ans:- ACE
177) Oesophageal atresia is associated with:-
A. Maternal polyhydramniose
B. Vertebral anomalies
C. Diaphragmatic hernia
D. Low birthweight
E. Duplex collecting system
Ans:- ABDE
178) Congenital Rubella:-
A. Is associated with cerebral calcification
B. Frequently lead to cataract
C. Is associated with VSD
D. Rarely occur following maternal infection in the third trimester
E. Should be prevented by vaccinating women found to be seronegative during the
first trimester
Ans:- B D
179) Human breast milk contain:-
A. Secretory IgA
B. Macrophages
C. Lysozyme
D. Vitamin c
E. Zinc
Ans:- ABCDE
180) The following are recommended daily requirement for 1 month old child /hg of
B wt:-
A. Fluids 150-180 ml orally
B. Calories 90-115 kcal
C. Sodium 1.25-2.5 mmol
D. Protein 2.2-3.5 gm
E. Potassium 2.0-3.5 mmol
Ans:- ABCDE
181) Compared with cow's milk, human breast milk contains:
A. Less sodium.
B. Less calcium.
C. Less protein.
D. Less fat.
E. Less carbohydrate.
Ans:-A B C

182) UK infants are routinely immunised against:


A. Meningococcus type B.
B. Diphtheria.
C. Cholera.
D. Polio.Pertussis.
Ans:-B D E
183) Mature breast milk contain per 100 ml;-
A. 60-75 kcal
B. 8-10 gm of protein
C. 0.1-0.5 gm of fat
D. 0.1-0.5 gm of carbohydrate
E. 5.2 mg of folic acid
Ans:- AE
carbohydrate MATURE BREAST MILK 7.4 COWS MILK gm 4.6 gm
Fat MATURE BREAST MILK 4.2 gm COWS MILK 3.9 gm
protein MATURE BREAST MILK 1.1 gm COWS MILK 3.4 gm
Calories MATURE BREAST MILK 70 COWS MILK 67
folic acid MATURE BREAST MILK 5.2 mg COWS MILK 3.7 mg
184) Treatment of PDA with Indomethacin:-
A. Should be first line therapy
B. Is safe in renal failure
C. Should not be given if there is thrombocythenia
D. Should be avoided in jaundiced babies
E. Should not be given in the presence of IVH
Ans:- D E
185) Retinopathy of prematurity ;-
A. Develop in the first week of life
B. Is more likely to occur in VLBW infants
C. Is recognized complication of hypoglycemia
D. Rarely resolve spontaneously
E. May be treated effectively with Laser therapy
Ans:- B E
186) Alpha fetoprotein level from maternal serum may be helpful in diagnosing:-
A. Congenital cardiac malformation
B. Prune-belly syndrome
C. Cleft lip & palate
D. Down syndrome
E. Fetal alcohol syndrome
Ans:- D
187) A 2-week-old infant is jaundiced. Findings include weight and length at the 75th
percentile for age; icterus; with hepatosplenomegaly; total bilirubin, 6.3 mg/dL;
direct bilirubin, 5.5 mg/dL; alanine aminotransferase activity, 130 U/L; aspartate
aminotransferase activity, 143 U/L; and gamma-glutamyl transpeptidase activity, 950
U/L.Of the following, the BEST study to evaluate the excretion of bile from the liver
is
A. computed tomography of the liver
B. hepatic ultrasonography
C. hepatobiliary scintigraphy
D. measurement of galactose-1-phosphate uridyltransferase activity
E. measurement of the serum alpha1-antitrypsin level
Ans:- C
188) Which of the following constellations of features BEST describes the fetal
alcohol syndrome?
A. Elfin facies, irritability, and supravalvular aortic stenosis
B. Growth deficiency with microcephaly, developmental delay, and short palpebral
fissures
C. Intrauterine growth retardation, triangular-shaped face, and clinodactyly of the
fifth finger
D. Short stature, webbed neck, and pulmonic stenosis
E. Weakness, club feet, immobile face, and inadequate respirations
Ans:- B
189) A female infant born to a 24-year-old woman has been diagnosed clinically as
having Down syndrome. The mother is concerned about her risk of having another
child who has achromosomal abnormalityThe statement that you are MOST likely to
include in your discussion is that her risk
A. can be estimated by determination of maternal serum alpha-fetoprotein in all
future pregnancies
B. cannot be estimated until her infant's chromosome complement has been
determined
C. is increased for Down syndrome, but not for any other chromosomal abnormality
D. is no greater than that of any other woman her age
E. is not increased until she reaches the age of 35
Ans:- B

190) During delivery of an infant who has an estimated gestational age of 42 weeks,
you note that the amniotic fluid looks like pea soup and contains thick particles of
meconium.Of the following, the MOST important initial step in resuscitation of the
infant is to :-
A. aspirate the gastric contents
B. determine the Apgar score initiate tracheal intubation
C. provide positive pressure ventilation
D. suction the hypopharynx
Ans:- D
191) In addition to irritability, sweating, and difficulty breathing with feeding, the
symptom that is MOST indicative of congestive heart failure in a 3-week-old infant is
A. ascites
B. cough
C. cyanosis
D. diminished feeding volume
E. pretibial edema
Ans:- D
192) The decreased incidence of enteric infections noted in breastfed infants
compared with formula-fed infants is MOST likely due to the
A. more alkaline stool pH in breastfed infants
B. nutritional benefits of human milk on the infant's immune system
C. predominance of Bacteroides and Clostridium in the gut of breastfed infants
D. presence of protective antibodies against enteric infection in human milk
E. sterility of human milk
Ans:- D
193) A newborn who weighs 600 g and whose estimated gestational age is 24 weeks at
birth is admitted to the neonatal intensive care unit after successful resuscitation in
the delivery room. Arterial blood gas measurements on room air are: pH, 7.35;
PCO2, 42 mm Hg; PO2,68 mm Hg; base deficit, 2 mEq/L. Of the following, the
MOST appropriate initial management is to
A. begin intravenous vancomycin
B. begin phototherapy
C. initiate enteral feeding
D. provide bicarbonate infusion
E. provide glucose infusion
Ans:- E
194) A 3-day-old infant presents to the emergency department with vomiting,
lethargy,hypotonia, and jaundice. Physical examination reveals hepatomegaly and
neurologic depression. A full sepsis evaluation is undertaken, and the Gram stain of
the cerebrospinal fluid reveals gram-negative organisms. Of the following, the BEST
additional laboratory test to obtain is
A. erythrocyte galactose-1-phosphate
B. liver glycogen content
C. plasma insulin level
D. plasma very long-chain fatty acids.
E. stool porphyrins
Ans:- A
195) You are examining a term newborn in the nursery. His weight is 3.27 kg (50th
percentile), and his length is 50.5 cm (50th percentile). The pregnancy, labor, and
delivery were unremarkable. There are no significant findings on physical
examination. The MOST likely head circumference in this child, if it is consistent
with his other growth parameters, is
A. 31 cm
B. 33 cm
C. 35 cm
D. 37 cm
E. 39 cm
Ans:- D
196) previously healthy 5-day-old male who was born at home develops bruising and
melena.The pregnancy, delivery, and postnatal course were unremarkable. The
infant is breastfeeding vigorously every 2 hours. Findings on physical examination
are unremarkable except for several large bruises. Laboratory testing reveals:
hemoglobin, 81 g/L (8.1 g/dL);white blood cell count, 9.4 x 109/L (9,400/mm3);
prothrombin time, 37 seconds; partial thromboplastin time, 98 seconds; platelet
count, 242 x 109/L (242,000/mm3); and fibrinogen,2.34 g/L (234 mg/dL). Of the
following, the MOST likely cause of the bleeding is
A. disseminated intravascular coagulation
B. factor VIII deficiency hemophilia
C. liver disease
D. vitamin K deficiency
E. von Willebrand disease
Ans:- D
197) A newborn whose estimated gestational age is 42 weeks is stained with
meconium.Tracheal intubation reveals meconium in the hypopharynx as well as
below the vocal cords.The infant has respiratory distress. A chest radiograph is
obtained. Of the following, the MOST likely radiographic finding is
A. coarse infiltrates
B. decreased lung volumes
C. mediastinal shift
D. pleural effusion
E. reticulogranular pattern
Ans:- A
198) A 20-year-old primigravida at 30 weeks of gestation has a blood pressure of
160/112 mm Hg, serum total bilirubin level of 44.5 mcmol/L (2.6 mg/dL), serum
alanine aminotransferase level of 150 U/L, and platelet count of 75 x 109/L
(75,000/mm3). She is hospitalized for observation and electronic fetal heart rate
monitoring. Of the following, the MOST ominous sign of fetal distress during
monitoring would be
A. early decelerations
B. increased beat-to-beat variability
C. late decelerations
D. spontaneous accelerations
E. variable decelerations
Ans:- C
199) A term newborn presents with bilious vomiting shortly after birth. Her
abdomen is distended slightly, and facial features are characteristic of Down
syndrome. She has passed a normal meconium stool. The pregnancy was complicated
by polyhydramnios. Of the following, the MOST likely diagnosis is
A. duodenal atresia
B. Hirschsprung disease
C. meconium ileus
D. midgut volvulus
E. pyloric stenosis
Ans:- A
200) An 18-year-old primigravida at 32 weeks' gestation has a blood pressure of
148/96 mm
Hg, proteinuria, oliguria, and visual disturbances. Labor is induced, and the infant is
delivered. His weight is 850 g (<10th percentile), crown-heel length is 38 cm (10th
percentile), and head circumference is 30 cm (50th percentile). Of the following, the
MOST
likely complication in this infant is
A. anemia of prematurity
B. hyaline membrane disease
C. hyperglycemia
D. meconium aspiration
E. perinatal asphyxia
Ans:- E
201) An infant is born at 27 weeks' gestation following a pregnancy complicated by
preterm labor that progressed despite administration of a tocolytic agent. Of the
following, the most appropriate INITIAL management is to
A. measure transcutaneous oxygen saturation
B. perform endotracheal intubation
C. place an umbilical arterial catheter
D. place the infant in an open bed warmer
E. provide nasal continuous positive airway pressure
Ans:- D
202) Of the following, the MOST important determinant of neurodevelopmental
outcome of VLBW infants is
A. antenatal obstetric management
B. infant gender
C. length of gestation
D. maternal education
E. socioeconomic status
Ans:- C
203) You are called to the newborn nursery to examine an infant who appears
dysmorphic.On physical examination, the baby is normally grown and vigorous. You
note overfolded pinnae, deviation of the nose to one side (Item Q25A), and a small
chin. The feet are maintained in dorsiflexion (Item Q25B), but can be corrected
passively. You review the pregnancy history. Of the following pregnancy
complications, the one MOST likely to be associated with this baby’s features is
A. maternal hypertension
B. polyhydramnios
C. preeclampsia
D. transverse lie
E. vaginal bleeding
Preferred Response: D
The infant described in the vignette displays the characteristic features of a deformation
sequence. Unlike malformations, which occur due to intrinsic problems within a
developing structure,deformations are due to mechanical forces acting on an otherwise
normally developing embryo or fetus. Causes of fetal deformation are many and include
oligohydramnios, prolonged breech positioning, a small or malformed uterus, fibroid
tumors of the uterus, and multiple gestations.Often, the affected infant has a pugilistic
facies, with deviation of the nose to one side. Limb positioning defects are common.
Barring any association with malformations or disruptions, the appearance of the affected
infant typically normalizes over time.Although maternal hypertension and preeclampsia
can be associated with placental insufficiency and, ultimately, decreased fetal movement,
typically they are not associated with fetal deformation. Polyhydramnios usually is
associated with unrestricted fetal movement,and fetuses are not deformed. Vaginal
bleeding has multiple potential causes, most of which are not associated with deformation.
CHILDREN HOSPITAL –BENGHAZI ABDULRAHMAN BASHIR 45Transverse lie is the only
pregnancy complication listed that is likely to be associated with deformation due to
unusual fetal position
204) A 10-day-old infant who weighed 1,750 g at birth and whose gestational age was
34 weeks is jaundiced. His total serum bilirubin concentration is 10.0 mg/dL and the
direct fraction is 0.8 mg/dL. He is receiving intermittent orogastric feeding of
expressed human milk and supplemental parenteral nutrition. Of the following, the
MOST likely explanation for these findings is
A. Crigler-Najjar syndrome
B. jaundice due to parenteral nutrition
C. neonatal hepatitis
D. physiologic jaundice
E. pyloric stenosis
Ans:- D
205) Early hospital discharge is defined as the discharge of a newborn earlier than 48
hours following vaginal delivery or 96 hours following cesarean delivery. Of the
following, the MOST common reason for readmission to the hospital within 7 days
following an early discharge is
A. bacterial sepsis
B. congenital heart disease
C. gastrointestinal malformation
D. hyperbilirubinemia
E. metabolic disorders
Ans:- D
206) A newborn infant is delivered by emergent cesarean section at 41 weeks'
gestation following a pregnancy complicated by a prolapsed umbilical cord and
meconium-stained amniotic fluid. At 6 hours of age, the infant has a generalized
tonic-clonic seizure. Of the following, the MOST likely explanation for this seizure is
A. hyponatremia
B. hypoxic-ischemic encephalopathy
C. intracranial hemorrhage
D. meningitis
E. pyridoxine dependency
Ans:- B
207) A 4-hour-old newborn has copious oral secretions and episodes of coughing,
choking, and cyanosis. The pregnancy was complicated by polyhydramnios. You
suspect esophageal atresia with tracheoesophageal fistula.Of the following, the
MOST helpful test to confirm the diagnosis is to
A. inject a contrast medium through an orogastric catheter and obtain a neck
radiograph
B. obtain computed tomography of the neck
C. perform flexible bronchoscopy
D. place an endotracheal tube and examine the endotracheal fluid
E. place an orogastric suction catheter and obtain a chest radiograph
Ans:- E
208) A newborn is delivered by emergent cesarean section because of fetal distress
following acute abruption of the placenta. The infant is resuscitated and transferred
to the nursery.On physical examination, she appears pale, and her extremities are
cold to touch. The capillary refill is 8 seconds. Results of an arterial blood gas
analysis show a Po2 of 48 mm Hg. Of the following, the MOST likely additional
finding is
A. decreased bicarbonate concentration
B. hypercalcemia
C. hyperglycemia
D. hyponatremia
E. normal anion gap
Ans:- A
209) Of the following, erythromycin prophylaxis is MOST likely to prevent ocular
infection due to
A. Chlamydia trachomatis
B. group B streptococci
C. Neisseria gonorrhoeae
D. Staphylococcus aureus
E. Trichomonas vaginalis
Ans:- C
210) A 1,300 g infant who is born at 34 weeks' gestation has a head circumference of
27 cm and crown-heel length of 40 cm. At 48 hours of age, she is irritable, tremulous,
and inconsolable. Her systolic blood pressure is 65 mm Hg and heart rate is 180
beats/min. Her face appears normal, and her cry is high-pitched. Cranial
ultrasonography reveals bilateral echo densities suggestive of periventricular
leukomalacia. Of the following, the MOST likely explanation for the findings in this
infant is maternal exposure to
A. alcohol
B. barbiturates
C. cocaine
D. marijuana
E. opiates
Ans:- C
211) A 4.3 kg infant is delivered to a woman whose diabetes mellitus is poorly
controlled. Of the following, the MOST likely neonatal manifestation of maternal
diabetes is
A. diabetic ketoacidosis
B. Hirschsprung disease
C. hypercalcemia
D. polycythemia
E. renal vein thrombosis
Ans:- D
212) Of the following, the MOST helpful finding to distinguish GBS pneumonia from
RDS is
A. a normal C-reactive protein level
B. an elevated erythrocyte sedimentation rate
C. diffuse alveolar infiltrates on chest radiography
D. increased ratio of bands to segmented neutrophils
E. persistent hypoxemia on blood gas analysis
Ans:- D
213) An 18-hour-old infant of a diabetic mother develops abdominal distension.
Physical examination reveals a protuberant, firm, but nontender abdomen; patent
anus; and no grossly visible anomalies. The infant has passed no meconium stool
since birth. A supine abdominal radiograph reveals multiple dilated loops of
intestine.Of the following, a contrast enema would MOST likely confirm a diagnosis
of
A. atresia of the colon
B. Hirschsprung disease
C. hypoplastic left colon syndrome
D. meconium ileus
E. midgut volvulus with malrotation
Ans:- C
214) You are attending the emergency delivery by cesarean section of a primiparous
woman.The gestation was complicated by pregnancy-induced hypertension. Deep
variable fetal
heart rate decelerations were noted during labor. At delivery, the infant is
acrocyanotic with
poor tone; spontaneous movement and minimal respiratory effort are present. Of the
following, your INITIAL management is to
A. ascertain the heart rate and assign a 1-minute Apgar score
B. begin tactile stimulation and provide blow-by oxygen supplementation
C. dry all skin surfaces and clear the oropharynx
D. initiate bag-mask ventilation
E. insert an umbilical catheter and administer naloxone
Ans:-C
215) A vigorous, normal-appearing term male newborn has not voided by 18 hours
after delivery. Perinatal history is negative for maternal illness or medications.
Amniotic fluid volume was reportedly normal, and the delivery was uneventful, with
Apgar scores of 6 and 9 at 1 and 5 minutes, respectively. Of the following, the MOST
likely reason why this 18-hour-old infant has not voided is
A. bilateral ureteropelvic junction obstruction
B. intravascular volume depletion
C. neurogenic bladder
D. posterior urethral valve
E. undocumented void in the delivery room
Ans:- E
216) A 2-week-old neonate who was born at 32 weeks’ gestation has recovered from
respiratory distress syndrome. He has been tolerating increasing volumes of enteral
feedings via gavage. Over the past several feedings, abdominal distension, gastric
residuals, and stools that are positive for blood have been noted. Of the following, the
radiographic finding MOST supportive of the diagnosis of necrotizing enterocolitis is
A. absence of luminal bowel gas
B. generalized bowel distension
C. intraperitoneal fluid
D. pneumatosis intestinalis
E. thickening of the bowel wall
Ans:- D
217) A term infant is placed under a radiant warmer, the skin is dried, and the
oropharynx and nose are suctioned. After tactile stimulation, there is minimal
respiratory effort, dusky color, and a heart rate of 86 beats/min. Bag/mask
ventilation is performed for 30 seconds with 100% oxygen at a rate of 40 to 60
breaths/min. The heart rate increases to 100beats/min. Of the following, the NEXT
best step is to:
A. administer sodium bicarbonate
B. continue bag/mask ventilation at a rate of 20 to 40 breaths/min
C .continue ventilation and begin chest compressions
D. observes for spontaneous respiration and discontinues ventilation
E. perform endotracheal intubation
Ans:-D
218) A 900 g male infant is delivered vaginally to a woman who had no prenatal care.
Of the following, the physical finding that is MOST consistent with prematurity
rather than intrauterine growth restriction is
A. creases over entire sole of foot
B. descended testes with deep rugae of the scrotum
C. formed and firm pinna with instant recoil
D. gelatinous translucent skin
E. raised areola and 3 mm breast buds\
Ans:-D
219) A 1-day-old infant develops bilious vomiting and gastric distension. She has
been afebrile and has been passing meconium-laden stools. Of the following, the most
appropriate INITIAL step in the management of this infant is
A. abdominal radiography to look for the “double-bubble” sign
B .culture of a catheterized urine specimen
C .insertion of a rectal tube for decompression
D .placement of a nasogastric tube and initiation of intravenous fluid therapy
E. upper gastrointestinal radiographic series to look for malrotation of the small bowel
Ans:- D
220) While performing ultrasonography on a 31-week fetus, an obstetrician notes
that the fetal heart rate ranges from 62 to 66 beats/min. Fetal growth appears
normal, and no structural cardiac anomalies are identified. Fetal echocardiography
reveals that the fetal atria appear to be contracting at 140 beats/min, with a
ventricular rate of 65 beats/min. Of the following, the NEXT step in the management
of this infant is to:-
A. administer beta-agonist drug therapy to the mother
B. assess the cardiac status of the infant following labor and delivery
C. counsel the parents that intrauterine fetal death is likely
D. perform amniocentesis to confirm lung maturity and if mature, perform immediate
cesarean section
E. repeat the fetal echocardiography and fetal ultrasonography in 1 week
Ans:-E
222) Term infant is cyanotic and requires intubation. Findings include: heart rate,
175beats/min; blood pressure, 60/30 mm Hg; increased right ventricular activity;
single S2;short systolic murmur; and equal arm and leg pulses; chest radiography,
normal heart size and pulmonary congestion. Arterial blood gases (right radial
artery on 100% FIO2): pH,7.31; PO2, 43 torr; PCO2, 48 torr. Of the following, the
MOST likely diagnosis is:-
A. hyaline membrane disease
B. hypoplastic left heart
C. intrauterine constriction of the ductus arteriosus
D. tetralogy of Fallot
E. total anomalous pulmonary venous connection
Ans:-E
223) A 7-day-old infant has copious purulent discharge from both eyes. The 17-year-
old mother currently complains of a yellowish vaginal discharge. The only
medications received by the infant were vitamin K and topical erythromycin
prophylaxis following delivery.Giemsa stain of a conjunctival scraping reveals
intracytoplasmic inclusions. After obtaining appropriate diagnostic studies, the
BEST management includes treatment with :-
A. oral erythromycin
B. oral penicillin
C. topical erythromycin
D. topical gentamicin
E. topical sulfonamide
Ans:-A
224) Of the following, the condition that is MOST likely to present with seizures
during the
first 24 hours of life is
A .fetal alcohol syndrome
B .herpes simplex infection
C. hypoxic-ischemic encephalopathy
D. organic acidemia
E. urea cycle defect
Ans:-C
225) Maternal serum alpha fetoprotein measured at 12 weeks gestation
A. If normal then neural tube defect are excluded
B. Is increased in twin pregnancy
C. Is reduced in Turner syndrome
D. Is increased in Down syndrome
E. AFT is the main blood protein fraction in the first trimester
Ans :- BE
Serum a fetoprotein is usually measured at 16-18 week gestation, normal screening would
not rule out neural tube defect or any other lesion such as twins, Turner's syndrome,
Exomphalus,which can all have raised value .
226) A newborn presents with an early onset of dyspnea with chest retractions,
expiratory grunting and cyanosis following an uneventful normal preterm labor. On
examination no cardiac murmurs are heard and the lungs appear clear. On a plain
X-Ray there is evidence of prominent pulmonary vascular markings and fluid lines
in the fissures. The cyanosis improves with minimal oxygen. The most probable
diagnosis is :-
a) Meconium aspiration syndrome
b) Fetal aspiration syndrome
c) Transient tachypnea of the newborn
d) Persistent fetal circulation
e) Hyaline membrane disease
Ans:- C
EXPLANATION
The clinical case given above indicates the likely diagnosis of transient tachypnea of the
newborn,(Choice C) which is otherwise called Respiratory distress syndrome type II. It is
characterized by the early onset of dyspnea with chest retractions, expiratory grunting and
cyanosis. Neonates usually recover within 3 days.The syndrome is believed to be
secondary to slow absorption of fetal lung fluid resulting in decreased pulmonary
compliance and tidal volume.The distinctive feature of transient tachypnea of the newborn
from hyaline membrane disease (Choice E) are the infant’s sudden recovery and the
absence of a radiographic reticulogranular pattern.In meconium aspiration syndrome
(Choice A) the chest X-ray is characterized by patchy infiltrate,coarsestreaking of both
lung fields and flattening of diaphragm. A normal chest X-ray in an infant with severe
hypoxia and no cardiac malformation suggests the diagnosis of persistent fetal
circulation. ( Choice D).Fetal aspiration syndrome(Choice B) is very common during
prolonged labor and difficult deliveries and may initiate vigorous respiratory movements
in-utero because of interference with oxygen supply
227) A neonate on the 5th day of life was diagnosed to have developed severe
meningitis.Serology of the CSF revealed that the causative organism was
Haemophilus influenza.The neonate was started on chloramphenicol. After 36hrs the
neonate started vomiting,and became flaccid. The baby was also found to be
hypothermic. These symptoms are result of the neonate having an immature
a) hepatic function
b) immune system
c) renal tubule system
d) hypothalamic regulatory system
e) peripheral neurological system
Ans: A
EXPLANATION
Chloramphenicol is the drug of choice in Haemophilus influenza due to its high efficacy
against H.influenza.Gray baby syndrome is the name given to chloramphenicol toxicity
seen in characteristically neonates even at low therapeutic levels. The cause for this is that
neonates have an immature hepatic function and lack an effective glucoronic acid
conjugation mechanism for degradation and detoxification of chloramphenicol. So even at
low therapeutic levels the drug accumulates in the body causing toxicity which is
manifested by vomiting, flaccidity, hypothermia, gray color and ultimately shock and
collapse.
1. Indomethacin give to preterm baby may cause :-
A. Hypernatremia
B. Hyperkalemia
C. Polyuria
D. Hypoglycemia
Ans:- B
Contraindications to indomethacin include thrombocytopenia (<50,000/mm3),
bleeding disorders, oliguria (<1 mL/kg/hr), necrotizing enterocolitis, isolated
intestinal perforation, and an elevated plasma creatinine level (>1.8 mg/dL).
2. Na- Valproate used in early pregnancy may result in fetal:-
A. Cardiac anomaly
B. Platelet dysfunction
C. Neural tube defect
Ans :-C
CHILDREN HOSPITAL –BENGHAZI ABDULRAHMAN BASHIR 52
3. α fetoprotein increased in :-
A. Turner syndrome
B. Trisomy 13
C. Rh isoimmunization
D. Spontaneous abortion
Ans:- A
4. During management of patient with sever meconium aspiration the following can
be used:-
A. IPPV
B. CPAP
C. Nitroprusside
D. Dexamethasone
Ans:-A
5. Hypoxic-ischemic encephalopathy all are correct except:-
A. Result from excessive use of oxytocin
B. IUGR is be first indication of fetal hypoxia
C. Associated with increased beat to beat variability
D. Prognosis depend on gestational age
E. Persistent of abnormal neurological sign at two week indicate poor prognosis
Ans:- C
6. Apgar score include the following except :-
A. Muscle tone
B. Response to catheter in nostrils
C. Heart rate
D. Respiratory rate
Ans:- D

7. Intracranial hemorrhage :
A. Primary hemorrhage disorder give rise to intraventricular hemorrhage
B. intraventricular hemorrhage is symptomatic immediately after birth
C. neurosurgical intervention is indicated early in the ?course of the condition
D. benefit of reducing intracranial pressure have not been established
8. HMD:-
A. Pathological finding are rarely seen in infant dying early than 8 hour after birth
B. In sever cases grunting is absent
C. Death is rare in the 1st day of illness
D. X- ray finding are pathognomic
Ans:- AB C
Death is rare on the 1st day of illness, usually occurs between days 2 and 7, and is
associated with alveolar air leaks (interstitial emphysema, pneumothorax), pulmonary
hemorrhage, or IVH, On xray,the lungs may have a characteristic, but not pathognomonic
appearance that includes a finereticular granularity of the parenchyma and air
bronchograms, which are often more prominent early in the left lower lobe because of
superimposition of the cardiac shadow
9. About HMD all correct except :-
A. Occur in 60% of infant (28 week of age)
B. Increased frequency is associated with previous history of affected infant
C. High oxygen lead to decreased surfactant
D. Atelectasis lead to ventilated but not perfused alveoli
E. HMD is accompanied by Rt → Lt shunt
Ans:- D
10.All of the following are true except:-
A. TTN isn't seen in normal delivered infant
B. Patient with TTN may have pleural effusion
C. Meconium stained amniotic fluid is seen in 10% of live birth
D. Meconium aspiration syndrome may lead to metabolic acidosis
E. Patient with meconium aspiration may get beneficial from ECMO
Ans:- A
11.natural thermal environment for 1-2 kg at 12 hour of age is :-
A. 37
B. 36
C. 34
D. 38
E. 32
Ans:- C
12.To increase the Hb of newborn by 1gm/dl the amount of packed cell required will
be :-
A. 2 cc/kg
B. 4 cc/kg
C. 5 cc/kg
D. 9 cc/kg
E. 15 cc/kg
Ans:- A
13.About birth injury all are correct except
A. C1-C2 level is the commonest site of injury
B. In spinal injury , the patient may die before appearance of neurological sign
C. Erb's palsy infant loss power to abduction
D. Erb's palsy infant, Bicep's reflex is absent
E. Erb's palsy infant, preserve of hand grasp is favorable prognosis
Ans: A
14.The following associated with polyhydramniose except;-
A. Indomethacin administration
B. Diaphragmatic hernia
C. Achondroplasia
D. Adenomatoid cystic lung
E. Trisomy 18
Ans:- A
15.Birth injury:-
A. Paralysis of the upper arm has better prognosis than paralysis of the lower
B. In nerve injury, neuroplasty is advised at the end of first year of life
C. In phrenic nerve paralysis, spontaneous recovery is expected
D. Facial nerve paralysis will result from the muscular agenesis of the facial nerve
Ans:-A C
16.All statement are true except:-
A. IVH is uncommon in infant > 34 week gestation
B. The incidence of VSD is high in VLBW infant ( PT > PDA)
C. Hypoglycemia seen more commonly in patient with SGA
D. Hypercarbia is risk factor for …..
E. Decrease vitamin E may cause hemolytic anaemia
17.Breast milk composition all true except:-
A. Protein 2gm/dl
B. Carbohydrate 7 gm /dl
C. Ca/Po4 33/15 mg/dl
D. Fat 4 gm/dl
E. Osmolality 290-300 mosm
Ans:- A
18.All statement are true except:-
A. Prematurity is predisposing factor for birth asphyxia
B. Cephalohematoma is accompanied with discoloration of the scalp
C. Sensation of central depression suggesting fracture accompanied
cephalohematoma
D. Occipital bone fracture is fatal
E. Subconjunctival hemorrhage is due to increase intrathoracic pressure
Ans:- B
19.Sign of ICH include all of the following except:-
A. Enlarged head
B. Fixed dilated pupil
C. Neck stiffness
D. Persistent cyanosis
E. Sweating
Ans:- E
20.Exchange transfusion in preterm baby may be complicated by:-
A. Hemoglobinuria
B. Hemoglobinemia
C. Thrombocytopenia
D. IVH ??
E. GVHD
Ans:-
21.For treatment of hyperbilirubinemia all are true except:-
A. Phototherpy ( 250-320 nm) ??
B. Intravenouse immunoglobuline
C. Metalloporphyrins (Tin (Sn)-protoporphyrin ).
D. Activated charcoal
Ans:- A
Clinical jaundice and indirect hyperbilirubinemia are reduced on exposure to a high
intensity of light in the visible spectrum. Bilirubin absorbs light maximally in the blue
range (420–470 nm).
22.Necrotizing enterocloitis :-
A. USS detect microbabbules of gas in the GIT wall is used recently
B. Vitamine E is used in the treatment of NEC
C. Maternal intake of cocaine may predispose to NEC
D. Breast milk protect against NEC
Ans:- C D
23.All of the following are correct except:-
A. Generalized oedema occur with Hurler syndrome
B. Cyanosis can be mask with pale
C. Postmature tend to have pale skin than term
D. Periodic respiration is common in the first 24 hour
E. Wormian bone associated with cliedo dystosis
Ans:-D
24.The following fact give false positive ( low) Apgar score except:-
A. Mg sulfate
B. Congenital myopathy
C. High fetal calcitonine level
D. Immaturity
E. Airway obstructed
Ans:-C
25.IDM:-
A. Increased insulin level in infant born to mother with gestational diabetus
B. Decrease level of FFA
C. Decrease Epinephrine & Glucagons response
D. Cortisol & human growth hormone are normal
Ans:- A BC D
26. Regarding thyroid problem during neonatal period all are false except:_
A. Maternal thyrotoxicosis during pregnancy can be treated safely with propranolol
& thiouracil
B. Mother should be kept mildly hypothyroidism
C. Breast milk may mask neonatal hypothyroidism
D. If the mother on thyroxine should stop breast feeding
E. Carbimazole can be used safely in nursing mother
Ans:-E
27. During morning rounds in the newborn nursery, you examine a healthy infant
who has blotchy erythematous macules that are 2 to 3 cm in diameter. The macules
are scattered over the trunk, face, and proximal extremities; the palms and soles are
spared. Each macule has a 1- to 3-mm central vesicle or pustule.Of the following, the
MOST likely additional finding in this patient is
A. pigmented macules located at sites of resolving pustules
B. presence of lesions at birth
C. pustules coalescing into bullae
D. Wright stain of a smear of the vesicopustular contents revealing a predominance of
eosinophils
E. Wright stain of a smear of the vesicopustular contents revealing a predominance of
polymorphonuclear neutrophils
Preferred Response: D
The infant described in the vignette exhibits the classic presentation of erythema toxicum
neonatorum. Tiny vesicles or pustules arise from blotchy erythematous macules, with
lesions characteristically appearing at 24 to 48 hours after birth. The pustules do not
coalesce into bullae. Wright staining of the pustular contents reveals a predominance of
eosinophils, not neutrophils. Because the lesions are seen in healthy infants, it has been
suggested that this benign condition be renamed; suggested names include “benign
neonatal rash” or “benign erythema neonatorum.”Transient neonatal pustular melanosis
(TNPM) is another well-recognized benign dermal eruption of infancy in which pustular
lesions spontaneously resolve into transient pigmented macules . TNPM may be present at
birth, and examination of pustular contents reveals apredominance of neutrophils.Infantile
acropustulosis is a chronic or recurring benign condition manifested by intensely pruritic
pustules on hands and feet. Characteristic papules and pustules may coalesce into
bullae.Infantile acropustulosis frequently is confused with scabies infestation. The lesions
resolve spontaneously at 1 to 2 years of age.
28.Maternal vaccination during pregnancy can be given safely except:-
A. BCG
B. HBV
C. Cholera vaccine
D. Tetanus
E. Meningococcal vaccine
Ans:- A
29. Newborn infants commonly have:-
A. Capillary hemangioma on the forehead.
B. Posterior cranial fontenalle.
C. Metopic sutures.
D. Impalpable coronal sutures.
E. Skin tag in front of the ear.
Ans:- ABC
53. The following statements are true about SIDS (sudden infant death
syndrome) except:
a) Both prone and side sleeping increase risk of SIDS.
b) All babies should sleep in supine position including infants with
micrognathia and obstructive sleep apnea.
c) Most common cause of postneonatal death 1 mo – 1 yr of age in
developed countries.
d) Future SIDS victims may have repeated fatigue during feeding
and profuse sweating during sleeping.
e) Caffeine and theophylline decrease apnea and improve
respiratory pattern. However, no medication can prevent SIDS.

53. (b) All babies should sleep in supine position except in infants with
micrognathia and obstructive sleep apnea.

124. A 2-year-old boy was brought to the ER with a history of difficulty


breathing and coughing for the last 2 days. His mother has asthma.
Three other siblings do not have asthma. Mother denies any foreign
body aspiration.
Physical examination: afebrile, mildly tachypneic, wheezing is noted in
the right chest, and the left lung sounds are clear. Chest X-ray reveals
overdistended right lung, especially in expiratory film. Appropriate
management should be:
a) Bronchodilator
b) Pulmonary physiotherapy
c) Thoracotomy
d) CPR (cardio pulmonary resuscitation)
e) Removal of the bronchial foreign body

294. A child was extubated and developed postextubation subglottic


edema. The next step in management:
a) Furosemide
b) Reintubate immediately
c) Spironolactone
d) Nasal CPAP (continuous positive airway pressure)
e) Inhalation of aerosolized racemic epinephrine

371. All of the following are indications for tonsillectomy except:


a) Obstruction due to tonsils
b) Tumor of tonsills
c) Middle-ear deafness
d) Peritonsillar abscess with prior history of tonsillitis
e) Recurrent peritonsillar abscess

379. A female child appears in the ER with a severe asthma attack. She
has been using albuterol inhalation pump.
Physical examination reveals tachypnea, intercostal and subcostal
retractions, and expiratory wheezes. She is in a respiratory failure. The
most important finding suggests impending respiratory failure:
a) Hypoxia
b) Hypercarbia
c) Fatigue
d) Lethargy
e) Hypocarbia

399. A 15-year-old girl appears in the clinic with history of a dry cough
with harsh, ‘barking’ quality for the last 2 weeks. She has no fever. She
had history of mild whitish vaginal discharge for the last 3 weeks. She
had no problems in sleeping at night. Her mother said that she did not
cough at night while sleeping. Physical examination reveals mildly
injected throat. Most likely diagnosis:
a) Pharyngitis
b) Chlamydia
c) Croup
d) Laryngeal papilloma
e) Habit cough

408. A 14-year-old girl appears with sudden onset of left chest pain for
the last 2 hours. She has no fever. She denies history of trauma. Her
menstrual period is regular, usually lasts for 5 days, and presently, she is
on day 3 of her period. Most likely diagnosis:
a) Pneumomediastinum
b) Pneumothorax
c) Pulmonary embolism
d) Asthma
e) Angina

408. (b) Catamenial pneumothorax

449. A 7-year-old boy is diagnosed to have a obstructive lung disease.


All of the following are abnormal except:
a) Total lung capacity
b) Residual volume
c) Vital capacity
d) Functional residual capacity
e) Diffusing capacity of carbon monoxide

449. (e) Diffusing capacity of carbon monoxide

451. A 2-week-old girl appears in the clinic for a routine check up. She
has biphasic stridor that is more prominent during inspiration. She has
hoarseness and barking cough. Most likely diagnosis:
a) Subglottic hemangioma
b) Laryngeal nodule
c) Tracheomalacia
d) Croup
e) Laryngeal web

471. A preterm infant with respiratory distress syndrome was placed on


a mechanical ventilator. The infant developed right pneumothorax. The
most important factor causing pneumothorax in a patient on mechanical
ventilator:
a) High PEEP
b) High PIP
c) Low oxygen
d) High flow rate
e) High expiratory time

536. Most common complication of a tympanostomy tube:


a) Hearing loss
b) Otorrhea
c) Cholesteatoma
d) Otitis media
e) Otitis externa

545. Pulmonary function test results in a patient with cystic fibrosis:


a) Restrictive lung disease
b) Obstructive lung disease
c) Initially restrictive, then obstructive lung disease
d) Initially obstructive, then restrictive lung disease
e) Normal lung function

569. A 6-year-old boy had a modified Blalock-Tausig shunt for tetralogy


of Fallot. About 12 hours after surgery, the boy developed respiratory
distress. Chest X-ray revealed pleural effusion. Most likely diagnosis:
a) Hemothorax
b) Chylothorax
c) Pneuthorax
d) Hydrothorax
e) Diaphragmatic hernia

592. A child is diagnosed to have a pleural effusion. All of the following


are the characteristics of a transudative pleural fluid except:
a) Total protein is less than 3 g/dL.
b) Lactate dehydrogenase level is low.
c) Total WBC count is less than 2000/mm3
d) Predominance of monocytes
e) A ratio of pleural protein to serum protein is more than 0.5
640. A 10-day-old male newborn is diagnosed to have a cystic fibrosis.
Sweat chloride test results are normal. The most likely interpretation:
a) He does not have a cystic fibrosis.
b) Sweat chloride test results are always abnormal in a cystic fibrosis
newborn.
c) A sweat chloride test should not be performed before 1 year of age.
d) Sweat chloride test results are not reliable in first week of life.
e) Sweat chloride test should be performed after 6 months of age.

840. A 4-year-old male asymptomatic child appears for a routine check


up. He had an ear infection 3 years ago. Physical examination reveals
bilateral middle-ear effusions. Most likely diagnosis:
a) Acute otitis media
b) Chronic otitis media
c) Otitis externa
d) Middle ear cholesteatoma
e) Eustachian tube dysfunction or obstruction

864. Most common cause of childhood nasal polyposis:


a) Allergic rhinitis
b) Allergic sinusitis
c) Vasomotor rhinitis
d) Cystic fibrosis
e) Deviated nasal septum
888. Most common clinical manifestation of cystic fibrosis:
a) Failure to thrive
b) Abnormal stools
c) Meconium ileus
d) Nasal polyps
e) Respiratory symptoms
889. The best initial diagnostic study for a child with respiratory
distress:
a) Arterial blood gas
b) Chest X-ray
c) Chest CT scan
d) Chest MRI
e) Oxygen saturation by pulse oximetry

977. A 3-year-old girl appears with a sudden onset of cough, dyspnea,


and hoarseness for the last 3 hours. Mother noticed a small amount of
fresh blood with mucus inside her mouth. She was playing and eating
prior to this episode. Physical examination reveals a croupy cough and
mild cyanosis. The appropriate management:
a) Throat culture
b) Nasopharyngeal culture
c) Remove a foreign body from esophagus
d) Remove a foreign body from larynx
e) Remove a foreign body from nasopharynx

978. A 2-year-old boy appears with a sudden onset of cough, dyspnea,


and hoarseness for the last 6 hours. Mother denies fever. He was playing
prior to this episode. Physical examination reveals bilateral wheezing,
audible slap, and palpable thud. The appropriate step in management:
a) Throat culture
b) Nasopharyngeal culture
c) Remove a foreign body from esophagus.
d) Remove a foreign body from trachea.
e) Remove a foreign body from stomach
981. Preferred investigative procedure for a lung mass:
a) MRI
b) Fluroscopy
c) CT scan
d) Chest x-ray
e) Pulmonary function test
1001. Least likely complication of bronchoscopy:
a) Hypoxia
b) Bronchospasm
c) Arrythmia
d) Pneumopericardium
e) Laryngospasm

30. Inhaled glucocorticoid therapy can cause dysphonia in patients with


asthma. The cause of dysphonia is:
a) Vocal cord myopathy
b) Fungal infection in vocal cord
c) Vocal card paralysis
d) Pharyngitis
e) Laryngitis

31. Most common side effects of inhaled glucocorticoids are oral thrush
and dysphonia. The cause of oral thrush formation is:
a) Local immunosuppression
b) Dry mouth
c) Excessive salivation
d) Herpes simplex virus
e) Infectious mononucleosis
164. The mean airway pressure (MAP) is increased by all of the
following parameters except:
a) Increased positive inspiratory pressure (PIP)
b) Increased positive end expiratory pressure (PEEP)
c) Increased inspiratory flow
d) Prolonged expiratory time
e) Increased ventilatory rate without changing the inspiratory time
164. (d) Prolonged expiratory time does not increase the MAP.
Prolonged inspiratory time without changing the rate causes reversal of
inspiratory–expiratory ratio (I:E) resulting in an increased of MAP.
170. Pulmonary interstitial emphysema (PIE) can be prevented by:
a) Avoiding high positive inspiratory pressure (PIP)
b) Avoiding high oxygen use
c) Increasing high PIP
d) Increasing mean ventilatory pressure
e) Avoiding low positive end-expiratory pressure (PEEP)

170. (a) Avoidance of high PIP and mean ventilatory pressure can
prevent the development of PIE.

171. The treatment of pulmonary interstitial emphysema (PIE) includes


all of the following except:
a) Selective intubation and ventilation of the involved lung and bronchus
b) High frequency ventilation
c) Oxygen
d) Bronchoscopy performed in patients with mucus plug
e) General respiratory care

172. The predominant source of bleeding in pulmonary hemorrhage is:


a) Tracheal
b) Bronchial
c) Interstitial
d) Laryngeal
e) Alveolar

173. The treatment of pulmonary hemorrhage includes all of the


following except:
a) Blood transfusion
b) Increased PEEP
c) Intratracheal administration of epinephrine
d) Suctioning to clear the airway
e) Increased PIP
173. (e) Increased PIP does not stop pulmonary hemorrhage. High
frequency ventilation (HFV) is useful to stop pulmonary hemorrhage in
some cases.

316. The preferred therapy for patients with recurrent respiratory


papillomatosis is:
a) Intralaryngeal sclerosing agent
b) Intralaryngeal epinephrine
c) Intralaryngeal corticosteroids
d) Reassurance
e) Excision with CO2 laser

317. A newborn developed noisy breathing and mild respiratory distress.


He was diagnosed with recurrent respiratory papillomatosis.
He acquired the disease from his mother:
a) At the time of delivery
b) In utero
c) Immediately after birth
d) During nursing
e) Within 7 days after birth

318. The recurrent respiratory papillomatosis (RRP) is caused by the


following types of human papillomavirus (HPV):
a) Types 1 and 3
b) Types 2 and 4
c) Types 6 and 11
d) Types 12 and 14
e) Types 13 and 15

319. All of the following environmental irritants can cause cough,


asthma, or chronic lung disease except:
a) Ozone
b) Nitrogen dioxide
c) Marijuana smoke
d) Tobacco smoke
e) Carbon dioxide
319. (e) Carbon dioxide and oxygen are not irritants; particulate matters
are also irritants.

794. The restrictive lung disease includes all of the following conditions
except:
a) Asthma
b) Pneumonia
c) Scoliosis
d) Pulmonary edema
e) Respiratory distress syndrome

795. The obstructive lung disease includes all of the following


conditions except:
a) Pulmonary fibrosis
b) Emphysema
c) Bronchiolitis
d) Meconium aspiration
e) Bronchopulmonary dysplasia

795. (a) Pulmonary fibrosis is a restrictive lung disease. Restrictive lung


disease also includes pneumothorax, pulmonary collapse, diaphragmatic
hernia, persistent pulmonary hypertension due to collapse, pulmonary
interstial emphysema, and pulmonary tuberculosis.

796. All of the following conditions cause fixed extrathoracic


obstruction except:
a) Laryngomalacia
b) Tracheal stenosis
c) Epiglottitis
d) Enlarged tonsils
e) Polyp of the vocal cord
796. (a) Laryngomalacia causes variable extrathoracic obstruction.
836. The preferred test for pulmonary function is:
a) Pulmonary scan
b) Pulmonary angiogram
c) Lung CT scan
d) Lung MRI
e) Arterial blood gas

837. The most common cause of acquired laryngotracheal stenosis is:


a) Infection
b) Reflux of gastric acid
c) Reflux of gastric pepsin
d) Bleeding
e) Endotracheal intubation
870. The most common organism in patients with Lemierre disease is:
a) Group A Streptococcus
b) S. aureus
c) Haemophilus influenzae
d) Klebsiella
e) Fusobacterium necrophorum

870. (e) Fusobacterium necrophorum (anaerobic oropharyngeal bacteria)

871. A 16-year-old healthy adolescent appears with sudden onset fever,


respiratory distress, and cough. He has been suffering from acute
tonsillopharyngitis for the last 3 days. The chest x-ray reveals multiple
bilateral cavitary nodules and mild pleural effusions. The preferred
therapy is:
a) Penicillin
b) Ciprofloxacin
c) Vancomycin
d) Gentamicin
e) Ceftazidime

871. (a) Lemierre disease is treated with intravenous penicillin or


cefoxitin; surgical drainage is indicated in patients with extrapulmonary
metastatic abscess.

872. The following medication should be avoided in patients with


influenza:
a) Acetaminophen
b) Ibuprofen
c) Aspirin
d) Antihistamine

893. A 15-yar-old healthy boy appears with a stabbing pain in the chest.
The pain radiates to the neck. He denies history of trauma. The
physical examination reveals subcutaneous emphysema. The most likely
diagnosis is:
a) Pneumothorax
b) Fracture ribs
c) Critical aortic stenosis
d) Myocardial infarction
e) Pneumomediastinum

894. A child is suspected to have recurrent aspirations. The initial study


should be:
a) Milk scan
b) Barium-swallow
c) Upper GI series
d) Ultrasonography
e) Plain chest x-ray

895. A 3-month-old child with SIDS (sudden infant death syndrome)


was brought to the ER. The physical examination most likely reveals:
a) External bruises
b) Enlarged liver
c) Enlarged spleen
d) Proptosis
e) Petechial hemorrhage
921. The methacoline challenge testing is performed to diagnose a
suspected case of:
a) Food allergy
b) Bronchial asthma
c) Cystic fibrosis
d) Tuberculosis
e) Cardiac arrythmias
926. If a mother is PPD positive and has a negative chest x-ray, the
newborn infant should be:
a) Separated from the mother
b) Given INH prophylaxis
c) Evaluated with a chest radiography
d) Evaluated with a PPD testing
e) With the mother

927. A mother has an active pulmonary tuberculosis. All of the


following statements are true about management of the newborn except:
a) The newborn should receive INH therapy.
b) The newborn should be isolated from the mother regardless of her
symptoms during INH therapy.
c) The newborn should be isolated from the mother if she has drug-
resistant tuberculosis, she is noncompliance, and she is ill enough to
require hospitalization.
d) The newborn should receive INH therapy until the mother’s sputum
cultures are negative for at least 3 months.
e) The newborn should receive a Mantoux tuberculin skin test after 3
months of age. If positive, INH should be continued for a total duration
of 9-12 months.
927. (b) The INH treatment of the newborn is very effective. Therefore,
a separation of the mother and infant is no longer mandatory if the
mother is asymptomatic.

928. The preferred therapy for a pregnant woman with an active


pulmonary tuberculosis is a combination of:
a) INH, pyrazinamide, and rifampin
b) INH, ethionamide, and ethambutol
c) Rifampin, ethambutol, and ethionamide
d) INH, rifampin, and ethambutol
e) Streptomycin, INH, and rifampin

928. (d) INH, rifampin, and ethambutol are preferred therapies;


aminoglycosides and ethambutol are teratogenic; the use of
pyrazinamide may not be safe during pregnancy.

929. A 12-year-old boy appears with a persistent cough, headache, fever,


malaise, and hoarseness for the last 6 days. Coryza is absent.
He produces a frothy, white sputum. Initially the cough was
nonproductive. The physical examination reveals a fine crackles on the
right chest. The chest x-ray reveals right lower lobe interstitial
infiltrates. The preferred therapy for this patient is:
a) Symptomatic therapy
b) Ampicillin
c) Cefotaxime
d) Ceftriaxone
e) Azithromycin

929. (e) Azithromycin, erythromycin, or clarithromycin is used in


Mycoplasma pneumonia. Coryza is usually present in viral infections.
Pneumonia in a school–aged children, especially if cough is a persistent
finding, is always indicate Mycoplasma pneumonia.

930. Neurologic complications of patients with Mycoplasma pneumonia


includes all of the following except:
a) Guillain-Barre syndrome
b) Bell palsy
c) Transverse myelitis
d) Hyperacusis
e) Aseptic meningitis
930. (d) Deafness, cerebellar ataxia, brainstem syndrome and acute
demyelinating encephalitis are also neurologic complications of
Mycoplasma pneumonia

1. The most common site of pulmonary atelactasis in children is:


a) Right upper lobe
b) Right middle lobe
c) Right lower lobe
d) Left upper lobe
e) Left lower lobe
26. Tonsillectomy is indicated in all of the following conditions except:
a) A patient who experienced six tonsillar infections that were treated
with antibiotics in the preceding year.
b) A patient who experienced five tonsillar infections that were treated
with antibiotics in each of the preceding two years.
c) A patient who experienced three tonsillar infections that were treated
with antibiotics in each of the preceding three years.
d) A patient who experienced four tonsillar infections that were treated
with antibiotics in each of the preceding three years.
e) A patient who experienced seven tonsillar infections that were treated
with antibiotics in the preceding year.

26. a) Tonsillectomy is indicated in a patient with seven or more


tonsillar infections that were treated with antibiotics in the preceding
year,
five or more tonsillar infections that were treated in each of the
preceding 2 years,
or three or more tonsillar infections that were treated in each of the
preceding 3 years.

27. The most common organism in patients with a empyema (purulent


pleurisy) is:
a) Staphylococcus aureus
b) Group A Streptococcus
c) Tuberculosis
d) E. coli
e) Streptococcus pneumoniae

28. The most common organism in patients with a post-traumatic


empyema is:
a) S. pneumoniae
b) S. aureus
c) E. coli
d) Pseudomonas
e) S. epidermidis

53. The most common foreign body in patients with aspiration is:
a) Popcorn
b) Dried beans
c) Sunflower seeds
d) Watermelon seeds
e) Nuts

54. A child appears with hemoptysis. A screening method includes all of


the following tests except:
a) Complete blood counts
b) Chest x-ray
c) Prothrombin time
d) Chest CT scan
e) Partial thromboplastin time
54. d) CT-scan is not a screening tool but may be useful to make a
diagnosis. Bronchoscopy is both diagnostic and therapeutic. A rigid
bronchoscopy is used to remove foreign body. A flexible bronchoscopy
is used in patients with noncopious bleeding.

102. A child is intubated for respiratory failure. An objective of


mechanical ventilation is the following:
a) Not to normalize arterial blood gas tension
b) Not to maintain an adequate gas exchange
c) Oxygen saturation should be maintained between 95-100%.
d) Arterial PCO2 should be maintained between 35-45 mm Hg.
e) Arterial pH should be maintained between 7.35-7.45.

102. a) An objective of mechanical ventilation is not to normalize


arterial blood gas tension but to maintain adequate gas exchange (i.e.,
some degree of hypoxia, O2 saturation 85-90%; moderate hypercarbia,
PCO2 60-80 mm Hg are acceptable if the patient’s condition is stable).

103. All of the following organisms cause acute pharyngitis in children


except:
a) Virus
b) Group A beta-hemolytic Streptococcus
c) Group C Streptococcus
d) Haemophilus influenzae
e) Arcanobacterium hemolyticum

103. d) Haemophilus influenzae and Streptococcus pneumoniae may be


cultured from throat but their role in causing pharyngitis is not clear.
Viruses are the most common cause of pharyngitis. However, group A-
beta-hemolytic Streptococcus (GABHS) is the most common bacterial
cause of pharyngitis.

109. Posteroanterior x-ray of a neck reveals a “steeple sign”. The most


likely diagnosis is:
a) Fracture clavicle
b) Fracture cervical vertebra
c) Croup
d) Laryngeal polyps
e) Tracheoesophageal fistula

109. c) “Steeple sign” represents typical subglottic narrowing that is


present in a patient with croup. This sign may be absent in croup or may
be present in a normal child or a patient with subglottic stenosis.

110. Lateral x-ray of the upper airway in a 2-year-old boy reveals a


“thumb sign”. The most likely diagnosis is:
a) Laryngeal polyps
b) Epiglottitis
c) Subglottic hemangioma
d) Laryngeal edema
e) Subglottic stenosis

117. The first symptom noted in patients with a common cold is:
a) Nasal congestion
b) Cough
c) Fever
d) Sore throat
e) Runny nose

118. A 7-year-old boy appears with cough, mild fever, nasal obstruction,
and rhinorrhea for the last 5 days. The symptoms began with sore throat
about 7 days ago. The mother also noted a change in the color and
consistency of the nasal secretions. A physical examination reveals a
stuffy nose, swollen, erythematous nasal turbinates, and mildly injected
throat. The next step in management
is:
a) Amoxicillin
b) Erythromycin
c) Cefuroxime
d) Amoxicillin-clavulanate
e) Symptomatic therapy

118. e) Symptomatic therapy is indicated for patients with a common


cold. Please remember, a change in color and consistency of the nasal
secretions does not indicate a sinusitis or bacterial infection.

119. A nasal secretion of a child reveals predominant


polymorphonuclear leukocytes. The most likely diagnosis is:
a) Rhinovirus infection
b) Haemophilus influenzae infection
c) Allergic rhinitis
d) Moraxella catarrhalis infection
e) Group A beta-hemolytic Streptococcus infection

119. a) Rhinovirus; the presence of polymorphonuclear leukocytes does


not mean a bacterial superinfection.

120. A nasal secretion of a child reveals predominant eosinophils. The


most likely diagnosis is:
a) Rhinovirus infection
b) Streptococcus pneumoniae infection
c) Influenza virus infection
d) Respiratory syncitial virus
e) Allergic rhinitis

138. A child appears with a drooling, trismus, unilateral throat pain, and
ipsilateral referred otalgia. A physical examination reveals the right
tonsil displaced medially and downward by the ipsilateral swollen palate
and anterior tonsillar pillar. The preferred diagnostic study is:
a) Throat culture
b) ASLO titer
c) CT scan of the affected area
d) Lateral x-ray of the neck
e) X-ray of maxillary sinuses
138. c) CT scan of an affected area or needle aspiration and culture can
make the diagnosis of peritonsillar abscess.

139. The preferred therapy for patients with a retropharyngeal or lateral


pharyngeal abscess without respiratory distress is:
a) Surgical drainage and penicillin
b) Azithromycin and gentamicin
c) A third-generation cephalosporin and penicillin
d) Nafcillin and surgical drainage
e) A third-generation cephalosporin and ampicillin-sulbactum

139. e) A third-generation cephalosporin and ampicillin-sulbactum or


clindamycin; surgical drainage is indicated in a patient with respiratory
distress or failure to improve with antibiotic therapy.

140. The preferred therapy for patients with a retropharyngeal or lateral


pharyngeal abscess with respiratory distress is:
a) Surgical drainage and penicillin
b) Surgical drainage and ampicillin
c) Surgical drainage and ampicillin-sulbactum
d) Surgical drainage and clindamycin
e) Surgical drainage and a third-generation cephalosporin and
clindamycin

148. Clinical uses of pulmonary function testing (PFT) include all of the
following except:
a) PFT usually makes the specific diagnosis.
b) PFT can detect an obstructive lung disease.
c) PFT can detect a restrictive lung disease.
d) PFT can detect a degree of functional impairment.
e) PFT is useful in a preoperative evaluation.

148. a) PFT rarely makes a diagnosis. PFT can detect a functional


impairment in patients with a normal physical examination but minor
complaint. PFT is useful in determining responses of a bronchodilator in
patients with an obstructive lung disease.

149. All of the following conditions are true in patients with a cystic
fibrosis (CF) except:
a) Autosomal recessive
b) Chromosome 7
c) Chronic obstructive lung disease
d) Pancreatic insufficiency
e) The most prevalent mutation of the CF transmembrane regulator
(CFTR) is the deletion of a single tyrosine residue at amino acid 508.

149. e) A deletion of a single phenylalanine residue at amino acid 508.

160. The following statement is not true about restrictive lung disease:
a) The duration of inspiration is increased.
b) The duration of expiration is reduced.
c) The chest x-ray reveals a decreased lung volume.
d) The physical examination reveals grunting and crackles.
e) The amplitude of respiratory movements is shallow.

160. a) Duration of inspiration is reduced. Respiratory rate is increased.


Inspiratory muscles are accessory muscles. Retractions are present.

161. All of the following statements are true about extrathoracic


obstructive disease except:
a) Duration of inspiration is reduced.
b) Respiratory rate is decreased.
c) Duration of expiration is unchanged.
d) Physical examination reveals a inspiratory stridor.
e) Chest x-ray findings are normal.
161. a) Duration of inspiration is prolonged. Inspiratory muscles are
accessory muscles. Retractions are present. Amplitude of respiratory
movements are normal or decreased.
162. The following statement is not true about the intrathoracic
obstructive lung disease except:
a) Duration of inspiration is prolonged.
b) Respiratory rate is either normal or increased.
c) Duration of expiration is prolonged.
d) A physical examination reveals expiratory wheezes.
e) A chest x-ray reveals an increased lung volume.

162. a) Duration of inspiration is unchanged. Both inspiratory and


expiratory (abdominal) muscles are accessory muscles.
Retractions are present. Amplitude of respiratory movements are either
normal or reduced.

163. The most common presentation in patients with a tracheal foreign


body is:
a) Wheezing
b) Stridor
c) Positive chest x-ray
d) Choking and aspiration
e) Positive soft tissue x-ray of the neck

163. e) Positive posteroanterior and lateral soft tissue x-ray of the neck
(92% of cases); answer (d) is present in 90% of cases; answer (b) is
present in 60% of cases; answer (c) is present in 58% of cases; answer
(a) is present in 50% of cases

164. A 2-month-old infant appears with a brassy cough. The most likely
diagnosis is:
a) Subglottic hemangioma
b) Bronchiolitis
c) URI
d) Chlamydia infection
e) Vascular ring
164. e) Vascular ring causes brassy cough. Brassy cough is also noted in
patients with tracheitis or habit cough. Brassy cough and stridor are
noted in patients with a laryngeal obstruction or pertussis

165. A child appears with a staccato cough. The most likely diagnosis is:
a) Habit cough
b) Subglottic hemangioma
c) Laryngomalacia
d) Croup
e) Chlamydia pneumonitis

166. A 10-year-boy appears with a history of cough only during the


daytime for the last 2 months. However, he sleeps well at night without
coughing. The boy is coughing during the physical examination. A
physical examination reveals a normal throat and clear breath sounds in
both lungs. The most likely diagnosis is:
a) Chlamydia infection
b) Bronchial asthma
c) Sinusitis
d) Habit cough
e) Carrier of group A Streptococcus

167. A congenital subglottic hemangioma is associated with the


following anomaly:
a) Renal
b) CNS
c) Liver
d) Spleen
e) Cutaneous

167. e) Cutaneous lesions are present in 50% of cases.

435. A child who is a known asthmatic appears with a sudden onset of


cough, dyspnea, tachypnea, shallow respiration, and tachycardia.
The child has no fever. A physical examination reveals a mild cyanosis,
decreased breath sounds and crackles over the right upper thoracic
cavity. The most likely diagnosis is:
a) Right pneumothorax
b) Left pneumothorax
c) Right upper lobe pneumonia
d) Right upper lobe atelactasis
e) Right lower lobe bronchiectasis

435. d) Pulmonary atelactasis can occur in asthma, pneumonia, and


trauma. Massive collapse of one or both lungs most commonly occurs
after a surgery. The most common site of atelactasis is the right upper
lobe.

436. A child appears with a moderate amount of bleeding through the


mouth. All of the following findings indicate a hemoptysisvexcept:
a) Bright red color
b) Frothy
c) Nausea
d) Accompanied by cough
e) Alkaline pH

437. A child appears with a massive bleeding through the mouth and
nose. The following finding does not indicate a hematemesis:
a) Dark red blood
b) Acidic pH
c) Nausea
d) Frothy
e) Presence of food particles

438. A child appears with a fever, cough, sputum production, tachypnea,


dyspnea, hemoptysis, and weight loss. A physical examination reveals
intercostals retractions, crackles, decreased breath sounds, and dullness
to percussion in the right upper chest area. The chest x-ray reveals
parenchymal inflammations with a cavity containing air-fluid level in
the right upper lobe. The most likely diagnosis is:
a) Right upper lobe TB
b) Right upper lobe pneumonia
c) Right upper lobe abscess
d) Right upper lobe cyst
e) Right upper lobe obstruction

439. The preferred diagnostic study in patients with a pulmonary abscess


is:
a) Chest x-ray
b) Lung scan
c) Lung CT scan
d) Bronchoscopy
e) Sputum culture

440. The following statement is not true about pulmonary aspirations


and abscesses:
a) A secondary lung abscess is more common in the right side,
particularly in immunocompromised children.
b) A primary lung abscess is more common in the right side.
c) If the child is upright, the posterior segments of upper lobes are
affected.
d) If the child is in recumbent position, left and right upper lobes are
affected.
e) In a recumbant position, apical segment of the right upper lobe is
affected.

440. a) A secondary lung abscess is more common on the left side

541. A 2-year-old boy appears in the ER with a history of foreign body


inside the nose. A physical examination reveals a small perforation of
the nasal septum. The foreign body is removed. The most likely foreign
body is:
a) Bead
b) Bean
c) Crayon
d) Stones
e) Disk battery

541. e) Disk batteries cause pain and local tissue destructions within a
matter of hours

542. A child had a nasal operation that required nasal packing. A follow-
up appointment is given in ENT clinic. The mother missed the
appointment. Subsequently, the child became ill, developed fever and a
bad smell from the nasal packing area. The most serious complication is:
a) Dislodgement of the packing
b) Excessive bleeding
c) Apnea
d) Brain abscess
e) Toxic shock syndrome

542. e) Toxic shock syndrome most commonly occurs from nasal


surgical packing

543. A 5-year-old girl appears with a purulent unilateral nasal discharge


for the last 3 days. A physical examination reveals unilateral purulent
secretions but no foreign body is visualized. The next step in
management is:
a) Amoxicillin-clavulanate
b) Cefuroxime
c) Azithromycin
d) Reassurance
e) Suctioning the secretions

543. e) Suctioning the secretions is indicated; the foreign body is usually


located behind the secretions; often a topical decongestant and headlight
are needed to remove the foreign body.
544. A routine newborn physical examination at birth reveals a nasal
septal deviated to the right. The most common cause is:
a) Choanal stenosis
b) Choanal atresia
c) Congenital syphilis
d) Frontal encephalocele
e) Trauma from delivery

545. The preferred management for newborns with a deviated nasal


septum secondary to trauma during delivery is:
a) Reassurance
b) ENT outpatient appointment after 7 days
c) ENT outpatient appointment after 1 month
d) CT scan of the nose
e) Immediate realignment using blunt probes cotton applicators, and
topical anesthesia

546. The following statement is not true in patients with a pulmonary


sequestration:
a) The majority of sequestrations are intrapulmonary.
b) Extrapulmonary sequestrations are almost always involve the left lung
c) Extrapulmonary sequestrations are strongly associated with a
diaphragmatic hernia.
d) Surgical removal is indicated.
e) The arterial supply in sequestration comes from a pulmonary artery
and venous drainage returns to the left atrium through pulmonary veins.

546. e) The arterial supply in sequestration comes from systemic


arteries, especially from the aorta and venous drainage returns to the
right atrium, especially through inferior vena cava.

547. The most common presentations in patients with a primary ciliary


dyskinesia are:
a) Situs inversus, otitis media, and chronic sinusitis
b) Chronic sinusitis, otitis media, and wheezing
c) Bronchiectasis, chronic sinusitis, and asthma
d) Asthma, pneumonia, and bronchiectasis
e) Productive cough, sinusitis and otitis media

547. e) Productive cough, sinusitis, and otitis media are present in


almost all patients; about 50% of patients have Kartagener syndrome
(i.e., situs inversus, otitis media, chronic sinusitis and airway disease
leading to bronchiectasis). Primary ciliary dyskinesia is known as
immotile cilia syndrome

548. The mode of inheritance in patients with a primary ciliary


dyskinesia is:
a) Autosomal recessive
b) Autosomal dominant
c) X-linked recessive
d) X-linked dominant
e) Unknown

549. The gold standard to make the diagnosis of primary ciliary


dyskinesia is:
a) CT scan of paranasal sinuses
b) Pulmonary function tests
c) CT scan of lungs
d) Ultrasonography of sinuses
e) Nasal biopsy

549. e) Nasal (or bronchial) biopsy or scraping reveals quantitative


documentation of missing dynein arms or random orientation of cilia
under electron microscopic examination.

550. A pulmonary function test in older children with a primary ciliary


dyskinesia reveals:
a) Normal
b) Restrictive lung disease
c) Obstructive lung disease
d) Both restrictive and obstructive lung diseases with an equal frequency
e) More restrictive than an obstructive lung disease

550. c) Obstructive lung disease

551. The cardiac output in a normal healthy newborn is about:


a) 75 mL/kg/minute
b) 150 mL/kg/min
c) 200 mL/kg/minute
d) 300 mL/kg/min
e) 350 mL/kg/minute

590. An adolescent boy appears with a recurrent, severe nose bleeds for
the last 6 months. He experienced several bleeding episodes more
frequently for the last 1 month. He denies history of a major trauma.
However, he has a long nose and often experienced minor injuries. A
physical examination reveals a nasal mass. He experienced a major
laceration in the right leg but the bleeding sopped spontaneously. The
most likely diagnosis is:
a) Hemophilia A
b) Nasal septal hematoma
c) von Willebrand disease
d) Kiesselbach plexus bleeding
e) Juvenile nasopharyngeal angiofibroma

590. e) Juvenile nasopharyngeal angiofibromas peak in adolescent and


preadolescent boys and also noted less than 2 years of age. He has no
bleeding disorder, therefore, answers (a) and (c) are wrong. Answer (d)
is wrong because the incidence of Kiesselbach plexus bleeding decreases
during adolescence.

591. The preferred initial diagnostic study in patients with a juvenile


nasopharyngeal angiofibroma is:
a) AP x-ray of the nose
b) Lateral x-ray of the nose
c) Excisional biopsy
d) Examination under anesthesia
e) CT scan with contrast

591. e) CT scan with contrast or MRI is the initial procedure of choice.


Arteriography, embolization, and extensive surgery may be required.

592. The preferred therapy to eradicate streptococcal carriage is:


a) Penicillin
b) Amoxicillin
c) Erythromycin
d) Clindamycin
e) Azithromycin

593. A 2-year-old boy appears with fever, sore throat, neck pain,
drooling, and decreased oral intake for the last 48 hours. A physical
examination reveals neck stiffness, torticollis, muffled voice, and
bulging of the posterior pharyngeal wall. The most likely diagnosis is:
a) Laternal pharyngeal abscess
b) Peritonsillar abscess
c) Acute epiglotittis
d) Meningitis
e) Retropharyngeal abscess

593. e) Please remember, retropharyngeal nodes involutes after 5 years


of age. Retropharyngeal abscess is common in less than 3-4 years of age
and presents with bulging of posterior pharyngeal wall in less than 50%
of cases. The patient may also appear with stridor, respiratory distress,
and refuse to move the neck.

594. A 2½-year-old girl appears with a fever, dysphagia, and enlarged


cervical lymph nodes. A physical examination reveals a bulging of the
left lateral pharyngeal wall and medial displacement of the left tonsil.
The most likely diagnosis is:
a) Acute adenoiditis
b) Peritonsillar abscess
c) Acute pharyngitis
d) Acute left tonsillitis
e) Left lateral pharyngeal abscess

595. A 12-year-old boy appears with a fever, sore throat, dysphagia, and
trismus for the last 2 days. He was diagnosed with a viral acute
tonsillopharyngitis 5 days ago. A physical examination reveals a right
tonsillar bulging with displacement of the uvula to the left.
The most likely diagnosis is:
a) Right peritonsillar abscess
b) Left peritonsillar abscess
c) Retropharyngeal abscess
d) Acute right tonsillitis
e) Acute uvulitis

595. a) Right peritonsillar abscess; this is common in adolescents. There


is usually the recent history of acute tonsillopharyngitis. An assymmetric
tonsillar bulging is diagnostic.

596. A 3-year-old girl appears in the ER with a history of sudden


development of fever and sore throat for the last 6 hours. She developed
dysphagia, drooling, and dyspnea for the last 1 hour. The child is sitting
upright, learning forward with the chin up, and opening mouth while
bracing on the arms. She was completely asymptomatic prior to this
episode. Her immunization status in unknown. Recently, her parents
immigrated from a developing country. The most likely diagnosis is:
a) Acute tonsillar abscess
b) Acute epiglottitis
c) Acute retropharyngeal abscess
d) Acute adenoiditis
e) Acute peritonsillar abscess
596. b) Acute epiglottitis; less common in the USA due to H. influenzae
type b vaccinations but still common in unimmunized or
underimmunized children between 2-4 years of age.

597. A 2-year-old girl appears in the ER at night with a sudden onset of


barking, metallic cough, and respiratory distress for the last 2 hours. She
experienced a mild coryza and hoarseness for the last 24 hours. The
child is asymptomatic during the day except for the cough and
hoarseness. She experienced a similar episode about 2 weeks ago.
Mother denies history of fever. The child appears anxious and
frightened. A physical examination reveals bilateral noisy inspiratory
sounds and mild intercostal retractions. The most likely diagnosis is:
a) Spasmodic croup
b) Acute infections laryngitis
c) Bacterial tracheitis
d) Laryngomalacia
e) Acute infectious laryngotracheobronchitis

597. a) The patients with spasmodic croup presents without fever and
viral infections. The allergy and psychological factors are important. The
children are usually symptomatic during the evening and nights.
Laryngoscopy usually reveals pale, watery edematous larynx but normal
epithelium. In acute infectious laryngotracheobronchitis, the laryngeal
epithelium appears erythematous, edematous, and destroyed.

598. The most common organism in patients with a bacterial tracheitis


is:
a) Nontypable H. influenzae
b) Moraxella catarrhalis
c) Streptococcus pneumoniae
d) Pseudomonas aeruginosa
e) Staphylococcus aureus
598. e) S. aureus; answers (a) and (b) also cause bacterial tracheitis.

599. The most common organism in patients with an infectious upper


airway obstruction is:
a) Mycoplasma pneumoniae
b) H. influenzae type b
c) Streptococcus pyogenes
d) Influenza viruses
e) Parainfluenza viruses

599. e) Parainfluenza viruses (types 1, 2, and 3) are responsible for 75%


of cases.

600. The most common organism in patients with an epiglottitis and


underimmunized or unimmunized is:
a) Streptococcus pyogenes
b) Streptococcus pneumoniae
c) Mycoplasma pneumoniae
d) Parainfluenza viruses
e) Haemophilus influenzae type b

601. The most common organism in patients with an epiglottis and


immunized is:
a) Haemophilus influenzae type b
b) Staphylococcus epidermidis
c) Streptococcus pyogenes
d) Pseudomonas aeruginosa
e) Parainfluenza viruses

601. c) Streptococcus pyogenes, S. pneumoniae, and S. aureus are the


most common organisms.

602. The most common organism in patients with a croup


(laryngotracheobronchitis) is:
a) Influenza type A
b) Adenovirus
c) Respiratory syncitial virus
d) Mycoplasma pneumoniae
e) Parainfluenzae viruses

602. e) Parainfluenzae viruses (types 1 and 2); type 3 causes


bronchiolitis and pneumonia.

603. A 3-year-old boy appears with a barking cough, hoarseness, low-


grade fever, and dyspnea for the last 6 hours. The child has been
suffering from rhinorrhea, mild cough, pharyngitis, and fever for the last
3 days. His condition worsens with agitation and crying. A physical
examination reveals tachypnea, mild retractions, inflamed pharynx,
coryza, and hoarse voice. The most likely diagnosis is:
a) Acute epiglottitis
b) Spasmodic croup
c) Croup
d) Acute pharyngitis
e) Acute infectious laryngitis

805. The most common congenital laryngeal anomaly in infant is:


a) Laryngomalacia
b) Subglottic stenosis
c) Subglottic hemangioma
d) Laryngeal web
e) Vocal cord paralysis

806. A full term newborn appears with low pitched, inspiratory stridor at
birth. The stridor worsens with crying, agitation, and feeding. The
newborn is NPO and receiving intravenous fluids. He was born by an
elective cesarean section with Apgar scores are 8 and 9 at 1 and 5
minutes respectively. The most likely diagnosis is:
a) Vocal cord paralysis
b) Laryngeal web
c) Laryngeal atresia
d) Laryngeal nodule
e) Laryngomalacia

806. e) The infants with laryngomalacia has a low pitched, inspiratory


stridor, that worsens with crying, agitation, and feeding. The infants with
vocal cord paralysis, laryngeal web, and laryngeal atresia usually appear
with high pitched cry.

807. The preferred diagnostic study in infants with laryngomalacia is:


a) AP x-ray of the neck
b) Lateral x-ray of the neck
c) Flexible laryngoscopy
d) Barium swallow of the esophagus
e) Flexible bronchoscopy

810. The pulmonary arteriovenous fistulas are most commonly present


is:
a) Williams syndrome
b) Down syndrome
c) Turner syndrome
d) Prader-Willi syndrome
e) Osler-Weber-Rendu syndrome

810. e) Osler-Weber-Rendu syndrome (hereditary hemorrhagic


telangiectasia type 1) is associated with angiomas of the GI tract, liver,
nasal and buccal mucous membranes.

Match the flow-volume curve in different respiratory conditions (924-


928):
924. Bronchopulmonary dysplasia e) Prolonged expiration phase

924. e) Prolonged expiration phase (i.e., obstructive type) e.g., BPD,


asthma, bronchiolitis, emphysema, and meconium aspiration.
925. Respiratory distress syndrome d) Reduced inspiratory and
expiratory phases looks like a small normal curve

925. d) Reduced inspiratory and expiratory phases (e.g., restrictive type)


e.g., RDS, pneumothorax, pneumonia, collapse, diaphragmatic hernia,
persistent pulmonary hypertension due to collapse, pulmonary interstitial
emphysema, and TB.

926. Epiglottitis
927. Laryngomalacia
928. Malacia of intrathoracic trachea
a) Reduced inspiratory and expiratory phases looks like a sandwich
b) Reduced inspiratory flow and plateau of inspiratory loop
c) Reduced and flattened expiratory loop
d) Reduced inspiratory and expiratory phases looks like a small normal
curve
e) Prolonged expiration phase

Match the characteristic of sinusitis and site of pain/headache (955- 959):


955. Frontal e) Above and medial to eyes

956. Sphenoidal a) Suboccipital area

957. Anterior ethmoidal b) Temporal area and eyes

958. Posterior ethmoidal c) Trigeminal area particularly mastoid

959. Maxillary d) Over maxilla and teeth

a) Suboccipital area
b) Temporal area and eyes
c) Trigeminal area particularly mastoid
d) Over maxilla and teeth
e) Above and medial to eyes

Match different types of cough and diseases (965-969):


965. Tracheitis c) Brassy
966. Chlamydia pneumonitis e) Staccato

967. Laryngeal obstruction a) Cough with stridor


968. Habit cough b) Disappears with sleep

969. Allergic reaction d) Noctural cough

a) Cough with stridor


b) Disappears with sleep
c) Brassy
d) Noctural cough
e) Staccato

Match different characteristics of cough and diseases (970-974):


970. Bronchitis e) Loose (discontinuous), productive

971. Cystic fibrosis a) Most severe on awakening in morning

972. Reactive airways d) Tight (wheezy)

973. Pertussis airway c) Paroxysmal

974. Asthma b) With vigorous exercise

a) Most severe on awakening in morning


b) With vigorous exercise
c) Paroxysmal
d) Tight (wheezy)
e) Loose (discontinuous), productive

46. A 5-month-old boy appears with respiratory distress for the last 24
hours. He had two episodes of pneumonia and four episodes of otitis
media and six episodes of diarrhea. He is delayed for his growth and
development. A physical examination reveals bilateral rales and right
otitis media. All of the following findings are true for this patient except:
a) Total CBC lymphocyte counts are 1600/mm3.
b) Chest x-ray reveals bilateral infiltrates.
c) Blood culture is positive for Streptococcus pneumoniae.
d) Serum immunoglobulin levels are elevated.

212. The most common cause of bronchiectasis in the developed


countries is:
a) Cystic fibrosis
b) Ciliary dyskinesia
c) TB
d) Measles
e) Right middle lobe syndrome

213. The preferred diagnostic study in patients with bronchiectasis is:


a) Chest x-ray (AP view)
b) Bronchoscopy
c) Chest x-ray (lateral view)
d) Bronchography
e) CT scan

213. e) Thin-section high-resolution CT scanning has replaced


bronchography as the gold standard. CT scan typically reveals
cylindrical (“tram lines,” “signet ring appearance”), varicose (bronchi
with “beaded contour”), cystic (cysts appear in “strings and clusters”),
or mixed types.
214. The most common combination of pathogens causing pneumonia
is:
a) RSV and Mycoplasma pneumoniae
b) RSV and Chlamydia trachomatis
c) Streptococcus pneumonia and S. aureus
d) Streptococcus pneumonia and C. trachomatis
e) Streptococcus pneumonia and RSV

214. e) Streptococcus pneumoniae with either RSV or Mycoplasma


pneumonia

215. The most common causes of empyema are:


a) S. aureus and S. pneumoniae
b) S. aureus and Pseudomonas aeruginosa
c) S. pneumoniae and RSV
d) S. pneumoniae and Mycoplasma pneumoniae
e) S. pneumoniae and Chlamydia trachomatis

216. The top five most common clinical manifestations in patients with
cystic fibrosis are the following except:
a) Acute or persistent respiratory symptoms
b) Failure to thrive (or malnutrition)
c) Abnormal stools
d) Meconium ileus
e) Nasal polyps

216. e) Nasal polyps or sinus diseases occur less frequently than rectal
prolapse, electrolytes or acid-base abnormalities. Answers
(a), (b), (c), and (d) are in the order of decreasing frequencies (i.e.,
answer (a) is the most common presentation).
Hepatobiliary diseases occur less frequently than nasal polyps

217. The parents noticed salty taste when they kiss the child. The most
likely diagnosis is:
a) Facial eczema
b) Acrodermatitis enteropathica
c) SLE
d) Cystic fibrosis
e) Ectodermal dysplasia

218. The diagnostic criteria in patients with cystic fibrosis (CF) include
all of the following except:
a) Presence of typical clinical manifestations and two elevated sweat
chloride concentrations (60 mEq/L or more) performed on separate days
b) A history of cystic fibrosis in a sibling and identification of two CF
mutations
c) A positive newborn screening test and two elevated sweat chloride
concentrations performed on separate days
d) A history of CF in a sibling and an abnormal nasal potential
difference measurement
e) A history of CF in a sibling and positive newborn screening test

218. e) Diagnostic criteria for cystic fibrosis are the following:


Presence of typical clinical manifestations (respiratory, GI, or GU) or
history of CF in a sibling or positive newborn
screening test
PLUS
Laboratory evidence for CFTR (cystic fibrosis transmembrane regulator)
dysfunction: Two elevated sweat chloride concentrations performed on
separate days or identification of two CF mutations or an abnormal nasal
potential difference measurement

219. The following statement is not true in patients with cystic fibrosis:
a) Sexual development is often delayed.
b) Sexual function is usually impaired.
c) Majority of males are azoospermic.
d) Female fertility rate is reduced.
e) Females may have cervicitis and accumulation of tenacious mucus in
the cervical canal.
219. b) Sexual function is usually unimpaired. Sexual development is
often delayed by an average of 2 years. Females may experience
secondary amenorrhea. Pregnancy is well tolerated in CF women with
good pulmonary functions.

220. All of the following complications are due to use of


aminoglycosides in patients with cystic fibrosis except:
a) Hypomagnesemia
b) Hyperuricemia
c) Hearing loss
d) Vestibular dysfunction
e) Renal tubular dysfunction

220. b) Hyperuricemia and colonic stricture can occur due to the use of
very large doses of pancreatic extracts.

221. A child has cystic fibrosis. The PFTs (pulmonary function tests) are
not performed until:
a) 1-2 years of age
b) 3-4 years of age
c) 5-6 years of age
d) 7-8 years of age
e) 9-10 years of age

221. c) 5-6 years of age; by this time most patients have obstructive
pulmonary disease. A decrease in mid-maximal flow rate is an early
finding that suggests small airway obstructions.

222. The following statement is not true about pulmonary function tests
in patients with cystic fibrosis:
a) Restrictive lung changes occur early in the disease process.
b) Obstructive lung changes typically present by 5-6 years of age.
c) Small airway obstruction is manifested early by decrease in mid-
maximal flow rate
d) Residual volume and functional residual capacity are increased early
in the course of the disease.
e) Presence of an obstructive lung disease and modest response to
bronchodilator are diagnostic of cystic fibrosis.

222. a) Restrictive lung changes occur late in the disease process (i.e.,
decreased total lung capacity and vital capacity) and are due to extensive
lung injuries and fibrosis

223. The presence of the following organisms on culture of lower


airways or sputum is diagnostic of cystic fibrosis in children:
a) Streptococcus pneumoniae
b) Mucoid forms of Pseudomonas
c) Mycoplasma pneumoniae
d) RSV
e) Chlamydia trachomatis

223. b) Mucoid forms of pseudomonas; the presence of Staphylococcus


aureus, Pseudomonas aeruginosa, or Burkholderia cepacia organism
suggests CF

752. The most common mechanism of arterial hypoxemia in pulmonary


disease is:
a) Ventilation-perfusion mismatch
b) Ventilation failure
c) Perfusion abnormalities
d) Arterial obstruction
e) Venous obstruction

752. a) Ventilation-perfusion mismatch (or decreased) i.e., alveolar PO2


is lower (e.g., 60) than normal (e.g., 100), therefore blood that passes
through this unit of alveoli achieve lower (e.g., 89%) than normal (e.g.,
95-100%) oxygen saturation. Supplemental oxygen therapy increases
alveolar PO2 and arterial PO2.

753. All of the following statements are true in infants who are sleeping
supine or on their side except:
a) They develop less fever at 1 month of age.
b) They develop less stuffy nose at 6 months of age.
c) They have less trouble sleeping at 6 months of age.
d) They have less outpatient visits for ear infection at 1 and 2 months for
sleeping supine.
e) They have less outpatient visits for ear infection at 3 months for
sleeping on their side.

753. d) Infants have less outpatient visits for ear infection at 3 and 6
months for those who are sleeping supine.

754. The preferred therapy in patients with antral choanal polyps is:
a) Surgery
b) Intranasal steroid sprays
c) Systemic steroids
d) Local decongestants
e) Systemic decongestants

755. The preferred therapy in patients with nasal polyps is:


a) Intranasal steroid sprays
b) Systemic steroids
c) Local decongestants
d) Systemic decongestants
e) Functional endoscopic sinus surgery

755. e) Functional endoscopic sinus surgery includes removal of polyps


and other associated nasal disease. Answers (a) and (b) provide some
shrinkage in nasal polyps. Answers (c) and (d) do not shrink nasal
polyps but reduce mucosal edema.
756. The most common organism for common cold is:
a) Coronavirus
b) Adenovirus
c) Influenza virus
d) Rhinovirus
e) Respiratory syncytial virus

757. The most common complication of cold is:


a) Sinusitis
b) Asthma
c) Bronchiolitis
d) Croup
e) Otitis media

758. The best method of prevention of common cold is:


a) Influenza vaccine
b) Vitamin C
c) Echinacea
d) Good handwashing
e) Chicken soup

Match all the different diseases and their clinical manifestations (759-
763):
759. Pertussis a) Persistent or paroxysmal cough

760. Allergic rhinitis d) Itching, sneezing, and nasal eosinophilia

761. Streptococcal nasopharyngitis b) Nasal discharge that excoriates


the nares

762. Foreign body in nose e) Unilateral, foul-smelling discharge


763. Sinusitis c) Persistent rhinorrhea or cough for more than 10-14
days, headache, facial pain, or periorbital edema

a) Persistent or paroxysmal cough


b) Nasal discharge that excoriates the nares
c) Persistent rhinorrhea or cough for more than 10-14 days, headache,
facial pain, or periorbital edema
d) Itching, sneezing, and nasal eosinophilia
e) Unilateral, foul-smelling discharge

764. Prolonged use of nasal topical adrenergic medications (e.g.,


xylometazoline, oxymetazoline, phenylephrine) can produce the
following complication:
a) Nasal ulcers
b) Hypertension
c) Tachycardia
d) Irritability
e) Rhinitis medicamentosa

764. e) Rhinitis medicamentosa is a rebound effect that produces a


sensation of nasal obstruction when the medication is discontinued

765. A young woman appears with recurrent pneumothorax during


menstruation. The investigation of the following organ is indicated:
a) Uterus
b) Fallopian tubes
c) Ovary
d) Vagina
e) Diaphragm

765. e) Diaphragmatic defect results in passage of intra abdominal air


into the pleural cavity. This girl has catamenial pneumothorax.
766. The most common organism isolated in patients with
pneumothorax, especially in infants is:
a) S. aureus
b) S. epidermidis
c) Streptococcus pneumoniae
d) Pseudomonas aeruginosa
e) Group B Streptococcus

766. a) Staphylococcus aureus; Mycoplasma pneumoniae infection can


cause pneumothorax

767. The most common cause of chylothorax is:


a) Chest injury
b) Metastatic lung disease
c) Child abuse
d) Thrombosis of thoracic duct
e) Rupture of thoracic duct during cardiac surgery

767. e) Rupture of thoracic duct during cardiac surgery for complex


congenital heart diseases. Answers (a), (b), (c), and (d) can cause
chylothorax

769. The most of inheritance in patients with Jeune syndrome is:


a) Autosomal recessive
b) Autosomal dominant
c) X-linked recessive
d) X-linked dominant
e) Multifactorial

769. a) Autosomal recessive. Jeune syndrome is called asphyxiating


thoracic dystrophy (thoracic-pelvic-phalangeal dystrophy).

935. The most common cause of community-acquired bacterial


pneumonia is:
a) Haemophilus influenzae type b
b) Streptococcus pneumoniae
c) Staphylococcus aureus
d) Pseudomonas aeruginosa
e) E. coli

936. The most common cause of community-acquired otitis media is:


a) Pseudomonas aeruginosa
b) E. coli
c) Haemophilus influenzae type b
d) Staphylococcus aureus
e) Streptococcus pneumonia

937. A child appears with upper respiratory tract infection and a


characteristic rash. The rash appeared within 24-48 hours after the onset
of symptoms. The rash began around the neck and subsequently
involved trunk and extremities. The rash is diffuse, finely papular, and
erythematous. The bright red discoloration of the skin, which blanches
on pressure. The skin feels rough and has goose-pimple appearance. The
rash is more prominent along the creases of the elbows, axillae, and
groins. She also has pharyngitis. The most likely diagnosis is:
a) Kawasaki disease
b) Rubella
c) Roseola
d) Toxic shock syndrome
e) Scarlet fever

938. The most common cause of bacterial pharyngitis is:


a) Group C Streptococcus
b) Group G Streptococcus
c) Group A Streptococcus
d) Corynebacterium diphtheriae
e) Arcanobacterium haemolyticum
Match all different diseases and sites of injury resulting in
respiratory failure (1001-1005):

1001. Werdnig-Hoffman disease e) Spinal cord

1002. Cystic fibrosis c) Peripheral airway obstruction

1003. Prune-belly syndrome a) Chest wall deformity

1004. Guillain-Barre syndrome b) Neuromuscular

1005. Croup d) Central airway obstruction

a) Chest wall deformity


b) Neuromuscular
c) Peripheral airway obstruction
d) Central airway obstruction
e) Spinal cord

3. The frontal sinuses first appear radiologically around:


a) Birth
b) 2 years of age
c) 4 years of age
d) 6 years of age
e) 8 years of age

4. The ethmoid sinuses reach their maximum size during:


a) 0-1 year of age
b) 1-3 years of age
c) 4-7 years of age
d) 7-14 years of age
e) 14-18 years of age
4. d) 7-14 years of age. All other sinuses (e.g., frontal, maxillary,
sphenoid) reach their maximum size after puberty.

28. A child develops retractions and respiratory distress after extubation


following a surgical procedure. A physical examination reveals
inspiratory stridor and wheezing in both lungs. The next step in
management is:
a) Chest physiotherapy
b) Selective intubation in right lung
c) Selective intubation in left lung
d) Intravenous corticosteroids
e) Racemic epinephrine aerosols
28. e) Racemic epinephrine aerosols therapy are effective in
patients with postoperative stridor. In severe cases, reintubation is
indicated.

389. A 2-hour-old baby becomes cyanotic. The infant is transferred to


NICU from regular nursery and is placed under oxyhood. The following
test can distinguish cardiac from pulmonary disease:
a) Chest x-ray
b) EKG
c) Pulmonary function test
d) Hyperoxia test
e) Chest CT scan

389. d) Hyperoxia test (i.e., infant receives 100% oxygen and


arterial blood gas is performed after 15-20 minutes). If the Pao2 is
above 150 mm Hg, cyanotic cardiac anomaly is probably
excluded and pulmonary cause is most likely etiology of hypoxia.
Echocardiogram is the best diagnostic study to diagnose cardiac
anomaly.
390. The hyperoxia test in a newborn reveals Pao2 is 200 mm Hg. The
most likely diagnosis is:
a) Tricuspid atresia
b) Transposition of great vessels
c) Pulmonary disease
d) Truncus arteriosus
e) Total anomalous pulmonary venous return
590. Arterial blood gas (ABG) in patients with BPD (bronchopulmonary
dyplasia) usually reveals the following result:

a) Baseline hypoxemia, elevated bicarbonate level, and reduced Pco2


level
b) Normal Po2 level, elevated bicarbonate level, and elevated Pco2 level
c) Elevated Po2 level, elevated bicarbonate level, and elevated Pco2
level
d) Baseline hypoxemia, decreased bicarbonate level, and elevated Pco2
level
e) Baseline hypoxemia, elevated bicarbonate level, and elevated Pco2
level

590. e) ABG in BPD patients usually reveals baseline hypoxemia


(requiring oxygen to maintain oxygen saturation above 90%), elevated
bicarbonate level, and elevated Pco2 level (due to chronic respiratory
insufficiency).

591. The following metabolic and respiratory conditions are noted in


patients with BPD:
a) Respiratory acidosis and metabolic acidosis
b) Respiratory alkalosis and metabolic alkalosis
c) Respiratory acidosis and metabolic alkalosis
d) Absence of respiratory acidosis but presence of metabolic alkalosis
e) Presence of respiratory acidosis but absence of metabolic alkalosis
592. The treatment in patients with BPD includes all of the following
except:
a) High calorie intake
b) Supplemental oxygen
c) Adequate fluid
d) Furosemide prn
e) Treatment for gastroesophageal reflux, if present

592. c) Fluid should be restricted along with high calorie intake in


patients with BPD. Gastroesophageal reflux is common in
patients with BPD. Inhaled glucocorticoids can be used in young
children with BPD.

593. An infant with BPD develops wheezing. The infant’s condition


worsen with beta-agonist therapy. Most likely cause is:
a) BPD with concomitant airway malacia
b) BPD with cardiac failure
c) BPD with bronchial smooth muscle irritation
d) BPD with bronchial smooth muscle hypertrophy
e) BPD with inflammation

593. a) BPD with concomitant airway malacia usually worsen with


beta-agonist therapy because beta-agonist relaxes bronchial
smooth muscles resulting in airway collapse due to airway
malacia.

594. The following immunoprophylaxis should be given to all patients


with BPD during winter months (October to April):
a) Influenza vaccine
b) Synagis vaccine
c) Meningococcal vaccine
d) Varicella vaccine
e) Oral amoxicillin prophylaxis
594. b) Synagis vaccine should be given to prevent RSV infectins
to all premature babies with or without BPD.

595. Analysis of the pleural fluid after thoracocentesis reveals a milky


fluid containing fat, protein, and lymphocytes. The definitive test to
diagnose the chylous fluid is:
a) Examination of the fluid under microscope
b) Measurements of sodium in the fluid
c) Measurements of chloride in the fluid
d) Measurements of cholesterol in the fluid
e) Quantitative measurements of triglyceride in the fluid

595. e) Triglyceride levels are elevated in chylous fluid.


Cholesterol levels are elevated in chronic serous effusions.

596. Majority of infants younger than 1 year of age with chylothorax


have the following outcome:
a) Placement of chest tube for a prolonged period of time
b) Need surgical intervention as soon as possible
c) Develop chronic lung disease
d) Develop pleural thickening
e) Spontaneously recover

596. e) More than 50% of cases recover spontaneously.


Repeated aspirations may be needed to release the pressure.
However, chyle accumulates rapidly and repeated aspirations
cause loss of calories, protein, and lymphocytes.

597. The therapy in patients with chylothorax includes all of the


following except:
a) Low-fat diet
b) High-protein diet
c) Adequate salt
d) Diuresis
e) Increased caloric intake
597. c) Salt restrictions are required in most patients. Fat soluble
vitamins (A, D, E and K) should be added.

598. A child has chylothorax for the last 4 weeks. The child is on a
mechanical ventilator and receiving TPN. Conservative therapy failed.
The child required several pleural fluid aspirations. The preferred
definitive therapy is:
a) Pleuroperitoneal shunt placement
b) Inhalation of nitric oxide (20 ppm)
c) Subcutaneous octreotide
d) Pressure controlled ventilation with positive end-expiratory pressure
e) Surgical ligation of the thoracic duct
693. A child with BPD (bronchopulmonary dysplasia) is treated with
furosemide. Furosemide therapy can cause the
following condition:
a) Metabolic acidosis
b) Metabolic alkalosis
c) Respiratory alkalosis
d) Respiratory acidosis
e) Both respiratory and metabolic acidosis

694. The respiratory compensation for a metabolic acidosis usually


takes:
a) 12-24 hours
b) 24-48 hours
c) 48-72 hours
d) 72-96 hours
e) 4-5 days

695. The metabolic compensation for a respiratory acidosis usually


takes:
a) 1- 2 days
b) 2 - 3 days
c) 3-4 days
d) 4-5 days
e) 5-6 days
759. A child is on a mechanical ventilator and is receiving 80% oxygen
(FIO2 0.8). ABG (arterial blood gas) reveals Po2 of 100 mm Hg, Pco2
of 40, and pH 7.35. The alveolar-arterial oxygen gradient (A-a gradient)
is:
a) 306
b) 350
c) 400
d) 420
e) 480

759. d)
A-a gradient is 420 (i.e., alveolar oxygen gradient minus arterial
oxygen gradient).
Arterial Po2 is 100 mm Hg.
Calculation of alveolar oxygen gradient:
PAO2 = [FIO2 (Pb-PH2O)]-(Paco2 / R)
= [0.8 (760-47)]-(40 / 0.8)
= (0.8 x 713)-(50)
= 570-50
= 520
(FIO2 = 0.8 i.e., 80% oxygen, Pb is 760 i.e., barometric pressure,
PH2O is 47 i.e., water vapor pressure,
Paco2 is 40, R is 0.8 i.e., respiratory quotient, PAo2 = alveolar
oxygen gradient).
A-a gradient = 520-100 = 420 mm Hg.
Normal A-a gradient in a healthy person is less than 10 mm Hg. If
a person is receiving 100% oxygen
(F102 1.0) and A-a gradient is more than 300 mm Hg, patient
needs intubation.
760. A child develops respiratory acidosis. The preferred method to
distinguish between the intrinsic lung disease and the poor respiratory
effort is:
a) Arterial pH
b) Arterial Po2
c) Arterial Pco2
d) Venous pH
e) Alveolar-arterial oxygen gradient

760. e) Alveolar-arterial oxygen gradient that is increased in


intrinsic lung disease unlike in poor respiratory effort

761. Acute respiratory alkalosis manifests with all of the following


findings except:
a) Bradycardia
b) Chest tightness
c) Circumoral numbness
d) Light headedness
e) Paresthesias of the extremities

761. a) Bradycardia is absent. Patient usually have palpitations. Less


common findings are cramps, tetany, syncope, and seizures. The
lightheadedness and syncope are due to the reduction in the cerebral
blood flow. The paresthethias, tetany, and seizures may be partially due
to the decreased ionized calcium because alkalemia causes more calcium
to bind with albumin

762. The following condition produces tissue hypoxia without


hypoxemia is:
a) Bacterial pneumonia
b) Viral pneumonia
c) Diaphragmatic hernia
d) Cyanotic heart disease
e) Carbon monoxide poisoning
762. e) Carbon monoxide poisoning, severe anemia, and congestive
heart failure produce tissue hypoxia without hypoxemia.

763. The following statement is not true about pulse oximetry:


a) It measures partial pressure of oxygen
b) Accuracy depends upon the adequate tissue perfusions
c) Abnormal hemoglobin and nail polish can affect results
d) Pulse oximetry is not very sensitive in detecting a mildly low Po2
e) Pulse oximetry is not adequate to eliminate hypoxia as a cause of
respiratory alkalosis

763. a) Pulse oximetry measures real-time oxygen saturation and


does not measure partial pressure of oxygen. Only ABG (arterial
blood gas) is adequate to eliminate hypoxia as a cause of a
respiratory alkalosis. Pulse oximetry does not give direct
measurements of CO2 tension, bicarbonate level, or acid-base status.
845. Most common anatomical cause of obstructive sleep apnea and
hypoventilation (OSA/H) in children is:
a) Enlarged tongue
b) Anterior nasal stenosis
c) Micrognathia
d) Midface hypoplasia
e) Adenotonsillar hypertrophy

845. e) Adenotonsillar hypertrophy. Answer (d) is noted in Down,


Cruzon, and Apart syndromes

846. Most common functional process contributing to obstructive sleep


apnea and hypoventilation (OSA/H) is:
a) Rapid eye movement sleep
b) Generalized hypotonia
c) Birth asphyxia
d) Cerebral palsy
e) Narcotics

846. a) Rapid eye movement (REM) sleep occurs about one


fourth of a typical night sleep. During REM sleep, apnea
frequency, apnea duration, and degree of hypoxia are severe in
patients with OSA/H. Answer (b) is noted in Down syndrome

847. Risk factors for obstructive sleep apnea and hypoventilation are all
of the following except:
a) Obesity
b) Chronic rhinitis
c) Asthma
d) White race
e) Positive family history

848. Chronic hypoxia that develops due to obstructive sleep apnea and
hypoventilation (OSA/H) resulting in all of the following conditions
except:
a) Growth failure
b) Polycythemia
c) Arrythmias
d) Death
e) Left ventricular failure

848. e) Chronic hypoxia due to OSA/H causes pulmonary hypertension


and right ventricular failure.

849. Most common symptom in patients with obstructive sleep apnea


and hypoventilation is:
a) Habitual snoring
b) Chronic mouth breathing
c) Restlessness during sleep
d) Frequent awakenings during sleep
e) Abnormal position during sleep
849. a) Habitual snoring is the most common symptom in patient with
OSA/H. Please remember, not all children with snoring are at risk for
the development of OSA/H. Most children with OSA/H breathe
normally while awake. Answers (b), (c), (d), and (e) are all clinical
manifestations in patients with OSA/H. Abnormal position during sleep
(e.g., hyperextended neck or prone with the bottom up in the air) is
required to maintain an upper airway.

850. A patient with Down syndrome develops pulmonary hypertension


and right ventricular failure. He has a history of habitual snoring.
Physical examination reveals obesity and pectus excavatum deformities.
EKG reveals a right ventricular hypertrophy. Most likely diagnosis is:
a) Large VSD
b) Hypoplastic lungs
c) Pleural effusion
d) Endocardial cushion defect
e) Obstructive sleep apnea / hypoventilation

851. Preferred diagnostic study in patients with obstructive sleep apnea /


hypoventilation is:
a) Arterial blood gas
b) Lateral x-ray of the neck
c) Audio / video taping during sleep
d) Home polysomnography

e) An overnight recording of multiple physiologic sensors during sleep

852. The following findings may be noted in patients with obstructive


sleep apnea and hypoventilation except:
a) Anemia
b) Right ventricular hypertrophy
c) Dysfunction in echocardiography
d) Respiratory acidosis with a metabolic alkalosis
e) Enlarged adenoid in the lateral x-ray of the neck

852. a) Anemia is not present. Polycythemia due to chronic hypoxia may


be noted. Polycythemia and answer (d) support the diagnosis but are
absent in majority of the pediatric patients.

853. Most severe complication of obstructive sleep apnea and


hypoventilation is:
a) Pulmonary hypertension
b) Daytime somnolence
c) Decreased memory
d) Decreased cognitive function
e) Increased behavioral problems

854. Preferred therapy in patients with obstructive sleep apnea and


hypoventilation (OSA/H) is:
a) Adenotonsillectomy in patients with a normal adenoid and tonsils
b) Nasal CPAP (continuous positive airway pressure)
c) Nasal steroids
d) Medroxyprogesterone acetate
e) Depends on the underlying abnormalities

854. e) Depends on the underlying abnormalities (e.g, -


- adenotonsillectomy is performed in patients with hypertrophied
adenoid and tonsils;
- maxillomandibular reconstruction surgery for children with
craniofacial disorders;
- nasal steroids can reduce snoring in some children; -
- medroxyprogesterone acetate increases ventilatory drive but is
effective only daytime hypoventilation associated with obesity-
hypoventilation syndrome;
- nasal CPAP can be used in children in whom adenotonsillectomy
failed;
- tracheotomy is indicated in severe cases.)

855. The adverse effect of medroxyprogesterone acetate when used in


female patients with OSA/H is:
a) Amenorrhea
b) Dysmenorrhea
c) Premenstrual syndrome
d) Pubertal development
e) Abnormal uterine bleeding

855. d) Pubertal development and growth are affected by


medroxyprogesterone acetate. This medication is also used for an
excessive menstrual bleeding, primary amenorrhea, and secondary
amenorrhea.

856. A complete resolution of symptoms may not occur after


adenotonsillectomy in all of the following conditions except:
a) Down syndrome
b) Extreme obesity
c) Cerebral palsy
d) Arnold chiari malformation
e) Onset of symptoms after 2 years of age

856. e) Incomplete resolution (i.e., symptoms persist even after


adenotonsillectomy) occurs in children who had onset of
symptoms before 2 years of age.

857. A child appears with a rapid enlargement of the right tonsil. He also
has fever, weight loss, and ipsilateral lymphadenopathy. Physical
examination reveals a normal left tonsil and an abnormal right tonsil.
Most likely diagnosis is:
a) Acute tonsillitis
b) Chronic tonsillitis
c) Peritonsillar abscess
d) Tonsillar abscess
e) Lymphoma

857. e) Lymphoma; a rapid unilateral enlargement of one tonsil


when associated with fever, weight loss, and lymphadenopathy, is
highly indicative of tonsillar malignancy. Lymphoma is the most
common malignancy of tonsil in children

858. Major virulence factor of group A beta-hemolytic Streptococci


(GABHS) is:
a) M protein
b) N protein
c) A protein
d) B protein
e) C protein

858. a) M protein produces resistance to phagocytosis by neutrophils.

859. The only accurate method to diagnose sinusitis is:


a) CT scan of the sinuses
b) Plain x-ray of the sinuses
c) History and physical examination
d) Transillumination of the sinuses
e) Sinus aspirate culture
863. A child has virus-induced reactive airway disease. She has been
coughing for the last 2 weeks. Most effective therapy is:
a) Vitamin C
b) Bronchodilator
c) Guaifenesin
d) Codeine
e) Dextromethorphan hydrobromide

864. Most common site of bleeding from the nose is:


a) Nasopharynx
b) Fractured nasal bone
c) Floor of the nose
d) Nasal sinuses
e) Kieselbach plexus

864. e) Kieselbach plexus is located in the anterior part of nasal septum


and consists of branches from the internal carotid (e.g., anterior and
posterior ethmoidal arteries) and external carotid (e.g., sphenopalatine
and terminal branches of internal maxillary arteries.)

865. Most common cause of nose bleed from anterior septum is:
a) Digital trauma
b) Sinusitis
c) Foreign bodies
d) Dry air
e) URI

866. The initial treatment in patients with nose bleed is:


a) Cold compress on the nose
b) Oxymetazoline drops
c) Neo-Synephrine drops
d) Anterior nasal packing
e) The nares should be compressed, an upright position, and head tilted
forward

866. e) Most nose bleeds stop spontaneously within a few minutes. The
nares should be compressed, an upright position, kept as quiet as
possible, and head titled forward to avoid blood goes back into the throat
and ultimately swallowed into the stomach. Answers (a), (b), (c), and (d)
are used if the previous therapies fail. Some patients need posterior nasal
packing when the bleeding occurs from the back of the nasal cavity.
Anterior and posterior nasal packing should be done by an ENT
specialist. A cautarization by silver nitrate may be required.
867. A child has nose bleeds during the winter months. He lives in an
apartment building. There is plenty of heat supply in the apartment. The
preferred preventive measure to avoid nose bleeds in this patient is:
a) Cold compress on the nose
b) Neo-Synephrine drops
c) Oxymetazoline drops
d) A room humidifier
e) Prophylactic anterior nasal packing

867. d) A room humidifier, saline drops, and petrolatum (vaseline)


applied to the septum may prevent epistaxis.

868. A child appears with mouth breathing and hyponasal speech.


Physical examination reveals widen bridge of the nose, eroded adjacent
bony structures, fleshy, glistening, gray, grapelike masses noted between
the nasal turbinates and the septum. Most likely diagnosis is:
a) Syphilis
b) Ethmoidal polyps
c) Sphenoidal polyps
d) Frontal encephalocele
e) Juvenile nasopharyngeal angiofibroma

868. b) This is a characteristic presentation of ethmoidal polyps.


Prolonged presence of ethmoidal polyps can cause widen bridge of the
nose and erode adjacent bony structures

869. A child appears in the ER with unilateral nasal discharge. Physical


examination reveals a small piece of crayon and mildly swollen
surrounding tissue. Preferred therapy is:
a) Keep the foreign body but start antibiotic therapy to prevent an
infection
b) Remove the foreign body under general anesthesia
c) Topical steroid therapy is to reduce the swelling
d) Topical Neo-Synephrine drops
e) Remove the foreign body by forcep or nasal suction under local
anesthesia

869. e) Remove the foreign body by forcep or nasal suction under


local anesthesia. General anesthesia is used if there is marked
swelling, tissue overgrowth, or bleeding. Infection improves
promptly when the foreign body is removed

870. Most common congenital anomaly of the nose is:


a) Choanal atresia
b) Hypoplastic nose
c) Single nasal cavity
d) Upturned nose
e) Long nose

871. A newborn appears cyanotic at birth. She becomes pink when


crying and again becomes cyanotic when stops crying. Most likely
diagnosis is:
a) Choanal stenosis
b) Mother received morphine
c) Perinatal asphyxia
d) Subarachnoid bleeding
e) Bilateral choanal atresia

872. Internal nasal airway doubles in size by:


a) 3 months of age
b) 6 months of age
c) 9 months of age
d) 1 year of age
e) 2 years of age

872. (b). By 6 months of age. Therefore, symptoms of nasal


congestion improve after 6 months of age in many children.

873. Acute bacterial sinusitis occurs after the following condition:


a) Acute otitis media
b) Chronic otitis media
c) Periorbital cellulitis
d) Orbital cellulitis
e) Viral upper respiratory tract infection

873. e) Viral upper respiratory tract infection; first, viral rhinosinusitis


develops (i.e., edema and inflammation within the sinuses), nose
blowing propels the nasal secretions into the sinus cavities,
nasopharyngeal bacteria enter the sinuses (normally cleared readily) and
grow inside due to an inadequate sinus drainage resulting in bacterial
sinusitis

Match the following complications of acute bacterial sinusitis and their


clinical manifestations (874-878):
874. Meningitis e) Headache, vomiting, nuchal rigidity, and
altered mental status

875. Pot puffy tumor b) Osteomyelitis of the frontal bone

876. Mucoceles a) Chronic inflammatory lesions in the frontal


sinus that can expand and displaced the eye causing diplopia.

877. Periorbital cellulitis c) Erythema and swelling of the tissues


surrounding the globe

878. Orbital cellulitis d) Proptosis, chemosis, double vision,


impaired extraocular movements, and eye pain

a) Chronic inflammatory lesions in the frontal sinus that can expand and
displaced the eye causing diplopia.
b) Osteomyelitis of the frontal bone
c) Erythema and swelling of the tissues surrounding the globe
d) Proptosis, chemosis, double vision, impaired extraocular movements,
and eye pain
e) Headache, vomiting, nuchal rigidity, and altered mental status

879. All of the following statements are true about the treatment in
patients with an acute sinusitis except:
a) Antibiotic therapy has substantial benefit in clinically diagnosed
acute bacterial sinusitis.
b) In uncomplicated cases, initial therapy is amoxicillin (45
mg/kg/day) for an acute bacterial sinusitis.
c) Penicillin-allergic patients should receive cefuroxime axetil,
cefpodoxime, clarithromycin, or azithromycin.
d) A “high-dose” amoxicillin-clavulanate (80-90 mg/kg/day of
amoxicillin and 6.4 mg/kg/day of clavulanate)
should be given in patients who are at high risk (e.g., younger than
2 years of age, daycare attendance, received antibiotic in the last 1-
3 months).
e) Unresponsive patients should be referred to an ENT specialist
for further evaluation and maxillary sinus aspiration and culture.

879. a) It is not clear whether antibiotic therapy has substantial benefit in


clinically diagnosed acute bacterial sinusitis. A study reveals that a 14-
day treatment with amoxicillin, amoxicillin-clavulanate, or placebo did
not affect resolution and duration of symptoms, or days missed from
school. About 50-60% children will recover without antibiotic therapy in
acute bacterial sinusitis. However, the American Academy of Pediatrics
recommend antibiotic therapy to promote resolution of symptoms and to
prevent suppurative complications. The duration of antibiotic therapy is
unclear but should be given 7 days after resolution of symptoms.

880. The preferred method of prevention in sinusitis is:


a) Frequent hand washing
b) Influenza vaccine
c) Oseltamivir
d) Zanamivir
e) Use face mask
880. a) Frequent handwashing, avoiding persons with cold; influenza
vaccine can prevent only some cases because influenza is responsible for
only a small portion of all colds. Answers (c) and (d) have complications
in children. Answer (e) does not make sense.
Prpared &Revised By
Dr.Wahid Helmi Rifahie
Consultant pediatrician
Zarka Hospital –Dymiate-Egypt

Second Revision All answered

100 Questions with


answer&Eplaination

WhatsApp
01004313142
Which one of the following is the most common cause
of congenital hydrocephalus?
A. Craniosynostosis
B. Intra uterine meningitis
C. Aqueductal stenosis
D. Malformations of great vein of Galen

Correct answer : C. Aqueductal stenosis

The most common malignant neoplasm of infancy is:


A. Malignant teratoma
B. Neuroblastoma
C. Wilms’ tumor
D. Hepatoblastoma

Correct answer : B. Neuroblastoma

ln a child, non functioning kidney is best diagnosed


by:
A. Ultrasonography
B. IVU
C. DTPA renogram
D. Creatinine clearance

Correct answer : C. DTPA renogram


The most common cause of renal scarring in a 3 year
old child is:
A. Trauma
B. Tuberculosis
C. Vesicoureteral reflux induced pyelonephritis
D. Interstitial nephritis

Correct answer : C. Vesicoureteral reflux induced


pyelonephritis

A child with recurrent urinary tract infections is most


likely to show:
A. Posterior urethral valves
B. Vesicoureteric reflux
C. Neurogenic bladder
D. Renal and ureteric calculi

Correct answer : B. Vesicoureteric reflux


Vesicoureteric reflux is the most common cause of
urinary tract infections in childhood. (upto 50%)

Which of the following is the most common renal


cystic disease in infants?
A. Polycystic kidney
B. Simple renal cyst
C. Unilateral renal dysplasia.
D. Calyceal cyst

Correct answer : C. Unilateral renal dysplasia


The most common type of total anomalous pulmonary
venous connection is:
A. Supracardiac
B. Infracardiac
C. Mixed
D. Cardiac

Correct answer : A. Supracardiac

One of the intestinal enzymes that is generally


deficient in children following an attack of severe
infectious enteritis is:
A. Lactase
B. Trypsin
C. Lipase
D. Amylase

Correct answer : A. Lactase


Even in the normal intestine, the lactase activity is
limited. Hence it is most prone to become deficient
following an attack of infectious enteritis.
Eisenmenger syndrome is characterized by all except:
A. Return of left ventricle & right ventricle to normal
size
B. Pulmonary veins not distended
C. Pruning of peripheral pulmonary arteries
D. Dilatation of central pulmonary arteries

Correct answer : A. Return of left ventricle & right


ventricle to normal size
Right ventricular hypertrophy that develops in
Eisenmenger syndrome will not return to normal size.

Diagnosis of beta Thalassemia is established by:


A. NESTROFT Test
B. HbA1c estimation
C. Hb electrophoresis
D. Target cells in peripheral smear

Correct answer : C. Hb electrophoresis


NESTROFT Test – Naked Eye Single Tube Red Cell
Osmotic Fragility Test – used for screening for
Thalassemia
HbA1c – Used to assess long term glycemic control in
diabetics (blood sugar control over past 3 months)
Target cells – A feature of thalassemia, but not
diagnostic
The coagulation profile in a 13-year old girl with
Menorrhagia having von Willebrands disease is:
A. Isolated prolonged PTT with a normal PT
B. Isolated prolonged PT with a normal PTT
C. Prolongation of both PT and PTT
D. Prolongation of thrombin time

Correct answer : A. Isolated prolonged PTT with a


normal PT
In VWF, there is impairment of intrinsic coagulation
pathway. Hence there is isolated prolonged PTT.

The most common leukocytoclastic vasculitis


affecting children is:
A. Takayasu disease
B. Mucocutaneous lymph node syndrome (Kawasaki
disease)
C. Henoch–Schönlein purpura
D. Polyarteritis nodosa

Correct answer : C. Henoch–Schönlein purpura


HSP is the most common childhood vasculitis. It is
produces leukocytoclastic vasculitis.
Bart‘s hydrops fetalis is lethal because:
A. Hb Bart’s cannot bind oxygen
B. The excess alpha globin form insoluble precipitates
C. Hb Bart’s cannot release oxygen to fetal tssues
D. Microcytic red cells become trapped in the placenta

Correct answer : C. Hb Bart’s cannot release oxygen


to fetal tssues
Hb Barts is formed of 4 gamma chains. The oxygen
affinity is so high that it releases very little oxygen
into the fetal tissues.

A child with a small head, minor anomalies of the face


including a thin upper lip, growth delay, and
developmental disability can have all of the following,
except:
A. A chromosomal syndrome
B. A teratogenic syndrome
C. A mendelian syndrome
D. A polygenic syndrome

Correct answer : D. A polygenic syndrome


The features given can occur as a part of a
chromosomal / teratogenic / mendelian syndrome.
Polygenic inheritance is the answer of exclusion. In a
polygenic inheritance, multiple genes are involved in
the phenotypic expression. Some examples of
polygenic inheritance are hypertension and

An affected male infant born to normal parents could


be an example of all of the following, except
A. An Autosomal dominant disorder
B. An Autosomal recessive disorder
C. A polygenic disorder
D. A vertically transmitted disorder

Correct answer : A. An Autosomal dominant disorder


An autosomal dominant disorder if present in the
parents will always express itself phenotypically. So it
is not possible for normal parents to have an child
affected with an autosomal dominant disorder.

The process underlying differences in expression of a


gene according to which parent has transmitted is
called:
A. Anticipation
B. Mosaicism
C. Non penetrance
D. Genomic imprinting

Correct answer : D. Genomic imprinting


In genomic imprinting, a gene / group of genes on the
paternal or maternal chromosome is selectively
inactivated. The functional allele will be provided by
the counterpart.
Example:
Angelman syndrome – deletion of maternal gene
Prader Willi syndrome – deletion of paternal gene

A malignant tumor of childhood, that metastasizes to


bones most often is:
A. Wilm’s tumor
B. Neuroblastoma
C. Adrenal gland tumors
D. Granulosa cell tumor of ovary
.

Correct answer : B. Neuroblastoma


60-70% of metastasis in case of neuroblastoma are
skeletal metastasis

The most common etiological agent for acute


bronchiolitis in infancy is:
A. Influenza virus
B. Para influenza virus
C. Rhinovirus
D. Respiratory syncytial virus

Correct answer : D. Respiratory syncytial virus (It is


responsible for more than 50% cases of bronchiolitis)

Late onset hemorrhagic disease of newborn is


characterized by all of the following features except:
A. Usually occurs in cow-milk fed babies
B. Onset occurs at 4-12 week of age
C. lntracranial hemorrhage can occur
D. Intramuscular vitamin K prophylaxis at birth has a
protective role

Correct answer : A. Usually occurs in cow-milk fed


babies
Haemorrhagic disease of newborn is more common in
breast milk fed babies. (Breast milk is a poor source
of Vitamin K)

With reference to mumps which of the following is


true?
A. Meningoencephalitis can precede parotitis
B. Salivary gland involvement is limited to the
parotids
C. The patient is not infectious prior to clinical parotid
enlargement
D. Mumps orchitis frequency leads to infertility

Correct answer : A. Meningoencephalitis can precede


parotitis
Meningoencephalitis can occur 1 week before onset of
parotitis.

The earliest indicator of response after starting iron in


a 6-year-old girl with iron deficiency is:
A. Increased reticulocyte count
B. Increased hemoglobin
C. Increased ferritin
D. Increased serum iron

Correct answer : A. Increased reticulocyte count


Sequence of changes during correction of iron
deficiency:
Clinical improvement in the child (increase in appetite,
improvement in irritability)
Inital bone marrow response
Increased reticulocyte count
Haemoglobin levels return to normal
Body iron stores return to normal (ferritin levels)
The following features are true for tetralogy of Fallot,
except:
A. Ventricular septal defect
B. Right ventricular hypertrophy
C. Atrial septal defect
D. Pulmonary stenosis

Correct answer : C. Atrial septal defect


Components of Tetralogy of Fallot:
Pulmonary stenosis
Overriding aorta
Ventricular septal defect
Right ventricular hypertrophy

Blalock and Taussig shunt is done between:


A. Aorta to pulmonary artery
B. Aorta to pulmonary vein
C. Subclavian artery to pulmonary vein
D. Subclavian vein to artery

Correct answer : A. Aorta to pulmonary artery


Blalock and Taussig shunt is used in the surgical
management of tetralogy of fallot. It is shunts blood
from the subclavian artery to the pulmonary artery.
That is not given among the list of options. As
subclavian artery is a branch of aorta, the best answer
would be ‘aorta to pulmonary artery.’

The most important determinant of prognosis in


Wilms tumor:
A. Stage of disease
B. Loss of heterozygosity of chromsome lp
C. Histology
D. Age less than one year at presentation

Correct answer : C. Histology


Histology and stage of disease are important
prognostic factors. But histology is more important.

A 1 month old boy is referred for failure to thrive. On


examination, he shows feature of congestive heart
failure. The femoral pulses are feeble as compared to
branchial pulses. The most likely clinical diagnosis is:
A. Congenital aortic stenosis
B. Coarctation of aorta
C. Patent ductus arteriosus
D. Congenital aortoiliac disease

Correct answer : B. Coarctation of aorta


Feeble femoral pulses compared to brachial pulse
indicates coarctation of aorta.
ln which of the following conditions left atrium is not
enlarged:
A. Ventricular septal defect
B. Atrial septal defect
C. Aortopulmonary window
D. Patent ductus arteriosus

Correct answer : B. Atrial septal defect


Right atrium and ventricle is enlarged in ASD. Left
atrium can enlarge once Eisenmenger’s syndrome
develops.

Which of the following haemoglobin (Hb) estimation


will be diagnostically helpful in a case of beta
thalassemia trait?
a) HbF
b) HbA1c
c) HbA2
d) HbH

Correct answer : c) HbA2


Normal hemoglobin is composed predominantly of
HbA1 – which is composed of 2 alpha chains and 2
beta chains. In beta thalassemia trait, there is
decrease in production of beta chains of hemoglobin.
But since the defect is minor, they will not have any
clinical features and the peripheral smear may be
normal. But there will be a compensatory increase in
HbA2 which is composed of 2 alpha chains and 2
delta chains. Normally HbA2 constitutes about 1.5 to
3.5% of total hemoglobin. But in beta thalassemia trait,
this increases to about 3.6 to 8%.

The sodium content of ReSoMal (rehydration solution


for malnourished children) is:
A. 90 mmol/L
B. 60 mmol/L
C. 45 mmol/L
D. 30 mmol/L

Correct answer : C. 45 mmol/L


Components of of ReSoMal:
Glucose 125 mmol/l
Sodium 45 mmol/l
Potassium 40 mmol/l
Chloride 70 mmol/l
Magnesium 3 mmol/l
Zinc 0.3 mmol/l
Copper 0.045 mmol/l
Citrate 7 mmol/l
A 15-year old female presented to the emergency
department with history of recurrent epistaxis,
hematuria and hematochezia. There was a history of
profuse bleeding from the umbilicus stump at birth.
Previous investigations revealed normal prothrombin
time, activated partial thromboplastin time, thrombin
time and fibrinogen levels. Her platelet counts as well
as platelet function tests were normal but urea clot
lysis test was positive. Which one of the following
clotting factor is most likely to be deficient?
a) Factor X
b) Factor XI
c) Factor XII
d) Factor XIII

Correct answer : d) Factor XIII


Clinical features of Factor XIII deficiency
History of prolonged bleeding from umbilical stump
Delayed bleeding
Recurrent abortions
Diagnosis (Investigations)
Prothrombin time and Activated partial thromboplastin
time are normal
Factor XIII is needed for stabilization of fibrin clot
Intrinsic and extrinsic pathways are not affected
Clot stability in 5M urea – qualitative test
Normal clot remains stable in 5M urea
But in cases of Factor XIII deficiency, the clot
dissolves
But it becomes positive only in very severe deficiency
Quantitative factor XIII assay – using photometry

The requirement of potassium is a child is:


A. 1-2 mEq/kg
B. 4-7 mEq/kg
C. 10-12 mEq/kg
D. 13-14 mEq/kg

Correct answer : A. 1-2 mEq/kg


Daily recommended intake of potassium is 1-2mEq/kg.

Which of the following malformation in a newborn is


specific for maternal insulin dependent diabetes
mellitus?
A. Transposition of great arteries
B. Caudal regression
C. Holoprosencephaly
D. Meningmyelocele
.
Correct answer : B. Caudal regression
Caudal regression is the most specific anomaly in a
child born to a diabetic mother

All of the following may occur in Down’s syndrome


except:
A. Hypothyroidism
B. Undescended testis
C. Ventricular septal defect
D. Brushfield`s spots

Correct answer : B. Undescended testis

The following are characteristic of autism except:


A. Onset after 6 years of age
B. Repetitive behavior
C. Delayed language development
D. Severe deficit in social interaction
Correct answer : A. Onset after 6 years of age
Features of autism usually appear before 3 years of
age. (OP Ghai)
Which of the following is the principal mode of heat
exchange in an infant incubator?
A. Radiation
B. Evaporation
C. Convection
D. Conduction
Correct answer : C. Convection
Which one of the following in the characteristic
feature of juvenile myoclonic epilepsy?
a) Myoclonic seizures frequently occur in the morning
b) Complete remission is common
c) Response to anticonvulsants is poor
d) Associated absence seizures are present in
majority of patients

Juvenile myoclonic epilepsy –


Clinical features, treatment
It starts in early adolescence
Bilateral myoclonic jerks are seen
Mostly in the morning
Precipitated by sleep deprivation
The patient usually remains conscious during the
episode
Associated with absence seizures and generalised
tonic clonic seizures
Benign condition
Complete remission is uncommon
Positive family history may be present
Responds well to anticonvulsants
Drug of choice – Valproate

A 12 year old Boy with hematemesis, melena and mild


splenomegaly presented to the paediatrics OPD. Examination
revealed absence of jaundice / ascites. Most probable diagnosis
is?
A. Extrahepatic Portal Venous Obstruction (EHPVO)
B. Cirrhosis
C. Non Cirrhotic Portal Fibrosis (NCPF)
D. Malaria with disseminated intravascular coagulation
Correct answer : A. Extrahepatic Portal Venous Obstruction
(EHPVO)Hematemesis, melena and splenomegaly are suggestive
of a diagnosis of portal hypertension
The first three options can cause portal hypertension
But considering the age and sex of the child, Extrahepatic Portal
Venous Obstruction (EHPVO) is the most probable diagnosis
Non Cirrhotic Portal Fibrosis (NCPF) is usually seen in adult
females in the third or fourth decade
Cirrhosis is not very common in children, and it is usually
accompanied by jaundice / ascites

Most common malignant orbital tumor in children is?


A. Acute myeloid leukemia
B. Acute lymphoblastic leukemia
C. Rhabdomyosarcoma
D. Cavernous hemangioma
Correct answer : C. Rhabdomyosarcoma
Most common malignant orbital tumor in children is
rhabdomyosarcoma.
Most common benign orbital tumor in children is dermoid cyst.

Right sided isomerism is found in association with?


A. Asplenia
B. Single spleen
C. Two spleens
D. Multiple spleens
Correct answer : A. Asplenia
Right sided isomerism
Alternate names – Ivemark syndrome / asplenia syndrome
It is a type of situs anomaly
Features : asplenia, malrotation of bowel, transverse liver, gall
bladder agenesis, imperforate anus, horseshoe adrenal gland,
urethral valves
The patients are immunocompromised due to absence of spleen,
most die before 1 year of age

Which of the following is the treatment of choice for true precocious


puberty?
A. Surgical removal of hypothalamic tumor
B. GnRH
C. Androgen blocking agents
D. LH
Correct answer : B. GnRH
Treatment of precocious puberty :
Pulsatile secretion of Gonadotropin Releasing Hormone is
responsible for the secretion of FSH and LH. By exogenous
administration of GnRH, it is possible to maintain a constant level
of GnRH in the blood. This inhibits the release of FSH and LH.

Which of the following is the most common complication of mumps in


children?
A. Orchitis
B. Conjunctivitis
C. Meningoencephalitis
D. Myocarditis

Correct answer : C. Meningoencephalitis


Meningoencephalitis is the most common complication of mumps
in children.

Which is not used for the treatment of juvenile myoclonic epilepsy?


A. Zonisamide
B. Topiramate
C. Carbamazepine
D. Valproate

Correct answer : C. Carbamazepine


Use of carbamazepine / phenytoin may increase myoclonus
in juvenile myoclonic epilepsy.

An eight year old boy presented to the casualty with high fever,
pruritic erythematous rash, joint pain and lymph node
enlargement. There is a history of upper respiratory tract infection
for which he was on cefaclor – 8 days completed of a 10 day
course. The most likely diagnosis is?
A. Serum sickness like illness
B. HSP
C. Type III hypersensitivity
D. Kawasaki disease

Correct answer : A. Serum sickness like illness


Serum sickness like reaction can occur following the use of
certain drugs, especially cefaclor in children. It presents with an
urticarial / purpuric rash, arthritis, lymphadenopathy and fever. But
unlike true serum sickness (a type III hypersensitivity response), it
is not caused by circulating immune complexes.
A one year old child presented to the OPD with the history of short
stature, tiredness and constipation. Examination revealed a
palpable goitre. Serum T4 was decreased and TSH levels were
increased. Which is the most probable diagnosis?
A. Thyroid dysgenesis
B. Thyroid Dyshormonogenesis
C. TSH receptor blocking antibody
D. Central hypothyroidism

Correct answer : B. Thyroid Dyshormonogenesis


Among the options given, only Thyroid Dyshormonogenesis
presents with a palpable goitre.

All of the following therapies may be required in a 1 hour old infant


with severe birth asphyxia except:
A. Glucose
B. Dexamethasone
C. Calcium gluconate
D. Normal saline
Correct answer : B. Dexamethasone
Steroids should not be used in management of infants with
asphyxia.

The karyotype of a patient with Androgen Insensitivity Syndrome


is:
A. 46XX
B. 46XY
C. 47XXY
D. 45XO
Correct answer : B. 46XY
Androgen insensitivity is seen in genetic males (XY) who exhibit a
female phenotype.

Which one of the following drugs is used for fetal therapy of


congenital adrenal hyperplasia?
A. Hydrocortisone
B. Prednisolone
C. Fludrocortisone
D. Dexamethasone
Correct answer : D. Dexamethasone
Dexamethasone acts by suppressing the secretion of steroids by
fetal adrenals.

Blood specimen for neonatal thyroid screening is obtained on:


A. Cord blood
B. 24 hours after birth
C. 48 hours after birth
D. 72 hours after birth
Correct answer : A. Cord blood D. 72 hours after birth
Update: The correct answer is 72 hours after birth.
Guidelines for neonatal thyroid screening – American Academy of
Pediatrics
When to screen:
Normal hospital delivery at term – Filter paper collection ideally at
2-4 days of age or at time of discharge
NICU / preterm home birth – Within 7 days of birth
Maternal history of thyroid medication / family history of
congenital hypothyroidism – cord blood for screening
Cord blood is to be used only if there is family history of
hypothyroidism. In normal delivery, blood is taken at 2-4 days of
age.

The appropriate approach to a neonate presenting with vaginal


bleeding on day 4 or life is:
A. Administration of vitamin K
B. Investigation for bleeding disorder
C. No specific therapy
D. Administration of 10 ml/kg of fresh frozen plasma over 4 hours
Correct answer : C. No specific therapy
Vaginal bleeding can occur in 3rd – 7th day after birth. It occurs due
to withdrawal of maternal hormones. It usually subsides in 2-3
days. No specific treatment is required.

In unconjugated hyperbilirubinemia, the risk of kernicterus


increases with the use of:
A. Ceftriaxone
B. Phenobarbitone
C. Ampicillin
D. Sulphonamide
Correct answer : D. Sulphonamide
Sulphonamide can displace bilirubin from albumin.
The following bacteria are most often associated with acute
neonatal meningitis except:
A. Escherichia coli
B. Streptococcus agalactiae
C. Neisseria meningitidis
D. Listeria monocytogenes
Correct answer : C. Neisseria meningitidis
Meningitis by Neisseria meningitidis is seen in 2months-12 years
of age.

Which one of the following is the most common cause of


congenital hydrocephalus?
A. Craniosynostosis
B. Intra uterine meningitis
C. Aqueductal stenosis
D. Malformations of great vein of Galen
Correct answer : C. Aqueductal stenosis

The most common malignant neoplasm of infancy is:


A. Malignant teratoma
B. Neuroblastoma
C. Wilms’ tumor
D. Hepatoblastoma
Correct answer : B. Neuroblastoma

The most common presentation of a child with Wilms tumor is:


A. An asymptomatic abdominal mass
B. Haematuria
C. Hypertension
D. Hemoptysis due to pulmonary secondary
Correct answer : A. An asymptomatic abdominal mass
ln a child, non functioning kidney is best diagnosed by:
A. Ultrasonography
B. IVU
C. DTPA renogram
D. Creatinine clearance
Correct answer : C. DTPA renogram
Isotope renogram is the best modality for renal function
estimation.

The most common cause of renal scarring in a 3 year old child is:
A. Trauma
B. Tuberculosis
C. Vesicoureteral reflux induced pyelonephritis
D. Interstitial nephritis
Correct answer : C. Vesicoureteral reflux induced pyelonephritis
A child with recurrent urinary tract infections is most likely to
show:
A. Posterior urethral valves
B. Vesicoureteric reflux
C. Neurogenic bladder
D. Renal and ureteric calculi
Correct answer : B. Vesicoureteric reflux
Vesicoureteric reflux is the most common cause of urinary tract
infections in childhood. (upto 50%)

Which of the following is the most common renal cystic disease in


infants?
A. Polycystic kidney
B. Simple renal cyst
C. Unilateral renal dysplasia
D. Calyceal cyst
Correct answer : C. Unilateral renal dysplasia

One of the intestinal enzymes that is generally deficient in children


following an attack of severe infectious enteritis is:
A. Lactase
B. Trypsin
C. Lipase
D. Amylase
Correct answer : A. Lactase
Even in the normal intestine, the lactase activity is limited. Hence it
is most prone to become deficient following an attack of infectious
enteritis.

The most common type of total anomalous pulmonary venous


connection is:
A. Supracardiac
B. Infracardiac
C. Mixed
D. Cardiac
Correct answer : A. Supracardiac

Eisenmenger syndrome is characterized by all except:


A. Return of left ventricle & right ventricle to normal size
B. Pulmonary veins not distended
C. Pruning of peripheral pulmonary arteries
D. Dilatation of central pulmonary arteries
Correct answer : A. Return of left ventricle & right ventricle to
normal size
Right ventricular hypertrophy that develops in Eisenmenger
syndrome will not return to normal size.
Diagnosis of beta Thalassemia is established by:
A. NESTROFT Test
B. HbA1c estimation
C. Hb electrophoresis
D. Target cells in peripheral smear
Correct answer : C. Hb electrophoresis
NESTROFT Test – Naked Eye Single Tube Red Cell Osmotic
Fragility Test – used for screening for Thalassemia
HbA1c – Used to assess long term glycemic control in diabetics
(blood sugar control over past 3 months)
Target cells – A feature of thalassemia, but not diagnostic

The most common leukocytoclastic vasculitis affecting children is:


A. Takayasu disease
B. Mucocutaneous lymph node syndrome (Kawasaki disease)
C. Henoch–Schönlein purpura
D. Polyarteritis nodosa
Correct answer : C. Henoch–Schönlein purpura
HSP is the most common childhood vasculitis. It is produces
leukocytoclastic vasculitis.
The coagulation profile in a 13-year old girl with Menorrhagia
having von Willebrands disease is:
A. Isolated prolonged PTT with a normal PT
B. Isolated prolonged PT with a normal PTT
C. Prolongation of both PT and PTT
D. Prolongation of thrombin time
Correct answer : A. Isolated prolonged PTT with a normal PT
In VWF, there is impairment of intrinsic coagulation pathway.
Hence there is isolated prolonged PTT.
Bart‘s hydrops fetalis is lethal because:
A. Hb Bart’s cannot bind oxygen
B. The excess alpha globin form insoluble precipitates
C. Hb Bart’s cannot release oxygen to fetal tssues
D. Microcytic red cells become trapped in the placenta
Correct answer : C. Hb Bart’s cannot release oxygen to fetal tssues
Hb Barts is formed of 4 gamma chains. The oxygen affinity is so
high that it releases very little oxygen into the fetal tissues.
An affected male infant born to normal parents could be an
example of all of the following, except
A. An Autosomal dominant disorder
B. An Autosomal recessive disorder
C. A polygenic disorder
D. A vertically transmitted disorder
Correct answer : A. An Autosomal dominant disorder
An autosomal dominant disorder if present in the parents will
always express itself phenotypically. So it is not possible for
normal parents to have an child affected with an autosomal
dominant disorder

A child with a small head, minor anomalies of the face including a


thin upper lip, growth delay, and developmental disability can have
all of the following, except:
A. A chromosomal syndrome
B. A teratogenic syndrome
C. A mendelian syndrome
D. A polygenic syndrome
Correct answer : D. A polygenic syndrome
The features given can occur as a part of a chromosomal /
teratogenic / mendelian syndrome. Polygenic inheritance is the
answer of exclusion. In a polygenic inheritance, multiple genes are
involved in the phenotypic expression. Some examples of
polygenic inheritance are hypertension and diabetes.

The process underlying differences in expression of a gene


according to which parent has transmitted is called:
A. Anticipation
B. Mosaicism
C. Non penetrance
D. Genomic imprinting
Correct answer : D. Genomic imprinting
In genomic imprinting, a gene / group of genes on the paternal or
maternal chromosome is selectively inactivated. The functional
allele will be provided by the counterpart.
Example:
Angelman syndrome – deletion of maternal gene
Prader Willi syndrome – deletion of paternal gene
A malignant tumor of childhood, that metastasizes to bones most
often is:
A. Wilm’s tumor
B. Neuroblastoma
C. Adrenal gland tumors
D. Granulosa cell tumor of ovary
Correct answer : B. Neuroblastoma
60-70% of metastasis in case of neuroblastoma are skeletal
metastasis.
Earliest sign of pathological gastroesophageal reflux in infants is?
A. Upper GI bleeding
B. Respiratory symptoms
C. Oesophageal stricture
D. Postprandial regurgitation
Correct answer : B. Respiratory symptoms
Gastroesophageal reflux in infants
Gastroesophageal reflux in infants can be of 2 types –
Physiological or Pathological.
Physiological reflux
The infant has regurgitation after feeding, but weight gain is
normal.
There are no respiratory symptoms / features of oesophagitis.
Pathological reflux (gastroesophageal reflux disease)
Regurgitation is associated with refusal to eat, excessive
vomiting, weight loss and failure to thrive.
The infant has recurrent respiratory features like like wheezing,
stridor, chronic cough, aspiration, sore throat, hoarseness and
recurrent pneumonitis.
Features of oesophagitis may be present – upper GI bleeding,
anemia and Sandifer syndrome (torticollis and dystonia associated
with GERD).
The most frequent complications are failure to thrive and recurrent
respiratory symptoms.
Investigations
Upper GI imaging series
Upper GI endoscopy
Esophageal pH monitoring
Management
Conservative management
Small frequent feeds thickened with cereal.
Burping the infant.
Hold the infant in an upright position for 20-30 minutes after
feeding.
Medical management
H2 blockers like ranitidine.
Proton pump inhibitors like omeprazole and pantoprazole.
Antacids like magnesium hydroxide.
Surgical management
Gastrostomy or fundoplication is done in a very small minority.
Differentiating an ASD from a VSD using a chest X-ray is by detection
of?
A. Enlargement of pulmonary artery
B. Enlarged left atrium
C. Dilated aorta
D. Pulmonary plethora
Correct answer : B. Enlarged left atrium

Differentiating an ASD from a VSD using a chest X-ray


Left atrium is not dilated in a case of atrial septal defect (ASD).
It is dilated in patients with ventricular septal defect. (VSD).
Pulmonary plethora is present in both ASD and VSD.
Pulmonary artery is dilated in both.
Size of the aorta is normal in both.

Least common cause of ambiguous genitalia in females?


A. 21 hydroxylase deficiency
B. 11 hydroxylase deficiency
C. WNT4 gene mutation
D. Foetal placental steroid sulfatase deficiency
Correct answer : D. Placental steroid sulfatase deficiency
Foetal placental steroid sulfatase deficiency is a rare X-linked
condition, and is seen only in males.
It is not a cause for development of ambiguous genitalia in
females.
The most common cause of ambiguous genitalia in females is
Congenital Adrenal Hyperplasia (CAH).
It is responsible for about 70% of cases.
21 hydroxylase deficiency and 11 hydroxylase deficiency are
subtypes of CAH.
Other causes include WNT4 mutation and foetal placental
aromatase deficiency.

All of the following are example of peripheral neuropathies except?


A. Abetalipoproteinemia
B. Charcot-Marie-Tooth disease
C. Werdnig Hoffman disease
D. Dejerine Sottas disease
Correct answer : C. Werdnig Hoffman disease
Abetalipoproteinemia is a progressive ataxic neuropathy with
retinitis pigmentosa and steatorrhoea.
Charcot-Marie-Tooth disease is a hereditary motor and sensory
neuropathy. It is characterised by weakness of extremities and
hammer toe.
Dejerine Sottas disease is also a hereditary motor and sensory
neuropathy. It is associated with distal weakness and ataxia.
Werdnig Hoffman disease is spinal muscular atrophy type 1. It
presents within first 6 months of life. It presents with generalised
symmetrical weakness, more in proximal muscle groups with
hypotonia.
Dr.Wahid Helmi
Pediatric Consultant
Egypt- Dymiate
&Cairo(Giza –ALharam)
01004313142
Third Revision
X-ray
1. Diagnosis?
2. What are the first two steps in treatment of hypoxic spell?
3. In a cyanotic newborn, how can you distinguish pulmonary disease from
cyanotic congenital heart disease?
4. most common neurological complications associate with disease
5. Which cardiac conditions are associated with following
Egg Shaped Heart
Snowman silhouette
Rib notching

Answers
1.cyano c congenital heart diseseas ,most probably TOF
Pulmonary vascular markings are decreased, low PBF
RVH on with upturn of apex
A hypoplastic main pulmonary artery segment contributes to the formation
-

2.knee chest posi on , morphine

3.Hyperoxia Test

4.Cerebral thrombosis, and brain abscess.


5.X-ray appearances
Egg Shaped Heart Transposition of great arteries
Snowman silhouette TAPVD (supracardiac)
Rib notching Co-arctation of aorta (long standing)
1. What is this sign called?
2. What are the structures that cause this appearance in this condition?
3. What is the diagnosis?
4. What are the treatment?

Answer

1. i.The heart shape is ovoid (egg-shaped).


ii. The upper mediastinum is narrow.
iii. The lung fields appear normal , no VSD.
2. Transposi on of the great arteries.
The diagnosis should be confirmed by echocardiography.
A prostaglandin infusion should be started and the case should be discussed
with the nearest pediatric cardiac centre as soon as possible.
Chest X ray of baby IDMM

1. Describe the X-ray.


2. What is your diagnosis?
3. What will you do next?

Answer

1.The heart is much larger than normal.


The lung fields appear well aerated.
UAC on the left very high
UVC on the right, very high

2. A diabetic cardiomyopathy is the most likely diagnosis.


Also baby IDMM may have TGA

3.Rx Reposi on the lines.

Echocardiography.

Seek expert advice if haemodynamically unstable.


1. What is the diagnosis?
2. What is this sign called?
3. What are the structures that cause this appearance in this condition?
4. What are the clinical presentation of the infant?

Answer

1. Total anomalous pulmonary venous drainage- supracardiac type

2. Snow man appearance, figure of 8 appearance

3.Cardiomegaly with increased vascular markings


Dilatation of both the left and right innominate veins and right SVC .
enlargement of sup. mediastinum secondary to right SVC, innominate artery

and ascending vertical vein.

4. Mild cyanosis, cardiac failure, recurrent chest infection, pulmonary HT.


1. What was the lesion?
2. What was the procedure carried out?
3. What are the complications ?

Answer

1. VSD
2. Device implant
3. Device displacement
Emboli formation
Haemolytic anaemia
1. Identify the procedure, where the device shown in this X-ray is used.
2. List two indications of this procedure.
3. What is the long-term non-cardiac complication, which may occur, if
the device is not placed in asymptomatic children?

Answer
1. ASD device closure
2. All symptomatic patients:
Asymptoma c pa ents with a Qp : Qs ra o of at least 2 : 1
3. Paradoxical (right to left shunt )
systemic embolization.
1. Identify the procedure, where the device shown in this X-ray is used.
2. List five complications, which may occur, if the device is not placed in
children with the above diagnosis.

Answer
1. PDA coil closure
2. Cardiac failure:
Infective endarteritis
Aneurysmal dilatation of the pulmonary artery
Calcification of the ductus
Non-infective thrombosis of the ductus with embolization
Paradoxical emboli
Pulmonary hypertension.
1) What is abnormalities , your diagnosis ?
2) Name three risk factor for this diagnosis?
3) What is the treatment?
4) What is the dose?

Answer
1. Respiratory distress syndrome
widespread opacification throughout both lung fields.
There are clear air bronchograms on both sides.
The heart border is not clearly defined.
The costophrenic and cardiophrenic angles are not clearly visualised.
There is an endotracheal tube.
2. Preterm, male, elective LSCS, gestational diabetes, multiple gestation, asphyxia
3. R x Antenatal corticosteroids - Betamethasone
Betamethasone 12 mg /12hour interval/I.M. /24 hours prior to delivery
Surfactant should be given if the dose has not been repeated since birth.
Ventilatory requirements are high and some centers
would consider high frequency oscillatory ventilation at this point. If this is not
available ventilation will probably need to be adjusted to improve the blood gases.
1. Describe the X ray of post term NB?
2. Diagnosis?
3. Give three recent advances in management ?

Answer

1. Hyperinflated lung with diffused patch infiltration , and atelectasis


2. MAS
3. Lung lavage , surfactant instillation , HFOV
1) Iden fy the condi on in the CXR of an ELBW newborn
2) Give the defini on of this condi on
3) Men on the stages of this condi on in a 34 wk old
4) What are the pharmacological strategies in the management of this condition
5) Expand INSURE

Answer
1. Bronchopulmonary dysplasia (BPD)
2. Current definitions include
total dura on of oxygen supplementa on requirement for >28 days,
degree of prematurity (<32 weeks gestational age at birth), and
Oxygen dependency at 36 weeks Postmenstrual age.
3. Stages:
a. Mild: Breathing room air at 56 days postnatal age or discharge*
b. Moderate: Need for <30% oxygen at 56 days postnatal age or discharge*
c. Severe: Need for > 30% oxygen and/or positive pressure (IMV/CPAP) at
56 days postnatal age or discharge* (* whichever comes first)
4. Pharmacological strategies
a. Vitamin A
b. Postnatal steroids
c. Superoxide dismutase
d. Furosemide
e. Intubate SURfactant Extubate
RADIOLOGY12

1. Diagnosis?

2. What is the clinical picture?

3. What is the requirement of echo before surgery?

Answers

1. Tracheo-esophageal fistula

2. Excessive drooling ,Respiratory distress

3. To rule out associated Congenital heart diseases and Right sided aorta
1. What is the abnormality?
2. What is the diagnosis ?
3. How is it suspected clinically ?
4. What is the management ?
5. What are three causes of respiratory distress in a baby born with this condition?

Answer
1. Bowel loops in Right hemithorax, mediastinal shift to left
2. Congenital Right diaphragmatic hernia
3. Respiratory distress
Mediastinal shift
Bowel sounds in the thorax
Scaphoid abdomen
4. no baging using a mask and mask ventilation avoiding lung inflation prevent the
bowel from distending any more with swallowed air.
NGT on drainage
Intubation and ventilation immediately after birth till pt stabilizes
Refer to NICU
Treat PPHN
5. Surgical correction A pediatric surgical opinion should be sought.
1. a) Mechanical compression of the lungs from the herniated viscera
b) Pulmonary hypoplasia from compression of the developing lungs in utero
c) Pulmonary hypertension
1. Diagnosis ?

2. Di eren al diagnosis of this Xray ?

3. Maternal condi on associated ?

4. Prognos c factors associated with be er outcome ?

Answer

1. Diagnosis: Congenital le Diaphragmatic Hernia (CDH)

2. Di erential diagnosis of this Xray:

-Congenital cystic adenomatoid malformation (CCAM)

-Cystic pulmonary interstitial emphysema

-Staphylococcal pneumonia with pneumatocele formation.

3. Maternal condi on associated: Polyhydramnios

4. Prognos c factors associated with better outcome:

-Hernia on a er 2nd trimester

-Absence of liver herniation

-Late onset of postnatal symptoms


Congenital left diaphragmatic hernia
Left Congenital lobar emphysema

Hyperinflation of the left upper lobe, paucity of vascular markings of the left
upper lobe, mediastinal shift to the right, atelectasis of the left lower lobe,
flattening of the left hemidiaphragm.

Common site of involvement In Left upper lobe.


Congenital left upper lobe emphysema.

Extension of the emphysematous lobe into the left lower lobe and its
displacement of the mediastinum toward the right.

In 50% of cases, a cause of CLE can be iden fied.


Congenital deficiency of the bronchial cartilage
external compression by aberrant vessels
bronchial stenosis
redundant bronchial mucosal flaps
kinking of the bronchus.
Describe chest xray ,whts Rx ?

Huge a left tension pneumothorax with mediastinal shift to the right.

endotracheal tube that is slightly high.

There is a central venous line with the tip at the thoracic inlet.

There is an umbilical venous line with a very low tip.

Rx A chest drain must be inserted immediately


Describe chest xray ,whts Rx ?

Large tension pneumothorax on the left.

Mediastinal shift to the right with tracheal deviation.

Transcutaneous oxygen electrode on left upper chest.

Rx Immediate drainage of the pneumothorax, needle in 2nd LT ICS then

Intubation and ventilation is very likely to be needed.


1. What is the radiological diagnosis?

2. Name 2 risk factors for the development of this condi on

Answer

1. Right pneumothorax mediastinal shift to left with left upper zone haziness

2. Risk Factors-

Assisted ventilation (including CPAP)

MAS

RDS

Other Air Leak Syndromes (e.g. PIE)

Pulmonary hypoplasia

CHD

Idiopathic or spontaneous
1. What is the abnormality
2. Three high risk situations when this condition is imminent
3. Management

Answer

1. Pneumothorax
2. Put a needle in second intercostal space then Intercostal drain(done)
1. Describe the X ray findings .
2. What is the diagnosis?
3. What is the treatment of choice ?
4. Write 4 life threatening complica ons of Kawasaki disease.
5. Write side effects pertaining to CVS of Digitalis

Answer

1. X ray chest PA view with air trapped s/o


2. Pneumopericrdium
3. O2 and monitoring if severe distress prompt evacua on.
4. MI, Coranry aneurusm, Thrombosis, DIC
5. Atrial, ventricular extrasystole, heart block,AV block, VT,VF
1. Identify the abnormality?
2. Clinical presentation?
3. Treatment ?

Answer

1. Pneumopericardium
2. Shock with weak pulses
3. Drainage
1. What is the abnormality ?
2. What is it a complication of ?

Answer
1. Pneumomediastinum
2. Forceful ventilation
Autosomal dominant.

It results from a muta on in the CBFA1 gene, which controls a key


transcription factor in osteoblast differentiation.

The anterior fontanelle often closes late , may be delayed eruption of teeth.

There can be bossing of the forehead.


A 17-year-old boy states that he is healthy, although he admits to being
treated for three cases of left lower lobe pneumonia over the past 10 years.

1. Diagnosis?
2. What is the treatment ?

Answers

1. Pulmonary sequestration.
2. Surgical lobectomy generally is curative.
1. What is the X ray suggestive of ?
2. What is the likely organism?
3. What are the complications?
4. What is the drug of choice?

Answer

1. Lobar Pneumonia
2. Pnemococous/Staphylococcus
3. pneumonic effusion, empyema
4. Penicillin for susceptible org and cefotaxime / vancomycin for penicillin
resistant org for 10-14 days
Consolidation in the right lower lobe , S. pneumoniae.

indications for admission in this patient?

Immunocompromised state
Toxic appearance
Moderate to severe respiratory distress
Requirement for supplemental oxygen
Complicated pneumonia
Dehydration
Vomiting or inability to tolerate oral fluids or medications
No response to appropriate oral antibiotic therapy
Social factors (e.g., inability of caregivers to administer medications at
home or follow-up appropriately)
If patient presents with fever and toxaemia.

1. give three differential diagnosis


2. Give three modalities of management

Answer

1. differential diagnosis
Lung Abcess
Infected Bronchogenic cyst
Infected Hydatid Cyst
2. management
Antibiotics (anaerobic +aerobic)
Chest physiotherapy
Percutaneous CT guided aspiration
6-year-old boy, who is presented with high fever, respiratory distress, and hypoxia. He
had an infected varicella lesion on his ear.

1. What are the CXR findings?

2. What is the most likely diagnosis?

3. What is your immediate management?

Answer

1. Complete opacifica on of the le lung field and a medias nal shi to right.

2. Empyema

3. Chest tube inser on and an bio cs.


1. What is the diagnosis?
2. plural fluid finding in bacterial pneumonia ?
3. What does VATS stand for?

Answers
1. Pleural Effusion (Right)

2. plural fluid finding / exudative pleural fluid

Proteins > 3.0 g/dL


Pleural Fluid LDH > 200 IU/L
Fluid to serum LDH ra o > 0.6
Cell count > 1000
3. Video Assisted thoracoscopy
17 month old boy brought with H/o inges ng Kerosene.
First X ray( A) was taken at 3 hours and second (B) a er few hours.
1. What is the role of gastric aspiration here on admission?
2. Ingestion of what amount is considered at risk for Pneumonitis ?
3. How long would observe this child ,if no abnormal symptoms develop.

Answer
1. Not to be done.
2. 30 ml
3. 8 12 hours
7-month-old child with worsening cough. On examination his temperature is
37.8oC. Respiratory rate 60 bpm, heart rate 130 bpm. He has moderate
intercostal recessions and his arterial oxygen satura ons are 88% on room air.
Ausculta on of his chest reveals bilateral sca ered fine crepts. weight is 7.3
kg (10th percen le), and length and head circumference are at the 95th
percentile for age.

1. Finding ?
2. Diagnosis?
3. What is the treatment (mention complete schedule)?

Answers

1. Interstitial infiltrates beginning in the perihilar region and spreading to


the periphery. Apices spared until later in the disease.
2. Pneumocystis pneumonia.
RADIOLOGY36

4. Finding ?
5. Diagnosis?
6. What is the treatment (mention complete schedule)?

Answers

1. reticulonodular infiltrate distributed fairly uniformly throughout the lungs


,reflect nodular interstitial spread without alveolar involvement
2. Miliary tuberculosis
3. 2HRZE + 7HR
1. Xray finding?
2. DX ?
3. DDx?

Answer

1. thin-walled, air-filled cysts within the RT lung parenchyma


2. Pneumatoceles
3. Staphylococcus Aureus Infection
Bronchogenic Cyst
Cystic Adenomatoid Malformation
Hyperimmunoglobulinemia E (Job) Syndrome
Pneumococcal Infections
Pneumonia
Pulmonary Sequestration
Tuberculosis
Pneumatoceles are thin-walled, air-filled cysts that develop within the lung
parenchyma frequent with staphylococcal infections, and they should not be
confused with a pulmonary abscess.
Pneumatoceles have thin, smooth walls and are seen with an improving
clinical picture thought to be a form of localized pulmonary interstitial
emphysema and are self limiting with only the rare case of a large, persisting
pneumatocele needing surgery, whereas pulmonary abscesses have thick, irregular
walls with an air fluid level and the child tends to be very ill.
1. Mention the abnormality in this CXR.
2. Men on 3 causes of this appearance.
3. What further investigations would you order?

ANSWER

1. Mediastinal enlargement
2. Lymphoma
Thymoma
Teratoma
3. CT chest,CT guided biopsy and HPE
This the Chest X ray of an 11 year old female child with h/o recurrent lower
respiratory infections.

1)What is the diagnosis?

2)Write the a) clinical features and b) one important diagnos c clinical sign

3)Which syndrome is associated with the above condi on?

4)Write the management

5)What is inves ga on of choice

Answer
1)Bronchiectasis
2)a) Productive cough with expectoration Hemoptysis ,FTT ,Cyanosis
b) Chest Deformities (Harrison's sulci) ,Crepitations, wheeze, crackles may
be heard on auscultation ,Clubbing
3) Kartagener's syndrome may be associated.
4) Management-
i. Treatment of underlying disorder
ii. Postural drainage
iii. Chest Physiotherapy
iv. Antibiotics
v. Surgical removal of the affected area
5) HRCT
1. What is the anatomical structure in which coin is lodged?

2. What is the location of carina with respect to thoracic vertebrae?

3. What are the anatomical areas of esophageal narrowing?

4. How can this foreign body be removed?

5. What complications you expect?

Answers

Esophagus (When foreign bodies lodge in the esophagus, the flat surface of
the object is seen in the AP view)
T4
Anatomic areas of esophageal narrowing
a. Cricoid
b. Tracheal bifurcation
c. Gastro-esophageal junction
4. Emergent Endoscopy

5. Airway compromise.

Esophageal rupture.

Erosion into the mediastinal structures.


Child admitted with sudden breathing problems . There was history of playing
with marbles at the time of development of marbles. X-RAY done shows ?

1. Describe X RAY
2. Diagnosis
3. Treatment

Answer

1. X-Ray findings:-
The right lung volume is increased and has herniated across the mid-line.
The left lung is compressed by the displaced heart and mediastinum.
The left lung remains airated and normal bronchi are seen on that side.
The right main bronchus cannot be traced from its origin.
2. Rt main bronchus partially obstructed by non opaque foreign body
3. Bronchoscopy and removal
Three year old toxic boy with fever, drooling, stridor, respiratory distress.

1.what Xray finding?

2. what Diagnosis ?

3. what Treatment ?

Answer
1. Xray finding:
-Increased space between the pharyngeal air shadow and the vertebrae.
-Posterior pharyngeal wall is bulging
2. Diagnosis: Retropharyngeal Abscess
3. Treatment:
- Intravenous antibiotics with or without surgical drainage.
A third generation cephalosporin with ampicillin-sulbactam or clindamycin to
provide anaerobic coverage.
RADIOLOGY44

Scenario: this is a 5 years old child who presented to you with history of cough
and fever for 4 days who got lethargic, his vaccine history is incomplete, this
hisX-ray

Q1: Describe the finding?

Q2: What is your diagnosis?

Q3: How would you approach this child?

Q4: what is the possible organism?

Q5: What is other considera on you need to make?

Answer
1. Lateral view of the neck with "thumb" sign
2. Epiglottitis
3. ABC, don't do throat exam, Antibiotics, consult anesthesia for possible
emergency tracheostomy, oxygen, IV fluid
4. H. Influenza
5. To give prophylaxis for the family contact
1. What is the x-ray finding?

2. What is the most likely diagnosis?

3. What are the Indica ons for hospitalization?

Answer
1. AP radiograph of the neck showing typical steeple sign (subglottic narrowing).
2. Viral croup (Laryngotracheobronchitis).
3. Indications for hospitalization include:
„h Persistent or worsening signs of respiratory distress despite therapy
„h Signs of impending or frank respiratory failure or compensated respiratory failure
„h Stridor at rest
„h Unreliable caretaker
„h Poor oral fluid intake
3 day neonate with Lethargy Feed refusal Abdominal distension (see x ray)
1. Describe the abnormalities on the X-ray ?
2. What is the diagnosis?
3. What is the radiological feature of Bell stage III NEC?
4. Name other conditions associated with pneumatosis intestinalis?

Answers
1. dilated bowel with thickened wall no air in the rectum
widespread intramural gas ,Pneumoperitoneum
2. NEC
3. Hirschsprung's disease
Pseudomembranous enterocolitis
Neonatal ulcerative colitis
Ischemic bowel disease

Stage I (suspected NEC)


Normal or mild dilatation or ileus
Stage II (definite NEC)
Intestinal dilatation and pneumatosis intestinalis (subserosal,submucosal air)
Stage III (advanced NEC, severely ill)
Same as II with portal vein gas and pneumoperitoneum
3 day neonate with Lethargy Feed refusal abdominal distension (see x ray)
1. What you see in this film?

2. What is your diagnosis?

3. What is the most likely cause in this preterm newborn?

Answers
1. Air under the diaphragm

2. Pneumoperitonium

3. Necro zing enterocoli s


Answer Following questions based on X Ray seen
1) What is abnormal in this X ray?
2) What is the ideal position of placement of umbilical arterial and U venous line?
2. After putting in a UA line, the right lower limb appears pale ,what would you do?
3. What is the level of the renal artery?
4. How do you maintain a UA line?

Answers
1) Abnormally placed umbilical arterial line in the subclavian artery
2) For umbilical arterial line - High: Between T7- T10; Low: Between L2-L3
For umbilical vein - Just above the diaphragm
3) Warm the other limb; If s ll pale >1/2 hour, remove the UA line
4) L-1
5) Use heparin infusion at rate of 0.5-1.0 Unit per hour
Single bubble -Pyloric atresia
1. What are two imp radiological abnormalities?
2. What is the diagnosis ?
3. What are the three imp investigations ?
4. What are the metabolic abnormalities expected?
5. What is the management ?

Answer

1. Distension of stomach ,No gas in the intestines


2. Pyloric Stenosis
3. USS abdomen: pyloric thickness >4mm/ pyloric length >14 mm
S electrolytes
Ba studies

4. Hypochloremic, hypokalemic, metabolic alkalosis

5. Management of the fluid & electrolytes + Ramstedt's operation


Double bubble- Duodenal atresia
Triple bubble- Jejunal atresia
1. What is the abnormality?
2. What is the likely diagnosis?
3. Delineate management.
4. Men on 3 complica ons

Answer
1. Multiple fluid levels ,gasless lower abdomen
2. Small bowel obstruction
3. Surgical correction
4. Dyselectrolytemia
Perforation
Exaggerated hyperbilirubinemia
Dysmotility syndrome
1. What is the diagnosis?
2. Describe three features seen on the X-ray of the disease?
3. What biochemical test would help clinch the diagnosis?
4. What is the treatment of the condition?

Answers
1. Rickets
2. features seen on the X-ray
a) Cupping
b) Widening of distal end of metaphysis
c) Fraying
3. Calcium, Phosphorus, Alkaline phosphatase
4. Injec on Vitamin D 6 lac unit IM stat and PO Calcium
1. What is the diagnosis?
2. This infant is 8 months old, what is the most likely type?
3. What is the earliest sign of this disorder?
4. What is the first radiological change that occurs in response to specific Rx?
5. How could this have been prevented?
6. What are the non - specific urinary findings in this disorder ? (at least 2)

Answer
1. Rickets
2. Vitamin D deficiency
3. Craniotabes
4. Appearance of provisional zone of calcification
5. Supplement of 400IU of vitamin D
6. Generalized aminoaciduria
> Glycosuria
> Phosphaturia
> Elevated urinary citrate
> Impaired renal acidification.
Short mother brings in her 1-year-old boy for the first time because she is

1-What is the most likely diagnosis?


2-What are the most likely serum laboratory findings?

Answer

1- Familial hypophosphatemic rickets of either the autosomal dominant or


sex-linked type.

2- The typical laboratory findings in this disorder are normal serum calcium
and low serum phosphate values.
1. what is the test you will ask for?
2. what is the treatment in this case?

Answer

1. Serum Ca+ and phosphate level


2. Vitamin D with Ca+ supplement
6 weeks infant K/C of Cholesta c jaundice (Extra-Hepatic Biliary Atresia)

1. What is the likely cause of fracture femur in this case?

2. How can this complica on be prevented?

3. How do you manage pruritus in these pa ents?

4. An infant with cholestasis, triangular facies, and a pulmonic stenosis


murmur is likely to have what syndrome?

Answers

1. Metabolic Bone disease (secondary to Vitamin D deficiency due to


malabsorption of fat soluble vitamins)

2. Replace 5,000-8,000 U /d of D 2, or 3 -5 µg/kq/d of 25 hydroxycholecalciferol

3. Ursodeoxycholic acid 15-20 mg/kg/day

4. Alagille syndrome (Arteriohepa c dysplasia)


Give five radiological findings ?

What is the diagnosis ?

What is the management ?

ANSWER

1. Ground glass appearance of bone


Thinned cortex
Periosteal calcification
White line of Fraenkel (well calcified cartilage)
Wimberger's sign (white ring)
2. Scurvy

3. Vit C 100-200 mg/ day + Dietary Therapy


X ray pictures of a 11 year old boy presen ng with recurrent long bone
fractures
1. Identify the condition?
2. Mode of inheritance ?
3. Underlying pathology?
4. Men on 1 di eren al diagnosis
5. Other clinical Features in this condi on? (Any 4)

1. Pyknodysostosis
2. Autosomal recessive
3. Lysosomal disorder due to genetic deficiency of Cathepsin K, which is
important for normal osteoclast function
4. Osteopetrosis
5. Short stature, Delayed closure of cranial sutures, fronto-parietal bossing,
short broad hands with hypoplasia of nails, nasal beaking, proptosis,
obtuse mandibular gonial angle
1. Findings in this x ray ?
2. What are the two important conditions which produce similar findings?
3. How do you differentiate radiologically these two conditions ?
4. What hematological problems can occur in a child with such x-ray findings ?

Answer
1. - Increased density of bone
- Changes suggestive of of rickets
2. - Osteopetrosis
- Pyknodysostosis
3. - Angle of mandible normal in osteopetrosis
- Increased angle of mandible in pyknodysostosis
- Distal phalanges normal in osteopetrosis
- Narrow distal phalanges in pyknodysostosis.
5. Anemia
1) Identify this condition.
2) Men on 3 treatment op ons
3) Men on 2 complica ons

ANSWER

1. Dense bones suggestive of osteopetrosis


2. Steroids,Interferon,Bone marrow Transplantation
3. Infections,Bleeds,Loss of vision
16-year-old boy who has a 3-month history of bilateral leg pain and lower back
pain. He also reports occasional low-grade fever and a 5- to 10-lb (2.25- to 4.5-kg)
weight loss over the last 3 months. He denies rashes or other symptoms.
His physical examination reveals loss of mobility of the lower spine when bending
forward and tenderness of both knees, hip radiograph was ordered.

1- What is the most likely diagnosis?


2- What other features you may see in this patient?
3- What finding you see in the hip radiograph?

answer
1- Ankylosing spondylitis, one of the diseases categorized as spondyloarthropathies,
or more recently, enthesitis-related arthritides.
2- Other features, including anterior uveitis, renal involvement, and rarely aortic
insufficiency, have been described in adolescents who have this disease.
3- Hip radiographs show evidence of sacroiliitis.
1. Describe the findings on the spine ?

2. Name the disease where this is seen?

3. Name 2 other skeletal complica ons of this condi on ?

Answer
1. Infarction affecting the central part of the vertebrae (fed by a spinal artery branch)
results in the characteristic H. vertebrae of sickle cell disease. The outer portions of
the plates are spared because of the numerous apophyseal arteries.
The lateral cxray shows multiple vertebral end-plate irregularities and depressions.
The peripheral portions of the end-plates are spared.
The appearance is due to bone infarction and subsequent collapse. This is an early
example of the classical h-shaped vertebrae seen in sickle-cell anaemia.
2. Sickle cell anemia (also seen in Gauchers disease)
3. Dactylitis, avascular necrosis of femoral head and humerus, osteomyelitis
Baby 4 wks old girl never seen her turn her head, which makes it difficult for
her to feed at the breast. They also note that her back does not appear
normal. On physical examination, her hairline appears low posteriorly. You
confirm that she does not turn her head, and when placed prone, does not
turn her head to the side. Her right scapula appears to be higher than the left,
and you note that the spine does not appear to be perfectly straight,
suggesting congenital scoliosis x ray done .
1. Diagnosis?
2. What other associated defects may be seen??

Answers

1- The Klippel-Feil syndrome involves the fusion of cervical vertebrae and


occurs in approximately 1 in 42,000 births, with a 65% female predominance. It
is usually a sporadic event. Due to neck immobility, affected individuals are at
risk of cervical spine injury.
2-Associated defects may include deafness (conduction or
sensorineural, occurring in up to 30% of pa ents), congenital heart
defects (usually ventricular septal defect), rib defects,hemivertebrae, Sprengel
anomaly (elevation of the scapula), scoliosis, and renal anomalies.
1. Describe the lesion?
2. Give two D/D
3. What is the triad of tumor lysis syndrome?

Answers

1. Osteolytic lesion of skull


2. Histiocytosis
Ewing`s Sarcoma
Lymphoma
Bone cyst
3. Hyperuricemia, hyperkalemia, and hyperphosphatemia
1. What is the abnormality seen on this radiograph?
2. Name 2 disorders which may give rise to this abnormality

ANSWER

1. Xray skull lateral view showing multiple lytic lesions


2. Histiocytosis
3. Secondaries
4. Multiple myeloma
1. Describe the X-ray appearance
2. Pathogenesis of the appearance
3. Possible Diagnosis
4. Which disorder is most commonly associated with an elevated MCHC?
5. How is the corrected reticulocyte count calculated?

Answers

1. Sunray appearance
2. Medullary widening
3. Chronic hemolytic anaemia
4. Hereditary spherocytosis
5. Corrected retic count = reticulocyte % × (patient Hct/normal Hct)
Dr=Wahid Helmi

Revised & prepared


Dr- Wahid Helmi
Consutant Pediatrician
Pediatric Department Zarka
Hospital –Dymiate=Egypt
MCQ IN PEDIATRIC
CARDIOLOGY
Dr=Wahid Helmi

You are seeing a 10-year-old girl in the clinic for a wellchild check. During the
visit, her parents mention that she has been complaining of her heart "fluttering."
It happens sometimes while she is just watching TV and other times when she is
playing. You are able to elicit that it seems to start suddenly and stop equally
suddenly. Her father mentions that he has felt his daughter's heart beat during the
episode and that it is "too fast to count." She is otherwise fairly asymptomatic
during the episode. Of the following, what is the most likely diagnosis?
a. Sinus arrhythmia
b. Second-degree heart block (Mobitz I)
c. Second-degree heart block (Mobitz II)
d. Atrial fibrillation
e. Reentrant tachycardia.

A 3-year-old boy is brought in to the emergency department by his parents for


fatigue and decreased appetite. On examination, he has cool extremities, a gallop
rhythm, and hepatomegaly. His heart rate is 140 beats/ min, and his respiratory
rate is 54 breaths/min. His parents report that he and his older sister were sick
with cough and congestion 3 weeks ago, but he never seemed to completely
recover. You decide to obtain a chest x-ray. What radiologic finding is most likely
for this suspected diagnosis?
a. Narrowed mediastinum
b. Complete whiteout of bilateral lung fields
c. Cardiomegaly .
d. Pneumothorax
e. Rib notching
Dr=Wahid Helmi

You are seeing a 15-year-old male patient in your clinic for the first time, along
with his mother and father. His mother reports that her father and brother had
heart attacks at 5 7 and 44, respectively. There are many other members of the
family, including your patient's mother, who have been diagnosed with
hypercholesterolemia. Your patient is overweight with a BMI of 28. You send him
for a fasting lipid panel and find that his total cholesterol is slightly elevated, LDL
is normal, HDL is normal, and triglycerides are moderately elevated. What is your
next step in managing this patient?
a. Diet and exercise recommendations .
b. Statin initiation
c. Echocardiogram
d. Genetic testing
e. Liver ultrasound

Answer d.
Dr=Wahid Helmi

Answer---B

A 3-year-old boy presents to the Emergency Department with 5 days of fever and
irritability. On examination, he has cervical lymphadenopathy, his eyes appear
red, and he is noted to have a maculopapular rash on his abdomen. Laboratory
results reveal a hemoglobin of 10.3 g/dL and platelets of 600,000/ mL. The next
best step in management is:
a. Obtain an echocardiogram
b. Culture the oropharynx and order an antistreptolysin 0 (ASO) titer
c. Treat the patient with 2 g/kg of IVIG.
d. Order antibody titers to Borrelia burgdorferi e. Obtain a heterophile antibody test.

A 10-year-old boy presents to the ED with 5 days of fever. He complains of chest


pain and difficulty catching his breath. Chest radiography shows an enlarged
cardiac silhouette. On examination, he is leaning forward to catch his breath, and
his heart rate is 130 beats per minute. His blood pressure is normal without pulsus
paradoxus. The next best step to establish the diagnosis is:
a. Obtain an electrocardiogram
b. Culture the oropharynx and order an antistreptolysin 0 (ASO) titer
c. Treat the patient with 2 g/kg of IVIG
d. Order antibody titers to Borrelia burgdorferi
e. Obtain large volume blood cultures.
Dr=Wahid Helmi

A 3-year-old recent immigrant from Somalia presents for his first well-child care
visit. On examination, the child is noted to be below the 3rd percentile for weight.
AII/VI soft, vibratory systolic ejection murmur is auscultated along the left sternal
border. The murmur becomes louder when the child is laid supine. No
hepatomegaly is noted. What is the next best step in management?
a. Referral to the emergency department for urgent
echocardiogram and evaluation by a cardiologist
b. Referral to cardiology clinic
c. Well-child visit in 1 year
d. Investigation of causes of failure to thrive.
Answer--d. This child's murmur has multiple characteristics of abenign murmur of
childhood: It is soft, systolic ejection
in quality, and becomes louder when the child is supine.
The vibratory nature of the murmur is also suggestive of a
Still' s murmur, a common murmur in this age range. The
child should be evaluated for other etiologies of failure to
thrive before consultation with a cardiologist.

15-year-old girl presents for evaluation after a syncopal event during track
practice. She was approaching the finish line when observers saw her fall. By the
time bystanders reached her, she had regained consciousness and appeared to be
back to baseline. She sustained several abrasions and a bruise on her shin during
the fall.She reports having experienced dizziness with exertion in the past, but she
has no recollection for this specific event. She denies any prior history of loss of
consciousness.The next step in management is:
a. Referral to an eating disorder specialist
b. Recommendation to increase fluid intake to 2 L or
more per day
c. Referral to a cardiologist and restriction from all exercise
until she is evaluated.
d. Referral to a cardiologis
Answer--c.
Dr=Wahid Helmi

Answer---a.

A 6-week-old, full term, previously healthy infant presents to your clinic because the
parents feel that the child has not been eating well and grunts with every feed. They
also report that she occasionally appears to be sweating with feeds and has to be woken
up to breastfeed. Vital signs reveal that her weight is at the 8th percentile (birth
weight at the 45th percentile), heart rate is 163 beats/min, respiratory rate is 62
breaths/minute, and saturation is 97%. Her right arm blood pressure is 77/48 mm
Hg, and her right leg blood pressure is 84/45 mm Hg.On physical examination, she
displays increased work of breathing, and diffuse rales can be auscultated. She has
a soft, I/VI holosystolic murmur that radiates throughout the precordium. What is the
most likely cause of this child's congestive heart failure symptoms?
A. Coarctation of the aorta
B. TGA
C. TOF
D. VSD
E. Bicuspid aortic valve
Answe-- d. Congestive heart failure can have a variety of etiologies,
but most fall into the categories of ventricular dysfunction,volume overload, and
pressure overload. In the setting of a large VSD, there is unrestricted blood flow
from the left ventricle into the right ventricle, causing a volume and pressure load to the
pulmonary circulation,worsening as pulmonary vascular resistance falls.
The increased pulmonary blood flow leads to pulmonary edema and respiratory
distress.

You are called to the newborn nursery to evaluate a2-day-old infant who appears
to be in distress. After placing her on monitors, you note that her right arm
saturation is 94% and her right leg saturation is 74%. She has a heart rate of 1 7 5
beats/ min and a respiratory rate of 7 7breaths/min. Her right arm blood pressure
is 87145 mm Hg, andherrightlegbloodpressureis 62/31 mmHg. On examination,
she has diminished 1 + pedal pulses with cool extremities. Of the following, what is
the most likely diagnosis and subsequent management strategy?
A. TGA, balloon atrial septostomy
B. TOF, prostaglandin infusion
Dr=Wahid Helmi

C. Interrupted aortic arch, prostaglandin infusion.


D. Total anomalous pulmonary venous return, emergent surgery
E. VSD, oxygen
Answer ---. c. In an interrupted aortic arch, the aorta is discontinuous,
with the proximal portion usually supplying the head and neck vessels and ending at the
level of the ductus,and with the distal portion arising from the ductus directly. Because
of this, the blood supply to the lower portion of the body is completely dependent on
patency of the ductus, making it a ductal-dependent lesion

The most common cyanotic congenital heart defect seen in infants of


diabetic mothers is which of the following?
a. d-Transposition of the great arteries .
b. Hypoplastic left heart syndrome
c. Tricuspid atresia
d. Tetralogy of Fallot
Dr=Wahid Helmi

Physiologic responses to asphyxia include which of the following?


a. Tachycardia, decreased central venous pressure,vasoconstriction of the
cerebral vessels
b. Bradycardia, increased central venous pressure,vasoconstriction of the
skeletal muscle vessels.
c. Bradycardia, decreased central venous pressure,vasodilation of the
cerebral vessels
d. Tachycardia, increased central venous pressure,vasoconstriction of the
skeletal muscle vessels

A 1-week-old infant with tetralogy of Fallot is undergoing a rule-out sepsis


evaluation. The infant’s baseline saturations are 98%, with a 2/6 systolic ejection
murmur along the left sternal border. During the blood draw, the infant’s
saturations drop to 60% and the infant becomes visibly cyanotic. On auscultation,
the lung sounds remain clear, but the murmur is no longer audible. What
interventions should be performed?
a. Intubation with 100% O2
b. STAT echocardiogram
c. Arterial blood gas
d. Morphine and phenylephrine administration.
Answer----d. The infant is this scenario is having a hypercyanotic spell caused by an acute decrease in
pulmonary blood flow. These are commonly triggered by medical proce-dures such as blood draws.
Morphine and phenylephrine are first-line agents to counteract the hypercyanotic spell.
Dr=Wahid Helmi

Newborn infant was found to have interrupted aortic arch type B, with truncus
arteriosus. What other features is the infant likely to have?
a. Epicanthal folds with a single palmar crease
b. Elfin facies
c. Broad thumbs and toes
d. Absent thymus on chest x-ray and hypocalcemia.
d. Both type B interrupted arch and truncus arteriosus are associated with 22q11
deletion syndrome. Infants with 22q11 deletion syndrome frequently have hypo-
calcemia and an absent thymus.

A term infant undergoes echocardiography after birth due to a murmur.


Numerous small cardiac tumors are identified throughout the heart. Which
genetic disease is likely?
a. Down syndrome
b. Williams syndrome
c. Tuberous sclerosis.
d. Hunter syndrome
Answer---- c. Numerous cardiac tumors found in a neonate are almost always
rhabdomyomas, which are strongly asso-ciated with tuberous sclerosis

An infant was born at full term to a 29-year-old woman with lupus by cesarean
section due to persistent fetal bradycardia. The newborn appears vigorous,with
normal capillary refill, blood pressure of 80/55 mm Hg, and 2+ femoral pulses.
The heart rate, however,is 65 beats/min, with a narrow QRS complex not
associated with p waves. What is the best course of action?
a. Begin isoproterenol
b. Begin cardiopulmonary resuscitation
c. Monitor closely on telemetry.
d. Use transcutaneous pacing
c. This infant has complete heart block related to trans-placental transfer of SSA and/or
SSB antibodies related to maternal lupus. Although the heart rate is low, the infant has
adequate cardiac output, as evidenced by good capillary refill and blood pressure.
Monitoring is all that is required in this situation.
Dr=Wahid Helmi

A 3-day-old infant suddenly develops tachycardia to 240 beats/min. Adenosine is


administered, which slows the ventricular rate briefly to 140 beats/min, revealing a
sawtooth-like pattern between the QRS complexes. Transesophageal pacing is used to
terminate the tachycardia. Which drug is best to use as prophylaxis
to prevent future tachycardia?
a. Propranolol
b. No prophylactic therapy needed.
c. Digoxin
d. Amiodarone
b. This infant had an episode of atrial flutter, which is very unlikely to recur and thus
does not warrant pro-phylaxis

An infant with a large ventricular septal defect and congestive heart failure is
being medically managed with digoxin, enalapril, and furosemide (Lasix).
Spironolactone was added yesterday due to persistent hypokalemia.
Overnight, the infant developed Mobitz II heart block. This is a toxicity of which
medication?
a. Spironolactone
b. Digoxin.
c. Enalapril
d. Lasix
b. Atrioventricular block is a well-described result of digoxin toxicity. Digoxin toxicity
is more likely to occur in the setting of hypokalem

A 10-day-old infant is being treated for sepsis with antibiotics and dopamine. The
infant is requiring escalating doses of dopamine. All the interventions below can
increase the infant’s response to the current dose of dopamine, except for one.
Which intervention does not increase an infant’s sensitivity to catecholamines?
a. Beginning steroid administration
b. Normalizing pH
c. Normalizing serum calcium levels
d. Normalizing potassium levels.
Answer----d.
Dr=Wahid Helmi

1. You are called to the newborn nursery to evaluate a 2-hour-old newborn who
has suddenly become cyanotic. The oxygen saturation on room air is 69%, and the
patient is tachycardic and tachypneic. Oxygen is administered without
improvement in the patient’s oxygen saturation.On examination, you hear a loud
S2 and no murmur. A chest radiograph shows increased pulmonary vascular
markings, a narrow mediastinum, and a small heart. Which of the
following would be the next step in management?
A. Start digoxin.
B. Refer the patient to surgery for placement of a Blalock–Taussig shunt.
C. Refer the patient to surgery for repair of a ventricular septal defect.
D. Proceed with pulmonary balloon valvuloplasty.
E. Begin infusion of prostaglandin E (PGE).
This patient’s clinical presentation and physical examination are
most consistent with transposition of the great arteries (TGA).

2. A 20-month-old boy with tetralogy of Fallot is admitted for evaluation of


cyanosis that is increasing in frequency. As you conclude your history and
physical examination, you witness an episode of cyanosis when the patient’s
brother makes him cry. As the crying increases, the
patient becomes more and more cyanotic. On examination, his cardiac murmur is
now much softer than before he began crying. What is the next most appropriate
step in management?
A. Intubate and begin mechanical ventilation.
B. Administer intravenous dopamine.
C. Place the patient in a knee-chest position.
D. Administer subcutaneous epinephrine.
E. Call for a cardiology consult.

Tetralogy of Fallot (hypercyanotic or “tet”) spells are


defined as paroxysmal episodes of hyperpnea, irritability, and prolonged crying that
Dr=Wahid Helmi

result in
increasing cyanosis and decreasing intensity of the heart murmur.

3. A 10-year-old girl is seen for a routine health maintenance evaluation. Five


years ago, she underwent surgical repair of coarctation of the aorta. On
examination, the blood pressure in her right arm is 173/81 mm Hg, and her
oxygen saturation is 97% in room air. Auscultation reveals a systolic ejection
murmur audible throughout the precordium. The patient is otherwise
asymptomatic. Which of the following would be the most appropriate next step in
management?
A. Check the blood pressure in all extremities.
B. Refer the patient to a cardiac surgeon promptly.
C. Obtain an echocardiogram to rule out a bicuspid aortic valve.
D. Recheck the oxygen saturation in 100% oxygen.
E. Have the patient return in 6 months for reevaluation

Restenosis is a known complication from repair of coarctation of the aorta, and these
clinical features are consistent with restenosis. Patients with coarctation of the aorta
classically present with hypertension in the right arm, and commonly in the left
arm,and reduced blood pressures in the lower extremities.

4. A 7-year-old boy presents with a 3-day history of fever (temperature to 39.7°C


[103.5°F]),shortness of breath, and weakness. He also complains of chest pain,
which is most intense when he lies down and improves when he sits upright. His
past medical history is significant for closure of a ventricular septal defect 2 weeks
ago. Which of the following findings is consistent with the most likely diagnosis?
A. Splinter hemorrhages
B. Pulsus paradoxus
C. Heart rate of 260 beats/minute with absent P waves
D. Prolongation of his QT interval
E. Tenderness at two of his costochondral joints
Dr=Wahid Helmi

This patient’s clinical presentation is most consistent with


pericarditis. The likely cause of his pericarditis is postpericardiotomy syndrome, given
the
recent closure of his ventricular septal defect before the onset of his symptoms.

5. A 1-month-old female infant is seen in your office for a routine health


maintenance evaluation.On examination, you hear a grade 4 holosystolic murmur
at the left sternal border. Femoral pulses and oxygen saturation in room air are
normal. The infant is otherwise well and growing normally. Which of the
following statements regarding this patient’s condition is correct?
A. Without intervention, congestive heart failure will develop.
B. Eisenmenger syndrome will eventually occur.
C. Surgical closure of the patent ductus arteriosus is indicated.
D. The murmur may disappear without intervention.
E. Balloon valvuloplasty is indicated.

This patient’s murmur is consistent with a small ventricular septal


defect (VSD). With a small VSD, a patient is likely to remain asymptomatic with
normal
growth and development.
Dr=Wahid Helmi

6. You are called to the nursery to evaluate a male newborn with cyanosis. On
auscultation, you hear a single S2 but no murmur. Pulse oximetry shows an
oxygen saturation of 72% in room air. An electrocardiogram reveals left axis
deviation and left ventricular hypertrophy. What is his likely diagnosis?
A. Tetralogy of Fallot
B. Transposition of the great arteries
C. Truncus arteriosus
D. Total anomalous pulmonary venous return
E. Tricuspid atresia with intact ventricular septum
Tricuspid atresia is the only cause of cyanosis in the
newborn period that manifests with left axis deviation and left ventricular hypertrophy
on
electrocardiogram (ECG).

Patients with tetralogy of Fallot present with a systolic murmur of pulmonary


stenosis and right ventricular hypertrophy (RVH) on ECG.
Patients with tricuspid atresia without a ventricular septal defect
have a single S2 as a result of the usual coexistence of pulmonary atresia and do not
have a
murmur.
Patients with transposition of the
great arteries also have no murmur and a single S2 but will have RVH on ECG.
Similarly, RVH
is present in total anomalous pulmonary venous return, along with a systolic murmur.
Truncus arteriosus manifests as combined ventricular hypertrophy with both a systolic
and
diastolic murmur.
Dr=Wahid Helmi

7. A 4-year-old boy is in the office for a routine health maintenance evaluation. His
examination is normal except for multiple deep dental cavities. You plan on
referring him for dental evaluation and possible dental extraction. His mother
reminds you that he has a “heart condition.” Which of the following cardiac
conditions requires antibiotic prophylaxis against endocarditis?
A. Patch repair of ventricular septal defect repaired 4 months ago
B. History of uncomplicated Kawasaki disease
C. Wolff–Parkinson–White syndrome
D. Patent ductus arteriosus
E. Ostium secundum atrial septal defect

8. You see a 7-week-old male infant with cough and poor feeding. Examination
reveals a respiratory rate of 72 breaths/minute and a heart rate of 170
beats/minute. His weight is 7 pounds 6 oz, just 2 oz more than his birth weight.
You hear diffuse rales throughout the lung fields and a systolic murmur on
auscultation. The liver is 4 cm below the right costal margin.
Which of the following conditions is the most likely cause of his signs and
symptoms?
A. Large ventricular septal defect
B. Ostium secundum atrial septal defect
C. Small patent ductus arteriosus
D. Critical or severe aortic stenosis
E. Mild to moderate pulmonary stenosis
Dr=Wahid Helmi

9. A thin 5-year-old boy presents for a routine health maintenance evaluation. He


feels well and is growing normally. On examination, you hear a continuous
murmur below the right midclavicle. The murmur is loudest while the patient is
sitting and disappears while he is supine. The femoral pulses are normal. Which of
the following conditions is the most likely diagnosis?
A. Aortic stenosis with regurgitation
B. Venous hum
C. Patent ductus arteriosus
D. Still’s murmur
E. Pulmonic systolic murmur
1-This murmur is most consistent with a venous hum, an innocent heart murmur.
Aortic stenosis with 2-regurgitation presents with systolic ejection and decrescendo
diastolic murmurs.
The murmur of a patent ductus arteriosus (PDA) is generally
continuous and machinery-like and does not vary with position. Patients with PDAs
generally
also have brisk pulses.
3-Both a Still’s murmur and a pulmonic systolic murmur are innocent
systolic murmurs. A Still’s murmur is usually a grade 1–3 systolic murmur best heard
at the
mid-left sternal border.
4-A pulmonic systolic murmur is a grade 1–2 high-pitched systolic
murmur best heard at the upper left sternal border.
5-Both the Still’s murmur and the pulmonic systolic murmur are loudest supine.
Dr=Wahid Helmi

10. A 15-year-old boy complaints of chest pain that occurs during basketball
practice. He is otherwise healthy and has no history of cardiac problems.
Examination is normal except for aharsh systolic ejection murmur at the apex that
worsens with standing and the Valsalva maneuver. An electrocardiogram
demonstrates left ventricular hypertrophy and left axis
deviation. Which of the following is the most appropriate initial management at
this time regarding the likely diagnosis?
A. Start propranolol to reduce left ventricular outflow tract obstruction.
B. Admit for urgent aortic balloon valvuloplasty.
C. Reassure the patient that the murmur is innocent and allow complete athletic
participation.
D. Admit for surgical myomectomy for septal hypertrophy.
E. Begin albuterol, as the patient’s chest pain is likely caused by asthma.
Dr=Wahid Helmi

Second Part

1. The most common cyanotic heart disease manifesting as congestive cardiac


failure during first week of life is:
(a) Pulmonary stenosis
(b) Fallot’s tetralogy
(c) Tricuspid atresia
(d) Hypoplastic left heart syndrome.

2. Congenital heart disease which causes death in the first week of life
(a) VSD
(b) TOF
(c) Epsteins anomaly
(d) Hypoplastic left ventricle.

3. The most common cause of heart failure in infants is:


(a) Myocarditis .
(b) Rheumatic fever
(c) Fluid over load
(d) Cardiomyopathy

1D 2D 3A
4. Congestive cardiac failure is diagnosed in an infant by:
(a) Basal crepts
(b) Elevated JVP
(c) Pedal oedema.
(d) Liver enlargement
Dr=Wahid Helmi

5. Excessive sweating in a young infant may be manifestation of


(a) Polycythaemia
(b) Phenylketonuria
(c) Heart failure.
(d) Anxiety

6. The most consistent early clinical sign of congestive cardiac failure in infants is:
(a) Oedema of feet .
(b) Basal rates
(c) Hepatomegaly
(d) Raised jugular venous pressure.

7. Which of the following is not a characteristic of right sided failure?


(a) Pulmonary oedema.
(b) Ascites
(c) Oliguria
(d) Dependent oedema

8. Digitalis toxicity can be diagnosed if ECG shows:


(a) Prolonged PR interval.
(b) T wave inversion\
(c) Shortening of QT interval
(d) Ventricular bigeminy
9. Compared to the oral digitalizing dose parenteral dose of digoxin should be:
(a) 1/4
(b) 1/2
(c) 1/3
(d) 2/3.
Dr=Wahid Helmi

10. Carey Coombs’ murmur of rheumatic carditis is:


(a) An apical pansystolic murmur
(b) An apical mid diastolic murmur .
(c) An apical early diastolic murmur
(d) A basal ejection systolic murmur

11. Essential feature of diagnosis of a acute rheumatic fever is:


(a) Recent sore throat infection.
(b) Erythema marginatum
(c) 1 major and 2 minor Jone’s criteria
(d) Prior H/o of rheumatic fever

4 C 5 C 6 A 7 A 8 A 9 D 10 B 11 A

12. Acute rheumatic fever is characterised by


(a) Erythema marginatum
(b) Atypical rugling pansystolic murmur
(c) Tender nodule
(d) Haemoptysis

13. Major manifestations of rheumatic fever are A/E


(a) Streptococcal antibody
(b) Polyarthritis
(c) Subcutaneous nodule
(d) Chorea

14. All of following are recognized manifestation of acute rheumatic fever except
(a) Abdominal pain
(b) Epistaxis
(c) Chorea
(d) Subcutaneous nodules
Dr=Wahid Helmi

15. The diagnosis of rheumatic fever is best confirmed by:


(a) Throat swab culture
(b) Raised ESR
(c) ASLO titre
(d) ECG changes
16. Rheumatic fever is suggested by the presence of:
(a) Symmetrical polyarthritis
(b) Nail clubbing
(c) Anemia
(d) ECG evidence of prolonged PR interval

17. An 8-year-old male child is admitted with a diagnosis of rheumatic fever with
arthritis, carditis and congestive heart failure. with reference to this case, consider
the following as initial lines of management:
(1) Eradication of remnant streptococcal infection.
(2) Administration of an anti-inflammatory drug.
(3) Institution of decongestive therapy
(4) Institution of graded and gradually increasing exercise.
Which of these should actually consist of initial line of management?
(a) 1, 2 and 3 (b) 1, 2, 3 and 4 (c) 2, 3 and 4 (d) 1 and 4
12 A 13 A 14 B 15 C 16 D 17 A

18. Bacterial endocarditis is most commonly seen in:


(a) VSD .
(b) PDA
(c) ASD
(d) AS
Dr=Wahid Helmi

19. Infective endocarditis not seen in:


(a) ASD .
(b) TOF
(c) VSD
(d) MR
20. A wide and fixed split second heart sound occurs in:
(a) Mitral stenosis
(b) Atrial septal defects .
(c) VSD
(d) Coarctation of aorta
21. Which one of the following does not produce cyanosis in the first year of life
(a) Artrial septal defect .
(b) Hypoplastic left heart syndrome
(c) Truncus arteriosus
(d) Double outlet right ventricle.
22. True about ASD:
(a) Patient foramen ovale
(b) Increase pulmonary artey flow leads to left parasternal heave .
(c) S2 widen
and variable
(d) Systolic murmur due to rapid flow of blood

23. VSD shunt reversal is called:


(a) Eisenmenger’s syndrome
(b) Eisenmenger’s complex .
(c) Ebstein’s anomaly
(d) None of the above
Dr=Wahid Helmi

24. Natural course of events in untreated ventricular septal defects except:


(a) Spontaneous closure of defect
(b) Development of pulmonary insufficiency
(c) Subacute bacterial endocarditis
(d) A normal life without symptoms .

25. A young boy had developed congestive failure and was found to have
membranous VSD. He Spontaneously showed improvement in his condition. The
is most likely to be due to:
(a) Perimembranous closure of VSD
(b) Development of AR
(c) Pulmonary vascular changes .
(d) Infective endocarditis

26. Differential cyanosis and clubbing is seen in


(a) PDA and reversal of shunt .
(b) TOF
(c) VSD + ASD
(d) MS + MR
18 A 19 A 20 B 21 A 22 B 23 B 24 D 25 C 26 A

27. Pure left sided failure may be seen with:


(a) ASD
(b) Aortic atresia
(c) Patent ductus arteriosus .
(d) Pulmonary valvular obstruction
28. One-year-old child with PDA; which is true:
(a) Symptoms similar to artopulmonary window .
(b) Chances of spontaneous closure high
(c) Indomethacin may help in closure
(d) Indocarditis is rare
Dr=Wahid Helmi

29. Preterm infant with PDA is given...to effect closure of the PDA:
(a) Corticosteroids
(b) Indomethacin .
(c) Aspirin
(d) Hyperbaric oxygen

30. The great danger of patients with patent ductus arteriosus is:
(a) Syncope
(b) Convulsions
(c) Arrhythmia
(d) Bacterial endocarditis .

31. All are characteristic of Fallot’s tetralogy except:


(a) Infundibular stenosis
(b) VSD
(c) Overriding aorta
(d) Left ventricular hypertrophy.

32. Which one of the following congenital heart disease has cyanosis without
cardiomegaly and /or congestive heart
failure?
(a) Transportation of great arteries
(b) Fallot’s tetralogy .
(c) Congenital mitral regurgitation
(d) Congenital pulmonary stenosis
33. Anoxic spells in tetralogy of Fallot’s are precipitated by:
(a) Fever
(b) Exertion
(c) Crying on feeding
(d) All .
Dr=Wahid Helmi

34. The congenital heart disease that has least chance of heart failure in infants:
(a) TOF .
(b) VSD
(c) TAPVC
(d) ASD

35. Cyanosis and breathlessness in a child is seen in:


(a) Transposition of great vessels
(b) Tetralogy of Fallot .
(c) Ventricular septal defect
(d) Atrial septal defect
36. Which of the following conditions does not present with CHF in neonate:
(a) Total anomalous pulmonary venous connection
(b) Coarctation of aorta
(c) Tetralogy of Fallot.
(d) Transposition of great vessels

37. In TOF:
(a) Cynotic spells are due to arrhythmias
(b) CXR and ECG are typically normal
(c) Wide split 2nd HS on inspiration
(d) Central cyanosis with clubbing.

27 C 28 A 29 B 30 D 31 D 32 B 33 D 34 A 35 B 36 C 37 D

38. A blue infant was found to have oligemic lung fields with normal sized heart.
The diagnosis is:
(a) Transposition of great vessels
(b) Tricuspid atresia
(c) TOF .
(d) Pulmonary stenosis
Dr=Wahid Helmi

39. Which one of the following is the most common cause of cyanotic congential
heart disease?
(a) Dextrocardia
(b) Fallot’s tetralogy.
(c) Atrial septal defect
(d) Coarctation of aorta

40. Pulmonary plethora is seen in all except:


(a) VSD
(b) ASD
(c) Fallot’s tetralogy.
(d) PDA

41. Blalock Taussing’s operation involves:


(a) Right pulmonary artery with aorta
(b) Left pulmonary artery with ipsilateral subclavian A .
(c) Right
pulmonary artery with descending aorta
(d) Left pulmonary artery with ascending aorta

42. Potts shunt is running from:


(a) Right subclavian artery to right pulmonary
(b) Descending aorta to left pulmonary.
(c) Left subclavian
to left pulmonary
(d) Ascending aorta to right pulmonary
Dr=Wahid Helmi

43. Tricuspid atresia all are true except:


(a) R.V. hypoplasia
(b) Left axis deviation
(c) Split S2 in inspiration.
(d) Pulmonary oligaemia

44. A child with central cyanosis and enlarged left ventricle the probable diagnosis
is:
(a) Tricuspid atresia.
(b) Eiesenmenger’s syndrome
(c) Tetralogy of Fallot
(d) Anomalous pulmonary artery

45. Left axis deviation with left ventricular hypertrophy is seen:


(a) Tricuspid atresia .
(b) Tetralogy of Fallot
(c) Coarctation of aorta
(d) Ventricular septal defect

46. True of tricuspid atresia are A/E:


(a) Right axis deviation .
(b) Left axis deviation
(c) Left ventricular hypertrophy
(d) Severe cyanosis

47. The best position for examination of cardiac murmurs in a child is:
(a) Sitting
(b) Standing
(c) Right lateral
(d) Recumbent.
Dr=Wahid Helmi

48. Which of the following congenital cyanotic disease presents with gallop
rhythm, left parasternal murmur, pericardial
friction rub:
(a) Ebstein anomaly.
(b) TA PVC (c) TGA
(d) Eissenmenger’s complex

38 C 39 B 40 C 41 B 42 B 43 C 44 A 45 A 46 A 47 D 48 A

49. In transposition of great vessels, all are true except:


(a) Aorta arises from the right ventricle
(b) Mitral valve is continuous with the aortic valve
(c) Causes.
jaundice immediately after birth
(d) None of the above

50. Emergency treatment for TGV:


(a) Balloon septostomy .
(b) Oxygen
(c) Ventilation
(d) Digoxin

51. Congestive heart failure, LVH and systolic heart murmur in a 29-day-old child
would suggest:
(a) TGA.
(b) VSD
(c) Rheumatic fever
(d) TOF (OPG/5th413)
Dr=Wahid Helmi

52. A neonate has central cyanosis and short systolic murmur on the 2nd day of
birth. The diagnosis is:
(a) TGV.
(b) TOF
(c) VSD
(d) ASD

53. A five-day-old. Full term male infant was severely cyanotic at birth.
Prostaglandin E was administered initially and later balloon atrial septostomy was
done which showed improvement in oxygenation. The most likely
diagnosis of this infant is:
(a) Tetralogy of Fallot
(b) Transposition of great vessels .
(c) Truncus arteriosus
(d) Tricuspid atresia

54. A neonate is found to have central cyanosis and a cardiac murmur at 30 hours
of age. The most probable diagnosis is.
(a) Endocardial cushion defect
(b) Ventricular septal defect
(c) Transposition of great vessels.
(d) Patent ductus arteriosus
55. True in total anomalous pulmonary connection are A/E
(a) The total pulmonary venous blood reaching the right atrium
(b) Always associated with VSD.
(c) The oxygen saturation of the blood in the pulmonary artery is higher to that in the
aorta
(d) Infracardiac type is always obstructive
Dr=Wahid Helmi

56. Figure of eight appearance is seen in:


(a) Partial anomalous pulmonary vessels
(b) Total anomalous pulmonary vessels .
(c) Truncus arteriosus
(d) Transposition of great vessels

49 C 50 A 51 A 52 A 53 B 54 C 55 B 56 B

THIRD PART
1. What is the most common congenital heart defect with a left to right shunt
causing congestive heart failure in the pediatric age group?
a. Atrial septal defect
b. Atrioventricular canal
c. Ventricular septal defect VSD
d. Patent ductus arteriosus
e. Aortopulmonary window
ANSWER: C

2-All true regarding ASD Exept:


a. Atrial septal defect is the second most common congenital heart defect in
children and adults.
b. Patients with atrial septal defects may have an embolic stroke as the initial
presentation.
c. Most children with atrial septal defects are asymptomatic.
d. The most common yet least serious type of atrial septal defect is an ostium
secundum defect.
e. The most common yet least serious type of atrial septal defect is ostium
primum defect.
ANSWER: E

3. What is the most likely age an infant with a large ventricular septal defect
will begin manifesting symptoms of congestive heart failure?
a. 1 day
b. 1 week
c. 1 month
d. 6 months
e. 1 year
ANSWER: C
Dr=Wahid Helmi

4. What is the dominant mechanism with which infants and young children
increase their cardiac output?
a. By increasing ventricular contractility
b. By increasing heart rate
c. By increasing ventricular end-diastolic volume
d. By decreasing heart rate
e. By increasing respiratory rate
ANSWER: B

5. The earliest sign of congestive heart failure on a chest X-ray is:


a. Increased heart size.
b. Kerley B lines.
c. Central pulmonary vascular congestion.
d. Pulmonary edema.
e. Pleural effusion.
ANSWER: A

6. A two day old cyanotic infant with a grade 3/6 ejection systolic murmur is
noted to have decreased pulmonary vascular markings on
chest x-ray and left axis deviation on EKG. The most likely diagnosis is:
a. Tetralogy of Fallot
b. Transposition of Great Vessels
c. Truncus Arteriosus
d. Tricuspid Atresia.
ANSWER: D

7. A 2 year old infant is noted to have mild cyanosis who assumes a


squatting position during long walking. He is noted to have
increasing fussiness followed by increasing cyanosis, limpness and
unresponsiveness. The most likely underlying lesion is:
a. Hypoplastic left heart
b. Transposition of the Great Vessels
c. Anomalous Pulmonary Venous Return
d. Tetralogy of Fallot
e. Aspiration with obstruction to air passages
ANSWER: D

8. An infant with a marked cyanotic congenital heart defect with decreased


pulmonary vascularity should be treated with:
a. Digoxin
b. Indomethacin
c. Prostaglandin E1
d. Epinephrine
ANSWER: C
Dr=Wahid Helmi

9. Cyanosis is produced by the presence of deoxygenated hemoglobin of at


least:
a. 1-2 gm/dL
b. 3-5 gm/dL
c. 6-8 gm/dL
d. 9-10 gm/Dl
ANSWER: B

10. A "tet spell" or "blue" spell of tetralogy of Fallot is treated with all of
the following except:
a. oxygen
b. knee chest position
c. morphine
d. digoxin
e. propranolol
f. phenylephrine
g. sodium bicarbonate
ANSWER: D

11. Pulmonary vascularity is increased in all of the following except:


a. TAPVR
b. Tricuspid atresia
c. TGV
d. Hypoplastic left heart
ANSWER: B

12. Pulmonary vascularity is decreased in all of the following except:


a. Tetralogy of Fallot
b. Pulmonary atresia
c. TAPVR
d. Tricuspid atresia
ANSWER: C

Fourth Revision
The side effect of hydralazine medication is:
a) Bronchospasms
b) Bradycardia
c) Hyperglycemia
d) Drug-induced lupus
e) Orthostatic hypotension
Ans… (d)
Hydralazine cause drug–induced lupus and tachycardia. Hydralazine is an
arterial vasodilator like diazoxide,nitroprusside, and minoxidil.
Dr=Wahid Helmi

The antihypertensive medication that causes bronchospasms is:


a) Propranolol
b) Clonidine
c) Diazoxide
d) Enalaprilat
e) Captopril
Ans (a) Propranolol and labetalol cause bronchospasms.
Propranolol also causes bradycardia and vivid dreams.

The preferred therapy for patients with hypertensive emergencies is:


a) Intravenous labetalol
b) Intravenous propranolol
c) Intravenous enalaprilat
d) Intravenous hydralazine
e) Intravenous phentolamine
Ans (a) Intravenous labetalol or nitroprusside or sublingual
nifedipine is the preferred therapy in patients with hypertensive
emergencies.

All of the following therapies are indicated initially in adolescents with


an essential hypertension except:
a) Weight reduction in obese
b) Reduce sodium intake
c) Aerobic exercise
d) Avoid tobacco
e) A small amount of red-wine (15 mL per day)
Ans (e) Adolescents should be counseled to avoid alcohol
and tobacco because of their adverse effects on blood pressure.
In a patient with hypertensive crisis, the blood pressure should be
reduced to about:
a) 95% of the total planned reduction within the first 2 hours and the
remaining 5% reduction over the next 4 hours.
b) 80% of the total planned reduction within the first 30 minutes and the
remaining 20% reduction over the next 24 hours.
c) 33% of the total planned reduction within the first 6 hours and the
remaining 67% reduction over the next 48-72 hours.
d) 20% of the total planned reduction within the first 3 hours and the
remaining 80% reduction over the next 12 hours.
e) 100% within the first 3 hours in order to avoid hypertensive
encephalopathy that is a potential complication.
Ans (c) The blood pressure should be reduced about one
third of the total planned reduction during the first 6 hours and remaining
two thirds over the following 2–3 days. A rapid reduction in blood pressure
may reduce the organ perfusions.
Dr=Wahid Helmi

The following antihypertensive medication may be avoided in patients


with abnormal renal functions:
a) Nifedipine
b) Furosemide
c) Captopril
d) Propranolol
e) Nitroprusside
(c) Captopril causes proteinuria, neutropenia, chronic cough, rash, and dysgeusia
(dysfunction of the taste sensation).

All children should have blood pressure


measurements at every well child visit at the age of:
a) 1 year
b) 2 years
c) 3 years
d) 4 years
e) 5 years
Ans (c) 3 years of age

All of the following are risk factors for developing cardiovascular


diseases in children except:
a) A child with elevated lipid levels
b) A child with a high blood pressure
c) The family history of a premature cardiovascular disease in a parent
(less than 55 years of age) or grandparent
d) The family history of high blood cholesterol levels
e) A 22-month-old child with a body weight that is above 97th percentile for
his/her age.
Ans
(e) The obesity for a child should be monitored at 2 years of age.
The most common clinical manifestation of myocarditis is:
a) Cardiac failure.
b) Bradycardia
c) Heart block
d) Hypertension
e) Bounding peripheral pulses
Ans (a) Cardiac failure. Arrhythmia and sudden death may be the first
clinical sings
Dr=Wahid Helmi

The most common cause of myocarditis is:


a) Diptheria
b) Viral infection
c) Toxoplasmosis
d) Histoplasmosis
e) Bacterial infections
Ans…. (b) Viral infection; answer (a), (b), (c), (d), and (e) also
cause myocarditis.

The most common primary malignant cardiac tumors in children is:


a) Sarcomas
b) Leukemia
c) Lymphomas
d) Wilms tumor
e) Mesotheliomas
Ans…. (a) Sarcomas

Furosemide therapy can cause all of the following complications


except:
a) Hyponatremia
b) Hypokalemia
c) Acidosis
d) Hypochloremia
e) Increased extracellular fluid volume (ECF)
Ans…. (c) Chronic furosemide therapy causes ‘contraction alkalosis’ due to
contraction of ECF volume.

The most common cause of death in pediatric patients with heart


transplantation is:
a) Rejection
b) Malignancy
c) Pulmonary hypertension
d) Cytomegalovirus infection
e) Graft coronary artery disease
Ans (d) Viral infections (32% of cases), especially CMV
infections (25% of cases) are the most common causes of death. Answer
(a), (b), (c), and (e) also cause death.
Dr=Wahid Helmi

The most common tumor that occurs in patients after cardiac


transplantation is:
a) Sarcomas
b) Leukemia
c) Mesotheliomas
d) Skin cancers
e) Lymphoproliferative disease
Ans... (e) PTLD (post –transplant–lymphoproliferative disease)
is associated with Epstein–Barr virus. The children can also develop. skin
cancers.

The most common cause of hypertension in newborn is:


a) Umbilical venous catheterization
b) Umbilical artery catheterization
c) Acute glomerulonephritis
d) Acute tubular necrosis
e) Excessive administration of intravenous fluids
Ans (b) Umbilical artery catheterization and renal artery thrombosis are the most
common causes

The side effects of propranolol include all of the following except:


a) Bradycardia
b) Bronchospasm
c) Hyperglycemia
d) Hypotension
e) Heart block
Ans (c) Hypoglycemia; other side effects are congestive heart failure and loss of
concentration or
memory
.

The following therapy is contraindicated in patients with supraventricular


tachycardia (SVT):
a) Verapamil
b) Ice bag over the face
c) Phenylephrine
d) Edrophonium
e) Synchronized DC cardioversion
Ans….. Verapamil causes hypotension, bradycardia, and cardiac arrest in
infants younger than 1 year of age
Dr=Wahid Helmi

The preferred therapy in a stable patient with SVT is:


a) Propranolol
b) Quinidine
c) Procainamide
d) Submersion of the face in iced saline
e) Adenosine
Ans (e) Adenosine

The preferred therapy in patients with SVT and severe cardiac failure
is:
a) Adenosine
b) Propranolol
c) Phenylephrine
d) Synchronized DC cardioversion
e) Quinidine
Ans…. (d) Synchronized DC cardioversion

Jervell-Lange-Nielsen syndrome (JLNS) is usually associated with:


a) Cataract
b) Deafness.
c) Hydronephrosis
d) Hepatitis
e) Choanal atresia
Ans (b) JLNS is associated with congenital deafness.JLNS is an autosomal recessive
disorder. It is an uncommon form of long Q–T syndrome.A newborn is admitted to
the NICU with respiratory distress.
Dr=Wahid Helmi

The newborn was born by NSVD with Apgar scores 8


and 9 at 1 and 5 minutes respectively. The physical examination reveals a
grade 2/6 systolic vibratory murmur at the left sternal border. The newborn
received 100% oxygen. A peripheral arterial blood gas reveals pH 7.34,
PCO2 45, and Po2 170. The preductal oxygen saturation is 96% and the
postductal oxygen saturation is 97%. The all for extremities BPs are the
following: LA 60/30, LL 65/34, RA 62/32, and RL 66/35. The chest x-ray
reveals hyperinflation, fluids in the fissure, and very small pleural effusions.
The right atrial and right ventricular oxygen saturations are 75%. The left
atrial and left ventricular oxygen saturations are 95%. The left ventricular
pressure is 100/8 and the aortic pressure is 100/60. The right ventricular
pressure is 25/3 and the pulmonary artery pressure is 25/10. The oxygen
consumption is 160 L/minute. The oxygen capacity is 200 mL/L. The
systemic and pulmonary blood flows are 5 L/minute/meter square. The
most likely diagnosis is:
a) Transposition of great vessels
b) Total anomalous pulmonary venous return
c) Tetralogy of Fallot
d) Critical Pulmonic stenosis
e) Normal cardiac anatomy
Ans: . (e) Normal cardiac
anatomy because all the parameters are normal. Please remember, the all
normal values. This newborn has
the x–ray findings of TTNB (transient tachypnea of the newborn).

The most common site the anomalous pulmonary veins drain into:
a) Right atrium
b) Infracardiac
c) Coronary sinus
d) Right superior vena cava
e) Left superior vena cava.
Ans (e) Left superior vena cava (40% of case)
791. The most common obstruction type of anomalous pulmonary venous
return is:
a) Infracardiac
b) Right atrium
c) Right superior vena cava
d) Left superior vena cava
e) Coronary sinus
Ans… (a) Infracardiac (95–100% of cases)
Dr=Wahid Helmi

A newborn appears with severe tachypnea and cyanosis at birth. The


newborn was intubated and placed on a mechanical ventilator. The
newborn remains ill. The physical examination reveals absence of heart
murmur but presence of mild intercostals retractions. The chest x-ray
reveals a small heart and a perihilar pulmonary edema. The EKG reveals a
right ventricular hypertrophy and the tall and spiked P waves. The oxygen
saturation in all four chambers are almost the same. The next step in
management is:
a) Blalock-Taussig shunt
b) Switch operation
c) Fontan operation
d) Repair of the truncus arteriosus
e) Surgical correction of total anomalous pulmonary venous return
Ans… (e) Surgical correction of TAPVR (obstructive type)

A newborn appears with mild tachypnea


and cyanosis at birth. The newborn received 100% oxygen by oxyhood.
The physical examination reveals a grade 2/6 systolic murmur along the left
sternal border and a gallop rhythm. The chest x-ray reveals a normal heart
size and an increased pulmonary vascularities. The EKG reveals a right
ventricular hypertrophy and the tall and spiked P waves. The next step in
management is:
a) Prostaglandin E1 infusions.
b) Indomethacin
c) Fontan operation
d) Switch operation
e) Blalock-Taussig shunt
Ans… (a) Prostaglandin E1 infusions are indicated in patients
with TAPVR (nonobstructive type) before the surgical correction.
A newborn appears with severe cyanosis and respiratory distress at birth. The
physical examination reveals aholosystolic murmur, gallop rhythm, and multiple
clicks over the left sternal border. The chest x-ray reveals a massive
cardiomegaly. The EKG reveals a right bundle branch block and WPW
syndrome. The child is placed on amechanical ventilator and infusion of PGE1
started. The next step in
management is:
a) Aortopulmonary shunt.
b) Blalock-Taussig shunt
c) Switch operation
d) Surgical closure of VSD
e) Surgical closure of ASD
Ans . (a) Ebstein anomaly of the tricuspid valve is treated with either
aortopulmonary shunt alone or by surgical patch closure of
tricuspid valve, atrial septectomy, and aortopulmonary shunt (Starnes
procedure).
Dr=Wahid Helmi

The most common organism in patients with Lemierre disease is:


a) Group A Streptococcus
b) S. aureus
c) Haemophilus influenzae
d) Klebsiella
e) Fusobacterium necrophorum
Ans (e) Fusobacterium necrophorum (anaerobic oropharyngeal bacteria)

A child appears with nausea, vomiting,decreased appetite, lethargy, and fever.


The child had a surgical repair of VSD 7 days age. He denies chest pain. The
findings in echocardiography may reveal:
a) Recurrence of VSD
b) Mitral valve stenosis
c) Aortic valve stenosis
d) Pulmonary valve regurgitation
e) Pericardial fluid.
Ans (e) In postpericardiotomy syndrome, patients may develop pericardial
effusion and cardiac tamponade.

The preferred therapy for patients with a postpericardiotomy syndrome


is:
a) Salicylates
b) Propranolol
c) Cefuroxime
d) Furosemide
e) Temporary pace maker
Ans (a) Salicylates or indomethacin and bed rest are indicated.
Sometimes, steroid therapy or pericardiocentesis is required.

The following is a contraindication to surgical closure of VSD:


a) Severe pulmonary vascular disease with VSD.
b) Supracrystal VSD
c) Qp : Qs ratio greater than 2:1 in a patient older than 24 months of age
d) Large VSD with failure to thrive
e) Large VSD with a developing pulmonary hypertension
Ans… (a) Severe pulmonary vascular disease
Dr=Wahid Helmi

The most common cause of heart transplants in children younger than 1 year of
age is:
a) Sinus arrythmia
b) Sinus bradycardia
c) Anomalous coronary artery
d) Total anomalous pulmonary venous return
e) Hypoplastic left heart syndrome
Ans (e) Congenital heart lesions (75% of cases)

[
In a case of suspected digoxin toxicity,serum digoxin level should be obtained:
a) Immediate after a dose
b) 30 minutes after a does
c) 1 hour after a dose
d) 2 hours after a dose
e) Immediately before a dose but at minimum 4 hours after the last dose
Ans (e) Immediately before a
dose but at minimum 4 hours after the last dose

The following condition increases digitalis toxicity:


a) Hyperkalemia
b) Hypernatremia
c) Hyponatremia
d) Hypocalcemia
e) Hypokalemia
Ans….. (e) Hypokalemia and hypercalcemia increase digitalis
toxicities
.
The digoxin therapy should be discontinued in the following condition:
a) Prolongation of P-R intervals
b) Changes in ST segments
c) Changes in T-waves
d) Blood level is 3 ng/mL in infants
e) Disturbances in new rhythms
Ans (e) A new rhythm disturbance is noted. The normal
blood levels of digoxin are 2–4 ng/dL in infants and 1–2 ng/dL in older
children.
.
Dr=Wahid Helmi

Blood pressure rises steadily beginning at approximately:


a) 1 month of age
b) 1 year of age
c) 2 years of age
d) 4 years of age
e) 6 years of age
Ans….( E ) 6 years of age

A 12 year-old obese boy has a strong family history of coronary heart disease.
His plasma total cholesterol level is 200mg/dL, HDL cholesterol level is
74/dL,and triglyceride level is 110mg/dL. His plasma LDL cholesterol level:
a) 96 mg/dL
b) 100 mg/dL
c) 104 mg/dL
d) 110 mg/dL
e) 112 mg/dL
Ans…….( C ) 104 mg/dL.LDL cholesterol= total cholestero l_ [HDL cholesterol+
(triglyceride/5)]
=200 _ [ 74+(100/5)]
=200 _96
=104 mg/dL

The preferred therapy for children older than 2 years with average LDL
cholesterol level more than 110 mg/dL is:
a) Statins
b) Nicotinic acid
c) Bile acid-binding resins
d) Dietary modification
e) Reassurance
Ans….( D )
Dietary modification I.e., their daily food intake should not provide more
than 30%of total calories as fat (approximately equal amount of saturated,
monosaturated,and polysaturated fats),and no more than 100 mg
cholesterol/1,000 calories (maximum 300 mg/day).This diet is called step I
diet from American Heart Association. The ideal goal is to lower LDL
cholesterol less than 110 mg/dL, but the minimal goal is to lower it less
than 130 mg/ Id
Dr=Wahid Helmi

A dietary intervention in children with hyperlipidemia usually reveals the


following result:
a) Lowers LDL cholesterol levels by more than 50%
b) Lowers LDL cholesterol levels by more than 40%
c) Lowers LDL cholesterol levels by more than 30%
d) Lowers LDL cholesterol levels by more than 20%
e) Lowers LDL cholesterol levels by no more than 10-15%
Ans...( E )
Lowers LDL cholesterol levels by no more than 10-15 %
The dietary management of hyperlipidemia in children is safe in the following
ages except:
a) 5 months.
b) 2 1/2 years
c) 3 years
d) 3 1/2 years
e) 4 years
Ans….( A )
The dietary management is safe in children older than 2 years of age.
Children younger than 2 years need large amount of calories to maintain
their rapid growth. Some authorities have demonstrated that a lower fat diet
can maintain normal growth in children as young as 6 months old

All of the following are risk factors in patients with hyperlipidemia except:
a) Diabetes
b) Hypertension
c) Smoking
d) Physical inactivity
e) Mild obesity
Ans…..( E ) Mild obesity is not a risk factor. However, severe obesity and low HDL
cholesterol levels are also risk factors

The first-line pediatric medicine for the therapy of hypercholesterolemia in


children is:
a) Statins
b) Bile acid -binding resin
c) Nicotinic acid
d) Fibrates
e) Absolute removal of fat from diet
Ans….( B ) Bile acid- binding resins (e.g., cholestyramine, colestipol, colesevelam)
primarily reduce LDL cholesterol levels in plasma. However, they can
enhance hypertriglyceridemia in children with triglyceride levels greater
than 300 mg/dL. Answers (a), (c), and(d) are also lipid-lowering
medications
Dr=Wahid Helmi

All of the following drugs raise triglyceride levels except:


a) Oral contraceptives
b) 13-ci-retinoic acid
c) Thiazide diuretics
d) HIV protease inhibitors
e) Phenytoin.
Ans…...( E )
Phenytoin does not increase triglyceride levels. Other drugs that raise
triglyceride levels are steroids, immunosuppressants, and some
beta-adrenegic blocking agents

All of the following conditions are associated with Hyperlipidemia except:


a) Hyperthyroidism
b) Diabetes mellitus
c) Nephrotic syndrome
d) Anorexia nervosa
e) Congenital biliary atresia
Ans…..( A )
Hyperthyroidism is not associated with hyperlipidemia.Other conditions
associated with hyperlipidemia and hypothyroidism, storage diseases (e.g.,
Tay-Sachs disease, Niemann-pick disease, glycogen storage disease),
hepatitis, systemic lupus erythematosus, and excessive alcohol intake (in
teenagers)
Dr=Wahid Helmi

The following statement is not true about the drug treatment in children with
hyperlipidemia aged 10 years and older,after an adequate trial (6-12 months)of
diet therapy:
a) General pediatrician should be able to treat children who need drug
therapy
b) Drug therapy is indicated when LDL cholesterol level is greater than 190
mg/dL
c ) Drug therapy is indicated when LDL cholesterol level is greater than
160 mg/dL and family history of premature coronary heart disease
d) Drug therapy is indicated when LDL cholesterol level is greater than 160
mg/dL in children with diabetes and hypertension
e) Drug therapy is indicated when LDL cholesterol level is greater than 160
mg/dL in children with severe obesity and low HDL cholesterol level
Ans…..( A )
General pediatrician should refer all children who need drug therapy to a
specialized pediatric lipid center. Drug therapy is also indicated when LDL
cholesterol level is greater than 160 mg/dL in children with smoking and
physical inactivity.

Hypertensive encephalopathy is most commonly associated with the following


disease:
a) Renal.
b) Hepatic
c) Adrenal
d) Pituitary
e) Thyroid
Ans…..( A ) Renal disease (e.g., acute glomerulonephritis, chronic pyelonephritis,
end-stage renal disease)

A child with congenital heart disease receiving loop diuretic. His serum
bicarbonate level is 31.The preferred management is:
a) Stop loop diuretic
b) Start digoxin
c) Add potassium chloride
d) Switch to another diuretic
e) No intervention is indicated
Ans...( E ) No intervention is indicated because the patient has mild metabolic
alkalosis ( I.e.,bicarbonate level is less than 32)
Dr=Wahid Helmi

A mother is admitted at 20th week of pregnancy. The fetus developed cardiac


failure due to arrhythmia. The preferred therapy is:
a) Lasix to the mother
b) Lasix to the fetus
c) Digoxin to the mother
d) Digoxin to the fetus
e) Deliver the fetus
Ans...( C )
Digoxin should be given to the mother and it crosses the placenta to the
fetus

A child has dilated cardiomyopathy. She is receiving adequate digoxin and


furosemide therapies. Here condition improve but not significantly. The
preferred therapy is:
a) Continue digoxin and increase furosemide dose
b) Increase digoxin and furosemide doses
c) Discontinue digoxin and add ACE inhibitor along with furosemide
d) Continue digoxin and furosemide, add ACE inhibitor
e) Continue the same management
Ans….( D ) Afterload-reducing agents and ACE inhibitors ( e.g., captopril, enalapril)
are used in conjunction with digoxin and furosemide. Afterload-reducing agents
(e.g., ACE inhibitors, Hydralazine, nitroglycerin, nitroprusside, prazocin,
carvedilol) are not generally used in patient with stenotic lesions of the left
ventricular outflow tract

All of the following are mechanisms of action of ACE inhibitors (e.g., captopril )
except:
a) Arterial dilatation
b) Venous dilatation
c) Increase aldosterone production
d) Reduced afterload and preload
e) Cardiac remodeling independent on their
influence on afterload
Ans...( C ) ACE inhibitors (e.g., captopril) decrease aldosterone production and
reduce salt and water retention. These medications are used in hypertension and
cardiac failure (nonsteotic lesions).Arterial dilatation causes afterload
reduction and venous dilatation causes preload reduction.The principal
effect of ACE inhibitors is arterial dilatation
Dr=Wahid Helmi

Adverse reactions of captopril include all of the following conditions:


a) Maculopapular rash
b) Chronic cough
c) Neutropenia
d) Hypotension
e) Improves renal function
Ans…...( E ) Captopril produces renal toxicity and failure
A child has been receiving high dose intravenous nitroprusside for
the past several days. He has an uncontrolled hypertension. The
following parameter should be monitored:
a) Serum BUN
b) Serum creatinine
c) Liver function tests
d) Pancreatic enzymes
e) Blood thiocyanate level
Ans….( E ) Blood thiocyanate level (more than 10 g/dL causes
toxicity)

A child developed nausea, fatigue,disorientation, acidosis, and


muscular spasms.She is in ICU.Most likely the child is receiving the
following medication:
a) Captopril
b) Hydralazine
c) Digoxin
d) Dopamine
e) Nitroprusside
Ans…..( E )
Nitroprusside (intravenous)administration causes thiocyanate poisoning

The use of nitroprusside is contradicted in


the following condition:
a) Seriously ill patients
b) Pre-existing hypotension
c) Increased preload
d) Hypertension
e) Increased afterload
Ans….( B )
Nitroprusside should not be used in patients with pre-existing
hypotension
because it can cause sudden hypotension. Nitroprusside is used in
critically
ill patients with hypertension. It primarily causes peripheral arterial
vasodilatation and afterload reduction, but may cause venodilatation and
preload reduction by decreasing venous return to the heart
Dr=Wahid Helmi

The following statement is not true abou dopamine:


a) Predominantly alph -adrenergic agonist
b) Selective renal vasodilator effect
c) Useful in patients with a compromised renal function
d) Vasoconstriction predominates if the dose is increased above 15
microgram/kg/minute
e) Increase cardiac contractility with little peripheral vasoconstriction at a
dose 2-10 microgram/kg/minute
Ans….( A ) Dopamine is predominantly beta-adrenegic agonist

Major side effect of milrinone


(phosphodiestrase inhibitor)is:
a) Hypertension
b) Hypotension
c) Bradycardia
d) Thrombocytopenia
e) Apnea
Ans...( B ) Hypotension is secondary to a peripheral vasodilatation.
Milrinone is used
in patients with low cardiac output who are unresponsive to standard
therapy. Milrinone is used as an adjunct to dopamine or dobutamine

Preferred therapy in patients with milrinone-induced hypotension is:


a) Hydralazine
b) Decrease intravenous fluids
c) Increased intravenous fluids
d) Captroil
e) Nitroprusside
Ans...( C ) Increased administration of intravenous fluids increase the
intravascular
volume. Other choices cause hypotension
Dr=Wahid Helmi

The side effect of amrinone therapy is:


a) Thrombocytosis
b) Thrombocytopenia
c) Leukocytosis
d) Polycythemia
e) Lymphocytosis
Ans….( B )
Amrinone is a phosphodiestrase inhibitor.It causes thrombocytopenia that
depends on the rate of infusion and duration of therapy

A child was admitted to the ICU for dilated cardiomyopathy. He has


improved and the acute condition is over. The following medication is
added as a part of chronic comprehensive heart failure treatment
program:
a) Vitamin E
b) Metoprolol
c) Carvedilol
d) Milrinone
e) Vitamin A
Ans...( B )
Metoprolol (a selective beta 1-adrenergic receptor antagonist)is used
commonly. Answer (c) is currently under study .Answer (d)is used in
ICU
patients. Answers (a ) and (e) do not make sense. Metoprolol improves
exercise tolerance; decreases hospitalization and mortality
Dr=Wahid Helmi

A 3.9 kg newborn appears with cyanosis at 2 hours of age, cyanosis


progressively gets worse, also dyspnea. An examination reveals lower
extremities are less cyanotic than upper extremities, hyperactive
precordium, either no murmur or PDA murmur of grade 3/6, 2nd heart
sound single and loud, or occasionally it may be split. Chest X-ray reveals
mild cardiomegaly, normal or increased pulmonary flow, narrow
mediastinum. ABG reveals Pao2 15 to 30, pH 7.30, Pco2 35, O2 saturation
30 to 70%, base deficit – 15.0. Pao2 increases slightly after hyperoxia test
(i.e., 100% oxygen). EKG reveals biventricular hypertrophy. Newborn was
intubated and placed on mechanical ventilator. PGE1 and NaHCO3 are
given. Baby’s oxygen saturation improved up to 85 to 90%. Next
appropriate step in management:
a) Fontan procedure
b) Blalock-Taussig shunt
c) Rashkind balloon atrial septostomy.
d) Norwood operation
e) Starnes procedure
Ans….(c) Newborn is diagnosed with transposition of great
vessels, therefore Rashkind baloon atrial septostomy is the initial
procedure of choice.
Dr=Wahid Helmi

A newborn male appears with rapid heart rate. He is afebrile. In room


air, oxygen saturation is 100% recorded by pulse oximeter. Heart rate is
260 per minute, respiratory rate is 46 per minute, and temperature is
98.6°F. Management of this patient includes all of the following except:
a) Verapamil
b) Adenosine
c) Ice pack over face
d) DC cardioversion
e) Propranolol
Ans…. (a) Verapamil is not used in children less than 1 year of age
because it reduces cardiac output and produces hypotension and cardiac
arrest.

A full-term newborn boy was admitted to NICU (neonatal intensive care


unit) with mild respiratory distress. He was born by cesarean section with
Apgar scores 8 an 8 at 1 and 5 minutes respectively. Initially baby was
Dr=Wahid Helmi

placed under 40% oxygen by oxyhood. Oxygen saturation was 85% by


pulse oximeter. Physical examination reveals tachypnea but no heart
murmur. Then, oxygen was increased to 50%. Chest X-ray reveals
increased pulmonary vascular markings. ABG: pH 7.30 / Pco2 42 / Po2 65
in 40% oxygen. Umbilical arterial and venous lines were placed. Po2 is
higher in umbilical venous line than that of arterial line. Repeat CXR
reveals umbilical venous line is in the right atrium and umbilical arterial line
at T8 level. Most likely diagnosis:
a) TTNB (transient tachypnea of newborn)
b) TGA (transposition of great arteries)
c) VSD
d) ASD
e) TAPVR (total anomalous pulmonary venous return).
Ans…. (e) TAPVR because all four pulmonary veins drain into right
atrium. Pulmonary veins contain oxygenated blood to right atrium.
Therefore, umbilical catheter in right atrium has higher oxygen than that of
umbilical arterial line.
Dr=Wahid Helmi

3. A 17-year-old male athlete suddenly collapsed while playing football.


He was brought to the ER. He expired despite adequate CPR. Past
medical history was unremarkable. Autopsy was performed. He had no
history of smoking, drugs, and alcohol. Most likely cause of death:
a) Critical aortic stenosis
b) Mitral valve prolapse
c) Aortic incompetence
d) Hypoplastic left heart syndrome
e) Long QT interval syndrome.
Ans…. (e) A malignant form of ventricular arrhythmia called
torsades de pointes occurs in patients with LQT syndrome and is a cause
of syncope or sudden death.

Heart disease is most common with:


a) Trisomy 21
b) Trisomy 18.
c) Trisomy 13
d) Turner syndrome
e) Trisomy 22
Ans (b) Heart disease is most common in trisomy 18 (more than
90% of cases).
Dr=Wahid Helmi

A 3-day-old newborn infant was diagnosed with congenital cyanotic


heart disease. Cardiac surgery was performed. The cardiac output was
decreased after surgery. The clinical manifestations suggestive of
decreased cardiac output:
a) Tachycardia
b) Bradycardia
c) Hypertension
d) Hypotension and prolonged capillary refilling time.
e) Hypotension and normal capillary refilling time.
Ans... (d) Hypotension and prolonged capillary refilling time indicate
decreased cardiac output.

The onset of routine screening for hypertension should be:


a) 6 months
b) 12 months
c) 3 years
d) 6 years
e) 12 years
Ans….. (c) Hypertension screening should begin at 3 years of age.
Dr=Wahid Helmi

A 3-year-old boy was admitted to pediatric cardiac ICU after surgical


repair of a coarctation of aorta. After the surgery, physical examination
reveals grade 2/6 systolic ejection murmur in the upper right sternal border.
Most likely cause of this murmur:
a) Congenital bicuspid aortic valve
b) Persistent coarctation of aorta
c) Functional murmur
d) Anemia causing murmur
e) Atrial septal defect
Ans…. (a) Congenital bicuspid aortic valve is an associated
anomaly in coarctation of aorta.

Physical activities should be limited in the following cardiac disease:


a) VSD
b) ASD
c) PDA
d) Aortic valve stenosis
e) Pulmonary artery stenosis
Ans….. (d) Aortic valve stenosis 320. A well-developed, well-nourished infant appears
with systolic ejection murmur in the right upper sternal border. The murmur heard best
during expiration.
Dr=Wahid Helmi

The 2nd heart sound was split. Most likely diagnosis:


a) Pulmonic stenosis
b) ASD
c) VSD
d) Aortic stenosis
e) Mitral regurgitation
Ans…. (a) Pulmonic stenosis

A female child is diagnosed with hyperkalemia. Physical examination


reveals irregular and rapid heart rates. The serum K level is 7.5 mEq/L. The
rest of the SMA 6 results are normal. The first drug of choice in this patient:
a) Intravenous glucose and insulin
b) Oral kayexalate
c) Slow intravenous calcium gluconate with ECG monitoring
d) Intravenous sodium bicarbonate
e) Intravenous salbutamol
Ans… (c) IV calcium gluconate
Dr=Wahid Helmi

A preterm 26 weeks gestational age newborn required surgical ligation


for PDA (patent ductus arteriosus). All of the following are expected after
surgical ligation of PDA except:
a) Child gains weight
b) Cardiac failure disappears
c) Less respiratory infections
d) EKG becomes normal
e) Absence of a functional systolic murmur over pulmonary area.
Ans… (e) A functional systolic murmur over pulmonary area may
be present.

Most common cause of thromboembolic stroke in children:


a) Coagulation disorder
b) Polycythemia
c) CNS trauma
d) Aortic injury
e) Cardiac abnormalities
Ans…. (e) Cardiac abnormalities
Dr=Wahid Helmi

A child was brought to the ER. He was resuscitated but expired.


Mother told that the boy was scheduled to have a cardiac surgery
6 weeks from now. Autopsy was performed. Most likely diagnosis:
a) Mitral stenosis
b) Mitral valve prolapse
c) Pulmonic stenosis
d) Aortic coarctation
e) Congenital aortic stenosis
Ans…. (e) Congenital aortic stenosis.

A full-term male newborn is delivered by NSVD. His mother has


history of lupus. The most common complication:
a) Mitral stenosis
b) Aortic stenosis
c) Cardiac tumor
d) Congenital heart block.
e) VSD
Ans…. (d) Congenital heart block

Most common cardiac lesion in trisomies:


a) ASD b) VSD c) Aortic stenosis d) Mitral stenosis e) Tricuspid
regurgitation
Ans….b) VSD
Dr=Wahid Helmi

A child appeared in the ER with lethargy and palpitation. Physical


examination revealed a hypotension and irregular heart beats.
Cardiopulmonary resuscitation was failed. Mother stated that the child was
receiving a prescribed drug:
a) Azithromycin
b) Alpha methyl dopa
c) Desipramine
d) AZT
e) Prednisone
Ans... (c) Desipramine

A 5-year-old boy appears in a clinic for routine check-up. Physical


examination reveals grade 2/6, systolic, musical heart murmur in the left
sternal border. Most likely diagnosis:
a) PDA
b) VSD
c) ASD
d) Aortic stenosis
e) Innocent murmur.
Ans... (e) Functional murmur
Dr=Wahid Helmi

A 14-year-old girl is diagnosed with scoliosis. The other organ defect


associated with scoliosis:
a) Renal
b) Eye
c) Liver
d) Cardiac.
e) ENT
Ans…. (d) Cardiac, mitral valve prolapse

A foramen ovale usually closes by:


a) 1 month
b) 2 months
c) 3 months.
d) 4 months
e) 6 months
Ans (c) Foramen ovale usually close by 3 months of age

A full-term, LGA male was born by a cesarean section due to fetal


distress. Apgar scores were 8 and 9 at 1 and 5 minutes respectively. His
mother had diabetes during this pregnancy. Most common cardiac
anomaly:
a) VSD b) ASD c) Cardiac failure d) Cardiomegaly.
e) Asymmetric septal hypertrophy
Ans (d) Cardiomegaly
Dr=Wahid Helmi

A 8-year-old boy has been taking digoxin for the last 3 months. He
appears in the ER with history of vomiting, diarrhea, diplopia, yellow or
green vision, and photophobia for the last 12 hours. Physical examination
reveals
bradycardia and dry mucous membranes. EKG reveals prolonged P – R
intervals. The next step in management:
a) Intravenous hydration
b) Oral hydration
c) Stop digoxin and give digoxin-specific Fab antibodies.
d) Opthalmology consult
e) Increase the digoxin dose
Ans….c) Stop digoxin and give digoxin-specific Fab antibodies

The hypertension is a characteristic feature of:


a) 21-hydroxylase deficiency
b) Carboxylase deficiency
c) 11 beta-hydroxylase deficiency
d) Beta glucosidase deficiency
e) 3 beta-hydroxysteroid dehydrogenase defect
Ans….   (c) 11 beta-hydroxylase deficiency causes hypertension.
A full-term newborn male was intubated in the delivery room. Apgar
scores were 2, 5, 7 at 1, 5, and 10 minutes respectively. He was brought to
NICU and was placed on a mechanical ventilator. Physical examination
revealed grade 2-3/6 systolic murmur in lower sternal border and equal air
entry in both lungs. Chest X-ray revealed mild cardiomegaly. EKG revealed
Dr=Wahid Helmi

S-T and T-wave changes. Reexamination at 12 hours of age revealed


louder murmur and progressive deterioration of his clinical condition. The
ventilator settings were adjusted accordingly. After 24 hours of age clinical
condition improved and heart murmur disappeared. Most likely diagnosis:
a) Meconium aspiration
b) VSD
c) Mitral valve regurgitation
d) Tricuspid valve regurgitation
e) PDA
Ans….   (d) Tricuspid regurgitation
A 6-year-old boy had a modified Blalock-Tausig shunt for tetralogy of
Fallot. About 12 hours after surgery, the boy developed respiratory distress.
Chest X-ray revealed pleural effusion. Most likely diagnosis:
a) Hemothorax
b) Chylothorax
c) Pneuthorax
d) Hydrothorax
e) Diaphragmatic hernia
Ans….   (b) Chylothorax
A full-term male infant is delivered by an emergency cesarean section
for a fetal distress. At birth, the boy is limp, cyanotic, and has no respiratory
effort. He is dried, suctioned, and stimulated. However, no clinical
improvement is noted. His Apgar score is 1 for heart rate 60/minute. The
next step in the management:
a) Chest compression
Dr=Wahid Helmi

b) Intubation STAT
c) Free-flow oxygen at a rate of 10 L/minute
d) Vigorous stimulation
e) Bag-and-mask ventilation with a rate of at least 10 L/minute.
Ans…..   (e) The boy needs oxygen by bag and mask. The infant
usually improves with that therapy.
Most common indication of cardiac transplantation:
a) Hypoplastic left heart syndrome
b) Cardiomyopathies
c) Multiple cardiac tumors
d) Tricuspid atresia
e) Single ventricle
Ans…...   (b) Cardiomyopathies
A full term newborn male appears cyanotic at birth. He was intubated
in the delivery room and received 100% oxygen. Maternal history of type I
diabetes. The boy was placed on a mechanical ventilator with the setting of
an IMV 40, PIP 20, PEEP 5, FiO2 100%, inspiratory time 0.5 second, and
flow rate 10 L/minute. Oxygen saturation is 100%. Physical examination
reveals plethora, heart rate is 190 per minute, respiratory rate is 80 per
minute, blood pressure 58/30, grade 3/6 systolic ejection murmur in the
mid-left sternal border, and liver is palpable 5 cm below the right costal
margin. Chest X-ray reveals increased pulmonary congestion. EKG reveals
increased anterior and posterior forces. CBC reveals WBC count 21,000,
60% polymorphs, 40% lymphocytes, platelet count 210,000, hemoglobin 21
g/dL and hematocrit 63. Most likely diagnosis:
Dr=Wahid Helmi

a) VSD
b) ASD
c) Pulmonic stenosis
d) Diabetic cardiomyopathy
e) Aortic stenosis
Ans…..    (d) Diabetic cardiomyopathy
Mitral valve prolapse is found in all of the following conditions except:
a) Marfan syndrome
b) Down syndrome
c) Scoliosis
d) Pectus excavatum
e) Straight back syndrome
Ans….   (b) Down syndrome patients do not have mitral valve prolapse
Most common cause of secondary hypertension in children:
a) Aortic coarctation
b) Portal vein thrombosis
c) Umbilical arterial catheter
d) Pheochromocytoma
e) Renal abnormality
Ans….   (e) Renal abnormality
A 4-day-old 26 weeks gestational age preterm infant on a mechanical
ventilator developed PDA (patent ductus arteriosus) murmur. The fluid was
restricted. However, the murmur was audible and PDA was confirmed by
an echocardiogram. Indomethacin therapy is contraindicated when:
a) BUN value is 18 mg/dL
Dr=Wahid Helmi

b) Platelet count is 55,000


c) Creatinine level is 1.9 mg/dL
d) Necrotizing enterocolitis
e) Urine output is 1.1 mL/Kg/hour
Ans….   (c) Indomethacin is contraindicated in patients with bleeding
disorder, NEC, oliguria (less than 1 mL/kg/hr), elevated creatinine level
(more than 1.8 mg/dL), and thrombocytopenia (less than 50,000/mm3).
Most common organism causing bacterial endocarditis after a dental
procedure:
a) S. aureus
b) Streptococcus viridans
c) Group D enterococcus
d) Pseudomonas aeruginosa
e) Serratia marcescens
Ans….   (b) Streptococcus viridans
A 15-year-old girl appears with a history of occasional chest pain and
palpitations for the last 6 months. She is a very bright student. Physical
examination reveals a late systolic murmur at apex with click. The murmur
is prolonged in late systole when the patient is standing or sitting condition.
She also has a thoracic scoliosis: Most likely diagnosis:
a) Mitral valve prolapse (MVP)
b) Tricuspid valve regurgitation
c) Mitral valve stenosis
d) Tricuspid atresia
e) Mild aortic stenosis
Dr=Wahid Helmi

Ans….   a) Mitral valve prolapse (MVP


Treatment of choice for moderate-to-severe hypertension in neonate:
a) Diazoxide
b) Propranolol
c) Hydralazine
d) Furosemide
e) Methldopa
Ans….   c) Hydralazine
A 2-day-old full term female newborn appears with poor feeding and
dyspnea. Physical examination reveals blood pressure 35/20, respiratory
rate 70 per minute, temperature 98.6°F, weak peripheral pulses, grayish
blue color of skin, hepatomegaly, and right ventricular heave. Most likely
diagnosis:
a) Aortic stenosis
b) Coarctation of aorta
c) Hypoplastic right heart
d) Hypoplastic left heart syndrome
e) Bicuspid aortic valve
Ans….   d) Hypoplastic left heart syndrome
A full term female newborn is admitted in NICU with meconium
aspiration. A few hours later, she developed PPHN (persistent pulmonary
hypertension). The best diagnostic study for PPHN:
a) Chest X-ray
b) Hyperoxia test
c) Echocardiogram
Dr=Wahid Helmi

d) Pulmonary function test


e) Arterial blood gas
Ans….   c) Echocardiogram

A 16-year-old boy appears in a clinic for a routine check up. Physical


examination reveals an average blood pressure 145/90, respiratory rate 20
per minute, pulse rate 74 per minute, and rest of the findings are
unremarkable. His father and uncle have a high blood pressure. His
grandfather died of a stroke at the age of 62. His weight and height are
above 95 th percentile. Laboratory test results reveal a normal CBC, SMA
6, and urinalysis. He is aware that he should control his weight and eating
habits. After 1 month, repeat blood pressure is 140/90 (checked twice).
Most likely diagnosis:
a) Renal artery stenosis
b) Aortic coarctation
c) Essential hypertension.
d) Hyperparathyroidism
e) Normal blood pressure
Ans….   (c) Essential hypertension
A routine newborn physical examination of a newborn reveals a grade
2/6, short, harsh systolic murmur at the apex. The newborn is completely
asymptomatic. The most likely diagnosis is:
a) Atrial septal defect
b) Ventricular septal defect.
c) Mitral stenosis
d) Mitral regurgitation
Dr=Wahid Helmi

e) Patent ductus arteriosus


Ans….   b) VSD

The first retinal sign in patients with a hypertensive retinopathy is:


a) Edema
b) Cotton-wool spots
c) Flame-shaped hemorrhages
d) Papilledema.
e) Generalized constrictions of retinal arterioles.
Ans….e) Generalized constrictions and irregular narrowing of
retinal arterioles are the first signs of hypertensive retinopathy. Answers
(a), (b), (c), (d), and thickening of the retinal vessels (i.e., silver-or
copper-wire appearance) also occur in hypertensive retinopathy.

The most common cardiovascular anomaly in patients with Alagille


syndrome is:
a) Transposition of great vessels
b) VSD
c) ASD
d) Tetralogy of Fallot
e) Peripheral pulmonic stenosis.
Ans... e) Peripheral pulmonic stenosis is usually present;
sometimes TOF is present.
Dr=Wahid Helmi

A newborn is diagnosed with a Shone complex. The following features


are present in this newborn:
a) Right heart obstructive lesions
b) Left-sided cardiac obstructive lesions.
c) Right-sided cardiac nonobstructive lesions
d) Left-sided nonobstructive lesions
e) Coarctation of the aorta
Ans….b) Left-sided cardiac obstructive lesions (e.g., a combination
of mitral valve, subaortic, aortic valve, and aortic arch) are called the Shone
complex.

A surgical repair of the coarctation of aorta was performed in an infant.


The next day after surgery, the infant developed vomiting, abdominal
distension, bloody stools, hypertension, and leukocytosis. The most likely
etiology is:
a) Gastric ulcers
b) Duodenal ulcers
c) Intestinal perforations
d) Gram-negative sepsis
e) Mesenteric arteritis.
Ans... e) Mesenteric arteritis can occur in patients with
postcoarctectomy syndrome. A patient may develop bowel necrosis and
small bowel obstruction. A patient improves with antihypertensive
medication (e.g., nitroprusside, esmolol, captopril). A surgical exploration of
the abdomen is rarely indicated.
Dr=Wahid Helmi

A high-output cardiac failure occurs in the following condition:


a) Myocarditis
b) Constrictive pericarditis
c) Large systemic arteriovenous fistulas.
d) Aortic stenosis
e) Hypoplastic left heart syndrome
Ans... c) Arteriovenous fistulas. In a patient with high-output cardiac
failure, cardiac output is more than normal and myocardial abnormalities
are absent
.

The most common cause of morbidity in patients with Marfan syndrome


is:
a) Progressive aortic root dilatation
b) Scoliosis
c) Dislocated lens
d) Funnel chest
e) Progressive mitral valve prolapse.
Ans….e) Marfan syndrome patients with progressive mitral valve prolapse appear with
arrythmias, endocarditis, cardiac failure, or
thromboemboli

A surgical repair of the coarctation of aorta was performed in an infant.The next day after surgery, the infant
developed vomiting, abdominal distension, bloody stools, hypertension, and leukocytosis. The most likely
etiology is:
a) Gastric ulcers
b) Duodenal ulcers
c) Intestinal perforations
d) Gram-negative sepsis
e) Mesenteric arteritis.
Dr=Wahid Helmi

A high-output cardiac failure occurs in the following condition:


a) Myocarditis
b) Constrictive pericarditis
c) Large systemic arteriovenous fistulas.
d) Aortic stenosis
e) Hypoplastic left heart syndrome
Ans... c) Arteriovenous fistulas. In a patient with high-output cardiac
failure, cardiac output is more than normal and myocardial abnormalities
are absent.

The most common cause of morbidity in patients with Marfan syndrome


is:
a) Progressive aortic root dilatation
b) Scoliosis
c) Dislocated lens
d) Funnel chest
e) Progressive mitral valve prolapse.
Ans….e) Marfan syndrome patients with progressive mitral valve
prolapse appear with arrythmias, endocarditis, cardiac failure, or
thromboemboli
Dr=Wahid Helmi

The most common cardiac defect in patients with Marfan syndrome is:
a) Aortic root dilatation.
b) Aortic stenosis
c) Mitral valve prolapse
d) Mitral stenosis
e) Coarctation of aorta
Ans….a) Aortic root dilatation

The sudden death in children is most commonly involved the following


organ system:
a) CNS
b) Pulmonary
c) Cardiac
d) Gastrointestinal
e) Hepatic
Ans…..c) Cardiac
Dr=Wahid Helmi

The following factor reduces a risk of SIDS (sudden infant death


syndrome):
a) Breast feeding
b) Prone (and side) sleep position
c) Pacifier (dummy) use.
d) Male gender
e) Recent febrile illness
Ans….c) Pacifier (dummy) use significantly reduces the risk of SIDS in infants when used
routinely and during sleep. The mechanism is unknown. It may be due to a direct effect of
pacifier or associated infant or parental behaviors. Breast feeding is recommended for
many advantages but is not recommended to reduce the SIDS. Answers (b), (d), and (e) can
increase a risk of SIDS.

Risk of SIDS is highest between:


a) 0-1 month of age
b) 1-2 months of age
c) 2-4 months of age.
d) 4-6 months of age
e) 6-9 months of age
Ans….c) 2-4 months of age.

A child appears with a history of syncopal episode during exercise,


fright, or sudden startle response. The preferred diagnostic study is:
a) EEG
Dr=Wahid Helmi

b) Head CT scan
c) Head MRI
d) ECG
e) Hearing test
Ans….d) ECG can reveal long Q-T syndrome.
An ECG finding in a patient reveals a heart-rate corrected Q-T interval
is 0.49 second. The most likely diagnosis is:
a) First-degree heart block
b) Second-degree heart block
c) Third-degree heart block
d) Long Q-T syndrome
e) Normal
Ans….d) Long Q-T syndrome; a heart-rate corrected Q-T interval
of more than 0.47 second is highly indicative of long Q-T syndrome
and more than 0.44 second is suggestive of long Q-T syndrome.
143. A child appears with fainting spells. A physical examination reveals a
normal neurological examination and positional (supine vs erect) character
of murmur on the left sternal border near mitral area. The preferred therapy
is:
a) Prophylactic antibiotic
b) Dilatation of the mitral valve
c) Repair of VSD
d) Repair of ASD
e) Removal of myxomas
Ans….e) A patient with myxoma appears with fainting spells and
have a positional character of murmur. Myxomas should be removed
Dr=Wahid Helmi

completely.
The most common location of cardiac myxomas is:
a) Right atrium
b) Left atrium
c) Right ventricle
d) Left ventricle
e) Interventricular septum
Ans….b) Left atrium (75% of cases); these smooth, pedunculated
masses arise from an interatrial septum and protrude into the left atrium.
150. The most common cause of congenital heart disease is:
a) Chromosomal abnormalities
b) Maternal alcohol intake..
c) Single gene mutation
d) Maternal drug intake
e) Unknown
Ans….e) Unknown
A congenital heart disease is most common in the following condition:
a) Trisomy 21
b) Trisomy 18
c) Trisomy 13
d) Turner syndrome
e) Trisomy 22
Ans….b) Trisomy 18 (90% of cases); trisomy 21 (50% of cases);
Turner syndrome (40% of cases)
All of the following heart diseases are more common in girls than that
of boys except:
Dr=Wahid Helmi

a) ASD
b) VSD
c) PDA
d) Pulmonic stenosis
e) Transposition of great vessels
Ans…..e) Transposition of great vessels and left-sided obstructive
lesions (e.g., aortic stenosis, hypoplastic left heart syndrome) are
more common in boys than in girls.
A 12-year-old boy came for a routine physical examination. The boy is
asymptomatic. The blood pressure recorded in the right upper extremity is
140/90. The blood pressure recorded in the left upper extremity is 138/88.
The next step in management is:
a) Measure blood pressures in both lower extremities
b) ECG
c) Chest x-ray
d) Serum electrolytes
e) Liver function tests
Ans….a) Measure blood pressures in both lower extremities. In a
normal person, systolic blood pressure in legs is 10-20 mm Hg higher than
in arms. In patients with a coarctation of the aorta, blood pressures in lower
extremities are lower than in upper extremities.

The most common complication of patients with a supracrystal VSD is:


a) Right ventricular failure
b) Left ventricular failure
Dr=Wahid Helmi

c) Pulmonic stenosis
d) Aortic insufficiency
e) Arrythmias
Ans….d) Aortic insufficiency (50-90% of cases) is due to a
prolapse of aortic valve into a defect.
The most common clinical presentation in patients with a mild or
moderate Pulmonic stenosis is:
a) Right ventricular failure
b) Hepatic failure
c) Tachypnea
d) Tachycardia
e) Asymptomatic
Ans... e) Asymptomatic
The most common cardiac lesion in patients with a rheumatic heart
disease is:
a) Mitral stenosis
b) Aortic stenosis
c) Aortic insufficiency
d) Tricuspid stenosis
e) Mitral insufficiency
Ans... e) Mitral insufficiency

In newborn infants with a congenital heart disease, the chance that a


murmur heard at birth is:
a) 1/2
Dr=Wahid Helmi

b) 1/4
c) 1/8
d) 1/12
e) 1/25
Ans... d) 1/12 (i.e., 8.3%)
Transposition of great vessels are most commonly associated with:
a) ASD
b) VSD
c) Pulmonic stenosis
d) Aortic stenosis
e) Tricuspid regurgitation
Ans….b) VSD (about 50% of cases)
A 9-month-old appears with a respiratory distress. He was previously
admitted for recurrent pneumonia. A physical examination reveals
wheezing, intercostals retractions, mild cyanosis, hyperdynamic
precordium, and a loud to-and-fro murmur over theleft upper sternal border.
The most likely diagnosis is:
a) Pulmonic stenosis
b) Aortic stenosis
c) Aortic regurgitation
d) Truncus arteriosus
e) Congenital absence of the pulmonary valve
Ans….e) Congenital absence of the pulmonary valve produces a
syndrome that manifests with signs of an upper airway obstruction. Marked
aneurysmal dilatation of the main and right or left pulmonary artery
resulting in compression of the bronchi that causes wheezing and recurrent
Dr=Wahid Helmi

pneumonia. Cyanosis may be absent, mild, or moderate. This syndrome


may be associated with TOF
431. The following chromosome is involved in patients with a William
syndrome:
a) Chromosome 5
b) Chromosome 7
c) Chromosome 11
d) Chromosome 13
e) Chromosome 17
Ans….b) Chromosome 7
The preferred therapy for children with a moderate to severe aortic
valvular stenosis is:
a) Balloon valvuloplasty
b) Konno procedure
c) Surgical valve replacement
d) Aortopulmonary anastomosis
e) Rashkind procedure
Ans…..a) Balloon valvuloplasty
A child was diagnosed with a coarctation of aorta. The operative repair
was performed. Several months after surgery, the child developed a
systolic ejection murmur along the left sternal border. The most likely
condition that becomes apparent after a surgical procedure for coarctation
of aorta is:
a) Aortic insufficiency
b) Recurrence of aortic coarctation
c) Mitral stenosis
Dr=Wahid Helmi

d) Mitral insufficiency
e) Subvalvular aortic stenosis
Ans…..e) Subvalvular aortic stenosis may become apparent after a
successful repair of other cardiac defects (e.g., VSD, PDA, or
coarctation of aorta).
A child appears with symptoms of shortness of breath and fatigue with
a mild exertion. The private MD appreciated a systolic ejection murmur at
the sternal border. The patient was referred to a pediatric cardiologist. The
echocardiogram reveals an aortic stenosis and severe tunnel-like subaortic
obstruction. The preferred therapy is:
a) Kenno procedure
b) Balloon valvuloplasty
c) Aortopulmonary anastomosis
d) Put a stent in the obstructed area
e) Create an artificial VSD
Ans…..a) Konno procedure (i.e., obstruction of a left ventricular
outflow tract can be relieved by borrowing space anteriorly from a right
ventricular outflow tract).
Heterotaxy syndromes are associated with all of the following cardiac
conditions except:
a) Dextrocardia
b) Transposition of great arteries
c) Common atrioventricular valve
d) Single atrium
e) Total anomalous pulmonary venous return
Ans…..   d) Single atrium is not present; other cardiac conditions are
Dr=Wahid Helmi

single ventricle, pulmonic stenosis, and pulmonary atresia.


A child is diagnosed to have a congenital heart disease. The oxygen
saturation in superior vena cava is 74%. The oxygen saturation and
pressure in the right atrium are 74% and 0-3 mm Hg respectively. The
oxygen saturation and pressure in the right ventricle are 74% and 40/6 mm
Hg. The oxygen saturation and pressure in the pulmonary artery are 74%
and 5 mm Hg. The oxygen saturation in the pulmonary veins are 98%. The
oxygen saturation and pressure in the left atrium are 98% and 4-8 mm Hg.
The oxygen saturation and pressure in the left ventricle are 80% and
105/10 mm Hg. The oxygen saturation and pressure in the aorta are 80%
and 100/60 mm Hg. The preferred surgical procedure is:
a) Arterial switch (Jatene) operation
b) Norwood operation
c) Starnes procedure
d) Fontan procedure
e) Blalock-Taussig shunt
Ans….e) Blalock-Taussig shunt or total repair is performed in
patients with a tetralogy of Fallot. This patient has a high right ventricular
pressure 40/6 mm Hg (normal 25/3 mm Hg), low pulmonary artery pressure
5 mm Hg (normal 25/10 mm Hg), reduced left ventricular saturation 80%
(normal 94-100%), and reduced aortic saturation 80% (normal 94-100%).
Dr=Wahid Helmi

A child is diagnosed to have a congenital heart disease. The oxygen


saturation in superior vena cava is 74%. The oxygen saturation and
pressure in the right atrium are 74% and 0-3 mm Hg. The oxygen
saturation and pressure in the right ventricle are 74% and 100/8 mm Hg.
The oxygen saturation and pressure in the pulmonary artery are 98% and
40/2 mm Hg. The oxygen saturation in the pulmonary veins are 98%. The
oxygen saturation and pressure in the left atrium are 98% and 4-8 mm Hg.
The oxygen saturation and pressure in the left ventricle are 98% and 40/2
mm Hg. The oxygen saturation and pressure in the aorta are 74% and
100/60 mm Hg. The preferred surgical procedure is:
a) Blalock-Taussig shunt
b) Arterial switch (Jatene) operation.
c) Glenn shunt
d) Fontan procedure
e) Norwood operation
Ans... b) Arterial switch (Jatene) operation is the preferred
procedure for transposition of great vessels. However, Rashkind balloon
atrial septostomy is the initial procedure of choice. This patient has a high
right ventricular pressure 100/8 mm Hg (normal 25/3 mm/Hg), high
pulmonary artery pressure 40/2 mm Hg (normal 25/10 mm Hg), decreased
left ventricular pressure 40/2 mm Hg (normal 100/8 mm Hg).
Dr=Wahid Helmi

A child is diagnosed to have a congenital heart disease. The oxygen


saturation in superior vena cava is 74%. The oxygen saturation and
pressure in the right atrium are 74% and 0-3 mm Hg. The oxygen
saturation and pressure in the right ventricle are 80% and 35/5 mm Hg. The
oxygen saturation and pressure in the pulmonary artery are 80% and 25/10
mm Hg. The oxygen saturation in the pulmonary veins are 98%. The
oxygen saturation and pressure in the left atrium are 98% and 4-8 mm Hg.
The oxygen saturation and pressure in the left ventricle are 98% and 100/8
mm Hg. The oxygen saturation and pressure in the aorta are 98% and
100/60 mm Hg. The preferred surgical procedure is:
a) Repair of VSD
b) Repair of ASD
c) Surgical closure of PDA
d) Surgical repair of truncus arteriosus
e) Total repair of anomalous pulmonary venous return
Ans... a) Repair of VSD is indicated. This patient has an increased
right ventricular pressure 35/5mm Hg (normal 25/3mm Hg),
increased right ventricular saturation 80% (normal 74%), and increased
pulmonary artery saturation 80% (normal 74%).
Dr=Wahid Helmi

A 15-year-old boy came for a routine physical examination. He is a


good athlete. A physical examination reveals heart rate 40 beats per
minute. The rest o the physical examination is unremarkable. The next step
in management is:
a) EKG
b) Stress test
c) Cardiology consult
d) Pace maker
e) Reassurance
Ans... e) Reassurance; the heart rate can normally drop up to 40
beats per minute in athletic adolescents.

All of the following statements indicate pathologic conditions except:


a) A newborn’s heart rate is more than 200 beats per minute persistently.
b) An infant’s heart rate is more than 150 beats per minute persistently.
c) An older child’s heart rate is more than 120 beats per minute
persistently.
d) A full term infant’s heart rate is 68 beats per minute during sleep.
e) A 16-year-old athlete’s heart rate is 40 beats per minute.
Ans... e) A 16-years-old athlete’s heart rate up to 40 beats per
minute is normal
Dr=Wahid Helmi

A newborn appears with a pink right arm and blue lower extremities.
The most likely diagnosis is:
a) Tetralogy of Fallot
b) Transposition of great arteries
c) Truncus arteriosus
d) Coarctation of aorta
e) Total anomalous pulmonary venous return
Ans….d) Differential cyanosis is noted in coarctation of aorta,
interrupted aortic arch, and persistent pulmonary hypertension (PPHN).

A newborn appears with a blue upper arm and less blue lower
extremities. The most likely diagnosis is:
a) Transposition of great arteries
b) Tetralogy of Fallot
c) Truncus arteriosus
d) Tricuspid atresia
e) Total anomalous pulmonary venous return
Ans….a) Transposition of great arteries
Dr=Wahid Helmi

A newborn appears with a circumoral cyanosis. The mucous


membranes (i.e., lips and tongue) are pink. The most likely diagnosis is:
a) Cold lips
b) VSD
c) ASD
d) PDA
e) Prominent venous plexus in lips
Ans….e) Prominent venous plexus in lips

A new born appears with a blueness around the forehead. The rest of
the physical examination is unremarkable. The most likely diagnosis is:
a) Cold forehead
b) Meningocele
c) Hydrocephalus
d) Encephalocele
e) Prominent venous plexus on the forehead
Ans….e) Prominent venous plexus on the forehead
Dr=Wahid Helmi

A newborn appears with a cyanosis of both hands and feet. The rest
of the physical examination is unremarkable. The most likely diagnosis is:
a) Transposition of great vessels
b) Gram-negative septic shock
c) Patent foramen ovale
d) Unwrapped and cold extremities
e) Persistent pulmonary hypertension
Ans….d) Unwrapped and cold extremities (i.e., acrocyanosis)

The common organisms in patients with a native valve and infective


endocarditis are all of the following except:
a) Staphylococcus epidermidis
b) Staphylococcus aureus
c) Streptococcus mutans
d) Streptococcus sanguis
e) Streptococcus bovis
Ans... a) S. epidermidis is common in the presence of an indwelling
central venous catheter or prosthetic valve. For a native valve, the common
organisms are viridans group streptococci (e.g., S. mutans, S. sanguis, S.
mitis), S. aureus, group D Streptococcus (enterococcus) (S. bovis, S.
faecalis).
Dr=Wahid Helmi

The most common organism in patients with an infective endocarditis


without an underlying heart disease is:
a) S. aureus
b) Pseudomonas aeruginosa
c) Serratia marcescens
d) Streptococcus mitis
e) Streptococcus faecalis
Ans... a) S. aureus

The most common organism in patients with an infective endocarditis


after a dental procedure is:
a) Streptococcus mutans
b) S. aureus
c) Streptococcus bovis
d) Streptococcus pneumoniae
e) Streptococcus faecalis
Ans... a) Viridans group streptococci (e.g., S. mutans, S. sanguis,
S. mitis)
The most common organism in patients with an infective endocarditis
after a lower bowel surgery is:
a) Streptococcus bovis
b) Streptococcus mutans
c) Streptococcus sunguis
d) E. coli
e) Serratia marcescens
Ans….a) Group D streptococcus (enterococcus)(e.g., S. bovis, S.
Dr=Wahid Helmi

faecalis)
The most common organism in patients with an infective endocarditis
after a genitourinary intervention is:
a) Streptococcus pneumoniae
b) E. coli
c) Pseudomonas aeruginosa
d) Campylobacter fetus
e) Streptococcus faecalis
Ans….e) Group D streptococcus (enterococcus) (e.g., S. bovis, S.
faecalis)
The most common organism in patients with an infective endocarditis
and intravenous drug abusers is:
a) Streptococcus aureus
b) Pseudomonas aeruginosa
c) Streptococcus epidermidis
d) Streptococcus mitis
e) Candida albicans
Ans….b) Pseudomonas aeruginosa or Serratia marcescens
The most common organism in patients with an infective endocarditis
and indwelling central venous catheter is:
a) S. aureus
b) Serratia marcescens
c) S. epidermidis
d) Candida albicans
e) Haemophilus influenzae
Ans….c) Staphylococcus epidermidis
Dr=Wahid Helmi

The following condition is an important risk factor for developing an


infective endocarditis in children with a cyanotic heart disease:
a) Diarrhea
b) RSV infection
c) MMR vaccination
d) Poor dental hygiene
e) Arrythmias
Ans…..d) Poor dental hygiene
All of the following postoperative cardiac conditions are risk factors for
developing infective endocarditis except:
a) Replaced aortic valve
b) Norwood aortopulmonary anastomosis
c) Blalock-Taussig shunt
d) Repair a mitral valve
e) Repair of a simple atrial septal defect
Ans... e) Repair of a simple ASD or closure of PDA almost
eliminates the risk of developing endocarditis. A surgical correction of a
congenital heart disease can reduce but does not eliminate the risk of
endocarditis except the two conditions mentioned above
All of the following preoperative cardiac conditions are risk factors for
developing infective endocarditis except:
a) VSD
b) Tetralogy of Fallot
c) Aortic stenosis
d) Tricuspid regurgitation in newborns
e) Transposition of great arteries
Dr=Wahid Helmi

Ans….d) Tricuspid regurgitation (TR) is not a preoperative


condition and does not produce infective endocarditis. TR is common in
newborns with a perinatal asphyxia secondary to transient papillary
muscles dysfunction. TR is a self-limiting condition. However, mitral valve
prolapse, bicuspid aortic valves, PDA, ASD, and other anatomical cardiac
defects, especially involving the left side of the heart can develop infective
endocarditis.

The foramen ovale is functionally closed in a normal healthy newborn


by:
a) 12 hours of life
b) 24 hours of life
c) 7 days of life
d) 1 month of life
e) 3 months of life
Ans….e) 3 months of life
The ductus arteriosus is functionally closed in a normal healthy
newborn by
a) 0-5 hours of life
b) 5-10 hours of life
c) 10-15 hours of life
d) 15-20 hours of life
e) 20-25 hours of life
Ans….c) 10-15 hours of life
554. The ductus arteriosus closes when the PO2 of the blood at the ductus
is about:
Dr=Wahid Helmi

a) 45 mm Hg
b) 50 mm Hg
c) 55 mm Hg
d) 60 mm Hg
e) 65 mm Hg
Ans….b) 50 mm Hg

The most common cardiac anomaly in patients with an autosomal


dominant polycystic kidney disease is:
a) ASD
b) VSD
c) Mitral valve prolapse
d) Peripheral pulmonic stenosis
e) Tricuspid atresia
Ans….c) Mitral valve prolapse
Facio-auriculo-vertebral spectum (FAVS) is associated with the
following cardiac anomaly:
a) ASD
b) Tetralogy of Fallot
c) Tricuspid stenosis
d) Truncus arteriosus
e) Transposition of great arteries
Ans….b) TOF or VSD
Cat-eye syndrome has the following chromosomal abnormality:
a) Trisomy 9
Dr=Wahid Helmi

b) Trisomy 21
c) Trisomy 21P
d) Trisomy 13
e) Trisomy 5P
Ans….c) Trisomy 21p is cat-eye syndrome. Deletion 5p is cat-cry
syndrome (or cri du chat syndrome).

The most common cause of death in pediatric patients with a heart


transplantation is:
a) CMV infection
b) Candida albicans infection
c) Protozoal infection
d) Toxoplasma infection
e) S. aureus infection
Ans…. a) CMV infections
All of the following suggestions are useful as a prevention of high
blood pressure in pediatric population except:
a) Control obesity
b) Reduce dietary sodium intake
c) Physical exercise
d) Reduce serum cholesterol levels
e) Consume a small amount of red-wine
Ans... e) Alcohol and tobacco use cause hypertension
A 17-year-old boy for a routine physical examination. He is
asymptomatic but experienced occasional headaches. His recorded blood
Dr=Wahid Helmi

pressure is above the 95th percentile for his age. The most likely diagnosis
is:
a) Aortic coarctation
b) Renal artery stenosis
c) Pheochromocytoma
d) Hyperthyroidism
e) Suspected hypertension
Ans... e) Suspected hypertension (BP is above 95th percentile)
A 16-year-old boy appears for a routine physical examination. He is
asymptomatic. His recorded blood pressures are between 90th and 95th
percentiles. He is obese. In addition to weight loss, the next step in
management is:
a) A follow-up examination after 1 month
b) A follow-up examination after 3 months
c) A follow-up examination after 6 months
d) Furosemide
e) Aldomet
Ans... c) A follow up examination after 6 months is recommended
(BP is between 90th and 95th percentiles)
A 13-year-old boy appears for a routine physical examination. He is
asymptomatic. His recorded blood pressure is below the 90th percentile.
The next step in management is:
a) Echocardiography
b) Renal ultrasonography
c) Follow-up after 1 year
d) Low-salt diet
Dr=Wahid Helmi

e) Blood pressure should be measured on three separate occasions.


Ans…,e) BP should be measured on three separate occasions
prior to further interventions (BP is less than 90th percentile).
The lowest limit of a normal systolic blood pressure in a full-term
newborn is:
a) 45 mm Hg
b) 50 mm Hg
c) 60 mm Hg
d) 70 mm Hg
e) 80 mm Hg
Ans….c) Less than 60 mm Hg in a full-term newborn is considered
low BP
The lowest limit of a normal systolic BP from 1 month to 1 year of age
is:
a) 65 mm Hg
b) 70 mm Hg
c) 75 mm Hg
d) 80 mm Hg
e) 85 mm Hg
Ans….b) Less than 70 mm Hg from 1 month to 1 year of age is
considered low BP.
The lowest limit of a normal systolic BP in a 5-year-old child is:
a) 75 mm Hg
b) 80 mm Hg
c) 85 mm Hg
d) 90 mm Hg
Dr=Wahid Helmi

e) 95 mm Hg
Ans….b) Less than 80 mm Hg in a 5 year old child is considered
low BP; the formula:Less than 70 mm Hg + 2 x age from 1-10 years.
The lowest limit of a normal systolic BP in a child older than 10 years
of age is:
a) 80 mm Hg
b) 85 mm Hg
c) 90 mm Hg
d) 95 mm Hg
e) 100 mm Hg
Ans….c) Less than 90 mm Hg in children older than 10 years of
age is considered low BP.
A routine physical examination in a 12-year-old boy reveals heart rate
85 per minute, systolic BP 135 mm Hg, diastolic BP 85 mm Hg, respiratory
rate 18 per minute, and the rest of the examination is unremarkable.
Occasionally, he complains of headaches. The next step in management
is:
a) Exercise
b) Weight loss
c) Low-salt diet
d) Furosemide
e) Reassurance
Ans….e) Reassurance because all findings are the upper limit of
normal for a 12-year-old child. However, exercise is good for all
healthy children and weight loss is indicated in obese children.
A routine physical examination of the child reveals pulsation of the
Dr=Wahid Helmi

aorta. He is thin and asymptomatic. The next step in management is:


a) Ultrasonography of the aorta
b) Measure four extremities BP
c) Aortic angiography
d) Abdominal x-ray
e) Reassurance
Ans….e) Reassurance; this is a normal finding in this young
children.
A 2-day-old newborn male appears with cyanosis and tachypnea for
the last 2 hours. He was born by NSVD with Apgar scores are 8 and 9 at 1
and 5 minutes respectively. The physical examination reveals a grade 2/6
systolic murmur at the left sternal border. The arterial PO2 is 80 in
hyperoxia test. The oxygen saturation values are as follows: RA 58%, RV
58%, LA 100%, LV 94%, pulmonary artery 58%, and ascending aorta 80%.
The volume of blood passes through different structures are as follows: RA
3 L/min/m2, RV 3 L/min/m2, LA 2 L/min/m2, LV 2 L/min/m2, pulmonary
artery 2 L/min/m2, and ascending aorta 3 L/min/m2. The newborn is
receiving PGE1 infusions. The preferred surgical therapy is:
a) Switch operation (Jatene)
b) Modified Blalock-Taussig shunt
c) Starnes procedure
d) Norwood operation
e) Fontan procedure
Ans... b) Modified Blalock-Taussig shunt (i.e., Gore-Tex conduit
anastomosed side to side from the subclavian artery to the
homolateral branch of the pulmonary artery) is the preferred surgical
Dr=Wahid Helmi

procedure in patients with a tetralogy of Fallot


A newborn appears with cyanosis and respiratory distress within the
1st hour of life. She was born by NSVD with Apgar score of 7 and 8 at 1
and 5 minutes respectively. The physical examination reveals a soft
systolic ejection murmur at the midleft sternal border and single, loud 2nd
heart sound. The arterial PO2 is 60 in hyperoxia test. The oxygen
saturation values are as follows: both vena cava 58%, RA 73%, RV 73%,
LA 90%, LV 90%, pulmonary artery 90%, right and left pulmonary arteries
80%, pulmonary veins 100%, and ascending aorta 73%. The newborn is
placed on a mechanical ventilator and PGE1 infusions have started. The
preferred initial surgical therapy is:
a) Norwood operation
b) Glenn operation
c) Fontan procedure
d) Aortic valvuloplasty
e) Rashkind procedure
Ans….e) Rashkind balloon atrial septostomy is the initial procedure
of choice in patients with transposition of great arteries. Arterial
switch (Jatene) operation is the final procedure of choice
The pulmonary arteriovenous fistulas are most commonly present is:
a) Williams syndrome
b) Down syndrome
c) Turner syndrome
d) Prader-Willi syndrome
e) Osler-Weber-Rendu syndrome
Ans... e) Osler-Weber-Rendu syndrome (hereditary hemorrhagic
Dr=Wahid Helmi

telangiectasia type 1) is associated with angiomas of the GI tract, liver,


nasal and buccal mucous membranes
A newborn appears with bradyarrythmias. ECG reveals that the P-P
intervals are constant, the P-R intervals increase progressively until a P
wave is not conducted; following a dropped beat, the P-R interval is
shorter. The most likely diagnosis is:
a) Long Q-T syndrome
b) 1st-degree heart block
c) Mobitz type I
d) Mobitz type II
e) 3rd-degree heart block
Ans….c) Mobitz type I (Wenckebach type) is one variant of
2nd-degree heart block.
A child appears with history of syncope. He is otherwise healthy. ECG
reveals that occasional atrial beats are not conducted to the ventricles. The
most likely diagnosis is:
a) 1st-degree heart block
b) Wenckebach type heart block
c) Mobitz type II heart block
d) 3rd-degree heart block
e) Normal variant
Ans... c) Mobitz type II
A newborn female appears with edema of the dorsa of the hands and
feet, loose skinfolds at the nape of the neck, low birthweight, and
decreased length. The most common cardiac anomaly in this patient is:
a) Aortic coarctation
Dr=Wahid Helmi

b) Pulmonic stenosis
c) Aortic stenosis
d) Mitral valve prolapse
e) Nonstenotic bicuspid aortic valve
Ans….e) Nonstenotic bicuspid aortic valve (33-50% of cases) is
the most common cardiac defect in patients with Turner syndrome

Match the congenital heart diseases and surgical procedures


998. Transposition of great arteries
999. Hypoplastic left heart syndrome
1000. Tetralogy of Fallot
1001. Ebstein anomaly
1002. Aortic stenosis
a) Blalock-Taussig shunt
b) Norwood procedure
c) Balloon valvuloplasty
d) Starnes procedure
e) Arterial switch (Jatene)
Dr=Wahid Helmi

Ans.... e) Arterial switch (Jatene); Rashkind balloon atrial


septostomy is the initial procedure of choice.
999. b) Norwood procedure
1000. a) Blalock-Taussig shunt
1001. d) Starnes procedure
1002. c) Balloon valvuloplasty
Dr=Wahid Helmi

Match the following syndromes with different clinical findings (1003-1007):


1003. Aortic stenosis
1004. Pulmonic stenosis
1005. Mitral valve prolapse
1006. Total anomalous pulmonary venous return 1007. Tricuspid atresia
a) Well-developed, well-nourished child
b) Sudden death
c) “Snowman sign” (figure eight)
d) Scoliosis
e) ECG reveals characteristic left axis deviation
Ans.. 1003. b) Sudden death
1004. a) Well-developed, well-nourished child
1005. d) Scoliosis
1006. c) “Snowman sign” (figure eight)
1007. e) ECG reveals a characteristic left axis deviation that is also noted
in endocardial cushion defects.
Dr=Wahid Helmi

A child is diagnosed to have restrictive


pericardial effusion. The important clinical signs are the following:
a) Tachycardia, hypotension, and decreased oxygen saturation
b) Bradycardia, hypotension, and decreased perfusion
c) Tachycardia, hypertension, and swelling of face
d) Arrythmia, normal BP, and venous engorgement of neck
e) Fever, tachycardia, and hypertension
Ans---A
Tachycardia, hypotension, and decreased oxygen saturation

day-old male newborn appears with heart murmur. Oxygen saturation values are the
following: right atrium (72%), right ventricle (83%), left atrium (100%), left ventricle
(100%),pulmonary artery (87%), and aorta (100%). Blood flow volume
(L/minute/m2) through the heart and blood vessels is the following: right
atrium (3), right ventricle (5), left atrium (6), left ventricle (6), pulmonary
artery (5), and aorta (4). The most likely diagnosis is:
a) Tetralogy of Fallot
b) VSD
c) Pulmonary stenosis
d) Aortic stenosis
e) Transposition of great arteries
Ans….B VSD
Dr=Wahid Helmi

child had surgery for right ventricular outflow tract obstruction. The expected
postoperative complication is:
a) Pulmonic stenosis
b) Tricuspid regurgitation
c) Pulmonary valvular insufficiency
d) Tricuspid stenosis
e) Rupture interventricular septum
Ans C Pulmonary valvular insufficiency also occurs due to severe pulmonary
hypertension.

child has congenital absence of the pulmonary valve. The most common associated
anomaly is:
a) ASD
b) VSD
c) Aortic stenosis
d) Pulmonic stenosis
e) Congenital absence of tricuspid valve
AnsB VSD
Dr=Wahid Helmi

Match the different cardiac conditions with different types of murmurs (242-246):
242. Coarctation of the aorta
243. Mitral valve prolapse
244. Pulmonary valvular insufficiency
245. Congenital mitral insufficiency
246. Congenital mitral stenosis
a) A low-pitched decrescendo diastolic murmur at the upper and midleft
sternal border
b) A short systolic murmur is noted along the left sternal border at the 3rd
and 4th intercostal spaces; murmur is transmitted
to the left infrascapular area.
c) Apical murmur is late systolic and associated with a click.
d) A high-pitched (“cooing dove”), apical holosystolic murmur
e) A rumbling apical diastolic murmur is followed by loud 1st sound; 2nd
sound is loud and split.
Ans….242/ b - A short systolic murmur is noted along the left sternal border at the
3rd and 4th intercostal spaces; murmur is transmitted to the left infrascapular area.
243/ c - Apical murmur is late systolic and may be associated with a click.
244/ a - A low-pitched decrescendo diastolic murmur is noted at the upper and
midleft sternal border
245/ d - A high-pitched (“cooing dove”), apical holosystolic murmur; moderate to
severe insufficiency causes low-pitched, mid- diastolic rumbling murmur.
246/ e - A rumbling apical diastolic murmur is followed by loud 1st sound; 2nd
sound is loud and split.
Dr=Wahid Helmi

Match all the clinical findings with the different clinical conditions (451-453):
451. Sinus bradycardia in neonates
452. Sinus bradycardia after neonatal period
453. Supraventricular tachycardia in neonates
a) Heart rate is 231 beats/minute
b) Heart rate is less than 90 beats/minute.
c) Heart rate is less than 60 beats/minute.
Ans...
451/ b - In neonates with sinus bradycardia, heart rate is less than 90 beats/minute.
452/c - After neonatal period, heart rate less than 60 beats/minute represents sinus
bradycardia
453/ a - In neonates with SVT, heart rate is usually more than 230 beats/minute
with abnormal P-wave axis. In other age groups with SVT, heart rate is
above 180 and up to 300 beats/minute.

routine physical examination in a full-term newborn reveals heart rate 90 beats/minute


during sleeping. The newborn is asymptomatic. The next step in management is:
a) Cardiology consultation
b) EKG
c) Sepsis work up
d) Chest x-ray
e) Reassurance
AnsE Reassurance
Dr=Wahid Helmi

routine physical examination of anewborn infant reveals heart rate 180 beats/minute
during crying. The newborn is asymptomatic. The next step in management is:
a) EKG
b) Cardiology consultation
c) Chest x-ray
d) Sepsis work up
e) Reassurance
AnsE Reassurance

The preferred therapy for patients with re-stenosis of coarctation of the aorta after earlier
surgery is:
a) Aortic stent placement
b) Repeat surgical procedure
c) Antihypertensive agent
d) Balloon angioplasty.
e) Small dose of acetyl salicylic acid daily
Ans….D Balloon angioplasty

All of the following clinical manifestations are present in newborns with critical pulmonic
stenosis except:
a) Pulmonary edema . b) Peripheral edema
c) Hepatomegaly d) Cyanosis
e) Dyspnea
Ans ---A--- Pulmonary circulation is decreased significantly in patients with critical PS.
They appear with right-sided heart failure.
Dr=Wahid Helmi

All of the following clinical manifestations are present in newborns with critical aortic
stenosis except:
a) Pulmonary edema
b) Poor perfusion
c) Hepatomegaly
d) Peripheral edema
e) Hypertension
Ans….E
Patients with critical AS manifest with hypotension and subsequently, total
circulatory collapse. They appear with both left- sided and right-sided
cardiac failure.

The most common cardiac defect in patients with autosomal dominant Holt-Oram
syndrome is:
a) PDA
b) ASD
c) VSD
d) Aortic stenosis
e) Pulmonic stenosis
Ans ---B ---Secundum ASD (atrial septal defect)
Dr=Wahid Helmi

The most common cardiac defect in patients with Alagille syndrome is:
a) VSD
b) ASD
c) Mitral regurgitation
d) Tricuspid regurgitation
e) Pulmonary stenosis.
Ans…..E Pulmonary stenosis (e.g., pulmonary valve or branch pulmonary artery) is
common in patients with arteriohepatic dysplasia (Alagille syndrome).

PHACE syndrome includes all of the following features except:


a) Posterior brain fossa anomalies
b) Hemangiomas (facial)
c) Arterial anomalies
d) Cardiac anomalies and aortic dilatation.
e) Eye anomalies
Ans….D Cardiac anomalies and aortic coarctation
Dr=Wahid Helmi

CATCH 22 syndrome can be associated with the following cardiac defect:


a) Tetralogy of Fallot.
b) Transposition of great vessels
c) Total anomalous pulmonary venous return
d) Tricuspid atresia
e) Hypoplastic left heart syndrome
Ans—A ---CATCH 22 syndrome can be associated with tetralogy of Fallot, pulmonary
atresia with VSD (a severe form of TOF), or interrupted aortic arch.

CATCH 22 syndrome includes all of the following findings except:


a) Cardiac defects
b) Abnormal facies
c) Thymic hypoplasia
d) Cleft lip
e) Hypocalcemia
Ans….D Cleft palate
Dr=Wahid Helmi

Match all different postoperative complications with clinical manifestations (779-783):


779. Phrenic nerve injury
780. Recurrent laryngeal nerve injury
781. Sympathetic nerve chain injury
782. Hyponatremia
783. Spinal artery ischemia
a) Horner syndrome
b) Seizures
c) Diaphragmatic paralysis
d) Vocal cord paralysis
e) Paraplegia
Ans….
779/ c Phrenic nerve injury causes diaphragmatic paralysis.
780/ d Recurrent laryngeal nerve injury causes vocal cord paralysis.
781/ a Sympathetic nerve chain injury causes Horner syndrome.
782/ b Hyponatremia causes seizures that are also produced by hypoglycemia,
CNS ischemia, and emboli.
783/ e Spinal artery ischemia after repair of coarctation causes paraplegia.
Dr=Wahid Helmi

Match all of the following complications


after cardiac surgery with different manifestations (784-788):
784. Postcoarctectomy syndrome
785. Repair of TAPVR
786. ARDS postpump syndrome
787. SIADH
788. Serous postpericardiotomy syndrome
a) Hyponatremia
b) Hypertension
c) Pericardial tamponade
d) Pulmonary hypertension
e) Possible release of vasoactive substances
Ans
784/ b Postcoarctectomy syndrome causes hypertension and mesenteric arteritis.
785/ d Repair of TAPVR (total anomalous pulmonary venous return) can cause
pulmonary hypertension that can occur trisomy 21 and Norwood 1st stage
operation.
786/ e ARDS postpump syndrome can be due to possible release of vasoactive
substances but the exact mechanism is unknown.
787/ a SIADH produces hyponatremia.
788/ c Serous postpericardiotomy syndrome produces pericardial tamponade
Dr=Wahid Helmi

All of the following statements are true in patients with Wolff-Parkinson-White syndrome
except:
a) WPW syndrome may be associated with Ebstein anomaly.
b) WPW syndrome is absent in patients with a normal heart.
c) WPW syndrome is usually seen when the patient does not have
tachycardia.
d) EKG reveals a short P-R interval and slow upstroke of QRS (delta
wave).
e) The anatomic re-entrant circuit are the AV node and an accessory
preexcitation pathway consisting of a muscular bridge
connecting atrium to the ventricle.
Ans- B WPW syndrome most often present in patients with a normal heart.

Ebstein anomaly is associated with the following:


a) Tricuspid stenosis
b) Tricuspid regurgitation
c) Mitral stenosis
d) Pulmonic stenosis
e) Aortic regurgitation
Ans- (b) Tricuspid regurgitation
Dr=Wahid Helmi

Mitral valve prolapse is commonly associated with all of the following


conditions except:
a) Marfan syndrome
b) Scoliosis
c) Pectus excavatum
d) Straight back syndrome
e) Noonan syndrome
Ans- (e) Noonan syndrome

The immediate postoperative complication after surgical repair of coarctation of aorta is:
a) Rebound hypertension
b) Hypotension
c) Aortic aneurysm
d) Upper extremities hypotension
e) Anastomotic leak
Ans (a) Rebound hypertension

The treatment for postoperative hypertension after surgical repair of


coarctation of aorta is:
a) Antihypertensive medication
b) Reoperation because of operative failure
c) Renal failure
d) Cardiac failure
e) Fluid retention because of increased ADH secretion
Ans…. (a) Antihypertensive medication
Dr=Wahid Helmi

The development of recoarctation after surgical repair of coarctation of


the aorta is best treated with:
a) Repeat surgical repair
b) Reassurance
c) Aspirin
d) Vasodilator medication
e) Balloon angioplasty
Ans.... (e) Balloon angioplasty. Intravascular stents are commonly used.
Dr=Wahid Helmi

A 5-month-old girl appeared with history of respiratory infections


followed by congestive cardiac failure. She was completely asymptomatic
prior to this episode. She appeared with dyspnea, cough, and failure to
thrive. Physical examination revealed edema, hepatomegaly, and
pulmonary congestion. Chest x-ray revealed cardiomegaly and clear lung
fields. EKG reveals left atrial and left ventricular hypertrophy.
Echocardiogram revealed a bright endocardial surface and a dilated, poorly
contracting left ventricle. The most likely diagnosis is:
a) Hypoplastic left ventricle
b) Aortic stenosis
c) Tetralogy of Fallot
d) Cardiomyopathy
e) Endocardial fibroelastosis
Ans (e) Endocardial fibroelastosis

The preferred therapy for patients with end-stage endocardial


fibroelastosis is:
a) Digoxin b) Lasix
c) Pacemaker d) Removal of endocardial tissue
e) Heart transplantation
Ans (e) Heart transplantation. Initial therapy includes prevention of respiratory infections
and treatment of congestive heart failure
Dr=Wahid Helmi

A mother is pregnant with her second child. Her first child suffered from a congenital
heart disease. The incidence of heart disease in her second child is:
a) 0.8%
b) 1-2%
c) 2-6%.
d) 10-20%
e) 20-30%
Ans
(c) 2–6%; the same incidence if one parent was affected.

321. A mother is pregnant with her third child. Her first and second children
suffered from congenital heart disease. The incidence of heart disease in
her third child is:
a) 2-6%
b) 10-20%
c) 20-30%.
d) 30-50%
e) 50-100%
Ans (c) 20–30%; the same incidence if two first–degree
Dr=Wahid Helmi

relatives were affected.


The lateral thoracotomy was performed on a child with tetralogy of
Fallot. All of the following complications can occur shortly after surgery
except:
a) Chylothorax
b) Horner syndrome
c) Diaphragmatic paralysis
d) Cardiac failure
e) Hyperactive radial pulse.
Ans (e) Radial pulse may be diminished on the side of
Blalock–Taussig shunt.
An infant is diagnosed with tetralogy of Fallot and admitted for the
modified Blalock-Taussig shunt operation. Two days prior to surgery, in the
early morning the infant developed cyanosis, tachypnea, and restlessness.
The resident who was on call made the diagnosis hypoxic, ‘blue’, or ‘tet’
spells. The infant was placed on the abdomen in knee-chest position,
received oxygen, and subcutaneous injection of morphine (0.2 mg/kg). The
infant remained cyanotic and developed metabolic acidosis. The infant
received a rapid intravenous infusion of sodium bicarbonate but did not
improve. The next step in management is:
a) Sepsis work up
b) Repeat ABG
c) Repeat EKG
d) Repeat echocardiogram
e) Intravenous methoxamine.
Dr=Wahid Helmi

Ans… (e) Intravenous methoxamine or phenylephrine


improves right ventricular flow, decreases right–to–left shunt, and
improves the cyanosis and symptoms. Intravenous propranolol
(beta–adrenergie blocker) is also used.

An infant is diagnosed with a severe form of tetralogy of Fallot. She is


receiving prostaglandin E1 infusion. The preferred therapy for the infant is:
a) Prostaglandin E2 infusion
b) Continue PGE1 infusion
c) Oral propranolol
d) Iron therapy
e) Open heat surgery and total correction.
Ans… (e) Open heart surgery and total corrections are
indicated. The modified Blalock–Taussig shunt (i.e, a Gore–Tex conduit
anastomosed side to side from the subclavian artery to the homolateral
branch of pulmonary artery) can be performed to improve pulmonary
circulation.
Dr=Wahid Helmi

An infant is diagnosed with a less severe form of tetralogy of Fallot.


He is growing normally and has no cyanotic spells. The time to perform
surgical repair is:
a) Between 1 and 3 months
b) Between 4 and 12 months.
c) Between 12 and 15 months
d) Between 15 and 18 months
e) Between 18 and 24 months
Ans. (b) Between 4 and 12 months of age
An infant is diagnosed with a less severe form of tetralogy of Fallot.
She is waiting for a surgical repair but needs careful observation. All of the
following treatments are useful except:
a) Hydration
b) Iron
c) Propranolol
d) Oxygen prn
e) Sodium bicarbonate.
Ans (e) Sodium bicarbonate therapy is not indicated.
Hydration is useful to prevent hemoconcentration. Iron therapy prevents the
paroxysmal dyspneic attacks in infancy and early childhood. Propranolol
decreases the hypercyanotic spells. Oxygen is given when indicated
1
1.Questions and Answers after the questions.

1. Which of the following is not associated with esophageal webs?

A. Plummer-Vinson syndrome
B. Epidermolysis bullosa
C. Lupus
D. Psoriasis
E. Stevens-Johnson syndrome

2. An 11 year old boy complains that occasionally a bite of hotdog “gives mild
pressing pain in his chest” and that “it takes a while before he can take another
bite.” If it happens again, he discards the hotdog but sometimes he can finish it.
The most helpful diagnostic information would come from

A. Family history of Schatzki rings


B. Eosinophil counts
C. UGI
D. Time-phased MRI
E. Technetium 99 salivagram

3. 12 year old boy previously healthy with one-month history of difficulty swallowing
both solid and liquids. He sometimes complains food is getting stuck in his
retrosternal area after swallowing. His weight decreased approximately 5% from last
year. He denies vomiting, choking, gagging, drooling, pain during swallowing or
retrosternal pain. His physical examination is normal.

What would be the appropriate next investigation to perform in this


patient?

A. Upper Endoscopy
B. Upper GI contrast study
C. Esophageal manometry
D. Modified Barium Swallow (MBS)
E. Direct laryngoscopy

4. A 12 year old male presents to the ER after a recent episode of emesis. The parents
are concerned because undigested food 3 days old was in his vomit. He admits to a
sensation of food and liquids “sticking” in his chest for the past 4 months, as he
points to the upper middle chest. Parents relate a 10 lb (4.5 Kg) weight loss over the
past 3 months. Past medical history and family history are unre- markable. Vital
signs are stable, and physical exam is unremarkable. The ER physician obtains a chest
2
X-ray AP and lateral that shows dilatation of the esophagus with an air fluid level.
What is the best diagnostic test for this patient’s condition?

A. Endoscopy
B. 24 hour PH monitoring
C. Barium swallow
D. Esophageal manometry
3

5. A 26 month old female is referred to the GI clinic with a history of spitting up since 10
month of age. Mom noticed that symptoms began after the introduction of table food.
The pediatrician diagnosed GERD and started the patient on an H2 blocker.
Medication was changed to a proton pump inhibi- tor without improvement and the
patient continued to spit up and have difficulty swallowing with solids but not with
liquids. She was in the 50%ile for height and the 25%ile for weight. Her diet was
mainly liquid. The pediatrician was concerned because in the last month she did not
gain weight. Af- ter your history and physical exam you ordered a barium swallow
which showed a posterior impres- sion of the upper-middle esophagus. Which of the
followings is the next step in management?

A. Esophagoscopy
B. 24 hours PH monitoring
C. Barium swallow
D. Esophageal manometry
D. Chest MRI

6. Eosinophilic Esophagitis is associated with the following diseases except:

A. Atopic Dermatitis
B. Asthma
C. Helicobacter Pylori
D. Allergic Rhinitis

7. You are seeing an 8 year old male in clinic as a follow-up from a recent EGD you
performed for the sensation of “things getting stuck” while swallowing. A distal
esophageal biopsy showed 10 eosino- phils/HPF. The EGD was otherwise
endoscopically and histologically normal, which included a total of 6 esophageal
biopsies. What is the most appropriate next step:

A. Start oral fluticasone.


B. Start proton pump inhibitor therapy
C. Start elemental diet
D. Refer to an allergist

8. You have diagnosed a 1 year old child with eosinophilic esophagitis. All of the
following are treat- ment options except:

A. Oral fluticasone.
B. Directed food elimination diet based on food allergy testing (skin prick and patch testing)
C. 6-food elimination diet (eliminating milk, soy, egg, wheat, peanut, and fish/shellfish)
D. Elemental diet
E. Lactose-free diet

9. A 10 year old African-American female presents with complaints of several months


of intermittent symptoms including trouble keeping eyelids open, inability to brush her
hair, and trouble getting out of chairs at school. Her speech is sometimes slurred. She
4

complains of double vision occasionally. Her symptoms are usually worse in the
evening after school. On exam, she has bilateral ptosis. When asked to raise both
extended arms over her head, she can raise them only 3-4 times before tiring. On
laboratory evaluation, she is acetylcholinesterase receptor antibody positive. Which of
the following structures is most likely to be affected?

A. Duodenal villi chloride channels


B. Colonic motility
C. Bile canaliculi
D. Upper esophageal sphincter
E. Pancreatic duct
5

10. The ER calls you at 7 PM to see a 2 year old who swallowed an unknown quantity of
vanilla scented hair relaxer. You ask about the presence of facial or oral lesions and
you are told none are evident but the patient is not fully cooperative for a complete
exam. You know the endoscopy suite is only on emergency status so you

A. Request the ER attending to call the ENT service.


B. Request the ER attending to notify the endoscopy suite for an emergent
study.
C. Proceed to the ER for your own assessment and finding no lesions or
respiratory distress, you recommend sending the patient home on bismuth
subsalicylate to return for F/U in one week.
D. Proceed to the ER for your own assessment and finding no lesions you or
respiratory distress, you recommend symptomatic treatment and endoscopy
the following morning

11. The patient with achalasia may have:

A. Incomplete relaxation of the lower esophageal sphincter


B. Ineffective peristalsis
C. Absent peristalsis
D. Dysphagic chest pain
E. A and C
F. All of the above

12. Bloody emesis in a 2 day old healthy full term neonate is likely to be secondary to:

A. Mallory-Weiss tear
B. Esophageal varices
C. Foreign body aspiration
D. Swallowed maternal blood

13. Which of the following statements is true regarding reflux:

A. Thickening formula reduces reflux episodes


B. Proton pump inhibitors have been found to improve infant irritability
C. Treatment with PPI’s for three months is indicated in patients with
endoscopically proven reflux esophagitis
D. Acute life threatening events have definitively been linked to gastroesophageal
reflux disease
E. Erythromycin has been proven to be beneficial in patients with GERD

14. Nissen fundoplication is indicated for all except:

A. Institutionalization
B. Intractable pain
6

C. Recurrent bleeding
D. Recurrent aspirations
E. Neurological impairment

15. ENT complications of GERD may include all of the following except:

A. Sinusitis
B. Otalgia
C. Laryngitis hoarseness
D. Glue ear
E. Recurrent epistaxis
7

16. An older sibling finds his 18m/o brother has taken apart a small non-functioning LED
flashlight and the lithium battery is missing. He informs his parents who then bring
the toddler to the ER. The ER attending calls and informs you that by x-ray the
battery in the stomach. You proceed to the ER, review the history, examine the
child and find he is in no distress. You then:

A. Call the endoscopy suite to set up for immediate removal


B. Tell the parents since the battery size is <10mm there is no need for concern or follow-
up
C. Tell the parents a “dead” battery will not cause tissue damage
D. Arrange to follow the course of the battery with daily radiographs
E. Discharge the patient for F/U in one week but ask the parents to notify you if
pain or vomit- ing evolves and to examine diaper stools for the battery over
the next 2-4 days

17. A one week old male infant has crying after feeds that last 2 hours. He spits up and
often calms down after passing gas. He stools after each feed. He takes a standard
cows’ milk formula. Mother recently noted small flecks of blood in the stools.
The most likely etiology is

A. Malrotation
B. Pyloric stenosis
C. Hirschsprung’s Disease
D. Milk-protein intolerance
E. Mild ulcerative colitis

18. Which statement is false?

A. E. histolytica infections are asymptomatic in 90% of patients.


B. E. histolytica liver abscesses tend to occur only in
those children who develop severe dysentery
C. G. lamblia can be zoonotic.
D. B. hominis produces diarrhea, bloating and eosinophilia
E. G. lamblia is predominantly contracted through water.

19. Which statement is false?

A. Salmonella infections can result from contaminated eggs, chicken, salads and cheese.
B. Campylobacter and Shigella sp infection can be very similar in presentation.
C. Yersinia infection of the terminal ileum can mimic appendicitis
D. Bacillus cereus constitutes a major component of probiotic therapy
E. Bacillus bifidum and Streptococcus thermophilus constitutes major
components of probiotic therapy

20. Which is a common association found amongst gastric polyps?


8

A. Peutz-Jeghers syndrome and juvenile polyps


B. H. pylori infection and hyperplastic polyps
C. Juvenile polyposis syndrome and fundic gland polyps
D. Familial adenomatous polyposis syndrome and hamartomas.

21. With respect to gastric tumors in childhood, which statement is false?

A. Fundic gland polyps associated with familial adenomatous polyposis


may undergo malignant transformation
B. Fundic gland polyps associated with long-term PPI use rarely appear before six years
C. Fundic gland polyps associated with long-term PPI require
surveillance for malignant transformation
D. Nearly all patients with Peutz-Jeghers syndrome require surveillance for gastric
hamartomas
E. Gastric teratomas with fetal elements occur exclusively in females
9

22. During a fraternity initiation, a 18y/o is forced to swallowed two live minnows.
Three days later he presents to the ER with severe abdominal cramps, nausea and
blood tinged vomitus. Physical exami- nation reveals diffuse abdominal tenderness.
You decide to consult a surgeon because you suspect

A. A perforated duodenal ulcer


B. Outlet obstruction by a minnow
C. Gastritis from schistosomiasis
D. Gastritis and perforation by Eustrongylides

23. Regarding H pylori which statement is true?

A. The natural reservoir for H pylori is zoonotic


B. Once gastritis becomes chronic it is usually irreversible
C. 90% of children with duodenal ulcer have antral H pylori infection
D. Approximately 15-20% of children with GERD response to H pylori
eradication
C. H Pylori

24. Regarding H pylori infection which statement is false?

A. Patients with MALT lymphoma experience regression of the


tumor with eradication of H pylori
B. H pylori gastric “cobblestoning” is more common in children
C. Biopsy for H pylori is best obtained from the antrum
D. H pylori grows best on chocolate agar medium

25. The metabolic disturbance most typical of pyloric stenosis is:

A. Hypochloremic acidosis
B. Hyperchloremic acidosis
C. Hypochloremic alkalosis
D. Hyperchloremic alkalosis

26. Which object(s) is LEAST likely to require endoscopic removal from the stomach?

A. 8 cm metal rod
B. A toy with known lead paint
C. 4 magnet balls
D. A nickel from a 9 year old boy

27. Which items is MOST likely to need endoscopic removal from the stomach?

A. A 3 cm-long pen cap in a 14 year old boy


B. A closed safety pin
10

C. A quarter ingested 4 weeks ago.


D. A 1 cm long toy piece

28. Which of the following is matched with its major site of injury?

A. Acid ingestion  stomach


B. Alkali ingestion  stomach
C. Doxycycline  duodenal bulb
D. Ibuprofen  colon
11

29. An 8 year old female comes to the ER with 12 hours of abdominal pain, vomiting,
and low grade temperature of 100.2. On exam, she has diffuse tenderness with
guarding in the right lower quad- rant and rebound tenderness. The next best
step in management is:

A. Oder a CBC, blood culture, and urine culture


B. Order an abdominal CT without contrast
C. Order a surgical consult
D. Order a pelvic ultrasound

30. A 14 year old male has had diarrhea and vomiting for 3 days with fevers up to 102.
Today he has right-sided lower abdominal pain with rebound tenderness. Laparoscopy
only shows mild periappen- diceal involvement. Which of the following is most
likely to yield the correct etiology:

A. Blood culture for gram negative sepsis


B. Stool culture for yersinia
C. WBC to look for immunosuppression
D. Colonoscopy for inflammatory bowel disease

31. 3 month old male infant is referred to you because of secretory diarrhea and
hypoglycemia. His work up included normal CBC, and normal Immunoglobulin
levels except for elevated IgE. You are sus- pecting autoimmune enteritis (AIE) as a
diagnosis. Which one of the following is true about AIE?

A. IPEX usually presents later in childhood.


B. It only affects boys.
C. Histology is significant for intraepithelial T cell infiltration.
D. Patients usually have normal lymphocyte count.
E. Only the IPEX form has extra-intestinal manifestations.

32. In you discussion with the parents of the above patient, which of which statement is correct?

A. Most of these cases will resolve by adolescence.


B. Diarrhea will decrease with bowel rest.
C. Immune suppression with monotherapy is effective.
D. BMT helps control diabetes and eczema in patients with AIE.

33. Appendicitis is usually characterized by periumbilical pain which moves to the RLQ
within the first 12-24 hours. In what situation may a patient not experience the
“classic” RLQ pain associated with appendicitis?

A. A long appendix
B. A fecalith impacted appendix
C. A perforated appendix
12

D. A retrocecal appendix.

34. Which of these would be most likely to cause pain in the LLQ?
A. Sigmoid volvulus
B. Pancreatitis
C. Acute Cholecystitis
D. Peptic ulcer

35. A full-term neonate has nonbilious emesis after each feed since birth. An abdominal
radiograph shows a dilated stomach and you suspect possible gastric outlet
obstruction. The next diagnostic test performed should be:

A. Nuclear medicine gastric emptying study


B. Barium contrast upper gastrointestinal series
C. Magnetic resonance imaging of the abdomen
D. Nuclear medicine biliary scan
E. Left lateral decubitus radiograph
13

36. A 2 year old male undergoes an abdominal ultrasound to evaluate the kidneys after
an abnormal urinalysis is discovered. A gastric duplication cyst is an incidental
finding on the ultrasound. The cyst is not large enough to cause compression and the
child has no vomiting. The recommended timing of treatment is:

A. Emergent surgery the same day


B. Surgical excision soon after the lesion is discovered
C. Surgical excision after 5 years of age
D. Surgical excision during adolescence
E. No surgical excision because the child is asymptomatic

37. A 14 m/o male presents with a three month history of chronic diarrhea, anorexia and
a fall from the 75th to the 25th percentile in weight. His height however remains on
track. There is no vomiting, but he does have increased foul and rancid flatulence and
hydrogen sulfide eructations. This patient merits

A. Serum tTG antibodies screening


B. IgA/IgG antigliadin antibodies screening
C. Stool analysis for giardiasis
D. DQ2-DQ8 screening
E. A trial of a milk-free diet

38. With respect to celiac disease which statement is false?

A. It is the most common non-surgical cause of asplenia


B. It can be associated with Down syndrome and idiopathic pericarditis
C. The biopsy reveals intraepithelial lymphocytosis
D. African-Americans are at increased risk if they are lactose intolerant
E. Up to 6% of patients with irritable bowel syndrome turn out to have celiac
disease.

39. Most duplication cysts arise in:

A. Stomach
B. Colon
C. Ileocecal region
D. Jejunum

40. Most duplication cysts present in:

A. Early adulthood
B. Puberty
C. Neonatal period
D. Before age 2
14

41. The best treatment option for enteric duplication cysts is:

A. Percutaneous drainage
B. Medical management
C. Surgical excision
D. Establish communication with main intestinal lumen
15

42. Which of the following neonates has the greatest risk potential of
developing necrotizing Enterocolitis?

A. Stable 1 day old, full term, female infant with no significant medical
findings consuming breast milk enterally.
B. 2 week old, 32 week gestation, male infant with a birth weight of 1200 grams
on hyperos- molar formula.
C. 3 week old, large for gestational age full term male infant of a diabetic mother.
D. 3 week old, 36 week gestation, male infant with birth weight of 2000 grams
and a menin- gomyelocele

43. A 10 day old, ex- 28 week premature female is confirmed to have Stage IIB NEC.
You would expect this infant to display all of the following signs and
symptoms except:

A. Temperature instability, lethargy and abdominal distension


B. Pneumatosis intestinalis on abdominal radiograph
C. Severely perforated bowel
D. Thrombocytopenia, diminished bowel sounds and grossly bloody stool

44. Most patients with malrotation present:

A. After one year of age


B. In the first month of life
C. In adolescence
D. Immediately after birth

45. The best modality for diagnosis of malrotation in a child is:

A. Barium enema
B. Upper GI series
C. Abdominal Ultrasoound
D. Upper endoscopy

46. On an UGI series the location of the duodenal-jejunal flexure ( Ligament of Treitz) is found:

A. Upper right of the spine


B. Upper left of the spine
C. Midline
D. Lower left of the spine

47. You will be following a patient who is s/p successful surgical repair of a
gastroschisis. Your long term concerns will be for:

A. Evolving intestinal dysmotility


16

B. Cantrell pentalogy defects


C) Development of GERD
D) All of the above
E) A and C

48. Newborns with surgically corrected omphalocele or gastroschisis are both at risk for:

A. Adhesions
B. GER and dysmotility
C. Atresias
D. Bowel ischemia
E. All of the above
17

49. Which of the following laboratory findings is NOT likely to be found in a


patient presenting with Small Bowel Bacterial Overgrowth?

A. Elevated D-lactate
B. Macrocytic Anemia
C. Microcytic Anemia
D. Elevated Stool pH
E. Hypocalcemia

50. Which of the following pairings of anatomic location and bacterial concentration is
INCORRECT?

A. Colon: <1011 – 1012 CFU/mL


B. Ileum: <108-1010 CFU/mL
C. Duodenum and Proximal Small Bowel: <106-108 CFU/mL
D. Stomach: <103 CFU/mL

51. A 4 year old male presented to the ED with a 5d h/o diarrhea, which became
bloody for the past 2 days. He was admitted overnight. Labs on admission include
WBC 17,000 Hgb 12.5, Plts 195; Na 142, K 4, Creat 1.5 BUN 30. Which of the
following organisms is most likely?

A. Yersinia
B. Toxigenic E. Coli
C. Norwalk-like virus
D. C. difficile
E. E. Coli O157:H7

52. A 2yo girl, who attends daycare, is brought to your outpatient clinic with a 3 day
history of watery, nonbloody diarrhea. Mom reports that she had low-grade fever,
not checked; since yesterday the stools have become bloody. She has not received
any recent antibiotic therapy. On exam the pa- tient’s vital signs reveal temperature
40 deg C, blood pressure 85/63, pulse 110, respiratory rate 20, oxygen saturations
100% room air; she has dry mucous membranes; stool testing reveals multiple
leukocytes. The most likely cause of this girl’s illness is:

A. Salmonella typhi
B. C difficile
C. Enteroinvasive E coli
D. Shigella
E. Giardia

53. What is the most common cause of diarrhea in children worldwide?

A. Rotavirus
18

B. Norwalk virus
C. Enterotoxigenic E. Coli
D. Salmonella
E. Shigella

54. You were called to evaluate a 48-hr-old male in the newborn nursery for a 12 hr
history of bilious emesis. The patient was born full-term vaginally without
complication. A prenatal ultrasound exami- nation revealed polyhydramnios during
the third trimester. On physical exam, the patient is mildly jaundiced, with a slight
abdominal distension and hypoactive bowel sounds. There is no abdominal
tenderness, respiratory distress, or signs of dehydration. He passed meconium 24 hrs
after delivery. You order an abdominal radiograph that shows dilation of the stomach
and proximal duodenum and absence of distal gas. The most appropriate next study
for this newborn is:

A. Fetal Karyotyping
B. Upper gastrointestinal radiograph
C. Echocardiography
D. Renal Ultrasound
E. Abdominal CT scan
19

55. Which of the following statements about tropical sprue (TS) are true?

A. Tropical sprue (TS) is seen only in visitors to the tropics


B. TS does not occur in epidemic form
C. TS has a uniform geographical distribution in all tropical regions
D. Patients can present with nutritional deficiencies in the absence of diarrhea.

56. Which of the following statements about TS are true?

A. Bacterial colonization of the small intestine is rare in patients with TS


B. Colonic absorption of water remains unaffected in TS
C. Morphological changes in the mucosa in patients with TS resemble those of celiac
disease
D. Small bowel transit is reduced in TS.

57. Which of the following is true regarding Clostridium difficile?

A. It’s presence always indicates active infection


B. Primarily causes watery diarrhea
C. Prevalence increases with age
D. Retesting should be done after 2 weeks of treatment

58. Which of the following causes of colitis presents as focal lesions without surrounding
inflammation

A. Crohn’s colitis
B. Microscopic colitis
C. Eosinophilic colitis
D. Behçet ‘s disease

59. Of the following, which best describes graft vs. host disease of the gut?

A. Exclusively involved the upper gastrointestinal tract


B. Significant involvement of the lamina propria
C. Apoptosis is commonly seen
D. Precipitated by food or drug allergies

60. A 14 year old female presents with lower abdominal pain for the past 4 months,
diarrhea, weight loss and intermittent fevers. Blood work shows a hematocrit of
10.2, mean cell volume of 65%, platelet count of 525, and sedimentation rate of 45.
Colonoscopy reveals moderate chronic, active colitis and ileitis with granuloma. You
decide to begin medical therapy for Crohn’s disease. A week later, her mother calls
and informs you she has developed pain in her legs. Which of the following
medications is most likely the cause of her new symptom?
20

A. Prednisone
B. Mesalamine
C. Metronidazole
D. Infliximab
E. Lactobacillus

61. An 8 year old male presents with chronic diarrhea and abdominal pain waking him
from sleep at night. His school performance has been declining due to frequent
absences and inability to concen- trate. Upper intestinal endoscopy and colonoscopy
reveal active esophagitis and linear ulcerations in the colon. The most common
extraintestinal manifestation of this disorder is which of the following:

A. Iritis
B. Erythema nodosum
C. Arthritis
D. Arthralgia
E. Aphthous stomatitis
21

62. Which of the following medications for ulcerative colitis works by inhibition of
prostaglandin and leukotriene synthesis?

A. Azathioprine
B. Mesalamine
C. Tacrolimus
D. Infliximab
E. Cyclosporin

63. A 14 year old male comes to your office with 6 weeks of persistent diarrhea
containing streaks of blood and crampy lower abdominal pain. Perianal inspection
reveals no lesions, and occult blood testing confirms the presence of blood. His
height is at the 45 percentile for his age. You suspect ulcerative colitis. Which of the
following findings on colonoscopy would most strongly support this diagnosis?

A. Inflammatory pseudopolyps
B. Areas of normal colon mucosa between inflamed regions
C. Inflammation of the terminal ileum
D. Nodularity of the colon mucosa.
E. Linear ulcerations

64. A child with Hirschsprung’s disease would have the following finding on anorectal
manometry after rectal dilation with the balloon:

A. Increased internal sphincter tone and decreased external sphincter tone.


B. Increased internal sphincter tone and increased external sphincter tone.
C. Decreased internal sphincter tone and decreased external sphincter tone.
D. Decreased internal sphincter tone and increased external sphincter tone.

65. The most common cause of constipation in childhood is:

A. Celiac disease
B. Inflammatory bowel disease
C. Hirschsprung disease
D. Functional constipation
E. Cystic fibrosis

66. A 4 year old otherwise healthy male has large-caliber, painful bowel movements,
which occur every 5-7 days. On physical exam, there is a hard palpable mass in the
left lower quadrant of his abdo- men. His exam is otherwise normal. He is taking
no medications. Which of the following is the most appropriate next step?

A. Obtain an abdominal x-ray to confirm presence of fecal impaction


B. Start this patient on daily maintenance oral laxative
C. Treat fecal impaction with oral and/or rectal therapies, followed by a
22

maintenance regimen and close followup.


D. Encourage increased fiber in his diet and close followup.
E. Order a barium enema.

67. Encopresis typically occurs when:

A. The external anal sphincter is no longer able to function to prevent defecation


B. The internal anal sphincter is no longer able to function to prevent defecation
C. The puborectalis is no longer able to function to prevent defecation
D. The external anal sphincter is contracted.
E. The anal canal is lengthened.
23

68. Which of the following is the correct statement about hemorrhoids?

A. Internal hemorrhoids cause painful bleeding.


B. Hemorrhoids develop later in life due to chronic straining.
C. Bleeding from hemorrhoids can cause anemia.
D. Patients with portal hypertension have higher risk of developing hemorrhoids.

69. Which statement is incorrect about the management of Hemorrhoids?

A. Acute thrombosed external hemorrhoids can be excised within 24-48 hr.


B. High fiber diet helps shrink hemorrhoids and decrease bleeding.
C. Sclerotherapy is more effective than rubber banding.
D. 5-10 % of patients with hemorrhoids will need surgical treatment.

70. Which of the following statements regarding Hirschsprung disease is TRUE?

A. HD is commonly associated with other congenital malformations.


B. Presence of recto-anal inhibitory reflex is characteristic.
C. Early diagnosis is important to avoid development of enterocolitis and
subsequent toxic megacolon.
D. Surgery for HD is usually performed between 2 and 3 years of age.

71. Which of the following is a common manifestation of HSP?

A. Painful palpable purpura


B. Colicky abdominal pain and occult GI bleed
C. Arthritis of large joints in the arms
D. Proteinuria

72. What of the following is part of the pathophysiology of E coli in HUS?

A. It produces Shiga toxin


B. It produces cholera toxin
C. It produces C diff toxin
D. It is enteroinvasive and ulcerogenic

73. Which of the following is the appropriate diet for an infant with primary intestinal
lymphangiectasia?

A. Low protein and low fat diet


B. Low protein and high fat diet
C. High protein and low fat diet
D. High protein and high fat diet

74. You follow a 6 y/o boy with juvenile polyposis coli. Part of your care involves
24

A. Screening all at risk family members


B. Colonoscopy performed annually with polypectomies PRN
C. Gastroscopy of the index patient beginning at adolescence
D. Only B and C
E. A, B and C

75. A 9 year old girl with presumed Peutz Jeghers syndrome lacks the STK11 mutation
on chromosome number 11. In this patient:

A. The risk of malignancy is greatest in the small bowel over the colorectal area
B. There is no risk for precocity
C. Her risk for breast tumor is under 25%
D. All statements are true
E. All statements are false
25

76. 12 month old girl present with her third episode of rectal prolapse during the last 2
months. No constipation reported by the parents but was prescribed lactulose in
the last month with a good response. Physical examination normal.

Which investigation should be the next step?

A. colonoscopy
B. barium enema
C. sweat test
D. abdominal series

77. An 8 month-old male infant presents to your clinic with 6 weeks of crying with
stooling. The parents have noted small steaks of blood in the diaper after he stools.
His nutrition consists of breast milk and a variety of fruits, vegetables and cereal. He
has had no vomiting, signs of abdominal pain, fe- vers or diarrhea. He was passing
hard-consistency stools at the beginning of this course, but this has improved with
daily MiraLax prescribed by his primary physician. Currently, he is passing three softly
formed stools daily. On your physical exam, you note a fissure in the posterior midline,
an associated small non-inflamed skin tag and hypertonicity of the anal sphincter. In
addition to careful hygiene, what is the best management approach?

A. As 0.2% nitric oxide preparation topically three times daily


B. Daily anal dilatation at home
C. Cow’s milk restriction
D. Increased dietary fiber
E. Lateral internal sphincterotomy

78. A 2-year old female has had a three week history of decreased stooling frequency.
Her stools have become hard in consistency and require straining. Her parents have
not noted blood in her stools, but are concerned with the amount of discomfort she
has with defecation. She has become ex- tremely apprehensive with diaper changes
and is exquisitely tender to the touch in the diaper area. On your physical exam, you
note a well-demarcated and moist area of erythema in her perineum, radiating from
the anus without induration. She has not had a similar perineal rash in the past. What
is the most appropriate diagnostic test?

A. Direct GABHS antigen


B. ASLO or anti-DNase B
C. Perianal swab culture
D. Sigmoidoscopy with biopsies
E. Stool ova & parasites

79. A previously healthy 7-year-old male was diagnosed with a perirectal abscess by his
primary physi- cian. Initial management of oral antibiotics and sitz baths has been
initiated. Which of the following is considered an indication for surgical
26

management?

A. Persistent perineal pain despite antibiotics


B. Persistent drainage despite 3 months of medical management
C. Underlying inflammatory bowel disease
D. The presence of fistula in ano at initial presentation
E. Current immunosuppressive medications for JIA
27

80. A 2-year-old female presents with distended abdomen, feeding intolerance, vomiting,
and chronic severe constipation since infancy. One year ago she had been at the
40% weight for age on the CDC growth curves and she is now on the 10% weight
for age. Past medical history is significant for multiple urinary tract infections which
have required antibiotic prophylaxis. Previous abdominal ultrasound showed
megaureters and hydronephrosis. An abdominal radiograph is notable for a
distended small bowel with multiple air fluid levels and stool-filled colon. Previous
home clean-outs prescribed with osmotic and stimulant laxatives have not helped to
resolve her symptoms. She has no abnormalities in thyroid studies, metabolic
problems, celiac testing, or hematocrit. She had a previ- ous normal anorectal
manometry. Exam in your office was notable for distended abdomen which is
tympanic to percussion with mild abdominal pain on palpation throughout. Rectal
exam was normal without stool mass noted.

What next study would give the most diagnostic yield?

A. Upper gastrointestinal contrast study with small bowel follow through


B. Radio-opaque marker study
C. Colonoscopy with biopsies for histopathology
D. Antroduodenal and colonic manometry

81. The hepatic lobule is centered on

A. Bile duct
B. Central vein
C. Portal vein
D. Hepatic artery

82. You follow an eleven year old boy with ulcerative colitis and he comes in for a
routine evaluation. He claims his symptoms have been well controlled with
mesalamine and at this visit he says his stools are formed with no blood, but he has
had new tenderness in his abdomen, occasional nausea, new
fatigue and itching. He denies recent trauma, fever or new medications. On physical
examination you note mild hepatomegaly and tenderness, and subtle conjunctival
icterus. You are concerned he may have acquired hepatitis or is evolving primary
sclerosis cholangitis. You order all the following studies except one.

A. Hepatitis serologies
B. JAG-1 and NOTCH-2 mutational studies
C. Fresh liver function studies
D. Blood cultures
E. MRCP

83. Findings in a patient with Alagille syndrome may include all of the following findings except
which?
28

A. Posterior embryotoxon
B. Hemivertebra
C. Periductal hyperplasia
D. Peripheral pulmonary stenosis
E. Portal tracts with bile duct ratio of less than 0.5
29

84. During his routine one month check-up a four week old African American male is
found to have woody hepatomegaly and a conjunctiva suggestive of icterus. His
mother is concerned he is not gain- ing weight and that his suck is poor. Your initial
studies reveal conjugated hyperbilirubinemia, moder- ate increase of transaminases
but a 4-fold increase of GGT. You order an ultrasound which reveals a small
gallbladder but the sonographer is unable to measure the hepatic ducts. Hepatic
scintography shows normal uptake but delayed and diminished excretion at 24 hours.
Your next step is to order

A. A repeat ultrasound to locate the “triangular cord sign.”


B. Schedule a liver biopsy
C. Repeat scintography with a double dose of Phenobarbital
D. Order serologies for CMV, HBV, HCV and Cocksackie B
E. Order a bone marrow for hemophagocytosis

85. You are following a five month old male who is status post Kasai. His liver is firm
but not hard and he has no splenomegaly. He is slowly gaining weight and his
nutrition appears adequate. There is no diarrhea but both his stools and urine are dark.
He is on ADEK supplementation and ursodeoxycholic acid. At this visit, his total
bilirubin is 6.4 mg/dL and his stools are guaiac positive. You should at this point:

A. Schedule endoscopy for possible varices


B. Increase his dose of ADEK and ursodeoxycholic acid
C. Change his antibiotics for possible ascending cholangitis
D. Request a HIDA scan to assess Kasai function
E. Refer the child to the transplant team for preliminary assessment and
registration.

86. A 16-year-old female has a 6-month history of intermittent epigastric and right upper
quadrant ab- dominal pain. Her BMI is 35. The pain is often worse after fatty meals
and it has not improved with 8 weeks of appropriate proton pump inhibitor therapy.
Laboratory testing for pancreatitis, hepatitis, liver function and celiac disease have
been unrevealing. Her WBC is normal. An abdominal ultra- sound with focus in the
right upper quadrant showed a normal appearing gallbladder without any evidence
of gallstones or sludge. The liver parenchyma and pancreas appeared normal. The
com- mon bile duct diameter was 3mm. You suspect that she may suffer from
chronic acalculous chole- cystitis/biliary dyskinesia. Which of the following is the
best next step in establishing that diagnosis?

A. Magnetic resonance cholangiopancreatography (MRCP)


B. Endoscopic retrograde cholangiopancreatography (ERCP) with sphincter of
Oddi manometry
C. Abdominal CT scan
D. Upper endoscopy with bile sampling for analysis
E. HIDA scan with fatty meal stimulation
30

87. The risk of childhood cholelithiasis is increased in all these circumstances except:

A. Cystic fibrosis with the mutation of F508 or I507


B. Crohn’s disease limited to the terminal ileum
C. Total abdominal situs inversus
D. Thalassemia and hereditary spherocytosis
E. A pregnant Pima Indian adolescent
31

88. A 2-day-old otherwise healthy full-term male infant undergoes an abdominal


ultrasound for surveil- lance of pelviectasis initially diagnosed on a prenatal
ultrasound. The kidneys appear normal but the ultrasound reveals a 3mm, mobile
echogenic mass in the gallbladder consistent with a gallstone. No other gallbladder or
liver abnormalities are seen and there is no pericholecystic fluid noted. The child has a
transcutaneous bilirubin consistent with a value of 4.3 mg/dl and is feeding well.
What is the most appropriate next step in the management/evaluation of the
gallstone?

A. Start Ursodeoxycholic Acid therapy


B. Consult pediatric surgery for cholecystectomy
C. Order a magnetic resonance cholangiopancreatography (MRCP)
to assess for bile duct anatomy
D. No further evaluation if the gallstone does not cause symptoms
E. Technetium 99m–hepatic iminodiacetic acid (HIDA) scan

89. A 7-year-old boy with nephrotic syndrome who has been hospitalized for 3 weeks
for the manage- ment of edema develops intermittent right upper quadrant
abdominal pain. An abdominal ultra- sound shows the presence of a mobile, 6mm
stone in the gallbladder. Which of his medications is most likely to have played a
role in the development of the gallstone.

A. Hydralazine
B. Omeprazole
C. Furosemide
D. Lisinopril
E. Dalteparin

90. Which of the following is a risk factor for the development of black pigment gallstones?

A. Hereditary Spherocytosis
B. Obesity
C. Pregnancy
D. Bile infection
E. Hyperlipidemia

91. 17 year old male has ALF. He was well until 2 wks before admission, when jaundice
developed. One week later he was hospitalized because of progressive confusion. He
has slight asterixis. Labs: Hemo- globin 9.8, ALK PHOS 60, T bilirubin 40, D
bilirubin 12, UA 1.1, AST 300, ALT 170, INR 2.5. SMA and ANA neg.
Ceruloplasmin 24 (22-43). Which of the following is true?

A. Normal value for ceruloplasmin excludes Wilson


B. Slit-lamp examination should be done to look for KF rings
C. He is too ill to be considered for OLT
32

D. Liver Bx should be the next study

92. 20 year old female was admitted for “Liver Transplant”. Three months ago she
began gaining wt (16 lbs). Two mo ago she had dark urine and yellow skin. Tests
for HBsAg, HB core AB, HAV IgM, HCV AB, CMV IgM and IgM to VCA for
EBV were all negative. PE showed jaundice, shifting dullness,
hepatomegaly. LABS: AST 624, T Bilirubin 12, TP 8.5, Alb 2, INR 2, ANA 1:40.
Tests for SMA and LKM were neg. What is the next step in her treatment?

A. OLT
B. Observation for 3 mo
C. IFN and Ribavirin
D. Prednisone 60 mg daily
33

93. A 15 year old female is admitted to the ICU following intentional acetaminophen
overdose. She is unresponsive, mildly hypotensive and has no stigmata of chronic
liver disease. She is most likely to die of:

A. Cerebral herniation
B. Coagulopathy with bleeding
C. Liver synthetic failure
D. Renal failure

94. A 12 year old boy is seen for FTT, episodic irritability, lethargy, and refusal to eat
animal protein (milk, eggs, and meat). Which of the following abnormalities is most
characteristic of OTC deficiency?

A. Elevated ammonia
B. Elevated plasma citrulline
C. Metabolic acidosis
D. Aminotransaminase levels more than 1000

95. A 5 day old breastfed infant is admitted to the hospital because of lethargy and a
poor suck. The infant appears jaundiced. The total bilirubin is 12 with 25%
conjugated. The metabolic disease most likely to cause jaundice in this infant is:

A. A HFI
B. B Galactosemia
C. C Hypothyroidism
D. D PKU

96. A 2 month old infant who presents with a 1 wk h/o intermittent vomiting appears
jaundiced on PE. At 4 weeks of age the exclusively breastfed baby was gaining wt
well and was not icteric. The mother subsequently returned to work and the infant
has been receiving supplements of formula and apple juice. A urine test for
reducing substance is positive.

A. alpha-1-antitrypsin deficiency
B. Biliaryatresia
C. hereditary Fructose Intolerance
D. Cystic Fibrosis

97. Which of the following statements about pyogenic abscess of the liver is true?

A. The right lobe is more commonly involved than the left lobe.
B. Appendicitis with perforation and abscess is the most common
underlying cause of hepatic abscess.
C. Mortality is not determined by the underlying disease.
D. Mortality from hepatic abscess is currently greater than 80%.
34

98. You follow a 7 m/o cholestatic male with biopsy proven non-syndromic paucity of
bile ducts. At this visit the mother is pleased to report he is no longer scratching
himself and is gaining weight. Surveil- lance LFT reveal persistent cholestasis,
elevated AlkP, progressive increase in ALT/AST, but a progres- sive drop in GGT.
You next order

A. Urinary bile acid profile


B. Repeat liver sonography
C. Repeat liver biopsy
D. A followed by B if the profile is abnormal
E. A followed by C if the profile is abnormal
35

99. You receive a request for a second opinion on a 4 wk/o male with hepatomegaly,
increased LFTs with low GGT, evidence of steatorrhea, and a liver biopsy revealing
giant cell transformation, fibrosis and bile lakes. You suggest as a primary next
step to:

A. Seek a hematology consult to r/o a hemophagocytosis syndrome


B. Obtain a urinary bile acid profile
C. Begin an empiric trial of Phenobarbital at 3 mg/kg/day
D. Begin an empiric trial of ursodeoxycholic acid at 15 mg/kg/day
E. Seek a infectious disease consult to r/o enterovirus infection

100. A 5- month old baby presents to the ER with vomiting, diarrhea and poor weight
gain for the past month. He is a full term baby born to a healthy mother with no
prenatal or perinatal complications. There have been no sick contacts. No fevers,
rashes, or recent antibiotic use. He has been solely breastfed, and one month ago he
started eating jarred baby foods, but has not been taking them well. A deficiency in
which enzyme should be considered in this case?

A. Aldolase B
B. Galactokinase
C. Fumarylacetoacetate hydrolase
D. Glucose-6-phosphatase
D. Galactose-1-phosphate uridyl transferase

101. A 3-day old female develops a fever prior to discharge home. She undergoes a
complete sepsis evaluation including blood, urine, and CSF cultures. Urine and
blood cultures are both positive for E Coli. Which of the following should be
the initial next step for this patient?

A. Check thyroid studies


B. Obtain an abdominal ultrasound
C. Switch to a soy-based formula
D. Check PT, PTT, INR
E. Obtain chest x-ray

102. Which of the following chemotherapy medications is NOT associated with veno-occlusive
disease?

A. 6-thioguanine
B. Busulfan
C. Cytosine arabinoside
D. Dactinomycin
E. L-asparaginase

103. Ground glass cytoplasmic inclusions are typically seen in which types of drug-induced liver
36

injury?

A. Amiodarone
B. Isoniazid
C. Isotretinoin
D. Mycophenolate mofetil
E. Oral Contraceptives
37

104. A two-year old boy develops URI symptoms 2 days prior to admission. His
pediatrician diagnoses him with a viral pharyngitis. The boy drinks minimal fluids
over the next two days. On the day of admission, parents note that he is pale and
extremely sleepy. They drive him to the ER, and the boy has a seizure in the car on
the way there. In the ER, his serum glucose is 25 mg/dL. Blood gas re- veals pH
7.29, pCO2 31, and HCO3 of 15. Electrolytes reveal Na 131, K 4.4, and Cl 99.
Ammonia is elevated at 95. Urinalysis demonstrates no glucose, no protein, and 1+
ketones. The most likely diagnosis is:

A. Sepsis
B. Urea cycle defect
C. Congenital heart defect
D. Organic acidemia
E. Fatty acid oxidation disorder

105. A 27-year old woman at 33 weeks gestation presents to the emergency room with
nausea, vomiting, and abdominal pain. She is found to have a mild elevation in her
transaminases, and is subsequently admitted for IV fluid hydration and observation.
Over the next 3 days, she develops worsening elevation in her transaminases and a
coagulopathy, and progresses to fulminant liver failure. An emergency cesarean
section is scheduled and the baby is delivered. Both mom and baby go on to do well.
Screening for which of the following mutations should be considered for the
baby?

A. MCAD mutation
B. G1528C mutation
C. Trifunctional protein deficiency
D. SCAD mutation

106. A 16-year-old white female presented with a few weeks history of low-grade-fever
and arthritis. On physical exam, she was noted to have erythema nodosum on both
her shins, arthritis of her ankles, and hepatomegaly. Chest radiograph showed
bilateral hilar adenopathy. Liver biopsy revealed non- caseating granulomas mainly
in the portal tract. Skin tuberculin test was negative. What is the most likely
diagnosis:

A. Tuberculosis
B. Sarcoidosis
C. Coccidioidomycosis
D. Histoplasmosis

107. The oncology service consults you for a febrile 12 y/o with hepatomegaly,
splenomegaly, hyperbili- rubinemia, elevated alkaline phosphatase and leucocytosis
Ultrasound reveals “bull’s eye” lesions in the hepatic parenchyma most likely
caused by
38

A. Systemic candidiasis
B. Coccidiomycosis
C. Histoplasmosis
D. Coxiella infection

108. With regards to hepatic granuloma, which statement is false?

A. Up to 10% of all liver biopsies reveal granulomas


B. Brucellosis, Q-fever and Hansen’s disease may cause hepatic granulomas
C. Granulomas occur in about 10% of patients with miliary tuberculosis
D. Acid-fast bacilli can actually be seen in TB granulomas

109. All of the following are associated with NAFLD EXCEPT:

A. Increased TGF-beta
B. Increased adiponectin levels
C. Increased hepatic Fe stores
D. Increased reactive oxygen species
E. Increased free fatty acids
39

110. The most common area of presentation of PTLD in pediatric liver transplant recipients include;

A. The kidneys
B. The pancreas
C. The liver
D. The pharyngeal and cervical lymphatic chain

111. Which of the following statements are true regarding the outcome of liver transplantation in
children

A. Expected one year patient survival is approximately 99%


B. Chronic rejection results in graft loss in approximately 5% of children
C. Primary non-function has been virtually eliminated as a clinical problem
D. All of the above

112. Nephrotoxicity secondary to calcineurin inhibitors causes the following

A. Afferent arteriole vasoconstriction


B. Endothelial cell damage
C. Interstitial fibrosis
D. Tubular atrophy
E. All of the above

113. In the setting of acute liver failure it is appropriate to proceed with transplantation:

A. Before establishing the etiology of the liver injury


B. When patients exhibit clinical signs of stage II encephalopathy
C. When patients have clinical signs of irreversible neurologic injury
D. Only if some form of bridge therapy is not available
E. None of the above

114. Accepted contraindications to liver transplantation in children include all except the following:

A. Alpers syndrome
B. Cystic fibrosis
C. Advanced pulmonary hypertension
D. Uncontrolled systemic infection
E. The above are all accepted contraindications

115. As compared to patients receiving tacrolimus, patients receiving cyclosporine are less likely to:

A. Develop DM
B. Develop hypertension
C. Dyslipidemia
D. Seizures
40

116. Primary bile acid therapy with cholic acid is effective in which of these peroxisomal disorders?

A. Zellweger syndrome
B. Adrenoleukodystrophy (ALD)
C. Refsum disease
D. Methylacyl-CoA racemase deficiency
E. Rhizomelic chondrodysplasia punctata
41

117. An eight month old infant presents with cholestasis, poor growth and chronic
diarrhea. He had negative TORCH serologies, normal alpha 1 antitrypsin, and normal
metabolic screening. His labora- tory tests revealed GGT 23, Bilirubin 6.0/4.0 and
ALT 350. His most likely diagnosis is:

A. PFIC-1, FIC-1 disease


B. PFIC-2, BSEP disease
C. PFIC-3, MDR3 disease
D. BRIC

118. Match the finding or symptoms to the gene defect:

A. MDR3 disease 1. Bile duct proliferation


B. BSEP disease 2. Episodes of severe pruritus which resolve
C. FIC-1 disease 3. Associated with liver tumors
D. BRIC 4. Recurrent pancreatitis

119. The family of a child newly diagnosed with AGS asks you what is the risk of their
older child or future children also being affected with AGS. You advise them
that:

A. The majority of mutations in AGS occur for the first time in the identified
individual
B. If there is a family history then the chance is 50% of a sibling being
affected
C. The sibling could carry the same gene mutation as the newly diagnosed child
and yet have a very different range and severity of clinical symptoms
D. All of the above

120. A child with AGS is likely to have:

A. A lifetime risk of needing a Liver transplant of 90%


B. Poor growth
C. Severe itching
D. All of the above
E. B and C only

121. Graft-versus-host disease (GVHD) is caused when:

A. Host lymphocytes are depleted


B. Immunosuppressive therapy is increased
C. Donor lymphocytes attack host tissues
D. Host granulocytes attack donor tissues

122. After bone marrow transplantation, present in the graft, either as contaminants or
42

inten- tionally introduced into the host, attack the tissues of the transplant recipient
after perceiving host tissues as antigenically foreign.

A. T Cell
B. T Helper Cell
C. Cytotoxic T Cell
D. Regulatory T Cell

123. The progression of disease in GVHD goes in which order?

A. GI Tract  Liver  Skin


B. GI Tract  Skin  Liver
C. Liver  GI Tract  Skin
D. Liver  Skin  GI Tract
E. Skin  GI Tract  Liver
F. Skin  Liver  GI Tract
43

124. During an upper endoscopy where are you most likely to find
histologic changes associated with GVHD?

A. Esophagus
B. Stomach
C. Duodenum
D. Rectum
E. Liver

125. When looking at a histologic sample, what finding is most suggestive of GVHD?

A. Crypt abnormalities
B. Epithelial cell apoptosis
C. Focal fibrosis
D. Focal reactive surface epithelium
E. Lymphocytic infiltrate

126. A 14 year old boy is referred to you for conjunctival icterus. He feels well and has
no complaints. He volunteers that has noted his eyes “yellow” when he has a cold.
At the visit, there is no icterus evident, no visceromegaly and all his liver function
tests and hemogram are normal. You next:

A. Schedule a liver biopsy


B. Order an ultrasound
C. Order hepatitis serologies
D. Order a 10 day course of low dose phenobarbital
E. Request a screen from UGT1a1 mutation

127. A 15 year old boy with sickle cell disease presents with jaundice and fatigue.
Review of systems reveals intermittent dark urine and icteric conjunctiva over the last
month. Physical exam is unremark- able except for scleral conjunctiva. Differential
diagnosis includes the following except:

A. Hemolysis related to his sickle cell disease


B. Hepatocellular dysfunction
C. Obstruction due to biliary stones
D. Acute cholecystitis

128. A three month-old infant of Canadian descent presents with irritability, mild
jaundice and hepa- tomegaly. Labs are concerning for AST of 130, ALT 170, PT
28, INR 2.4, and serum glucose of 40. Which of the following tests would be most
helpful in confirming the underlying diagnosis?

A. CBC with differential


B. Urine succinylacetone
44

C. Factor 5 level
D. Serum alpha-fetoprotein level
E. Serum albumin

129. Which condition or syndrome is NOT associated with hepatoblastoma?

A. Beckwith-Wiedemann Syndrome
B. Familial Adenomatous Polyposis Syndrome
C. Glycogen Storage Disease 1a
D. Biliary Atresia
E. Cleft Palate
45

130. A 3-week old full term, otherwise healthy formula-fed infant presents with new-
onset poor feeding and fever to 38.3°C. Laboratory findings at presentation are
significant for leukocytosis, as well as a mild transaminase elevation (approximately
1.5 times upper limits of normal). Total bilirubin is el-
evated to 6.7 mg/dL with conjugated bilirubin of 2.3 mg/dL. A bacterial infection is
found on culture analysis. What is the most likely bacterial cause and route of
infection?

A. Staphylococcus aureus infection from skin


B. Streptococcus pyogenes infection from throat
C. Escherichia coli infection from urine
D. Listeria monocytogenes infection from blood

131. A 1-week old infant is noted to have fulminant liver failure with coagulopathy after
presenting with vesicular rash and seizures. Disseminated herpes simplex virus
infection is confirmed by nasopharyn- geal culture and PCR analysis of CSF.
Which of the following is true regarding this infection?

A. Parenteral acyclovir is indicated only in instances of disseminated disease


B. Liver transplantation is contraindicated due to the presence of active viral
infection.
C. Mortality rate is high even with acyclovir therapy
D. The majority of mothers of infants with disseminated HSV will have
symptomatic infection at time of delivery
E. Both C and D.

132. Which serologic profile best reflects the “immune tolerant” state of chronic Hepatitis
B infection?

A. HBsAg positive, HBsAb positive, HBeAg positive, HBeAb positive


B. HBsAg positive, HBsAb negative, HBeAg positive, HBeAb positive
C. HBsAg positive, HBsAb negative, HBeAg positive, HBeAb negative
D. HBsAg negative, HBsAb positive, HBeAg negative, HBeAb positive

133. Which of the following individuals should receive HBV immune globulin
therapy?

A. All infants and children


B. Asymptomatic children with chronic HBV infection
C. Post-HBV exposure, within 24 hours after exposure
D. All household contacts of HBV infected child

134. True or False: Serum Hepatitis C antibody (IgG) is protective and will prevent
recurrent infection upon re-exposure of HCV.
46

135. All of the following hepatitis viruses can result in a chronically infected state,
except:

A. HEV
B. HCV
C. HDV
D. HBV

136. 6 week old presents with neonatal jaundice, direct hyperbilirubinemia, markedly
elevated GGT and pale-colored stools. Possible diagnoses include all of the
following, except:

A. Biliary atresia
B. PFIC2
C. PFIC3
D. A1AT deficiency
47

137. Cholestatic infants should be monitored for deficiencies in many vitamins, including:

A. Vitamin K
B. Folic acid
C. Vitamin A
D. Iron
E. A and C
F. All of the above

138. Which of the following is true regarding autoimmune hepatitis?

A. Children represent a very small subset of patients diagnosed with autoimmune hepatitis
B. Acute hepatitis is the most common presentation
C. Giant cell transformation is a classic histological feature
D. In children, the incidence is equal in males and females
E. HLA A2 is a common association.

139. Patients with Type 1 autoimmune hepatitis:

A. Typically exhibit anti-perinuclear antibody (pANCA)


B. Present at an earlier age than do patients with Type 2 disease
C. Do not have other autoimmune disorders
D. Typically progress to liver transplant in 5-8 years
E. Often demonstrate mildly abnormal cholangiograms

140. An 8 year old girl is being treated for Type 2 AIH with a standard
prednisone/azathioprine regimen. She responded readily with ALT falling from a
high of 650 IU/l consistently. A slow prednisone taper is in progress. With the most
recent dosage decrease, ALT was noted to jump from 75 to 180 with 6-MP
metabolites in the normal range. The appropriate response should be:

A. This is a treatment failure. Consider addition of CSA


B. Consider overlap syndrome and order MRCP
C. Stop current regimen and start MMF
D. Refer for transplantation
E. Increase prednisone temporarily and resume taper when labs improve

141. Which of the following arteries contributes to the blood supply of the pancreas?

A. Left gastric artery


B. Right gastric artery
C. Proper hepatic artery
D. Superior mesenteric artery
E. Inferior mesenteric artery
48

142. The major mediator of meal-stimulated pancreatic enzyme secretion is:

A. Secretin
B. Cholecystokinin
C. Pancreatic polypeptide
D. Peptide YY
E. Somatostatin
49

143. A ten-year-old male presents to the emergency room with a one day history of
vomiting, decreased appetite, and abdominal pain after falling off his bicycle the
day before. Labs reveal an elevated lipase of 3500 and abdominal ultrasound reveals
peripancreatic fluid and pancreatic fullness. Patient is admitted with a diagnosis of
acute pancreatitis. He is initially managed with bowel rest, IV fluids, and IV pain
medications. Within 4 days, he is no longer requiring IV pain medications, is
tolerating a regular diet, and his lipase has decreased to 485. He is subsequently
discharged home. Two weeks later at his follow-up visit, he complains of mild to
moderate diffuse abdominal pain and decreased appetite. He is noted to have
epigastric fullness on physical exam. Lipase is rechecked and is now 1205. Which
of the following steps should be taken next in the evaluation of this patient?

A. Follow serum lipase on a daily basis for the next week


B. Repeat abdominal ultrasound
C. Admit the patient for bowel rest and IV fluids
D. Send genetic testing for CFTR mutations
E. Schedule for an upper endoscopy

144. A 2-year old male presents with a cough and diarrhea. Weight is below the 5th
percentile and height is at the 10th percentile on the CDC growth curve. His
mother states his stool is very foul- smelling. Stool culture for bacteria and
examination for ova and parasites are negative. The parents want the fecal elastase test
performed to rule out pancreatic insufficiency. Which statement is correct to tell the
parents?

A. No sweat test is necessary in this child.


B. A 72-hour fecal fat test is easy to perform.
C. Fecal trypsin testing is more specific than fecal elastase testing
D. Fecal elastase can be artificially lowered if the child has short bowel
syndrome.

145. You are giving a medical student lecture regarding the embryology of pancreatic
development. What statement is correct to present to the students?

A. The ventral and dorsal pancreatic segments fuse at the 4th week of
gestation.
B. Pancreatic function occurs at the 12th week of gestation.
C. Sonic hedgehog protein is the hedgehog protein necessary for
pancreatic cellular differentiation.
D. The dorsal aspect of the pancreatic segment contains the
connection to the common bile duct.

146. A 2 year old female comes to your office with failure to thrive, steatorrhea, small
teeth, nasal ab- normalities, and microcephaly. An outside lab test reveals
50

hypothyroidism. You suspect pancreatic insufficiency due to what diagnosis?

A. Johansson-Blizzard syndrome
B. Homozygous F508 mutation cystic fibrosis
C. Shwachman-Diamond syndrome
D. Congenital Rubella

147. Pancreas divisum:

A. Results from lack of fusion of the ventral and dorsal pancreatic ducts
B. Is easily diagnosed with ultrasound
C. Is associated with ectopic pancreatic tissue
D. Is treated with duodenoduodenostomy

148. Annular pancreas

A. Is best diagnosed by ultrasound


B. Results from the failure of atrophy of the ventral bud
C. Is not associated with congenital malformations
D. Occurs at the third portion of the duodenum
51

149. Which of the following statements regarding Shwachman-Diamond Syndrome is TRUE?

A. Exocrine pancreatic insufficiency is a universal manifestation in all patients with SDS


B. Exocrine pancreatic insufficiency in SDS arises from ductular inflammation
C. Few SDS patients exhibit intermittent neutropenia
D. SDS adolescents commonly have hepatomegaly

150. How would you assess for vitamin D deficiency in a child in whom you suspected
inadequate intake/ inadequate sun exposure?

A. Serum 25 (OH) D level


B. Serum phosphorus
C. Serum 1, 25 (OH) D
D. Serum calcium
E. Urinary 1, 25 dihydroxy D

151. A 4-year-old child with biliary atresia, status post Kasai, presents with a direct
bilirubin of 3.0, an ALT of 230, AST of 340 and GGT of 850. Recently his mother
noticed that he was walking ‘funny’. On examination he is jaundiced, has a large
spleen and liver. You note that his gait is wide and irregular. What vitamin
deficiency is the most likely cause of his ‘funny’ walking?

A. Vitamin A
B. Vitamin D
C. Vitamin E
D. Vitamin K
E. Carnitine

152. You are seeing a family who just moved to the US from Greenland. A family
brings in a 1 year old infant who has failure to thrive, diarrhea, and abdominal
distension. The baby has been breastfed ex- clusively till 6 months of age when solid
foods were introduced. Between 6 and 12 months of age, the weight decreased from
the 50th to the 5th percentile, while the height remained at the 50th per- centile. On
physical examination, the alert but thin infant has a distended abdomen and a perianal
rash. The stool is watery and foul-smelling and has a pH of 3. No parasites are
identified in the stool. Fecal fat and fecal alpha-1-antitrypsin measurements are both
within normal limits. Of the following, the MOST likely diagnosis is:

A. Congenital lactose intolerance


B. Cystic fibrosis
C. Hereditary fructose intolerance
D. Intestinal lymphangiectasia
E. Sucrase-isomaltase deficiency
52

153. A 15 month old female with history of failure to thrive and a mild, persistent diarrhea
is brought to your clinic for further evaluation of a 2 week history of “being
wobbly” and “running into things, especially at night”. Laboratory analysis reveals
the abnormality shown below. What is the patient’s diagnosis?

A. Acrodermatitis enterohepatica
B. Congenital chloride diarrhea
C. Abetalipoproteinemia
D. Syndromic diarrhea
E. Vitamin E deficiency

http://www.wadsworth.org
53

154. Which is the only amino acid malabsorption disorder that presents with
gastrointestinal manifesta- tions of diarrhea, failure to thrive, and possible
hyperammonemic coma with ingestion high protein diet?

A. Cystinuria
B. Lysinuric protein intolerance
C. Hartnup disease
D. Iminoglycinuria
E. Renal tubular acidosis

155. An 18 month old presents to your clinic with history of or diarrhea since about 9
months of age. Dietary history is positive for 3-4 servings of fruit daily and at least
16 to 24 ounces of juice daily. In addition, he drinks 16 ounces of whole milk and eats
a variety of food including chicken, pasta, and vegetables. Parents have noticed some
relationship to fruit and juice intake. What is likely cause of his diarrhea and the
treatment you would recommend?

A. Congenital Glucose –Galactose malabsorption requiring him to be on Ross


Carbohydrate Free formula supplemented with Fructose as his sole
source of carbohydrate
B. Acrodermatitis enterohepatica requiring treatment with zinc
C. Abetalipoproteinemia requiring supplementation with medium chain
triglycerides and fat-soluble vitamins (with especially high doses of
vitamin E)
D. Fructose malabsorption requiring them to take fructose in the form of fruit
and fruit juice out of his diet. They can reintroduce in small amounts when
he is school-aged.
E. Load dependent lactose intolerance with secondary disaccharide
deficiencies.

156. In chronic pancreatic insufficiency, which of the following is absorbed normally?

A. Fat
B. Folic acid
C. Vitamin B12
D. Protein
E. amylopectin

157. Medium chain triglycerides account for 40-50% of the fat content of formulas
fed to low-birth weight infants. Of the following, the BEST explanation for
this practice is that

A. Absorption of MCT is similar to that of butterfat


B. Low-birth weight infants have a large pool of bile acids
C. Low-birth weight infants have normal capacity to synthesize bile acids
D. MCT cause less steatorrhea
54

E. MCT require micelle formation for absorption

158. Lactose enhances the intestinal absorption of which one of the following
nutrients?

A. Calcium
B. Chloride
C. Lipid
D. Potassium
E. Sodium

159. Of the following, the most beneficial formula for patients with gastrointestinal
allergy, short gut or cystic fibrosis is:

A. Protein hydrolysate
B. Carbohydrate free
C. Lactose free
D. Low iron
E. Soy based
55

160. Of the following fatty acids, which must be added to cow milk-based infant formula?

A. Arachidonic acid
B. Linoleic acid
C. Oleic acid
D. Palmitic acid
E. Stearic acid

161. In regards to gastric emptying, which of the following statements is TRUE?

A. Solids empty faster after vagotomy


B. Proteins delay emptying more than lipids
C. Solids are emptied in an exponential pattern
D. Liquids are emptied faster than solids
E. Cholecystokinin accelerates emptying

162. An Asian-American family is concerned that their 2-month-old infant’s abdominal


distention is due to lactose intolerance. She is intermittently fussy, with frequent
vomiting and poor weight gain, but no diarrhea. She takes a standard cow’s-milk
based infant formula. Which of these options is the most appropriate first
intervention?

A. Trial of a soy-based formula


B. Trial of lactase supplementation
C. Upper GI series
D. IgE RAST testing for cow’s milk
E. Stool pH and reducing substances

163. An adolescent patient with recurrent abdominal pain has a duodenal biopsy
showing low lactase activity, but lactose breath hydrogen test is normal. What is one
likely explanation for these conflict- ing results?

A. His pain is strictly functional in nature.


B. He recently completed a course of antibiotics.
C. He suffers from sucrase-isomaltase deficiency.
D. He ate a high-fiber meal prior to breath testing.
E. He has cow’s-milk protein intolerance.

164. After three weeks of nursing a new mother develops fissured nipples and has to use a
breast pump. At the baby’s one month check-up the mother expresses concerns that
when she begins the pump- ing process the milk seems watery and she is worried
about the baby’s nutrition. You assure her that:

A. Foremilk is normally thinner and primarily serves to assure the baby’s hydration
B. The milk has the normal whey : casein ratio of 30 : 70
56

C. Consuming a diet higher in fat will increase the lipid concentration and nutrition of her
milk
D. Consuming more cow milk in her diet will increase the carbohydrate content of her
milk
E. It provides sufficient vitamin D for her baby

165. Which of the following statements regarding nutritional evaluation in children is TRUE?

A. Marasmus is defined as severe malnutrition with nutritional edema


B. Prealbumin is a sensitive marker of chronic malnutrition
C. Iron deficiency anemia is the most common nutritional deficiency in children
D. Folate and vitamin B12 deficiency cause microcytic anemia
E. The first step in management of acute moderate malnutrition is always aggressive
hydration
with intravenous fluids
57

166. A 10 year old female and her mother present to your office with dietary questions.
The patient has just recently decided to follow a vegetarian diet and her mother is
concerned that this diet will be inadequate for her. Supplementation of which
vitamin is recommended for this patient?

A. Vitamin C
B. Thiamine (Vitamin B1 )
C. Folate (Vitamin B9 )
D. Cobalamin (Vitamin B12 )
E. Niacin (Vitamin B3 )

167. A 12 year old male with Crohn’s disease obtains a chest x-ray prior to the start of
Remicade therapy. CXR reveals cardiomegaly. Patient is referred to cardiologist,
and Echo reveals cardiomyopathy. Deficiency of which of the following
micronutrients has been associated with cardiac complications?

A. Selenium
B. Iron
C. Copper
D. Vitamin C
E. Zinc

168. What is the most appropriate IV formulation within the first 24 hrs of life for a
patient born at 28 week gestation weighing 1,100 grams?

A. Normal Saline
B. 5% Dextrose with electrolytes
C. 5% Dextrose with amino acids
D. 5% Dextrose

169. What is the caloric requirement for a healthy 13 year old male?

A. 100-110kcal/kg/day
B. 70-90 kcal/kg/day
C. 20-30 kcal/kg/day
D. 45-55 kcal/kg/day

170. A 16 year old boy with a Body Mass Index (BMI) of 47, Obstructive Sleep Apnea,
and Type 2 Diabetes Mellitus is interested in a laparoscopic Roux-en-Y Gastric
Bypass surgery for weight reduction. A true statement regarding this procedure is:

A. Patients undergoing Roux-en-Y Gastric Bypass may experience paradoxical


weight gain
B. Post-surgery follow-up pediatric care should be limited to a single office visit
C. Roux-en-Y Gastric Bypass may lead to iron deficiency and other micronutrient
deficiencies
58

D. A post-surgical decrease in insulin resistance is not seen until the BMI decreases
by 30%
E. The stomach is removed completely during the Roux-en-Y Gastric Bypass

171. Which of the following statements regarding the ketogenic diet is FALSE?

A. The ketogenic diet is high in fat content and low in protein and
carbohydrates.
B. Ketones have a direct anti-seizure effect on the brain.
C. The ketogenic diet is recommended for children with disorders of fatty acid
oxidation.
D. Patients following a ketogenic diet require vitamin and mineral
supplementation.
59

172. A twelve-year-old girl who was diagnosed with cystic fibrosis and pancreatic
insufficiency as an infant presents for her annual evaluation. Her BMI has fallen from
the 60th percentile to the 40th percentile for age. She is receiving pancreatic enzyme
replacement therapy at a dose of 2,500 units of lipase per kg per meal. Which is the
most appropriate next step in management?

A. Reassure the patient and family that a BMI at the 40th percentile is adequate.
B. Provide oral nutritional supplements and conduct a full nutritional and behavioral
evaluation.
C. Increase her dose of pancreatic enzymes to 3,000 units of lipase per kg per meal.
D. Refer the patient for surgical gastrostomy tube placement.

173. A three-year-old boy presents for evaluation of diarrhea. His mother states that he
has had up to 6 watery bowel movements per day for one week. He also has a fever
and nasal congestion. Which of the following is the most appropriate
recommendation?

A. Restrict the patient’s diet to clear liquids until the diarrhea resolves.
B. Remove all sources of lactose from the patient’s diet until the diarrhea resolves.
C. Begin a diet consisting of only bananas, rice, applesauce, and toast.
D. Encourage oral rehydration with fluids followed by an unrestricted, age-appropriate
diet.

174. A 2 year old with history of short bowel syndrome (secondary to necrotizing
enterocolitis), TPN dependence, and TPN induced liver disease presents to your clinic
with a history of persistent anemia over the last 6 months. On exam, he is afebrile,
pale, and significantly jaundiced. He has hepato- splenomegaly. He has a Mickey
button in place and well-healed incision from his previous surger- ies. Labs are
significant for a total bilirubin of 8 mg/dL and a direct bilirubin of 4 mg/dL. His AST
54 units/L and ALT 68 units/L, Albumin 2.5 g/dL, INR 1.5. His hemoglobin is 7.5
g/dL and MCV 70. What micronutrient deficiency has resulted in his persistent
anemia?

A. Selenium
B. Copper
C. Niacin
D. Folate

175. A 6 year old male develops a duodenal hematoma after routine endoscopy for
evaluation of his chronic diarrhea and failure to thrive. Biopsies shows subtotal
villous atrophy, atrophic villi, enlarged crypts with large amounts of inflammatory
cells. The patient is diagnosed with celiac disease Marsh 3b. What vitamin
deficiency contributed to the formation of the duodenal hematoma during the
endoscopy?
60

A. Vitamin A
B. Vitamin D
C. Vitamin E
D. Vitamin K

176. A healthy 1 week old male presents to your clinic for evaluation of a questionable
anal abnormality noted by the pediatrician at 2 days of age. In turns out, everything
is normal – however, the mother asks you about vitamin supplementation,
specifically of vitamin D, as she is completely breastfed infant. When should
vitamin D supplementation begin and how much?

A. Immediately, 200 IU till patient is 2 months old then 400 IU.


B. Immediately, 400 IU.
C. Not till 2 months of age, 400 IU.
D. Never, as long as patient is breastfed he will get all the vitamin D he needs from mom.

177. Which of the following patients does not require folate supplementation?

A. A 10 year old male with inflammatory bowel disease on


methotrexate for maintenance therapy.
B. A 2 year male with seizure disorder on phenytoin.
C. A 6 year old female with cystic fibrosis.
D. A 12 year old female with eosinophilic colitis being tried on sulfasalazine therapy.
61

178. A 6 month old male develops a weepy, crusted dermatitis around the eyes, nose,
mouth, diaper area, hands and feeds about 4 weeks after being weaned from breast
milk to formula. He recently devel- oped a watery diarrhea and has stopped gaining
weight over the last month. On exam, he appears listless. His hair is sparse. What is
there is fine and lightly pigmented. What nutritional deficiency has resulted in this
patient’s current condition?

A. Copper
B. Aluminum
C. Zinc
D. Molybdenum

179. A 6 month old female with a history of short bowel syndrome secondary to multiple
intestinal atre- sias presents with complaints of increased work of breathing, poor
feeding, and cough over the last week. She is diagnosed with RSV. It is noted on chest
x-ray that her heart size is significant enlarged and that on exam her heart rate seems
irregular. Further evaluation is concerning for evolving car- diomyopathy. The ICU
physician is concerned that the patient receives her TPN from a new, small emerging
pharmaceutical company by her home in rural Texas. What micronutrient deficiency
is the ICU physician concerned about?

A. Selenium
B. Copper
C. Pyridoxine
D. Iodine

180. A 2 year old male with diagnosis of abetalipoproteinemia moves to your area. He
has recently been diagnosed and comes to your office for establishment of care.
The mother’s chief complaint for this visit is that he stumbles while walking despite
walking with a wide-gait. The stumbling is much worse at night. In addition to a
single supplementation with AquaDEK – which of the following vitamins should
be supplemented further that the standard dosage in AquaDEK.

A. Vitamin A
B. Vitamin D
C. Vitamin E
D. Vitamin K

181. Considered one of three major nutrient deficiencies in the world by the World Health
Organization, this deficiency is the primary cause of blindness in children in the
developing world.

A. Iodine
B. Iron
C. Zinc
62

D. Vitamin A

182. A 16 year old adolescent male has undergone resection of the terminal ileum
because of an ileal stricture. Of the following nutrients, which is MOST likely to
become deficient in this patient?

A. Folic acid
B. Thiamin
C. Pantothenic acid
D. Cyanocobalamin
E. Vitamin K
63

183. A 15 year old boy with well–controlled ulcerative colitis is noted to have a
hemoglobin of 10.2 gm%. He denies any symptoms of bowel disease. He has a good
appetite and eats a varied diet. He takes sulfasalazine and azathioprine for
maintenance therapy. Which of the following vitamin levels is likely to be
abnormally low.

A. Folate
B. Thiamine
C. Vitamin C
D. Cobalamin

184. A healthy 13 y/o male is referred to GI clinic due to elevated alkaline phosphatase
which is 2-3 times the upper limit of normal. Aspartate aminotransferase, alanine
aminotransferase, total bilirubin, and albumin are normal. What should be your next
step to assess for cholestatic disease?

A. Nothing. This is definitely not hepatobiliary disease as this patient is a rapidly


growing male and elevated alkaline phosphatase is due to increased bone
activity.
B. Liver Ultrasound
C. Obtain gamma glutamyltransferase or 5’-nucleotidase
D. Repeat Alkaline Phosphatase, AST,ALT, and total bilirubin in 6 months
E. HIDA scan

185. The endoscopic definition for hiatal hernia involves:

A. Ascension of the Z line more than 2 cm above the hiatus


B. Ascension of the Z line 3 cm above the hiatus
C. Presence of folds in the distal esophagus
D. Open an incompetent hiatus.

186. Six hours after an upper endoscopy a 6 year old develops abdominal pain
and vomiting. Upper abdomen is tender, KUB is unremarkable. Best
next step:

A. Pain medication and antiemetic


B. Imaging study
C. Surgery consult
D. Reassure mom and follow patient in 1 week

187. A 5 year old with abdominal pain and diarrhea is undergoing combined breath
hydrogen/methane testing. A standard dose of lactulose is given while the patient is
NPO. Results from the study show an increase in hydrogen production and peak 3
hours after ingestion of the test dose. No change in methane production is noted.
64

The most correct interpretation of these results is:

A. The results indicate small bowel bacterial overgrowth


B. The results indicate lactose intolerance
C. The results are normal
D. The results are inadequate for interpretation
E. The results indicate fast intestinal transit time

188. Which of the following represents the most distinctive feature of the duodenum, as
compared to the rest of the small bowel?

A. Generally taller and more slender villi


B. Increased proportion of goblet cells within the epithelium
C. Abundance of lymphoid aggregates localized within the lamina propria
D. Predominance of tubuloalveolar glands known as Brunner’s glands
throughout the mucosa and submucosa
E. The plicae circularis are the tallest and most numerous within the small bowel
65

189. Which of the following could distinguish celiac disease from autoimmune
enteropathy on small bowel biopsies?

A. The degree of villous blunting is far less in autoimmune enteropathy


B. An increased number of plasma cells in the lamina propria in autoimmune
enteropathy
C. The lack of increased intraepithelial lymphocytes in autoimmune
enteropathy
D. The patchy nature of scalloped and ulcerated mucosa seen in autoimmune
enteropathy
E. The increased presence of submucosal eosinophils in autoimmune
enteropathy

190. Fecal elastase may be used to diagnose chronic pancreatitis. Which of the following
conditions does not result in abnormal levels of fecal elastase?

A. Celiac disease
B. Crohn’s disease
C. Diarrheal illness
D. Primary sclerosing cholangitis
E. Short bowel syndrome

191. Which of the following tests can detect mild pancreatic dysfunction?

A. Lundh test meal


B. Fecal elastase
C. Secretin stimulation test
D. Serum immunoreactive trypsinogen
E. Fecal fat measurement

192. An contrast UGI reveals a corkscrewed duodenum ending blindly


suggesting the possible diagnosis of:

A. Duodenal web
B. Duodenal protein enteropathy
C. Malrotation
D. Congenital microcolon of disuse
E. Annular pancreas

193. The radiological signs of necrotizing enterocolitis can include

A. pneumatosis intestinalis, pneumoperitoneum and hepatic pneumatosis


B. Colonic “thumb printing”
C. Punctate adrenal calcification
D. A and B
66

E. A and C

194. You see in follow-up a 12 y/o boy who sustained a bicycle handlebar injury 6 weeks
prior. He reports he is feeling well with no fever, vomiting or pain, but on
physical examination, you notice he grimaces when you palpate a smooth fullness
just to the left of the umbilicus. You next order:

A. Plainfilm of the abdomen


B. Contrast UGI
C. US of the abdomen
D. CAT scan
E. Tc99m scan
67

195. A 5 year old boy is referred to your clinic for the evaluation of chronic diarrhea and
failure to thrive. You would like the parent to perform a 3 day collection for fecal
fat. Which of the following is NOT true regarding this test?

A. Steatorrhea is present in a child or adult if more than 7% of ingested fat is excreted.


B. It allows one to determine whether fat malabsorption is pancreatic or
nonpancreatic in nature.
C. The test involves the meticulous weighing of food and careful dietary
records in order to calculate mean daily fat intake.
D. Every single stool during a 3 day period should be collected.

196. Limitation of stool electrolyte testing include:

A. Can only be done on solid stool


B. Special media are needed
C. Subjects often ingest non-absorbable sugars
D. No standard exists for electrolytes in stool therefore results must be used in a
comparative values to prior or future stools
E. There are few genetic tests available for differentiation of congenital causes of diarrhea

197. A 3 year old male with a past medical history significant for small bowel resection
including the terminal ileum following necrotizing enterocolitis at 2 weeks of
age presents with perianal excoriation and diarrhea. Which antidiarrheal
agent would be most appropriate?

A. Bismuth subsalicylate
B. Cholestyramine
C. Loperamide
D. Octreotide
E. Clonidine

198. All of the following are true EXCEPT:

A. Pruritus related to cholestatic liver disease is thought to be due to centrally


mediated causes related to endogenous opioid neurotransmission.
B. Diphenhydramine can help ameliorate pruritus in cholestatic liver disease
C. Opiate antagonists is the first line treatment for pruritus associated with
cholestasis in children
D. Cholestyramine is a hydrophilic, water insoluble anion-exchange resin that
binds bile acids, preventing their absorption through the enterohepatic
circulation

199. Regarding blood transfusions in children, which statement is false?

A. Blood volume in children varies more dramatically in the first year of life than in latency
68

B. A unit of pRBCs has a hematocrit of 90%


C. Transfusion in children raises hemoglobin approximately 2 to 2.5 g/dL for
every 10 ml/kg of pRBCs given.
D. Hematocrit equilibrium post transfusion is generally evident within 24 hours
E. Newborns have a blood volume of about 85 ml/kg

200. What is the gold standard for measuring dehydration?

A. Serum electrolytes
B. BUN/Cr
C. Urine specific gravity
D. Percent loss of body weight
E. Heart rate
69

201. A 7 month old boy with biliary atresia underwent liver transplantation. His
medications include tacro- limus and prednisone. Two months postoperatively he is
noted to have elevated transaminases. An infectious work up is initiated and a liver
biopsy is performed, which shows dense periportal lympho- cytic and eosinophilic
infiltration with endotheliitis and bile duct damage. One week after starting high
dose steroids his transaminases are still elevated. Which of the following agents may
be consid- ered in this situation?

A. Tacrolimus at an increased dose


B. Cyclosporine
C. MMF
D. OKT3

202. A 13 year old girl has a liver transplantation for autoimmune hepatitis and is started
on prednisone and tacrolimus in the post operative period. Her hospital course is
complicated by a central line infec- tion and adenovirus infection. She is discharged
home on Prednisone, tacrolimus and MMF. Several months after discharge she
develops pharyngitis and is treated with an antibiotic.

One week later, her labs are as follows: Albumin - 4.1 mg/dL; total protein - 6.3
mg/dL; total biliru- bin – 2.0 mg/dL; direct bilirubin - 1.0 mg/dL; ALT – 35 IU/L,
AST – 22 IU/L, GGT – 45 IU/L; Na: 135 mEq/L; K: 6.8 mEq/L; CL – 110 mEq/L;
CO2 – 24 mEq/L; glucose – 120mg/dL, BUN – 45 mg/dL; Cr -
2.3 mg/dL; Mg – 1.1

Which antibiotic is most likely to be the cause of these abnormal laboratory


findings?

A. Penicillin
B. Amoxicillin
C. Erythromycin
D. Ceftriaxone

203. A 3 year old girl underwent liver transplantation for hepatoblastoma. Her post
operative course is complicated by biliary leak and multiple episodes of ascending
cholangitis. She receives 10 days of parenteral antibiotics with one week of
fluconazole. She is discharged home on Tacrolimus, Predni- sone and
Multivitamins.

Two weeks later laboratory assessment shows Albumin – 3.5 mg/dL; total protein -
6.8 mg/dL, total bilirubin - 5.0 mg/dL, direct bilirubin 3.2 mg/dL, ALT – 375 IU/L,
AST – 450 IU/L, ALP – 315 IU/L. A liver biopsy is suggestive of rejection.

Which of the following factors may be responsible?


70

A. Tacrolimus toxicity
B. Discontinuation of Fluconazole
C. Intercurrent viral infection
D. Incorrect HLA typing of donor

204. A twelve year old male who was diagnosed with Crohn disease 3 years ago has
been on q8 wk Inf- liximab infusions for one year. The parents reported that he
developed erythematous, vesicular, skin lesions in the right posterior rib cage area.
The patient was afebrile but complained of skin tingling sensation. According to
above history, your recommend is:

A. Observation
B. Start 3rd generation cephalosporin
C. Discontinue Infliximab
D. Start p.o acyclovir
E. Admit for IV acyclovir
71

205. A 16 year old female has been on maintenance Infliximab mono therapy for 3 years.
She was previ- ously treated with 6-mercaptopurine. She presents with night
sweats and splenomegaly.

What rare life threatening tumor must you be concerned about:

A. Burkitt’s Lymphoma
B. Hepatosplenic T-cell Lymphoma
C. Acute Myelogenous Leukemia
D. Hodgkin’s Lymphoma
E. Signet Ring Carcinoma

206. A 13-year-old female has a draining peri-rectal fistula. You choose to use infliximab
therapy to heal the fistula. What problem with using the therapy must you think
about before starting therapy?

A. Walled off abscess formation


B. Systemic infection
C. Increased pain
D. Alternative site of fistulization
E. Constipation

207. A 17-year-old male has Crohn disease and has responded to Infliximab monotherapy
for two years without complication. He has been on 5 mg/kg/dose every 8 weeks. He
develops 6 bloody stools per day at 7 weeks and a trough level is obtained. It is
undetectable. Stool studies for infection and CMV PCR of colonic tissue is
negative. Your best choice to treat this patient is:

A. Change to Adalimumab
B. Obtain a HACA
C. Increase to 10 mg/kg/dose or change interval to 4 weeks
D. Start prednisone
E. Start methotrexate or 6-mercaptpurine

208. A 7 year old female with cystic fibrosis is seen in follow up. She has not lost
weight but has had little weight gain over the last six months despite adequate
caloric intake. She has 2 stools per day, and sometime sees oil droplets. She has no
other symptoms. She takes 2000 units/kg of pancreatic enzymes with all meals.
The next best step in management is:

A. Increase the pancreatic enzyme dose to 4000 units/kg before each meal
B. Offer reassurance that no intervention is needed unless there is weight loss
C. Add loperamide to her daily medications
D. Add a PPI to her daily medications
72

209. You are interested in assessing the possibility that a child has steatorrhea. What
question would you not ask the parents?

A. Are the stools hard to wipe off during a diaper change?


B. Do the stools float?
C. Do the stools smell rancid?
D. Are the stools difficult to flush?
E. Are the stools loose and oily?

210. A 4 month old infant presents to the emergency department with a four day history
of progressive lethargy, bradycardia, loose stools, and diminished deep tendon
reflexes. She is afebrile and has no rhinorrhea or exanthema. History revealed that her
parents were mixing antacids into her bottles to help with reflux symptoms. Her
symptoms are MOST likely explained by

A. Hypermagnesemia
B. Hypomagnesemia
C. Hyperkalemia
D. Hypokalemia
E. Hypoglycemia
73

211. In a 12 year old boy with Zollinger-Ellison Syndrome, which of the


following is the BEST medical therapy?

A. Sodium bicarbonate
B. Nizatidine
C. Lansoprazole
D. Famotidine
E. Calcium carbonate-ranitidine compound

212. Probiotics have been shown to be of benefit in

A. Ulcerative colitis
B. Crohn’s disease
C. Both
D. Neither

213. In regard to the colonization of the newborn gut, which statement is false?

A. It is initially colonized by facultative maternal vaginal and fecal flora


consisting of
Streptococcus, Enterococcus and Coliform genera
B. C-section babies are initially colonized by Klebsiella, Enterobacter and
Clostridia genera
C. Breast fed babies have higher concentrations of Bifidobacterium and
Lactobacillus genera
D. Formula fed babies have higher concentrations of Bifidobacterium and
Lactobacillus genera
E. After 10 days both vaginal and C-section babies share the same flora

214. Probiotics are live microorganisms, that when consumed in adequate numbers, confer
a health ben- efit to the host. They also:

A. Lower intestinal pH and decrease the adherence of pathologic bacteria


B. Increase the production of short chain fatty acids and increase motility
C. Contribute to the production of riboflavin and folate
D. All above statements are false
E. All above statements are true

215. Misoprostol is thought to treat and prevent NSAID-associated peptic


ulcers by which of the following mechanisms:

A. Decreasing gastric acid secretion


B. Increasing mucosal resistance to injury by decreasing bicarbonate secretion
C. Providing mucosal protection by decreasing mucosal blood flow
D. Blocking histamine receptors
74

E. Activating adenylate cyclase in parietal cells.

216. Which of the following is true concerning the properties and action of Sucralfate on
gut mucosa?

A. Sucralfate is a sulfated aluminum hydroxide that selectively binds to ulcers and


erosions.
B. Sucralfate is a sulfated magnesium hydroxide that selectively binds to ulcers
and erosions.
C. Sucralfate binds to ulcers and erosions and is activated by non-acidic
conditions.
D. Sucralfate should be taken together with antacids for optimal binding
activity.

217. A dialysis patient being treated for a gastric ulcer presents with bone pain, mental
status changes and proximal muscle weakness. Which medication was
responsible for these side effects?

A. Ranitidine
B. Cimetidine
C. Sucralfate
D. Terfenadine
75

218. Which one of these drugs have reduced bioavailability when co-administered with
Sucralfate?

A. Ciprofloxacin
B. Erythromycin
C. Aspirin
D. Prednisone

219. You are called at 3 am from the ER about a 3 year old female who has a history of
severe constipa- tion. The ER physician performed a history and a physical exam and
there is no evidence of constipa- tion or any acute problem but the mother demanded
an X-ray of the abdomen to be taken since “that is how her doctor diagnosed the
constipation”. The X-ray showed scattered stool around the colon. Thereafter the
mother insisted on an admission for a clean-out. The ER physician is asking you,
would you admit this patient under your service for a Go-Lytely clean-out?

A. The abdominal X-ray and the history/physical exam are enough evidence that
this patient has constipation and needs a clean-out.
B. Parental insistence on admission may be reasonable given what appears to be a
knowledge- able parent.
C. The history/physical exam and X-ray do not support a diagnosis of
constipation. You will request history of admissions, ER visits and test
results before deciding.
D. You recommend no admission; there is no indication to admit the patient.
Refer the patient and the mother to a psychiatrist.
E. You Recommend a CT scan of the abdomen since is more reliable than X-
ray to diagnose constipation.

220. You are consulted for admission of a 5 year old male who has G-tube issues. The
patient has a his- tory of severe recurrent asthma and is followed by the pulmonology
service. He was diagnosed with GERD for which he received a Nissen-
fundoplication and G-tube. Recently, he started spitting up again at home but it has
not been witnessed by nurses. Mother believes the vomiting is caused by
fundoplication failure and a blocked G-tube and that an endoscopy is the best test to
diagnose failed fundoplication. At the end of the interview she gives you a gift and
thanks for your time and asks when you would do the scope and if you can also
change her G-tube to a G-J tube since this will help with the vomiting. The next
best course of action is:

A. Accuse mother of Munchausen Syndrome by Proxy and call hospital security


to protect the child and the mother
B. Ask the mother to go out of the room to interview the child alone and ask
him questions about child abuse
C. Immediately leave the room and call Child Protective Services to take him into
custody.
76

D. Call physicians and staff involved in his care to get details of the history.
Obtain medical records and document details of the encounter.
E. Schedule upper endoscopy and G-J tube placement as soon as possible. Consult
surgeon for repeat fundoplication.

221. Infant rumination best responds to

A. Anticholinergics
B. Soy formula
C. NG tube feedings
D. Frequent holding and social interaction
E. PPI therapy
222. Clostridium difficile colitis is found in what percentage of children with antibiotic
associated diarrhea: A. 5%-10%
B. 15%-20%
C. 25%-30%
D. Greater than 30%
77

223. Which of the following statements regarding congenital chloridorrhea is true?

A. Stool Cl- level less than 90 mmol/L


B. Results in metabolic alkalosis
C. Results in metabolic acidosis
D. First detected after the patient first gets solid food
E. Due to defective chloride-bicarbonate exchange in the colon

224. Which of the following conditions is usually suspected after protracted diarrhea
begins after the first few weeks of life?

A. Microvillous inclusion disease


B. Tufting enteropathy
C. Congenital chloridorrhea
D. Autoimmune enteropathy
E. Enterocyte heparan sulfate deficiency

225. Erythromycin when used as a prokinetic:

A. Stimulates motilin
B. If given to a one week old increased the risk of pyloric stenosis
C. Can safely be given to a nursing mother
D. All are true

226. Pill esophagitis is commonly associated with these medications except:

A. Doxycycline
B. Tetracycline
C. Bismuth subsalicylate
D. Theophylline
E. KCl

227. What is the most common finding in a 15 year old male with IgA deficiency?

A. Recurrent infections with E. histolytica


B. Celiac disease
C. Inflammatory bowel disease
D. None of the above

228. Which immunodeficiency should be suspected in a 6yo male with recurrent


infections and perianal disease with gastric granulomas?

A. Leukocyte adhesion defect


B. Severe combined immunodeficiency
C. X-linked chronic granulomatous disease
78

D. IgA deficiency

229. Regarding IgA deficiency, which statement is false?

A. IgA deficiency is the most common form of immunodeficiency.


B. Most patients with confirmed IgA deficiency are asymptomatic.
C. There is variable penetrance of the IgA deficiency defect but all cases have
<2 SD of normal IgA levels for age.
D. Sinopulmonary infection is the most common manifestation, followed by
recurrent gastroin- testinal parasitosis of various genera including protozoa,
flagellate, nematodes and platyhel- minthes.
E. The diagnosis is most accurate after the age of four years.
79

230. You are consulted for a jaundiced 9 y/o male who is 3 weeks post bone marrow
transplant. He is afebrile and has right upper quadrant tenderness. All viral serology
is negative except HBsAg. ALT is 78, AST is 59, TB is 8.2 with a direct of 3.9. AP is
181. His coagulation studies are normal. His RUQ ultrasound report is
unremarkable. You suspect hepatic GVHD and advise beginning with

A. Treatment with steroids


B. Treatment with steroids plus ursodeoxycholic acid
C. Liver biopsy to confirm the diagnosis
D. HIDA scan
E. Bone marrow aspiration for culture.

231. A 14 year old female with poorly controlled hyperthyroidism is evaluated for
diarrhea. Which is the following is not a possible cause for diarrhea is in this
patient?

A. Decreased bile output


B. Decreased trypsinogen levels
C. Small intestinal bacterial overgrowth
D. Lactase deficiency
E. Accelerated oral to cecal transit

232. Nonalcoholic steatohepatitis is associated with which one of the following conditions?

A. Autoantibodies against liver antigens


B. Chronic hepatitis C
C. Insulin-dependent diabetes mellitus
D. Iron overload
E. Copper overload

233. Which of the following is not associated with hypothyroidism?

A. Ascites
B. Esophageal compression
C. Delayed gastric emptying
D. Delayed small bowel transit
E. Pernicious anemia

234. Graft-versus-host disease (GVHD) is caused when:

A. Host lymphocytes are depleted


B. Immunosuppressive therapy is increased
C. Donor lymphocytes attack host tissues
D. Host granulocytes attack donor tissues
80

235. The ethical precepts described in the Belmont Report include adherence to the
principles of Autonomy, Beneficence, Non-maleficence and:

A. Consent
B. Self-determination
C. Justice
D. Morality

236. In human subjects research involving minors, ethical considerations include:

A. Assent must be obtained from all children.


B. Clinical trials may not involve a greater than a minor
increase over minimal risk to individual subjects.
C. Investigator conflicts of interest are permissible, as long as they are reported to the local
IRB.
D. A subpart D (Code of Federal Regulation) determination is required for all studies.
81

237. Choose the technique used to assess a statistical models assumptions:

A. Specificity
B. Cost-effective analysis
C. Sensitivity analysis
D. Validity
238. Match each word with the corresponding answer
choice: Validity A. Measure of
agreement
Reliability B. Measurement of what is intended
Accuracy C. Measure gives same results on repeated trials
Precision D. Measure of the internal consistency of
a test Kappa statistic E. How close measurement is to actual
value Cronbach’s alpha F. How close repeated measures
are to each other

239. If a researcher wants to increase the power of the study from 80% to 90% but
keep all other parameters the same, the original sample size will:

A. Decrease
B. Stay the same
C. Increase
D. Unable to tell

240. A control group compared to the intervention group should vary by:

A. Gender
B. Age
C. Education
D. None of the above

241. Analysis of variables by the original group assignment regardless if they


remained or adhered to that group is called:

A. Intention to treat analysis


B. Per protocol analysis
C. Post-hoc analysis
D. Cost-effectiveness analysis

242. The measure of how results of a study can be generalized to the population as a
whole:

A. External validity
82

B. Accuracy
C. External variation
D. Power

243. A 17-year-old boy who has evidence of Crohn’s disease in the terminal ileum
develops severe radiating inguinal and scrotal pain.

The MOST likely visceral source for the referred pain in this patient
is the

A. Appendix
B. Diaphragm
C. Gallbladder
D. Small bowel
E. Ureter
83

244. Which of the following represents the MOST likely mechanism for peptic injury
associated with chronic administration of nonsteroidal anti-inflammatory
drugs (NSAIDS)?

A. Generalized ischemia of the gut


B. Genetic predisposition
C. Inhibition of prostaglandin
D. Promotion of growth of Helicobacter pylori
E. Topical caustic injury

245. A 6 year old girl has had abdominal pain and nonbilious vomiting for 8 hours. History
reveals cough and fever for the past 3 days. Findings on physical examination include
temperature, 39°C (102.2°F); tachypnea; toxic appearance; diffuse, voluntary
guarding; and quiet bowel sounds.

Of the following, the examination MOST likely to establish the etiology of the
abdominal pain and fever in this patient is a(n):

A. Abdominal radiograph
B. Chest radiograph
C. Complete blood count
D. Rectal examination
E. Upper gastrointestinal series

246. A 9-year-old girl has the height age of a 7-year-old and the bone age of
a 6-year-old. Among the following, the MOST likely cause of her
short stature is

A. Achondroplasia
B. Hypothyroidism
C. Malnutrition
D. Normal variant short stature
E. Silver-Russell syndrome

247. An antral or pyloric web (diaphragm) is considered in the differential diagnosis of a 6-


month-old girl with failure to thrive syndrome and nonbilious vomiting.

The MOST specific study for diagnosing this condition is

A. Gastric emptying study


B. Plain abdominal radiography
C. Ultrasonography
D. Upper gastrointestinal contrast study
E. Upper gastrointestinal endoscopy
84

248. A 10-year-old child has had intermittent diarrhea and weight loss over
the past year. A TRUE statement regarding testing with guaiac or
orthotolidine for occult blood in this patient’s stool is:

A. Microscopic examination of the stool is a better test for detecting occult blood
B. Negative results exclude lower gastrointestinal bleeding
C. Positive results confirm the presence of occult blood
D. These tests detect peroxidase activity in hemoglobin
E. These tests quantitate the amount of hemoglobin in the stool
85

249. An 18-year-old girl who is taking tolmetin for juvenile rheumatoid arthritis
develops gastritis. Which of the following medications would have been MOST
likely to prevent the development of peptic disease in this patient?

A. Antacids
B. Corticosteroids
C. H2-blockers
D. Misoprostol
E. Sucralfate

250. A 3 year old boy with acute lymphoblastic leukemia in hematologic remission is
receiving vincristine, methotrexate, and 6-mercaptopurine. He develops abdominal
pain and distention and nausea with- out fever or diarrhea.

These findings are MOST likely caused by

A. Escherichia coli gastroenteritis due to granulocytopenia


B. Intestinal candidiasis due to lymphopenia
C. Intestinal mucosal ulcerations due to methotrexate toxicity
D. Necrotizing enterocolitis due to 6-mercaptopurine toxicity
E. Reduced intestinal motility due to vincristine toxicity

251. A term infant is born with gastroschisis that is repaired at birth. The infant is
placed on total parenteral nutrition.

The serum level of which of the following is likely to become abnormal


FIRST?

A. Alkaline phosphatase activity


B. Bile salt concentration
C. Direct bilirubin concentration
D. Indirect bilirubin concentration
E. Transaminase activity

252. A previously healthy 12-year-old boy is icteric. Physical examination reveals a


noncommunicative, moderately ill-appearing boy who has an enlarged, soft, tender
liver and ascites; there is no sple- nomegaly. Pitting edema of the ankles and sacral
area and scattered bruising of the extremities are noted.

Among the following, the MOST critical set of studies to include in the initial
laboratory evaluation is

A. Hepatitis A and B serologies


B. Hepatocellular enzyme activities
C. Prothrombin time and partial thromboplastin time
86

D. Total and fractionated serum bilirubin levels


E. Total serum protein and albumin concentrations

253. A 2 year old girl has a history of recurrent pneumonia, short stature, and failure to
thrive. Studies reveal absolute neutropenia and thrombocytopenia, normal sweat
chloride concentration, and me- taphyseal dysplasia of the head of the left femur.
The MOST likely diagnosis for this patient is:

A. Alagille-Watson syndrome
B. Shwachman-Diamond syndrome
C. Sideroblastic anemia
D. Trypsinogen deficiency
E. Wiskott-Aldrich syndrome
87

254. Of the following, which condition is the MOST common presentation of


infection with enterotoxigenic Escherichia coli?

A. Dysentery-like illness with fever


B. Hemolytic-uremic syndrome
C. Hemorrhagic colitis
D. Self-limited illness with watery stools and cramps
E. Severe diarrhea and dehydration

255. A 7-day-old breastfed infant born at term has had decreased appetite, irritability, and
vomiting for 24 hours. On physical examination, the infant appears listless.
Respiratory rate is 40/min; heart rate, 160/min; and blood pressure, 68/38 mm Hg. The
skin and conjunctiva are icteric but no other abnor- malities are noted. Laboratory
studies reveal: hemoglobin, 12 gm/dL; total bilirubin, 16 mg/dL; and direct bilirubin,
8 mg/dL. Urinalysis is negative for reducing substances.

Of the following, the MOST likely diagnosis is

A. Bacterial sepsis
B. Blood group incompatibility
C. Breast milk jaundice
D. Hypothyroidism
E. Intrauterine infection

256. Which of the following metabolic alterations is most commonly seen with re-feeding syndrome?

A. Hyperlipidemia secondary to increased serum ketone bodies


B. Wernicke’s encephalopathy secondary to thiamine deficiency
C. Severe hypophosphatemia affecting myocardial and respiratory function
D. Hypernatremia and hypertonic dehydration affecting mental status

257. A 29 year old male is referred from an optometrist for evaluation. The patient’s liver
profile shows AST 78 IU/L, ALT 92 IU/L, Bili 1.4 mg/dL, Alk Phos 88 IU/L, and
albumin 3.4 g/dL. The photo of the patient’s eyes is below. All of the following
statements are true except:

A. The patient’s diagnosis is Wilson’s disease if the ceruloplasmin is low.


88

B. This finding on the eye exam can be seen in primary biliary


cirrhosis and autoimmune hepatitis.
C. The pigmentation will disappear with effective therapy.
D. Neurologic symptoms typically do not occur in the absence of this finding.
E. The mechanism that causes the disorder associated with this
finding occurs as a result of over-absorption of copper.
89

258. A 10-old male with HIV on HART therapy is evaluated for elevated liver enzymes
(ALT 119 IU/L, AST 101 IU/L, bilirubin 1.3 mg/dL, Alk Phos 390 IU/L). A liver
biopsy showed numerous blood-filled cysts that do not have an endothelial lining.
This liver biopsy finding is most likely secondary to:

A. Cytomegalovirus
B. Protease inhibitors
C. Rochalimaea henselae
D. Caroli disease
E. Congenital factors (e.g., cystic Von Meyenburg complexes)

259. An infant boy does not pass stool during the first 36 hours of life. Following rectal
examination, he passes a meconium plug. During the next 2 weeks, he has
intermittent episodes of both watery and hard, pellet-like stools. Barium enema
reveals dilation of the large bowel with narrowing immediately proximal to the
rectum.

The MOST appropriate management of this patient’s problem is to

A. Add table sugar to the present formula


B. Arrange for a rectal biopsy
C. Change to a soy-based formula
D. Observe without intervention
E. Order an upper gastrointestinal series

260. Lactose enhances intestinal absorption of which ONE of the following


nutrients?

A. Calcium
B. Chloride
C. Lipid
D. Potassium
E. Sodium

261. A 2 week old boy has short-bowel syndrome following surgery for severe necrotizing
enterocolitis. Management has included total parenteral nutrition. Clinical findings
include a wasted appearance; dry, flaky skin; a poorly healing abdominal
incision; and thrombocytopenia.

The patient MOST likely has a deficiency of

A. Calories
B. Essential amino acids
C. Essential fatty acids
D. Iron
90

E. Vitamin E

262. A 14 month old African-American infant, exclusively breastfed since birth, has just
begun walking. Physical examination reveals prominence of the costochondral
junctions. Radiographs reveal widen- ing of the distal end of the radii. The laboratory
test MOST likely to confirm the diagnosis is measure- ment of the serum
concentration of:

A. Albumin
B. Lactate dehydrogenase
C. Phosphorus
D. Vitamin A
E. Vitamin C
91

263. A 6 year old boy who has had a relapse of acute lymphoblastic leukemia has had a
4.8 kg (10 lb) weight loss during a course of combination chemotherapy with
prednisone, vincristine, and high- dose methotrexate. He has had a poor
appetite but no vomiting or diarrhea.

Of the following, the MOST appropriate next course of management would be

A. Administration of anabolic steroids


B. Administration of intravenous gamma globulin
C. Discontinuation of methotrexate
D. Enteral hyperalimentation
E. Parenteral hyperalimentation

264. The basal energy or metabolic requirement for children is calculated MOST accurately by
considering

A. Body surface area


B. Creatinine-height index
C. Serum protein concentration
D. Total lymphocyte count
E. Triceps skinfold thickness

265. An infant boy born at term is delivered at home without medical supervision. At 48
hours of age, he is brought to the emergency room because of a bloody discharge from
the umbilical cord and bloody stools. Until the results of laboratory studies are
available, the BEST initial management is to adminis- ter intravenous:

A. Ampicillin and gentamicin


B. Cryoprecipitate
C. Factor VIII concentrate
D. Fresh frozen plasma
E. Vitamin K

266. A 3 1/2 year old boy with chronic diarrhea and failure to thrive is diagnosed with
cystic fibrosis. Neurologic examination reveals absent deep tendon reflexes, truncal
ataxia, and muscle weakness. A nutrient deficiency is suspected. Given this
constellation of findings, what additional physical sign is MOST likely to be present
in this child?

A. Desquamating skin lesions


B. Ophthalmoplegia
C. Positive Trousseau sign
D. Purpura
E. Stooped posture
92

267. Which of the following BEST explains why solutions containing 1.2 to 2.5%
glucose, rather than 5% glucose, are used for oral rehydration?

A. Absorption of sodium and water in the gut is maximized


B. Glomerular filtration rate is enhanced
C. Hyperglycemia and glycosuria are less likely to occur
D. Potassium absorption is decreased
E. Stomach distention with vomiting is less likely to occur

268. A TRUE statement about the sugar content of infant formulas is:

A. All lactose-containing formulas are cow milk-based


B. All cow milk-based formulas contain only simple sugars
C. All soy-based formulas are corn syrup-free
D. All soy-based formulas contain lactose
E. Proprietary formulas do not contain sucrose
93

269. Shortly after birth, a 3,500 g term newborn is found to be jittery and to have a high-
pitched cry. Physical examination reveals tachypnea and a liver edge that is palpable
several centimeters below the umbilicus. Blood glucose concentration is 14 mg/dL.
Among the following, the MOST likely cause of the hypoglycemia in this newborn
is:

A. Galactokinase deficiency
B. Glycogen storage disease
C. Insulinoma
D. Maternal diabetes mellitus
E. Prolonged maternal labor

270. A breastfed infant who appeared healthy at birth develops chronic diarrhea, failure
to thrive, and hepatomegaly during the first few weeks of life. Ultrasonography
reveals adrenal enlargement and calcification.

Of the following, the MOST likely explanation for these findings is:

A. Cystic fibrosis
B. Glucose-galactose malabsorption
C. Glycogen storage disease
D. Niemann-Pick disease
E. Wolman disease

271. Among the following, the gastrointestinal disease MOST likely to respond to
treatment with anti- cholinergic medications is:

A. Constipation
B. Dysentery
C. Gastroesophageal reflux
D. Irritable bowel syndrome
E. Poor motility

272. Examination of a developmentally normal 7-month-old boy reveals moderately


enlarged cervical lymph nodes; a hemorrhagic seborrhea-like rash on the forehead,
scalp, and trunk; and hepato- splenomegaly. Laboratory findings include:
hemoglobin, 12.0 g/dL; mean corpuscular volume, 82 fL; white blood cell count,
10,700/mm³, with 40% neutrophils and 60% lymphocytes; and platelet count,
260,000/mm³.

These findings are MOST consistent with

A. Acute lymphoblastic leukemia


B. Aplastic anemia
C. Langerhans cell histiocytosis
94

D. Neuroblastoma
E. Niemann-Pick disease

273. A 14 year old boy is being evaluated for jaundice that was first noted 1 week ago
following an upper respiratory tract infection. He reports not feeling very hungry for
the past month. Physical examina- tion reveals a firm liver, an enlarged spleen,
and an intention tremor.

Among the following, the test that would be MOST helpful for making a
definitive diagnosis in this patient is a:

A. Liver biopsy for copper content


B. Serum bilirubin concentration
C. Serum ceruloplasmin level
D. Serum transaminase activity
95

274. A 3 month old infant girl is admitted to the hospital for evaluation of recurrent
episodes of hypogly- cemia. Physical examination reveals hepatomegaly. After 2
hours of fasting, she develops hypoglyce- mia. Measurement of which of the
following would be MOST helpful in determining the etiology of this patient’s
hypoglycemia?

A. Ammonia in the serum and the arterial pH


B. Cortisol and growth hormone in the serum
C. Insulin and glucose in the serum
D. Ketones and reducing substances in the urine
E. Organic acids in the urine and lactate in the serum

275. The mother of a 3 month old infant reports that the boy is demanding frequent
feedings and has a noticeably protuberant abdomen. Physical examination reveals
doll-like facies and marked hepa- tomegaly. Laboratory findings include a serum
glucose level of 20 mg/dL and an elevated venous lactate level of 44 mg/dL
(normal, <18 mg/dL).

Of the following, the most appropriate INITIAL management of this infant is:

A. Administration of a formula that has high concentrations of fructose and galactose


B. Daily injections of glucagon
C. Insertion of a portacaval shunt
D. Nocturnal infusion of glucose via a nasogastric tube
E. Referral for liver transplantation

276. The mother of a 3 month old boy reports that he has a poor appetite and
constipation. Findings on physical examination, when compared to those 2
months ago, include poor interim growth, increased lethargy, hoarse cry, decreased
tone, large fontanelles, and a more pronounced umbilical hernia.

Of the following, the MOST likely cause of this infant’s problem is

A. Agenesis of the thyroid gland


B. Endemic goitrous hypothyroidism
C. End-organ unresponsiveness to thyroid hormone
D. Inborn error of thyroxine synthesis
E. Thyroid gland unresponsiveness to thyrotropin

277. Which of the following is the most common cause of pancreatitis in childhood?

A. Viral
B. Drug induced
C. Idiopathic
D. Familial
96

E. Abdominal trauma

278. Which of the following is not part of the Currarino triad characterizing caudal
regression syndrome which can present as infantile constipation?

A. Dysplastic sacrum
B. Anal abnormalities
C. Tethered cord
D. Pre-sacral mass

279. Which of the following is NOT associated with Wilson’s Disease:

A. Fatty liver
B. High serum uric acid
C. Low serum zinc level
D. Low serum alkaline phosphatase
E. High serum bilirubin
97

280. Which statement is false?

A. 90% of Vitamin A is stored within the liver


B. Vitamin A deficiency Xerophthalmia is irreversible
C. Can cause a hypochromic microcytic anemia with low Fe but normal Fe
stores
D. Hypervitaminosis A is associated with head aches

281. Which of the following is NOT a common feature of BOTH kwashiorkor and
marasmus:

A. Irritability
B. Decreased serum lipoproteins
C. Markedly Depressed serum albumin
D. Increased susceptibility to infection
E. Anemia

282. Tyrosinemia is associated with:

A. Boiled cabbage smell


B. Mousey smell
C. Blue Cheese Vinaigrette smell
D. Maple syrup
E. All of the above except D

283. Which statement about Hepatitis A is true?

A. Leading cause of fulminant hepatitis in Pediatrics


B. Has been associated with chronic hepatitis
C. Recurrence of the disease can occur up to 6 months after primary infection
D. Treatment for non-fulminant hepatitis A includes Lamivudine for 4 weeks
E. Severity of disease decrease with increasing age

284. Acute lower GI hemorrhage in HIV infected patients is most often caused by:

A. CMV colitis
B. Lymphoma
C. Kaposi’s sarcoma
D. Idiopathic chronic colitis
E. Nonspecific colitis

285. First line of treatment of esophageal candidiasis in HIV infected patient is:

A. Clotrimazole
B. Ketoconazole
98

C. Fluconazole
D. Amphotericin B

286. Organism causing intestinal microsporidiosis in AIDS patients is:

A. Encephalitozoon intestinalis
B. Cryptosporidium
C. Isospora Belli
D. Enterocytozoon Bieneusi
99

287. A 7 year old girl who had undergone a surgical repair for long-segment
Hirschsprung’s disease in early infancy presents with a fever, abdominal distention,
and bloody diarrhea for 2 days. Which of the following is the most likely
diagnosis?

A. Diversion colitis
B. Enterocolitis
C. Ulcerative colitis
D. Colonic stricture
E. Viral gastroenteritis

288. A 6 year old boy just arriving from Eastern Europe has had malodorous diarrhea since
early infancy, even though he was breast-fed. He is small, has some bruises from
bumping into furniture going to the bathroom at night, and has recently developed
some difficulty walking. Physical examination shows that he is small and
undernourished, with depleted subcutaneous fat. He has a protuber- ant abdomen
and 1+ edema in his lower extremities. He has no deep tendon reflexes in his lower
extremities. Which one of the following explains the finding on the small intestinal
biopsy from this patient?

A. Gluten enteropathy
B. Congenital lactase deficiency
C. Abetalipoproteinemia
D. Glucose-galactose transport defect
E. Chronic nonspecific diarrhea of childhood

289. A 5 year old boy is referred for evaluation of liver disease after presenting to his
primary physi- cian with chronic pruritus. His evaluation reveals a small child
(below the fifth percentile for height;
weight for height tenth percentile) with excoriations on his trunk and extremities. He
has no icterus. A grade 2/6 systolic murmur is heard at the left upper sternal border.
His liver is soft, about 1 cm below the right costal margin and nontender. Spleen was
not palpable. He has diminished but sym- metric deep tendon reflexes in his lower
extremities. Laboratory studies reveal:

Hemoglobin 12.8
Platelet count 239,00
0
AST 129
ALT 134
Alkaline phosphatase 678
GGTP 948
Total bilirubin 0.7
Prothrombin time 13.9
INR 1.2

Which one of the following is the most likely diagnosis?


100

A. Progressive familial intrahepatic cholestasis (e.g., Byler’s disease)


B. Sclerosing cholangitis
C. Niemann-Pick disease, type A
D. Alagille syndrome
E. Alpha-1-antitrypsin deficiency

290. All of the following statements about hepatitis E are true, except

A. Outbreaks of hepatitis E tend to be very large because of the high rate


of secondary (case-to-case) spread
B. Cases of hepatitis E in the United States are rare
C. Infection with hepatitis E virus (HEV) in pregnancy is associated with high mortality rate
D. Anti-HEV appears to be protective, and prospects for developing a vaccine are good
E. HEV is not closely related in structure or function to any of the other viral hepatitis agents
101

291. All of the following agents are effective for both induction of remission and
maintenance of remission in patients with ulcerative colitis except

A. Oral mesalamine formulations including Asacol and Pentasa


B. Sulfasalazine
C. Olsalazine (Dipentum)
D. Rectal mesalamine (Rowasa enemas and suppositories)
E. No exception

292. Which one of the following statements is false with respect to the use of
cyclosporine in patients with inflammatory bowel disease?

A. Cyclosporine administered as a continuous intravenous infusion at a high dose


of 4 mg/kg/d is effective for severely active ulcerative colitis
B. Cyclosporine administered orally at a dose of 5 mg/kg/d is ineffective for the
induction of improvement or remission in patients with active Crohn’s
disease
C. Cyclosporine administered orally at a dose of 5 mg/kg/d is ineffective for
maintenance of remission in patients with Crohn’s disease
D. Cyclosporine is slow acting and thus is not useful as a bridge therapy to other
slower acting medications such as 6-mercaptopurine, azathioprine, or
methotrexate
E. None of the above

293. A young adult with a life-long history of mild jaundice, but no bilirubinemia or
evidence of chronic hepatitis or hemolysis is likely to have a genetic defect in:

A. Sinusoidal bilirubin uptake pump


B. MRP2 (canalicular multispecific organic anion transporter)
C. Bilirubin-UGT
D. UDP glucuronic acid synthetase
E. Cholesterol 7-á-hydroxylase

294. Alpha- l-antitrypsin deficiency leads to liver injury by way of

A. Uncontrolled proteolytic enzyme activity in the portal tracts


B. Chronic pancreatitis and focal biliary cirrhosis
C. Inability to transport divalent cations into the endoplasmic reticulum
D. Accumulation of abnormal glycoprotein in the liver cells
E. Pulmonary fibrosis and the development of cardiac cirrhosis

295. Which serological test is most commonly abnormal in autoimmune hepatitis?

A. Anti-nuclear antibody (ANA)


B. Anti-smooth muscle antibody
102

C. Anti-LKM antibody
D. Anti-SLA
E. Anti-HCV

296. A patient with photosensitivity is referred to you because of abnormal


transaminases (ALT=120, AST=150). Physical examination shows pigmentation
and blisters on the dorsa of the hands. What results of laboratory testing is the
least likely to be found?

A. Positive hepatitis C antibody


B. High levels of excretion of ALA (aminolevulinic acid) and
porphobilinogen (PBG)
C. Serum ferritin of 300 ng/mL
D. Elevated MCV (mean corpuscular volume)
E. Elevated gamma glutamyl-transpeptidase
103

297. Crigler-Najjar syndrome type II is typically treated by:

A. Liver transplantation
B. Lifelong phototherapy
C. Ursodeoxycholic acid
D. Gene transfer of UGT using adenovirus vectors
E. Phenobarbital

298. A 15 year old boy with decompensated cryptogenic cirrhosis presents with a 2 week
history of increasing anorexia and weakness. Four weeks prior to presentation he
was treated for an episode of spontaneous bacterial peritonitis and was discharged
after 5 days of IV antibiotics on prophylac- tic Bactrim, spironolactone and
furosemide. Physical examination is remarkable for jaundice and ascites.
Laboratory data reveal a serum creatinine of 3.1 mg/dl (4 weeks prior: 0.8 mg/dl), and
BUN of 52 mg/dl (4 weeks prior: 14 mg/dl). At the present time, all of the following
are appropriate steps except:

A. Decrease diuretics by half, liberalize sodium intake and obtain follow-up blood studies.
B. Obtain urinalysis, urine sodium measurement, and urine eosinophil count
C. Renal ultrasound
D. Discontinue diuretics and give saline or colloid challenge
E. Repeat diagnostic paracentesis

299. Hepatitis A occurs in cyclical outbreaks in the


United States. These outbreaks spread largely
because of:

A. HAV infection among injection drug users


B. Promiscuous sexual behavior
C. Infected food handlers
D. Widespread vaccination programs
E. Close personal contact with infected but asymptomatic individuals, particularly
children

300. A previously healthy two year old boy is referred to you for elevation of liver
function tests. When a liver profile was drawn during an episode of fever, his serum
alkaline phosphatase concentration was elevated. He has no recent history of
fractures. His growth and development have been normal. He did not have neonatal
liver disease. Review of symptoms is negative for pruritus, chronic diarrhea, or acholic
stools. His physical examination is normal. Laboratory studies at your institution
confirm the biochemical findings. Serum 25-hydroxy vitamin D levels are
within the normal range.
Which of the following is the most appropriate next step to manage this child?

Patient’s results Normal range


104

Calcium 9.2 mg/dL 8.8-10.7 mg/dL


Phosphorus 4.2 mg/dL 3.0-5.0 mg/dL
Blood urea nitrogen 8 mg/dL 5-20 mg/dL
SGOT 28 IU/L 20-60 IU/L
SGPT 18 IU/L 5-45 IU/L
GGT 12 IU/L 6-20 IU/L
Conjugated bilirubin 0.1 mg/ dL < 0.3 mg/dL
Alkaline phosphatase 2800 IU/L 65-525 IU/L

A. Abdominal ultrasound
B. Liver biopsy
C. Radiographs for rickets survey
D. 1,25 dihydroxy vitamin D level
E. No further laboratory tests
105

301. The following statements regarding the management of foreign bodies in


the stomach are true EXCEPT:

The clinician should consider removing objects that are more than 2 cm in
A.
diameter or more than 5 cm in length, because they are unlikely to pass
through the duodenum.
B. In the case of battery ingestion, levels of heavy metal in the
blood and urine should be measured.
C. Batteries that have passed through the esophagus to the stomach
should always be removed.
D. Between 80% to 90% of ingested foreign bodies that reach the
stomach will pass without specific therapy.
302. Which of the following metabolic alterations is most commonly seen with re-feeding
syndrome?

A. Hyperlipidemia secondary to increased serum ketone bodies


B. Wernicke’s encephalopathy secondary to thiamine deficiency
C. Severe hypophosphatemia affecting myocardial and respiratory function
D. Hypernatremia and hypertonic dehydration affecting mental status

303. Which of the following statements concerning hereditary hemochromatosis (HH) is


false?

A. The phenotypic expression in the United States is 1/200-1/250.


B. HH is one of the most common, identified, genetic disorder in Caucasians.
C. The genetic defect causes an excessive absorption of iron.
D. Compound heterozygosity (C282Y, H63D) accounts for 3-5% of cases.
E. HH should be considered in any male patient with transferrin
saturation greater than 30 percent.

304. What is not a result of ascorbic acid deficiency?

A. Perifollicular hemorrhage
B. Subperiosteal hemorrhage
C. Hyperkeratotic hair follicles
D. Cheilosis

305. An asymptomatic duplication cyst that is found incidentally should be

A. Watched expectantly
B. Excised in a patients under the age of three years
C. Excised regardless of age
D. Undergo MRI and if it communicates with intestinal lumen, then excise
E. Undergo MRI and if it does not communicate with intestinal lumen, then
watch
106

306. With respect to candidal esophagitis which statement is false?

A. It results in dysphagia, substernal pain and odynophagia


B. It can be treated with oral fluconazole
C. It is highly unlikely in the absence of thrush
D. It can be mimicked by cryptococcosis
E. Patients with AIDS may have candidal esophagitis with thrush

307. With respect to herpetic esophagitis which statement is false?

A. It is highly unlikely in the absence of herpetic stomatitis


B. It may be accompanied by drooling, fever and generalized malaise
C. Esophageal biopsy may show multinucleated giant cells
D. It can occur in children with normal immunity
E. Ulcerations may resemble CVM esophagitis
107

308. A previously healthy 2-month-old infant suddenly develops bilious vomiting.


Physical examination reveals an ill-appearing child with abdominal distention and
diminished bowel sounds. Which of the following is the MOST likely diagnosis?

A. Antral web
B. Cholelithiasis
C. Duodenal atresia
D. Malrotation with midgut volvulus
E. Peptic ulcer disease

309. Meckel’s diverticulum

A. Can be lined by ileal mucosa with external serosa but no muscularis layers
B. Can be lined by ileal mucosa, ectopic gastric mucosa and muscularis – serosa layers (*)
C. Can be lined with total gastric mucosa
D. Often has pancreatic rest which ulcerate
E. Can be lined with non-HCL secreting gastric mucosa

310. A toddler presents with edema of the hands, feet, and scrotum. Hypoproteinemia,
lymphocytopenia, and decreased levels or serum albumin, immunoglobulins,
transferrin, and ceruloplasmin are noted. Small bowel contrast study shows thickened
mucosal folds. Characteristic histopathology will most likely reveal

A. Eosinophilia in the lamina propria


B. Dilated lymphatics in the villous tips
C. Neutrophils invading the crypts
D. Basal lymphocytosis
E. Haloed inclusions bodies within the enterocytes

311. You are consulted on a 19 m/o with corrected tricuspid atresia and moderate anasarca.
Upon hearing the history and performing a physical examination you ask for a
screening stool alpha-1-antitrypsin level because you suspect

A. Hypoproteinemia secondary to alpha-1-antitrypsin liver disease


B. Protein losing enteropathy from heart surgery
C. Pulmonary hypertension secondary to alpha-1-antitrypsin deficiency fibrosis
D. Chaperon disease of enterocytes
E. Neutrophil elastase enteropathy

312. Which of the following laboratory findings is NOT likely to be found in a patient
presenting with Small Bowel Bacterial Overgrowth?

A. Elevated D-lactate
B. Macrocytic Anemia
C. Microcytic Anemia
108

D. Elevated Stool pH
E. Hypocalcemia
109

313. A 9-year-old girl with no previous illness is admitted with RLQ abdominal pain of
one month dura- tion, worsening over the last few days. Her pain is constant, non-
radiating, moderate to severe
in intensity, and associated with nausea and vomiting. She has suffered a weight loss
of10 lb. On examination, she has RLQ tenderness without abdominal rigidity,
guarding or rebound pain. Bowel sounds are normal. She has no fever. CBC, serum
chemistries, CRP, amylase and lipase were normal except for mild normocytic
anemia and moderately elevated CRP. Abdominal CT scan with contrast reveals
mesenteric adenopathy measuring 3 cm maximum and an irregular filling defect
involving the terminal ilium. EGD and colonoscopy reveals 3 polypoidal mucosal
lesions in the cecum measuring
2.5 cm maximum. The ileocecal valve is edematous and the ileum is hard to intubate.
Biopsies reveal mild focal active cecitis and normal mucosa in the rest of the colon
and upper GI tract. The most ap- propriate next step is:

A. Small Bowel Series


B. Start Treatment with Steroids
C. Diagnostic laparoscopy
D. TB skin Test
E. Send IBD panel
F. Abdominal/Pelvic Ultrasound

314. Gastric acid secretion is inhibited by

A. Gastrin
B. Secretin
C. Motilin
D. CCK
E. Histamine
110

Answers for Board Review

1. C (Congenital Anomalies of the Esophagus)


2. C (Congenital Anomalies of the Esophagus)
3. B (Deglutition)
4. A (Dysphagia)
5. E (Dysphagia)
6. C (Eosinophilic Esophagitis).
7. B (Eosinophilic Esophagitis).
8. E (Eosinophilic Esophagitis).
9. D (Esophageal Anatomy, Development and Physiology)
10. D (Esophageal Caustic Injury)
11. F (Esophageal Motility)
12. D (Upper GI Bleed)
13. C (Gastroesophageal Reflux and Gastroesophageal Reflux Disease)
14. A (Gastroesophageal Reflux and Gastroesophageal Reflux Disease)
15. E (Gastroesophageal Reflux and Gastroesophageal Reflux Disease)
16. E (Esophageal Caustic Ingestions)
17. D milk-protein intolerance (Colic)
18. B (Food and Waterborne Illness)
19. D (Food and Waterborne Illness)
20. B (Abdominal Masses)
21. A (Abdominal Masses)
22. D (Gastritis)
23. D (H Pylori)
25. C (Pyloric Stenosis)
26. D (Gastric Foreign Body)
27. C (Gastric Foreign Body)
28. A (Gastric Foreign Body)
29. C (Appendicitis)
30. B (Appendicitis)
31. D (Autoimmune Enteropathy)
32. D (Autoimmune Enteropathy)
33. D (Abdominal Pain)
34. A (Abdominal Pain)
35. B (Stomach Congenital Anomalies)
36. B (Stomach Congenital Anomalies)
37. A (Celiac Disease)
38. D (Celiac Disease)
39. C (Small Intestine – Congenital Anomalies)
40. D (Small Intestine – Congenital Anomalies)
41. C (Small Intestine – Congenital Anomalies)
42. B (Necrotizing Enterocolitis)
43. C (Necrotizing Enterocolitis)
111

44. B (Small Intestine – Congenital Anomalies)


45. B (Small Intestine – Congenital Anomalies)
46. B (Small Intestine – Congenital Anomalies)
47. E (Small Intestine – Congenital Anomalies)
48. E (Small Intestine – Congenital Anomalies)
49. D (Small Bowel Bacterial Overgrowth)
50. C (Small Bowel Bacterial Overgrowth)
51. B (Enteric Infections)
52. D (Enteric Infections)
53. A (Enteric Infections)
54. B (Small Intestine – Obstruction)
112

55. D (Tropical Sprue)


56. C (Tropical Sprue)
57. B (Colitis not due to inflammatory bowel disease)
58. D (Colitis not due to inflammatory bowel disease)
59. C (Colitis not due to inflammatory bowel disease)
60. C (Inflammatory bowel disease – Crohn’s Disease)
61. C (Inflammatory bowel disease – Crohn’s Disease)
62. B (Inflammatory bowel disease – Ulcerative Colitis)
63. A (Inflammatory bowel disease – Ulcerative Colitis)
64. B (Colonic Motility)
65. D (Constipation)
66. C (Constipation)
67. A (Constipation)
68. C (Hemorrhoids)
69. C (Hemorrhoids).
70. C (Hirschsprung Disease)
71. B (Other inflammatory lesions of the bowel)
72. A (Other inflammatory lesions of the bowel)
73. C (Other inflammatory lesions of the bowel)
74. E (Polyps)
75. E (Polyps)
76. C (Rectal Prolapse)
77. B (Perianal Disease)
78. B (Perianal Disease)
79. B (Perianal Disease)
80. D (Pseudo Obstruction)
81. B (Biliary Tree Normal Microanatomy)
82. B (Other disorders of the bile ducts)
83. C (Other disorders of the bile ducts)
84. B (Biliary Atresia)
85. E (Biliary Atresia)
86. E (Cholecystitis)
87. C (Cholecystitis)
88. D (Cholecystitis)
89. C (Cholecystitis)
90. A (Cholecystitis)
91. B (Fulminant liver failure)
92. D (Fulminant liver failure)
93. A (Fulminant liver failure)
94. A (Fulminant liver failure)
95. B (Fulminant liver failure)
96. C (Fulminant liver failure)
97. A (Bacterial, parasitic and other infections of the liver)
98. E (Bile acid synthetic defects)
99. B (Bile acid synthetic defects)
113

100. A (Carbohydrate Metabolism)


101. C (Carbohydrate Metabolism)
102. E (Drug Induced Liver Injury)
103. D (Drug Induced Liver Injury)
104. E (Disorders of Lipid Metabolism)
105. B (Disorders of Lipid Metabolism)
106. B (Granulomatous hepatitis)
107. A (Granulomatous hepatitis)
108. C (Granulomatous hepatitis)
109. B (Non alcoholic fatty liver disease and steatohepatitis (NAFLD/NASH))
110. D (Liver transplantation)
111. B (Liver transplantation)
112. E (Liver transplantation)
113. A (Liver transplantation)
114. B (Liver transplantation)
114

115. A (Liver transplantation)


116. D (Peroxisomal Disorders)
117. A (Familial hepatocellular cholestatic disorders)
118. A = 1, B = 3, C = 4, D = 2 (Familial hepatocellular cholestatic disorders)
119. D (Familial hepatocellular cholestatic disorders)
120. E ( Familial hepatocellular cholestatic disorders)
121. C (Acute graft vs Host disease and veno occlusive disease)
122. A (Acute graft vs Host disease and veno occlusive disease)
123. F (Acute graft vs Host disease and veno occlusive disease)
124. B(Acute graft vs Host disease and veno occlusive disease)
125. B (Acute graft vs Host disease and veno occlusive disease)
126. E (Disorders of bilirubin metabolism)
127. A (Jaundice)
128. B (Disorders of amino acid metabolism)
129. D (Liver masses)
130. C (Congenital hepatic infections)
131. E (Congenital hepatic infections)
132. C (Viral hepatitis)
133. C (Viral hepatitis)
134. F (Viral hepatitis)
135. A (Viral hepatitis)
136. B (Neonatal Cholestasis)
137. E (Neonatal Cholestasis)
138. B (Chronic hepatitis - Autoimmune Hepatitis and Crossover Syndromes in Children)
139. A (Chronic hepatitis - Autoimmune Hepatitis and Crossover Syndromes in Children)
140. E (Chronic hepatitis - Autoimmune Hepatitis and Crossover Syndromes in Children)
141. D (Pancreas - Normal anatomy, development and physiology)
142. B (Pancreas - Normal anatomy, development and physiology)
143. B (Acute Pancreatitis)
144. D (Pancreas – Exocrine Function)
145. B (Pancreas – Exocrine Function)
146. A (Pancreas – Exocrine Function)
147. A (Congenital anomalies of the pancreas)
148. B (Congenital anomalies of the pancreas)
149. A (Shwachman-Diamond syndrome)
150. A (Nutritional consequences of cholestasis)
151. C (Nutritional consequences of cholestasis)
152. C (Congenital enzyme and transport defects)
153. C (Congenital enzyme and transport defects)
154. B (Congenital enzyme and transport defects)
155. D (Congenital enzyme and transport defects)
156. B (Normal digestion and absorption)
157. D (Normal digestion and absorption)
158. A (Normal digestion and absorption)
159. A (Normal digestion and absorption)
115

160. B (Normal digestion and absorption)


161. D (Normal digestion and absorption)
162. C (Disaccharidase deficiency)
163. B (Disaccharidase deficiency)
164. A (Comparison of human milk and cow-milk based formulas)
165. C (Malnutrition)
166. D (Vitamin and mineral absorption, function and deficiency states)
167. A (Vitamin and mineral absorption, function and deficiency states)
168. C (Nutritional requirements of pre-term and term infants, children and adolescents)
169. D (Nutritional requirements of pre-term and term infants, children and adolescents)
170. C (Obesity)
171. C (Nutritional Therapy)
172. B (Nutritional Therapy)
173. D (Nutritional Therapy)
174. B (Vitamin and mineral absorption, function and deficiency states)
116

175. D (Vitamin and mineral absorption, function and deficiency states)


176. B (Vitamin and mineral absorption, function and deficiency states)
177. C (Vitamin and mineral absorption, function and deficiency states)
178. C (Vitamin and mineral absorption, function and deficiency states)
179. A (Vitamin and mineral absorption, function and deficiency states)
180. C (Vitamin and mineral absorption, function and deficiency states)
181. D (Vitamin and mineral absorption, function and deficiency states)
182. D (Vitamin and mineral absorption, function and deficiency states)
183. A (Vitamin and mineral absorption, function and deficiency states)
184. C (Alkaline phosphatase)
185. A (Endoscopy)
186. B (Endoscopy)
187. C (Breath tests)
188. D (Intestinal Biopsy)
189. C (Intestinal Biopsy)
190. D (Pancreatic Function Testing)
191. C (Pancreatic Function Testing)
192. C (Radiologic Evaluations)
193. D (Radiologic Evaluations)
194. C (Radiologic Evaluations)
195. B (Stool Testing)
196. C (Stool Testing)
197. B (Anti-diarrheals)
198. C (Anti-pruritic agents)
199. B (Blood Replacement)
200. D (Fluid Therapy)
201. D (Anti-rejection and anti-inflammatory)
202. C (Anti-rejection and anti-inflammatory)
203. B (Anti-rejection and anti-inflammatory)
204. E (Biologics)
205. B (Biologics)
206. A (Biologics)
207. C (Biologics)
208. D (Pancreatic Enzymes)
209. B (Pancreatic Enzymes)
210. A (Acid Control)
211. C (Acid Control)
212. A (Prebiotics/Probiotics)
213. D (Prebiotics/Probiotics
214. E (Prebiotics/Probiotics)
215. A (Prostaglandins)
216. A (Sucralfate)
217. C (Sucralfate)
218. A (Sucralfate)
219. C (Munchausen by proxy syndrome)
117

220. D (Munchausen by proxy syndrome)


221. D (Rumination)
222. B (Chronic Diarrhea)
223. C(Chronic Diarrhea)
224. D (Chronic Diarrhea)
225. D (Drug induced bowel injury)
226. C (Drug induced bowel injury)
227. D (GI manifestations of immunodeficiency).
228. C (GI manifestations of immunodeficiency)
229. D (GI manifestations of immunodeficiency)
230. C (GI manifestations of immunodeficiency)
231. C ( GI manifestations of endocrine disorders)
232. C (GI manifestations of endocrine disorders)
233. D (GI manifestations of endocrine disorders)
234. C (Graft versus host disease)
118

235. B C (Ethics)
236. D (Ethics)
237. C (Study Design and Statistics)
238. Validity- B; Reliability- C; Accuracy- E; Precision- F;
Kappa statistic-A; Cronbach’s alpha- D (Study Design
and Statistics)
239. C (Study Design and Statistics)
240. D (Study Design and Statistics)
241. A (Study Design and Statistics)
242. A (Study Design and Statistics)
243. E (Abdominal Pain)
244. C (Prostaglandins)
245. B (Abdominal Pain)
246. B (GI Manifestations of Endocrine Disease)
247. E (Endoscopy)
248. D (Stool Testing)
249. D (Prostaglandins)
250. E (Drug Induced Bowel Injury)
251. B (TPN)
252. C (Acute Liver Failure)
253. B (Schwachman Diamond Syndrome)
254. D (Enteric Infection)
255. A (Neonatal Cholestasis)
256. C (Malnutrition)
257. E (Wilson’s Disease)
258. C (GI Manifestations of Immune Deficiency)
259. B (Hirschsprung Disease)
260. E (Normal Digestion and Absorption)
261. C (Essential Fatty Acids)
262. C (Vitamin and Mineral Absorption, function and deficiency states)
263. D (TPN)
264. E (Nutritional Assessment)
265. E (Vitamin and Mineral Absorption, function and deficiency states)
266. B (Vitamin and Mineral Absorption, function and deficiency states)
267. A (Normal Digestion and Absorption)
268. A (Infant Formula)
269. B (Glycogen Storage Disease)
270. E (Disorders of Lipid Metabolism)
271. D (Irritable Bowel Syndrome)
272. C (Hematologic Manifestations of GI Disease)
273. A (Wilson’s Disease)
274. E (Hepatomegaly)
275. D (Glycogen Storage Disease)
276. D (Constipation)
277. E (Acute Pancreatitis)
119

278. C (Constipation)
279. B (Wilson’s Disease)
280. B (Vitamin and Mineral Absorption, function and deficiency states)
281. C (Malnutrition)
282. A (Tyrosinemia)
283. C (Viral Hepatitis)
284. A (GI manifestations of Immunodeficiency)
285. C (GI manifestations of Immunodeficiency)
286. C (GI manifestations of Immunodeficiency)
287. B (Hirschsprung Disease)
288. C (Abetalipoproteinemia)
289. D (Alagille syndrome)
290. A (Viral Hepatitis)
291. E (IBD – Ulcerative Colitis)
292. D (IBD – Crohn’s Disease)
293. C (Disorders of bilirubin metabolism)
120

294. D (Alpha-1-antitrypsin deficiency)


295. A (Chronic Hepatitis)
296. B (Porphyria)
297. E (Bilirubin Metabolism disorders)
298. A (Liver Transplant)
299. E (Viral Hepatitis)
300. E (Alkaline Phosphatase)
301. B (Gastric Foreign Body)
302. C (Malnutrition)
303. E (Iron Storage Disease)
304. D (Vitamin and Mineral Absorption, function and deficiency states)
305. C (Congenital Anomalies of Small Intestine)
306. C (Infectious Esophagitis)
307. A (Infectious Esophagitis)
308. D (Congenital Anomalies of Small Intestine)
309. B (Congenital Anomalies of Small Intestine)
310. B (Protein-losing Enteropathy)
311. B (Protein-losing Enteropathy)
312. D (Small bowel bacterial Overgrowth)
313. C (Abdominal Mass)
314. B (Stomach Anatomy, Development and Physiology)
Review Questions
1. A 3-year-old girl presents to your office with 3 months of diarrhea. She is
otherwise well, and her growth has been normal. Dietary history reveals low
fiber intake and high juice intake. What is the first step in her management?
a. Initiation of a BRAT diet
b. Elimination of juice and other sugar-sweetened foods
and beverages from her diet
c. Screening laboratories
d. Stool pH and reducing substances
e. 24 hours of oral rehydration solution only
1. b. This 3-year-old girl has diarrhea but has continued to have normal growth. As a
result, a true malabsorptive or inflammatory process is less likely. Given that
she has a high amount of juice intake, this provides ahigh osmotic load to the intestine
and results in toddler's diarrhea. The first step in her management is minimally
invasive and involves a trial of eliminated sugar-sweetened foods and beverages from
her diet to see whether the diarrhea resolves. Her process is chronic, and treating it as
an acute process with 24hours of oral rehydration solution will not help long
term. A BRAT diet is no longer recommended for diarrhea,but especially not for
chronic diarrhea. Screening laboratories and stool studies can be obtained but are
likely to be low yield in this setting.
2. You have been seeing a 16-year-old girl with Crohn disease for several years. She
recently had an ileocecectomy and has remained on medical treatment after surgery.
She reports that after surgery, she has been having increased stool frequency but is
otherwise feeling well.Colonoscopy and MRE before surgery demonstrated
that the areas of active disease were limited to the ileum and cecum. What is the most
likely etiology of her diarrhea?
a. Clostridium difficile colitis
b. Short bowel syndrome
c. Bile acid diarrhea
d. Active Crohn disease
e. Pancreatitis
2. c. This patient has Crohn disease, and evaluation before surgery showed that active
disease was limited to the ileum and cecum. This portion of the bowel was
resected, and she should no longer have active Crohn disease. Only a small segment
was resected, and she should not have short bowel syndrome. Though possible,
it would be unusual for her to develop Clostridium difficile colitis or pancreatitis
immediately after surgery.Additionally, pancreatitis should not give her diarrhea.
After resection of the ileum, she has less reuptake of bile acids, which can lead to an
osmotic diarrhea. She developed diarrhea immediately after surgery, which
makes this most likely.

3. A 3-year-old boy comes to your office due to chronic


emesis. He has previously been healthy. Further history
reveals that emesis tends to occur in the morning.
His medical record notes dysmetria on examination last
week, but this was attributed to his young age. What are
your next steps?
a. Physical examination with thorough neurologic
evaluation and fundoscopic examination
b. Upper GI to rule out anatomic abnormality
c. Trial of acid suppression
d. Screening laboratories
e. Referral to neurology
3. c. This 3-year-old boy has morning emesis, which immediately should trigger
concern for possible causes of increased intracranial pressure. Neurologic
examinations can be tough on young patients, but dysmetria was recently noted. The
next best step would be a thorough neurologic examination, including a fundoscopic
examination. If there is concern for increased intracranial pressure, head imaging is
warranted. An upper GI,trial of acid suppression, and screening laboratories
may be helpful, but a neurologic examination needs to be perf armed first to help
narrow the differential for emesis. Referral to neurology seems premature, and
if there is concern for increased intracranial pressure,more emergent attention is
needed.

4. A 2-week-old male is transferred to your hospital due to


the presence of profuse diarrhea and an inability to gain
weight since birth. What is the first step in your diagnostic
evaluation?
a. Upper endoscopy with biopsies
b. Trial of pancreatic enzymes
c. Observe whether diarrhea persists or stops while
patient is fasting
d. Upper GI with small bowel follow-through to evaluate
for congenital short bowel syndrome
e. Celiac panel
4. c. This patient has congenital diarrhea, which has along list of possible etiologies.
The first and least invasive step is to see what happens to the diarrhea if the
patient is not fed. This will help distinguish between osmotic and secretory diarrhea,
which will help narrow the differential. An upper endoscopy or an upper GI
may be needed in the future. A celiac panel will not be helpful because this infant
would not be exposed to gluten to have celiac disease. A trial of pancreatic enzymes
is also less likely to be helpful until it has been established that there is a reason for
pancreatic insufficiency.
5. Which of the following results is not consistent with carbohydrate
malabsorption?
a. Stool pH <5.5
b. Positive stool-reducing substances
c. Positive hydrogen breath test after administration of
lactose
d. Stool osmotic gap <SO mOsm/L
e. Increased flatus
5. d. When carbohydrates are not absorbed, they are fermented by bacteria present in
our gastrointestinal tract, which leads to increased gas production. The increased
production of gas, including hydrogen and methane, leads to a positive hydrogen
breath test and also increased flatus. The breakdown of carbohydrates
by bacteria also results in the production of short-chain fatty acids, which decreases
stool pH to less than S.S. Stool-reducing substances are also an indicator of
carbohydrate malabsorption. Stool osmotic gap is used to distinguish osmotic and
secretory diarrhea. It does not indicate which dietary component is malabsorbed.
6. A 2-month-old baby boy presents to the emergency room
with a week of large-volume nonbilious emesis after
every meal. Physical examination reveals an olive-like
mass in the upper abdomen. His abdominal x-ray demonstrates
a distended stomach. Which of the following
electrolyte abnormalities is this baby most likely to have?
a. Hypochloremic, hypokalemic metabolic alkalosis
b. Hyperchloremic, hypokalemic metabolic acidosis
c. Hypochloremic, hyperkalemic metabolic alkalosis
d. Hypochloremic, hyperkalemic metabolic acidosis
e. Hyperchloremic, hyperkalemic metabolic alkalosis
6. a. In any form of gastric outlet obstruction, classically due to pyloric stenosis, loss
of hydrochloric acid from the stomach results in hypochloremia and a metabolic
alkalosis. In trying to compensate for volume contraction and metabolic alkalosis, the
kidneys retain sodium and excrete potassium and hydrogen ions. The patient
will have a hypochloremic, hypokalemic metabolic alkalosis.
7. A female full-term neonate has been vomiting copious
amounts of breast milk since birth. There was a maternal
history ofpolyhydramnios. What is the best next step?
a. Two-view abdominal x-ray
b. Continue to try to feed breast milk
c. Surgical consult
d. Placement of nasogastric tube for feeding
e. Abdominal ultrasound
7. a. This patient has had high-volume emesis since birth.Additionally,
polyhydramnios in utero may suggest a GI tract obstruction that did not allow the
fetus to swallow amniotic fluid. GI tract obstruction needs to be considered in this
situation, and a two-view abdominal x-ray can serve as an initial screen for this.
Continuing to feed orally or via nasogastric tube will result in more emesis. A surgical
consult is likely needed, but further assessment can be done first. An abdominal
ultrasound is not likely to be as helpful to evaluate for GI tract obstruction in this
scenario.
8. A teenage girl presents to your office with diarrhea for the past 6 weeks. She
reports that ever since she came home from a camping trip, she acutely developed
diarrhea.Upon further questioning, she tells you that she drank from a stream. She has
been having abdominal pain. There is no blood or mucus in the stool. She has
had weight loss. Her abdominal examination is notable for generalized abdominal
tenderness. What is the most likely cause of her symptoms?
a. Lactose intolerance
b. Celiac disease
c. Inflammatory bowel disease
d. Infectious gastroenteritis
e. Irritable bowel syndrome
8. d. This teenage girl presents with chronic diarrhea of acute onset after a camping
stream with exposure to contaminated drinking water. An infectious etiology,
especially Giardia, is most likely in this scenario. Celiac disease and inflammatory
bowel disease can both present as chronic diarrhea with abdominal pain, but they
tend to have a gradual onset. Irritable bowel syndrome can also present with diarrhea,
but it does not tend to cause weight loss within a short period of time.
9. A 3-month-old baby girl presents to your office with nonbilious emesis that started
soon after birth. She vomits after most feeds. She is happy, developing normally, and
gaining weight well. What are the next steps in management?
a. Upper endoscopy with biopsies
b. Change to an elemental formula
c. Abdominal ultrasound
d. Initiation of metoclopramide
e. Reassurance
9. e. This baby is a "happy spitter" and has reflux, the most common cause of emesis
in this age group. She has emesis with every feed but is feeding well and growing
well. Further testing or a formula change is not warranted because this is a problem
that should improve with time. Testing is likely to be normal.Though initiation of
ranitidine could be considered, it is not necessary. Certainly, a physician would not
start with metoclopramide for emesis due to a higher side effect profile than
ranitidine. The best answer is reassurance that the infant will outgrow the reflux with
time.

10. A 15-year-old boy presents to your office with several months of diarrhea. Upon
further questioning, he reveals that he has been having right lower quadrant
abdominal pain and blood in his stool. His energy level is low. His growth chart
demonstrates weight loss. On examination, he has focal right lower quadrant
tenderness and pallor. Diagnostic testing is most likely to reveal which of the
following:
a. Low fecal elastase
b. Elevated stool pH
c. Elevated fecal calprotectin
d. Elevated serum creatine kinase
e. Elevated serum gastrin
10. c. This patient presents with chronic diarrhea, localized right lower quadrant pain,
blood stool, weight loss, and fatigue. His clinical picture is most consistent
with inflammatory bowel disease. His fatigue is suggestive of active disease but also
anemia. His localized right lower quadrant pain suggests ileocecal disease,
Review Questions
1. A 10-week-old ex-38-week exclusively breastfed infant presents to
your outpatient pediatric office for asick visit. Her mother reports that she
has had browncolored emesis for the past 24 hours. She is otherwise
well-appearing. She has a normal physical examination in your office;
however, she has an episode of darkbrown emesis while you are in the
room. A Gastroccult test of the emesis is positive for blood. What is the
most likely etiology of this brown-colored emesis?
a. N ecrotizing enterocolitis
b. Swallowed maternal blood
c. Volvulus
d. Gastric vascular lesion
e. Milk protein allergy
1. b. An otherwise healthy breastfed infant with Gastroccult positive
emesis and a normal examination is most likely to have swallowed
maternal blood from a mother's cracked nipple during breastfeeding. Use
of an Apt test can aid in distinguishing whether the blood is the
infant's blood or her mother's blood.
2. A 6-year-old male presents to your office for evaluation of a 3-month
history of abdominal pain and a 2-week history of blood in the stool. He
is passing pellet-like stools approximately every 3 to 4 days. He is
resistant to sitting on the toilet and often cries while he is passing stool.
He reports that the abdominal pain is often relieved with stooling. His
weight for age has remained in the 50th percentile over the past 3 years.
He has a good appetite.He is not on any medications at this time. His
mother has noticed that he has had streaks of bright-red blood on top
of his stools intermittently over the past several weeks.What is the most
likely etiology for the blood in his stool?
a. Anal fissure
b. Juvenile polyp
c. Crohn disease
d. Infectious colitis
e. Intussusception
2. a. Functional constipation is a common cause of abdominal pain in a
toddler or school-aged patient.Children with constipation will often strain
to stool.Chronic straining can lead to anal fissures, which often
result in streaks of bright-red blood in the stool.
3. A 2-year-old previously healthy male presents to the emergency
department with sudden-onset brisk, painless rectal bleeding. Which of
the fallowing studies is most likely to diagnose the source of the
bleeding?
a. Abdominal x-ray
b. Abdominal ultrasound
c. Meckel scan
d. Clostridium difficile toxin testing
e. Hemoccult
3. c. Meckel diverticulum most commonly presents with brisk, painless
rectal bleeding. An abdominal ultrasound and abdominal x-ray imaging
would be unlikely to be helpful in the diagnosis of a Meckel
diverticulum.Clostridium difficile colitis is often associated with
abdominal cramping and diarrhea. Although the hemoccult would be
positive, it would not be diagnostic for this patient.

4. What is the most important initial management step if a child presents


to the emergency department with an acute gastrointestinal hemorrhage?
a. Perform an upper endoscopy
b. Perform a bleeding scan
c. Ensure hemodynamic stabilization
d. Start IV proton pump inhibitor therapy
e. Place a nasogastric tube
4. c. The most important initial management step in an acute
gastrointestinal hemorrhage is to resuscitate the patient and ensure
hemodynamic stabilization.This should be done before any additional
procedures or imaging studies. It should be noted that initial CBC
will likely not be reflective of total volume of blood loss as hemoglobin
measure will not drop until there is re-equilibration of fluid amongst the
body's compartments.Consequently, it is imperative that packed red
blood cells be administered empirically at least at the rate of blood loss,
ideally warmed and with adequate plasma components to prevent
coagulopathy (e.g.,fresh frozen plasma, cryoprecipitate).

5. You are caring for a 12-year-old patient who has a4-year history of
inflammatory bowel disease. She is currently anemic and has been having
intermittently bloody stools for the past 1 month. She also reports
intermittent cramping abdominal pain for the past 1month. She is
currently maintained on oral Pentasa for her inflammatory bowel disease
maintenance therapy.An upper endoscopy and colonoscopy were
performed by her gastroenterologist last week and were both visually
normal. You suspect that she may have a small bowel lesion, and you
would like to do additional testing at this time. What would be the next
study that you or the gastroenterologist should order?
a. Capsule endoscopy
b. Patency capsule
c. Abdominal x-ray
d. Bleeding scan
e. Upper GI endoscopy
5. b. A patient with a several-year history of inflammatory bowel disease
is at increased risk for a posb sible intestinal stricture. Before performing
a capsule endoscopy to look for a small-intestinal lesion, the patient hould
have a patency capsule test to ensure that the capsule will pass through
the intestine. If the patency capsule does not pass through with ease,
then the patient should not proceed with the capsule endoscopy.

6. A false positive hemoccult may be due to:


a. Iron supplements
b. Ascorbic acid
c. Orange juice
d. Redmeat
6. d. Red meat ingestion can yield a false positive hemoccult result. Iron
supplements do not produce a false positive test result. Ascorbic acid and
orange juice can produce a false negative hemoccult result.
7. An 8-year-old previously healthy female presents to the emergency
department after an episode of brightred vomit this morning. She has had
several days of previously nonbloody vomiting, nonbloody diarrhea,
and fever. She has been vomiting approximately four to five times per
day for the past 3 days. Several children in her class are out sick as well.
What is the most likely cause of her bloody vomit?
a. Stress ulcer
b. Esophageal varices
c. Vascular malformation
d. Toxic ingestion
e. Mallory-Weiss tear
7. e. Mallory-Weiss tear is the most common cause of sudden-onset
bloody emesis in an otherwise healthy child who has had several days of
acute vomiting.These tears typically heal spontaneously, and the children
make a full recovery.
8. "Currant jelly" stools are typical of what gastrointestinal
disease process?
a. Intussusception
b. Meckel diverticulum
c. Intestinal duplication
d. Clostridium difficile colitis
e. Inflammatory bowel disease
8. a. "Currant jelly" stools are typical of intussusception.In addition to
this mixed bloody and mucusy stool,intussusception is also often
associated with vomiting,lethargy, and intermittent abdominal pain
episodes.
9. All of the following are acceptable potential endoscopic
interventions to perform to stop a gastrointestinal
hemorrhage, except:
a. Injection of sclerosing agent
b. Injection of epinephrine
c. Thermocoagulation
d. Argon plasma coagulation
e. Injection ofBotulinum toxin
9. e. Injection of sclerosing agent (i.e., ethanolamine oleate), injection of
epinephrine, thermocoagulation,and argon plasma coagulation are all
endoscopy interventions used to stop acute gastrointestinal bleeding.
Injection of Botulinum toxin is not used in gastrointestinal bleeds. Rather,
it can be injected around the pyloric sphincter to relax the sphincter and
allow more food to pass through.

10. A 12-year-old boy presents to your pediatric outpatient office for


evaluation of lack of weight gain over the past 1 year. Upon further
questioning, he reports that he has had some intermittent bloody diarrhea
with bright-red blood mixed in with his stools for the past 3months. He
has also had daily periumbilical abdominal pain for the past several
months. His examination is significant for pallor, diffuse tenderness to
palpation throughout the abdomen with no rebound tenderness and no
guarding, and multiple skin tags around the anus. What is the most likely
diagnosis?
a. Crohn Diseas
b.Ulcerative colitis
c. Juvenile polyps
d. Anal fissure
e. Meckel diverticulum
10. a. Poor weight gain, bloody stool, abdominal pain,and perianal lesions
raise suspicion for Crohn disease.Ulcerative colitis, juvenile polyps, anal
fissure, and Meckel diverticulum are not typically associated with
perianal disease.
Dr-Wahid Helmi

Question 1:
A 14-year-old youth has had intermittent periumbilical and lower abdominal pain, some bloating,
and increased flatus for 2 years. He denies a relationship of the pain to food or drink, and a milk-free
diet did not relieve his symptoms. He has had no fevers, diarrhea, constipation, bleeding, or other
systemic problems and his growth is normal. He eats a regular diet, including diet soft drinks and
sugar-free gum. His family is intact and he denies undue stress in his school or social life. He also
denies alcohol, drug use, or smoking. Physical examination is normal, including heme occult
negative rectal examination. Screening laboratory tests including complete blood count, ESR, liver
enzymes and amylase, and urinalysis are normal. Which one of the following would be the most
likely etiology of the abdominal complaints in this teenager?
A. lactose intolerance
B. excessive sorbitol intake
C. irritable bowel syndrome
D. teenage stresses
E. acid-peptic disease

Suggested answer: B or C. Sorbitol is a FODMAP (fermentable oligo-, di-, and


monosaccharides and polyols), metabolized by colonic bacteria to produce short chain fatty
acids and gas. Sorbitol is commonly found in diet soft drinks and sugar-free gum. When taken
in excess, sorbitol produces enough gas to produce abdominal pain and bloating as in this
patient. Sorbitol can also cause a persistent, osmotic diarrhea.

Irritable bowel syndrome, the most commonly diagnosed gastrointestinal condition, is diagnosed
by the presence of 3 things: i) chronic abdominal pain, ii) altered bowel habits (constipation or
diarrhea, which is may or may not be present in this patient), and iii) no organic cause. The
etiology of IBS in unclear; however, one hypothesis is that bacteria cause IBS, by fermenting
FODMAPs in the colon. FODMAPs have been shown to produce IBS symptoms in clinical
trials. Treatment for IBS is broad, and includes a strong physician-patient relationship,
avoiding certain foods (lactose, allergens, gluten), anti-spasmodics, anti-depressants, anti-
diarrheal agents, and/or antibiotics.

This patient does not have lactose intolerance, as a milk-free diet did not relieve his symptoms.
He should resume drinking milk, or at least receive calcium and Vitamin D supplements to
support growth. In addition, the patient does not report excessive stress, and does not have
typical GERD symptoms.

Question 2:
A 9-year-old girl has been losing hair for about 2 years and now presents with a large mass in her
epigastrum. Her parents separated about 3 years ago. The patient and her siblings have been
spending part of each week at each parent's home. A plain film of the abdomen reveals a large
mass in the stomach. Endoscopy reveals a trichobezoar. What do you now recommend to the
family?
A. administer meat tenderizer orally
B. endoscopic removal of bezoar under anesthesia
C. abdominal CT scan to rule out gastric tumor
D. surgical consultation for removal of bezoar
Dr-Wahid Helmi

E. reassurance that the bezoar will pass

Suggested answer: D. Trichobezoars usually occur in young women with psychiatric disorders,
and in some cases may extend through the small bowel and even cecum (Rapunzel syndrome).
Phytobezoars (from vegetable matter) are responsive to enzymatic dissolution, including
cellulase, papain, and carbonated soda. Trichobezoars are resistant to enzymes and must be
removed manually. Endoscopy can be attempted but often fails, with the endoscopic tools
becoming snared in the strands of hair. Surgical removal is more effective. All patients should
receive psychiatric therapy to prevent the problem from reoccurring.

Question 3:
Among the hepatitis viruses, the hepatitis B virus (HBV) is unique because:
A. liver injury is mediated through the immune system
B. the presence of antibodies to HBV indicates protective immunity
C. it has similarities to human retroviruses because it is a DNA virus which replicates
through an RNA intermediate
D. it is the only agent to cause fulminant hepatic failure
E. it has a glycoprotein coat enclosing a viral nucleocapsid

Suggested answer: C. HBV is a DNA virus that enters the hepatocyte nucleus and is read by
host-cell machinery to make RNA particles. The RNA particles in turn are exported to the
cytoplasm, where HBV-encoded reverse transcriptase converts them to new viral DNA particles.
Infection can be acute and cause fulminant hepatic failure; however, chronic infections
characterized by a series of stages are more common. First, in the “immune tolerant” phase,
the host allows HBV infection and replication as seen by high HBV DNA levels and HBeAg
positivity. Second, in the “immune clearance” phase, the host mounts an immune response
against infected hepatocytes, signified by the appearance of anti-HBe, disappearance of HBe-Ag,
transaminase elevation, and hepatitis symptoms. In some cases, the host clears the virus
completely (anti-HBs appears, HBsAg disappears); in other cases, the virus becomes latent and
can reactivate/stimulate the “immune clearance” phase at unpredictable times. The virus has a
glycoprotein envelop enclosing a proteinaceous nucleocapsid.

HAV is an RNA virus that never enters the nucleus; rather, in the cytoplasm, it makes more
particles using its own RNA-dependent RNA polymerase. Infections are acute and can cause
fulminant hepatic failure. Infections have a biphasic pattern, first with viral replication (akin to
the HBV “immune tolerant” phase”) quickly followed by immune attack of virus and cells
harboring the virus (akin to the HBV “immune clearance” phase, and characterized by
appearance of anti-Hep A IgM and hepatitis symptoms). The virus does not have an envelope,
but does have a proteinaceous nucleocapsid surrounding the RNA.

HCV is an RNA virus like HAV but has an envelope. It also never enters the nucleus, but rather
enters the cytoplasm, takes over the cell’s ribosomal machinery, and converts the cell into a
factory producing viral proteins that generate more viral nucleic acid and particles. Viral
particles then bud off the cells (using the cell’s plasma membrane for an envelope) and infect
other cells. Acute infections are mild or even asymptomatic, and fulminant hepatic failure is
rare. Chronic infections are more common, and occur when the host cannot completely clear
Dr-Wahid Helmi

the infection. This long-term, low-level battle between host and virus leads to liver
inflammation, cirrhosis, and hepatocellular carcinoma.

HDV is an RNA virus with an envelope like HCV; however, the envelope has HBsAg, making
HDV replication possible only in cells also infected with HBV. HDV enters the nucleus, and
host-cell RNA polymerases use the original HDV strand to make more RNA particles. Acutely,
HDV is thought to cause cytopathic damage to hepatocytes and may be confused with HBV
reactivation (see above). Fulminant hepatic failure is possible. Chronically, the host immune
response to HDV infected cells causes hepatocyte damage.

HEV is an RNA virus without an envelope, similar to HAV. The life cycle is less well-
understood, but HEV appears to never enter the nucleus (similar to HAV). Instead, in the
cytoplasm, the virus makes more particles using its own RNA-dependent RNA polymerase.
Infections are usually acute and can cause fulminant hepatic failure. Chronic infections have
only been reported in solid-organ recipients taking post-transplant immunosuppression.

Question 4:
Common routes of the spread of hepatitis A virus include all of the following except:
A. consumption of contaminated water or food
B. close personal contact
C. infants in daycare centers
D. homosexual men by sexual contact
E. transfusion of packed red blood cells

Suggested answer: E. Hepatitis A is shed in bile, found in stools, and spread fecal-orally.
Hence, it can be spread through consumption of contaminated water/food, close personal
contact (without proper hand-washing), infants in close proximity in a day care, and other close
contact such as anal-oral sexual contact. Interestingly, infants that acquire HAV infection have
a less severe course, perhaps because of a less developed immune system. HAV is not spread
through blood transfusions.

Question 5:
Serological changes associated with being a healthy carrier of the hepatitis B virus are:
A. HBsAg positive, HBeAg positive, HBV DNA negative by hybridization assay, with
normal serum aminotransferases
B. HBsAg positive, HBV DNA negative by hybridization assay, anti-HBe positive, with
normal serum aminotransferases
C. HBsAg negative, anti-HBs positive, anti-HBe positive
D. HBsAg negative, anti-HBs negative, anti-HBe positive
E. HBsAg positive, HBV DNA positive by hybridization assay, HBeAg negative, with only
slightly elevated serum aminotransferases

Suggested answer: E. Inactive carries are those who were once infected with Hepatitis B and
mounted a successful immune response against it. During the infection or “immune tolerant”
phase, these patients had HBV antigens and DNA in their serum (HBsAg positive, HBeAg
positive, HBV DNA virus positive) but have not yet mounted an immune response (anti-HBe
Dr-Wahid Helmi

negative as well as anti-HBs negative). During the immune response or “immune clearance”
phase, these patients developed antibodies against the virus (anti-HBe positive) as well as a
clinical picture of hepatitis (elevated transaminases). Their immune response is effective but not
complete, rendering them carriers with evidence of the virus still present (HBsAg positive, HBV
DNA hybridization positive) but also evidence of a successful response (anti-HBe positive
causing HBeAg negativity). Carriers may also have slightly elevated aminotransferases. Had
their immune response been complete, the patients would be cured of the disease and be anti-
HBs positive, HBsAg negative, anti-HBe positive, HBeAg negative, and HBV DNA negative.

Question 6:
All of the following statements about hepatitis E are true, except:
A. outbreaks of hepatitis E tend to be very large because of the high rate of secondary (case-
to-case) spread
B. cases of hepatitis E in the United States are rare
C. infection with hepatitis E virus (HEV) in pregnancy is associated with high mortality rate
D. anti-HEV appears to be protective, and prospects for developing a vaccine are good
E. HEV is not closely related in structure or function to any of the other viral hepatitis
agents

Suggested answer: A. While the seroprevalence of HEV is ~20% in the US, symptomatic HEV
infection is rare (perhaps because most infections may be with the less virulent genotype 3).
HEV is acquired through contaminated water and rarely spreads person-to-person, making it
less readily transmissible than HAV. Following HEV infections, humans develop anti-HEV IgM
and then anti-HEV IgG. There are no HEV vaccines readily available yet, though some are
being developed. HEV has a unique genome and structure and is the only member of the genus
hepevirus in the family Hepeviridae.

HEV is most dangerous during pregnancy. For unclear reasons, women infected in the third
trimester have an increased risk of hepatic failure and mortality (15-25% mortality rate). This
may because the virus replicates faster in the pregnancy state. Furthermore, infants born to
infected mothers are also at risk, with some presenting with massive hepatic necrosis shortly
after birth.

Question 7:
Recognized complications of acute hepatitis A include all of the following except:
A. fulminant hepatic failure
B. relapsing hepatitis C
C. chronic hepatitis
D. cholestatic hepatitis
E. triggering autoimmune hepatitis

Suggested answer: B. HAV most commonly causes an acute, self-resolving hepatitis. However,
it has been associated with a variety of other presentations: i) fulminant hepatic failure (rare);
ii) relapsing, chronic hepatitis characterized by HAV-caused hepatitis interspersed between long
periods without symptoms (prognosis is excellent, with no reports of cirrhosis or chronic liver
disease); iii) cholestatic hepatitis characterized by prolonged jaundice, itching, and laboratory
Dr-Wahid Helmi

abnormalities, which eventually self-resolves with supportive care; and iv) triggering of
autoimmune hepatitis, which develops months after the initial HAV infection.

Question 8:
A 45-year-old woman is undergoing treatment for chronic hepatitis C with the combination of
interferon (3 three times a week) and ribavirin (1000 mg per day). Prior to therapy she was
found to have cirrhosis but there was no evidence of hepatic decompensation. Her baseline blood
count was hemoglobin 13.9 g/dL, MCV 89, total white cell count 4200/mm3, platelet count
92,000/mm3. After 6 weeks of therapy, her blood count was hemoglobin 9.9 g/dL, MCV 102,
total white cell count 2500/mm3, platelet count 47,000/mm3. The best course of action is:
A. decrease the dose of ribavirin to 600 mg/day
B. decrease the dose of interferon to 1.5 three times a week
C. administer recombinant human erythropoietin
D. decrease the dose of both ribavirin (to 600 mg/day) and interferon (to 1.5 three times a
week)
E. check serum levels of folate and vitamin B12 and correct if deficient

Suggested answer: D. This patient is being treated with interferon and ribivarin, which implies
she has either genotype 2, 3, or 4 (genotype 1 is treated with interferon, ribivarin, and a protease
inhibitor). Interferon strengthens the innate and adaptive immune response, whereas ribivarin is
a nucleoside analog.

80% of patients taking interferon and ribivarin have side-effects. Anemia usually appears in the
first 12 weeks, caused by ribivarin-induced hemolysis (ribivarin is concentrated in erythrocytes
leading to oxidative damage) and interferon-related bone marrow suppression to blunt a
compensatory response. Neutropenia and thrombocytopenia occur soon after treatment
initiation and are due to interferon-related bone marrow suppression. Ribivarin dose reduction
would treat the anemia, whereas Interferon dose reduction would treat the thrombocytopenia.
Erythropoietin is another medication that has been shown to improve the anemia, possibly
without needing to reduce the ribivarin dose.

Question 9:
An 18-year-old Asian woman is being treated for hepatitis B. Prior to therapy she was found to
have ALT 198 U/L, AST 91 U/L, normal bilirubin, albumin, and prothrombin time. Liver biopsy
results showed chronic hepatitis B, grade 3, stage 3. After 12 weeks of therapy, serum ALT is
found to have increased to 1,082 U/L, bilirubin 2.1 mg/dL but albumin and prothrombin time
remain normal. Apart from some fatigue, the patient is tolerating interferon well. The best course
of action is:
A. check for antinuclear antibodies and total immunoglobulin level in serum and consider
instituting corticosteroid therapy
B. stop interferon
C. recheck lab work again in 2 weeks time
D. add lamivudine to the regimen
E. ask the patient to skip three scheduled doses of interferon
Dr-Wahid Helmi

Suggested answer: C. Interferon and the antiviral lamivudine are used to treat HBV infection in
children. Interferon is dosed for 16 weeks, has numerous side effects, and leads to serological
conversion (presence of anti-HBe positive and HBeAg negative) in 40% (genotypes A and B) to
5-15% (genotypes C and D) cases. Lamivudine is taken indefinitely and has similar rates of
seroconversion after 2 years (~30-35%). Lamivudine, however, induces viral resistance, with
rates greater than 60% reported after 3 years of use.

This patient is taking interferon. Interferon causes a rise in AST/ALT in 30-40% of cases, likely
because it stimulates the immune system to destroy infected hepatocytes. Hence, this rise is a
sign the medication is working. Had the patient been taking lamivudine, a rise in AST/ALT could
also signify development of viral resistance. Interferon does induce autoantibodies that can be
symptomatic (i.e. hypothyroidism, hyperthyroidism) but this is less common. Interferon should
not be stopped, because the patient’s fatigue is only mild. Adding lamivudine to the interferon
regimen may decrease viral load faster, but has not been shown to produce better virological
outcomes after the 16 week course has finished. (On the other hand, combination therapy with
lamivudine plus interferon, versus lamivudine alone, may prevent development of viral
resistance.)

Question 10:
All of the following are primary cellular components of the immune system and the gut mucosa
except:
A. Peyer's patches
B. lamina propria lymphocytes
C. intrapithelial lymphocytes
D. IgG-secreting B lymphocytes
E. IgA-secreting B lymphocytes

Suggested answer: D. The gut is the largest lymphoid organ in the body. Specialized epithelial
cells, called M cells, sit above Peyer’s patches and sense luminal antigens. The M cells then
stimulate naïve B and T cells in the Peyer’s patches. The naïve cells undergo a long maturation
process, during which they take a long circular course to eventually return to the gut mucosa
(Peyer’s patches  regional lymph nodes  lymphatics  thoracic duct  systemic circulation
 exit in the lamina propria). In the lamina propria, they reside and perform their immune
surveillance functions. In addition, special memory T-cells called intraepithelial lymphocytes
are anchored to the epithelial layer. They respond to a subset of luminal antigens, and secrete
cytokines to mediate the inflammatory response.

B cells that mature into plasma cells ultimately residing in the lamina propria have special
functions. They mainly secrete IgA2 and very little IgM and IgE, but virtually no IgG. IgA2
differs from the IgA1 made by plasma cells in the circulation. IgA2 molecules are dimeric and
secreted into the luminal space only after transiting through epithelial cells (via endocytosis
followed by exocytosis). Furthermore, IgA2 molecules are essentially non-inflammatory because
they bind antigens but do not activate complement. As a result, IgA2 molecules neutralize
antigens without triggering excessive inflammation to the countless number of gut antigens.

Question 11:
Dr-Wahid Helmi

Cells from intestinal lymphoid tissues migrate to all of the following immune tissues except:
A. gastrointestinal mucosal immune tissues
B. pulmonary mucosal immune tissues
C. genitourinary tract
D. lactating mammary glands
E. skin

Suggested answer: E. The “common mucosal immune system” refers to the interconnected
mucosal organs that house IgA2 secreting plasma cells (which initially started as naïve B cells in
Peyer’s patches). These mucosal organs include the lamina propria of the small intestine, the
salivary and lacrimal glands, the lactating mammary glands, the genitourinary tract, and the
lungs. Migration to these areas is mediated by specific cell adhesion molecules.

Question 12:
Which one of the following most accurately reflects the epidemiologic features of inflammatory
bowel disease?
A. the prevalence is approximately 100 cases per 100,000 general population
B. the prevalence is approximately 1,000 cases per 100,000 general population
C. non-Jews are more likely to develop Crohn's disease than Jews
D. people who smoke are less likely to get Crohn's disease
E. people who smoke are more likely to get ulcerative colitis

Suggested answer: A. The epidemiology of inflammatory bowel disease has been well studied.
In North America, UC has prevalence of 27-246 per 100,000 persons, and Crohn disease has a
prevalence of 26-201 per 100,000 persons. There is a negative correlation between smoking and
UC, and a positive correlation between smoking and Crohn disease recurrence. People of
Jewish descent have a higher risk of developing Crohn disease compared to non-Jews,
highlighting the partly-genetic etiology of the disease.

Question 13:
Which one of the following extraintestinal manifestations of inflammatory bowel disease do not
parallel the course of intestinal inflammation and do not improve in parallel with improvement in
intestinal symptoms?
A. peripheral arthritis
B. apthous ulcers
C. spondylitis and sacroiliitis
D. erythema nodosum
E. uveitis and iritis

Suggested answer: C. Arthritis is the most common extra-intestinal complications of IBD.


Peripheral arthritis usually involves large joints, does not cause synovial destruction, and
parallels the course of intestinal symptoms. Central axial arthritis, such as ankylosing
spondylitis (characterized by back and progressive spinal stiffness) is similar to primary
sclerosing cholangitis in that it does not follow the course of intestinal disease. Treatments for
axial arthritis include NSAIDs (despite concern for inducing worsening intestinal inflammation),
Dr-Wahid Helmi

methotrexate, sulfasalazine, and/or anti-TNF therapy. Apthous ulcers, erythema nodosum,


uveitis, and iritis often (but not always) parallel intestinal disease.

Question 14:
All of the following urinary tract complications may occur as a consequence of Crohn's disease
except:
A. calcium oxalate renal stones
B. calcium phosphate renal stones
C. uric acid renal stones
D. ureteral obstruction due to retroperitoneal fibrosis
E. fistulous communication between the terminal ileum and the bladder

Suggested answer: B. There are two general mechanism by which Crohn disease affects the
renal system. First, transmural inflammation can affect the underlying ureters, causing
occlusion usually on the right side and hydronephrosis. Inflammation can also create fistulas
between inflamed bowel and the bladder, promoting cystitis and urinary tract infections.

Second, Crohn disease is associated with calcium oxalate and uric acid renal stones. Calcium
oxalate stones form when the ileum is affected, preventing proper absorption of fats.
Unabsorbed long-chain fatty acids compete with the insoluble calcium oxalate for calcium.
Without calcium, oxalate binds sodium, becomes soluble, is absorbed by the colon (“enteric
hyperoxaluria”), and eventually re-precipitates as calcium oxalate stones when excreted in the
urine. Calcium is an effective treatment. Uric acid stones form secondary to bicarbonate loss
with diarrhea. With low serum bicarbonate, the kidneys excrete acid in compensation. Uric
acid, which is soluble in alkalotic conditions, precipitates as stones in the low pH urine.

Question 15:
All of the following statements regarding the release of mesalamine (5-aminosalicylate) by the
following delivery systems are true except:
A. the dose form Pentasa releases throughout the small and large intestine beginning in the
duodenum and continuing through to the rectum
B. the dose form Asacol releases beginning in the terminal ileum and cecum and continuing
through to the rectum
C. the dose form olsalazine (Dipentum) releases throughout the colon beginning in the cecum
and extending to the rectum
D. mesalamine administered as a Rowasa suppository extends to the left colon and splenic
flexure
E. no exception

Suggested answer: A. Sulfasalazine is used to treat topical inflammatory bowel disease. It is


metabolized by colonic bacteria into two components: 5-ASA, which has anti-inflammatory
properties, and sulfapyridine, which has anti-bacterial products. More recently 5-ASA products
alone have been developed, because many patients are intolerant to the sulfapyridine component.
Sulfasalazine and 5-ASA behave similarly in trials, suggesting that the sulfapyridine anti-bacterial
function has little significance.
Dr-Wahid Helmi

Because ingested 5-ASA is rapidly absorbed in the jejunum, enemas have been used to deliver
medication directly to the colon. In addition, two delayed release preparations have been made.
The first involves coating 5-ASA with resins or microgranules, which dissolve and release 5-ASA in
settings of pH>7 (distal small bowel and colon). Pentasa and Asacol have this coating. The second
involves dimerizing 5-ASA, so that it is only released after bacterial cleavage (in the colon).
Olsalazine (Dipentum) is an example of this dimer form. Importantly, neither delivery system has
proven more efficacious over the other.

Question 16:
All of the following agents are effective for both induction of remission and maintenance of
remission in patients with ulcerative colitis except:
A. oral mesalamine formulations including Asacol and Pentasa
B. sulfasalazine
C. olsalazine (Dipentum)
D. rectal mesalamine (Rowasa enemas and suppositories)
E. no exception

Suggested answer: E. For mild to moderate ulcerative colitis, sulfasalazine and 5-ASA preparations
have been shown effective in inducing and maintaining remission (compared to placebo controls).
There were no significant differences between sulfasalazine versus 5-ASA, or among the different 5-
ASA preparations. Furthermore, for distal colitis, 5-ASA enemas achieve remission in as many as
90% of cases and maintain remission in as many as 75% of cases. When combined, oral plus enema
therapy is more efficacious in inducing and maintaining remission when compared to either agent
alone.

Question 17:
All of the following side effects associated with azathioprine and 6-mercaptopurine are idiosyncratic
reactions except:
A. pancreatitis
B. leukopenia
C. fever
D. rash

Suggested answer: B. Idiosyncratic reactions are those that occur rarely, are unpredictable, and
are dose-independent. For AZA and 6-MP use, these include pancreatitis, fever, rash, and
pneumonitis. Leukopenia, on the other hand, is an intended side-effect used to reduce host cell
inflammation. Leukopenia occurs when the 6-MP metabolites are shunted away from TPMT-
mediated production of 6-MMP (which causes hepatotoxicity). Instead, 6-MP is metabolized
through a different pathway to 6-TG. 6-TG accumulates in tissues, inhibits purine metabolism and
subsequent DNA/RNA synthesis, and prevents lymphocyte proliferation. If too much, 6-TG can
cause severe leukopenia and dangerous host immunosuppresion.

Question 18:
All of the following statements are true with respect to the treatment of inflammatory bowel
disease with azathioprine and 6-mercaptopurine except:
Dr-Wahid Helmi

A. controlled clinical trials have demonstrated that azathioprine doses of 2.0-2.5 mg/kg/d
and 6-mercaptopurine at doses of 1.0-1.5 mg/kg/d are effective
B. these agents are thought to be relatively slow acting requiring up to 3 months or more to
reach the full clinical effect
C. these agents are effective for the treatment of Crohn's disease including patients with
chronically active disease, patients with fistulas who are steroid dependent and patients
who require maintenance of remission therapy
D. these agents are not useful in patients with ulcerative colitis for maintenance of remission
E. none of the above

Suggested answer: D. Both 6-MP and its pro-drug AZA have been used successfully to treat UC
and Crohn disease. AZA’s molecular weight is >2 times that of 6-MP, and ~88% of AZA
converts to 6-MP, so dosing for AZA is ~2 times that of 6-MP. The drugs require 3-6 months to
achieve their full effect, so they are better at maintaining – rather than inducing – remission.
They have been tested numerous times in clinical trials, with results supporting their ability to
increase remission maintenance by ~40-70% and decrease corticosteroid requirements in ~70%
of patients with corticosteroid-dependent disease.

Question 19:
All of the following statements regarding the use of methotrexate for the treatment of
inflammatory bowel disease are true except:
A. methotrexate administered as a 25 mg intramuscular dose weekly is effective for inducing
remission in patients with active Crohn's disease and for steroid sparing
B. methotrexate administered at doses of 15-25 mg/week orally or intramuscularly is
effective for induction of improvement and remission in patients with active ulcerative
colitis
C. there is no controlled clinical trial data to indicate that methotrexate is effective for
maintenance of remission in patients with Crohn's disease
D. there is no controlled clinical trial data to indicate that methotrexate is effective for
maintenance of remission in patients with ulcerative colitis
E. no exception

Suggested answer: B and C. Methotrexate blocks inflammation, perhaps by inhibiting


methylation reactions vital for immune cells to function and proliferate. Methotrexate has been
best studied in Crohn disease, with results seen with the intramuscular (versus oral) form. In
Crohn disease, intramuscular methotrexate has been found to induce remission (25 mg q week)
and maintain remission (15 mg qweek) in clinical trials. Studies with oral preparations have
been less promising, and positive controlled clinical trial data for UC is lacking. Methotrexate
side effects include nausea, vomiting, and abdominal distress, relieved in part by folic acid.
Chronic methotrexate can also cause liver toxicity, but the doses given for IBD are not high
enough to warrant surveillance liver biopsies.

Question 20:
Which one of the following statements is false with respect to the use of cyclosporine in patients
with inflammatory bowel disease?
Dr-Wahid Helmi

A. cyclosporine administered as a continuous intravenous infusion at a high dose of 4


mg/kg/d is effective for severely active ulcerative colitis
B. cyclosporine administered orally at a dose of 5 mg/kg/d is ineffective for the induction of
improvement or remission in patients with active Crohn's disease
C. cyclosporine administered orally at a dose of 5 mg/kg/d is ineffective for maintenance of
remission in patients with Crohn's disease
D. cyclosporine is slow acting and thus is not useful as a bridge therapy to other slower acting
medications such as 6-mercaptopurine, azathioprine, or methotrexate
E. none of the above

Suggested answer: D. Cyclosporin binds cyclophilins to inhibit calcineurin and subsequent


transcription of pro-inflammatory cytokines. Cyclosporin has a rapid onset of action, allowing it
to be used as a bridge to 6-MP, AZA, or methotrexate therapy. In ulcerative colitis, cyclosporin
(4 mg/kg/day continuous infusion) can prevent colectomy in severe, steroid-refractory cases.
More recent data shows that lower dose infusions (2 mg/kg/day) may have similar potency with a
possible (but not proven) decrease in side-effects. On the other hand, in Cohn disease, low dose
oral cyclosporin (5 mg/kg/day) does not induce remission and higher doses and/or parenteral
administration have yet to be properly tested. Cyclosporin does have significant side effects
limiting its use, including opportunistic infections, nephrotoxicity, seizures (especially if
cholesterol levels are low), peripheral neuropathy, and anaphylaxis.

Question 21:
The gene product of the hemochromatosis locus, HFE is:
A. a component of the ferritin complex which stores iron
B. a surface molecule that associates with the transferrin receptor
C. a divalent cation transport protein
D. a component of the endoplasmic reticulum
E. a subunit of the apotransferrin molecule

Suggested answer: B. Hereditary hemochromatosis is caused by mutations in the HFE (high Fe)
locus. HFE is a transmembrane protein expressed in intestinal crypt and liver cells. It controls
liver absorption in two proposed ways:

i) expressed in crypt cells  in crypts, binds transferrin receptor and promotes uptake of
transferrin-bound iron from the circulation  enterocytes have high transferrin bound
iron  enterocytes down-regulate the apical iron transporter DMT1 (“divalent metal
transporter 1”, a divalent cation transport protein)  enterocytes migrate up to villi but
without DMT1 cannot absorb iron
ii) expressed in liver cells  increases liver hepicidin expression  liver hepicidin travels
to enterocytes and downregulate basal transporter ferroportin  without ferroportin,
iron is absorbed by villi enterocytes but cannot be transported into the circulation 
enterocytes sloughed into lumen and absorbed iron lost

Question 22:
The liver disease that results from mutations in the Wilson's disease gene is associated with
which one of the following?
Dr-Wahid Helmi

A. inability to take up copper from the plasma ceruloplasmin pool


B. dysfunction of a copper transporter that is embedded in the canalicular membrane
C. inability to transport copper from the cytosol to the trans-Golgi network
D. copper-dependent antibody mediated immune reactions
E. abnormal primary structure of the ceruloplasmin molecule

Suggested answer: C. Wilson’s disease is caused by a mutation in the copper-transporting


adenosine triphosphatase (ATPase) gene (ATP7B). ATP7B has at least two functions: i)
coordinates transport of copper from the cytoplasm, through the trans-Golgi network to vesicles,
and eventually out into the bile cannaliculi via exocytosis; and ii) promotes the binding of
copper with apoceruloplasmin to form ceruloplasmin, which is secreted into the bloodstream.
Without ATP7B, copper is neither excreted into the bile nor the circulation (via ceruloplasmin).
As a result, copper levels rise in hepatocytes, ultimately leading to oxidative damage and spilling
of copper into the circulation.

Question 23:
A young adult with a life-long history of mild jaundice, but no bilirubinemia or evidence of
chronic hepatitis or hemolysis is likely to have a genetic defect in:
A. sinusoidal bilirubin uptake pump
B. MRP2 (canalicular multispecific organic anion transporter)
C. bilirubin-UGT
D. UDP glucuronic acid synthetase
E. cholesterol 7-á-hydroxylase

Suggested answer: C. This patient has Gilbert’s syndrome, which is caused by reduced uridine
diphosphoglucuronate glucuronosyltransferase (UGT) activity. Normally, hepatocytes take up
unconjugated bilirubin through the sinusoidal space. Hepatocytes then conjugated the bilirubin
using bilirubin-UGT and excrete it into the cannalicular space. This process is independent of
bile acid secretion, so isolated defects in bilirubin production/secretion should not lead to the
liver damage often seen with defects in bile acid secretion.

The bilirubin conjugation process can be impaired at many steps. First, hepatocyte uptake of
unconjugated bilirubin can be blocked by drugs. Second, hepatocyte conjugation can be
impaired by defects in UGT activity. Gilbert’s syndrome is caused by mutations in the
regulatory elements controlling UGT expression and has mild, jaundice phenotypes. Crigler-
Najjar syndrome, on the other hand, is caused by mutations in the gene itself, and can result in
severe kernicterus-like phenotypes. Third, hepatocytes can have impaired secretion of bilirubin
after it is conjugated. Dubin-Johnson syndrome results from mutations in MRP-2, which
transports conjugated bilirubin from the cytoplasm to the cannalicular space. Rotor syndrome
results from defective hepatocyte storage of conjugated bilirubin, leading to leakage into the
cytoplasm.

Question 24:
Alagille syndrome is differentiated from other causes of cholestasis by:
A. typical facies and cardiovascular abnormalities
B. high unconjugated bilirubin levels
Dr-Wahid Helmi

C. Unusually high transaminases plus high alkaline phosphatase


D. high hepatic copper content
E. the frequent finding of coexistent pulmonary fibrosis

Suggested answer: A. Alagille’s syndrome is an autosomal dominant disease in the Notch


signaling pathway. It has an incidence of 1:40,000 to 1:100,000, with variable penetrance and
variable presentation. Common findings include paucity of bile ducts, characteristic facies
(triangular face with pointed chin and broad forehead), pulmonic stenosis, Tetrology of Fallot,
posterior embryotoxin, and butterfly vertebrate. Conjugated bilirubin, not unconjugated
bilirubin levels, can be high, with pruritis often the most bothersome symptom. High hepatic
content is more characteristic of Wilson’s disease (though Alagille’s patients may have
increased copper staining on biopsies, secondary to poor excretion), and Hepatitis C infection is
associated with increased risk of idiopathic pulmonary fibrosis.

Question 25:
Alpha- l-antitrypsin deficiency leads to liver injury by way of:
A. uncontrolled proteolytic enzyme activity in the portal tracts
B. chronic pancreatitis and focal biliary cirrhosis
C. inability to transport divalent cations into the endoplasmic reticulum
D. accumulation of abnormal glycoprotein in the liver cells
E. Pulmonary fibrosis and the development of cardiac cirrhosis

Suggested answer: D. A1AT “deficiency” refers to a deficiency of A1AT in lung tissue, leading
to unchecked elastase activity and pulmonary damage over many years. A1AT is produced in
and secreted by hepatocytes. However, in many forms of A1AT such as Z and M alleles, a
misfolded protein is made that polymerizes in the endoplasmic reticulum, cannot be secreted,
and accumulates abnormally in hepatocytes. This, in turn, can lead to liver damage.
Accumulated A1AT protein can be detected by with periodic acid-Schiff (PAS) reagent staining
on liver biopsy. Importantly, patients with deletions in the A1AT gene will not have liver
disease, because there will be no protein to abnormally accumulate.
Dr-Wahid Helmi

Question 1:
A 10 month old boy undergoes resection of hepatoblastoma limited to the right lobe of his liver. He has
no underlying liver disease and completed his chemotherapy prior to surgery without complication.

Assuming that his post-operative course is unremarkable, which of the following best describes the
regenerative response of the residual liver segment to resection.

A. The liver mass will be restored only after infusion of hepatocyte stem cells
B. The liver mass will be restored only after the remaining hepatocytes have undergone 12 rounds of cell
doubling
C. The liver mass will be stored only after the remaining hepatocytes have undergone 1-2 rounds of
replication
D. The liver mass will be restored only after infusion of recombinant hepatocyte growth factor
E. The liver mass will be restored only after infusion of TNF-

Suggested answer: C. Liver regeneration is thought to proceed through one of two routes: i) in healthy
tissue, such as following resection, proliferation of all cell types; and ii) in diseased tissue proliferation of
stellate cells, oval cells, or other cells with stem-cell properties. In the above question, the cells in the
remaining normal liver (hepatocytes, cholangiocytes, endothelial cells, etc) will divide soon after
resection to restore the missing liver mass. 1-2 rounds of replication is sufficient to double the liver
mass.

Hepatic growth factor has mitogenic, anti-apoptotic, and anti-inflammatory properties, and in
experimental settings does promote liver regeneration. TNF-alpha, among other things, promotes
hepatic endothelial cell proliferation and also has been used in experiments to promote the regenerative
program. However, neither compound has proven clinical utility in regeneration, and neither is
administered following liver resection.

For a review, see Michalopoulos GK. Liver Regeneration. J Cell Physiol. 2007 Nov; 213(2):286-300.

Question 2:
An 8 year old young girl is referred to you for evaluation of increased weight. The patient and her parents
want to know her risk for developing obesity in adulthood. She has no other complaints. Her school
performance is good and she gets along well with peers. Her mother is 34 years old and her body mass
index is 24.5. Her father is 36 years old and his body mass index is 32. Physical examination shows that
her body mass index is greater than 95th percentile for age and sex. Examination is otherwise normal and
she has no biochemical evidence of endocrine disease.

Of the following, what is the most accurate answer to the parent's question.

A. Her risk of obesity as an adult is 1 in 5, similar to that of the general population


B. She has a 2 fold increased risk of obesity by age 30 compared to her peers
C. Her increased weight likely reflects the prepubertal growth surge.
D. Weight before puberty does not correlate with risk of obesity during adulthood.
E. Since one of her parents is obese, she has an 90% likelihood of obesity by age 30

Suggested answer: B. Many childhood factors have been studied to determine whether they predict adult
obesity, including childhood obesity after age 3, parental obesity, television time, birthweight, and in
utero insults. The patient in this question has two important risk factors: she is obese (BMI > 95%) and
at least one of her parents is obese. As a result, her risk for obesity increases above that of the
Dr-Wahid Helmi

population. The exact increase may be anywhere from 2-4 times depending on the population studied. A
child having one obese child is thought to have a 50% increased chance of becoming obese as an adult.

See Whitaker RC, Wright JA, Pepe MS, Seidel KD, Dietz WH. Predicting obesity in young adulthood
from childhood and parental obesity. N Engl J Med. 1997 Sep 25;337(13):869-73.

Question 3:
The emergency room calls about a 2 year old who swallowed some toilet bowl cleaner. Mom says she
was cleaning the bathroom and thought the toddler was in another room. She turned around and saw her
son with the bottle at his lips. He did some coughing, crying, and spitting. 2 hours later, physicians in the
ED say that the lips look red and perhaps there is a burn on the posterior oropharynx. The child appears
well. The next appropriate step would be:

A. send home with follow up in GI clinic in 1 month


B. send home with UGI in 1 month and follow up as needed
C. observe in ED for 6 hours and if no symptoms send home
D. admit NPO with EGD the next morning
E. admit NPO on steroids and antibiotics with EGD the next morning.

Suggested answer: D. Toilet bowl cleaners are acidic, allowing them to dissolve stains made from
minerals found in pipe water. Acid ingestions cause coagulation necrosis. Acids pass quickly through
the GI tract, so symptoms commonly occur in the stomach and small intestine (though oropharynx and
esophageal lesions can also occur). EGD should be performed within 12-36 hours of the ingestion (the
delay allows time for some damage to occur, so that it can be visualized during endoscopy).

In contrast to acid ingestions, base ingestions cause liquefaction necrosis. The base reacts with stomach
acid to create heat, which burns the mucosa. Symptoms can present in the oropharynx; however, lack of
oropharynx findings does not mean that more distal (i.e. esophagus) mucosa is intact. Alkaline
ingestions have a 1000-fold lifetime risk of developing esophageal carcinoma.

Question 4:
A 13 year old young woman developed recurrent seizures one year after receiving a liver transplant one
year ago because of acute liver failure. She has been started on treatment with dilantin after she was
determined to have an idiopathic seizure disorder. The family is concerned that dilantin may affect the
metabolism of tacrolimus. Which of the following is the most likely effect of dilantin on tacrolimus
pharamcokinetics?

A. Decreased tacrolimus level due to competition with dilantin for intestinal absorption
B. No effect
C. Decreased tacrolimus level due to activation of the CYP3A4 by dilantin
D. Increased tacrolimus level due to inhibition of the CYP3A4 by dilantin
E. Increased tacrolimus level due to competition with dilantin for renal excretion

Suggested answer: C. Numerous medications affect the cytochrome P450 3A4 system, the major enzymes
involved in clearing Tacrolimus. Azole antifungals, calcium channel blockers, and macrolides inhibit the
P450 system, thereby increasing the Tacrolimus levels. Antiseizure agents (such as dilantin) and anti-TB
agents can induce the P450 system and decrease Tacrolimus levels.

Question 5:
Dr-Wahid Helmi

A 16 year old young woman with steroid-dependent Crohn's disease has a 3 week history of increased
diarrhea and weight loss. Her disease is well-controlled when she is taking prednisone at 40 mg each day.
When her dose of prednisone is decreased below 20 mg each day, she has an exacerbation of symptoms.
On examination, her body mass index is 17 and she has mild tenderness to palpation throughout her
abdomen. Endoscopic and radiologic evaluation show inflammatory disease primarily in the jejunum and
ileum. You recommend that treatment be started with mercaptopurine and discuss potential nutritional
intervention. Which of the following nutritional interventions is most likely to promote recovery?

A. Complete bowel rest and institution of total parenteral nutrition


B. Complete bowel rest and institution of peripheral parenteral nutrition
C. Oral zinc supplementation
D. Elemental diet delivered by nasogastric tube
E. Supplement present diet with a lactose free formula

Suggested answer D. This patient is having a flare of small bowel Crohn’s. There are two things to
consider when making a nutrition plan: i) stopping the flare, and ii) reversing the malnutrition. In terms
of stopping the flare, both TPN/bowel rest and enteral nutrition (elemental or non-elemental) have
produced remission rates up to 80% in some studies. However, relapse rates are generally higher after
enteral nutrition vs. prednisolone. In terms of reversing malnutrition, sufficient enteral nutrition is hard
to achieve via PO feedings and usually needs an NG tube. Parental nutrition via IV bypasses this, but
has increased cost, greater chance of mixing error, and risk of line infection. Because this patient has no
contraindication to using the gut, enteral feeds are preferable. No consistent differences have been found
between elemental or non-elemental formulas.

Supplements (choice E) are controversial, as they may simply displace other foods and result in no net
gain in calorie consumption.

Question 6:
A 10 month old boy with biliary atresia has developed progressive jaundice and is listed for liver
transplantation. His 27-year-old mother has been evaluated and found to be a suitable living liver
transplant donor. When discussing the procedure for transplantation with the family, the surgeon explains
that only the left lateral segment of the mother's liver will be removed and transplanted into the recipient.
Which of the following describes the segment(s) of the liver to be used for transplantation in this case?

A. Segments 5,6,7, and 8


B. Segments 2,3, and 4
C. Segment 4
D. Segment 1
E. Segments 2 and 3

Suggested answer: E. Left lateral segment transplants comprise segments 2 and 3. The left lateral
segment represents approximately 20-25% of total volume, and includes the left hepatic vein, left branch
of the portal vein, and left branch of the hepatic artery. In split liver transplantation, the left lateral
segment goes to a child, whereas the right graft (segments I and IV to VIII, including the vena cava, right
branch of the hepatic artery, and portal vein) goes to an adult.

Question 7:
In addition to the regulatory mechanisms intrinsic to the epithelium itself, electrolyte transport is
regulated by all of the following pathways except:

A. Endocrine and paracrine regulation


Dr-Wahid Helmi

B. ATP
C. Enteric neural regulation
D. Intestinal immune system

Suggested answer: A. Electrolyte transport is controlled by a number of processes. The enteric nervous
system controls electrolyte transport, as acetylcholine and vasoactive intestinal polypeptide from nerve
endings stimulate epithelial cells to secrete chloride, which in turn leads to water in the lumen for food
lubrication. Paracrine factors (from immune and other surrounding cells) also control electrolyte
transport. For example, histamine from mast cells increases chloride secretion and decreases
bicarbonate secretion; prostaglandins from myofibroblasts, on the other hand increase secretion of both.
ATP has a central role in electrolyte transport, providing the energy to create gradients of cations (i.e.
Na+/K+ ATPase). Classic endocrine hormones are not thought to be major regulators of electrolyte
transport.

Question 8:
Which of the following is not a feature of Reye’s syndrome?

A. elevated transaminases
B. cholestasis
C. encephalopathy
D. microvesicular steatosis
E. vomiting

Suggested answer: B. Reye’s syndrome involves mitochondrial defects in fatty acid oxidation, caused by
a double insult of viral infection (flu, gastroenteritis) and mitochondrial toxins (i.e. salicylates). It is best
characterized as an event occurring after aspirin is given for a viral illness, and presents as
vomiting/lethargy followed by encephalopathy and hepatocyte damage (ALT and AST 3X > normal).
Liver biopsies show microvesicular steatosis without necrosis, and electron microscopy shows
mitochondrial changes. Bilirubin levels are usually normal or only slightly increased, and if cholestasis
is present other diagnoses should also be considered.

Question 9:
What is the most sensitive indicator for renal cyclosporine toxicity?

A. cyclosporine level
B. blood pressure
C. GFR calculated using creatinine
D. Creatinine
E. GFR based on inulin

Suggested answer: E. Cyclosporine binds with cyclophilin, which in turn inhibits calcineurin. Active
calcineurin normally dephosphorylates nuclear factor of activated T cells (NFAT-1), allowing it to enter
the nucleus and promote transcription of IL-2 and other pro-T and B cell cytokines. Hence, cyclosporine
is known as a calcineurin inhibitor (Tacrolimus works in a similar way, binding to its partner FK-506
binding protein to inhibit calcineurin).

Cyclosporine causes two forms of renal toxicity: acute and chronic. Acute toxicity is characterized by
afferent and efferent arteriole vasoconstriction, secondary to endothelial cell dysfunction. This results in
a drop in GFR and acute renal failure. Chronic toxicity occurs from vasoconstriction leading to ischemia
and structural changes in the kidneys. Acute toxicity is reversible by withdrawing the medication,
whereas chronic toxicity is thought to be permanent.
Dr-Wahid Helmi

Clinically, patients with cyclosporine toxicity present with decreased GFR. They have hypertension
secondary to sodium/volume retention, as the kidneys try to preserve perfusion in the setting of arteriole
constriction. The lowered GFR also leads to an increase in BUN and creatinine. Hence, the most
sensitive test to determine toxicity would be one that identifies an impaired GFR. Inulin, an inert
substance that is freely filtered through the glomerulus and not reabsorbed, is used to accurately
determine GFR.

Question 10:
A 7 month old infant is brought to you for evaluation because of recurrent vomiting and lethargy for 3
months. Dietary history reveals that she was exclusively breastfed until 5 months and her diet now
consists of cereal, fruits, fruit juice, and vegetables. What enzyme deficiency is likely?

A. fructose-6-phosphate
B. triokinase
C. aldolase B
D. lactase
E. sucrase isomaltase

Suggested answer C. This patient tolerates lactose from breast milk but does not tolerate foods with
sucrose/fructose (lactose is a disaccharide made from glucose and galactose, whereas sucrose is a
disaccharide made from glucose and fructose). Sucrose is digested by the brush border enzyme complex
sucrose isomaltase, which has both sucrase and alpha1,6 dextrin hydrolyzing activity. Deficiencies in
this enzyme result in sugar malabsorption and diarrhea.

Fructose, once transported into the enterocytes by facilitate diffusion, becomes phosphorylated to
fructose-1-phosphate by fructokinase. Aldolase B then controls the fate of the molecule, cleaving it into
products that enter the glycolytic, gluconeogenic, or glycogen synthesis pathways. Without this enzyme,
fructose-1-phosphate accumulates. Such patients with hereditary fructose intolerance develop poor
feeding, vomiting, lethargy, hypoglycemia, failure to thrive, liver disease, and proximal renal tubular
dysfunction when fed sucrose or fructose.

Question 11:
Cobalamin (Vitamin B12) absorption may be impaired in each of the following conditions EXCEPT:

A. pernicious anemia
B. cholestatic jaundice
C. Crohn’s ileitis
D. Zollinger-Ellison syndrome
E. Small bowel bacterial overgrowth
F. Pancreatic insufficiency

Suggested answer: B. Vitamin B12 (cobalamin) comes mainly from the cobalamin-containing meats, but
gut bacteria produce small amounts that are absorbed. The absorption of B12 from foodstuffs is a well-
characterized process. First, cobalamin must make it through the acidic stomach, by binding to
haptocorrin (R binder) at low pH. Most of the gastric haptocorrin originates from saliva. In the
duodenum, pancreatic proteases activate in the presence of bicarbonate, hydrolyze haptocorrin, and
liberate cobalamin. The cobalamin in turn binds to intrinsic factor (made by gastric parietal cells) and
becomes resistant to pancreatic proteases. The cobalamin/intrinsic factor complex binds to an ileal
brush border receptor and enters enterocytes.
Dr-Wahid Helmi

Given this sequence, B12 absorption can be impaired at multiple steps. Pernicious anemia leads to low
intrinsic factor, due to autoimmune attack on parietal cells. Zollinger-Ellison syndrome causes the
duodenum to be too acidic, preventing protease activation and degradation of the haptocorrin/cobalamin
complex. In the same way, pancreatic insufficiency prevents degradation of haptocorrin/cobalamin.
Bacterial overgrowth in the small intestine disrupts the cobalamin/intrinsic factor interaction, and ileal
disease affects cobalamin/intrinsic factor binding at the ileal brush border. Cholestasis does not alter
B12 absorption. Bile does contain haptocorrin, but salivary haptocorrin is clinically the most important.

Question 12:
True statements regarding the Schilling test include each of the following EXCEPT:

A. The stage I test result is normal in patients receiving acid suppressive therapy
B. The Schilling test relies on normal renal function for adequate interpretation
C. The stage II test result is abnormal in healthy patients who have undergone total gastrectomy
D. Results may be abnormal in patients with small bowel bacterial overgrowth

Suggested answer: C. In patients with B12 deficiency, the Schilling test is used to determine which step
in the absorption process is impaired. There are 4 steps to the test. Step 1 involves feeding radiolabeled
B12 and measuring the amount excreted in the urine, as an indicator of how much is absorbed.
Unlabeled B12 is injected concurrently that saturates tissue B12 receptors, ensuring that labeled B12
passes to the urine. Step 2-4 then repeat the first step with various adjuncts: step 2 adds intrinsic factor
(if test becomes normal, then assume IF deficiency or pernicious anemia), step 3 adds antibiotic (if test
becomes normal, then assume small bowel overgrowth), and step 4 adds pancreatic enzymes (if test
becomes normal, then assume pancreatic insufficiency).

The Schilling test should be normal in patients with acid suppression, as too much acid (not too little)
disrupts B12 absorption by interfering with pancreatic protease activation. The Schilling test relies on
normal renal function to excrete radiolabeled B12, and results are impaired with bacterial overgrowth
(hence step 3 of the test). Patients without a stomach should have impaired B12 absorption because of
reduced IF levels; however, step 2 provides supplemental IF and bypasses this defect.

Question 13:
Folate supplementation is indicated in all of the following situations EXCEPT:

A. Anticonvulsant therapy with phenytoin or carbamazepine


B. Celiac sprue
C. Chronic sulfasalazine therapy
D. Pancreatic insufficiency
E. Methotrexate therapy

Suggested answer: D. Folate is absorbed via a sodium-dependent carrier, after being hydrolyzed from
polyglutamate to monoglutamate forms by brush border enzymes. Once inside epithelial cells, folic acid
becomes methylated and reduced into its metabolically active form 5-methyltetrahydrofolate. Celiac
sprue impairs brush border activity. Phenytoin impairs brush border enzyme activity converting
polyglutamate to monoglutamate forms, and sulfasalazine interferes with absorption through the carrier.
Methotrexate prevents reduction of folate that enters cells into its active form, by inhibiting dihydrofolate
reductase (DHFR). Pancreatic enzymes are not involved in folate absorption.

Question 14:
Brush border saccharidase activity is decreased by:
A. A diet rich in sucrose
Dr-Wahid Helmi

B. Fasting
C. Diabetes mellitus
D. Pancreatic exocrine insufficiency
E. Intestinal bacterial overgrowth

Suggested answer: E, There are 3 brush border saccharidases: sucrase-isomaltase, lactase-phlorizin


hydrolase, and maltase-glucoamylase. The brush border, and subsequently the saccharidases, are
destroyed by proteases created by overgrowing bacteria. Lactase-phlorizin hydrolase expression can be
increased in infants by enteral feeds, but fasting has never been proven to decrease activity. In adults, all
enzymes except lactase can be induced by substrate (lactase expression declines with age in the majority
of people, and persistence of lactase expression is an autosomal recessive trait). Pancreatic amylase is
important to start alpha1,6 digestion of starch, which is continued by sucrase-isomaltase and lactase-
phlorizin hydrolase. However, salivary amylase can compensate in settings of pancreatic insufficiency.

Question 15:
Which of the following is not a recognized disease association of hepatitis C viral infection?

A. Cryoglobulinemia
B. Porphyria cutanea tarda
C. Membranoproliferative glomerulonephritis
D. Diabetes mellitus
E. Increased risk of myocardial infarction

Suggested answer: E. There are many extrahepatic manifestations of Hepatitis C. Cryoglobulinemia


occurs when Hepatitis C-antibody complexes form in the blood. “Mixed” cryoglobulinemia refers to
complexes with IgG against the Hepatitis C antigen and IgM against the IgG. To diagnose
cryoglobulinemia, blood samples must be kept warm because the complexes precipitate when cooled.

Porphyria cutanea tarda is a common skin manifestation of Hepatitis C. As a result of liver dysfunction,
the UROD gene (uroporphyrinogen decarboxylase) is impaired, heme synthesis is halted, and porphyrins
build up. The porphyrins collect in the skin absorb visible violet light, producing free radicals that
damage nearby tissue. In the skin, the most common findings are erosions, blisters, and scarring.

Membranoproliferative glomerulonephritis is also seen with Heptitis C, perhaps from a vasculitis


produced by the antibody-antigen complexes. Diabetes mellitus (adult-onset, insulin resistant) is 4X
more likely in Hepatitis C patients for unknown reasons. Some have speculated that the antibody
response induced by Hepatitis C results in an autoimmune attack of the pancreas. No association
between Hepatitis C and myocardial infarction has been demonstrated.

Question 16:
All of the following statements regarding the pathogenesis of diarrhea in patients with Zollinger-Ellison
syndrome are true EXCEPT:
A. Hypergastrinemia leads to colonic hypersecretion of fluid and electrolytes.
B. Acid hypersecretion leads to extreme acidification of the duodenum, resulting in inactivation of
pancreatic enzymes and concomitant steatorrhea.
C. Extreme acidification of the small intestine can lead to mild disruption of enterocyte integrity,
resulting in mild malabsorption.
D. Gastric hypersecretion can lead to significant volume overload of the small intestine.

Suggested answer: A. Zollinger-Ellison syndrome is a disorder of acid hypersecretion, caused by tumors


(commonly pancreatic) secreting gastrin. Children have increased gastric secretions, as well as diarrhea
Dr-Wahid Helmi

secondary to acid osmotic load, mucosal damage, and pancreatic lipase inactivation. Most changes from
acid hypersecretion occur in the small intestine rather than the colon.

Question 17:
The following statements regarding the management of foreign bodies in the stomach are true EXCEPT:

A. The clinician should consider removing objects that are more than 2 cm in diameter or more than 5
cm in length, because they are unlikely to pass through the duodenum.
B. In the case of battery ingestion, levels of heavy metal in the blood and urine should be measured.
C. Batteries that have passed through the esophagus to the stomach should always be removed.
D. Between 80% to 90% of ingested foreign bodies that reach the stomach will pass without specific
therapy.

Suggested answer: B. When disk button batteries are swallowed, X-ray imaging must be done to
determine the location of the battery. 90% of batteries pass spontaneously within 14 days. When
batteries lodge in the esophagus, they must be removed immediately as even 1 hour of contact has been
shown to be injurious (and 4 hours of contact can erode through all esophageal layers). Injury occurs
from electrolyte leakage, liquefaction necrosis secondary to alkali leakage such as sodium or potassium
hydroxide, mercury toxicity, pressure necrosis, and direct current flow causing low voltage burns.
Batteries found in the stomach can be removed if they are not passed after 1 week. Levels of heavy metals
are not routinely measured in battery ingestion, as serum levels have never been reported to be high
enough after battery ingestion to cause toxicity.

Question 18:
All of the following are treatments for symptomatic bezoars EXCEPT:

A. Cellulase
B. Acetylcysteine
C. Atropine
D. Mechanical fragmentation at the time of endoscopy

Suggested answer: C. Bezoars refer to a mass trapped in the gastrointestinal tract (usually the stomach).
Removing the mass can be accomplished via mechanical fragmentation or enzymatic digestion.
Mechanical digestion involves breaking the bezoar into small pieces, so that it can pass through the
intestinal tract. This approach risks causing pylorous/ intestinal obstruction. For digestion, the enzymes
used depend on the constituents of the mass, and include cellulase (which digests cellulose) and
acetylcycteine (which splits disulfide bonds in proteins). Atropine does not have digestive activity.

Question 19:
All of the following are potent acid secretagogues EXCEPT:

A. Coffee
B. Decaffeinated coffee
C. Milk
D. Alcohol

Suggested answer: C. Secretagogues are substances that cause the secretion of other substances, i.e.
acid by parietal cells. Alcohol and coffee directly stimulate parietal cells through uncharacterized
mechanisms. Alcohol also impairs gastric mucosa which contributes to acid-mediated damage. Caffeine
increases cAMP levels in parietal cells leading to increased acid production. However, coffee
independent of caffeine also stimulates parietal cells, consistent with observations that both caffeinated
Dr-Wahid Helmi

and decaffeinated coffee increases acid production. Milk is not a secretagogue; rather, it is thought to
neutralize stomach acid because it has a pH higher than that in the stomach.

Question 20:
A 21 year old woman who has had type I diabetes for 12 years had a 2-year history of intermittent nausea
and vomiting that has worsened in the past 3 months. A scintigraphic scan to evaluate solid-phase gastric
emptying shows 8% emptying at 2 hours (normal, 42% to 80%). All of the following statements are true
EXCEPT:

A. This condition is usually associated with other peripheral and/or autonomic neuropathies.
B. Delays in gastric emptying correlate well with symptoms of nausea and vomiting in this condition.
C. Findings on gastrointestinal manometry may include loss of fed and fasting antral motility and
increased phasic and tonic pyloric motility.
D. A gastric acid analysis may show a reduction in acid production in response to sham feeding.
E. The delay in gastric emptying may be exacerbated by periods of worsening hyperglycemia.

Suggested answer: B. While delayed gastric motility is best-documented in long-standing diabetics, a few
case reports have documented the phenomenon in children as well. The mechanism is thought to be
diabetic neuropathy of the vagal nerve. Patients feel fullness and bloating, though symptoms of nausea
and vomiting correlate poorly with delayed gastric emptying. Furthermore, with poor vagal input, acid
secretion decreases during sham feeding, migrating motor complexes are reduced (but cells of Cajal are
normal in number), antral contractions are less frequent, and the pylorous has increased tone.
Interestingly, in addition to the chronic diabetic changes, episodes of hyperglycemia in normal and
diabetic patients can cause delayed gastric emptying.

For further discussion, see Kashyap P, Farrugia G. Diabetic gastroparesis: what we have learned and
had to unlearn in the past 5 years. Gut. 2010 Dec;59(12):1716-26.

Question 21:
Which of the following statements concerning management of the above case is TRUE?

A. Avoidance of a high fat diet might reduce symptoms.


B. Increasing the oral dose of erythromycin might worsen the nausea.
C. The efficacy of domperidone in reducing nausea may be partially explained by CNS effects on
dopamine receptors in the brain stem.
D. Endoscopic destruction of a gastric bezoar may alleviate symptoms temporarily.
E. All of the above statements are correct.

Suggested answer: E. In order to improve gastric motility, a number of approaches are available. The
first is to improve glycemic control. The second is to use motility agents, including erythromycin,
domperidone (a dopamine receptor antagonist), cisparide, and IV grehlin. Erythromycin can be titrated
up as tolerated, until side-effects such as diarrhea, nausea, or vomiting occur. Removing any bezoars
that have formed secondary to poor motility may also improve symptoms, as well as avoiding meals that
further slow gastric motility, i.e. high fat meals.

Question 22:
A one month old infant boy recovery from heart surgery develops post-prandial emesis. He was the
product of a 35 week gestation. He was found to have congenital heart disease and received prostaglandin
E1 for 21 days to maintain patency of his ductus arteriosus prior to surgery. Post-operatively he did well.
He began feeding with a standard infant formula 2 days ago.
Dr-Wahid Helmi

A. Chest X-ray
B. Upper GI
C. Upper endoscopy
D. Change to elemental formula
E. Begin treatment with H2 blocker

Suggested answer: C. Post-prandial emesis occurs for a number of reasons, including simple reflux,
malrotation, milk allergies, dysmotility, and pyloric stenosis. In the setting of heart disease, gastric
hypoperfusion must also be considered which may delay motility and/or decrease intestinal barrier
function, allowing large protein epitopes to enter the bloodstream and cause allergies. Finally, with
chronic prostaglandin use, case reports have documented antral hyperplasia and gastric outlet
obstruction. Unlike pyloric stenosis which involves muscle, this phenomenon involves the mucosa. The
condition reverses with drug withdrawal. Endoscopy could diagnose this definitively, as well as rule-out
allergic causes. (Upper GI could also suggest antral hyperplasia but could not distinguish it from pyloric
stenosis).

Question 23:
A 17-year-old girl presents with a complaint of a fatigue and jaundice for 4 weeks. She has enlargement
of her liver, scleral icterus and 5 fold elevation of serum transaminases. Her INR is 1.2. Her hepatitis A,
B, and C serologies are negative but her anti-nuclear antibody titer is 1:640. She has not taken
medications. Review of systems reveals that she has had loose stools for 6 months and that she has lost 5
pounds.

Which of the following is the most appropriate next step?

A. Observe for six months


B. Measure anti-mitochondrial antibody titers
C. Colonoscopy
D. Liver biopsy
E. UGI and small bowel follow through

Suggested answer: D. This patient has liver disease and a positive ANA titer (1:640 is lower limit of
abnormal), making type I autoimmune hepatitis likely. Her liver disease also accompanies intestinal
disease suspicious for possible inflammatory bowel disease. Indeed, approximately <1% of children with
ulcerative colitis develop autoimmune hepatitis and 3-5% develop primary sclerosing cholangitis. The
first step is to confirm he liver diagnosis with a liver biopsy, looking for mononuclear and plasma cell
infiltrates in the portal tracts. Next, a colonoscopy can be performed to further investigate the possibility
of inflammatory bowel disease.

Question 24:
The gastric mucosa is a hostile environment for most bacteria. Helicobacter pylori is highly adapted to
survival in the gastric mucosa. Which of the following is most likely explanation for the survival of
H.pylori in the acidic environment of the gastric mucosa?

A. Expression of a vacuolating cytotoxin which impairs acid secretion


B. Production of urease
C. Inhibition of histamine stimulated acid secretion
D. Inhibition of the H+/K+ pump
E. Strong adhesion to the mucosal surface
Dr-Wahid Helmi

Suggested answer: B. Helicobacter pylori are uniquely situated to handle the acidic environment of the
stomach. H. pylori has a urease, which hydrolyzes urea into carbon dioxide and ammonia, and the
ammonia in turn buffers the stomach pH. H. pylori uses adhesins to adhere to the mucosal surface, as
well as a vacuolating cytotoxin (VacA) to create channels in which nutrients can escape from mucosal
cells to feed the bacteria. Proton pump inhibitors block the H+/K+ pump, and ranitidine inhibits
histamine-induced parietal cell acid secretion.

Question 25:
What is one of the primary differences between pediatric enteral formulas and adult enteral formulas?

A. Pediatric formulas have less protein/same volume


B. Pediatric formulas utilize different sources of carbohydrates
C. Pediatric formulas are only made with hydrolyzed protein
D. Pediatric formulas utilize only LCT to enhance fat absorption

Suggested answer: A. Pediatric formulas (versus adult formulas) have more fat but decreased protein
and carbohydrates. They utilize similar carbohydrates, are made full proteins, pepstides, or amino acids,
and use various triglycerides (including long chain fatty acids to prevent essential fatty acid deficiency)
for the fat component.
Dr-Wahid Helmi

Question 1:
A 5-month-old infant has been fed only goat milk. Laboratory studies reveal: hemoglobin concentration,
9.5 g/dL; mean corpuscular volume, 98 fL; white blood cell count, 4200/mm3; and reticulocyte count,
0.2%.

For initial treatment, it would be MOST appropriate to recommend dietary supplementation with

A. ascorbic acid
B. folic acid
C. iron
D. pyridoxine
E. vitamin B12

Suggested answer: B. Goat’s milk can be used for infants suffering from cow’s milk protein allergy.
Goat’s milk contains only trace amounts of the allergenic protein alpha-S1-casein normally found in
cow’s milk (however, goat’s milk contains equivalent amounts of another allergenic protein beta-
lactoblobulin). Goat’s milk is also deficient in other vitamins, including vitamin D (supplemented in
cow’s milk and formulas), vitamin B12, and especially folate. Goat’s milk contain 6 ug/L folate, whereas
breast milk contains 45 ug/L and cow’s milk 50 ug/L. Hence, infants drinking goat milk are most
susceptible to folate deficiency and resulting megoblastic anemia.

Of note, milk from other animals have been tested on children with cow’s milk protein allergy, with better
results (see Vita et al. Ass's milk in children with atopic dermatitis and cow's milk allergy: crossover
comparison with goat's milk. Pediatr Allergy Immunol (2007) 18: 594-8).

Question 2:
The basal energy or metabolic requirement for children is calculated MOST accurately by considering

A. body surface area


B. creatinine-height index
C. serum protein concentration
D. total lymphocyte count
E. triceps skinfold thickness

Suggested answer: E. The basal metabolic rate (BMR) is the amount of energy used by an organism at
rest. In children it includes the energy of growing, and differs between boys and girls in accord with the
sex differences in the intensity and duration of the adolescent growth spurt. BMR is measured by the heat
given off per unit of time, and is expressed as calories released per kilogram of body weight (or per
square meter of body surface area) per hour. BMR is a function of an organism’s fat free mass (FFM),
as this comprises the bulk of active metabolic tissue.

BMR correlates best with body surface area. Creatinine-height index may estimate lean body mass/FFM,
and hence correlate with BMR, but it is not well validated in children. Triceps skinfold thickness
measures subcutaneous fat, not FFM. Certain methods using multiple skin fold measurements to estimate
FFM (“Dauncey” method and the “Durnin/ Womersley” method) are fraught with multiple problems.

Question 3:
Of the antibodies found in human colostrum and milk, the immunoglobulin (Ig) that is MOST likely to
prevent organisms from adhering to the infant's intestinal mucosa is

A. IgA
Dr-Wahid Helmi

B. IgD
C. IgE
D. IgG
E. IgM
Suggested answer: A. Infants passively acquire many antibodies from their mothers (“passive humoral
immunity”). During gestation, they receive maternal IgG transplacentally. After birth, they receive IgA
from colostrum and breast milk, which provides local protection in the gut but does enter the systemic
circulation. They also receive IgG from breast milk, which can bind to transepithelial Hc receptors and
enter the circulation. Infant do not receive IgE, IgM, or IgD (an immunoglobulin associated with IgM)
passively from their mothers.

Question 4:
A 3-year-old boy, who has sustained second- and third-degree burns of the esophagus from ingesting a
lye solution, requires placement of a gastrostomy tube for nutritional support. The parents are counseled
about the risks and benefits of tube feedings.

In your discussion, you tell them that the MOST frequent complication of enteral feeding is

A. clogging of the feeding tube


B. diarrhea
C. electrolyte disturbances
D. gastric irritation
E. hypoalbuminemia

Suggested answer: B. Gastrostomy tubes (G tubes) allow for nutrition in patients with oral or esophageal
disease. There are many complications with G tubes, including aspiration, electrolyte imbalances (i.e.,
refeeding syndrome), and fluid imbalances as most formulas only contain 70-80% water. Other long
term complications may include dysmotility, as the procedure attaches the stomach to the abdominal
wall. The most frequent complication is osmotic diarrhea. Diarrhea results when the rate of feeding
exceeds the gut’s capacity to absorb, leading to increased sugar delivery to the colon where it is
metabolized by bacteria into osmotically active substances.

Question 5:
A 1,220 gm infant develops necrotizing enterocolitis. After the infant recovers, the BEST type of formula
to use to initiate feedings is

A. 20 calories/oz elemental formula


B. 20 calories/oz premature formula
C. 20 calories/oz standard infant formula
D. 24 calories/oz premature formula
E. 27 calories/oz premature formula

Suggested answer: D. Breast milk has many advantages over formula, including the prevention of NEC.
Multiple studies have demonstrated that infants started on expressed breast milk (from their mother or a
donor) have lower NEC rates compared to those started on formula. He current recommendations are to
“prime” the with breast milk early. It is unclear whether the rate of milk increase affects NEC rates.

Despite the benefits of breast milk, breast milk does not have enough calcium and phosphorous for
preterm infants. Preterm formulas account for this deficiency and have adequate amounts. For the
patient in the vignette, the infant should first undergo “priming” with human milk. When a rate of 100
cc/kg is achieved, the human milk can be fortified to 24 kcal/ounce and advanced to reach a goal of 150-
Dr-Wahid Helmi

180 cc/kg/day. If human milk is unavailable, elemental formula (better) or preterm formula fortified to 24
kcal can be used (there is no data favoring 20 kcal formula over 24 kcal formula).

Question 6:
A TRUE statement regarding anthropometric measurements in the assessment of nutritional status is:

A. Acute changes in weight reflect changes in muscle mass


B. Arm circumference is the best screening tool for malnutrition
C. Single measurements are the most sensitive indicators of nutritional problems
D. Standard growth curves are equally applicable to all ethnic groups
E. Standard growth curves overestimate the early gains to be made by breastfed infants

Suggested answer: B. Mid-upper arm circumference (MUAC) is an easy way to screen for malnutrition,
applicable to all children >1 year of old or >6 months old and taller than 65 cm. To measure MUAC,
measure the circumference (to the nearest millimeter) at the midpoint of the left arm, half-way between
the tip of the shoulder (olecranon) and the tip of the elbow (acromium). MUAC compared to BMI is less
affected by edema. Despite the ease of MUAC, the most sensitive indicators of nutritional problems
account for multiple variables, such as social factors in addition to anthropometric calculations. Acute
changes in weight usually reflect fluid shifts, not changes in muscle mass.

Standard growth curves are convenient ways to compare a child’s height and weight with same-aged
children in the population. There are CDC and WHO growth curves. The CDC curves reflect growth of
US children over the last 30 years, whereas the WHO curves reflect growth of children all over the world,
including Brazil, Ghana, India, Norway, Oman, and the United States. Initially, the CDC recommended
using the CDC growth curves for all children in the US. Now, however, they recommend using the WHO
growth curves for children <2 years old, because the WHO growth curves better account for breast
feeding. When the curves for <2 years old are compared, the WHO curve (more breast fed infants) starts
off faster but plateaus quicker. As a result, the CDC curve overestimates the percentiles for breast fed
infants early and underestimates their percentiles later. The CDC recommends switching to the CDC
curve at age 2, resulting in a situation where a child may be overweight at age 2 on the WHO curve but
normal on the CDC curve.

Question 7:
An 8-month-old girl recently recovered from a gastrointestinal illness. When the mother reintroduced the
formula, cereal, and baby foods the child had been eating before the illness, the girl developed persistent,
watery diarrhea; abdominal distention; flatulence; and recurrent abdominal pain.

Of the following, the MOST likely cause of this patient's problem is

A. cow milk allergy


B. gluten sensitivity
C. lactose intolerance
D. new-onset infectious enteritis
E. persistent infectious enteritis

Suggested answer: C. This patient suffers from secondary/acquired lactose intolerance. It occurs after a
viral infection, in which intestinal brush border (and their disaccharidases such as lactase) are
destroyed. As a result, lactose will not be digested in the small intestine, will pass to the colon, and will
be metabolized by gut bacteria into gas and osmotically active substances. Lactase is the last
disaccharidase to return to normal function after injury, so secondary lactose intolerance may last for
months.
Dr-Wahid Helmi

Question 8:
In humans, vitamin E functions PRIMARILY as a:

A. coenzyme of carboxylase
B. component of rhodopsin
C. membrane antioxidant
D. methyl donor
E. regulator of calcium absorption

Suggested answer: C. Vitamin E is a fat-soluble antioxidant that stops production of reactive


oxygen species normally formed when fat undergoes oxidation. Biotin (Vitamin B7) is a
coenzyme for many carboxylases. Vitamin A is a component of rhosopsin, the photopigment
used in black/white vision. Folic acid (Vitamin B9) and colbalamin (Vitamin B12) are methyl
donors. Vitamin D is a regulator of calcium absorption.

Question 9:
Zinc is MOST easily absorbed from:

A. casein-dominant, cow milk-based formula


B. cow milk
C. human milk
D. soy-based formula
E. whey-dominant, cow milk-based formula

Suggested answer: C. Zinc bioavailability has been studies by labeling zinc with a radioisotope, mixing it
into various milk/formula preperations (human milk, cow’s milk, cow’s milk formula, and soy protein
formula), and assaying it on newborn sucking rates. Zinc bioavailability was 28% in human milk, 24%
from cow’s milk formula, 15% from cow’s milk, and 10% from soy formula.

Cow’s milk-based formula is the first choice for infants who are not breast fed. Cow’s milk has 50%
more protein, and the ratio of whey protein to casein protein is 20:80 (compared to 70:30 in human
milk). Caesin is composed of proteins held together in granular structures called micelles, precipitates
out in the acidic environment of the stomach, but is nicely digested in the alkaline environment of the
small intestine, providing a slowly absorbed source of amino acids. Whey, on the other hand, consists
mainly of beta-lactoglobulin (major component of bovine whey, and allergenic), alpha-lactalbumin
(major component of human whey), albumin, immunoglobulins, hormones, and growth factors. It is
easily digested in the acidic environment of the stomach and produces a rapid surge in plasma amino
acids. Pasteurization destroys many of the immunoprotective properties of whey.

The ratio of whey to casein in popular formulas are as follows: 100:0 in Carnation Good Start Supreme,
60:40 in Enfamil Lipil, and 48:52 in Similac Advance.

Question 10:
An infant boy born at term is delivered at home without medical supervision. At 48 hours of age, he is
brought to the emergency room because of a bloody discharge from the umbilical cord and bloody stools.

Until the results of laboratory studies are available, the BEST initial management is to administer
intravenous
Dr-Wahid Helmi

A. ampicillin and gentamicin


B. cryoprecipitate
C. factor VIII concentrate
D. fresh frozen plasma
E. vitamin K

Suggested answer: E. Newborns are vulnerable to hemorrhagic disorders secondary to Vitamin K


deficiency. Very little Vitamin K is transferred transplacentally, and the storage of Vitamin K in the
neonatal liver is also limited. Furthermore, until the gut is colonized with bacteria (especially
Bacteroides species), there is very little microbial production of Vitamin K. Breast milk also contains
little Vitamin K. To prevent bleeding, Vitamin K is administered intramuscularly immediately after birth
in the US. With a Vitamin K injection, the incidence of Vitamin K deficiency-related bleeding varies from
0.25-1.7% in the first week of life.

Question 11:
Physical examination of a 2-year-old malnourished girl reveals a "rachitic rosary."

Among the following, the indicator MOST likely to be associated with this child's problem is:

A. bronchopulmonary dysplasia
B. chronic hepatitis
C. galactosemia
D. sucrase-isomaltase deficiency
E. tyrosinemia

Suggested answer: E. “Rachitic rosary” refers to prominent knobs of bone at the costochondrial
joints. It is associated with rickets, and is caused by excessive urinary losses of calcium and
phosphate and/or defect in renal hydroxylation of 25-OH Vitamin D3 into 1,25-diOH Vitamin
D3. Tyrosinemia can involve renal tubules, leading to a Fanconi-like syndrome with normal
anion-gap, metabolic acidosis, hyperphosphaturia, hypophosphatemia, and Vitamin D resistant
rickets. Galactosemia is characterized by inability to metabolize galactose, and is caused by
mutations in one of 3 genes. It can involve renal tubular dysfunction and a Fanconi-like
syndrome, but (unlike this case) typically presents with jaundice and seizures in the first few days
of life. Sucrase-isomaltase deficiency leads to improper digestion of sucrose, resulting in
sucrose delivery to colonic gut bacteria. It is characterized by diarrhea and malnutrition.

Question 12:
The immunoglobulin (Ig) MOST abundant in human milk is:

A. IgA
B. IgD
C. IgE
D. IgG
E. IgM

Suggested answer: A. Human milk contains all the immunoglobulins (M, A, D, G, E), but secretory IgA
is the most abundant. sIgA from breast milk is an important source of passive immunity before the
neonate generates its own sIgA; furthermore, assuming the mother and child share the same environment
and flora, breast milk sIgA will confer protection against the relevant organisms. sIgA is packaged in
breast milk in acid-resistant packages, allowing it to survive the acidic stomach and makes its way to the
Dr-Wahid Helmi

intestines. Early milk contains 2 g/L IgA, 0.12 g/L IgM, and 0.34 g/L IgG. Mature milk contains 1 g/L
IgA, 0.2 g/L IgM, and 0.05 g/L IgG.

Question 13:
An infant born at term has a Coombs positive ABO blood group incompatibility. On the third day of life,
laboratory findings include: venous hemoglobin concentration, 14 gm/dL, and total serum bilirubin level,
10.8mg/dL.

Among the following, the MOST appropriate therapy is

A. blood transfusion
B. expectant follow-up
C. intravenous administration of immune globulin
D. therapeutic iron supplementation
E. vitamin K supplementation

Suggested answer: B. This child is at risk for elevated unconjugated bilirubin requiring phototherapy,
given his Coombs positive/ABO blood group incompatibility status. Currently, his total serum bilirubin
level is 10.8 mg/dL which is below the recommended phototherapy range for a healthy, full-term
newborn. His future care should include expectant follow-up, as well as at least one bilirubin
fractionation measured early to ensure that the there is no elevation in conjugated bilirubin.

Question 14:
A 1, 450 gm infant is not growing well on 24 calories/oz premature formula. The caloric density is
increased to 27 calories/oz by the addition of medium-chain triglycerides.

Of the following complications, this change in formula is MOST likely to produce:

A. bloody stools
B. diarrhea
C. gastric distention
D. increased gastric emptying time
E. steatorrhea

Suggested answer: B. This patient is being fed a formula with increased osmolarity, and is at risk for an
osmolar diarrhea. Medium-chain triglycerides (MCTs) are considered more easily absorbed than that
other fatty acids, because they are absorbed directly into the enterocyte and carried through the portal
circulation to the liver (they do not travel through lymphatics). Furthermore, MCTs do not require bile
acids, lipase, or co-lipase for their absorption. Recently, studies have challenged the benefits of MCTs,
showing infants fed long chain fatty acids grew just as well as those taking MCTs.

Question 15:
Which of the following drugs administered to the mother is MOST likely to have an adverse effect on the
infant who is breast feeding?

A. Furosemide
B. Gentamicin
C. Hydralazine
D. Metronidazole
E. Penicillin
Dr-Wahid Helmi

Suggested answer: D. Metronidazole is secreted in breast milk in small quantities, just as furosemide,
gentamicin, hydralazine, and penicillin are. None of the medications have been shown to pose a risk to
the infant. However, metronidazole poses a theoretical risk of cancer to children, as it increases the risk
of certain cancers in mice and rats. As a result, some practioners will recommend mothers to stop breast
feeding for 12 to 24 hours after taking metronidazole.

Question 16:
During the first 4 months of life, the USUAL caloric requirement (in Kcal/kg per day) for appropriate
postnatal growth in the healthy term infant who is formula fed is

A. 90
B. 105
C. 120
D. 135
E. 150

Suggested answer: B. Newborns need extra calories to support rapid growth. On average, a 0-3 month
bottle fed infant will take from 90-135 kcal/kg/day to achieve an increase in weight of 25 to 30 g/day
(135-202.5 cc/kg/day). A 3-12 month infant requires approximately 100 kcal/kg/day. Hence, on average,
during the 4 months of life a child will need 105 kcal/kg/day.

Question 17:
Medium-chain triglycerides account for 40 to 50% of the fat content of formulas fed to low-birthweight
infants.

Of the following, the BEST explanation for this practice is that:

A. absorption of medium-chain triglycerides is similar to that of butterfat


B. low-birthweight infants have a large pool of bile acids
C. low-birthweight infants have increased capacity to synthesize bile salts
D. medium-chain triglycerides cause less steatorrhea
E. medium-chain triglycerides require micelle formation for absorption

Suggested answer: D. Preterm infants have low intraluminal bile salts, and all infants have low
pancreatic lipase (their salivary lipase compensates). Medium chain triglycerides (MCTs) can be
absorbed directly into enterocytes and passed to the liver via the portal vein. Their absorption is not
dependant on bile salts, pancreatic enzymes, chylomicron formation, or lymphatic transport. Recently,
studies have challenged the benefit of MCTs over long-chain fatty acids by showing that MCTs offered no
extra growth advantage in preterm infants. MCTs carry the risk of increased urinary dicarboxylic acid
excretion, which is produced after incomplete beta oxidation of MCTs.

Question 18:
A 31/2-year-old boy with chronic diarrhea and failure to thrive is diagnosed with cystic fibrosis.
Neurologic examination reveals absent deep tendon reflexes, truncal ataxia, and muscle weakness. A
nutrient deficiency is suspected.

Given this constellation of findings, what additional physical sign is MOST likely to be present in this
child?

A. Desquamating skin lesions


B. Ophthalmoplegia
Dr-Wahid Helmi

C. Positive Trousseau sign


D. Purpura
E. Stooped posture

Suggested answer: B. Vitamin E deficiency is characterized by neurological and neuromuscular findings,


including ophthalmoplegia. Vitamin A deficiency can present as desquamating skin lesions and night
blindness. Vitamin D deficiency can present with hypocalcemia leading to a positive Trousseau sign
(spasms of the hand when blood flow if occluded via a blood pressure cuff). Vitamin D deficiency can
also lead to vertebral fractures producing a stooped posture. Vitamin K deficiency leads to bleeding seen
as petechiae (<3 mm) or purpura (3-10 mm).

Question 19:
Which of the following amino acids, present in adequate amount in cow milk-based formula, must be
added to soy-based infant formula?

A. Isoleucine
B. Leucine
C. Lysine
D. Methionine
E. Valine

Suggested answer: D. Methionine is a sulfur-containing essential amino acid, and is two times more
abundant in cow’s milk than soy milk. Methionine supplementation improves the biological quality of soy
protein, by improving weight gain, urea nitrogen excretion, and albumin synthesis. Hence,
supplementation began in the early 70s; before that, infants feeding soy formulas were at risk for severe
hypoalbuminemia and edema. In addition to methionine, carnitine (fatty acid transport), and taurine
(neurodevelopment, bile acid conjugation) are also supplemented in soy protein.

Question 20:
A 2-year-old child has undergone resection of the terminal ileum because of an ileal-ileal intussusception.
Of the following nutrients, which is MOST likely to become deficient in this patient?

A. Folic acid
B. Thiamine
C. Vitamin A
D. Vitamin B12
E. Vitamin K

Suggested answer: D. Vitamin B12 is exclusively absorbed in the terminal ileum. Vitamin B12
(cobalamin) comes mainly from cobalamin-containing meats, but gut bacteria produce small amounts
that are absorbed. The absorption of B12 from foodstuffs is a well-characterized process. First,
cobalamin must make it through the acidic stomach, by binding to haptocorrin (R binder) at low pH.
Most of the gastric haptocorrin originates from saliva. In the duodenum, pancreatic proteases activate in
the presence of bicarbonate, hydrolyze haptocorrin, and liberate cobalamin. The cobalamin in turn binds
to intrinsic factor (made by gastric parietal cells) and becomes resistant to pancreatic proteases. The
cobalamin/intrinsic factor complex binds to an ileal brush border receptor and enters enterocytes.

Question 21:
Of the following, the MOST beneficial formula for patients with gastrointestinal allergy, short gut
syndrome, or cystic fibrosis is:
Dr-Wahid Helmi

A. a protein hydrolysate
B. carbohydrate free
C. lactose free
D. low in iron
E. soy based

Suggested answer: A. A protein hydrolysate formula provides proteins in the form of small peptides. As
such, they are less allergenic and less likely (vs. full proteins) to cause allergies in patients with
gastroenterology allergies such as cow’s milk protein allergy. They are also less likely to cause allergies
in children with poor gut integrity, such as those with short gut syndrome. Protein hydrolysate formulas
requires less peptidase digestion because they are pre-digested, making them ideal for patients with
Cystic Fibrosis or patients with other reasons for pancreatic sufficiency. Sugars such as lactose have
nothing to do with allergy, but may cause an osmotic diarrhea in patients with malabsorption. Soy
proteins are both allergenic and require pancreatic enzymes to fully digest.

Question 22:
Of the following fatty acids, which MUST be added to cow milk-based infant formula?

A. Arachidic
B. Linoleic
C. Oleic
D. Palmitic
E. Stearic

Suggested answer: B. There are 2 essential fatty acids for humans: alpha-linolenic acid (ALA, an
omega-3 fatty acid) and linoleic acid (LA, an omega-6 fatty acid). LA is the only fatty acid
whose amount is required to be stated on formulas. ALA and LA are not independently added,
but rather part of the vegetable oils mixed in formulas to provide fatty acids.

ALA is converted to eicosapentaenoic acid (EPA) and docosahexaenoic acid (DHA), which are
anti-inflammatory. LA is used to make arachidonic acid (ARA) which is a precursor to
prostaglandins and other proinflammatory mediators. Both DHA and ARA are supplemented in
available formulas.

Question 23:
A 2-year-old boy with developmental delay receives daily tube feedings. Among the following, the side
effect MOST likely to result from high osmolality or high carbohydrate content of these feedings is:

A. diarrhea
B. fecal impaction
C. gastritis
D. mucosal atrophy
E. occlusion of the feeding tube

Suggested answer: A. High osmolality and high carbohydrate content formulas can both lead to
diarrhea. High osmolality results in high osmolar concentration in the colon, which draws water into the
lumen creating loose stools. High carbohydrate contents can result in excess carbohydrates reaching the
colonic gut bacteria. The bacteria ferment the carbohydrates into osmotically-active short chain fatty
acids, which also attract water in the lumen to produce diarrhea.
Dr-Wahid Helmi

Question 24:
When the composition of colostrum is compared to that of mature human milk, colostrum is found to be
LOWER in:

A. ash
B. immunoglobulin A
C. polymorphonuclear leukocytes
D. sodium
E. total fat

Suggested answer: E. Colostrum is the first stage of breast milk and lasts for many days after birth.
Colostrum is high in protein (especially immunoglobulins), fat-soluble vitamins, and minerals (“ash”).
Compared to mature milk, it has higher concentrations of protein, sodium, potassium, and chloride. It
has lower fat concentrations than that found in mature milk.

Question 25:
The RECOMMENDED daily dose of vitamin D for a healthy 1-week-old, preterm infant taking oral
feedings is (in IU):

A. 4
B. 40
C. 100
D. 400
E. 1,000

Suggested answer: D. Vitamin D is best known for its role in calcium homeostasis; however, it is also a
hormone that has far ranging effects in the host. Because infants often receive inadequate sunlight, and
because breast milk has low concentration of Vitamin D, infants are advised to supplement their
milk/formula feeds with Vitamin D supplementation. The current recommendation for preterm or term
neonates is to supplement with 400 IU/day.
Dr-Wahid Helmi

Question 1
Which of the following statements regarding parenteral nutrition (PN)-associated complications and
toxicity is true?

A. Premature infants develop protein toxicity rapidly with early introduction of PN.
B. The most common feature of neonatal PN-associated liver disease is steatosis.
C. Experimental models have demonstrated impaired pulmonary function and displacement of
bilirubin from albumin binding sites with excessive intravenous (IV) lipid administration.
D. PN-associated liver disease usually presents within one week of initiating PN.

Suggested answer: C. Parenteral nutrition has many associated complications. In general, within 1-2
weeks LFTs become elevated and micro/macro steatosis develops. Cholestasis (conjugated bili > 1.5
mg/dL), extramedullary hematopoiesis, fibrosis and then cirrhosis ensue. Moderate fibrosis is present
after approximately 40 months of parenteral nutrition. Cholestasis is the prominent initial finding in
infants (steatosis and extramedullary hematopoiesis resolves), whereas steatosis is most prominent in
adults.

Specific macronutrient complications include: i) protein toxicity with time, perhaps from amino acids
such as methionine, cysteine, and tryptophan, as documented in premature infants receiving 3.6 g/kg/day
vs. 2.5 g/kg/day; ii) glucose toxicity leading to steatosis, especially with GIRs>12.6; and iii) lipid toxicity,
possibly through phytosterols in lipid preparations. Lipids have also been shown to alter pulmonary
membrane diffusion and displace bilirubin from albumin.

Question 2
Which of the following metabolic alterations is most commonly seen with re-feeding syndrome?

A. Hyperlipidemia secondary to increased serum ketone bodies


B. Wernicke’s encephalopathy secondary to thiamine deficiency
C. Severe hypophosphatemia affecting myocardial and respiratory function
D. Hypernatremia and hypertonic dehydration affecting mental status

Suggested answer: C. Refeeding syndrome occurs within 4 days when transitioning from the starving to
the fed state. During starvation, the body metabolizes protein and fat, and uses little insulin. Protein/fat
catabolism leads to low total body phosphate stores, even if serum levels are normal. With feeding, the
body starts metabolizing sugar and generates large amounts of insulin. The insulin surge promotes
cellular uptake of phosphate, resulting in profound hypophosphatemia. The low phosphate impairs ATP
production resulting in a myriad of symptoms, from non-specific signs to rhabdomyolysis, leukocyte
dysfunction, respiratory failure, cardiac failure, hypotension, arrhythmias, seizures, coma, and even
sudden death. Hypophosphatemia can also rarely cause Wernicke’s encephalopathy, but thiamine
deficiency (not associated with refeeding syndrome) is a more likely cause.

Question 3
Which of the following statements is false concerning failure to thrive (FTT)?

A. FTT is a term used to describe infants and children whose growth deviates from that expected
for their sex and age.
B. It is a symptom not a diagnosis
C. Dysfunction of any organ system may result in FTT.
D. It is a common disorder that may affect 10-20% of outpatients.
E. The majority of patients with FTT have organic causes.
Dr-Wahid Helmi

Suggested answer: E. There is no consensus definition for FTT. It usually refers to children with
abnormally low weight for age and gender. Others suggest that it should include failure in other aspects
of development as well. Still other definitions for older children are: <75% median weight/height, <2
standard deviations below mean weight/height, or weight that crosses 2 percentiles. FTT accounts for 1-
5% of pediatric hospital referrals and 10-20% of outpatient visits. Usually an organic cause is not found,
though oro-motor dysfunction is increasingly becoming recognized as a common cause for FTT.

Question 4
A 29 year-old male is referred from an optometrist for evaluation. The patient’s liver profile shows AST
78 IU/L, ALT 92 IU/L, Bili 1.4 mg/dL, Alk Phos 88 IU/L, and albumin 3.4 g/dL. The photo of the
patient’s eyes is attached. All of the following statements are true except: 3.18.04

A. The patient’s diagnosis is Wilson’s disease if the ceruloplasmin is low.


B. This finding on the eye exam can be seen in primary biliary cirrhosis and autoimmune hepatitis.
C. The pigmentation will disappear with effective therapy.
D. Neurologic symptoms typically do not occur in the absence of this finding.
E. The mechanism that causes the disorder associated with this finding occurs as a result of over-
absorption of copper.

Suggested answer: E. Wilson’s disease, the commonest metabolic cause of ALF in children over 3,
results from defective copper trafficking out of hepatocytes. The disease is caused by a mutation in
ATB7B, which moves copper ions against concentration gradients i) into the Golgi (for formation of
ceruloplasmin) and ii) out of the cell. As a result, serum ceruloplasmin is low, though it may sometimes
be normal or high because ceruloplasmin is also a negative acute phase reactant. Other diagnostic tests
include a blood smear for hemolysis, low alkaline phosphatase, and raised urine copper before and after
pencillamine challenge. The best diagnostic test is elevated copper levels on liver biopsy.

Kayser-Fleisher rings are gold or gray/brown rings in the peripheral cornea representing copper/sulfur
deposits. They are usually (but not always) present in older patients with neurological problems, such as
mood disorders/depression (adolescents), bradykinesia (pseudo-parkinsonian Wilson’s), multiple sclerosis
with tremor (pseudo-sclerotic Wilson’s) , or dyskinesia. Treatment reverses KF rings. Some treatments
include pencillamine which binds free copper for urinary excretion, and zinc which traps copper in
enterocytes to be lost in the gut lumen when the cells are sloughed.

Serum copper is not a reliable test for Wilson’s disease. Serum copper reflects a combination of free
copper and copper bound to ceruloplasmin. Therefore, in Wilson’s disease it may be high (from free
copper released when hepatocytes lyse) or low (because of poor ceruloplasmin production).

Question 5
All of the following medications can cause acute cholestasis resembling obstructive jaundice except:
Dr-Wahid Helmi

A. Erythromycin estolate
B. Nitrofurantoin
C. Sulindac
D. Amoxicillin-clavulanic acid
E. Chlorpromazine

Suggested answer: C. Most drug-induced liver disease is cytotoxic. However, some drugs induce
hepatitis-cholestasis disease characterized by jaundice, itching, elevated alkaline phosphatase, and
histological evidence that resembles obstructive jaundice. Drugs that cause this include erythromycin,
chlorpromazine, nitrofurantoin, and amoxicillin-clavulanic acid. Eosinophils can be seen on liver
biopsies, suggesting a hypersensitivity/allergic etiology. Sulindac is an NSAID that can also cause
cholestasis through inhibiting cannilicular secretion of bile acids. Rather than an obstructive picture,
histology shows more hepatocellular damage as the retained bile damages hepatocytes.

Question 6
A 17 year-old female is gravida 21 weeks. She presents with nausea, emesis and jaundice. Her
medications are prenatal vitamins. Her laboratory exam is significant for ALT 639 IU/L, AST 459 IU/L,
bilirubin 4.8 mg/dL, Alk Phos 320 IU/L, and albumin 3.4 g/dL. The most likely cause of her jaundice is:

A. Intrahepatic cholestasis of pregnancy


B. Viral hepatitis
C. Acute fatty liver of pregnancy
D. Hyperemesis gravidarum
E. Drug-induced hepatitis

Suggested answer: B. The most common cause of jaundice in pregnancy is viral infection, with HSV and
Hepatitis E most concerning for the developing fetus. In the case of Hepatitis B infections, the newborn
needs active (vaccine) and passive (immunoglobulins) immunoprophylaxis. Intrahepatic cholestasis of
pregnancy usually has more itching than jaundice, and occurs in the 3rd trimester when hormone levels are
highest (estrogen inhibits BSEP and progesterone inhibits MDR3). Mutations in BSEP and MDR3 may
also manifest for the first time during pregnancy. Acute fatty liver of pregnancy presents more severely,
and hyperemesis gravidarum has extreme nausea and vomiting. Prenatal vitamins should not cause
hepatitis.

One algorithm is as follows: 1) rule out viral hepatitis; then 2) if ALT>1000, consider medication toxicity
(Tylenol); 3) if ALT<1000 and with renal failure or DIC, consider acute fatty liver of pregnancy; 4) if
ALT<1000 with RUQ pain, consider stones; and 5) otherwise if ALT<1000 consider hyperemesis
gravidarum or other drugs.

Question 7
A 10-old male with HIV on HART therapy is evaluated for elevated liver enzymes (ALT 119 IU/L, AST
101 IU/L, bilirubin 1.3 mg/dL, Alk Phos 390 IU/L). A liver biopsy showed numerous blood-filled cysts
that do not have an endothelial lining. This liver biopsy finding is most likely secondary to:
A. Cytomegalovirus
B. Protease inhibitors
C. Rochalimaea hensalae
D. Caroli’s disease
E. Congenital factors (e.g., cystic Von Meyenburg complexes)
Dr-Wahid Helmi

Suggested answer: C. Peliosis hepatis refers to blood filled cavities in the liver that develop from neo-
angiogenesis. Peliosis often also occurs in the spleen. The neo-angiogenesis is induced by Bartonella
spp. infection (formerly Rochalimaea, such as those that cause Cat Scratch disease) in the setting of
HIV/AIDS (CD4<100). In HIV negative individuals, hepatis peliosis can be caused by anabolic steroids,
Castleman’s disease, Hodgkin’s lymphoma, leukemia, or other malignancies. Diagnosis is made by
visualizing gram-negative bacilli in blood or biopsy specimens, and cultures are seldom positive.

Caroli’s disease and cystic von Meyenburg complexes are both developmental diseases associated with
polycystic kidney disease. Caroli’s disease has numerous dilated intrahepatic bile ducts lined with
defective cholangiocytes. Cystic von Meyenburg complexes are benign cystic tumors, also referred to as
bile duct hamartomas.

Question 8
All of the following statements regarding coagulation disorders in cirrhosis are true except:

A. Thrombopoietin is decreased in patients with cirrhosis.


B. Factor VIII is not depressed until the late stages of cirrhosis.
C. Bleeding is a common cause of death in patients with cirrhosis who undergo abdominal surgery.
D. Thrombotic complications are rare in patients with cirrhosis because of thrombocytopenia and
decreased clotting factor levels.

Suggested answer: D. Coagulopathies accompany liver disease, as hepatocytes synthesize both pro- and
anti- coagulation factors. As a result, bleeding is a common cause of death in cirrhotic patients who
undergo surgery, while thrombotic complications such as DVTs and PEs also occur. Factor VIII is high
(made in spleen in lymph nodes) in cirrhosis and is not depressed until late stages. The liver also makes
thrombopoietin, which promotes megakarocytes to make platelets. Thrombopoietin is low in cirrhosis,
and, along with platelet congestion in the spleen from portal hypertension, explains thrombocytopenia in
cirrhotic patients (liver transplant restores thrombopoietin levels to normal).

Question 9
Which of the following statements concerning esophageal varices is not true?

A. Esophageal variceal bleeding ceases spontaneously in 40% of patients.


B. The mortality from an episode of variceal hemorrhage is 30%.
C. Overall mortality is not significantly reduced with beta-blocker prophylaxis.
D. A hepatic venous portal gradient (HVPG) of greater than or equal to 20 mm HG predicts a poorer
one-year survival in a patient who has bled from varices.
E. Sclerotherapy is an effective method of preventing first time variceal bleeding and reducing
mortality.

Suggested answer: E. Variceal bleeding must be managed effectively, as the mortality from variceal
hemorrhage is 30-50%. Varices are unlikely to bleed when the HVPG is less than 12, whereas 1/3 of
patients in one study bled if the gradient was greater than 12. Prophylactic measures such as beta
blockers have been shown to prevent bleeds but have not improved mortality from variceal hemorrhage.
Prophylactic sclerotherapy or banding has not become standard practice. Prophylactic sclerotherapy did
reduce variceal bleeding but increased congestive hypertensive gastropathy in one study (no difference in
survival). In another study, 42% of patients bled after prophylactic sclerotherapy.

In the event of bleeding, only 40-50% episodes stop spontaneously. Sclerotherapy and band ligation have
proven effective measures to stop bleeding temporarily, and are considered standard measures to stabilize
a patient ahead of liver transplant.
Dr-Wahid Helmi

Question 10
All of the following drugs can cause microvesicular steatosis except:

A. Tetracycline
B. Aspirin
C. AZT
D. Alcohol
E. Troglitazone
F. Valproic acid

Suggested answer: E. Troglitazone is a PPARg agonist used to treat DM2, that has since been pulled off
the market for causing hepatitis. Histology shows hepatocellular injury, and liver failure can ensure
within 1-7 months. The other choices (tetracycline, aspirin, AZT, alcohol, and valproic acid) can all
affect mitochondria and cause mircovesicular steatosis, analogous to that seen with aspirin in Reye’s
syndrome.

Question 11
A 14 year-old female presents for evaluation of jaundice and fatigue. She has been taking phenytoin for
six years for a seizure disorder and levothyroxine for three years for hypothyroidism. Her review of
systems is positive for amenorrhea (three months). Laboratory exam shows ALT 684 IU/L, AST 388
IU/L, bilirubin 6.1 mg/dL, Alk Phos 199 IU/L, total protein 8.2 g/dL, albumin 3.1 g/dL, and PT 16
seconds. The most likely explanation for her illness is:

A. Phenytoin liver toxicity


B. Autoimmune hepatitis
C. Hyperthyroidism
D. Primary Biliary Cirrhosis
E. Choledocholithiasis

Suggested answer: B. This patient likely has autoimmune hepatitis. She has some early symptoms of
AIH, such as fatigue and amenorrhea. Other extra-hepatic symptoms include low grade fever, skin rash,
and joint pain. She has a pre-existing history of likely autoimmune thyroid disease. And her total protein
is elevated in the context of a low albumin, suggesting she is making copious immunoglobulins as would
occur in autoimmune states. Phenytoin toxicity and PBC are not associated with amenorrhea.
Hyperthyroidism usually does not elevate liver numbers. Choledocholithiasis would present with RUQ
pain.

Question 12
Which of the following statements concerning hereditary hemochromatosis (HH) is false?

A. The phenotypic expression in the United States is 1/200-1/250.


B. HH is the most common, identified, genetic disorder in Caucasians.
C. The genetic defect causes an excessive absorption of iron.
D. Compound heterozygosity (C282Y, H63D) accounts for 3-5% of cases.
E. HH should be considered in any male patient with transferrin saturation greater than 60 percent.

Suggested answer: B. Hereditary hemochromatosis is a disease of excessive gut iron absorption. Iron (in
the form of free iron or attached to heme) is processed by the brush border and then stored in enterocytes
as ferritin. The enterocytes can either pass the iron into the circulation via the transporter ferroportin or
carry the iron into the intestinal lumen and out of the body when the cells are sloughed. Iron that passes
Dr-Wahid Helmi

through the ferroportin channel is carried to the liver via the portal circulation and stored in zone 1
hepatocytes. Kupffer cells in the liver store iron from the systemic circulation, whereas reticulendothelial
cells in the spleen store iron from spent red blood cells.

Hereditary hemochromatosis is most commonly caused by the HFE (high Fe) gene. The gene controls
liver hepcidin expression. Hepcidin is a hormone which negatively regulates ferroportin. With mutant
HFE, then, hepcidin levels are low. As a result, ferroportin activity is increased, and more iron stored in
the enterocyte is allowed to enter the systemic circulation, even when excess iron is already present. The
iron becomes deposited in various end organs, where it causes a series of systemic complications. The
iron binds to its carrier transferrin, so a transferrin saturation >60% in males and >45-50% in females
should be concerning for hemochromatosis.

The genetics of HFE are complex. The incidence of HFE homozygosity in Caucasian/Celtic populations
is as high as 1:150-1:200. However, phenotypic expression ranges from 1:2 to 1:150, depending on the
what symptoms are accepted as consistent with disease (in the 1:2 study, many of the symptoms were
general and could be found in control subjects as well). As a result, the incidence of HH based on
phenotype may be as rare as 1:200 (incidence of homozygosity) X 1:150 (incidence of symptoms in
homozygous individuals) = 1:30,000. This is rarer than the 1:3200 incidence of CF, which is the most
common identified genetic disorder in Caucasians.

Question 13
All of the following statements regarding TPN liver disease are true except:

A. TPN liver disease is commonly associated with the development of gallbladder stones and sludge.
B. TPN liver disease is usually cholestatic in adults and associated with steatosis in infants.
C. TPN liver disease usually resolves following institution of enteral nutrition if cirrhosis has not
developed.
D. Low birth-weight infants are more susceptible than adults to TPN liver disease.
E. The combination of CCK and ursodiol does not reduce mortality from TPN-related liver disease.

Suggested answer: B. Liver disease is a mjor complication of TPN. Clinical cholestasis develops in 25%
of premature infants on TPN, with low birth-weight infants and those developing sepsis at highest risk.
Steatosis is transient in infants, whereas it is the defining characteristic in adults. 100% of infants on TPN
for more than 6 weeks develop calcium bilirubinate sludge (stones are much less common). Studies have
shown that CCK does not reduce gallstones, cholestasis, or mortality. Ursodiol can improve liver
numbers initially, but has not been shown to improve liver histology or mortality.

Question 14
Pellegra is associated with what vitamin deficiency?

A. Riboflavin
B. Thiamin
C. Ascorbic acid
D. Pyridoxine
E. Niacin

Suggested answer: E. Vitamin B1 (thiamine) deficiency causes “beri beri,” characterized by confusion,
peripheral paralysis, muscle weakness, tachycardia, and cardiomegaly. Vitamin B2 (riboflavin)
deficiency causes anemia, angular stomatitis, and seborrheic dermatitis. Vitamin B3 (niacin) deficiency
causes pellagra, consisting of dermatitis, diarrhea, and dementia. Vitamin B6 (pyridoxine) deficiency
causes tongue swelling, rash, and neuropathy. Vitamin B9 (folate) deficiency causes macrocytic anemia
Dr-Wahid Helmi

and macroglossia. Vitamin B12 (cyanocobalmin) deficiency also causes macrocytic anemia. Vitamin C
(ascorbic acid) deficiency causes bleeding gums, poor wound healing, and scurvy.

Question 15
A 24 month old infant who is TPN-dependent secondary to short gut syndrome has persistent anemia.
She does have TPN-cholestasis but does not have esophageal varices. There has been no evidence of
bleeding. On exam she is pale and mildly jaundiced. Her hemoglobin is 7.5 gm/dl and her MCV is 70.
What deficiency would result in her persistent anemia?

A. Carnitine
B. Selenium
C. B12
D. Copper

Suggested answer: D. Copper deficiency can occur with TPN use, especially because copper is often
omitted from formulations to avoid excessive copper accumulation (copper is secreted via bile, and
cholestasis impairs this secretion). Copper deficiency can cause iron anemia and microcytic anemia,
because copper is needed for enzymes involved in iron absorption (i.e. copper-dependent ferroxidase).
B12 deficiency causes megoblastic anemia, and can result from ileal resection in short bowel syndrome.
Selenium deficiency occurs if it is not supplemented in TPN, and leads to cardiomyopathy and Keshan’s
disease (perhaps in association with Coxsackie virus infection).. Carnitine deficiency also occurs if it is
not supplemented in TPN. Carnitine is needed to shuttle long chain fatty acids into the mitochondria, and
without it myopathy (including cardiomyopathy) develops.

Question 16
What is not a result of ascorbic acid deficiency?

A. perifollicular hemorrhage
B. subperiosteal hemorrhage
C. hyperkeratotic hair follicles
D. cheilosis

Suggested answer: D. Ascorbic acid (Vitamin C) deficiency is associated with enlargement and
hyperkeratosis of hair follicles, followed by proliferation of blood vessels around the follicle leading to
perifollicular hemorrhage. Subperiosteal hemorrhage is also possible, especially in infants.
Cheilosis/angular stomatitis is a characteristic of riboflavin (vitamin B2) deficiency.

Question 17
Vitamin A deficiency is manifested by all of the following symptoms except

A. keratomalacia
B. follicular hyperkeratosis
C. night blindness
D. xerosis
E. seborrhea dermatitis

Suggested answer: E. Vitamin A deficiency is characterized by night blindness, keratomalacia (dryness


and keratinized epithelium of the cornea), xerosis especially of the conjunctivae and cornea, and follicular
hyperkeratosis from blockage of hair follicles with keratin. Excessive vitamin A can cause seborrhea
dermatitis.
Dr-Wahid Helmi

Question 18
The standard deviation is defined as

A. The average of the squared differences between each observation and the mean.
B. Measure of the inaccuracy of the sample mean as a representative of the mean of the entire
patient population from which the sample was drawn.
C. A standardized measure of variation used to compare dispersion for variables with different units
of measurement.
D. Mean of the differences between each observation and their mean.

Suggested answer: C. Standard deviation measures how much variation there is in a data set from the
mean. It quantifies the variability of the population. It can be viewed as a standarized measure of
“dispersion,” or variation, around the mean. One standard deviation below and above the mean accounts
for 68.26% of the population, whereas two standard deviations below and above the mean accounts for
95.44% of the population.

If the data points represent the entire population, then the true mean can be calculated. In this case, the
population variance is calculated by subtracting the value of each point from the mean, squaring the
result, and then averaging it by dividing by the total number of values (choice A). The population
standard deviation is then calculated by taking the square root of the population variance.

If data points represent a sample of the population, then the mean of the data points will estimate the true
mean of the population. To account for this estimation, the variance of a sample is calculated by
subtracting the value of each point from the mean, squaring the result, and then dividing by the total
number of values minus 1. The sample standard deviation is calculated by taking the square root of the
sample variance.

When the data points represent a sample of the population, the standard error of the mean can be
calculated. It quantifies the how accurately the sample mean representing the true mean. In other words,
it quantifies the certainty/uncertaintly of the calculated mean (choice B). It is calculated by dividing the
sample standard deviation by square root of the number of data points (standard deviation of the sampling
distribution of the mean).

Question 19
The following statement is false regarding a correlation coefficient r:

A. A correlation coefficient is a measurement of the strength of the linear association between two
variables.
B. The correlation coefficient varies between -1 and +1.
C. It represents the proportion of the variability in an outcome variable that can be explained by its
linear association with a predictor variable and vice versa.
D. It can only be used when the variables of interest are continuous.

Suggested answer: C. The Pearson’s correlation coefficient r measures how well two continuous
variables vary with one another. -1 reflects perfect negative correlation, +1 perfect positive correlation,
and 0 no correlation. The square of the correlation (coefficient of determination) measures the amount of
variance in one group that can be associated with the amount of variance in the other group (choice C).
This question assumes r refers to the Pearson correlation coefficient; however, other correlation
coefficients, like the Spearman rank correlation coeficient, does not require variables to be continuous.

Question 20
Dr-Wahid Helmi

The following equation represents a linear regression line


A. r2
B. y = β1x1 +β2x2 + E
C. sd/√n
D. logit P(x) = α + βE+ γ1V1 + γ2V2 +δ1EV1

Suggested answer: B. A linear regression line models the relationship between two variables using a line.
A pre-requisite is that a correlation between two variables exists. The most common method for making
the line is the method of least-squares. This method calculates the best-fitting line for the data by
minimizing the sum of the squares of the vertical deviations from each data point to the line. Choice A is
the coefficient of determination and choice C is the standard error of the mean.

Question 21
The following statements are true regarding sensitivity

A. Probability of testing positive if the disease is truly present


B. Probability of having the disease given the results of the test
C. Should not be increased relative to specificity when the penalty associated with misdiagnosis is
high.
D. Can be increased by increasing the prevalence of disease in the screened population

Suggested answer: A. Sensitivity is the probability of a test being positive if the disease is truly present,
whereas specificity is the probability of the test being negative if the disease is truly absent. These
calculations do not depend on the prevalence of the disease. For screening tests (i.e. HIV screening tests),
in which the goal is to catch all diseased individuals even at the risk of having false positives, sensitivity
should be increased relative to specificity. In contrast to sensitivity and specificity, positive and negative
predictive values reflect the probability of having or not having the disease given a test result. These
values are influenced by the prevalence of the disease in the population.

Question 22
Using the following number set determine the mean, median, and mode

1, 2 ,3 ,4, 5 ,5 , 5

Suggested answer: Mean is (1+2+3+4+5+5+5)/7 = 3.57. Median is the middle number, or 4. Mode is the
most common value, or 5.

Question 23
The reaction of the internal anal sphincter illustrated below is mediated via which pathway?

A. Cholinergic inhibitory
B. Cholinergic excitatory
C. Adrenergic inhibitory
D. Adrenergic excitatory
E. Interstitial cells of Cahal

IAS
Pressure
(mmHg)
EAS

Rectal
balloon
Dr-Wahid Helmi

Suggested answer: E. When the rectum is distended, the IAS relaxes to allow stools to descend and the
EAS contracts to prevent defecation. The IAS relaxation is accomplished by non-adrenergic, non-
cholinergic (NANC) nerves that release nitric oxide (NO), vasoactive intestinal peptide (VIP), and
perhaps carbon monoxide. Rodent studies have also shown that Interstitial Cells of Cajal, the pacemaker
cells throughout the gut stimulating slow waves of peristalsis, are located in the IAS and may be involved
in IAS relaxation. (See Gut 2005;54:1107-1113. Interstitial cells of Cajal are involved in the afferent
limb of the rectoanal inhibitory reflex)

Question 24
The pathogenesis of Hirschsprung’s disease is thought to be related to which of the following?

A. Increased acetylcholine
B. Decreased glutamine
C. Decreased GABA
D. Decreased nitric oxide
E. Increased serotonin

Suggested answer: D. Hirschprung’s disease is caused by lack of ganglion cells in the mucosal
(Meissner’s) and myenteric (Auerbach’s) plexuses of the distal colon. These ganglion cells normally
come from migrating neural crest cells during development, and mutations in genes involved in neural
crest cell migration have been found in Hirschprung’s patients. The classic manometry finding in
Hirshprung’s disease is absent IAS relaxation when the rectum is distended. The relaxation normally is
mediated by nitric oxide, suggesting that nitric oxide-generating neurons are missing or defective in
Hirschprung’s disease.

Question 25
A 14 year old boy with biliary atresia who underwent a Kasai procedure at 5 weeks of age has progressed
to end stage liver disease and is listed for liver transplantation. He has had 2 major variceal bleeds in the
last 3 months and recently had a transjugular intrahepatic postosystemic shunt (TIPS) placed. The most
likely complication following this shunt placement is:

A. Infection
B. Encephalopathy
C. Progressive liver failure
D. Stent migration

Suggested answer: B. The most common complication of the TIPS procedure is encephalopathy, as the
procedure allows blood from the gut to bypass the liver filter and enter the systemic circulation directly.
Encephalopathy happens in as many as 25-34% adult cases, and usually occurs immediately after the
procedure. The second most common complication is shunt occlusion, which can develop within 24
hours of the procedure. Heart failure can also occur, as more blood is returned to right heart following
TIPS. Stent migration has been reported, including cases in which the stent caused atrial puncture.
Dr-Wahid Helmi

Question 1:
In the pediatric population, which of the following is the most common route of HCV transmission:

A. materno-foetal
B. IV drug use
C. blood products
D. sexual contacts
E. tattoos

Suggested answer: A. Since blood product screening started in 1991, most childhood HCV cases occur
through perinatal transmission. Adolescents and adults acquire HCV through percutaneous (i.e. IV drug use)
and non-percutaneous (i.e. sexual transmission) routes. Acute infections are often asymptomatic. Chronic
infections, caused by incomplete immune-mediated attack against infected hepatocytes, lead to fibrosis and
cirrhosis, and accounts for the most number of adult liver transplantations worldwide.

Question 2:
Which of the following is NOT associated with Wilson’s Disease:

A. Fatty liver
B. High serum uric acid
C. Low serum zinc level
D. Low serum alkaline phosphatase
E. High serum bilirubin

Suggested answer: B. Wilson’s disease is characterized by poor copper intracellular trafficking and
extracellular transport, caused by a defect in the copper-transporting adenosine triphosphatase (ATPase)
gene (ATP7B). It is the most common reason for fulminant hepatic failure in children greater than 3 years.
The earliest histological changes include microvesicular and macrovesicular fatty deposition and
glycogenylated nuclei, which progresses to inflammation and fibrosis.

Wilson’s disease can present as a wide spectrum, from modest elevations in transaminases to acute hepatitis
with high serum bilirubin levels. Wilson’s disease also has characteristically low zinc and alkaline
phosphatase levels, though the mechanism has not been completely resolved (zinc is a treatment in Wilson’s
disease, inducing metallothionein which traps copper in enterocytes). Uric acid levels are low in Wilson’s
disease, because copper accumulates in renal tubular cells. This causes a Fanconi-like syndrome and uric
acid wasting into the urine.

Question 3:
Which statement is false:

A. 90% of Vit. A is stored within the liver


B. Vit. A deficiency Xeropthalmia is irreversible
C. Can cause a hypochromic microcytic anemia with low Fe but normal Fe stores
D. Hypervitaminosis A is associated with head aches

Suggested answer: B. Vitamin A comes from plant and animal sources. Vitamin A from plant sources is in
the form of Provitamin A, which is converted to Vitamin A in a series of highly regulated steps that depend on
whole-body Vitamin A status. Vitamin A from animal sources is already in the active form, so excessive
intake can lead to Vitamin A toxicity. Once Vitamin A is absorbed, in enters the lymphatic system through
chylomicrons, then the venous system at the thoracic duct where it is metabolized into remnants such as
Dr-Wahid Helmi

apoliproteins B and E, and eventually endocytosed into the liver. 50-90% of total body retinol is stored in
stellate (Ito) cells, and transports to other organs through the serum by binding to retinol binding protein.

Vitamin A deficiency is uncommon in the developed world, but the third most common nutritional deficiency
world-wide. It presents as xerophthalmia (abnormal keratinization of conjunctiva secondary to poor lacrimal
gland secretion), poor bone growth, non-specific skin problems, and decreased humoral and cell mediated
immune function. Vitamin A deficiency can also lead to anemia, presumably by inhibiting the normal
metabolism of iron. Supplementation can reverse many of these problems; however, advanced corneal
scarring may be irreversible.

Vitamin A toxicity, on the other hand, presents with a myriad of symptoms, including dry skin, headaches,
hepatomegaly, and increased CSF pressures (pseudotumor cerebri).

Question 4:
The most common route of transmission for new onset Hepatitis C in the pediatric population in United States
and Europe is:

A. Drug Abuse
B. Male to male sex
C. Tainted blood supply
D. Vertical transmission

Suggested answer: D. Please see question1 in this set. Vertical transmission accounts for as much as 65% of
pediatric cases of HCV infection.

Question 5:
Which of the following variables DOES NOT predict a higher likelihood of response to interferon therapy in
children with chronic hepatitis B:

A. Active inflammation on liver biopsy


B. Higher interferon dose
C. Female gender
D. Low level of baseline HBV DNA
E. Elevated serum transaminase levels

Suggested answer: B. Interferon has at least two antiviral mechanisms: (1) increases expression of anti-
viral genes, and (2) stimulates the immune system to eliminate HBV. The immune-stimulatory effects of
interferon explains why responders first have a burst of elevated aminotransferases (presumably from the
host immune system attacking infected hepatocytes), followed by normalization of aminotransferases, loss
of serum HBV-DNA, and loss of viral antigens HbeAg and HbsAg.

Hence, interferon may help those children already trying to mount an immune response against infected
hepatocytes (those with active inflammation on liver biopsy and those with ALT greater than or equal to 2
times the upper limit of normal). Furthermore, interferon may promote a better response when there is
less virus to fight, i.e. in those with lower baseline serum HBV DNA levels. Interferon has also been
shown to have a better response in females. Increasing interferon doses or priming with prednisone has
not been shown to improve outcomes.

Question 6:
Which of the following is NOT a common feature of BOTH kwashiorkor and marasmus:
Dr-Wahid Helmi

A. Irritability
B. Decreased serum lipoproteins
C. Markedly Depressed serum albumin
D. Increased susceptibility to infection
E. Anemia

Suggested answer: C. There are two subtypes of severe protein-energy malnutrition (PEM): kwashiorkor and
marasmus. Kwashiorkor is characterized by muscle atrophy and increased body fat, secondary to poor
protein intake in the setting of adequate energy intake. Marasums, on the other hand, is characterized by
muscle wasting and depleted fat stores, secondary to inadequate intake of all nutrients. Severe PEM of both
types produces a number of signs and symptoms: irritability, decreased serum lipoproteins, increased
infection risk, and anemia. However, kwashiorkor classically presents with severely low serum albumin
concomitant with edema. Marasmus presents with low-normal serum albumin with wasting but no edema.

Question 7:
Tyrosinemia is associated with:

A. Boiled cabbage smell


B. Mousey smell
C. Blue Cheese Vinaigrette smell
D. Resident post call smell
E. All of the above except D

Suggested answer: A. Tyrosinemia is caused by a defect in metabolizing tyrosine into its two products,
acetoacetate (ketogenic) and fumarate (glucogenic). Five enzymes are involved in tyrosine metabolism,
and defects in fumarylacetoacetate hydrolase (FAH) catalyzing the last enzymatic step leads to the most
severe form of tyrosinemia (Hereditary tyrosinemia Type 1). Fumarylacetoacetate (FAA) accumulates,
damaging renal and liver cells. FAA is metabolized into succinylacetoacetate and succinylacetone which
are measured in the urine to make the diagnosis. Nitisinone, which blocks an earlier step in the tyrosine
metabolism pathway, is used as treatment to prevent FAA accumulation.

Patients with tyrosinemia may present with a “boiled cabbage” smell. Others describe the smell as
“rotten mushrooms” or “rancid butter.” Patient with PKU (inability to break down phenylalanine) have
a “mousy,” “musty,” “wolf-like,” “barny,” “horsey,” or “stale” smell. Isovaleric acidemia patients
have a “cheesy” “acrid” odor of “smelly feet.”

Question 8:
Which statement about Hepatitis A is true:

A. Leading cause of fulminant hepatitis in Pediatrics


B. Has been associated with chronic hepatitis
C. Recurrence of the disease can occur up to 6 months after primary infection
D. Treatment for non-fulminant hepatitis A includes Lamivudine for 4 weeks
E. Severity of disease decrease with increasing age

Suggested answer: C. Hepatitis A is an RNA virus which replicates in hepatocytes and spreads. The
virus per se does not cause symptoms; rather, cell-based immunity targeting infected hepatocytes leads to
jaundice, elevated aminotransferases, and other symptoms. Older children are more severely affected,
perhaps because they have a more developed immune response (70% under 6 years old are
Dr-Wahid Helmi

asymptomatic, whereas 70% of older children/adults are symptomatic). Treatment is largely supportive,
though vaccine and/or immunoglobulin administration to previously-unvaccinated exposed individuals
has been shown to limit symptoms. Relapse can occur, typically only once within 6-9 months after the
initial infection (“relapsing Hepatitis A”). HAV infection may also trigger autoimmune hepatitis in
predisposed children, but HAV on its own has never been to cause a chronic infection.

Fulminant hepatitis occurs when the host’s immune response is excessive. HAV infection accounts for
only 1% of pediatric acute liver failure cases in the US (but up to 60% in Latin America).

Question 9:
Symptoms and signs NOT associated with Kwashiorkor (see picture) include:

A. Flaky paint sign


B. Hanover’s sign
C. Flag sign
D. Proportional weight for height
E. Fatty liver

Suggested answer: B. Kwashiorkor is characterized by decreased protein intake but normal or above
normal energy intake, and presents with edema and low serum albumin. Kwashiorkor patients have
normal weight and height for age, anasarca, and dry, atrophic, peeling skin (“flaky-paint sign”) with
areas of hyperkeratosis and hyperpigmentation. They also have hepatomegaly (from fatty liver
infiltrates) and a distended abdomen with dilated intestinal loops. The hair of kwashiorkor patients is
dry, fargile, and hypopigmented. During periods of adequate protein intake, hair color is restored and
produces bands of normal color (the “flag sign”).

Question 10:
Acute lower GI hemorrhage in HIV infected patients is most often caused by:

a. CMV colitis
b. Lymphoma
c. Kaposi's sarcoma
d. Idiopathic chronic colitis
e. Nonspecific colitis

Suggested answer: C. HIV infected patients, especially those with low CD4 counts in the pre-HAART
era, were especially vulnerable to GI problems of infectious or malignant origin. CMV colitis presents
similarly to inflammatory bowel disease, with severe cases causing mucosal ulceration and hemorrhage.
Dr-Wahid Helmi

Additionally, HIV-related non-Hodgkin’s lymphoma frequently presents in the GI tract, with all
segments from mouth to anus including biliary system vulnerable (vs. non-HIV related GI lymphomas, in
which the stomach is most commonly affected). Severe cases may also produce rectal bleeding. The most
common cause of GI bleeding in HIV patients is Kaposi’s sarcoma. This malignancy is a vascular tumor
caused by HHV-8, and appears as hemorrhagic nodules on colonoscopy. It has been associated with
both upper and lower GI bleeds.

Question 11:
First line of treatment of esophageal candidiasis in HIV infected patient is:

A. Clotrimazole
B. Ketoconazole
C. Fluconazole
D. Amphotericin B

Suggested answer: C. Esophageal candidiasis presents with pain on swallowing, and occurs in HIV
infected patients with CD4 counts <100 cells/ul. Unlike oropharyngeal thrush which can be treated with
topical treatments such as clotrimazole, esophageal candidiasis always requires systemic antifungal
therapy. Fluconazole is the intial treatment of choice, and has been shown to be more effective than
ketoconazole in randomized trials. Amphotericin B must be given IV and has a number of side effects,
making it a poor first choice.

Question 12:
Organisms causing intestinal microsporidiosis in AIDS patients is:

A. Encephalitozoon intestinalis
B. Cryptosporidium
C. Isospora belli
D. Enterocytozoon bieneusi

Suggested answer: A and D. Microsporidiosis is one of the commonest enteropathogens in HIV/AIDS


patients. Microsporidiosis are small, spore-forming, intracellular organisms most closely related to
fungi. In enterocytes, they distort villous architecture and impair absorption without much concomitant
inflammation, leading to watery diarrhea. The most common species affecting humans is Enterocytozoon
bieneusi (normally remain local), followed by Encephalitozoon intestinalis (can spread systemically via
macrophages). Treatment is with albendazole, though it less effective for E. bieneusi.

Isospora belli and Cryptosporidium also affect immunocompromised patients. In contrast to


microsporidiosis, they are protozoa. Both cause gastric/small bowel symptoms, and cryptosporidiosis is
associated with voluminous watery diarrhea.

Question 13:
The most common non-opportunistic protozoon parasite is AIDS patients is:

A. Cryptosporidium
B. Gardia lamblia
C, Blastomyces hominis
D. Entamoeba histolytica
E. Toxoplasma gondii
Dr-Wahid Helmi

Suggested answer: D. The most common gastrointestinal pathogens in HIV patients include
Cryptosporidium parvum, Isospora belli, and Entamoeba histolytica. Of these, Cryptosporidium and
Isospora are considered opportunisitc because they require the host to be immunocompromised to cause
disease. Entamoeba histolytica, on the other hand, affects immunocompromised and immunocompetent
individuals and is considered non-opportunistic.

Blastomyces dermatitidis (not hominis) is an opporunisitic fungus that typically does not affect the GI
tract, whereas Blastocystis (not Blastomyces) hominis is a non-opportunistic protozoa whose role as a
commensal versus pathogenic organism is still unclear. Giardia lamblia is a protozoan causing
prolonged diarrhea in HIV/AIDS patients as well as immunocompetent patients. Finally, Toxoplasma
gondii is an opportunistic protozoan that presents typically with encephalitis, pneumonitis, or
chorioretinitis, but many also involve the gastrointestinal tract.

Question 14:
The most common opportunistic infection in HIV patients is:

A. Mycobacterium Avium Complex


B. Mycobacterium Tuberculosis
C. Salmonella Typhimurium
D. Clostridium Difficile

Suggested answer: B. While the statistics and definition differ in various studies (some report esophageal
candidiasis and Pneumocystitis jiroveci as the most common opportunistic infections), Mycobacterium
tuberculosis is one of the commonest opportunistic infections in HIV. Ongoing studies are investigating
the relationship between TB and HIV, with results suggesting that TB may not only increase after HIV but
lead to a more serious course of disease.

Other common opportunistic pathogens include Mycobacterium Avium Complex (MAC, comprising M.
avium and M. intracellulare) causing chronic abdominal cramps and bloating, as well as Salmonella
typhimurium causing hemtochezia, tenesmus, and lower abdominal cramps. Clostridium difficile is not
an opportunistic infection, though it is very common in HIV patients presumably secondary to frequent
antibiotic use.

Question 15:
Which of the following is NOT found in Wolman's disease (cholesterol ester storage disease)

A. Orange coloured liver


B. Lipid laden macrophages in the portal triad
C. Blueish hue to some hepatocytes
D. Diffuse steatosis
E. Inflammation

Suggested answer: C. Wolman disease is caused by a defect in lysosomal acid lipase (LAL). As a result,
lysozymes receive endocytosed lipoproteins properly, but they cannot hydrolyze the triglycerides and
cholesterol esters. The triglycerides and cholesterol esters accumulate inside cells, leading to bowel wall
thickening (accumulation in enterocytes and macrophages) and severe life-threatening diarrhea and
malnutrition. In Wolman disease, the adrenal glands are also calcified.

The liver in Wolman disease is enlarged and appears yellow/orange and greasy in appearance (the orange
comes more from the cholesterol esters than the triglycerides). There is extensive fibrosis, associated with
lymphoid infiltration and accumulation of triglycerides/cholesterol esters in hepatocytes, Kupffer’s cells, and
Dr-Wahid Helmi

portal area macrophages. In iron storage disorders, rather than Wolman disease, hepatocytes stained for
iron may have bluish-hue reflecting excess ferritin in the cytoplasm.

Question 16:
A 5-month-old girl is referred for evaluation of poor growth and irritability. The history is significant for
constant spitting up of formula. An upper gastrointestinal series reveals some gastroesophageal reflux but
is otherwise normal. Which one of the following most likely explains these findings?

A. Prone positioning after feedings


B. Stress in the home
C. Immaturity with low pressures of the lower esophageal sphincter
D. Inappropriate relaxation of the lower esophageal sphincter
E. Pyloric stenosis

Suggested answer: D. This patient has gastroesophageal reflux disease (GERD) as opposed to
uncomplicated reflux, because the reflux is causing pathological symptoms such as poor growth. She
does not have anantomical defects such as malrotation, annular pancreas, or even pyloric stenosis
because her UGI was normal. She may have cow’s milk protein allergy, which can cause similar
symptoms. However, the most common explanation for GERD is transient relaxations of the LES
separate from the normal LES relaxation that occurs with swallowing.

The prone position has actually been shown to decrease reflux events, though in most cases the risk of
SIDS outweighs risks caused by reflux. The patient is 5 months old and should not have an immature LES
with consistently low pressures. Finally, stress in the home is more often correlated with adolescent, and
adult GERD.

Question 17:
A 7-year-old girl who had undergone a surgical repair for long-segment Hirschsprung's disease in early
infancy presents with a fever, abdominal distention, and bloody diarrhea for 2 days. Which of the
following is the most likely diagnosis?

A. diversion colitis
B. enterocolitis
C. ulcerative colitis
D. colonic stricture
E. viral gastroenteritis
Suggested answer: B. This patient has Hirschsprung-associated enterocolitis (HAEC). The disease most
commonly occurs within the year after correction though may occur before or many years after repair. It
presents with diarrhea, fever, vomiting, and sometimes bloody stools. Imaging shows air-filled loops of
bowel with no air in the rectosigmoid colon (“cut-off sign”). The pathogenesis is thought to be related to
intestinal stasis and bowel overgrowth proximal to the agangiolic segment or proximal to an anastomotic
stricture. Bacteria invade the intestinal wall, leading to disease. HAEC occurs in ~1/4 of patients with
Hirschsprung’s disease, and at one time had a mortality as high as 33%.

Diversion colitis is also caused by stasis; however, this patient does not have any blind loops of bowel.
UC should have a more chronic course. Colonic (anastomotic) strictures are associated with HAEC, but
HAEC can occur without strictures and strictures can exist without HAEC. Finally, viral gastroenteritis
would not be expected to produce bloody diarrhea and the severe symptoms found in this case.
Dr-Wahid Helmi

Question 18:
A 6-year-old boy just arriving from Eastern Europe has had malodorous diarrhea since early infancy, even
though he was breast-fed. He is small, has some bruises from bumping into furniture going to the
bathroom at night, and has recently developed some difficulty walking. Physical examination shows that
he is small and undernourished, with depleted subcutaneous fat. He has a protuberant abdomen and 1+
edema in his lower extremities. He has no deep tendon reflexes in his lower extremities. Which one of the
following explains the finding on the small intestinal biopsy from this patient?

A. gluten enteropathy
B. congenital lactase deficiency
C. abetalipoproteinemia
D. glucose-galactose transport defect
E. chronic nonspecific diarrhea of childhood

Suggested answer: C. This patient has evidence of fat malabsorption (wasting), including Vitamin K
(bruising), Vitamin E (decreased deep tendon reflexes), and perhaps even Vitamin A (night blindness or
retinitis pigmentosa). Abetalipoproteinemia (ABL) can cause such symptoms.

The body uses lipoproteins to shuttle cholesterol esters and triglycerides among organs. Lipoproteins
have a core of cholesterol esters and triglycerides, surrounded by a monolayer of cholesterol,
phospholipids, and dedicated lipoprotein peptides called apolipoproteins. There are many types of
apolipoproteins, with beta-apolipoproteins being the largest. Furthermore, different lipoprotein particles
have different apolipoproteins. For example, chylomicrons (a lipoprotein particle made in enterocytes)
carry beta-apolipoprotein 48 (ApoB-48), whereas VLDL (a lipoprotein particle made in hepatocytes)
carry beta-apoliprotein 100 (ApoB-100).

In ABL, enterocytes can absorb lipids properly but cannot exocytose the chylomicon particles from their
basolateral membrane into the systemic circulation. The defect is caused by a mutation in microsomal
triglyceride transfer protein (MTP). As a result, patients with ABL have lipids trapped in their
enterocytes, leading to fat malabsorption, diarrhea, and severe malnutrition. Without chylomicrons to
deliver ingested lipids, they have low levels of lipoprotein particles and their corresponding
apolipoproteins. Perhaps the most serious consequence is Vitamin E malabsorption. By 2-6 years,
infants show profound Vitamin E deficiency manifested by symptoms such as retinopathy and
spinocellular degeneration. Treatment is a medium chain triglyceride diet and copious Vitamin E
supplementation.

Gluten enteropathy (celiac disease) would have started only after exposure to gluten, not while the child
was still exclusively breast feeding. Congenital lactase deficiency is rare and would have led to diarrhea
and malnutrition, without the extra neurological signs. Similarly, glucose-galactose transporter defects
are rare and lead to severe diarrhea and dehydration in infants. They are caused by defects in the
sodium/glucose cotransporter (SGLT1), and can be treated with fructose-based formulas.

Question 19:
An 11-year-old girl presents with a history of epigastric abdominal pain for 2 months. The pain is worse
after meals and has occasionally awakened her from sleep. She denies diarrhea or constipation, skin rash,
fever, or mouth sores. Physical examination is completely normal. An upper gastrointestinal radiograph
reveals a duodenal ulcer. Which one of the following is indicated at this time?

A. upper endoscopy with biopsy


B. Helicobacter pylori antibody titers
C. fasting serum gastrin level
Dr-Wahid Helmi

D. colonoscopy with biopsy


E. cytomegalovirus antibody titer

Suggested answer: A. This patient has a duodenal ulcer. Endoscopy is the gold-standard for diagnosing
ulcers, because radiography depends on patient compliance/flexibility and can miss up to 50% of
duodenal ulcers. Endoscopy in this case has many advantages: i) allows for therapeutics in cases where
the ulcer is bleeding/at risk for bleeding; ii) allows for visualization of other ulcers as in hypersecretory
disorders; and iii) allows for diagnosis of H. pylori.

H. pylori antibody titers are not used in pediatrics, in part because standards for IgG levels do not exist
and infections are usually too chronic for abnormal IgM levels. A fasting serum gastrin level could be
useful to diagnose Zollinger-Ellison syndrome; however, this disease usually is diagnosed in patients
between 20-50 years old. A colonoscopy would be useful if the upper endoscopy showed signs of Crohn
disease, but would not be an appropriate first test. CMV gastrointestinal disease is very rare in
immunocompetent hosts. If present, upper endoscopy with viral studies on biopsy specimens would be a
better test than measuring serum antibody titers.

Question 20:
A 9-month-old boy has irritability and regurgitation. Growth and development are normal, as is his
physical examination. Upper gastrointestinal radiograph is normal with no gastroesophageal reflux,
normal anatomy, and no obstructive lesions. Upper endoscopy is normal. The results of an esophageal
biopsy are shown in the figure (Genevay et al. Archives of Pathology and Laboratory Medicine 2010,
134: 815-825). What is the most likely mechanism that explains these findings?

A. achalasia of infancy
B. inappropriate relaxation of the lower esophageal sphincter
C. protein intolerance
D. infection of the gastrointestinal tract
E. gluten enteropathy

Suggested answer: C or B (Note: No picture was included in the original question) Picture A shows
esophageal findings that may be present in infants with protein intolerance. There are many eosinophils
with superficial layering, basal cell hyperplasia (>50% of epithelial cell thickness), and dilated
intercellular spaces. Eosinophilic abscesses are seen in the inset. Picture B shows reflux, or
Dr-Wahid Helmi

inappropriate relaxation of the LES. Basal cell hyperplasia, papillary elongation, and fewer eosinophils
are present.

Achalasia of infancy is very rare, would appear on UGI (dilated esophagus, distal taper), and usually
does not have histological findings on routine upper endoscopy biopsy. Reflux may or may not be present
on UGI, and endoscopy findings are nonspecific. Mild protein intolerance would be undetected by UGI
or endoscopy, though severe cases in older children may be seen as narrowing on imaging and furrowing
on endoscopy. Celiac disease has small intestinal, not esophageal, findings.

Question 21:
A 15-year-old girl with chronic ulcerative pancolitis complains of recent recurrence of diarrhea and
bleeding while taking Azulfidine 4.0 gm/d. Although she had a difficult course during the first year of
disease, she subsequently has not required corticosteroid therapy for over 9 years. Which one of the
following endoscopic findings has the most ominous prognosis?
A. pancolitis
B. stricture in the ascending colon
C. pseudopolyps in the rectum and sigmoid colon
D. inflammatory infiltrates noted on endoscopic ileal biopsy
E. lack of haustra in the transverse colon

Suggested answer: B. Prognosis in UC relates to the extent of disease, with 20% of adults with
proctitis/distal colitis resolving spontaneously in some studies. Pseudopolyps in the rectum and sigmoid
colon suggest relatively distal disease, whereas lack of haustra (a sign of chronic UC) in the transverse
colon identifies more proximal disease. Pancolitis is consitent with the entire colon being involved, with
some inflammatory infiltrates spilling over to the terminal ileum in 10-15% of cases (“backwash ileitis”).

Unlike in Crohn disease, in UC strictures are rare and concerning for neoplasm. UC patients most
vulnerable are those with long-standing (>7 years), more extensive (beyond the splenic flexure) disease
as in this case. Strictures should be sampled by endoscopy to rule out colon cancer. Benign strictures
can also occur in UC, usually from repeated bouts of inflammation and muscle hypertrophy.

Question 22:
A 9-year-old presents with abdominal pain, nausea, and vomiting continuing for several weeks. Upper
endoscopy reveals some erythema and a few superficial erosions in the antrum. Gastric biopsies reveal
mild chronic inflammatory infiltrates and tight ~5-7 m spiral-shaped bacteria in the mucosa. A 13C-Urea
breath test was negative. Which one of the following is the most appropriate next step in treating this
child?

A. penicillin G, single intramuscular dose


B. omeprazole, amoxicillin, and metronidazole orally for 2 weeks
C. amoxicillin orally for 6 weeks
D. H-2 blocking agent for 8 weeks
E. amoxicillin and metronidazole for 1 week

Suggested answer: B. This patient has chronic gastritis caused by Helicobacter heilmanni. Both H.
heilmanni and H. pylori are spiral-shaped; however, H. heilmanni gives a negative urea breath test. H.
heilmanni compared to H. pylori usually produces a less severe gastritis, with fewer ulcers/erosions that
are limited to the antrum. Intestinal metaplasia and MALT is less common with H. heilmanni as well.
Treatment is similar to treatment for H. pylori, and consists of a PPI plus two antibiotics, usually
Dr-Wahid Helmi

amoxicillin and clarithromycin or flagyl and clarithromycin for 2 weeks. Older regimens includes a PPI,
amoxicillin, and flagyl for 2 weeks.

Question 23:
The barium enema shown in the figure (no figure included in original question) is obtained in a young
infant with abdominal distention and vomiting. Which one of the following is the most likely diagnosis?
A. intussusception
B. Hirschsprung's disease
C. malrotation
D. distal intestinal obstruction syndrome
E. anal stenosis

Suggested answer: B. In Hirschsprung’s disease, a barium enema can identify the aganglionic segment.
The area usually shows a thin lumen and colonic dilation proximally. Barium enemas are no longer used
when there is a risk for perforation, as in intussusception and distal intestinal obstruction syndrome
(partial or complete obstruction of the ileocecal area by intestinal contents, commonly seen in CF
patients with pancreatic insufficiency and/or dysmotility). Rather, water-soluble contrast materials are
used to reduce the intussusception or diagnose DIOS/clear the obstruction. A barium swallow, not
barium enema, makes the diagnosis of malrotation, whereas no imaging is needed to diagnose of anal
stenosis.

Question 24:
A 4-month-old formula-fed girl presents for evaluation of rectal bleeding. The remainder of the history is
unremarkable, and the physical examination is normal except for heme occult positive yellow stools. The
figure (no figure included in the original question) is a photograph taken at sigmoidoscopy. Which one of
the following is the appropriate next step?
Dr-Wahid Helmi

A. remove all cow and soy protein from the infant's diet
B. begin sulfasalazine
C. obtain an abdominal CT scan
D. blood test for APC gene abnormalities
E. give reassurance only

Suggested answer: A. This patient likely has cow’s milk protein allergy in response to cow’s milk in
formula. The condition leads to mild colitis with patches of erythema in the rectum, and histology shows
eosinophils. Treatment is eliminating cow’s milk protein from the diet and using semi-elemental formulas
(soy formulas are discouraged because a number of infants will also be sensitive to proteins in soy milk).
After switching formulas, bleeding may resolve as fast as 72 hours or as long as a few weeks. Finally,
nearly all children will outgrow the condition by 1 year of age.

In cow’s milk protein allergy, the histological findings resolve with diet change and there is no need for
anti-inflammatory medication. Furthermore, the bleeding is usually mild and imaging, such as a CT
scan, is not needed. Colonic polyps and cancer are extremely rare in infancy, and without a family
history of colon cancer gene testing is not warranted. In a select group of patients – those thriving on
breast milk without any signs of anemia, whose mothers have already eliminated dairy and insist on
breast-feeding – reassurance may be appropriate next step.

Question 25:
A 5-year-old boy is referred for evaluation of liver disease after presenting to his primary physician with
chronic pruritus. His evaluation reveals a small child (below the fifth percentile for height; weight for height
tenth percentile) with excoriations on his trunk and extremities. He has no icterus. A grade 2/6 systolic
murmur is heard at the left upper sternal border. His liver is soft, about 1 cm below the right costal margin
and nontender. Spleen was not palpable. He has diminished but symmetric deep tendon reflexes in his lower
extremities. Laboratory studies reveal:

Hemoglobin 12.8

Platelet count 239,000

AST 129

ALT 134

Alkaline phosphatase 678

GGTP 948

Total bilirubin 0.7

Prothrombin time 13.9

INR 1.2

Which one of the following is the most likely diagnosis?


A. progressive familial intrahepatic cholestasis (e.g., Byler's disease)
B. sclerosing cholangitis
C. Niemann-Pick disease, type A
D. Alagille syndrome
Dr-Wahid Helmi

E. alpha-1-antitrypsin deficiency

Suggested answer: D. With pruritis and a heart defect, this patient likely has Alagille syndrome. Alagille
syndrome is characterized by paucity of bile ducts, pulmonic stenosis, butterfly vertebrate, posterior
embryotoxin, and dysmorphic facies (triangular-broad forehead with small pointed mandible). Patients often
are malnourished secondary to fat malabsorption, and show signs of fat-soluble vitamin deficiency (including
Vitamin E deficiencies causing diminished reflexes, as in this case). Some patients also experience extreme
pruritis, which may require a biliary diversion or even liver transplantation. Alagille syndrome is most
commonly caused by mutations in Jag1, which serves as a ligand for Notch receptors during cholangiocyte
cell fate specification.

PFIC1, or Byler’s disease, causes growth failure and pruritis. It is associated with diarrhea, and has coarse
granular bile on EM. However, similar to PFIC2, GGT levels are usually normal. Sclerosing cholangitis is
characterized by pruritis, can precede a diagnosis of IBD (especially UC), and is not associated with heart
findings. Niemann-Pick disease type A is caused by pathological accumulation of the phospholipid
sphingomyelin in the monocyte-macrophage system, leading to fatty accumulation in the liver, spleen, and
CNS (leading to early neurodegeneration). The disease is caused by a mutation in the lysozymal enzyme
sphingomyelinase encoded by the SMPD1 gene. Finally, A1AT deficiency has a variety of liver presentation,
with some children showing mild increases in transaminases and others progressing quickly to cirrhosis and
liver failure. Similar to PSC, A1AT is not associated with cardiac defects.
Dr-Wahid Helmi

Question 1:
A 5-month-old infant who was bottle fed cow milk-based formula had severe watery diarrhea for 4 days.
It resolved within 24 hours of beginning an oral rehydration solution. Efforts to restart the previous
formula result in a return of severe diarrhea.

At this time, you are MOST likely to recommend a 5-day course of:

A. elemental formula
B. evaporated milk
C. goat milk
D. lactose-free formula
E. oral rehydration solution

Suggested answer: D. This patient suffers from secondary lactose malabsorption. One possible
pathogenesis model is: a) viral infection blunts small intestine villi; b) without villi, there is insufficient
lactase (lactase sits on the tips of villi); c) lactose from milk cannot be digested; and d) lactose is
delivered to colonic bacteria, which ferment it into osmotically active substances, causing diarrhea.
Temporarily switching to a lactose-free formula would address the lack of lactase.

One unexplained issue is why secondary lactose malabsorption can take many weeks to resolve, even if
villi are reformed properly. Some have proposed that there is something special about the disaccharidase
lactase. Others have suggested that other factors rather than lactase deficiency may be responsible,
including chronic small intestinal infection after the initial insult.

Question 2:
In which of the following conditions is intermittent (bolus) feeding PREFERRED to continuous
nasogastric feeding?

A. Congenital heart disease with failure to thrive


B. Gastroesophageal reflux with failure to thrive
C. Inflammatory bowel disease with bloody diarrhea
D. Malabsorption syndrome with severe villous atrophy
E. Oromotor discoordination due to birth asphyxia

Suggested answer: C. Continuous feeds are problematic because they require the patient to be attached to
the pump constantly, and because they ignore normal physiological patterns of feeding and fasting.
However, they are useful when the stomach cannot handle large volumes. For example, in congenital
heart disease, large volumes may divert too much blood to the gut and away from other vital organs. In
reflux, large volumes may promote more vomiting and, if there is oromotor dysfunction, may lead to
aspiration pneumonia. In malabsorption syndromes, neither bolus nor continuous feeding would be
expected to provide adequate nutrition, and parenteral nutrition would be needed. Patients with
inflammatory bowel disease affecting the colon should have intact gastric tissue, and should be able to
handle bolus feeds.

Question 3:
In the preterm infant, supplementation of human milk or use of formulas containing increased
concentrations of fat and protein often is necessary for adequate nutrition.

Of the following, the BEST explanation for this need for supplementation is that:

A. amino acids are poorly absorbed in the preterm infant


Dr-Wahid Helmi

B. bile acid production in the preterm infant is normal but pancreas activity is deficient
C. fat in human milk is of low caloric value
D. malabsorption of up to 20% of ingested fat is common in the preterm infant
E. medium-chain triglycerides are poorly absorbed in the preterm infant

Suggested answer: D. Premature infants need increased nutrients for a number of reasons: 1) they may
have been malnourished in utero (prompting the early delivery); 2) they undergo a number of challenges
post-natally, including respiratory problems, sepsis, and temperature control; and 3) they have an
immature digestive system and impaired absorptive capacity. Premature infants have the most trouble
digesting lipids, as both bile acid and pancreatic lipase secretion are reduced. Amino acids, on the other
hand, should be absorbed well because intestinal amino acid/peptide transporters are intact.

Fat in human milk is the major calorie source. Furthermore, fat from human milk is better absorbed by
premature infants, in part because human milk also contains lipase. Human milk fat only contains
approximately 12% medium-chain triglycerides. However, because MCTs can be absorbed without bile
salts and lipase, premature formulas have MCT concentrations as high as 40%. Importantly, in infants
this increased MCT concentration in formulas has never been shown to improve absorption or growth.

Question 4:
A 6-month-old infant who has congenital heart disease has grown poorly due to insufficient caloric
intake. You recommend nasogastric feeding to enhance weight gain.

Of the following, the MOST common complication of nasogastric feeding is:

A. allergic reaction to the nasogastric tube


B. dehydration due to diarrhea
C. gastric perforation
D. gastritis
E. gastroesophageal reflux

Suggested answer: E. Children with failure to thrive require supplemental feeds. When oral intake is not
sufficient, nasogastric tube feeding can be used. NG feeds have a number of advantages, including
controlling precisely how many calories are delivered and adjusting fluid volumes according to the
patient’s needs. Complications such as gastric perforation and allergic reaction to the tube are rare,
whereas complications such as gastritis may occur but are not prohibitive. Diarrhea can occur when high-
sugar or osmolarity formulas are delivered, but dehydration usually is prevented because total fluid intake
is controlled. Reflux is a common complication, because (a) the LES is stented open by the feeding tube;
and (b) infants are often fed larger volumes than they are accustomed to, in order to promote catch-up
growth.

Question 5:
A 2-week-old boy is admitted to the hospital with sepsis due to Escherichia coli. He is being breastfed
and has been vomiting frequently. Findings include failure to thrive, lethargy, hypotonia, jaundice,
hepatomegaly, and positive nonglucose reducing substances in the urine.

Of the following, the MOST likely explanation for these findings is:

A. galactosemia
B. glycogen storage disease
C. lactose intolerance
D. maple syrup urine disease
Dr-Wahid Helmi

E. urea cycle defect

Suggested answer: A. Galactosemia classically presents in the context of E. coli sepsis, and is associated
with hypotonia, jaundice, hepatomegaly, and failure to thrive. Reducing substances (galactose) are
present in the urine. Galactosemia results from misprocessing of galactose. Normally galactose and
glucose are generated from lactose by lactase. Galactose enters the cells and is phosphorylated to
galactose-1-phosphate by galactokinase (GALK), which is then converted to uridine diphosphate
galactose by glactose-1-phosphate uridyl transferase (GALT). GALK deficiency, a mild form of
galactosemia, only causes cataracts from galactose accumulation. GALT deficiency, the severe from of
galactosemia, causes cataracts as well as the more severe symptoms present in this vignette, presumably
from galactose-1-phosphate accumulation.

Glycogen storage disease usually causes hypoglycemia when feeding intervals are lengthened, and
hepatomegaly from metabolite accumulation. Lactose intolerance causes diarrhea from malabsorbed
sugars. Maple syrup urine disease is caused by a defect in branched-chain amino acid metabolism,
leading to poor motor and feeding problems and urine with a maple syrup odor. Urea cycle defects cause
defects in ammonia disposal, generating very high levels of ammonia in newborns.

Question 6:
A 3-month-old infant girl is admitted to the hospital for evaluation of recurrent episodes of hypoglycemia.
Physical examination reveals hepatomegaly. After 2 hours of fasting, she develops hypoglycemia.

Measurement of which of the following would be MOST helpful in determining the etiology of this
patient's hypoglycemia?

A. Ammonia in the serum and the arterial pH


B. Cortisol and growth hormone in the serum
C. Insulin and glucose in the serum
D. Ketones and reducing substances in the urine
E. Organic acids in the urine and lactate in the serum

Suggested answer: D. The case vignette is most characteristic of glycogen storage disease, with
symptoms caused by impaired hepatic glycogenolysis during fasting. Glycogen remains in the liver
(causing hepatomegaly), and glucose is not deposited in the serum (causing hypoglycemia). Typically
patients will have urine ketones from fat breakdown but no reducing substances in the urine.

A number of other diseases also lead to hypoglycemia. Defects in sugar metabolism such as galactosemia
and hereditary fructose intolerance impair gluconeogenesis from galactose and fructose, respectively, and
lead to hypoglycemia after many hours of fasting. They are not associated with urine ketones, but do
have positive urine reducing substances. Defects in amino acid metabolism may cause hypoglycemia
because of concomitant liver disease resulting in poor gluconeogenesis, while defects in fatty acid
metabolism can cause hypoglycemia because of liver disease resulting in poor gluconeogenesis, as well as
because of overused and depleted sugar stores in the absence of being able to utilize fats. Fatty acid
metabolism defects can be detected by examining urine for organic acids.

Hypoglycemia may also be caused by too much sugar utilization, as in the case of hyperinsulin states.
Finally, low cortisol (Addison’s disease, congenital adrenal hyperplasia) and low growth hormone cause
hypoglycemia, presumably from poor gluconeogensis. For these children, hormone replacement is the
treatment of choice.

Question 7:
Dr-Wahid Helmi

The mother of a 3-month-old infant reports that the boy is demanding frequent feedings and has a
noticeably protuberant abdomen. Physical examination reveals doll-like facies and marked hepatomegaly.
Laboratory findings include a serum glucose level of 20 mg/dL and an elevated venous lactate level of 44
mg/dL (normal, <18 mg/dL).

Of the following, the most appropriate INITIAL management of this infant is:

A. administration of a formula that has high concentrations of fructose and galactose


B. daily injections of glucagon
C. insertion of a portacaval shunt
D. nocturnal infusion of glucose via a nasogastric tube
E. referral for liver transplantation

Suggested answer: D. This patient has glycogen storage disease type 1, or Von Gierke disease. It is
caused by a deficiency is glucose-6-phosphatase, the last step in glycogenolysis (and the final step in
gluconeogenesis). Patients have doll-like facies and hepatomegaly, and laboratory results are significant
for marked hypoglycemia and lactic acidosis. Patients usually present when they begin to sleep through
the night and have longer periods without feeds. Treatment includes frequent glucose feeds during the
day and continuous feeds at night, usually with uncooked starch that releases glucose slowly. Efforts to
increase glycogenolysis (via glucagon) or gluconeogenesis (via fructose, galactose, or amino acid
formulas) do not help. There is no evidence of portal hypertension, so a shunt is not needed, and diet
modification – not liver transplant – is the standard treatment.

Question 8:
A 5-year-old girl was treated with amoxicillin for otitis media. One week after antibiotic therapy was
completed, she developed crampy abdominal pain and has been passing six stools daily that contain both
bright red blood and mucus. Physical examination reveals a temperature of 38.5°C (101°F), abdominal
distension, and diffuse abdominal tenderness.

Among the following, the most appropriate INITIAL diagnostic study to perform is a(n):

A. barium enema
B. colonoscopy
C. culture of stool for Clostridium difficile
D. evaluation of stool for ova and parasites
E. evaluation of stool for rotavirus

Suggested answer: C. C. difficile infection typically occurs during or shortly after antibiotic use, and
presents with watery stools, abdominal pain, and leukocytosis. In more severe cases, fever, bloody stools,
and (in the most severe forms) toxic megacolon is present. Diagnosis can be made by anaerobic cultures.
However, because cultures take time to grow, various toxin assays – including cytotoxin assays,
immunoassays, and PCR tests – have been developed. Colonoscopy can also be used to make the
diagnosis, which may show a spectrum of findings from erythema to ulcers to pseudomembranes (ulcers
filled with inflammatory material). First line treatment is flagyl, with oral vancomycin used for
reoccurring cases.

Question 9:
A previously healthy 2-year-old boy presents with irritability, drooling, and refusal to eat. Results of
physical examination include: drooling; symmetric aeration with normal breath sounds; and absence of
wheezes, retractions, or rhonchi. You suspect a foreign body was ingested.
Dr-Wahid Helmi

The MOST likely location of the foreign body is the:

A. esophagus
B. oral cavity
C. right main stem bronchus
D. stomach
E. trachea

Suggested answer: A. Foreign body ingestion is a common problem in the 6 month to 3 year old age
group, occurring approximately 80,000 times each year in the US. The most important initial step is to
rule out airway involvement. Drooling in this case suggests the object has lodged in the esophagus.
Imaging can confirm the diagnosis, with most esophageal objects trapped at physiological areas of
narrowing (UES, aortic arch, LES) or areas of stricture (usually mid-esophagus). Treatment depends on
the object and symptoms. If the object is sharp, a battery, causing complete obstruction, or causing other
systemic symptoms (fever, pain), the object should be removed immediately. If the object is blunt and
not causing any other symptoms, the object may be allowed to pass into the stomach on its own.
However, waiting beyond 24 hours for the object to pass is not advised, because the object may cause
local trauma after long periods of time.

Question 10:
A 6-week-old previously well girl is brought to the emergency department because she is irritable,
jaundiced, and feeds poorly.

Of the following, the most appropriate INITIAL laboratory investigation to determine the etiology of this
child's illness is:

A. an upper gastrointestinal series


B. cultures of the blood, cerebrospinal fluid, and urine
C. complete blood count and reticulocyte count
D. determination of serum electrolyte levels
E. measurement of serum acetaminophen level

Suggested answer: B. The differential diagnosis for this patient is sepsis vs. toxin ingestion vs. metabolic
disease. After taking a history negative for toxin ingestion, a sepsis work-up is an appropriate first step.
This patient likely has galactosemia, which presents with poor feeding, jaundice, and hepatomegaly.
Galactosemia also commonly occurs alongside E. coli sepsis, explaining the irritability and sepsis-like
presentation. In addition to blood work, a urinalysis showing reducing substances would also help
establish the diagnosis.

Question 11:
Vitamins and minerals are incorporated into infant formulas in the United States to provide an essentially
complete diet.

Which of the following minerals or trace minerals must be SUPPLEMENTED in ready-to-feed infant
formulas to meet the recommended daily allowances?

A. Calcium
B. Fluoride
C. Iron
D. Magnesium
E. Selenium
Dr-Wahid Helmi

Suggested answer: B. Ready-to-feed infant formulas are pre-mixed, compared to powdered formulas that
must be mixed by the parent before feeding. Ready-to-feed formulas have certain advantages, including
avoiding mixing mistakes and preventing infectious diseases from dirty water. However, ready-to-feed
formulas are made without fluorinated water. As a result, infants drinking ready-to-feed formulas (or
living in areas with fluorinated water) should receive fluoride supplementation after 6 months of age.

Formulas (whether pre-mixed or powdered) contain many supplements. Formulas are supplemented with
calcium (milk-based ~50-55 mg/100ml and soy-based ~70 mg/100ml vs. breast milk ~33 mg/100ml).
Formulas is also supplemented with Vitamin D, while breast milk contains very little Vitamin D. As a
result, all breast fed infants, and formula fed infants drinking less than 1 liter of formula, require
supplementation to achieve 400 IU.

Iron is also supplemented in formula, because iron deficiency is the most common nutrient deficiency in
children. Children require 0.5-0.8 mg/day of iron. Breast milk provides approximately 0.3-1 mg/L of
iron and is 50% bioavailable; unsupplemented formula, on the other hand, provides 1-2 mg/L but is only
4-6% bioavailable. Supplemented formula, in contrast, contains 12 mg of iron per liter of formula. With
a 4-6% bioavailability, this gives the infant 0.48-7.2 mg absorbed iron per liter of formula consumed.

Magnesium in breast milk and formula is abundant and readily bioavailable. Magnesium is present at 4-5
mg/100ml in routine formula, 5-8 mg/100ml in soy formula, and ~3.4 mg/100ml in breast milk. Finally,
selenium is present in higher quantities in breast milk, and selenium supplementation of formula has been
shown to improve infants’ selenium status. Formula selenium supplementation may be a change seen in
the future.

Question 12:
You suspect the cause of abdominal pain in a 17-year-old child is Helicobacter pylori-associated
gastroduodenitis.

Of the following, the most appropriate INITIAL step in management is to:

A. begin an empiric 2 week course of amoxicillin therapy


B. begin a trial of omeprazole therapy
C. obtain serologic testing for anti-Helicobacter antibodies
D. obtain upper gastrointestinal series to exclude gastric adenocarcinoma
E. refer for endoscopy and gastric biopsy

Suggested answer: E. Even though H. pylori is the most common cause of uncomplicated
gastric/duodenal ulcers in patients not taking NSAIDs, other causes do exist. As a result, establishing the
diagnosis is important before starting treatment. H. pylori can be diagnosed with endoscopy, urease
breath tests, and stool antigen tests. Serological testing for anti-pylori IgG was common (most infections
are chronic, so IgM serologies are less useful), but has since fallen out of favor because of its high false
positive rate and low positive predictive value. A negative serology test, however, does suggest no
infection.

Stool antigen tests are preferred over serology, and are especially useful for pediatric population in which
serological ranges are not available. Stool antigen tests may produce false-negative results in the setting
of PPI use.

Question 13:
An 8-year-old boy recently was diagnosed as having severe ulcerative colitis.
Dr-Wahid Helmi

Of the following, the best INITIAL management is:

A. bowel resection
B. continuous nasogastric feedings
C. intravenous corticosteroids
D. oral azathioprine
E. oral 5-aminosalicylates

Suggested answer: C. This patient is having a flare, and short-term IV steroids are the treatment of choice to
reduce inflammation acutely. Bowel resection is too aggressive as a first step, though may be needed if the
disease is steroid-resistant. NG feeds are used to treat mild-to-moderate IBD in some places such as Canada,
but more severe disease requires extra therapy. Similarly, oral 5-ASAs may be effective for mild-to-moderate
disease but alone would be inadequate for a severe flare. Finally, oral azathioprine would be an appropriate
maintenance medication. However, it takes weeks to have full effect, and would be inadequate in the short-
run to improve symptoms.

Question 14:
In a child who has chronic diarrhea and weight loss, the PREFERRED method for providing nutritional
support is:

A. continuous nasogastric feedings


B. intermittent (bolus) nasogastric feedings
C. nasoduodenal feedings
D. oral feedings
E. parenteral nutrition

Suggested answer: D. For ill patients needing supplemental nutrition, the first decision is whether to feed
enterally or parenterally. Those that can absorb nutrients (i.e. no small bowel disease) should be tried on
enteral feeds first. Enteral feeds provide trophic support to the gut, and can be given by mouth (as in this
case), by NG tube (if the oropharynx or esophagus is diseased), or by ND tube (if the stomach empties
poorly). On the other hand, those with small bowel disease (Crohn disease, celiac disease, tufting
enteropathy) have poor absorptive function and may require parenteral nutrition. Parenteral nutrition has
many disadvantages including a high rate of complications such as line infections.

Question 15:
A 16-month-old boy who has leukemia is receiving prednisone, vincristine, L-asparaginase, and intrathecal
methotrexate. On the seventh day of chemotherapy, broad-spectrum antibiotics were administered for fever.
All cultures were negative, and antibiotics were stopped after 14 days. Over the past 2 weeks, abdominal
distension and pain have developed. Hematologic findings are normal.

The MOST likely etiology of the abdominal findings is:

A. candidal mucositis
B. chemotherapy-induced mucositis
C. chemotherapy-induced neurotoxicity
D. Clostridium difficile infection
E. leukemic infiltration of the bowel wall

Suggested answer: D. The most common possibilities for this case are C. difficile versus teflitis. Teflitis is a
necrotizing enterocolitis that develops after induction chemotherapy, thought to be due to a combination of
damaged mucosa from chemotherapeutics and microorganism invasion from neutropenia. It typically
Dr-Wahid Helmi

presents in neutropenic patients with RLQ abdominal pain and fever. Given the normal neutrophil count and
recent antibiotic use, C. difficile infection is the more likely. Chemotherapy-induced neurotoxicity usually
affects peripheral nerves, and damage of the enteral nervous system causing ileus would take longer to
develop. Leukemic infiltration of the bowel wall – especially after starting chemotherapy – would be rare.
Pancreatitis with ileus is another possibility, given that L-asparaginase often induces pancreatitis.

Question 16:
Among the following, the MOST specific clinical manifestation of an antral web in an infant is:

A. diarrhea
B. distension of the lower abdomen
C. extreme irritability
D. hematemesis
E. nonbilious vomiting

Suggested answer: E. Antral webs are rare, as are all non-pyloric stenosis causes of gastric outlet obstruction
in the neonate. The incidence is thought to be 1 in 100,000, and they are commonly associated with other GI
or cardiac abnormalities. Antral webs usually are located 1-2 cm proximal to the pylorus and are 1-4 mm
thick. Webs with apertures greater than 1 cm are rarely asymptomatic, whereas those with near complete
closure cause nonbilious vomiting in the first feeds after birth. The diagnosis can be made by UGI showing
two compartments to the stomach, and the definitive treatment is surgical removal. Lower GI symptoms
(diarrhea, distension of the lower quadrants) would not occur. Hematemesis could occur but nonbloody
vomiting would be more common. Finally, although patients with antral webs may be hungry and irritable,
irritability is a general sign and not specific to any particular disease.

Question 17:
An 18-month-old infant who has intestinal atresia has been maintained on total parenteral nutrition via a
central venous catheter since birth. He is brought to your office with a 1-day history of lethargy, fever, and
jaundice.

Of the following, the MOST likely cause of this infant's findings is:

A. cholelithiasis
B. fulminant hepatic failure
C. infection of the central venous catheter
D. intrahepatic cholestasis
E. viral hepatitis

Suggested answer: C. Parenteral nutrition, though life-saving, has a number of acute and chronic
disadvantages. Acutely, patients commonly develop central line infections. Infected children may have mild
symptoms initially, that rapidly progress to shock requiring resuscitation. Chronically, patients develop TPN-
associated cholestasis, likely from compounds present in the intralipid formulation. The cholestasis leads to
bile salt back-up into the liver and liver damage, which ultimately could lead to liver failure. Cholelithiasis,
secondary to poor bile flow, is another chronic complication of TPN. It could lead to cholecystitis and
produce similar symptoms, though abdominal pain would be a more prominent finding. Finally, fulminant
hepatic failure and viral hepatitis are both acute and would not be more likely to occur in a patient on
parenteral nutrition.

Question 18:
A 1-week-old boy has a seizure. Significant findings on physical examination include a "cherubic" face,
protuberant abdomen, and profound hepatomegaly. Glycogen storage disease is suspected.
Dr-Wahid Helmi

The MOST important issue in the initial management of this patient is to:

A. decrease serum lactate


B. prevent hypoglycemia
C. reduce hypercholesterolemia
D. reduce serum free fatty acids
E. treat hyperuricemia

Suggested answer: B. This patient is having hypoglycemic seizures, which can be managed by
administering sugars to return the infant to a euglycemic state. Patients with glycogen storage have a
number of other serum findings (particularly those with GSD 1/von Gierke disease, or glucose-1-
phosphatase deficiency, as in this case). They may have elevated lactate levels from massive amounts of
glucose-1-phosphate, created from the breakdown of glycogen during fasting but trapped in the liver,
undergoes glycolysis. Furthermore, in the absence of usable glucose, de novo triglyceride production
increases dramatically and leads to elevated cholesterol concentrations. This hyperlipidemia is associated
with xanthomas but not premature atherosclerosis, and, as a result, cholesterol-lowering drugs are not
recommended. Finally, the excessive glycolysis creates large amounts of phosphorylated intermediate
compounds, which inhibits re-phosphorylation of adenine nucleotides. This causes nucleic acid
degradation and produces excessive uric acid, which may lead to nephrolithiasis.

Question 19:
A 5-month-old boy with frequent vomiting is switched from human milk to formula. His symptoms
immediately worsen, and he becomes highly irritable. Shortly afterward, he becomes comatose. Physical
examination reveals a small, hypotonic child responsive only to pain. Laboratory studies reveal: increased
anion gap; metabolic acidosis; serum ammonia concentration, 150 mg/dL; and serum glucose level, 85
mg/dL.

Which of the following classes of inborn errors of metabolism is MOST likely in this patient?

A. Disorder of fatty acid oxidation


B. Glycogen storage disease
C. Lysosomal disease
D. Organic acidemia
E. Urea cycle defect

Suggested answer: Urea cycle defects should be considered in patients with an anion gap, high ammonia,
and normal glucose. Urea cycle defects lead to high ammonia, because there is a defect in converting
ammonia to the secreted product urea. High ammonia causes neurological deficits through mechanisms
incompletely understood. High ammonia also causes an anion gap, possibly by impairing brain
mitochondria, forcing anaerobic metabolism of sugar, and producing lactic acid as a by-product. This
patient showed mild symptoms with human milk (i.e. frequent vomiting), because human milk has low
protein levels (2.3 g/dL protein at birth and decreasing to 1.5-1.8 g/dL after 2-4 weeks). The symptoms
increased with formula, because formula has more protein (approximately 2.1-2.2 g/dL) which led to an
increased accumulation of ammonia.

Question 20:
Of the following, the inborn error of metabolism that exists MOST frequently in a vitamin-dependent
form is:

A. argininosuccinase deficiency
Dr-Wahid Helmi

B. branched-chain ketoaciduria
C. cytochrome C oxidase deficiency
D. homocystinuria
E. phenylketonuria

Suggested answer: D. Homocystinuria occurs with a defect in the conversion of methionine to cysteine,
leading to accumulation of the intermediate product homocysteine. Normally cystathionine beta synthase
(CBS) uses B6 to transsulfurate homocysteine to cysteine. However, in CBS deficiency, homocysteine
accumulates and causes developmental delay, osteoporosis, ocular abnormalities, thromboembolic
disease, homocystinuria, and premature atherosclerosis.. Treating with excessive B6 can drive the
reaction forward and reduce symptoms in some cases. Homocysteine can also be remethylated to
methionine, using methyl donors such as folate and B12.

Argininosuccinate synthetase/lyase deficiencies are responsible for urea cycle defects. Branched-chain
alpha-ketoacid dehydrogenase complex (BCKD) deficiency causes branched chain ketoaciduria (maple
syrup urine disease), a disease characterized by improper metabolism of branched chain amino acids.
This results in a shortage of substrates for gluconeogenesis, energy production, and fatty acid/cholesterol
synthesis. Cytochrome C oxidase deficiency is the most common respiratory chain defect causing
myopathy in newborns, resulting in poor energy production. Finally, phenylalanine hydroxylase causes
phenylketonuria because of an inability to convert phenylalanine to tyrosine. Excessive phenylalanine
accumulates and causes mental retardation.

Question 21:
A previously healthy 20-month-old boy develops gastroenteritis with anorexia, vomiting, and diarrhea; 24
hours later he develops seizures and has a decreased level of consciousness. The serum glucose level is
25 mg/dL, electrolyte concentrations are normal, and urinalysis reveals no ketones.

Among the following, the MOST likely diagnosis for this patient is

A. alcohol ingestion
B. glycogen storage disease type I
C. hereditary fructose intolerance
D. insulinoma
E. medium-chain acyl-CoA dehydrogenase deficiency

Suggested answer: E. For patients with hypoglycemia and no ketones, a disorder in fatty acid metabolism
should be suspected. Normally, with fasting or illness, the liver creates energy for the brain by beta-
oxidizing fatty acids from adipose tissue into ketones. However, in fatty acid oxidation defects such as
medium-chain acyl-CoA dehydrogenase deficiency (MCAD), the liver cannot produce ketones and the
brain is left without an energy source. Without ketones, sugars are rapidly used as the sole energy source
by all tissues, eventually creating hypoglycemia and further depriving the brain of energy.
Alcohol ingestion can create hypoglycemia, but is inconsistent with the clinical history. Glycogen storage
disease produces ketones, as fatty acids must be mobilized because glucose is limiting. Hereditary
fructose intolerance results in hypoglycemia and low ketones, but presents at the onset of fructose
ingestion with vomiting and failure to thrive. Finally, insulinomas also have hypoglycemia and low
ketones (insulin prevents fatty acid mobilization), but insulinomas are very rare in infants.

Question 22:
Dr-Wahid Helmi

The mother of a 3-month-old boy reports that he has a poor appetite and constipation. Findings on
physical examination, when compared to those 2 months ago, include poor interim growth, increased
lethargy, hoarse cry, decreased tone, large fontanelles, and a more pronounced umbilical hernia.

Of the following, the MOST likely cause of this infant's problem is

A. agenesis of the thyroid gland


B. endemic goitrous hypothyroidism
C. end-organ unresponsiveness to thyroid hormone
D. inborn error of thyroxine synthesis
E. thyroid gland unresponsiveness to thyrotropin

Suggested answer: A. Congenital hypothyroidism occurs in 1:2000-4:000 births and is the leading reason
for preventable mental retardation. Congenital hypothyroidism is most commonly caused by thyroid
gland dysgenesis (agenesis, hypoplasia, ectopy). Other causes include defects in thyroid hormone
synthesis, secretion, transport, metabolism, and/or responsiveness. Most infants appear normal at birth
because of maternal contributions of T4; hence, screening tests to measure free T4 and TSH are
administered to identify hypothyroid infants before symptoms described in the vignette develop.

Question 23:
A 3-month-old infant who has a history of gastroesophageal reflux has had increasing vomiting for 2
days. This morning she developed rapid, deep, labored breathing; lethargy; and shock. Findings include:
serum sodium, 144 mEq/L; potassium, 4.5 mEq/L; chloride, 89 mEq/L; bicarbonate, 5 mEq/L; pH, 7.16;
glucose, 48 mg/dL; ammonia, 128 mcmol/L; and ketonuria.

The MOST likely explanation for these findings is:

A. aminoacidopathy
B. ethylene glycol poisoning
C. metoclopramide toxicity
D. organic acidemia
E. urea cycle defect

Suggested answer: D. This patient has organic academia. These diseases are usually caused by defects in
enzymes that metabolize amino acids, and can also be caused by defects in fat and carbohydrate
metabolism. Infants present with poor feeing, vomiting, and lethargy in the first weeks of life, similar to
infants with sepsis. Laboratory results show a profoundly high anion gap metabolic acidosis (in this case,
the gap is 50), with increased ammonia, decreased glucose, and electrolytes consistent with dehydration.
Organic acid analysis in urine is used to confirm the diagnosis. Acute management involves rehydration
and glucose infusion; after stabilization, protein feeds can be reintroduced as long as the offending amino
acid is omitted.

Aminoacidopathies refer only to defects in amino acid metabolism; however, the patient in this vignette
could have problems with fat or sugar metabolism leading to organic acidemia. Ethylene glycol
poisoning also causes a high anion gap metabolic acidosis, but gaps at 50 would only occur very acutely
after ingestion. Metoclopramide toxicity usually causes tardive dyskinesia, which may or may not be
reversible. Finally, urea cycle defects usually have higher ammonia levels and a less severe, lactic acid
metabolic acidosis.

Question 24:
Which of the following is the most common cause of pancreatitis in childhood?
Dr-Wahid Helmi

A. viral
B. drug induced
C. idiopathic
D. familial
E. abdominal trauma

Suggested answer: A. Pancreatitis in childhood has a number of causes. Viral infections are the most
common cause (up to 13-33%), including infections by mumps, rubella, coxsackie B virus, CMV, and HIV.
Other causes include trauma, drug-induced, anatomical (pancreatic divisum, choledochal cyst), toxin-induced,
gall-stone induced, and hereditary (defects in cationic trypsinogen, CF gene).

Question 25:
Which of the following is not part of the Currarino triad characterizing caudal regression syndrome which can
present as infantile constipation?

a. dysplasic sacrum
b. anal abnormalities
c. tethered cord
d. pre-sacral mass

Suggested answer: C. Caudal regression syndrome refers to a constellation of diseases with poorly
formed caudal (sacral and lower lumbar) vertebrate. They are more common in infants of diabetic
mothers, and often is accompanied with a tethered cord. Three types of caudal regression syndromes are:
VACTERL (Vertebral, Anorectal, Cardiac, Tracheal-Esophageal fistula, Renal, and Limb abnormalities);
OEIS (Omphalocele, cloacal Exstrophy, Imperforate anus, Spinal malformation); and Currarino syndrome
(partial sacral agenesis, a pre-sacral mass, and recto-anal abnormalities).
Dr-Wahid Helmi

Question 1:
A TRUE statement regarding gastrointestinal functioning in the premature infant is:
A. Bile acid pools are elevated
B. Intestinal motility is normal
C. Lactose is digested effectively
D. Vegetable oils are digested poorly
E. Vitamins are absorbed adequately

Suggested answer: D. Premature infants have a decreased bile acid pool, for a combination of reasons:
a) bile acid synthesis proteins are not fully expressed, b) the gall bladder does not concentrate well,
thereby diluting bile acids in bile, c) the terminal ileum machinery to reabsorb bile acids is immature,
and d) the liver machinery to re-uptake bile acids from the portal circulation is also immature (explaining
why infants have high circulating bile acids for up to 6 months). All of these reasons, in combination with
low pancreatic lipase and colipase secretion in infants, lead to poor absorption of vegetable oils and fat
soluble vitamins.

Premature infants have reduced lactase expression, which may be induced with early feeding. Premature
infants also have altered motility, with poor antral contractions before 32 weeks in some studies. Other
studies report altered migrating motor complex (MMC) patterns lacking the strong contractions of phase
3 in infants less than 32 weeks old.

Question 2:
Which of the following BEST explains why solutions containing 1.2 to 2.5% glucose, rather than 5%
glucose, are used for oral rehydration?
A. Absorption of sodium and water in the gut is maximized
B. Glomerular filtration rate is enhanced
C. Hyperglycemia and glycosuria are less likely to occur
D. Potassium absorption is decreased
E. Stomach distention with vomiting is less likely to occur

Suggested answer: A. Enterocytes on villi express Na-coupled glucose transporter-1 (SGLT-1). The
transporter is located on the apical side of the cell, and uses a sodium concentration gradient to bring 2
molecules of sodium and 1 molecule of glucose into the cell. (The gradient is created by the basal Na/K
pump, which pumps sodium out of the cell). Water follows the sodium, promoting rehydration. The
glucose is either used by the enterocyte, or enters circulation via facilitated diffusion through the basal
transporter Glut2.

Excessive glucose can be harmful, because it may be unabsorbed in the small intestine. When it reaches
the large intestine, it may be metabolized by gut bacteria to create osmotically active substances.
Diarrhea, and further dehydration, could result.

Question 3:
A TRUE statement concerning a low protein diet in patients with renal insufficiency is:
A. Appetite will increase because the diet is more palatable
B. Cognitive capacity will decrease
C. Nutrient absorption will increase
D. Renal osteodystrophy will be easier to control
E. The progression of renal insufficiency will be slowed

Suggested answer: E. In renal failure, the kidneys have trouble clearing toxins related to urea. As a
result, a constellation of symptoms may develop, including decreased cognitive capacity, decreased taste/
Dr-Wahid Helmi

appetite, decreased growth, and further renal damage by increasing GFR. A low protein does lead to an
improvement of symptoms, presumably from less uremic toxins. A low protein diet is associated not only
with lower creatinine measurements, but also slower progression to renal failure. Hence, children with
renal insufficiency should be given limited amounts of high-quality protein.

Neural function such as taste/appetite and cognitive capacity may increase with less uremic toxins (but
not because the low-protein diet tastes better). Absorption will not change with a low-protein diet. Renal
osteodystrophy is related more to renal 1-hydroxylation of 25-OH Vitamin D and less to uremic toxins.

Question 4:
A 6-week-old boy with known congestive cardiomyopathy weighs 4 kg. He takes only 12 oz of formula
daily. Of the following, the MOST likely consequence of decreased intake in this infant is
A. hypocalcemia
B. hypoglycemia
C. hypokalemia
D. poor growth in length
E. poor weight gain

Suggested answer: E. Children with congestive cardiomyopathy have increased metabolic needs.
Nutritional support is complicated because these children also have strict fluid requirements. The patient
in this question is only taking 72 kcal/kg/day (assuming a 24 kcal/oz formula), which is insufficient even
for an infant without heart disease. As a result, he will have poor weight gain, and if persistent, poor
growth in length later.

Question 5:
The weight of an adolescent boy is twice ideal body weight for height. If he remains obese as an adult, he
is likely to develop each of the following EXCEPT
A. infertility
B. learning disorder
C. pulmonary insufficiency
D. sleep disorder
E. thromboembolism

Suggested answer: B. Obesity has not been shown to cause an adolescent to develop learning disorders
as he transitions to adulthood. However, some cases of obesity are related to neurological disorders (i.e.
Prader Willi Syndrome, Bardet-Biedl Syndrome), and obesity in an otherwise normal child may cause
low self-esteem and poor school performance. Obesity is associated with a number of other co-
morbidities, including infertility (possibly related to leptin abnormalities), pulmonary insufficiency,
obstruction sleep apnea, and hypercoagulation.

Question 6:
A 6-month-old infant has been fed only goat milk. Laboratory studies reveal: hemoglobin concentration,
9.5 gm/dL; mean corpuscular volume, 100; white blood cell count, 4,200/mm^3; and reticulocyte count,
0.4%. In addition to dietary counseling, you would MOST likely recommend initial supplementation
with:
A. ascorbic acid
B. folic acid
C. iron
D. pyridoxine
E. vitamin B12
Dr-Wahid Helmi

Suggested answer: B. Goat milk is an alternative for infants with cow’s milk protein allergy. However,
goat milk lacks folate, which can lead to a megoblastic anemia as seen in this patient. Folate is absorbed
in the small intestine, when the brush border enzyme folate conjugase hydrolyzes dietary polyglutamates
into folic acid. Hence, severe mucosal disease can also lead to folate deficiency. Finally, sulfasalazine
use can lead to folate deficiency, because sulfasalazine competes with enzymes of the folate absorption
pathway.

Question 7:
A 4-year-old girl who attends preschool has had diarrhea for 10 days. Several other children have had
similar symptoms, including fever and vomiting that persisted for 3 to 5 days. This child, however, has
continued to have abdominal discomfort, excessive gas with abdominal distention, and watery stools. Her
appetite is good, and she is not acting ill. Which of the following laboratory tests is MOST likely to
confirm the diagnosis?
A. Breath nitrogen test
B. Clostridium difficile toxin and antigen
C. Stool culture
D. Stool for ova and parasites
E. Stool for pH and reducing substance

Suggested answer: D. This patient has several risk factors for giardia (in preschool, watery diarrhea,
excessive gas, appears well) and sending stool for ova and parasites is appropriate. If available, sending
stool for giardia antigen may have a higher yield. Giardia is commonly treated with Flagyl
(metronidazole) or Alinia (nitozoxanide).

This patient’s symptoms are less concerning for acute gastritis, so a breath nitrogen test to detect H.
pylori would not be warranted (and if H. pylori was a concern, a stool antigen test would be better).
Clostridium difficile is less likely because there is no prior antibiotic use, stools are non-bloody,
symptoms are both upper and lower, and the symptoms seem to be rapidly contagious. Bacterial stool
cultures would be more useful to work-up acute bloody diarrhea. Finally, the patient has no evidence of
sugar malabsorption, so stool for pH and reducing substances would not be helpful.

Question 8:
Of the following, the BEST reason for administering parenteral alimentation through a peripheral vein
rather than through a central vein is:
A. inability to insert and maintain a catheter in a central vein
B. lower incidence of hyperglycemia
C. lower incidence of infection
D. need for parenteral alimentation after discharge from the hospital
E. need to deliver solutions of high osmotic load

Suggested answer: C. Lines in peripheral veins (including PICCs) have a lower incidence of
infection. Peripheral IVs, however, last only a few days and are can only deliver a limited
osmotic load. PICC and central lines last for long periods and can deliver a high osmotic load.
Central veins are placed surgically, and may be used when peripheral access is limited.
Hyperglycemia is a risk with all forms of parenteral nutrition, and involves many factors
including glucose infusion rate, the patient’s insulin production, and the patient’s insulin
sensitivity.

Question 9:
Dr-Wahid Helmi

During a routine health supervision visit, a 16-year-old girl is noted to have a hemoglobin concentration
of 10 gm/dL and mean corpuscular volume of 72. The MOST common cause for anemia in such a patient
is:
A. chronic hemolysis
B. Crohn disease
C. folic acid deficiency
D. iron deficiency
E. parasitic infestation

Suggested answer: D. For a menstruating female, the most common reason for microcytic
anemia is low iron from blood loss. Chronic hemolysis can lead to anemia but is rare. Crohn
disease can also cause microcytic anemia from poor duodenal iron absorption; however, in this
patient without Crohn disease symptoms, iron loss is much more likely. Folic acid deficiency
causes a megoblastic anemia. Finally, parasites can also cause Fe-deficiency anemia, and is
more common in the developing world.

Question 10:
The mother of a 4-year-old boy reports that he has developed hives within 1 hour after ingesting
acetaminophen syrup, brompheniramine elixir, and hard candies. He also has developed hives and
wheezing following injection of penicillin. He inadvertently was given an acetaminophen tablet but had
no reaction. Of the following, the MOST likely cause of this boy's symptoms is
A. allergy to sucrose
B. idiopathic urticaria
C. idiosyncratic reaction to artificial coloring or preservatives
D. multiple drug allergies
E. penicillin contamination of the acetaminophen syrup and brompheniramine elixir

Suggested answer: D. This patient has an allergy to penicillin. He also has an allergy to
brompheniramine elixir, because he tolerates Tylenol alone and hard candies are generally
hypoallergenic. Interestingly, brompheniramine is an anti-histamine, which in most children would be
expected to improve allergic symptoms. However, in this child with multiple drug allergies, the
brompheniramine actually induces more symptoms. The pathophysiology of multiple drug allergy
syndrome remains unknown.

Question 11:
A 33-year-old pregnant woman wishes to breastfeed her baby. She has had systemic lupus erythematosus
for 2 years and takes prednisone and hydralazine. Among the following, your BEST advice to her
regarding her current medication regimen and breastfeeding is:
A. Both prednisone and hydralazine usually are compatible with breastfeeding
B. Captopril should be substituted for hydralazine
C. Cyclophosphamide should be substituted for prednisone
D. Cyclosporine should be substituted for prednisone
E. High dose aspirin should be substituted for prednisone

Suggested answer: A. Although prednisone and hydralazine can be passed in breast milk, the absolute
amount in negligible and poses no risk to the infant. Captopril is not contraindicated during
breastfeeding, though has been associated with birth defects if taken during pregnancy.
Cyclophosphamide and cyclosporine are contraindicated in breastfeeding mothers, as they may cause
pancytopenia and other problems in the child. High dose aspirin must be used with caution in
Dr-Wahid Helmi

breastfeeding mothers, because aspirin is passed in breast milk and may cause bleeding and platelet
abnormalities in infants.

Question 12:
The daily caloric requirement in kcal/kg for a healthy full-term infant is approximately:
A. 40
B. 60
C. 80
D. 100

Suggested answer: D. The dietary recommended intake for a newborn is 102 kcal/kg/day, or
approximately 150 cc/kg/day of human milk (approximately 16.5 ounces/day for a 3.5 kg newborn).
From 4 months to approximately 3 years, the DRI is in the 80 kcal/kg/day range. From age 5-8 years old
the DRI is the 60 kcal/kg/day range, and post-pubertal adolescents generally have a DRI below 40
kcal/kg/day.

Question 13:
The MAIN advantage of enteral nutrition over parenteral nutrition is that enteral nutrition:
A. facilitates monitoring caloric intake
B. has fewer complications
C. permits maintenance of longer periods of positive nitrogen balance
D. prevents toxins from accumulating in the colon
E. provides improved caloric intake in the outpatient setting

Suggested answer: B. Enteral nutrition is favored over parenteral nutrition, because parenteral
nutrition has major complications. These include line infections, cost, TPN associated cholestasis, and
calculation errors on nutrients delivered. With enteral nutrition, it may be harder to calculate caloric
intake because everything that is given may not be absorbed. Both enteral and parenteral nutrition can
permit positive nitrogen balance, depending on how much protein is delivered. Enteral nutrition may be
associated with more toxin accumulation in the colon, as bacteria metabolize unabsorbed foods and
create short-chain fatty acids and other compounds. Finally, both enteral and parenteral nutrition can be
used in the outpatient setting.

Question 14:
A TRUE statement about the sugar content of infant formulas is:
A. All lactose-containing formulas are cow milk-based
B. All cow milk-based formulas contain only simple sugars
C. All soy-based formulas are corn syrup-free
D. All soy-based formulas contain lactose
E. Proprietary formulas do not contain sucrose

Suggested answer: A. Most cow’s milk-based infant formulas contain the disaccharide lactose and not
the disaccharide sucrose (there are exceptions). Soy-based formulas contains a combination of corn
syrup solids, sucrose, and malto-dextrin. Proprietary formulas (i.e. Alimentum, Isomil, and Alsoy) also
contain sucrose. The sucrose content in formula becomes important, especially in the work-up of
hereditary fructose intolerance. Infants consuming cow milk-based formulas generally will not be
exposed to sucrose, which is made of fructose and glucose. However, infants on soy or specialty formulas
will be exposed to sucrose and hence fructose.

Question 15:
Dr-Wahid Helmi

Continuous, rather than intermittent (bolus), tube feeding is the PREFERRED choice for providing
nutrition to the patient with:
A. a transpyloric feeding tube
B. anorexia nervosa
C. coma following head injury
D. esophageal obstruction
E. multiple facial fractures

Suggested answer: A. Bolus feedings are preferred to continuous feedings, because they a) more closely
replicate normal eating, and b) allow the patient to be disconnected from the pump for many hours
during the day. However, if feeds are administered directly to the intestine, a continuous rate is needed.
The intestine does not have the storage capacity of the stomach, and large bolus feeds would lead to
dumping syndrome.

Question 16:
A full-term infant, who weighed 2,200 gm at birth, has been breastfed for 4 months. General growth and
development appear normal. Physical examination reveals enlargement of the costochondral junctions
and subtle thickening of the wrists and ankles. The disorder that is MOST consistent with these findings
is:
A. cleidocranial dysostosis
B. rickets
C. Rubenstein-Taybi syndrome
D. Russell-Silver syndrome
E. Scurvy

Suggested answer: B. This patient’s clinical symptoms are characteristic of rickets. Vitamin-D
deficiency rickets is common between 3 months and three years, because calcium needs are high for
growth and sunlight exposure is limited. Furthermore, vitamin D in breast milk is scant. Infants may be
protected for the first 3 months, because 25-OH Vitamin D (clacidiol) is transferred via placenta from
mother to fetus. Calcidiol has a half life of 3 to 4 weeks; afterwards, serum 25-OH Vitamin D
concentration will fall unless supplemented through multivitamins, formula, or sunlight exposure.

Question 17:
A TRUE statement regarding introduction of solid foods before the age of 4 to 6 months is:
A. Delaying introduction of solid foods beyond this age increases the likelihood of food allergy later
B. Feeding of solid foods helps the infant sleep through the night
C. Feeding of solid foods improves maternal-infant bonding
D. Starches are poorly digested because of low levels of amylase in the gut
E. Supplementation with solid foods is necessary for adequate nutrition

Suggested answer: D. Solid foods should be introduced between 4 to 6 months of age. This is the ideal
time, as introducing foods may provide extra calories to support extra growth, satisfy hunger, and reduce
the occurrence of allergies. Introducing foods before this time may interfere with adequate energy intake,
overload the kidneys, promote food allergies, and place the child at risk for aspiration. In addition,
pancreatic enzyme secretion (including amylase)is immature for the first months of life, making starches
difficult to digest. When foods are introduced after 6 months, infants may suffer from decreased growth
(from decreased calorie intake), iron-deficiency anemia in breastfed infants, delayed oral motor function,
and solid food aversion.

Question 18:
Dr-Wahid Helmi

A 6-week-old infant born at term has a hemoglobin level of 11 gm/dL and is diagnosed with physiologic
anemia of the newborn. The MOST likely cause is:
A. inadequate iron stores in the bone marrow
B. inadequate serum levels of vitamin E
C. increased excretion of iron in the stool
D. low levels of serum erythropoietin
E. persistent fetal hemoglobin

Suggested answer: D. Physiologic anemia of the newborn occurs when fetal hemoglobin normally
declines at 6 to 8 weeks. Because fetal hemoglobin has a higher affinity for oxygen, serum erythropoietin
levels are initially low. Erythropoietin levels eventually rise again with time, correcting the anemia.
Some have investigated whether Vitamin E mitigate the normal hemoglobin nadir, with equivocal results.

Question 19:
Most authorities encourage the early introduction of human milk in the very-low-birthweight (VLBW)
infant. However, mothers must be informed early in the feeding process that supplementation of their
milk with protein and other nutrients may be necessary. The MOST likely explanation for why protein
supplementation of human milk often is required in the VLBW infant is that

A. human milk contains less than half the protein of cow milk formula
B. the hepatic metabolism of protein is ineffective in most preterm infants
C. the protein in preterm human milk is of poor nutritional quality
D. VLBW infants have excessive gastrointestinal losses of ingested protein
E. VLBW infants require an increased protein intake because of their high catabolic rate

Suggested answer: E. Though VLBW benefit most from human milk, VLBW infants require higher
protein intake to sustain adequate growth. They also have higher protein turnover rates. As a result, for
VLBW infants, human milk should be fortified with extra protein to meet these increased needs. VLBW
infants fed human milk without protein fortification have slower growth rates, lower BUN, and lower
serum albumin, demonstrating the importance of protein supplementation.

Human milk and formula contain almost equivalent amounts of protein; however, human milk contains
more whey protein (quickly digested) whereas many formulas contain more casein protein (more slowly
digested). There is no evidence that VLBW have increased protein losses from their gut.

Question 20:
A 3-year-old boy underwent a hepatoportoenterostomy (Kasai procedure) for extrahepatic biliary atresia
at 6 weeks of age. He has been receiving cholestyramine to treat severe pruritus for the past 2 months. Of
the following, the nutrient MOST likely to be malabsorbed because of this patients underlying liver
disease and its treatment is:
A. carbohydrate
B. fat
C. protein
D. trace elements
E. water-soluble vitamins

Suggested answer: B. This patient will have low luminal bile acids for two reasons: 1) poor bile acid
secretion into the gut, secondary to biliary atresia; and 2) poor bile acid activity in the gut, secondary to
binding with cholestyramine. Bile acids, pancreatic lipase, and pancreatic colipase work together to
digest fats efficiently. Without bile acids, fat malabsorption will occur. Neither BA nor cholestyramine
impairs duodenal or pancreatic function, so protein and carbohydrate absorption should be intact.
Dr-Wahid Helmi

Question 21:
A 5-year-old girl who has severe developmental delay is fed exclusively through a gastrostomy tube.
Within 1 day of a change in her tube feeding formula, she develops diarrhea, abdominal distress, and
flatulence. The stool pH is less than 5.0. These findings are MOST consistent with malabsorption of:
A. carbohydrate
B. fiber
C. lipid
D. protein
E. vitamins

Suggested answer: A. This child has signs of symptoms of excessive carbohydrates in her feeds. Sugars
that are not absorbed in the small intestine are delivered to gut bacteria in the colon. Here bacteria
metabolize sugars into short chain fatty acids. The byproduct of this reaction is gas (causing bloating)
and an increased colonic osmotic load (causing diarrhea). Furthermore, the short-chain fatty acids
cause the stool to be acidic.

Question 22:
A 10-month-old African-American infant who was born at 30 weeks gestation weighed 1,400 g at birth.
She has been breastfed exclusively and has received no supplemental vitamins. General growth velocity
has been slow, but development has been normal. Physical examination reveals enlargement of the
costochondral junctions and slight thickening of the wrists and ankles. This infant MOST likely has a
deficiency of:
A. folic acid
B. vitamin A
C. vitamin C
D. vitamin D
E. vitamin E

Suggested answer: D. This child has Vitamin D-deficiency rickets. Premature babies are at higher risk
for developing rickets, possibly secondary to poor maternal transfer of calcidiol through the placenta,
poor bile acid production to absorb fat-soluble vitamins, and lack of sunlight in the NICU. In addition,
breastfed infants receive inadequate Vitamin D in milk, as breast milk only contains 12-60 IU/L
(recommended dose for infants controversial, and spans 200-400 IU/day). Hence, breastfed infants must
receive supplements. Formula fed infants do not need supplements, because formula contains adequate
amounts of added Vitamin D.

Question 23:
You have volunteered to work in a refugee camp in Rwanda. You are examining a 15-month-old infant
whose diet has been a dilute carbohydrate gruel since he was weaned from the breast at 9 months of age.
Physical examination reveals pallor; apathy; thin, pale hair; a desquamating skin rash; pitting edema of
the lower extremities; and weight at the 80th percentile for age. Among the following, the MOST likely
diagnosis is:
A. kwashiorkor
B. marasmus-kwashiorkor
C. marasmus
D. vitamin A deficiency
E. vitamin C deficiency

Suggested answer: A. This patient has kwashiorkor. Marasmus occurs secondary to inadequate total
calories, and is characterized by low muscle and fat mass. Children look wasted. Kwashiorkor occurs
Dr-Wahid Helmi

secondary to adequate total calories but inadequate protein, and is characterized by low muscle mass
and normal fat mass. Children look edematous and may have a scaly dermatitis resembling flaky paint.
In the mixed picture, patients start with marasmus but then develop a pro-inflammatory state (i.e. from
infection), leading to an edematous appearance. Vitamin A deficiency causes corneal dryness
(xerophthalmia) leading to scarring and night blindness, whereas Vitamin C deficiency causes impaired
collagen synthesis and scurvy.

Question 24:
A 3-week-old boy who was born at 28 weeks gestation is being fed 100 kcal/kg per day of fortified (24
kcal/oz) breast milk by gavage. He also is receiving aminophylline for apnea and low-flow oxygen and
diuretic therapy for chronic lung disease. He is gaining weight at a rate of 5 g/day. Of the following, the
MOST likely explanation for this child’s slow weight gain is that:
A. administration of a loop diuretic causes excessive loss of glucose and protein
B. aminophylline therapy impairs fat absorption in the upper small intestine
C. multiple episodes of apnea/bradycardia increase the basal metabolic rate
D. the daily caloric intake is less than the recommended range for preterm infants
E. the metabolic rate of a growing preterm infant is twice that of a term infant

Suggested answer: D. Preterm infants have increased energy needs, in part because they have lower
body stores of fat and glycogen, expend more energy controlling their temperature, and have to support
high rates of growth relative to their body size. Their metabolic rate is more than that of a term infant
(but not twice as much), and hence they need more calories that the standard 100 kcal/kg/day needed by a
term infant.

Infants are commonly given theophylline/aminophylline (caffeine) for apnea and bradycardia, which also
increases energy expenditure and may impair growth but does not impair fat absorption. Loop diuretics
should not cause protein or glucose loss. Finally, apnea/bradycardia episodes may increase energy
expenditure but would not affect the basal metabolic rate.

Question 25:
A 6-week-old boy who was born at home and who has been exclusively breastfed has had diarrhea for 5
days. His parents bring him to the emergency department because he has multiple deep ecchymoses and
bloody stools. Until the results of laboratory studies become available, the best INITIAL management of
this patient is to administer intravenous?
A. cryoprecipitate
B. factor VIII concentrate
C. fresh frozen plasma
D. platelets
E. vitamin K

Suggested answer: E. Newborns receive Vitamin K supplementation at birth because many are vitamin
K deficient. There are many reasons for this deficiency: 1) low vitamin K stores at birth, 2) poor
placental transfer of vitamin K, 3) low levels of vitamin K in breast milk (but adequate amounts in
formula), and 4) low colonic bacteria that normally generate vitamin K. Without adequate vitamin K,
infants can develop bleeding at the umbilicus, in the mucous membranes, in the GI tract, at circumcision
sites, and at IV sites. They can also have hematomas at sites of trauma, as well as life-threatening
intracranial bleeding.

Vitamin K administration is the first treatment of choice. For active bleeding, fresh frozen plasma should
be administered. Prothrombin complex concentrates (PCC) can be given in life-threatening situations.
Dr-Wahid Helmi

Question 1:
Shortly after birth, a 3,500 g term newborn is found to be jittery and to have a high-pitched cry. Physical
examination reveals tachypnea and a liver edge that is palpable several centimeters below the umbilicus.
Blood glucose concentration is 14 mg/dL.

Among the following, the MOST likely cause of the hypoglycemia in this newborn is:

A. galactokinase deficiency
B. glycogen storage disease
C. insulinoma
D. maternal diabetes mellitus
E. prolonged maternal labor

Suggested answer: B. Hypoglycemia in an infant suggests either too much insulin (insulinoma, maternal
DM) or too little glucose release into the bloodstream in non-feeding states (GSD). MDM is much more
common than insulinomas, and neither are associated with hepatomegaly,. On the other hand, GSD is
associated with a large liver secondary to glycogen entrapment. Other signs include lactic acidosis,
slight ketosis, and hyperuricemia.

Galatosemia is caused by defects in one of two enzymes. Classic galactosemia occurs from defects in
galactose-1-phosphate uridyltransferase (GALT), which results in accumulation of galactose-1-
phosphate, galactose, and oxidative and reductive products galactitol and galactonate. Patients develop
hepatosplenomegaly and mental retardation, in addition to cataracts and galactosuria. The other
mutation is in galactokinase, which works one step ahead of GALT and is used to create galactose-1-
phosphate. Infants with mutations in this gene have cataracts and galactosuria, but do not have
hepatosplenomegaly or mental retardation (presumably because galactose-1-phosphate is not produced
so does not accumulate). Cataracts cccur in both types of galactosemia, when excessive galactose is
converted to the osmotically active galactitol in the lens/eye.

Question 2:
A breastfed infant who appeared healthy at birth develops chronic diarrhea, failure to thrive, and
hepatomegaly during the first few weeks of life. Ultrasonography reveals adrenal enlargement and
calcification.

Of the following, the MOST likely explanation for these findings is:

A. cystic fibrosis
B. glucose-galactose malabsorption
C. glycogen storage disease
D. Niemann-Pick disease
E. Wolman disease

Suggested answer: E. Wolman disease results from recessive mutations in lysosomal acid lipase. This
enzyme acts on endocytosed lipoproteins, hydrolyzing cholesteryl esters and triacylglycerols. Without the
enzyme, patients accumulate cholesteryl esters and triacylglycerols in various tissues. Patients have
severe diarrhea (presumably from intestinal involvement), malnutrition, abdominal distension,
hepatosplenomegaly, and calcification of the adrenal glands. Patients do not survive past infancy.
Niemann-Pick Types A, B, and C disease are associated with neurological changes rather than diarrhea.
Types A and B are caused by defects in acid sphingomyelinase activity. Sphingomyelin accumulates
intracellularly, producing hepatosplenomegaly, “foam cells,” and, in Type A, severe neurodegenerative
Dr-Wahid Helmi

disease and death by 3 years. Type C disease, caused by mutations in NPC1 or NPC2, involves defects in
cholesterol trafficking from lysosomes to other compartments. As a result, excessive lipids are found in
lysosomes.

Glucose-galactose malabsorption causes life-threatening diarrhea and dehydration in the first few weeks,
associated with renal (not adrenal) calcium deposits. The disease is caused by recessive mutations in the
sodium/glucose cotransporter SGLT1, which is the main transporter involved in transporting glucose and
galactose into enterocytes. As a result, glucose and galactose remain in the gut lumen and act as an
osmotic drive causing diarrhea. SGLT1 is also found in the kidney, and mutations can also lead to
glucosuria. Treatment involves fructose-based formulas that do not contain glucose or galactose.

Question 3:
A 4-week-old infant is jaundiced. Findings include weight and length at the 75th percentile for age;
icterus; hepatosplenomegaly; total bilirubin, 6.3 mg/dL; direct bilirubin, 5.5 mg/dL; alanine
aminotransferase activity, 130 U/L; aspartate aminotransferase activity, 143 U/L; and gamma-glutamyl
transpeptidase activity, 950 U/L.

Of the following, the BEST study to evaluate the excretion of bile from the liver is:

A. computed tomography of the liver


B. hepatic ultrasonography
C. hepatobiliary scintigraphy
D. measurement of galactose-1-phosphate uridyltransferase activity
E. measurement of the serum alpha1-antitrypsin level

Suggested answer: C. Bile excretion can be documented with the HIDA (hepatobiliary iminodiacetic
acid) scan. The procedure is based on monitoring a radiolabeled compound as it travels from the
hepatocyte, into the cannalicular space, through the biliary tree, and into the intestine. HIDA scans have
been shown to be more sensitive than specific for biliary atresia; another better, yet more invasive test, to
demonstrate patent bile ducts is the intraoperative cholangiogram.

The other choices do not measure excretion. CT is not needed in this case, whereas US is useful to
demonstrate presence of a choledochal cyst, polysplenia (found in approximately 10% of BA cases), the
triangular cord sign (thickening of left branch of portal vein in BA), and absence of gall bladder. GALT
activity is useful if classic galactosemia is suspected. Finally, low A1AT levels may hint at A1AT
deficiency, though PI (protease inhibitor) typing is a better test.

Question 4:
A 14-year-old boy has a 36-hour history of severe, continuous midepigastric pain radiating to the back,
persistent vomiting, and fever. Physical examination reveals: blood pressure, 70/40 mm Hg; temperature,
39.5ºC (103.1ºF); marked midepigastric tenderness, guarding, and rebound; absent bowel sounds; and
abdominal distention. You suspect acute pancreatitis.

The test or procedure that will be MOST specific in confirming the diagnosis is a(n):

A. abdominal ultrasonogram
B. endoscopic retrograde cholangiopancreatogram
C. serum alanine aminotransferase activity
D. serum amylase activity
E. white blood cell count
Dr-Wahid Helmi

Suggested answer: D. Amylase can be used to make the diagnosis of acute pancreatitis; however, lipase
has been shown to be sensitive and more specific. Amylase elevations can also occur in salivary disease,
intestinal disease, gynecological disease, and neoplasms. Abdominal ultrasound may or may not show
changes, and may be hampered in this case by air from abdominal distension. ERCP outlines the
pancreatic duct to identify stones or strictures, but does not outline the pancreatic parenchyma. ALT/AST
would be useful to identify gall-stone related common bile duct obstruction, leading to pancreatitis and
hepatitis. Finally, WBC counts could be high in a number of infectious or inflammatory diseases.

Question 5:
Enteric diseases commonly occur in young children who attend out-of-home child care facilities.

Of the following, the enteropathogen most likely to cause CHRONIC diarrhea in an immunocompetent
child is:

A. adenovirus
B. Cryptosporidium
C. Escherichia coli 0157:H7
D. rotavirus
E. Shigella

Suggested answer: D. Chronic diarrhea is diarrhea that occurs for more than 4 weeks. Viruses such as
rotavirus and adenovirus can cause “post-enteritis syndrome,” a form of chronic diarrhea thought to be
secondary to the acute infection. Post-enteritis syndrome may be caused by a number of factors: a)
disaccharide deficiency because of blunted villi, b) leaky mucosa leading to protein translocation and
sensitivity/allergy, and c) repeated enteric infections following the initial insult. Cryptosporidium causes
chronic diarrhea in immunocompromised hosts. E. coli and Shigella usually cause acute symptoms.

Question 6:
Among the following, the gastrointestinal disease MOST likely to respond to treatment with anti-
cholinergic medications is:

A. constipation
B. dysentery
C. gastroesophageal reflux
D. irritable bowel syndrome
E. peptic ulcer disease

Suggested answer: D. Cholinergic agonists are used to stimulate the parasympathetic nervous system,
thereby activating motility. Anti-cholinergic agents, then, would be expected to make dysmotility
problems such as constipation and reflux worse. Anti-cholinergics can be used to treat IBS patients
suffering from too much motility. Theoretically, they can also be used in peptic ulcer disease
(acetylcholine is one input to parietal cells), but more often proton-pump inhibitors or histamine receptor
agonists are chosen. Finally, dysentery should be treated by targeting the offending agent rather than by
altering motility though anti-cholinergic medications.

Question 7:
For the past 6 weeks, a 4-year-old boy has had painless, bright red rectal bleeding associated with bowel
movements. Examination of the abdomen and anus reveals normal findings. The rectal vault is empty, and
no blood is noted on gross inspection.
Dr-Wahid Helmi

Of the following, the MOST likely cause for the hematochezia is

A. hemolytic-uremic syndrome
B. Henoch-Schönlein purpura
C. intussusception
D. juvenile polyposis
E. Meckel diverticulum

Suggested answer: D. Juvenile polyposis is characterized by painless bright red blood associated with
bowel movements. Histology usually shows hamartomatous changes. Meckel’s diverticulum presents
with melena (if slow bleeding) or bright red rectal bleeding not associated with bowel movements (if fast
bleeding). HUS (a coagulopathy), HSP (a vasculitis), and intussusception (an anatomical defect) all
present with abdominal pain.

Question 8:
A 2-year-old boy has had bilious vomiting and bloody stools since last night. Physical examination
reveals a moderately ill, dehydrated child who has a scaphoid abdomen and absent bowel sounds. Stools
are maroon-colored and strongly positive for blood.

After stabilizing the patient, the INITIAL diagnostic study that should be performed is a(n)

A. acute abdominal radiographic series


B. barium swallow
C. computed tomogram of the abdomen
D. Meckel scan
E. upper endoscopic examination

Suggested answer: A. This patient has signs of symptoms of an acute abdomen, likely secondary to
volvulus causing bilious vomiting and ischemic gut. To confirm the diagnosis, radiographic films can be
used to document ileus and presence of free air. A barium swallow would be poorly tolerated because of
the intestinal obstruction, and may cause harm if perforation is present. CT of the abdomen would better
outline the patient’s anatomy, though may be more than what is needed before surgery. A Meckel’s
diverticulum would not cause upper GI symptoms, and an EGD would provide no diagnostic or
therapeutic benefit.

Question 9:
A 13-year-old girl is being evaluated for diarrhea, abdominal pain, and weight loss.

Of the following, the feature that BEST distinguishes Crohn disease from ulcerative colitis is:

A. development of crypt abscesses


B. hepatic involvement
C. mucosal ulcerations on endoscopy
D. noncaseating granulomas on mucosal biopsy
E. poor growth

Suggested answer: D. Although the treatments overlap, the type of IBD does play a role in diagnostic
tests and prognosis. Generally, UC is a mucosal disease which starts in the rectum. As the disease
progresses, it involves continuous parts of the colon but does not extend into the ileum. Crypt abscesses
are common, and hepatic involvement can be seen in the form of primary sclerosing cholangitis or
autoimmune hepatitis. Crohn disease, on the other hand, extends throughout the thickness of the bowel
Dr-Wahid Helmi

and can be present in skip lesions throughout the GI tract. Liver involvement is less common, though can
be present. The most characteristic pathological finding is the noncaseating granuloma, comprised of
macrophages coordinating an autoimmune reaction in the mucosa. Both UC and Crohn disease can be
associated with ulcers on endoscopy and poor growth.

Question 10:
A 6-month-old boy has chronic diarrhea. Findings include: weight less than the 5th percentile and length
at the 25th percentile for age; marked cachexia with protuberant abdomen; sodium, 125 mEq/L; chloride,
90 mEq/L; albumin, 2.5 g/dL; and total protein, 4.3 g/dL. Stool is negative for reducing sugars but
positive for neutral fats; a 72-hour fecal fat collection shows a coefficient of absorption of 45% (normal,
>93%).

These findings are MOST consistent with:

A. celiac sprue
B. cow milk-soy protein allergy
C. Crohn disease
D. cystic fibrosis
E. giardiasis

Suggested answer: C. Cystic fibrosis is associated with pancreatic insufficiency, secondary to


inspissation of pancreatic secretions and poor secretion. As a result, patients develop fat malabsorption,
protein malabsorption, chronic diarrhea, and malnutrition. Celiac disease starts when children consume
gluten-containing foods, so would not be present as a chronic disease in a 6 month old. Cow milk-soy
protein allergy presents as bloody stools, rectal eosinophilia, and possibly constipation, but does not lead
to failure to thrive. Crohn disease is less common in a 6 month old, and does not necessarily lead to fat
malabsorption. Finally, giardiasis can cause chronic diarrhea but would not be expected to cause
steatosis.

Question 11:
The clinical manifestations of cholecystitis differ depending on the age of the patient.

Which of the following findings is MORE likely to occur in an affected child than in an affected adult?

A. Fat intolerance
B. Fever
C. Jaundice
D. Pain radiating to the right scapula
E. Palpable mass in the right upper quadrant

Suggested answer: C. Cholecystitis is defined as inflammation of the gall bladder, usually secondary to
bile stasis and bacterial growth. It can be acute (from stone obstruction) or chronic (from stone
obstruction or poor gall bladder motility). Symptoms include fever and pain in the RUQ and radiating to
the right scapula. Pain intensifies with fatty meals that stimulate gall bladder contraction. Children
more commonly present with jaundice, even if a stone is not identified. This jaundice is presumably from
edema of the bile duct walls, preventing bile outflow and bilirubin back-up into the circulation.

Question 12:
A febrile 1-month-old infant has a generalized seizure. Findings include healthy appearance; weight and
length, 90th percentile; liver span, 11 cm; serum glucose, 20 mg/dL; alanine aminotransferase activity,
Dr-Wahid Helmi

123 U/L; aspartate aminotransferase activity, 153 U/L; total bilirubin, 2.0 mg/dL; and lactic acid, 4.7
mmol/L.

These findings are MOST suggestive of:

A. alpha1-antitrypsin deficiency
B. congenital hepatic fibrosis
C. galactosemia
D. glycogen storage disease
E. perinatal cytomegalovirus infection

Suggested answer: D. This patient is having a hypoglycemic seizure, likely from a glycogen storage
disease. In type Ia GSD, there is a deficiency of glucose-6-phosphatase. In the liver, glucose-6-
phosphatase converts glucose-1-phosphate (generated from stored glycogen) into glucose to be released
in the circulation during times of fasting. In addition to hypoglycemia and related seizures, patients with
GSD type I have hepatomegaly and hepatocyte damage from excessive stored glycogen.

A1AT could cause jaundice, elevated AST/ALT, and hepatomegaly; however, it is not associated with
hypoglycemia at diagnosis. Similarly, congenital hepatic fibrosis and CMV infection involve the liver but
would not lead to hypoglycemia so early. Galactosemia causes hepatosplenomegaly from galactose-1-
phosphate accumulation; however, it is not characteristically associated with hypoglycemia.

Question 13:
A 6-year-old boy who has chronic constipation has been treated with cathartic medications for 1 week.
You decide to place him on maintenance therapy with a lubricating laxative.

Among the following, the agent you are MOST likely to recommend is:

A. bisacodyl
B. docusate sodium
C. magnesium hydroxide
D. malt soup extract
E. mineral oil

Suggested answer: E. Mineral oil is a lubricating laxative, which coats the mucosa allowing stools to
slide easily and preventing colonocytes from reabsorbing water. Docusate sodium acts like a soap,
reducing the surface tension of stool and allowing more fat and water to enter. Magnesium hydroxide
(along with polyethylene glycol and lactulose) are common osmotic laxatives, whereas bisacodyl is a
stimulant laxative that increases persistalsis by irritating mucosal smooth muscle. Malt soup extract is a
bulk forming fiber laxative. Its cellulose content absorbs water from the intestine and causes stool to
become bulky and soft.

Question 14:
A 4-month-old boy regurgitates after all feedings. His weight has remained at the 10th percentile for age.
Normal findings on an upper gastrointestinal barium study have excluded anatomic abnormalities. You
suspect gastroesophageal reflux.

The best INITIAL management of this child would be:

A. administration of cisapride
B. administration of ranitidine
Dr-Wahid Helmi

C. administration of small, thickened oral feedings


D. change to an elemental formula
E. referral for fundoplication

Suggested answer: C. This patient has uncomplicated GER with no pain, arching, or failure to thrive.
The first intervention is to thicken oral feeds. The next intervention would be to try a hypoallergenic (not
elemental) formula, because cow’s milk allergy presents with symptoms similar to GER. Cisapride, a
pro-motility agent, is not available due to associations with heart arrhythmias. Ranitidine or a proton-
pump inhibitor may be good choice if the patient showed symptoms of pain, such as back-arching;
however, without these symptoms, acid suppression is not warranted. Finally, a fundoplication is
excessive for uncomplicated GER.

Question 15:
Examination of a developmentally normal 7-month-old boy reveals moderately enlarged cervical lymph
nodes; a hemorrhagic seborrhea-like rash on the forehead, scalp, and trunk; and hepatosplenomegaly.
Laboratory findings include: hemoglobin, 12.0 g/dL; mean corpuscular volume, 82 fL; white blood cell
count 10,700/mm³, with 40% neutrophils and 60% lymphocytes; and platelet count 260,000/mm³.

These findings are MOST consistent with:

A. acute lymphoblastic leukemia


B. aplastic anemia
C. Langerhans cell histiocytosis
D. neuroblastoma
E. Niemann-Pick disease

Suggested answer: C. Langerhans cell histocytosis results from abnormal proliferation of the Langerhan
cell, a dendritic cell subtype. Multisystem LCH involves the skin (rash), lymph nodes, liver (liver
dysfunction, primary sclerosing cholangitis), spleen, and various other organs. LCH is treated with
chemotherapy. In this scenario, ALL and aplastic anemia are unlikely because there is no anemia.
Neuroblastoma can present with an abdominal mass. Niemann-Pick disease involves cholesterol
trafficking, and leads to neurodegenerative changes.

Question 16:
A previously healthy 12-year-old boy presents with an upper gastrointestinal tract hemorrhage. Findings
include: hepatosplenomegaly, ascites, a prominent vascular pattern on the abdomen, thrombocytopenia,
mildly elevated aminotransferase activity, markedly elevated gammaglutamyl transpeptidase activity, and
a prolonged partial thromboplastin time.

Which of the following laboratory studies is MOST likely to provide a diagnosis?

A. Alpha1-antitrypsin level
B. Fasting blood glucose level
C. Galactose-1-phosphate uridyltransferase activity
D. Hepatobiliary scintigraphy
E. Sweat test

Suggested answer: E. This patient has chronic liver disease (as demonstrated by the many signs of portal
hypertension) caused by a biliary etiology (as suggested by high GGT levels). The most likely diagnosis
is cystic fibrosis-related liver disease. CFTR is present in biliary epithelium and promotes proper bile
flow. Without it, bile backs up and leads to liver cirrhosis. In addition, as in this case, children with CF-
Dr-Wahid Helmi

related liver disease may have minimal lung disease and no growth problems. Biliary atesia causes
similar symptoms and uses hepatobiliary scans in its work-up. BA, however, presents in infancy.

Alpha-1-antitrypsin deficiency can present with cirrhosis in adolescents, but usually AST/ALT levels are
higher secondary to misfolded A1AT protein accumulating in hepatocytes. Also, A1AT is best diagnosed
with PI typing, because alpha-1-antitrypsin levels may be falsely elevated in times of inflammation.
Fasting blood glucose may be low secondary to liver dysfunction, but would not identify the primary liver
disease. GALT deficiency leads to classic galactosemia, which presents in infancy.

Question 17:
A 14-year-old boy is being evaluated for jaundice that was first noted 1 week ago following an upper
respiratory tract infection. He reports not feeling very hungry for the past month. Physical examination
reveals a firm liver, an enlarged spleen, and an intention tremor.

Among the following, the test that would be MOST helpful for making a definitive diagnosis in this
patient is a:

A. liver biopsy for copper content


B. serum bilirubin concentration
C. serum ceruloplasmin level
D. serum transaminase activity
E. slit lamp examination of the cornea

Suggested answer: A. In an adolescent with liver disease and neurological findings, Wilson disease
should be suspected. Wilson disease is caused by a mutation in the ATP7B copper transporter.
Mutations result in abnormal copper trafficking at 2 levels: 1) impaired copper binding to
apoceruloplasmin inside the hepatocyte, and 2) decreased secretion of copper into the biliary system.
Copper accumulates in hepatocytes and other tissues (brain, kidneys, heart), leading to clinical
symptoms.

Only 50% of patients have the classical findings of Kayser-Fleischer rings (detected by slit lamp
examination) and low serum ceruloplasmin. Serum ceruloplasmin is especially tricky, because although
it should be low secondary to low serum copper levels, it may be elevated as an acute phase reactant in
the setting of chronic liver inflammation. Elevated serum transaminase activity or bilirubin
concentration may initiate the work-up but are not specific to Wilson disease. The gold standard for
diagnosing Wilson disease is hepatic copper content. 85% of patients have a hepatic copper
concentration greater than 250mcg/g of liver tissue.

Question 18:
A 4-month-old infant has had diarrhea for 6 weeks. Findings include: weight less than the 5th percentile
and length at the 10th percentile for age; cachexia; a protuberant abdomen; total protein, 4.3 g/dL;
albumin, 2.8 g/dL; stool pH, 4.5; stool for reducing substances, 2+; stool culture, negative for enteric
pathogens; and sweat chloride, 10 mEq/L.

The study that would be MOST helpful in determining the cause of this child’s diarrhea is

A. D-xylose test
B. serum trypsinogen level
C. small bowel biopsy
D. stool alpha1-antitrypsin level
E. upper gastrointestinal barium study
Dr-Wahid Helmi

Suggested answer: C. This patient has severe malnutrition, with poor weight compared to height, low
serum proteins, and sugar malabsorption. The differential diagnosis for this child’s condition is broad,
and includes infection, pancreatic insufficiency, and villi disease (i.e. tufting enteropathy, microvillus
inclusion disease). EGD to visualize the mucosa and collect samples for routine histology as well as
electron microscopy will help distinguish between some of these possibilities.

It is already clear that the child has malabsorption, so a D-xylose test is unnecessary. (In the test, D-
xylose, a substance absorbed in the SI, is given and then measured in blood and urine. Low values
suggest malabsorption). Serum trypsinogen is used to detect pancreatitis, which is unlikely in this child.
Elevated stool alpha-1-antitrypsin identifies protein loss in the gut, which in an infant raises the
possibility of congenital intestinal lymphangiectasia. However, in congenital intestinal lymphangiectasia,
lymphatics are blocked and drain backwards into the bowels, leading to fat – rather than sugar –
malabsorption. Finally, an UGI barium swallow is used to detect malrotation when the primary symptom
is vomiting.

Question 19:
Physical examination of a 13-year-old boy who is being evaluated for short stature reveals aphthoid
lesions in the mouth and skin tags and fissures around the anus.

Of the following, these findings are MOST consistent with

A. Crohn disease
B. herpes simplex virus infection
C. immunoglobulin A deficiency
D. pseudomembranous colitis
E. ulcerative colitis

Suggested answer: A. Crohn disease is characterized by involvement of all parts of the GI tract,
including mouth (aphthous ulcers), small intestine (poor growth), and perianal areas (skin tags and
fissures, present in 1/3 of cases). Ulcerative colitis, on the other hand, usually presents sub-acutely with
bloody diarrhea and symptoms restricted to the colon. HSV infection could cause mouth ulcers but would
not cause chronic growth problems and anal findings. Those with IgA deficiency are usually
asymptomatic. The 10% with symptoms may have diarrhea secondary to persistent Giardia infection.
Pseudomembranous colitis occurs acutely in response to C. difficile infection. It can be seen
endoscopically as membranes of host protein, mucus, and inflammatory cells over C. difficile-induced
mucosal ulcers.

Question 20:
A 4-week-old boy who was born at 28 weeks' gestation and weighed 800 g at birth is gaining weight at a
rate of only 5 g/day even though his caloric intake is 125 kcal/kg. The stool is poorly formed and bulky.
The infant is receiving a lactose-free formula containing 24 kcal/oz.

Which of the following interventions is MOST likely to result in a decrease in steatorrhea and improved
weight gain?

A. Increase the daily caloric intake to 150 kcal/kg


B. Increase the glucose polymers in the formula
C. Increase the protein intake
D. Increase vitamin A and vitamin E supplements
E. Substitute medium-chain triglycerides for long-chain triglycerides
Dr-Wahid Helmi

Suggested answer: E. Premature infants absorb long-chain triglycerides poorly, because they have not
yet developed the machinery to digest and absorb fats. They have poor pancreatic and bile excretion,
leading to low amounts of lipase, colipase, and bile acids in the intestinal lumen. Without these enzymes,
micelles do not form, lipids are not absorbed into the intestinal lymphatics, and, as a result, fats are
passed into the stool. Medium chain triglycerides can circumvent this problem because they are
absorbed directly into enterocytes without digestion by bile acids. Hence, changing the fat content of the
formula should lead to better absorption and better weight gain.

Question 21:
A 14-year-old female adolescent who has severe juvenile rheumatoid arthritis presents to your office with
epigastric abdominal pain. Six weeks earlier she began taking a nonsteroidal anti-inflammatory drug
(NSAID) because of worsening joint complaints.

Of the following, the most appropriate INITIAL management of her symptoms would be

A. administration of an antibiotic effective against Helicobacter pylori


B. administration of an H2-receptor antagonist
C. dietary modification
D. substitution of salicylate for NSAID
E. upper endoscopy and gastric biopsy for gastric adenocarcinoma

Suggested answer: B. NSAIDs (and aspirin) cause gastric/duodenal damage by inhibiting COX enzymes
and prostoglandin production. Prostoglandins have a number of protective roles in the upper GI tract,
including decreasing acid secretion, stimulating mucus production, stimulating bicarbonate production,
and promoting vasodilation of gastric vessels to increase oxygen delivery. To counter these effects, acid
suppression therapy with H2-receptor antagonists and proton pump inhibitors can be used. Misoprostol,
a prostaglandin E analog, seems to have even more gastroprotective effects, but also causes diarrhea and
abdominal discomfort.

H. pylori may predispose patients taking NSAIDs to develop peptic ulcer disease, so testing and treating
positive cases is appropriate. There is no evidence that this patient consumes foods likely to increase
acid secretion, so dietary modification would not be the primary intervention. Upper endoscopy may be
appropriate to confirm ulcers (not adenocarncinoma) caused by NSAID/aspirin use.

Question 22:
The essential difference between elemental formulas and casein hydrolysate formulas is that elemental
formulas are MOST likely to include:

A. a hyperosmolar composition
B. an increased concentration of vitamins and minerals
C. lactose as the predominant carbohydrate source
D. medium-chain triglycerides
E. protein in the form of amino acids rather than oligopeptides

Suggested answer: E. Cow’s milk and soy milk based formulas contain large protein epitopes which can
trigger an allergic reaction. To address this, casein hydrolysate formulas are used, consisting of smaller
peptides rather than full proteins. If the peptides still induce allergy, then elemental formulas consisting
of individual amino acids are used. Individual amino acids do not produce allergic responses.
Dr-Wahid Helmi

Elemental formulas have a slightly higher osmolarity than hydrolysate formulas, but this difference is not
as important as their differing protein contents. The two formulas have equivalent vitamins and minerals,
and use corn syrup solids, cornstarch, and in some cases sucrose (Similac Alimentum) as carbohydrate
sources. The amount of MCT oils added is brand-dependent.

Question 23:
Among the following disorders, rickets is MOST likely to develop in patients who have:

A. acute pancreatitis
B. adrenal insufficiency
C. cirrhosis
D. congenital heart disease
E. lactose intolerance

Suggested answer: C. Patients with liver disease have two reasons to be Vitamin D deficient. First, if
cholestatic, they have impaired absorption of fat soluble vitamins including Vitamin D2. (The skin can
compensate to some extent, creating Vitamin D3). Second, the Vitamin D2/D3 is converted in the liver to
25-hydroxy-Vitamin D, which then passes to the kidneys and is converted to the active 1,25-dihydroxy-
Vitamin D. Patient with cirrhosis have impaired 25-hydroxylation of Vitamin D2 and D3. Acute
pancreatitis would not cause Vitamin D deficiency; however, chronic pancreatitis and pancreatic
insufficiency would result in impaired lipase/colipase secretion, and would impact absorption of fat-
soluble vitamins.

Question 24:
A 6-month-old boy developed a weepy, crusted dermatitis around the eyes, nose, mouth, diaper area,
hands, and feet about 4 weeks after being weaned from human milk to formula. He is listless, recently
developed diarrhea, and has stopped gaining weight. In addition to the dermatitis, he has sparse hair that
is fine and lightly pigmented.

Of the following, this constellation of findings is MOST consistent with a deficiency in

A. copper
B. thiamine
C. vitamin A
D. vitamin C
E. zinc

Suggested answer: E. Acrodermatitis enteropathica, a recessive mutation in the zinc transporter


SLC39A4, is responsible for this child’s symptoms. Affected infants absorb zinc poorly, and have
dermatitis (around orifices and the limbs), alopecia, diarrhea, growth retardation, frequent infections,
neuropsychiatric problems, and delayed sexual maturation. Symptoms usually start when the infant is
weaned from breast milk to formula, suggesting that breast milk has some compound that aids in zinc
absorption. Treatment is with daily zinc supplementation.

Also on the differential in this case is Menkes syndrome. Menkes syndrome is caused by a mutation in the
X-linked gene ATP7A, which is important in copper trafficking. In the small intestine, it is responsible for
copper absorption. Symptoms arise when copper-dependent mitochondrial enzymes are impaired, and
when copper abnormally accumulates in certain tissues. Infants present with “kinky” hair, growth
failure, and neurological deterioration.
Dr-Wahid Helmi

Poor nutrition can lead to deficiencies in many vitamins and minerals. Copper deficiency is associated
with sideroblastic anemia, neutropenia, failure to thrive, and skeletal abnormalities. Thiamine deficiency
(Vitamin B1) is associated with beriberi, consisting of cardiomyopathy and polyneuritis. Vitamin A
deficiency causes night blindness, and Vitamin C deficiency causes scurvy. Zinc deficiency leads to
dermatitis around perioral and perianal areas, poor appetite, diminished taste acuity, hypogonadism, and
short stature.

Question 25:
Following an acute febrile illness, a 6-month-old infant has had diarrhea for 3 weeks. When the diarrhea
began, the mother said she fed the infant an oral electrolyte solution exclusively. As the diarrhea slowed,
the mother reintroduced cow milk infant formula, and the diarrhea recurred. The infant continues to
receive oral electrolyte solution exclusively. She appears thin but is otherwise healthy.

The BEST management at this time is to administer:

A. a gluten-free diet
B. a hypoallergenic diet
C. a lactose-free diet
D. clindamycin
E. metronidazole

Suggested answer: C. Post-enteritis syndrome refers to chronic diarrhea persisting after an initial
enteric infection. It was thought that such patients have lactose malabsorption secondary to villi damage
and poor lactase production, which then leads to an osmotic diarrhea. Patients were instructed to avoid
lactose and consume formulas with glucose polymers. If intolerance persisted, then protein
hypersensitivity was thought to be the cause, presumably from damaged mucosa allowing translocation of
large peptides. To treat this scenario, patients were put on hypo-allergenic formulas.

Recent work has shown that disaccharidase deficiency and protein sensitization are actually not that
common. Rather, the mechanism of disease appears to be from repeated enteral infections following the
initial infection. Some recommend treating with probiotics to hasten recovery times.
Dr-Wahid Helmi
Question 1:
A 12 year old girl with cirrhosis due to alpha-1-antitrypsin deficiency presents for evaluation of a 6-
month history of slowly progressive exertional dyspnea. Physical examination reveals multiple spider
angiomas, clubbing, and acrocyanosis. Chest and cardiac examination are unremarkable and chest
radiography and pulmonary function tests are normal. Arterial blood gas reveals a respiratory
alkalosis with hypoxemia. Which one of the following is the best screening test to define the
diagnosis?

A. Pulmonary angiography
B. Contrast echocardiography
C. Abdominal ultrasonography with Doppler examination of the hepatic vasculature
D. Bronchoscopy
E. Right heart catheterization to assess pulmonary arterial pressure

Suggested correct Answer: B. Alpha-1-antitrypsin deficiency is caused by mutations in the A1AT


gene. The best diagnostic test is PI (protease inhibitor) typing, which is commonly mistaken as a
genotyping test. PI typing is actually a phenotype test, in which different protein forms are identified
by Western blot. There are 3 common protein variants: M (normal), S (somewhat abnormal), Z
(abnormal). There is also a null variant, caused by a mutation that leads to no protein production.

A1AT deficiency leads to liver and lung disease. Lung disease happens in adults, from progressive
damage of lung tissue from neutrophil serine proteases (there is no A1AT to counter these enzymes).
The liver injury occurs earlier, because misfolded A1AT cannot leave the hepatocyte. It accumulates
and causes damage. ZZ and SZ phenotypes are most likely to have accumulation and hepatocyte
damage in children, whereas MZ may lead to liver issues later in life. Interestingly, the null phenotype
will have no accumulation or liver damage but will have significant lung injury.

In this question, the child likely has a ZZ or SZ phenotype causing her liver symptoms. Her
respiratory symptoms could be due to concomitant A1AT lung disease. However, lung disease in
children is very rare, and would present as emphysema on exam and pulmonary function tests. In
one study, adolescents with ZZ phenotype showed no significant abnormality in pulmonary function
tests compared to age-matched controls. Another study showed a slight degree of hyperinflation in
affected children, but no studies have conclusively shown major lung disease in the pediatric
population.

The pulmonary findings of hypoxemia and alkalosis (presumably secondary to hyperventilation) are
more likely from one of two processes related to any liver disease: hepatopulmonary syndrome vs.
portopulmonary hypertension. Hepatopulmonary syndrome consists of hypoxemia, intrapulmonary
vascular dilations, and liver disease. The vascular dilations lead to pulmonary R->L shunts detected
on contrast (bubble) echocardiography or technetium 99m-labeled microalbumin study. Classic
physical findings are clubbing, cyanosis, and spider nevi. Furthermore, because the shunts are more
often found at the lung bases, symptoms usually worsen when standing.

Portopulmonary hypertension is the opposite of hepatopulmonary syndrome, because it involves


pulmonary vasoconstriction (as opposed to dilation) and increased pulmonary pressures. It is not as
well described in children. An echocardiogram showing increased tricuspid regurgitation would
suggest this diagnosis, and right heart catherization would confirm it.

Question 2:
Hepatitis A occurs in cyclical outbreaks in the United States. These outbreaks spread largely
because of:
A. HAV infection among injection drug users
Dr-Wahid Helmi
B. Promiscuous sexual behavior
C. Infected food handlers
D. Widespread vaccination programs
E. Close personal contact with infected but asymptomatic individuals, particularly children

Suggested correct Answer: E. Hepatitis A is an RNA virus acquired by direct contact or through
contaminated water/food. Most infections are asymptomatic, and nearly 100% of 18 year olds in
developing countries (before the vaccine was available) had serological evidence of past infection.
Children appear to be the reservoirs, making answer E the correct choice.

Infants and toddlers are less likely than older children or adults to develop jaundice, with one study
showing that only 1 in 12 infants with Hepatitis A developing jaundice. Small children can present
simply with diarrhea, and may be diagnosed as general acute viral gastroenteritis (whereas jaundice
is more common in adults, diarrhea is more common in children). Finally, children may be completely
asymptomatic with Hepatitis A (“anicteric hepatitis”). Despite these mild symptoms, vaccinations are
warranted in children to prevent them from harboring the virus and ultimately spreading it to adults, in
which the infection is more severe.

Question 3:
A previously healthy two year old boy is referred to you for elevation of liver function tests. When a
liver profile was drawn during an episode of fever, his serum alkaline phosphatase concentration was
elevated. He has no recent history of fractures. His growth and development have been normal. He
did not have neonatal liver disease. Review of symptoms is negative for pruritis, chronic diarrhea, or
acholic stools. His physical examination is normal. Laboratory studies at your institution confirm the
biochemical findings. Serum 25-hydroxy vitamin D levels are within the normal range.

Patient's results Normal range


Calcium 9.2 mg/dL 8.8- 10.7 mg/dL
Phosphorus 4.2 mg/dL 3.0 -5.0 mg/dL
Blood urea nitrogen 8 mg/dL 5-20 mg/dL
SGOT 28 IU/L 20-60 IU/L
SGPT 18 IU/L 5-45 IU/L
GGT 12 IU/L 6-20 IU/L
Conjugated bilirubin 0.1 mg/ dL < 0.3 mg/dL
Alkaline phosphatase 2800 IU/L 65-525 IU/L

Which of the following is the most appropriate next step to manage this child?
A. Abdominal ultrasound
B. Liver biopsy
C. Radiographs for rickets survey
D. 1,25 dihydroxy vitamin D level
E. No further laboratory tests

Suggested correct Answer: E. Some clinicians will further fractionate the alkaline phosphatase, to
determine whether it came from bone, intestine, or liver. High bone alkaline phosphatase results from
high osteoclast activity and bone turnover, such as with calcium or vitamin D deficiency. High
intestinal alkaline phosphatase has been reported in intestinal disease, such as intestinal ulcers.
High liver alkaline phosphatase comes from cholangiocyte injury and points to bile duct disease.

High alkaline phosphatase with no other clinical signs, however, often leads to a long course of lab
work and other tests with little diagnostic yield (i.e. X-rays, CT scans, liver biopsies, endoscopies). A
few case reports have been published on the issue, showing that alkaline phosphatase levels
Dr-Wahid Helmi
eventually return to normal with time. Other names for this phenomenon are benign transient hyper-
alkaline phosphatasia or benign transient hyperphosphatasemia. It is also known as “Ulysses
syndrome,” reflecting the long journey of diagnostic tests leading the clinician to no concrete
diagnosis.

Question 4:
Mutations in the cystic fibrosis transmembrane regulator (CFTR) may cause cystic fibrosis. The
protein is present on the apical surface of epithelial cells and regulates chloride secretion. Which of
the following is the best evidence that the CFTR is a c-AMP dependent protein kinase A activated
chloride channel?

A. Demonstration of c-AMP regulated chloride channels after transfection of the CFTR gene into
cells lacking CFTR
B. The presence of mutations in the CFTR in patients with elevated sweat chloride concentrations
C. Structural similarity of the CFTR to other ATP Binding Cassette (ABC) membrane transporters
known to regulate ion transport
D. Failure of heat stable enterotoxin to stimulate chloride secretion in cell with a mutation in the
CFTR
E. Demonstration of message for the CFTR in lung, pancreas, and biliary epithelium

Suggested correct Answer: A. This is a basic science question about experimental method. One
time-tested way to study a gene or protein is to introduce it into cells without it, and then study what
changes occur in the cell. In this case, introducing CFTR into a cell line without CFTR will allow the
scientist to manipulate the system to answer specific questions about CFTR.

One way to diagnose CF is the sweat test. This technique works because children with CF have high
concentrations of sodium and chloride in their sweat. They have higher concentrations of chloride
because the CFTR channel is used to reabsorb chloride from sweat as it passed through the ducts to
the skin. In CF patients, the CFTR works improperly and chloride cannot be reabsorbed, leading to
more chloride deposited in sweat.

Question 5:
A 2 year old boy with biliary atresia underwent liver transplantation for biliary atresia 6 weeks ago. He
was CMV seronegative at transplantation and received a left lateral segment graft from his mother
who was CMV seropositive. He has developed low-grade fever and vomiting. He is referred to you
for evaluation. He is taking cyclosporine, prednisone 10 mg each day and no antiviral medications.

Based on the clinical picture, which of the following would be the most specific test to confirm the
presence of active CMV disease?

A. Positive urine for CMV antigen


B. Blood sample which is CMV PCR positive
C. Rise in CMV IgG titers
D. Presence of increased CMV IgM titers
E. In situ hybridization positive for CMV in a gastric body biopsy

Suggested correct answer: E. CMV infection in normal hosts is usually inconsequential or consists of
mononucleosis-like symptoms with mild hepatitis. On the other hand, CMV infection in
immunosuppressed hosts (such as those status-post liver transplantation) can be devastating. CMV
may come from the new organ, transfused blood, or reactivation of latent infection. It can lead to a
clinical presentation similar to that of rejection, and if allowed to persist may progress to liver failure.
Dr-Wahid Helmi
To diagnose CMV disease, the best test is to identify CMV in serum, urine, or biopsy tissues. In this
case, the child has signs of CMV gastritis (vomiting), so identifying CMV is stomach tissue would
confirm the diagnosis. Urine positive for CMV antigen and PCR positive serum documents infection
somwhere in the body, but does not confirm or refute CMV causing GI symtpoms. Furthermore, PCR
is more sensitive than specific, and may be positive in patients with latent CMV with no disease.
Antibody tests can be confusing, because of maternal contributions (in <18 months) as well as
antibodies transferred in transfusions. Treatment for CMV includes IV ganciclovir until the infection
subsides, followed by PO valganciclovir.

Question 6:
Which of the following statements regarding transient lower esophageal sphincter relaxations
(TLESRs) is false?
A. TLESRs represent the major mechanism by which acid reflux occurs.
B. TLESRs reduce or obliterate the resting pressure of the lower esophageal sphincter.
C. Gastric distension increases the frequency of TLESRs
D. TLESRs cannot be induced by pharyngeal stimulation.

Suggested correct Answer: D. The LES is tonically contracted at rest, a process mediated by vagal
cholinergic and some sympathetic fibers. The LES relaxes with the esophageal peristaltic wave. The
LES also relaxes with TLSERs which may be due to increased NO production from iNOS. TLSERs
are thought to be the mechanism allowing GER to occur, and their frequency increases with gastric
distension. Simulating the pharynx, as in swallowing, also promotes LES relaxation.

Question 7:
In a double-blinded placebo controlled trial involving 200 patients, investigators observed that drug A
did not improve pulmonary function in children with cystic fibrosis. The change in pulmonary function
was normally distributed for both groups. The investigator is concerned that that drug A improved
pulmonary function but that the effect was not detected.

If the investigator is correct, which of the following is the most likely explanation for the observation:

A. Type 1 error
B. The analysts failed to use non-parametric analysis to compare the groups
C. Type II error
D. The patients in the study changed eating behavior while on drug A
E. The analysts failed to use Chi-square analysis to compare the groups

Suggested correct Answer: C. There are two types of common errors, type I and type II. Type I
errors occur when an association is found that does not really exist (false positive), whereas type II
errors occur when an association is not found when one really does exist (false negative). In
epidemiological terms, if a null hypothesis is incorrectly rejected when it is in fact true then a type I
error occurs. When a null hypothesis is not rejected despite it being false a type II error occurs.
Alpha is used to denote type I errors, whereas Beta is used to denote type II errors.

Question 8:
A 17 year old young man presents with a 3 year history of dysphagia. There has been no weight
loss, and the patient’s nutrition is good. The findings of upper endoscopy and esophageal
manometry are consistent with a diagnosis of achalasia. Which of the following statements is true?

A. The risk of esophageal perforation with pneumatic dilation is about 1 in 500.


B. Esophageal dilation must be repeated in fewer than 10% of patients.
C. Heller myotomy affords a good to excellent response in more than 80% of cases.
Dr-Wahid Helmi
D. Acid reflux is more common after pneumatic dilation than after surgical myotomy.

Suggested correct Answer: C. Achalasia is a rare disorder in children, resulting in improper


relaxation of the lower esophageal sphincter. One proposed mechanism is less NO formation,
leading to tonic LES contraction. On physical exam, patients have trouble swallowing solids and
eventually liquids. The classic manometry findings are: (1) poor esophageal peristalsis, and (2) no
LES relaxation upon swallowing. The first finding is always present, whereas the second finding may
be present to variable degrees. Other diagnostic modalities used are barium swallow study (proximal
dilations present) and endoscopy (to rule out infection, carcinoma, and leiomyoma).

Therapies commonly used (with increasing success) include: (1) calcium channel blockers, (2) Botox
injections, (3) pneumatic dilation, and (4) Heller myotomy. Pneumatic dilations can be done by a
gastroenterologist, though they pose particular challenges. Technically they are difficult, because
they require manipulations that cut the muscular layer but leave the mucosa intact. The incidence of
perforation can be as much as 5%, and most patients require repeat dilation over time. Acid reflux is
a common in all procedures that open the LES, and sometimes reflux control is attempted with
fundoplication following myotomy.

Question 9:
Cytokines act as messengers to regulate immune responses. One of the mechanisms by which the
calcineurin inhibitors, tacrolimus and cyclosporin, promote allograft survival following liver
transplantation is inhibition of transcription of the cytokine, interleukin 2. Which of the following is the
dominant mechanism by which interleukin 2 enhances the immune response to foreign tissue?

A. Promotes activation of macrophages


B. Promotes clonal expansion of activated T-cells
C. Promotes clonal expansion of plasma cells
D. Promotes activation of eosinophils
E. Promotes clonal expansion of B-cells

Suggested correct Answer: B. IL-2 binds to the IL-2 receptor and promotes T-cell expansion. IL-2
transcription is mediated by NF-AT (nuclear factor of activated T-cells), which moves into the nucleus
and directs transcription only after it is dephosphorylated. Calcineurin is the molecule that
accomplishes this dephosphorylation in the cytoplasm, and calcineurin activation occurs only through
an increase in intracellular calcium secondary to T-cell receptor activation. Both tacrolimus and
cyclosporine form complexes with different T-cell proteins. These complexes in turn bind to
calcineurin and inhibit its dephosphorylation activity, even in the presence of high intracellular calcium
levels.

Question 10:
A four year old boy presents to you with a history of bloody diarrhea. He was well until one week ago
when he developed bloody diarrhea while on vacation with his family. His family had been traveling
by car from California to Michigan and stopped at several fast-food restaurants. He was seen in a
local emergency room and treated empirically with amoxicillin. His symptoms have persisted. Today,
he appears pale and is listless. His serum hemoglobin is 7.8 and his white blood count is 14,000. His
stool contains blood and mucus. Which of the following is the most appropriate next step in
managing this child?

A. Stool culture and sensitivity


B. Treatment with Trimethoprim-Sulfa
C. Observation and reevaluation tomorrow
D. Colonoscopy
Dr-Wahid Helmi
E. Complete blood count, renal group and urinalysis

Suggested correct answer: E. This patient has signs of hemolytic uremic syndrome (HUS), a
vasculitis characterized by microangiopathic anemia, thrombocytopenia, and uremia. In children the
majority of cases follow a diarrheal illness caused by Shiga-like toxin, usually from E. coli type
O157:H37. The bacteria cause symptoms through 2 mechanisms: (1) local effacement of villi,
leading to an early watery diarrhea, and (2) toxin secretion, producing damage in epithelial and
microvascular endothelial cells throughout the body including the intestines and kidney, leading to
renal damage and bloody diarrhea. Damage to endothelial cells results in localized clotting, which
traps platelets and shears passing red blood cells. Coagulation times are normal in HUS.

Stool cultures should be done for public health reasons, especially in the cases of severe bloody
diarrhea. However, the most appropriate immediate step is to confirm the diagnosis through a CBC
and smear, and monitor for ongoing kidney damage (HUS is a leading pediatric cause of acute renal
failure in many countries). Antibiotic treatment is considered a contraindication, because lysis of
bacteria may release more Shiga-like toxin and worsen the vasculitis.

Question 11:
A 14 year old African-American young woman is referred for evaluation of asymptomatic elevation of
serum transaminases. She has not received blood transfusions. She is not taking medications.
Examination shows that her body mass index is 34. She is anicteric and has no signs of portal
hypertension but has prominent acanthosis nigricans in her neck folds and axilla.

Laboratory evaluation

Hepatitis B surface Antigen Negative


Hepatitis B surface antibody Positive
Hepatitis C antibody Negative
AST 100 IU/l
ALT 120 IU/l
Total serum bilirubin level 0.4 mg/dl
Serum immunoglobulin level Normal

If a liver biopsy were performed, the most probable histologic findings would be:

A. Mixed portal infiltration with necrotic hepatocytes at the limiting plate


B. Macrovesicular hepatic steatosis with mild portal inflammation
C. Microvesicular hepatic steatosis
D. Cirrhosis with portal inflammation and Mallory bodies
E. Normal histology

Suggested correct answer: B. This patient likely has non-alcoholic fatty liver disease (NAFLD). The
term encompasses a wide variety of liver pathology, from steatosis to steatohepatitis to cirrhosis. The
pathophysiology of this disease centers around insulin resistance, which is probably present in this
obese adolescent with acanthosis nigricans. The diagnosis of NAFLD is suggested by
hyperglycemia, hyperlipidemia, elevated transaminases (ALT>AST), and increased echogenicity on
liver ultrasound. A recent article on the topic concluded that normal ALT ranges at most children’s
hospitals are not sensitive for abnormal ALT levels, and that an ALT greater than 25 for boys and 23
girls should raise suspicion for liver disease.

The gold-standard diagnosis for NAFLD is (1) absence of other liver disease (i.e. infectious,
autoimmune, toxin, or genetic causes, and (2) liver biopsy showing characteristic NAFLD findings.
Dr-Wahid Helmi
The three most consistent biopsy findings are: i) macrovesicular > microvesicular steatosis, ii) lobular
inflammation consisting of neutrophils and lymphocytes (“mixed”), and iii) hepatocyte ballooning.
Fibrosis (portal > perisinusoidal in children) and glycogenated nuclei may also be present. In severe
cases, extensive fibrosis leading to cirrhosis is present.

Treatment for NAFLD is evolving. Weight loss has proven to be beneficial. Other therapies being
tested include insulin sensitizing agents (i.e. metformin), anti-oxidants to prevent oxidative damage
(i.e. Vitamin E), and N-acetylcysteine, among many others. Liver transplant for patients with NAFLD-
related cirrhosis has been performed, though fatty changes can reoccur in the transplanted organ.
Bariatric surgery is also a potential new therapy, because it often improves insulin sensitivity which
may in turn treat the pathophysiology underlying NAFLD.

Question 12 and 13:


A 19-month-old female infant came to the emergency room with a 5-day history of intermittent
abdominal pain and diarrhea. She has been passing 8 to 10 brown, watery stools a day. There was
no blood or mucus in her stools. She had vomited twice (once at the onset of the illness and on the
day of presentation), and the vomitus was nonbloody and nonbilious. She had no fever or other
systemic symptoms. There was no prior antibiotic use. Her mother had diarrhea for 2 days at the
onset of her illness.

Physical examination results were within normal limits except for right lower quadrant tenderness. No
masses were palpated. Rectal examination results were normal, but her stool was guaiac positive.
Laboratory study results included a hemoglobin concentration of 11.9 g/dL and a leukocytosis of
17,400/mm 3 with a normal white blood cell differential count. Renal profile and urinalysis results were
normal. Her stool was negative for leukocytes but positive for Clostridium difficile toxin and antigen.
Abdominal radiographs were obtained (Fig 1), and when she continued to have significant abdominal
pain, an abdominal computed tomography was requested (Fig 2).

Figure 1: Figure 2:

12. Based on the findings on the imaging studies what is your diagnosis?
Dr-Wahid Helmi

A. Pseudomembranous colitis
B. Ileocolonic intussusception
C. Lymphoma
D. Colonic polyp
E. Distal intestinal obstruction syndrome

13. What is the next appropriate step in the treatment of the patient?

A. Surgical resection
B. Colonoscopy
C. Air enema
D. Sweat chloride
E. Oral Flagyl

Suggested correct answers: A, E. Clostridium difficile is a gram-positive anaerobic bacillus that can
produce an infectious colitis. This usually happens after antibiotics are administered, which allows C.
difficile to flourish by clearing the bowels of normal flora. Infections are also commonly seen in
patients receiving anti-neoplastic therapy. C. difficile elaborates Toxin A (disease causing) and Toxin
B, which are identified in ELISA (protein) or PCR (gene) assays. C. difficile usually causes a mild
limited watery diarrhea, but long-term diarrhea and/or bloody diarrhea can also occur. In
pseudomembranous colitis, the most severe form of C. difficile infection, high fever, leukocytosis, and
hypoalbuminemia are present. Imaging shows boggy, thickened areas of bowels.

Treatment consists of stopping causative antibiotics and/or starting anti-C. difficile antibiotics. Oral
flagyl is the first line choice, followed by oral vancomycin. IV Flagyl is also acceptable, though IV
vancomycin is thought to be ineffective because it does not penetrate into the gut lumen. Surgery
may be needed in especially severe cases of pseudomembranous colitis.

Question 14:
The colon is able to absorb each of the following EXCEPT:
A. Monosaccharides
B. Acetate
C. Propionate
D. Butyrate

Suggested correct answer: A. The colon’s best known functions are water absorption and fecal
storage. However, the colon is also an enormous reservoir of gut bacteria. Bacteria metabolize
carbohydrates that make it past the small intestine, including digestion-resistant starches, excess
fructose and lactose, and poorly digestible monosaccharides such as lactulose, sorbitol, and
sucrolose. Bacteria ferment these sugars, producing gas and short-chain fatty acids such as acetate,
propionate, and butyrate as a by-product. These short-chain fatty acids in turn are absorbed by
colonocytes and may have numerous benefits including trophic properties for enterocytes.

Monosaccharides are absorbed in the small intestine, and are the only form in which sugars can be
absorbed. Salivary and pancreatic amylase digests starch into oligosaccharides, which are further
digested by brush border enzymes into the monosaccharide glucose. The brush border enzyme
lactase digests lactose into the monosaccharides glucose and galactose, whereas the brush border
enzyme sucrase digests sucrose into the monosaccharides glucose and fructose. The
monosaccharides then enter enterocytes through channels, because they are too big for passive
diffusion. Glucose and galactose are co-transported with sodium driven by a sodium gradient.
Fructose has its own channel that operates via diffusion from high fructose (lumen) to low fructose
Dr-Wahid Helmi
(inside the cell) gradients. The Na-glucose transporter on villi is the bases for oral rehydration
solution containing 2 molecules of sodium for every molecule of glucose. The diffusion-driven
fructose channel explains why children who drink too much juice have excessive fructose that cannot
enter enterocytes, leading to high fructose in the lumen and subsequent osmotic diarrhea (toddler’s
diarrhea).

Question 15:
A 2-year-old boy was referred to the Pediatric Gastroenterology Unit because of a short period of
vomiting, dysphagia, and decreased appetite. He had been operated at birth for an esophageal
atresia; the postoperative course and recovery had been uneventful. He remained healthy until the
present episode of acute vomiting and dysphagia, which began 48 hours before referral to our unit.
Physical examination was unremarkable. Complete blood count and sedimentation rate were normal.
Contrast radiography of the upper gastrointestinal tract ruled out anastomotic esophageal stricture but
strongly suggested the presence of a foreign body obstructing the lower esophagus. The child
underwent esophagogastroscopy under general anesthesia. At esophagogastroscopy, no foreign
body was present, but an ulcerative lesion of the lower esophageal region was noted, corresponding
to the location of the presumed esophageal foreign body before spontaneous migration.
Esophagoscopy was followed by a systematic examination of the stomach, pylorus, and duodenum.
Surprisingly, a 10-mm diameter lesion was noted in the lumen of the prepyloric region (Fig 1); the
surrounding mucosa was normal. Biopsies of this lesion were performed.

Figure 1

This lesion most likely represents:

A. Gastric leiomyoma
B. Heterotopic pancreas
C. Peptic ulcer
D. Gastric adenocarcinoma
E. Gastric hyperplastic polyp
Dr-Wahid Helmi
Suggested correct answer: B. Heterotopic pancreas, also known as a pancreatic rest, is a congenital
anomaly characterized by pancreatic tissue in ectopic locations. The defect results from abnormal
migration of pancreatic tissue. The most common site of pancreatic rests is in prepyloric gastric
antrum, though numerous other sites have been reported such as the duodenum, ileum, gallbladder,
common bile duct, and splenic hilum. There are a few case reports of distal esophagus pancreatic
rests in patients with isolated esophageal atresia.

Pancreatic rests are usually asymptomatic and found incidentally. They appear as a round, smooth
submucosal mass with a central crater. They can be symptomatic when large (>1.5 cm), leading to
abdominal pain, dyspepsia, and bleeding. Pancreatitis has also been described in pancreatic rests.

Neoplasms such as gastric adenocarcinoma usually do not occur in such young patients. Gastric
polyps commonly come in two varieties: (1) parietal cell hyperplasia secondary to PPI use, and (2)
hamartomas such as those in Peutz-Jehger syndrome. Reflux and strictures related to reflux may
occur after repair of esophageal atresia, because an intact LES sphincter is not present. However,
stomach protective mechanisms are functional and there is no increased incidence of peptic ulcers.

Question 16:
A one month old infant boy presents to you with complaints of persistent jaundice. The infant was a
term product of an unremarkable pregnancy. Since going home, he has been breast fed and has
nursed well. His mother has a prominent forehead with deep set eyes. Examination of the infant
shows an icteric infant boy. He is afebrile. Heart examination shows an III/VI holosystolic murmur. His
liver is palpable 2 centimeters below the right costal margin in the midclavicular line and he does not
have ascites. An abdominal ultrasound showed a small gall bladder. Pertinent laboratory studies
include:

Alkaline phosphatase 650 IU/L


Total serum bilirubin 7 mg/dl
Conjugated serum bilirubin 6.2 mg/dl
AST 95 IU/L
-glutamyl transpeptidase 650 IU/L
Serum albumin 3.6 mg/dL

Which of the following is the most likely diagnosis?

A. Biliary atresia
B. Alagille Syndrome
C. Galactosemia
D. Sclerosing cholangitis
E. Cystic fibrosis

Suggested correct answer: B. This patient has neonatal cholestasis. The differential of this condition
is broad, and includes infectious, genetic, metabolic, and toxin causes. Biliary atresia must be ruled-
out, because Kasai operations performed earlier (<45 days of life) may have the best outcome.
Galactosemia, in which the child is unable to convert galactose to glucose secondary to deficiencies
in one of 3 enzymes, leads to high serum galactose levels. It presents with weight loss, diarrhea,
vomiting, lethargy, positive reducing substances (i.e. galactose) in the urine, and, commonly, E. coli
sepsis. CF can present as neonatal cholestasis but cardiac findings are not characteristic. Neonatal
sclerosing cholangitis is a rare entity that presents histologically like biliary atresia. It differs clinically
because in sclerosing cholangitis the stools are pigmented. Sclerosing cholangitis is best diagnosed
by ERCP.
Dr-Wahid Helmi
In this patient, the combination of cardiac findings (usually pulmonary stenosis), cholestasis, and
family history points to Alagille’s syndrome. The disorder is characterized by paucity of
intrahepatic/interlobular bile ducts. It is caused by defects in the Notch signaling pathway, which is
critical for proper cholangiocyte differentiation. The most common mutation in Alagille’s syndrome is
in Jagged 1, the ligand for the Notch receptor. Because the Notch pathway is used in many
developmental processes, Alagille’s syndrome patients can have numerous extra-hepatic
phenotypes. These include cardiac, skeletal (i.e. butterfly vertebrate), ocular (posterior embryotoxin),
vascular (intracranial bleeding), and renal defects (in light of all these issues, pruritis from cholestasis
is still the complaint most often made by patients). For still unclear reasons, the penetrance of Notch
mutations varies, so that individuals in the same family with the same mutation may be affected to
different degrees.

Question 17:
A 14 year old young man is referred for evaluation of abdominal pain. He had a history of
gastroesophageal reflux disease that was refractory to medical therapy. He underwent a laparoscopic
fundoplication 6 months ago. He now complains of postprandial abdominal pain and distension. The
pain is not associated with vomiting, diarrhea, dysphagia or heartburn.

Which of the following is the most-likely cause for his symptoms?

A. Non-ulcer dyspepsia
B. Biliary tract dyskinesia
C. Recurrence of gastroesophageal reflux disease
D. Gas-bloat syndrome
E. Eosinophilic esophagitis

Suggested correct answer: D. The fundoplication operation takes proximal portions of the stomach
and wraps it around the lower esophagus to recapitulate LES tone. In the past it was a mainstay for
medication-resistant reflux. Subsequent studies, however, have shown that a sizeable number of
children must return to PPI use within a few years of the surgery. Other complications include “gas
bloat syndrome,” which occurs when stomach gas cannot escape through the esophagus. Symptoms
include abdominal distension, abdominal pain, wretching, and gagging.

Question 18:
All of the following statements regarding familial pancreatitis are true EXCEPT:

A. Pattern of inheritance is autosomal recessive


B. The mutated gene encodes cationic trypsinogen
C. Males and females are equally affected
D. Usually presents before 15 years of age
E. Patients with this disorder have a 50-fold increased risk of developing pancreatic cancer

Suggested correct answer: A. Familial pancreatitis involving the cationic trypsinogen gene (PRSS1)
is autosomal dominant. Mutations cause trypsinogen to autoactivate in acinar cells, producing trypsin
which in turn activates other pancreatic proenzymes. Together, the active enzymes collectively
destroy pancreatic tissues (normally, trypsinogen is activated only in the intestinal lumen by
enterokinase on the brush border). Other genetic causes of pancreatitis include mutations in the
serine protease inhibitor Kazal type 1 (SPINK-1). Recessive mutations in the CFTR gene can also
lead to chronic pancreatitis, by thickening the composition of secretions in the duct thus causing
backflow into the pancreas. Cases of hereditary pancreatitis usually present before age 10, and
patients with hereditary pancreas have a much higher risk for developing pancreatic cancer,
presumably due to many years of chronic inflammation.
Dr-Wahid Helmi

Question 19:
A 17-year-old girl is referred to you for evaluation of persistent episodic burning epigastric pain. An
adult gastroenterologist previously saw her. She had a history of using non-steroidal anti-
inflammatory drugs once each week for headaches. Serology for H. pylori was positive and
esophagogastroduodenoscopy showed a duodenal ulcer. She was given a 14-day treatment of
omeprazole and amoxicillin. Her symptoms have persisted. A stool antigen test for H. Pylori
performed more than 4 weeks after treatment was positive.

Which of the following is the most likely cause for her persistent symptoms?
A. Unrecognized esophagitis
B. Failure to adhere to the medical regimen
C. Primary resistance of H.pylori to amoxicillin
D. Failure to use an effective treatment for H.pylori eradication
E. Peptic disease due to NSAID

Suggested correct answer: D. This patient’s symptoms could be attributed to many causes, including
H. pylori gastritis, peptic ulcer disease, and eosinophilic disease. Given the positive H. pylori test and
duodenal ulcer, H. pylori disease is the most likely. H. pylori is a gram negative rod. Stool antigen
testing is the noninvasive diagnostic test of choice (serology, as performed in this case, is not
recommended, because standard values have never been established in children). Treatment
consists of a PPI plus 2 antibiotics, such as amoxicillin and clarithromycin. Antibiotic resistance has
been identified, and current guidelines recommend susceptibility testing only if the first round of
treatment fails to eradicate the organism.

One important area of active research is determining when to test children for H. pylori. Many
children are carriers (some populations have infection rates >50%), so results may return positive
even if H. pylori is not responsible for the child’s symptoms. H. pylori should be tested when ulcers
are found endoscopically, in iron deficiency refractory to therapy, in patients with MALT lymphoma, in
those with a family history of gastric cancer, and when checking the success of eradication therapy.
H. pylori should not be checked in patients with recurrent abdominal pain/non-ulcer dyspepsia,
GERD, and asymptomatic children. Practitioners who discover H. pylori incidentally (i.e. during
foreign body removal, because a primary care physician ordered the test) must make a decision on
whether or not to treat. Many practitioners will treat, given the H. pylori-associated long terms risks of
MALT lymphoma and gastric cancer.

Question 20:
A 22-month-old boy is referred for evaluation of poor growth and chronic diarrhea since birth. His
stools are large and foul-smelling. Stool cultures and examination of stool for ova and parasites has
been negative. His past medical history is remarkable for recurrent otitis media. His physical
examination shows that his weight and height are both below the 5th percentile for age. His abdomen
is protuberant. A sweat chloride was normal. His serum trypsinogen is low. Abdominal ultrasound
shows a large echogenic pancreas.

Which of the following laboratory abnormalities is most likely to be detected in this boy?

A. Increased stool alpha-1 antitrypsin levels


B. Elevated anti-endomysial antibody titers
C. Neutropenia
D. Peripheral eosinophilia
E. Macrocytic anemia
Dr-Wahid Helmi
Suggested correct answer: C. This patient shows evidence of pancreatic insufficiency, including poor
fat absorption, failure to thrive, low serum trypsinogen, and presumably low fecal elastase. The
negative sweat test argues against CF, making Shwachman-Diamond syndrome the most likely
diagnosis. Shwachman-Diamond syndrome leads to improper development of acinar cells (vs.
pancreatic duct problems in CF). Pancreatic tissue is replaced with fatty tissue, and pancreatic
lipomatosis produces an echogenic image on ultrasound. Treatment includes pancreatic enzyme and
fat soluble vitamin replacement. With time, pancreatic function does appear, with half of the patients
demonstrating pancreatic sufficiency by age 4.

Shwachman-Diamond syndrome also has bone marrow and skeletal abnormalities. Bone marrow
problems include recurrent neutropenia, anemia, thrombocytopenia, and possible leukemia. Skeletal
problems involve long bones and the thoracic cage. The inheritance is autosomal recessive, in an
incompletely characterized gene names SBDS. The product of this gene is likely to play a
fundamental role, because it is present in all archaea and eukaryotes. Another syndrome
characterized by pancreatic insufficiency secondary to fatty replacement of the pancreas is
Johanson-Blizzard syndrome. This syndrome comes with multiple congenital abnormalities
(deafness, imperforate anus, urogenital malformations, dental anomalies, “beak-shaped” face) as well
as significant endocrine involvement (hypothyroidism, panhypopituitarism, diabetes, growth hormone
deficiency).

Question 21:
A two year old boy with biliary atresia presents with anemia and hematemesis. He underwent a
hepatic portoenterostomy when he was 5 weeks old. Today, he is pale. His abdomen is non
distended but he has prominent abdominal wall vessels and his spleen is palpable 5 cm below the left
costal margin. After hemodynamic stabilization, you plan to perform upper endoscopy and
sclerotherapy. Under which of the following conditions is antibiotic prophylaxis recommended?

A. The patient has aortic stenosis


B. The patient has a history of spontaneous bacterial peritonitis
C. The patient has a history of recurrent cholangitis
D. The patient has multiple dental caries
E. The patient has an nasojejunal feeding tube

Suggested correct answer: A. Prophylactic sclerotherapy therapy versus watchful waiting in patients
with portal hypertension and esophageal varices is an active debate (banding is impractical in young
children, because the equipment needed for banding is too large). Sclerotherapy has been shown to
reduce to incidence of first-time variceal bleeds. However, these patients also had a higher incidence
of gastric variceal bleeding if gastric varices were present, presumably secondary to congestive
hypertensive gastropathy. Survival rate was not affected by prophylactic sclerotherapy.

Sclerotherapy can lead to transient bacteremia, so patients with heart disease are at increased risk.
The patients that qualify for prophylactic antibiotics have recently been modified. High risk patients
should receive antibiotics, including those with a prosthetic valve, history of endocarditis, complex
cyanotic heart disease, and systemic-pulmonary shunts. Aortic stenosis does not fit into this
category, so antibiotic therapy will be at the discretion of the endoscopist.

In cases of active bleeding, prophylactic antibiotics for all patients are warranted. Prophylactic
antibiotics have numerous benefits: i) protects patient from bacteremia during the procedure, ii) may
prevent infection from introducing endoscope, and iii) treats infections that may have caused
increased portal pressures and promoted onset of the bleed.

Question 22:
Dr-Wahid Helmi
A 22-month-old boy is referred for evaluation of chronic diarrhea. He has had persistent diarrhea
following an episode of acute gastroenteritis 4 months ago. His stools are loose and watery and of
large volume. He has never passed gross or occult blood per rectum. Despite his diarrhea, he has
grown normally and remains active. Stool cultures and examination of stool for ova and parasites has
been negative. Because of his diarrhea, his parents stopped giving him milk and he currently drinks
more than 1 liter each day of juice or Kool-aid. His physical examination is normal. Which of the
following is the cause for his chronic diarrhea?

A. Giardiasis
B. Excessive carbohydrate intake
C. Lactose intolerance
D. Celiac disease
E. Inflammatory bowel disease

Suggested correct answer: B. This patient has “toddler’s diarrhea,” an osmotic diarrhea from
excessive sugar intake. Clues to the diagnosis include normal growth and excessive juice intake.
Toddler’s diarrhea occurs with increased luminal sucrose. In the small intestine, sucrase digests
sucrose into glucose and fructose. Glucose is readily absorbed through sodium-glucose channels, in
a process that depends on sodium gradients. Fructose, on the other hand, is absorbed by diffusion
through fructose channels. When intracellular fructose levels reach a certain point, more fructose
cannot diffuse intracellularly and instead remains in the intestinal lumen. This excess fructose
becomes an osmotic force, drawing water into the lumen and leading to a watery diarrhea. The
patient’s symptoms will improve once juice intake is returned to normal.

Question 23:
A 15-year-old girl with steroid-resistant Crohn's disease who is in remission has been started on
mercaptopurine (dose 1.5 mg/kg/day). You find that she has decreased activity of the enzyme,
thiopurine methyltransferase (TPMT) (the enzyme responsible for a major alternative catabolic
pathway of mercaptopurine).

For which of the following is the patient at risk due to decreased TPMT activity

A. Relapse of Crohn's disease


B. pancytopenia
C. opportunistic infection
D. bacterial overgrowth
E. sclerosing cholangitis

Suggested correct answer: B. 6-MP is an antimetabolite initially developed to destroy immune cells in
the setting of leukemia. It is converted to a purine analogue that disrupts DNA replication. At lower
doses, it is used as immunosuppressive therapy in IBD. Azathioprine is converted to 6-MP non-
enzymatically. 6-MP then has 3 fates: i) 6-thiouracil (inactive), ii) 6-MMP (hepatotoxic), and iii) 6-TG
(active immunosuppressive compound). TPMT, or thiopurine methyltransferase, determines how
much 6-MP is converted away from the active metabolite 6-TG to 6-MMP. Patients with low TPMT
activity, or those with mutant alleles, produce excess amounts of 6-TG leading excessive immune
suppression and bone marrow failure. For this reason, TMPT activity or genotype is often tested
before administering 6-MP. Other important side effects of 6-MP include pancreatitis, hepatotoxicity
(presumably in part from 6-MMP metabolites), and possible hematopoetic neoplasm.
Dr-Wahid Helmi

Question 24:
A 16-month-old boy received a cadaveric liver transplant at age 10 months for biliary atresia. His
immunosuppressive medications are tacrolimus and prednisone. He presents to you with complaints
of diarrhea and intermittent fever. A colonoscopy shows findings consistent with post-transplant
lymphoproliferative disease (PTLD) and biopsy of the lesions shows predominantly lymphocytic
infiltrate. Which of the following is the best way to confirm the diagnosis (EBV associated PTLD)?

A. Quantitation of peripheral blood EBV viral load by polymerase chain reaction


B. Qualitative identification of peripheral blood EBV by polymerase chain reaction
C. Identification of B-cells in the biopsy by monoclonal antibody against CD20 (a specific marker for
B-cell)
D. Flow cytometry of blood to identify activated B cells
E. In situ hybridization of biopsy with EBER-1 probe (labels EBV-encoded RNA in infected cells)

Suggested correct answer: E. PTLD is a post-transplant complication related to EBV infection and
immunosuppression. When a child has been infected with EBV prior to transplant, he or she may do
better on minimal post-transplant immunosuppression to prevent EBV reactivation while also
preventing rejection. More often, children acquire EBV infection either i) during transplant from the
donor or ii) after transplant while being heavily immunosuppressed. EBV titers can be accurately
followed using serum EBV PCR measurements.

PTLD occurs when EBV-infected B cells proliferate excessively, usually (but not always) in the setting
on increased EBV PCR titers. There are at least two broad steps to this process. First, viral genes
overtake cellular machinery and promote polyclonal proliferation (ultimately, the fittest B cell clone is
selected and the lesion becomes monoclonal). Second, immunosuppression impairs T cells, which
normally check B cell proliferation. The final result is neoplastic proliferation of B cells, diagnosed
through histology using two criteria: i) evidence of lymphoproliferation on tissue biopsy, and ii)
presence of EBV (i.e. EBV RNA detected by in situ hybridization) in tissue.

Clinically, PTLD presents with mononucleosis-type symptoms, isolated lymph node involvement,
and/or lymphocyte invasion of tissues (liver, gut, iris). Gut PTLD presents with diarrhea, GI bleeding,
and weight loss. Treatment includes reduced immunosuppression (at the risk of inducing rejection),
anti-virals, surgical excision of the lesion, localized radiation therapy, chemotherapy, monoclonal
antibodies against the B cell antigen CD20, interferon therapy, and T-cell therapy.

Question 25:
A 6 year old boy is referred to you for post-prandial abdominal pain of 9-12 months duration. He has
no associated vomiting, heartburn, dysphagia, or diarrhea. His appetite is decreased. Past medical
history reveals that he found to have a Stage II Wilm’s tumor at age 2 years. He was treated with
Dr-Wahid Helmi
resection and received 3000 rads to his abdomen. Examination shows no evidence of malnutrition.
He has mild epigastric and right lower quadrant tenderness to deep palpation.

Laboratory evaluation
Hemoglobin 14 g/dL
Serum amylase Normal

Which of the following would be the most appropriate next step?

A. CT scan of the abdomen


B. Esophagogastroduodenoscopy with small intestinal biopsy
C. Hydrogen breath test
D. Upper GI contrast study with small-bowel follow through
E. Colonoscopy with biopsy

Suggested correct answer: A. Radiation enteritis is common after radiation treatment to the intestines
in pediatric patients. Radiation treatment creates free radicals from water, which in turn damage
neoplastic and normal tissue. Acute changes include edema and inflammation, producing abdominal
pain, diarrhea, and tenesmus. Chronic changes are due to obliterative arteritis that causes ischemia,
leasing to fibrosis, stricture, and/or ulceration. Patients may have dysmotility, bacterial overgrowth,
obstruction, altered absorption, in bleeding (with colonic involvement).

The first step in diagnosing the patient’s GI problem is to obtain a CT image to look for signs of
stricture/obstruction vs. tumor recurrence (though recurrence of stage I and II Wilm’s tumor is
uncommon). An UGI/SBFT may then be used to better evaluate the caliber of the small bowel and
the presence of strictures. Colonoscopy would be performed if there were concerns for colonic
disease, such as proctitis.
Dr-Wahid Helmi

Question 1:
A 3-month-old boy has failure to thrive and chronic diarrhea. Stools are pale, foul-smelling, and
greasy. Physical examination reveals a wasted, nonicteric boy who has coarse breath sounds and
rhonchi. The abdomen is distended and tympanitic. The liver has normal texture and is palpable
at the right costal margin.

Of the following, the MOST likely diagnosis is:

A. biliary atresia
B. celiac disease
C. congenital lactase deficiency
D. cystic fibrosis
E. Schwachman-Diamond syndrome

Suggested answer: D. Cystic fibrosis has a number of GI presentations. Meconium ileus at birth
is pathognomonic for the disease. Most infants have some degree of pancreatic insufficiency,
clinically leading to diarrhea (steatorrhea, secondary to fat malabsorption) and failure to thrive.
Some infants also develop cholestasis from inspissated bile which progresses to chronic liver
damage. Finally, older children and adults can develop intermittent small bowel obstruction,
called distal intestinal obstructive syndrome (DIOS) or “meconium equivalent.”

Biliary atresia would present with jaundice and pale stools. Celiac disease only presents after
exposure to gluten, which occurs after the child starts taking foods. Congenital lactase
deficiency is a rare cause of sugar malabsorption, which is not associated with steatorrhea or
pulmonary problems. Schwachman-Diamond syndrome is a rare recessive condition
characterized by: i) pancreatic insufficiency, leading to steatorrhea; ii) hypocellular bone
marrow with some or all cell lines down, resulting in frequent infections; iii) skeletal
abnormalities; and iv) poor growth.

Question 2:
A 15-year-old girl who is being evaluated for poor school performance and acting-out behaviors
is noted to have a large, firm liver. Rings of brown pigment are seen in Descemet membrane on
ophthalmologic examination.

Determination of which of the following is MOST likely to confirm the diagnosis in this patient?

A. Hepatic copper concentration


B. Hepatic iron concentration
C. Hepatitis B antibody titers
D. Serum alpha1-antitrypsin level
E. Sweat chloride level

Suggested answer: A. Wilson’s disease should be considered in any teenager with new onset
mental status changes. The disease is caused by mutations in ATP7B, which is involved in
hepatocyte copper trafficking. Tests favoring the diagnosis of Wilson’s disease include low
serum ceruloplasmin (because the liver cannot incorporate copper into apo-ceruloplasmin to
Dr-Wahid Helmi

create ceruloplasmin), high urinary copper excretion, and presence of corneal Kayser-Fleischer
rings. Hepatic copper concentration is considered the gold standard test, with concentrations
>250 mcg/g of tissue diagnostic (normal is <50mcg/g). However, because reports exist of
patients with Wilson’s disease and normal hepatic copper concentration, all factors must be
considered when making the diagnosis.

Question 3:
A 17-year-old boy who has evidence of Crohn’s disease in the terminal ileum develops severe
inguinal pain.

The MOST likely visceral source for the referred pain in this patient is the:

A. appendix
B. diaphragm
C. gallbladder
D. small bowel
E. ureter

Suggested answer: E. Crohn disease patients, especially those with terminal ileum disease, are
susceptible to renal calcium oxalate stone formation and resulting pain. Oxalate normally binds
to calcium in the gut lumen to form an insoluble precipitate passed in stools. However, in the
setting of fat malabsorption, calcium binds to free fatty acids instead and allows oxalate to bind
sodium. Sodium oxalate is soluble and is absorbed into the circulation. The oxalate is then
delivered to the renal system, where it binds calcium and precipitates out as stones.

Question 4:
Which of the following represents the MOST likely mechanism for peptic injury associated with
chronic administration of nonsteroidal anti-inflammatory drugs (NSAIDS)?

A. Generalized ischemia of the gut


B. Genetic predisposition
C. Inhibition of prostaglandin
D. Promotion of growth of Helicobacter pylori
E. Topical caustic injury

Suggested answer: C. NSAIDs can cause gastrointestinal injury by two mechanisms. First, they
can cause topical injury. NSAIDs are carboxylic acid derivates and are still protonated in the
low pH stomach environment. As a result, they remain soluble, are absorbed, and within the
more neutral gastric epithelial cells, deprotonate and cause injury. Second NSAIDs block
cyclooxygenase (COX) activity, preventing conversion of arachidonic acid into prostaglandin
precursors. Prostaglandins protect gastric mucosa from injury through various mechanisms,
including stimulating mucin production and increasing bicarbonate excretion. Because both
oral and IV NSAIDs cause peptic injury, the prostaglandin effect is thought to be more important
than the topical effect.
Dr-Wahid Helmi

NSAIDs that reach the distal small intestine and colon can cause intestinal damage and
increased permeability through similar mechanisms. As a result, NSAIDs have been known to
aggravate IBD.

Question 5:
A 7-year-old boy who has dysentery develops self-limited generalized seizures.

Of the following, the enteric pathogen MOST likely to cause these clinical findings is:

A. Campylobacter jejuni
B. Giardia lamblia
C. rotavirus
D. Salmonella typhimurium
E. Shigella dysenteriae

Suggested answer: E. Shigella species are resistant to stomach acid, multiply in the small
intestine, and then invade colonic cells. They lead to a number of intestinal manifestations,
including obstruction, perforation, toxic megacolon, and proctitis. They also have a number of
systemic effects, including bacteremia, arthritis, and hemolytic-uremic syndrome. Shigella has
also been associated with generalized seizures usually in the setting of high fevers in children
<15 years old. A Shigella toxin is thought to cause the seizures, though the exact toxin has not
been indentified.

Campylobacter jejuni is associated with Guillain-Barre syndrome, likely caused by antibodies


made in response to C. jejuni antigens. Rotavirus infections have been associated with seizures,
but less commonly that Shigella infections. Nontyphoidal Salmonella can lead to bacteremia and
meningitis, usually in children <1 year old. Giardia manifestations are usually limited to the
intestinal tract.

Question 6:
A 2-year-old girl is brought to the emergency department immediately after she swallowed some
drain cleaner. Physical examination reveals a crying child who is drooling; there are no abnormal
findings in the mouth.

Among the following, the BEST next step in the management of this child is to:

A. admit her for observation


B. arrange for follow-up in 1 week
C. encourage her to drink milk
D. obtain a consultation for upper endoscopy
E. order a barium swallow

Suggested answer: D. All patients with caustic ingestions should be evaluated with endoscopy,
because esophageal/gastric injury can be present without oropharynx symptoms. Endoscopy is
performed between 24-48 hours, because earlier endoscopy may not detect the full extent of
damage and later endoscopy may cause perforation. Diluting the ingested substance with milk
Dr-Wahid Helmi

or water is not recommended, because the amount of liquid required may induce vomiting and
cause further caustic damage. Barium swallows are not useful in the acute setting, because they
do not adequately detect injury or risk for stricture. However, in severe ingestions, they can be
used 2 to 3 weeks post- ingestion to evaluate for strictures.

In severe esophageal injuries, a nasogastric tube is placed with endoscopy-assisted visual


guidance to prevent perforation. The tube serves two purposes: i) allows a conduit for nutrition,
and ii) prevents complete stricturing of the esophagus. Corticosteroids are not recommended,
and they have shown no benefit in reducing strictures.

Question 7:
The physical features of a newborn girl suggest trisomy 21. She spits up the first feeding and
develops bilious emesis with subsequent feedings. Findings include a scaphoid abdomen, no
evidence of an abdominal mass, and quiet bowel sounds; the double-bubble sign is noted on a
plain radiograph of the abdomen (see radiograph).

Among the following, the MOST likely cause for the findings in this patient is:

A. antral web
B. duodenal atresia
C. Hirschsprung disease
D. meconium ileus
E. tracheoesophageal fistula

Suggested answer: B. 5% of Down syndrome patients have congenital gastrointestinal


abnormalities, including: i) duodenal atresia with our without annular pancreas, ii) imperforate
anus, iii) esophageal atresia with tracheoesophageal fistula, iv) Hirschsprung disease, and v)
celiac disease. Antral webs have been reported in Down syndrome patients but are not
common. Meconium ileus is most often seen in cystic fibrosis. Duodenal atresia can be detected
by KUB, which shows the “double bubble” sign (first bubble is stomach, second bubble is
duodenum with atretic outlet distally).

Question 8:
An 18-month-old girl has had inconsolable screaming, abdominal distention, and nonbilious
vomiting for the past 9 hours. Findings include irritability alternating with lethargy, quiet bowel
Dr-Wahid Helmi

sounds, a palpable mass in the right upper quadrant, hemoccult-positive stool, and decreased
bowel gas in the right lower quadrant.

The MOST appropriate next step is to obtain a(n):

A. air reduction enema


B. computed tomogram of the abdomen
C. laparotomy
D. ultrasonogram of the abdomen
E. upper gastrointestinal series

Suggested answer: A. Intussusception is the most common abdominal emergency in childhood,


occurring most frequently in the ileo-cecal junction. When the diagnosis is uncertain, abdominal
ultrasound or plain films may be useful. However, when the diagnosis is clear by the clinical
symptoms (as in the case) and there are no clinical signs of perforation, non-operative reduction
using air or water/saline is warranted. Surgical reduction is used if perforation is present, or if
the intussusception is limited to the small intestine and is causing symptoms (most
intussusceptions limited to the small bowel are asymptomatic and self-resolve).

Question 9:

For the past 3 months a 5-year-old girl has had intermittent, painless, bright red blood per rectum
in association with bowel movements. Inspection of the anus reveals no fissures, but blood is
present on the examiner's finger following digital examination of the rectum.

Of the following, the MOST likely cause of this patient's rectal bleeding is:

A. intussusception
B. juvenile polyp
C. Meckel diverticulum
D. peptic ulcer disease
E. ulcerative colitis

Suggested answer: B. Juvenile polyps and Meckel diverticulum – as opposed to intussusception,


peptic ulcer disease, and ulcerative colitis – causes painless rectal bleeding. Juvenile polyps are
benign hemartomas (non-neoplastic) usually found in the rectosigmoid region in children ages
2-8. They bleed when injured by stool in transit, and often present as bright red blood
depending on how distal they are located. Meckel diverticulum is a remnant of the
omphalomesenteric duct that persists in 2% of the general population. They consist of ectopic
gastric mucosa, and bleeding results from acid-related ulceration of tissue. Only 4% of affected
patients ultimately develop clinical symptoms, most before the age of 20 years.

Question 10:
A 6-year-old girl has had abdominal pain and nonbilious vomiting for 8 hours. History reveals
cough and fever for the past 3 days. Findings on physical examination include temperature 39°C
(102.2°F); tachypnea; toxic appearance; diffuse, voluntary guarding; and quiet bowel sounds.
Dr-Wahid Helmi

Of the following, the examination MOST likely to establish the etiology of the abdominal pain
and fever in this patient is a(n):

A. abdominal radiograph
B. chest radiograph
C. complete blood count
D. rectal examination
E. upper gastrointestinal series

Suggested answer: B. This patient likely has pneumonia and associated abdominal pain, which
can best be diagnosed with a chest X-ray. A chest X-ray would also be able to detect air under
the diaphragm, if perforation is a concern. Abdominal pain in the setting of pneumonia is
referred, because the T9 dermatome distribution is shared by both the abdomen and lung. A
CBC could indicate infection, but would not identify which part of the body is affected. A rectal
examination would be useful if constipation causing abdominal pain is suspected. An UGI series
is best used to define anatomy; however, given that this child has no history of previous emesis,
malrotation is unlikely.

Question 11:
An 1800 g infant born at 34 weeks' gestation is being fed enterally with a 24 kcal/oz formula
developed for preterm infants. The formula contains supplemental medium-chain triglycerides.

Among the following, the BEST explanation for using medium-chain triglycerides is to

A. decrease watery stools


B. enhance absorption of calcium
C. enhance absorption of iron
D. enhance absorption of vitamin C
E. prevent malabsorption of fat

Suggested answer: E. Premature infants have impaired bile and pancreatic lipase secretion,
and, as a result, have fat malabsorption. Medium chain triglycerides can bypass these
deficiencies, because they are digested and absorbed without the need of bile acids, pancreatic
lipase, or co-lipase. In addition, MCTs enter the circulation directly from the enterocyte,
without passing through the lymphatic system.

Despite the theoretical benefits of MCTs in preterm infants, their actual benefits remain
controversial. In formulas with MCTs versus long chain triglycerides, there was no benefit in
weight gain, lipid absorption, or mineral absorption. Furthermore, MCTs may be incompletely
oxidized leading to increased urine dicarboxylic acid excretion and metabolic inefficiency.

Question 12:
A 10-year-old boy adopted from Romania has pruritus, mild icterus, and hematemesis. Physical
examination reveals an anxious boy who has normal vital signs, ascites, hepatosplenomegaly,
Dr-Wahid Helmi

and a prominent venous pattern over the abdomen. Stools are charcoal colored and guaiac
positive.

Of the following, the MOST likely cause for the hematemesis is:

A. esophageal varices
B. gastric polyp
C. peptic ulcer disease
D. posterior nasal bleeding
E. thrombocytopenia

Suggested answer: A. This patient likely has liver disease and resulting portal hypertension,
causing esophageal varices. Liver disease is suggested by pruritis (bile acid back-up into the
circulation) and icterus. Portal hypertension is suggested by splenomegaly, ascites, and a
prominent venous pattern on the abdomen. When esophageal varices bleed, they can result in
hematemesis. Blood can also transit through the GI tract and appear as black, charcoal colored
stool. Gastric lesions (such as a gastric polyp or peptic ulcer disease), posterior nasal bleeding,
or thrombocytopenia would not give extra-intestinal symptoms such as ascites or splenomegaly.

Question 13:
A 17-year-old adolescent who is receiving nonsteroidal anti-inflammatory agents for juvenile
rheumatoid arthritis has the acute onset of epigastric pain. He vomits "coffee ground" material
containing flecks of blood that clears quickly with gastric lavage.

Among the following, the MOST likely cause for these clinical findings is

A. duodenal ulcer
B. gastritis
C. Helicobacter pylori infection
D. Mallory-Weiss tear
E. posterior nasal bleeding

Suggested answer: B. NSAIDs cause gastritis, by impairing prostaglandin production by gastric


mucosa (see question 4 above). Prostaglandins exert their protective role by inducing mucin
and bicarbonate secretion. NSAIDs and aspirin can also cause duodenal ulcers, though H.
pylori infection is a more common cause. The mechanism by which NSAIDs cause duodenal
ulcers is acid dependent and prevented by acid blockade. Mallory-Weiss tears are esophageal
tears associated with vomiting, whereas posterior nasal bleeding should not cause epigastric
pain.

Question 14:
A 9-year-old girl has the height age of a 7-year-old and the bone age of a 6-year-old.

Among the following, the MOST likely cause of her short stature is

A. achondroplasia
Dr-Wahid Helmi

B. hypothyroidism
C. malnutrition
D. normal variant short stature
E. Silver-Russell syndrome

Suggested answer: B. This patient has short stature in the context of delayed bone age. Both
hypothyroidism and growth hormone deficiency are associated with short stature and
corresponding delay in bone maturation. As a result, when re-supplemented with either thyroid
hormone or growth hormone, patients demonstrate good growth potential.

Achondroplasia, caused by a dominant mutation over-activating the Fgf3 receptor, leads to


shortened bones that would produce a shorter stature than in this patient. At the growth plates,
achondroplasia patients do demonstrate delayed bone age. Severe malnutrition will stunt
growth but not affect bone age. Normal variant short stature by definition is short stature in the
presence of normal bone age. Finally, Russell-Silver syndrome patients typically have a
constellation of symptoms, including IUGR, prominent forehead, triangular face, and body
asymmetry. They have delayed bone age early with fast advancement later.

Question 15:
A 15-month-old boy who is a recent immigrant from the Caribbean area has rectal prolapse.
Stools sometimes contain mucus and blood. Physical examination reveals a child who appears
well nourished, but short for age. The mucosal prolapse is easily reducible; other findings on
rectal examination are normal.

Of the following, the MOST likely cause for this boy's rectal prolapse is:

A. celiac disease
B. chronic constipation
C. cystic fibrosis
D. rectal polyp
E. trichuriasis

Suggested answer: E. Rectal prolapse results from intussusception of the upper rectum and
sigmoid colon. There are a number of predisposing factors, including: i) increased intra-
abdominal pressure, from constipation, coughing, or vomiting; ii) diarrheal illnesses, from
parasites such as Ascaris lumbricoides and Trichuris trichiura or from malabsorptive syndromes
such as celiac disease; iii) malnutrition (hypoproteinemia leading to mucosal edema and smaller
fat pads reducing perirectal support); iv) inflammatory lesions such as polyps producing lead
points; and v) cystic fibrosis (may occur in 20% of CF patients under 3, from a combination of
coughing, malabsorptive diarrhea, and malnutrition). The patient’s clinical history is most
consistent with parasitic infection.

Question 16:
An antral or pyloric web (diaphragm) is considered in the differential diagnosis of a 6-month-old
girl with failure to thrive syndrome and nonbilious vomiting.
Dr-Wahid Helmi

The MOST specific study for diagnosing this condition is

A. gastric emptying study


B. plain abdominal radiography
C. ultrasonography
D. upper gastrointestinal contrast study
E. upper gastrointestinal endoscopy

Suggested answer: E. An antral/pyloric web is a circumferential mucosal septum in the pyloric


region, projecting into the lumen perpendicular to the long axis of the antrum. Antral webs may
have a large aperature and be clinically insignificant; alternatively, they may be nearly complete
and cause gastric outlet obstruction symptoms. Histologically, webs are made of gastric
mucosa, submucosa, and muscularis mucosae. Plain abdominal radiography may show a
dilated stomach, whereas an UGI study could show an extra compartment between the web and
antrum reminiscent of a second duodenal bulb. The most specific study would be to directly
visualize and sample the tissue through upper endoscopy.

Question 17:
A 10-year-old child has had intermittent diarrhea and weight loss over the past year.

A TRUE statement regarding testing with guaiac or orthotolidine for occult blood in this patient's
stool is:

A. Microscopic examination of the stool is a better test for detecting occult blood
B. Negative results exclude lower gastrointestinal bleeding
C. Positive results confirm the presence of occult blood
D. These tests detect peroxidase activity in hemoglobin
E. These tests quantitate the amount of hemoglobin in the stool

Suggested answer: D. The guaiac-based fecal occult blood test (gFOBT) is a qualitative test that
detects a peroxidase reaction (which turns guaiac-impregnated paper blue). Because heme from
hemoglobin has peroxidase activity, gFOBT has been used to detect the presence of a GI bleed.
However, gFOBTs can produce false positives and negatives. False positives occur because
certain foods have peroxidase activity. These include animal meats with high heme content
(beef, lamb) and raw peroxidase-rich fruits and vegetables (broccoli, cauliflower, radishes,
turnips, and some melons). False negatives occur depending on sampling. As a result, in
colorectal cancer screening, three to six gFOBTs are performed in sequence to establish a
gFOBT negative result. Microscopic examination may miss blood depending on what part of the
stool is studied.

Question 18:
An 18-year-old girl who is taking tolmetin for juvenile rheumatoid arthritis develops gastritis.

Which of the following medications would have been MOST likely to prevent the development
of peptic disease in this patient?
Dr-Wahid Helmi

A. Antacids
B. Corticosteroids
C. H2-blockers
D. Misoprostol
E. Sucralfate

Suggested answer: D. Tolmetin is an NSAID, which causes gastritis by inhibiting prostaglandin


production. Prostaglandins in turn have a number of effects, including increasing bicarbonate
secretion and mucin production. As a result, the prostaglandin E analog Misoprostol is the most
appropriate prevention strategy. Though less effective than Misoprostol, PPIs have also been
shown to be protective in preventing gastric ulcers or NSAID-related duodenal damage (NSAID-
related duodenal damage, compared to gastric damage, is more closely related to NSAIDs
ability to increase acid secretion). Antacids, H2-blockers, and sucralfate may all have some
benefit because each replicates some aspect of the protective effects of prostaglandins.
Corticosteroids promote gastritis.

Question 19:
A previously healthy 13-month-old boy has had two bloody stools in the past 6 hours. He has
had no nausea, vomiting, or diarrhea. The child appears well and has good peripheral perfusion.
The pulse is 130/min. Abdominal examination reveals normal findings except for hyperactive
bowel sounds. Maroon-colored, guaiac-positive stool is found on rectal examination.
Radiography of the abdomen shows a nonspecific bowel gas pattern.

Of the following, the MOST likely diagnosis is

A. anal fissure
B. Crohn disease
C. intussusception
D. Meckel diverticulum
E. peptic ulcer disease

Suggested answer: D. Four clinical aspects suggest a bleeding Meckel diverticulum: i) the
bleeding is painless (vs. painful in intussusception and IBD); ii) the bleeding is copious (vs. not
as much in anal fissures); iii) the patient is 13 months old and healthy (vs. usually older and
malnourished for Crohn disease); and iv) the blood is maroon, suggesting a bleed in the
proximal bowels. A Meckel scan may confirm the diagnosis, though false negatives are common.
The scan uses intravenous 99m technetium pertechnetate which is taken up by gastric mucosa.
Positive signal in the terminal ileum area indicates ectopic gastric mucosa and is consistent with
a Meckel diverticulum.

Question 20:
A 2-year-old boy has just swallowed a quarter. Physical examination reveals a quiet child in no
distress

The MOST appropriate first step in managing this patient is to


Dr-Wahid Helmi

A. administer glucagon intramuscularly


B. administer papain
C. admit for observation
D. obtain radiographs
E. perform an upper endoscopy

Suggested answer: D. The most appropriate step is to obtain radiographs for two reasons.
First, radiographs will confirm that the quarter in not in the trachea (if in the trachea, the side
instead of the face of the coin would be seen on PA films). Second, radiographs will identify the
quarter’s location in the GI tract. Coins in the esophagus that cause symptoms should be
removed immediately, whereas asymptomatic coins can be watched for 24 hours to allow
passage in the stomach. Coins in the stomach can be left to pass in the stools; however, if the
coin remains in the stomach after 4 weeks, it is unlikely to pass and should be removed
endoscopically.

Coins lodged in the esophagus for more than 24 hours should be removed via upper endoscopy,
because they may cause aspiration or other complications. There are 3 esophageal sites where
coins/foreign objects become lodged: the cricopharyngeus, mid-esophagus, and above the LES.
Glucagon, which lowers the LES, has been tried with little success in helping coins move to the
stomach. Papain has also been tried in cases of food impaction, to digest the food. Papain is
now contraindicated because it poses a risk for esophageal injury and possible perforation.

Question 21:
Of the following, the MOST common feature of chronic, nonspecific diarrhea of infancy and
childhood (toddler's diarrhea) is:

A. guaiac-positive stools
B. intermittent fever
C. lactose intolerance
D. significant weight loss
E. unimpaired growth

Suggested answer: E. Nonspecific diarrhea of infancy (toddler’s diarrhea) is defined as the


passage of three or more large, unformed stools during waking hours (not at night) for four or
more weeks. It begins in infancy or preschool years, does not cause failure to thrive, and does
not have a specific definable cause. Toddler’s diarrhea may be caused by rapid intestinal transit
secondary to uninterrupted MMCs, because food particles are often seen in the stool. Other
proposed mechanisms include dumping bile acids and hydoxyl fatty acids into the colon, as well
as excessive sugar intake promoting an osmotic diarrhea. Symptoms usually improve with
dietary modifications including increased fat and fiber intake, limited fluid intake, and avoidance
of fruit juices (especially apple, prune, and pear juice which have a high osmotic load).
Symptoms usually resolve by age 4.

Guaiac-positive stools, intermittent fever, and significant weight loss are associated with
inflammatory or infectious causes of diarrhea. Primary lactose intolerance is rare in children
and usually is accompanied by additional signs and symptoms including poor growth.
Dr-Wahid Helmi

Secondary lactose malabsorption can occur in infants following temporary villi blunting from an
infectious gastroenteritis.

Question 22:
Among the following, the LEAST important variable in the development of diarrhea is

A. gender
B. immunologic status
C. nutritional status
D. presence of chronic infection
E. presence of systemic disease

Suggested answer: A. Generally, diarrhea occurs when enterocytes are actively secreting
solutes that draw water with them (secretory diarrhea) or when they are unable to absorb
osmotically active solutes (osmotic diarrhea). There is a cyclic relationship between
immunological and nutritional status in causing diarrhea, whereby poor immunological status
leads to infection/diarrhea, which causes malnutrition, which in turn increases vulnerability to
further infection/diarrhea. Systemic diseases can also cause malnutrition, which increases
vulnerability to infection, which may in turn cause diarrhea. There is no evidence that gender is
a direct variable correlating with the development of diarrhea.

Question 23:
A 5-year-old girl has had a recent onset of postprandial emesis and epigastric pain that awakens
her at night. Physical examination reveals normal findings except for guaiac-positive stools.
There is a strong family history of peptic ulcer disease.

Among the following, the MOST specific diagnostic study for peptic ulcer disease is

A. abdominal ultrasonography
B. plain abdominal radiography
C. technetium pertechnetate scanning
D. upper gastrointestinal contrast study
E. upper gastrointestinal endoscopy

Suggested answer: E. Peptic ulcer disease is most accurately diagnosed by visualizing the
mucosa, via upper endoscopy with biopsies. UGI contrast studies have been used as a
noninvasive test to detect mucosal abnormalities; however, they have high false positive and
false negative rates. Technetium pertechnetate scanning (Meckel scan) is useful for detecting
gastric tissue especially in ectopic places, not for detecting gastritis or ulcers. Abdominal
ultrasonography and plain abdominal radiography are not used to diagnose peptic ulcer
disease.

Question 24:
A 3-year-old boy with acute lymphoblastic leukemia in hematologic remission is receiving
vincristine, methotrexate, and 6-mercaptopurine. He develops abdominal pain and distention and
nausea without fever or diarrhea.
Dr-Wahid Helmi

These findings are MOST likely caused by

A. Escherichia coli gastroenteritis due to granulocytopenia


B. intestinal candidiasis due to lymphopenia
C. intestinal mucosal ulcerations due to methotrexate toxicity
D. necrotizing enterocolitis due to 6-mercaptopurine toxicity
E. reduced intestinal motility due to vincristine toxicity

Suggested answer: E. In humans and animal models, vincristine reduces gastric motility
presumably from its neuropathic side effects. The most common motility problem with
vincristine is constipation, usually starting 3 days after the dose is given. However, other signs
of dysmotility, including abdominal distension and vomiting, have been known to occur. The
patient’s lack of diarrhea is inconsistent with enteritis. Methotrexate commonly causes
hepatotoxicity, vomiting, and mucositis, without abdominal distension. 6-MP commonly causes
vomiting and may cause elevation in liver numbers. It is also associated with a severe
hypersensitive reaction, with gastrointestinal symptoms of nausea, vomiting, diarrhea, and fever.

Question 25:
Of the following, the MOST likely cause of passage of bright red blood from the rectum in an
otherwise healthy, asymptomatic 3-year-old child is

A. intussusception
B. juvenile polyp
C. Meckel diverticulum
D. peptic ulcer
E. ulcerative colitis

Suggested answer: B. The two most common causes of rectal bleeding in the 2-5 year age group
are juvenile polyps (asymptomatic) and infectious colitis (symptomatic). A Meckel diverticulum
is also possible, but classically the majority of cases (60%) occur before age 2. Intussusception
causes pain and is accompanied by red “currant jelly” stool. Peptic ulcers usually present later
and produce black, melanotic stools. Ulcerative colitis would likely be accompanied by other
symptoms, including diarrhea.
Dr-Wahid Helmi
Pediatrics consultant
Dmiate-Egypt
#revision_6_All_Ped.
1. Which of the following is the most likely pathogen
responsible for bronchiectasis?
(A) Corynebacterium diphtheriae
(B) Streptococcus pneumoniae
(C) parainfluenza virus
(D) rhinovirus
1. (B) Bronchiectasis has numerous etiologies. Most
commonly, cultures reveal normal oral flora from the lower
respiratory tract: Streptococcus pneumoniae,
Staphylococcus aureus,Haemophilus influenzae,
Pseudomonas aeruginosa. Parainfluenza viruses typically
are responsible for croup. Corynebacterium diphtheriae is
the causative organism for diphtheria.
Rhinovirus is the most common pathogen isolated with acute
viral rhinitis or the common cold

2. A 3-month-old infant had a mild microcytic, hypochromic


anemia at birth and the screen was negative for sickle cell
disease/trait. She was started on iron therapy and presents
today for follow up. The hemoglobin (Hgb) electrophoresis
laboratory results are:
Hemoglobin 8.8 mg/dL (normal: 10.5–14.0)
Hematocrit 25% (normal: 33–42)
Mean corpuscular volume (MCV) 60 fL (normal: 70–90)
Mean corpuscularhemoglobin concentration (MCHC) 32
g/dL (normal: 33–37)
Hgb A2 27% (normal: 1.5%–4%); Hgb A1 30% (normal:
76%–99%); HgF (fetal hemoglobin)
50% (normal: 0%–20%); Bart Hgb 0% (normal: 0%)
Which of the following is the MOST likely diagnosis?
(A) heterozygous alpha thalassemia
(B) homozygous alpha thalassemia
(C) beta thalassemia major
(D) beta thalassemia minor
2. (D) The typical hemoglobin electrophoresis for beta
thalassemia minor has an elevated level of hemoglobin A2.
In a normal infant there is mainly HgF and HgA1 with
minimal amounts of A2.Bart hemoglobin is diagnostic for
the alpha thalassemias after the neonatal period is over. Beta
thalassemia major will only have fetal hemoglobin on
electrophoresis. Because of the high
incidence of false-negatives in hemoglobin screenings in the
neonatal period, it is important for the provider to do a full
work-up of microcytic, hypochromic anemias to ensure
proper diagnosis.

3. Which of the following is the most common etiologic agent


for acute tonsillitis in the United States?
(A) adenovirus
(B) group A beta-hemolytic Streptococcus Pyogenes
(C) Epstein–Barr virus
(D) Mycoplasma pneumoniae
3. (B) In children who present with symptoms of sore throat
and fever, approximately 50% to 70% of these cases are due
to a viral infection. Adenovirus is one of the most common
etiologic viral
agents. Epstein–Barr virus is the etiologic agent for
mononucleosis and while very common in the United States
it is still less than rhinoviruses and coronaviruses. The two
remaining choices are bacterial pathogens of which group A
beta-hemolytic streptococcus (GAS) is the most
common followed by the less common pathogens (group C
Streptococcus, Arcanobacterium haemolyticus, and
Streptococcus pneumoniae). As a single agent, GAS is the
most common etiology of acute tonsillitis and pharyngitis
4. In a pediatric patient with suspected congestive heart
failure, which of the following signs and symptoms would be
least likely seen on physical examination?
(A) bradycardia
(B) cardiomegaly
(C) hepatosplenomegaly
(D) tachypnea
4. (A) In left-sided congestive heart failure, the signs of
tachycardia, tachypnea, intercostals retractions, rales, and
rhonchi are found. Hepatosplenomegaly is a sign of right-
sided congestive heart failure. Bradycardia is not associated
with either left- or right-sided congestive heart
failure in the pediatric patient.

5. A 14-year-old girl presents to the office for a third visit


over the past month complaining of fatigue and pain in her
pelvic bones. Her previous evaluation included the following
laboratory tests:
Heterophile antibody test: negative
Hematocrit: 34% Since her last visit to the office, she has
lost 4 lb. Her mother reports she has a poor appetite. On
physical examination, she is noted to be pale, has several
large ecchymotic areas on her legs, and has inguinal
lymphadenopathy. Her complete blood cell count (CBC)
results are given below:
Which of the following is the MOST likely diagnosis?
(A) Hodgkin's lymphoma
(B) acute lymphoblastic leukemia
(C) acute myelogenous leukemia
(D) infectious mononucleosis/Epstein–Barr virus
5.B) Leukemia is the most common form of childhood cancer.
Acute lymphoblastic leukemia is the most common form of
leukemia in childhood, accounting for approximately 4 out of
100,000 children younger than the age of 15. The clinical
presentation is variable, ranging from severe
with a life-threatening infection to asymptomatic at a routine
well-child visit. Often, there is a 3-to 4-week history of an
illness prior to the diagnosis, with signs and symptoms including
malaise, anorexia, intermittent fever, bone tenderness, pallor,
petechiae, purpura, and abdominal pain.
Findings noted on the physical examination include pallor,
petechiae, purpura, retinal hemorrhages, lymphadenopathy
(either localized or generalized to cervical, axillary, or inguinal
areas), bone and joint tenderness (especially in the pelvis, lower
vertebral bodies, and femur،(
hepatosplenomegaly, and nephromegaly. Initially, the most
useful test is a complete blood count with differential, revealing
multiple cytopenias and leukemic blasts. The bone marrow
examination is diagnostic, revealing a homogeneous infiltration
of leukemic blasts replacing normal marrow. Acute
myelogenous leukemia typically presents with
hyperleukocytosis (WBC<
(100,000or with myeloblasts on peripheral smears and bone
marrow biopsies. It accounts for 25% of leukemias in childhood.
Patients with chronic Epstein–Barr virus infections present with
sore throat, fever, posterior cervical lymphadenopathy, and
malaise associated with atypical lymphocytosis and a positive
heterophile antibody test. Hodgkin's lymphoma typically
presents
with painless cervical adenopathy and a normal CBC. However,
typically the C-reactive protein and erythrocyte sedimentation
rates are elevated.

6. Which of the following is the most common etiologic agent


of bacterial meningitis in the pediatric population of the
United States?
(A) Streptococcus pneumoniae
(B) Haemophilus influenzae type B
(C) Listeria monocytogenes
(D) Neisseria meningitides

6. (A) Despite the increase in vaccination of infants in the


United States, Streptococcus pneumoniae remains the most
common etiologic agent for bacterial meningitis in the
pediatric population. Haemophilus influenzae type B is the
second most common, but has gone down
significantly due to the widespread vaccination of children.
Neisseria meningitides has approximately 2,400 to 3,000
cases a year. Meningitis due to Listeria monocytogenes is
typically seen in the neonatal period due to transmission
from the mother. It is present in normal
fecal matter in around 10% of the population. Its rates have
gone down due to strict guidelines for the food industry,
resulting in less than 1,000 cases per year.

7. Three weeks ago, an 8-year-old child was diagnosed with


Streptococcal pharyngitis based upon a positive throat
culture for group A beta-hemolytic streptococcus. Today,
she returns to the clinic with evidence of carditis. The
differential diagnosis includes rheumatic fever. What
additional finding would allow you to make the diagnosis of
rheumatic fever based upon the
modified Jones criteria?
(A) leukocytosis
(B) polyarthritis
(C) elevated erythrocyte sedimentation rate (ESR)
(D) erythema multiforme
7. (B) The diagnosis of rheumatic fever is based on clinical
grounds using the modified Jones criteria. Two major
manifestations or one major and two minor manifestations
in addition to supporting evidence of a preceding
streptococcal infection are needed to make the diagnosis of
rheumatic fever. The major manifestations are polyarthritis,
carditis, erythema marginatum,
subcutaneous nodules, and Sydenham chorea. The minor
manifestations are fever, arthralgia,previous rheumatic
fever or rheumatic heart disease, an elevated sedimentation
rate or Creactive protein, and a prolonged P–R interval. The
supporting evidence of a preceding streptococcal infection
includes elevated titers of antistreptolysin O or other
streptococcal
antibodies and positive throat culture for group A beta-
hemolytic streptococcus
8. In a patient who has newly diagnosed Hemophilia B,
which of the following laboratory results would be expected
on a coagulation panel?
(A) Increased aPTT (activated partial thromboplastin time),
normal PT (prothrombin time),
factor VIII deficiency
(B) + von Willebrand factor, decreased aPTT, and increased PT
(C) Increased PT, increased bleeding time, decreased platelets,
decreased fibrinogen
(D) Increased aPTT, normal PT, normal thrombin time
8. (D) Hemophilia B, also known as Christmas disease and
factor IX deficiency. Factor IX is activated on the intrinsic
side of the coagulation cascade right before the common
pathway and the result is an increased aPTT, with a normal
prothrombin time, thrombin time, and INR. It does
not affect platelets nor bleeding time.

9. A routine physical examination of a 12-year-old girl


demonstrates dark, coarse, curly pubic hair spread sparsely
over the pubic symphysis, as well as elevation of the breast
and areola without separation of their contours. According
to Tanner stages of sexual maturation, at what stage
would you assess her sexual maturity?
(A) Tanner Stage II
(B) Tanner Stage III
(C) Tanner Stage IV
(D) Tanner Stage V
9. (B) Tanner stages of sexual maturation categorize the
progression of pubertal development in girls according to
pubic hair and breast development. Menarche usually
occurs 18 to 24 months
following the onset of breast development. In female breast
development, Tanner Stage I is an absence of breast
development; Stage II is a small, raised breast bud; Stage III
shows further enlargement/elevation of breast and alveolar
tissue; Stage IV is the areola and papilla forming a
secondary mound on breast contour; and Stage V is the
mature breast with alveolar area as part of the breast
contour. For the stages of pubic hair development, Stage I is
prepubertal, an absence of hair; Stage II shows sparse, fine
hair, primarily on the border of labia; Stage III is pigmented
and curly and increases in quantity on the mons pubis; Stage
IV is increased quantity of coarser texture with labia and
mons pubis well covered; and Stage V is mature adult
distribution with spreading to medial thighs.

10. During the first year of life, what would be the expected
average growth for an infant who weighs 8 lb at birth?
(A) 7 lb at 2 weeks, 14 lb at 6 months, 21 lb at 12 months
(B) 7 lb at 2 weeks, 21 lb at 4 months, 28 lb at 12 months
(C) 8 lb at 2 weeks, 16 lb at 4 months, 24 lb at 12 months
(D) 8 lb at 2 weeks, 24 lb at 6 months, 32 lb at 12 months
10. (C) During the first year of life, the average, expected
increase in weight of a full-term infant is to regain the birth
weight by 2 weeks of age, double the birth weight by 4
months of age, and triple the birth weight by 1 year of age.

11. A 3-year-old child presents to the emergency department


(ED) with bruises on his body. His mother claims that her
son sustained these bruises when he tumbled down the stairs
3 days ago.Which of the following colors would you expect
the bruises to be if this occurred as stated?
(A) brown
(B) purple
(C) red
(D) yellow
11. (B) When evaluating children with physical injuries, the
major difficulty is distinguishing intentional injuries from
unintentional injuries. Inconsistencies between the stated
story and the injury are suspect. Discoloration caused by
healing bruises tends to follow a distinctive pattern.
On the first day, there is swelling without discoloration.
From day 1 through day 5, the bruise is purple in color. For
days 5 through 7, the bruise is green. Then, from day 7
through day 10, the bruise is yellow, followed by a brownish
color from day 10 to day 14.

12. An 8-month-old infant, whose parents elected not to


immunize, presents with a 5-day history of arunny nose in
late January. Then, over the past 3 days she has developed a
temperature of 101.2°F and vomiting (three times in 24
hours). This morning she developed watery, non-
bloody,nonmucous diarrhea. Which of the following is the
MOST likely causative organism for her illness?
(A) Clostridium difficile
(B) Giardia lamblia
(C) Shigella species
(D) rotavirus
12. (D) Rotavirus is one of the most important causes of
acute gastroenteritis in infants and young
children primarily 6 to 24 months of age. In the United
States, there are 65,000 to 70,000
hospitalizations and 200 deaths per annum. Peak incidences
occur in the fall and winter. Most
initial infections are characterized by diarrhea (watery,
nonbloody, nonmucous), fever, and vomiting. Nasal
congestion and coryza often precede the gastrointestinal
symptoms. Clostridium difficile produces a toxin that causes
a self-limited diarrhea in which symptoms characteristically
begin following the administration of antibiotics that reduce
normal bowel
flora. Giardia lamblia, a flagellated protozoa,
characteristically causes a mild diarrhea, with or without a
low-grade fever, anorexia, flatulence, and abdominal
cramps. It is not associated with vomiting nor upper
respiratory tract symptoms. Shigella gastroenteritis in young
children classically presents acutely with a high fever or
seizures along with vomiting followed by
bloody, mucoid, diarrheal stools.

13. In a 4-year-old female child who presents with “toeing


in,” which of the following is the likely etiology?
(A) femoral anteversion
(B) genu valgum
(C) genu varum
(D) tibial torsion
13. (A) “Toeing in” in children before the age of 2 is
typically due to tibial torsion; however, any “toeing in”
after the age of 2 to 3, is usually due to femoral anteversion.
The femur has more
internal rotation that results in the presentation Genu
varum is known as bowleg and genu
valgum is known as knock-kneed.

14. Which of the following is the first sign of puberty in a


normal male?
(A) appearance of axillary hair
(B) appearance of pubic hair
(C) deepening of the voice
(D) enlargement of the testes
14. (D) The first sign of pubertal development in boys is the
enlargement of testicular size and occurs at the mean age of
11.6 years. Genital stages accelerate before pubic hair
development, which occurs, on average, at 13.4 years of age.
The deepening of the voice and the development of chest and
axillary hair usually occurs in midpuberty or 2 years after
the growth of pubic hair.

15. Huntington disease has which of the following types of


genetic patterns of inheritance?
(A) autosomal dominant
(B) autosomal recessive
(C) X-linked dominant
(D) X-linked recessive
15. (A) Huntington disease is an autosomal dominant
hereditary disease. Its occurrence is between
1:5,000 and 1:20,000. It is caused by a defect on chromosome
4p16.3 that results in a repeat of
“CAG” in the “Huntington” protein gene.

16. A 9-year-old child, who was diagnosed with a viral upper


respiratory infection 2 weeks ago,returns to the clinic with a
complaint of a 2-day history of drooping of one side of her
mouth. She is afebrile with a blood pressure of 110/60 mm
Hg. Her physical examination reveals an inability to
completely close her left eye, inability to wrinkle her
forehead, and the drooping of her mouth on the left side.
Her smile is asymmetric. The remainder of her examination
is otherwise normal. Which of the following is the MOST
likely diagnosis?
(A) Bell's palsy
(B) botulism
(C) brainstem glioma
(D) Guillain–Barré syndrome
16. (A) Bell's palsy is the acquired peripheral facial
weakness (cranial nerve VII) of sudden onset and unknown
etiology. It often follows a viral illness with notable
improvement within 2 weeks and near complete recovery
within 2 months. Prednisone therapy may promote recovery
of facial strength. Guillain–Barré syndrome (acute
idiopathic polyneuritis) generally presents with symmetrical
weakness of the lower extremities, which may ascend rapidly
to the arms, trunk, and face. Nonspecific respiratory or
gastrointestinal symptoms may occur 5 to 14 days preceding
the
infection. Physical examination will yield symmetric flaccid
weakness, which is usually proximal in distribution. Rarely,
there is cranial nerve (III–VI, IX–XI) involvement. Botulism
is most often caused by the ingestion of food containing the
Clostridium botulinum toxin or rarely
from an infected wound. Children will present with blurred
or double vision, ptosis, or choking.Physical findings include
a weak swallow paralysis of accommodation and eye
movements. In this case, there was not a history of food
ingestion or wound infection to support this diagnosis.
Children with a brain stem tumor may present with facial
and extraocular muscle palsies,hemiparesis, gait
disturbances, and hydrocephalus (25%). Changes in
personality such as lethargy, irritability, and aggressive
behavior are particularly common findings. Speech and
swallowing difficulties are not unusual. Later in the illness,
patients will develop vomiting and
headaches.
17. A 5-year-old child presents to the office for a school
physical examination. His medical history is unremarkable,
including normal growth and development. His physical
examination is normal except for a grade II/VI high-pitched,
vibratory, systolic ejection murmur heard best at the left
lower sternal border with radiation to the apex. When the
child is in a supine position, the murmur is louder. Which of
the following murmurs is the MOST likely diagnosis?
(A) physiologic peripheral pulmonic stenosis murmur
(B) pulmonary ejection murmur
(C) Still's murmur
(D) venous hum
17. (C) Still's murmur is the most common innocent
murmur of early childhood and is usually appreciated in
children from 3 to 6 years of age. It is a grade I–III/VI early
systolic ejection murmur of musical or vibratory quality
heard best between the apex and the left lower sternal
border. It is loudest when the patient is in a supine position.
The murmur may diminish or disappear with inspiration,
during the Valsalva maneuver, or when the patient is
standing or seated. A physiologic peripheral pulmonic
stenosis murmur is a soft, short, high-pitched, grade I–II/VI
systolic ejection murmur. Typically, it is auscultated with
equal intensity at the left upper
sternal border, along the back, and in both axillae. It is
usually found in newborns and generally disappears by 3 to
6 months of age. A pulmonary ejection murmur is the most
common innocent murmur of later childhood and is usually
seen in children 8 to 14 years of age. It is a soft, early to
midsystolic ejection, grade I–III/VI murmur heard best
along the left upper sternal border. It is louder when the
patient is supine or with increased cardiac output. It
diminishes with standing or during the Valsalva maneuver.
A venous hum is a continuous musical, grade I–II/VI
murmur heard
at the right or left superior infraclavicular area. The
murmur is obliterated when the patient is in a supine
position, with head rotation, and with compression of the
jugular vein. It is usually auscultated in children from 3 to 6
years of age.

18. Which of the following sleeping positions for a healthy


infant should be recommended to parents during
anticipatory guidance in order to reduce the risk for sudden
infant death syndrome?
(A) prone position
(B) seated position
(C) side position
(D) supine position
18. (D) Sudden infant death syndrome (SIDS) is defined as
the sudden, unexplained death of an apparently healthy
infant that is unexpected and not adequately explained by a
comprehensive medical history, a postmortem physical, and
investigation of the death scene. SIDS is a leading cause of
death in infants between the ages of 1 month and 1 year,
second only to congenital
anomalies. The exact etiology of SIDS is unclear. Prevention
of SIDS has become a focus of public health measures. In
1994, The American Academy of Pediatrics initiated a
campaign called “Back to Sleep,” which recommended
placing infants in the supine position for sleep.Following the
institution of this campaign in the United States, the annual
death rate decreased from 1 3 per 1 000 to 0.7 per 1,000.

19. A previously healthy 12-month-old infant has been


coughing and experiencing fever on and off for 2 months. He
was diagnosed and treated for pneumonia approximately 3
months ago. On physical examination, he is noted to be in no
acute respiratory distress; however, he is tachypneic
with bibasilar rales and scattered rhonchi. Which of the
following is the MOST likely diagnosis?
(A) bronchiectasis
(B) chronic bronchitis
(C) croup
(D) bronchopulmonary dysplasia
19. (A) Bronchiectasis, meaning “dilation of the bronchi,”
results from destruction of the airway and poor drainage,
often associated with cystic fibrosis, foreign body aspiration,
or an infection. It is uncommon in the general population.
The presentation may vary from a chronic productive cough
to recurrent pneumonia with or without hemoptysis.
Persistent rhonchi, rales, and decreased breath sounds are
noted over the affected atelectatic area. Croup is an
inflammatory disease of the larynx most frequently affecting
young children during the fall and early winter months.
Typically, there is an upper respiratory tract prodrome
followed by stridor and a “barky cough”in the absence of
drooling. Subglottic narrowing with a normal epiglottis is
diagnostic on alateral neck X-ray. The most common
pathogen is parainfluenza virus. Bronchopulmonary
dysplasia is most commonly seen in infants in the neonatal
intensive care unit. It is a chronic
condition seen in patients whose clinical course included
hyaline membrane disease. These infants typically need
oxygen for a few months as they grow and some need
permanent tracheostomy and ventilation for up to 2 years.
Chronic bronchitis falls into the chronic obstructive
pulmonary disease category typically seen in older adults
and does not typically present with acute symptoms.
20. The second most common etiologic agent of otitis externa
is which of the following?
(A) Staphylococcus aureus
(B) Corynebacterium
(C) Anaerobes
(D) Streptococcus pyogenes
20. (A) Otitis externa is an infection of the auditory canal.
The most common etiologic agent is pseudomonas. However,
Staphylococcus aureus is a very close second and therefore
antibiotic treatment should provide coverage for both
organisms. Corynebacterium is part of the normal flora of
the auditory canal and does not typically cause infections.
Streptococcus pyogenes is the
most common cause of acute bacterial pharyngitis.

21. At 12 hours of age, a physical examination is performed


on a neonate who has intrauterine growth retardation. He is
noted to have microcephaly, jaundice, and
hepatosplenomegaly. Which of the following is the MOST
likely congenital viral infection in this neonate?
(A) cytomegalovirus
(B) herpes simplex virus
(C) rubella
(D) syphilis
21. (A) Cytomegalovirus (CMV) is one of the congenital
neonatal TORCH infections (toxoplasmosis, o ther [syphilis,
varicella-zoster, and parvovirus in this list], r ubella,
cytomegalovirus, and h erpes simplex/h epatitis/H IV). CMV
is the most common congenital infection. The disease-
specific manifestations for CMV include microcephaly with
periventricular calcifications, neonatal jaundice with direct
hyperbilirubinemia, and
hepatosplenomegaly. Other associated manifestations
include intrauterine growth retardation,thrombocytopenia,
and purpura. Disease-specific manifestations for herpes
simplex virus include skin/eye/mouth vesicles, encephalitis,
respiratory distress, and sepsis. Disease-specific
manifestations of rubella include congenital heart lesions
(patent ductus arteriosus, pulmonary artery stenosis, aortic
stenosis, ventricular defects), thrombocytopenic purpura
characterized by purple macular lesions (“blueberry
muffin” appearance), cataracts, retinopathy, and
sensorineural deafness. Disease-specific manifestations of
syphilis include mucocutaneous lesions (snuffles), periostitis,
osteochondritis, and hemolytic anemia. Often, these babies
are stillborn. Syphilis is caused by a spirochete, Treponema
pallidum, not a virus.

22. A previously healthy, 5-month-old infant is admitted to


the hospital due to lethargy progressing to
semiconsciousness. The physical examination reveals a
depressed mental status and bilateral retinal hemorrhages.
Which of the following is the MOST likely diagnosis?
(A) child abuse
(B) retinitis pigmentosa
(C) Reye syndrome
(D) viral encephalitis
22. (A) Approximately 40% of children who have been
physically abused showed evidence of ocular trauma. Retinal
hemorrhages are the most frequent ocular finding that
result from violent shaking. This form of child abuse is
termed shaken baby syndrome. The finding of retinal
hemorrhages in an infant without an appropriate medical
condition (eg, clotting disorder,leukemia) should raise
concerns about nonaccidental trauma. Some of the most
common presenting complaints of infants with shaken baby
syndrome are lethargy, coma, seizures,vomiting, and
respiratory distress. Retinal hemorrhages are not associated
with retinitis
pigmentosa, retinoblastoma, Reye syndrome, or viral
encephalitis. With Reye syndrome, an antecedent viral
illness is followed by vomiting and progressive lethargy. On
examination, there is usually fever, tachypnea, and stupor.
Laboratory hallmarks include elevated serum hepatocellular
enzyme assays and elevated serum ammonia. Retinitis
pigmentosa is a progressive
retinal degeneration and is characterized by pigmentary
changes, optic atrophy, and progressive impairment of
visual function. The presenting clinical manifestation is
usually an impairment of dark adaptation or night vision.
Clinical manifestations of viral encephalitis vary in severity
depending upon the etiologic organism (eg, cytomegalovirus,
mumps, echovirus). Some children will have mild symptoms
lapsing into a coma leading to death, whereas others are
febrile, with convulsions and hallucinations followed by full
recovery.

23. A 2-year-old child presents to the emergency department


via ambulance due to a seizure lasting approximately 2
minutes with jerking and somnolence. En route in the
ambulance her vital signs are: temperature 39°C rectal;
pulse 120/min; respirations 32/min; blood pressure 110/64
mm Hg.Upon further questioning, her mother claimed she
had a runny nose yesterday. On physical examination, she is
sleepy but arousable with negative Kernig and Brudzinski
signs. Which of the following seizures is the MOST likely
diagnosis?
(A) absence seizure
(B) complex partial seizure
(C) febrile seizure
(D) simple partial seizure
23. (C) A febrile seizure is a brief (less than 15 minutes),
generalized, symmetric, tonic–clonic seizure associated with
a febrile illness (temperature greater than 38.8°C) without
any central nervous system infection or neurologic cause. An
absence (petit mal) seizure is a brief (2 to 25
seconds) loss of consciousness that can occur multiple times
per day. There is no loss of tone,and frequently the only
observable behaviors are staring or minor movements such
as lip smacking and semipurposeful movements of the
hands. There is no postictal period. Complex partial seizures
(psychomotor) have varied symptoms including alterations
in consciousness,
unresponsiveness, and repetitive complex motor activities
that are purposeless. Often, at the beginning of the attack,
there is a psychoillusory phenomenon such as hallucinations,
visual distortions, visceral sensations, or feelings of intense
emotions. Simple partial seizures include
focal motor, adversive, and somatosensory seizures.
Manifestations of these seizures are varied including
hallucinatory, psychoillusory, or complex emotional
phenomena. Children will interact normally with their
environment, with the exception of those limitations imposed
by the seizure.
Following the seizure (minutes to hours), there may be
transient paralysis of the affected body
part.
24. A 6-month-old uncircumcised male infant presents with
a 2-day history of a fever (39.6°C rectal today), vomiting,
and poor feeding. Urinalysis of a catheterized specimen
reveals 50 to 100 white blood cells per high-power field and
moderate bacteria. Two days later, the urine culture results
are available. Which of the following is the MOST common
pathogen responsible for this
infant's first urinary tract infection?
(A) Enterococcus
(B) Escherichia coli .
(C) Klebsiella
(D) Staphylococcus saprophyticus
24. (B) Urinary tract infections (UTIs) are one of the most
common infections in children. Clinical features of a UTI
vary depending upon the age and sex of the child. In
newborns, the most common symptom is failure to thrive
associated with poor feeding, diarrhea, and vomiting. In
infants, the symptoms may be relatively nonspecific, such as
poor feeding, failure to gain weight,vomiting, fever, strong-
smelling urine, and irritability. As children grow older, the
initial signs and symptoms become more specific to the
urinary tract. In early infancy, males are two times more
likely than girls to have a UTI. Also, uncircumcised males
are 10 times more likely to be
affected than circumcised males. Escherichia coli is the most
common pathogen for the first UTI (80%) and of recurrent
infections (75%). Other organisms that cause infections
include Pseudomonas aeruginosa, Proteus, Enterobacter,
Klebsiella, and Enterococcus. An infection with
Staphylococcus saprophyticus, a coagulase-negative
staphylococcus, is primarily seen in
adolescents with a UTI.
25. A 6-month-old infant presents to the emergency
department with a 2-day history of vomiting and diarrhea.
Upon physical examination, she appears to be intermittently
irritable and restless with minimal tearing when crying and
dry mucous membranes. Her capillary refill is 2 to 3
seconds.Her urine sodium is less than 20 mEq/L and mildly
oliguric. On the basis of these clinical manifestations, what is
the magnitude of her dehydration?
(A) less than 3%
(B) approaching 3% to 5% (mild)
(C) approaching 6% to 10% (moderate)
(D) approaching 11% to 15% (severe)
25. (B) Dehydration is a common pathophysiologic alteration
in fluid and electrolyte balance in children. Children are at
an increased risk for dehydration because of their decreased
oral intake,especially when ill, and their higher ratio of
surface area to body weight, promoting significant
evaporative losses. Important clinical features to estimate
the degree of dehydration include postural blood pressure,
changes in heart rate, capillary refill time, skin turgor and
color, lack of tears, lack of external jugular venous filling
when supine, sunken fontanel (if present), and altered
mental status. This infant was estimated to have mild
dehydration (3% to 5% decrease in
body weight) with decreased tears, slightly longer capillary
refill time (2 to 3 seconds), and intermittent irritability and
restlessness. Severe dehydration (11% to 15% decrease in
body weight) manifests as markedly decreased skin turgor
with parched or mottled mucous membranes, absence of
tears, tachycardia, capillary refill greater than 4 seconds,
hypotension,
circulatory collapse, and anuria. Moderate dehydration (6%
to 10% decrease in body weight) manifests as decreased skin
turgor; dry mucous membranes; decreased tearing;
oliguria; and normal pulse, blood pressure, and perfusion.
26. A previously well, 15-month-old baby boy is brought to
the emergency department in the middle of the night with
increased irritability and severe paroxysmal colicky
abdominal pain followed by vomiting. On physical
examination, a tubular mass is palpated in the abdomen.
The rectal
examination reveals bloody mucus. Which of the following is
the MOST likely diagnosis?
(A) appendicitis
(B) infectious enteritis
(C) intussusception.
(D) pyloric stenosis
26. (C) Intussusception is the most common cause of
intestinal obstruction between 3 months and 6years of age. It
is twice as common in males than females. It is caused by
intestinal invagination,usually around the ileocecal valve.
The classic presentation is intermittent severe colicky
abdominal pain with legs drawn up, followed by periods of
comfort or falling asleep. Vomiting usually occurs in the
early phase, which later becomes bilious. A passage of blood
and mucus in the stool (“currant jelly stools”) occurs in
60% of the cases. Palpation of the abdomen usually
reveals a sausage-shaped mass in the right upper quadrant.
The classic presentation of pyloric stenosis is in first-born
males of 3 to 6 weeks of age, presenting with nonbilious
projectile vomiting leading to dehydration with
hypochloremia, hypokalemia, and metabolic alkalosis.
Afirm, movable, 2-cm olive-shaped mass (“olive”) is
palpable superior and to the right of the
umbilicus in the midepigastrium. In addition, peristaltic
waves may be visible on the physical examination. The
classic presentation of appendicitis presents with a period of
anorexia followed by steady periumbilical pain shifting to
the right lower quadrant; nausea and vomiting is followed
by a low-grade fever. Diarrhea (nonbloody and nonmucous),
if it occurs, is infrequent. Peritoneal signs are present. The
incidence increases with age and peaks during
adolescence. Infective enteritis usually begins with emesis
followed by crampy abdominal pain of hyperperistalsis. This
sequence of symptoms with emesis preceding pain is an
important factor in distinguishing it from intussusception.
Masses are not palpated with infective enteritis.

27. Which of the following daily maintenance fluid


requirements is the closest approximation for a24-kg child
who is refusing to eat?
(A) 1,080 mL
(B) 1,200 mL
(C) 1,580 mL.
(D) 2,000 mL

27. (C) Dehydration is a common pathophysiologic


alteration in fluid balance in children. The body has a
maintenance fluid requirement to replace daily normal
losses that occur through the skin,kidney, intestines, and
respiratory tract. The following formula can be used to
calculate the usual
amount of fluid a healthy child requires by mouth to
maintain hydration:
100 mL/kg for the first 10 kg of body weight
50 mL/kg for the next 10 kg of body weight
20 mL/kg for the weights above 20 kg
For this question, a 24-kg child would require:
100 mL/kg × 10 kg = 1,000 mL for the first 10 kg
50 mL/kg × 10 kg = 500 mL for the next 10 kg
20 mL/kg × 4 kg = 80 mL for the next 4 kg
Total = 1,580 mL 24 kg
28. Which of the following is the recommended treatment
for a 4-year-old child with presumed bacterial meningitis?
(A) cefotaxime or ceftriaxone plus ampicillin
(B) cefotaxime or ceftriaxone plus vancomycin.
(C) gentamicin plus ampicillin
(D) ampicillin plus chloramphenicol
28. (B) The most common etiologic organisms for bacterial
meningitis in children are Spneumoniae, N Meningitidis, and
H influenzae. Because of an increase in resistant
Spneumoniae, coverage with vancomycin and a third-
generation cephalosporin such as cefotaxime or ceftriaxone
is needed for best coverage. Gentamicin can be used but, as
with all aminoglycosides, caution is needed regarding
toxicity. Ampicillin, rifampin, and chloramphenicol are
alternative treatments if necessary.

29. At a 2-month-old well-child checkup, a female infant is


noted to have the following physical findings: widely open
anterior and posterior fontanels, large protruding tongue,
coarse facial features, low-set hair line, and an umbilical
hernia. In the newborn period, there was aprolongation of
physiologic icterus. The results of the newborn screening test
are abnormal.
Which of the following is the MOST likely diagnosis?
(A) congenital adrenal hyperplasia
(B) congenital hypothyroidism.
(C) Crigler–Najjar syndrome
(D) galactosemia
29. (B) Congenital hypothyroidism is one of the most
common disorders tested for in newborn screening tests,
revealing an elevated TSH (thyroid stimulating hormone)
and a decreased T4(thyroxine). Symptoms suggestive of
congenital hypothyroidism in the neonate include hypotonia,
coarse facial features, hirsute forehead, large fontanels
(anterior and posterior),
widely open sutures, umbilical hernia, protruding/large
tongue, hoarse cry, distended abdomen,and prolonged
jaundice. Signs of congenital hypothyroidism include
lethargy or hypoactivity,poor feeding, constipation, mottling,
and hypothermia. Congenital adrenal hyperplasia (CAH) is
not universally screened for in the newborn screening test,
as it is included in only 14 of the 50 states. In females with
CAH, there may be virilization with abnormalities of the
external genitalia varying from mild enlargement of the
clitoris to complete fusion of the labioscrotal folds. Signs of
adrenal insufficiency (salt loss) may present in the first few
days of life. Crigler–Najjar
syndrome is not one of the disorders tested for in the
standard newborn screening tests. It is an inherited disease
producing congenital nonobstructive, nonhemolytic,
unconjugated severe hyperbilirubinemia. The physical
findings in this infant do not correlate with Crigler–Najjar
syndrome. Galactosemia is tested for in the newborn
screening test in nearly all 50 states. The
infant may have symptoms of cataract, hepatomegaly, and
prolonged jaundice. Often, these
neonates have Escherichia coli sepsis, leading to death in the
first 2 weeks of life if not treated
promptly.

30. Within hours of birth, a healthy infant is noted to have a


superficial swelling over the right occipitoparietal region
that extends across the suture line. Which of the following
conditions is it MOST likely to be?
(A) caput succedaneum
(B) cephalohematoma
(C) craniotabes
(D) subgaleal hemorrhage
30. (A) Caput succedaneum is a result of fluid and blood
accumulation in the occipitoparietal region of the newborn's
scalp due to the vacuum effect of membrane rupture. A
cephalohematoma is afirm, tense external swelling of the
cranium that does not extend across suture lines because it is
limited to the surface of one cranial bone. It occurs most
often in the parietal area. This subperiosteal hemorrhage
usually is not present at birth, but develops within the first
24 hours of life. Craniotabes is a condition caused by the
osteoporosis of the outer table of the involved membranous
bone, generally over the temporoparietal or parietooccipital
areas, creating a “pingpong ball” sensation when gentle
pressure is applied. A subgaleal hemorrhage is a firm,
fluctuant external swelling of the cranium that does extend
across suture lines and increases in size over time.

31. A 2-year-old male child is brought to the emergency


department by his mother with a sudden onset of choking,
gagging, coughing, and wheezing. Vital signs are
temperature 37°C ; pulse 120/min; and respirations
28/min. The physical examination reveals decreased breath
sounds
over the right lower lobe with inspiratory rhonchi and
localized expiratory wheezing. The chest X-ray reveals
normal inspiratory views but expiratory views show
localized hyperinflation with mediastinal shift to the left.
Which of the following is the MOST likely diagnosis?
(A) asthma
(B) epiglottitis
(C) foreign body aspiration.
(D) pulmonary embolism
31. (C) Foreign body aspiration into the respiratory tract is
associated with an acute choking or coughing episode with
expiratory wheezing (indicative of a lower airway
obstruction) in children aged 6 months to 4 years of age.
Often, there is a history of the child playing with small toys
that are commonly aspirated. Asymmetrical physical
findings of decreased breath sounds and localized wheezing
are present with foreign body aspiration. A positive forced
expiratory chest X-ray shows a mediastinal shift away from
the affected side. Radiolucent foreign bodies such as plastic
toys may not appear on an X-ray, but there will be evidence
of this mediastinal
shift. Asthma is generally characterized by wheezing, but it
is not unilateral nor is it of sudden onset. A chest X-ray
reveals bilateral hyperinflation with flattening of the
diaphragm. Epiglottitis is a life-threatening upper airway
obstructive condition that presents with a sudden onset of
fever, dysphagia, drooling, and inspiratory retractions with
stridor. A lateral neck X-ray reveals an enlarged, indistinct
epiglottis (“thumb sign”); however, the chest X-ray is
normal. Pulmonary embolism, rare in children, presents
clinically with acute dyspnea, tachypnea, and
tachycardia.There may be mild hypoxemia, rales, and focal
wheezing. Chest X-rays may be normal, or there
may be a peripheral infiltrate, small pleural effusion, or
elevated hemidiaphragm.

32. A 16-year-old girl is brought to the emergency


department by ambulance after reportedly ingesting “a
bottle of aspirin.” Vital signs are temperature 37.8°C
oral; pulse 94/min;respirations 30/min; blood pressure
100/68 mm Hg. What would you expect the blood gases to
show that would confirm she had swallowed the aspirin?
(A) anion gap metabolic acidosis with respiratory acidosis
(B) nonanion gap metabolic acidosis with respiratory alkalosis
(C) anion gap metabolic acidosis with respiratory alkalosis.
(D) nonanion gap metabolic acidosis with respiratory acidosis
32. (C) An acute salicylate overdose (greater than 150
mg/kg) will produce symptoms of salicylate intoxication.
Chronic salicylate intoxication occurs with ingestion of
greater than 100 mg/kg/day for at least 2 days. Salicylates
affect most organ systems, leading to various metabolic
abnormalities. Because salicylates are a gastric irritant,
symptoms of vomiting and diarrhea
occur soon after the overdose, which may contribute to the
development of dehydration.Salicylates stimulate the
respiratory center leading to hyperventilation and
hyperpnea resulting in respiratory alkalosis and
compensatory alkaluria. A characteristic feature of
salicylate
intoxication is the coexistence of a respiratory alkalosis with
a widened anion gap metabolic acidosis.

33. A 16-year-old high school boy presents to the emergency


department 4 hours after sustaining an abrasion to his knee
after a fall while rollerblading on the school playground. His
school immunization record reveals that his last diphtheria,
tetanus, and pertussis (DTaP) booster was administered at
age 4. In this situation, which of the following is the MOST
appropriate plan?
(A) administer tetanus toxoid
(B) administer adult tetanus and diphtheria toxoid (Td)
(C) administer diphtheria, tetanus toxoid, and acellular pertussis
(Tdap) vaccine
(D) administer tetanus immune globulin
33. (C) Generalized tetanus (lockjaw) is a neurologic disease
caused by Clostridium tetani. Although any open wound is a
potential source for contamination with C tetani, those with
dirt,soil, feces, or saliva are at increased risk. Tetanusprone
wounds contain devitalized tissue,
especially those caused by punctures, frostbite, crush injury,
or burns. Recommendations for tetanus prophylaxis in a
child with a laceration or abrasion depend upon the number
of previous vaccinations, occurrence of last booster, type of
wound (clean or tetanus-prone), and age of child. In this
case, the patient is older than 7 years and had all of his
previous immunizations;
however, his most recent booster was greater than 10 years
ago. Thus, he should receive an adult-type diphtheria and
tetanus toxoid with acellular pertussis. In most cases, when
tetanus toxoid is required for wound prophylaxis in a child
older than 7 years, the Td instead of tetanus
toxoid alone is recommended so that diphtheria immunity is
maintained. If tetanus immunization is not up to date at the
time of wound treatment, then the immunization series
should be completed according to the primary immunization
schedule. If a child is younger than 7 years, then the
diphtheria, tetanus, acellular pertussis (DTaP) booster is
indicated, unless there is acontraindication for pertussis, in
which case the diphtheria and tetanus (DT) booster should
be administered. Tetanus immune globulin (TIG) is
recommended for treatment of tetanus. Under special
circumstances, a patient infected with the human
immunodeficiency virus (HIV) with a
tetanusprone wound should also receive TIG in addition to
the prophylactic vaccine.

34. Which of the following newborn reflexes should still be


present at the 9-month check-up?
(A) Galant reflex
(B) Landau reflex
(C) rooting reflex
(D) parachute reflex
34. (D) Normally, primitive reflexes are present at birth and
should not persist beyond the age of 6months. However, the
parachute reflex is a postural response that normally
appears around 7months of age to coincide with volitional
movement and persists for life. It occurs when an infant
is held prone by the waist over a surface and lowered with
the head downward and extends the arms and legs as a form
of protection. The rooting reflex occurs when the cheek is
stroked on the infant and they turn his/her head to feed.
Galant and Landau reflex disappear by the age of 2
months (trunk incurvation upon stroking the back) and 6
months (the baby lifts head and straightens spine upon being
held prone), respectively

35. A 3-day-old infant has bilateral copious, yellow-green eye


discharge and conjunctival inflammation. A Gram stain of
this discharge reveals gram-negative intracellular
diplococci.Which of the following antibiotics is the drug of
choice for this infection?
(A) ceftriaxone
(B) cephalexin
(C) erythromycin
(D) gentamicin
35. (A) Gonococcal ophthalmia neonatorum presents as a
unilateral or bilateral serosanguineous discharge and then
within 24 hours the discharge becomes mucopurulent,
followed by conjunctival injection and edema of the eyelids.
The usual incubation period for Neiserria gonorrhea is 2 to 5
days; however, the infection may be present at birth or
delayed greater than 5
days if there has been instillation of silver nitrate
prophylaxis. A presumptive diagnosis is made by the
demonstration of gram-negative intracellular diplococci on
Gram stain. Definitive diagnosis is made by culture.
Following a positive Gram stain and pending culture
results,treatment should be promptly initiated with
ceftriaxone (50 mg/kg/24 hours IV or IM for one dose
not to exceed 125 mg), a third-generation cephalosporin with
good coverage for gram-negative bacteria. An alternate drug
is cefotaxime (100 mg/kg/24 hours IV or IM every 12 hours
for 7days or 100 mg/kg as a single dose), which is also a
third-generation cephalosporin. Although erythromycin
drops (0.5%) are used prophylactically for N gonorrhea, this
is not an effective
treatment. Gentamicin would be used for Pseudomonas, and
Chlamydia is treated with erythromycin. Cephalexin as a
first-generation cephalosporin does not have coverage for
gramnegative
bacteria.

36. A 10-year-old boy presents to the office, complaining of a


painful, swollen area along his right jaw and neck. On
physical examination, he is noted to be febrile and has
diffuse tenderness over the right parotid gland. His
laboratory tests include an elevated serum amylase. His
parents elected not to vaccinate him. In this patient, based
on the most likely diagnosis, which of the following is a
complication of his disease?
(A) hepatitis
(B) nerve deafness.
(C) pneumonitis
(D) testicular torsion
36. (B) The most likely diagnosis in this patient is mumps. It
is endemic in most unvaccinated populations. The onset is
characterized by pain and swelling in one or both parotid
glands. The pain can be exacerbated by tasting sour liquids
such as lemon juice. An elevated serum amylase level is
common and coincides with the parotid swelling. Unilateral,
rarely bilateral, nerve
deafness is a complication of mumps that may be transient
or permanent. Other complications include
meningoencephalomyelitis, orchitis, epididymitis,
pancreatitis, arthritis, and rarely thyroiditis and
myocarditis.

37. Which of the following physical examination findings in


a newborn infant should cause the clinician to suspect a
genetic disorder?
(A) café au lait spots
(B) subconjunctival hemorrhages
(C) miliaria
(D) vernix caseosa
37. (A) Café au lait spots are brown macules that may be
found on any part of the body. The presentation of six or
more spots greater than 1.5 cm is a sign of
neurofibromatosis, a genetic disorder that results in
neurofibromas that can develop in any organ/tissue system.
Miliaria are blocked sweat gland ducts that are commonly
found on the face, scalp, or intertriginous areas.
Vernix caseosa is a normal finding in newborns and is a
whitish, greasy layering on the body— it decreases as an
infant comes to full term. Subconjunctival hemorrhages are
a common finding in infants secondary to birth trauma.

38. The newborn examination at 1 minute shows a heart rate


of 120 bpm, strong cry, some flexion in the upper
extremities, sneezing with nasal catheter suction, and bluish
hands and feet; but the remainder of the body is pink. What
is the Apgar score?
(A) 7
(B) 8.
(C) 9
(D) 10
38. (B) The Apgar score assesses the newborn at 1-minute
and 5-minute intervals to determine the need for
resuscitative care. The infant is evaluated by heart rate,
respiratory effort, muscle tone,response to catheter in
nostril, and color, and each is rated on a scale of 0, 1, or 2 for
a total
score of 10. The heart rate is scaled 0–2 for absent, less than
100 bpm (slow), and greater than 100 bpm; respiratory
effort of absent, slow/irregular, and good crying. Muscle
tone scale (0–2) consists of limp, some flexion, and active
motion; response to catheter stimulation (0–2) is scaled no
response, grimace, and cough/sneeze. Finally, color is scored
0–2 for blue/pale, body
pink with blue extremities, and completely pink.

39. Which of the following is the most common congenital


heart malformation?
(A) atrial septal defect
(B) tetralogy of Fallot
(C) ventricular septal defect
(D) transposition of the great vessels
39. (C) Ventricular septal defect, a hole between the two
ventricles, can be cyanotic or acyanotic based on the size of
the defect, and accounts for 30% of cases of congenital heart
disease. Atrial septal defect occurs in approximately 10% of
congenital heart disease cases. Transposition of great vessels
is an embryonic malformation resulting in the aorta arising
from the right ventricle
and the pulmonary artery arising from the left ventricle. It is
responsible for about 10% of all congenital malformations.
Tetralogy of Fallot, consisting of a ventricular septal
defect,overriding aorta, pulmonic/subpulmonic stenosis, and
right ventricular hypertrophy, accounts for 10% of
congential heart disease.

40. A 5-year-old male child presents to the office for his


kindergarten physical examination.Assuming that the
patient's immunizations have been up to date, which of the
following are the immunizations that the patient should
receive at the end of today's visit?
(A) hepatitis B, inactivated poliovirus (IPV), diphtheria, tetanus,
acellular pertussis (DTaP),
measles, mumps, rubella (MMR), varicella
(B) IPV, DTaP, MMR, pneumococcal (PCV)
(C) IPV, DTaP, MMR, Haemophilus influenzae type B (Hib)
(D) DTaP, IPV, MMR, varicella.
40. (D) The immunization schedule is developed biannually
by the Centers for Disease Control and Prevention.
Assuming that the child has had the appropriate
immunizations at the regularly scheduled examinations, the
recommended immunizations at the 4- to 6-year-old range
are the
DTaP (diphtheria, tetanus, acellular pertussis), IPV
(inactivated polio), and the MMR (measles,mumps, and
rubella). The hepatitis series should have been completed by
the age of 6 months and the Haemophilus influenzae type B
(Hib) should be completed by the age of 12 to 15
months.Varicella is given from 12 to 18 months and again
from 4 to 6 years; the PCV (pneumococcal)
should be finished by 12 to 15 months.

41. Which of the following is an absolute contraindication to


breastfeeding?
(A) tuberculosis of the mother
(B) methadone treatment (20 mg/d)
(C) maternal smoking
(D) infant with cystic fibrosis
41. (A) There are only two known absolute contraindications
to breastfeeding: tuberculosis of the mother and
galactosemia of the infant. The highly contagious nature of
tuberculosis makes the risk greater than the benefit, and
infants with galactosemia are unable to digest any lactose
due to an
enzyme deficiency. Infants of mothers in a methadone
program may be breastfed as long as the mother's dose is
less than 40 mg. While nicotine is transmitted in breast milk
and is therefore strongly discouraged, it is not an absolute
contraindication. As long as a breastfed infant with cystic
fibrosis is maintaining normal growth with supplemented
pancreatic enzymes,
breastfeeding is encouraged.

42. A 4-month-old infant presents to the office for her “well-


check.” The parents state that they have no concerns and think
she is doing well. She is being breastfed every 4 to 6 hours and
has four wet diapers a day and two dirty diapers a day. Her birth
weight was 7 lb 7 oz (50th percentile);she missed her 2-month
appointment and at today's visit her weight is 11 lb 5 oz (5th
percentile).
The clinician, however, is very concerned and diagnoses the
infant with which of the following?
(A) dwarfism
(B) growth deficiency
(C) lactose intolerance
(D) Beckwith–Wiedemann syndrome
42. (B) Failure to thrive is diagnosed in infants younger than
the age of 6 months with a decrease in growth velocity that
results in a decrease in two major percentile lines on the
growth chart. In the case of this patient, she was initially in
the 50th percentile and crossed the 25th and 10th percentile
and fell into the 5th percentile. Failure to thrive is also
known as growth deficiency
and may also be diagnosed if the child is younger than 6
months and has not grown for two consecutive months or if
a child is older than 6 months and has not grown for 3
consecutive months. Growth hormone deficiency/dwarfism
may present with decreased growth velocity later in
childhood; the drop in percentiles is grossly below the 5th
percentile mark. Lactose intolerance presents with varying
gastrointestinal symptoms without the marked decrease in
weight. Beckwith–Wiedemann syndrome consists of
macrosomia, macroglossia, and omphalocele and they are at
increased risk for malignancies, hypoglycemia, and
dysmorphism (usually of the ears).

43. During influenza season, a 15-year-old boy presents to


the emergency department, unresponsive.The parents state
that when they tried to wake him up in the morning he
would not get up and was barely breathing. They deny any
drug or alcohol use and state that he just had some cold
symptoms the past few days. A spinal tap shows decreased
glucose, increased pressure, and increased proteins, but
there were no cells found. The rest of the blood work shows
elevated liver enzymes, but normal serum bilirubin and
alkaline phosphatase. A liver biopsy demonstrates
microvesicular steatosis without glycogen and large
mitochondria. Which of the following is the
best treatment for this patient?
(A) high-dose steroids
(B) broad-spectrum antibiotics until the cultures come back
(C) supportive treatment, including maintenance fluids and
hyperventilation
(D) liver transplant
43. (C) This patient has presented with classical findings of
Reye syndrome—upper respiratory infection followed by
unresponsiveness. Reye syndrome is usually preceded by an
upper respiratory tract illness, which progresses into
vomiting, strange behavior, stupor, and coma.Liver
function tests (LFTs) will be markedly elevated (without
jaundice); however, the serum bilirubin and alkaline
phosphatase are normal. Unresponsive patients who have a
spinal tap will show no cells in the CSF and glucose may be
low with increased CSF pressure. If arterial blood gases are
ordered, they will show a mild respiratory alkalosis and
metabolic acidosis. A liver biopsy will show little
inflammatory changes with diffuse microvesicular steatosis
and absent
glycogen from the hepatocytes. The mitochondria of the
hepatocytes are large and polymorphic with decreased
matriceal density. Treatment for patients with Reye
syndrome is largely supportive—specifically decreasing
cerebral edema. There is no place for antibiotics or steroids.
The liver will fully recover if the cerebral edema is
decreased.

44. A 5-year-old female child presents for her kindergarten


physical examination and her mother mentions that she
thinks she looks a bit yellow to her. The clinician notes
diffuse jaundice,icterus, and Kayser–Fleischer rings. Which
of the following is the treatment of choice for this patient?
(A) alpha-interferon therapy
(B) D-penicillamine therapy
(C) methylprednisolone
(D) protease inhibitor therapy
44. (B) Wilson disease is a result in a genetic mutation on
chromosome 13 that causes decreased bile excretion of
copper and results in accumulation of copper by the liver,
specifically the ceruloplasmin. The build-up of copper causes
damage to the liver, basal ganglia, and other tissues.
Physical examination shows jaundice, hepatosplenomegaly,
Kayser–Fleischer rings (abrown band at the junction of the
iris and cornea under slit-lamp), and neurologic
manifestations later in the disease process. Laboratory tests
show marked decrease in ceruloplasmin of the liver, anemia,
hemolysis, and severely elevated bilirubin with decrease
alkaline phosphatase.Urinalysis shows severe elevation in
copper excretion, glycosuria, and aminoaciduria. Liver
biopsy is conclusive with evidence of copper greater than 250
μg/g of dry tissue. Treatment requires copper chelation
with D-penicillamine or trientine hydrochloride. Liver
transplant may be required with noncompliance and in
acute fulminant disease. Copper chelation is continued for
life with the addition of zinc (decrease copper absorption)
and vitamin B6 (decrease optic neuritis). Genetic screening
of siblings and future children should be strongly
encouraged. Alphainterferon therapy is mainly used to treat
hepatitis patients. There is no place for steroids in therapy
and protease inhibitors are antiviral medications that are
typically used in HIV patients.

45. A 24-month-old infant presents for his routine physical


examination. The parents state that he has been following all
of his developmental milestones. On examination, the
clinician hears a grade II/VI murmur along the left sternal
border, which radiates into the left axilla and the left side of
the back. The child also has decreased femoral pulses
bilaterally. The clinician orders a chest Xray.Which of the
following is the expected finding on X-ray based on the
presentation?
(A) notching or scalloping of the ribs
(B) boot-shaped heart—right ventricular hypertrophy
(C) “egg on string”—narrowed mediastinum
(D) absence of the main pulmonary artery
45. (A) The patient's presentation is consistent with findings
of coarctation of the aorta. The pathognomonic finding in
coarctation is decreased or absent femoral pulses. However,
the majority of children show no signs of coarctation in
infancy and develop signs and symptoms during childhood,
most notably unequal pulses and blood pressure between
arms and legs (arms
lower than legs). In addition, a grade II/VI ejection murmur
is heard at the aortic area and left sternal border that
radiates into the left axilla and left back. Chest X-ray shows
a normal-sized heart, a prominent aorta, indents at the level
of the coarctation, and a dilated poststenotic segment
resulting in the “figure 3” sign. Scalloping or notching of
the ribs is due to enlargement of the intercostal arteries.
Echocardiography is used to directly visualize the
coarctation and estimate the obstruction. Asymptomatic
infants and children are encouraged to have corrective
surgery prior to age 5, after which they are at increased risk
for myocardial dysfunction and hypertension, and require
exercise testing prior to participation in aerobic activities.
The bootshaped heart is seen in patients with tetralogy of
Fallot secondary to right ventricular hypertrophy; the
narrowed mediastinum finding with “egg on a string” is
typically seen in patients with transposition of the great
vessels.

46. An 8-year-old female child presents to the emergency


department with her parents. They state she has been
coughing all night the past few nights, to the point she
sounds like she is choking. On examination, the clinician
notes mild retractions at rest. Retractions worsen with the
lung examination and there is diffuse stridor on
auscultation. Pulse oximetry is 92% on room air and the
child is afebrile. Which of the following is the recommended
treatment for this patient?
(A) supportive care only—mist therapy
(B) IV (intravenous) antibiotics, with gram-negative coverage
(C) IM (intramuscular) dexamethasone
(D) nebulized racemic epinephrine and oral dexamethasone

46. (D) Viral croup usually presents with cough that may
sound like a dog or a seal barking. The
patients are usually afebrile and also present with stridor
either at rest, in severe cases, or when
agitated, in mild cases. In addition, the patient may be
cyanotic and have retractions and acute
shortness of breath. Radiologic examination of the neck
shows subglottic narrowing with a
normal epiglottis, “steeple sign.” However, X-rays are
usually not indicated in patients with the
common presenting symptoms. Treatment for viral croup
is mainly symptomatic, especially in
mild cases consisting of oral hydration and mist therapy.
Severe cases (stridor at rest) call for
oxygen in patients who have desaturated, and nebulized
racemic epinephrine and
glucocorticoids. Dexamethasone as an intramuscular
injection or oral as a one time dose is
effective in alleviating symptoms, decreasing the need for
intubation, and decreasing hospital
stays. Inhaled budesonide is also effective in decreasing
hospital stays and improving symptoms,
but dexamethasone is more cost-effective. Patients who are
unable to be stabilized need airway
maintenance either by intubation with endotracheal tube
or by tracheostomy if intubation fails.
Because it is a self-limiting disorder, unless there is a
secondary infection most children recover
in a few days.

47. A neonate presents with meconium ileus that is


successfully unobstructed. The infant returns at her 4-month
appointment with signs of failure to thrive. Which of the
following is the most likely diagnosis for this patient?
(A) cystic fibrosis
(B) Wilson disease
(C) intussusception
(D) volvulus
47. (A) Cystic fibrosis (CF) is a major cause of
gastrointestinal and pulmonary morbidity in children due to
mutations in the CF genes. The mutations lead to a
deficiency in cystic fibrosis transmembrane conductance
regulator protein that controls movement of salt and water
into and out of epithelial cells and results in production of
abnormally thick mucus. About 15% of patients
with CF present with meconium ileus at birth. This is
typically treated with enema for disimpaction and rarely
surgery. Approximately half of the infants with CF will
present with failure to thrive, which is diagnosed by lack of
growth for 2 consecutive months in patients younger than 6
months of age. They may also present with respiratory
compromise. However, not
all patients present in childhood. Diagnosis of CF is
confirmed by a sweat chloride level above 60 meq/L or with
genetic testing. Treatment for patients with CF is mainly
symptomatic therapy for obstructions of the digestive and
respiratory tract. In addition, there is pancreatic enzyme
supplementation to aid in digestion and vitamin and calorie
supplementation for deficiencies in
the diet. Gene therapy is now being looked at for future
treatment. Intussusception (telescoping of the small
intestine) typically presents in an infant with paroxysmal
abdominal pain, vomiting, and diarrhea that may progress
into bloody stools. Volvulus is normally the result of
intestinal
malrotation that causes occlusion of the superior mesenteric
artery and eventual bowel necrosis.Infants typically present
within 3 weeks of life with bile-stained vomiting and bowel
obstruction.Wilson's disease is the defect in the ability to
excrete copper in the bile that results inaccumulation of
copper in the liver.

48. Which of the following is NOT a cyanotic heart lesion?


(A) transposition of the great arteries
(B) atrioventricular septal defect
(C) hypoplastic left heart syndrome
(D) tricuspid atresia
48. (B) Cyanotic heart lesions are a result of a right-to-left
shunt. These include tetralogy of Fallot,pulmonary atresia
with and without ventricular septal defect, tricuspid atresia,
hypoplastic left heart syndrome, and transposition of the
great arteries. The right-to-left shunt results in
deoxygenated blood reaching the left ventricle, aorta, and
systemic arteries. The decreased oxygen in the blood results
in decreased oxygen to the tissue and subsequently causes
cyanosis.Atrial septal defect, ventricular septal defect,
atrioventricular septal defect, and patent ductusarteriosus
most commonly present with a left-to-right shunt.

49. A 2-week-old male infant presents for a routine checkup.


The mother complains that he nurses every hour, but vomits
(nonbilious) after every time he eats. He has only had three
bowel movements since he has been home. On examination,
the infant has not gained any weight since leaving the
hospital, and the clinician notes gastric peristaltic waves.
Which of the following is
the treatment of choice for this patient?
(A) pyloromyotomy
(B) metoclopramide
(C) laparotomy
(D) omeprazole

49. (A) This infant is presenting with signs and symptoms of


pyloric stenosis. Infants typically have vomiting (projectile
at times) after every feeding and it normally starts between
the age of 2 and 4 weeks. The infant nurses fervently and is
hungry. In addition, there may be dehydration,constipation,
weight loss, and apathy. Abdomen may be distended with
gastric peristaltic waves.
Occasionally, an olive-sized mass can be felt in the right
upper quadrant with deep palpation after the child has
vomited. Vomitus is typically nonbilious. Diagnosis is
confirmed by an upper gastrointestinal series with delayed
gastric emptying, enlarged pyloric muscle, and
characteristic semilunar impressions on the gastric antrum.
In addition, an ultrasound is needed to verify the
hypertrophic muscle. The treatment of choice for these
patients is pyloromyotomy, which can be done
laparoscopically. These patients make full recoveries and
have an excellent prognosis.

50. A 6-month-old infant presents for her checkup. Her


father mentions that they started solid foods after her 4-
month check. She has had foul-smelling diarrhea off and on
for the first month of solids; it now occurs after every meal
and looks greasy. They have tried different formulas and
different cereals without improvement. What is the
diagnostic test of choice for the most likely disorder?
(A) sweat chloride test
(B) RAST (radioallergosorbent assay test)
(C) gastrin level
(D) intestinal biopsy
50. (D) Celiac disease or gluten enteropathy typically
presents with diarrhea episodes in the first 6to 12 months of
life—when whole grains are first fed. Therefore, in strictly
breastfed babies,symptoms may not be noticed until solid
foods are begun. The diarrhea is usually intermittent at
first and then typically progresses into pale, greasy, foul-
smelling, frothy stools. Additional symptoms may be
constipation, vomiting, and abdominal pain, which may lead
the clinician to think of intestinal obstruction. Other findings
may be failure to thrive, anemia, and vitamin deficiencies.
Stool sample demonstrates excessive fecal fat excretion.
Blood tests show
hypoproteinemia and impaired carbohydrate absorption.
Intestinal biopsy is the diagnostic test of choice for celiac
disease. Results show shortened celiac mucosa, absent villi,
lengthened crypts of Lieberku$$$hn, plasma cell infiltration
of the lamina propria, and intraepithelial lymphocytes.
Treatment consists of dietary restriction of gluten—wheat,
rye, and barley. Steroids are given on an as needed basis.
Sweat chloride testing is utilized in patients suspected of
cystic fibrosis.Gastrin level is taken in patients suspected of
Zollinger–Ellison syndrome, and RAST (radioallergosorbent
assay test) is used in patients to determine different
environmental-type allergens

51. A 9-year-old child presents to the urgent care center with


her mother. The child is complaining of dark colored urine.
The mother mentions that the child was complaining of sore
throat and cold symptoms a few weeks ago. The urine shows
gross hematuria without nitrites or leukocytes.Which of the
following is the best test to help the clinician confirm the
diagnosis?
(A) monospot
(B) antistreptolysin O titer
(C) immunoglobulin electrophoresis
(D) renal biopsy
51. (B) The most likely diagnosis for this patient is
poststreptococcal glomerulonephritis. The diagnosis is
supported by a documented culture of group A beta-
hemolytic streptococcus infection. If a culture is not
available, like of the patient in this scenario, the clinician can
order
an antistreptolysin O titer. Antistreptolysin is an enzyme
released by group A streptococcus and is elevated for up to 1
month after strep infection. Glomerulonephritis presents
with gross hematuria with or without edema. Hypertension,
proteinuria, ascites, and headache may also be
present. Treatment with antibiotics is useful if infection is
still present, and, if necessary,symptomatic treatment for
renal failure is done with hemodialysis. Symptoms typically
resolve within a few weeks. The monospot is used to
diagnose infectious mononucleosis. Renal biopsy could be
performed on extreme cases of glomerulonephritis but is not
typically necessary.
Immunoglobulin electrophoresis would be utilized in
patients suspected of having immunoglobulinopathies or
IgA-mediated glomerulonephritis.

52. Upon performing a newborn examination, the clinician


notes a widened pulse pressure,paradoxical splitting of S2,
and a “machine”-like murmur heard best at the second
intercostals space, left sternal border, and inferior to the
clavicle. Which of the following is the most likely diagnosis?
(A) tetralogy of Fallot
(B) ventricular septal defect
(C) atrial septal defect
(D) patent ductus arteriosus
52. (D) Patent ductus arteriosus (PDA) is an isolated
abnormality that occurs in infants. The ductus arteriosus is
a normal fetal vessel that joins the aorta and the pulmonary
artery and spontaneously closes after 3 to 5 days. Lack of
closure results in the audible murmur that is
“machinelike” and
maximal at the second intercostal space (ICS), at the left
sternal border (LSB), and inferior to the clavicle. It is
typically a pansystolic murmur with bounding pulses and a
widened pulse pressure.There is also a paradoxical splitting
of S1 and S2. Echocardiography confirms the PDA, the
direction and degree of shunting, and the presence of lesions
for which the PDA is needed to keep. If there are no other
cardiac malformations requiring the PDA, then if the PDA is
large,surgery should be completed before 1 year of age.
Symptomatic PDAs that are relatively small may be closed
with indomethacin in preterm infants. The murmur heard in
atrial septal defect
(ASD) usually is an ejection type, systolic murmur heard
best at the LSB, second ICS with awide, fixed S2 and normal
pulses. Ventricular septal defect (VSD) presents with a
harsh,pansystolic murmur heard best at the third and fourth
ICS. With increasing size of the VSD,heaves, thrills, and lifts
are present along with radiation throughout the chest.
Tetralogy of Fallot
presents with a rough ejection, systolic murmur heard best
at the LSB and the third ICS with radiation to the back.

53. A 15-year-old boy suddenly collapses on the basketball


court; his sports physical conducted at the beginning of the
year did not elicit any abnormal findings. Basic life support
initiated at the scene, however, is unsuccessful in
resuscitation. Which of the following is the most likely
etiology of his sudden death?
(A) mitral valve prolapse
(B) surgically corrected aortic stenosis
(C) hypertrophic cardiomyopathy
(D) rheumatic heart disease
53. (C) Hypertrophic cardiomyopathy in adolescence is
typically due to familial hypertrophic cardiomyopathy with
an incidence of 1:500. Many patients are asymptomatic until
a sporting event, which may cause symptoms, specifically
sudden cardiac death. Examination may demonstrate a
palpable or audible S4, an LV (left ventricular) heave,
systolic ejection murmur
(may need to stimulate cardiac activity), and/or a left
precordial bulge. Echocardiography is the gold standard for
diagnosis but family history should be assessed. Stress
testing is indicated to assess for ischemia and arrhythmias.
Strenuous activities are prohibited for these patients. The
other cardiomyopathies (dilated and restrictive) are next but
are not as common. Congenital structural abnormalities of
the coronary arteries are the next most common cause.
Valvular disorders, including surgically repaired aortic
stenosis, are typically not causes of sudden death,but these
patients should be screened for symptoms and stress tested
as necessary.

54. A 7-year-old male child presents to the emergency


department with complaints of severe dyspnea, dysphagia,
drooling, muffled voice, and fever. The pulse oximetry is
91% on room air;lung examination shows stridor and
inspiratory retractions. Which of the following is the
expected chest X-ray finding for the suspected diagnosis?
(A) thumbprint sign
(B) Scottie dog sign
(C) steeple sign
(D) figure 3 sign
54. (A) This patient presentation describes epiglottitis.
Although there is a decreased incidence of epiglottitis
secondary to the introduction of the vaccine for
Haemophilus influenzae type B(Hib), patients still present
with sudden onset of fever, dysphagia, muffled voice,
drooling,cyanosis, inspiratory retractions, and soft stridor.
The patients are usually sitting in a tripod
position to aid their breathing. Recognition of the classic
symptoms needs to be immediate to stabilize the patient's
airway, as these patients will decompensate into respiratory
failure quickly. In the event that there is time, a lateral neck
X-ray will show the “thumb sign,” which is an
enlarged, undistinguished epiglottis. Treatment for the
patient requires intubation for airway stabilization, blood
cultures and throat/epiglottis cultures, and antibiotic
coverage for H.influenzae. The steeple sign is seen in patients
with croup and is due to a subglottic narrowing.The “figure
3” sign is seen in patients with coarctation of the aorta. The
“Scottie dog” sign is
seen in oblique lumbar films and is a normal finding
representing the pars interarticularis. Its absence signifies
spondylolysis.

55. Which of the following is one of the most common lethal


genetic disorders in the United States?
(A) trisomy 13
(B) trisomy 21
(C) cystic fibrosis
(D) neurofibromatosis
55. (C) With an incidence of 1:3,000 to 1:4,000 Caucasians,
cystic fibrosis is the most common lethal genetic disorder in
the United States. While trisomy 21 (Down syndrome) is one
of the most common genetic disorders with 1:500 newborns,
it is typically not a fatal disease. It is characterized with
mental retardation and physical malformations. Trisomy 13
is a fatal trisomy,
with most deaths occurring in early infancy or by the age of
2, but its incidence is approximately 1:12,000 live births.
Neurofibromatosis, a genetic disorder of typical autosomal
dominant inheritance, occurs in approximately 1:3,000 live
births. Most affected children have the skin lesions (café au
lait macules or neurofibromas) and other minor problems.

56. Which of the following is the initial treatment step in an


adolescent who presents to the emergency department with
status epilepticus?
(A) IV glucose
(B) stabilize airway
(C) arterial blood gas
(D) IV diazepam therapy

56. (B) Status epilepticus is a medical emergency and is


defined as seizure activity that lasts aminimum of 30
minutes. This results in hypoxia, acidosis, cerebral edema,
and structural damage.In addition, fever, respiratory
depression, hypotension, and death may occur. There are
both convulsive and nonconvulsive types of status
epilepticus. Because of its emergency status and potential
complications, the clinician needs to initiate the ABCs
(airway, breathing, circulation).
Therefore, the first line of treatment is to establish and
maintain an airway, oxygen is next, and then circulation,
which encompasses pulse, blood pressure, and IV access.
Once the IV is established, the orders should be for
administering glucose-containing fluids and IV drug therapy
with diazepam, lorazepam, or midazolam as well as
administer phenytoin and phenobarbital.Arterial blood
gases should be ordered and any abnormalities should be
corrected appropriately.Finally, the clinician should
determine the underlying cause: trauma, structural
disorder,infection, lactic acidosis, toxins, and uremia.
Maintenance drug therapy is necessary until the underlying
cause is determined and rectified.

57. A 12-year-old boy presents to the urgent care center


complaining of burning pain in his lower extremities with
weakness. On examination, the clinician notes symmetric
weakness with severely decreased active range of motion of
the lower extremities. In addition, there is decreased position
and vibratory sensation in the distal portions bilaterally.
Upon further questioning, the patient admits to being
diagnosed with mononucleosis 2 weeks ago. Which of the
following is the most likely diagnosis?
(A) poliomyelitis
(B) botulism
(C) Tick-bite paralysis
(D) Guillain–Barré syndrome
57. (D) Guillain–Barré syndrome is most likely due to a
delayed hypersensitivity with T-cell–mediated antibodies to
mycoplasma and viral infections (CMV, EBV, hepatitis B,
campylobacter jejuni). The patients may mention a
nonspecific respiratory or gastrointestinal infection 1 to 2
weeks prior to symptoms. Complaints may be paresthesias,
weakness in bilateral lower extremities with occasional
ascension into the arms, trunk, and face, and rarely ataxia
and ophthalmoplegia in the Miller–Fisher variant.
Examination findings demonstrate symmetric flaccid
weakness, with impairment of position, vibration, and touch
in the distal portions of the
extremities. If a spinal tap is performed, it may show few
polymorphonuclear neutrophils with high protein and
normal glucose. EMG is positive for decreased nerve
conduction. Laboratory tests may show high titers of
suspected infections or active infection of hepatitis/bacterial
pathogens. Guillain-Barré is normally a self-limiting
disorder within a few weeks, unless there are issues with
respiratory depression. Poliomyelitis is secondary to
polioviruses and presents with fever, paralysis, meningeal
signs, and asymmetrical weakness. Botulism secondary to
infection with Clostridium botulinum in older children
presents with blurred vision, diplopia,ptosis, choking, and
weakness. In infants, botulism presents as constipation, poor
suck and cry,apnea, lethargy, and choking. Tick-bite
paralysis presents with rapid onset with ascending flaccid
paralysis reaching upper extremities in a couple of days of
onset and patients often present with
paresthesia and pain. Finding of a tick is usually
confirmatory for these patients.

58. Which thoracic curvature is an indication for treatment


with bracing in an adolescent with scoliosis?
(A) less than 20°
(B) 20° to 40°
(C) 40° to 60°
(D) 40° with lumbar curvature of 30°
58. (B) Scoliosis is defined by lateral curvature of the spine
with rotation of vertebrae and is typically located in the
thoracic or lumbar spine in the right or left directions.
Idiopathic scoliosis most commonly presents as a right
thoracic curve in females from 8 to 10 years of age.Scoliosis
is typically asymptomatic unless curvatures are so severe
that there is pulmonary
dysfunction or there is an underlying disorder (bone or
spinal tumor) that is causing the scoliosis.X-rays need to be
taken of the entire spine to help determine the degree of
curvature. Treatment modalities are based on the degree of
curvature: 20° or less does not normally require treatment;
20° to 40° is an indication for bracing in an immature
child; and 40° and greater is resistant to bracing and
requires surgical fixation with spinal fusion, which is best
done at special centers.

59. A 6-year-old female child presents with complaints of


chronic hip pain so severe that she has not been able to walk
to the school bus. Examination shows severe tenderness at
the left hip with markedly decreased active and passive
range of motion. Radiologic examination demonstrates joint
effusion with widening. Which of the following is the most
likely diagnosis?
(A) osteochondritis dissecans
(B) slipped capital femoral epiphysis
(C) septic hip arthritis
(D) Legg–Calvé–Perthes disease
59. (D) Legg–Calvé–Perthes disease is also known as
avascular necrosis of the proximal femur. It typically occurs
in children between 4 and 8 years old and persistent hip pain
is the main symptom. On examination, the clinician notices a
limp and/or limitation of motion of the affected hip.
Radiologic examination demonstrates the necrosis with
effusion and joint space widening with a negative aspirate.
Treatment involves surgical hip replacement. Slipped capital
femoral epiphysis (SCFE) is due to the displacement of the
proximal femoral epiphysis owing to disruption of the
growth plate. The head is normally displaced medially and
posteriorly relative to the femoral neck. It typically occurs in
adolescence, specifically obese males, and can also be
associated with hypothyroidism. SCFE usually occurs after
direct trauma to the hip or a fall.Patients complain of vague
symptoms at first that progress into pain of the hip or of the
knee. On examination, there is decreased internal rotation of
the hip that can be confirmed by lateral X-ray of the hip.
Septic hip arthritis is not common in children between the
age of 5 and 12 years. The
legs are held in external rotation to minimize pain and will
have a positive aspirate.Osteochondritis dissecans typically
presents in the knee, elbow, and talus and is characterized
by a wedge-shaped necrosis of bone.

60. A 16-year-old boy presents to the office with thumb pain.


He just returned from a skiing trip. On examination, the
practitioner notes a positive ulnar collateral ligament laxity
test. What is the
most likely diagnosis?
(A) mallet finger
(B) gamekeeper thumb
(C) boxer fracture
(D) nondisplaced scaphoid fracture
60. (B) Gamekeeper thumb is a result of damage to the ulnar
collateral ligament during forced abduction of the
metacarpophalangeal joint, an injury that is most commonly
seen in skiers. An avulsed fragment may or may not be seen
on radiologic examination. If it is smaller than 2 mm,
there is no fragment, a thumb spica cast can be used as seen
in patients with no fragment. If the fragment is larger than 2
mm, surgery is required. Mallet finger is an avulsion of the
extensor tendon and occurs in ball-handling sports. Boxer
fracture is a distal neck fracture of the 5th metacarpal.
Scaphoid fractures are due to hyperextension of the wrist
injuries and present with
pain in the anatomic snuffbox and swelling.

61. A 3-year-old child is brought in by her parents to the


urgent care center stating that the child “will not bend her
arm.” They are obviously worried and distraught. The
clinician notices the elbow is held in strict pronation and
there is tenderness over the radial head. X-ray examination
shows no findings. Which of the following is the treatment of
choice for this disorder?
(A) place elbow in full supination and move from full extension
to full flexion
(B) immobilization of the elbow in a splint for 2 weeks
(C) referral to the orthopedic surgeon for suspected radial head
fracture
(D) call child protective services for suspected battery
61. (A) Nursemaid elbow is the subluxation of the radial
head due to a child or infant being lifted or pulled by the
hand. The patient will present with the elbow pronated and
painful and he or she will not bend the elbow. During the
radiologic examination, the dislocation is usually reduced by
placing the elbow in full supination and moving it slowly
from full extension to full flexion. This typically provides
immediate relief of pain and a sling may be given for
comfort for a couple of days. Otherwise, X-rays are normal.
Child protective services should be considered if this is
arecurrent problem or if there are other associated signs and
symptoms of battery. There is no
need for orthopedic referral unless reduction is not
commonly done in your setting.Immobilization of the elbow
is not recommended, because the patient then may have to
recover from frozen shoulder.

62. A 7-year-old child is brought into the office by her


mother who states that the child “is still wetting the bed at
night.” The child has already decreased liquid intake and
uses the bathroom before going to bed. The mother is
worried that there is something wrong with the child. Upon
examination there is no abnormality. Urinalysis is negative.
Which of the following is the treatment of choice for this
disorder?
(A) bed-wetting alarm
(B) desmopressin acetate (DDAVP)
(C) imipramine
(D) amitriptyline
62. (A) This patient is presenting with signs and symptoms of
primary nocturnal enuresis, which is the wetting only at
night during sleep without any sustained period of dryness.
It is mainly considered a parasomnia occurring in deep
sleep. The incidence of enuresis is higher in boys, is typically
related to a developmental delay, and most children become
continent by adolescence.
Patients need to be tested for structural abnormalities and
infections, in addition to neurologic diseases, diabetes
mellitus and insipidus, and seizure disorders. Treatment
includes limiting liquids at bedtime and routine bathroom
training during the day. If these are unsuccessful, the next
option is a bed-wetting alarm. This device is attached to the
child's undergarment and vibrates when the child is wet to
arouse the child to be aware of their need to urinate. If the
alarm is unsuccessful, then the next step is medication—
DDAVP (desmopressin acetate) or imipramine.

63. A 13-year-old boy presents with fever and blood in his


urine. Examination shows an asymptomatic mass in the left
lower quadrant. Urinalysis shows hematuria and small
leukocytes.Which of the following is the most likely
diagnosis?
(A) renal cell carcinoma
(B) intussusception
(C) volvulus
(D) nephroblastoma
63. (D) Nephroblastoma also known as Wilms tumor
typically presents with an asymptomatic abdominal mass
noticed by the parent or an increasing size of the abdomen.
On examination, the mass feels smooth and firm, is well
defined, and usually does not cross the midline. Gross
hematuria may be present, but rare, and some patients have
microscopic hematuria when tested.
Wilms tumor accounts for approximately 5% of cancers in
children younger than 15 years. Wilms tumor arises from
the kidney and the average age at diagnosis is 4 years.
Ultrasound and CT of the abdomen can be used to confirm
the presence of an intra-abdominal mass. Treatment
includes
exploratory abdominal surgery for removal and staging with
a mixture of chemotherapy.Intussusception (telescoping of
the small intestine) typically presents in an infant with
paroxysmal abdominal pain, vomiting, and diarrhea that
may progress into bloody stools.Volvulus is normally the
result of intestinal malrotation that causes occlusion of the
superior
mesenteric artery and eventual bowel necrosis. Infants
typically present within 3 weeks of life with bile-stained
vomiting and bowel obstruction.

64. A mother brings in her 20-month-old female child to the


office because she noticed pubic hair growing. On
examination, the clinician notices that the clitoris is
enlarged; the rest is unremarkable. Which of the following is
an expected laboratory finding on this patient?
(A) increased aldosterone
(B) increased estrogen
(C) increased androstenedione
(D) increased luteinizing hormone
64. (C) Infant girls presenting with signs of precocious
puberty need to be screened for congenital adrenal
hyperplasia (CAH). CAH most commonly presents with
pseudohermaphroditism in females—urogenital sinus,
enlarged clitoris, or other signs of virilization. In males,
there tends to
be isosexual precocity in older males and salt-losing crisis in
infant males. Both children show increased linear growth
and skeletal maturation. The most common type of CAH is a
deficiency in the enzyme 21-hydroxylase and laboratory tests
demonstrate increased urinary and plasma
androgens (DHEA, androstenedione). There may be elevated
progesterone, but typically there is no effect on estrogen.
There is also decreased aldosterone and elevated urinary
ketosteroids.There is also no effect on the levels of
leuteinizing hormone or follicle-stimulating
hormone.Treatment usually involves glucocorticoids,
mineralocorticoids, and reconstructive surgery, if
needed.

65. A 5-year-old child presents for her kindergarten


checkup. The clinician notes that over the past couple of
years, her height decreased from the 50th percentile to the
5th percentile. On examination, the clinician also notes
truncal adiposity. Her CBC and lead levels were
normal.Which of the following is the most likely diagnosis?
(A) growth hormone deficiency
(B) Cushing disease
(C) congenital hypothyroidism
(D) congenital adrenal hyperplasia
65. (A) Growth hormone (GH) deficiency is defined as a
decreased growth velocity, delay in skeletal maturation,
absence of other explanations for poor growth (lack of
intake), and laboratory tests demonstrating decreased GH
secretion. Etiology of GH deficiency can be congenital,
genetic, acquired, or idiopathic, which is the most common.
Infants usually have a
normal birth weight and may have a slightly decreased
length. In addition, most infants present with other
endocrine deficiencies like hypoglycemia, hypothyroidism,
and/or adrenal insufficiency. Children may present with
truncal adiposity because growth hormone promotes
lipolysis. Serum GH or intrinsic growth factor levels may or
may not be decreased. In patients who do not have a
demonstrated decrease in these hormones, a trial period
with GH is indicated.These patients and positive GH-
deficient patients receive a once-daily subcutaneous injection
of
recombinant human GH. Congenital hypothyroidism
typically presents with short stature (typically noted after
the 4-month newborn visit), delayed epiphyseal
development, delayed closure of fontanelles, and retarded
dental eruption in addition to other signs of
hypothyroidism.Cushing disease typically presents with
truncal adiposity with thin extremities, muscle
wasting,decreased growth rate, and moon facies. Laboratory
results show elevated adrenocorticosteroids both in urine
and serum, hypokalemia, eosinopenia, and lymphocytopenia.
Typically, in patients
younger than the age of 12, Cushing disease is secondary to
administration of ACTH or glucocorticoids. Congenital
adrenal hyperplasia typically presents with
pseudohermaphroditism in females or salt-losing crisis in
males with or without isosexual precocity. There is an
increased linear growth and advanced skeletal maturation.

66. An Rh-negative, 5-year-old male child presents with


acute onset of petechiae and purpura after an acute viral
illness. In addition, he has episodes of epistaxis. Which of the
following is atreatment option if his platelet count falls below
20,000/mm3, but he is not actively bleeding?
(A) platelet transfusions
(B) IV anti-D (WinRho SD) 50–70 mg/kg/dose
(C) prednisone 2.4 mg/kg/24 hours × 2 weeks
(D) splenectomy
66. (C) In patients with idiopathic thrombocytopenic
purpura, treatment options should be initiated when platelet
counts fall below 20,000, regardless of whether there is
active bleeding or not.Without active bleeding the treatment
options include prednisone 2–4 mg/kg/24 hours for 2
weeks; IV immunoglobulin 1 g/kg/24 hours for 1 to 2 days,
or IV anti-D 50–75 μg/kg/dose for Rh-positive patients.
Splenectomy is indicated for life-threatening bleeding. There
is currently no indication for platelet transfusion and none
of the above treatments are considered optimal,
because in the majority of children, it will resolve on its own
within 6 months.
67. A 9-year-old female child presents with tachycardia,
tachypnea, shortness of breath, bibasilar rales, and
distended jugular veins. Which of the following is the least
likely cause for her signs and symptoms?
(A) rheumatic heart disease
(B) sickle cell anemia
(C) viral myocarditis
(D) patent ductus arteriosus
67. (D) This patient is presenting with signs of congestive
heart failure. The most common causes of heart failure in
children/adolescents are due to acquired heart disease.
Congenital heart diseases,such as malformations of the
heart— patent ductus arteriosus and ventricular septal
defects, are
the most common causes of heart failure in infants–toddlers,
and are second to fluid overload in
neonates.

68. A 12 year-old girl patient was treated for a urinary tract


infection 3 days ago. She presents today with severe
conjunctivitis, target lesions on her trunk, and bullous
eruptions in her mouth. Which of the following medications
is the likely cause of her symptoms?
(A) ciprofloxacin
(B) erythromycin
(C) amoxicillin
(D) trimethoprim-sulfamethoxazole (TMP-SMX)
68. (D) This patient has the classic presentation of erythema
multiforme major or Stevens–Johnson syndrome. The most
common causes in children of erythema multiforme are
medications and Mycoplasma pneumoniae. Of the
antibiotics listed, the one most commonly causing Stevens–
Johnson syndrome is sulfonamide followed by penicillin and
tetracycline. The most common
medications causing SJS in children are nonsteroidal anti-
inflammatory drugs.

69. Which of the following is a complication of infection with


Parvovirus B19?
(A) aplastic crisis
(B) leukopenia
(C) aseptic meningitis
(D) congenital defects (if mother contracts during pregnancy)
69. (A) Infection with Human parvovirus B19 (also known as
fifth disease) resulting in the slapped cheek appearance, can
also cause aplastic anemia. This is because the virus infects
the precursors of erythrocytes and halts erythropoiesis.
Recovery is typically spontaneous with an occasional
transfusion for severe anemias.

70. A 9-year-old male child presents with a painful rash of


his upper extremity. His mom states it started 4 days ago
and seems like it is spreading. Physical examination
demonstrates a vesicular rash across the right upper arm
and chest but does not cross the midline. Which of the
following prescriptions would be most appropriate for this
patient at today's visit?
(A) hydration
(B) nonsteroidal anti-inflammatory drugs (NSAIDs)
(C) Varicella-Zoster immunoglobulin (VZIG)
(D) oral acyclovir
70. (D) As this patient is presenting with signs and symptoms
of herpes zoster within the appropriate time frame for
antiviral treatment, the treatment for this patient would be
oral acyclovir. NSAIDs may help with the pain associated
from zoster but will not hasten the length of the course of the
virus as acyclovir will. Varicella-Zoster immunoglobulin
(VZIG) is indicated for prophylaxis in exposed individuals
who are immunocompromised.

71. Which of the following findings would suggest a specific


child abuse diagnosis of Munchausen syndrome by proxy?
(A) fractures in various stages of healing
(B) retinal hemorrhages
(C) head or abdominal trauma
(D) recurrent polymicrobial sepsis
71. (D) Munchausen syndrome by proxy is when the
parent/caregiver is causing or complaining of signs and
symptoms of illnesses in his/her children. While it is a form
of child abuse and should be treated as such, it is also
considered a psychiatric disorder where the
parent/caregiver is
desiring to be in the sick role. The most common signs or
symptoms that should raise the level of suspicion for
Munchausen syndrome by proxy are: recurrent
polymicrobial sepsis, recurrent apnea, chronic dehydration,
or other unexplained symptoms like vomiting, diarrhea,
seizures,
failure to thrive, and hypoglycemia. The remaining signs are
seen in classical physical child
abuse.
72. Which of the following is a contraindication for the
meningococcal (Menomune) vaccine?
(A) history of Guillain-Barré
(B) complement deficiency
(C) college freshman in dormitories
(D) persons with functional asplenia
72. (A) Menomune is a tetravalent vaccine that is indicated
for prevention of meningococcemia caused by the bacterium
Neisseria meningitides. Menomune is indicated for patients
between 11and 12 years of age and at 15 years of age. It is
also indicated for college freshmen in dormitories, military
recruits, microbiologists working with the bacterium,
persons with complement deficiency and functional or
anatomic asplenia, and for those traveling to countries with
endemic disease. Guillain–Barré is a rare complication of the
Menomune vaccine, and if apatient has a history of
developing it, is the only relative contraindication other than
a known reaction to a previous administration of the vaccine
rubber latex and diphtheria toxoid severe
allergic reaction

73. A 7-year-old Caucasian female child presents to the


office with “an itchy head.” The child's mother, who is
with her, states that this has been bothering her daughter for
about a week and she has noticed a lot of “dandruff” in
the child's hair that will not come out. She also mentions
that several of her daughter's friends are having the same
problem. On the basis of the most likely diagnosis, what is
the best treatment for this patient?
(A) permethrin 1% shampoo
(B) ketoconazole cream
(C) tar-based shampoo
(D) silver sulfadiazine 1% cream
73. (A) The most likely diagnosis is pediculosis. This
parasitic infestation is most commonly seen in the young
school-aged child, and more often in female and Caucasian
children. The pediculosis louse lives in the hair and on the
scalp and intermittently “bites” into the skin to feed.
Discrete
urticarial papules or erosions may arise at the bite site. By
visualizing the live louse on the scalp,or in the hair, one can
easily make the diagnosis. However, the louse may be
difficult to see, as it is only 1 to 3 mm in size. Otherwise, nits,
or the casings of the eggs laid by the louse, can often be seen
on the proximal portion of the hair shaft. The nit adheres to
the hair shaft and is often
difficult to remove. Brown nits are representative of current
infestations and white nits past infestations. Treatment of
head lice can be difficult due to the increasing resistance to
some of the current treatment options. First-line treatment
includes permethrin (5%) and permethrin-based products.
Secondary treatment options for resistant infestations may
include Malathion (0.5%).Regardless of treatment, viable
ova should be removed by combing the patient's wetted hair
with a finely toothed comb until all are removed.
Ketoconazole cream and tar-based shampoos are utilized in
fungal and seborrheic dermatitis infections. Silver
sulfadiazine cream is a topical
antibiotic.
74. A young mother brings her 4-year-old son to the clinic
for evaluation of a rash on his umbilicus and hands. She has
been treating it with an over-the-counter ointment for about
a week, without success. She says that she has noticed that he
scratches the rash periodically, and it seems to bother him
the most at night. She also says that she noticed this same
rash on the hands on one of the other boys at his daycare
center. On examination, there are excoriated papules and
nodules on his hand and umbilicus. What is the most likely
diagnosis?
(A) herpes simplex
(B) scabies
(C) pediculosis
(D) tinea corporis

74. (B) Scabies, Sarcoptes scabiei, is the most common


arthropod infestation of children, and it is highly contagious.
However, its presentation varies widely and is dependent on
the child's age,
duration of the infestation, and immune status. Most often,
the presenting complaint is severe intermittent itching. The
linear papule or burrow commonly associated with scabies is
often difficult to identify. Instead, most children will present
with eczematous eruptions of red,
excoriated papules and nodules. Usually, the distribution of
the papules are the most diagnostic finding, and may include
the web spaces of the fingers and toes, axillae, umbilicus,
groin, penis,and the instep of the feet. Usually, in older
children and adults, the face and scalp are spared. The
treatment for scabies is a 12-hour application of permethrin
5% lotion. In addition, the parents
and all caregivers should be treated at the same time.
Clothing and bedding should be washed and dried (heat kills
scabies). The family should also be educated in the treatment
and prevention of future infestations. Moreover, they should
be advised that the itching associated with scabies
could persist for 7 to 14 days after successful treatment.
Pediculosis is an infestation of louse in the hair. Tinea
corporis is a fungal infection of the torso or “ring worm”
and presents with annual scaly plaques with central clearing
and pustules. Herpes simplex typically presents with
grouped
vesicles on erythematous base and is painful. It typically is
located in the lips, eyes, cheeks, or
hands of children.

75. A new mother brings her 3-month-old daughter to the


clinic for a rash on the infant's head. On examination, the
skin affected by the rash is thickened, yellowish white in
color, scaly, and looks waxy. In addition, it involves only the
scalp and bilateral postauricular areas. What is the most
likely diagnosis?
(A) contact dermatitis
(B) lichen planus
(C) pityriasis rosea
(D) seborrheic dermatitis
75. (D) Seborrheic dermatitis is common in all age groups.
In infants, this inflammatory skin disease is often manifested
as thickened, yellowish white, scaly, waxy appearing skin of
the scalp and commonly involves the postauricular areas
and the forehead. The more common name is “cradle
cap.” Cradle cap is a self-limiting disease of infants and
resolves by the child's first birthday. In all ages, the scalp
scale can be treated by shampooing with zinc pyrithione
(Head and Shoulders), selenium sulfide 1% to 2.5%, salicylic
acid (Tsal), or ketoconazole (Nizoral). The primary lesion in
lichen planus presents on the flexor surfaces and is
characterized by pruritic
papules that are polygonal and flat-topped. Pityriasis rosea
typically presents with the “herald
patch” that is a solitary pink, round patch with some
central clearing typically found on the torso. The rest of the
eruption is described as papulovesicular and develops a
Christmas tree pattern.
Contact dermatitis usually presents with red patches and
plaques with scales and is localized to the area exposed to the
irritant.

76. An 8-year-old male child presents with brown,


nonpruritic, annular lesions on the back of his hands and
feet. Intradermal nodules are seen on the extensor surfaces
of the elbows and knees that have been present for several
months. At today's visit, the lesions are essentially
unchanged since his last visit about a month ago. What is the
best treatment for this suspected disorder?
(A) excision and biopsy
(B) no treatment
(C) topical steroids
(D) wet to dry dressings
76. (B) This presentation is typical for granuloma annulare,
which is a benign skin disorder, and treatment is not
warranted. It is most commonly seen in children aged 6 to
10. The red to brown lesions are annular or circinate. These
asymptomatic lesions are often confused with tinea corporis.
The lesions will disappear on their own over a couple of
years.

77. Which of the following is the recommended treatment of


a 2- to 5-cm, single, nonpainful, common wart on the hand of
a 7-year-old?
(A) 40% salicylic acid plaster
(B) burning laser surgery
(C) electrocautery
(D) liquid nitrogen
77. (D) Liquid nitrogen is the treatment of choice for a single
isolated wart. Forty percent salicylic acid in a plaster
application is the most effective treatment of large and
painful warts.Electrosurgery, burning laser surgery, and
other destructive treatments should be avoided because of
the potential for scarring and subsequent problems often
associated with scars, as
well as the possible recurrence of the wart after destructive
treatment.

78. In treating uncomplicated, comedonal acne (open and


closed comedones) in adolescents, which of the following
treatments is best?
(A) topical antibiotics
(B) topical keratolytics
(C) oral retinoids
(D) systemic antibiotics
78. (B) Topical keratolytic agents applied to the skin either
as a single, once a day agent or in combination regime
(retinoic acid cream, azelaic acid, and adapalene) once a day
in the evening and benzoyl peroxide gel in the morning, will
control approximately 80% to 85% of cases of adolescent
acne. When treating inflammatory acne, papular or
pustular, a daily topical antibiotic
such as tetracycline, minocycline, or erythromycin can be
used in addition to a daily keratolytic.The oral retinoid, 13-
cis-retinoic acid (isotretinoin), Accutane is reserved for
treating nodulocystic acne (severe cystic acne). This
medication is not effective for the milder forms of acne such
as comedonal. Isotretinoin is teratogenic in women of
childbearing age and has other side effects. Therefore, strict
adherence to FDA guidelines is required
79. The most common fracture of newborns is a fracture of
the
(A) clavicle
(B) humerus
(C) radius
(D) ulna
79. (A) Clavicular factures are the most common fractures
in infants and children. In newborns, this fracture is usually
unilateral and often occurs after a difficult delivery. Many
times no treatment is required or a figure-of-eight bandage
can be used. For infants and children, a sling can be
used. The bump that can be seen after fracture consolidation
will usually resolve in a few months to a year. The next most
common fractures are of the extremities, humerus being the
most common and then the femur, but still much less
common than the clavicle.

80. A 13-year-old boy presents to the clinic for a complaint


of right knee pain that he first noticed about a year ago. It
started out as mild discomfort in the area just below the
kneecap, but has been getting progressively worse. Now, it
hurts anytime he uses his leg, even when walking. He does
not remember any injury to his knee. When you examine his
knee, you notice swelling and exquisite tenderness over the
tibial tubercle. X-rays are normal. What is the most likely
diagnosis?
(A) chondromalacia patellae
(B) Osgood–Schlatter disease
(C) patellar dislocation
(D) patellofemoral overuse syndrome
80. (B) Osgood–Schlatter disease is caused by microfractures
of the patellar ligament where it inserts into the tibial
tubercle. This condition usually occurs in the preteen and
adolescent years,and is more common in males than females.
The history of injury can be vague and the patient may not
remember a specific injury that precipitated the pain. Often,
the pain progresses to the point of interference of even
routine physical activities. X-rays may or may not show any
abnormalities. Upon X-ray, Type I disease appears normal,
but Type II will reveal fragmentation of the tibial tubercle.
Often, after healing there will be enlargement of the tibial
tubercle.
Generally, treatment consists of rest, limitation of activities,
and isometric exercises.Chondromalacia patellae can only be
diagnosed under an arthroscopic examination, not on the
basis of clinical features. Patellofemoral overuse syndrome
presents with medial knee pain and subpatellar pain.
Additional signs are swelling and crepitus in the knee and it
is more common in females than males. It is diagnosed by
increased Q-angles (anterosuperior iliac spine through
center of patella to tibial tubercle). Subluxation of the
patella or dislocation is more common in adolescent girls
and the patient presents with acute knee pain. The knee is in
flexion with a mass lateral to the knee and with absence of
the bony prominence of the patella (flat). X-ray confirms the
dislocation.

81. The eggs of this parasite are detected by microscopic


examination of clear adhesive tape that has been pressed to
the child's anus in the morning, prior to bathing. What
parasite is most likely to be identified by this test method?
(A) Ancylostoma duodenale (hookworm)
(B) Ascaris lumbricoides (ascaris)
(C) enterobiasis (pinworm)
(D) trichuriasis (whipworm)
81. (C) Enterobiasis or pinworms is a worldwide infection
that affects people of all ages and socioeconomic levels. It
especially affects children. The classic manifestation of this
problem is nocturnal anal pruritis and sleeplessness. The
sleeplessness may be secondary to the migration of female
worms to the perianal area to lay eggs, during which the
tape may pick up the larvae.
Transmission of the worms occurs when children ingest the
eggs that are present on their hands (from scratching), in the
bedclothes, or in house dust. After hatching in the stomach,
the larvae migrate to the cecum where they mature into
adults. The treatment of choice for pinworms is pyrantel
pamoate or mebendazole. Albendazole may also be used. For
eradication of this
parasite, often the entire family must be treated at once.
Ascaris is a helminthiasis infection that is ingested and
excreted in the stool. Diagnosis is made by stool examination
for the characteristic eggs. Hookworms are found in warm,
damp soil and penetrate the skin. From there the infection
can spread to the lungs where they ascend into the trachea to
be swallowed and live in the intestine. Diagnosis is made by
stool examination for the eggs. Whipworm is ingested from
the soil and lives in the intestine; detection is also made by
egg in the feces.

82. A 13-year-old boy presents with complaints of pain in


both knees and his right ankle. The pain is worse in the
morning. He denies any injuries, but does notice he tires
more easily when playing baseball. He says this has been
going on for about 8 weeks. His father admits to having
chronic low back pain, but otherwise the family medical
history is noncontributory. On the basis of this history,
which of the following is the most likely diagnosis?
(A) juvenile idiopathic arthritis
(B) Lyme arthritis
(C) psoriatic arthritis
(D) enteropathic arthritis
82. (A) Juvenile idiopathic arthritis (JIA) presents as three
distinct types. The types are based upon clinical
manifestations during the first 6 months of the illness. The
most common type is pauciarticular as presented by this 13-
year-old boy in the scenario mentioned. Second is
polyarticular disease with five or more joints being affected,
and the third is systemic onset of disease that begins with
high spiking fevers that are often associated with a rash that
comes and goes with the fever elevations. It is recommended
that patients with pauciarticular JIA have an
ophthalmologic evaluation and slit lamp examination every 3
months, if the antinuclear antibody
test (ANA) is positive and every 6 months, if the ANA is
negative, for 4 years after the JIA is identified to catch
iridocyclitis (untreated results in blindness). Lyme arthritis
usually presents with a monoarticular rash that typically
affects the larger joints, without morning
stiffness.Enteropathic arthritis is associated with
gastrointestinal symptoms occurring simultaneously as
lower extremity arthritis. It encompasses Reiter syndrome,
reactive arthritis (eg, postsalmonella,shigella), and arthritis
associated with celiac disease and inflammatory bowel
disease. Psoriatic arthritis is the arthritis accompanying the
dermatological disorder of psoriasis.The build-up of
epidermal cells over the joints causes inflammation and
thickening that results in arthralgia.
83. An 8-year-old female child presents with complaints of a
red itchy right eye with a lot of yellowish green color
discharge for 3 days. She denies any injury. Her visual
acuity is normal but she does have moderate tearing and
mild photophobia. What is the most likely diagnosis?
(A) allergic conjunctivitis
(B) bacterial conjunctivitis
(C) viral conjunctivitis
(D) reactive arthritis/Reiter syndrome
83. (B) Bacterial conjunctivitis is often unilateral and
presents with a mucopurulent discharge.Common bacterial
causes of this problem include nontypable Haemophilus,
Streptococcus pneumoniae, Moraxella catarrhalis, and
Staphylococcus aureus. These infections usually respond to
topical antibiotics such as sulfacetamide and erythromycin.
Systemic treatment is
indicated for conjunctivitis caused by chlamydia
trachomatis, Neisseria gonorrhea, or Neisseria meningitides.
Allergic conjunctivitis is usually associated with moderate to
severe itching of the eyes and clear mucoid drainage. Viral
conjunctivitis is usually associated with minimal itching,
profuse tearing, and minimal clear mucoid drainage. While
reactive arthritis
typically presents with a conjunctivitis, it is also
concomitantly present with arthritis and urethritis.

84. While seeing a 12-week-old baby girl for her well-child


checkup, it is noticed that she has tearing from her left eye.
There is a small reddened area that is swollen and she cries
when it is touched. The swollen area is just below the medial
inferior eyelid. There is also constant tearing from this same
eye. Her mother says it just started about 2 days ago and is
getting worse. What is the most likely cause of this problem?
(A) blepharitis
(B) conjunctivitis
(C) dacryocystitis
(D) anterior uveitis
84. (C) Dacryocystitis, whether acute or chronic, is usually
secondary to bacterial infections. It presents as an acutely
inflamed swelling and tender area over the lacrimal sac just
medial and inferior to the inner canthus of the eye. Because
the lacrimal sac is inflamed and blocked there is tearing and
usually purulent discharge from the eye. There may also be
an orbital cellulitis.Treatment consists of oral and topical
antibiotics and warm compresses, and surgical drainage
may also be indicated. After the acute episode and for
chronic cases, surgical correction of the nasolacrimal
obstruction is required. Anterior uveitis typically presents
with pain, photophobia,blurred vision, and injection without
exudates. Blepharitis is an inflammation of the lid margin
that presents with crusty debris along the lashes. Unless
there is a concomitant conjunctival infection there is
typically no injection noted.

85. The majority of cases of halitosis in young children can


be traced to which of the following causes?
(A) dental caries
(B) nasal foreign body
(C) poor dietary habits
(D) upper respiratory tract infection
85. (B) While halitosis can be caused by pharyngitis,
sinusitis, and poor hygiene, the most common cause of
halitosis in children is a nasal foreign body. Seeds and beads
are the leading objects inserted into the nose. If not
promptly removed, they can cause nasal obstruction,
infection,
rhinorrhea, bleeding, halitosis, or a foul smell. They are
usually easy to remove, but if there is difficulty in removing
the foreign body, the child should be referred to an
otolaryngologist for definitive care. Tobacco use in
adolescents is a common cause of halitosis. Dental disease is
the most common cause of halitosis in adults.

86. Of the following, which is the most frequent cause of


epistaxis in children?
(A) bleeding disorders
(B) choanal atresia
(C) digital trauma
(D) foreign bodies
86. (C) Most cases of epistaxis in the anterior portion of the
nose are caused by digital trauma (nose
picking) or some other mechanical cause such as nose
blowing or repeated nose rubbing. Other
causes may include incorrect use of steroid nasal sprays.
Examination of the anterior nose will
usually reveal irritation of the Kiesselbach area. Less than
5% of recurrent nosebleeds are
caused by bleeding disorders. Choanal atresia, unilateral,
usually appears as a chronic nasal
discharge that may be mistaken for chronic sinusitis.
Foreign bodies typically present with
purulent discharge instead of bleeding.

87. A 5-year-old male child in the clinic is being evaluated


for a firm, painful lump that is slightly reddened and
approximately 3 cm in diameter, in his right axilla. His
mother tells you the lump has been there for a couple of
days. The boy does not look acutely ill. The mother informs
you that they got a new kitten and puppy about a month ago
but otherwise nothing else is new at home. Which of the
following is the most likely etiology for his rash?
(A) Bartonella henselae
(B) parvovirus
(C) Hodgkin disease
(D) Osgood–Schlatter disease
87. (A) Cat scratch disease (CSD) is caused by the gram-
negative bacillus, Bartonella henselae.The disease is more
common in the fall and winter months and more males than
females are affected. Typically (approximately 90%),
patients report handling a cat or kitten and up to 70%
will report a scratch by a cat. The most common
complication of CSD is encephalitis. About half of the
patients with CSD will develop a primary cutaneous papule
at the site of inoculation, most often (approximately 50%) on
the hands or upper extremities, 3 to 10 days after the
exposure.
Regional lymphadenopathy will usually develop in about 1
to 7 weeks after the cutaneous lesions
and will affect the nodes draining the site of the scratch or
bite. The affected lymph nodes may be inflamed and are
usually tender. Occasionally, the involved nodes may
suppurate. The lymphadenopathy resolves in about 2
months, but may last as long as 4 to 8 months. Treatment is
usually not indicated for this self-resolving disease. However,
suppurative lesions may need to be aspirated for pain relief.
It has been shown that 5 days of treatment with
azithromycin has helped to speed recovery for some patients.
Because Hodgkin disease involves the lymph nodes,it should
be considered as a differential diagnosis when evaluating a
child for CSD. However, it
typically presents as a cervical lymphadenopathy. Fifth
disease (erythema infectiosum) is achildhood disease caused
by the human parvovirus. This common community-
acquired disease does not usually require treatment, but
respiratory isolation is recommended for 7 days following
the onset of symptoms. The initial stage of the disease
presents as red cheeks that appear to be “slapped” or
“slapped cheeks” with circumoral pallor. Osgood–
Schlatter disease is an orthopedic problem in children. It is
the result of repetitive microtraumas to the patellar ligament
at its point of insertion into the tibial tubercle. Usually, rest
and anti-inflammatory medications
are helpful in alleviating the pain associated with this
condition.

88. When considering infections caused by nematodes, which


of the following is most consistent with iron deficiency
anemia, abdominal discomfort, weight loss, and the presence
of ova in the
feces?
(A) ascariasis
(B) hookworm
(C) pinworms
(D) whipworm
88. (B) Hookworm (Ancylostoma duodenale and Necator
americanus) infections, if severe, can cause iron deficiency
anemia. Abdominal discomfort, weight loss, and ova in the
stool are more commonly associated with these nematodes.
Both types of human hookworms are found in tropic
and subtropical climates, which include the southeastern
United States, primarily the coastal areas. The larva of this
parasite is passed in the feces and incubates in warm, damp
soil when they hatch into larvae. The larvae penetrate
directly into the skin of humans, enter the bloodstream, and
migrate to the lungs. From the lungs they move up to the
trachea and are
swallowed. Once swallowed, they mature in the intestines.
The worms attach their mouth to the mucosal lining of the
intestine where they suck blood and shed new ova. Mild
infections are usually asymptomatic, but severe infestations
can cause anemia. Treatment for the infestation is achieved
with albendazole. In severe cases of anemia, parenteral iron
or transfusion may be
indicated. Pinworms are associated only with localized
pruritus, specifically the anus. Treatment may help
recurrent urinary tract infections in some young girls when
the pinworm has infected the urethra. Ascariasis is usually
asymptomatic; however, in severe cases it may be associated
with
anorexia, diarrhea, vomiting, weight loss, and abdominal
pain. Whipworm is also asymptomatic until the infection is
severe, with general gastrointestinal symptoms—pain,
diarrhea, and mild abdominal distention. Eosinophilia may
also be present, although slight.

89. Erythema migrans, the characteristic rash of Lyme


disease, occurs in what percent of patients with this disease?
(A) 20% to 40%
(B) 40% to 60%
(C) 60% to 80%
(D) 80% to 100%
89. (C) Appearing in 60% to 80% of cases, the characteristic
rash may not be present in all cases of acute Lyme disease.
Following the bite of a deer tick (Ixodes species), infected
with the spirochete Borrelia burgdorferi, an erythematous
ring forms around the bite site and spreads outward. The
ring may have a raised border and usually a clear center.
The ring can attain a
diameter of up to 20 cm. Multiple rings may form and they
can form at sites distal to the original bite site. If left
untreated, the rash will usually resolve within 3 weeks.
Erythema migrans is aminimally tender to nontender,
nonscaly rash that persists longer than many of the other
erythematous rashes of childhood.
90. A 2-week-old male infant is being seen in the clinic for a
profuse mucoid discharge from both eyes, with some
associated tearing. On examination, you notice both eyes are
hyperemic and the eyelids are red and swollen. Which of the
following is the most likely cause of this patient's ophthalmia
neonatorum (conjunctivitis in the newborn)?
(A) allergic
(B) gonococcal
(C) chlamydial
(D) viral
90. (C) Chlamydial infections are the most common cause of
conjunctivitis in newborns in developed countries. Other
causes of ophthalmia neonatorum include reactions to silver
nitrate prophylaxis, other bacterial infections such as
gonococcal or staphylococcal, or viral organisms such as
adenovirus or echovirus. Chlamydia trachomatis causes
conjunctivitis and pneumonia in
neonates. Treatment for chlamydial conjunctivitis should be
with systemic erythromycin to treat the conjunctivitis and as
prophylaxis against pneumonia.

91. Which of the following neurologic disorders is least likely


to be associated with Lyme disease?
(A) aseptic meningitis
(B) Bell palsy
(C) polyradiculitis
(D) seizures
91. (D) Seizures have not been associated with Lyme disease.
Neurologic manifestations occur in up to approximately
20% of patients with Lyme disease. Primarily, these are Bell
palsy,lymphocytic, aseptic meningitis, and polyradiculitis.
Cranial neuropathies, such as Guillain–Barré syndrome and
ataxias are less common. Additional neurologic
manifestations include
peripheral neuropathy, pseudotumor cerebri, and
encephalitis. If untreated, most neurological symptoms are
self-limited but some will persist or become permanent.

92. A young mother brings her 3-week-old daughter for care


of a rash in her mouth. The mother indicates the baby was
doing fine until 2 days ago when she noticed white spots in
the infant's mouth. On examination, they do not come off
easily with a tongue blade. She is bottle-feeding the infant
without any problem. Which of the following is the most
likely diagnosis of this problem?
(A) leukoplakia
(B) hand–foot–mouth disease
(C) herpangina
(D) oral candidiasis
92. (D) Oral candidiasis (thrush) is very common in the first
few weeks of infancy. The diagnosis is usually done by visual
inspection and does not usually require further laboratory
testing. On visual examination, white, creamy plaques are
found on the buccal mucosa and occasionally the
gingival and lingual mucosa. For this age group, direct
topical application of nystatin in oral suspension to the
lesions should suffice. If the lesions are resistant to treatment
or if they occur in older children, consideration should be
given to the possibility of the patient being
immunocompromised. All sources of candida, such as toys
and bottle nipples, should be
sterilized daily. Herpangina and hand–foot–mouth disease
are ulcerating lesions of the oral cavity due to viruses and
are self-limiting, but can be very painful. Leukoplakia is a
precursor lesion to oral cancer, seen most commonly in oral
tobacco users.

93. A 14-year-old boy presents for evaluation of behavior


problems that his mother reports have been present for
about a year, but have been worsening in the past few
months. She complains that her son has been having
problems in school, is not behaving, and is getting into fights.
He seems to only want to talk about science fiction movies
and occasionally seems to be talking to people who are not
really there. Sometimes, he seems really depressed and at
other times, “full of energy and happy.” On the basis of
this mother's observations, which of the following is the most
likely diagnosis?
(A) attention-deficit/hyperactivity disorder (ADHD)
(B) bipolar disorder
(C) conduct disorder
(D) depression
93. (B) Bipolar affective disorder is the most likely diagnosis
for this patient. Although ADHD,bipolar disorder, and
conduct disorder share many similarities in behavior
disorders, such as varying degrees of school and behavior
problems, defiant attitude, and distractibility, the
obsession with ideas (in this case, science fiction movies) is
not present in ADHD and conduct disorder. The mood
swings described here, as depression and elation are
consistent with bipolar disorder, which is confirmed by the
presence of hallucinations. Hallucinations, when considering
a differential diagnosis in a behavior disorder, are diagnostic
for bipolar disorder. In up to 70% of patients with bipolar
disorder, their first symptom of the disorder may be
depression.However, hallucinations are not typically a
manifestation of depression.

94. A 9-year-old male child presents in August with


complaints of a red rash on the palms of his hands, soles of
his feet, and a little on his legs. His mother states that this
rash started about 2days ago, and just before it appeared
her son had been complaining of a severe headache and
aching all over. She said he felt “hot to the touch” during
that time, as well. The child mentions he was camping in
Arkansas about 10 days ago with his dad but did not eat
anything abnormal.
On the basis of this history, what is the most likely
diagnosis?
(A) endemic typhus
(B) human ehrlichiosis
(C) Q fever
(D) Rocky Mountain spotted fever
94. (D) Rocky Mountain spotted fever (RMSF) is the most
common rickettsial infection in the United States, especially
in the eastern, southeastern, and western states, and it is
very common in 5- to9-year-old children. A known tick
exposure may or may not be documented. Most exposures to
ticks carrying Rickettsia rickettsii, the causative organism of
this disease, occur in the warmer months of April to
September when victims are most likely to participate in
outdoor activities in wooded areas. The incubation period of
RMSF is 3 to 12 days (mean 7) after a tick exposure.
The tick must be attached for 6 hours or greater in order to
transmit the disease. Clinical presentation includes fever,
often 40°C, myalgias, headache, and less characteristic,
red-rose macular or maculopapular rash. The rash usually
appears within 2 to 6 days, after the fever. The rash is
especially prevalent on the palms, soles, and extremities.
After several days, the rash,
which starts peripherally and spreads centrally, becomes
petechial. Conjunctivitis, edema,splenomegaly,
meningismus, and confusion may occur. Up to 5% to 7% of
patients with RSMF will die, and therefore, delays in
treatment should be avoided. Treatment for children is
doxycycline, regardless of age and the possible side effect of
stained teeth. In endemic areas,treatment should be started
early and is often based on suspicion alone, and prior to the
appearance of the rash. Endemic typhus (murine typhus) is
not transmitted by ticks but instead by the fleas from
infected rodents. The rash of endemic typhus differs from
that of RMSF in that it does not involve the palms and soles.
Q fever is spread by inhalation instead of ticks. The cause
of this rickettsial disease is Coxiella burnetii hosted by
domestic animals including dogs, cats,cattle, and sheep.
Unpasteurized milk from infected animals may also be a
source of this infection. One form of human monocytic
ehrlichiosis is carried by ticks that have fed on infected hosts
that may include deer, wild rodents, and sheep, most
commonly in the southeast, north, and
south central United States. The presentation is usually a
viral syndrome without any rash.Although this is usually a
self-limiting disease, deaths do occur in children; therefore,
treatment should be carried out with the antibiotic of choice,
doxycycline, regardless of side effects.

95. In young children, which of the following is the most


common cause of lower respiratory tract infections?
(A) adenovirus
(B) human parvovirus
(C) parainfluenza virus
(D) respiratory syncytial virus
95. (D) In young children, respiratory syncytial virus (RSV)
accounts for more than 70% of bronchiolitis, approximately
40% of the cases of pneumonia, and about 10% of cases of
croup. This seasonal disease occurs in the winter and early
spring months of the year. More than 50% of
children have been infected with RSV by age 1, and by the
age of 2, almost all children have been infected. Reinfection
commonly occurs but is mild. Adenovirus infections, though
common in early childhood, only account for approximately
up to 10% of all respiratory diseases. The
peak incidence of adenovirus respiratory infections occurs in
the spring, summer, and early winter. Human parvovirus
infection is typically seen in school-aged children. This
disease is characterized by the “slapped-cheek” appearing
rash on the face that appears about 10 to 17 days
following the infection. About 2 days after the appearance of
this facial rash, a similar rash appears on the extremities,
trunk, neck, and buttocks. The rash often persists for a few
days to afew weeks (average of 10 days) and often will recur
with exposure to bathing in warm water,exercise, sunlight,
and stress. Parainfluenza viruses fall into four categories
and are responsible
for the majority of cases of croup (65%), laryngitis (50%),
and tracheobronchitis (25%). Types 1to 3 occur as seasonal
outbreaks with types 1 and 2 in the fall and type 3 in the
spring and summer. Type 4 is an endemic virus. Clinical
symptoms of these viruses include laryngotracheitis
(croup), laryngitis, bronchiolitis, and less commonly
pneumonia (especially in
immunocompromised children).

96. A 3-year-old male child presents to the clinic for a cough


that occurs only after he has been running, according to his
mother. She says she first noticed this about 6 months ago,
after he had had one of his usual winter colds, and his cough
persisted for about a week. On the basis of this history, what
is the most likely diagnosis?
(A) airway foreign body
(B) asthma
(C) cystic fibrosis
(D) laryngomalacia
96. (B) Asthma, in this case exercise-induced, is the most
likely cause of this problem. The symptoms commonly
associated with acute exacerbations of asthma include
wheezing, cough,dyspnea, and chest pain. Some symptoms
that might be suggestive of asthma include exerciseinduced
cough, nighttime cough, cough after cold air exposure, and
cough after laughing. Airway
foreign bodies, though not common, are an acute problem
that may present as sudden cough,choking, and wheezing.
Cystic fibrosis (CF) is the most common, lethal, genetic
disease affecting the Caucasian population. Up to 50% of
patients with CF are diagnosed in infancy, but others
may not be diagnosed until adolescence or adulthood.
Chronic or recurrent cough should be an indicator for
consideration of CF as a differential diagnosis.
Laryngomalacia is the most common cause of stridor in
infants. It is the incomplete development of the cartilaginous
support of the
laryngoglottic structures. This congenital condition is
usually self-limiting and occurs most commonly in infants at
or just after birth. The inspiratory collapse of the epiglottis
or arytenoids cartilages is heard as stridor.

97. A 12-month-old male infant presents with his mother's


concerns that he does not seem to play with other children as
his brother and sister did at this age. She indicates she has
noticed that he does not seem to respond when she or other
children call him by name, he is indifferent to other children
or adults when they are present, and he does not seem to
know any and “just grunts.” On the basis of this history,
the most likely diagnosis for this problem is which of the
following?
(A) attention-deficit/hyperactivity disorder (ADHD)
(B) autism
(C) fragile X syndrome
(D) schizophrenia
97. (B) Autism is the most likely diagnosis for this child. The
signs of autism often present before the second year of life
such as the child's failure to respond to their name, failed
speech development, and appearing self-absorbed and
withdrawn in the presence of other children or
adults. Often in childhood, autistic children may develop
ritualistic behaviors and intense interests that if interrupted
may cause tantrums and rages. When speech does begin to
develop, it may be nonsensical: reversal of speech patterns,
echolocation, and other abnormal patterns.Goals of
treatment include early intervention to address behavior and
communication skills.
ADHD is characterized by easy distractibility, inattention,
and overactivity. Estimates for the presence of ADHD in
school-aged children range from 2% to 20%. Fragile X
syndrome is the most common cause of functional mental
retardation. This syndrome, affecting approximately 1 in
1,250 males, is caused by a trinucleotide expansion (CGG
repeated sequence) in the Fragile X Mental Retardation I
(FMR1) gene. Fragile X syndrome is characterized by a wide
range of symptoms, which may include language delay,
hyperactivity, autistic behavior, and variable
levels of mental retardation. Schizophrenia is usually
detected in adolescence, with prepubertal onset occurring
rarely. Patients may initially present with somatic or social
behavior problems.Schizophrenic children and adolescents
often have the same symptoms as adults, such as
hallucinations, bizarre thought processes, and rambling
speech.

98. When evaluating a newborn, the inability to pass a small


catheter through the nasal cavity is most
indicative of which of the following conditions?
(A) choanal atresia
(B) meconium ileus
(C) nasal infection
(D) nasal polyps
98. (A) Choanal atresia, whether unilateral or bilateral, is a
nasal obstruction that occurs relatively rarely in newborns.
If bilateral choanal atresia occurs at birth, it causes a
respiratory distress that requires immediate treatment (due
to infants being obligate nose breathers) by placing an oral
airway and subsequent surgical correction. Unilateral
choanal atresia can present as a

chronic, single-sided, nasal discharge that may not appear


until later in childhood. Meconium ileus, intestinal
obstruction secondary to inspissated meconium, occurs in
approximately 10% of newborns with cystic fibrosis. Cystic
fibrosis affects approximately 1 in 2,500 live Caucasian

births, and is a leading cause of death in young adults. Nasal


infections may occur secondary to afuruncle (infected hair
follicle) in the anterior nares or as a nasal septal abscess
following spread of a furuncle. Common causes of nasal
infections include picking at the nose and pulling out nose
hair. Nasal polyps are uncommon in children younger than
age 10, and when they do occur it is usually in older children
and adults with allergic rhinitis.
99. Anorexia nervosa is an eating disorder commonly
affecting teenage girls. Which of the following best
represents the percentage of the teenage girls affected?

(A) 1% to 5%

(B) 5% to 10%

(C) 10% to 15%

(D) 15% to 20%

99. (A) It is estimated that 1% to 5% of adolescents are affected


by anorexia nervosa. There are two types of anorexia nervosa.
The first is the nonpurging type when patients restrict their total
caloric intake and the second involves binge eating and purging
in association with the restrictive dietary habits. Otherwise,
intensive exercise regimes may be used as a means to control
weight.Anorexia nervosa occurs in boys but is more prevalent in
girls (2:1). The specific etiology of this familial problem is
unknown; there are genetic and environmental factors. DSM-IV
criteria also include refusal to keep weight at 85% of ideal
weight, intense fear of gaining weight even though underweight
amenorrhea and disturbance in the way one's body shape is
experienced.
100. Which of the following is the most common childhood
nutritional disorder in the United States?

(A) binge eating disorder

(B) folate deficiency

(C) obesity

(D) rickets

100. (C) Obesity is the number one nutritional disorder in


children in the United States. In 2004, 17% of American
children aged between 9 and 19 were considered obese. Risk
factors for obesity include other obese family members and
infants born to diabetic mothers. Associated environmental
factors include sedentary lifestyle, total caloric intake, television
watching, and computer games. All are considered contributory
factors in childhood obesity. Binge eating disorder is a relatively
new eating disorder category. It is most frequent in overweight
or obese individuals. This disorder includes recurrent episodes
of binge eating (eating more than most individuals would in a 2-
hour period) and a sense of lack of control over the impulse to
eat,marked distress over the episode at least 2 days a week, and
is not associated with regular compensatory activity such as
purging or fasting. Folate deficiency anemia (megaloblastic) can
occur in infants within a few weeks after birth. This deficiency
may be a result of malabsorption,low dietary intake such as with
goat's milk or home-prepared formulas that have been sterilized
by heating, or formulas based on pasteurized milk. Infants who
are breastfed or given supplemented cows’ milk formulas do
not have a problem with folate deficiency. In children,

rickets is most commonly a result of poor dietary intake of


vitamin D and inadequate exposure to direct sunlight. Vitamin D
sources include milk, cheese, and baby formula. Vitamin D in
humans is produced by activation of its inactive precursors in
the skin after exposure to ultraviolet light.
101. By which age do infants develop heart failure secondary
to congenital heart lesions?

(A) birth

(B) 6 months

(C) 9 months

(D) 12 months

101. (B) The symptoms for congestive heart failure in infants


are typically failure to thrive,tachycardia, and poor feeding.
These will typically not present at birth and will be identified by

102. Which of the following is the antibiotic of choice for a


patient who is diagnosed with Bordetella pertussis?

(A) erythromycin 40-50 mg/kg/24 hours in divided doses X 14


days

(B) ampicillin 100 mg/kg/34 hours in divided doses X 7 days

(C) amoxicillin 80-90 mg/kg/24 hours in divided doses X 10


days

(D) cephalexin 30 mg/kg/24 hours in divided doses X 7 days

102. (A) Bordetella pertussis is a gram-negative bacillus and,


therefore, of all the choices, the antibiotic with good gram-
negative coverage is erythromycin. The other
macrolides,azithromycin and clarithromycin may also be given
for shorter durations, however they are more expensive.
Ampicillin, amoxicillin, and cephalexin provide mainly gram-
positive coverage.

103. Which of the following is indicated for an incarcerated


inguinal hernia present for more than 12 hours?

(A) watchful waiting

(B) manual reduction

(C) surgical reduction

(D) bilateral surgical reduction

103. (C) Surgical reduction is the treatment of choice for


incarcerated hernias over 12 hours. At that point the likelihood
that the hernia will manually reduce is very small and the bowel
is becoming necrotic and needs to be removed as soon as
possible. Bilateral surgical reduction is required only in the
event of two hernias, and there is no place for prophylaxis
surgery for inguinal hernia repairs.
104. A 6-year-old female child presents with neck pain and
fever for 2 days. Her remote history consists of 2 to 3 days of
diarrhea and vomiting. She attends a local daycare where
other kids had similar nausea/vomiting, but recovered. The
LP was positive for gram-negative bacilli, decreased protein,
and increased neutrophils. Which of the following is the
most likely etiologic agent?

(A) rotavirus

(B) Salmonella species

(C) Corynebacterium diphtheria

(D) Clostridium botulinum

104. (B) Salmonella species are gram-negative bacilli that are


classified as Enterobacteriaceae,along with E Coli. While
extremely uncommon as an etiology for meningitis, salmonella
can cause lethal meningitis infections and must be watched.
While there is typically no treatment for mild to moderate
diarrhea from salmonella infections, these patients should be
monitored for complete resolution. Viral meningitis typically
does not have a positive Gram stain, unless there is
contamination. Corynebacterium and clostridium are gram-
positive bacilli
105. In an infant with highly suspected vitamin K deficiency,
which laboratory finding would be expected?

(A) prolonged PT (prothrombin time)

(B) elevated fibrinogen

(C) decreased platelet count

(D) decreased aPTT (activated partial thromboplastin time)

105. (A) Vitamin K deficiency causes hemorrhagic disease of


the newborn. Vitamin K is one of the compounds required for
conversion of prothrombin, factors VII, IX, and X of the
coagulation cascade. In addition, proteins C & S are also
Vitamin K dependent. Therefore, the result is an increased
prothrombin time and this would result in an increased aPTT.
There is no effect on platelets or fibrinogen.
106. In a 12-month-old male infant presenting with acute
onset ear pain that is disrupting his sleep, which of the
following findings on clinical examination would confirm a
diagnosis of acute otitis media?

(A) erythematous tympanic membrane

(B) tenderness upon palpation of the tragus (C) bulging


tympanic membrane

(D) flat tracing on tympanometry

106. (C) The diagnosis of otitis media requires the presence of


middle ear effusion, acute onset ofsymptoms, and signs and
symptoms of middle ear inflammation. Presence of the middle
ear effusion can be determined by the bulging of the tympanic
membrane, air-fluid levels, absent mobility of the tympanic
membrane by pneumatic otoscopy, or otorrhea from perforation.
Office tympanometry can be performed to confirm a diagnosis
of effusion. Tenderness on palpation of the tragus typically is a
sign of otitis externa.
107. Which of the following requirements for child safety
restraints is TRUE?

(A) Infants weighing less than 20 lb and longer than 20 inches


may sit in forward-facing seats.

(B) Children weighing between 20 and 30 lb may sit in upright


booster seats.

(C) Children who weigh less than 80 lb should be in a certified


child safety seat/booster.

(D) Children weighing more than 40 lb may sit in the fiont seat
in four-door vehicles.

107. (C) While different states have different requirements for


child safety restraints, the most common guidelines state that
infants must be 20 lb and 1 year of age before switching to
forwardfacing seats. Children between 20 lb and 40 lb should be
in front-facing safety seats, typically with a 5-point harness;
children between 40 and 80 lb may be in booster seats in which
the back is typically required based on the height of the child.
Lastly, children should be older than 12years of age and
typically at least 80 lb as the front air bags are dangerous.
108. Which of the following is the most common cause for
childhood gynecomastia?

(A) neoplasms

(B) medications

(C) illicit drug use

(D) idiopathic

108. (D) The most common etiology of gynecomastia is


idiopathic. Occurring in 50 % to 60% of adolescent males,
idiopathic gynecomastia typically is self-limited. Additional
uncommon etiologies of gynecomastia include liver disease,
hyperthyroidism, illicit drugs (marijuana heroin), neoplasms
(adrenal, testicular), and medications (eg, antacids,
chemotherapy)
109. In the evaluation of a child with newly diagnosed
hypertension, which of the following evaluations will help
rule in the most common etiology?

(A) urinalysis

(B) serum uric acid 7 V D tewelatdalaegltull

(D) chest radiography

109. (A) A urinalysis should be performed because renal disease


is the most common etiology of hypertension in children.
Electrocardiograms and chest radiography should be considered
as part of the evaluation for end-organ disease as well as an
initial basic metabolic panel to include serum and creatinine.
Although rare, elevated uric acid has also been shown to cause
essential hypertension in children

.
110. The Centers for Disease Control and Prevention
recommend the first lead screening for children living in
high risk areas in the United States at which age?

(A) 6 months

(B) 9 months

(C) 15 months

(D) 24 months

110. (B) The CDC recommends that there are two age ranges for
testing lead in children in the United States: 9 to 12 months and
again at 24 months. These high-risk areas include poverty-
stricken areas, use of lead paint pottery, lead painted homes
(peeling or cracking), industrial exposures,and use of diarrhea
remedies in Mexico. The CDC recommends using questions to
screen all children between 6 months and 6 years of age.

Answers and Explanations


1. (B) Bronchiectasis has numerous etiologies. Most commonly,
cultures reveal normal oral flora
from the lower respiratory tract: Streptococcus pneumoniae,
Staphylococcus aureus,
Haemophilus influenzae, Pseudomonas aeruginosa.
Parainfluenza viruses typically are
responsible for croup. Corynebacterium diphtheriae is the
causative organism for diphtheria.
Rhinovirus is the most common pathogen isolated with acute
viral rhinitis or the common cold.
2. (D) The typical hemoglobin electrophoresis for beta
thalassemia minor has an elevated level of
hemoglobin A2. In a normal infant there is mainly HgF and
HgA1 with minimal amounts of A2.
Bart hemoglobin is diagnostic for the alpha thalassemias after
the neonatal period is over. Beta
thalassemia major will only have fetal hemoglobin on
electrophoresis. Because of the high
incidence of false-negatives in hemoglobin screenings in the
neonatal period, it is important for
the provider to do a full work-up of microcytic, hypochromic
anemias to ensure proper
diagnosis.
3. (B) In children who present with symptoms of sore throat and
fever, approximately 50% to 70%
of these cases are due to a viral infection. Adenovirus is one of
the most common etiologic viral
agents. Epstein–Barr virus is the etiologic agent for
mononucleosis and while very common in
the United States it is still less than rhinoviruses and
coronaviruses. The two remaining choices
are bacterial pathogens of which group A beta-hemolytic
streptococcus (GAS) is the most
common followed by the less common pathogens (group C
Streptococcus, Arcanobacterium
haemolyticus, and Streptococcus pneumoniae). As a single
agent, GAS is the most common
etiology of acute tonsillitis and pharyngitis
4. (A) In left-sided congestive heart failure, the signs of
tachycardia, tachypnea, intercostal
retractions, rales, and rhonchi are found. Hepatosplenomegaly is
a sign of right-sided congestive
heart failure. Bradycardia is not associated with either left- or
right-sided congestive heart
failure in the pediatric patient.
5. (B) Leukemia is the most common form of childhood cancer.
Acute lymphoblastic leukemia is the
most common form of leukemia in childhood, accounting for
approximately 4 out of 100,000
children younger than the age of 15. The clinical presentation is
variable, ranging from severe
with a life-threatening infection to asymptomatic at a routine
well-child visit. Often, there is a 3-
to 4-week history of an illness prior to the diagnosis, with signs
and symptoms including malaise,
anorexia, intermittent fever, bone tenderness, pallor, petechiae,
purpura, and abdominal pain.
Findings noted on the physical examination include pallor,
petechiae, purpura, retinal
hemorrhages, lymphadenopathy (either localized or generalized
to cervical, axillary, or inguinal
areas), bone and joint tenderness (especially in the pelvis, lower
vertebral bodies, and femur),
hepatosplenomegaly, and nephromegaly. Initially, the most
useful test is a complete blood count
with differential, revealing multiple cytopenias and leukemic
blasts. The bone marrow
examination is diagnostic, revealing a homogeneous infiltration
of leukemic blasts replacing
normal marrow. Acute myelogenous leukemia typically presents
with hyperleukocytosis (WBC >
100,000) or with myeloblasts on peripheral smears and bone
marrow biopsies. It accounts for
25% of leukemias in childhood. Patients with chronic Epstein–
Barr virus infections present with
sore throat, fever, posterior cervical lymphadenopathy, and
malaise associated with atypical
lymphocytosis and a positive heterophile antibody test.
Hodgkin's lymphoma typically presents
with painless cervical adenopathy and a normal CBC. However,
typically the C-reactive protein
and erythrocyte sedimentation rates are elevated.
6. (A) Despite the increase in vaccination of infants in the
United States, Streptococcus
pneumoniae remains the most common etiologic agent for
bacterial meningitis in the pediatric
population. Haemophilus influenzae type B is the second most
common, but has gone down
significantly due to the widespread vaccination of children.
Neisseria meningitides has
approximately 2,400 to 3,000 cases a year. Meningitis due to
Listeria monocytogenes is
typically seen in the neonatal period due to transmission from
the mother. It is present in normal
fecal matter in around 10% of the population. Its rates have gone
down due to strict guidelines
for the food industry, resulting in less than 1,000 cases per year.
7. (B) The diagnosis of rheumatic fever is based on clinical
grounds using the modified Jones
criteria. Two major manifestations or one major and two minor
manifestations in addition to
supporting evidence of a preceding streptococcal infection are
needed to make the diagnosis of
rheumatic fever. The major manifestations are polyarthritis,
carditis, erythema marginatum,
subcutaneous nodules, and Sydenham chorea. The minor
manifestations are fever, arthralgia,
previous rheumatic fever or rheumatic heart disease, an elevated
sedimentation rate or Creactive
protein, and a prolonged P–R interval. The supporting evidence
of a preceding
streptococcal infection includes elevated titers of
antistreptolysin O or other streptococcal
antibodies and positive throat culture for group A beta-
hemolytic streptococcus
8. (D) Hemophilia B, also known as Christmas disease and
factor IX deficiency. Factor IX is
activated on the intrinsic side of the coagulation cascade right
before the common pathway and
the result is an increased aPTT, with a normal prothrombin time,
thrombin time, and INR. It does
not affect platelets nor bleeding time.
)
9. (B) Tanner stages of sexual maturation categorize the
progression of pubertal development in
girls according to pubic hair and breast development. Menarche
usually occurs 18 to 24 months
following the onset of breast development. In female breast
development, Tanner Stage I is an
absence of breast development; Stage II is a small, raised breast
bud; Stage III shows further
enlargement/elevation of breast and alveolar tissue; Stage IV is
the areola and papilla forming a
secondary mound on breast contour; and Stage V is the mature
breast with alveolar area as part
of the breast contour. For the stages of pubic hair development,
Stage I is prepubertal, an absence
of hair; Stage II shows sparse, fine hair, primarily on the border
of labia; Stage III is pigmented
and curly and increases in quantity on the mons pubis; Stage IV
is increased quantity of coarser
texture with labia and mons pubis well covered; and Stage V is
mature adult distribution with
spreading to medial thighs.
10. (C) During the first year of life, the average, expected
increase in weight of a full-term infant is
to regain the birth weight by 2 weeks of age, double the birth
weight by 4 months of age, and
triple the birth weight by 1 year of age.
11. (B) When evaluating children with physical injuries, the
major difficulty is distinguishing
intentional injuries from unintentional injuries. Inconsistencies
between the stated story and the
injury are suspect. Discoloration caused by healing bruises tends
to follow a distinctive pattern.
On the first day, there is swelling without discoloration. From
day 1 through day 5, the bruise is
purple in color. For days 5 through 7, the bruise is green. Then,
from day 7 through day 10, the
bruise is yellow, followed by a brownish color from day 10 to
day 14.
12. (D) Rotavirus is one of the most important causes of acute
gastroenteritis in infants and young
children primarily 6 to 24 months of age. In the United States,
there are 65,000 to 70,000
hospitalizations and 200 deaths per annum. Peak incidences
occur in the fall and winter. Most
initial infections are characterized by diarrhea (watery,
nonbloody, nonmucous), fever, and
vomiting. Nasal congestion and coryza often precede the
gastrointestinal symptoms. Clostridium
difficile produces a toxin that causes a self-limited diarrhea in
which symptoms
characteristically begin following the administration of
antibiotics that reduce normal bowel
flora. Giardia lamblia, a flagellated protozoa, characteristically
causes a mild diarrhea, with or
without a low-grade fever, anorexia, flatulence, and abdominal
cramps. It is not associated with
vomiting nor upper respiratory tract symptoms. Shigella
gastroenteritis in young children
classically presents acutely with a high fever or seizures along
with vomiting followed by
bloody, mucoid, diarrheal stools.
13. (A) “Toeing in” in children before the age of 2 is typically
due to tibial torsion; however, any
“toeing in” after the age of 2 to 3, is usually due to femoral
anteversion. The femur has more
internal rotation that results in the presentation Genu varum is
known as bowleg and genu
valgum is known as knock-kneed.
14. (D) The first sign of pubertal development in boys is the
enlargement of testicular size and
occurs at the mean age of 11.6 years. Genital stages accelerate
before pubic hair development,
which occurs, on average, at 13.4 years of age. The deepening of
the voice and the development
of chest and axillary hair usually occurs in midpuberty or 2
years after the growth of pubic hair.
15. (A) Huntington disease is an autosomal dominant hereditary
disease. Its occurrence is between
1:5,000 and 1:20,000. It is caused by a defect on chromosome
4p16.3 that results in a repeat of
“CAG” in the “Huntington” protein gene.
16. (A) Bell's palsy is the acquired peripheral facial weakness
(cranial nerve VII) of sudden onset
and unknown etiology. It often follows a viral illness with
notable improvement within 2 weeks
and near complete recovery within 2 months. Prednisone
therapy may promote recovery of facial
strength. Guillain–Barré syndrome (acute idiopathic
polyneuritis) generally presents with
symmetrical weakness of the lower extremities, which may
ascend rapidly to the arms, trunk, and
face. Nonspecific respiratory or gastrointestinal symptoms may
occur 5 to 14 days preceding the
infection. Physical examination will yield symmetric flaccid
weakness, which is usually
proximal in distribution. Rarely, there is cranial nerve (III–VI,
IX–XI) involvement. Botulism is
most often caused by the ingestion of food containing the
Clostridium botulinum toxin or rarely
from an infected wound. Children will present with blurred or
double vision, ptosis, or choking.
Physical findings include a weak swallow paralysis of
accommodation and eye movements. In
this case, there was not a history of food ingestion or wound
infection to support this diagnosis.
Children with a brain stem tumor may present with facial and
extraocular muscle palsies,
hemiparesis, gait disturbances, and hydrocephalus (25%).
Changes in personality such as
lethargy, irritability, and aggressive behavior are particularly
common findings. Speech and
swallowing difficulties are not unusual. Later in the illness,
patients will develop vomiting and
headaches.
17. (C) Still's murmur is the most common innocent murmur of
early childhood and is usually
appreciated in children from 3 to 6 years of age. It is a grade I–
III/VI early systolic ejection
murmur of musical or vibratory quality heard best between the
apex and the left lower sternal
border. It is loudest when the patient is in a supine position. The
murmur may diminish or
disappear with inspiration, during the Valsalva maneuver, or
when the patient is standing or
seated. A physiologic peripheral pulmonic stenosis murmur is a
soft, short, high-pitched, grade I–
II/VI systolic ejection murmur. Typically, it is auscultated with
equal intensity at the left upper
sternal border, along the back, and in both axillae. It is usually
found in newborns and generally
disappears by 3 to 6 months of age. A pulmonary ejection
murmur is the most common innocent
murmur of later childhood and is usually seen in children 8 to 14
years of age. It is a soft, early to
midsystolic ejection, grade I–III/VI murmur heard best along the
left upper sternal border. It is
louder when the patient is supine or with increased cardiac
output. It diminishes with standing or
during the Valsalva maneuver. A venous hum is a continuous
musical, grade I–II/VI murmur heard
at the right or left superior infraclavicular area. The murmur is
obliterated when the patient is in
a supine position, with head rotation, and with compression of
the jugular vein. It is usually
auscultated in children from 3 to 6 years of age.
18. (D) Sudden infant death syndrome (SIDS) is defined as the
sudden, unexplained death of an
apparently healthy infant that is unexpected and not adequately
explained by a comprehensive
medical history, a postmortem physical, and investigation of the
death scene. SIDS is a leading
cause of death in infants between the ages of 1 month and 1
year, second only to congenital
anomalies. The exact etiology of SIDS is unclear. Prevention of
SIDS has become a focus of
public health measures. In 1994, The American Academy of
Pediatrics initiated a campaign
called “Back to Sleep,” which recommended placing infants
in the supine position for sleep.
Following the institution of this campaign in the United States,
the annual death rate decreased
from 1 3 per 1 000 to 0.7 per 1,000.
19. (A) Bronchiectasis, meaning “dilation of the bronchi,”
results from destruction of the airway and
poor drainage, often associated with cystic fibrosis, foreign body
aspiration, or an infection. It is
uncommon in the general population. The presentation may vary
from a chronic productive cough
to recurrent pneumonia with or without hemoptysis. Persistent
rhonchi, rales, and decreased
breath sounds are noted over the affected atelectatic area. Croup
is an inflammatory disease of
the larynx most frequently affecting young children during the
fall and early winter months.
Typically, there is an upper respiratory tract prodrome followed
by stridor and a “barky cough”
in the absence of drooling. Subglottic narrowing with a normal
epiglottis is diagnostic on a
lateral neck X-ray. The most common pathogen is parainfluenza
virus. Bronchopulmonary
dysplasia is most commonly seen in infants in the neonatal
intensive care unit. It is a chronic
condition seen in patients whose clinical course included hyaline
membrane disease. These
infants typically need oxygen for a few months as they grow and
some need permanent
tracheostomy and ventilation for up to 2 years. Chronic
bronchitis falls into the chronic
obstructive pulmonary disease category typically seen in older
adults and does not typically
present with acute symptoms.
20. (A) Otitis externa is an infection of the auditory canal. The
most common etiologic agent is
pseudomonas. However, Staphylococcus aureus is a very close
second and therefore antibiotic
treatment should provide coverage for both organisms.
Corynebacterium is part of the normal
flora of the auditory canal and does not typically cause
infections. Streptococcus pyogenes is the
most common cause of acute bacterial pharyngitis.
21. (A) Cytomegalovirus (CMV) is one of the congenital
neonatal TORCH infections
(toxoplasmosis, o ther [syphilis, varicella-zoster, and parvovirus
in this list], r ubella, c
ytomegalovirus, and h erpes simplex/h epatitis/H IV). CMV is
the most common congenital
infection. The disease-specific manifestations for CMV include
microcephaly with
periventricular calcifications, neonatal jaundice with direct
hyperbilirubinemia, and
hepatosplenomegaly. Other associated manifestations include
intrauterine growth retardation,
thrombocytopenia, and purpura. Disease-specific manifestations
for herpes simplex virus include
skin/eye/mouth vesicles, encephalitis, respiratory distress, and
sepsis. Disease-specific
manifestations of rubella include congenital heart lesions (patent
ductus arteriosus, pulmonary
artery stenosis, aortic stenosis, ventricular defects),
thrombocytopenic purpura characterized by
purple macular lesions (“blueberry muffin” appearance),
cataracts, retinopathy, and
sensorineural deafness. Disease-specific manifestations of
syphilis include mucocutaneous
lesions (snuffles), periostitis, osteochondritis, and hemolytic
anemia. Often, these babies are
stillborn. Syphilis is caused by a spirochete, Treponema
pallidum, not a virus.
22. (A) Approximately 40% of children who have been
physically abused showed evidence of
ocular trauma. Retinal hemorrhages are the most frequent ocular
finding that result from violent
shaking. This form of child abuse is termed shaken baby
syndrome. The finding of retinal
hemorrhages in an infant without an appropriate medical
condition (eg, clotting disorder,
leukemia) should raise concerns about nonaccidental trauma.
Some of the most common
presenting complaints of infants with shaken baby syndrome are
lethargy, coma, seizures,
vomiting, and respiratory distress. Retinal hemorrhages are not
associated with retinitis
pigmentosa, retinoblastoma, Reye syndrome, or viral
encephalitis. With Reye syndrome, an
antecedent viral illness is followed by vomiting and progressive
lethargy. On examination, there
is usually fever, tachypnea, and stupor. Laboratory hallmarks
include elevated serum
hepatocellular enzyme assays and elevated serum ammonia.
Retinitis pigmentosa is a progressive
retinal degeneration and is characterized by pigmentary changes,
optic atrophy, and progressive
impairment of visual function. The presenting clinical
manifestation is usually an impairment of
dark adaptation or night vision. Clinical manifestations of viral
encephalitis vary in severity
depending upon the etiologic organism (eg, cytomegalovirus,
mumps, echovirus). Some children
will have mild symptoms lapsing into a coma leading to death,
whereas others are febrile, with
convulsions and hallucinations followed by full recovery.
23. (C) A febrile seizure is a brief (less than 15 minutes),
generalized, symmetric, tonic–clonic
seizure associated with a febrile illness (temperature greater than
38.8°C) without any central
nervous system infection or neurologic cause. An absence (petit
mal) seizure is a brief (2 to 25
seconds) loss of consciousness that can occur multiple times per
day. There is no loss of tone,
and frequently the only observable behaviors are staring or
minor movements such as lip
smacking and semipurposeful movements of the hands. There is
no postictal period. Complex
partial seizures (psychomotor) have varied symptoms including
alterations in consciousness,
unresponsiveness, and repetitive complex motor activities that
are purposeless. Often, at the
beginning of the attack, there is a psychoillusory phenomenon
such as hallucinations, visual
distortions, visceral sensations, or feelings of intense emotions.
Simple partial seizures include
focal motor, adversive, and somatosensory seizures.
Manifestations of these seizures are varied
including hallucinatory, psychoillusory, or complex emotional
phenomena. Children will interact
normally with their environment, with the exception of those
limitations imposed by the seizure.
Following the seizure (minutes to hours), there may be transient
paralysis of the affected body
part.
24. (B) Urinary tract infections (UTIs) are one of the most
common infections in children. Clinical
features of a UTI vary depending upon the age and sex of the
child. In newborns, the most
common symptom is failure to thrive associated with poor
feeding, diarrhea, and vomiting. In
infants, the symptoms may be relatively nonspecific, such as
poor feeding, failure to gain weight,
vomiting, fever, strong-smelling urine, and irritability. As
children grow older, the initial signs
and symptoms become more specific to the urinary tract. In
early infancy, males are two times
more likely than girls to have a UTI. Also, uncircumcised males
are 10 times more likely to be
affected than circumcised males. Escherichia coli is the most
common pathogen for the first UTI
(80%) and of recurrent infections (75%). Other organisms that
cause infections include
Pseudomonas aeruginosa, Proteus, Enterobacter, Klebsiella, and
Enterococcus. An infection
with Staphylococcus saprophyticus, a coagulase-negative
staphylococcus, is primarily seen in
adolescents with a UTI.
25. (B) Dehydration is a common pathophysiologic alteration in
fluid and electrolyte balance in
children. Children are at an increased risk for dehydration
because of their decreased oral intake,
especially when ill, and their higher ratio of surface area to body
weight, promoting significant
evaporative losses. Important clinical features to estimate the
degree of dehydration include
postural blood pressure, changes in heart rate, capillary refill
time, skin turgor and color, lack of
tears, lack of external jugular venous filling when supine,
sunken fontanel (if present), and
altered mental status. This infant was estimated to have mild
dehydration (3% to 5% decrease in
body weight) with decreased tears, slightly longer capillary refill
time (2 to 3 seconds), and
intermittent irritability and restlessness. Severe dehydration
(11% to 15% decrease in body
weight) manifests as markedly decreased skin turgor with
parched or mottled mucous
membranes, absence of tears, tachycardia, capillary refill greater
than 4 seconds, hypotension,
circulatory collapse, and anuria. Moderate dehydration (6% to
10% decrease in body weight)
manifests as decreased skin turgor; dry mucous membranes;
decreased tearing; oliguria; and
normal pulse, blood pressure, and perfusion.
26. (C) Intussusception is the most common cause of intestinal
obstruction between 3 months and 6
years of age. It is twice as common in males than females. It is
caused by intestinal invagination,
usually around the ileocecal valve. The classic presentation is
intermittent severe colicky
abdominal pain with legs drawn up, followed by periods of
comfort or falling asleep. Vomiting
usually occurs in the early phase, which later becomes bilious. A
passage of blood and mucus in
the stool (“currant jelly stools”) occurs in 60% of the cases.
Palpation of the abdomen usually
reveals a sausage-shaped mass in the right upper quadrant. The
classic presentation of pyloric
stenosis is in first-born males of 3 to 6 weeks of age, presenting
with nonbilious projectile
vomiting leading to dehydration with hypochloremia,
hypokalemia, and metabolic alkalosis. A
firm, movable, 2-cm olive-shaped mass (“olive”) is palpable
superior and to the right of the
umbilicus in the midepigastrium. In addition, peristaltic waves
may be visible on the physical
examination. The classic presentation of appendicitis presents
with a period of anorexia
followed by steady periumbilical pain shifting to the right lower
quadrant; nausea and vomiting
is followed by a low-grade fever. Diarrhea (nonbloody and
nonmucous), if it occurs, is
infrequent. Peritoneal signs are present. The incidence increases
with age and peaks during
adolescence. Infective enteritis usually begins with emesis
followed by crampy abdominal pain
of hyperperistalsis. This sequence of symptoms with emesis
preceding pain is an important factor
in distinguishing it from intussusception. Masses are not
palpated with infective enteritis.
27. (C) Dehydration is a common pathophysiologic alteration in
fluid balance in children. The body
has a maintenance fluid requirement to replace daily normal
losses that occur through the skin,
kidney, intestines, and respiratory tract. The following formula
can be used to calculate the usual
amount of fluid a healthy child requires by mouth to maintain
hydration:
100 mL/kg for the first 10 kg of body weight
50 mL/kg for the next 10 kg of body weight
20 mL/kg for the weights above 20 kg
For this question, a 24-kg child would require:
100 mL/kg × 10 kg = 1,000 mL for the first 10 kg
50 mL/kg × 10 kg = 500 mL for the next 10 kg
20 mL/kg × 4 kg = 80 mL for the next 4 kg
Total = 1,580 mL 24 kg
28. (B) The most common etiologic organisms for bacterial
meningitis in children are S
pneumoniae, N Meningitidis, and H influenzae. Because of an
increase in resistant S
pneumoniae, coverage with vancomycin and a third-generation
cephalosporin such as cefotaxime
or ceftriaxone is needed for best coverage. Gentamicin can be
used but, as with all
aminoglycosides, caution is needed regarding toxicity.
Ampicillin, rifampin, and
chloramphenicol are alternative treatments if necessary.
29. (B) Congenital hypothyroidism is one of the most common
disorders tested for in newborn
screening tests, revealing an elevated TSH (thyroid stimulating
hormone) and a decreased
T4(thyroxine). Symptoms suggestive of congenital
hypothyroidism in the neonate include
hypotonia, coarse facial features, hirsute forehead, large
fontanels (anterior and posterior),
widely open sutures, umbilical hernia, protruding/large tongue,
hoarse cry, distended abdomen,
and prolonged jaundice. Signs of congenital hypothyroidism
include lethargy or hypoactivity,
poor feeding, constipation, mottling, and hypothermia.
Congenital adrenal hyperplasia (CAH) is
not universally screened for in the newborn screening test, as it
is included in only 14 of the 50
states. In females with CAH, there may be virilization with
abnormalities of the external genitalia
varying from mild enlargement of the clitoris to complete fusion
of the labioscrotal folds. Signs
of adrenal insufficiency (salt loss) may present in the first few
days of life. Crigler–Najjar
syndrome is not one of the disorders tested for in the standard
newborn screening tests. It is an
inherited disease producing congenital nonobstructive,
nonhemolytic, unconjugated severe
hyperbilirubinemia. The physical findings in this infant do not
correlate with Crigler–Najjar
syndrome. Galactosemia is tested for in the newborn screening
test in nearly all 50 states. The
infant may have symptoms of cataract, hepatomegaly, and
prolonged jaundice. Often, these
neonates have Escherichia coli sepsis, leading to death in the
first 2 weeks of life if not treated
promptly.
30. (A) Caput succedaneum is a result of fluid and blood
accumulation in the occipitoparietal region
of the newborn's scalp due to the vacuum effect of membrane
rupture. A cephalohematoma is a
firm, tense external swelling of the cranium that does not extend
across suture lines because it is
limited to the surface of one cranial bone. It occurs most often in
the parietal area. This
subperiosteal hemorrhage usually is not present at birth, but
develops within the first 24 hours of
life. Craniotabes is a condition caused by the osteoporosis of the
outer table of the involved
membranous bone, generally over the temporoparietal or
parietooccipital areas, creating a “pingpong
ball” sensation when gentle pressure is applied. A subgaleal
hemorrhage is a firm, fluctuant
external swelling of the cranium that does extend across suture
lines and increases in size over
time.
31. (C) Foreign body aspiration into the respiratory tract is
associated with an acute choking or
coughing episode with expiratory wheezing (indicative of a
lower airway obstruction) in
children aged 6 months to 4 years of age. Often, there is a
history of the child playing with small
toys that are commonly aspirated. Asymmetrical physical
findings of decreased breath sounds
and localized wheezing are present with foreign body aspiration.
A positive forced expiratory
chest X-ray shows a mediastinal shift away from the affected
side. Radiolucent foreign bodies
such as plastic toys may not appear on an X-ray, but there will
be evidence of this mediastinal
shift. Asthma is generally characterized by wheezing, but it is
not unilateral nor is it of sudden
onset. A chest X-ray reveals bilateral hyperinflation with
flattening of the diaphragm. Epiglottitis
is a life-threatening upper airway obstructive condition that
presents with a sudden onset of
fever, dysphagia, drooling, and inspiratory retractions with
stridor. A lateral neck X-ray reveals
an enlarged, indistinct epiglottis (“thumb sign”); however, the
chest X-ray is normal. Pulmonary
embolism, rare in children, presents clinically with acute
dyspnea, tachypnea, and tachycardia.
There may be mild hypoxemia, rales, and focal wheezing. Chest
X-rays may be normal, or there
may be a peripheral infiltrate, small pleural effusion, or elevated
hemidiaphragm.
32. (C) An acute salicylate overdose (greater than 150 mg/kg)
will produce symptoms of salicylate
intoxication. Chronic salicylate intoxication occurs with
ingestion of greater than 100 mg/kg/day
for at least 2 days. Salicylates affect most organ systems,
leading to various metabolic
abnormalities. Because salicylates are a gastric irritant,
symptoms of vomiting and diarrhea
occur soon after the overdose, which may contribute to the
development of dehydration.
Salicylates stimulate the respiratory center leading to
hyperventilation and hyperpnea resulting in
respiratory alkalosis and compensatory alkaluria. A
characteristic feature of salicylate
intoxication is the coexistence of a respiratory alkalosis with a
widened anion gap metabolic
acidosis.
33. (C) Generalized tetanus (lockjaw) is a neurologic disease
caused by Clostridium tetani.
Although any open wound is a potential source for
contamination with C tetani, those with dirt,
soil, feces, or saliva are at increased risk. Tetanusprone wounds
contain devitalized tissue,
especially those caused by punctures, frostbite, crush injury, or
burns. Recommendations for
tetanus prophylaxis in a child with a laceration or abrasion
depend upon the number of previous
vaccinations, occurrence of last booster, type of wound (clean or
tetanus-prone), and age of
child. In this case, the patient is older than 7 years and had all of
his previous immunizations;
however, his most recent booster was greater than 10 years ago.
Thus, he should receive an
adult-type diphtheria and tetanus toxoid with acellular pertussis.
In most cases, when tetanus
toxoid is required for wound prophylaxis in a child older than 7
years, the Td instead of tetanus
toxoid alone is recommended so that diphtheria immunity is
maintained. If tetanus immunization
is not up to date at the time of wound treatment, then the
immunization series should be completed
according to the primary immunization schedule. If a child is
younger than 7 years, then the
diphtheria, tetanus, acellular pertussis (DTaP) booster is
indicated, unless there is a
contraindication for pertussis, in which case the diphtheria and
tetanus (DT) booster should be
administered. Tetanus immune globulin (TIG) is recommended
for treatment of tetanus. Under
special circumstances, a patient infected with the human
immunodeficiency virus (HIV) with a
tetanusprone wound should also receive TIG in addition to the
prophylactic vaccine.
34. (D) Normally, primitive reflexes are present at birth and
should not persist beyond the age of 6
months. However, the parachute reflex is a postural response
that normally appears around 7
months of age to coincide with volitional movement and persists
for life. It occurs when an infant
is held prone by the waist over a surface and lowered with the
head downward and extends the
arms and legs as a form of protection. The rooting reflex occurs
when the cheek is stroked on the
infant and they turn his/her head to feed. Galant and Landau
reflex disappear by the age of 2
months (trunk incurvation upon stroking the back) and 6 months
(the baby lifts head and
straightens spine upon being held prone), respectively
35. (A) Gonococcal ophthalmia neonatorum presents as a
unilateral or bilateral serosanguineous
discharge and then within 24 hours the discharge becomes
mucopurulent, followed by
conjunctival injection and edema of the eyelids. The usual
incubation period for Neiserria
gonorrhea is 2 to 5 days; however, the infection may be present
at birth or delayed greater than 5
days if there has been instillation of silver nitrate prophylaxis. A
presumptive diagnosis is made
by the demonstration of gram-negative intracellular diplococci
on Gram stain. Definitive
diagnosis is made by culture. Following a positive Gram stain
and pending culture results,
treatment should be promptly initiated with ceftriaxone (50
mg/kg/24 hours IV or IM for one dose
not to exceed 125 mg), a third-generation cephalosporin with
good coverage for gram-negative
bacteria. An alternate drug is cefotaxime (100 mg/kg/24 hours
IV or IM every 12 hours for 7
days or 100 mg/kg as a single dose), which is also a third-
generation cephalosporin. Although
erythromycin drops (0.5%) are used prophylactically for N
gonorrhea, this is not an effective
treatment. Gentamicin would be used for Pseudomonas, and
Chlamydia is treated with
erythromycin. Cephalexin as a first-generation cephalosporin
does not have coverage for gramnegative
bacteria.
36. (B) The most likely diagnosis in this patient is mumps. It is
endemic in most unvaccinated
populations. The onset is characterized by pain and swelling in
one or both parotid glands. The
pain can be exacerbated by tasting sour liquids such as lemon
juice. An elevated serum amylase
level is common and coincides with the parotid swelling.
Unilateral, rarely bilateral, nerve
deafness is a complication of mumps that may be transient or
permanent. Other complications
include meningoencephalomyelitis, orchitis, epididymitis,
pancreatitis, arthritis, and rarely
thyroiditis and myocarditis.
37. (A) Café au lait spots are brown macules that may be found
on any part of the body. The
presentation of six or more spots greater than 1.5 cm is a sign of
neurofibromatosis, a genetic
disorder that results in neurofibromas that can develop in any
organ/tissue system. Miliaria are
blocked sweat gland ducts that are commonly found on the face,
scalp, or intertriginous areas.
Vernix caseosa is a normal finding in newborns and is a whitish,
greasy layering on the body— it
decreases as an infant comes to full term. Subconjunctival
hemorrhages are a common finding in
infants secondary to birth trauma.
38. (B) The Apgar score assesses the newborn at 1-minute and
5-minute intervals to determine the
need for resuscitative care. The infant is evaluated by heart rate,
respiratory effort, muscle tone,
response to catheter in nostril, and color, and each is rated on a
scale of 0, 1, or 2 for a total
score of 10. The heart rate is scaled 0–2 for absent, less than 100
bpm (slow), and greater than
100 bpm; respiratory effort of absent, slow/irregular, and good
crying. Muscle tone scale (0–2)
consists of limp, some flexion, and active motion; response to
catheter stimulation (0–2) is
scaled no response, grimace, and cough/sneeze. Finally, color is
scored 0–2 for blue/pale, body
pink with blue extremities, and completely pink.
39. (C) Ventricular septal defect, a hole between the two
ventricles, can be cyanotic or acyanotic
based on the size of the defect, and accounts for 30% of cases of
congenital heart disease. Atrial
septal defect occurs in approximately 10% of congenital heart
disease cases. Transposition of
great vessels is an embryonic malformation resulting in the aorta
arising from the right ventricle
and the pulmonary artery arising from the left ventricle. It is
responsible for about 10% of all
congenital malformations. Tetralogy of Fallot, consisting of a
ventricular septal defect,
overriding aorta, pulmonic/subpulmonic stenosis, and right
ventricular hypertrophy, accounts for
10% of congential heart disease.
40. (D) The immunization schedule is developed biannually by
the Centers for Disease Control and
Prevention. Assuming that the child has had the appropriate
immunizations at the regularly
scheduled examinations, the recommended immunizations at the
4- to 6-year-old range are the
DTaP (diphtheria, tetanus, acellular pertussis), IPV (inactivated
polio), and the MMR (measles,
mumps, and rubella). The hepatitis series should have been
completed by the age of 6 months and
the Haemophilus influenzae type B (Hib) should be completed
by the age of 12 to 15 months.
Varicella is given from 12 to 18 months and again from 4 to 6
years; the PCV (pneumococcal)
should be finished by 12 to 15 months.
41. (A) There are only two known absolute contraindications to
breastfeeding: tuberculosis of the
mother and galactosemia of the infant. The highly contagious
nature of tuberculosis makes the risk
greater than the benefit, and infants with galactosemia are
unable to digest any lactose due to an
enzyme deficiency. Infants of mothers in a methadone program
may be breastfed as long as the
mother's dose is less than 40 mg. While nicotine is transmitted
in breast milk and is therefore
strongly discouraged, it is not an absolute contraindication. As
long as a breastfed infant with
cystic fibrosis is maintaining normal growth with supplemented
pancreatic enzymes,
breastfeeding is encouraged.
42. (B) Failure to thrive is diagnosed in infants younger than the
age of 6 months with a decrease in
growth velocity that results in a decrease in two major percentile
lines on the growth chart. In the
case of this patient, she was initially in the 50th percentile and
crossed the 25th and 10th
percentile and fell into the 5th percentile. Failure to thrive is also
known as growth deficiency
and may also be diagnosed if the child is younger than 6 months
and has not grown for two
consecutive months or if a child is older than 6 months and has
not grown for 3 consecutive
months. Growth hormone deficiency/dwarfism may present with
decreased growth velocity later
in childhood; the drop in percentiles is grossly below the 5th
percentile mark. Lactose
intolerance presents with varying gastrointestinal symptoms
without the marked decrease in
weight. Beckwith–Wiedemann syndrome consists of
macrosomia, macroglossia, and
omphalocele and they are at increased risk for malignancies,
hypoglycemia, and dysmorphism
(usually of the ears).
43. (C) This patient has presented with classical findings of
Reye syndrome—upper respiratory
infection followed by unresponsiveness. Reye syndrome is
usually preceded by an upper
respiratory tract illness, which progresses into vomiting, strange
behavior, stupor, and coma.
Liver function tests (LFTs) will be markedly elevated (without
jaundice); however, the serum
bilirubin and alkaline phosphatase are normal. Unresponsive
patients who have a spinal tap will
show no cells in the CSF and glucose may be low with increased
CSF pressure. If arterial blood
gases are ordered, they will show a mild respiratory alkalosis
and metabolic acidosis. A liver
biopsy will show little inflammatory changes with diffuse
microvesicular steatosis and absent
glycogen from the hepatocytes. The mitochondria of the
hepatocytes are large and polymorphic
with decreased matriceal density. Treatment for patients with
Reye syndrome is largely
supportive—specifically decreasing cerebral edema. There is no
place for antibiotics or
steroids. The liver will fully recover if the cerebral edema is
decreased.
44. (B) Wilson disease is a result in a genetic mutation on
chromosome 13 that causes decreased
bile excretion of copper and results in accumulation of copper
by the liver, specifically the
ceruloplasmin. The build-up of copper causes damage to the
liver, basal ganglia, and other
tissues. Physical examination shows jaundice,
hepatosplenomegaly, Kayser–Fleischer rings (a
brown band at the junction of the iris and cornea under slit-
lamp), and neurologic manifestations
later in the disease process. Laboratory tests show marked
decrease in ceruloplasmin of the
liver, anemia, hemolysis, and severely elevated bilirubin with
decrease alkaline phosphatase.
Urinalysis shows severe elevation in copper excretion,
glycosuria, and aminoaciduria. Liver
biopsy is conclusive with evidence of copper greater than 250
μg/g of dry tissue. Treatment
requires copper chelation with D-penicillamine or trientine
hydrochloride. Liver transplant may
be required with noncompliance and in acute fulminant disease.
Copper chelation is continued
for life with the addition of zinc (decrease copper absorption)
and vitamin B6 (decrease optic
neuritis). Genetic screening of siblings and future children
should be strongly encouraged. Alphainterferon
therapy is mainly used to treat hepatitis patients. There is no
place for steroids in
therapy and protease inhibitors are antiviral medications that are
typically used in HIV patients.
45. (A) The patient's presentation is consistent with findings of
coarctation of the aorta. The
pathognomonic finding in coarctation is decreased or absent
femoral pulses. However, the
majority of children show no signs of coarctation in infancy and
develop signs and symptoms
during childhood, most notably unequal pulses and blood
pressure between arms and legs (arms
lower than legs). In addition, a grade II/VI ejection murmur is
heard at the aortic area and left
sternal border that radiates into the left axilla and left back.
Chest X-ray shows a normal-sized
heart, a prominent aorta, indents at the level of the coarctation,
and a dilated poststenotic segment
resulting in the “figure 3” sign. Scalloping or notching of the
ribs is due to enlargement of the
intercostal arteries. Echocardiography is used to directly
visualize the coarctation and estimate
the obstruction. Asymptomatic infants and children are
encouraged to have corrective surgery
prior to age 5, after which they are at increased risk for
myocardial dysfunction and
hypertension, and require exercise testing prior to participation
in aerobic activities. The bootshaped
heart is seen in patients with tetralogy of Fallot secondary to
right ventricular
hypertrophy; the narrowed mediastinum finding with “egg on a
string” is typically seen in
patients with transposition of the great vessels.
46. (D) Viral croup usually presents with cough that may sound
like a dog or a seal barking. The
patients are usually afebrile and also present with stridor either
at rest, in severe cases, or when
agitated, in mild cases. In addition, the patient may be cyanotic
and have retractions and acute
shortness of breath. Radiologic examination of the neck shows
subglottic narrowing with a
normal epiglottis, “steeple sign.” However, X-rays are
usually not indicated in patients with the
common presenting symptoms. Treatment for viral croup is
mainly symptomatic, especially in
mild cases consisting of oral hydration and mist therapy. Severe
cases (stridor at rest) call for
oxygen in patients who have desaturated, and nebulized racemic
epinephrine and
glucocorticoids. Dexamethasone as an intramuscular injection or
oral as a one time dose is
effective in alleviating symptoms, decreasing the need for
intubation, and decreasing hospital
stays. Inhaled budesonide is also effective in decreasing hospital
stays and improving symptoms,
but dexamethasone is more cost-effective. Patients who are
unable to be stabilized need airway
maintenance either by intubation with endotracheal tube or by
tracheostomy if intubation fails.
Because it is a self-limiting disorder, unless there is a secondary
infection most children recover
in a few days.
47. (A) Cystic fibrosis (CF) is a major cause of gastrointestinal
and pulmonary morbidity in children
due to mutations in the CF genes. The mutations lead to a
deficiency in cystic fibrosis
transmembrane conductance regulator protein that controls
movement of salt and water into and
out of epithelial cells and results in production of abnormally
thick mucus. About 15% of patients
with CF present with meconium ileus at birth. This is typically
treated with enema for
disimpaction and rarely surgery. Approximately half of the
infants with CF will present with
failure to thrive, which is diagnosed by lack of growth for 2
consecutive months in patients
younger than 6 months of age. They may also present with
respiratory compromise. However, not
all patients present in childhood. Diagnosis of CF is confirmed
by a sweat chloride level above
60 meq/L or with genetic testing. Treatment for patients with CF
is mainly symptomatic therapy
for obstructions of the digestive and respiratory tract. In
addition, there is pancreatic enzyme
supplementation to aid in digestion and vitamin and calorie
supplementation for deficiencies in
the diet. Gene therapy is now being looked at for future
treatment. Intussusception (telescoping of
the small intestine) typically presents in an infant with
paroxysmal abdominal pain, vomiting, and
diarrhea that may progress into bloody stools. Volvulus is
normally the result of intestinal
malrotation that causes occlusion of the superior mesenteric
artery and eventual bowel necrosis.
Infants typically present within 3 weeks of life with bile-stained
vomiting and bowel obstruction.
Wilson's disease is the defect in the ability to excrete copper in
the bile that results in
accumulation of copper in the liver.
48. (B) Cyanotic heart lesions are a result of a right-to-left shunt.
These include tetralogy of Fallot,
pulmonary atresia with and without ventricular septal defect,
tricuspid atresia, hypoplastic left
heart syndrome, and transposition of the great arteries. The
right-to-left shunt results in
deoxygenated blood reaching the left ventricle, aorta, and
systemic arteries. The decreased
oxygen in the blood results in decreased oxygen to the tissue and
subsequently causes cyanosis.
Atrial septal defect, ventricular septal defect, atrioventricular
septal defect, and patent ductus
arteriosus most commonly present with a left-to-right shunt.
49. (A) This infant is presenting with signs and symptoms of
pyloric stenosis. Infants typically have
vomiting (projectile at times) after every feeding and it normally
starts between the age of 2 and
4 weeks. The infant nurses fervently and is hungry. In addition,
there may be dehydration,
constipation, weight loss, and apathy. Abdomen may be
distended with gastric peristaltic waves.
Occasionally, an olive-sized mass can be felt in the right upper
quadrant with deep palpation
after the child has vomited. Vomitus is typically nonbilious.
Diagnosis is confirmed by an upper
gastrointestinal series with delayed gastric emptying, enlarged
pyloric muscle, and characteristic
semilunar impressions on the gastric antrum. In addition, an
ultrasound is needed to verify the
hypertrophic muscle. The treatment of choice for these patients
is pyloromyotomy, which can be
done laparoscopically. These patients make full recoveries and
have an excellent prognosis.
50. (D) Celiac disease or gluten enteropathy typically presents
with diarrhea episodes in the first 6
to 12 months of life—when whole grains are first fed. Therefore,
in strictly breastfed babies,
symptoms may not be noticed until solid foods are begun. The
diarrhea is usually intermittent at
first and then typically progresses into pale, greasy, foul-
smelling, frothy stools. Additional
symptoms may be constipation, vomiting, and abdominal pain,
which may lead the clinician to
think of intestinal obstruction. Other findings may be failure to
thrive, anemia, and vitamin
deficiencies. Stool sample demonstrates excessive fecal fat
excretion. Blood tests show
hypoproteinemia and impaired carbohydrate absorption.
Intestinal biopsy is the diagnostic test of
choice for celiac disease. Results show shortened celiac mucosa,
absent villi, lengthened crypts
of Lieberku$$$hn, plasma cell infiltration of the lamina propria,
and intraepithelial lymphocytes.
Treatment consists of dietary restriction of gluten—wheat, rye,
and barley. Steroids are given on
an as needed basis. Sweat chloride testing is utilized in patients
suspected of cystic fibrosis.
Gastrin level is taken in patients suspected of Zollinger–Ellison
syndrome, and RAST
(radioallergosorbent assay test) is used in patients to determine
different environmental-type
allergens
51. (B) The most likely diagnosis for this patient is
poststreptococcal glomerulonephritis. The
diagnosis is supported by a documented culture of group A beta-
hemolytic streptococcus
infection. If a culture is not available, like of the patient in this
scenario, the clinician can order
an antistreptolysin O titer. Antistreptolysin is an enzyme
released by group A streptococcus and
is elevated for up to 1 month after strep infection.
Glomerulonephritis presents with gross
hematuria with or without edema. Hypertension, proteinuria,
ascites, and headache may also be
present. Treatment with antibiotics is useful if infection is still
present, and, if necessary,
symptomatic treatment for renal failure is done with
hemodialysis. Symptoms typically resolve
within a few weeks. The monospot is used to diagnose
infectious mononucleosis. Renal biopsy
could be performed on extreme cases of glomerulonephritis but
is not typically necessary.
Immunoglobulin electrophoresis would be utilized in patients
suspected of having
immunoglobulinopathies or IgA-mediated glomerulonephritis.
52. (D) Patent ductus arteriosus (PDA) is an isolated
abnormality that occurs in infants. The ductus
arteriosus is a normal fetal vessel that joins the aorta and the
pulmonary artery and spontaneously
closes after 3 to 5 days. Lack of closure results in the audible
murmur that is “machinelike” and
maximal at the second intercostal space (ICS), at the left sternal
border (LSB), and inferior to the
clavicle. It is typically a pansystolic murmur with bounding
pulses and a widened pulse pressure.
There is also a paradoxical splitting of S1 and S2.
Echocardiography confirms the PDA, the
direction and degree of shunting, and the presence of lesions for
which the PDA is needed to
keep. If there are no other cardiac malformations requiring the
PDA, then if the PDA is large,
surgery should be completed before 1 year of age. Symptomatic
PDAs that are relatively small
may be closed with indomethacin in preterm infants. The
murmur heard in atrial septal defect
(ASD) usually is an ejection type, systolic murmur heard best at
the LSB, second ICS with a
wide, fixed S2 and normal pulses. Ventricular septal defect
(VSD) presents with a harsh,
pansystolic murmur heard best at the third and fourth ICS. With
increasing size of the VSD,
heaves, thrills, and lifts are present along with radiation
throughout the chest. Tetralogy of Fallot
presents with a rough ejection, systolic murmur heard best at the
LSB and the third ICS with
radiation to the back.
53. (C) Hypertrophic cardiomyopathy in adolescence is
typically due to familial hypertrophic
cardiomyopathy with an incidence of 1:500. Many patients are
asymptomatic until a sporting
event, which may cause symptoms, specifically sudden cardiac
death. Examination may
demonstrate a palpable or audible S4, an LV (left ventricular)
heave, systolic ejection murmur
(may need to stimulate cardiac activity), and/or a left precordial
bulge. Echocardiography is the
gold standard for diagnosis but family history should be
assessed. Stress testing is indicated to
assess for ischemia and arrhythmias. Strenuous activities are
prohibited for these patients. The
other cardiomyopathies (dilated and restrictive) are next but are
not as common. Congenital
structural abnormalities of the coronary arteries are the next
most common cause. Valvular
disorders, including surgically repaired aortic stenosis, are
typically not causes of sudden death,
but these patients should be screened for symptoms and stress
tested as necessary.
54. (A) This patient presentation describes epiglottitis. Although
there is a decreased incidence of
epiglottitis secondary to the introduction of the vaccine for
Haemophilus influenzae type B
(Hib), patients still present with sudden onset of fever,
dysphagia, muffled voice, drooling,
cyanosis, inspiratory retractions, and soft stridor. The patients
are usually sitting in a tripod
position to aid their breathing. Recognition of the classic
symptoms needs to be immediate to
stabilize the patient's airway, as these patients will
decompensate into respiratory failure quickly.
In the event that there is time, a lateral neck X-ray will show the
“thumb sign,” which is an
enlarged, undistinguished epiglottis. Treatment for the patient
requires intubation for airway
stabilization, blood cultures and throat/epiglottis cultures, and
antibiotic coverage for H.
influenzae. The steeple sign is seen in patients with croup and is
due to a subglottic narrowing.
The “figure 3” sign is seen in patients with coarctation of the
aorta. The “Scottie dog” sign is
seen in oblique lumbar films and is a normal finding
representing the pars interarticularis. Its
absence signifies spondylolysis.
55. (C) With an incidence of 1:3,000 to 1:4,000 Caucasians,
cystic fibrosis is the most common
lethal genetic disorder in the United States. While trisomy 21
(Down syndrome) is one of the
most common genetic disorders with 1:500 newborns, it is
typically not a fatal disease. It is
characterized with mental retardation and physical
malformations. Trisomy 13 is a fatal trisomy,
with most deaths occurring in early infancy or by the age of 2,
but its incidence is approximately
1:12,000 live births. Neurofibromatosis, a genetic disorder of
typical autosomal dominant
inheritance, occurs in approximately 1:3,000 live births. Most
affected children have the skin
lesions (café au lait macules or neurofibromas) and other minor
problems.
56. (B) Status epilepticus is a medical emergency and is defined
as seizure activity that lasts a
minimum of 30 minutes. This results in hypoxia, acidosis,
cerebral edema, and structural damage.
In addition, fever, respiratory depression, hypotension, and
death may occur. There are both
convulsive and nonconvulsive types of status epilepticus.
Because of its emergency status and
potential complications, the clinician needs to initiate the ABCs
(airway, breathing, circulation).
Therefore, the first line of treatment is to establish and maintain
an airway, oxygen is next, and
then circulation, which encompasses pulse, blood pressure, and
IV access. Once the IV is
established, the orders should be for administering glucose-
containing fluids and IV drug therapy
with diazepam, lorazepam, or midazolam as well as administer
phenytoin and phenobarbital.
Arterial blood gases should be ordered and any abnormalities
should be corrected appropriately.
Finally, the clinician should determine the underlying cause:
trauma, structural disorder,
infection, lactic acidosis, toxins, and uremia. Maintenance drug
therapy is necessary until the
underlying cause is determined and rectified.
57. (D) Guillain–Barré syndrome is most likely due to a delayed
hypersensitivity with T-cell–
mediated antibodies to mycoplasma and viral infections (CMV,
EBV, hepatitis B, campylobacter
jejuni). The patients may mention a nonspecific respiratory or
gastrointestinal infection 1 to 2
weeks prior to symptoms. Complaints may be paresthesias,
weakness in bilateral lower
extremities with occasional ascension into the arms, trunk, and
face, and rarely ataxia and
ophthalmoplegia in the Miller–Fisher variant. Examination
findings demonstrate symmetric
flaccid weakness, with impairment of position, vibration, and
touch in the distal portions of the
extremities. If a spinal tap is performed, it may show few
polymorphonuclear neutrophils with
high protein and normal glucose. EMG is positive for decreased
nerve conduction. Laboratory
tests may show high titers of suspected infections or active
infection of hepatitis/bacterial
pathogens. Guillain-Barré is normally a self-limiting disorder
within a few weeks, unless there
are issues with respiratory depression. Poliomyelitis is
secondary to polioviruses and presents
with fever, paralysis, meningeal signs, and asymmetrical
weakness. Botulism secondary to
infection with Clostridium botulinum in older children presents
with blurred vision, diplopia,
ptosis, choking, and weakness. In infants, botulism presents as
constipation, poor suck and cry,
apnea, lethargy, and choking. Tick-bite paralysis presents with
rapid onset with ascending flaccid
paralysis reaching upper extremities in a couple of days of onset
and patients often present with
paresthesia and pain. Finding of a tick is usually confirmatory
for these patients.
58. (B) Scoliosis is defined by lateral curvature of the spine with
rotation of vertebrae and is
typically located in the thoracic or lumbar spine in the right or
left directions. Idiopathic
scoliosis most commonly presents as a right thoracic curve in
females from 8 to 10 years of age.
Scoliosis is typically asymptomatic unless curvatures are so
severe that there is pulmonary
dysfunction or there is an underlying disorder (bone or spinal
tumor) that is causing the scoliosis.
X-rays need to be taken of the entire spine to help determine the
degree of curvature. Treatment
modalities are based on the degree of curvature: 20° or less
does not normally require treatment;
20° to 40° is an indication for bracing in an immature child;
and 40° and greater is resistant to
bracing and requires surgical fixation with spinal fusion, which
is best done at special centers.
59. (D) Legg–Calvé–Perthes disease is also known as avascular
necrosis of the proximal femur. It
typically occurs in children between 4 and 8 years old and
persistent hip pain is the main
symptom. On examination, the clinician notices a limp and/or
limitation of motion of the affected
hip. Radiologic examination demonstrates the necrosis with
effusion and joint space widening
with a negative aspirate. Treatment involves surgical hip
replacement. Slipped capital femoral
epiphysis (SCFE) is due to the displacement of the proximal
femoral epiphysis owing to
disruption of the growth plate. The head is normally displaced
medially and posteriorly relative
to the femoral neck. It typically occurs in adolescence,
specifically obese males, and can also be
associated with hypothyroidism. SCFE usually occurs after
direct trauma to the hip or a fall.
Patients complain of vague symptoms at first that progress into
pain of the hip or of the knee. On
examination, there is decreased internal rotation of the hip that
can be confirmed by lateral X-ray
of the hip. Septic hip arthritis is not common in children
between the age of 5 and 12 years. The
legs are held in external rotation to minimize pain and will have
a positive aspirate.
Osteochondritis dissecans typically presents in the knee, elbow,
and talus and is characterized by
a wedge-shaped necrosis of bone.
60. (B) Gamekeeper thumb is a result of damage to the ulnar
collateral ligament during forced
abduction of the metacarpophalangeal joint, an injury that is
most commonly seen in skiers. An
avulsed fragment may or may not be seen on radiologic
examination. If it is smaller than 2 mm,
there is no fragment, a thumb spica cast can be used as seen in
patients with no fragment. If the
fragment is larger than 2 mm, surgery is required. Mallet finger
is an avulsion of the extensor
tendon and occurs in ball-handling sports. Boxer fracture is a
distal neck fracture of the 5th
metacarpal. Scaphoid fractures are due to hyperextension of the
wrist injuries and present with
pain in the anatomic snuffbox and swelling.
61. (A) Nursemaid elbow is the subluxation of the radial head
due to a child or infant being lifted or
pulled by the hand. The patient will present with the elbow
pronated and painful and he or she
will not bend the elbow. During the radiologic examination, the
dislocation is usually reduced by
placing the elbow in full supination and moving it slowly from
full extension to full flexion. This
typically provides immediate relief of pain and a sling may be
given for comfort for a couple of
days. Otherwise, X-rays are normal. Child protective services
should be considered if this is a
recurrent problem or if there are other associated signs and
symptoms of battery. There is no
need for orthopedic referral unless reduction is not commonly
done in your setting.
Immobilization of the elbow is not recommended, because the
patient then may have to recover
from frozen shoulder.
62. (A) This patient is presenting with signs and symptoms of
primary nocturnal enuresis, which is
the wetting only at night during sleep without any sustained
period of dryness. It is mainly
considered a parasomnia occurring in deep sleep. The incidence
of enuresis is higher in boys, is
typically related to a developmental delay, and most children
become continent by adolescence.
Patients need to be tested for structural abnormalities and
infections, in addition to neurologic
diseases, diabetes mellitus and insipidus, and seizure disorders.
Treatment includes limiting
liquids at bedtime and routine bathroom training during the day.
If these are unsuccessful, the next
option is a bed-wetting alarm. This device is attached to the
child's undergarment and vibrates
when the child is wet to arouse the child to be aware of their
need to urinate. If the alarm is
unsuccessful, then the next step is medication—DDAVP
(desmopressin acetate) or imipramine.
63. (D) Nephroblastoma also known as Wilms tumor typically
presents with an asymptomatic
abdominal mass noticed by the parent or an increasing size of
the abdomen. On examination, the
mass feels smooth and firm, is well defined, and usually does
not cross the midline. Gross
hematuria may be present, but rare, and some patients have
microscopic hematuria when tested.
Wilms tumor accounts for approximately 5% of cancers in
children younger than 15 years. Wilms
tumor arises from the kidney and the average age at diagnosis is
4 years. Ultrasound and CT of
the abdomen can be used to confirm the presence of an intra-
abdominal mass. Treatment includes
exploratory abdominal surgery for removal and staging with a
mixture of chemotherapy.
Intussusception (telescoping of the small intestine) typically
presents in an infant with
paroxysmal abdominal pain, vomiting, and diarrhea that may
progress into bloody stools.
Volvulus is normally the result of intestinal malrotation that
causes occlusion of the superior
mesenteric artery and eventual bowel necrosis. Infants typically
present within 3 weeks of life
with bile-stained vomiting and bowel obstruction.
64. (C) Infant girls presenting with signs of precocious puberty
need to be screened for congenital
adrenal hyperplasia (CAH). CAH most commonly presents with
pseudohermaphroditism in
females—urogenital sinus, enlarged clitoris, or other signs of
virilization. In males, there tends to
be isosexual precocity in older males and salt-losing crisis in
infant males. Both children show
increased linear growth and skeletal maturation. The most
common type of CAH is a deficiency
in the enzyme 21-hydroxylase and laboratory tests demonstrate
increased urinary and plasma
androgens (DHEA, androstenedione). There may be elevated
progesterone, but typically there is
no effect on estrogen. There is also decreased aldosterone and
elevated urinary ketosteroids.
There is also no effect on the levels of leuteinizing hormone or
follicle-stimulating hormone.
Treatment usually involves glucocorticoids, mineralocorticoids,
and reconstructive surgery, if
needed.
65. (A) Growth hormone (GH) deficiency is defined as a
decreased growth velocity, delay in
skeletal maturation, absence of other explanations for poor
growth (lack of intake), and
laboratory tests demonstrating decreased GH secretion. Etiology
of GH deficiency can be
congenital, genetic, acquired, or idiopathic, which is the most
common. Infants usually have a
normal birth weight and may have a slightly decreased length. In
addition, most infants present
with other endocrine deficiencies like hypoglycemia,
hypothyroidism, and/or adrenal
insufficiency. Children may present with truncal adiposity
because growth hormone promotes
lipolysis. Serum GH or intrinsic growth factor levels may or
may not be decreased. In patients
who do not have a demonstrated decrease in these hormones, a
trial period with GH is indicated.
These patients and positive GH-deficient patients receive a
once-daily subcutaneous injection of
recombinant human GH. Congenital hypothyroidism typically
presents with short stature
(typically noted after the 4-month newborn visit), delayed
epiphyseal development, delayed
closure of fontanelles, and retarded dental eruption in addition to
other signs of hypothyroidism.
Cushing disease typically presents with truncal adiposity with
thin extremities, muscle wasting,
decreased growth rate, and moon facies. Laboratory results
show elevated adrenocorticosteroids
both in urine and serum, hypokalemia, eosinopenia, and
lymphocytopenia. Typically, in patients
younger than the age of 12, Cushing disease is secondary to
administration of ACTH or
glucocorticoids. Congenital adrenal hyperplasia typically
presents with pseudohermaphroditism
in females or salt-losing crisis in males with or without
isosexual precocity. There is an
increased linear growth and advanced skeletal maturation.
66. (C) In patients with idiopathic thrombocytopenic purpura,
treatment options should be initiated
when platelet counts fall below 20,000, regardless of whether
there is active bleeding or not.
Without active bleeding the treatment options include
prednisone 2–4 mg/kg/24 hours for 2
weeks; IV immunoglobulin 1 g/kg/24 hours for 1 to 2 days, or
IV anti-D 50–75 μg/kg/dose for
Rh-positive patients. Splenectomy is indicated for life-
threatening bleeding. There is currently no
indication for platelet transfusion and none of the above
treatments are considered optimal,
because in the majority of children, it will resolve on its own
within 6 months.
67. (D) This patient is presenting with signs of congestive heart
failure. The most common causes of
heart failure in children/adolescents are due to acquired heart
disease. Congenital heart diseases,
such as malformations of the heart— patent ductus arteriosus and
ventricular septal defects, are
the most common causes of heart failure in infants–toddlers, and
are second to fluid overload in
neonates.
68. (D) This patient has the classic presentation of erythema
multiforme major or Stevens–Johnson
syndrome. The most common causes in children of erythema
multiforme are medications and
Mycoplasma pneumoniae. Of the antibiotics listed, the one most
commonly causing Stevens–
Johnson syndrome is sulfonamide followed by penicillin and
tetracycline. The most common
medications causing SJS in children are nonsteroidal anti-
inflammatory drugs.
69. (A) Infection with Human parvovirus B19 (also known as
fifth disease) resulting in the slapped
cheek appearance, can also cause aplastic anemia. This is
because the virus infects the
precursors of erythrocytes and halts erythropoiesis. Recovery is
typically spontaneous with an
occasional transfusion for severe anemias. (Scott, 2006, pp. 701-
702; Levin and Weinberg 2009,
pp. 1100-1101)
70. (D) As this patient is presenting with signs and symptoms of
herpes zoster within the appropriate
time frame for antiviral treatment, the treatment for this patient
would be oral acyclovir. NSAIDs
may help with the pain associated from zoster but will not
hasten the length of the course of the
virus as acyclovir will. Varicella-Zoster immunoglobulin
(VZIG) is indicated for prophylaxis in
exposed individuals who are immunocompromised.
71. (D) Munchausen syndrome by proxy is when the
parent/caregiver is causing or complaining of
signs and symptoms of illnesses in his/her children. While it is a
form of child abuse and should
be treated as such, it is also considered a psychiatric disorder
where the parent/caregiver is
desiring to be in the sick role. The most common signs or
symptoms that should raise the level of
suspicion for Munchausen syndrome by proxy are: recurrent
polymicrobial sepsis, recurrent
apnea, chronic dehydration, or other unexplained symptoms like
vomiting, diarrhea, seizures,
failure to thrive, and hypoglycemia. The remaining signs are
seen in classical physical child
abuse.
72. (A) Menomune is a tetravalent vaccine that is indicated for
prevention of meningococcemia
caused by the bacterium Neisseria meningitides. Menomune is
indicated for patients between 11
and 12 years of age and at 15 years of age. It is also indicated
for college freshmen in
dormitories, military recruits, microbiologists working with the
bacterium, persons with
complement deficiency and functional or anatomic asplenia, and
for those traveling to countries
with endemic disease. Guillain–Barré is a rare complication of
the Menomune vaccine, and if a
patient has a history of developing it, is the only relative
contraindication other than a known
reaction to a previous administration of the vaccine rubber latex
and diphtheria toxoid severe
allergic reaction
73. (A) The most likely diagnosis is pediculosis. This parasitic
infestation is most commonly seen in
the young school-aged child, and more often in female and
Caucasian children. The pediculosis
louse lives in the hair and on the scalp and intermittently
“bites” into the skin to feed. Discrete
urticarial papules or erosions may arise at the bite site. By
visualizing the live louse on the scalp,
or in the hair, one can easily make the diagnosis. However, the
louse may be difficult to see, as it
is only 1 to 3 mm in size. Otherwise, nits, or the casings of the
eggs laid by the louse, can often
be seen on the proximal portion of the hair shaft. The nit adheres
to the hair shaft and is often
difficult to remove. Brown nits are representative of current
infestations and white nits past
infestations. Treatment of head lice can be difficult due to the
increasing resistance to some of the
current treatment options. First-line treatment includes
permethrin (5%) and permethrin-based
products. Secondary treatment options for resistant infestations
may include Malathion (0.5%).
Regardless of treatment, viable ova should be removed by
combing the patient's wetted hair with
a finely toothed comb until all are removed. Ketoconazole
cream and tar-based shampoos are
utilized in fungal and seborrheic dermatitis infections. Silver
sulfadiazine cream is a topical
antibiotic.
74. (B) Scabies, Sarcoptes scabiei, is the most common
arthropod infestation of children, and it is
highly contagious. However, its presentation varies widely and
is dependent on the child's age,
duration of the infestation, and immune status. Most often, the
presenting complaint is severe
intermittent itching. The linear papule or burrow commonly
associated with scabies is often
difficult to identify. Instead, most children will present with
eczematous eruptions of red,
excoriated papules and nodules. Usually, the distribution of the
papules are the most diagnostic
finding, and may include the web spaces of the fingers and toes,
axillae, umbilicus, groin, penis,
and the instep of the feet. Usually, in older children and adults,
the face and scalp are spared. The
treatment for scabies is a 12-hour application of permethrin 5%
lotion. In addition, the parents
and all caregivers should be treated at the same time. Clothing
and bedding should be washed
and dried (heat kills scabies). The family should also be
educated in the treatment and prevention
of future infestations. Moreover, they should be advised that the
itching associated with scabies
could persist for 7 to 14 days after successful treatment.
Pediculosis is an infestation of louse in
the hair. Tinea corporis is a fungal infection of the torso or
“ring worm” and presents with annual
scaly plaques with central clearing and pustules. Herpes simplex
typically presents with grouped
vesicles on erythematous base and is painful. It typically is
located in the lips, eyes, cheeks, or
hands of children.
75. (D) Seborrheic dermatitis is common in all age groups. In
infants, this inflammatory skin disease
is often manifested as thickened, yellowish white, scaly, waxy
appearing skin of the scalp and
commonly involves the postauricular areas and the forehead.
The more common name is “cradle
cap.” Cradle cap is a self-limiting disease of infants and
resolves by the child's first birthday. In
all ages, the scalp scale can be treated by shampooing with zinc
pyrithione (Head and
Shoulders), selenium sulfide 1% to 2.5%, salicylic acid (Tsal),
or ketoconazole (Nizoral). The
primary lesion in lichen planus presents on the flexor surfaces
and is characterized by pruritic
papules that are polygonal and flat-topped. Pityriasis rosea
typically presents with the “herald
patch” that is a solitary pink, round patch with some central
clearing typically found on the torso.
The rest of the eruption is described as papulovesicular and
develops a Christmas tree pattern.
Contact dermatitis usually presents with red patches and plaques
with scales and is localized to
the area exposed to the irritant.
76. (B) This presentation is typical for granuloma annulare,
which is a benign skin disorder, and
treatment is not warranted. It is most commonly seen in children
aged 6 to 10. The red to brown
lesions are annular or circinate. These asymptomatic lesions are
often confused with tinea
corporis. The lesions will disappear on their own over a couple
of years.
77. (D) Liquid nitrogen is the treatment of choice for a single
isolated wart. Forty percent salicylic
acid in a plaster application is the most effective treatment of
large and painful warts.
Electrosurgery, burning laser surgery, and other destructive
treatments should be avoided
because of the potential for scarring and subsequent problems
often associated with scars, as
well as the possible recurrence of the wart after destructive
treatment.
78. (B) Topical keratolytic agents applied to the skin either as a
single, once a day agent or in
combination regime (retinoic acid cream, azelaic acid, and
adapalene) once a day in the evening
and benzoyl peroxide gel in the morning, will control
approximately 80% to 85% of cases of
adolescent acne. When treating inflammatory acne, papular or
pustular, a daily topical antibiotic
such as tetracycline, minocycline, or erythromycin can be used
in addition to a daily keratolytic.
The oral retinoid, 13-cis-retinoic acid (isotretinoin), Accutane is
reserved for treating
nodulocystic acne (severe cystic acne). This medication is not
effective for the milder forms of
acne such as comedonal. Isotretinoin is teratogenic in women of
childbearing age and has other
side effects. Therefore, strict adherence to FDA guidelines is
required
79. (A) Clavicular factures are the most common fractures in
infants and children. In newborns, this
fracture is usually unilateral and often occurs after a difficult
delivery. Many times no treatment
is required or a figure-of-eight bandage can be used. For infants
and children, a sling can be
used. The bump that can be seen after fracture consolidation will
usually resolve in a few months
to a year. The next most common fractures are of the
extremities, humerus being the most common
and then the femur, but still much less common than the
clavicle.
80. (B) Osgood–Schlatter disease is caused by microfractures of
the patellar ligament where it
inserts into the tibial tubercle. This condition usually occurs in
the preteen and adolescent years,
and is more common in males than females. The history of
injury can be vague and the patient
may not remember a specific injury that precipitated the pain.
Often, the pain progresses to the
point of interference of even routine physical activities. X-rays
may or may not show any
abnormalities. Upon X-ray, Type I disease appears normal, but
Type II will reveal fragmentation
of the tibial tubercle. Often, after healing there will be
enlargement of the tibial tubercle.
Generally, treatment consists of rest, limitation of activities, and
isometric exercises.
Chondromalacia patellae can only be diagnosed under an
arthroscopic examination, not on the
basis of clinical features. Patellofemoral overuse syndrome
presents with medial knee pain and
subpatellar pain. Additional signs are swelling and crepitus in
the knee and it is more common in
females than males. It is diagnosed by increased Q-angles
(anterosuperior iliac spine through
center of patella to tibial tubercle). Subluxation of the patella or
dislocation is more common in
adolescent girls and the patient presents with acute knee pain.
The knee is in flexion with a mass
lateral to the knee and with absence of the bony prominence of
the patella (flat). X-ray confirms
the dislocation.
81. (C) Enterobiasis or pinworms is a worldwide infection that
affects people of all ages and
socioeconomic levels. It especially affects children. The classic
manifestation of this problem is
nocturnal anal pruritis and sleeplessness. The sleeplessness may
be secondary to the migration of
female worms to the perianal area to lay eggs, during which the
tape may pick up the larvae.
Transmission of the worms occurs when children ingest the eggs
that are present on their hands
(from scratching), in the bedclothes, or in house dust. After
hatching in the stomach, the larvae
migrate to the cecum where they mature into adults. The
treatment of choice for pinworms is
pyrantel pamoate or mebendazole. Albendazole may also be
used. For eradication of this
parasite, often the entire family must be treated at once. Ascaris
is a helminthiasis infection that
is ingested and excreted in the stool. Diagnosis is made by stool
examination for the
characteristic eggs. Hookworms are found in warm, damp soil
and penetrate the skin. From there
the infection can spread to the lungs where they ascend into the
trachea to be swallowed and live
in the intestine. Diagnosis is made by stool examination for the
eggs. Whipworm is ingested from
the soil and lives in the intestine; detection is also made by egg
in the feces.
82. (A) Juvenile idiopathic arthritis (JIA) presents as three
distinct types. The types are based upon
clinical manifestations during the first 6 months of the illness.
The most common type is
pauciarticular as presented by this 13-year-old boy in the
scenario mentioned. Second is
polyarticular disease with five or more joints being affected, and
the third is systemic onset of
disease that begins with high spiking fevers that are often
associated with a rash that comes and
goes with the fever elevations. It is recommended that patients
with pauciarticular JIA have an
ophthalmologic evaluation and slit lamp examination every 3
months, if the antinuclear antibody
test (ANA) is positive and every 6 months, if the ANA is
negative, for 4 years after the JIA is
identified to catch iridocyclitis (untreated results in blindness).
Lyme arthritis usually presents
with a monoarticular rash that typically affects the larger joints,
without morning stiffness.
Enteropathic arthritis is associated with gastrointestinal
symptoms occurring simultaneously as
lower extremity arthritis. It encompasses Reiter syndrome,
reactive arthritis (eg, postsalmonella,
shigella), and arthritis associated with celiac disease and
inflammatory bowel
disease. Psoriatic arthritis is the arthritis accompanying the
dermatological disorder of psoriasis.
The build-up of epidermal cells over the joints causes
inflammation and thickening that results in
arthralgia.
83. (B) Bacterial conjunctivitis is often unilateral and presents
with a mucopurulent discharge.
Common bacterial causes of this problem include nontypable
Haemophilus, Streptococcus
pneumoniae, Moraxella catarrhalis, and Staphylococcus aureus.
These infections usually
respond to topical antibiotics such as sulfacetamide and
erythromycin. Systemic treatment is
indicated for conjunctivitis caused by chlamydia trachomatis,
Neisseria gonorrhea, or
Neisseria meningitides. Allergic conjunctivitis is usually
associated with moderate to severe
itching of the eyes and clear mucoid drainage. Viral
conjunctivitis is usually associated with
minimal itching, profuse tearing, and minimal clear mucoid
drainage. While reactive arthritis
typically presents with a conjunctivitis, it is also concomitantly
present with arthritis and
urethritis.
84. (C) Dacryocystitis, whether acute or chronic, is usually
secondary to bacterial infections. It
presents as an acutely inflamed swelling and tender area over
the lacrimal sac just medial and
inferior to the inner canthus of the eye. Because the lacrimal sac
is inflamed and blocked there is
tearing and usually purulent discharge from the eye. There may
also be an orbital cellulitis.
Treatment consists of oral and topical antibiotics and warm
compresses, and surgical drainage
may also be indicated. After the acute episode and for chronic
cases, surgical correction of the
nasolacrimal obstruction is required. Anterior uveitis typically
presents with pain, photophobia,
blurred vision, and injection without exudates. Blepharitis is an
inflammation of the lid margin
that presents with crusty debris along the lashes. Unless there is
a concomitant conjunctival
infection there is typically no injection noted.
85. (B) While halitosis can be caused by pharyngitis, sinusitis,
and poor hygiene, the most common
cause of halitosis in children is a nasal foreign body. Seeds and
beads are the leading objects
inserted into the nose. If not promptly removed, they can cause
nasal obstruction, infection,
rhinorrhea, bleeding, halitosis, or a foul smell. They are usually
easy to remove, but if there is
difficulty in removing the foreign body, the child should be
referred to an otolaryngologist for
definitive care. Tobacco use in adolescents is a common cause
of halitosis. Dental disease is the
most common cause of halitosis in adults.
86. (C) Most cases of epistaxis in the anterior portion of the
nose are caused by digital trauma (nose
picking) or some other mechanical cause such as nose blowing
or repeated nose rubbing. Other
causes may include incorrect use of steroid nasal sprays.
Examination of the anterior nose will
usually reveal irritation of the Kiesselbach area. Less than 5% of
recurrent nosebleeds are
caused by bleeding disorders. Choanal atresia, unilateral, usually
appears as a chronic nasal
discharge that may be mistaken for chronic sinusitis. Foreign
bodies typically present with
purulent discharge instead of bleeding.
87. (A) Cat scratch disease (CSD) is caused by the gram-
negative bacillus, Bartonella henselae.
The disease is more common in the fall and winter months and
more males than females are
affected. Typically (approximately 90%), patients report
handling a cat or kitten and up to 70%
will report a scratch by a cat. The most common complication of
CSD is encephalitis. About half
of the patients with CSD will develop a primary cutaneous
papule at the site of inoculation, most
often (approximately 50%) on the hands or upper extremities, 3
to 10 days after the exposure.
Regional lymphadenopathy will usually develop in about 1 to 7
weeks after the cutaneous lesions
and will affect the nodes draining the site of the scratch or bite.
The affected lymph nodes may be
inflamed and are usually tender. Occasionally, the involved
nodes may suppurate. The
lymphadenopathy resolves in about 2 months, but may last as
long as 4 to 8 months. Treatment is
usually not indicated for this self-resolving disease. However,
suppurative lesions may need to
be aspirated for pain relief. It has been shown that 5 days of
treatment with azithromycin has
helped to speed recovery for some patients. Because Hodgkin
disease involves the lymph nodes,
it should be considered as a differential diagnosis when
evaluating a child for CSD. However, it
typically presents as a cervical lymphadenopathy. Fifth disease
(erythema infectiosum) is a
childhood disease caused by the human parvovirus. This
common community-acquired disease
does not usually require treatment, but respiratory isolation is
recommended for 7 days following
the onset of symptoms. The initial stage of the disease presents
as red cheeks that appear to be
“slapped” or “slapped cheeks” with circumoral pallor.
Osgood–Schlatter disease is an
orthopedic problem in children. It is the result of repetitive
microtraumas to the patellar ligament
at its point of insertion into the tibial tubercle. Usually, rest and
anti-inflammatory medications
are helpful in alleviating the pain associated with this condition.
88. (B) Hookworm (Ancylostoma duodenale and Necator
americanus) infections, if severe, can
cause iron deficiency anemia. Abdominal discomfort, weight
loss, and ova in the stool are more
commonly associated with these nematodes. Both types of
human hookworms are found in tropic
and subtropical climates, which include the southeastern United
States, primarily the coastal
areas. The larva of this parasite is passed in the feces and
incubates in warm, damp soil when
they hatch into larvae. The larvae penetrate directly into the skin
of humans, enter the
bloodstream, and migrate to the lungs. From the lungs they
move up to the trachea and are
swallowed. Once swallowed, they mature in the intestines. The
worms attach their mouth to the
mucosal lining of the intestine where they suck blood and shed
new ova. Mild infections are
usually asymptomatic, but severe infestations can cause anemia.
Treatment for the infestation is
achieved with albendazole. In severe cases of anemia, parenteral
iron or transfusion may be
indicated. Pinworms are associated only with localized pruritus,
specifically the anus. Treatment
may help recurrent urinary tract infections in some young girls
when the pinworm has infected the
urethra. Ascariasis is usually asymptomatic; however, in severe
cases it may be associated with
anorexia, diarrhea, vomiting, weight loss, and abdominal pain.
Whipworm is also asymptomatic
until the infection is severe, with general gastrointestinal
symptoms—pain, diarrhea, and mild
abdominal distention. Eosinophilia may also be present,
although slight.
89. (C) Appearing in 60% to 80% of cases, the characteristic
rash may not be present in all cases of
acute Lyme disease. Following the bite of a deer tick (Ixodes
species), infected with the
spirochete Borrelia burgdorferi, an erythematous ring forms
around the bite site and spreads
outward. The ring may have a raised border and usually a clear
center. The ring can attain a
diameter of up to 20 cm. Multiple rings may form and they can
form at sites distal to the original
bite site. If left untreated, the rash will usually resolve within 3
weeks. Erythema migrans is a
minimally tender to nontender, nonscaly rash that persists longer
than many of the other
erythematous rashes of childhood.
90. (C) Chlamydial infections are the most common cause of
conjunctivitis in newborns in
developed countries. Other causes of ophthalmia neonatorum
include reactions to silver nitrate
prophylaxis, other bacterial infections such as gonococcal or
staphylococcal, or viral organisms
such as adenovirus or echovirus. Chlamydia trachomatis causes
conjunctivitis and pneumonia in
neonates. Treatment for chlamydial conjunctivitis should be
with systemic erythromycin to treat
the conjunctivitis and as prophylaxis against pneumonia.
91. (D) Seizures have not been associated with Lyme disease.
Neurologic manifestations occur in up
to approximately 20% of patients with Lyme disease. Primarily,
these are Bell palsy,
lymphocytic, aseptic meningitis, and polyradiculitis. Cranial
neuropathies, such as Guillain–
Barré syndrome and ataxias are less common. Additional
neurologic manifestations include
peripheral neuropathy, pseudotumor cerebri, and encephalitis. If
untreated, most neurological
symptoms are self-limited but some will persist or become
permanent.
92. (D) Oral candidiasis (thrush) is very common in the first few
weeks of infancy. The diagnosis is
usually done by visual inspection and does not usually require
further laboratory testing. On
visual examination, white, creamy plaques are found on the
buccal mucosa and occasionally the
gingival and lingual mucosa. For this age group, direct topical
application of nystatin in oral
suspension to the lesions should suffice. If the lesions are
resistant to treatment or if they occur in
older children, consideration should be given to the possibility
of the patient being
immunocompromised. All sources of candida, such as toys and
bottle nipples, should be
sterilized daily. Herpangina and hand–foot–mouth disease are
ulcerating lesions of the oral
cavity due to viruses and are self-limiting, but can be very
painful. Leukoplakia is a precursor
lesion to oral cancer, seen most commonly in oral tobacco users.
93. (B) Bipolar affective disorder is the most likely diagnosis for
this patient. Although ADHD,
bipolar disorder, and conduct disorder share many similarities in
behavior disorders, such as
varying degrees of school and behavior problems, defiant
attitude, and distractibility, the
obsession with ideas (in this case, science fiction movies) is not
present in ADHD and conduct
disorder. The mood swings described here, as depression and
elation are consistent with bipolar
disorder, which is confirmed by the presence of hallucinations.
Hallucinations, when considering
a differential diagnosis in a behavior disorder, are diagnostic for
bipolar disorder. In up to 70%
of patients with bipolar disorder, their first symptom of the
disorder may be depression.
However, hallucinations are not typically a manifestation of
depression.
94. (D) Rocky Mountain spotted fever (RMSF) is the most
common rickettsial infection in the United
States, especially in the eastern, southeastern, and western
states, and it is very common in 5- to
9-year-old children. A known tick exposure may or may not be
documented. Most exposures to
ticks carrying Rickettsia rickettsii, the causative organism of this
disease, occur in the warmer
months of April to September when victims are most likely to
participate in outdoor activities in
wooded areas. The incubation period of RMSF is 3 to 12 days
(mean 7) after a tick exposure.
The tick must be attached for 6 hours or greater in order to
transmit the disease. Clinical
presentation includes fever, often 40°C, myalgias, headache,
and less characteristic, red-rose
macular or maculopapular rash. The rash usually appears within
2 to 6 days, after the fever. The
rash is especially prevalent on the palms, soles, and extremities.
After several days, the rash,
which starts peripherally and spreads centrally, becomes
petechial. Conjunctivitis, edema,
splenomegaly, meningismus, and confusion may occur. Up to
5% to 7% of patients with RSMF
will die, and therefore, delays in treatment should be avoided.
Treatment for children is
doxycycline, regardless of age and the possible side effect of
stained teeth. In endemic areas,
treatment should be started early and is often based on suspicion
alone, and prior to the
appearance of the rash. Endemic typhus (murine typhus) is not
transmitted by ticks but instead by
the fleas from infected rodents. The rash of endemic typhus
differs from that of RMSF in that it
does not involve the palms and soles. Q fever is spread by
inhalation instead of ticks. The cause
of this rickettsial disease is Coxiella burnetii hosted by domestic
animals including dogs, cats,
cattle, and sheep. Unpasteurized milk from infected animals
may also be a source of this
infection. One form of human monocytic ehrlichiosis is carried
by ticks that have fed on infected
hosts that may include deer, wild rodents, and sheep, most
commonly in the southeast, north, and
south central United States. The presentation is usually a viral
syndrome without any rash.
Although this is usually a self-limiting disease, deaths do occur
in children; therefore, treatment
should be carried out with the antibiotic of choice, doxycycline,
regardless of side effects.
95. (D) In young children, respiratory syncytial virus (RSV)
accounts for more than 70% of
bronchiolitis, approximately 40% of the cases of pneumonia,
and about 10% of cases of croup.
This seasonal disease occurs in the winter and early spring
months of the year. More than 50% of
children have been infected with RSV by age 1, and by the age
of 2, almost all children have
been infected. Reinfection commonly occurs but is mild.
Adenovirus infections, though common
in early childhood, only account for approximately up to 10% of
all respiratory diseases. The
peak incidence of adenovirus respiratory infections occurs in the
spring, summer, and early
winter. Human parvovirus infection is typically seen in school-
aged children. This disease is
characterized by the “slapped-cheek” appearing rash on the
face that appears about 10 to 17 days
following the infection. About 2 days after the appearance of
this facial rash, a similar rash
appears on the extremities, trunk, neck, and buttocks. The rash
often persists for a few days to a
few weeks (average of 10 days) and often will recur with
exposure to bathing in warm water,
exercise, sunlight, and stress. Parainfluenza viruses fall into four
categories and are responsible
for the majority of cases of croup (65%), laryngitis (50%), and
tracheobronchitis (25%). Types 1
to 3 occur as seasonal outbreaks with types 1 and 2 in the fall
and type 3 in the spring and
summer. Type 4 is an endemic virus. Clinical symptoms of these
viruses include laryngotracheitis
(croup), laryngitis, bronchiolitis, and less commonly pneumonia
(especially in
immunocompromised children).
96. (B) Asthma, in this case exercise-induced, is the most likely
cause of this problem. The
symptoms commonly associated with acute exacerbations of
asthma include wheezing, cough,
dyspnea, and chest pain. Some symptoms that might be
suggestive of asthma include exerciseinduced
cough, nighttime cough, cough after cold air exposure, and
cough after laughing. Airway
foreign bodies, though not common, are an acute problem that
may present as sudden cough,
choking, and wheezing. Cystic fibrosis (CF) is the most
common, lethal, genetic disease affecting
the Caucasian population. Up to 50% of patients with CF are
diagnosed in infancy, but others
may not be diagnosed until adolescence or adulthood. Chronic
or recurrent cough should be an
indicator for consideration of CF as a differential diagnosis.
Laryngomalacia is the most common
cause of stridor in infants. It is the incomplete development of
the cartilaginous support of the
laryngoglottic structures. This congenital condition is usually
self-limiting and occurs most
commonly in infants at or just after birth. The inspiratory
collapse of the epiglottis or arytenoid
cartilages is heard as stridor.
97. (B) Autism is the most likely diagnosis for this child. The
signs of autism often present before the
second year of life such as the child's failure to respond to their
name, failed speech
development, and appearing self-absorbed and withdrawn in the
presence of other children or
adults. Often in childhood, autistic children may develop
ritualistic behaviors and intense
interests that if interrupted may cause tantrums and rages. When
speech does begin to develop, it
may be nonsensical: reversal of speech patterns, echolocation,
and other abnormal patterns.
Goals of treatment include early intervention to address
behavior and communication skills.
ADHD is characterized by easy distractibility, inattention, and
overactivity. Estimates for the
presence of ADHD in school-aged children range from 2% to
20%. Fragile X syndrome is the
most common cause of functional mental retardation. This
syndrome, affecting approximately 1 in
1,250 males, is caused by a trinucleotide expansion (CGG
repeated sequence) in the Fragile X
Mental Retardation I (FMR1) gene. Fragile X syndrome is
characterized by a wide range of
symptoms, which may include language delay, hyperactivity,
autistic behavior, and variable
levels of mental retardation. Schizophrenia is usually detected in
adolescence, with prepubertal
onset occurring rarely. Patients may initially present with
somatic or social behavior problems.
Schizophrenic children and adolescents often have the same
symptoms as adults, such as
hallucinations, bizarre thought processes, and rambling speech.
98. (A) Choanal atresia, whether unilateral or bilateral, is a nasal
obstruction that occurs relatively
rarely in newborns. If bilateral choanal atresia occurs at birth, it
causes a respiratory distress
that requires immediate treatment (due to infants being obligate
nose breathers) by placing an
oral airway and subsequent surgical correction. Unilateral
choanal atresia can present as a
chronic, single-sided, nasal discharge that may not appear until
later in childhood. Meconium
ileus, intestinal obstruction secondary to inspissated meconium,
occurs in approximately 10% of
newborns with cystic fibrosis. Cystic fibrosis affects
approximately 1 in 2,500 live Caucasian
births, and is a leading cause of death in young adults. Nasal
infections may occur secondary to a
furuncle (infected hair follicle) in the anterior nares or as a nasal
septal abscess following
spread of a furuncle. Common causes of nasal infections include
picking at the nose and pulling
out nose hair. Nasal polyps are uncommon in children younger
than age 10, and when they do
occur it is usually in older children and adults with allergic
rhinitis.
99. (A) It is estimated that 1% to 5% of adolescents are affected
by anorexia nervosa. There are two
types of anorexia nervosa. The first is the nonpurging type when
patients restrict their total
caloric intake and the second involves binge eating and purging
in association with the restrictive
dietary habits. Otherwise, intensive exercise regimes may be
used as a means to control weight.
Anorexia nervosa occurs in boys but is more prevalent in girls
(2:1). The specific etiology of
this familial problem is unknown; there are genetic and
environmental factors. DSM-IV criteria
also include refusal to keep weight at 85% of ideal weight,
intense fear of gaining weight even
though underweight amenorrhea and disturbance in the way
one's body shape is experienced.
100. (C) Obesity is the number one nutritional disorder in
children in the United States. In 2004, 17%
of American children aged between 9 and 19 were considered
obese. Risk factors for obesity
include other obese family members and infants born to diabetic
mothers. Associated
environmental factors include sedentary lifestyle, total caloric
intake, television watching, and
computer games. All are considered contributory factors in
childhood obesity. Binge eating
disorder is a relatively new eating disorder category. It is most
frequent in overweight or obese
individuals. This disorder includes recurrent episodes of binge
eating (eating more than most
individuals would in a 2-hour period) and a sense of lack of
control over the impulse to eat,
marked distress over the episode at least 2 days a week, and is
not associated with regular
compensatory activity such as purging or fasting. Folate
deficiency anemia (megaloblastic) can
occur in infants within a few weeks after birth. This deficiency
may be a result of malabsorption,
low dietary intake such as with goat's milk or home-prepared
formulas that have been sterilized
by heating, or formulas based on pasteurized milk. Infants who
are breastfed or given
supplemented cows’ milk formulas do not have a problem with
folate deficiency. In children,
rickets is most commonly a result of poor dietary intake of
vitamin D and inadequate exposure to
direct sunlight. Vitamin D sources include milk, cheese, and
baby formula. Vitamin D in humans
is produced by activation of its inactive precursors in the skin
after exposure to ultraviolet light.
101. (B) The symptoms for congestive heart failure in infants
are typically failure to thrive,
tachycardia, and poor feeding. These will typically not present at
birth and will be identified by
the 6-month well visit.
102. (A) Bordetella pertussis is a gram-negative bacillus and,
therefore, of all the choices, the
antibiotic with good gram-negative coverage is erythromycin.
The other macrolides,
azithromycin and clarithromycin may also be given for shorter
durations, however they are more
expensive. Ampicillin, amoxicillin, and cephalexin provide
mainly gram-positive coverage.
103. (C) Surgical reduction is the treatment of choice for
incarcerated hernias over 12 hours. At that
point the likelihood that the hernia will manually reduce is very
small and the bowel is becoming
necrotic and needs to be removed as soon as possible. Bilateral
surgical reduction is required
only in the event of two hernias, and there is no place for
prophylaxis surgery for inguinal hernia
repairs.
104. (B) Salmonella species are gram-negative bacilli that are
classified as Enterobacteriaceae,
along with E Coli. While extremely uncommon as an etiology
for meningitis, salmonella can
cause lethal meningitis infections and must be watched. While
there is typically no treatment for
mild to moderate diarrhea from salmonella infections, these
patients should be monitored for
complete resolution. Viral meningitis typically does not have a
positive Gram stain, unless there
is contamination. Corynebacterium and clostridium are gram-
positive bacilli
105. (A) Vitamin K deficiency causes hemorrhagic disease of
the newborn. Vitamin K is one of the
compounds required for conversion of prothrombin, factors VII,
IX, and X of the coagulation
cascade. In addition, proteins C & S are also Vitamin K
dependent. Therefore, the result is an
increased prothrombin time and this would result in an increased
aPTT. There is no effect on
platelets or fibrinogen.
106. (C) The diagnosis of otitis media requires the presence of
middle ear effusion, acute onset of
symptoms, and signs and symptoms of middle ear inflammation.
Presence of the middle ear
effusion can be determined by the bulging of the tympanic
membrane, air-fluid levels, absent
mobility of the tympanic membrane by pneumatic otoscopy, or
otorrhea from perforation. Office
tympanometry can be performed to confirm a diagnosis of
effusion. Tenderness on palpation of
the tragus typically is a sign of otitis externa.
107. (C) While different states have different requirements for
child safety restraints, the most
common guidelines state that infants must be 20 lb and 1 year of
age before switching to forwardfacing
seats. Children between 20 lb and 40 lb should be in front-facing
safety seats, typically
with a 5-point harness; children between 40 and 80 lb may be in
booster seats in which the back
is typically required based on the height of the child. Lastly,
children should be older than 12
years of age and typically at least 80 lb as the front air bags are
dangerous. 108. (D) The most common etiology of
gynecomastia is idiopathic. Occurring in 50 % to 60% of
adolescent males, idiopathic gynecomastia typically is self-
limited. Additional uncommon
etiologies of gynecomastia include liver disease,
hyperthyroidism, illicit drugs (marijuana
heroin), neoplasms (adrenal, testicular), and medications (eg,
antacids, chemotherapy)
109. (A) A urinalysis should be performed because renal disease
is the most common etiology of
hypertension in children. Electrocardiograms and chest
radiography should be considered as part
of the evaluation for end-organ disease as well as an initial basic
metabolic panel to include
serum and creatinine. Although rare, elevated uric acid has also
been shown to cause essential
hypertension in children.
110. (B) The CDC recommends that there are two age ranges for
testing lead in children in the United
States: 9 to 12 months and again at 24 months. These high-risk
areas include poverty-stricken
areas, use of lead paint pottery, lead painted homes (peeling or
cracking), industrial exposures,
and use of diarrhea remedies in Mexico. The CDC recommends
using questions to screen all
children between 6 months and 6 years of age.

You might also like